You are on page 1of 985

REBECCA CALDWELL OGLESBY, MSN, RN

The new edition of the most-trusted resource for the NCLEX-RN licensure exam has arrived!
NCLEX-RN Review, Sixth Edition continues to be the most reliable and practical tool for successful
preparation for the nurse licensure examination. Based on the latest test plan issued by the National
Council of State Boards of Nursing (NCSBN), this resource will help you reach your goal of becoming
a registered nurse. The comprehensive, outline-format content review is accompanied by over 3,000
unique and challenging NCLEX-style questions with answers, rationales, and detailed coding that
pinpoint cognitive level, area of client need, and nursing process.
Buy it today and nd out why this is the book of choice for thousands of successful nursing students
and carries the endorsement of the National Student Nurse Association (NSNA).

Key Features:
New charting type questions that reect the latest changes to the examination.
Robust CD-ROM with over 2,000 questions and comprehensive practice tests simulates
the test experience in presentation, interactivity, and layout.
Emphasizes delegation, prioritization, and pharmacology.

Additional Resources Available From Delmar Cengage Learning:


Complete Review for NCLEX-RN by Donna Faye Gauwitz, MS, RN
ISBN 10: 0-7668-6237-2
ISBN 13: 978-0-7668-6237-1
Complete Review CD-ROM for NCLEX-RN CD-ROM by Donna Faye Gauwitz, MS, RN
ISBN 10: 0-7668-6238-0
ISBN 13: 978-0-7668-6238-8
Practice Questions for NCLEX-RN by Donna Faye Gauwitz, MS, RN
ISBN 10: 1-4018-0590-6
ISBN 13: 978-1-4018-0590-6

To learn more about Delmar, visit www.cengage.com/delmar


Purchase any of our products at your local college store or at our
preferred online store www.ichapters.com

1418053155_cvr_se_p.indd 1 3/6/09 9:52:20 PM


53155_00_FM_pi-xxii.qxd 3/2/09 10:30 PM Page i


NCLEX-RN
REVIEW
Sixth Edition

Rebecca Caldwell Oglesby, MSN, RN


Instructor of Nursing , BSN Program
The Presbyterian School of Nursing at Queens University of Charlotte
Charlotte, North Carolina

Australia Canada Mexico Singapore Spain United Kingdom United States


53155_00_FM_pi-xxii.qxd 3/2/09 10:30 PM Page ii

NCLEX-RN Review, Sixth Edition Copyright 2010, 2005, 2000, 1997, 1994, 1992 Delmar,
Rebecca Caldwell Oglesby Cengage Learning

Vice President, Career and Professional Editorial: ALL RIGHTS RESERVED. No part of this work covered by the copyright
Dave Garza herein may be reproduced, transmitted, stored, or used in any form or
by any means graphic, electronic, or mechanical, including but not
Director of Learning Solutions: Matthew Kane
limited to photocopying, recording, scanning, digitizing, taping, Web
Senior Acquisitions Editor: Maureen Rosener distribution, information networks, or information storage and retrieval
Managing Editor: Marah Bellegarde systems, except as permitted under Section 107 or 108 of the 1976 United
Senior Product Manager: Patricia Gaworecki States Copyright Act, without the prior written permission of the
publisher.
Editorial Assistant: Meaghan OBrien
Vice President, Career and Professional Marketing: For product information and technology assistance, contact us at
Jennifer McAvey Cengage Learning Customer & Sales Support, 1-800-354-9706
Marketing Director: Wendy Mapstone For permission to use material from this text or product, submit all
Senior Marketing Manager: Michele McTighe requests online at www.cengage.com/permissions.
Further permissions questions can be e-mailed to
Marketing Coordinator: Scott Chrysler permissionrequest@cengage.com
Production Director: Carolyn Miller
Production Manager: Andrew Crouth Library of Congress Control Number: 2008930400
Senior Content Project Manager: Stacey Lamodi ISBN-13: 978-1-4180-5315-4
Senior Art Director: Jack Pendleton
ISBN-10: 1-4180-5315-5
Technology Project Manager: Erin Zeggert

Delmar
5 Maxwell Drive
Clifton Park, NY 12065-2919
USA

Cengage Learning is a leading provider of customized learning solutions


with office locations around the globe, including Singapore, the United
Kingdom, Australia, Mexico, Brazil, and Japan. Locate your local office at:
international.cengage.com/region

Cengage Learning products are represented in Canada by Nelson


Education, Ltd.

To learn more about Delmar, visit www.cengage.com/delmar


Purchase any of our products at your local college store or at our
preferred online store www.ichapters.com
Notice to the Reader
Publisher does not warrant or guarantee any of the products described herein or perform any independ-
ent analysis in connection with any of the product information contained herein. Publisher does not
assume, and expressly disclaims, any obligation to obtain and include information other than that
provided to it by the manufacturer. The reader is expressly warned to consider and adopt all safety
precautions that might be indicated by the activities described herein and to avoid all potential hazards.
By following the instructions contained herein, the reader willingly assumes all risks in connection with
such instructions. The publisher makes no representations or warranties of any kind, including but not
limited to, the warranties of fitness for particular purpose or merchantability, nor are any such represen-
tations implied with respect to the material set forth herein, and the publisher takes no responsibility
with respect to such material. The publisher shall not be liable for any special, consequential, or exemplary
damages resulting, in whole or part, from the readers use of, or reliance upon, this material.

Printed in Canada
1 2 3 4 5 6 7 12 11 10 09
53155_00_FM_pi-xxii.qxd 3/2/09 10:30 PM Page iii

Dedication

For my husband Mike, who patiently endured this work;


For my colleagues, who never doubted me;
For dear friends, who believed in me;
For Echo Heron, who inspired me;
For nursing students, whose compassion I revere.

iii
53155_00_FM_pi-xxii.qxd 3/2/09 10:30 PM Page iv

A Message from the National


Student Nurses Association

The National Student Nurses Association (NSNA) is pleased to bring you the NSNA endorsed
NCLEX-RN Review, Sixth Edition. Using this book will better prepare you to meet the challenge
of passing the exam the first time around.
NSNA is committed to the professional development of todays nursing student. We recognize
the challenges of succeeding in todays complex health care environment. This outstanding book
maintains high standards both in content and in presentation. The contributing experience of the
clinicians and educators will help you achieve NCLEX success!

iv
53155_00_FM_pi-xxii.qxd 3/2/09 10:30 PM Page v

CONTENTS

Message from the National Student Nurses Analysis 10


Association iv Planning and Implementation 10
Techniques of Drug Administration 10
Contributors xi
Medication Calculations 18
Preface xvii
Central Nervous System Drugs 23
List of Abbreviations xix
Local Anesthetics 23
Nonnarcotic Analgesics and Antipyretics 25
UNIT 1 Narcotic Analgesics 27
PREPARING FOR THE NCLEX Narcotic Antagonists 28
EXAMINATION 1 Sedatives and Hypnotics 29
Anticonvulsants 31
Understanding the NCLEX Examination 2 Muscle Relaxants 32
The Test plan 2 Antipsychotic Agents 32
How the Test Is Constructed 2
How the Test Is Scored 3 Autonomic Nervous System Drugs 37
How Candidates Are Notified of Results 3 Adrenergic Drugs 37
Adrenergic Blocking Agents 41
Preparation and Test Taking 3 Cholinergics 42
Using the Test Plan to Your Best Advantage 3 Anticholinergics 43
Final Preparation for Test Taking 4 Antiparkinson Agents 44
Taking the Test 5
How to Use this Book 6 Drugs Affecting the Endocrine System 46
Antidiabetic Agents 46
References and Suggested Readings 6 Pituitary Hormones 48
Corticosteroids 49
Thyroid Hormones 50
UNIT 2 Thyroid Antagonists 50
DRUGS AND NURSING Womens and Mens Health Agents 51
IMPLICATIONS 7 Oxytocics 52

Factors Affecting Drug Action 8 Eye Drugs 53


Definition of a Drug 8 Mydriatics and Cycloplegics 53
Factors Affecting Drug Action 8 Miotics 54

Drug Administration 10 Cardiovascular Drugs 55


Assessments Appropriate to All Medication Cardiac Glycosides 55
Administration 10 Antianginal Drugs 57

v
53155_00_FM_pi-xxii.qxd 3/2/09 10:30 PM Page vi

Peripheral Vasodilators 59 Antineoplastics Affecting Hormonal Balance 100


Antidysrhythmics 59 Mitotic Inhibitors 102
Cardiac Stimulants 62 Miscellaneous Antineoplastic Agents 102
Anticoagulants 62
Thrombolytic Drugs 64 Immunosuppressants 103
Antilipemic Agents 65 Azathioprine (Imuran) 103
Antihypertensives 65 Cyclosporine (Sandimmune) 104

Renal Drugs 69 Vitamins and Minerals 104


Diuretics 69 Vitamins 104
Potassium-Removing Resin 72 Minerals 105
Heavy Metal Antagonists 105
Respiratory Drugs 72
Antiasthmatic Drugs 72 Herbs and Herbal Health Products 106
Antihistamines 74 History 106
Mucolytics 74 Source and Use 106
Expectorants and Antitussives 75 Natural Pharmacy 106
Complications 106
Gastrointestinal Tract Drugs 76 Professional Responsibilities 106
Histamine (H2) Antagonists 76
Gastrointestinal (GI) Anticholinergics 77 Vaccines and Toxoids 108
Antacids 77 Vaccines and Toxoids 108
Antidiarrheal Agents 78 Immune Serums 109
Laxatives 79
Antiemetics 81 References and Suggested Readings 110
Emetics 81
Sucralfate (Carafate) 82

Arthritis Drugs 83 UNIT 3


Arthritis Drugs 83 UNIVERSAL PRINCIPLES OF
Antigout Drugs 84 NURSING CARE
MANAGEMENT 111
Antimicrobials 85
General Information 85
Nursing Practice Standards 112
Aminoglycosides 86
Nursing: Scope & Standards of Practice (2004) 112
Penicillins 86
Standards of Practice 112
Cephalosporins 87
Standards of Professional Performance 114
Macrolides 89
Tetracyclines 89
Legal and Ethical Aspects of Nursing 116
Chloramphenicol 90
Overview 116
Sulfonamides 91
Guide to the Code of Ethics for Nurses: Interpretation
Urinary Anti-Infectives 91
and Application (ANA, 2001) 116
Vancomycin Hydrochloride (Vancocin) 92
Fluoroquinolones 93
Managing Client Care 118
Antitubercular Drugs 93
Priorities of Client Care 118
Assignment Methods for Delivery of Care 119
Antiviral Agents 95
Acyclovir (Zovirax) 95
Safety 121
Fire Safety/Preparedness Practices 121
Antifungal Agents 95
Equipment 122
Antifungals 95
Restraints 122
Principles and Interventions for Specific Aspects of
Anthelmintic Agents 96
Care 122
Anthelmintics 96
Universal/Clinical Issues 128
Antineoplastic Agents 97
Cultural Diversity in Health Practices 130
Antineoplastic Agents 97
Alkylating Agents 97
References and Suggested Readings 142
Antimetabolites 98
Antibiotic Antineoplastic Agents 99

vi CONTENTS
53155_00_FM_pi-xxii.qxd 3/2/09 10:30 PM Page vii

Evaluation 241
UNIT 4 Disorders of the Cardiovascular System 242
ADULT NURSING 143
The Hematologic System 264
Overview of Anatomy and Physiology 264
Multisystem Stressors 144 Assessment 268
Stress and Adaptation 144 Analysis 269
Inflammatory Response 144 Planning and Implementation 269
Immune Response 144 Evaluation 270
Nutrition 146 Disorders of the Hematologic System 271
Infection 155
The Respiratory System 283
Pain 155
Overview of Anatomy and Physiology 283
Fluids and Electrolytes 158
Assessment 286
Acid-Base Balance 161
Analysis 286
Intravenous Therapy 162
Planning and Implementation 286
Shock 164
Evaluation 291
Multiple Trauma 166
Disorders of the Respiratory System 292

Aging 173 The Gastrointestinal System 307


General Information 173 Overview of Anatomy and Physiology 307
Patterns of Health and Disease in the Older Adult 176 Assessment 310
Assessment 176 Analysis 312
Analysis 177 Planning and Implementation 312
Planning and Implementation 177 Evaluation 314
Interventions 177 Disorders of the Gastrointestinal System 314
Evaluation 178 Disorders of the Liver 326
Conditions 178 Disorders of the Gallbladder 331
Elder Mistreatment 179 Disorders of the Pancreas 332
Osteoporosis 180
Cerebral Vascular Accident 180 The Genitourinary System 341
Benign Prostatic Hypertrophy 180 Overview of Anatomy and Physiology 341
Cataracts 180 Assessment 343
Glaucoma 180 Analysis 345
Planning and Implementation 345
Perioperative Nursing 182 Evaluation 348
Overview 182 Disorders of the Genitourinary System 348
Preoperative Period 183
Intraoperative Period 184 The Musculoskeletal System 361
Postoperative Period 186 Overview of Anatomy and Physiology 361
Assessment 362
Oncologic Nursing 191 Analysis 363
Pathophysiology and Etiology of Cancer 191 Planning and Implementation 363
Diagnosis of Cancer 192 Evaluation 366
Treatment of Cancer 192 Disorders of the Musculoskeletal System 366

The Endocrine System 381


The Neurosensory System 199 Overview of Anatomy and Physiology 381
Overview of Anatomy and Physiology 199 Assessment 383
Assessment 203 Analysis 384
Analysis 207 Planning and Implementation 385
Planning and Implementation 207 Evaluation 385
Evaluation 210 Disorders of the Endocrine System 386
Disorders of the Nervous System 210
Disorders of the Eye 224 The Integumentary System 406
Disorders of the Ear 227 Overview of Anatomy and Physiology 406
Assessment 407
The Cardiovascular System 236 Analysis 407
Overview of Anatomy and Physiology 236 Planning and Implementation 408
Assessment 238 Evaluation 408
Analysis 240 Disorders of the Integumentary System 408
Planning and Implementation 240

CONTENTS vii
53155_00_FM_pi-xxii.qxd 3/2/09 10:30 PM Page viii

Complementary and Alternative Medicine (CAM) 413 Planning and Implementation 493
Overview 413 Evaluation 493
Disorders of the Gastrointestinal System 494
References and Suggested Readings 418
The Genitourinary System 501
Variations from the Adult 501
UNIT 5 Assessment 501
PEDIATRIC NURSING 421 Analysis 501
Planning and Implementation 501
Evaluation 502
Growth and Development 422 Disorders of the Genitourinary System 502
General Principles 422
Assessment 422 The Musculoskeletal System 506
Analysis 431 Variations from the Adult 506
Planning and Implementation 431 Assessment 507
Evaluation 432 Analysis 507
Growth and Development Issues 432 Planning and Implementation 507
Death and Dying 436 Evaluation 508
Disorders of the Musculoskeletal System 508
Multisystem Stressors 441
Genetic Disorders 441 The Endocrine System 515
Fluid and Electrolyte, Acid-Base Balances 444 Variations from the Adult 515
Accidents, Poisonings, and Ingestion 446 Analysis 515
Planning and Implementation 515
The Neurosensory System 450 Evaluation 515
Variations from the Adult 450 Disorders of the Endocrine System 515
Assessment 451
Analysis 452 The Integumentary System 517
Planning and Implementation 452 Variations from the Adult 517
Evaluation 453 Assessment 517
Disorders of the Nervous System 453 Analysis 517
Planning and Implementation 518
The Cardiovascular System 464 Evaluation 518
Variations from the Adult 464 Disorders of the Integumentary System 518
Assessment 465
Analysis 466 Pediatric Oncology 521
Planning and Implementation 466 Overview 521
Evaluation 467 Assessment 521
Disorders of the Cardiovascular System 467 Analysis 522
Planning and Implementation 522
The Hematologic System 475 Stages of Cancer Treatment 523
Variations from the Adult 475 Cancers 525
Assessment 475
Analysis 475 References and Suggested Readings 532
Planning and Implementation 476
Evaluation 476
Disorders of the Hematologic System 476 UNIT 6
MATERNITY AND FEMALE
The Respiratory System 482 REPRODUCTIVE NURSING 533
Variations from the Adult 482
Assessment 482
Analysis 483 Overview of Anatomy and Physiology of the Female
Planning and Implementation 483 Reproductive System 534
Evaluation 484 Anatomy 534
Disorders of the Respiratory System 484 Physiology 536

The Gastrointestinal System 492 The Childbearing Cycle 538


Variations from the Adult 492 Conception 538
Assessment 493 Nidation (Implantation) 538
Analysis 493 Developmental Stages 538

viii CONTENTS
53155_00_FM_pi-xxii.qxd 3/2/09 10:30 PM Page ix

Special Structures of Pregnancy 539


Physical and Psychologic Changes of
UNIT 7
Pregnancy 541 PSYCHIATRIC-MENTAL HEALTH
NURSING 621
The Antepartal Period 543
Assessment 543 Overview of Psychiatric-Mental Health Nursing 623
Analysis 544 Theoretical Basis 623
Planning and Implementation 544 Nursing Process 626
Evaluation 547 Analysis 627
Complications of Pregnancy 547 Planning and Implementation 627
Pre- and Coexisting Diseases of Pregnancy 551 Evaluation 630
Behaviors related to Emotional Distress 630
Labor and Delivery 560
Overview 560 Psychiatric Disorders (DSM-IV-TR) 638
Assessment during Labor 564 Disorders of Infancy, Childhood, and Adolescence 638
First Stage of Labor 565 Pervasive Developmental Disorders 640
Second Stage of Labor 567 Delirium, Dementia, and other Cognitive Disorders 641
Third Stage of Labor 568 Substance Use Disorders 642
Fourth Stage of Labor 568 Psychoactive Substance-Induced Organic Mental
Complications of Labor and Delivery 569 Disorders 642
Analgesia and Anesthesia 573 Schizophrenia and other Psychotic Disorders 647
Operative Obstetrical Procedures 575 Mood Disorders 649
Neurotic Disorders 653
The Postpartum Period 581 Anxiety Disorders 653
Overview 581 Somatoform Disorders 655
Postpartal Psychosocial Changes 582 Dissociative Disorders 656
Assessment 583 Personality Disorders 656
Analysis 583
Planning and Implementation 583 Psychologic Aspects of Physical Illness 674
Evaluation 585 Stress-Related Disorders 674
Complications of the Postpartum Period 585 Victims of Abuse 674
Critical Illness 677
The Newborn 590 Chronic Illness 677
Physiologic Status of The Newborn 590 AIDS 678
Assessment 592 Death and Dying 679
Analysis 593 Grief and Mourning 679
Planning and Implementation 593
Evaluation 594 References and Suggested Readings 683
Variations from Normal Newborn
Assessment Findings 594

The High-Risk Infant 599


Overview 599
APPENDIX 685
Assessment 599
Analysis 599
Diabetic Diet and Exchange Lists 686
Planning and Implementation 599
Evaluation 599 Renal Diet 690
High-Risk Disorders 599
Special Conditions in the Neonate 601 Bariatric Diet 691
High-Fiber Diet 692
Conditions of the Female Reproductive System 607
Infertility and Fertility 607 1500-Kilocalorie Diet 692
Menstrual Disorders 610 1000-MilIigram Sodium-Restricted Diet 694
Infectious Disorders 612
Bland Diet 695
References and Suggested Readings 620 Low-Residue Diet 695
20-Gram Fat-Restricted Diet 696
Fat-Controlled Diet 697

CONTENTS ix
53155_00_FM_pi-xxii.qxd 3/2/09 10:30 PM Page x

COMPREHENSIVE
PRACTICE TESTS 699

Practice Test 1 699


2 723
3 744
4 765
5 786
6 806
7 824
8 845
9 865
10 882
11 899

Index 916

x CONTENTS
53155_00_FM_pi-xxii.qxd 3/2/09 10:30 PM Page xi

CONTR I BUTOR S

EDITOR Nancy Clarkson, MEd, RN, BC


Professor and Chairperson
Rebecca Caldwell Oglesby, MSN, RN Department of Nursing
Instructor of Nursing, BSN Program Finger Lakes Community College
The Presbyterian School of Nursing at Queens University Canandaigua, New York
of Charlotte
Charlotte, North Carolina Deborah L. Dalrymple, RN, MSN, CRNI
Associate Professor of Nursing
Montgomery County Community College
CONTRIBUTORS Blue Bell, Pennsylvania
Margaret Ahearn-Spera, RN, C, MSN
Director, Medical Patient Care Services Judy Donlen, DNSc, RN
Danbury Hospital Executive Director, Southern New Jersey
Danbury, Connecticut Perinatal Cooperative
Assistant Clinical Professor Pennsauken, New Jersey
Yale University School of Nursing
New Haven, Connecticut Judith L. Draper, APRN, BC
Assistant Professor
Cynthia Blank-Reid, RN, MSN, CEN Drexel University College of Nursing and Health Professions
Trauma Clinical Nurse Specialist Philadelphia, Pennsylvania
Temple University Hospital
Philadelphia, Pennsylvania Theresa M. Fay-Hillier, MSN, RN, CS
Clinical Adjunct Associate Professor Adjunct Faculty
Drexel University College of Nursing and Health Professions Drexel University College of Nursing and Health Professions
Philadelphia, Pennsylvania Philadelphia, Pennsylvania

Elizabeth Blunt, PhD (c), MSN Marcia R. Gardner, MA, RN, CPNP, CPN
Assistant Professor and Director, Graduate Nursing Assistant Professor
Programs Drexel University College of Nursing and Health
Drexel University College of Nursing and Health Professions Professions
Philadelphia, Pennsylvania Philadelphia, Pennsylvania

Margaret Brenner, RN, MSN Jeanne Gelman, MSN, MA, RN


Senior Consultant Professor Emeritus, Psychiatric-Mental Health Nursing
Pinnacle Healthcare Group, Inc. Widener University
Paoli, Pennsylvania Chester, Pennsylvania

xi
53155_00_FM_pi-xxii.qxd 3/2/09 10:30 PM Page xii

Theresa M. Giglio, MS, RD Judith C. Miller, MSN, RN


Instructor, LaSalle University President, Nursing Tutorial and Consulting Services
Philadelphia, Pennsylvania Clifton, Virginia

Judith M. Hall, RNC, MSN, IBCLC, LCCE, FACCE Eileen Moran, MSN, RN, C
Mary Washington Hospital Clinical Educator
Fredericksburg, Virginia Abington Memorial Hospital
Abington, Pennsylvania
Marilyn Herbert-Ashton, MS, RN, BC
Marie OToole, EdD, RN
Virginia Western Community College
Associate Professor, College of Nursing
Roanoke, VA
Rutgers, The State University of New Jersey
Newark. New Jersey
Marilyn Herbert-Ashton, RN, C, MS
Director, Wellness Center Faye A. Pearlman, MSN, RN, MBA
F. F. Thompson Health Systems, Inc. Assistant Professor
Adjunct Professor of Nursing Drexel University College of Nursing and Health Professions
Finger Lakes Community College Philadelphia, Pennsylvania
Canandaigua, New York
Janice Selekman, DNSc, RN
Holly Hillman, RN, MSN Professor and Chair
Assistant Professor Department of Nursing
Montgomery County Community College University of Delaware
Blue Bell, Pennsylvania Newark. Delaware

Lorraine C. Igo, MSN, EdD, RN Robert Shearer, CRNA, MSN


Assistant Professor Assistant Professor
Drexel University College of Nursing and Drexel University College of Nursing and Health Professions
Health Professions Philadelphia, Pennsylvania
Philadelphia, Pennsylvania
Magdeleine Vasso, MSN, RN
Nancy H, Jacobson, MSN, APRN-BC, CS Assistant Professor
Staff Development Coordinator Drexel University College of Nursing and Health Professions
Rydal Park Philadelphia, Pennsylvania
Rydal, Pennsylvania
Anne Robin Waldman, MSN, RN, C, AOCN
Clinical Nurse Specialist
Nancy H. Jacobson, RN, CS, MSN Albert Einstein Medical Center
Senior Manager Philadelphia, Pennsylvania
The Whitman Group
Huntington Valley, Pennsylvania Virginia R. Wilson, MSN, RN, CEN
Assistant Professor, Graduate Nursing Programs
Charlotte D. Kain, EdD, RN, C Drexel University College of Nursing and Health Professions
Professor Nursing, Health Care of Women Philadelphia, Pennsylvania
Montgomery County Community College
Blue Bell, Pennsylvania
TEST ITEM WRITERS
Roseann Tirotta Kaplan, MSN, RN, CS Frances Amorim, MSN, RN
Adjunct Faculty Clinical Lecturer
Drexel University College of Nursing and Health Professions University of Pennsylvania, School of Nursing
Philadelphia, Pennsylvania Philadelphia. Pennsylvania
Constance O. Kolva Taylor, MSN, RN Eileen B. Augente, MA, RN, CDE
Kolva Consulting Instructor, Long Island University
Harrisburg, Pennsylvania Brooklyn, New York
Clinical Nurse Specialist, Diabetes
Mary Lou Manning, PhD, CPNP, RN Amityville, New York
Director, Infection Control and Occupational Health
The Childrens Hospital of Philadelphia Pamela Bellefeuille, RN, CS, CEN, TRNCCP, MN
Adjunct Assistant Professor Clinical Nurse Specialist, Manager
University of Pennsylvania School of Nursing Emergency Department, Kaiser
Philadelphia, Pennsylvania Santa Rosa, California

xii CONTRIBUTORS
53155_00_FM_pi-xxii.qxd 3/2/09 10:30 PM Page xiii

Jean E. Berry, MSN, RN Nancy Kirk, MSN, RN


Assistant Professor Lead Instructor, Medical Surgical Nursing and
Bloomsburg University Pharmacology
Bloomsburg, Pennsylvania The Health Institute of Tampa Bay
St. Petersburg, Florida
Janice Caie-Lawrence, MSN, RN
Assistant Professor Kathleen I. Marchiondo, MSN, RN
University of Detroit Mercy Assistant Professor
Detroit, Michigan Research College of Nursing
Kansas City, Missouri
Susan Chaney, MSE, RN, C
Staff Nurse, Harbor View Mercy Hospital Cheryl Martin, MSN, RN, C
Psychiatric Division of St. Edward Mercy Associate Professor of Nursing
Medical Center Bethel College
Fort Smith, Arkansas Mishawaka, Indiana

Jane A. Claffy, MSN, RN, C Patricia A. Middlemiss, MSN, RN


St. Vincents Hospital School of Nursing Assistant Professor of Nursing
New York City, New York Gwynedd-Mercy College
Gwynedd Valley, Pennsylvania
H. Allethaire Cullen, RNP, CS, MSN, CEN
Director of Education Anna P. Moore, MS, RN
Education Empowerment Assistant Professor
Coventry, Rhode Island Southside Regional Medical Center
School of Nursing
Janet Curley, EdD(C), RN, CEN Petersburg, Virginia
Adjunct Lecturer
Pace University Patricia A. Mynaugh, PhD, RN
New York, New York Associate Professor
Villanova University College of Nursing
Sandra H. Faria, DSN, RN Villanova, Pennsylvania
Florida State University School of Nursing
Tallahassee, Florida Lynda C. Opdyke, MSN, RN
Facilitator, Academic Affairs
Elizabeth A. Fisk, MS, RN Mercy School of Nursing
Associate Professor Charlotte, North Carolina
Fitchburg State College
Fitchburg, Massachusetts Joan Opelia, MSN, RN
Consultant
Judith M. Hall, RNC, MSN, IBCLC, LCCE, FACCE Nursing Tutorial and Consulting Services
Lactation Consultant and Childbirth Educator Clifton, Virginia
Mary Washington Hospital
Fredericksburg, Virginia Deborah L. Panning, MSN, RN, C
Unit Manager
Shirley Sullivan Hall, MSN, RN Brandywine Hospital School of Nursing
Assistant Professor Coatesville, Pennsylvania
School of Nursing Hampton University
Hampton, Virginia Carol Park, MSN, RN, CS
School of Nursing
Janice L. Hinkle, MSN, RN, CNRN Sentara Norfolk General Hospital
Instructor Norfolk, Virginia
Thomas Jefferson University
Philadelphia, Pennsylvania Mary Reuland, MS, RN
College of St. Catherine
Susan Jaskowski, MSN, CCRN, CRNP, RN Minneapolis Campus
Pediatric Neurosurgery Nurse Practitioner Minneapolis, Minnesota
Penn State University Hershey Medical Center
Hershey, Pennsylvania

CONTRIBUTORS xiii
53155_00_FM_pi-xxii.qxd 3/2/09 10:30 PM Page xiv

Kathy Rodgers, MSN, RN, CNS, CCRN, CEN Wanda May Webb, MSN, RN, CS
Critical Care Clinical Nurse Specialist Level Coordinator
St. Elizabeth Hospital Brandywine School of Nursing
Beaumont, Texas Coatesville, Pennsylvania

Mary Jean Ricci, MSN, RN, C Mary Jo Westien, MS, APRN, CS


Holy Family College Brigham Young University
Philadelphia, Pennsylvania Provo, Utah

Nancy G. Runton, MSN, CPNP Diane Wieland, MSN, RN, CS


Pediatric Nurse Practitioner Assistant Professor
Virginia Pediatric Group, LTD Thomas Jefferson University
Fairfax, Virginia College of Allied Health Sciences
Department of Nursing
Linda F. Samson, PhD, RNC, CNAA Philadelphia, Pennsylvania
Dean, School of Health Sciences
Clayton State College Judy Winterhalter, DNSc, RN, CS
Morrow, Georgia Associate Professor
Gwynedd-Mercy College
Fran Schuda, MSN, RN, CCRN Gwynedd Valley, Pennsylvania
Director, Training and Development Psychotherapist
Episcopal Hospital North Penn Counseling Center
Philadelphia, Pennsylvania Lansdale, Pennsylvania

Ruth Schumacher, MSN, RN Evelyn Wolynies, MSN, RN


Lecturer, Department of Maternal-Child Nursing Clinical Nurse Specialist
College of Nursing Neuropsych Huntingtons Disease
The University of Illinois at Chicago University of Medicine and Dentistry of New Jersey
Chicago, Illinois Cooper Hospital
Camden, New Jersey
Cheryl J. Smally, MSN, MSS, RN
St. Lukes College of Nursing and Health Sciences Catherine Y. Zion, MS, RN
Sioux City, Iowa Clinical Instructor
Macon College School of Nursing
Lorry L. Smith, MSN, RN Macon, Georgia
Clinical instructor, Baccalaureate Nursing
Eastern Kentucky University REVIEWERS
Richmond, Kentucky
Judy Bourrand, MSN, RN
Diane L. Spatz, PhD, RN, C Ida V. Moffett School of Nursing
University of Pennsylvania Samford University
Philadelphia, Pennsylvania Birmingham, Alabama

Martha L. Tanicala, MSN, RN, CPN Mary Kathie Doyle, BS, CCRN
Instructor Instructor
St. Vincent Medical Center School of Nursing Maria College
Toledo, Ohio Troy, New York

Susanne M. Tracy, MN, MA, RN Mary Lashley, PhD, RN, CS


Associate Professor Associate Professor
RivierSt. Joseph School of Nursing Towson University
Nashua, New Hampshire Towson, Maryland

Rosemarie Trouton, MSN, RN Melissa Lickteig, EdD, RN


Clinical Nurse Educator, Pediatrics Instructor, School of Nursing
Hershey Medical Center Georgia Southern University
Hershey. Pennsylvania Statesboro, Georgia

xiv CONTRIBUTORS
53155_00_FM_pi-xxii.qxd 3/2/09 10:30 PM Page xv

Darlene Mathis, MSN, RN APRN, BC, NP-C, CRNP Carol Meadows, MNSc, RNP, APN
Assistant Professor Eleanor Mann School of Nursing
Ida V. Moffett School of Nursing University of Arkansas
Samford University Fayetteville, Arkansas
Birmingham, Alabama
Maria Smith, DSN, RN, CCRN
Barbara McGraw, MSN, RN Professor, School of Nursing
Instructor Middle Tennessee State University
Central Community College Murfreesboro, Tennessee
Grand Island, Nebraska

CONTRIBUTORS xv
53155_00_FM_pi-xxii.qxd 3/2/09 10:30 PM Page xvi
53155_00_FM_pi-xxii.qxd 3/2/09 10:30 PM Page xvii

P R E FA C E

Endorsed by the National Student Nurses Association Test-Taking Software


(NSNA), the NCLEX-RN Review, Sixth Edition has
been developed expressly to meet your needs as you The CD-ROM included with this text holds a pool of
study and prepare for the all-important NCLEX-RN over 3000 questions (2000 new, 1100 from text) in an
licensure examination. Taking this exam is always a environment that simulates the test taking experience.
stressful event in the best of circumstances; it Tests are downloaded in varying lengths just like the
constitutes a major career milestone and NCLEX actual exam. You can test your knowledge and test
success is the key to your future ability to practice as a taking skills in two ways: learning and test modes. In
registered nurse. learning mode, the rationale for correct and incorrect
responses are immediately given after each question.
In test mode, you will receive a score after completing
NEW TO THIS EDITION a test. Questions answered incorrectly may be
The NCLEX-RN Review, Sixth Edition has been revised reviewed after completing the exam.
to meet the standards set by the National Councils In either mode, once the test is completed, you have
State Boards of Nursings (NCSBN) most current test the option to view and print the results displayed as
plan. More than 3000 unique and challenging NCLEX bar-graph percentages that represent the areas of the
questions have been included. Additionally, each test plan, cognitive levels, subject area, and nursing
question is followed by a comprehensive rationale for process. This element gives you a clear and concise
each answer given as well as the identifying areas of visual presentation of the results that further enhance
cognitive level, client need, nursing process, and and maximize study time.
subject area. Additional emphasis on pharmacology
and delegation in the form of 500 new questions Free PDA Downloads
has been added throughout the practice material Practice for the exam on-the-go with PDA portability!
since these areas have a greater emphasis on the Downloads can be accessed from the CD-ROM
actual exam. providing over 500 practice questions with rationales.
Alternative format questions are throughout the Practice tests are downloaded in tests of varying
practices tests and include the newest addition to the lengths just like the actual examination. Test and
exam: the charting question. These new questions learning modes are available in both Windows and
have been added to the test-taking software included Palms operation systems.
with the book.
New content currently being tested on the exam is Organization, Content, and Features
included in the text such as conscious sedation,
complementary and alternative medicine (CAM), and Unit 1, Preparing for the NCLEX Examination is an
herbal medicines. introductory unit that covers:
More user-friendly charts and images have been Explanation of the test plan
included throughout the review content, followed by Test construction
diet and nutrition appendices and eleven 100-question How computerized adaptive testing (CAT) works
practice tests. Study tips and techniques

xvii
53155_00_FM_pi-xxii.qxd 3/2/09 10:30 PM Page xviii

Unit 2, Drugs and Nursing Implications groups The Nursing Process integrated with a body
drugs by classifications and similarities to help you in systems approach:
consolidating this important but sometimes Assessment: review of both history and
overwhelming information. Unit 2 includes: physical examination
Drug classification prototypes Analysis: includes appropriate NANDA nursing
Related drug variances from the prototype diagnoses
Drug action mechanisms Planning: discusses client goals
Drug uses and adverse effects Implementation: identifies the interventions to
Nursing implications and discharge teaching achieve client goals
Unit 3, Universal Principles of Nursing Care and Evaluation: lists outcome criteria
Management includes: Review of the pertinent disorders for each system
Nursing practice standards that includes:
Legal and ethical aspects of nursing General characteristics
Delegation Pathophysiology
Prioritization Psychopathology
Coordinating the health care team and client care Medical/surgical management
Units 4 through 7 cover adult, pediatric, maternity, Assessment data
and psychiatric-mental health nursing. Each of these Nursing interventions and client education
units covers a systematic approach to review the The concept, scope, and design of this text represent
subject matter: the commitment of the author and publishing team to
Introductory review of anatomy and physiology help the graduate nurse reach full professional potential.
along with basic theories and principles Good luck on your NCLEX-RN examination!

xviii PREFACE
53155_00_FM_pi-xxii.qxd 3/2/09 10:30 PM Page xix

LI ST O F

A B B R E V I AT I O N S

AA Alcoholics Anonymous BUN blood urea nitrogen


ABGs arterial blood gases CABG coronary artery bypass graft
ABE acute bacterial endocarditis C Celsius
ac before meals Ca calcium
ACOA Adult Children of Alcoholics CAD coronary artery disease
ACE angiotensin-converting enzyme CAT computerized adaptive testing
ACh acetylcholine CBC complete blood count
ACTH adrenocorticotropic hormone CCK-PZ cholecystokinin and pancreozymin
ADA American Dietetic Association CCU coronary care unit
ADH antidiuretic hormone CDC Centers for Disease Control and Prevention
ADL activities of daily living CEA carcinoembryonic antigen
AFB acid-fast bacillus CF cystic fibrosis
AIDS acquired immune deficiency syndrome CHD congenital heart disease
AKA above the knee amputation CHF congestive heart failure
ALG antilymphocytic globulin CHO carbohydrate
ALL acute lymphocytic leukemia CI chloride
ALT alanine aminotransferase CL cognitive level
An analysis cm centimeter
ANA American Nurses Association or antinuclear CN client need
antibodies CNM certified nurse midwife
ANLL acute nonlymphocytic leukemia CNS central nervous system
ANS autonomic nervous system Co comprehension
Ap application CO2 carbon dioxide
A-P anterior-posterior COPD chronic obstructive pulmonary disease
APTT activated partial thromboplastin time CP cerebral palsy
ARC AIDS-related complex CPAP continuous positive airway pressure
ARDS adult respiratory distress syndrome CPD cephalopelvic disproportion
As assessment CPK creatine phosphokinase
ASA acetylsalicyclic acid (aspirin) CPR cardiopulmonary resuscitation
ASD atrial septal defect C&S culture and sensitivity
ASO antistreptolysin CSF cerebrospinal fluid
AST aspartate aminotransferase CST contraction stress test
ATG antithymocytic globulin CT computed tomography
ATN acute tubular necrosis CTZ chemoreceptor trigger zone
ATP adenosine triphosphate CV cardiovascular
AV atrial-ventricular CVA cerebrovascular accident
BCG Bacillus Calmette-Gurin CVP central venous pressure
BID twice a day CVS chorionic villi sampling
BKA below the knee amputation D&C dilatation and currettage
BMR basal metabolic rate DDAVP desmopressin
B&O suppositories containing belladonna DDST Denver Developmental Screening Test
BP blood pressure DES diethylstilbestrol
BPD bronchopulmonary dysplasia DIC disseminated intravascular coagulation
BPH benign prostatic hypertrophy dL deciliter
BSE breast self-examination DMT a hallucinogen

xix
53155_00_FM_pi-xxii.qxd 3/2/09 10:30 PM Page xx

DNA deoxyribonucleic acid ICP intracranial pressure


DPT diphtheria, pertussis, and tetanus toxoid ICU intensive care unit
dr dram ID identification
DT diphtheria and tetanus toxoid IDDM insulin-dependent diabetes mellitus
DTs delirium tremens IDM infant of diabetic mother
DTaP diphtheria-tetanus-acellular pertussis vaccine IgG immunoglobulin G
DTP diphtheria, tetanus, and pertussis toxoid IM intramuscular
DVT deep venous thrombosis Im implementation
ECG electrocardiogram IMV intermittent mandatory ventilation
ECT electroconvulsive therapy in inch
ED Emergency department IPPB intermittent positive pressure breathing
EDC estimated date of confinement IQ Intelligence quotient
EEG electroencephalogram ISG immune serum globulin
EMG electromyography ITP idiopathic thrombocytopenic purpura
ENT ear, nose, throat IUD intrauterine device
EP erythrocyte protoporphyrin IUGR intrauterine growth retardation
ERT estrogen replacement therapy IV intravenous
ESR erythrocyte sedimentation rate IVP intravenous pyelogram
ETOH ethyl alcohol JRA juvenile rheumatoid arthritis
Ev evaluation K potassium or knowledge (in question code for
fl dr fluid dram comprehensive practice tests)
fl oz fluid ounce kcal kilocalories
F Fahrenheit KCL potassium chloride
FAD flavin adenine dinucleotide kg kilogram
FDA Food and Drug Administration KUB kidney, ureter, bladder
FHT fetal heart ones L liter
FHR fetal heart rate lb pound
FSH follicle-stimulating hormone LDH lactic dehydrogenase
FSP fibrin split products LE lupus erythematosus
ft feet LGA large for gestational age
FTT failure to thrive LH luteinizing hormone
g gauge or gram LOA left occiput anterior
GI gastrointestinal LOC level of consciousness
gr grain LOP left occiput posterior
gtt(s) drop(s) LP lumbar puncture
GTT glucose tolerance test LPN licensed practical nurse
GU genitourinary L/S lecithin/sphingomyelin
GVHD graft versus host disease LSD lysergic acid diethylamide
h hour LVN licensed vocational nurse
HA headache m meter or minim
H2 histamine 2 min minim or minutes
HBIG hepatitis B immunoglobulin MAO monamine oxidase
HBV hepatitis B vaccine MAOI monamine oxidase inhibitors
HCG human chorionic gonadotropin MAR medication administration record
HGI hydrochloric acid or hydrochloride mcg microgram
HCO3 bicarbonate MCT medium chain triglycerides
HCS/HPL human chorionic somatomammotropin/ MD medical doctor
human placental lactogen mEq milliequivalent
He health promotion/maintenance mg milligram
HELLP hemolysis, elevated liver enzymes, lowered MI myocardial infarction
platelets mL milliliter
Hct hematocrit MLC mixed leukocyte culture
Hg mercury mm millimeter
Hgb hemoglobin MMR measles, mumps, rubella
HbA1c hemoglobin A1c MRI magnetic resonance imaging
HgbS abnormal hemoglobin seen in sickle-cell MSH melanocyte-stimulating hormone
anemia mU milliunit
Hib Haemophilus influonzae type B Na sodium
HIV human immunodeficiency virus NANDA North American Nursing Diagnosis
HLA human leukocyte antigen Association
HMD hyaline membrane disease NEC necrotizing enterocolitis
HNP herniated nucleus pulposus ng nanograms
H2O water NG nasogastric
H2O2 hydrogen peroxide NIDDM non-insulin-dependent diabetes mellitus
hr(s) hour(s) NP nursing process
HSV2 herpes simplex virus type 2 NPO nothing by mouth
I&O intake and output NS normal saline
I & Os intake and outputs NSAIDs nonsteroidal anti-inflammatory drugs
IA intra-arterial NSS normal saline solution

xx LIST OF ABBREVIATIONS
53155_00_FM_pi-xxii.qxd 3/2/09 10:30 PM Page xxi

NST nonstress test R respirations


O2 oxygen RA rheumatoid arthritis
OB-Gyn obstetrics-gynecology RAIU radioactive iodine uptake
OBS organic brain syndrome RBC red blood cell
OCD obsessive-compulsive disorder RDA recommended daily allowances
OCT oxytocin challenge test RDS respiratory distress syndrome
OD right eye RF rheumatic fever
OMD organic mental disorder RIA radioimmunoassay
OOB out of bed RN registered nurse
OPV oral polio vaccine RNA ribonucleic acid
OR operating room ROA right occiput anterior
OS left eye ROM range of motion
OTC over-the-counter ROP right occiput posterior
OU both eyes S3 third heart sound
oz ounce SA sinoatrial
P pulse Sa safe, effective care environment
PA pulmonary artery SBE subacute bacterial endocarditis
PABA para-aminobenzoic acid Sub-Q subcutaneous
PACU postanesthesia care unit SGA small for gestational age
PAP pulmonary artery pressure SGOT serum glutamic-oxylacetic transaminase
Pap Papanicolaou SGPT serum glutamic-pyruvic transaminase
PCA patient-controlled analgesia SIDS sudden infant death syndrome
pCO2 partial pressure of carbon dioxide SL sublingual
PCP Pneumocystis carinii pneumonia or SLE systemic lupus erythematosus
phencyclidine STD sexually transmitted disease
PCWP pulmonary capillary wedge pressure T temperature or thoracic
PDA patent ductus arteriosus TB tuberculosis
PEEP positive-end expiratory pressure TBI total body irradiation
PG phosphatidylglycerol tbsp tablespoon
PGE2 prostaglandin E2 TCAs tricyclic antidepressants
Ph physiological integrity Td adult tetanus toxoid and diphtheria toxoid
PICCs peripherally implanted central catheters TEF tracheoesophageal fistula
PID pelvic inflammatory disease TENS transcutaneous electrical nerve stimulator
PIH pregnancy-induced hypertension TET tetralogy
PKU phenylketonuria THC tetrahydrocannabinol
Pl planning TIA transient ischemic attack
PMI point of maximal impulse TID three times a day
PND paroxysmal nocturnal dyspnea TPN total parenteral nutrition
PNS parasympathetic nervous system or TPP thiamin pyrophosphate
peripheral nervous system TSE testicular self-examination
PO by mouth TSH thyroid-stimulating hormone
PO2 partial pressure of oxygen tsp teaspoon
post-op postoperative (after surgery) TUR transurethral resection
PPD purified protein derivative TURP transurethral prostatectomy
PPN peripheral parenteral nutrition ug microgram
pre-op preoperative UC ulcerative colitis
prep preparation URI upper respiratory infection
PRN as needed UTI urinary tract infection
Ps psychosocial integrity VBAC vaginal birth after cesarean
PSA prostate-specific antigen VDRL Venereal Disease Reactive Laboratory
PT prothrombin time VER visual evoked response
PTCA percutaneous transluminal coronary VF ventricular fibrillation
angioplasty VMA vanillylmandelic acid
PTH parathormone VS vital signs
PTSD post-traumatic stress disorder VSD ventricular septal defect
PTT partial thromboplastin time VT ventricular tachycardia
PTU propylthiouracil WBC white blood count or white blood cell
PUBS percutaneous umbilical blood sampling WBCs white blood cells
PUC pediatric urine collector wk week
PVC premature ventricular contraction or WNL within normal limits
polyvinyl chloride < less than
PVD peripheral vascular disease > greater than
q every
QID four times a day

LIST OF ABBREVIATIONS xxi


53155_00_FM_pi-xxii.qxd 3/2/09 10:30 PM Page xxii
53155_01_Ch01_p001-006.qxd 2/21/09 9:47 AM Page 1

U N I T 1

P R E PA R I N G F O R
THE NCLEX
E X A M I N AT I O N

This first unit of the NCLEX-RN Review will provide you with UNIT OUTLINE
the important information you need to know about the 2 Understanding the NCLEX
construction of the National Council Licensure Examination for Examination
Registered Nurses (NCLEX-RN, often referred to as state 3 Preparation and Test Taking
boards), with tips on how to study and with test-taking
techniques you can use to improve your success when writing
the examination.

1
53155_01_Ch01_p001-006.qxd 2/21/09 9:47 AM Page 2

Understanding the NCLEX Examination

THE TEST PLAN not limited to, abuse/neglect, behavioral


interventions, cultural diversity, end of life, grief and
The NCLEX-RN examination questions are based on a loss, psychopathology sensory/perceptual alterations,
test plan of client needs with concepts and processes stress management, and therapeutic communications.
fundamental to the practice of nursing integrated D. Physiological Integrity
throughout the categories. These categories are nursing The fourth major category has four subcategories:
process, caring, communication and documentation, basic care and comfort, pharmacological and
and teaching and learning. The latest plan went into parenteral therapies, reduction of risk potential,
effect in April 2007. and physiological adaptation. These subcategories
account for 5367% of test items and measure the
nurses ability to promote physical health and
Categories of Client Needs wellness by providing care and comfort, reducing
The health care needs of clients across the life span in risk potential, and managing health alterations.
a variety of settings are grouped under four broad 1. Basic care and comfort comprises 612% of test
categories, some with several subcategories. The four items and includes, but is not limited to, alter-
main categories are safe, effective care environment; native and complementary therapies elimination,
health promotion and maintenance; psychosocial mobility/immobility, and rest and sleep.
integrity; and physiological integrity. 2. Pharmacological and parenteral therapies
A. Safe, Effective Care Environment comprises 1319% of test items and includes,
This category has two subcategories: management but is not limited to, central venous access
of care and safety and infection control. These two devices, dosage calculation, intravenous
subcategories account for 2133% of test items, therapy, and medication administration.
which measure the nurses ability to provide and 3. Reduction of risk potential comprises 1319%
direct nursing care that enhances the care delivery of test items and includes, but is not limited
setting in order to protect clients and their families/ to, diagnostic tests, laboratory values,
significant others and other health care personnel. monitoring conscious sedation, and
1. Management of care comprises 1319% of test therapeutic procedures.
items. This includes, but is not limited to, 4. Physiological adaptation comprises 1117% of
advocacy, client rights, confidentiality, test items and includes, but is not limited to,
delegation, ethical practice, legal rights and fluid and electrolyte imbalances,
responsibilities, and supervision. hemodynamics, pathophysiology, and
2. Safety and infection control comprises 814% radiation therapy.
of test items. This includes, but is not limited
to, disaster planning, emergency response Levels of Cognitive Ability
plan, and error prevention.
B. Health Promotion and Maintenance The practice of nursing requires application of
The second major category comprises 612% of knowledge, skills, and abilities. To assess the
test items and measures the nurses ability to candidates ability in these areas, a majority of the test
provide and direct nursing care of clients and their items are written at the application or higher levels of
families/significant others that incorporates the cognitive ability, which requires more complex
knowledge of expected growth and development thought processing. Blooms taxonomy for the
principles, prevention, and/or early detection of cognitive domain serves as a basis for writing and
health problems and strategies to achieve optimum coding the test items.
health. This includes, but is not limited to, the
aging process, ante/intra/postpartum and newborn
care, developmental stages and transitions, growth HOW THE TEST IS CONSTRUCTED
and development, health and wellness, self-care,
and techniques of physical assessment. A. The National Council of State Boards of Nursing
C. Psychosocial Integrity Inc. is the central organization for the independent
The third category comprises 612% of test items and member boards of nursing, which includes the
measures the nurses ability to provide and direct care 50 states, the District of Columbia, Guam, and the
that promotes and supports the emotional, mental, Virgin Islands. The member boards are divided
and social well-being of the families/significant others into four regional areas, which supervise the

1
experiencing stressful events as well as clients with selection of test item writers (representing
acute or chronic mental illness. This includes, but is educators and clinicians), whose names are

2 NCLEX-RN Review
53155_01_Ch01_p001-006.qxd 2/21/09 9:47 AM Page 3

suggested by the individual state boards of D. The exam is given at Pearson Professional Testing
nursing. This provides for regional representation Centers across the United States. The candidate
in the testing of nursing practice. All test items are submits credentials to the State Board of Nursing
validated in at least two approved nursing in the state in which licensure is desired. Once the
textbooks or references. credentials are accepted, the candidate calls the
B. The National Council contracts with a professional testing service for an appointment, which will be
testing service to supervise writing and validation scheduled within 30 days.
of test items by the item writers. This professional
service works closely with the Examination
Committee of the National Council in the test HOW THE TEST IS SCORED
development process. The National Council and
the state boards are responsible for the A. The NCLEX-RN is scored by computer and a
administration and security of the test. pass/fail grade is reported.
C. The exam is a computer exam known as CAT, B. A criterion-referenced approach is used to set the
which stands for Computerized Adaptive Testing. passing score. This provides for the candidates
The exam is taken on a computer utilizing test performance to be compared with a consistent
state-of-the-art technology. standard of criteria. Passing the exam will
There are several formats for questions. Multiple- determine if the candidate is safe to practice as an
choice questions with four choices, single-answer entry-level nurse by using critical thinking skills to
items or ones that require more than one response. make nursing judgements.
There may be fill-in-the-blank questions or ones
that ask the test taker to identify the area on a
picture or a graphic, or drag-and-drop. The
computer screen displays the question and the
HOW CANDIDATES ARE
answer choices. There may also be questions that NOTIFIED OF RESULTS
require responses to be placed in priority order.
Each of these types of questions are integrated A. Candidates in the following states may access their
throughout the sample tests in this book. unofficial results within two days via the NCLEX
Each candidate is oriented to the computer Candidate Web site or from the NCLEX-RN. Quick
before the exam starts. Because the exam is geared Results line: Arizona, Colorado, Connecticut,
to the candidates skill level, each candidate will District of Columbia, Florida, Georgia, Illinois,
have a unique exam. Each exam will include Idaho, Indiana, Iowa, Kansas, Kentucky, Louisiana,
approximately 15 experimental questions dispersed Maine, Maryland, Massachusetts, Michigan,
throughout the exam, so the candidate will be able Minnesota, Missouri, Montana, Nebraska, Nevada,
to answer all the questions with equal effort. The New Jersey, New Mexico, New York, North
experimental questions will not be counted for or Carolina, North Dakota, Ohio, Oklahoma, Oregon,
against the candidate. Some candidates will be Pennsylvania, South Carolina, South Dakota,
finished in a little over an hour; others will use the Tennessee, Texas, Utah, Vermont, Washington,
entire allotted time. The minimum number of Wisconsin, Wyoming.
questions candidates must answer is 75, and the Website: www.pearsonvue.com/nclex
maximum they may answer is 265. There is not a NCLEX Quick Results line: 1-900-776-2539.
time limit for each question, but a 6 hour limit for B. Unsuccessful candidates are provided with a
the entire exam, which includes the exam instruc- diagnostic profile that describes their overall
tions explaining how to use the mouse, the space performance on a scale from low to high, and their
bar, and the enter key; samples representing each performance on the questions testing their abilities
type of question in the exam; and rest breaks. to meet client needs.

Preparation and Test Taking

USING THE TEST PLAN TO YOUR identify your knowledge base in relation to the
information provided in the test plan.
BEST ADVANTAGE A. Look carefully at the elements of the test plan
(Categories of Client Needs), which are also
Performing a Self-Needs Analysis reported to those who fail the test.

1
The first step to take when preparing to study for the
NCLEX-RN is to perform a self-needs analysis to

PREPARATION FOR THE NCLEX EXAMINATION 3


53155_01_Ch01_p001-006.qxd 2/21/09 9:47 AM Page 4

B. Go through your notes and text references. Select promote retention of information being studied.
what is important and star, underline, or highlight Concentrate on the information you identified in
this information. your self-needs analysis as needing to be learned.
C. Categorize this information in terms of material F. The final step of your study program involves
that needs to be learned or material that needs organizing the material so that you will be able to
only to be reviewed. learn all the need to learn and review all the
need to review information within the allotted
study time period. Your schedule should have
Planning for Study allowed you to complete your review so you can
A. Look at the period of time available to you for close your books and do something relaxing on the
study between now and when you are scheduled night before the examination.
to take the NCLEX-RN. Ideally, plan to study up
to four nights before the test, allow three nights
for review, and the night before the test for FINAL PREPARATION FOR
relaxation. If you have limited time for study,
plan your time so that you have at least one night TEST TAKING
for nothing but review.
B. Identify your maximum concentration time for In addition to having studied appropriately to assure
profitable study. It is better to block out short yourself of a good knowledge base, there are measures
periods of time (4560 minutes, interspersed with you can take to be in prime physiologic and
planned breaks) that can be quality study time, psychologic shape for writing the examination.
rather than setting aside 3 hours of time to study,
which may only produce 90 minutes of quality Physiologic Readiness
study time.
C. When you decide what your maximum time for To prime yourself physiologically, you should meet
profitable study is, then that is the block of time your own needs for nutrition, sleep, and comfort.
you should set aside on a regular basis for study A. You will function best if you are well nourished.
purposes. Within the confines of your allocated 1. Plan to eat three well-balanced meals a day for
study time, make sure you establish a schedule at least 3 days prior to the examination.
that permits you to cover completely all the 2. Be careful when choosing the food you
material to be learned. consume within 24 hours of the examination.
D. Nursing research has shown that reviewing more a. Avoid foods that will make you thirsty or
than 5,000 questions before sitting for the exam cause intestinal distress.
produces greater success rates of passing. b. Minimize the potential of a full bladder
midway through the examination by
limiting the amount of fluids you drink
How to Study and by allowing sufficient time at the test
A. To promote maximum concentration, ensure that site to use the bathroom before entering
your study materials are your prime area of focus. the room.
B. Make sure you are mentally alert and in a room B. Assess your sleep needs.
where you will be free from outside interruptions. 1. Determine the minimal amount of sleep you
If possible, choose a room with no telephone. need in order to function effectively.
C. Do not smoke, do not nibble on snacks, and do not 2. Plan to allow sufficient time in your schedule
answer the telephone. This will allow you to direct the week before the examination to provide
your energy to the study activity. yourself with the minimum sleep you need to
D. Proceed with your planned study periods in an function effectively for at least 3 days prior to
organized manner by choosing an approach that the examination.
will be meaningful to you. Some content lends C. Plan your wardrobe ahead of time.
itself to study using concepts, while other content 1. Shoes and clothes that fit you comfortably will
is best studied using systems. not distract your thought processes during the
E. Use methods of memory improvement that will examination.
work for you. Mnemonic devices (where a letter 2. Include a comfortable sweater.
represents the first letter of each item in a 3. Your clothes for the test day should be ready
sequence) are an effective means of retrieving to wear by the night before the examination.
material. Mental imagery is the technique of D. If you wear glasses or contact lenses, take along an
forming pictures in your mind to help you extra pair of glasses.
remember details of the sequence of events, such as E. If you are taking medications on a regular basis,
the administration of an injection. Try practicing continue to do so during this period of time.
self-recitation to improve your study habits. Introduction of new medications should be

1
Reciting to yourself the material being learned will avoided until after completion of the examination.

4 NCLEX-RN Review
53155_01_Ch01_p001-006.qxd 2/21/09 9:47 AM Page 5

Reducing Psychologic Stress A. Take your seat and give yourself an opportunity to
implement the method of relaxation you have been
While a certain amount of anxiety will stimulate your practicing.
nervous system to focus keenly on the examination, B. Read the directions carefully, and then be sure to
excess anxiety will interfere with your ability to follow them carefully.
concentrate on the examination and, indeed, hinder C. Plan to manage your time effectively. While taking
your success. You must approach the examination the CAT test, work steadily. You do not have to
with a positive attitude. You have graduated from a answer a specific number of questions in a given
school of nursing that has prepared you to provide time period. If you take the maximum length of
safe and effective nursing care to your clients. Trust time, your score will reflect the number of
that the curriculum in your school of nursing was questions you have completed.
designed to include all the important concepts and D. Read the stem of the question carefully. This is the
principles necessary for safe nursing practice. Feel part of the question that describes what is being
confident that you accessed multiple resources to asked.
allow you to learn the content. Most of the tests you E. Read the stem a second time to key in on
wrote while in school were developed in the style important words and then reword the question to
used for the NCLEX-RN. Keeping these points in determine the purpose of the question.
mind will enable you to approach the examination F. Move to the answer choices. In a single multiple-
with a positive frame of reference for success. choice item there will be one correct and three
Minimize the anxiety-producing situations related incorrect choices. Incorrect answers are called
to writing the examination by carefully planning your distractors. A multiple-choice item that has more
pre-examination activities. Make a list of the than one correct answer may have fewer distractors.
important things you need to accomplish. G. Consider if the question is asking about
A. Rehearse the route or means of transportation you 1) a needed assessment that should be done first,
plan to take to the test location, preferably at the 2) Maslows Hierarchy of Needs, or 3) a safety
same time of the day on which you actually will be issue. Keep these in mind for each question.
going. Check your local resources for road H. Carefully evaluate the answer choices for key
conditions that might necessitate altering your words. Be sure to appreciate the universality of
planned route. In your time assessment, include words such as each, all, never, and none; the
parking your car, locating where you are to report limitations of words such as rarely, most, and
for registration, and locating the bathrooms. To least; and the latitude offered by words such as
ensure adequate travel time and to minimize stress usually, frequently, and often.
related to getting to the test site on time the I. Read each option twice. Use the space bar on the
morning of the test, add an extra 30 minutes to the keyboard to highlight each answer choice.
total time needed for the rehearsal run. J. Answer it by saying to yourself
B. Have your admission materials readily available. 1. Yes, it answers what is being asked.
C. If you are staying overnight near the test site, be 2. No, it does not answer what is being asked.
sure you pack everything you will need. Before 3. Maybe it answers what is being asked.
retiring for the night, make your rehearsal run to K. Use this procedure for all the answer choices.
the test location in preparation for the next day. When you first read the question, if an obvious
D. Plan to use relaxation exercises to control your answer comes to mind, restrain your desire to
anxiety level. If you have been using a specific look for it in the answer choices. For a single
method of relaxation successfully, then continue multiple-choice item, read all the choices to
using it during this period of time. If you have not, make sure your thought was indeed the only
consider trying one of the following. yes answer. For this type of question, if you are
1. Yoga or meditation before the exam fortunate enough to have only one yes answer,
2. Guided imagery: requires using your then you have eliminated the three distractors.
imagination to create a relaxing sensory scene For a multiple multiple-choice item, you likely
on which to concentrate. have the correct answers and have eliminated
3. Breathing exercises. the other distractors.
E. For any of the methods to achieve the desired L. If you identified more than one yes option for a
results, you must be willing to commit the time single multiple-choice item, then evaluate those
necessary to implement their prescribed protocols. other options in terms of which is more yes than
maybe. If you have no yes answer, then evaluate the
maybe choices for one that leans more toward yes.
TAKING THE TEST M. Always choose the answer that has the highest
likelihood for being yes (correct). Look critically
While having a good knowledge base is important for at the answer choices for clues. If you see choices
success in test-taking situations, the following that are opposites, frequently one is the correct

1
strategies can be used to maximize your skill in answer. For example:
choosing the correct answers.

PREPARATION FOR THE NCLEX EXAMINATION 5


53155_01_Ch01_p001-006.qxd 2/21/09 9:47 AM Page 6

R. Assume you have a doctor's order and all the


During insertion of a central venous catheter, in
equipment or supplies you need. Remember, the
which position should the client be placed?
questions are based on texts and not the real
1. A supine position.
world.
2. Trendelenburgs position.
S. When you have decided on an answer, press the
3. Reverse Trendelenburgs position.
return key. The computer will ask if this is your
4. A high-Fowlers position.
desired answer choice. If it is, then press the enter
key again and your choice will be recorded. If you
Choices 2 and 3 are opposites, and in this case 2 is
wish to review the answer choices again, use the
the correct answer.
space bar to do so and then press the enter key when
N. If you have an answer that contains more than one
you have made your decision. The computer will
option, all of the options must be correct for that
again ask if this is your desired answer choice. If it is,
choice to be correct. If you can eliminate one of the
then press the enter key again to record your choice.
options in an answer, you can automatically
T. A calculator will be accessible on the desktop of
eliminate the other answer choices with that
the computer screen.
option. For example:

What complications can occur from the


administration of TPN? HOW TO USE THIS BOOK
1. Hyperglycemia and hypocalcemia.
2. Hyperglycemia and hyperkalemia. As you go through each unit of this book, use it to
3. Hypoglycemia and hypercalcemia. perform your self-needs analysis and as a basis for
4. Hyperkalemia and hypercalcemia. study. You will make the best use of both your time
and the book if you use the study skills suggested
Hypercalcemia and hyperkalemia are distractors. earlier in this unit.
Eliminate the choices with these options and there A. For more detailed information on the particular
is only one correct choice remaining. subjects you feel need extra study, review those
O. Look for options that do not meet the requirements subjects in current textbooks, and scan the list
of the stem. For example: of suggested readings at the end of each unit for
further resources.
Which medication is used to lyse (break up) B. The questions interspersed throughout Units 2
already formed clots? through 7 are typical of the board questions, and they
1. Warfarin sodium (Coumadin). will help you to become more familiar with the
2. Heparin sodium (Lipo-Hepin). style of questions found in the NCLEX-RN.
3. Streptokinase (Kabikinase). C. When setting up your study time include time to take
4. Vitamin K. the practice tests on the enclosed disk in the back of
this book. You may take these tests once, before you
Choices 1, 2, and 3 are all used in the treatment of start studying, to help you with your needs analysis.
formed clots. Choice 4 is necessary for clot formation When you have completed studying, you may take
and therefore does not meet the requirements for the them again, to evaluate for test performance.
stem and must be eliminated as a possibility. Now D. While taking the practice tests, apply the test-
choose among the remaining options. 3 is correct. taking strategies discussed earlier in this unit.
P. If the stem asks for the exception or which choice
is not the answer, you are looking for the no
answer rather than the yes answer. For example:
REFERENCES AND SUGGESTED READINGS
Benson, H. RelaxationResponse.org. Retrived November 8, 2008,
Individuals from which group would be excluded from http://relaxationresponse.org.
from getting TB? Bloom, B. S., Engelhart, M. D., Furst, E. G., Hill, W. H.,
1. Lower socioeconomic groups. Krathwohl, D. R., et al. (Eds.). (1956). Taxonomy of
2. Malnourished and debilitated individuals. educational objectives: The classification of educational goals.
3. Individuals on steroid therapy. Handbook I, The cognitive domain. New York: David McKay.
4. White females with a history of alcohol abuse. Flynn, P. (1980). Holistic health: The art and science of care.
Bowie, MD: Brady.
Levy, J. (1989). The fine arts of relaxation and meditation.
Choices 1, 2, and 3 are all at risk and therefore, yes New York: Wisdom Publications.
responses. Choice 4 is the no response and, National Counsel of State Boards of Nursing, Inc. (2008).
therefore, the correct answer. NCLEX-RN Examination. Chicago: Author.
Q. Be careful to avoid reading elements into the Wilson, L. O. (2006). Beyond BloomA new Version of the Cognitive
question that are not specifically included in the Taxonomy. Retrieved August 14, 2008, from http://www.uwsp.edu/
stem and answer choices. education/lwilson/curric/newtaxonomy.htm.

1 6 NCLEX-RN Review
53155_02_Ch02_p007-110.qxd 2/21/09 10:48 AM Page 7

U N I T 2

DRUGS AND
NURSING
I M P LI C AT I O N S

Todays nurse needs to have a firm foundation in pharmacology. UNIT OUTLINE


Important aspects of current nursing practice deal with 8 Factors Affecting Drug Action
effectiveness of medications, detecting adverse effects, drug 10 Drug Administration
interactions, and incorporating client teaching concerning drug 23 Central Nervous System Drugs
use. Clients, including pregnant women and nursing mothers, 37 Autonomic Nervous System
should be taught adverse effects of drugs and not to take OTC Drugs
drugs or herbals while on prescribed medication without 46 Drugs Affecting the Endocrine
consulting their physician. System
53 Eye Drugs
This unit, Drugs and Nursing Implications, provides a review of
55 Cardiovascular Drugs
all major drug classifications. The first part of the unit contains
69 Renal Drugs
a review of pharmacokinetics, drug administration, and
72 Respiratory Drugs
calculations. Keep in mind that pediatric drug doses typically are
76 Gastrointestinal Tract Drugs
calculated per kilograms of body weight. Following these
83 Arthritis Drugs
introductory concepts, multiple choice questions for each drug
classification are provided. 85 Antimicrobials
95 Antiviral Agents
Each major drug classification is represented by a prototype 95 Antifungal Agents
drug. A review of action, use, adverse effects, nursing 96 Anthelmintic Agents
implications, discharge teaching (where applicable), and related
97 Antineoplastic Agents
drugs is in chart form when appropriate or otherwise listed with
103 Immunosuppressants
pertinent comments. At times, a prototype drug may not be used
104 Vitamins and Minerals
because one particular drug might not represent a drug group.
106 Herbs and Herbal Health
For example, in the laxative drug group, there is not one single
Products
representative or prototype drug. If no prototype drug is used,
108 Vaccines and Toxoids
this is indicated under the drug classification.
7
53155_02_Ch02_p007-110.qxd 2/21/09 10:48 AM Page 8

Factors Affecting Drug Action

DEFINITION OF A DRUG 2) Constipation may delay drug


absorption, potentially causing
A. According to the Food and Drug Administration toxicity
(FDA), a drug is any substance used to diagnose,
cure, mitigate, treat, or prevent a condition or
disease.
Distribution
B. Drugs come from three main sources: plants (e.g., A. Once in bloodstream, drugs are distributed within
digoxin), animals (e.g., insulin), and synthetic the body. Distribution can take as long as several
chemicals (e.g., meperidine). hours, depending on blood flow (in various areas
C. Most of the drugs used today are synthetic of the body) and cardiac output.
chemicals and are associated with fewer allergic B. Plasma-protein binding
reactions. 1. Medications connect with plasma proteins
(primarily albumin) in vascular system.
2. Strong attachments have a longer period of
FACTORS AFFECTING drug action.
3. Clients with reduced plasma proteins such as
DRUG ACTION in kidney or liver disease could receive a
heightened drug effect.
Absorption C. Volume of distribution
1. Client with edema has an enlarged area in
A. Absorption refers to the time the drug enters the which a drug can be distributed and may need
body until it enters the bloodstream. an increased dose.
B. Many factors affect the rate and amount of 2. Smaller dose may be needed for client with
absorption. dehydration.
1. Dosage form D. Barriers to drug distribution: prevent some
2. Route of administration medications from entering certain body organs.
a. Parenteral: absorption generally rapid 1. Blood-brain barrier
b. Intravenous (IV) and intra-arterial (IA): a. Helps preserve homeostasis in brain.
most rapid absorption b. To pass through this barrier, drug must be
c. Intramuscular (IM) and subcutaneous (SC) lipid soluble and loosely attached to
1) Absorption is relatively fast if given in plasma protein.
an aqueous base but can be delayed if 2. Placental barrier
given in an oil base a. Shields fetus from possibility of adverse
2) Speed of absorption depends on drug effects.
condition of blood flow b. Many substances (drugs, nicotine, alcohol)
3) Impaired peripheral circulation and do cross placental barrier.
shock will delay absorption E. Obesity: body weight plays a role in drug
d. Intradermal: absorption is slow and distribution because blood flows through fat
confined to area injected (e.g., purified slowly, thus increasing time before drug is
protein derivativePPD) released.
e. Oral: rate and degree of absorption can F. Receptor combination
vary depending on GI motility, presence of 1. A receptor is an area on a cell where drug
food in stomach, gastric pH, and use of attaches and response takes place.
other drugs a. Receptor is usually protein or nucleic
3. Lipid solubility: affects absorption as it passes acid.
through gastric intestinal mucosa. b. Other substances that can be receptors are
4. Gastrointestinal (GI) motility enzymes, lipids, and carbohydrate
a. Stomach empties more slowly with food residues.
and will delay oral drug absorption. 2. Drugs can have an agonist or antagonist effect.
b. Most oral drugs are best absorbed if given a. Agonist will connect itself to the
before meals or between meals. receptor site and cause pharmacological
1) Diarrhea can cause drugs not to be response.
absorbed

2 8 NCLEX-RN Review
53155_02_Ch02_p007-110.qxd 2/21/09 10:48 AM Page 9

b. Antagonist will attempt to attach, but E. Half-life: time required for total amount of drug to
because attachment is uneven, there is no decrease by 50%.
drug response.
3. There can be competition at receptor site when
more than one drug tries to occupy it.
Accumulation
A. Therapeutic levels
1. Important goal is for drug to reach therapeutic
Metabolism levels and maintain therapeutic level.
A. Process of metabolism is a sequence of chemical 2. Can be maintained when liver or renal
events that change a drug after it enters the body. function remain unchanged.
B. Liver is principal site of drug metabolism. B. Loading dose
C. Oral medications 1. Sometimes given to raise therapeutic level
1. Go directly to the liver via the portal quickly before drug has chance to be
circulation before entering systemic eliminated.
circulation. 2. For client safety, loading doses are given in
2. Many medications become entirely several smaller doses over short periods of time.
inactivated by the liver the first time they go 3. Once therapeutic level is achieved, a smaller
through it. daily maintenance dose is given to maintain
D. Age therapeutic levels; digoxin may be given this
1. Age of an individual influences metabolism of way.
drugs. C. Toxicity: occurs when drug is eliminated more
2. Infants and elderly have reduced ability to slowly than it is absorbed, causing excessive drug
metabolize some drugs. concentration.
E. Nutrition: liver enyzmes involved in metabolism
rely on adequate amounts of amino acids, lipids,
vitamins, and carbohydrates.
Underlying Disease
F. Insufficient amounts of major body hormones such A. Disease can lead to variable drug response.
as insulin or adrenal corticosteroids can reduce B. Diseases that may affect drug response
metabolism of drugs in liver. 1. Cardiovascular disease
2. Gastrointestinal disease
3. Liver disease
Excretion 4. Kidney disease
A. Process by which drugs are eliminated from body
1. Drugs can be excreted by kidneys, intestines,
lungs, mammary, sweat, and salivary glands.
Clients Age
2. Most important route of excretion for most A. Pediatric: drug dosages are based on body
drugs is kidneys. weightmilligrams per kilogram (mg/kg).
B. Renal excretion B. Geriatric: careful drug history should be obtained,
1. Carried out by glomerular filtration and including over-the-counter (OTC) drugs to
tubular secretion, which increase quantity of determine whether there are drug interactions or
drug excreted. adverse effects.
2. Another renal process that results in excretion
is tubular reabsorption.
a. Drug metabolites in urine can be reverted
back into bloodstream. Sample Questions
b. Decreases quantity of drug excreted.
C. Drugs can affect elimination of other drugs
1. Example: probenecid is sometimes 1. What is the result of taking antibiotics with
administered with penicillin to prevent food?
excretion of penicillin and thus increase 1. Prevent side effects.
effects of penicillin. 2. Enhance action of drug.
2. Example: antacids increase elimination of
3. Delay rate of absorption.
aspirin, thus decreasing its effects.
D. Blood concentration levels 4. Increase rate of absorption.
1. Affect drug elimination
2. When peak blood level of drug is reached, 2. What occurrence may be caused due to the
excretion becomes greater than absorption decreased serum albumin levels in the elderly?
and blood levels of drug begin to drop. 1. Toxic drug effects.

DRUGS AND NURSING IMPLICATIONS

2 9
53155_02_Ch02_p007-110.qxd 2/21/09 10:48 AM Page 10

2. Enhanced absorption.
3. Enhanced drug distribution. Answers and Rationales
4. An increase in the therapeutic effects.

3. If a central nervous system (CNS) depressant is 1. 3. Taking food will decrease the rate of absorption.
administered to an infant, toxic effects can occur Furthermore, taking dairy products with an
due to what action? antibiotic such as tetracycline will cause calcium
(Ca+) to bind to the drug and decrease absorption.
1. Increased drug absorption.
2. Increased drug distribution. 2. 1. Toxic drug effects occur because there is less
3. Decreased drug half-life. albumin or protein for the drug to bind to in the
4. Decreased drug excretion. elderly.

3. 3. The blood-brain barrier is not fully developed in


infants and CNS depressants can readily penetrate.

Drug Administration

ASSESSMENTS APPROPRIATE TECHNIQUES OF DRUG


TO ALL MEDICATION ADMINISTRATION
ADMINISTRATION General Principles for All Medications
A. Confirm client diagnosis and appropriateness of A. Verify all new or questionable orders on the
medication. medication administration record (MAR) against
B. Identify all concurrent medications. physician orders for completeness.
C. Identify any potential contraindications or B. Prepare medications in a quiet environment.
allergies. C. Wash your hands. Observe standard precautions,
D. Identify clients knowledge of medications. as appropriate.
D. Collect all necessary equipment, including straws,
juice or water, stethoscope.
ANALYSIS E. Review MAR for each client carefully to ensure
safety: note medication, dosage, route, expiration
Nursing diagnoses for the client receiving medications date, and frequency.
may include: F. Research drug compatibilities, action, purpose,
A. Risk for injury related to side effects of contraindications, side effects, and appropriate routes.
medications. G. Find medication for individual client and
B. Deficient knowledge: drug effects related to lack of calculate dosage accurately. Confirm normal range
previous experience. of dose, particularly in pediatrics.
C. Noncompliance related to side effects, financial, or H. Check expiration date on medication and look for
other difficulties, limiting ability to take any changes that may indicate decomposition
medications. (color, odor, and clarity).
I. Compare label three times with the medication to
decrease risk of error.
PLANNING AND 1. When removing package from drawer
IMPLEMENTATION 2. Before preparing medication
3. After preparing medication
A. Identify appropriate goals such as, Client will J. Check need for prn medications.
explain rationale for medication prior to discharge. K. Be sure medications are identified for each client.
B. Prepare and administer medications according to L. Check for any allergies and perform all special
the following principles. assessments before administration.

2 10 NCLEX-RN Review
53155_02_Ch02_p007-110.qxd 2/21/09 10:48 AM Page 11

M. Confirm clients identity by checking at least two 3. To reduce chance of contamination, place any
of the three possible mechanisms for identification removable lids open side up; place necessary
to ensure safety. medications into cap of container; transfer to
1. Ask client his name. med cup; replace lid and container.
2. Check clients identi-band and ask him to state E. Prepare liquid medications.
his date of birth. 1. Shake liquid medications, if necessary, to mix.
3. Check bed tag (this is least reliable method). 2. Pour away from bottle label.
N. Provide privacy, if needed. 3. Read liquid amount at meniscus of med cup at
O. Inform client of medication, any procedure, eye level to ensure accuracy.
technique, purpose, and client teaching as 4. If needed, a syringe may be used to measure
applicable. and administer liquid medications.
P. Stay with client until medication is gone; do not 5. Wipe lip of bottle with damp towel to prevent
leave medication at bedside. stickiness.
Q. Assist client as needed, and leave in position of comfort. 6. Replace lid and container.
R. Give medication within 30 minutes of prescribed 7. Do not administer alcohol-based products,
time. such as elixirs, to alcohol-dependent persons.
S. Chart administration immediately in ink. F. Sit client upright to enhance swallowing.
T. Circle initials and document rationale if drug not G. Have client swallow medication except with the
administered. following:
U. Report any errors immediately and complete 1. Sublingual (SL) route: have client place
appropriate institutional documentation. medication under tongue (high rate of
V. Liquid medicationsall routes of administration absorption). Do not allow fluids for 30 minutes
must not be mixed together unless compatibility is following administration.
verified. 2. Buccal route: have client place medication
W. Observe for any reactions and document both between gum and cheek. Do not allow
positive and negative responses. fluids for 30 minutes following
X. Observe the five rights: give the right dose of the administration.
right drug to the right client at the right time by the 3. Iron: have client use straw to prevent staining
right route. teeth.
Y. To ensure safety do not give a medication that H. Stay with client until medication is gone. Use
someone else prepared. Institution policies may gloves if you need to place your finger in clients
require having a colleague double check mouth.
medications such as insulin and heparin. If you I. Special concerns
are unsure in any way, have a colleague verify. 1. Use a calibrated dropper, nipple, or syringe to
Z. If using a computer-controlled dispensing system, give medications to an infant.
follow agency policy for administration and 2. Keep infant at 45 angle.
documentation. 3. See whether medication is available in liquid
form if client is a child or unable to swallow
solid medication.
Administration of Oral Medications 4. Be sure not to use a childs favorite food, as
A. Special assessment: assess clients knowledge this may result in distrust.
level, diet status, oral cavity, and ability to 5. If using an NG or stomach tube for medication
swallow medication. administration, check for correct placement
B. Use agency equipment to crush tablets, if before administration and follow medication
appropriate. In general, enteric-coated tablets with water. Be sure to check for food
should not be crushed. Only scored tablets can be interaction.
broken.
C. With the exception of time-release capsules, Administration of Rectal Drugs
capsule contents may be mixed with food to
enhance swallowing. A. Special assessment: assess clients bowel function
D. Prepare solid medications (tablets, capsules, etc.). and ability to retain suppository/enema.
1. All solid medications can be placed in one B. Obtain suppository from storage area or
medicine cup unless an assessment needs to refrigerator.
be made before administering a particular C. Provide privacy.
medication (e.g., blood pressure, apical pulse). D. Position client left laterally.
2. Unit dose containers can remain in original E. Put on glove or finger cot.
individual package. F. Moisten suppository with water-soluble lubricant.

DRUGS AND NURSING IMPLICATIONS

2 11
53155_02_Ch02_p007-110.qxd 2/21/09 10:48 AM Page 12

Figure 2-1 A rectal suppository is inserted about 2 inches in adults so it will be placed in the
internal anal sphincter

G. Insert suppository, tapered end first, Administration of Inhalants


approximately 2 inches (to pass internal
sphincter) (Figure 2-1). A. Special assessment: monitor vital signs before and
H. Hold buttocks together. after treatments.
I. Encourage client to retain suppository for 1020 B. Have client inhale and exhale deeply.
minutes to allow suppository to melt. C. Have client place lips around mouthpiece without
J. If drug administered via enema, have client retain touching and inhale medication until lungs are
solution 2030 minutes. fully inflated. Inhale slowly and deeply while
depressing the top of the canister, close mouth,
hold breath 10 seconds, then exhale. A specially
Administration of Nasal Medications designed spacer device is available to assist the
client who may have difficulty with this.
A. Have client blow nose to clear mucus.
D. Have client remove mouthpiece, hold breath as
B. Position client so that head can be tilted back to
long as able, and then exhale completely.
aid in gravitational flow or in specific position to
E. If necessary, repeat procedure until medication is
reach sinuses.
gone. Allow 25 minutes between inhalations.
C. Push up on tip of nostril.
F. Wash mouthpiece with warm water.
D. Place dropper or atomizer angled slightly upward
G. Special concerns
just inside nostril; be careful not to touch nose
1. Have tissues handy; encourage expectoration
with applicator.
of sputum.
E. Squeeze atomizer quickly and firmly or instill
2. Be sure client is aware that coughing is
correct number of drops.
expected after treatment.
F. Remind client to keep head tilted for 5 minutes.
3. If mouth is placed directly on inhaler, it is possible
G. Inform client the drops may produce an
that the tongue will absorb medication, resulting in
unpleasant taste.
inadequate dosing and tongue irritation.
H. Leave tissues with client; instruct just to wipe
nose, not blow, to allow for absorption.
I. Special concerns Administration of Ophthalmic
1. If client aspirates and begins to cough, sit
client upright, stay until clients distress is
Medications
relieved. A. Check solution for color and clarity before
2. If client is an infant, lay infant on its back. administering.

2 12 NCLEX-RN Review
53155_02_Ch02_p007-110.qxd 2/21/09 10:48 AM Page 13

J. Wipe away any excess medication starting from


inner canthus.
K. Repeat if necessary using clean tissue.
L. Special concerns
1. Ophthalmic medications are for individual
clients; droppers and ointments should not be
shared.
2. Restrain infants and children if necessary.

Administration of Otic Medications


A. Warm medication in hands prior to administration.
B. Put on gloves.
C. Have client turn to unaffected side to aid
gravitational flow.
D. Clean outer ear using a wet gauze pad. Assess ear
condition.
E. Straighten ear canal by pulling pinna up and back
for adults or down and back for infants and
Figure 2-2 Instilling eye drops children under 3 (Figure 2-4).

B. Warm solution in hands before administration.


C. Have client lie on back or sit with head turned to
affected side to aid in gravitational flow.
D. Cleanse eyelid and eyelashes with sterile gauze
pad soaked with physiologic saline. Assess eye
condition.
E. Have client look up.
F. Assist client in keeping eye open by pulling down
on cheekbone with thumb or forefinger and
pulling up on eyelid. Be sure lower conjunctiva is
exposed (Figure 2-2).
G. Place necessary number of drops into lower
conjunctiva near outer canthus (less sensitive than
cornea).
H. If using ointment, squeeze into lower conjunctiva
moving from inner to outer canthus (Figure 2-3).
1. Do not touch eye with applicator.
2. Twist tube to break medication stream.
I. Have client blink 23 times.

Figure 2-3 Instilling eye ointment Figure 2-4 Administering ear drops

DRUGS AND NURSING IMPLICATIONS

2 13
53155_02_Ch02_p007-110.qxd 2/21/09 10:48 AM Page 14

F. Instill necessary number of drops along side of Administration of Parenteral


canal without touching ear with dropper.
G. Maintain position of ear until medication has Medications
totally entered canal. General Principles
H. Have client remain on side for 510 minutes to
allow medication to reach inner ear. A. Special assessments for parenteral medications:
I. Cotton may be used to keep medication in canal, 1. Assess area for presence of lesions, rashes, or
but only if it is premoistened with medication. abscesses prior to administration.
J. Repeat procedure for other ear if necessary. 2. Assess for discomfort or impaired mobility,
K. Special concern: restrain infants and children if which may affect site selection.
necessary. 3. Assess client ability for self-injection, if
appropriate.
Administration of Topical Agents B. Select appropriate needle size and syringe.
1. Use tuberculin 1 mL syringe for volumes less
A. Provide privacy and expose only appropriate area than 1 mL.
to promote comfort. 2. Needle lumen must be larger for solutions
B. Cleanse area of old medications using gauze pads with increased viscosity.
with soap and warm water. C. When medication comes in a vial, cleanse rubber
C. Use gloves and gauze, tongue depressor, or sterile stopper with alcohol wipes/swab.
applicator, if integument broken. D. Without contaminating plunger, draw up air equal
D. Assess area for any changes or contraindications of to the amount of medication needed.
application. E. Inject the air into the vial to prevent negative
E. Spread medication over site evenly and thinly. pressure and aid in aspirating medication.
F. If necessary, cover area loosely with a dressing. F. Remove the appropriate amount of medication (the
G. Special concerns vial may be multidose).
1. Clients often receive topical agents for image- G. Check to ensure no air bubbles are present; if
altering problems. Applying the medication bubbles are a problem, draw up slightly more
offers a good opportunity to talk about these medication than is needed, return all medication
problems and to share information about to vial, and withdraw medication again or tap
improvements. syringe until air is all collected at top of barrel and
2. When applying nitroglycerin ointment, take can be expelled.
clients blood pressure 5 minutes before and
after application.
3. Use gloves to administer medication to
prevent self-absorption.
4. When using transderm patches, use gloves to
avoid inadvertent drug absorption. Remove
backing and place patch in area with little
hair. Press edges down to secure patch.

Administration of Vaginal Medications


A. Provide privacy.
B. Put on gloves.
C. Have client void.
D. Place client on a bedpan in dorsal recumbent
position with hips and knees flexed (Figure 2-5).
E. Cleanse perineum with warm, soapy water,
working from outer to inner position.
F. Moisten applicator tip with water-soluble lubricant
or just water.
G. Separate labia to insert applicator approximately
2 inches, angled downward and back.
H. Instill medication.
I. If giving douche, dry clients buttocks; otherwise
have client remain in position approximately
1520 minutes (there is no sphincter to hold
suppository in place).
J. Wash applicator with warm, soapy water. Figure 2-5 The client should be in a dorsal recumbent
position for the administration of vaginal creams

2
K. Provide client with pads if needed.

14 NCLEX-RN Review
53155_02_Ch02_p007-110.qxd 2/21/09 10:48 AM Page 15

H. When using an ampule: tap neck to force Q. Record site when documenting medication.
medication into ampule, wrap neck with alcohol R. Variations on preparing medications
wipe/swab, snap off top away from self, place 1. Disposable injection systems have already-
needle into ampule to withdraw medication. A prepared cartridges with attached needle
filter needle should be used to avoid glass shards. appropriate to route and viscosity. To add
Discard filter needle after use into sharps medication, add sterile air from cartridge to
container. vial, then add medication from vial to
I. When mixing a powder, use a filter needle when cartridge.
drawing up medication. Reconstitute according to 2. When combining two medications from an
manufacturers recommendations. ampule and a vial, first determine appropriate
J. Replace protective cover on needle before volumes, as well as total volume. Withdraw
proceeding, using a one-hand scoop method. appropriate volume of medication from vial,
K. Select appropriate site, avoiding bruised or tender followed by medication in ampule.
areas; rotate sites as much as possible. 3. When combining medications from two vials,
L. Cleanse site with alcohol wipe/swab to decrease determine appropriate volume for each drug
contamination. Use gloves to avoid contact with and total volume. Inject air into vial A, then
blood. into vial B. Withdraw medication from vial B,
M. Insert needle quickly with bevel up, leaving a then return to vial A.
small amount of needle showing, and release hold
(to decrease pain). Subcutaneous (SC) Administration
1. With the exception of heparin and insulin,
aspirate to check for blood. A. Use size 25 g to 27 g, 121-inch needle, maximum
2. If blood present, remove needle and start volume 1.5 mL.
again. B. Put on gloves.
3. When giving medications IV, a blood return is C. Pinch skin to form SC fold.
desired. D. Insert needle at 45 angle in thigh or arm or 90
N. Inject medication slowly. angle in abdomen (to avoid entering muscle)
O. Quickly withdraw needle and immediately place (Figure 2-6).
pressure over the site with a new swab. Massage E. Possible sites
area if giving Z-track injection. 1. Lateral aspect of upper arm
P. Dispose of syringe in appropriate manner, but do 2. Anterior thigh
not recap. Utilize safety cover for needle, if 3. Abdomen: 1 inch away from umbilicus
available, before placing in sharps container. 4. Back, in scapular area

Intramuscular Subcutaneous Intravenous Intradermal

90-degree 45-degree 25-degree 10- to 15-


angle angle angle degree angle
Epidermis
Dermis
Subcutaneous
tissue
Muscle

Intramuscular Subcutaneous Intravenous Intradermal


(IM) (SC) (IV) (ID)

Figure 2-6 Angles of injections

DRUGS AND NURSING IMPLICATIONS

2 15
53155_02_Ch02_p007-110.qxd 2/21/09 10:48 AM Page 16

Hypodermoclysis
A. A method of giving large volume solutions SC at a
slow rate.
B. Reserved for clients unable to receive fluids IV.
C. Gloves must be worn.

Intradermal Administration
A. Use size 26 g to 27 g, 1-inch needle on a 1 mL or
tuberculin syringe (volume will be approximately
0.1 mL).
B. Put on gloves.
C. Stretch skin taut.
D. Insert needle at 1015 angle approximately
12 mm depth with needle bevel upward.
E. Possible sites
1. Ventral forearm
2. Scapula Figure 2-8 When using the dorsogluteal site, injection
3. Upper chest is made into the gluteus medius muscle
F. When wheal appears, remove needle; do not
massage site.
3. Rectus femoris (medial thigh): a handbreadth
Intramuscular (IM) Administration above knee and below greater trochanter; good
A. Use size 18 g to 23 g, 12-inch needle, maximum site for infants and self-injection.
volume 5 mL. 4. Gluteus medius (dorsogluteal) (Figure 2-8):
B. Put on gloves. landmarks are posterior superior iliac spine,
C. Stretch skin taut. iliac crest, greater trochanter of femur.
D. Insert needle at 90 angle. 5. Deltoid (Figure 2-9): landmarks are acromium
E. Possible sites process, axilla base; for small doses less than
1. Gluteus minimus (ventrogluteal): landmarks 2 mL only.
are anterior-superior iliac spine, iliac crest, F. Z-track injection (IM variation) for irritating
greater trochanter of femur. solutions.
2. Vastus lateralis (anterior thigh) (Figure 2-7): 1. Needle size: replace needle used to draw up
a handbreadth above the knee and below medication with one 23 inches long, 2022 g.
greater trochanter; good site for children. 2. Pull skin away from site laterally with
nondominant hand to ensure medication
enters muscle.
DEEP 3. Wait 10 seconds after injecting medication
FEMORAL
ARTERY
before withdrawing needle.
4. Release skin; do not massage (seals needle
track).
SCIATIC 5. Encourage physical activity.
NERVE
6. Possible sites: gluteus medius best, but may
use any IM site except deltoid.
RECTUS
FEMORIS G. A 45 angle may be sufficient for infants and
children.
INJECTION
AREA Administration of Intravenous (IV)
VASTUS
LATERALIS Medications
A. General principles
1. Check site for complications (redness,
FEMORAL swelling, tenderness).
ARTERY 2. Check for blood return.
AND VEIN
3. Prepare medication according to
manufacturers specifications.
Figure 2-7 Anterior view of the location of the vastus 4. Appropriate tubing selection varies according
lateralis muscle in a young child to institution policy. Generally, rates greater

2 16 NCLEX-RN Review
53155_02_Ch02_p007-110.qxd 2/21/09 10:48 AM Page 17

C. Secondary piggyback/add-a-line (added to an


Acromion process existing IV line) (Figure 2-10).
1. With regulator turned off, spike tubing into IV
bag with medication.
2. Squeeze drip chamber; fill halfway with solution.
3. Run fluid through tubing.
Head of humerus 4. If using add-a-line tubing, lower main IV bag
on hanger provided, otherwise hang bag at
Injection site same level as primary bag.
Deltoid 5. Swab most proximal port with alcohol for add-
a-line systems, otherwise lower port is
acceptable.
6. Attach 20 g 1-inch needle to tubing, if a
needleless system is not being used.
Deltoid groove 7. Insert needle into injection port.
8. Regulate rate with control and watch to count
drops.
9. When medication absorbed, main line will
start to drip again.
10. Turn off secondary tubing.
11. Return main bag to original position.
12. Special concerns
a. Be sure to label tubing with date.
b. Use new tubing every 2472 hours
(according to institution policy).
D. Intravenous push medications
1. Using an appropriately sized needle, prepare
medication as ordered.
Figure 2-9 Deltoid injection site 2. Cleanse injection port with alcohol or other
appropriate cleanser.
3. Unless otherwise recommended, turn off
than or equal to 12 hours require macrotubing primary IV bag; flush with saline if indicated.
(60 gtts/mL), all others require macrotubing 4. Insert needle and administer medication at
(10, 15, or 20 gtts/mL). prescribed rate.
5. Gloves should be worn when contact with E. Electronic regulators
blood or other body fluids is a possibility. 1. Syringe infusers
B. A scheduled routine flushing of the IV site is a. Check for drug compatibility, flush with
required to check for patency and if intravenous saline if necessary.
catheter is still in the vein. Flushing is also done b. Place syringe into infuser and prime
before and after intermittent medications. Site may appropriate tubing with prepared medication.
be called: male adapter, heplock, heparin lock, c. Secure unit and activate unit according to
capped jelco, INT. manufacturers recommendations.
1. Clean injection port with alcohol at each step. 2. Pumps and controllers
2. Use SASH method to give medication. a. Prime tubing according to manufacturers
a. S: flush with 23 mL saline. recommendations; do not purge when
b. A: administer medication at prescribed rate attached to client.
using either the needleless system, a blunt b. Prior to connecting IV to client, check to
needle, or a short needle with a gauge equal determine if tubing allows gravity free-
to or smaller than catheter (25 g 12 in). flow. If it does, be sure to turn off regulator.
c. S: flush with 23 mL saline (maintain c. Connect tubing to client and turn on
positive pressure to prevent blood back-up electronic regulator.
into catheter). d. Confirm alarm function by keeping tubing
d. If a central line (CVP) or PICC line is clamped while machine is turned on. Do
present, then an additional flush of not turn off alarms.
Heplock flush is administered (to prevent e. Follow manufacturers directions for
blood clotting in line). Most facilities deactivating alarm and starting IV flow.
protocol require the use of 10-cc syringes f. Explain regulator and alarms to client.
for all flushes in CVPs or PICCs due to the g. Confirm flow rate with hourly checks on
less pressure force exerted on the lines. client, fluid, and regulator.

DRUGS AND NURSING IMPLICATIONS

2 17
53155_02_Ch02_p007-110.qxd 2/21/09 10:48 AM Page 18

Primary set Piggyback set Primary set Piggyback set

Extension
hook

Clamp closed

Piggyback port

Secondary port

A. B.

Figure 2-10 (A) In this setup, the tubing to the primary set is clamped to allow the piggyback unit to
empty first. The tubing on the primary setup is unclamped once the piggyback unit empties. (B) In this setup,
the primary bottle is hung on an extension hook to allow the piggyback unit to empty first. The primary unit
then begins to empty

MEDICATION CALCULATIONS 2. Units of measurement are


a. Meter (m) for length
b. Gram (g) for weight
Conversions c. Liter (l or L) for volume
Conversions need to be made within systems (from 3. Multiples and fractions of 10 are identified by
one measurement to another) and also among systems prefixes (Table 21).
(from apothecary to metric or household to metric). a. 0.001 liter would then be equal to
1 milliliter (mL).
Conversions within Systems b. 1000 grams would be equal to a kilogram (kg).
4. Commonly used terms of weight include
A. Metric system kilogram, gram, milligram, and microgram.
1. Based on decimal system, basic unit is 10.

Table 2-1 Metric Prefixes Denoting Multiples and Fractions of 10

Multiples Measure Fractions Micro

1000 100 10 0.1 0.01 0.001 .000001


Decimal
Thousands Hundreds Tens LGM Tenths Hundredths Thousandths Millionths
Metric

2
Kilo- Hecto- Deka- Deci- Centi- Milli- Micro-

18 NCLEX-RN Review
53155_02_Ch02_p007-110.qxd 2/21/09 10:48 AM Page 19

Table 2-2 Household Units of Liquid Measure Table 2-3 Approximate Equivalents to Remember

60 drops (gtts) = 1 teaspoon (tsp) Household Metric


3 tsp = 1 tablespoon (tbsp)
1 drop (gtt) = .06 milliliter (mL)
6 tsp = 1 ounce (oz)
15 drops (gtt) = 1 mL [1 cc]
2 tbsp = 1 oz
1 teaspoon (tsp) = 5 (4) mL
6 oz = 1 teacup
1 tablespoon (tbsp) = 15 mL
8 oz = 1 glass
2 tbsp = 30 mL
8 oz = 1 measuring cup
1 ounce (oz) = 30 mL
1 teacup (6 oz) = 180 mL
5. Commonly used terms of volume include liter 1 glass (8 oz) = 240 mL
and milliliter. Note: A cube measuring 1 cm per 1 measuring cup (8 oz) = 240 mL
side holds one milliliter, so a cubic centimeter 2 measuring cups (1 pint) = 500 mL
(cc) equals a milliliter (mL): 1 cc 5 1 mL.
6. To convert within the metric system, set up a
ratio with the conversion factor on the right 4. (15 gtts) (X) (90 gtts) (1 mL)
5
and the desired information on the left, cross 15 gtts 15 gtts
multiply, divide to find X, and complete 5. X 5 6 mL
needed math. Remember to keep ratios equal:
whatever is done to one side must be done to
the other. Dosage Calculations
7. Example: convert 5000 mg to g A. Calibrated containers are available for oral liquids
a. 1000 mg 5 1 g (Figure 2-11), and liquid injectables (Figure 2-12).
b. 5000 mg 1000 mg B. Formula for calculations
5
X 1g ordered amount of drug unknown quantity needed (X)
c. (X) (1000 mg) 5 (5000 mg) (1 g) 5
amount of drug on hand known quantity of drug
d. (X) (1000 mg) (5000 mg) (1 g) C. Be sure all conversions are done first. The
5 technique of using ratios is the same.
(1000 mg) (1000 mg)
e. X 5 (5) (1 g)
f. X 5 5 g Technique
g. 5000 mg 5 5 g
A. Dosage calculation for scored tablet
B. Household system
1. 2000 mg of a drug is ordered. It is available as
1. Approximate measures that vary according to
a scored tablet containing 4 grams.
manufacturer, temperature.
2. Calculation
2. Common units include teaspoon (tsp),
a. Convert 4 grams into mg
tablespoon (tbsp), ounce (oz), cup, and drop (gtt)
1. 4g 1g
(Table 22). 5
3. Conversions within the system are the same. X mg 1000 mg
4. Example: convert 3 tsp to drops 2. 4000 = X Therefore, 4 g = 4000 mg
a. 60 drops 5 1 tsp
b. 3 tsp 1 tsp
5
X 60 gtts
c. (1 tsp) (X) 5 (3 tsp) (60 gtts)
d. (1 tsp) (X) (3 tsp) (60 gts)
5
1 tsp 1 tsp
e. X 5 180 gtts

Conversions from One System to Another


A. Conversions are done in the same manner. Some of
the equivalents must be memorized (Table 23).
B. Example: convert 90 gtts to ml
1. 15 gtts 5 1 mL
2. 90 gtts 15 gtts
5
X 1 mL
3. (15 gtts) (X) 5 (90 gtts) (1 mL) Figure 2-11 Disposable medicine containers

DRUGS AND NURSING IMPLICATIONS

2
19
53155_02_Ch02_p007-110.qxd 2/21/09 10:48 AM Page 20

2) 5 gr X mg
60 mL syringe 5
1 gr 60 mg
3) X 5 300 mg
30 mL syringe
b. Now calculate the dosage
1) ordered amt is 300 mg
10 mL syringe 5
amt on hand 100 mg
5 mL syringe
X (unknown) capsules
1 (known) capsules
2) 300 mg = X
100 mg = 1
3) 100 X = 300
3 mL syringe
4) X = 3
Tuberculin 3. Give 3 capsules.
Insulin syringe with needle
D. Dosage calculation for parenteral medications
1. Order reads: Furosemide (Lasix) 35 mg IV. The
vial is labeled 40 mg = 4 mL. How many mL
should be given?
Figure 2-12 Commonly used syringes (not to scale)
2. Calculations
a. (ordered amt) 35 mg (unknown quantity)
b. Now calculate dosage 5
(amt on hand) 40 mg 4 mL (known quantity)
1.
2000 mg (ordered amt) X (unknown quantity) 1) 35 mg X
4000 mg (amt on hand) 5 1 tablet (known quantity) 40 mg
5
4 mL
2. 2000 mg 5 4000 mg X 2) 40 X = 140
Divide each by 4000 3) X = 3.5
3. 2000 mg 4000 mg b. Give 3.5 mL
5
4000 mg 4000 mg c. Another method:
4. 2000 mg 1) (ordered amt) (known quantity)
5X
4000 mg (amt on hand)
5. X = 0.5 tablet
3. Give 1 2 tablet. 2) (35 mg) (4 mL) = 3.5 mL
B. Dosage calculation for liquid (40 mg)
1. The order is for potassium chloride (KCl) E. Dosage calculation for units (some medications
20 mEq. The bottle is labeled KCl elixir such as heparin and penicillin are ordered in units)
10 mEq/mL. How many mL will be given? 1. The order is penicillin 750,000 units. The vial
a. Ordered amount of drug is 20 mEq; amount reads 300,000 units/2 mL. How many mL will
of drug on hand is 10 mEq. be given?
b. Unknown quantity is X; known quantity of 2. Ordered amount of drug is 750,000 units;
drug on hand is 1 mL. amount of drug on hand is 300,000 units.
2. Calculations 3. Unknown quantity is X; known quantity is 2 mL.
a. 20 mEq X 4. Calculations
5 a. 750,000 units X
10 mEq 1 mL 5
b. (10 mEq) (X) 5 (20 mEq) (1 mL) 300,000 units 2 mL
c. (10 mEq) (X) (20 mEq) (1 mL) b. (300,000 units) (X) = (750,000 units) (2 mL)
5 c. 300,000 X 1,500,000
10 mEq 10 mEq 5
3,000,000 units 300,000
d. X 5 2 mL
3. Give 2 mL potassium chloride. 150
d. X 5
C. Dosage calculation for a capsule 30
1. The order reads: Phentoin Sodium capsules e. X = 5 mL
(Dilantin) gr V orally. Available is a bottle 5. Give 5 mL penicillin.
labeled Dilantin 100 mg per capsule. How F. Dosage calculation for powders that need to be
many capsules will be given? reconstituted by adding sterile water or normal
2. Calculations saline solution (the total amount of solution is
a. First, convert gr 5 to mg used for calculations)
1) 1 gr 5 60 mg

2 20 NCLEX-RN Review
53155_02_Ch02_p007-110.qxd 2/21/09 10:48 AM Page 21

1. Mefoxin 1 g is ordered; mefoxin 2 g is on


hand. Add 4.3 mL to equal 5 mL solution.
2. Ordered amount of drug is 1 g; amount of drug
on hand is 2 g.
3. Unknown quantity is X; known quantity is
5 mL.
4. Calculations
a. 1 g X
5
2g 5 mL
b. (2 g) (X) 5 (5 mL) (1 g)
c. (2 g) (X) (5 mL) (1 g)
5
2g 2g
d. X 5 2.5 mL
5. Give 2.5 mL mefoxin.
G. Dosage calculation in children (pediatric dosages)
1. Body surface area (BSA): most accurate
method for calculating pediatric dosages
a. West nomogram, Figure 2-13, if BSA is not
known: draw a line from height on the
nomogram; the point of intersection on
surface area is the BSA.
b. Formula using surface area (m2)
surface area (m2)
3 adult dose 5 child dose
1.73 m2
c. Example of calculating a pediatric dosage
using BSA:
1) The adult dose is 100 mg Demerol; the
child weighs 20 kg and is 40 inches
tall.
2) BSA is 0.77 m2
3) Calculations
a) 0.772
3 100 mg 5 X
1.73 m2
b) 0.45 3 100 mg 5 X
c) 45 mg 5 X
4) Childs dose is 45 mg. Nomogram for determining body surface of children from
2. Pediatric dosages may also be calculated by height and weight. (S = W 0.425  H 0.725  71.84, or
weight (mg/kg) log S = log W  0.425  log H  0.725  1.8564 [S =
body surface in cm 2 ; W = weight in kg; H = height in
a. Example: the order is for phenobarbital cm]).
2 mg/kg of body weight; the client weighs
25 kg.
b. Calculations Figure 2-13 Nomogram. In this example,
1) 2 mg 1 kg a child who weighs 15 kilograms and is about
5
X 25 kg 92 centimeters in height has a body surface area
2) (1 kg) (X) 5 (2 mg) (25 kg) of .60 square meter. (Source: A Formula to
3) (1 kg) (X) (2 mg) (25 kg) Estimate the Approximate Surface Area if Height
5
1 kg 1 kg and Weight Be Known by D. Dubois and
4) X 5 50 mg E. F. Dubois, 1916, Archives of Internal Medicine,
c. Give 50 mg of phenobarbital. 17, p. 863. Modified with permission.)
H. Dosage calculation for IV medications
1. Macrodrop
a. More commonly used in adult IVF 2. Microdrop
(intravenous fluid) administration a. More commonly used for children, elderly
b. In order to calculate the flow rate, need to or critically ill where exact control is
know drop factor: 10, 15, or 20 gtt/mL required
b. Drop factor is always 60 gtt/mL

DRUGS AND NURSING IMPLICATIONS

2 21
53155_02_Ch02_p007-110.qxd 2/21/09 10:48 AM Page 22

3. Formula for calculation


amount of solution
3 Drop factor 5 gtt/min
Sample Questions
time in minutes
4. Example: the order is for 1000 mL NS over
8 hours; drop factor is 10 gtt/mL 4. The order reads Digoxin 0.375 mg once
a. Calculation daily. The bottle reads Digoxin 0.25 mg per
1) 1000 mL 10 gtt/mL tablet. How much should the nurse
3 5X administer?
480 min mL
2) 1000 mL/gtt
5 20.8 gtt/min 5. The order is chloral hydrate 200 mg. The bottle
480 min reads chloral hydrate 0.1 g/capsule. Give
3) X = 20.8 gtt/min capsules.
5. The same formula can be used for IVs
requiring microdrop rates, or the following 6. The order is penicillin 50,000 units. The vial
formula can be used: reads penicillin 500,000 units. Add 4.3 mL to
mL/hour = microdrops/min yield 5 mL. Give mL.
This formula works because the drop factor for
microdrop tubing is always 60 microgtts/mL 7. The order is ampicillin 0.4 mg/kg. Client weighs
and an hour is 60 minutes. 38.5 pounds. The bottle reads ampicillin
a. Example: order is for 1000 mL D5NS over 10 mg/mL. Give mL.
24 hours. Drop factor is 60 gtt/mL
b. Calculation with the first formula 8. Phenytoin (Dilantin) 5 mg/kg is ordered for a
1) 1000 mL 60 gtt 40-pound child. It is to be administered in three
3 5X
1440 min mL equal doses. The drug is available in an oral
2) (1000 mL) 3 (60 gtt) suspension containing 125 mg/mL. How many
5 41.66
1440 min/mL ml should be administered per dose?
3) X = 41.66 gtt/min
c. Cannot give a partial drop, so rate is 42 9. The order is for 1.2 million units of penicillin G
gtt/min. (Bicillin) IM. Available is 600,000 units/mL.
d. Calculation with the second formula How much should the nurse administer?
1) First determine the mL/hour:
total volume 10. Order is for 2500 mL D5W over 24 hours. Drop
factor is 15 gtt/mL. Run IV at
total hours
gtts/minute.
2) 1000 mL
5 42 mL/hour 5 42 gtt/min
24 hours 11. Order is for 2000 mL D5W over 24 hours. Drop factor
6. Readjusting IV rates may be necessary when is 60 gtt/mL. Run IV at gtt/minute.
the prescribed rate for an existing IV is
changed 12. Enoxaparin sodium (Lovenox) 25 mg SC q 12
a. Formula for calculation hours is ordered. The label reads 30 mg /0.3 mL.
amount of solution remaining How much should the nurse administer?
3 drop factor
remaining hours in minutes
b. Example: the order is to infuse remaining 13. The order is for meperidine 50 mg IM q 4 hours
700 mL over 3 hours; drop factor is prn. The label reads meperidine 75 mg/mL. How
15 gtt/mL. much should the nurse administer?
c. Calculation
1) 700 mL 15 gtt 14. An adult is on continuous IV heparin therapy for
3 5X thrombophlebitis. The IV contains 15,000 units
180 min mL
of heparin in 500 mL of 5% dextrose (D5W) at the
2) 700 mL 3 15 gtt
5 58.33 rate of 20 mL per hr. How many units per hour is
180 the client receiving?
3) 700 gtt
5 58.3 gtt/min 1. 60 units.
12 min
d. Cannot give partial drop, so run IV at 2. 25 units.
58 gtt/min. 3. 600 units.
4. 700 units.

2 22 NCLEX-RN Review
53155_02_Ch02_p007-110.qxd 2/21/09 10:48 AM Page 23

15. The order is for Ancef 1 gram IV in 50 cc 5% 8. 1.2 mL/dose. First convert pounds to kg.
dextrose (D5W) to run in over 30 minutes every 6 Divide 40 pounds by 2.2 pounds/kg for a
hours. The administration set delivers 10 gtts/cc. weight of 18.18 kg. Multiply 5 mg/kg for a
What should the drip rate be? total daily dose of 91 mg. Calculate the dose
1. 8 gtt/min. using ordered over amount on hand times
2. 15 gtt/min. volume.
3. 17 gtt/min. 91 mg
3 5 mL 5 3.6 mL total daily dose
4. 25 gtt/min. 125 mg
Divide 3.6 mL by 3 doses 5 1.2 mL/dose.

9. 2 mL.
Answers and Rationales 1,200,000 units
3 1 mg 5 2 mL
600,000 units
4. 1.5 tablets. The formula to use is: 10. 26 gtt/min. Divide 2500 mL by 24 hours. Then
divide the result by 60 minutes per hour and
ordered amt multiply by 15 gtt/mL.
3 quantity
amt on hand
11. 83 gtt/min. Divide 2000 mL by 24 hours for
0.375 83 mL/hour. This is divided by 60 min/hour and
3 1 tablet 5 1.5 tablets
0.25 multiplied by 60 gtt/mL for a total of 83 gtt/min.
5. 2 capsules. First convert grams to mg.
12. 0.25 mL. Ordered amount is 25 mg. Available is
1000 mg = 1 g
30 mg in 0.3 mL.
1000 mg 3 0.1 5 100 mg 0.25 mg
Then use ordered over amount on hand times 3 0.3 mL 5 0.25 mL
0.30 mg
quantity.
200 mg 13. 0.67 mL. Ordered amount is 50 mg. Available is
3 1 capsule 5 2 capsules 75 mg/mL.
100 mg
50 mg
3 1 mL 5 0.66667 mL
6. 0.5 mL. The formula to use is ordered over 75 mg
amount on hand times volume.
50,000 units 14. 3. Divide 500 mL by 20 mL to determine the
3 5 mL5 0.5 mL number of hours of the infusion: 25 hours. Next,
500,000 units
divide 25 hours into 15,000 units to get
7. 0.7 mL. First convert pounds to kg. 38.5 pounds units/hour: 600 units of heparin/hour.
divided by 2.2 pounds/kg 5 17.5 kg. Calculate
15. 3. total volume infused
total mg to be given. 3 drop factor 5 gtt per minute
time in minutes
0.4 mg
3 17.5 kg 5 7 mg
kg 50 mL 10 gtt
3 5 16.6 5 17 gtt/min
Then use ordered over amount on hand times 30 minutes 1 mL
volume.
7 mg
3 1 mL 5 0.7 mL
10 mg

Central Nervous System Drugs

LOCAL ANESTHETICS enzymes; produces temporary loss of sensation


and motion in a limited area of the body.
A. Prototype: Lidocaine (Xylocaine) 2. Use. Topical anesthesia, regional anesthesia
1. Action. Amide-type anesthetic that blocks (Unit 4), antiarrhythmic (discussed in
nerve conduction; metabolized by hepatic Cardiovascular Drugs).

DRUGS AND NURSING IMPLICATIONS

2 23
53155_02_Ch02_p007-110.qxd 2/21/09 10:48 AM Page 24

3. Adverse effects. Drowsiness, dizziness, light-


headedness, restlessness, numbness of lips Sample Questions
and tongue; headache with spinal anesthesia;
hypotension, bradycardia, cardiovascular
collapse; convulsions; tinnitus; muscle 16. Why is epinephrine added to local anesthetic
weakness; anaphylaxis; respiratory depression. preparations?
4. Nursing implications 1. Prolong anesthetic action.
a. Force fluids with spinal anesthesia.
2. Lower blood pressure.
b. When used for spinal or epidural
anesthesia, should be preservative free. 3. Prevent arrhythmias.
c. Monitor VS, and keep siderails up. 4. Increase blood flow to injection site.
d. Epinephrine, a vasoconstrictor, may be
added to lidocaine (Xylocaine). 17. A client complains of a severe sore throat after the
e. May interfere with swallowing reflex. extraction of 2 wisdom teeth. Viscous lidocaine
f. Discard drug without preservatives after (Xylocaine) is ordered. Which of the following
immediate use. should be included in client teaching concerning
g. Due to adverse effects, elderly clients the use of viscous lidocaine (Xylocaine)?
should be closely monitored. 1. Take viscous lidocaine (Xylocaine) with
h. Do not use discolored, cloudy solutions. fluids to soothe sore throat.
B. Related drugs. See Table 2-4.
2. Instruct client to use a humidifier while
taking viscous lidocaine (Xylocaine).

Table 2-4 Local Anesthetics

Drug Use Adverse Effects Nursing Implications


Procaine (Novocain) Nerve block Anaphylaxis is seen more with Procaine is an ester-type anesthetic
Spinal anesthesia ester-type anesthetics metabolized by esterase found in
Infiltration plasma
anesthesia Emergency resuscitation equipment
should be available
Monitor VS (see lidocaine [Xylocaine])
Benzocaine (Americaine) Topical anesthesia See procaine (Novocain) and Benzocaine (Americaine) is similar to
lidocaine (Xylocaine) procaine (Novocain) and is an ester-
type anesthetic
Commonly found in OTC
preparations to treat sunburn, rashes,
sore throats, and hemorrhoids
Mepivacaine (Carbocaine) Infiltration nerve See lidocaine (Xylocaine) Amide-type local anesthetic that has
block anesthesia two times the potency and toxicity of
lidocaine
Bupivacaine (Marcaine) Epidural blocks See lidocaine (Xylocaine) Long-acting amide-type anesthetic
Infiltration Toxicity seen more often in children
anesthesia and elderly
Peripheral nerve
block
Etidocaine (Duranest) Infiltration See lidocaine (Xylocaine) Analgesia effects last 1122
anesthesia times longer than lidocaine
Peripheral nerve
block
Central neural
blocks

2 24 NCLEX-RN Review
53155_02_Ch02_p007-110.qxd 2/21/09 10:48 AM Page 25

3. Advise client to wait 60 minutes before c. Monitor CBC, prothrombin time, kidney
eating after drug application. and liver function studies for clients on
4. Encourage client to take viscous lidocaine long-term therapy.
(Xylocaine) with food to reduce GI distress. d. Additive effect for clients on anticoagulant.
e. Stop therapy 1 week before surgery.
5. Discharge teaching
a. Drink plenty of fluids to prevent salicylate
Answers and Rationales crystalluria.
b. Take with glass of water, antacid, milk, or
food to reduce gastric irritation.
16. 1. Epinephrine prolongs anesthetic action, while c. Parents should not give to children or
shortening the onset of action and reducing adolescents with flu or chickenpox
blood flow to injection site. because Reyes syndrome may occur.
d. Report signs of bleeding and bruising to
17. 3. Viscous lidocaine (Xylocaine) can interfere physician.
with swallowing reflex and clients should wait e. Discontinue use if tinnitus, dizziness, or GI
at least 60 minutes after use before eating. distress occur.
f. Pregnant women should not use.
g. Do not crush enteric-coated tablets.
NON-NARCOTIC ANALGESICS h. Do not ingest large amounts of alcohol as
this increases risk of GI bleeding.
AND ANTIPYRETICS B. Related drugs. See Table 2-5.
C. Prototype: acetaminophen (Tylenol)
A. Prototype: salicylates 1. Action. Analgesic and antipyretic action (see
Acetylsalicylic Acid (aspirin) (ASA) aspirin); does not have anti-inflammatory or
1. Action antiplatelet action.
a. Analgesia: inhibits formation of 2. Use. Mild to moderate pain, fever control.
prostaglandins involved with pain. 3. Adverse effects. Rash, thrombocytopenia, liver
Analgesia also occurs by action of toxicity. Toxicity can occur 224 hours after
hypothalamus and blocking generation of ingestion.
pain impulses. 4. Nursing implications
b. Antipyretic: inhibits formation of a. Monitor liver and kidney function, and
prostaglandins in production of fever. CBC periodically for clients on long-term
Aspirin acts on the hypothalamus to therapy.
produce vasodilation. b. Can cause psychologic dependence.
c. Anti-inflammatory: inhibits prostaglandin c. Antidote: acetylcysteine (Mucomyst)
synthesis causing anti-inflammatory action. 5. Discharge teaching. Notify physician if no
d. Antiplatelet action occurs when aspirin relief of symptoms within 5 days of therapy.
inhibits prostaglandin derivative,
thromboxane A2.
2. Use. Mild to moderate pain; control of fever;
inflammatory conditions; reduce TIA Sample Questions
occurrence; reduce risk of MI in men with
unstable angina.
3. Adverse effects. Tinnitus, confusion, 18. Which of the following should be included in
dizzinessall are symptoms of salicylism; teaching concerning the administration of
drowsiness; epistaxis, bleeding, bruising; indomethacin (Indocin)?
edema, hypertension; nausea, vomiting, 1. Have periodic ophthalmic examinations.
diarrhea, gastritis; hypersensitivity;
2. Take on an empty stomach.
hypoglycemia, sweating; impaired renal
function; respiratory alkalosis and metabolic 3. Take aspirin for headache relief.
acidosis are associated with aspirin toxicity. 4. Eat high-fiber foods to prevent constipation.
4. Nursing implications
a. Clients with history of nasal polyps, 19. In comparing aspirin to acetaminophen
asthma, rhinitis, chronic urticaria (Tylenol), what is true pertaining to Tylenol?
have high incidence of aspirin 1. It is contraindicated in clients with peptic
hypersensitivity. ulcer disease.
b. Clients with diabetes should have glucose 2. It is contraindicated in clients with asthma.
monitored.
3. It is as effective as aspirin for reducing fever.

DRUGS AND NURSING IMPLICATIONS

2 25
53155_02_Ch02_p007-110.qxd 2/21/09 10:48 AM Page 26

Table 2-5 Nonsteroidal Anti-Inflammatory Drugs (NSAIDs)*

Drug Use Adverse Effects Nursing Implications


Ibuprofen (Motrin, Relief of mild to moderate May cause sodium or water Do not take with aspirin
Advil) pain retention Take with food or with milk to
Primary dysmenorrhea Thrombocytopenia, hemolytic decrease GI distress
Rheumatoid and anemia Monitor liver enzymes
osteoarthritis Acute renal failure, hematuria
Can elevate liver enzymes
Naproxen (Naprosyn) Rheumatoid and See aspirin and ibuprofen See aspirin and ibuprofen
osteoarthritis (Motrin) (Motrin)
Ankylosing spondylitis
Primary dysmenorrhea
Acute gout attacks
Juvenile diabetes
Indomethacin (Indocin) Close patent ductus GI distress, anorexia Clients need to have periodic
arteriosus in premature Severe headache ophthalmic examinations
infant Corneal cloudiness, visual field Do not take aspirin; see aspirin
Acute gouty arthritis changes
Moderate severe refractory
rheumatoid and
osteoarthritis
Ankylosing spondylitis
Piroxicam (Feldene) Acute or long-term manage- Higher incidence of GI See aspirin, indomethacin
ment of rheumatoid or bleeding (Indocin), and ibuprofen
osteoarthritis (Motrin)
Ketorolac (Toradol) Short-term pain Risk of renal impairment and Do not give longer than 5 days
management GI bleeding in prolonged Anaphylaxis can occur with
use first dose
Celecoxib (Celebrex) Primary dysmenorrhea Peripheral edema Separate administration with
Acute pain Abdominal pain magnesium or aluminum
Rheumatoid and Upper respiratory infection containing antacids by
osteoarthritis 2 hours
Monitor liver enzymes, renal
function, H & H and electrolytes
Valdecoxib (Bextra) Primary dysmenorrhea Anemia Can take without regard to
Osteoarthritis Fluid retention food
Adult rheumatoid arthritis Increased blood pressure Do not take aspirin or other
NSAIDs with this medication
*NSAIDs are prostaglandin Inhibitors

4. It has a stronger anti-inflammatory effect than 21. Which drug is the drug of choice used to treat
aspirin. primary dysmenorrhea?
1. Acetaminophen (Tylenol).
20. Which condition is an indication for aspirin use?
2. Piroxicam (Feldene).
1. Asthma.
3. Indomethacin (Indocin).
2. TIA.
4. Ibuprofen (Motrin).
3. Gout.
4. Nasal polyps.

2 26 NCLEX-RN Review
53155_02_Ch02_p007-110.qxd 2/21/09 10:48 AM Page 27

g. Advise client to change position slowly.


Answers and Rationales h. Check for signs of urinary retention.
i. Keep stool record and institute measures to
prevent constipation; e.g., fluids, foods high
18. 1. Indomethacin (Indocin) may cause visual field in fiber, and activity as tolerated; administer
changes or corneal cloudiness. Clients should stool softeners and laxatives as ordered.
have periodic ophthalmic examinations to j. Have narcotic antagonist (naloxone
monitor for visual change. [Narcan]) available for reversal of effects if
necessary.
19. 3. Acetaminophen (Tylenol) is as effective as k. Teach client not to drink alcoholic
aspirin in reducing fever. Both have similar beverages while taking narcotics.
antipyretic actions. l. Monitor for withdrawal symptoms and
decrease dose slowly because these drugs
20. 2. Due to aspirins antiplatelet effect, aspirin can may produce physical dependence.
be used to decrease TIA. m. Use special caution with clients with
increased intracranial pressure, chronic
21. 4. Ibuprofen (Motrin) is the drug of choice to obstructive pulmonary disease (COPD),
treat primary dysmenorrhea. alcoholism, severe hepatic or renal disease,
and in elderly or debilitated clients who
may not metabolize the drug efficiently.
NARCOTIC ANALGESICS 5. Discharge teaching
a. Take before pain intensifies to receive
A. Prototype: morphine sulfate fullest analgesic effect.
1. Action. Acts on opioid receptors in CNS and b. No alcohol or CNS depressants should be
induces sedation, analgesia, and euphoria. taken.
2. Use. Relief of moderate to severe pain, c. No smoking or ambulating alone after drug
preoperative and/or postoperative medication, has been taken.
pain relief in MI, relief of dyspnea occurring in d. Avoid activities requiring alertness.
pulmonary edema or acute left ventricular B. Related drugs. See Table 2-6.
failure. C. Narcotic agonists/antagonists
3. Adverse effects. Sedation, confusion, 1. Examples: pentazocine (Talwin), nalbuphine
euphoria, impaired coordination, dizziness; (Nubain), butorphanol (Stadol), buprenorphine
urinary retention, constipation, (Buprenex), fentanyl (Duragesic)
hyperglycemia; respiratory depression; 2. Mechanism of action
hypotension, tachycardia, bradycardia; a. Term agonist refers to the fact that they
nausea, vomiting, decreased uterine bind to opioid receptors to produce
contractility; allergic reactions; tolerance, analgesia.
physical and psychological dependence; b. Term antagonist refers to the fact that
pupil constriction. they counteract the effects of the pure
4. Nursing implications narcotic agonists (i.e., morphine,
a. Assess clients pain before giving meperidine [Demerol]).
medication. 3. Use. Relief of moderate to severe pain; may
b. Evaluate effectiveness of analgesic be used for clients who cannot tolerate
including onset and duration of response pure narcotic agonists. Caution: May
to medication. produce withdrawal in a client who has
c. Observe for signs of tolerance with been taking pure narcotic agonists for a
prolonged use. week or more.
1) Tolerance means that a larger dose of 4. Adverse effects. Drowsiness, nausea,
narcotic analgesic is required to psychotomimetic effects, e.g., hallucinations;
produce the original effect. respiratory depression and constipation but
2) The first sign of tolerance is usually a less of a risk than with pure narcotic agonists
decreased duration of effect of the such as butorphanol (Stadol), nalbuphine
analgesic. (Nubain), pentazocine (Talwin).
d. Monitor respiratory rate and depth before D. Combination drugs
giving drug, and periodically thereafter. 1. Some narcotics can be combined with other
e. Encourage sighing, coughing, and deep drugs.
breathing. 2. Examples of this would be codeine combined
f. Warn ambulatory clients to avoid activities with Empirin, Fiorinal, or Tylenol.
that require alertness.

DRUGS AND NURSING IMPLICATIONS

2 27
53155_02_Ch02_p007-110.qxd 2/21/09 10:48 AM Page 28

3. Meperidine (Demerol) will be most effective


Table 2-6 Narcotic Analgesics if you take it before your pain becomes
severe. This will decrease your needing
Drug Use Comments
excess amounts of a narcotic.
Codeine Moderate to Less potential for 4. Meperidine (Demerol) is ordered for you to
severe pain, dependence than help manage your pain. You really should
cough relief morphine sulfate take it.
Take oral form with food
Monitor for cough 23. Before administering meperidine (Demerol) to a
suppression client, which assessment is most important for
Smoking can reduce the nurse to make?
pain relief
1. Apical pulse rate.
Cautious use with client
on MAO inhibitor 2. Respiratory rate.
3. Blood pressure.
Hydromorphone Moderate to Take oral form with food
(Dilaudid) severe pain Mix with 5 mL of sterile 4. Level of consciousness.
water or normal saline
for IV use 24. Which adverse effect would the nurse expect to
Smoking reduces pain observe in a client receiving a narcotic
relief analgesic?
1. Urine retention.
Meperidine Moderate to Take oral form with food
(Demerol) severe pain, PO dose < 50% as 2. Diarrhea.
preoperative effective as parenteral 3. Hypoglycemia.
medication IM preferred route for 4. Hypertension.
duplicate doses
Methadone Severe pain, IM preferred route
(Dolophine) narcotic
withdrawal Answers and Rationales
Oxycodone Moderate to Monitor liver and blood
Hydrochloride severe pain studies
Give oral form with food 22. 3. Narcotic drugs should be taken before pain
(Percocet,
Percodan) High abuse potential becomes intense so the client can receive the
fullest analgesic effects. By adhering to this, the
Oxycontin Moderate to Monitor respirations client will have good pain control and will not
severe pain Take with food be requesting additional doses.
Not intended for use as
an as needed analgesic 23. 2. Respiratory rate needs to be assessed before
(See also oxycodone) giving the client a narcotic as narcotics can have
a life-threatening effect.

24. 1. An adverse side effect of narcotic analgesics


is urinary retention. You would also monitor
Sample Questions constipation, hyperglycemia, and
hypotension.

22. A client is 1 day post-op. She tells the nurse that


shes worried she will become addicted to NARCOTIC ANTAGONISTS
painkillers so she wants to wait for the pain to
become very intense before she takes anything. A. Prototype: naloxone hydrochloride (Narcan)
What is the best response by the nurse? 1. Action. Occupies opiate receptor sites and
prevents or reverses effects of agonists such as
1. Meperidine (Demerol) is very addicting. You
morphine sulfate.
shouldnt request pain medication unless you
2. Use. Postoperative respiratory depression
really need it.
caused by narcotics, therapy in suspected or
2. Meperidine (Demerol) is not appropriate confirmed narcotic overdose.
for you. Ill ask the physician to order a 3. Adverse effects. Excess dosage in narcotic
non-narcotic pain medication for you. depression: hypertension, tremors, reversal of

2 28 NCLEX-RN Review
53155_02_Ch02_p007-110.qxd 2/21/09 10:48 AM Page 29

analgesia, hyperventilation, increases PTT. In SEDATIVES AND HYPNOTICS


too-quick reversal: nausea, vomiting; sweating,
tachycardia. A. Prototype for the Barbiturates: phenobarbital
4. Nursing implications sodium (Luminal)
a. Emergency resuscitative equipment needs 1. Action. Hinders movement of impulses from
to be available. the thalamus to the brain cortex, thus creating
b. Monitor VS, especially respirations. depression in the CNS, which can range from
c. Monitor client closely as activity of some mild to severe. Considered a long-acting
narcotics last longer than that of naloxone barbiturate.
hydrochloride (Narcan). 2. Use. Sedation, hypnosis, seizure disorders.
d. Monitor surgical clients for bleeding. 3. Adverse effects. Dizziness, ataxia, drowsiness,
e. Withdrawal symptoms will be seen in hangover, anxiety, irritability, hand tremors,
client addicted to narcotics. vision difficulties, insomnia; bradycardia, low
B. Related drug: naltrexone (Trexan) blood pressure; chest tightness, wheezing,
1. Used in narcotic detoxification and to prevent apnea, respiratory depression; nausea, vomiting,
readdiction in drug users. constipation; hypersensitivity reactions.
2. Initial treatment of overdose is naloxone 4. Nursing implications
hydrochloride (Narcan) and then naltrexone a. High doses for long periods of time can
(Trexan) is given as it has a longer duration of cause physical dependence.
action. b. Drug has extended half-life so steady
C. Related drug: nalmefene (Revex) plasma level may take 34 weeks of
1. Used to treat opioid overdose. medication before occurring.
2. Can be given IM or SC if IV access is lost. c. Give reconstituted solutions within
30 minutes of mixing.
d. Give IM deeply in large muscle mass and
observe IM sites.
Sample Questions e. IV administration: client must be
monitored constantly: take VS frequently;
have emergency equipment available;
25. Which of the following observations by the monitor for extravasation at infusion site.
nurse indicates an adverse effect of naloxone f. Pill can be crushed and mixed with food or
hydrochloride (Narcan) on the client? fluid.
1. Hypotension. g. Will cause restlessness in client in pain.
2. Bradycardia. h. Geriatric, pediatric, and debilitated clients
can have paradoxical reactions.
3. Tremors.
i. Monitor liver and blood studies with long-
4. Increased urine output. term therapy.
j. Schedule IV drug under Federal Controlled
26. Which client should have cautious use of Substances Act.
naloxone hydrochloride (Narcan)? k. Many drug interactions.
1. Meperidine (Demerol) addict. 5. Discharge teaching
2. Client with brittle diabetes. a. Drowsiness occurs in first few weeks of
3. Client with asthma. therapy and will decrease.
b. Avoid potentially dangerous activities
4. Postoperative radical neck. until response to drug is known.
c. Alcohol is prohibited.
d. Do not alter dosing schedule or amount.
e. Do not stop abruptly.
Answers and Rationales f. Teratogenic. Prolonged use necessitates
alternative contraception methods if taking
birth control pills.
25. 3. Tremors are an adverse effect of naloxone g. Do not keep at bedside due to potential for
hydrochloride (Narcan) and indicate an overdosing.
overdose of the drug. B. Related drugs. See Table 2-7.
Note: actions, adverse effects, and nursing
26. 1. If naloxone hydrochloride (Narcan) is given to implications are similar to phenobarbital sodium
a client who is addicted to narcotics, the client (Luminal).
will experience withdrawal syndrome. Thus,
narcotic addicts should use this drug cautiously.

DRUGS AND NURSING IMPLICATIONS

2 29
53155_02_Ch02_p007-110.qxd 2/21/09 10:48 AM Page 30

5. Discharge teaching
Table 2-7 Barbiturates a. Avoid alcohol.
b. Avoid potentially dangerous activities
Drug Use until response to drug is known.
Amobarbital sodium (Amytal) Sedation, hypnosis, c. Smoking decreases drug effect.
(intermediate acting) preoperative medication, d. Avoid abrupt discontinuance of drug.
labor, chronic and acute e. If pregnant or planning a pregnancy,
seizures discuss ending drug therapy with
physician.
Butabarbital sodium (Butisol) Sedation, hypnosis,
f. Long-term high dose use can cause
(intermediate acting) preoperative medication
physical dependence.
Pentobarbital sodium Sedation, hypnosis, D. Related drugs. See Table 2-8.
(Nembutal) (short acting) preoperative medication Note: actions, adverse effects, and nursing
Secobarbital sodium (Seconal) Hypnosis, preoperative implications are similar to diazepam (Valium).
(short acting) medication E. Other sedative/hypnotic drugs
Thiopental sodium (Pentothal Induction of general 1. Drugs which produce sedation and/or sleep
Sodium) (ultrashort acting) anesthesia, acute seizures, that are not barbiturates or benzodiazepines.
decrease of intracranial 2. Examples
pressure in neurosurgery, a. Buspirone (BuSpar): used for anxiety
narcoanalysis and disorders.
narcosynthesis in psychiatry b. Ethchlorvynol (Placidyl): used for short-
term insomnia (lasting 1 week).
c. Zolpidem (Ambien): used for short-term
C. Prototype for Benzodiazepines (antianxiety insomnia (lasting 1 week).
agents): diazepam (Valium)
1. Action. Not fully understood. Depresses the
CNS at the limbic system and reticular
formation.
Sample Questions
2. Use. Anxiety disorders, acute alcohol
withdrawal, muscle relaxant, tetanus, 27. The nurse would monitor the client who has
convulsive disorders, preoperative medication.
been given pentobarbital sodium (Nembutal) for
3. Adverse effects. Dry mouth, constipation,
which adverse effects?
urinary retention, photophobia and blurred
vision; for other effects see adverse effects 1. Tachycardia.
listed under pentobarbital sodium (Luminal). 2. Hypertension.
4. Nursing implications 3. Dry mouth.
a. Adverse effects typically dose related. 4. Anxiety.
b. Two weeks of therapy needed before steady
plasma levels seen.
c. Tablet can be crushed.
d. Do not mix with other drugs in the same
syringe.
e. Cautious IV use as drug can precipitate in Table 2-8 Benzodiazepines
IV solutions.
f. IM should be deep into large muscle mass; Drug Use
rotate IM sites.
Alprazolam (Xanax) Anxiety
g. Parenteral administration can cause low
Clorazepate (Tranxene) Anxiety
blood pressure, increased heart rate,
Flurazepam (Dalmane) Hypnosis
muscle weakness, and respiratory
Midazolam (Versed) Preoperative medication,
depression.
conscious sedation
h. For extended therapy, monitor liver and
Triazolam (Halcion) Hypnosis
blood studies.
Chlordiazepoxide (Librium) Anxiety, alcohol withdrawal
i. Adverse effects more likely in geriatric
Clonazepam (Klonopin) Seizures, restless leg
clients.
syndrome, panic attacks
j. Monitor I&O.
Lorazepam (Ativan) Anxiety, preoperative
k. Schedule IV drug under Federal Controlled
medication
Substances Act.

2 30 NCLEX-RN Review
53155_02_Ch02_p007-110.qxd 2/21/09 10:48 AM Page 31

28. Which group should not receive barbiturates? e. Discharge teaching


1. Children <5 years of age. 1) Relate signs of fatigue, dry skin, deepening
voice with extended therapy as drug can
2. Pregnant women.
mask decreased thyroid reserve.
3. Adults prone to seizures. 2) Report jaundice as drug is metabolized
4. Adults with bleeding ulcers. in the liver and liver dysfunction
causes elevated blood levels of drug.
3) Abrupt drug withdrawal can cause
seizures or status epilepticus.
Answers and Rationales 4) Withdraw gradually.
5) Avoid potentially dangerous activities
until drug response is known.
27. 4. Anxiety is an adverse side effect. 6) Alcohol use can cause drug toxicity.
7) Cautious use in pregnancy and lactation.
28. 2. Barbiturates are teratogenic and are
8) Flu shot during therapy can increase
contraindicated for pregnant women. seizure occurrence.
9) Family members need instruction in
care of client during a seizure.
ANTICONVULSANTS 4. Succinimides (ethosuximide [Zarontin]): used
in treatment of absence seizures.
A. Several categories of drugs are used to treat seizure 5. Acetazolamide (Diamox): diuretic used as an
activity. Each group will be addressed and there adjunct or alone in treatment of absence, tonic-
will be no prototype. clonic, or myoclonic seizures.
1. Barbiturates (phenobarbital) 6. Carbamazepine (Tegretol)
a. Used for generalized and absence a. Chemically similar to tricyclic
seizures. antidepressants.
b. Refer to discussion under sedatives and b. Used in treatment of tonic-clonic, complex
hypnotics on barbiturates. partial, and mixed seizures.
2. Benzodiazepines (Diazepam [Valium]) 7. Adjunct anticonvulsants
a. Drug of choice for status epilepticus. Also a. Valproic acid (Depakene)
used for absence seizures. 1) Used in treatment of absence seizures.
b. Refer to discussion under Sedatives and 2) Low incidence of side effects as
Hypnotics on Benzodiazepines. compared to other anticonvulsants.
3. Hydantoins (phenytoin [Dilantin]) b. Felbamate (Felbatol): used to treat Lennox-
a. Action. Prevents dissemination of electrical Gastaut Syndrome in children and partial
discharges in motor cortex area of the brain. seizures.
b. Use. Tonic-clonic and complex partial c. Lamotrigine (Lamictal): used to treat
seizures, status epilepticus, prevention of partial seizures.
seizures that accompany neurosurgery. d. Gabapentin (Neurontin): used to treat
c. Adverse effects. Confusion, slurred speech, partial seizures.
slow physical movement; blood dyscrasias;
nausea, vomiting, constipation; gingival
hyperplasia; hirsutism; rash; acne;
hypotension, circulatory collapse, cardiac Sample Questions
arrest.
d. Nursing implications
1) May take 710 days to achieve 29. The nurse should teach clients to watch for
therapeutic serum concentration. which common adverse effect of phenytoin
2) Tablet can be crushed and should be (Dilantin)?
mixed with food or fluid. 1. Alopecia.
3) Suspension must be shaken well.
4) Can turn urine pink, red, or red-brown. 2. Edema.
5) IM route not recommended. 3. Gingival hyperplasia.
6) Do not mix with other drugs. 4. Hallucinations.
7) Monitor CBC, liver, thyroid, and urine
tests. 30. What is the drug of choice for status epilepticus?
8) Gingival hyperplasia seen most often 1. Phenytoin (Dilantin).
in children and adolescents. 2. Carbamazepine (Tegretol).
9) Stop drug immediately if a measles-
3. Phenobarbital (Luminal).

2
like rash occurs.
4. Diazepam (Valium).

DRUGS AND NURSING IMPLICATIONS 31


53155_02_Ch02_p007-110.qxd 2/21/09 10:48 AM Page 32

b. Used for muscle spasms associated with


Answers and Rationales cerebral vascular accident, spinal cord
injury, cerebral palsy, and multiple
sclerosis. Also given intravenously for
29. 3. Gingival hyperplasia is a common adverse malignant hyperthermia.
effect of phenytoin (Dilantin) seen most often in c. Adverse effects. Drowsiness; malaise,
children and adolescents. diarrhea; hepatotoxicity (in extended
use at high doses).
30. 4. Diazepam (Valium) is the drug of choice for d. Nursing implications. Capsule can be
status epilepticus. opened and contents can be mixed with
juice or other liquid, monitor ambulation,
liver and kidney function tests should be
MUSCLE RELAXANTS monitored, monitor IV site for
extravasation, should be withdrawn after
A. Various drugs used to treat musculoskeletal 45 days if no improvement has been
problems. Three common drugs will be discussed. seen.
There will be no prototype drug. 4. Cyclobenzaprine (Flexeril)
1. Baclofen (Lioresal) a. Acts on brainstem to reduce tonic somatic
a. Mechanism of action not known but drug muscle activity.
inhibits nerve activity in the spinal cord, b. Used for managing acute and painful
thus decreasing spasms of skeletal muscle spasm.
muscles. c. Adverse effects. Dizziness, drowsiness,
b. Used in multiple sclerosis and spinal cord confusion, fatigue, headache, nervousness,
injuries. blurred vision, arrhythmias, constipation,
c. Adverse effects. CNS depression ranging dyspepsia, nausea, unpleasant taste,
from sedation to coma and seizures; urinary retention.
urinary frequency; hirsutism, d. Nursing implications. Take with food,
photosensitivity, acne-like rash; nausea CNS depressant, drowsiness is common.
and vomiting.
d. Nursing implications. May be taken with
food, monitor ambulation, depressant
effects will be increased if mixed with
other CNS depressants, monitor VS, client
Sample Questions
should avoid potentially dangerous
activities until response is known, do not
31. Which statement indicates a need for more
abruptly withdraw.
teaching about baclofen (Lioresal) by the nurse?
2. Carisoprodol (Soma)
a. Mechanism of action not known but is 1. Ill take my pills with my meals.
believed due to drugs central depressant 2. Ill drive myself to work each day.
action. 3. I wont have wine with dinner anymore.
b. Used in cerebral palsy, muscle stiffness, 4. Ill use sunscreen when I go outside.
and spasm found in various
musculoskeletal disorders.
c. Adverse effects. Sedation; headache;
syncope, tachycardia, postural Answers and Rationales
hypotension; nausea, vomiting; hiccups,
allergic reactions.
d. Nursing implications. May be taken with 31. 2. Due to the depressant effects of baclofen
food, drowsiness is common effect and (Lioresal), the client should not engage in any
client may need dosage reduced, client potentially dangerous activities until the client
should not take alcohol or other CNS response to the drug is known.
depressants, do not abruptly stop, allergic
reactions usually occur from the first to
the fourth dose. ANTIPSYCHOTIC AGENTS
3. Dantrolene (Dantrium)
a. Interferes with calcium release from the A. Prototype: phenothiazines (Chlorpromazine
muscle, which causes a decrease in muscle [Thorazine])
contraction. 1. Action. Not fully understood, but is thought to
block dopamine receptors in the brain.

2 32 NCLEX-RN Review
53155_02_Ch02_p007-110.qxd 2/21/09 10:48 AM Page 33

Chlorpromazine causes a sedative effect h. With long-term therapy, drug is gradually


known as a neuroleptic effect and reduced before discontinuing therapy.
antipsychotic effect. Also causes an anti- i. Incompatible with many drugs. Note: also
emetic effect by depressing the chemoreceptor review the anticholinergic drug atropine
trigger zone (CTZ). Potentiates the effects of sulfate for nursing implications and
other CNS depressant drugs. Blocks peripheral discharge teaching.
acetylcholine (Ach) receptors, histamine (H1) B. Related drugs. See Table 2-9.
receptors, and alpha-adrenergic receptors.
These actions cause anticholinergic, alpha-
anti-adrenergic, and antihistamine effects that
can produce adverse effects. Table 2-9 Phenothiazines
2. Use. Management of acute and chronic
schizophrenia, manic phase of bipolar Drug Use Comments
disorder, management of nausea and vomiting,
control of excessive anxiety before surgery, Promethazine Pre- and Rarely causes
treatment of acute intermittent porphyria, (Phenergan) postoperative extrapyramidal
treatment of intractable hiccups, tetanus. sedation symptoms
3. Adverse effects. Extrapyramidal symptoms; Prophylaxis for
dizziness, sedation, seizures; orthostatic nausea,
hypotension, tachycardia, arrhythmias; vomiting, motion
cholestatic jaundice; agranulocytosis; sickness
photosensitivity; anticholinergic effects: Red, Adjunct to
Hot, Dry, Blind, Mad; urticaria; changes in analgesics
menses and libido; potentiates CNS effects of Allergic conditions
narcotic analgesics, sedatives, hypnotics, and Thioridazine Psychotic disorders Rarely causes
alcohol; neuroleptic malignant syndrome. (Mellaril) Short-term use in extrapyramidal
4. Nursing implications depression symptoms
a. Monitor blood pressure (standing, lying, Attention deficit
and sitting), pulse, respirations, and I&O. disorder
b. Wear gloves when handling parenteral or
liquid form to prevent contact dermatitis. Fluphenazine Psychotic disorders Clients at great risk
c. Give deep IM injection into gluteal muscle (Prolixin) for extrapyramidal
and massage well. (Prolixin symptoms
d. Monitor client for extrapyramidal Decanoate) Most potent
symptoms, which can occur 160 days phenothiazine
after therapy is begun. Tardive dyskinesia Haloperidol Psychotic disorders Clients at great risk
can occur several months or years after (Haldol) Tourettes for extrapyramidal
therapy. syndrome symptoms
e. Monitor CBC, liver function studies, Short-term Causes less sedation
glucose levels, and urinalysis and treatment in and hypotension
encourage periodic ocular examinations. hyperactive
f. Supervise ambulation to prevent falls until children
client develops a tolerance.
g. Protect drug from light. Mesoridazine Schizophrenia Increased sedation
5. Discharge teaching (Serentil) Acute/chronic but less risk of
a. Take with food or milk to decrease GI alcoholism extrapyramidal
distress. symptoms
b. Take drug at bedtime. Thiothixene Psychotic disorders Clients at great risk
c. Wear protective clothing and/or sunscreen (Navane) for extrapyramidal
before exposure to sun. symptoms
d. With initial therapy, change positions
Pimozide Tourettes Decreased sedation
gradually to reduce orthostatic
(Orap) syndrome but increased risk of
hypotension.
extrapyramidal
e. Report fever, sore throat to physician.
symptoms
f. Stop or reduce cigarette smoking, as this
shortens the half-life and higher doses may Molindone Psychotic disorders Clients at low risk
be needed. (Moban) for sedation and
g. Mix liquid form in juice, water, milk, or extrapyramidal
baby food. symptoms

DRUGS AND NURSING IMPLICATIONS

2 33
53155_02_Ch02_p007-110.qxd 2/21/09 10:48 AM Page 34

35. 1. Haloperidol (Haldol) is also used to treat


Sample Questions Tourettes syndrome and causes fewer sedative
effects than other phenothiazines.

32. The psychiatrist orders chlorpromazine Antipsychotic Agents (Continued)


(Thorazine) IM. If the medication is not properly
handled, what effect could this have on the C. Prototype: lithium (lithium carbonate [Eskalith])
nurse? 1. Action. Exact mode of action unknown.
1. Skin discoloration. Thought to alter neurotransmitters in CNS that
produce antidepressant and antimanic effects.
2. Skin irritation. 2. Use. Treatment and prophylaxis of manic
3. Headache. phase of bipolar disorder.
4. Dizziness. 3. Adverse effects. Confusion, restlessness,
fatigue, weakness, hand tremors; arrhythmias,
33. Which of the following interventions should be circulatory collapse, palpitations,
included in a care plan concerning hypotension; blurred vision; dry mouth, thirst,
chlorpromazine (Thorazine) therapy? weight gain; nausea, diarrhea; leukocytosis.
1. Supervise ambulation. 4. Nursing implications
a. Monitor serum lithium levels (blood tests
2. Take a hot bath to reduce agitation.
usually done monthly).
3. Restrict fluid intake to prevent edema. b. Monitor for lithium intoxication.
4. Discontinue drug if sedation occurs. c. Treatment for lithium intoxication
includes IV therapy with normal saline,
34. Which phenothiazine is used specifically diuretics, and hemodialysis.
as an antiemetic and rarely causes d. Monitor thyroid function studies
extrapyramidal symptoms? periodically.
1. Thioridazine (Mellaril). e. May take 12 weeks to achieve therapeutic
2. Fluphenazine (Prolixin). effects.
5. Discharge teaching
3. Promethazine (Phenergan). a. Drink 2.53 liters of fluid per day to relieve
4. Chlorpromazine (Thorazine). thirst and dry mouth.
b. Maintain sodium intake of 610 g daily to
35. Which antipsychotic agent is also used to treat reduce lithium toxicity.
Tourettes syndrome and causes less sedation c. Take with food to decrease GI distress.
than other phenothiazines? d. Do not drive or operate machinery until
1. Haloperidol (Haldol). drug response established.
2. Thioridazine (Mellaril). e. Report to physician: nausea, vomiting,
edema, weight gain, tremors, and
3. Fluphenazine (Prolixin).
drowsiness (may be signs of lithium
4. Chlorpromazine (Thorazine). toxicity or hypothyroidism).
f. Record weight on a weekly basis.
D. Related drugs: lithium citrate (Cibalith-S):
available in liquid form
Answers and Rationales

32. 2. Handling the parenteral or liquid forms of


chlorpromazine (Thorazine) may cause contact
Sample Questions
dermatitis, so gloves should be worn.
36. Which of the following interventions should the
33. 1. Chlorpromazine (Thorazine) during initial use nurse stress while a client is on lithium therapy?
can cause orthostatic hypotension; ambulation
1. Weigh self once a month.
should be supervised to prevent falls until
tolerance develops. 2. Restrict fluid intake to prevent edema.
3. Do not restrict sodium intake.
34. 3. Promethazine (Phenergan) is used as an 4. Avoid eating cheese and bananas and
antiemetic and rarely causes extrapyramidal drinking wine.
symptoms.

2 34 NCLEX-RN Review
53155_02_Ch02_p007-110.qxd 2/21/09 10:48 AM Page 35

38. When should the client expect to see


Answers and Rationales improvement of depression while on
imipramine (Tofranil)?
1. 12 days.
36. 3. Clients need to maintain sodium intake
2. 1 week.
(usually 610 g daily) to prevent lithium
toxicity. 3. Several weeks.
4. Immediately after first dose.
Antipsychotic Agents (Continued)
E. Prototype: tricyclic antidepressants (imipramine
[Tofranil]) Answers and Rationales
1. Action. Structurally related to phenathiazines.
Blocks reuptake of the neurotransmitters
norepinephrine and serotonin at the neuronal 37. 3. Imipramine (Tofranil) can cause drowsiness;
membrane, which increases and prolongs the the client should avoid driving or operating
response of the neurotransmitters. machinery.
2. Use. Endogenous and reactive depression;
childhood enuresis. 38. 3. It takes several weeks (24 weeks)
3. Adverse effects. Sedation, confusion; before clients may see improvement of
anticholinergic effects; orthostatic depression.
hypotension, arrhythmias; clients recovering
from an acute MI should not take drug; blood
dyscrasias; extrapyramidal symptoms; Antipsychotic Agents (Continued)
gynecomastia; jaundice.
4. Nursing implications G. Prototype: monoamine oxidase inhibitors (MAO
a. May take 24 weeks to achieve therapeutic inhibitors) (phenelzine [Nardil])
effects. Monitor for suicidal tendencies. 1. Action. Inhibits MAO, which increases
b. Monitor CBC for clients on long-term therapy. neurotransmitter levels (dopamine,
c. Monitor I&O. norepinephrine, serotonin).
d. Drug therapy is discontinued gradually. 2. Use. Neurotic and atypical depression.
5. Discharge teaching 3. Adverse effects. Orthostatic hypotension; dry
Take with food to decrease GI distress mouth, blurred vision, constipation;
Note: review the anticholinergic atropine sulfate hypertensive crisis; liver dysfunction;
for further nursing implications and discharge leukopenia.
teaching as imipramine (Tofranil) has 4. Nursing implications
anticholinergic adverse effects a. Monitor blood pressure while standing,
F. Related drugs sitting, and supine.
1. Amitriptyline (Elavil) b. Interacts with many drugs.
2. Nortriptyline (Aventyl): more useful in elderly c. Monitor I&O.
clients due to fewer anticholinergic effects. d. Therapeutic effectiveness takes 24 weeks.
3. Desipramine (Norpramin) e. Monitor liver function studies, glucose,
4. Doxepin (Sinequan) and CBC.
5. Amoxapine (Asendin) 5. Discharge teaching
a. Avoid foods or beverages containing
tyramine or tryptophan including: caffeine
beverages, soy sauce, red wine, beer,
Sample Questions cheese, yogurt, sour cream, raisins,
bananas, avocado, herring, beef and
chicken liver, Italian green beans.
37. Which of the following is included in client b. Change position slowly.
teaching concerning imipramine (Tofranil)? H. Related drugs
1. Expect to see improvement of depression in 1. Tranylcypromine (Parnate): contraindicated in
23 days. clients over age 60.
2. Isocarboxazid (Marplan)
2. Stop taking drug if dizziness occurs.
3. Do not drive or operate machinery.
4. Take drug on an empty stomach.

DRUGS AND NURSING IMPLICATIONS

2 35
53155_02_Ch02_p007-110.qxd 2/21/09 10:48 AM Page 36

3. Citalopram (Celexa)
Sample Questions 4. Fluvoxamine (Luvox)
5. Escitalopram (Lexapro)
K. Miscellaneous antidepressants
39. Which foods/beverages should be avoided while 1. Bupropion (Wellbutrin)
taking phenelzine (Nardil)? 2. Venlafaxine (Effexor)
1. Cheese. 3. Nefazodone (Serzone)
4. Trazodone (Desyrel)
2. Apples.
5. Clozapine (Clozaril)
3. Pasta. 6. Olanzapine (Zyprexa)
4. Cereal. 7. Risperidone (Risperdal)

Answers and Rationales Sample Questions

39. 1. Foods such as cheese that contain tyramine or 40. Which statement indicates a need for more
tryptophan should be avoided while taking teaching by the nurse concerning fluoxetine
MAO inhibitors to prevent hypertensive crisis. (Prozac) therapy?
1. I will take this medication in the morning.
Antipsychotic Agents (Continued) 2. I will use calamine lotion if I get a skin
rash.
I. Prototype: selective serotonin reuptake inhibitors
(fluoxetine [Prozac]) 3. It will take a month before I feel better.
1. Action. Blocks serotonin reuptake and 4. I will check with my physician before I take
increases transmission at serotonergic any other medications.
synapses.
2. Use. Major depression; obsessive-compulsive
disorder.
3. Adverse effects. CNS stimulation, sexual Answers and Rationales
dysfunction, nausea, headache, anorexia,
weight loss, skin rash.
4. Nursing implications 40. 2. A skin rash resulting from use of fluoxetine
a. Can take up to 4 weeks to achieve (Prozac) indicates an allergic reaction and
therapeutic effects. should be reported to the physician
b. Interacts with warfarin (Coumadin). immediately.
c. Cannot be combined with monoamine
oxidase inhibitors.
5. Discharge teaching
a. Take in the morning.
b. Report skin rash immediately.
J. Related drugs
1. Paroxetine (Paxil)
2. Sertraline (Zoloft)

2 36 NCLEX-RN Review
53155_02_Ch02_p007-110.qxd 2/21/09 10:48 AM Page 37

Autonomic Nervous System Drugs

4. Nursing Implications
ADRENERGIC DRUGS a. Use great caution in preparing and
A. Prototype calculating doses as this is a potent drug.
Adrenergic drugs are divided into two groups, b. Tolerance occurs with extended use.
direct-acting and mixed-acting. The direct-acting c. Solutions should be clear and colorless
contain most of the adrenergic drugs. (except suspension for injection). Protect
solutions from light, heat, and freezing.
d. Suspension for injection must be shaken
Direct-Acting Adrenergics well.
e. Rotate SC sites and monitor for necroses.
Nonselective (Alpha and Beta) Agonists
f. Have a fast-acting alpha-adrenergic blocker
Prototype: Epinephrine (Adrenalin Chloride) such as phentolamine (Regitine) or
1. Action. Epinephrine (Adrenalin Chloride) vasodilator such as nitrite available for
has the same actions stimulated as the excessive hypertensive reaction.
sympathetic nervous system. It increases the g. Have an alpha-adrenergic blocker available
force of myocardial contraction; increases for pulmonary edema.
systolic blood pressure, cardiac rate and h. Have a beta-adrenergic blocker available
output; relaxes bronchial smooth muscle; for cardiac arrhythmias.
inhibits histamine release; increases tidal i. Monitor VS.
volume and vital capacity; prevents insulin 5. Discharge Teaching
release and raises blood sugar; prevents a. For inhalation products: do not exceed
uterine contractions and relaxes uterine recommended dosage; take drug during
smooth muscle; lowers intraocular pressure second half of inspiration, take second
and decreases formation of aqueous humor; inhalation 35 minutes after first dose.
constricts arterioles in kidneys, mucous b. For nasal solutions: do not use for more
membranes, and skin; and dilates blood than 35 days; burning and stinging may
vessels in skeletal muscle. occur initially but are transient.
2. Use. Treatment of anaphylaxis and c. For ophthalmic solution: slight stinging
bronchospasm, cardiac resuscitation, control or may occur initially but is usually
prevention of low blood pressure during spinal transient; headache and browache are
anesthesia, lengthening effects of local also transient.
anesthesia, promotion of mydriasis, treatment d. Do not take any OTC medications without
of acute hypotension. physician approval.
3. Adverse Effects. Systemic: anxiety, Prototype: norepinephrine bitartrate (Levophed)
headache, fear, arrhythmias, hypertension, 1. Action. Norepinephrine bitartrate (Levophed) is
cerebral/subarachnoid hemorrhage, an alpha and beta-1 receptor agonist and has no
hemiplegia, pulmonary edema, insomnia, effect on beta-2 receptors. Its biggest action is
anginal pain in clients with angina pectoris, seen on the cardiovascular system, where the
tremors, vertigo, sweating, nausea, vomiting, following happens: an increase in total
agitation, disorientation, paranoid peripheral resistance (vasopressor response); and
delusions; prolonged use at high doses increased force, rate, and impulse conduction of
causes increased serum lactic acid levels, the heart, which is usually overridden by
metabolic acidosis, and increased blood activation of baroreceptors, thus causing
glucose. Local injection: necrosis at sites bradycardia. Other actions are mydriasis and
when injections are repeated. Nasal elevated blood glucose and insulin.
solution: stinging and burning locally, 2. Use. Revives blood pressure in acute
rebound congestion. Ophthalmic solutions: hypotensive states (sympathectomy, spinal
stinging on initial use, eye pain, headache, anesthesia, poliomyelitis, septicemia, blood
browache, blurred vision, photophobia, transfusion, drug reactions); adjunct in
problems with night vision, pigment treatment of cardiac arrest.
deposits in conjunctiva, cornea, and eyelids 3. Adverse effects. Bradycardia; cardiac
with prolonged use. arrhythmias; headache.

DRUGS AND NURSING IMPLICATIONS

2 37
53155_02_Ch02_p007-110.qxd 2/21/09 10:48 AM Page 38

4. Nursing implications c. Phentolamine (Regitine) should be


a. Do not mix drug in 100% saline solutions available for hypertensive crisis seen in IV
(NS) as oxidation will occur. Mix in 5% administration.
dextrose solution or 5% dextrose in saline d. Levodopa (L-Dopa) is used to decrease
solution. excess mydriatic effect.
b. Give into large vein to prevent e. If systemic adverse effects are seen from
extravasation. nasal and eye use, stop drug and notify
c. Do not infuse in femoral vein in elderly physician.
clients or those with occlusive vascular f. Apply pressure to lacrimal sac of eye
disease. during and for 12 minutes after
d. Check blood pressure every 2 minutes after administration of eye drops.
start of infusion until desired blood g. Incompatible with butacaine, oxidizing
pressure is attained; then check blood agents, ferric salts, metals, and
pressure every 5 minutes if infusion alkalies.
continued. h. Wash hands after handling drug as blurred
e. Monitor IV site for extravasation. vision and unequal pupil size can result if
f. Have phentolamine (Regitine) available drug-contaminated finger rubs eye.
in case of extravasation. 510 mg of 5. Discharge teaching
phentolamine (Regitine) in 1015 mL a. Client should not change dose in
of saline should be infiltrated into any way.
area. b. If drug has been taken for 5 days without
g. Drug solution should be clear and relief, notify physician.
colorless. c. Clear nasal passages before using nasal
preparations.
Selective Alpha Agonists d. Wear sunglasses after eye administration if
eyes sensitive to light.
Prototype: phenylephrine (Neo-Synephrine) e. Call physician if eye sensitivity lasts more
1. Action. Phenylephrine (Neo-Synephrine) than 12 hours after drug has been given.
produces vasoconstriction and increased blood f. Ophthalmic solutions can stain contact
pressure. Topical application produces lenses.
vasoconstriction of mucous membranes. g. Tips and droppers of nasal solutions
Application to eye causes mydriasis and should be cleaned with hot water after
vasoconstriction and promotes flow of each use.
aqueous humor. h. Do not touch droppers of eye solutions.
2. Use. Stabilizes blood pressure during 6. Related drugs
anesthesia; vascular failure in shock; subdues a. Methoxamine (Vasoxyl): used for treating
paroxysmal supraventricular tachycardia; acute hypotension seen during surgery. It
rhinitis of allergy and common cold; is given IV for immediate effect or IM for
sinusitis; wide-angle glaucoma; longer lasting effects.
ophthalmoscopic examination or surgery; b. Agents found in OTC cough, cold, and
uveitis. allergy remedies and in eye decongestant
3. Adverse effects. Eye tearing and stinging, products include naphazoline,
headache, browache, blurred vision, oxymetazoline, tetrahydrozoline,
increased sensitivity to light; nasal rebound xylometazoline.
congestion; nasal burning, stinging,
dryness, and sneezing; palpitations, Nonselective Beta (Beta-1 and Beta-2) Agonists
tachycardia, bradycardia (overdose);
hypertension; trembling, sweating, feeling Prototype: isoproterenol (Isuprel)
of fullness in the head; sleeplessness, 1. Action. Isoproterenol (Isuprel) has
dizziness, light-headedness, tingling in cardiovascular actions of vasodilation, which
extremities. decreases diastolic blood pressure and
4. Nursing implications peripheral resistance, and actions of increased
a. For IV infusion, check blood pressure, cardiac output. Other actions are
pulse, and central venous pressure every bronchodilation; raising levels of blood
25 minutes. glucose, insulin, and free fatty acids;
b. IV overdose can cause ventricular and causing release of renin from the
arrhythmias. kidney.

2 38 NCLEX-RN Review
53155_02_Ch02_p007-110.qxd 2/21/09 10:48 AM Page 39

2. Use. Acute heart failure; management of 4. Nursing implications


intraoperative bronchospasm; additive a. Must be administered cautiously as even
treatment in cardiac arrest, AV heart block, small errors can produce deleterious
Stokes-Adams syndrome; treatment of chronic effects.
bronchoconstriction; management of syncope; b. Always dilute drug if not prediluted.
treatment of bronchospasm in COPD and c. Dose must be decreased by 1/10 in clients
asthma. who have been receiving MAO inhibitors.
3. Adverse effects. Restlessness, anxiety, CNS d. Do not mix with other drugs.
stimulation, hyperkinesia, insomnia, tremors, e. Protect drug from light.
irritability, vertigo, headache; arrhythmias, f. Infuse into large vein.
tachycardia, angina, blood pressure changes; g. Monitor for extravasation and have
pulmonary edema, respiratory difficulties; phentolamine (Regitine) available if this
flushing, pallor, sweating; nausea, vomiting, occurs.
heartburn. h. Closely check blood pressure, urine
4. Nursing implications output, and cardiac output.
a. Tolerance can develop with prolonged use. 5. Related drugs: Dobutamine (Dobutrex) is used
b. A beta-adrenergic blocker should be in treatment of acute heart failure. It is given to
available if arrhythmias occur. adults via IV infusion.
c. Client needs continuous ECG monitoring
during IV administration. Selective Beta-2 Agonists
d. IV infusion must be given via infusion
pump with guidelines from the physician. All drugs in this group are similar so there will be
5. Discharge teaching no prototype. Two representative examples will be
a. Client should not alter dosage. mentioned.
b. Inhalation form should be taken during 1. Metaproterenol sulfate (Alupent) is used
second half of inspiration; second to treat bronchial asthma and bronchospasm
inhalation should be taken that accompanies emphysema and bronchitis.
35 minutes later. It is given orally and via metered-dose inhaler
c. Client should not chew or swallow or inhalant solution to adults. Children
sublingual tablets. 60 pounds or less receive syrup orally and
d. Avoid OTC drugs unless approved by children more than 12 years can receive
physician. inhalation therapy.
6. Related drugs a. Adverse effects. CNS stimulation; cardiac
a. Isoxsuprine hydrochloride arrhythmias, tachycardia, palpitations,
(Vasodilan) is used in cerebrovascular changes in blood pressure; respiratory
insufficiency and peripheral vascular difficulties; sweating, pallor, flushing;
disease. It is given to adults IM nausea, vomiting, heartburn.
or PO. b. Nursing implications
b. Ritodrine (Yutopar) is used for 1. Tolerance can develop with
management of preterm labor. prolonged use.
2. Have a beta-adrenergic blocker
available in case of arrhythmia.
Selective Beta-1 Agonists 3. Give inhalant during second half of
inspiration.
Prototype: dopamine hydrochloride (Intropin) 4. Teach clients to not alter dose and not
1. Action. Dopamine hydrochloride (Intropin) to take any OTC drugs without
increases cardiac output and systolic blood physician approval.
pressure. In low doses it reduces renal 2. Terbutaline sulfate (Brethine): refer to data on
vascular resistance, which increases metaproterenol sulfate (Alupent). It is given PO,
glomerular filtration rate and urinary output. SC, and via inhaler to adults and children
2. Use. Corrects hemodynamic imbalance in more than 12 years.
shock caused by myocardial infarction,
trauma, septicemia, congestive heart failure, Mixed-Acting Adrenergics
and open heart surgery.
3. Adverse effects. Tachycardia, palpitations, Prototype: ephedrine (various products)
hypotension, vasoconstriction; nausea, 1. Action. Ephedrines actions are similar to the
vomiting; dyspnea, headache; piloerection. peripheral autonomic effects of

DRUGS AND NURSING IMPLICATIONS

2 39
53155_02_Ch02_p007-110.qxd 2/21/09 10:48 AM Page 40

norepinephrine. The main effects of the drug 42. Which nursing action is contraindicated while
are reduced nasal congestion, increased blood receiving IV dopamine hydrochloride (Intropin)?
pressure, bronchodilation, cardiac stimulation, 1. The nurse will monitor vital signs frequently.
and stimulation of the CNS.
2. The nurse will check the IV infusion site
2. Use. Relief of allergies and mild asthma;
frequently for extravasation.
therapy in shock and hypotension.
3. Adverse effects. Systemic with increased 3. The nurse will infuse the drug via macrodrip
doses: headache, insomnia, nervousness; tubing and will adjust the rate manually.
palpitations, tachycardia, arrhythmias, 4. The nurse will check client extremities for
urinary retention; nausea, vomiting, anorexia; temperature and color.
sweating, thirst. Topical use: burning,
stinging, sneezing, dry nasal mucosa, 43. A client experiences extravasation at the
rebound congestion. Overdose: confusion, insertion site of dopamine hydrochloride
delirium, convulsions, pyrexia, coma; (Intropin) IV. The infusion is stopped. What
hypertension; respiratory depression; should be done next?
paranoid psychosis; auditory and visual 1. Warm compresses should be applied to the IV
hallucinations. site.
4. Nursing implications
a. Parenteral solution must be clear and 2. An ice pack should be applied to the IV site.
should be protected from light. 3. The extremity with the IV site should be
b. Monitor urine output. elevated on two pillows.
c. Clients with cardiovascular problems need 4. The IV site should be infiltrated with
monitoring of cardiac response and blood phentolamine (Regitine).
pressure.
d. Client receiving IV ephedrine needs close 44. When high doses of dopamine hydrochloride
monitoring of vital signs. (Intropin) are given IV for treatment of shock,
5. Discharge teaching what effect would the nurse be looking for?
a. Client should not use nasal decongestant 1. Increased blood pressure.
longer than 5 days.
2. Decreased heart rate.
b. Anxiety reaction can occur with extended
use of systemic ephedrine. 3. Increased respirations.
c. Ephedrine is commonly abused. Client 4. Elevated body temperature.
needs to be aware of adverse effects and
proper use. 45. Which drug produces effects that closely mimic
d. Client should not take any OTC high doses of dopamine hydrochloride (Intropin)?
preparations without consulting 1. Atropine sulfate.
physician. 2. Ephedrine.
e. Insomnia is a common effect and doses
should be spaced accordingly. 3. Isoproterenol (Isuprel).
6. Related drugs: Metaraminol (Aramine): 4. Norepinephrine (Levophed).
used for acute hypotension and can be
given preoperatively to prevent
hypotension; given SC, IM, or IV; given to
adults and children. Answers and Rationales

41. 3. Dopamine hydrochloride (Intropin) at low


Sample Questions doses causes dilation of renal and mesenteric
arteries, which in turn causes increased urine
output.
41. What can the nurse expect if a low dose level of
dopamine hydrochloride (Intropin) is given 42. 3. Dopamine hydrochloride (Intropin) needs to
intravenously? be infused via a minidrip tubing and attached to
1. A decrease in glomerular filtration an infusion pump for accurate administration.
rate.
43. 4. If extravasation occurs when dopamine
2. A decrease in the force of myocardial hydrochloride (Intropin) is administered, the IV
contractions. site should be infiltrated with phentolamine
3. An increase in urine output. (Regitine) immediately after discontinuing the

2
4. An increase in tactile sensation. infusion.

40 NCLEX-RN Review
53155_02_Ch02_p007-110.qxd 2/21/09 10:48 AM Page 41

44. 1. High doses of dopamine hydrochloride 5. Nursing implications


(Intropin) stimulate alpha-adrenergic activity, a. Take drug at beginning of migraine attack.
which causes increased blood pressure. b. Avoid prolonged use.
c. Give antiemetics for nausea and vomiting.
45. 4. Dopamine hydrochloride (Intropin) given in d. Monitor extremities.
high doses has effects that closely mimic e. Pregnant client should not receive this
norepinephrine (Levophed). drug.
f. Client should not increase dose of drug
without consulting physician.
ADRENERGIC BLOCKING AGENTS g. Use lowest effective dose.

A. Prototype for alpha-adrenergic blocking agents: Beta-Adrenergic Blocking Agents


phentolamine (Regitine)
1. Action. Phentolamine (Regitine) blocks alpha- Note: Refer to Beta Blockers in the section on
1 receptors, thus causing blood vessel dilation; Cardiovascular Drugs.
decreased blood pressure; increased cardiac Timolol maleate (Timoptic) is an optic beta-adrenergic
output; miosis; increased tearing, mucus blocker. It decreases intraocular pressure whether
secretion, gastric acid secretion, and glaucoma is present or not. It also decreases aqueous
gastrointestinal motility. humor formation and increases aqueous humor outflow.
2. Use. Diagnosis of pheochromocytoma; It is used to treat glaucoma and hypertension. It is given
management of hypertensive episodes in orally and via eye drops to adult clients.
pheochromocytoma; treatment of extravasation
from norepinephrine (Levophed) or dopamine
hydrochloride (Intropin); adjunctive therapy
in cardiogenic shock or other situations of Sample Questions
decreased cardiac output.
3. Adverse effects. Hypotension, orthostatic
hypotension; MI, cerebrovascular occlusion 46. A client calls the physicians office and states
(these effects can occur with hypotensive that she has vomited each time she has taken
states that can occur after parenteral ergotamine tartrate (Ergomar). What is the
administration); tachycardia, arrhythmias; nurses best response?
dizziness, weakness, flushing; nausea, 1. Vomiting is a common adverse effect. Tell
vomiting, diarrhea; nasal stuffiness. the physician so he can prescribe an
4. Nursing implications antiemetic for you.
a. For parenteral administration client must
be in supine position. Blood pressure and 2. Stop taking the drug immediately. Vomiting
pulse should be checked every 2 minutes is a toxic drug effect.
until stable. 3. You must be allergic to the drug. Notify the
b. Use reconstituted solutions immediately. physician.
c. Have client lie down or put head down if 4. Vomiting is a transient effect and it will
feeling dizzy or light-headed. eventually go away.
d. Treatment for overdose: keep client lying
down with head lowered, supportive 47. Which group of individuals would be excluded
measures, IV infusion of levarterenol from receiving ergotamine tartrate (Ergomar)?
(norepinephrine). 1. Clients with diabetes.
B. Related drug: ergotamine tartrate (Ergomar)
2. Clients with asthma.
1. Along with adrenergic blocking activity, has a
direct stimulatory effect on smooth muscle. It 3. Clients suffering from alcoholism.
also decreases pulsation in cranial arteries and 4. Pregnant women.
has emetic and oxytocic effects.
2. Used for treatment of vascular headaches such 48. Which of the following information would be
as migraines or cluster headaches. excluded from the client teaching about
3. Given to adults via sublingual or inhalation ergotamine tartrate (Ergomar)?
route. 1. Take the drug as soon as you feel a migraine
4. Adverse effects. Nausea, vomiting; numbness, headache coming on.
tingling, and muscle pain in extremities;
2. Have your blood pressure checked routinely
pulselessness in legs; precordial pain;
when taking this drug.
transient tachycardia or bradycardia;
ergotism (ergot poisoning); dependency and 3. Increase the dose as needed to help control

2
abuse. your migraines.

DRUGS AND NURSING IMPLICATIONS 41


53155_02_Ch02_p007-110.qxd 2/21/09 10:48 AM Page 42

4. Tell the physician if you have any numbness C. Prototype: acetylcholinesterase inhibitors:
or tingling in your toes and fingers. Neostigmine (Prostigmin)
1. Action. Neostigmine (Prostigmin) inhibits the
neurotransmitter acetylcholine, which
produces a cholinergic response, and produces
Answers and Rationales reversible cholinesterase inactivation, which
permits a prolonged effect of acetylcholine at
cholinergic synapses.
46. 1. Ergotamine tartrate (Ergomar) has emetic effects; 2. Use. Treatment and diagnosis of myasthenia
vomiting is a common side effect. The client needs gravis; prevention of postoperative abdominal
an antiemetic to help control this problem. distension; treatment and prevention of
postoperative bladder distension; postoperative
47. 4. Ergotamine tartrate (Ergomar) has oxytocic reversal of nondepolarizing muscle relaxants.
effects; it is contraindicated in pregnant women. 3. Adverse effects. Nausea, vomiting, cramping,
diarrhea, increased salivation; muscle tremor
48. 3. Ergotamine tartrate (Ergomar) is a drug that is and weakness; dyspnea, bronchospasm,
abused by clients by altering dosage amount. Only increased bronchial secretions, respiratory
the physician should change the dose of the drug. depression; hypo- or hypertension, arrhythmias,
bradycardia; miosis; cholinergic crisis.
4. Nursing implications
CHOLINERGICS a. Keep atropine and emergency resuscitation
equipment readily available, especially for
A. Prototype: acetylcholine chloride (Miochol) parenteral use.
1. Action. A neurotransmitter that mediates b. Monitor vital signs, breath sounds, I&O.
synaptic activity in the nervous system; c. Report to physician if client does not void
stimulates the vagus nerve and parasympathetic within 1 hour after receiving dose.
nervous system (PNS) causing vasodilation and 5. Discharge teaching
cardiac depression; causes miosis of the eye as a. Encourage client to take drug with food or
it contracts the iris sphincter muscle; contracts milk if GI distress occurs.
and relaxes the urinary bladder, causing b. Instruct client to keep a record of response
micturition. Acetylcholine chloride (Miochol) to drug.
is identical to synthesized acetylcholine (Ach). c. Instruct client to monitor and report
2. Use. To produce miosis in eye surgery. adverse effects.
3. Adverse effects. Systemic absorption: d. Advise client to wear a medic alert bracelet
hypotension, bradycardia; bronchospasm; (for myasthenia gravis).
flushing, sweating. e. Instruct client to cough, breathe deeply,
4. Nursing implications and perform range of motion exercises
a. Reconstitute vial just before use and regularly.
discard unused portion. D. Related drugs
b. Shake vial gently to mix drug. 1. Pyridostigmine (Mestinon, Regonol): used to
B. Related drugs: Bethanechol chloride (Urecholine) treat myasthenia gravis and postoperative
1. Used to treat postoperative urinary retention. reversal of nondepolarizing skeletal muscle
2. See acetylcholine chloride (Miochol); also relaxants. Additional adverse effects: rash;
nausea, vomiting, diarrhea, abdominal thrombophlebitis with IV use.
cramping, dizziness, faintness; cholinergic 2. Edrophonium chloride (Tensilon): used to
crisis can occur with overdose. diagnose myasthenia gravis.
3. Nursing implications 3. Tacrine (Cognex): used to treat mild to
a. Monitor VS, breath sounds, and I&O. moderate Alzheimers disease.
b. PO drug should be given one hour before 4. Pilocarpine (Akarpine): used in open-angle
meals or two hours after meals. glaucoma.
c. Never give IM or IV as drug may cause life- 5. Donepezil (Aricept): used in Alzheimers disease.
threatening effects.
d. Atropine sulfate is antidote.
4. Discharge teaching
a. Encourage client not to drive or operate Sample Questions
heavy machinery while taking drug.
b. Teach client to change positions slowly.
Note: Carbachol (Isopto Carbachol) and 49. The client is prescribed bethanechol chloride
pilocarpine (Almocarpine) are discussed under (Urecholine). What information about this drug

2
Miotics in section on Eye Drugs. is important for the nurse to know?

42 NCLEX-RN Review
53155_02_Ch02_p007-110.qxd 2/21/09 10:48 AM Page 43

1. IM or IV is the preferred route. c. Monitor for constipation and check


2. Bethanechol chloride (Urecholine) should be bowel sounds.
given with food. d. Monitor geriatric clients for CNS
stimulation and heat stroke (infants and
3. Breath sounds should be monitored.
small children should also be monitored
4. Constipation is a frequent adverse effect. for heat stroke).
e. Smaller doses usually are given to geriatric
50. Which drug would be given to treat neostigmine clients due to adverse effects.
(Prostigmin) overdose? 5. Discharge teaching
1. Acetylcholine acetate (Miochol). a. Take drug 30 minutes before meals.
2. Atropine sulfate. b. Eat foods high in fiber and drink plenty of
3. Bethanechol (Urecholine). liquids to overcome constipation.
c. Keep dental appointments as decreased
4. Lidocaine (Xylocaine).
salivation makes clients more prone to
tooth decay.
d. Use good oral hygiene, i.e., rinse mouth,
brush teeth, hard candy, saliva substitute,
Answers and Rationales fluids.
e. Maintain periodic eye appointments to
49. 3. Breath sounds should be monitored to assess monitor for increased intraocular
for wheezing and bronchospasm. pressure.
f. Avoid hot baths and showers, sun, and
50. 2. Atropine sulfate, an anticholinergic, is the heat to prevent heat stroke.
antidote for neostigmine (Prostigmin). g. Change position gradually.
h. Do not drive or operate machinery.
B. Related drugs. See Table 2-10.
ANTICHOLINERGICS
A. Prototype: atropine sulfate
1. Action. Atropine sulfate is a plant alkaloid
derived from the atropa belladonna plant that Table 2-10 Anticholinergics
blocks the neurotransmitter acetylcholine and
inhibits parasympathetic actions. Drug Use Comments
2. Use. To produce mydriasis and cycloplegia for Scopolamine Preanesthetic Place transderm
eye examinations; treat uveitis; preoperative medication scop patch
medication to reduce secretions and (Transderm-scop) behind ear the
bradycardia; treat sinus bradycardia or asystole; used for night before trip.
hypermotility of GU tract; adjunct in treating prophylaxis of Replace patch
asthmatic bronchospasm; GI disorders, peptic motion sickness after three days if
ulcer, GI hypermotility and biliary colic; Mydriatic and more prolonged
antidote for overdoses of parasympathomimetic cycloplegic for effects are
drugs; prevention of adverse effects when eye exam needed.
reversing neuromuscular blockade Irritable bowel
postoperatively with acetylcholine inhibitor; syndrome,
antidote to organophosphate pesticides. diverticulitis
3. Adverse effects. Disorientation, restlessness, Management of
hallucinations, headache, dizziness; postencephalitic
palpitation, hypertension or hypotension, parkinsonism
ventricular tachycardia; blurred vision,
photophobia; suppression of sweating; urinary Glycopyrrolate Preanesthetic Has fewer CNS
hesitancy and retention, constipation; dry (Robinul) medication effects than
mouth; flushed, dry skin. Adjunct in peptic atropine. Do not
4. Nursing implications ulcer disease mix with
a. Do not give to clients with myasthenia therapy barbiturates or
gravis, acute glaucoma, prostatic Reverse alkaline drugs.
hypertrophy. neuromuscular
b. Monitor VS, especially pulse and blood blockade
pressure and I&O.

DRUGS AND NURSING IMPLICATIONS

2 43
53155_02_Ch02_p007-110.qxd 2/21/09 10:48 AM Page 44

2. Use. Treat Parkinsons disease, prevent or


Sample Questions control antipsychotic drug-induced
extrapyramidal tract symptoms.
3. Adverse effects. Note phrase Red, hot, dry,
51. Which of the following effects of atropine sulfate blind, mad; dry mouth; constipation;
would the nurse expect a client to exhibit? tachycardia, hypotension; dizziness, drowsiness,
1. Dry mouth. confusion; decreased bronchial secretions;
blurred vision, photophobia, acute glaucoma;
2. Increased bronchial secretions.
urinary retention; suppression of sweating.
3. Tachycardia. 4. Nursing implications
4. Miosis. a. Drug can be taken before or after meals.
b. See atropine sulfate.
52. Which statement indicates that the client needs c. Drug should be gradually withdrawn.
more teaching concerning the use of atropine 5. Discharge teaching. See atropine.
sulfate? C. Related drugs. See Table 2-11.
1. I will brush my teeth and see my dentist D. Prototype: dopaminergic agents (levodopa
regularly. [Larodopa])
1. Action. Levodopa (Larodopa) is a metabolic
2. I will eat low-residue foods to prevent
precursor of the catecholamine
diarrhea.
neurotransmitter dopamine that readily
3. I will take my medication at least a half hour crosses the blood-brain barrier and restores
before meals. dopamine levels in extrapyramidal centers.
4. I will stay in my air conditioned house on 2. Use. Treat Parkinsons disease (except drug-
hot and humid days. induced Parkinsons).
3. Adverse effects. Anorexia, nausea, vomiting;
orthostatic hypotension, dizziness, headache;
constipation, dry mouth; mydriasis; urinary
Answers and Rationales retention, darkened urine; increase BUN, AST
(SGOT), ALT (SGPT), LDH, bilirubin, alkaline
phosphatase; decreased WBCs, hemoglobin,
51. 1. Atropine sulfate causes dry mouth and and hematocrit, decreased glucose tolerance;
decreases secretions, which is why it is given as blurred vision; muscle twitching,
a preanesthetic. blepharospasm, ataxia, increased hand
tremors; disturbed breathing; confusion,
52. 2. Atropine sulfate can cause constipation; high- anxiety, agitation.
fiber foods and fluids should be encouraged. 4. Nursing implications
a. Monitor vital signs and client for adverse
effects.
ANTIPARKINSON AGENTS b. Monitor client for behavior changes.
c. Monitor CBC, glucose, and kidney and
General Considerations liver function studies.
d. With long-term therapy levodopa
There are 2 major categories of antiparkinsons agents: (Larodopa) may lose its effectiveness and
(1) anticholinergics and (2) dopaminergic agents adjunctive drugs may be used.
A. Antiparkinson drugs control rather than cure
symptoms of Parkinsons. Antiparkinson agents
can cause or worsen other disorders, and clients,
especially the elderly, need to be closely
monitored for adverse effects. Antiparkinson drugs Table 2-11 Antiparkinson Agent (Anticholinergic)
are initiated and discontinued gradually. Drugs
Drug Use Comments
should not be abruptly withdrawn. Antiparkinson
agents are contraindicated in clients with Benztropine Treat Parkinsons disease Longer lasting
glaucoma, prostatic hypertrophy, duodenal ulcers, Mesylate and as adjunct with sedative effects
tachycardia, and biliary obstruction. (Cogentin) trihexyphenidyl HCl and muscle
B. Prototype: anticholinergics (trihexyphenidyl HCl (Artane) relaxation than
[Artane]) This drug is similar to atropine sulfate. Prevent or control trihexyphenidyl
1. Action. Blocks the neurotransmitter drug-induced HCl (Artane)
acetylcholine at certain cerebral synapses extrapyramidal tract
and inhibits parasympathetic responses. symptoms

2 44 NCLEX-RN Review
53155_02_Ch02_p007-110.qxd 2/21/09 10:48 AM Page 45

5. Discharge teaching d. Do not take OTC medications without


a. Restrict foods high in Vitamin B6 consulting physician.
(pyridoxine) (i.e., liver, green vegetables, e. Take drug between meals.
fortified cereals, whole grain cereals). 6. Related drugs. See Table 2-12.
Vitamin B6 reverses therapeutic effects of
levodopa (Larodopa).
b. Change positions gradually.
c. Do not abruptly stop taking drug as sudden Sample Questions
withdrawal can lead to parkinsonian crisis.

53. Which adverse effect would be absent with


Table 2-12 Antiparkinson Agents (Dopaminergic Agents) trihexyphenidyl HCl (Artane) therapy?
1. Dizziness.
Drug Use Comments
2. Dry mouth.
Carbidopa/ Treatment of Carbidopa prevents 3. Diarrhea.
Levodopa Parkinsons metabolism of levodopa
4. Suppression of sweating.
(Senemet) disease and allows more levodopa
for transport to brain. 54. Which statement by the client indicates
Adverse CNS effects may understanding of proper use of levodopa
occur sooner.
(Larodopa)?
Levodopa (Larodopa)
should be discontinued 1. If my symptoms get worse I will stop taking
8 hours before starting this drug.
Senemet. 2. I will not eat liver or boxed cereals.
Bromocriptine Parkinsons Treat Parkinsons. 3. If I have a cold I will take aspirin or a cold
(Parlodel) Treatment of Often given with levodopa remedy.
amenorrhea, (Larodopa) or carbidopa/ 4. I will eat foods low in residue to prevent
galactorrhea levodopa (Senemet). diarrhea.
Female Give with food or milk.
infertility Contraindicated in clients 55. Which instruction should the nurse give to
Suppression of with hypersensitivity to clients taking bromocriptine (Parlodel)?
postpartum ergot alkaloids. 1. Take 1 hour before meals.
lactation Oral contraceptives 2. Do not use birth control pills as
Acromegaly antagonize effects of
contraceptives.
bromocriptine (Parlodel).
Advise client to use 3. This drug causes infertility and use of
another method of contraceptives will not be necessary.
contraception. 4. Adverse effects will be reduced if taken
This drug causes during the day.
increased fertility.
Hypotension is a
frequently seen adverse
effect. Answers and Rationales
Ropinirole Idiopathic Directly stimulates
(Requip) Parkinsons dopamine receptors. 53. 3. Trihexylphenidyl HCl (Artane) is an
disease Given alone in early anticholinergic that can cause constipation, not
stages of Parkinsons diarrhea.
disease and with
levodopa (Larodopa) in 54. 2. Vitamin B6 (pyridoxine) reverses the
later stages. therapeutic effects of levodopa (Larodopa);
Give with food to clients should restrict their intake of foods high
decrease nausea. in vitamin B6 such as whole-grain cereals,
Must be discontinued
fortified cereals, liver, green vegetables.
gradually.
Assess for hypotension 55. 2. Oral contraceptives antagonize the effects of
as dose is increased. bromocriptine (Parlodel). Another method of
Assess liver and renal birth control should be used.
function during treatment.

DRUGS AND NURSING IMPLICATIONS

2 45
53155_02_Ch02_p007-110.qxd 2/21/09 10:48 AM Page 46

Drugs Affecting the Endocrine System

Refer also to Table 4-24, Hormone Functions. p. Insulin analog: insulin lispro (Humalog) is a
synthetic insulin with a faster onset and
shorter duration of action than human insulin.
ANTIDIABETIC AGENTS q. Injections should be given immediately
after mixing two insulins.
A. Prototype: insulin 5. Discharge teaching
1. Action. Hormone that increases glucose a. Available without a prescription (except
transport across cell membranes; transforms insulin injection, concentrated).
glycogen into glucose, prevents breakdown of Prescription is needed for needles and
fats to fatty acids, and inhibits protein syringes (depending on state law).
breakdown. b. Change of insulin brand, type, etc., is done
2. Use. Clients with Type 1 diabetes; Clients with by physician.
Type 2 diabetes not controlled with oral c. In initial period of dosage regulation client
hypoglycemic agents, diet, and exercise; may have visual problems. Should not get
Clients with Type 2 diabetes undergoing lens changes until vision is balanced.
stressful situations: infection or surgery; d. Remove prefilled syringes from refrigerator
pregnant women with diabetes emergency 1 hour before administration.
management of diabetic coma. e. Inject at a 90 angle if you can pinch an
3. Adverse effects. Allergic reaction: local or inch, otherwise inject at a 45 angle.
systemic; hypoglycemia; ketoacidosis. f. Report symptoms of reactions at injection
4. Nursing implications site.
a. There is a difference between insulin g. Know symptoms of hypoglycemic reaction
injection and insulin injection and have some type of fast-acting
concentrated, for which 500 units 5 1 mL. carbohydrate available at all times.
b. Human insulins should only be mixed h. If ill, continue taking insulin and drink
with each other. freely noncaloric liquids. Notify physician
c. IV insulin can be absorbed by the container if diet cannot be followed.
or tubing. i. Monitor blood glucose at home and
d. Stable at room temperature for 1 month. instruct on use.
e. Do not inject cold insulin, causes j. Smoking decreases insulin absorption.
lipodystrophy. k. When traveling, needs to have necessary
f. Drug solution should not be used if supplies.
discolored or contains precipitate. Do not l. Carry a medical identification card.
shake vial. Gently roll (all except regular B. Refer to Table 4-25, Characteristics of Insulin
insulin) vial between palms before drawing Preparations, Unit 4.
up medicine.
g. Check expiration date.
h. When mixing two insulins, rapid-acting
insulin should be drawn up first. Sample Questions
i. Syringe must coordinate with strength of
insulin.
j. Injection sites must be rotated. 56. The nurse is teaching the client about insulin
k. Treat severe hypoglycemic reaction with injections. Which statement is correct?
glucagon or 1050% IV glucose. 1. Insulin needs to be shaken well before being
l. Treat ketoacidosis with IV insulin and IV drawn up into the syringe.
fluids.
m. Diet is prescribed by physician. 2. Long-acting insulins are clear in color.
n. Monitor blood glucose levels. 3. When putting regular and NPH insulin in the
o. Fixed-combination insulins such as 70/30 same syringe, draw regular insulin up first.
insulin are available. Contains 70% NPH 4. NPH is compatible with regular and lente insulin.
and 30% regular insulin. 50/50 insulin is
also available and contains 50% NPH and 57. What information will the nurse instruct the client

2
50% regular insulin. about minimizing local skin reactions to insulin?

46 NCLEX-RN Review
53155_02_Ch02_p007-110.qxd 2/21/09 10:48 AM Page 47

1. Injecting it slowly. e. Monitor and teach symptoms of


2. Always refrigerating it. hypoglycemic reactions and how to treat.
f. Monitor blood and urine glucose levels.
3. Giving it in divided doses.
5. Discharge teaching
4. Bringing it to room temperature before a. Reinforce that drug is not oral insulin
administering. and will control diabetes.
b. Use form of birth control other than oral
58. Which statement by the client indicates a need contraceptives.
for further teaching by the nurse? c. Alcohol can trigger a hypoglycemic
1. I will inject my insulin at a 90 angle. reaction.
2. I will take more insulin when I go to my d. Cover body in sunshine. Use sunscreen.
exercise class. e. Weigh weekly and report progressive gain.
3. I will always have some kind of sugar with f. Carry medical identification.
me in case I have a hypoglycemic reaction. B. Refer to Table 4-26, Oral Hypoglycemic
Agents, Unit 4.
4. I will carefully draw up my doses of
insulin.

Sample Questions
Answers and Rationales
59. The client tells the nurse that his brother has
Type 1 diabetes and he takes insulin. The nurse
56. 3. Regular insulin should be drawn up before
is asked why his brother cannot take an oral
NPH insulin when putting the two together in
antidiabetic agent. The nurse explains that oral
one syringe.
antidiabetic agent. What explanation will the
57. 4. Insulin should be at room temperature before nurse give regarding oral antidiabetic agents in
injecting to decrease occurrence of Type 1 diabetes?
lipodystrophy. 1. He has little or no endogenous insulin that
can be released.
58. 2. Exercise increases glucose use in the body, so 2. He is allergic to oral antidiabetic agents.
a decreased dose of insulin may be needed.
3. He would need so much of an oral
antidiabetic agent that it would be financially
Oral Hypoglycemic Agents prohibitive for him to take one.
A. Prototype: tolbutamide (Orinase) 4. He would have more episodes of
1. Action. Lowers blood glucose concentrations hypoglycemia with oral antidiabetic agents.
by stimulating secretion of endogenous insulin
from beta cells in the pancreas. Increases 60. Tolbutamide (Orinase) should not be taken if a
peripheral sensitivity to insulin. From the person is allergic to what substance?
class of sulfonylureas. 1. Penicillin.
2. Use. Type 2 diabetes: not controlled by diet 2. Insulin.
and exercise, used with insulin in client with 3. Sulfa.
Type 2 diabetes when neither insulin nor oral
hypoglycemic agents work well alone. 4. Caffeine.
3. Adverse effects. Hypoglycemia; increased
61. The client will need more teaching about
chance of cardiovascular disease; anorexia,
tolbutamide (Orinase) if he makes which of the
nausea, vomiting, diarrhea; hemolytic anemia;
allergic skin rashes; photosensitivity; following statements?
inappropriate ADH secretion. 1. I will get a medic alert bracelet that says Im
4. Nursing implications a diabetic taking tolbutamide (Orinase).
a. Tablet can be crushed. 2. Im glad I can still have wine with my
b. Monitor closely during initial therapy. meals.
c. If client stabilized on tolbutamide 3. If I go outside, Ill stay out of the sun or use
(Orinase) is exposed to stress (infection, sunscreen.
surgery), the oral agent may be
discontinued and replaced by insulin. 4. I know that tolbutamide (Orinase) will help
d. Can transfer from one sulfonylurea to control my diabetic condition.

2
another easily.

DRUGS AND NURSING IMPLICATIONS 47


53155_02_Ch02_p007-110.qxd 2/21/09 10:48 AM Page 48

1. Desmopressin (DDAVP)
Answers and Rationales a. Can be given PO, SC, IV, or intranasally;
monitor for extravasation.
b. Keep refrigerated.
59. 1. Oral antidiabetic agents can only work when 2. Lypressin spray (Diapid): given intranasally.
the client has endogenous insulin, which is not
the case in Type 1 diabetes.
Sample Questions
60. 3. Clients who are allergic to sulfa cannot take
tolbutamide (Orinase), which is a
sulfonylurea. 62. Which of the following is the desired response
of vasopressin (Pitressin)?
61. 2. Alcohol combined with an oral hypoglycemic
1. Lower urine specific gravity.
agent can trigger a hypoglycemic reaction.
2. Lower urine output.
3. Treat hypotension.
PITUITARY HORMONES 4. Control polyphagia.

A. Prototype: hormone corticotropin (ACTH) 63. A client complains of GI distress following


(cosyntropin [Cortrosyn]) administration of vasopressin (Pitressin). What
1. Action. Synthetic corticotropin that stimulates instructions should the nurse provide to the client?
corticosteroid release from functional adrenal 1. Eat crackers after taking the dose.
cortex. 2. Take a warm bath to reduce abdominal
2. Use. As a diagnostic test to diagnose adrenal cramping.
insufficiency.
3. Lie down for 30 minutes following
3. Adverse effects. (see Corticosteroids) Cushings
administration.
syndrome if given over a period of time,
hypersensitivity reactions. 4. Drink a glass of water with each dose.
4. Nursing implications. (see Corticosteroids)
Administer deep IM.
B. Prototype: ADH (antidiuretic hormone) Answers and Rationales
(vasopressin [Pitressin]).
1. Action. Hormone released by posterior
pituitary gland that regulates water 62. 2. The goal of vasopressin (Pitressin) is to lower
metabolism and prevents dehydration. Has urine output; replacement for the ADH hormone.
vasoconstrictor effect that elevates blood
pressure. In diabetes insipidus a deficiency in 63. 4. Drinking a glass of water with each dose will
ADH is characterized by polyuria and decrease GI distress.
polydipsia. Vasopressin (Pitressin) acts as a
replacement for ADH.
2. Use. Replacement therapy for diabetes PITUITARY HORMONES (CONTINUED)
insipidus.
D. Prototype: somatotropin (growth hormone)
3. Adverse effects. Hypersensitivity, anaphylaxis;
(somatropin [Humatrope])
water intoxication, hyponatremia; nausea,
1. Action. Somatotropin stimulates growth of
diarrhea, cramping; hypertension; nasal
skin, connective tissue, and long bones.
irritation, headache.
2. Use. Replacement therapy in children with growth
4. Nursing implications
retardation caused by lack of somatotropin.
a. Monitor BP, weight, and I&O.
3. Adverse effects. Hyperglycemia; pain at
b. Vasopressin is available SC, IM, IV, and
injection site; myalgia; headache;
intra-arterially.
hypercalciuria; allergic reactions.
5. Discharge teaching
4. Nursing implications
a. Keep record of I&O, weight.
a. Record height.
b. If URI and use drug intranasally,
b. Monitor blood glucose.
absorption may be affected.
c. Rotate IM sites. Do not give SC.
c. Report sudden changes in output.
d. Store in refrigerator.
d. Drink water with dose to reduce GI
e. Discontinue if epiphyses have fused.
distress.
5. Discharge teaching. Annual bone age
C. Related Drugs
assessment test.

2 48 NCLEX-RN Review
53155_02_Ch02_p007-110.qxd 2/21/09 10:48 AM Page 49

CORTICOSTEROIDS g. Carry Medic Alert card.


h. Follow directions for topical use. Do not
use an occlusive dressing, and apply
General Information ointment sparingly.
A. The adrenal cortex secretes three natural steroids: i. Rinse mouth after using inhaled steroids.
glucocorticoids, mineralcorticoids, and adrenal C. Related drugs
androgens and estrogens. 1. Dexamethasone (Decadron)
1. Glucocorticoids (Cortisol) a. Given IV to treat cerebral edema or allergic
a. Have anti-inflammatory effects. symptoms.
b. Regulate carbohydrate, protein, and fat b. Betamethasone (Celestone)
metabolism. c. Dexamethasone and betamethasone are
2. Mineralcorticoids (Aldosterone, 1030 times more potent than
Desoxycorticosterone) hydrocortisone.
Regulate water and electrolyte metabolism. d. Both drugs can be inhaled to treat asthma.
3. Adrenal androgens and estrogens 2. Methylprednisolone (Solu-Medrol) (has little
a. Supplement sex hormones from gonads. mineralcorticoid action)
b. Corticosteroids suppress immune response a. Prednisone (Deltasone)
and affect all body systems. b. Prednisolone (Delta-Cortef)
B. Prototype: hydrocortisone (Cortisol) Note: All of these drugs are 45 times as
1. Action. Glucocorticoid, mineralcorticoid, and potent as hydrocortisone. Prednisone and
immunosuppressive actions. prednisolone have half the mineral-
2. Use. Replacement therapy for adrenocorticoid corticoid action of hydrocortisone.
insufficiency; anti-inflammatory for many 3. Fludrocortisone (Florinef)
allergic, inflammatory, or immunoreactive a. An oral synthetic mineralcorticosteroid.
disorders. b. Used to treat Addisons disease.
3. Adverse effects. Increased susceptibility to
infection; hypokalemia, hypocalcemia; sodium
and fluid retention; increased appetite, nausea,
peptic ulcer; headache, hypertension,
Sample Questions
congestive heart failure; osteoporosis; acne,
impaired wound healing, hirsutism, skin 64. Which statement should be included in
thinning; ecchymosis, petechiae;
teaching concerning hydrocortisone (Cortisol)
hyperglycemia, impaired glucose metabolism,
therapy?
growth retardation, menstrual disorders;
glaucoma, cataract formation; mental 1. Take aspirin to treat fever.
disturbances, insomnia; thrombophlebitis; 2. Take hydrocortisone (Cortisol) before meals.
masks symptions of infection. 3. Restrict caffeine and alcohol intake.
4. Nursing implications 4. Restrict potassium intake.
a. Observe for mental changes.
b. Monitor BP, weight, I&O, blood glucose, 65. What occurrence is prevented by gradually
and serum potassium. discontinuing hydrocortisone (Cortisol)?
c. IM use: inject deep IM. Do not give SC.
1. Anaphylaxis.
d. Corticosteroid doses are not
interchangeable. 2. Diabetic coma.
e. Corticosteroids are not abruptly 3. Adrenal insufficiency.
withdrawn. Doses are tapered to allow the 4. Cardiovascular collapse.
adrenal gland to function independently.
5. Discharge teaching
a. Take drug before 9 A.M. (This causes less
suppression on the adrenal cortex.) Answers and Rationales
b. Take with food or milk to decrease GI
effects.
c. Never abruptly stop taking. This could 64. 3. Hydrocortisone (Cortisol) can cause GI
precipitate acute adrenal crisis. distress and even lead to a peptic ulcer with
d. Eat foods high in potassium. long-term use. Caffeine and alcohol can
e. Avoid individuals with infections. further increase GI distress and should be
f. Restrict sodium, alcohol, and caffeine intake. restricted.

DRUGS AND NURSING IMPLICATIONS

2 49
53155_02_Ch02_p007-110.qxd 2/21/09 10:48 AM Page 50

65. 3. Adrenal insufficiency can occur with abrupt 2. Use. Management of hyperthyroidism.
removal of corticosteroids. Corticosteroids are 3. Adverse effects. Hypothyroidism;
gradually discontinued so that the adrenal glands agranulocytosis, thrombocytopenia, bleeding;
can begin to secrete corticosteroids independently. nausea, vomiting, loss of taste; rash, urticaria,
skin pigmentation; jaundice, hepatitis;
nephritis.
THYROID HORMONES 4. Nursing implications
a. Take same time daily with respect to
A. Prototype: levothyroxine (Synthroid) meals. Food can change absorption rate.
1. Other various agents used to treat hypothyroid b. Drug response occurs 23 weeks after
conditions include desiccated thyroid: starting drug.
thyroglobulin (Proloid); liotrix (Thyrolar); c. Therapy may last 6 months to several years
liothyronine sodium (Cytomel). with remission in 25% of clients.
2. Use. Replacement or substitution of d. Can be given during pregnancy. Stopped
diminished or absent thyroid function due to 23 weeks before delivery.
thyroid disease or thyroidectomy. e. Do not nurse baby.
3. Adverse effects. Headache, nervousness, f. Check pulse daily.
insomnia, irritability; palpitations, increased 5. Discharge teaching
blood pressure, tachycardia, dysrhythmias, a. Report signs of agranulocytosis (fever,
angina; weight loss, nausea, vomiting; chills, sore throat).
menstrual irregularities; allergic skin reaction; b. Report signs of bleeding promptly.
heat intolerance. c. Ask physician about use of iodized salt
4. Nursing implications and seafood in diet.
a. Baseline weight and thyroid studies. B. Related drugs
b. Avoid aspirin use. 1. Methimazole (Tapazole): similar to
c. Protect from light. propylthiouracil (PTU) except it is 10 times
d. Check pulse before taking. more potent. Given once daily due to long
5. Discharge teaching duration of action. Risk of hepatotoxicity is
a. Do not alter dosage. less.
b. Carry Medic Alert card. 2. Iodines: cause dose-related effects on thyroid
function. Low doses necessary for thyroid
function. High amounts inhibit thyroid function.
Used to decrease size and vascularity of the
Sample Questions thyroid before thyroid surgery, management
of thyroid storm, treatment of hyperthyroidism,
and treatment of thyroid cancer. Adverse effect:
66. What action should the nurse perform before GI distress.
administering levothyroxine (Synthroid)?
1. Check the clients pulse.
2. Listen to the clients chest. Sample Questions
3. Take the clients temperature.
4. Assess the clients neuro status.
67. The client calls the physicians office and
complains of chills, fever, and sore throat.
Which nursing action is appropriate?
Answers and Rationales 1. Tell the client it sounds like she has the flu
and that she should drink lots of fluids, take
66. 1. An adverse effect of levothyroxine (Synthroid) aspirin, and get extra rest.
is tachycardia; the nurse should check the 2. Tell the client to come in immediately for a
clients pulse before administration. throat culture and blood work as this may be
a serious drug reaction.
3. Expect the physician to prescribe another
THYROID ANTAGONISTS thyroid antagonist drug as this is an allergic
reaction.
A. Prototype: propylthiouracil (PTU)
1. Action. Prevents synthesis of thyroid 4. Tell the client that these are expected drug
hormones. Partially prevents peripheral reactions and that they will subside in
conversion of T4 to T3. a few days.

2 50 NCLEX-RN Review
53155_02_Ch02_p007-110.qxd 2/21/09 10:48 AM Page 51

4) Nursing implications
Answers and Rationales a) Stop taking 1 week before surgery
to decrease risk of
thromboembolism.
67. 2. Symptoms of chills, fever, and sore throat b) If one menstrual period is missed
while receiving propylthiouracil (PTU) require and tablets were taken correctly,
throat culture and blood work right away. continue pills; if two periods are
missed, stop pills and have
pregnancy test; will need
WOMENS AND MENS additional birth control method for
first month of drug therapy.
HEALTH AGENTS c) Smoking increases risk of
A. Prototype: progesterone (Progestin) thromboembolism.
1. Action. Changes a proliferative endometrium d) May take longer to conceive after
into a secretory one; causes a change in stopping pills. See Nursing
consistency of cervical mucus; stops Implications for progesterone.
spontaneous uterine contractions. b. Progestin-only preparations
2. Use. Amenorrhea; abnormal uterine bleeding; 1) Referred to as minipills.
endometrial cancer; prevention of conception. 2) Less effective than estrogen-progestin
3. Adverse effects combinations.
a. In parenteral administration: Breakthrough 3) Action not understood.
bleeding, spotting, dysmenorrhea, breast 4) Adverse effects and nursing
tenderness; headache, dizziness; edema, implications are same as for estrogen-
thromboembolism, hypertension; nausea, progestin combinations.
vomiting, bloating, weight gain; jaundice; 5) If two consecutive doses are missed,
rash, hirsutism, acne, oily skin; vision client must stop drug, use alternative
changes. birth control, wait until menses
b. Other effects: Hypertension; reduced occurs, and start therapy again.
glucose tolerance; thromboembolism in C. Fertility agents
high doses in specific groups of women. 1. Prototype: clomiphene citrate (Clomid)
4. Nursing implications given PO.
a. Take oral forms with food. 2. Menotropins (Pergonal) given IM.
b. Monitor weight. 3. Chorionic gonadotropin (A.P.L.) given IM.
c. Monitor BP. 4. Action. Increases the release of gonadotropins
d. For intramuscular injection and stimulates the growth and maturation of
1) Inject deeply into gluteal muscle. ovum.
2) Rotate injection sites. 5. Use. Infertility, induces ovulation.
3) Shake vial to ensure uniform dispersion. 6. Adverse effects. Multiple births, headache,
5. Discharge teaching tachycardia, nausea, vomiting, constipation,
a. Client should not smoke. anxiety, DVT, breast pain, diplopia.
b. Client should have regular Pap tests and 7. Nursing implications. Monitor for adverse
should do breast self-exam. effects, support client and partner throughout
c. Client should report calf pain, breast their attempt to achieve fertility.
lumps, or severe headache. D. Bisphosphonates
B. Related drugs 1. Alendronate (Fosamax).
1. Hydroxyprogesterone (Delalutin), 2. Tiludronate (Skelid).
medroxyprogesterone (Provera), and 3. Action. Bisphosphonates inhibit normal and
megesterolacetate (Megace). abnormal bone resorption of bone by
2. Oral contraceptives decreasing osteoclast activity.
a. Estrogen-progestin combinations 4. Use. Prevention and treatment of osteoporosis
1) Action. Suppress ovulation by in men and postmenopausal women, and
preventing release of follicle- Pagets disease.
stimulating hormone (FSH) and 5. Adverse effects. Flatus, gastritis, acid
luteinizing hormone (LH). Act directly regurgitation, dysphagia, muscle pain,
on reproductive organs. constipation or diarrhea, and headache.
2) Use. Prevention of pregnancy; 6. Nursing implications
amenorrhea; functional bleeding; a. Alendronate: take in the morning with 8
endometriosis. ounces of water 30 minutes before meals or
3) Adverse effects. Same as for progesterone. other medications. Sit upright after taking

DRUGS AND NURSING IMPLICATIONS

2 51
53155_02_Ch02_p007-110.qxd 2/21/09 10:48 AM Page 52

for at least 30 minutes to prevent


esophageal irritation. Sample Questions
b. Tiludronate: take on an empty stomach at
least 2 hours before or after eating.
E. Selective estrogen receptor modulators (SERMs) 68. The client calls the gynecology clinic and states
1. Raloxifene (Evista): used to prevent she thinks she is pregnant even though she has
postmenopausal osteoporosis. consistently taken her birth control pills. What
2. Tamoxifen (Novadex): used for prevention and would be the nurses best response?
treatment of breast cancer. 1. Continue taking the pills and see the
3. Action. SERMs work by stimulating estrogen
physician.
receptors on bone and blocking estrogen
receptors on breast tissue. 2. Stop taking the pills and see the physician.
4. Adverse effects. Hot flashes and leg cramps; 3. Continue taking the pills and see if menses
can increase the risk of thromboembolism. occurs in the next cycle.
5. Nursing implications. Periodically monitor 4. Stop taking the pills for this cycle; wait
CBC and platelet counts. 28 days and start them again.
F. Mens health agents
1. Androgens: testosterone, danazol (Danocrine), 69. Which conditions could be present in clients
fluoxymesterone (Halotestin), who are prescribed the medication danazol
methytestosterone (Android). (Danocrine)?
2. Action. Stimulate spermatogenesis and 1. Infertility.
maintenance of secondary sex characteristics,
and stimulate the formation and maintenance 2. Muscular dystrophy.
of muscular and skeletal protein. 3. Sexual dysfunction in men and women.
3. Use. Primarily for replacement therapy; 4. Endometriosis and fibrocystic breast disease.
treatment of breast cancer in women.
4. Adverse effects. Fluid retention, headaches,
nausea, vomiting, constipation or diarrhea,
acne, gynecomastia, priapism, depression,
Answers and Rationales
jaundice, and bleeding.
5. Nursing implications. Available PO. 68. 2. If the client is taking birth control pills and
Testosterone also available IM and
believes she is pregnant, she should stop taking
transdermally. Give with food to decrease GI
the pills and see the physician.
upset. Transdermal testosterone: Matrix type
placed on the scrotum; reservoir patch placed 69. 4. Danazol is given in the treatment of
on the abdomen, back, thighs, or upper arms. endometriosis and helps relieve the symptoms
Matrix type doesnt need to be removed for
of fibrocystic breast disease.
bathing, sexual activity, or swimming;
reservoir patch needs to be removed for these
activities. For IM use give deep IM in the OXYTOCICS
gluteus. Therapeutic effects may take
34 months and should not be abruptly A. Prototype: oxytocin (Pitocin)
discontinued. 1. Action. Posterior pituitary gland hormone that
G. Androgen inhibitors: 5-Alpha-Reductase Inhibitor: may initiate labor by stimulating uterine
finasteride (Proscar) smooth muscle contractions. Releases milk
1. Given orally to treat BPH and may take up to 6 from breast in breastfeeding women.
months before relief of BPH symptoms occurs. 2. Use. Labor induction; control postpartum
May cause impotency, decreased libido, or bleeding; treatment of incomplete abortion;
ejaculatory dysfunction. stimulate breast milk ejection.
H. Phosphodiesterase inhibitor: Sildenafil (Viagra), 3. Dose. IV infusion: add 0.52 milliunits/min,
tadalafil (Cialis), vardenafil (Levitra) increase by 12 milliunits/min every 1560
1. Given orally to treat male erectile dysfunction. minutes until contraction pattern (maximum
2. Contraindicated in clients taking nitrates. 20 milliunits/min). IV for uterine bleeding:
Sildenafil potentiates hypotensive effects of 1040 units to 1 liter of dextrose or
nitrates. Adverse effects: flushing, headache, electrolyte solution.
GI upset. IM: 10 units after delivery of placenta.
3. Nursing implications. Take 1 hour before sexual 4. Adverse effects. Water intoxication,
intercourse. Clients with a history of cardiac hypotension; postpartum hemorrhage; PVCs,
disease or angina should use cautiously. cardiac arrhythmias; uterine rupture; nausea,

2
vomiting; hypertension, cardiovascular

52 NCLEX-RN Review
53155_02_Ch02_p007-110.qxd 2/21/09 10:48 AM Page 53

Table 2-13 Oxytocin-Related Drugs

Drug Use Adverse Effects Nursing Implications


Ergonovine (Ergotrate) Control late postpartum Severe hypertension; Monitor BP, heart rate, and
Ergot Alkaloid (produce bleeding bradycardia; nausea, fundus
vasoconstriction and Treatment and prevention vomiting; diarrhea Notify physician if BP increases
intense oxytocic effects) of postpartum and Contraindicated to induce
postabortion bleeding labor
Methylergonovine Prophylaxis after delivery of Fewer adverse effects than Never administer before
(Methergine) placenta ergonovine. See above. delivery of placenta
Ergot Alkaloid (see above) Management of postpartum See above
bleeding

collapse; fetus: bradycardia, hypoxia, 3. Use an infusion pump and piggyback


intracranial hemorrhage, death; anaphylactic infusion into primary infusion line.
reactions. 4. Monitor clients temperature every 15
5. Nursing implications minutes.
a. Infusion pump for IV infusion.
b. Monitor BP, heart rate, I&O. 71. Which statement is correct regarding the reason
c. Use fetal monitor to monitor fetal heart rate why Methylergonovine (Methergine) is given?
and uterine contractions.
1. To induce labor.
d. Notify physician and stop infusion if fetal or
maternal distress. Place mother on left side. 2. In the first stage of labor.
e. Drug IV line should be piggybacked into a 3. After placental delivery.
primary infusion line. 4. IV prophylactically to prevent postpartum
f. Physician should be readily available to hemorrhage.
manage maternal or fetal complications.
B. Related drugs. See Table 2-13.

Answers and Rationales


Sample Questions
70. 3. Oxytocin (Pitocin) infusion should be
administered on an infusion pump and
70. The nurse sets up an oxytocin (Pitocin) infusion. piggybacked into a primary infusion line to
Which of the following are nursing control rate of infusion and to minimize/prevent
considerations in caring for clients receiving potentially dangerous adverse effects of oxytocin
oxytocin infusions? (Pitocin).
1. The nurse should increase the infusion by
3 milliunits/min every 15 minutes until there 71. 3. It is given after placental delivery because
is a pattern of contractions. methylergonovine (Methergine) can cause
2. Time-tape the solution and use a microdrop uterine tetany.
tubing to monitor the rate.

Eye Drugs

MYDRIATICS AND CYCLOPLEGICS cycloplegia, which paralyzes the lens and


eye muscles.
A. Prototype: atropine (Isopto Atropine) 2. Use. Facilitate eye exams and treat uveitis.
1. Action. An anticholinergic that causes 3. Adverse effects. Photophobia, reduced

2
mydriasis (dilation) of the pupil and lacrimation, impaired distant vision,

DRUGS AND NURSING IMPLICATIONS 53


53155_02_Ch02_p007-110.qxd 2/21/09 10:48 AM Page 54

increased intraocular pressure, eye pain, 73. 2. Atropine can raise intraocular pressure.
blurred vision. Clients with glaucoma have increased
4. Nursing implications intraocular pressure and a further increase in
a. Sunglasses to reduce photophobia. intraocular pressure could lead to an acute crisis
b. Artificial tears for reduced lacrimation. and blindness.
c. Elderly clients should be screened prior to
receiving atropinecan increase 74. 1. Vision is temporarily impaired following the
intraocular pressure. examination. This client should not drive, as
d. Should not drive until drug effects have distant vision is impaired.
worn off.
B. Related drugs
1. Sympathomimetic agents: MIOTICS
a. Apraclonidine (Iopidine)
b. Dipivefrin (Propine) A. Prototype: acetylcholine (Miochol)
2. Cyclopentolate (Cyclogyl) 1. Action. A cholinergic drug that causes miosis
(contraction) of the pupil and contraction of
the ciliary muscle in the eye.
2. Use. Decreases intraocular pressure in
Sample Questions glaucoma and achieves miosis in cataract
surgery.
3. Adverse effects. Low toxicity after systemic
72. To reduce the chance of having systemic effects
absorption; transient hypotension, decreased
related to atropine, which intervention will be heart rate; bronchospasm; flushing, sweating.
performed after administration? 4. Nursing implications
1. Place a warm compress over both eyes. a. Reconstitute just before use due to
2. Rinse the eye with water following instability of solution.
instillation. b. Systemic reactions treated with
3. Maintain pressure on inner canthus for 12 intravenous atropine.
minutes. B. Related drugs
1. Carbachol (Isopto Carbachol): Tell client of
4. Have client wipe eyes with gauze after brief stinging in eye after use; symptoms of eye
instillation. and brow pain, photophobia, and blurred
vision will usually be lessened with prolonged
73. Which of the following conditions should be
use.
assessed for prior to topical atropine 2. Echothiophate (Phospholine Iodine): Solutions
application? are unstable, client must wash hands before
1. Cataracts. use.
2. Glaucoma. 3. Pilocarpine (Pilocar, Isopto Carpine): Causes
3. Uveitis. blurred vision and focusing difficulty. Client
needs to understand that glaucoma treatment
4. Conjunctivitis.
is long and needs adherence to prevent
74. Which statement by the client indicates that he blindness; eyedropper tip should not be
contaminated; clients with asthma and lung
understands the instructions given following
disorders should be observed for respiratory
instillation of atropine?
difficulties.
1. My son will drive me home after the exam. 4. Physostigmine (Isopto Eserine)
2. If my eyes itch its OK to rub them. a. Beta blockers
3. I plan to go to the beach after this 1) Betaxolol (Beoptic)
appointment. 2) Timolol (Timoptic)
b. Carbonic anhydrase inhibitors (CAIs):
4. I will mow the lawn as soon as I get home.
Indicated for treatment of glaucoma.
1) Acetazolamide (Diamox)
2) Dorzolamide (Trusopt)
Answers and Rationales

72. 3. Applying pressure to the inner canthus


(lacrimal sac) will reduce systemic effects.

2 54 NCLEX-RN Review
53155_02_Ch02_p007-110.qxd 2/21/09 10:48 AM Page 55

3. I will stop the pilocarpine as soon as my


Sample Questions vision improves.
4. I know that pilocarpine can cause side
effects in my eye as well in other areas of my
75. Eye medication that treats glaucoma has what body.
effect?
1. Both mydriatic and miotic.
2. Mydriatic.
3. Miotic.
Answers and Rationales
4. Malaise.
75. 3. Miotic eye medication causes a contraction of
76. Which statement made by the client indicates a the eye pupil and contraction of the ciliary
need for more teaching about pilocarpine? muscle, which helps to decrease intraocular
1. I know a side effect of pilocarpine is blurred pressure.
vision.
2. I wont touch the eyedropper tip of the 76. 3. Treatment for glaucoma will continue
pilocarpine to my eye when instilling the throughout the clients life. Eye medication
drops. should not be discontinued.

Cardiovascular Drugs

CARDIAC GLYCOSIDES clients receiving quinidine, serum digoxin


levels could double, leading to possible
A. Prototype: digoxin (Lanoxin) toxicity.
1. Action. Increases force of myocardial 4. Nursing implications
contraction (positive inotropic effect). a. Half-life is longer in elderly.
Decreases rate of conduction (negative b. Monitor CBC, serum electrolytes, liver and
chronotropic effect) while increasing renal function studies, and ECG.
refractory period of the AV node. Positive c. Hold if apical rate is below 60 or greater
inotropic effect improves blood supply to vital than 120 beats per minute in adults, below
organs and kidneys, providing a diuretic 90 beats per minute in infants, or below 70
effect. Has a slow onset and shorter duration beats per minute in children up to
of action than other cardiac glycosides. Is adolescence.
eliminated through the kidneys. Digoxin elixir d. Monitor I&O and daily weights; potassium
is better absorbed by the GI tract than digoxin levels. Encourage foods high in potassium.
tablets. e. Monitor serum digoxin levels therapeutic
2. Use. Congestive heart failure (CHF); atrial range (0.52.0 ng/mL).
fibrillation; atrial flutter; paroxysmal atrial f. Give after meals if GI distress.
tachycardia. g. Do not confuse digoxin with digitoxin
3. Adverse effects. Cumulative with a narrow (Crystodigin) as they are not the
margin of safety. With toxicity there are same.
many symptoms that make it difficult to h. IM injections are painful and absorption is
distinguish from the condition being treated. erratic. Avoid IM injections if possible and
Arrhythmias, bradycardia: arrhythmias more give in large muscle mass.
frequently seen in children; anorexia, nausea, i. Digoxin antidote: Digoxin Immune Fab
vomiting, diarrhea; headaches, fatigue, (Digi-bind).
confusion, insomnia, convulsions; visual 5. Discharge teaching
disturbances: blurred vision, green or yellow a. Take radial pulse and notify physician if
tint or halos; hypersensitivity. Toxicity toxicity symptoms occur.
occurs more quickly in presence of a low b. Take dose the same time each day and do
serum potassium. Quinidine-digoxin reaction not skip or double up on dose.
may occur. When digoxin is stabilized in c. Daily weights.

DRUGS AND NURSING IMPLICATIONS

2 55
53155_02_Ch02_p007-110.qxd 2/21/09 10:48 AM Page 56

Table 2-14 Review of Antiarrhythmic Drugs

Drug Action Adverse Effects Nursing Implications


Quinidine Depresses myocardial excitability; Hematologic/Dermatologic: Administer drug with food to minimize
slows conduction time in atria Agranulocytosis; thrombocytopenia GI symptoms (nausea and vomiting)
and ventricles, prolongs P-R purpura; urticaria Carefully monitor electrolyte levels, blood
interval and QRS complex; CNS: vertigo; blurred vision; diplopia; counts, and kidney and liver function
prolongs refractory period; confusion, syncope Advise clients that diarrhea is common in
depresses myocardial GI: vomiting; cramping early therapy and should disappear
contractility, reduces vagal tone. CV: AV heart block; atrial or Encourage client to report dizziness or
Used in atrial fibrillation and flutter ventricular arrhythmias; faintness immediately
ventricular tachycardia. hypotension; severe bradycardia; Instruct patient to avoid fatigue, excessive
arterial embolism caffeine, alcohol, smoking, heavy meals,
stressful situations, OTC medications
Procainamide Depresses ectopic pacemakers; Same as quinidine, plus severe Inform client that drug may cause light-
(Pronestyl) action on the heart similar to hypotension with parenteral use, headedness and dizziness
quinidine. Used to treat PVCs, possible development of a Periodic ECG determinations and blood
ventricular tachycardia, and erythematosus-like syndrome in counts in clients on prolonged therapy
some atrial arrhythmias. some clients
Lidocaine Suppresses automaticity of ectopic CNS: dizziness; slurred speech; Closely monitor IV flow rate to ensure
(Xylocaine) pacemaker; shortens refractory apprehension; muscle twitching; maintenance of adequate plasma levels
period; decreases duration of tremors; convulsions Observe carefully for signs of CNS
action potential in Purkinje fibers; CV: hypotension, bradycardia toxicity (e.g., confusion, tremors),
local anesthetic action. Dermatologic: urticaria, peripheral particularly during IV infusion
Used for acute ventricular edema Monitor cardiac function and blood
arrhythmias. pressure closely. IM use may increase
CPK (creatine phosphokinase) levels
Disopyramide Increases action potential duration CV: hypotension; precipitation or Caution client to avoid driving until
(Norpace) and effective refractory period of aggravation of CHF effects of drug are known because
the atria and ventricles which GU: urinary hesitancy dizziness may occur
decreases automaticity and CNS: dry mouth; blurred vision; Monitor I&O because urinary retention
conduction velocity. Useful for fatigue, headache, malaise, dizziness may occur
PVCs and episodes of ventricular GI: nausea, constipation Encourage use of hard candy to relieve
tachycardia. dry mouth
Verapamil Calcium channel blockers inhibit the CV: hypotension, bradycardia; Carefully monitor blood pressure during
(Calan, Isoptin) influx of extracellular calcium ions palpitations, peripheral edema initial therapy and whenever dosage
Nifedipine into cardiac and smooth muscle CNS: flushing; weakness; dizziness; changes are made because hypotension
(Procardia) cells. Antianginal effects include light-headedness. may occur
Diltiazem dilation of coronary arteries and GI: nausea; cramping; heartburn; Monitor liver enzymes periodically
(Cardizem) arterioles. Verapamil also constipation with verapamil during therapy
decreases the influx of calcium Respiratory: dyspnea; cough; Be alert for signs of CHF, which can
into the cardiac contractile and wheezing occur especially if client is also receiving
conduction cells of the SA and a beta blocker
AV node. Useful in management
of chronic, stable angina and
treatment of supraventricular
tachyarrhythmias.
Digitalis Stimulates the force of cardiac Digitalis toxicity: Closely observe client for signs of toxicity
contraction with improvement of CNS: headache, fatigue, malaise, Check with physician regarding what pulse
cardiac output. Decreases cardiac drowsiness, muscle weakness, rates (both high and low) should be used
oxygen demands, diastolic heart insomnia, agitation, seizures, as indicators for withholding medication
size, and heart rate. Useful in paresthesias of hands and feet, Watch for changes in pulse rate (sudden
treatment of CHF and certain personality changes, impaired increase above 120 or fall below 60)
arrhythmias, such as atrial memory, hallucinations Recognize signs of hypokalemia, which
fibrillation and atrial flutter. CV: arrhythmias (all types are increases the incidence of digitalis toxicity
possible) Encourage client to take digitalis at
ENT: yellow-green halos prescribed times only
GI: anorexia, nausea and vomiting, Advise client that protracted diarrhea or
abdominal distension and pain vomiting can create an electrolyte
imbalance and lead to digitalis toxicity
Recommend adherence to prescribed diet

2 56 NCLEX-RN Review
53155_02_Ch02_p007-110.qxd 2/21/09 10:48 AM Page 57

d. Avoid high-sodium foods. Increase dietary 78. 2. The client should be taught how to monitor
intake of potassium. the radial pulse, and to hold the medication if the
e. Separate digoxin from other pills in pulse is less than 60 beats per minute for an
pillbox. adult, less than 70 for children, or less than 90
B. Related drugs: Phosphodiesterase inhibitors: for an infant.
milrinone (Primacor) and inamrinone (Inocor),
used for short-term management of CHF. Also see 79. 4. The normal range of digoxin is 0.52.0 ng/mL,
Table 2-14. with a toxic threshold of 2.5 ng/mL.

80. 4. Vomiting and diarrhea are adverse effects of


digoxin therapy and may also be symptoms of
Sample Questions digoxin toxicity. The client should report this
occurrence to the physician.

77. What is the main action of cardiac glycosides?


1. Release free calcium with cardiac muscle. ANTIANGINAL DRUGS
2. Increase the rate of impulse formation at the
SA node. A. Prototype: nitrites and nitroglycerin (Nitro-bid,
Nitrodur, Nitrostat IV)
3. Decrease the conduction of electrical impulses
1. Action. Dilates the peripheral vascular smooth
in the heart.
muscles of smaller vessels, which decreases
4. Decrease the force of myocardial contractions. cardiac preload and afterload leading to
decreased myocardial oxygen needs.
78. What should the nurse teach the client about Selectively dilates large coronary arteries,
cardiac glycosides? which helps to decrease anginal pain and
1. Avoid fruits with potassium. hypoxia of the myocardium. Given by many
2. How to monitor the radial pulse. different routes of administration including
3. Always take digoxin on an empty stomach. PO, SL, buccal, topical, transdermal. Tolerance
may develop with continued use.
4. Return to the health care provider in 1 year. 2. Use. Treatment and prophylaxis of angina
pectoris. IV nitroglycerin manages congestive
79. Which lab value would be a concern to the
heart failure associated with acute MI and
nurse?
controls intraoperative hypotension or
1. 0.5 ng/mL manages hypertension.
2. 1.0 ng/mL 3. Dose (adults)
3. 1.5 ng/mL a. SL: 0.150.6 mg at onset of attack or
4. 2.2 ng/mL anticipation of attack.
b. PO: Sustained release 2.52.6 mg TID or
80. Which statement by the client will assure the QID. Topical ointment: 12 inches every
nurse that the client understands teaching about 8 hours up to 45 inches every 4 hours.
the adverse effects of digoxin? c. Transdermal: 0.10.6 mg/hr, can increase up
to 0.8 mg/hr; patch worn 1214 hours/day.
1. Ill call the physician if my pulse is below d. Spray: 12 sprays, can repeat every 5
70. minutes for 15 minutes.
2. If a rash develops, Ill apply a topical cream. e. Buccal: 1 mg every 5 hours; dose and
3. I will call the physician daily to report my frequency increased as needed.
weight. f. IV: 5 mcg/min in 5% dextrose in water or
4. I will notify the health care provider if 0.9% sodium chloride and titrate every
vomiting or diarrhea develops. 35 minutes until response.
4. Adverse effects. Headache, usually disappears
with long-term therapy; flushing; hypotension,
dizziness; reflex tachycardia; skin rash with
Answers and Rationales ointment.
5. Nursing implications
a. No more than 3 tablets SL should be taken
77. 1. It is believed that free calcium is released in a 15-minute period (1 tablet every
within the cardiac muscle cell, potentiating the 5 minutes). If pain not relieved by 3 tablets
action of actin and myosin, which are the major over 15 minutes, could indicate an acute MI
and physician should be contacted.

2
proteins responsible for muscle contraction.

DRUGS AND NURSING IMPLICATIONS 57


53155_02_Ch02_p007-110.qxd 2/21/09 10:48 AM Page 58

b. Leave tablets at bedside and allocate a 2. Replace the transdermal patch every 8 hours.
specific number of tablets in container. 3. Do not stand near microwave ovens while in
Instruct client to tell nurse when having an use.
attack and number of tablets taken.
4. Put on an extra transdermal patch if chest
c. Sustained-release tablets or capsules
pain occurs.
should be taken 1 hour before meals or
2 hours after meals. 83. Which of the following should the nurse include
d. Nitroglycerin ointment should be applied
in teaching about taking sublingual
to a hairless or shaved area to promote
nitroglycerin?
absorption. New site should be used with
each new dose. Use ruled applicator paper 1. To replace tablets on a yearly basis.
that comes with ointment to measure dose. 2. Keep tablets in a moist warm environment.
Wear gloves when applying ointment to 3. Take the tablet before exercise to prevent
applicator. Leave applicator paper on site. angina.
Cover the applicator paper with plastic
4. Notify physician if after 5 consecutive doses
wrap and secure with tape.
the chest pain persists.
e. Transdermal nitroglycerin has aluminum
backing and patch. Remove before
defibrillation. Avoid standing near
microwave ovens to prevent burns. Patches
are usually applied in morning and Answers and Rationales
removed in evening to prevent tolerance.
f. Dilute IV nitroglycerin in 5% dextrose or
0.9% sodium chloride. Avoid using 81. 4. A headache is a frequently seen adverse
polyvinyl chloride (PVC) plastic as it can effect that usually disappears with long-
absorb nitroglycerin. Non-PVC is provided term therapy. The physician may order
by the manufacturer. IV use requires aspirin or acetaminophen for headache
continuous hemodynamic monitoring. relief.
6. Discharge teaching
a. Rise slowly to prevent dizziness. 82. 3. The back of a transdermal nitroglycerin patch
b. Store in original dark glass container in a contains aluminum, which could cause burns to
cool place. Date bottle when opening and clients standing near microwave ovens or if
discard after 3 months. defibrillation is needed.
c. Headache will discontinue with long-term
use. 83. 3. Nitroglycerin should be taken before exercise
d. Keep diary of the number of anginal to prevent an anginal attack.
attacks and tablets taken.
e. Do not drink alcohol.
B. Related drugs: Isosorbide dinitrate (Isordil): ANTIANGINAL DRUGS (CONTINUED)
used to treat and prevent anginal attacks; given
SL or PO. C. Prototype for Calcium Channel Blockers:
verapamil (Calan, Isoptin)
1. Action. Inhibits myocardial oxygen demand by
inhibiting the influx of calcium through
Sample Questions muscle cell, which leads to reduced afterload
and coronary vasodilation. Decreases
myocardial contractility, causing peripheral
81. The client states that he is getting headaches vasodilation leading to decreased heart
after taking nitroglycerin. How does the nurse workload.
interpret this occurrence? 2. Use. Angina; essential hypertension (PO form);
1. Toxic effect. cardiac dysrhythmias (IV use).
2. Symptom of tolerance. 3. Adverse effects. Constipation; nausea and
vomiting; hypotension; bradycardia; AV block;
3. Hypersensitivity reaction.
dizziness.
4. Adverse effect. 4. Nursing implications
a. Monitor VS, I&O, and ECG.
82. The physician decides to order nitroglycerin b. Encourage high-fiber foods and increased
transdermal patches. Which instruction is fluid intake (condition permitting).
important? 5. Discharge teaching

2
1. Remove patch when showering. a. Take radial pulse before taking verapamil.

58 NCLEX-RN Review
53155_02_Ch02_p007-110.qxd 2/21/09 10:48 AM Page 59

b. Avoid caffeine. B. Antiplatelet agents


c. Avoid driving or operating 1. Dipyridamole (Persantine): Potent vasodilator
machinery/heavy equipment until that also decreases platelet aggregation and
response to drug is established. clotting time. Selectively dilates small
d. Change positions slowly to decrease resistance vessels of coronary vascular bed.
orthostatic hypotension. Used in the prevention of thromboembolism in
e. Do not abruptly discontinue verapamil cardiac valve replacement surgery; also used in
therapy as rebound angina could occur. other thromboembolic disorders to decrease
Dose is generally tapered. platelet aggregation. Adverse effects: headaches,
f. Continue with nitroglycerin therapy if dizziness, weakness, hypotension, GI distress,
prescribed. flushing, and skin rashes. Monitor BP. Other
6. Related drugs. See Table 2-14. antiplatelets: Aspirin, cilostazol (Pletal),
clopidogrel (Plavix), and teclopidine (Ticlid).

Sample Questions
Sample Questions
84. A client is taking nitroglycerin with verapamil
(Isoptin). What occurrence should the nurse 86. The client complains of nausea while taking
watch for? isoxsuprine HCl (Vasodilan). What instruction
1. Hyperkalemia. will the nurse give the client?
2. Hypotension. 1. Stop taking the medication and report this to
3. Seizures. the physician.
4. Insomnia. 2. Take an antacid with the isoxsuprine HCl
(Vasodilan).
85. Which statement by the client indicates 3. Keep taking the drug as this effect is only
understanding concerning the use of verapamil transient.
(Isoptin)? 4. Report this to the physician so he can reduce
1. If I get dizzy I will stop taking the pills and the dose of isoxsuprine HCl (Vasodilan).
call the physician.
2. Im glad that I can continue to drink coffee.
3. I will only have to take the pills for a couple Answers and Rationales
of weeks.
4. I will take my pulse before taking my pill.
86. 4. Adverse effects of isoxsuprine HCl (Vasodilan)
are dose related and can be dealt with by
reduction of the dose.
Answers and Rationales
ANTIDYSRHYTHMICS
84. 2. Verapamil (Isoptin) reduces afterload and with
concurrent use of nitroglycerin can cause A. Prototype: quinidine (Quinaglute) class 1A
hypotension. 1. Action. Alkaloid from the bark of the
cinchona tree. Related to quinine, an
85. 4. Clients should take pulse before taking antimalarial drug. Decreases myocardial
verapamil (Isoptin) as this drug can cause excitability and slows conduction velocity,
bradycardia. while prolonging the refractory period. PR
interval and QRS complex may be prolonged.
Has anticholinergic effects that reduce vagus
PERIPHERAL VASODILATORS nerve activity, which slows AV conduction.
2. Use. Atrial dysrhythmias, atrial fibrillation,
A. Prototype: isoxsuprine HCl (Vasodilan) and atrial flutter; ventricular dysrhythmias.
Relaxation of the smooth muscle of blood vessels. 3. Adverse effects. Cinchonism: GI distress,
Used to treat peripheral vascular disorders such as tinnitus, visual disturbances, dizziness,
Raynauds and Buergers disease (thromboangitis headache; AV block, hypotension;
obliterans), diabetic vascular disease, and varicose thrombocytopenia; hypersensitivity; nausea,

2
ulcers. vomiting, diarrhea.

DRUGS AND NURSING IMPLICATIONS 59


53155_02_Ch02_p007-110.qxd 2/21/09 10:48 AM Page 60

4. Nursing implications 2. Use. Ventricular arrhythmias, i.e., VT; VF; PVCs.


a. Monitor ECG and VS, serum electrolytes, 3. Dose (given parenterally only)
CBC, kidney and liver function. a. Adult
b. Monitor serum quinidine levels. Normal 1) IV Bolus: 50100 mg at a rate of
range 36 mcg/mL. 2550 mcg/kg/minute; once
c. Take an apical pulse. arrhythmia controlled continue
d. Take with food if GI upset. infusion of 14 mg/minute.
e. Clients taking digoxin and quinidine are 2) IM: 200300 mg and repeat in
more prone to digitalis toxicity. 6090 minutes if needed.
5. Discharge teaching b. Pediatric:
a. Take radial pulse before taking. 1) IV: 1 mg/kg followed by an infusion of
b. Report symptoms of cinchonism, 30 mcg/kg/minute.
palpitations, faintness, or breathlessness. 4. Adverse effects. Drowsiness; CNS stimulation
B. Related drugs. See Table 2-14. can develop leading to seizures; ventricular
1. Procainamide tachycardia, heart block, hypertension,
2. Disopyramide bradycardia.
5. Nursing implications
a. Monitor ECG, VS, neurologic status, and
serum lidocaine levels.
Sample Questions b. Therapeutic lidocaine levels range between
1.56 mcg/mL.
c. Use an infusion pump.
87. Which of the following adverse effects is unique d. Cardiac IV lidocaine should not contain
to quinidine (Quinaglute)? preservatives or epinephrine.
1. SLE (systemic lupus erythematosus). e. Deltoid muscle is preferred for IM use.
2. Agranulocytosis. f. Do not mix with other drugs.
D. Related drugs
3. Cinchonism.
1. Mexiletine (Mexitel): related to lidocaine
4. Hypoglycemia. 2. Tocainide (Tonocard): related to lidocaine
3. Phenytoin sodium (Dilantin)
88. What client teaching should be included Also see Table 2-15.
concerning quinidine administration? E. Class IC drugs: Both drugs given orally to treat
1. Drink plenty of orange juice. ventricular dysrhythmias.
2. Maintain a high-fiber diet. 1. Flecainide (Tambocor)
3. Take Pepto Bismol if diarrhea occurs. 2. Propafenone (Rythmol)
F. Class I A, B, C drugs
4. Take medication with meals.
1. Moricizine (Ethmozine): a sodium channel
blocker used to treat life-threatening
ventricular dysrhythmias; given orally.
Answers and Rationales

87. 3. Cinchonism is a syndrome seen specifically


Sample Questions
when using quinidine; manifested by tinnitus, GI
distress, dizziness, visual disturbances, and 89. Which of the following statements is correct
headache. concerning the administration of phenytoin
(Dilantin)?
88. 4. Taking quinidine with meals will decrease GI
distress. 1. Phenytoin is administered by continuous
infusion.
2. Phenytoin can be mixed in dextrose
ANTIDYSRHYTHMICS (CONTINUED) solutions.
3. 0.9% normal saline should be used to flush
C. Prototype: lidocaine (Xylocaine) class 1B
1. Action. Prolongs refractory period in the IV line and site before and after
myocardium and Purkinje fibers. Has administration.
little effect on atria. Depresses automaticity 4. Phenytoin is given by rapid IV push.
but therapeutic doses do not depress
myocardial contractility. Also used as a

2
local anesthetic.

60 NCLEX-RN Review
53155_02_Ch02_p007-110.qxd 2/21/09 10:48 AM Page 61

Table 2-15 Drugs Related to Lidocaine

Drug Action Use Adverse Effects Nursing Implications


Tocainide Oral analog of Ventricular Drug-induced SLE; Give with food to reduce GI
(Tonocard) lidocaine arrhythmias dyspnea; GI distress; see distress
(Xylocaine) lidocaine Avoid driving or operating
heavy machinery until drug
response known
Take radial pulse
Phenytoin sodium Also an Ventricular and Drowsiness; slurred speech; Should not be given with other
(Dilantin) antiepileptic supraventricular ataxia; nystagmus; antidysrhythmics
drug; see arrhythmias hypotension; IV: do not mix with dextrose as
lidocaine unresponsive to agranulocytosis; rash; crystallization can occur. Flush
lidocaine or nausea, vomiting IV line with saline before and
procainamide. after administration.
Also used to treat Do not mix with other drugs
digitalis-induced Monitor CBC
arrhythmias. Therapeutic levels range
between 1020 mcg/mL

Answers and Rationales Sample Questions

89. 3. Phenytoin (Dilantin) has a high alkalinity and 90. Which adverse effect should the nurse monitor
can precipitate easily. Flushing the IV line and while a client is on maintenance bretylium
site with 0.9% normal saline will minimize therapy?
venous irritation and prevent precipitation. 1. Hypotension.
2. Tachycardia.
ANTIDYSRHYTHMICS (CONTINUED) 3. Insomnia.
4. Hearing loss.
F. Prototype: bretylium (Bretylol) class III
1. Action. An antifibrillatory drug. Initially
releases norepinephrine to increase
conduction velocity and strengthen the Answers and Rationales
heartbeat.
2. Use. Life-threatening arrhythmias.
3. Adverse effects. Hypotension, dizziness; 90. 1. Hypotension occurs because after the initial
worsening arrhythmias, hypertension; nausea, release of norepinephrine, bretylium blocks
vomiting, diarrhea. further release of norepinephrine.
4. Nursing implications
a. Monitor ECG, vital signs, I&O.
b. Gradually reduce dose. BETA BLOCKERS (CLASS II)
c. Change position slowly.
G. Related drugs A. Prototype: propranolol (Inderal)
1. Amiodarone (Cordarone) 1. Action. Beta-adrenergic blocker that decreases
a. Given orally to treat chronic recurrent heart rate, force of contraction, myocardial
ventricular tachycardia or ventricular irritability, and conduction velocity, and
fibrillation that is unresponsive to other depresses automaticity.
drugs. 2. Use. Cardiac arrhythmias caused by excessive
2. Ibutilide (Corvert): given parenterally to treat cardiac stimulation of sympathetic nerve
atrial dysrhythmias. impulse; digitalis-induced arrhythmias; essential
H. Unclassified antidysrhythmic: Adenosine hypertension; angina pectoris; preoperative
(Adenocard) is given IV to treat PSVT. management of pheochromocytoma; prevention

2
of migraine headaches.

DRUGS AND NURSING IMPLICATIONS 61


53155_02_Ch02_p007-110.qxd 2/21/09 10:48 AM Page 62

3. Adverse effects. Dizziness, drowsiness, CARDIAC STIMULANTS


insomnia, depression; hypoglycemia;
bronchospasm; bradycardia, heart block, A. Cardiac stimulants are also autonomic nervous
hypotension; rash. system drugs. The autonomic nervous system
4. Nursing implications drugs are discussed in more detail in a previous
a. Take apical pulse. section in this unit.
b. Monitor I&O, daily weights. B. Representative drugs
c. Gradually reduce dose before discontinuing. 1. Atropine sulfate
d. Pulse rate may not rise following exercise a. Blocks vagal stimulation of the SA node in
or stress, due to beta-blocking effects. the heart, thus increasing heart rate. Acts
5. Discharge teaching systemically to block cholinergic activity
a. Take radial pulse before administering drug. throughout the body.
b. Avoid alcoholic beverages. b. Cardiac uses: treatment of sinus
c. Avoid cold exposure to extremities. bradycardia or asystole; management of
d. Change positions slowly. symptomatic sinus bradycardia; diagnosis
B. Related drugs of sinus node dysfunction.
Note: olol is present in generic names. c. Adverse effects are related to blocking of
1. Esmolol (Brevibloc): class II antidysrhythmic, cholinergic activity in the body.
used to treat tachycardia, supraventricular 2. Isoproteronol (Isuprel)
tachycardia, atrial fibrillation, and atrial flutter. a. Stimulates beta-1 adrenergic receptors in
2. Nadolol (Corgard): used to treat essential heart to increase cardiac output. Is also a
hypertension and angina. bronchodilator.
3. Pindolol (Visken): used to treat essential b. Cardiac uses: cardiac standstill; carotid
hypertension. sinus hypersensitivity; Stokes-Adams
4. Timolol (Blocadren): used to treat essential syndrome; ventricular arrhythmias.
hypertension. c. Adverse effects: headache, palpitations,
5. Atenolol (Tenormin): class II antidysrhythmic, dry mouth, flushing, sweating, and
also used to treat angina and hypertension. bronchial edema.
6. Metoprolol (Lopressor): class II
antidysrhythmic, given after MI to decrease
risk of sudden cardiac death, and also used to ANTICOAGULANTS
treat angina and hypertension.
7. Sotalol (Betapase): class III antidysrhythmic, A. Prototype: anticoagulants hinder one or more
generally used to treat life-threatening ventricular steps of the coagulation process. They do not
dysrhythmias, i.e., ventricular tachycardia. dissolve existing blood clots but prevent further
coagulation from occurring.
B. Related drugs: low-molecular-weight heparin
(LMWH). See Table 2-16 comparing Heparin and
Sample Questions Warfarin sodium (Coumadin).
1. Enoxaparin (Lovenox)
2. Dalteprin (Fragmin)
91. Which occurrence should clients be monitored 3. Action. Enzymatically removes part of heparin
for if taking beta-adrenergic blocking agents? molecule, making a smaller, more accurate
1. Hyperglycemia. heparin.
4. Use. Prophylaxis in deep venous thrombosis
2. Heat intolerance.
(DVT) or pulmonary embolism (PE) especially
3. Respiratory difficulties, bradycardia. after hip/knee or abdominal surgery.
4. The development of arthritis. 5. Adverse effects. Bleeding, anemia, and
thrombocytopenia.
6. Nursing implications. Assess and monitor for
symptoms of bleeding. Special monitoring of
bleeding times not necessary. Antidote:
Answers and Rationales protamine sulfate.

91. 3. Beta-adrenergic blocking agents reduce heart


rate and force of contraction, as well as possibly
causing bronchoconstriction.

2 62 NCLEX-RN Review
53155_02_Ch02_p007-110.qxd 2/21/09 10:48 AM Page 63

Table 2-16 Comparison of Heparin and Warfarin (Coumadin)

Heparin Coumadin
Action Blocks conversion of prothrombin to thrombin and fibrinogen to Blocks prothrombin synthesis. Action takes
fibrin. Immediate action. 1224 hrs to occur.
Use Prophylaxis and treatment of thrombosis and embolism. Prophylaxis and treatment of thrombosis and
Anticoagulati on for vascular and cardiac surgery. embolism.
Prevention of clotting in heparin lock sets, blood samples, and Atrial fibrillation with embolization.
during dialysis. Adjunct in treatment of coronary occlusion and
Treatment of disseminated intravascular clotting syndrome (DIC). small cell carcinoma of lung with chemotherapy
Adjunctive treatment of coronary occlusion with acute MI. and radiation.
Adult: Adult:
Dose SC (deep, intrafat): initially, 10,00020,000 units, then Oral: 510 mg PO initially, then 210 mg PO per day
800010,000 units every 8 hours or 15,00020,000 units based on PT or INR.
every 12 hours or as determined by coagulation test results.
Intermittent IV injection: 10,000 units initially followed by
500010,000 units every 46 hours. Continuous IV infusion:
inject 5000 units initially followed by 20,00040,000
units/day in 1000 mL of sodium chloride solution.
Pediatric:
IV: Initially 50 units per kilogram.
Maintenance: 50100 units per kilogram IV drip every 4 hours.
Hemorrhage, bruising, thrombocytopenia. Hemorrhage from any tissue or organ.
Adverse Effects Alopecia Anorexia, nausea, vomiting, diarrhea.
Osteoporosis Hypersensitivity: dermatitis, urticaria, fever
Allergic reactions: fever, chills, urticaria, bronchospasm. Jaundice, hepatitis
Elevated AST (SGOT), ALT (SGPT). Overdosage: petechicre, paralytic ileus; skin necrosis of
toes (purple toes syndrome), and others tissues.
Protamine sulfate Vitamin K
Antidote Laboratory test used to monitor therapy: partial Laboratory tests used to monitor therapy:
thromboplastin time (PTT) prothrombin time (PT), INR
Read label carefully as drug is supplied in differing strengths. Known for highest adverse drug interactions of all
Nursing Do not give IM. groups.
Implications SC injection: given in fatty layer of abdomen or just above iliac Tablet can be crushed and taken with any fluid.
crest; use 1/25/8 needle after drawing heparin into syringe; Monitor prothrombin time (PT) and INR.
do not inject within 2 inches of umbilicus, scars, or bruises; do Have antidote vitamin K available.
not aspirate; do not massage injection site; rotate injection sites Many drug interactions.
and document. Smoking increases dose requirement.
Continuous IV infusion should be given via IV volume
control device.
Observe needle sites daily for signs of hematoma.
Monitor CBC, PTT and other coagulation tests.
Test stool for occult blood daily.
Have antidote protamine sulfate available.
Monitor VS.
Report: hematuria, bloody stools, hematemesis, bleeding gums,
petechiae, nosebleed, bloody sputum.
Alcohol and smoking alter drug response.
Aspirin, antihistamines, ginseng, ginkgo biloba, and NSAIDS
shouldnt be taken while on heparin therapy as these agents
may cause platelet function interference.
Do not abruptly withdraw.
Generally followed with oral anticoagulant therapy.
Heparin used only in hospital setting. Stress importance of not skipping doses.
Discharge Client will need to report frequently for blood tests.
Teaching Client shouldnt take any medication or herbals
without checking first with physician.
Client should report signs of bleeding.
Use soft toothbrush; floss teeth with waxed floss.
Shave with electric razor.
Client should tell other health care personnel such
as dentists, dental hygienists, etc., that he is taking
Coumadin.
Client should carry medical identification (Medic
Alert) stating name of drug, name of physician, etc.
Teach client measures to avoid venous stasis.

DRUGS AND NURSING IMPLICATIONS

2 63
53155_02_Ch02_p007-110.qxd 2/21/09 10:48 AM Page 64

93. 3. Protamine sulfate is the antidote for heparin


Sample Questions overdose.

94. 3. A client on heparin therapy should not take


92. Which action should the nurse perform while aspirin due to increased potential for bleeding.
the client is on IV heparin? The nurse needs to contact the physician for a
1. Protect the medication from light. different pain medication.
2. Use IM route if there are frequent intravenous 95. 1. Changing dose of warfarin sodium
site changes. (Coumadin) by missing a dose on one day and
3. Attach the IV heparin to an infusion pump. doubling the dose on the following day is
4. Explain to the client that stools may turn unacceptable as it will negatively affect blood
gray. coagulation.

93. What should the nurse have readily available for


a heparin overdose? THROMBOLYTIC DRUGS
1. Platelets.
2. Urokinase. A. Prototype: streptokinase (Streptase)
1. Action. Transforms plasminogen to plasmin
3. Protamine sulfate. which degrades fibrinogen, fibrin clots, and
4. Vitamin K. other plasma proteins.
2. Use. Pulmonary emboli; coronary artery
94. The client requests pain medication for a thrombosis; deep venous thrombosis;
headache. The physician has ordered aspirin arteriovenous cannula occlusion.
X grains PO every 4 hours for pain. What action 3. Adverse effects. Bleeding; allergic reaction;
should the nurse take? arrhythmias.
1. Substitute Tylenol for the aspirin. 4. Nursing implications
2. Give the aspirin as ordered. a. Start therapy as soon as possible after
thrombus appears as thrombi older than
3. Call the physician for a different pain reliever.
7 days react poorly to streptokinase.
4. Give 5 grains of aspirin now and 5 grains in b. When used in treatment of an acute MI, start
2 hours. therapy within 6 hours of attack. When used
in treatment of a stroke, therapy should be
95. The nurse is teaching about warfarin sodium started within a 3-hour window of the attack.
(Coumadin) therapy. Which statement by the c. Heparin is discontinued before
nurse needs correcting? streptokinase is started.
1. If you miss your daily dose of Coumadin, d. Corticosteroids can be given to decrease
take 2 tablets the next day. allergic reaction.
2. Use waxed dental floss while on Coumadin e. Reconstitute streptokinase with normal saline
therapy. (preferred solution) or 5% dextrose solution.
f. IM injections are contraindicated.
3. Notify your physician before taking any
g. Monitor blood coagulation studies and VS.
other medication.
h. Maintain bed rest while receiving drug.
4. Avoid drinking alcohol in any form while i. Monitor for excessive bleeding every 15
on Coumadin therapy. minutes for the first hour of treatment,
every 30 minutes for second to eighth
hours, then every 8 hours.
j. Keep whole blood available.
k. Aminocaproic acid is the antidote for
Answers and Rationales streptokinase.
B. Related drugs
92. 3. Continuous intravenous infusion of heparin 1. Alteplase (Activase)
needs constant monitoring to ensure accuracy in 2. Anistreplase (Eminase)
dose. An infusion pump or volume controller 3. Reteplase (Retavase)
4. Tenecteplase (TNKase)
should be used for this purpose.
5. Urokinase (Abbokinase)

2 64 NCLEX-RN Review
53155_02_Ch02_p007-110.qxd 2/21/09 10:48 AM Page 65

2. Reductase inhibitors: Atorvastatin (Lipitor),


Sample Questions fluvastatin (Lescol), lovastatin (Mevacor),
pravastatin (Pravachol), and simvastatin
(Zocor) decrease cholesterol levels by stopping
96. What will the nurse assess during and after the body from making its own cholesterol.
streptokinase treatment? Used to treat hypercholesterolemia types IIa
1. Urticaria. and IIb. Adverse effects: headache; insomnia,
fatigue, blurred vision, myalgias, nausea,
2. Diarrhea.
hepatotoxicity, elevated CPK, alkaline
3. Sore throat. phosphatase, and transaminase. Nursing
4. Peripheral edema. implications: monitor renal and hepatic
studies; take with meals to increase absorption.
3. Gemfibrozil (Lopid) decreases triglycerides
Answers and Rationales and increases HDL cholesterol. May cause
diarrhea or GI upset.
4. Niacin: vitamin B3 (Nicobid): reduces liver
96. 1. Streptokinase (Streptase) is a foreign protein synthesis and reduces cholesterol and total
and does cause allergic reactions. Urticaria would lipid levels. Used in treatment of
be a sign of this. hyperlipidemia. Adverse effects: tingling,
flushing, jaundice, GI upset, pruritus. Nursing
implications: dosage is individualized. Niacin
ANTILIPEMIC AGENTS is an OTC preparation that should be taken
under a physicians care.
A. Prototype: cholestyramine (Questran)
1. Action. Prevents the metabolism of cholesterol
in the body.
2. Use. Type IIa hyperlipoproteinemia; pruritus Sample Questions
caused by partial biliary obstruction.
3. Adverse effects. Constipation, nausea, and
vomiting; deficiencies in fat-soluble vitamins 97. Which statement by the client indicates a need
A, D, K; rash and skin irritation; osteoporosis; for more teaching about cholestyramine
headache, dizziness, syncope; arthritis; fever. (Questran) by the nurse?
4. Nursing implications 1. I will continue going to exercise class.
a. Monitor cholesterol and serum triglyceride
2. I will take the drug after meals.
levels.
b. Assess preexisting constipation problems. 3. I will dissolve the drug in liquid before
c. Long-term use increases bleeding tendencies: taking it.
oral vitamin K may be given prophylactically. 4. I will increase the fiber in my diet.
5. Discharge teaching
a. Take with water or preferred liquid and
dissolve.
b. Take before meals. Answers and Rationales
c. Eat a high-bulk diet low in cholesterol and
saturated fats with increased fluids.
d. Do not omit doses or change dose 97. 2. Cholestyramine (Questran) should be taken
intervals. before meals for better absorption.
e. Do not take cholestyramine (Questran) at
the same time as other medications as
there will be interference with
absorption. ANTIHYPERTENSIVES
f. Encourage exercise and weight loss.
g. Give for several months or years if it is General Information forAdministration
effective.
B. Related drugs
of Antihypertensives
1. Colestipol (Colestid) is similar in action, use, 1. Primary objective of antihypertensive therapy
adverse effects, and nursing implications to is to control essential hypertension and
cholestyramine (Questran). maintain BP with minimal adverse effects.

DRUGS AND NURSING IMPLICATIONS

2 65
53155_02_Ch02_p007-110.qxd 2/21/09 10:48 AM Page 66

2. Antihypertensives reduce peripheral 1. Action. Blocks alpha receptors in arterial


resistance and decrease volume of circulating smooth-muscle vasculature and mediates
blood. vasoconstriction.
3. Orthostatic hypotension is a common adverse 2. Use. Essential hypertension and hypertension
effect for all antihypertensives. caused by pheochromocytoma.
4. Should not be abruptly discontinued as 3. Adverse effects. Postural hypotension and
rebound hypertension could occur. syncope with initial therapy; reflex
5. Discharge instructions for all tachycardia. See clonidine (Catapres).
antihypertensives 4. Nursing implications. Monitor vital signs.
a. Have adverse effects that may affect client 5. Discharge teaching
compliance in taking medication. It is a. Take with food to reduce dizziness,
important for client to receive thorough light-headedness.
teaching and support to maintain b. Take initial dose at bedtime to reduce
compliance. effects of syncope.
b. Do not abruptly discontinue or skip doses c. Report sexual difficulties.
of medications. 6. Related drugs
c. Change positions gradually; avoid alcohol, a. Doxazosin (Cardura)
hot showers and baths. b. Terazosin (Hytrin)
d. Do not take OTC drugs without consulting D. Prototype: peripheral-acting antiadrenergic agents
physician. (reserpine [Serpalan])
e. Monitor weight and eat low-sodium foods. 1. Action. Lowers BP by blocking norepinephrine
f. Take BP and record in diary. Report in CNS and peripherally.
changes to physician. 2. Use. Rarely used due to its adverse effects and
g. Do not drive or operate heavy machinery availability of other antihypertensives; agitated
until drug effects are established. psychosis; essential hypertension with a diuretic;
Note: thiazide diuretics are discussed under parenteral use to treat hypertensive emergencies.
Renal Drugs; MAO (monamine oxidase 3. Adverse effects. Suicidal depression,
inhibitors) are discussed under Central drowsiness; nasal obstruction; increased
Nervous System Drugs; beta blockers are incidence of breast cancer in women;
discussed under Antiarrhythmics; and calcium impotence; decreased cardiac output, postural
channel blockers are discussed under hypotension; diarrhea, increased gastric
Antianginal Drugs. secretions.
A. Prototype: central acting antihypertensives 4. Nursing implications
(clonidine [Catapres]) a. Monitor for depression; obtain a mental
1. Action. Blocks sympathetic nerve impulses health history for depression.
in brain, which causes decreased sympathetic b. Assess for family history of breast cancer.
outflow leading to decreased BP, c. Administer with meals to reduce GI
vasoconstriction, heart rate, and cardiac distress.
contractility. d. Monitor BP and I&O.
2. Use. Used either alone or in combination with e. Rinse mouth or use hard candy for dry
other antihypertensives. mouth.
3. Adverse effects. Orthostatic hypotension; 5. Related drugs. Guanethidine (Ismelin):
drowsiness, behavior changes; peripheral antihypertensive agent rarely used due to its
edema, CHF; Raynauds phenomenon; adverse effect; affects sympathetic nerve
impotence, urinary retention; dry mouth, endings by releasing norepinephrine; and then
constipation. interferes with release of norepinephrine; with
4. Nursing implications initial use may see transient hypertension and
a. Monitor I&O, weight, and BP. elevated heart rate.
b. Monitor clients with a history of mental E. Prototype: ACE inhibitor (angiotensin-converting
depression. enzyme inhibitor) (captopril [Capoten])
5. Discharge teaching. Take last dose of 1. Action. Lowers BP by inhibiting angiotensin-
medication in the evening to minimize converting enzyme, which inhibits angiotensin
drowsiness during the day. II (vasoconstrictor) and indirectly reduces
B. Related drugs. Methyldopa (Aldomet): may cause serum aldosterone levels.
blood dyscrasias and hepatotoxicity; monitor 2. Use. Initial therapy of essential hypertension
blood work and liver function tests. in clients with normal renal function; severe
C. Prototype: alpha-adrenergic receptor blocker hypertension in clients with renal
(prazosin [Minipress]) dysfunction; CHF.

2 66 NCLEX-RN Review
53155_02_Ch02_p007-110.qxd 2/21/09 10:48 AM Page 67

3. Adverse effects. Blood dyscrasias; 4. Nursing implications


hypotension; proteinuria; hyperkalemia; rash; a. Monitor CBC, ANA titer, and LE
loss of taste perception. preparation.
4. Nursing implications b. Clients receiving parenteral hydralazine
a. Monitor CBC, electrolytes, and urinalysis. (Apresoline) check BP and pulse every 5
b. Administer 1 hour before meals. minutes until stable while on parenteral
5. Discharge teaching agent.
a. Report fever, sore throat, and rash. 5. Related drugs
b. Use salt substitutes only if prescribed a. Minoxidil (Loniten)
(many substitutes contain K+). 1) Has more adverse effects than
6. Related drugs hydralazine. May cause cardiac muscle
a. Enalapril (Vasotec) lesions and hirsutism.
b. Lisinopril (Zestril) 2) Topical preparation (Rogaine) is
c. Benezepril (Lotensin) available to treat baldness.
d. Fosinopril (Monopril) b. Nitroprusside (Nipride)
e. Ramipril (Altace) 1) Given IV to treat hypertensive
f. Losarten (Cozaar) and valasartan (Diovan): emergencies.
angiotensin II receptor blockers 2) Should be administered in an
F. Prototype: direct-acting vasodilators (hydralazine infusion pump. IV tubing and
[Apresoline]) container should be wrapped in
1. Action. Direct relaxation of arteriolar smooth aluminum foil to protect from light.
muscle causing vasodilation. VS, neuro checks, and I&O should be
2. Use. Hypertension; parenteral hydralazine closely monitored.
(Apresoline) is used in hypertensive c. Diazoxide (Hyperstat)
emergencies. 1) Given IV to treat hypertensive
3. Adverse effects. Headache, dizziness, emergencies.
depression; tachycardia, angina, palpitations; 2) May cause hyperglycemia.
lupus-like syndrome, rash, fever; weight gain, See Table 2-17.
sodium retention; nausea, vomiting, anorexia.

Table 2-17 Review of Antihypertensive Drugs

Drug Action Adverse Effects Nursing Implications


Diuretics Block sodium reabsorption in Hyperuricemia, Monitor I&O and observe for excessive
Thiazide diuretics ascending tubule of kidney; hyperglycemia, diuresis.
Chlorothiazide water excreted with hypercalcemia, elevated Encourage client to make positional
(Diuril) sodium, producing BUN; hypokalemia, changes slowly to decrease occurrence
Hydrochlorothiazide decreased blood volume orthostatic hypotension, of orthostatic hypotension.
(Esidrex, Hydro anorexia, nausea, vomiting, Perform baseline and periodic determi-
Diuril, Oretic) light-headedness, nations of electrolytes, BUN, uric acid,
Chlorthalidone headache, drowsiness, rash blood sugar, weight, and blood pressure.
(Hygroton) If possible, administer in the morning to
Metolazone avoid nocturia.
(Zaroxolyn) Administer with food to minimize gastric
distress.
Encourage inclusion of high-potassium
foods in diet to decrease possibility of
hypokalemia. Advise client to avoid
high-sodium foods.
Stress the importance of taking the
drugs regularly as prescribed.

Loop (High-Ceiling) Inhibit reabsorption of Similar to thiazides but See above.


diuretics sodium and chloride at the intensity differs.
Furosemide (Lasix) proximal portion of Hypocalcemia, hearing
Ethacrynic acid ascending loop of Henle loss.
(Edecrin)
Bumetanide (Bumex)

2
(continues)

DRUGS AND NURSING IMPLICATIONS 67


53155_02_Ch02_p007-110.qxd 2/21/09 10:48 AM Page 68

Table 2-17 Review of Antihypertensive Drugs (continued)


Drug Action Adverse Effects Nursing Implications
Potassium-sparing Antagonizes the effect of Hyperkalemia, gynecomastia, Monitor I&O, blood pressure, and
diuretic aldosterone on the tubular hirsutism, irregular menses, weight regularly.
Spironolactone cells of the kidney; sodium rash, drowsiness or confusion Instruct client to be alert for signs of
(Aldactone) excreted in exchange for hyponatremia.
potassium Inform client that swelling and
tenderness of the breasts occur most
often with prolonged therapy.
Monitor serum potassium levels daily
during early stages of therapy.
Advise client to avoid excessive intake of
potassium-rich foods.
Drugs acting on the Metabolized into a false Orthostatic hypotension, Advise slow positional changes and avoid
CNS Methyldopa neurotransmitter displacing sedation, weakness, prolonged standing in one position.
(Aldomet) norepinephrine from its drowsiness, dry mouth, Inform client the drug may darken urine.
receptor sites; sympathetic liver damage Advise client to observe for signs of liver
activity reduced dysfunction.
Clonidine Stimulates alpha-adrenergic Orthostatic hypotension, Closely observe clients with prior history
(Catapres) receptor in brain, causing sedation, drowsiness, dry of mental depression because the drug
inhibition of sympathetic mouth, anxiety, depression may cause depressive episodes.
vasoconstriction Advise slow positional changes.
Encourage client to dangle feet for a few
minutes before standing.
Beta-Adrenergic Reversible competitive Drowsiness, light- Record I&O and clients weight and
Blockers blocking action at beta- headedness, lethargy, notify physician of any significant
Propranolol (Inderal) adrenergic receptor sites. cramping, nausea, and changes.
Nadolol (Corgard) Results in decreased heart bradycardia Advise client to rise slowly, avoid
rate and force of prolonged standing, and be careful
contraction, slowed AV when operating machinery.
conduction, decreased Inform client that smoking may reduce
plasma renin, and lowered effectiveness of drug.
blood pressure Evaluate heart rate and rhythm before
administration of the drug.
Warn client that alcohol may enhance
the hypotensive effect of the drug.
Perform urinary protein estimates prior to
therapy and at monthly intervals thereafter.
Drugs acting on renin- Depresses the functioning of Rash, pruritus, proteinuria Administer (captopril) one hour before meals.
angiotensin system the renin-angiotensin- (mainly captopril), Instruct client to report blood dyscrasias (sore
Captopril (Capoten) aldosterone mechanism by agranulocytosis (mainly throat, fever) and excessive hypotension
Enalapril (Vasotec) inhibiting angiotensin- captopril), excessive (dizziness) and rashes immediately.
converting enzyme (ACE) in hypotension, blood
the plasma dyscrasias

Vasodilators Direct relaxation of arteriolar Headache, nausea, vomiting, Advise client that headache and palpitations
Hydralazine smooth muscle producing diarrhea, sweating, may occur during early stages of therapy.
(Apresoline) decreased peripheral palpitations, and Perform periodic blood counts, LE cell
resistance tachycardia. Systemic preparations, and antinuclear antibody
lupus-like symptoms (high titer determinations.
doses). Advise client to make positional changes
slowly.
Observe clients receiving large amounts
of hydralazine closely for signs of
developing lupus-like reaction.

2 68 NCLEX-RN Review
53155_02_Ch02_p007-110.qxd 2/21/09 10:48 AM Page 69

102. Which side effect should the client be aware of


Sample Questions when taking clonidine (Catapres)?
1. Anxiety.
2. Diarrhea.
98. A client is started on prazosin (Minipress) 1 mg
PO daily. Which client teaching instruction 3. Dry mouth.
should the nurse stress? 4. Irritability.
1. Rise slowly from a lying or sitting position.
2. Take the drug on an empty stomach.
3. Force fluids to 2 liters/day.
4. Take the medication in the morning.
Answers and Rationales
99. Which of the following should the nurse 98. 1. As with most antihypertensives, initial therapy
specifically assess for prior to clients starting on of prazosin (Minipress) may produce orthostatic
captopril (Capoten) therapy? hypotension; client should rise slowly.
1. Depression.
2. Renal dysfunction. 99. 2. Renal damage is an adverse response to
3. Liver disease. captopril (Capoten) that is more apt to occur in
clients with renal dysfunction.
4. Hyperglycemia.
100. 3. Reserpine (Serpasil) has been known to cause
100. Which antihypertensive drug may be linked to
suicidal depression; an increased incidence of
breast cancer and may cause suicidal
breast cancer has also been noted with this
depression?
drug.
1. Guanethidine (Ismelin).
2. Hydralazine (Apresoline). 101. 4. Nitroprusside (Nipride) is sensitive to light
3. Reserpine (Serpalan). and becomes less active; therefore the IV tubing
and container should be covered.
4. Clonidine (Catapres).
102. 3. Clonidine (Catapres) can cause dry mouth due
101. Which of the following actions should the nurse
to decreased salivary flow.
take when administering nitroprusside (Nipride)?
1. Mix the solution in normal saline.
2. Administer the drug by IV push.
3. Monitor neuro checks and VS every hour.
4. Wrap IV bottle and tubing with aluminum foil.

Renal Drugs

DIURETICS hypotension, arrhythmias; anorexia, nausea,


vomiting, diarrhea; agranulocytosis.
A. Prototype: thiazide diuretics (hydrochlorothiazide 4. Nursing implications
(HCTZ) [Hydrodiruill]) Monitor I&O, weights, and serum electrolytes,
1. Action. Blocks sodium reabsorption in the distal glucose, and BUN.
convoluted tubule, which prevents water 5. Discharge teaching
reabsorption, increases urine output, and decreases a. Take medication in the early morning and
blood volume. Potassium is also excreted. after meals to prevent GI distress.
2. Use. Essential hypertension; edema associated b. Report symptoms of agranulocytosis such
with CHF. as fever, sore throat.
3. Adverse effects. Hypokalemia, hyponatremia; c. Change positions slowly.
drowsiness; hyperglycemia; photosensitivity, d. Eat foods high in potassium (i.e., oranges,
bananas, strawberries).

2
hypersensitivity: thiazides are chemically
related to sulfonamides; orthostatic e. Take daily weights.

DRUGS AND NURSING IMPLICATIONS 69


53155_02_Ch02_p007-110.qxd 2/21/09 10:48 AM Page 70

B. Related drugs b. Monitor CBC, serum and urine electrolytes,


1. Metolazone (Zaroxolyn) BUN, blood glucose, uric acid.
2. Chlorathiazide (Diuril) c. Monitor VS closely when client receiving
3. Chlorthalidone (Hygroton) IM or IV administration. Client should be
4. Bendroflumethiazide (Naturetin) switched to oral preparation when practical.
d. IM injection is painful; use Z-track technique.
e. Infusion rate must be closely monitored.
Should not be more than 4 mg per minute.
Sample Questions f. Elderly require close monitoring during
active diuresis. Watch for fluid and
electrolyte imbalances.
103. Which statement by the client indicates that she g. Monitor I&O.
needs more client teaching in regard to h. Weigh client daily.
hydrochlorothiazide (Hydrodiuril) therapy? i. Monitor for hearing loss.
1. I will take my medication with orange juice. j. Monitor clients with diabetes closely.
2. I will take my medication before going to bed. k. Tablets slightly discolored are still
considered potent; discolored parenteral
3. I will take my medication after meals.
solutions should be discarded.
4. I will notify the physician if I have a sore throat. l. Compatible with 5% dextrose in water, sodium
chloride, and lactated Ringers; use solution
mixed with furosemide (Lasix) within 24 hours.
5. Discharge teaching
Answers and Rationales a. Take dose early in day to avoid nocturia.
b. Usually allowed liberal salt intake; consult
103. 2. Taking hydrochlorothiazide (Hydrodiuril) physician.
before going to bed may cause nocturia and c. Caution client about orthostatic
hypotension if on high dose of furosemide
interrupted sleep. The client needs to be taught to
(Lasix) or on other antihypertensive agents.
take the medication in the morning as the diuretic
d. Needs diet high in potassium and maybe a
effect begins in 2 hours and peaks in 4 hours.
potassium supplement.
e. Stay out of the sun and use sunscreen.
6. Related drugs. Ethacrynic acid (Edecrin) and
DIURETICS (CONTINUED) bumetanide (Bumex) are similar to furosemide
(Lasix). Both drugs are used for treating edema
C. Prototype: loop diuretics (furosemide [Lasix])
and can be given orally and parenterally.
1. Action. Acts by inhibiting reabsorption of
sodium and chloride at the proximal portion
of the ascending loop of Henle, increasing
water excretion. This drug is considered to be Sample Questions
potent.
2. Use. Hypertension, pulmonary edema, edema 104. Which group of clients needs special monitoring by
seen with congestive heart failure, cirrhosis,
the nurse while receiving furosemide (Lasix) therapy?
and renal disease.
3. Adverse effects. Fluid and electrolyte imbalances: 1. Premature infants.
hypokalemia, hypochloremic alkalosis, 2. Client with diabetes.
hyperuricemia, hyponatremia, hypocalcemia, 3. Client with asthma.
hypomagnesemia, hyperglycemia; nausea, 4. Clients with peripheral vascular disease (PVD).
vomiting, anorexia, constipation; diarrhea in
children given high doses with sorbitol as the
vehicle; jaundice, acute pancreatitis; polyuria,
nocturia, urinary bladder spasm; dizziness, Answers and Rationales
paresthesias, headache, blurred vision,
irreversible hearing loss; leukopenia, anemia;
orthostatic hypotension, cardiac arrhythmias; 104. 2. Furosemide (Lasix) can cause hyperglycemia.
muscle spasms; photosensitivity, rash, pruritus. Clients with diabetes need close monitoring of urine
4. Nursing implications and blood glucose while on furosemide therapy.
a. Can take with food.

2 70 NCLEX-RN Review
53155_02_Ch02_p007-110.qxd 2/21/09 10:48 AM Page 71

DIURETICS (CONTINUED) 2. Use. Primary hyperaldosteronism; edema;


treatment and prevention of hypokalemia;
D. Prototype: carbonic anhydrase inhibitors essential hypertension.
(acetazolamide [Diamox]) 3. Adverse effects. Hyperkalemia; hyponatremia;
1. Action. Promotes renal excretion of sodium, gynecomastia; thirst, dry mouth, diarrhea;
potassium, bicarbonate, and water via impotence, irregular menses, hirsutism; headache,
reduction of hydrogen ion secretion in the dizziness, drowsiness, confusion; rash, urticaria.
renal tubule cells of the kidney. 4. Nursing implications
2. Use. Adjunct in treating congestive heart a. Give with food.
failure; adjunct in treating open-angle b. Tablet can be crushed.
glaucoma to decrease intraocular pressure; c. Monitor serum electrolytes.
acute mountain sickness; epilepsy. d. Duiretic effect may take until third day of
3. Adverse effects. Nausea, vomiting, anorexia, therapy and may last 23 days after drug is
melena, constipation; hematuria, renal colic, stopped.
renal calculi, crystalluria; liver damage; e. Monitor I&O and check for edema.
fatigue, nervousness, drowsiness, dizziness, f. Weigh client daily.
depression, headache, tremor, convulsions; g. Monitor blood pressure.
transient myopia; bone marrow depression; h. Adverse effects usually reversible if drug
urticaria, pruritus, rash, photosensitivity; discontinued.
weight loss, fever, acidosis; increased 5. Discharge teaching
excretion of calcium, potassium, magnesium, a. Teach signs of electrolyte imbalance and
and sodium; hyperglycemia; hyperuricemia. when to report.
4. Nursing implications b. Consult physician concerning potassium
a. Can be taken with food. and sodium intake. Usually client should
b. Tablets (regular only, not sustained-release) avoid a high-potassium diet.
can be crushed or dissolved in hot water. 6. Related drugs
Will not dissolve in fruit juice. a. Triamterene (Dyrenium): prevents sodium
c. Give in morning to avoid interrupting sleep. reabsorption in the distal tubule of the
d. Avoid IM route; alkalinity of solution kidney, which causes retention of
causes pain. potassium and sodium and water. It is used
e. Monitor clients for metabolic acidosis. to treat edema, hypertension, and
f. Monitor I&O and weight when using drug hypokalemia. Adverse effects:
for edema. hyperkalemia, renal calculi, nausea,
g. Maintain fluid intake to prevent kidney stones. vomiting, anorexia, diarrhea, headache,
h. Many adverse effects are dose related. fatigue, rash, photosensitivity, and blood
i. Monitor diabetic clients. dyscrasias. Nursing implications: give the
j. Potassium loss greatest in early treatment. drug with food to decrease GI upset; give
k. Observe for signs of hypokalemia; those at high the drug in the morning, monitor I&O, body
risk for this are clients receiving other diuretics weight, serum electrolytes, and BUN; client
or digitalis glycosides, and the elderly. should avoid foods high in potassium.
l. Client may need a potassium supplement. b. Amiloride (Midamor): resembles
m. Monitor serum electrolytes, blood gases, triamterene. The biggest difference is
urinalysis. that it is much more potent than
n. Parenteral solution should be used within triamterene.
24 hours after reconstitution. F. Prototype: osmotic diuretics (mannitol
5. Discharge teaching [Osmitrol])
a. Do not interchange brands without asking 1. Action. Acts by increasing the osmotic
physician. pressure of the glomerular filtrate inside the
b. Avoid excess salt intake. renal tubules. This causes less reabsorption
c. Report any adverse effects. of fluid and electrolytes by the tubules
d. Do not drive or perform other activities if and increased loss of fluid, chloride, and
experiencing CNS effects. sodium.
e. If taken in high doses or for long periods of 2. Use. Prevention and treatment of acute renal
time, client will need a diet high in potassium. failure; reduction of intracranial pressure;
E. Prototype: potassium-sparing diuretics reduction of intraocular pressure; urinary
(spironolactone [Aldactone]) excretion of drug overdoses.
1. Action. Acts by blocking aldosterone receptors 3. Adverse effects. Nausea, anorexia, thirst;
in the kidney tubule, thus causing excretion of diuresis, urinary retention; dizziness,
water and sodium and potassium retention. headache, convulsions; pulmonary

DRUGS AND NURSING IMPLICATIONS

2 71
53155_02_Ch02_p007-110.qxd 2/21/09 10:48 AM Page 72

congestion; tachycardia, chest pain, high or


low blood pressure; metabolic acidoses; Answers and Rationales
hypokalemia, hyponatremia, hypochloremia,
dehydration.
4. Nursing implications 105. 4. Mannitol is used primarily in the treatment of
a. Test dose given to clients with advanced intracranial pressure, but also to treat acute renal
oliguria. failure, where rapid reduction of pressure and
b. Monitor serum and urine electrolytes, volume is required.
central venous pressure, and renal
function.
c. Accurate I&O every 30 minutes. POTASSIUM-REMOVING RESIN
d. Monitor VS.
e. Monitor for signs of electrolyte imbalance. A. There is one drug in this category, sodium
f. Weigh client daily. polystyrene sulfonate (Kayexalate).
g. Avoid extravasation. B. Characteristics of polystyrene sulfonate
h. Drug may crystallize if exposed to low (Kayexalate)
temperatures. Warm solution to dissolve 1. A resin that exchanges sodium ions for
crystals. potassium ions in the large intestine.
i. Solutions above 15% have tendency to 2. Used in the treatment of hyperkalemia.
crystallize. IV filter must be used for 3. Given orally or rectally via high enema to both
infusing solutions 15% and above. adults and children.
4. Nursing implications
a. Retain rectal suspension for at least 3060
minutes.
Sample Questions b. Monitor for electrolyte deficiency.
1) Hypokalemia can occur.
2) Magnesium and calcium can also be
105. A client is receiving mannitol. The nurse is
lost.
aware that mannitol will help to decrease what
3) Sodium may be retained.
condition? c. Constipation can occur with oral
1. Hypertension. administration.
2. Hyperkalemia. d. Rectal administration helps prevent
3. Peripheral edema. constipation.
e. Mix resin with sorbitol and water (never
4. Increased intracranial pressure.
with oil).
f. Stop administration when serum
potassium is 45 milliequivalents.

Respiratory Drugs

ANTIASTHMATIC DRUGS more susceptible to developing CNS


stimulation effects); tachycardia, hypotension,
A. Prototype: theophylline palpitations (Note: shouldnt be used in clients
1. Action. Classified as a methylexanthine; a with cardiovascular disease); tachypnea,
bronchodilator that relaxes the bronchial flushing; nausea, vomiting, GI distress (Note:
smooth muscle cells. It also increases renal should not be used in clients with peptic ulcer
blood flow, thus producing a diuretic effect, disease or hyperthyroidism); rectal irritation
and acts as a CNS stimulant. with rectal suppository use.
2. Use. Emphysema; chronic bronchitis; asthma; 4. Nursing implications
CHF. a. Monitor theophylline levels (1020
3. Adverse effects. CNS stimulation: irritability, mcg/mL).
nervousness, restlessness (Note: children are b. Monitor vital signs and symptoms of
toxicity.

2 72 NCLEX-RN Review
53155_02_Ch02_p007-110.qxd 2/21/09 10:48 AM Page 73

c. Clients who smoke tobacco and marijuana c. Rinsing or gargling may reduce irritation in
require higher doses of theophylline. the mouth.
d. Administer with milk or meals if GI d. Discontinue use if an allergic reaction
distress is present, otherwise give 12 occurs.
hours before meals with water. D. Leukotriene inhibitors: Zileuton (Zyflo),
5. Discharge teaching zafirlukast (Accolate), and montelukast (Singulair),
a. Consult with the physician before taking used to prevent asthma attacks.
OTC drugs.
b. Avoid excessive caffeine use.
c. Do not crush or chew time-released or
enteric-coated preparations.
Sample Questions
B. Related drugs (See prototype: theophylline for
adverse effects) 106. Which statement by a clients mother best
1. Aminophylline (Somophyllin)
indicates her understanding of the use of
a. Can be given PO, rectal, IV, or IM.
cromolyn sodium?
b. IM injection is painful and generally
avoided. 1. I will have him take this medication during
c. IV infusion should not exceed an asthma attack.
25 milligrams per minute. 2. I will open the capsule and dilute it in
d. Vital signs should be monitored. juice.
e. Often used to treat severe 3. I will tell him to take a puff of medication
bronchoconstriction. upon exhalation.
f. Avoid mixing with other medications as it
4. I will have him use this medication to
is incompatible with many medications.
2. Theo-dur prevent asthma attacks.
3. Slow-Bid
107. Which adverse effect should a clients mother be
4. Quibron-T
alert for when administering theophylline?
5. Elixophylline
All of the above are derivatives of theophylline. 1. Drowsiness.
Note: they are less potent than theophylline and 2. Irritability and restlessness.
dosage adjustments may be needed. 3. Constipation.
C. Prototype: cromolyn sodium (Intal, Nasalcrom) 4. Bradycardia.
1. Action. Acts on lung mucosa to prevent
histamine release. Classified as a mast cell 108. Which of the following fluids should be avoided
stabilizer. while taking theophylline?
2. Use. Prophylactically to reduce the number of
1. Ginger ale.
asthmatic attacks. It is not used in the
treatment of acute asthmatic attacks; to treat 2. Apple juice.
allergic rhinitis; ophthalmically to treat 3. Hot chocolate.
allergic disorders. 4. Milk.
3. Adverse effects. Bronchoconstriction; cough;
nasal congestion; rash. 109. The clients mother asks the nurse the purpose
4. Discharge teaching of offering theophylline. What is the nurses best
a. Proper use of inhaler response?
1) With spinhaler place capsule in 1. This drug decreases inflammation in the
container and exhale fully.
bronchi.
2) Place mouthpiece between lips.
3) Tilt head back. 2. Theophyllines antihistamine effect will
4) Inhale deeply and rapidly to cause the counteract bronchospasm.
propeller to turn. 3. This drug will help to facilitate removal of
5) Remove the inhaler. secretions.
6) Hold breath a few seconds. 4. Theophylline dilates the bronchial tree and
7) Slowly exhale. will make breathing easier.
b. Capsules should not be swallowed or
opened.

DRUGS AND NURSING IMPLICATIONS

2 73
53155_02_Ch02_p007-110.qxd 2/21/09 10:48 AM Page 74

administration for allergy usually given before


Answers and Rationales meals (ac) or at bedtime (hs) as a single dose.
Monitor respiratory function especially in children
as drug can suppress cough reflex and thicken
106. 4. Cromolyn sodium is effective in preventing bronchial secretions. Can cause photosensitivity.
asthma attacks because it prevents histamine E. Second generation non-sedating
release. antihistamines
1. Desloratadine (Clarinex)
107. 2. Irritability and restlessness are symptoms of 2. Fexofenadine (Allegra)
CNS stimulation, which could lead to seizures. 3. Cetirizine (Zyrtec)
Children are very prone to CNS stimulation with
this drug.

108. 3. Hot chocolate contains caffeine, which can Sample Questions


further increase CNS effects of theophylline.

109. 4. Theophylline dilates the smooth muscle cells 110. What would the nurse teach the client to do to
in the bronchi, which enhances breathing and lessen the sedation effects of antihistamines?
counteracts bronchial constriction.
1. Increase caffeine intake during the day.
2. Take the antihistamine when going to bed.
ANTIHISTAMINES 3. Take the antihistamine with a vitamin.
4. Have a 2-hour nap in the morning and
A. Antihistamines reduce histamine activity by afternoon.
blocking histamine receptor sites. They act within
1530 minutes after administration but are 111. What is an adverse effect that is seen more often
eliminated slowly from the body. Antihistamines in children than adults who are taking
are used to suppress symptoms of histamine antihistamines?
release in allergy. Other uses of antihistamines 1. Dizziness.
include rhinitis, colds, motion sickness, vertigo,
Parkinsons disease, and as a sleep aid. It is 2. Dry mucous membranes.
important to remember to administer any 3. Constipation.
antihistamine before an allergy attack to prevent 4. CNS excitement.
histamine from occupying receptor sites and thus
decreasing the severity of the attack. There are a
few classes of drugs that contain antihistamine
properties. Sedation is the most common adverse
effect of antihistamines. Paradoxical excitation
Answers and Rationales
has been seen in children taking these drugs, and
symptoms such as dizziness, confusion, sedation, 110. 2. The sedation effects of antihistamines will be
and hypotension are seen in the elderly. There are
decreased if the client takes the drug at bedtime.
also anticholinergic effects from antihistamines,
which include dry nose, mouth, and throat; 111. 4. A side effect of antihistamines that is seen
urinary retention; constipation; tachycardia; and more commonly in children than adults is
blurred vision. excitation of the central nervous system.
B. Chlorpheniramine maleate (Chlor-Trimeton): given
PO, IM, SC, and IV. Available in a sustained-
release form. There are increased depressant
effects if taken with alcohol or other CNS MUCOLYTICS
depressants. Give oral forms with food if GI upset
occurs. A. Acetylcysteine (Mucomyst)
C. Diphenhydramine HCl (Benadryl): given PO, IM, 1. Action. Reduces the viscosity of mucus in the
and IV. IM should be given deeply in a large bronchial tree.
muscle mass. Hypersensitivity reactions occur 2. Use. Cystic fibrosis; acute and chronic
more with parenteral administration than with PO. bronchopulmonary diseases such as
Related drugs: Clemastine (Tavist) and pneumonia, bronchitis, and emphysema;
dimenhydrinate (Dramamine). acetylcysteine is the antidote for
D. Promethazine HCl (Phenergan): given PO, IM, acetaminophen (Tylenol) overdose.
rectally, and IV. Can be taken with food. Oral

2 74 NCLEX-RN Review
53155_02_Ch02_p007-110.qxd 2/21/09 10:48 AM Page 75

3. Adverse effects. May cause bronchospasm in B. Antitussives


asthmatic clients and should be discontinued; 1. Antitussives are given to reduce the force and
stomatitis, nausea, vomiting. amount of coughing. They can act centrally by
4. Nursing implications suppressing the cough center in the brain or
a. Suction equipment should be readily peripherally to reduce the susceptibility of
available. irritant receptors to activity. Some antitussives
b. Has a foul odor of rotten eggs. contain narcotics. The antitussives are given
c. Should rinse mouth after treatment. for the symptomatic relief of nonproductive
cough.
2. Dextromethorphan (Benylin DM, Pertussin):
This is the most frequently used non-narcotic
Sample Questions antitussive. Because of its safety record, it is
used for children as well as adults. Common
adverse effects are dizziness, drowsiness, and
112. Which of the following should be available for nausea. It shouldnt be given to clients
clients receiving acetylcysteine? receiving MAO inhibitors.
1. A glass of water. 3. Codeine: Due to its addicting capabilities,
2. Tracheostomy set-up. it should be given in the smallest dose
possible to decrease adverse effects and
3. Suction set-up.
tolerance. The client needs to be watched
4. Room deodorizer. for signs of dependency. Common adverse
effects are nausea, vomiting, and
constipation. Encourage clients to increase
fluid intake and take a laxative if
Answers and Rationales constipation occurs. Provide for client safety
due to codeines sedative effects. If the
client is taking other CNS depressants with
112. 3. Acetylcysteine (Mucomyst) can cause an
codeine, there is an increased chance of
outpouring of copious secretions, which may
CNS effects. Respiratory depression occurs
cause gagging. Suction may be needed to at high doses. Observe respiratory rate and
facilitate the removal of secretions to prevent use cautiously in clients with asthma or
aspiration. emphysema.

EXPECTORANTS AND
ANTITUSSIVES Sample Questions
A. Expectorants
1. Expectorants reduce the viscosity of bronchial 113. Which of the following points should be made
secretions, which allows for their removal regarding guaifenesin by the nurse?
from the lungs. They are used in the 1. Guaifenesin has a high incidence of adverse
management of cough associated with the effects.
common cold and in the treatment of 2. Increase fluid intake to help liquefy and
bronchitis. loosen secretions while taking guaifenesin.
2. Guaifenesin (Robitussin): can be given to
3. Guaifenesin has a high alcohol content.
adults and children. It increases the
respiratory tract fluid thus reducing viscosity 4. This drug can cause blood glucose level to
of secretions. It is the most frequently used rise.
OTC expectorant medication. Client should be
told to increase fluid intake and add 114. In which situation would the use of an
humidification. A common adverse effect is antitussive be inappropriate?
gastric upset, which is caused by its 1. The clients cough is interfering with eating
stimulatory effect on gastric secretions. meals.
3. Terpin hydrate elixir: directly stimulates the 2. The clients cough is associated with a
bronchial secretory glands. Is often used as a suppurative lung disorder.
vehicle for other cough medications. Terpin
3. The clients cough is the source of a
hydrate has a high alcohol content and
complication such as a rib fracture.
shouldnt be given to alcoholics. Also
shouldnt be given to children under 12 years. 4. The clients cough is irritating to the

2
respiratory tract.

DRUGS AND NURSING IMPLICATIONS 75


53155_02_Ch02_p007-110.qxd 2/21/09 10:48 AM Page 76

115. Which adverse effect is associated with the use 114. 2. An antitussive is not appropriate for a client
of high doses of codeine as an antitussive? with lung disease accompanied by increased
1. Diarrhea. sputum as pneumonia or atelectasis could occur.
2. Nasal congestion.
115. 3. Respiratory depression is an adverse effect
3. Respiratory depression. associated with use of codeine. It is a life-
4. Skin rash. threatening effect.

Answers and Rationales

113. 2. Clients taking guaifenesin (Robitussin) need to


increase fluid intake daily to help thin and loosen
secretions. This allows the drug to be more effective.

Gastrointestinal Tract Drugs

HISTAMINE (H2) ANTAGONISTS 5. Discharge teaching. Smoking decreases


effectiveness of cimetidine.
A. Prototype: cimetidine (Tagamet) 6. Related drugs. See Table 2-18.
1. Action. Decreases stomach acidity by B. Proton pump inhibitors
impeding the action of histamine. Competes 1. Prototype: omeprazole (Prilosec)
with histamine for occupancy of histamine 2. Used to decrease gastric acid concentration in
(H2) receptor sites on the parietal cells in the peptic ulcer and gastroesophageal reflux
stomach and suppresses the release of gastric disease.
acid. 3. Nursing implications
2. Use. Short-term treatment of active duodenal a. Do not chew, crush, or open capsule.
ulcer and benign gastric ulcer; decreased dose b. Can be administered with antacids.
after ulcer has healed to inhibit reappearance; 4. Related drugs. Esomeprazole (Nexium),
pathologic hypersecretory conditions, e.g., lansoprazole (Prevacid), pantoprazole
Zollinger-Ellison syndrome. (Protonix), rabeprazole (Aciphex).
3. Adverse effects. Diarrhea, muscle pain, rash; C. Peptic ulcers caused by Helicobacter pylori
CNS effects of dizziness, confusion, are commonly treated with a combination of
drowsiness, headache; changes in liver two antibiotics, a bismuth compound and a
function studies; agranulocytosis and histamine antagonist or proton pump
neutropenia; antiandrogenic effects inhibitor.
(impotence and gynecomastia).
4. Nursing implications
a. Oral form should be taken with meals. Table 2-18 Other H2 Antagonists
b. Antacids decrease absorption; give
antacid 1 hour before or after Drug Nursing Implications
administration. Ranitidine Avoid administration of antacids at
c. Usual course of treatment for ulcer disease (Zantac) same time.
is 46 weeks. Hemodialysis may reduce Zantac
d. Many drug interactions. blood levels.
e. Watch for CNS changes particularly in the Famotidine May be given with antacid dose.
elderly as confusion is a major toxic (Pepcid) Dosage modifications not necessary in
effect. elderly except with renal impairment.
f. Use cautiously with clients who have Nizatidine Can elevate serum salicylate levels in
impaired renal or hepatic function. (Axid) clients taking high doses of aspirin.

2
g. Monitor CBC and liver function studies.

76 NCLEX-RN Review
53155_02_Ch02_p007-110.qxd 2/21/09 10:48 AM Page 77

118. 2. Smoking decreases the effectiveness of


Sample Questions cimetidine (Tagamet) and is also contraindicated
in ulcer disease.

116. What is the primary action of cimetidine? 119. 1. Confusion is a major toxic effect in the
1. Suppresses the action of acetylcholine at the elderly.
receptor responsible for histamine release.
2. Decreases the pH of gastric fluids.
GASTROINTESTINAL (GI)
3. Antagonizes the action of histamine at its H2
receptor site. ANTICHOLINERGICS
4. Neutralizes gastric secretions.
With the advent of H2 antagonists, gastrointestinal
117. An antacid is ordered in conjunction with the anticholinergics are rarely used. H2 antagonists have
cimetidine. What instruction should the nurse a more prolonged action and fewer side effects, and
give concerning concurrent use of an antacid are considered more effective in treating gastric
with cimetidine? ulcers. Gastrointestinal anticholinergics delay gastric
emptying time, which prolongs the action of
1. Take both drugs together. antacids.
2. Take both drugs with milk.
3. Take both drugs with meals.
4. Take the drugs 1 hour apart. ANTACIDS
118. Which statement made by the client best A. There are five antacid categories with the same
indicates his understanding of cimetidine action of neutralizing gastric acid. There are
therapy? significant differences among each of the five
categories; therefore, no individual antacid is
1. I will stop taking cimetidine when my
considered a prototype. The categories are displayed
stomach pain is gone.
in Table 2-19: magnesium, compounds of
2. I will stop smoking. aluminum, sodium, calcium, alkaline, or a
3. I will take cimetidine on an empty stomach. combination.
4. I know that cimetidine will turn my stools 1. Use. Control ulcer pain; peptic ulcer;
black. esophageal reflux; prophylaxis for Curlings
ulcer.
119. In elderly clients taking cimetidine, which 2. Nursing implications for all antacids
adverse effect should the nurse be most a. Shake liquid antacids prior to use.
concerned with? b. Liquids tend to be more effective than
tablets.
1. Confusion.
c. Tablets must be chewed completely before
2. Diarrhea. swallowing.
3. Muscle pain. d. Take a sip of water following antacid
4. Constipation. administration to ensure passage to the
stomach.
e. Large amounts of water will dilute the antacid.
f. Aluminum and magnesium combinations
to reduce the side effects of diarrhea and
Answers and Rationales constipation.
g. Do not take other oral drugs within 12
hours of an antacid.
116. 3. Cimetidine (Tagamet) competes with h. Can interfere with the intended response of
histamine for occupancy of histamine (H2) enteric-coated medications.
receptor sites on the parietal cells in the B. See Table 2-19 for categories and adverse effects.
stomach.

117. 4. Antacids decrease cimetidine absorption.

DRUGS AND NURSING IMPLICATIONS

2 77
53155_02_Ch02_p007-110.qxd 2/21/09 10:48 AM Page 78

Table 2-19 Antacids

Drug Categories Adverse Effects Nursing Implications


Magnesium-containing antacids Diarrhea Doses greater than 1.5 mL may have a laxative
Magnesium Hydroxide effect.
(Milk of Magnesia) Contraindicated in clients with renal failure.
Monitor for symptoms of hypermagnesemia,
CNS changes, hypotension, nausea, vomiting.
Aluminim-containing antacids Constipation Check serum for hypophosphatemia.
Aluminum Hydroxide Monitor for fecal impaction and intestinal
(Amphogel) obstruction in elderly.
Aluminum can accumulate in CNS causing toxic
effects; not given over a long period of time.
Sodium Carbonate/Bicarbonate Constipation, sodium retention Must chew tablet.
Dihydroxyaluminum Not for long-term use.
Sodium Carbonate (Rolaids)
Sodium Bicarbonate Systemic alkalosis, bloating, sodium Contraindicated in congestive heart failure,
(Baking Soda) retention hypertension, or sodium-restricted diets.
Monitor for milk-alkali syndrome, nausea,
vomiting, headache, hypercalcemia,
hypercalciuria, hypophosphatemia.
Calcium Carbonate Acid rebound, milk-alkali syndrome, Monitor for milk-alkali syndrome.
(Tums, Maalox) hypercalcemia, constipation Not to be taken with milk or foods high in
vitamin D.
Aluminum Magnesium Constipation or diarrhea, Other common antacids in this group are
combinations hypermagnesemia Maalox, DiGel, Gelusil.
Magaldrate (Riopan) Use cautiously in clients with impaired renal
function and monitor magnesium levels.

Sample Questions Answers and Rationales

120. Which antacid is LEAST LIKELY to cause 120. 3. Antacid combinations containing aluminum
adverse effects? and magnesium have reduced adverse effects as
1. Magnesium hydroxide (Milk of Magnesia). aluminum antacids cause constipation and
2. Aluminum hydroxide (Amphogel). magnesium antacids cause diarrhea.
3. Aluminum/magnesium combination
121. 3. Antacids such as Maalox neutralize gastric
(Maalox).
acidity, thus reducing pain. Constipation and
4. Calcium carbonate (Tums). diarrhea are adverse effects with antacids and
may be decreased when taking a combination
121. What is one of the first effects that the nurse will
antacid acid such as Maalox.
tell the client to expect from taking Maalox?
1. Constipation.
2. Decrease of gastric acid secretions. ANTIDIARRHEAL AGENTS
3. Alleviation of burning pain.
4. Diarrhea. A. Antidiarrheal agents slow intestinal motility and
propulsion. There are two categories of drugs to
treat diarrhea.
1. Absorbents: act by binding drugs, digestive
enzymes, toxins, bacteria which may be

2
causing diarrheal condition.

78 NCLEX-RN Review
53155_02_Ch02_p007-110.qxd 2/21/09 10:48 AM Page 79

2. Opiates: act by reducing the propulsive


movement of small intestine and colon, Answers and Rationales
causing dehydration of intestinal contents.
B. Related drugs. See Table 2-20.
122. 1. Lomotil contains atropine, which is an
anticholinergic that can cause urinary retention.
Sample Questions 123. 4. Pepto-Bismol contains small amounts of
salicylate. Salicylates are connected with Reyes
122. The physician orders Lomotil 5 mg PO QID for syndrome and are not recommended in children
diarrhea. What adverse effect of Lomotil must under the age of 18.
the nurse be alert for?
1. Urinary retention.
LAXATIVES
2. Decreased peristalsis.
3. Tinnitus. A. Protype: Laxatives either stimulate or change the
4. Diarrhea. consistency of stools. Five of the most common
categories of laxatives are used to induce 1 or more
123. A clients daughter mentions to the nurse that bowel movements per day.
her 6-year-old son often takes Pepto-Bismol for 1. Use. Promote movement of feces through the
an upset stomach. What potential problem bowel to prevent/treat constipation, prevent
should the nurse reinforce that Pepto-Bismol straining during defecation, or for preparation
could cause in children? before diagnostic tests/surgery.
2. Saline cathartics attract and hold large
1. Interacts with multivitamins.
amounts of fluids thereby increasing the bulk
2. Will cause anorexia. of stools. The nurse needs to encourage fluid
3. Has an antibiotic effect. intake to prevent dehydration.
4. Contains salicylate.

Table 2-20 Antidiarrheals

Category Drug Adverse Effects Nursing Implications


Adsorbent Bismuth subsalicylate Salicylate poisoning, impaction, Over-the-counter (OTC) preparation
(Pepto-Bismol) darkening of stool and tongue Use cautiously with aspirin or other
aspirin-containing drugs
Kaolin and Pectin Constipation, fecal impaction OTC preparation
(Kaopectate) Regular and concentrated suspensions
Extended use can cause disruption of
nutrient absorption in body.
Opiate Loperamide Hydrochloride Toxic megacolon in clients with Synthetically related to meperidine
(Imodium) ulcerative colitis; drowsiness, (Demerol)
dizziness, abdominal discomfort, Shouldnt be taken together with other
constipation CNS depressants
Treat overdose with slurry of activated
charcoal and watch for CNS depression.
Diphenoxylate Sedation, flushing, palpitations, Withhold in severe dehydration or
Hydrochloride with blurred vision, dry mouth, urinary electrolyte imbalance
Atropine (Lomotil) retention Addiction possible in high doses and
with prolonged use
Opium Tincture (Paregoric) Respiratory depression, physical Reduced dose for elderly and clients
and psychological dependence, with respiratory problems
mental impairment Safety precautions
Give with 23 swallows of water to
ensure passage to stomach

DRUGS AND NURSING IMPLICATIONS

2 79
53155_02_Ch02_p007-110.qxd 2/21/09 10:48 AM Page 80

Table 2-21 Laxatives

Category Drug Adverse Effects Nursing Implications


Saline Magnesium citrate Hypermagnesia in renal Laxation in 26 hrs
(Citroma) failure, nausea Available in effervescent form
Most effective on empty stomach and
followed with a glass of water
Magnesium hydroxide See magnesium citrate Lower doses act as antacid
(Milk of Magnesia) Shake bottle before taking
Glycerin (Glycerol) Abdominal cramps, rectal Evacuation occurs in 1530 minutes after
discomfort enema/suppository
Bulk-forming Methylcellulose Fecal impaction, esophageal Laxation in 1272 hrs
(Citrucel, Cologel) obstruction, nausea Take with 1 glass of water to prevent fecal
impaction
Must not chew tablet form or take in dry
powder form as it may cause esophageal
obstruction
Psyllium hydrophilic Same as above Same as above
muciloid (Metamucil)
Lubricant Mineral oil Impairs absorption of fat- Administer in an upright position
soluble vitamins and Do not take with meals, first given in evening
nutrients, lipid pneumonia Retention enema is usually followed by a
cleansing enema
Keep in refrigerator

Stool softener Ducosate sodium Rare: rash, abdominal cramps Effective in 1272 hrs
(Colace) Take with plenty of fluids
Use caution with clients on sodium-restricted
diets

Stimulant Bisacodyl (Dulcolax) Rare Effect in 12 hr PO, and 15 min.1 hour in


rectal form
PO drug given before breakfast or at bedtime
Do not take tablets within 1 hour of
antacid or milk administration
Cascara Sagrada Large doses can cause Could discolor urine
hypokalemia, glucose Effect in 612 hrs
intolerance, calcium Prolonged use can cause rebound
deficiency, anorexia constipation
Castor oil (Emulsoil, Rebound constipation, Effect in 23 hrs
Neoloid) abdominal cramps, nausea, Do not schedule within 2 hours of taking
vomiting other oral drugs
Shake emulsion well
Give with juice or carbonated beverage to
mask castor oils unpleasant odor and taste
Phenolphthalein (Ex-Lax, Rash, lupus-like syndrome Effect in 68 hrs
Feen-a-Mint, Correctol) Urine/feces may have reddish
discoloration
Usually given at bedtime
Effect could last 34 days
Discontinue if rash occurs
Senna (Senokot) Abdominal cramps, nausea Laxation in 610 hrs
Administer at bedtime
Urine/feces may have yellowish-brown or
reddish-brown discoloration

2 80 NCLEX-RN Review
53155_02_Ch02_p007-110.qxd 2/21/09 10:48 AM Page 81

3. Bulk-forming laxatives increase the bulk of the sickness, CNS, disorders, administration of certain
feces by stimulating mechanical peristalsis drugs, and radiation therapy. There are five categories
and are considered the safest of all the laxative of antiemetics:
groups. 1. Antihistamines and anticholinergics: block
4. Lubricant laxatives coat the feces with an oil acetylcholine and histamine H2 receptors.
film and prevent the colon from reabsorbing 2. Neuroleptics: bind with dopamine2
water from the feces. 3. Prokinetics: stimulate acetylcholine to
5. Stool softeners prevent straining during increase gastric emptying.
defecation and prevent constipation by 4. Serotonin-blocking agents: block transmission
decreasing surface tension of feces. of afferent visceral and chemoreceptor triggers
6. Stimulant laxatives stimulate peristalsis. (mostly associated with chemotherapy).
7. Should not be given to clients with symptoms 5. Substance P Neurokinin-1 receptor antagonist:
of nausea, vomiting, abdominal pain, used in conjunction with serotonin
symptoms of appendicitis, or intestinal antagonists and a corticosteroid.
obstruction. See Table 2-22.
8. Used for a week or less to prevent rebound
constipation and dependence.
9. Appropriate fluid intake and a diet high in
fiber will help promote proper bowel function. Sample Questions
B. Related drugs. See Table 2-21.

126. What nursing diagnosis would be appropriate


for a client taking promethazine (Phenergan)?
Sample Questions
1. Disturbed body image.
2. Diarrhea.
124. What group of laxatives would the nurse tell the 3. Risk for injury.
client is considered the safest and most natural?
4. Chronic pain.
1. Saline-cathartics group.
2. Lubricant-laxative group.
3. Stool-softener group.
4. Bulk-forming laxative group.
Answers and Rationales
125. The nurse should withhold a laxative from the
126. 3. Due to the side effect of drowsiness, the client
client that has what condition?
needs to refrain from driving or operating
1. Nausea. dangerous machinery while taking this
2. Excess body weight. medication.
3. Increase in appetite.
4. Painful defecation. EMETICS
A. Prototype: ipecac syrup
Answers and Rationales Not fully known but probably stimulates the CTZ
and irritates the GI tract to induce vomiting, thus
delaying the absorption time of toxic substances.
124. 4. Bulk-forming laxatives are considered the Emesis should occur within 2030 minutes.
safest and most natural of all laxative groups 1. Use. Stimulate vomiting for clients (in a
because they are natural or semisynthetic hospital setting) who have taken toxic doses of
substances, they produce stools that are formed oral medications, and poisons.
normally, and are not absorbed systemically. 2. Adverse effects. Fluid and electrolyte
imbalance; stimulated and then suppressed
125. 1. A client with nausea should never be given a nervous system; hypotension; persistent
laxative because this person is at risk for fluid vomiting; aspiration.
and electrolyte imbalance. 3. Nursing implications
a. Should not be given after charcoal
administration as it antagonizes the effects.
ANTIEMETICS b. Administer with water and monitor vital
signs.
A. Protype: Antiemetics are given to prevent and treat c. Repeat dose once if vomiting does not occur.

2
nausea, vomiting, and nausea associated with motion

DRUGS AND NURSING IMPLICATIONS 81


53155_02_Ch02_p007-110.qxd 2/21/09 10:48 AM Page 82

Table 2-22 Antiemetics

Category Drug Adverse Effects Nursing Implications

Antihistamines Diphenhydramine Dry mouth, drowsiness If preventing nausea, give 30 minutes prior
(Benadry) Promethazine HO to noxious stimuli
Antivert Phenergan Not compatible with lactated Ringers IV
solution
Anticholinergics Scopolamine (Transderm- Increased heart rate, Monitor VS
Scop) disorientation

Neuroleptics Chlorpromazine HCl Drowsiness, blurred vision, If IM, use large muscle
(Thorazine) hypotension, Dizziness Do not give SC
Prochlorperazine
(Compazine)
Prokinetic Metoclopramide Anxiety Monitor for dehydration
(Reglan)
Serotonin Dolasetron mesylate Headache, hypotension, GI Monitor BP
blockers (Anzement) upset Give 30 min before chemo
Ondansetron HCL
(Zofran)
Substance P Aprepitant Asthenia, hiccups Give in morning
neurokinin-1 Use additional contraception
receptor antagonist

d. If a client is younger than age 10 only one


dose should be given. Answers and Rationales
e. Should not be given to semiconscious or
unconscious clients or clients having
seizures. 127. 4. The Poison Control Center should be called
f. Should not be given if substance ingested for instructions.
is corrosive, petroleum-based, or
cyanide.
g. Not to be used in home setting. SUCRALFATE (CARAFATE)
B. Prototype: apomorphine HCl.
A CNS depressant and a controlled substance that A. Prototype: sucralfate (Carafate)
is given subcutaneously. In adults is usually 1. Action. Reacts with gastric acid to form a
effective in 515 minutes. Clients who are allergic substance that adheres to the ulcer site to
to morphine or other opiates should not take protect the ulcer from bile salts, pepsin, and
apomorphine. acid. This permits healing.
2. Use. Treatment of duodenal ulcer: short term
(up to 8 weeks).
3. Adverse effects. Diarrhea, constipation; gastric
Sample Questions discomfort; dry mouth; pruritus, rash; back
pain; dizziness, sleeplessness.
4. Nursing implications
127. What action should the client perform after an a. Separate administration from other drugs
accidental poisoning? by 2 hours to decrease chance of
1. Drink milk. interaction.
b. Take on an empty stomach.
2. Drink water until vomiting occurs.
c. Antacids should not be given within 1/2
3. Take laxatives. hour before or after sucralfate dose.
4. Call the Poison Control Center. 5. Related drug. Misoprostol (Cytotec).

2 82 NCLEX-RN Review
53155_02_Ch02_p007-110.qxd 2/21/09 10:48 AM Page 83

a. Protects stomach lining by increasing 129. Which of the following teaching points about
mucus and bicarbonate production and sucralfate should the nurse emphasize to ensure
inhibiting secretion of gastric acid. success with the therapy?
b. Used to prevent NSAID and aspirin- 1. Discontinue the drug if indigestion occurs.
induced ulcers in clients at high risk of
2. Use the drug on an as-needed basis if future
complications from gastric ulcers.
ulcer problems occur.
3. Constipation may occur with long-term use.
4. Take the drug 1 hour before meals or 2 hours
Sample Questions after meals and at bedtime.

128. By what mechanism does sucralfate enable the


clients ulcer to heal? Answers and Rationales
1. Coating the stomach mucosa, thus preventing
gastric acid irritation.
128. 2. Sucralfate (Carafate) reacts with gastric acid
2. Forming a paste-like substance at the ulcer
and becomes a paste-like substance that forms a
site that serves as a protective barrier.
protective barrier at the ulcer site.
3. Binding with all gastric contents to inactivate
their function in the stomach. 129. 4. Sucralfate is more active in the lower-pH
4. Attracting white and red blood cells to the environment of an empty stomach. By taking the
stomach to increase circulation. drug 1 hour before meals or 2 hours after meals,
the stomach is empty and optimal results will be
obtained from the drug.

Arthritis Drugs

ARTHRITIS DRUGS d. VS should be monitored and resuscitation


equipment should be readily available
A. Prototype: auranofin (Ridaura) with test dose.
1. Action. Mechanism of action is unclear. This e. Diarrhea is more severe with oral gold
is the only group of drugs that may partially salts.
reverse or stop joint destruction. f. Oral auranofin is less toxic and better
2. Use. These drugs are most effective early in tolerated than IM.
rheumatoid arthritis. g. Overdose of gold salts can be treated with
3. Adverse effects. Most common: skin rash, dimercaprol (BAL).
proteinura, blood dyscrasias, gastric irritation, h. Gold salts given IM are best given in the
diarrhea. Nitroid crisis is an anaphylactic gluteal muscle.
reaction that resembles effects of a large dose 5. Discharge teaching. Be aware that therapeutic
of nitroglycerin: i.e., flushing, severe effects may not be seen for several months.
hypotension, tachycardia, light-headedness. B. Other gold salts
4. Nursing implications 1. Aurothioglucose (Solganol)
a. Gold salts should not be given to clients with 2. Gold sodium thiomalate (Myochrysine)
hepatic and renal disorders, hypertension, 3. Both drugs are given IM only.
uncontrolled diabetes, or heart failure, Noncompliance can be a problem with both
or to clients receiving radiation therapy. drugs as weekly injections may be needed for
b. Baseline CBC must be checked prior to several months.
administration and checked throughout C. Antimalarials
therapy. 1. Chloroquine (Aralen)
c. Nitroid crisis is more apt to occur with IM 2. Hydroxychloroquine (Plaquenil)
injection of gold salts; therefore a test dose 3. These drugs are used to treat malaria and
is given. rheumatoid arthritis that is unresponsive to
NSAIDs.

DRUGS AND NURSING IMPLICATIONS

2 83
53155_02_Ch02_p007-110.qxd 2/21/09 10:48 AM Page 84

4. Nursing implications 131. 2. It can take 6 months or longer for the gold
a. The nurse needs to remind clients that salts to show a therapeutic response. Because of
frequent ophthalmic examinations are this, clients are left on gold salt therapy even
required as visual impairment can though they are experiencing adverse effects
occur. before therapeutic response.
b. The nurse must be alert for blood dyscrasia,
GI distress, and dermatologic reactions.
D. Other drugs ANTIGOUT DRUGS
1. Adalimumab (Humira)
2. Etanercept (Enbrel) A. Prototype: allopurinal (Zyloprim)
3. Leflunomide (Avara) 1. Action. Prevents the production of uric acid by
4. Infliximab (Remicade) inhibiting the enzyme xanthine oxidose.
5. Penicillamine (Cupriminel) 2. Use. Used to manage primary or secondary
6. These are newer antiarthritis agents used to gout and to prevent attacks; used to treat
treat rheumatoid arthritis that no longer clients with recurrent calcium oxalate
responds to traditional therapy. calculi.
3. Adverse effects. GI symptoms: nausea,
vomiting, diarrhea; skin rash, maculopapular;
hepatomegaly; drowsiness.
Sample Questions 4. Nursing implications
a. Discontinue use at first sign of skin rash.
b. Force fluids (12 liters) to help prevent
130. When a client comes to the office for monthly formation of uric acid kidney stones.
visits, which of the following would be excluded c. Monitor liver function tests and CBC
from the nurses assessment? throughout therapy.
1. Assess clients skin. d. Administer drug after meals.
2. Examine clients oral cavity. 5. Discharge teaching. Advise client not to drive.
B. Prototype: colchicine (Novocolchine)
3. Question client about any itching. 1. Action. Drug of choice to treat acute gout
4. Check clients blood sugar levels. attacks and prophylaxis of recurrent gout. It
decreases the inflammatory response to
131. The client has been receiving auranofin for 2 deposition of monosodium urate crystals.
months and is showing little clinical response to 2. Adverse effects. Nausea, vomiting, diarrhea,
it; she is experiencing the following adverse abdominal pain; bone marrow depression; hair
effects of the drug: abdominal cramps, vomiting, loss; rash; thrombophlebitis if given IV.
diarrhea, and stomatitis. The physician decides to 3. Nursing implications
continue auranofin. What is the reason for this? a. Do not give IM or SC as this causes severe
1. Side effects of auranofin are rare and will irritation.
pass quickly. b. Monitor IV site.
c. Assess for rash.
2. It will take several months of therapy with
d. Monitor CBC.
auranofin to achieve therapeutic effects.
e. Take drug after meals.
3. The physician is not sure of the best effects f. Hair loss is reversible when drug is
from auranofin because it is such a new form stopped.
of treatment. g. During an acute attack is usually given
4. Side effects of auranofin always occur just every 12 hours until pain is relieved and
before the beginning of a therapeutic should be stopped if nausea, vomiting, or
response to the drug. diarrhea occurs.
h. No more than 12 tablets should be given in
a 24-hour period.

Answers and Rationales


Sample Questions
130. 4. Skin rashes, pruritus, and mouth lesions are
all adverse effects of gold salts and need to be
monitored frequently. Hyperglycemia has not 132. What instruction should the nurse give in regard
been associated with gold salt therapy. to colchicine therapy?

2 84 NCLEX-RN Review
53155_02_Ch02_p007-110.qxd 2/21/09 10:48 AM Page 85

1. The drug will be absorbed better if you take it 3. Naproxen (Naprosyn)


on an empty stomach. 4. Sulindac (Clinoril)
2. You should limit your fluid intake while
youre on this drug.
3. No more than 12 colchicine tablets should be
taken in a 24-hour period. Answers and Rationales
4. If you have another gout attack, you need to
wait 12 hours before taking colchicine.
132. 3. In order for the client to be treated safely
133. What client condition would alert the nurse to with colchicine (Novocolchine), not more
discontinue colchicine? than 12 tablets should be taken in a 24-hour
period.
1. The client becomes dizzy.
2. Diarrhea or vomiting occurs. 133. 2. Diarrhea and vomiting are both early signs of
3. Stools turn black. colchicine toxicity. Other signs of early toxicity
4. Serum uric acid level is below normal. are anorexia, nausea, abdominal discomfort, and
weakness. Colchicine should be stopped
134. In addition to treating gouty arthritis, which immediately before more serious toxicity
drug is also used to treat calcium oxalate occurs.
stones?
1. Allopurinol (Zyloprim) 134. 1. Allopurinol (Zyloprim) is used to manage and
prevent gout attacks. A new use for this drug is
2. Colchicine (Novocholchine)
treatment of calcium oxalate stones.

Antimicrobials

GENERAL INFORMATION 5. Agents that inhibit nucleic acid synthesis:


action involves the use of enzymes for
A. Terminology reproduction that are not found in human
1. Bacteriostatic: prevents multiplication and cells.
growth of bacterial organisms. D. Need to be administered at regular intervals so
2. Bactericidal: kills bacterial organisms. therapeutic blood levels can be maintained. This
B. Cultures need to be obtained before initiating will prevent development of resistant strains of
therapy. organisms. An order for QID administration means
C. Sites of action giving the drug at 6-hour intervals.
1. Agents that suppress bacterial cell wall E. Peak and trough levels
synthesis: action creates a defect in bacterial 1. Blood levels need to be high enough to be
cell wall structure and death of organism. therapeutic but not so high that severe toxicity
2. Agents that suppress protein synthesis within is caused.
the bacterial cell: action interferes with normal 2. Peak: clients blood is drawn 1 hour after IM or
growth and reproduction of bacterial cell, 30 minutes after IV administration.
which eventually causes its eradication. 3. Trough: clients blood is drawn just before next
3. Agents that interfere with bacterial cell dose of antibiotic is given.
membrane permeability: action causes F. Superinfection
intracellular parts to escape and leads to 1. Infection occurring when client is receiving or
bacterial cell death. has recently been given antibiotic treatment.
4. Agents with antimetabolite action: action 2. Develops when normal bacterial flora are
causes interference with a necessary metabolic changed by the use of an antibiotic. Allows
process that the bacterial cell needs for normal growth of bacteria that are resistant to the
growth and function. antibiotic being used.

DRUGS AND NURSING IMPLICATIONS

2 85
53155_02_Ch02_p007-110.qxd 2/21/09 10:48 AM Page 86

3. Clients more susceptible when placed on 3. Bactericidal.


broad-spectrum antibiotics. 4. Anthelmintic.
G. Resistance
1. Many bacteria have developed resistance to 136. The nurse will assess the client daily for which
antibiotic therapy. of the following adverse effects of gentamicin
2. Ways to prevent resistance (Garamycin)?
a. Use antibiotics only when necessary. 1. Constipation.
b. Do not use antibiotics to treat viral infections.
3. Streptogramins: antibiotics to treat resistant 2. Hearing loss.
strains of bacteria. Used to treat vancomycin 3. Tetany.
resistant enterococcus (VRE) and methicillin 4. Bradycardia.
resistant S. aureus (MRSA).
a. Quinupristin/dalfopristin (Synercid) 137. The nurse needs to monitor the client daily for
b. Linezolid (Zyvox) the presence of superinfection. How is a
superinfection defined in relation to an
infection?
AMINOGLYCOSIDES 1. It has an increasing number of the organisms
normal microbial competitors.
A. Prototype: gentamicin (Garamycin)
1. Action. Acts by suppressing protein synthesis 2. It necessitates increased amount of
in bacterial cell. Bactericidal. antibiotics for treatment.
2. Use. Serious gram-negative bacterial 3. It occurs if antibiotic therapy is abruptly
infections, eye infections. stopped.
3. Adverse effects. Ototoxicity, nephrotoxicity, 4. It develops when an antibiotic alters the
neuromuscular blockade, hypersensitivity, normal bacterial flora.
photosensitivity with topical preparations.
4. Nursing implications
a. Cautious use in clients with decreased
renal function, reduced hearing, Answers and Rationales
dehydration, neuromuscular disorders.
b. Monitor hearing and balance.
c. Monitor renal function tests and I&O. 135. 3. Gentamicin (Garamycin) acts by suppressing
d. Client needs adequate hydration. protein synthesis in the bacterial cell. This effect
e. Safety precautions if there are vestibular is bactericidal.
nerve effects.
f. Monitor drug levels. 136. 2. Gentamicin (Garamycin) can cause auditory
5. Discharge teaching and vestibular damage. The nurse must assess
a. Full course of treatment is essential. the client daily for hearing loss while on this
b. Report problems with balance or hearing drug.
changes.
c. Avoid sunlight. 137. 4. A superinfection occurs when normal bacterial
B. Related drugs. Amikacin (Amikin), kanamycin flora are changed by the use of an antibiotic. This
(Kantrex), neomycin (Neobiotic), streptomycin, allows growth of bacteria that are resistant to the
tobramycin (Nebcin), netilmicin (Netromycin), antibiotic currently being used.
paromomycin (Humatin). Kanamycin (Kanrex) and
neomycin (Neobiotic) are given orally to prepare the
bowel for surgery. Neomycin (Neobiotic) is given to
persons in hepatic failure to reduce ammonia levels. PENICILLINS
A. Prototype: penicillin G potassium (Pentids)
1. Action. Inhibits cell wall synthesis of
microorganisms. Bactericidal. Natural
Sample Questions penicillin.
2. Use. Systemic infections caused by gram-
positive cocci; syphilis; prophylaxis for
135. What is the antibacterial action of gentamicin
rheumatic fever and bacterial endocarditis.
(Garamycin)? 3. Adverse effects. Hypersensitivity reactions; GI
1. Anti-inflammatory. upset; anemia, thrombocytopenia, leukopenia;
2. Bacteriostatic. nephritis; potassium poisoning; irritation at
injection site.

2 86 NCLEX-RN Review
53155_02_Ch02_p007-110.qxd 2/21/09 10:48 AM Page 87

4. Nursing implications about penicillins. Which of the following


a. Monitor client for allergic reactions. Have statements is true?
emergency equipment available. 1. Penicillins are well absorbed from the
b. Clients with questionable serious gastrointestinal tract after oral ingestion.
penicillin allergy may be skin tested.
2. Penicillins have a long half-life due to their
c. Give oral form on empty stomach.
d. Oral form should be taken with a full glass extended excretion time.
of water. 3. Extended-spectrum penicillins have an
e. Monitor CBC, BUN, and creatinine. increased ability to penetrate outer
f. Probenecid (Benemid) may be given to membranes of gram-negative bacteria.
increase blood levels of penicillins. 4. Penicillins easily enter bacterial cell
g. Monitor IV and IM injection sites. membranes due to their lipid solubility.
h. IV solutions are stable at room temperature
for 24 hours only. 139. Penicillin is ordered for a client. The physician
5. Discharge teaching also orders probenecid therapy. What will the
a. Complete the therapy even if you feel well nurse tell the client that the action of probenecid
before the medicine is finished. will do?
b. Oral doses should be taken around the 1. Prevent a hypersensitivity reaction to the
clock.
penicillin.
c. Dont take for other infections.
B. Related drugs 2. Enhance metabolism of the penicillin.
1. Penicillinase-resistant penicillins 3. Stimulate the immune system.
a. Used to treat infections caused by 4. Increase blood levels of the penicillin.
penicillinase-producing organisms.
b. Examples: nafcillin sodium (Nafcil,
Unipen), cloxacillin (Tegapen),
dicloxacillin (Oxapen) Answers and Rationales
2. Aminopenicillins
a. Increased effectiveness against gram-
negative organisms. 138. 3. Extended-spectrum penicillins enter the
b. Examples: ampicillin (Amcill, Polycillin), membranes of gram-negative organisms more
amoxicillin trihydrate (Amoxil), readily than other penicillin groups.
bacampicillin (Spectrobid)
3. Extended-spectrum penicillins 139. 4. Probenecid (Benemid) is given with penicillin to
a. Structurally similar to ampicillin but have enhance therapeutic blood levels of the penicillin.
an increased spectrum of activity against
gram-negative bacteria.
b. Examples: carbenicillin sodium CEPHALOSPORINS
(Geocillin), piperacillin sodium (Pipracil),
ticarcillin (Ticar), mezlocillin (Mezlin) A. Prototype for first-generation cephalosporins:
4. Penicillin/beta-lactamase inhibitor cefazolin sodium (Ancef). Note: The
combinations cephalosporins are divided into four groups or
a. Combination of penicillin with beta- generations based on their spectrums of activity.
lactamase inhibitor which prevents 1. Action. Inhibits bacterial cell wall synthesis.
destruction of penicillin by enzymes and Bactericidal.
extends the penicillins spectrum of 2. Use. Infections caused by gram-positive cocci;
antimicrobial activity. septicemia; bone, joint, and skin infections;
b. Examples: Amoxicillin/potassium prophylactic use in surgery; serious intra-
clavulanate (Augmentin), abdominal infection.
piperacillin/tazobactam (Zosyn), 3. Adverse effects. Phlebitis at IV site; diarrhea,
ampicillin/sublactam (Unasyn), pseudomembranous colitis; hypersensitivity
ticarcillin/clavulanate (Timentin) reactions; fungal overgrowth; discomfort at IM
injection site; nephrotoxicity; hepatotoxicity;
bone marrow depression.
4. Nursing implications
Sample Questions a. Give IM injections deeply into large
muscle masses; rotate sites.
b. Assess for history of penicillin allergy as
138. Before giving a prescription for penicillin V there is a cross allergy between

2
potassium, the nurse reviews basic information cephalosporins and penicillin.

DRUGS AND NURSING IMPLICATIONS 87


53155_02_Ch02_p007-110.qxd 2/21/09 10:48 AM Page 88

c. Dose will be reduced with renal sodium (Rocephin), cefdinir (Omnicef),


impairment and decreased liver function. cefixime (Suprax), cefoperazone (Cefobid),
d. Increased risk of renal toxicity if given cefotaxime (Claforan), cefpodoxime (Vantin),
with other nephrotoxic drugs. ceftibuten (Cedax).
e. Monitor renal, liver function studies, and D. Prototype for fourth-generation cephalosporins:
I&O. cefepine (Maxipine).
f. Prolonged IV administration can cause 1. Action. See action for cefazolin sodium
thrombophlebitis. Assess and rotate IV (Ancef).
sites. 2. Use. Urinary tract infections caused by E. coli
g. Probenecid therapy will increase blood or Klebsiella; skin infections caused by S.
levels of cephalosporin. aureus; pneumonia caused by S. pneumoniae,
5. Discharge teaching Pseudomonas aeruginosa or Enterobacter.
a. Finish full course of therapy even if you 3. Adverse effects. See adverse effects for
feel well. cefazolin sodium (Ancef).
b. Promptly report diarrhea, rash, hives, 4. Nursing implications. Have Vitamin K
difficulty breathing, unusual bleeding. available if hypoprothrombinemia develops.
c. Report signs of superinfection. See Nursing Implications and Discharge
6. Related drugs. Cephalexin (Keflex), Teaching for cefazolin sodium (Ancef).
cephalothin sodium (Keflin), cephapirin 5. Related drug. Cefditoren (Spectracef).
sodium (Cefadyl), cepharadine (Velosef).
B. Prototype for second-generation cephalosporins:
cefoxitin sodium (Mefoxin)
1. Action. See action for cefazolin sodium Sample Questions
(Ancef).
2. Use. Infections caused by gram-negative and
gram-positive bacteria; septicemia; pelvic, 140. What action do penicillins and cephalosporins
skin, and soft-tissue infections; prophylaxis in have on bacteria?
abdominal or pelvic surgery; gonorrhea. 1. Inhibiting bacterial cell wall synthesis.
3. Adverse effects. See adverse effects for 2. Preventing bacterial protein synthesis.
cefazolin sodium (Ancef).
3. Increasing permeability of bacterial
4. Nursing implications. Lidocaine used as
membranes.
diluent for IM injection and helps reduce pain
of IM injection. See Nursing Implications and 4. Inhibiting metabolic processes in the
Discharge Teaching for cefazolin sodium bacterial cell.
(Ancef).
5. Related drugs. Cefaclor (Ceclor), cefamandole 141. Which medication will increase the chance of
naftate (Mandol), cefuroxime sodium (Ceftin), developing nephrotoxicity if the client is taking
cefmetazole (Zefazone), cefonicid (Monocid), cefazolin sodium also?
cefotetan (Cefotan), cefprozil (Cefzil), 1. An antacid.
loracarbef (Lorabid). 2. Vitamin D.
C. Prototype for third-generation cephalosporins:
3. Dimenhydrinate (Dramamine).
cefotaxime (Claforan)
1. Action. See action for cefazolin sodium 4. An aminoglycoside.
(Ancef).
142. Which statement by the client indicates a need
2. Use. Serious infections caused by gram-
negative and gram-positive bacteria; for more teaching about cephalosporins by the
meningitis, especially in neonates; nurse?
uncomplicated gonorrhea. 1. I will take every bit of this medication even
3. Adverse effects. See adverse effects for if I feel better.
cefazolin sodium (Ancef). 2. I will tell the doctor if I have diarrhea, a
4. Nursing implications rash, or any difficulty breathing.
a. Do not mix with aminoglycoside solutions. 3. I will continue to test my urine for glucose
Give these drugs separately.
with Clinitest tablets.
b. Protect IV solutions from light. See
Nursing Implications and Discharge 4. I will tell the dentist that Im taking a
Teaching for cefazolin sodium (Ancef). cephalosporin when I go for my
5. Related drugs. Ceftazidime (Fortaz), appointment.
ceftizoxime sodium (Cefizox), ceftriaxone

2 88 NCLEX-RN Review
53155_02_Ch02_p007-110.qxd 2/21/09 10:48 AM Page 89

143. Which lab result is indicative of an adverse stearate, azithromycin (Zithromax),


effect of cephalosporins? clarithromycin (Biaxin), dirithromycin
1. Increased potassium. (Dynabac).
2. Decreased AST (SGOT) and ALT (SGPT).
3. Elevated hemoglobin.
4. Increased BUN and creatinine. Sample Questions

144. The client is allergic to penicillin. What


Answers and Rationales explanation will the nurse tell the client that he
can be given erythromycin?
1. Erythromycin is more easily absorbed from
140. 1. Penicillins and cephalosporins have the same the GI tract than penicillin.
action on bacteria: inhibition of bacterial cell
2. Erythromycin has a spectrum of activity that
wall synthesis.
is similar to penicillin.
141. 4. There is an increased risk of renal toxicity 3. Erythromycin has fewer adverse effects than
when cephalosporins are given with other penicillin.
nephrotoxic drugs such as diuretics and 4. Erythromycin is not as toxic to the body as
aminoglycosides. penicillin.

142. 3. A diabetic client who is taking a 145. Which statement by the client indicates a need
cephalosporin drug will get a false-positive for more teaching about erythromycin?
glucose reaction if Clinitest tablets are used in 1. If I notice any change in my hearing I will
urine testing. The client needs to use Clinistix or call the doctor.
Tes Tape for urine testing. 2. I will take the erythromycin with orange
143. 4. Cephalosporins can cause renal toxicity for juice.
which an elevated BUN and creatinine would be 3. I wont take the erythromycin with meals.
indicative. 4. I wont crush the erythromycin tablets, Ill
swallow them whole.

MACROLIDES
A. Prototype: erythromycin base (E-Mycin) Answers and Rationales
1. Action. Inhibits protein synthesis in bacterial
cell. Bacteriostatic. Has broad spectrum of
activity. 144. 2. Erythromycin and penicillin have a similar
2. Use. Persons allergic to penicillin; spectrum of activity; so individuals who are
Legionnaires disease; mycoplasma allergic to penicillin can take erythromycin.
pneumonia; intestinal dysenteric amebiasis;
acne; staphylococcal and streptococcal 145. 2. Acidity decreases the activity of
infections. erythromycin; it should not be taken with acids
3. Adverse effects. Gastrointestinal irritation, such as fruit juices.
reversible hearing loss, hepatitis, allergic
reactions, superinfections.
4. Nursing implications
a. Do not crush enteric-coated tablet. TETRACYCLINES
b. Take on empty stomach with a full glass of
A. Prototype: tetracycline hydrochloride
water.
(Achromycin V)
c. Do not give with acids.
1. Action. Broad-spectrum drug with
d. Monitor liver function tests.
bacteriostatic action and, at higher doses,
e. GI symptoms are dose related.
bactericidal action. Inhibits bacterial wall
f. Give IM deeply into a large muscle mass.
synthesis. Reduces free fatty acids from
g. IV must be diluted sufficiently and
triglycerides, thus reducing acne lesions.
administered slowly to avoid venous
2. Use. Chlamydia, mycoplasma, rickettsia, acne
irritation and thrombophlebitis.
vulgaris, gonorrhea, spirochetes.
B. Related drugs. Erythromycin estolate (Ilosone),

2
erythromycin gluceptate (Ilotycin), erythromycin

DRUGS AND NURSING IMPLICATIONS 89


53155_02_Ch02_p007-110.qxd 2/21/09 10:48 AM Page 90

3. Adverse effects. Headache, dizziness;


neutropenia; nausea, vomiting, diarrhea, Answers and Rationales
colitis, abdominal cramping; hepatotoxicity;
photosensitivity, hypersensitivity;
superinfections; chelating to teeth and 146. 4. Tetracycline is best absorbed on an empty
new bone. stomach. Taking tetracycline with food or milk
4. Nursing implications impairs absorption.
a. Avoid use during pregnancy, in nursing
women, and in children under age 8 as drug 147. 4. Diarrhea should be reported to the physician,
binds to calcium in teeth and new bone who can rule out diarrhea as a symptom of
growth, which results in tooth discoloration superinfection or an adverse effect.
of permanent teeth and retarded bone growth.
b. Give deep IM.
c. Monitor CBC and signs of liver toxicity. CHLORAMPHENICOL
5. Discharge teaching
a. Avoid use with calcium supplements, A. Prototype: chloramphenicol (Chloromycetin)
antacids, iron, or dairy products as these 1. Action. A synthetic broad-spectrum agent.
may reduce tetracycline absorption. Primarily bacteriostatic but is bactericidal in
b. Avoid the sun while taking drug and for a higher doses. Inhibits protein synthesis.
few days after therapy is terminated. 2. Use. Haemophilus influenzae meningitis,
c. Use meticulous hygiene to reduce rickettsia, salmonella typhi, mycoplasma,
superinfections. bacteroides, typhoid fever. Note:
d. Complete prescribed course. chloramphenicol (Chloromycetin) used only in
B. Related drugs severe infections when other antibiotics cannot
1. Doxycycline (Vibramycin): can be be used due to its severe adverse effect of
administered with food. Safe to use in clients aplastic anemia.
with renal impairment. 3. Adverse effects. Aplastic anemia; neurotoxicity;
2. Minocycline (Minocin): can be taken with gray-baby syndrome (seen in premature infants,
food. Dizziness and fatigue may occur. newborns, and children less than 2 years old.
3. Demeclocycline (Declomycin): administer on Abdominal distention, vomiting, pallor,
an empty stomach. Foods high in calcium and irregular respirations, circulatory collapse, and
iron interfere with absorption. death can occur due to the infants immature
4. Oxytetracycline (Terramycin): administer on liver function); hypersensitivity; nausea,
an empty stomach. Food disrupts extent and vomiting, enterocolitis; superinfections; bitter
rate of absorption. taste especially after IV injection.
4. Nursing implications
a. Obtain and monitor baseline CBC,
platelets, and serum iron.
Sample Questions b. Monitor children less than 2 years old for
gray-baby syndrome.
c. Do not give by IM injection.
146. Which of the following should the nurse include 5. Discharge teaching
in teaching about taking tetracyclines? a. Inform physician immediately of fever,
1. Take tetracycline HCl with milk. fatigue, sore throat, or bruising.
2. Take tetracycline with food. b. Take drug on an empty stomach unless GI upset.
3. Encourage sitting in sun to enhance c. Notify physician and discontinue drug if
tetracyclines effects. symptoms of hypersensitivity occur.
4. Take tetracycline before meals.

147. Which of the following adverse effects should Sample Questions


the client report to the physician while taking
tetracycline?
1. Constipation. 148. Which of the following changes in an infant should
2. Hypertension. the nurse immediately report to the physician?
3. Tachycardia. 1. Vomiting.
4. Diarrhea. 2. Constipation.
3. Flushing.

2
4. Dry skin.

90 NCLEX-RN Review
53155_02_Ch02_p007-110.qxd 2/21/09 10:48 AM Page 91

contraindicated in clients allergic to


Answers and Rationales salicylates.
2. Sulfamethorazole (Gantanol) can be given in
combination with trimethoprin (Proloprim) as
148. 1. Abdominal distention and vomiting are early Septra or Bactrim. Used in treating urinary
symptoms of gray-baby syndrome and should be tract infections, bronchitis, and pneumocystis
reported immediately as this syndrome is life- pneumonia.
threatening.

Sample Questions
SULFONAMIDES
A. Prototype: sulfisoxazole (Gantrisin) 149. Eight days after taking sulfisoxazole, a child
1. Action. Prevents conversion of para- develops a fever. When the childs mother calls
aminobenzoic acid (PABA) to folic acid, the pediatricians office about this, what
which is required for bacterial growth. instructions will the nurse provide?
Effects are usually bacteriostatic but can be
1. Reduce the dose of the sulfisoxazole.
bactericidal in high urinary
concentrations. 2. Have the child rest in bed.
2. Use. Urinary tract infections, otitis media, 3. Call the pediatrician if more symptoms
nocardiosis (occurs in the lungs and spreads develop.
to skin, brain, and other areas), systemic 4. Stop the sulfisoxazole and bring the child in
infections, vaginitis, superficial eye to see the pediatrician today.
infections.
3. Adverse effects. Hypersensitivity; Stevens-
Johnson syndrome (acute onset of fever, bullae
on skin and ulcers on mucous membranes of Answers and Rationales
lips, eyes, mouth, nasal passages, and
genitalia. Pneumonia, joint pain, and
prostration are also seen); fever 710 days after 149. 4. A fever 710 days after starting sulfisoxazole is
starting therapy may indicate sensitization or an adverse effect of the drug that could indicate a
hemolytic anemia; renal dysfunction; sensitization to the drug or hemolytic anemia.
hematologic reaction; GI reaction; The drug should be stopped and the client
photosensitivity. should see the physician as soon as possible.
4. Nursing implications
a. Give oral form on empty stomach with full
glass of water. URINARY ANTI-INFECTIVES
b. Observe skin for presence of rash, ulcers.
c. Monitor temperature. A. The medications are dependent on the desired
d. Monitor I&O; force fluids; check urine goal, whether the client has an acute, recurrent or
pH; cautious use in clients with renal chronic UTI.
dysfunction; monitor renal function 1. Urinary anti-infectives are structurally
tests. different so there will be no prototype drug
e. Monitor CBC. 2. See Table 2-23.
5. Discharge teaching
a. Avoid direct sunlight.
b. Complete full course of treatment.
c. Diabetics who take oral hypoglycemic Sample Questions
agents need to be aware of increased
chance of hypoglycemic reactions with use
of sulfonamides. 150. The client should be taught which of the
d. Oral contraceptives may be unreliable following points about nitrofurantoin?
while client is receiving sulfonamides. 1. Take it on an empty stomach.
Alternate method of contraception should
2. You may experience nausea and vomiting.
be used.
B. Related drugs 3. You can crush the tablet if its too hard to
1. Sulfasalazine (Azulfidine) used in treatment of swallow whole.
ulcerative colitis. Contains aspirin, so is 4. It doesnt interact with any other drugs.

DRUGS AND NURSING IMPLICATIONS

2 91
53155_02_Ch02_p007-110.qxd 2/21/09 10:48 AM Page 92

Table 2-23 Urinary Anti-Infectives

Drug Use Adverse Effects Nursing Implications


Methenamine Converted to formaldehyde Nausea, vomiting, diarrhea, Give with food or milk to prevent GI
(Hiprex; (which is bactericidal) in abdominal discomfort. upset.
Mandelamine) the presence of acidic Avoid foods, fluids, and medications
urine. that alkalinize urine.
Monitor I&O.
Increase fluid intake.
Monitor liver function tests.
Nalidixic Acid Bactericidal effect on gram- Headache, dizziness, vertigo, Minor CNS reactions are common
(Neg Gram) negative bacteria by visual disturbances, and should decrease in 48 hours.
preventing transmission of photosensitivity. Client should wear sunglasses if
genetic information. bothered by bright lights.
Client should avoid sun exposure,
wear sunscreen and appropriate
clothing.
Give with food to decrease GI upset.
Nitrofurantoin Interferes with carbohydrate Pulmonary hypersensitivity, Monitor pulmonary status.
(Macrodantin) metabolism of bacteria. Is nausea, vomiting, lower Give drug with milk or meals.
bacteriostatic in low extremity paresthesias. Monitor neurologic status.
concentrations and Avoid tooth staining by not crushing
bactericidal in high tablets, dilute suspension, and rinse
concentrations. mouth after taking drug.
Phenazopyridine An azo dye excreted in the Rash, headache, GI Give after meals to prevent GI upset.
(Pyridium) urine which provides a disturbances, reddish-orange Tell client that urine may turn
topical analgesic effect to discoloration of urine. reddish-orange and can stain fabrics.
the urinary tract. Stop drug if skin or sclera turn yellow
which is a sign of drug accumulation.
Cinoxacin Prevents protein synthesis Dizziness, headache, nausea, No breastfeeding while taking this
(Cinobac) and DNA replication in tinnitus. drug.
bacterial cell. Use carefully in clients with liver,
kidney, and CNS disorders.
Give at evenly spaced time periods
during each 24 hours of drug
administration.

2. Use. Staphylococcus infections,


Answers and Rationales pseudomembranous colitis, gram-positive
organisms. Penicillin-G and methicillin-
resistant bacteria.
150. 2. GI irritation is the most frequent adverse effect 3. Adverse effects. Ototoxicity, nephrotoxicity,
of nitrofurantoin (Macrodantin). hypersensitivity, thrombophlebitis, red-neck
syndrome (flushing and hypotension from
rapid IV infusion), superinfections.
VANCOMYCIN HYDROCHLORIDE 4. Nursing implications
a. Monitor I&O.
(VANCOCIN) b. Obtain and monitor renal and auditory
function tests.
A. Prototype: vancomycin hydrochloride (Vancocin) c. Administer IV slowly to prevent phlebitis,
1. Action. Interferes with cell membrane extravasation, red-neck syndrome.
synthesis and exhibits a bactericidal and
bacteriostatic effect.

2 92 NCLEX-RN Review
53155_02_Ch02_p007-110.qxd 2/21/09 10:48 AM Page 93

3. I will drink an 8-oz glass of water with this


Sample Questions drug.
4. I will take this drug with a cup of tea or
coffee, as caffeinated beverages make this
151. Which of the following is an important nursing drug work more quickly.
consideration when administering IV
vancomycin hydrochloride (Vancocin)?
1. Mix the dose in 50 mL of dextrose in water.
2. Infuse over 30 minutes.
Answers and Rationales
3. Infuse over 60 minutes.
4. Administer IV push. 152. 3. Taking an 8-oz glass of water will help to
prevent crystalluria.

Answers and Rationales ANTITUBERCULAR DRUGS


A. Prototype: isoniazid (INH)
151. 3. IV vancomycin hydrochloride (Vancocin) 1. Action. Bacteriostatic and in high
should be given over 1 hour to help prevent concentrations becomes bactericidal.
thrombophlebitis and red-neck syndrome. Mechanism of action not known but is
believed to interfere with lipid and nucleic
acid biosynthesis in tubercle bacilli that are
FLUOROQUINOLONES actively growing.
2. Use. Initial treatment of tuberculosis;
A. Prototype: ciprofloxacin (Cipro) prophylactic treatment of tuberculosis in high-
1. Action. Inhibits DNA-gyrase (an enzyme risk groups.
needed for replication of bacterial DNA). 3. Adverse effects. Peripheral neuritis; jaundice,
Bactericidal effect. elevation in liver function tests; nausea,
2. Use. Pseudomonas infections, gram-negative vomiting; blood dyscrasias.
urinary tract infections or gram-negative 4. Nursing implications
systemic infections. a. Assess neuromuscular function and give
3. Adverse effects. Nausea, vomiting, diarrhea, pyridoxine (vitamin B6) to treat and/or
flatulence; headache, tremors, confusion, prevent problems.
dizziness, insomnia; fever; rash; elevated BUN, b. Regularly scheduled baseline liver
AST (SGOT), ALT (SGPT), creatinine; function studies.
decreased WBC, hematocrit. c. Monitor for hepatic dysfunction.
4. Nursing implications d. Take drug on empty stomach in a single
a. Administer with a large glass of water to daily dose.
prevent crystalluria. e. Give drug with meals and divide daily
b. Do not give with antacids. dose into 3 equal parts if GI upset
c. Give 2 hours after meals. occurs.
B. Related drugs. Norfloxacin (Noroxin), gatifloxacin f. Assess for bruising, bleeding, fever, sore
(Tequin), levofloxacin (Levaquin), lomefloxacin throat.
(Maxaquin), moxifloxacin (Avelox), sparfloxacin g. Monitor CBC.
(Zagam), trovafloxacin (Trevan). 5. Discharge teaching
a. Tyramine-containing foods may cause
hypertensive crises, so should be
avoided.
Sample Questions b. Avoid histamine-containing foods as may
cause an exaggerated drug response.
c. Avoid use of alcohol.
152. Which statement by the client indicates that she d. May cause a feeling of euphoria. Plan rest
understands correct use of ciprofloxacin (Cipro)? periods and dont overdo.
1. I will take this drug with Maalox to prevent e. Drug therapy must not be interrupted and
GI upset. must be continued for prescribed time.
2. I will stop taking this drug when I no longer B. Related drugs. See Table 2-24.
have pain upon urination.

DRUGS AND NURSING IMPLICATIONS

2 93
53155_02_Ch02_p007-110.qxd 2/21/09 10:48 AM Page 94

Table 2-24 Drugs Commonly Used to Treat Tuberculosis

Drug and Dosage Adverse Effects Nursing Implications


Isoniazid (INH) Peripheral neuritis, a numbness and tingling Tell client to report signs of neuritis and
10 to 20 mg/kg (up to in hands and feet. Vitamin B6 (pyridoxine) hepatitis (anorexia, nausea, vomiting, jaundice,
300 mg) PO or IM daily, may be given to prevent or treat this malaise, or dark urine).
or 15 mg/kg PO or IM condition. Isoniazid may interfere with phenytoin
twice a week Hepatitis, with the risk increasing with age. (Dilantin) metabolism, requiring a lower dose of
Liver enzymes may be routinely monitored in the TB medication; should be taken on an
elderly or symptomatic clients. empty stomach, and the client should not drink
Hyperexcitability may occur with single alcohol while on therapy.
300 mg dose.
Ethambutol (Myambutol) Optic neuritis, a loss of red-green color Tell client to notify physician if vision blurs or if
15 to 25 mg/kg PO daily, discrimination, and decreased visual acuity unable to see red or green.
or 50 mg/kg PO twice can occur with dosages of 25 mg/kg. Use the drug with caution if a visual exam
weekly Reversible, if medication discontinued. Skin cannot be done and in clients with renal
rash. impairment.
Rifampin Body fluids (urine, tears, saliva, etc.) may turn Tell clients to expect orange-tinged body fluids.
(Rifadin, Rimactane) orange. Tell clients to report anorexia, nausea, vomiting,
10 to 20 mg/kg (up to Hepatitis jaundice, malaise, or dark urine.
600 mg) PO daily, or Flu-like syndrome Use the drug with caution in cases of liver
600 mg PO twice a week Purpura (rare) disease.
Rifampin affects the actions of other drugs,
including anticoagulants, oral hypoglycemics,
corticosteroids, oral contraceptives, and
methadone.
Streptomycin Damage to cranial nerve VIII (vestibulocochlear). Baseline renal and audiology studies may be
15 to 20 mg/kg Damage to the vestibular portion causes obtained before therapy begins.
(up to 1 g) IM daily or dizziness, vertigo, tinnitus, and roaring in Tell client to report any ringing, roaring, or
25 to 30 m/kg IM ears. fullness in ears.
twice a week Auditory damage causes loss of hearing at Help coordinate outpatient arrangements for
high frequency ranges. IM injections, if necessary.
Renal toxicity.
Pyrazinamide Excess uric acid levels, which can cause gout Baseline uric acid and liver enzyme levels may
20 to 40 mg/kg or hepatitis. be obtained; monitor uric acid and liver
(up to 2 g) PO daily enzymes.
Instruct patient to report any signs of gout
(painful swelling in joints, chills, fever) and
hepatitis (anorexia, nausea, vomiting, jaundice,
malaise, and dark urine).
Use with caution in clients who have liver
disease, gout, or renal impairment.

Other drugs used: capreomycin (Capastat), kanamycin (Kantrex), ethionamide (Trecator-SC), para-amino-salicylic acid, and cycloserine (Seromycin).

4. Rifampin is taken in the usual adult dose of


Sample Questions 60 mg daily.

153. The nurse needs to tell the client which fact


about rifampin?
1. Rifampin increases the effectiveness of oral
Answers and Rationales
contraceptives.
2. Rifampin may cause soft-contact lenses to be 153. 2. Rifampin (Rimactane) discolors body
permanently discolored. secretions such as sweat, urine, feces, and tears a

2
3. Rifampin will increase the activity of red-orange color.
coumarin-type oral anticoagulants.

94 NCLEX-RN Review
53155_02_Ch02_p007-110.qxd 2/21/09 10:48 AM Page 95

Antiviral Agents

ACYCLOVIR (ZOVIRAX) 3. Rimantadine (Flumadine). Used in treatment


and prevention of influenza A.
A. Prototype: acyclovir (Zovirax) 4. Nevirapine (Viramune). Used with other antiviral
1. Action. Inhibits viral DNA replication. Does agents to treat HIV in children and adults.
not cure herpes infections but decreases the 5. Indinavir (Crixivan). Used to treat adults with
severity and duration of herpes. HIV.
2. Use. Herpes simplex virus 1 and 2, initial 6. Valacyclovir (Valtrex).
treatment of genital herpes infection.
3. Adverse effects. Nausea, vomiting, diarrhea;
rash; headache, vertigo; crystalluria; phlebitis
at injection site; transient burning with topical
Sample Questions
use.
4. Nursing implications
a. Do not give IV bolus. Give IV over 1 hour 154. Which statement concerning the use of acyclovir
to prevent crystalluria and phlebitis. is correct?
b. Clients should drink plenty of fluids. 1. Sexual relations can be resumed while using
5. Discharge teaching topical acyclovir.
a. Use topical preparation sparingly, and use 2. Acyclovir can be used for repeated genital
rubber gloves when applying. herpes infections.
b. Avoid sexual contact while lesions are
3. Acyclovir should be given IV over 1 hour.
visible.
c. Drug does not cure herpes nor prevent 4. Acyclovir prevents the recurrence of genital
transmission to others. herpes.
B. Related drugs
1. Ribavirin (Virazole): Given by inhalation to
treat respiratory syncytial virus (RSV) in
hospitalized infants and small children. Answers and Rationales
Inhalation of aerosol can be teratogenic.
2. Zidovudine (AZT, Retrovir): Developed to
154. 3. IV acyclovir (Zovirax) should be given over
control AIDS or ARC (AIDS-related complex)
1 hour to prevent phlebitis and crystalluria.
with Pneumocystis carinii. Causes
leukocytopenia; monitor blood work.

Antifungal Agents

ANTIFUNGALS azotemia; thrombophlebitis; hypotension,


tachycardia, or cardiovascular collapse with
A. Prototype: amphotericin B (Fungizone) rapid infusion; blood dyscrasia;
1. Action. Fungicidal or fungistatic. Alters the hypersensitivity.
fungal cell membrane permeability by binding 4. Nursing implications
to sterols. Clients must be monitored due to a. Monitor CBC, BUN, creatinine,
many toxic effects. electrolytes.
2. Use. Candida infections, histoplasmosis, b. Administer analgesics, antihistamines,
coccidiomycoses, blastomycosis, prior to infusion to minimize febrile
cryptococcoses. reactions.
3. Adverse effects. Febrile reactions, nausea, c. Infuse drug slowly.
vomiting; nephrotoxicity, hypokalemia, d. Monitor VS frequently.

DRUGS AND NURSING IMPLICATIONS

2 95
53155_02_Ch02_p007-110.qxd 2/21/09 10:48 AM Page 96

e. Monitor I&O.
f. Administer potassium supplements. Sample Questions
g. Do not mix with other drugs.
B. Related drugs
1. Nystatin (Mycostatin): Used to treat candida 155. Which electrolyte imbalance should the nurse
infections. monitor for while a client is on amphotericin B
2. Griseofulvin (Grisactin): Used to treat ringworm therapy?
infections. Adverse effects: headaches, blood 1. Hyponatremia.
dyscrasias, GI upset, rash from sunlight. Give
on full stomach. Clients allergic to penicillin 2. Hypokalemia.
should use this drug with caution. 3. Hyperkalemia.
3. Fluconazole (Diflucan): Used to treat candida 4. Hypercalcemia.
infections and cryptococcal meningitis.
4. Ketoconazole (Nizoral): Used to treat systemic
fungal infections.
5. Terbinafine (Lamisil): Used to treat Answers and Rationales
onychomycosis.

155. 2. Hypokalemia can occur due to nephrotoxocity


of this drug.

Anthelmintic Agents

the gastrointestinal tract. Approximately 98% of


ANTHELMINTICS the drug remains effective as it passes through the
GI tract and is excreted in the feces.
A. The anthelmintics are a group of drugs that affect B. See Table 2-25.
various systems within the worms, causing them
to die. Most of these drugs are poorly absorbed in

Table 2-25 Anthelmintics

Drug Action Use Adverse Effects Nursing Implications


Pyrantel Paralyzes intestinal Roundworm, pinworm, Nausea, vomiting, anorexia, Give with milk or fruit juice.
(Antiminth) tract of worm. hookworm abdominal cramps, Entire dose must be taken at
diarrhea. once.
Offer frequent, small meals.
Mebendazole Inhibits glucose Pinworm, roundworm, Abdominal cramping, Can be taken with or without
(Vermox) and other threadworm, occasional fever. food.
nutrient uptake hookworm Tablet can be chewed or
of helminth. crushed.
Examine stool for presence
of worms.
Thiabendazole Interferes with Threadworm, pinworm Dizziness, drowsiness, Give with food.
(Mintezol) parasitic headache, anorexia, Chew tablets before
metabolism. nausea, malodor of urine. swallowing.
Avoid activities such as
driving and working with
machinery.
Praziquantel Enhances perme- Shistosomes and Headache, dizziness, Give with food or liquids.
(Biltricide) ability of the cell flukes abdominal pain, Do not chew.
membranes of increased liver enzymes. Do not breastfeed.
the parasite to
calcium.

2 96 NCLEX-RN Review
53155_02_Ch02_p007-110.qxd 2/21/09 10:48 AM Page 97

Sample Questions Answers and Rationales

156. How is mebendazole (Vermox) quite effective 156. 2. Pinworms infect the intestines. Mebendazole
against pinworms? (Vermox) is poorly absorbed in the GI tract and
1. It is active in the stomach where the therefore is quite effective against these
pinworm eggs hatch. helminths.
2. It is poorly absorbed in the GI tract and kills
the pinworms that infect the intestines.
3. Its systemic activity kills pinworms all over
the body.
4. It stimulates peristalsis causing the pinworms
to be expelled before they reproduce.

Antineoplastic Agents

ANTINEOPLASTIC AGENTS 3. Frequent rest periods.


4. Expect alopecia and purchase scarves or wigs.
A. General considerations 5. Report fever, use good hand-washing
1. Combination of antineoplastic agents usually technique, avoid individuals with upper
used to destroy cancer cells. respiratory infections.
2. Clients must be closely monitored due to 6. Use soft toothbrush and baking soda rinse to
many toxic adverse effects. minimize stomatitis.
3. Agents destroy cancer cells and may also kill 7. Use progressive relaxation exercises or guided
normal cells. imagery to help cope with nausea.
B. General adverse effects. Nausea, vomiting,
anorexia; diarrhea and constipation; stomatitis;
alopecia; bone marrow depression (leukopenia, ALKYLATING AGENTS
anemia, and thrombocytopenia); hepatic toxicity;
hyperuricemia; fatigue. A. Prototype: cyclophosphamide (Cytoxan)
C. Nursing implications 1. Action. Produces cytoxic effects by damaging
1. Handle antineoplastic agents carefully; DNA and interfering with cell replication.
mutagenic and possibly carcinogenic. Most effective against rapidly dividing cells.
2. Nurses should wear gloves, long-sleeved cover 2. Use. Leukemias; multiple myeloma;
gown, protective goggles, and mask as neuroblastoma; ovarian, breast, lung cancers;
appropriate. Hodgkins disease; Ewings sarcoma.
3. Monitor IV site closely to assess for 3. Adverse effects. Gonadal suppression,
extravasation and stop IV if it occurs. hemorrhagic and nonhemorrhagic cystitis.
4. Treat used equipment as hazardous waste. 4. Nursing implications
5. Administer antiemetic if ordered prior to a. Force fluids.
chemotherapy and up to 48 hours afterwards. b. Assess for signs and symptoms of
6. Monitor CBC. unexplained bleeding.
7. Monitor I&O. c. Assess leukocyte count frequently.
8. Monitor liver and renal function studies. d. Monitor CBC, uric acid, electrolytes,
9. Inspect oral cavity daily. thrombocytes, and hepatic and renal
D. Discharge teaching function at least twice a week.
1. Eat frequent, small portions of high-calorie, e. Instruct client to report hematuria or
high-protein, bland, low-residue foods. dysuria immediately.
2. Avoid highly seasoned foods, drink clear B. Related drugs. See Table 2-26.
liquids if nauseated.

DRUGS AND NURSING IMPLICATIONS

2 97
53155_02_Ch02_p007-110.qxd 2/21/09 10:48 AM Page 98

Table 2-26 Alkylating Agents

Drug Use Comments


Cisplatin (Platinol) Lymphoma; myeloma; melanoma; Causes nephrotoxicity and ototoxicity, ensure
osteosarcoma; cervical, ovarian, testicular, adequate hydration and give diuretics prior to
lung, esophageal, prostatic cancers. therapy.
Have client void every hour or insert Foley catheter
before initiating treatment.
Assess for hearing deficits.
Busulfan (Myleran) Polycythermia vera, chronic myelogenous Discontinue drug when white blood cells (WBC)
leukemia. reach 15,000 mm3.
Monitor CBC as this drug can cause severe bone
marrow depression.
Mechlorethamine HCl Hodgkins disease, non-Hodgkins Assess for edema, ascites, weight gain.
(Mustargen) lymphomas; lung cancer. Assess for signs and symptoms of dehydration.
Wear gloves if applying solid preparation.
Thiotepa Bladder, breast, and ovarian cancers. Decreased dose for renal or hepatic impairment
and bone marrow depression.
Only given parenterally.
Chlorambucil Breast and ovarian cancer; non-Hodgkins Assess CBC, WBC, and serum uric acid levels
(Leukeran) lymphomas; chronic lymphocytic routinely.
leukemia. Avoid IM injections when platelet count is low.
Urge client to drink 1012 glasses of fluid per day.
Provide urine alkalinization if uric acid levels are
increased.

Drinking plenty of water and fluids reduces the


Sample Questions risk of developing both types of cystitis.

158. 1. Metaclopramide (Reglan) is an antiemetic


157. Which of the following should be included in used to treat the nausea and vomiting that are
client teaching about cyclophosphamide adverse effects seen with cyclophosphamide
(Cytoxan) therapy? (Cytoxan) therapy.
1. Omit dose if anorexic or nauseated.
2. Drink plenty of water. ANTIMETABOLITES
3. Take cyclophosphamide with meals.
4. Take cyclophosphamide before going to bed. A. Prototype: methotrexate with leucovorin rescue
1. Action. Leucovorin calcium is a folic acid
158. Which of the following medications might the analog that interferes with mitotic process by
physician order along with cyclophosphamide? blocking folinic acid.
1. Metoclopramide (Reglan). 2. Use. Acute lymphoblastic leukemia; cancer of
breast, lung, testes, ovary, head, and neck;
2. Diphenhydramine HCl (Benadryl).
choriocarcinoma.
3. Minoxidil (Loniten). 3. Adverse effects. See General Considerations.
4. Dexamethasone (Decadron). Intrathecal use may cause fever, headache, and
vomiting.
4. Nursing implications. Leucovorin calcium is
frequently given to prevent toxicity when high
Answers and Rationales doses of methotrexate are given.
5. Discharge teaching. See General
Considerations. Instruct client to avoid
157. 2. Hemorrhagic and nonhemorrhagic cystitis are self-medication with OTC vitamins (folic acid
related to cyclophosphamide (Cytoxan) therapy. and derivatives may alter drug response).

2 98 NCLEX-RN Review
53155_02_Ch02_p007-110.qxd 2/21/09 10:48 AM Page 99

B. Related drugs. 48 hours if refrigerated; protect from


1. 5-Fluorouracil (5-FU) sunlight; do not infuse in less than
2. Mercaptopurine (Purinethol) 5 minutes; red streaking over vein and
3. Cytarabine (Cytosar-U) facial flushing are signs of too-rapid
4. Floxuridine (FUDR) administration.
5. Fludarabine (Fludara) c. Do not mix with other drugs.
d. Monitor IV site; for local extravasation
pour normal saline on area, apply a cold
compress; infiltration with corticosteroid
Sample Questions may be ordered.
e. Monitor CBC, serum uric acid levels,
cardiac output (listen for S3), weight.
159. Which of the following drugs is given to prevent f. Frequent mouth care.
methotrexate toxicity? g. Client needs sufficient fluids to prevent
1. Trimethobenzamide HCl (Tigan) hyperuricemia.
2. Prednisone h. Assist client with information on wigs and
head coverings before hair loss starts.
3. Allopurinol (Zyloprim)
i. Offer support to client to deal with drug
4. Leucovorin calcium therapy and diagnosis.
j. Wear gloves to prepare this drug. Wash
160. Which of the following agents should be avoided skin with soap and water if powder or
by clients on methotrexate therapy? solution contacts skin.
1. Folic acid k. Urine is red colored for 12 days after
2. Vitamin C administration. Clears within 48 hours.
3. Iron B. Related drugs. See Table 2-27.
4. Vitamin D

Sample Questions
Answers and Rationales
161. While the client is receiving intravenous
159. 4. Leucovorin calcium is a reduced form of folic doxorubicin the nurse should monitor which of
acid that takes up binding sites to prevent the following?
methotrexate toxicity. 1. Chest X-rays.
160. 1. Folic acid can alter methotrexate response. 2. Sodium levels.
3. Liver function studies.
4. Electrocardiograms.
ANTIBIOTIC ANTINEOPLASTIC
162. What complaints would alert the nurse to stop
AGENTS the infusion of doxorubicin?
A. Prototype: doxorubicin HCl (Adriamycin) 1. Headache and dizziness.
1. Action. Attaches to DNA and prevents DNA 2. Burning and pain at the infusion site.
synthesis in vulnerable cells. 3. Upset stomach and heartburn.
2. Use. Cancer of thyroid, lung, bladder, breast, 4. Light-headedness and confusion.
and ovary; acute leukemia; sarcoma; Ewings
sarcoma; neuroblastoma; lymphomas. 163. Which nursing action is NOT appropriate for the
3. Adverse effects. Nausea, vomiting, stomatitis; client receiving doxorubicin?
ECG changes; agranulocytosis, leukopenia,
1. Provide frequent mouth care for the client.
thrombocytopenia; hyperpigmentation of skin
and nails; alopecia. 2. Tell the client his urine will be red-tinged for
4. Nursing implications the first couple of days after administration of
a. Do not give SC or IMlocal reaction and the drug.
skin necrosis can occur. 3. Put the client on fluid restriction.
b. IV use: reconstitute with normal saline or 4. Explain to the client he will lose his hair.
sterile water; reconstituted solution stable
for 24 hours at room temperature or

DRUGS AND NURSING IMPLICATIONS

2 99
53155_02_Ch02_p007-110.qxd 2/21/09 10:48 AM Page 100

Table 2-27 Antibiotic Antineoplastic Agents

Drug Action Uses Comments


Bleomycin Sulfate Prevents DNA, RNA, and Lymphomas, melanoma, Pulmonary side effects of dyspnea,
(Blenoxane) protein synthesis in cells cancers of head, neck, fever, rales, cough.
Cell cycle specific in G2 and esophagus, lung, skin, penis, Febrile reaction usually occurs on
M phases. testes, vulva, cervix, anus. first day of therapy.
Monitor respiratory status.
Dactinomycin Prevents synthesis of Testicular cancer, melanoma, Monitor IV site carefully.
(Actinomycin D) messenger RNA; cell cycle choriocarinoma, Wilms Do not expose drug solution to
nonspecific. tumor, neuroblastoma, direct sunlight.
retinoblastoma, Ewings
sarcoma, Kaposis sarcoma.
Daunorubicin Inhibits DNA synthesis; cell Acute myelocytic and ECG changes can occur.
(Daunomycin) cycle specific in S-phase lymphocytic leukemia. Urine turns red on day of
of cell cycle. administration.
Monitor IV site carefully
Never give IM or SC.
Do not mix with heparin.
Mitomycin Prevents DNA and protein Cancer of gastrointestinal tract, See Prototype Drug
(Mutamycin) synthesis in cells; cell cycle breast, lung, bladder, cervix.
nonspecific
Plicamycin Prevents DNA synthesis; Testicular cancer, Side effect of bleeding.
(Mithramycin) decreases serum calcium hypercalcemia. Monitor blood coagulation studies.
by unknown action; Monitor client for signs of
blocks action of hypocalcemia.
parathyroid hormone.

2. Believed that hormonal agents hinder use of


Answers and Rationales steroids necessary for cell growth.
3. Hormonal therapy keeps cancer cells in resting
phase, thus decreasing growth of tumor.
161. 4. Doxorubicin (Adriamycin) can cause ECG 4. No direct cytotoxic effect of hormonal agents
changes; the nurse should monitor so they are unable to cure cancer.
electrocardiograms while the client is receiving B. Estrogens (female hormones). See Table 2-28.
this drug. C. Androgens (male hormones) are also used as
replacement therapy for growth and development of
162. 2. Intravenous infusion of doxorubicin male sex organs and secondary sex characteristics
(Adriamycin) should be stopped if the client in androgen-deficient males. See Table 2-29.
complains of burning and pain at the infusion D. Antihormonal agents
site. These signs are indicative of extravasation, 1. Antiestrogen: Tamoxifen (Nolvadex)
which could lead to skin necrosis. a. Use. Advanced breast cancer in pre- and
postmenopausal women.
163. 3. Fluids should not be restricted for a client b. Adverse effects. Most common are similar
receiving doxorubicin; this drug can cause to signs of menopause (hot flashes and
hyperuricemia if fluid intake is not sufficient. flushing); nausea, vomiting; temporary
bone and tumor pain; temporary drop in
WBC count.
ANTINEOPLASTICS AFFECTING c. Nursing implications. Monitor WBC count;
tell premenopausal women to use
HORMONAL BALANCE contraception as short-term therapy causes
ovulation.
A. Mechanism of action of hormonal agents.
2. Antiadrenal: aminoglutethamide (Cytadren)
1. Exact mechanism is not completely
a. Use. Adrenal and metastatic breast

2
understood.
cancer.

100 NCLEX-RN Review


53155_02_Ch02_p007-110.qxd 2/21/09 10:48 AM Page 101

Table 2-28 Estrogens

Drug Use Adverse Effects Nursing Implications


Diethylstilbestrol (DES) Breast and prostate cancer. Headache; vertigo; insomnia; Monitor calcium levels.
nausea; weight changes; Monitor males for signs of
phlebitis; edema; uterine feminization.
bleeding; feminization in Monitor salt intake and keep it
males; changes in calcium reduced.
and folic acid metabolism Weigh client daily.
Ethinyl Estradiol (Estinyl) Breast and prostate cancer. See Diethylstilbestrol (DES) See Diethylstilbestrol (DES)

Table 2-29 Androgens

Drug Use Adverse Effects Nursing Implications


Fluoxymesterone Breast and renal cancer Nausea; vomiting; weight gain; Monitor client for
(Halotestin) edema and fluid retention; masculinizing effects.
vaginal dryness and itching; Monitor weight.
acne; hypercalcemia if bone Low salt intake.
cancer present; masculinization Restrict fluids if necessary.
of females Monitor blood pressure.
Testosterone Cypionate Breast cancer See Fluoxymesterone IM must be given deeply into
(Depo-Testosterone) (Halotestin) gluteal muscle.
See Fluoxymesterone (Halotestin)

b. Adverse effects. Drowsiness; anorexia, 3. Serum potassium.


nausea; vomiting, severe pancytopenia; 4. Serum calcium.
rash; and adrenal insufficiency.
c. Nursing implications. Possible 165. Which statement about diethylstilbestrol made
replacement therapy with hydrocortisone to a male client by the nurse is incorrect?
and mineralocorticoids; monitor blood 1. You may develop signs of increased
pressure, thyroid studies, and CBC; tell masculinity while taking diethylstilbestrol.
client that drug may cause drowsiness and
orthostatic hypotension. 2. You need to decrease your salt intake while
3. Gonadotropin releasing hormone: leuprolide taking diethylstilbestrol.
(Lupron) 3. You will need to weigh yourself each day
a. Use. Prostate cancer. while taking diethylstilbestrol.
b. Adverse effects. Hot flashes, transient bone 4. You can experience vascular problems while
pain, rash, alopecia, cardiac arrhythmias, taking diethylstilbestrol.
breathing difficulty, and hematuria.
c. Nursing implications. Monitor and rotate
injection sites; only use syringes
provided with drug; provide comfort
measures. Answers and Rationales

164. 4. Diethylstilbestrol (DES) can cause hypercalcemia;


Sample Questions serum calcium levels need to be monitored.

165. 1. Diethylstilbestrol (DES) can cause signs of


164. Which diagnostic test should the nurse monitor feminization in males, not increased masculinity
when the client is receiving diethylstilbestrol? in males.
1. Arterial blood gases.

2
2. Liver enzymes.

DRUGS AND NURSING IMPLICATIONS 101


53155_02_Ch02_p007-110.qxd 2/21/09 10:48 AM Page 102

Table 2-30 Mitotic Inhibitors

Drug Use Adverse Effects Nursing Implications


Etoposide Lymphomas; acute Myelotoxic; nausea; vomiting; Do not give IM or SC as will cause
(VP-16; VePesid) nonlymphocytic leukemia; diarrhea; somnolence; tissue necrosis.
cancer of lung, testes, peripheral neuropathy; Do not give IV push.
bladder, prostate, liver, hepatotoxicity Avoid skin contact with this drug.
uterus Hypotension can occur during
administration; monitor blood
pressure.
Vinblastine (Velban) Lymphomas; cancer of Peripheral neuropathies, Monitor CBC and platelets.
testes, breast, kidney, head parethesias, neuritis, muscle Frequent neuro checks.
and neck, Kaposis pain and weakness, pain in Monitor for extravasation.
sarcoma tumor site, urinary retention Monitor I&O.
Pacitaxel (Taxol) Advanced ovarian cancer Severe allergic reactions, bone Wear gloves when handling.
marrow suppression, Premedicate client with a steroid
peripheral neuropathy, and an H-1 and H-2 antagonist
muscle pain before administration.
Check blood pressure and pulse
during administration.

MITOTIC INHIBITORS 2. You wont lose any hair.


3. You may have constipation.
A. Prototype: vincristine (Oncovin) 4. You wont have any breathing problems.
1. Action. Acts on cells undergoing mitosis, thus
stopping cell division.
2. Use. Acute leukemia; lymphomas; cancer of
brain, breast, cervix, testes; Wilms tumor. Answers and Rationales
3. Adverse effects. Peripheral neuropathy;
paresthesias; loss of deep tendon reflexes; jaw
pain; cramps; muscle weakness; constipation; 166. 2. Alopecia is an adverse effect of vincristine
nausea, vomiting, stomatitis; phlebitis; (Oncovin).
alopecia; hyponatremia; leukopenia;
photosensitivity.
4. Nursing implications MISCELLANEOUS
a. Do not give IM or SC as tissue necrosis can
occur. ANTINEOPLASTIC AGENTS
b. For IV use, inject solution directly into
A. L-Asparaginase (Elspar)
vein or into tubing of running IV infusion.
1. Action. Enzyme that destroys asparagine, an
Infusion can be given over 1 minute.
amino acid necessary for protein synthesis of
c. Monitor bowel function.
leukemia cells. Causes death to leukemia cells.
d. Frequent neuro checks.
2. Use. Acute lymphocytic leukemia.
e. Monitor CBC and platelets.
3. Adverse effects. Anorexia, nausea, vomiting,
f. Advise client to avoid overexposure to sun.
azotemia, hemorrhagic pancreatitis, rash,
B. Related drugs. See Table 2-30.
hyperglycemia, increased serum ammonia,
anaphylaxis, and hepatotoxicity.
4. Nursing implications
a. Monitor CBC, platelets, renal and pancreatic
Sample Questions enzymes, coagulation studies, uric acid,
blood glucose, and serum albumin.
b. Dont shake vial.
166. The client requests information about the c. Only give drug in a clear solution; chance of
adverse effects of vincristine. Which statement hypersensitivity is increased with each dose.
by the nurse would need correcting? B. Hydroxyurea (Hydrea)
1. You may experience muscle cramps and 1. Action. Urea derivative that kills granulocytes.

2
muscle weakness. Prevents DNA synthesis in cell cycle.

102 NCLEX-RN Review


53155_02_Ch02_p007-110.qxd 2/21/09 10:48 AM Page 103

2. Use. Chronic myelogenous leukemia;


malignant melanoma and cancers of the head, Sample Questions
neck, ovary, and colon; sickle cell crisis.
3. Adverse effects. Anemia, leukopenia,
megaloblastosis, thrombocytopenia, anorexia, 167. Your client is receiving L-asparaginase (Elspar)
nausea, vomiting, and diarrhea. and complains of stomach pain. You will need
4. Nursing implications. Monitor CBC, platelets, to evaluate which laboratory test?
liver and renal enzymes; encourage fluids. 1. Platelet count.
C. Procarbazine (Matulane)
2. BUN.
1. Action. Similar to alkylating agents; inhibits
RNA, DNA, and protein synthesis in the cell. 3. Serum amylase.
2. Use. Hodgkins disease, multiple myeloma, 4. Serum potassium.
malignant melanoma, lung cancer, and brain
tumors.
3. Adverse effects. Anorexia, nausea, vomiting,
leukopenia, thrombocytopenia, and altered Answers and Rationales
reproductive potential.
4. Nursing implications
a. Advise client to avoid alcohol, sedatives, 167. 3. Stomach pain may be a sign of pancreatitis,
narcotics, and tricyclic antidepressants which is an adverse effect of L-asparaginase
(drug is an MAO inhibitor). (Elspar). The nurse should monitor serum
b. Restrict foods high in tyramines. amylase as an elevation of this enzyme indicates
c. Monitor CBC, platelets, and liver a need to discontinue the drug.
enzymes.

Immunosuppressants

AZATHIOPRINE (IMURAN) c. Use good handwashing and hygiene and


avoid individuals with URIs.
A. Prototype: azathioprine (Imuran) B. Related drugs
1. Action. Purine analog and derivative of 1. Mycophenolate (CellCept)
mercaptopurine that antagonizes purine a. Used to prevent and treat rejection of renal
metabolism, interferes with nucleic acid transplants.
synthesis, and alters antibody production. b. Given orally.
Immunosuppressant action not fully
understood.
2. Use. Adjunct to prevent rejection of renal
transplants; severe rheumatoid arthritis. Sample Questions
3. Adverse effects. Hypotension, pulmonary
edema; hepatotoxicity; nausea, vomiting,
diarrhea, stomatitis, anorexia; alopecia; 168. Which statement by the client indicates a need
bone marrow suppression, leukopenia, for further teaching in regard to azathioprine
thrombocytopenia; hypersensitivity therapy?
pancreatitis, skin rash; secondary 1. I will take one daily dose 1 hour before
infection. breakfast.
4. Nursing implications 2. I should avoid individuals who have the flu
a. Monitor CBC, liver and kidney function or a cold.
studies.
b. Monitor for symptoms of infection. 3. I will notify the physician if I notice any
c. Monitor for symptoms of rejection. bruising.
5. Discharge teaching 4. I will try to maintain good personal
a. Take with food. hygiene.
b. Report signs of infection and bruising.

DRUGS AND NURSING IMPLICATIONS

2 103
53155_02_Ch02_p007-110.qxd 2/21/09 10:48 AM Page 104

e. Notify physician if bruising present or


Answers and Rationales oliguria.
5. Discharge teaching. See Azathioprine (Imuran).
B. Related drugs
168. 1. Azathioprine (Imuran) should be taken with 1. Tacrolimus (Prograf)
food and in divided doses to reduce GI upset. a. Used to prevent rejection with kidney,
heart, and liver transplants.
b. Given orally or IV.
CYCLOSPORINE (SANDIMMUNE)
A. Prototype: cyclosporine (Sandimmune)
1. Action. Exact immunosuppressant action Sample Questions
unknown. Interferes with T-lymphocyte
activity.
2. Use. Prophylaxis for recipients of kidney, 169. Which lab values should be monitored in clients
heart, and liver transplants to prevent organ receiving cyclosporine?
rejection. 1. Electrolytes.
3. Adverse effects. Nephrotoxicity; 2. Glucose.
hepatotoxicity; hypertension; infections;
tremors; leukopenia; diarrhea, nausea, 3. BUN and creatinine.
vomiting; anaphylaxis in IV use; gum 4. Serum amylase.
hyperplasia.
4. Nursing implications
a. Monitor CBC, liver and kidney function
studies. Answers and Rationales
b. With IV use epinephrine; resuscitation
equipment should be available.
c. Protect IV infusion from light. 169. 3. Cyclosporine (Sandimmune) has nephrotoxic
d. Mix PO medication in milk or orange juice at effects; therefore, BUN and creatinine levels
room temperature. Stir and drink immediately should be monitored.
and rinse glass with more milk or juice to
ensure that entire dose has been taken.

Vitamins and Minerals

Vitamins and some minerals are vital substances B. See Table 4-3, Vitamins, Unit 4.
needed by the human body. Because nutrition plays an
important role, they are primarily discussed in the
nutrition section, Unit 4. Excessive quantities of
vitamins can cause adverse effects. Treatment of Sample Questions
poisonous minerals is included here as is fluoride, a
treatment for prevention of dental caries.
170. A client admits to taking Vitamin A in excess of
50,000 units daily. The nurse explains that this
VITAMINS is considered an overdose and that he can get
adequate amounts of vitamin A in his diet. What
A. General considerations related to vitamins. is an example of a good dietary source of
1. Vitamins are necessary for body metabolism of vitamin A?
carbohydrates, protein, and fat. 1. Spinach.
2. Dosage of vitamins is stated in RDAs
2. Pork.
recommended daily allowances.
3. Fat-soluble vitamins accumulate in the body; 3. Nuts.
therefore excessive amounts should not be 4. Tomatoes.
taken.

2 104 NCLEX-RN Review


53155_02_Ch02_p007-110.qxd 2/21/09 10:48 AM Page 105

HEAVY METAL ANTAGONISTS


Answers and Rationales
A. Heavy metals such as lead, iron, arsenic, gold, and
mercury can have toxic effects. Heavy metal
170. 1. Dark green vegetables such as spinach are antagonists prevent or reverse poisoning by
good food sources of vitamin A. neutralizing the heavy metals. Toxic effects or
poisoning can occur from drug overdose (e.g., gold
salts or iron), or accidental ingestion of lead chips
MINERALS from lead-containing paint, or pesticide ingestion.
B. Examples of heavy metal antagonists
A. General considerations related to minerals. 1. Deferoxamine mesylate for iron intoxication
1. Minerals are essential components of living a. May turn urine red.
tissues. b. When given IV, use an infusion pump and
2. Dosage of minerals is stated in RDAs. monitor blood pressure frequently.
3. Minerals are divided into 2 groups, major c. May cause hypotension, tachycardia,
elements and micronutrients. allergic reactions, and pain on injection.
a. Major elements: calcium, phosphorus, 2. Edetate calcium disodium (Calcium EDTA).
magnesium, sodium, potassium, chloride, For lead poisoning; can cause renal toxicity.
and sulfur. May increase intracranial pressure: do not give
b. Micronutrients: iron, copper, iodine, IV if lead encephalopathy.
manganese, zinc, fluorine, cobalt, 3. Dimercaprol (BAL in oil). Given IM in
chromium, molybdenum, and selenium. combination with calcium EDTA to treat lead
B. See Table 4-2, Minerals, Unit 4. poisoning. Also used to treat arsenic, mercury,
and gold toxicity. Painful on injection and can
cause hypertension and tachycardia in large
doses.
Sample Questions 4. Edetate disodium (Disodium EDTA) for
hypercalcemic crisis.
171. What information should the nurse provide the
client regarding iron administration?
1. Take it with milk. Sample Questions
2. Take it with meals.
3. Take after meals with an antacid.
172. What should be monitored when administering
4. Take with orange juice between meals. calcium EDTA for lead poisoning?
1. Liver function studies.
2. Kidney function.
Answers and Rationales 3. Hemoglobin and hematocrit.
4. Glucose levels.
171. 4. Orange juice contains vitamin C which helps
to absorb iron. Iron should be taken between
meals for maximum absorption. Note: sometimes Answers and Rationales
iron is given with meals to decrease GI side
effects, but absorption is reduced.
172. 2. Calcium EDTA can cause renal toxicity.

DRUGS AND NURSING IMPLICATIONS

2 105
53155_02_Ch02_p007-110.qxd 2/21/09 10:48 AM Page 106

Herbs and Herbal Health Products

HISTORY use. Herbs/herbal health products may be natural but


not necessarily safe.
A. Eighty percent of the worlds population currently B. Herbs are chemical compounds that are
use herbs for some aspect of primary health care. biologically active and as such require review and
B. Plants and plant products are still a common safety education before their use.
element today in the healing disciplines of C. Currently there is a lack of standardized and
ayurvedic (homeopathic) medicine, naturopathic scientific data to support the general use of the
(traditional oriental) medicine, and Native herbs/herbal health products that are sold as
American groups. nutritional supplements.
C. Americans embrace the use of herbs/herbal health D. These do not require FDA approval. The FDA
products and spend approximately requirements for the strength and purity of the
$5 billion dollars a year on natural herbal supplement on the label are in the early stages.
products.
D. Americans who use herbs/herbal health products
are generally better educated and are holistically COMPLICATIONS
health oriented. These individuals are more
likely to discuss and report their overall health A. Toxic impurities and incorrectly mixed herbs have
status to their health care provider. Americans resulted in kidney failure and death.
are also reluctant to tell health care providers of B. Allergic reactions and interaction with
their herb use, are risking adverse interactions prescription drugs have also been reported.
between herbs and their prescriptions, and are C. The FDA has become involved with herbs/herbal
putting themselves at risk for surgery or any health products when there have been serious
invasive procedure without disclosure of their health issues and deaths as in the case of ephedra
herbs/herbal health products use. and cascara sagrada. See Table 2-31 for commonly
used herbs and the possible side effects.

SOURCE AND USE PROFESSIONAL RESPONSIBILITIES


A. Herbs are flowering plants, shrubs, trees, moss The nurse, as a professional, should provide for
ferns, fungus, seaweed or algae plants, or plant clients the resources and client education materials
parts that are valued for medicinal qualities. Herbs that include common names and uses and side
and herbal health products are used in all shapes effects of herbs and herbal health products. The
and forms. nurse must include in the health assessment
B. Common uses include infusions, teas, tablets interview open-ended questions regarding use of
(pills), lozenges, extracts, salves, balms, ointments, herbal supplements and OTC medications. The nurse
and oils. must be aware of common herbs/herbal health
products and their interaction with commonly
prescribed medications.
NATURAL PHARMACY
A. Herbs are considered to be pharmacologic remedies
that are readily available over-the-counter for general

2 106 NCLEX-RN Review


53155_02_Ch02_p007-110.qxd 2/21/09 10:48 AM Page 107

Table 2-31 Commonly Used Herbs

Common Herbs Common Use Possible Clinical Side Effects


Ginko Improve memory; increase blood Increased risk of bleeding
circulation (peripheral and to the Changes in hemodynamic monitoring
brain); treat bronchial, asthmatic, and Interference with antiseizure medications
pulmonary conditions GI distress
St. John's Wort Antianxiety, anti-inflammatory, Decreased effectiveness of digoxin
antidepressive, and sedative agent Intensify or prolong effects of narcotics and anesthetic agents
Large doses may result in photosensitivity
Potential for constipation, abdominal cramps, dry mouth,
fatigue, dizziness, insomnia, or restlessness
Ginseng Increase physical stamina and mental Potential to falsely increase digoxin levels
concentration, enhance general health, Potential for bleeding in postmenopausal females
stimulate CNS, decrease the advances Potential for hypertension when combined with caffeine
of Alzheimers disease, reduce tinnitus May result in increased heart rates
for those with chronic ringing in the ear Contraindicated in pregnancy and lactation
Garlic Lower cholesterol and triglyceride levels Potential for dangerously low blood sugars when
and blood pressure; currently looked combined with diabetes medication
at for antioxidant, fibrinolytic, and Potential for headaches and potentiate myalgia and fatigue
antimicrobial properties destroying Potential for increased bleeding and bruising, especially for
bacteria, fungi, and parasites individuals taking anticlotting medications
Feverfew Prevent migraine headaches; decrease Potential for allergic reactions with allergies to ragweed,
the number and severity of headaches; asters, chrysanthemums, or daisies
arthritis and rheumatic disease Potential for abdominal pain, glossitis, stomatitis, and
allergic dermatitis; GI upsets and nervousness
Potential for interaction with thrombolytics, anticoagulants,
and aspirin, increasing bleeding
Potential for dangerously high levels of heart rate and blood
pressure if combined with Imitrex or other migraine medication
Potential for withdrawal syndrome in abrupt withdrawal,
including rebound headaches, insomnia, fatigue, and
nervousness
Contraindicated in pregnancy and lactation
Kava Kava Nervousness, anxiety, or restlessness; Potential for increased effects of certain antiseizure
used as a muscle relaxant, medications, intensifying psychoactive agents
antispasmodic, anticonvulsant Potentiate barbiturates and prolong effects of certain
psychotropic treatment for cystitis anesthetics
Potential for increased alcohol effects with risk of toxicity
Potential for allergic reactionsrash, GI discomfort, changes
in ocular movement, and hepatotoxicity
Potential for increased risk of suicide for people with
endogenous depression
Contraindicated in pregnancy and lactation
Licorice Treat coughs and chills; expectorant; Potential for hypertension, swelling, or electrolyte
anti-inflammatory; antiallergic; imbalance
treatment of, preventing, and healing Potential for headaches, lethargy, and sodium and water
stomach ulcers retention in long-term use
Potential for potassium loss and possible heart failure

(continues)

DRUGS AND NURSING IMPLICATIONS

2 107
53155_02_Ch02_p007-110.qxd 2/21/09 10:48 AM Page 108

Table 2-31 Commonly Used Herbs (continued)

Common Herbs Common Use Possible Clinical Side Effects


Ginger Motion sickness, nausea, vomiting, Potential for increased bleeding in clients already taking
and vertigo, stomachaches, to aid certain anticlotting medications, anyone at risk for
digestion, a mild stimulant to help hemorrhage
promote circulation Contraindicated for treating morning sickness associated
with pregnancy
Potential for CNS depression or dysrhythmia in overdose
Saw Palmetto Enlarged prostate and urinary Potential for GI upset, nausea, abdominal pain,
inflammations; mild diuretic hypertension, headache, urinary retention, and back pain
Potential for effects with other hormone therapies such as
adrenergic drugs and oral contraceptives; may result in oral
contraceptive failure
Reduces absorption of iron
Potential for false-positive prostate specific antigen test
results
Contraindicated in pregnancy and lactation due to its
hormonal effects
Valerian Mild sleep aid, muscle relaxant, for relief Potential for GI complaints, headache
of nervous stomachs, for stress relief, Potential for excitatory effects, cardiac function disorders,
or as a sedative; decrease time to and restless-sleepless states
sleep, does not result in sleep Potential for increased effects of certain antiseizure
hangover medications
Potential for prolonging the effects of certain anesthesia
agents
Potential for increased sedative effects of barbiturates
Potential to demonstrate an additive effect when used with
benzodiazepines
Potential for hepatomegaly when combined with other
herbs such as skullcap and mistletoe
Contraindicated in infants and pregnant women

Vaccines and Toxoids

VACCINES AND TOXOIDS in the body. Takes time to develop and


considered to be permanent. Acquired by
A. Vaccines and toxoids: general information the person having a specific disease or by
1. Given to prevent some infectious diseases and inoculation with toxoid or vaccines.
diseases transferred by animal bites and Passive immunity involves the individual
injuries. receiving antibodies against antigens that
2. Vaccine is composed of weakened or dead have been formed someplace other than
microorganisms that cause antibody formation. within the person. Is immediate but effects
3. Toxoid is a bacterial toxin that has reduced are short-lived. Is acquired through
toxicity but can cause antibody formation. injection of serum containing antibodies.
4. Immunity is the ability to fight or conquer 5. If immunosuppressed, receiving corticosteroid
infection. therapy, or has an active infection, should not
a. Natural immunity exists from birth and is a be inoculated.
basic form of resistance to disease. B. Specific vaccines and toxoids
b. Acquired immunity occurs after birth. Can 1. DPT (Diphtheria, Tetanus Toxoid, and
be active or passive. Involves the Pertussis Vaccine). Produces active immunity
manufacture of antibodies against antigens by forming antibodies.

2 108 NCLEX-RN Review


53155_02_Ch02_p007-110.qxd 2/21/09 10:48 AM Page 109

a. DTwP Vaccine (Tri-Immunol). Contains serious adverse effects. First dose given at
diphtheria and tetanus toxoids and whole- 12 months of age and second dose 612
cell pertussis vaccine. months after.
b. DTaP Vaccine (Tripedia, Acel-Imune, 10. Refer to Table 5-5, Recommended Schedule for
Certiva, Infanix). Contains diphtheria and Immunization, Unit 5.
tetanus toxoids and acellular pertussis
vaccine. Has fewer side effects and is more
effective than DTwP. Recommended for all
children, including those who began the Sample Questions
series with DTwP.
c. Doses are given at 2 months, 4 months,
6 months, between 15 and 18 months, and 173. A 4-month-old child is brought to the clinic for
4 and 6 years. the next set of immunizations. Which of the
2. MMR (Measles, Mumps, Rubella). Contains following would contraindicate receiving
live attenuated virus. Should not be given immunizations at this time?
during pregnancy. Give with caution to 1. Delayed development.
children who have a history of 2. Weight loss.
thrombocytopenia and anaphylactic-like
reactions to eggs, neomycin, and gelatin. 3. Anorexia.
a. Give between 12 and 15 months and 4 and 4. Active infection.
6 years. Second dose must be given before
age 12.
b. DPT can be given with MMR.
3. Inactivated Polio Vaccine (IPV). Contains Answers and Rationales
inactivated viruses of all three polio serotypes.
Four doses are given: at 2 months, at 4 months,
between 6 and 18 months, and between 4 and 173. 4. Any evidence of an active infection
6 years. Has no serious adverse effects. contraindicates immunization. Other
4. Bacillus Calmette-Guerin Vaccine (BCG). contraindications for immunization are
Produces active immunity to tuberculosis (TB). immunosuppression and corticosteroid therapy.
Give to infants in countries where TB is
endemic. Persons who have had BCG will have
a positive purified protein derivative (PPD) test. IMMUNE SERUMS
a. Also used to stimulate the immune system
in treating cancer. A. Immune serums
b. Should not be given to persons taking 1. Provide passive immunity. They are antibodies
antituberculosis drugs. that are formed in another person or animal
5. Hepatitis B Vaccine (Engerix-B). Effective and then given to the client. Offer immediate
against all types of hepatitis B and immunity but duration is short. Treatment
recommended for individuals at risk to considered to be only moderately effective.
contract hepatitis B. Now recommended for 2. Hepatitis B immune globulin, human. Given as
all children. a prophylactic treatment after exposure to
a. Does not prevent an unrecognized hepatitis B. Needs to be given to adults within
infection already present. 7 days of exposure and repeated in 2830 days.
b. Two IM doses are given 1 month apart and Newborns are immunized at birth and then
a third dose is given 6 months after the again at 3 and 6 months. Cautious use in
first dose. persons with hypersensitivity to immune
6. Hemophilus Influenza B (Hibtiter). Given at 2, globulins. Adverse effects: tenderness at
4, and 6 months and booster at 15 months. injection site and urticaria.
7. Td (Adult Tetanus Toxoid and Diphtheria 3. Immune serum globulin (immunoglobulin).
Toxoid). Give at age 1416 years and repeated Given to nonimmunized persons to prevent or
every 10 years. reduce severity of various infectious diseases
8. Varicella Virus Vaccine (Varivax). Contains and prophylactically in primary immune
live, attenuated varicella viruses. Has no deficiencies. Adverse effects: pain and redness
serious adverse effects. A single dose should at the injection site.
be given to children 12 to 18 months of age. 4. Tetanus immune globulin, human (Hypertet).
9. Hepatitis A Vaccine (Havrix). Contains Used if wound more than 24 hours old or if
inactivated hepatitis A virus. Recommended client has fewer than two previous tetanus
where there is high risk for disease. Has no toxoid injections. Is considered to be better

DRUGS AND NURSING IMPLICATIONS

2 109
53155_02_Ch02_p007-110.qxd 2/21/09 10:48 AM Page 110

than antitoxin. Adverse effect: discomfort at REFERENCES AND SUGGESTED READINGS


the injection site.
5. Rho (D) immune globulin, human (RhoGAM). Abrams, A. (2006). Clinical drug therapy; Rationales for
Given to Rh-negative mothers with Rh-positive nursing practice (8th ed.). Philadelphia: Lippincott
fetus, and also given to Rh-negative women Williams & Wilkins.
Aschenbrenner, D., Cleveland, L., & Venable, S. (2005). Drug
who have miscarriages or abortions. Must be
therapy in nursing (2nd ed.). Philadelphia: Lippincott
given within 72 hours of delivery. Contraindi- Williams & Wilkins.
cated in hypersensitivity to immune globulin. Barnes, P. M., & Powell-Griner, E. (2004). Complementary and
Adverse effect: local tenderness. alternative medicine use among adults: United States, 2002.
U.S. Department of Health and Human Services.
Curren, A. M. (2008). Math for meds: Dosage and solutions
Sample Questions (10th ed.). Clifton Park, NY: Delmar Cengage Learning.
Deglin, J. H., & Vallerand, A. H. (2008). Daviss drug guide for
nurses (11th ed.). Philadelphia: F. A. Davis.
Dubois, D., & Dubois, E. F. (1916). A formula to estimate the
174. The client is given the hepatitis B immune globulin approximate surface area if height and weight be known.
serum, which will provide passive immunity. What Archives of Internal Medicine, 17, 863.
is an advantage of passive immunity? Gutierrez, K., & Queener, S. (2003). Pharmacology for nursing
1. It has effects that last a long time. practice. St. Louis, MO: Mosby.
Hodgson, B., & Kizor, R. J. (2008). Saunders nursing drug
2. It is highly effective in treatment of disease. handbook (12th ed.). St. Louis, MO: Saunders.
3. It offers immediate protection. Karch, A. (2006). Focus on nursing pharmacology (4th ed.).
Philadelphia: Lippincott Williams & Wilkins.
4. It encourages the body to produce antibodies.
Lehne, R. (2007). Pharmacology for nursing care (6th ed.).
Philadelphia: Elsevier Science.
175. What common adverse effects will the nurse tell
Micozzi, M. S. (2005). Fundamentals of complementary
the client may be experienced after being given and alternative medicine (3rd ed.). New York: Elsevier
hepatitis B immune globulin? Health Science.
1. Tachycardia and chest tightness. National Council of State Boards of Nursing, Inc. (2008).
NCLEX-RN Examination. Chicago: Author.
2. Heartburn and diarrhea.
Pickar, G. D. (2007). Dosage calculations, a ratio-proportion
3. Dyspnea and upper respiratory infection. approach (2nd ed.). Clifton Park, NY: Delmar
4. Pain and tenderness at the injection site. Cengage Learning.
Saxton, D., ONeill, N. E., & Glavinspiehs, C. (2005). Math
and meds for nurses (2nd ed.). Clifton Park, NY: Delmar
Cengage Learning.
Answers and Rationales Spratto, G. R., & Woods, A. L. (2009). Delmar nurses drug
handbook 2009. Clifton Park, NY: Delmar Cengage Learning.
Wilson, B., Shannon, M., & Stang, C. (2008). Nurses drug guide
174. 3. Passive immunity provides immediate 2008. Upper Saddle River, NJ: Prentice Hall.
protection. Passive immunity is also
short-lived, is limited in effectiveness, and
does not stimulate the body to produce
antibodies.

175. 4. The most common adverse effects of hepatitis B


immune globulin are pain and tenderness at the
injection site.

2 110 NCLEX-RN Review


53155_03_Ch03_p111-142.qxd 2/21/09 11:35 AM Page 111

U N I T 3

UNIVERSAL
PRINCIPLES OF
NURSING CARE
MANAGEMENT

Nurses must frequently apply various management principles UNIT OUTLINE


while caring for their clients in various health care settings. This 112 Nursing Practice Standards
unit has been crafted to clarify these issues. It begins with a 116 Legal and Ethical Aspects
comprehensive view of nursing practice standards as well as of Nursing
legal and ethical aspects of nursing. 118 Managing Client Care
121 Safety
Client care management issues such as determining priorities,
130 Cultural Diversity in Health
working with the health care team, making assignments, Practices
delegating to unlicensed assistive personnel, and coordinating
client care as the client progresses from admission through
discharge have been described along with valuable principles
to facilitate the nurses application of this information.

Safety considerations regarding fire, disaster management,


electricity, equipment, and the use of physical restraints have
been incorporated.

This unit also includes selected principles and interventions


related to specific aspects of care such as body mechanics,
transfer techniques, positioning, the hazards and prevention of
immobility, application of cold and heat, asepsis, and the care
of clients who develop or are at risk for pressure ulcers.
Additionally, a section on cultural diversity in health practices
explores key issues related to cultural, religious, food, and death
practices in the process of nursing care delivery.

111
53155_03_Ch03_p111-142.qxd 2/21/09 11:35 AM Page 112

Nursing Practice Standards

NURSING: SCOPE & STANDARDS Measurement Criteria


OF PRACTICE (2004) A. Data collection process:
1. Systematic
Standards are authoritative statements by which the 2. Ongoing
nursing profession describes the responsibilities for B. Holistic data collection involves:
which its practitioners are accountable. Consequently, 1. Client
standards reflect the values and priorities of the 2. Family
profession. Standards provide direction for professional 3. Other health care providers as appropriate
nursing practice and a framework for the evaluation of 4. Environment
practice. Written in measurable terms, standards also C. Priority of data collection activities determined by:
define the nursing professions accountability to the 1. Clients immediate condition
public and the client outcomes for which nurses are 2. Anticipated needs of the client or situation
responsible. D. Uses appropriate evidence-based:
Nursing: Scope & Standards of Practice describes a 1. Assessment techniques
competent level of nursing practice and professional 2. Instruments
performance that is common to all registered nurses. E. Uses analytical models and problem-solving tools
The scope of the practice statement articulates the F. Synthesizes available relevant data, information,
who, what, when, where, and how of practice, for and knowledge to identify patterns and variances
nursing organizations, policy makers and the nurses G. Relevant data documented in a retrievable format
accountability to the public. The practice part of the
statement consists of 2 components: Standards of Standard 2. Diagnosis
Practice, which contains 6 standards and Standards
of Professional Performance, which contains Registered nurse analyzes the assessment data to
9 standards. These are presented in the following determine the diagnoses or issues.
section (ANA, 2004).
Nursing: Scope & Standards of Practice is used in Measurement Criteria
conjunction with Nursings Social Policy Statement
(ANA, 2003) and the Guide to the Code of Ethics for A. Diagnoses or issues from assessment data
Nurses: Interpretation and Application (ANA, 2008). B. Diagnoses are validated with:
Together these resources provide a complete and 1. Client
definitive description that best serves the publics 2. Family
health and the nursing profession. There are 3. Other health care providers, when possible
additional scope of practice statements specific to and appropriate
those registered nurses in the specialty practices, but C. Diagnoses or issues documented to facilitate
have been omitted from this text because the emphasis determination of:
of this text is preparation of the nurse generalist (one 1. Expected outcomes
who has graduated from a diploma, associates or 2. Plan of care
baccalaureate level program).
Standard 3. Outcomes Identification
Registered nurse identifies expected outcomes for a
STANDARDS OF PRACTICE plan individualized to the client or the situation.

(Reprinted with permission from American Nurses Measurement Criteria


Association, Nursing: Scope and Standards of Practice,
2004 nursebooks.org, American Nurses Association, A. Outcomes are formulated with:
Silver Spring, MD) 1. Client
2. Family
3. Other health care providers, when possible
Standard 1. Assessment and appropriate
Registered nurse collects comprehensive data B. Culturally appropriate expected outcomes derived
pertinent to the clients health or the situation. from diagnoses
C. Considers associated risks, benefits, costs, current

3
scientific evidence, and clinical expertise

112 NCLEX-RN Review


53155_03_Ch03_p111-142.qxd 2/21/09 11:35 AM Page 113

D. Defines expected outcomes considering associated C. Uses evidence-based interventions and treatments
risks, benefits and costs, and current scientific specific to the diagnosis or problem
evidence, in terms of: D. Uses community resources and systems to
1. Client implement plan
2. Client values E. Collaborates with nursing colleagues and others
3. Ethical considerations
4. Environment Standard 5a. Coordination of Care
5. Situation
E. Outcomes include a time estimate for attainment Registered nurse coordinates care delivery.
F. Outcomes provide direction for continuity of care
G. Modifies outcomes based on: Measurement Criteria
1. Changes in the status of the client
A. Coordinates implementation of the plan
2. Evaluation of the situation
B. Employs strategies to promote health and a safe
H. Documents expected outcomes as measurable goals
environment
C. Documents the coordination of the care
Standard 4. Planning
Registered nurse develops a plan that prescribes Standard 5b. Health Teaching
strategies and alternatives to attain expected outcomes.
and Health Promotion
Measurement Criteria Registered nurse employs strategies to promote health
and a safe environment.
A. Develops individualized plan considering client
characteristics or the situation, including:
Measurement Criteria
1. Age
2. Culturally appropriate A. Provides health teaching that addresses:
3. Environmentally sensitive 1. Healthy lifestyles
B. Plan is developed with: 2. Risk-reducing behaviors
1. Client 3. Developmental needs
2. Family 4. Activities of daily living
3. Others, as appropriate 5. Preventive self-care
C. Plan includes strategies that address: B. Uses health promotion and health-teaching
1. Each of identified diagnoses or issues methods appropriate to:
2. Promotion and restoration of health 1. Situation
3. Prevention of illness, injury, and disease 2. Clients developmental level
D. Provides for continuity within the plan 3. Learning needs
E. Incorporates a time line within the plan 4. Readiness
F. Establishes the plan priorities with: 5. Ability to learn
1. Client 6. Language preference
2. Family 7. Culture
3. Others, as appropriate C. Seeks opportunities for feedback/evaluation of
G. Utilizes the plan to provide direction to health effectiveness of strategies
care team
H. Plan reflects current statutes, rules and Standard 6. Evaluation
regulations, and standards
I. Integrates current trends and research affecting care Registered nurse evaluates progress toward
J. Considers the economic impact of the plan attainment of outcomes.
K. Plan uses standardized language/recognized
terminology Measurement Criteria
Standard 5. Implementation A. Evaluation of outcomes is:
1. Systematic
Registered nurse implements the identified plan. 2. Ongoing
3. Criterion-based
Measurement Criteria 4. Related to structures and processes in the plan
and time line
A. Implements plan in safe and timely manner
B. Client and other care providers are involved
B. Documents implementation of the identified plan,
in process, as appropriate
including:
C. Effectiveness of planned strategies evaluated by:
1. Any modifications
1. Client responses
2. Changes

3
2. Attainment of expected outcomes
3. Omissions

UNIVERSAL PRINCIPLES OF NURSING CARE MANAGEMENT 113


53155_03_Ch03_p111-142.qxd 2/21/09 11:35 AM Page 114

D. Documents the results of the evaluation Standard 8. Education


E. Uses ongoing assessment data to revise (as
needed): Registered nurse attains knowledge and competency
1. Diagnoses that reflects current nursing practice.
2. Outcomes
3. Plan Measurement Criteria
4. Implementation A. Participates in ongoing educational activities
F. Disseminates results (as appropriate, in accordance related to knowledge bases and professional issues
with state and federal laws and regulations) to: B. Demonstrates a commitment to lifelong learning:
1. Client 1. Self-reflection
2. Others involved in the care or situation 2. Inquiry to identify learning needs
C. Seeks experiences that reflect current practice to
maintain skills and competence in clinical practice
STANDARDS OF PROFESSIONAL or role performance
D. Acquires knowledge and skills appropriate to the
PERFORMANCE specialty area, practice setting, role, or situation
E. Maintains records that provide evidence of
Standard 7. Quality of Practice competency and lifelong learning
Registered nurse systematically enhances the quality F. Seeks experiences and formal and independent
and effectiveness of nursing practice. learning activities to maintain and develop clinical
and professional skills and knowledge
Measurement Criteria
Standard 9. Professional Practice
A. Documents application of the nursing process in a
responsible, accountable, and ethical manner Evaluation
B. Uses the results of quality improvement activities Registered nurse evaluates ones own nursing practice
to initiate changes in: in relation to professional practice standards and
1. Nursing practice guidelines, relevant statutes, rules, and regulations.
2. Health care delivery system
C. Uses creativity and innovation in nursing practice Measurement Criteria
to improve care delivery
D. Participates in activities to improve quality and A. Provides age-appropriate care in a culturally and
effectiveness of nursing practice. May include: ethnically sensitive manner
1. Identifying aspects of practice important for B. Engages in self-evaluation on a regular basis by
monitoring identifying:
2. Using indicators for monitoring 1. Areas of strength
3. Collecting data to monitor quality and 2. Areas for further professional development
effectiveness C. Obtains informal feedback regarding own practice
4. Analyzing quality data to identify from clients, peers, colleagues, and others
opportunities for improvement D. Participates in systematic peer review, as appropriate
5. Making recommendations to improve nursing E. Takes action to achieve goals identified during the
practice or outcomes evaluation process
6. Implementing activities to enhance the quality F. Provides rationales for practice beliefs, decisions,
of nursing practice and actions as part of the informal and formal
7. Developing, implementing, and evaluating evaluation processes
policies, procedures, and/or guidelines to
improve the quality of practice Standard 10. Collegiality
8. Participating on interdisciplinary teams to Registered nurse interacts with and contributes to the
evaluate clinical care or health services professional development of peers and colleagues.
9. Participating in efforts to minimize costs and
unnecessary duplication Measurement Criteria
10. Analyzing factors related to safety, satisfaction,
effectiveness, and cost/benefit options A. Shares knowledge and skills with peers and
11. Analyzing organizational systems for colleagues (i.e., client care conferences,
barriers presentations, or formal or informal meetings)
12. Implements processes to remove or decrease B. Provides peers with feedback regarding their
barriers within organizational systems practice/role performance
13. Incorporates new knowledge to initiate changes C. Interacts with peers and colleagues to enhance own
in nursing practice if desired outcomes not professional nursing practice/role performance

3
achieved D. Maintains compassionate and caring relationships
with peers and colleagues

114 NCLEX-RN Review


53155_03_Ch03_p111-142.qxd 2/21/09 11:35 AM Page 115

E. Contributes to an environment conducive to 1. Identifying clinical problems specific to nursing


education of health care professionals research (client care and nursing practice)
F. Contributes to a supportive and healthy work 2. Participating in data collection (surveys, pilot
environment projects, formal studies)
3. Participating in a formal committee or program
4. Sharing research activities and/or findings
Standard 11. Collaboration with peers and others
Registered nurse collaborates with client, family, and 5. Conducting research
others in the conduct of nursing practice. 6. Critically analyzing and interpreting research
for application to practice
Measurement Criteria 7. Using research findings in development of
policies, procedures, and standards of practice
A. Communicates with client, family, and health care in client care
providers regarding client care and the nurses role 8. Incorporating research as a basis for learning
in the provision of that care
B. Collaborates with appropriate individuals in
creating a documented plan focused on outcomes Standard 14. Resource Utilization
with decisions related to care and delivery of Registered nurse considers factors related to safety,
services that indicate communication effectiveness, cost, and impact on practice in the
C. Partners with others to effect change and generate planning and delivery of nursing services.
positive outcomes through knowledge of the client
or situation Measurement Criteria
D. Documents referrals, including provisions for
continuity of care A. Evaluates factors such as safety, effectiveness,
availability, cost and benefits, efficiencies, and
impact on practice when choosing practice options
Standard 12. Ethics that would result in the same expected outcome
Registered nurse integrates ethical provisions in all B. Assists client and family to identify and secure
areas of practice. appropriate/available services to address health-
related needs
C. Assigns or delegates tasks, based on needs and
Measurement Criteria
condition of the client, potential for harm, stability
A. Uses current Code of Ethics for Nurses with of the clients condition, complexity of the task,
Interpretive Statements (ANA) to guide practice and predictability of the outcome
B. Delivers care in a way that preserves and protects D. Assists client and family to become informed
client autonomy, dignity, and rights consumers about options, costs, risks, and benefits
C. Maintains client confidentiality within regulatory of treatment and care
parameters
D. Serves as a client advocate and fosters skills for Standard 15. Leadership
self-advocacy
E. Maintains a therapeutic/professional client-nurse Registered nurse provides leadership in the
relationship with appropriate role boundaries professional practice setting and the profession.
F. Demonstrates commitment to practicing self-care,
managing stress, and connecting with self and others Measurement Criteria
G. Contributes to resolving ethical issues of clients,
A. Engages in teamwork as a team player and a team
colleagues, or systems (i.e., ethics committees)
builder
H. Reports illegal, incompetent, or impaired practices
B. Works to create and maintain healthy work
environments in local, regional, national, or
Standard 13. Research international communities
C. Displays ability to define a clear vision, associated
Registered nurse integrates research findings into
goals, and a plan to implement and measure
practice.
progress
D. Demonstrates a commitment to continuous,
Measurement Criteria lifelong learning for self and others
A. Utilizes the best available evidence, including E. Teaches others to succeed by mentoring and other
research findings, to guide practice decisions strategies
B. Actively participates in research activities at various F. Exhibits creativity and flexibility through times of
levels appropriate to the nurses level of education change
and position. Such activities may include: G. Demonstrates energy, excitement, and a passion for

3
quality work

UNIVERSAL PRINCIPLES OF NURSING CARE MANAGEMENT 115


53155_03_Ch03_p111-142.qxd 2/21/09 11:35 AM Page 116

H. Accepts mistakes by self and others to create a culture and unlicensed personnel in any assigned or
where risk-taking is not only safe but expected delegated tasks
I. Inspires loyalty through valuing people as the K. Serves in key roles in work setting (committees,
most precious asset in organization councils, and administrative teams)
J. Directs coordination of care across settings and L. Promotes advancement of profession via
among caregivers, including oversight of licensed participation in professional organizations

Legal and Ethical Aspects of Nursing

OVERVIEW J. Distributive justice: allocation of goods and


services and how or to whom they are distributed.
It is important for nurses to recognize that nursing practice 1. Equality: everyone receives the same.
is guided by legal restrictions and professional obligations. 2. Need: greater services go to those with greater
Legal responsibilities are regulated by state nurse-practice needs (e.g., critically ill client receives more
acts and may vary from state to state. In addition, general intensive nursing care).
standards for the practice of nursing have been developed 3. Merit: services go to more deserving (used as a
and published by the American Nurses Association, criterion for transplant recipients).
which has also developed a code of ethics.
Nurses need to be aware of these standards, as well
as legal and ethical concepts and principles, because GUIDE TO THE CODE OF ETHICS
nurses are accountable for their actions in all these
areas in their professional role.
FOR NURSES: INTERPRETATION
AND APPLICATION (ANA, 2001)*
Ethical Concepts That Apply The Guide to the Code of Ethics for Nurses:
to Nursing Practice Interpretation and Application contains the full text
A. Ethics: rules and principles that guide nursing of the Code of Ethics for Nurses with Interpretative
decisions or conduct in terms of the Statements (ANA, 2001), in addition to a history,
rightness/wrongness of that decision or action. purpose, application, case studies and examples.
B. Morals: personally held beliefs, opinions, and This guide is used as a tool for teaching employees
attitudes that guide our actions. and students how to apply the values in the Code of
C. Values: appraisal of what is good. Ethics.
1. Dilemmas may occur when different values The Code of Ethics for Nurses serves the following
conflict. purposes:
2. Example: clients right to refuse treatment may It is a succinct statement of the ethical obligations
be in conflict with nurses obligation to benefit and duties of every individual who enters the
client and to carry out treatment. nursing profession.
D. Ethical dilemma: a problem in making a decision It is the professions non-negotiable ethical standard.
because there is no clearly correct or right choice. It is an expression of nursings own understanding
This may result in having to choose an action that of its commitment to society.
violates one principle or value in order to promote
another. Code of Ethics for Nurses
E. Autonomy: an individual has the right to make his
or her own decision regarding treatment and care. A. The nurse, in all professional relationships,
F. Paternalism: another person makes decisions practices with compassion and respect for the
about what is right or best for the individual. inherent dignity, worth, and uniqueness of every
G. Beneficence: promoting good or doing no harm to individual, unrestricted by considerations of social
another. or economic status, personal attributes, or the
H. Right to know: right to knowledge necessary or nature of health problems.
helpful in making an informed decision.
I. Principle of double effect: promoting good may
*Reprinted with permission from American Nurses Association, Guide to
involve some expected harm, such as adverse side the Code of Ethics for Nurses: Interpretation and Application, 2001.

3
effects of medication. nursebooks.org, American Nurses Association, Silver Spring, MD.

116 NCLEX-RN Review


53155_03_Ch03_p111-142.qxd 2/21/09 11:35 AM Page 117

B. The nurses primary commitment is to the client, 2. Valid: having capacity to give consent and also
whether an individual, family, group, or community. demonstrating an understanding of the nature
C. The nurse promotes, advocates for, and strives to of the treatment, expected effects, possible side
protect the health, safety, and rights of the client. effects, and alternatives to treatment.
D. The nurse is responsible and accountable for F. Assault: unjustifiable threat or attempt to touch or
individual nursing practice and determines the injure another.
appropriate delegation of tasks consistent with the G. Battery: unlawful touching or injury to another.
nurses obligation to provide optimum client care. H. Crime: act that is a violation of duty or breach of
E. The nurse owes the same duties to self as to others, law, punishable by the state by fine or
including the responsibility to preserve integrity imprisonment (see Table 3-1).
and safety, to maintain competence, and to I. Tort: a legal wrong committed against a person, his
continue personal and professional growth. or her rights, or property; intentional, willfully
F. The nurse participates in establishing, committed without just cause (see Table 3-1). The
maintaining, and improving health care person who commits a tort is liable for damages in
environments and conditions of employment a civil action.
conducive to the provision of quality health care 1. Negligence and malpractice are torts.
and consistent with the values of the profession 2. Victims of malpractice are entitled to receive
through individual and collective action. monetary awards (damages) to compensate for
G. The nurse participates in the advancement of the their injury or loss.
profession through contributions to practice, J. Good Samaritan doctrine: rescuer is protected
education, administration, and knowledge from liability when assisting in an emergency
development. situation or rescuing a person from imminent and
H. The nurse collaborates with other health serious peril, if attempt is not reckless and
professionals and the public in promoting persons condition is not made worse.
community, national, and international efforts to K. Licensure: Granted by states to protect public
meet health needs. 1. Purposes
I. The profession of nursing, as represented by a. Standards for entry into practice
associations and their members, is responsible for b. Defines what licensed person can do (e.g.,
articulating nursing values, for maintaining the Nurse Practice Acts)
integrity of the profession and its practice, and for 2. License revocation/suspension
shaping social policy. a. Criteria vary in each state.
b. Licensed nurses should be aware of their
states Nurse Practice Act.
Legal Concepts That Apply c. Nurses who are disciplined in one state
to Nursing Practice may also be disciplined in another state in
which they hold a license.
A. Standards: identify the minimal knowledge and
conduct expected from a professional practitioner.
Standards are applied as they relate to a practitioners Table 3-1 Examples of Crimes and Torts
experience and educational preparation. For
example, any nurse would be expected to be certain Crimes Torts
that an ordered medication was being given to the Assault and battery Assault and battery
correct client. However, more complex nursing Involuntary manslaughter: False imprisonment: intentional
actions, such as respirator monitoring, would require committing a lawful act confinement of a client
supervised experience and/or continuing education. that results in the death without consent
B. Negligence: lack of reasonable conduct or care. of a client Fraud
Omitting an action expected of a prudent person in Illegal possession or sale Negligence/malpractice:
a particular circumstance is considered negligence, of a controlled substance Medication errors
as is committing an action that a prudent person Carelessness resulting
would not. in loss of clients property
C. Malpractice: professional negligence, misconduct, Burns from hot water bottles,
or unreasonable lack of skill resulting in injury or heating pads, hot soaks
loss to the recipient of the professional services. Failure to prevent falls by
D. Competence: ability or qualification to make not using bed rails
informed decisions. Incompetence in assessing
E. Informed consent: agreement to the performance of symptoms (shock, chest pain,
a procedure/treatment based on knowledge of respiratory distress)
facts, risks, and alternatives. Administering treatment
1. Simple: having capacity to give consent for the to wrong client
treatment or procedure.

UNIVERSAL PRINCIPLES OF NURSING CARE MANAGEMENT

3 117
53155_03_Ch03_p111-142.qxd 2/21/09 11:35 AM Page 118

Legal Concepts Related to Psychiatric-Mental E. Rights of clients: rights that each state may grant to
Health Nursing its residents committed to a psychiatric hospital.
1. Right to receive treatment and not just be confined
A. Voluntary commitment: client consents to hospital 2. Right to the least restrictive alternative (locked
admission. vs unlocked units, inpatient vs outpatient care)
1. Client must be released when he no longer 3. Right to individualized treatment plan and to
chooses to remain in the hospital. participation in the development of that plan
2. State laws govern how long a client must and to an explanation of the treatment
remain hospitalized prior to release. 4. Right to confidentiality of records
3. Client has the right to refuse treatment. 5. Right to visitors, mail, and use of telephone
B. Involuntary commitment: client is hospitalized 6. Right to refuse to participate in experimental
without consent. treatments
1. Most states require that the client be mentally 7. Right to freedom from seclusion or restraints
ill and be a danger to others/self (includes 8. Right to an explanation of rights and assertion
being unable to meet own basic needs such as of grievances
eating or protection from injury). 9. Right to due process
2. In most states the client who has been
involuntarily committed may not refuse
treatment.
Legal Responsibilities of the Nurse
C. Insanity: a legal term for mental illness in which A nurse is expected to:
an individual cannot be held responsible for or A. Be responsible for his or her own acts
does not understand the nature of his or her acts. B. Protect the rights and safety of patients
D. Insanity defenses: not guilty by reason of insanity. C. Witness, but not obtain, informed consent for
1. MNaghten rule (right and wrong test): the medical procedures
accused is not legally responsible for an act if, D. Document and communicate information
at the time the act was committed, the person regarding client care and responses
did not, because of mental defect or illness, E. Refuse to carry out orders that the nurse
know the nature of the act or that the act was knows/believes are harmful to the client
wrong. F. Perform acts allowed by that nurses state nurse
2. Irresistible impulse: the accused, because of practice act
mental illness, did not have the will to resist G. Reveal clients confidential information only to
an impulse to commit the act, even though appropriate persons
able to differentiate between right and wrong. H. Perform acts for which the nurse is qualified by
3. Individuals who commit crimes and either education or experience
successfully plead insanity defenses may be I. Witness a will (this is not a legal obligation, but
involuntarily committed to psychiatric the nurse may choose to do so)
hospitals under civil commitment laws. There J. Restrain clients only in emergencies to prevent
is presently a trend toward finding individuals injury to self/others. Clients have the right to be
insane and guilty. free from unlawful restraint.

Managing Client Care

PRIORITIES OF CLIENT CARE Table 3-2 Maslows Hierarchy of Needs

For One Client Maturity


A. Maslows Hierarchy of Needs (1954) (see Self-Actualization
d
Table 3-2) ar Self-Esteem/Respect Needs
w
1. Principles To
a. An individuals needs are depicted in g Affection or Belonging Needs
in
ascending levels on the hierarchy. ow
r Safety and Security Needs
b. Needs on one level must be (at least G
partially) met before one can focus on a Physiologic/Survival Needs

3
higher-level need Adaptation based on Maslows Hierarchy of Needs.

118 NCLEX-RN Review


53155_03_Ch03_p111-142.qxd 2/21/09 11:35 AM Page 119

2. Levels of Maslows Hierarchy 2. Assesses available staff and their job


a. Physiologic/survival needs: basic human descriptions. Decides how to use human
needs (e.g., oxygen, water, food, elimination, resources to accomplish care.
physical and mental rest, activity, and B. Typical levels of staff
avoidance of pain) 1. Nursing Assistants
b. Safety and security needs a. Unlicensed assistive personnel (UAP)
1) Protection from physical harm (e.g., b. Assign to majority of the routine
mechanical, thermal, chemical, or procedures (e.g., baths, bed making,
infectious) routine VS, etc.)
2) Interpersonal, economic, and 2. Licensed Practical Nurse (LPN)/Licensed
emotional security Vocational Nurse (LVN)
c. Affection or belonging needs a. LPN/LVNs work under the direction of a
1) Giving and receiving of affection registered nurse or a physician.
2) Sense of belonging (e.g., including b. Performs most patient care except in some
client/family in planning of care) specialty areas.
d. Self-esteem/respect needs c. Some states prohibit IV push medications
1) Feeling of self-worth or to hang the first unit of blood.
2) Need for recognition 3. Registered Nurse (RN)
e. Self-actualization a. Performs the most complex procedures
1) Highest level: not reached by all (e.g., starting IVs, developing the plan of
2) Independence care, interpreting ECGs, correlating
3) Feeling of achievement or competency laboratory results with client status)
B. Application of Maslows Hierarchy in health care b. Applies the nursing process for each client
1. Client care c. Coordinates the medical plan with the
a. Basic physiologic needs should take nursing care plan
precedence over higher-level needs and on d. Coordinates client activities
up the continuum accordingly. 1) Other departments
b. Professional nurse often delivers care at 2) Health care workers
multiple levels simultaneously (e.g., while 3) Community
feeding a client, you position them to e. Performs client/family teaching
prevent aspiration and converse with them). f. Ensures documentation of care and outcomes
c. Tool to guide decision making of priorities in g. Directs and supervises care given by LPNs
emergencies and time management of care. and ancillary personnel
2. Also applies to families, staff, and yourself h. Acts as a client advocate; supporting,
pleading, or arguing in favor of the client
For Multiple Clients regarding:
1) Client rights
A. Maslows Hierarchy applies (e.g., more critically ill 2) Facility policy
clients will require more care to meet their 3) Treatment/care issues
physiologic/survival needs) 4) Personnel issues
B. Organizing multiple client assignments
1. Analyze and plan for entire shift.
2. Develop a working plan so that priorities get Delegation to Unlicensed Assistive
accomplished and all clients receive optimal care.
3. First consider schedules for nursing activities
Personnel (UAP)
(e.g., meds, treatments, VS, mealtimes, client A. Delegation is the transfer of authority to a
appointments, I&Os, etc.). competent individual to perform a selected
4. Then work in the nonscheduled activities that nursing task in a selected situation.
need to be accomplished to meet care plan goals 1. Based on principle of public safety.
(e.g., supporting family, teaching client, meeting 2. RN has ultimate accountability for the
with other departments about scheduling, provision and management of nursing care
writing care plan, discharge planning). (includes delegation decisions).
3. When done correctly, it allows more care to be
ASSIGNMENT METHODS provided in a given time period by distributing
the workload and allowing better use of the
FOR DELIVERY OF CARE RNs time.
B. Five Rights of Delegation
Principles 1. Right Task
a. Often defined by states Nurse Practice Act
A. Registered nurse (RN) is the decision maker/delegator

3
b. Facility policy
1. Assesses each client. Determines appropriate
plan of care.

UNIVERSAL PRINCIPLES OF NURSING CARE MANAGEMENT 119


53155_03_Ch03_p111-142.qxd 2/21/09 11:35 AM Page 120

c. Job description of UAP, or specific role d. Supervise or assign supervision to other


delineation for a specific UAP appropriate licensed nurses.
d. Tasks appropriate for consideration: e. Monitor performance, and get and provide
1) Repetitive custodial nature feedback as indicated (check intermittently).
2) Not require UAP to make clinical f. Intervene as needed.
judgment g. Provide education as needed.
3) Not require complex steps or decisions h. Ensure clear documentation.
4) Results predictable i. Evaluate the client outcome.
5) Potential risk is minimal j. Evaluate your delegation practice.
6) Uses standard unchanging procedure C. Other considerations
2. Right Circumstances 1. Plan and start delegating before you get too busy.
a. Assess the clients condition and stability. 2. The delegation relationship takes time to build.
b. Identify the environment/setting (e.g., ICU 3. Select the UAP for the task, if possible (e.g.,
vs. long-term care). one UAP might do best with a large, faster-
c. Identify the collective nursing care needs paced assignment, while another may do
of the whole assignment. better with clients who can benefit by a slower
d. Assess the clients plan of care and goals. conversational approach).
e. Provide the appropriate skill-mix and lines 4. Allow flexibility where possible.
of reporting. 5. Use positive feedback.
f. Provide the needed supplies and equipment. 6. Give credit.
g. Match complexity of the task with the
UAPs competence and level of
supervision available.
Admission of Client to Hospital
h. Identify any infection control or safety issues. A. Room assignment
3. Right Person 1. Check available data (e.g., diagnosis, age,
a. Organizations standards for competency of pertinent history)
UAPs. 2. Does client need to be close to nurses station
b. Instruct or assess the UAPs competence on for optimal monitoring?
a client-specific basis. 3. Does client need isolation or special
c. Perform UAP evaluations based on the precautions?
standards. 4. Who will be the clients roommate?
4. Right Direction/Communication 5. Consider the physical layout of available rooms
a. Communicate the task(s) clearly and on and bathrooms. What would be best for the
client-specific and UAP-specific bases. client based on his or her functional status?
b. Use oral and/or written vehicles to B. Perform a baseline admission assessment per
communicate, depending on the facility procedure.
circumstances. C. Obtain needed equipment (e.g., urinal, denture
c. Communicate specific information to be cup, etc.).
reported, specific data to collect, and time D. Explain and document the disposition of valuables
lines for reporting. per facility policy.
d. Communicate specific tasks to be E. Orient to facility/policies (e.g., visiting hours,
performed and any client-specific parking, telephone, chaplaincy services, TV,
instruction or limitations. mealtimes, electrical equipment, etc.).
e. Expected outcomes or potential F. Orient to unit (e.g., layout, lounges, smoking
complications and when to communicate policy, activities, menu selection, medication
this information. times, straight vs. prn orders, mealtimes, unit
f. What signs and symptoms to be alert for personnel, etc.).
and how to report it. G. Orient to room (e.g., roommate; bedside stand,
g. Communicate availability of support. table, and closet; call light, bathroom call system,
h. Verify understanding. bed operation, TV, telephone, etc.).
5. Right Supervision/Evaluation
a. Supervision may be provided by the
delegating nurse or other designated staff.
Caring for the Client
b. Supervising nurse must know the expected Who Leaves the Unit
method for supervision (direct or indirect),
A. Coordinate scheduling to consider clients
the competency of the UAP, nature of the
diagnosis, activity/test to be performed, and
delegated tasks, and the stability of the
clients other therapeutic goals.
client condition.
B. Prepare client physically and psychologically as
c. Ensure adequate time is allotted to providing
indicated.

3
needed supervision.

120 NCLEX-RN Review


53155_03_Ch03_p111-142.qxd 2/21/09 11:35 AM Page 121

C. Consider the clients condition; medication, diet, and 6. Ensure that client has needed prescriptions.
treatment regimes; as well as specific precautions 7. Provide written/audio/visual educational
and adjust the clients schedule as needed. materials at the level of the clients ability and
D. Communicate pertinent information to other appropriate community resource contact
departments/personnel. information.
8. Schedule or direct client to arrange for
Discharge of Client from the Hospital appropriate follow-up.
9. Communicate with individuals/agency(ies)
A. Discharge to home responsible for follow-up care.
1. Begin discharge plan on admission. B. Discharge of client to long-term care facility:
2. Teach client/significant other about disease communicate with facility nursing staff
process, needed precautions, restrictions, 1. Clients functional abilities and limitations
treatments, and medications. 2. Present medical regime and schedule
3. Assess and document knowledge of disease and 3. Mental and behavioral status
home-care regimen and ability to perform safely. 4. Family support/involvement
4. Make referrals as needed for added support 5. Nursing care plan and response
and care (e.g., community/home health nurses, 6. Existing advance directives
home health aide, community support groups, 7. Recent medication administration records
social worker, physical therapist, etc.). 8. History and physical
5. Arrange for client to obtain needed 9. Pertinent diagnostic reports
equipment/supplies (e.g., bedside commode, 10. Other: requirements per insurance
ostomy supplies, dressings, etc.).

Safety

FIRE SAFETY/PREPAREDNESS Table 3-3 Fire Hazards and Prevention


PRACTICES Fire Hazards Fire Prevention
A. Be aware of hazards and report immediately. Faulty electrical Report frayed or exposed electrical wires
B. Locate and remember: equipment Report sparks or excessive heat
1. Escape routes and wiring coming from electrical equipment
2. Fire drill procedures Overloaded Avoid overloaded circuits
3. Use of available equipment circuits Dont use adaptors or extension cords
Plugs that are Use only 3-pronged grounded plugs
a. Fire escapes
not properly Do not allow electrical equipment from
b. Fire doors grounded outside the institution to be used until it is
c. Fire alarms checked by the maintenance department
d. Fire sprinkler controls Clutter Avoid clutter
e. Fire extinguishers Unsafe practices No open flames or smoking in the area
f. Shut-off valves for O2 and/or medical air when O2 Remove flammable liquids from the area
4. Keep fire exits clear. in use Post Oxygen in Use signs as per
C. Fire safety institutional policy
1. Prevention is everyones responsibility. Secure O2 storage per institutional policy
2. Three elements needed for a fire to start Smoking Remove cigarettes and matches from room
Report suspicious odors of smoke or
a. Fuel: substance that will burn
burning immediately
b. Heat: flame or spark Control smoking practices per institutional
c. Oxygen: room air contains 21% O2 policy
3. See Table 3-3. Limit smoking to designated areas
D. In the event of a fire: No smoking in bed
1. Follow the RACE acronym: Directly supervise smoking of selected clients
R 5 Remove all persons in immediate danger Ensure use of safe ashtrays/metal receptacles
to safety Spontaneous Dispose of chemicals, rags, and combustible
A 5 Active alarm and have someone call 911 combustion substances in proper containers

UNIVERSAL PRINCIPLES OF NURSING CARE MANAGEMENT

3 121
53155_03_Ch03_p111-142.qxd 2/21/09 11:35 AM Page 122

C 5 Close doors to prevent spread of smoke PRINCIPLES AND INTERVENTIONS


and fire
E 5 Extinguish the fire using the PASS FOR SPECIFIC ASPECTS OF CARE
acronym:
P 5 Pull the pin Body Mechanics
A5 Aim on the base of fire
S 5 Squeeze the handle A. Safe and efficient use of appropriate muscle
S 5 Sweep from side to side groups to do the job
2. Shut off piped-in O2 and/or medical air. B. Principles for the safe movement of clients
3. Follow institutional policy concerning 1. Keep your back straight.
announcing the fire and location and notifying 2. Ensure a wide base of support (keep your feet
fire company. separated).
4. Avoid use of elevators. 3. Bend from the hips and knees (not the waist).
5. Follow institutional evacuation plan as 4. Use the major muscle groups (strongest).
needed. 5. Use your body weight to help push or pull.
6. Avoid twisting (pivot the whole body).
7. Hold heavy objects close to your body.
EQUIPMENT 8. Push or pull objects instead of lifting.
9. Ask for help as needed.
A. Follow facility procedure when using various 10. Synchronize efforts with client and other staff.
equipment. 11. Use turning or lifting sheets as needed.
B. Unfamiliar equipment 12. Use mechanical devices as needed.
1. Contact your staff development department or
supervisor for information. Transfer and Movement Principles
2. Read available manufacturers literature.
C. Suspected malfunction (i.e., equipment that and Techniques
does not do its task consistently or correctly, A. From bed to chair or wheelchair
makes unusual noises, or gives off an unusual odor 1. Identify clients strongest side.
or extreme temperature) 2. Place chair beside bed, on same side as clients
1. Dont try to repair. strongest side, so it faces the foot of bed.
2. Replace it immediately. Stabilize chair and lock wheels.
3. Contact maintenance department so that it can 3. Lower bed, lock wheels, and elevate head of bed.
be checked out safely and repaired. 4. If assistance is needed:
a. Place one arm under clients shoulders.
The other arm should be placed over and
RESTRAINTS around the knees.
b. Bring legs over the side of bed while
A. Physical restraints should be used only if raising the clients shoulders off the bed.
necessary to prevent injury to the client or c. Dangle client and watch for signs of fainting
others. or dizziness. (Stand in front of client for
1. Signed, dated, physicians order needs to be protection in case of balance problems.)
written specifying the form of restraint and a d. Protect paralyzed arm during transfer. (Use
time limit for restraint use. (At that time the sling or clothing for support.)
client will be reevaluated for restraint need to e. Place clients feet flat on the floor. (If client
determine if a less restrictive method is has a weak leg, use your leg and foot to
appropriate.) brace the weak foot and knee.)
2. Least restrictive form of restraint should be f. Face the client and grasp firmly by placing
used your arms under the armpits. Have client
a. Maintain functional abilities lean forward so that your control of the
b. Decrease risk of complications clients upper body is stabilized.
c. Minimize behavioral reaction g. Using a wide base of support and bending
3. Remove restraints for 10 min q2h for ROM, at your knees, coach the client to assist as
repositioning/ambulation, toileting, and much as possible by using verbal
preventative skin care. instruction and counting.
4. Document rationale for restraint, other h. Stand client (if weight bearing is
measures tried in lieu of restraint (e.g., permitted) by pivoting the feet, legs, and
distraction, family notification, environmental hips to a standing position.
modifications), client response, and i. Continue the slow pivotal movement until
preventative care. client is positioned over chair. Lower

3
client into chair.

122 NCLEX-RN Review


53155_03_Ch03_p111-142.qxd 2/21/09 11:35 AM Page 123

B. Log rolling
1. Performed when spinal column must be kept
straight (post-injury or surgery).
2. Two or more persons needed
a. Both staff should be on side opposite
where client is to be turned.
1) One staff places hands under clients
head and shoulders.
2) One staff places hands under clients
hips and legs.
3) Move client as a unit toward you.
4) Cross arms over chest and place pillow
between legs.
5) Raise side rail.
b. Both staff move to side of bed to which
client is being turned.
1) One staff should be positioned to keep
clients shoulders and hips straight.
2) One staff should be positioned to keep
thighs and lower legs straight.
3) At the same time the client is drawn
toward both staff in a single unified
motion. The clients head, spine, and
legs are kept in a straight position.
c. Position with pillows for support and raise
side rails.

Positioning of the Client


Figure 3-1 (A) Semi-Fowlers position, (B) high
A. General principles Fowlers position
1. Privacy/draping
2. Universal precautions as needed
3. Knowledge of clients condition when moving
client (e.g., paresis or paralysis of a limb; need 3. Supine (dorsal/horizontal recumbent)
to support joints or limbs in a specific manner; a. Client lies on back.
awareness of pressure points) b. Clients head and shoulders slightly
4. Good posture and body alignment elevated with pillow (modified per client
5. Use of added supports as needed (e.g., pillows, condition, physician order, or agency
wedge cushions, handrolls, foot boards) policy regarding spinal injury, surgery or
6. Comfort: reduce pressure and strain on body post spinal anesthesia)
parts c. Small pillow under lumbar curvature
7. Safety d. Prevent external rotation of legs with
8. Bed in a low position once repositioned supports placed laterally to trocanters
9. Access to personal items and care (e.g., call e. Knees slightly flexed
bell, drinking water, tissues, telephone, etc.) f. Prevent footdrop with foot board, rolled
10. Clients should change position fairly pillow or high top sneakers (depends on
frequently (at least every 2 hours). persistence of client condition)
B. Positions 4. Prone (see Figure 3-2)
1. Semi-Fowlers (see Figure 3-1A) a. Client lies on abdomen.
a. Backrest elevated at 458 angle b. Head turned to one side on small pillow or
b. Knees supported in slight flexion on flat surface.
c. Arms rest at sides c. Small pillow just below diaphragm to
2. High Fowlers (see Figure 3-1B) support lumbar curve, facilitate breathing,
a. Backrest elevated at 908 angle (right angle) and decrease pressure on female breasts.
b. Knees slightly flexed d. Pillow under lower legs to reduce plantar
c. Arms supported on pillows or bedside flexion and flex knees.
table e. May be modified in amputees where
d. Allows for good chest expansion in clients flexion of hips and knees may be
with cardiac or respiratory problems contraindicated.

UNIVERSAL PRINCIPLES OF NURSING CARE MANAGEMENT

3 123
53155_03_Ch03_p111-142.qxd 2/21/09 11:35 AM Page 124

Figure 3-2 Prone position Figure 3-3 Sims position

5. Trendelenburg
a. Client lies on back with head lower than
rest of body.
b. Enhances circulation to the heart and brain.
Sometimes used when shock is present.
c. In emergencies, the entire lower bed may
be elevated on shock blocks.
d. May be used for prolapsed cord outside of
the hospital.
6. Modified Trendelenburg
a. Client is positioned with legs elevated to an
angle of approximately 208, knees straight,
trunk horizontal, and head slightly elevated.
b. Used for persons in shock to improve cerebral Figure 3-4 The knee-chest position
circulation and venous return to the heart
without compromising respiration.
(Contraindicated when head injury is present.)
7. Lateral (side-lying)
a. Client lies on side.
b. Pillow under head to prevent lateral neck
flexion and fatigue.
c. Both arms are slightly flexed in front of the
body. Pillow under the upper arm and
shoulder provides support and permits
easier chest expansion.
d. Pillow under upper leg and thigh prevents
internal rotation and hip adduction.
e. Rolled pillow behind clients back.
8. Sims (semiprone; see Figure 3-3) Figure 3-5 Dorsal lithotomy position
a. Similar to lateral, but with weight
supported on anterior aspects of the ilium,
humerus, and clavicle.
b. Used for vaginal and rectal exams, enema 9. Knee-chest (see Figure 3-4)
administration, and drainage of oral a. Client first lies on abdomen with head
secretions from the unconscious client. turned to one side on a pillow.
Comfortable for the client in the last b. Arms flexed on either side of head.
trimester of pregnancy. c. Finally the client is assisted to flex and
c. Client placed on side (left side for enema or draw knees up to meet the chest.
rectal exam) with head turned to side on a d. Difficult position to be maintaineddo not
pillow. leave client alone. Used for rectal and
d. Lower arm is extended behind the body. vaginal exams.
e. Upper arm flexed in front of body and 10. Dorsal lithotomy (see Figure 3-5)
supported by a pillow. a. Used for female pelvic exam.
f. Upper leg is sharply flexed over pillow b. Have client void before assuming this

3
with the lower leg slightly bent. position.

124 NCLEX-RN Review


53155_03_Ch03_p111-142.qxd 2/21/09 11:35 AM Page 125

c. Client lies on back with the knees well 2. Alcohol or sponge bath (tepid solutions,
flexed and separated. 8581008 F)
d. Frequently stirrups are used. (Adjust for a. Alcohol bathcombination of alcohol and
proper feet and lower leg support.) water (alcohol has a drying effect on skin
e. If prolonged use of stirrups, be alert to used less frequently). Alcohol increases
signs of clot formation in the pelvis and heat loss by evaporation.
lower extremities. b. Sponge bathcool or tepid (not cold) water.
c. Frequent and regular VS monitoring (T, P,
R, and BP).
Immobility d. Large areas sponged at one time allowing for
A. Definition: inability to move in environment freely transfer of body heat to the cooling solution.
1. May be prescribed to limit movement of e. Wet cloths applied to forehead, ankles,
body/body part(s) as part of treatment/care wrists, armpits, and groin where blood
plan circulates close to skin surface.
a. Bed rest objectives may be: f. Identify temperature to cease treatment
1) Reduce physical activity due to potential for continued downward
2) Allow rest temperature drift.
3) Reduce oxygen needs 3. Discontinue systemic cold applications and
4) Allow to regain strength report and document findings if:
5) Prevent further injury a. Shivering occurs (this mechanism will
6) Promote healing raise body temperature);
7) Restrict movement of specific body b. Cyanosis of the lips or nails occurs; or
part(s) c. Accelerated weak pulse occurs.
2. May be related to physical inactivity, B. Local
cognitive, and/or emotional changes 1. Purposes
B. Conditions that may require bed rest include a. Control bleeding by constriction of blood
cardiovascular, neurological, musculoskeletal, vessels.
cancer, AIDS, etc. b. Reduce inflammation:
C. Factors affecting immobility 1) Inhibit swelling.
1. Length of immobility 2) Decrease pain.
2. Severity of illness or injury 3) Reduce loss of motion at site of
3. Premorbid physical condition inflammation.
4. Emotional state c. Control accumulation of fluid.
D. Hazards of immobility (see Table 3-4) d. Reduce cellular activity (e.g., check
bacterial growth in local infections).
e. Effective initial treatment after trauma
Cold Application (24-48 hours). This application of cold is
A. Systemic then frequently followed by a phase of
1. Lowers metabolic rate application of heat.
a. Client lies on top of one, or between two, 2. Ice caps or ice collars
cooling blankets. Blanket(s) are attached to a. Covered with cotton cloth, flannel, or
a machine that circulate(s) coolant towel to absorb moisture from
solution. condensation. Change as needed.
1) Follow agency policy/procedure for b. Not left on for longer than 1 hour.
care of client treated with hypothermia c. Cease treatment and report if client
blanket(s). complains of cold or numbness, or if area
2) Monitor VS (T, P, R, and BP) regularly appears mottled.
and frequently. 3. Cold compresses
3) Attention to skin hygiene and a. Use sterile technique for open wounds.
protection with oil as required. Check site of application after 510
4) Frequent repositioning and assessment minutes for signs of intolerance (cyanosis,
of body surface areas. blanching, mottling, maceration, or
5) Observe for signs of tissue damage and blisters).
frostbite (pale areas). b. Remove after prescribed treatment period
6) Assist client in basic needs (e.g., (usually 20 minutes).
hygiene, elimination, nutrition, etc.). C. Special considerations
7) Identify client temperature at which to 1. Elderly clients and clients with impaired
cease the treatment (temperature may circulation have decreased tolerance to cold.
continue to drift downward). Monitor 2. Moist application of cold penetrates better

3
VS frequently until stable for 72 hours. than dry application.

UNIVERSAL PRINCIPLES OF NURSING CARE MANAGEMENT 125


53155_03_Ch03_p111-142.qxd 2/21/09 11:35 AM Page 126

Table 3-4 Hazards of Immobility

Potential Negative Effects of Immobility Nursing Interventions


Cardiovascular:
Orthostatic hypotension:
Impaired ability to equalize blood supply upon assuming Monitor VS
an upright position (BP drop, weakness, dizziness, or Dangle clients legs 2-3 times/day, if appropriate
fainting) Tilt tables
Encourage progressive weight-bearing, as indicated
Monitor for change in lying and sitting/standing BP
Increased cardiac workload:
Blood volume redistributes and increases circulating Monitor tolerance for various ADLs
volume (increased heart rate) Monitor characteristics of pulses
Valsalva maneuver:
Holding breath and fixing thorax, breath forced against Teach to exhale rather than hold breath when moving in bed
closed glottis during movement Overhead trapeze for repositioning
Thrombus formation:
Venous stasis, external pressure against veins Proper positioning
Assess for Homans sign
Elastic stockings, sequential compression devices, etc.
Ensure adequate hydration
Anticoagulants

Respiratory:
Limited chest expansion Monitor respiratory rate and depth
Monitor for use of accessory muscles
Decreased movement and pooling of secretions Check breath sounds in all lobes and for degree of aeration
Teach to perform deep breathing and coughing exercises
Impaired oxygen exchange Assess for effective cough
Note any evidence of adventitious lung sounds

Metabolic:
Reduced metabolic rate (except with fever) Encourage to be up and about during day, if possible
Tissue atrophy and protein catabolism Provide diet with increased protein and calories
Nutritional supplements
Check weights
Bone demineralization Watch for peripheral edema
Fluid and electrolyte imbalances Monitor laboratory studies

Gastrointestinal:
Slower peristalsis (risk for constipation/nausea and vomiting, Monitor frequency and consistency of BMs
fecal impaction) Check for bowel sounds in all four quadrants of abdomen
Prevent or treat constipation
Assess for signs of fecal impaction

Urinary Elimination:
Stasis of urine (risk of infection) Monitor I&O
Assist client to empty bladder
Renal calculi Assess for signs of urinary tract infection and renal calculi

Musculoskeletal:
Decreased strength Consult PT and OT, as indicated and endurance
Rehab techniques as indicated
Muscle atrophy - Active and passive ROM
Contractures - Isokinetic/resistive
Osteoporosis - Stretch and flexibility
Change position at least q of 2 h
Monitor height over time
Restorative nursing care
Check ROM

3
(continues)

126 NCLEX-RN Review


53155_03_Ch03_p111-142.qxd 2/21/09 11:35 AM Page 127

Table 3-4 Hazards of Immobility (continued)

Pressure Ulcers:
Prolonged pressure on area disturbs blood supply and nutrition Monitor skin condition
to a body part Use pressure reduction/relieving devices
Position to avoid injury to tissues and promote lung expansion
Check intertriginous areas for accumulation of sweat and
loss of fluid
Psychological:
Depression, disorientation, social isolation, altered body concept, Provide education
anxiety, etc. Consult other interdisciplinary team members, as needed
Sleep disturbances- disrupted sleep and wake cycles Encourage participation as capable in ADLs
Provide emotional support
Create pleasant environment
Coordinate care to allow client to get through sleep cycles
Provide orienting materials (e.g., clocks, newspapers,
eyeglasses, hearing aids, etc.)

Application of External Heat blistering, maceration, pronounced pallor,


or if the client reports pain or discomfort.
A. Rationale e. Remove the device after 1525 minutes or
1. Relaxes muscles in spasm. as ordered/necessary.
2. Softens exudates for easy removal. D. Special considerations
3. Hastens healing due to vasodilation. 1. Moist heat penetrates deeper than dry and is
4. Localization of infection. Note: Do not usually better tolerated.
apply heat to the abdomen with suspected 2. The skin area involved may vary in any
appendicitis as it may precipitate individual depending on the number of heat
rupture. receptors present.
5. Hastens suppuration. 3. Heat is less tolerated in the very young,
6. Warms a body part. elderly, and clients with circulatory problems.
7. Reduces congestion of an underlying
organ. Asepsis
8. Increases peristalsis.
9. Reduces pressure from accumulated A. Defined as the absence of disease-producing
fluids. organisms.
10. Comforts and relaxes. B. Medical asepsis
B. Dry heat 1. Practices to reduce the number of
1. Hot water bottle/bag, electric heating pad, microorganisms after they leave the body or to
lamp, cradle, or aquamatic pad. reduce transmission.
2. Deeper tissue penetration modes: ultrasound, 2. Often referred to as clean technique.
and shortwave and microwave diathermy 3. Includes:
(administered by Licensed Physical a. Hand washing/decontamination
Therapist). b. Standard precautions
3. Follow agency policy for heat application c. Isolation technique (i.e., contact, droplet,
mode ordered: airborne)
a. Check temperature of water and machine d. Cleaning/disinfecting of equipment
setting carefully; C. Surgical asepsis
b. Assess site of application frequently for 1. Practices aimed at destroying pathological
signs of tissue damage or burns; and organisms before they enter the body through
c. Be alert to potential bleeding resulting an open wound.
from vasodilation. 2. Referred to as sterile technique.
C. Moist heat 3. Includes:
1. Soaks, compresses, hot packs a. Physical barriers: gloves, masks, gowns,
a. Follow agency policy. drapes, protective eyewear
b. Check temperature of application. b. High-risk procedures:
c. Use sterile technique for open wounds. 1) Catheter insertion
d. Assess skin condition after 5 minutes for 2) Surgical wound dressing changes

3
increased swelling, excessive redness, 3) Administration of injections

UNIVERSAL PRINCIPLES OF NURSING CARE MANAGEMENT 127


53155_03_Ch03_p111-142.qxd 2/21/09 11:35 AM Page 128

c. Associated with populations with high a. Cleanse immediately and apply protective
risk for infection. The clients in this barrier as indicated.
category are: 5. Avoid massage over bony prominences.
1) Transplant recipients (Massage around but not directly over
2) Burn victims pressure sites.)
3) Neonates 6. Change position frequently, every 15 minutes
4) Immunosuppressed/AIDS, clients to 2 hours, to decrease prolonged pressure.
with cancer receiving chemotherapy 7. Reduce friction and shearing (e.g., promote
4. Principles of surgical asepsis lifting rather than dragging).
a. Sterile field: area where sterile materials 8. Support surfaces:
for a sterile procedure are placed (e.g., a a. Pressure relieving: static surfaces (e.g., air,
table covered with sterile drape). gel, foam, or a combination)
b. Sterile field remains sterile throughout b. Pressure reducing: dynamic surfaces (e.g.,
procedure. low air-loss systems or air fluidized beds)
c. Movement in and around field must not 9. Positioning devices.
contaminate it. 10. Nutritional intake (especially calories,
d. Keep hands in front of you and above your protein, and fluids if not contraindicated).
waist (never reach across the field with Also vitamin A and C, iron, zinc, and
unsterile items). arginine supplemental products.
e. Barrier techniques (gown, gloves, masks, 11. ROM, ambulation, or activities as appropriate
and drapes are used as indicated to to promote increased circulation.
decrease transmission). 12. Avoid pressure from appliances and care
f. Edges of sterile containers are not sterile equipment.
once opened. D. Staging of pressure ulcers
g. Dry field is necessary to maintain sterility 1. Stage I
of field. a. Observable pressure-related alteration of
intact skin as compared to adjacent or
opposite area on body
UNIVERSAL/CLINICAL ISSUES b. May include changes in color (red, blue,
purple tones), temperature (warmth or
Pressure Ulcer (Dermal Ulcer, coolness), skin stiffness (hardness, edema)
and/or sensation (pain) (Temporary
Decubitis Ulcer) blanching from pressure can last up to
A. Any lesion caused by unrelieved pressure that 30 minutes.)
causes local interference with circulation and 2. Stage II
subsequent tissue damage. a. Partial thickness loss of skin involving
B. Risk factors epidermis and/or dermis.
1. Immobility (e.g., bed- and chair-bound clients b. Superficial breakdown characterized by
as well as those with impaired ability to blister, abrasion, or shallow crater. Wound
reposition themselves) base is pink and moist, painful, and free
2. Incontinence from necrosis.
3. Impaired nutritional status/intake 3. Stage III
4. Impaired level of consciousness a. Full thickness skin loss involving
5. Impaired physical condition (e.g., stability of subcutaneous damage or necrosis. May
condition, chronicity, and severity) extend to but not through underlying
6. Skin condition impaired (e.g., nourishment, fascia.
turgor, integrity) b. Infection is generally present.
7. Predisposing conditions (e.g., diabetes c. Characterized by deep crater or eschar.
mellitus, neuropathy, vascular disease, May include undermining and exudate.
anemia, cortisone therapy) Wound base is not usually painful.
C. General prevention, care, and treatment 4. Stage IV
1. Inspect skin and document status and a. Full thickness loss of skin with severe
interventions daily. destruction, tissue necrosis, or damage to
2. Cleanse when soiling occurs (e.g., avoid hot muscle, bone, or supporting structures
water, harsh or drying cleansing agents). (e.g., tendon or joint capsule).
3. Minimize dry skin (e.g., avoid cold or dry air b. Infection, undermining, and sinus tracts
and use moisturizers as needed). are frequently present.
4. Minimize moisture from irritating substances 5. If wound contains necrotic tissue or eschar,
(e.g., urine, feces, perspiration, wound accurate staging cannot be confirmed until

3
drainage). wound base is visible.

128 NCLEX-RN Review


53155_03_Ch03_p111-142.qxd 2/21/09 11:35 AM Page 129

E. Specific wound care treatments j. Skin sealant protects high-risk skin from
1. Goals moisture and/or chemical breakdown.
a. Support moist wound healing 4. Debridement: Removal of necrotic devitalized
b. Prevent or treat infection tissue (eschar or slough). Necrotic tissue
c. Avoid trauma of tissue and surrounding skin provides nutrients for bacterial growth and
d. Comfort needs to be removed for healing to occur.
2. Solutions a. Methods of debridement
a. Cleansing products 1) Enzymatic/chemical
b. Control of bacteria 2) Mechanical
3. Dressings or coverings 3) Surgical
a. Damp to dry dressing (e.g., gauze dressing 4) Physiologic/autolytic
put on damp and removed at tacky dry b. Be alert to bleeding and damage to adjacent
status) debrides slough and eschar. viable tissue.
1) If dries completely and adheres to 5. Miscellaneous
viable tissue, moisten dressing before a. Whirlpool: for cleansing.
removal. b. Hyperbaric O2: application of high O2
b. Nonadherent dressing impregnated with concentration for healing.
sodium chloride to draw in wound c. Electrical stimulation: stimulates
exudate and decrease bacteria. healing.
1) Change at least daily. d. Growth factor: cell growth stimulation.
c. Transparent films, semipermeable e. Vacuum-assisted closure (VAC): uses
membrane to promote moist healing by gas negative pressure.
exchange and prevention of bacterial and F. Documentation
fluid penetration. 1. Interventions and response to interventions
1) Change when seal is lost or excessive 2. Address:
amount of fluid collected underneath. a. Location of lesions
d. Hydrocolloid wafers contain water-loving b. Dimensions: measure and record size
colloids. Wound exudate mixes with wafer (length, width, and depth in cm)
to form a gel, moist environment, and 1) Measuring guides with concentric
nonsurgical debridement. circles available.
1) Wafers are occlusive and should not 2) Use sterile applicator to determine
be used on infected wounds. accurate depth.
e. Gels/hydrogels available in sheets or gels 3) Photographs: need clients written
and are nonadherent. They provide a moist permission.
environment and some absorption of c. Stage
bacteria and exudate from the wound. d. Undermining, pockets, or tracts (e.g.,
1) Not highly absorptive measuring underdetermined areas of a
a) Do not use on wounds with wound by length, width, depth)
copious exudate. e. Condition of tissue
b) Be alert to maceration of periwound 1) Granulation: red, moist, beefy.
areas. (Use moisture barriers.) 2) Epithelialized: new pink, shiny
f. Exudate absorptive dressings, beads, epidermis.
pastes, or powders, which, when mixed, 3) Necrotic tissue: avascular.
conform to the wound shape. Attracts a) Slough: yellow, green, gray,
debris, exudate, and bacteria via osmosis. brown.
1) Removed only by irrigation. Do not b) Eschar: hard, black, leathery.
use with deeply undermined wounds f. Drainage
or tracts. 1) Volume (scant, small, moderate,
g. Foams create a moist environment and copious, number of soaked dressings)
absorption. 2) Color
1) Nonadherent to wound. Many require 3) Consistency
a secondary dressing to secure. 4) Odor
h. Calcium alginate pads or ropes made from g. Periwound condition and wound margins
seaweed that convert to a firm substance (e.g., erythema, crepitus, induration,
when mixed with exudate. maceration, hematoma, desiccation,
1) Highly absorptive: will dry out blistering, denudation, pustule, tenderness,
wounds that have little exudate. temperature)
i. Moisture barrier (e.g., A & D ointment) h. Pain: related to procedures or constant,
protects high-risk skin from moisture and location, severity

3
breakdown.

UNIVERSAL PRINCIPLES OF NURSING CARE MANAGEMENT 129


53155_03_Ch03_p111-142.qxd 2/21/09 11:35 AM Page 130

Cultural Diversity in Health Practices

A. Culture: socially transmitted behavioral a. May choose medicine people or local


patterns, rules of conduct, arts, values, beliefs, healers for one problem and modern
customs, rituals, lifeways, and products of medicine for others.
existence that guide the worldview and 4. Seventh-Day Adventists
decision making a. Prohibit consumption of pork, shellfish,
B. Key component of the nursing assessment process alcohol, coffee, and tea (all meat and fish
in order to plan care in a manner that is sensitive avoided by the most devout).
and respectful of the individual needs of the 5. Hindus
client/significant others a. Prohibit consumption of beef (all meat
C. Cultural considerations and alcohol are avoided by the most
1. An individual may not necessarily identify devout).
strongly with a specific group just because b. Food is eaten with right hand (regarded as
he/she was born into it. clean).
2. An individual may identify with more than 6. Muslims
one group. a. Prohibit consumption of pork and pork
3. Clients may choose to practice selected products (e.g., lard) and alcohol.
customs of a group while not honoring others. Consumption of blood is forbidden;
4. How a client identifies with a culture, ethnic therefore, all meat and poultry are cooked
group, or religion may affect his/her health to well done.
practices and care up to the end of life. b. Bread is required with each meal (a gift
5. Rituals tend to become most important to from God).
individuals at times of significant life transitions. c. Food is eaten with the right hand (regarded
6. When ethical dilemmas arise, the leader of as clean).
the spiritual or cultural group might be d. Beverages are not consumed until after the
consulted. meal (some believe it unhealthy to eat and
D. Assessment drink at the same time). Some Muslims do
1. Does the client identify strongly with a not mix hot and cold foods at the same meal.
specific group or groups? e. Fasting as the start of a remedy: Prophet
2. What are the beliefs, customs, practices, and Mohammed said the stomach is the house
rules that are most important to the client? of every disease.
3. How can the health care team support the f. High concern for ingredients in mouthwash,
client and plan care that will address these nonhome-prepared food, medication (gelatin
needs? Are there special wishes/needs? capsules derived from pig, insulin, etc.).
4. Is the client part of a community, congregation, g. Special daily prayer times
or extended family structure? Does this play an 1) Need basin of water to wash before
important part in his/her life? praying
5. How do the identified culture(s) influence 2) Bed or chair facing Mecca
feelings about health and care? It is important 3) Read or listen to the Quran
to assess areas related to the situation (e.g., for h. Death is Gods will and foreordained. The
a client newly diagnosed with AIDS: What worldly life is preparation for eternal life.
gives the clients life meaning? What does pain i. Death rituals
mean to this client?). 1) Body washed three times by a Muslim
E. Selected examples of diverse cultures that might of the same gender and wrapped in
influence health care practices white
1. Jehovahs Witness 2) Buried as soon as possible in a brick-
a. Urge members to refuse blood transfusions. or cement-lined grave with the body
2. Christian Scientists, Orthodox Jews, Greeks, facing Mecca (no cremation and
and some Spanish-speaking societies may not typically no autopsy)
allow organ donation because of their belief 7. Roman Catholic
that the body needs to remain intact and whole. a. Anointing of the Sick (Last Rites) for the
3. Native Americans believe that health is seriously ill. This Sacrament of Healing
universal. It is a balancing of mind, body, discusses Gods grace and brings physical
spirit, and nature. and spiritual strength.

3 130 NCLEX-RN Review


53155_03_Ch03_p111-142.qxd 2/21/09 11:35 AM Page 131

8. Jewish 3. An adult is offered the opportunity to participate


a. Prohibit the consumption of pork and in research on a new therapy. The researcher asks
shellfish. the nurse to obtain the clients consent. What is
b. Kosher means properly preserved. the most appropriate action for the nurse to take?
1) Properly slaughtered, prepared, and 1. Be sure the client understands the project
served before signing the consent form.
2) Do not mix dairy products and meat in
the same meal 2. Read the consent form to the client and give
3) Plates and utensils for preparation and him/her an opportunity to ask questions.
serving of meat and dairy products 3. Refuse to be the one to obtain the clients
kept separate consent.
c. Death rituals 4. Give the form to the client and tell him/her to
1) Death is expected part of the life cycle; read it carefully before signing it.
after death the soul continues to flourish
2) Respectful to stay with a dying person 4. An adult has signed the consent form for a research
3) No autopsy, cremation, or embalming study but has changed her mind. The nurse tells
(keep the body whole, no desecration) the client that she has the right to change her mind
4) Burial as soon as possible based upon which of the following principles?
5) Shiva: Immediate family sits Shiva 1. Paternalism and justice.
for 7 days beginning with the burial;
2. Autonomy and informed consent.
the mourners do not work; they think
about the deceased; they cover the 3. Beneficence and double effect.
mirrors of the home, receive visitors, 4. Competence and right to know.
and conduct an evening service
9. Male circumcision is religiously practiced by 5. The nurse is preparing to move an adult who has
the followers of Judaism and Islam. right-sided paralysis from the bed into a
a. Ceremony, with great festivities, usually wheelchair. Which statement describes the best
conducted 8 days after birth for Jews and action for the nurse to take?
within the early few years of age among 1. Position the wheelchair on the left side of
Muslims the bed.
2. Keep the head of the bed elevated 108.
3. Protect the clients left arm with a sling
Sample Questions during the transfer.
4. Bend at the waist while helping the client
into a standing position.
1. A female client tells the nurse that she has tested
positive for HIV, but she does not want the nurse 6. An adult has experienced a cerebrovascular
to tell anyone. What is the best action for the accident that has resulted in right-sided weakness.
nurse to take? The nurse is preparing to move the client to the
right side of the bed so that he may then be turned
1. Document this information on the clients chart.
to his left side. What is an important principle
2. Tell the clients physician. when moving the client in bed?
3. Inform the health care team who will come in 1. To keep the feet close together.
contact with the client.
2. To bend from the waist.
4. Encourage the client to disclose this
3. To use body weight when moving objects.
information to her physician.
4. A twisting motion will save steps.
2. A young woman who has tested positive for HIV
tells her nurse that she has had many sexual 7. Which statement by the nurse best indicates a
partners. She tells her nurse that she believes correct understanding of log rolling when
that she will die soon. What would be the best moving a client?
response for the nurse to make? 1. One nurse may perform this task alone.
1. Where theres life theres hope. 2. Pillows are needed for positioning in order to
2. Would you like to talk to the nurse who provide support.
works with HIV-positive clients? 3. The legs should be moved before the head is
3. You are a long way from dying. moved.
4. Not everyone who is HIV positive will 4. Keeping the neck in a straight position is the

3
develop AIDS and die. primary concern.

UNIVERSAL PRINCIPLES OF NURSING CARE MANAGEMENT 131


53155_03_Ch03_p111-142.qxd 2/21/09 11:35 AM Page 132

8. The nurse is caring for a client who has a 13. An adult has developed a stage II pressure ulcer.
temperature of 1058F (40.58C). The physician He is scheduled to receive wet to dry dressings
orders the application of a cooling blanket. The every shift. What will the nurse state is the
nurse should know that which of the following purpose of receiving this type of dressing?
statements is true about the use of a cooling 1. Draw in wound exudate and decrease bacteria.
blanket? 2. Debride slough and eschar.
1. Cold application will increase the metabolic 3. Promote healing by gas exchange.
rate.
4. Promote a moist environment and soften
2. Vital signs should be monitored every exudate.
8 hours.
3. The client should remain in one position to 14. The nurse is performing a wound irrigation and
conserve energy. dressing change. Which action, if taken by the
4. Skin hygiene and protection of body surface nurse, would be a break in technique?
areas is essential. 1. Consistently facing the sterile field.
2. Washing hands before opening the sterile set.
9. Topical heat is ordered for all of the following
3. Opening the bottle of irrigating solution and
clients. The order should be questioned for
pouring directly into a container on the
which client?
sterile field.
1. A teenager who is active and rapidly
4. Opening the sterile set so that the initial flap
growing.
is opened away from the nurse.
2. A new mother who is breastfeeding.
3. A middle-aged adult with a cardiac 15. An adult is homeless and has gangrene on his
dysrhythmia. foot. The physician has recommended
4. An adult with arteriosclerosis obliterans. hospitalization and surgery. The client has
refused. The nurse knows which of the
10. The nurse is preparing to administer a following is true?
sponge bath to an infant with a high fever. 1. The client can be restrained if one physician
What should be included in the administration declares him incompetent.
of the bath?
2. The client can be hospitalized against his will.
1. Large amounts of alcohol to increase
3. The client cannot choose which treatment to
evaporation of heat.
refuse.
2. Adjustment of the water temperature to
4. The client may sign against medical advice
608708F.
(AMA).
3. Wet cloths applied to all areas where blood
circulates close to skin surfaces. 16. An adult has been medicated for her surgery.
4. Small areas of the body sponged at a time to The operating room (OR) nurse, when going
avoid rapid heat loss. through the clients chart, realizes that the
consent form has not been signed. Which of
11. The nurse is instructing the family of a the following is the best action for the nurse
homebound, bedridden client in the general to take?
prevention of pressure sores. What should be
1. Assume it is emergency surgery and the
included in the teaching?
consent is implied.
1. Promoting lifting rather than dragging when
2. Get the consent form and have the client sign it.
turning the client.
3. Tell the physician that the consent form is
2. Massaging directly over pressure sites.
not signed.
3. Changing the clients position every 4 hours.
4. Have a family member sign the consent form.
4. Cleaning soiled areas with hot water.
17. A licensed nurse in one state receives a job offer
12. A nurse is assessing a client with a Stage I as a nurse in an adjoining state. Which of the
pressure ulcer. Which finding would be following should the nurse do first?
noted? 1. Contact the first states board of nursing to
1. Superficial skin breakdown. cancel the 1st received license.
2. Deep pink, red, or mottled skin. 2. Contact the hospital the nurse wants to work
3. Subcutaneous damage or necrosis. in and ask them to contact its state board of

3
4. Damage to muscle or bone. nursing.

132 NCLEX-RN Review


53155_03_Ch03_p111-142.qxd 2/21/09 11:35 AM Page 133

3. Contact the new states board of nursing and 22. A nurse comes upon a motor vehicle accident
ask for reciprocity. when driving to work. The nurse administers care
4. Take the examining test in the new state. to the people involved. Under the Good
Samaritan Act, for what could the nurse be
18. An adult has just returned to the unit from liable?
surgery. The nurse transferred him to his bed but 1. For nothing, any action is covered.
did not put up the siderails. The client fell and 2. For gross negligence.
was injured. What kind of liability does the 3. For not providing the standard of care found
nurse have? in a hospital.
1. None. 4. For not stopping and offering care.
2. Negligence.
23. The nurse is supervising a newly trained
3. Intentional tort.
certified nurse aide (CNA). An adult has just
4. Assault and battery. arrived on the unit after surgery. Which of the
following is the most appropriate task for the
19. The nurse is in the hospitals public cafeteria nurse to delegate to the CNA?
and hears two nursing assistants talking about
1. Taking the clients vital signs while the nurse
the client in 406. They are using the clients
watches.
name and discussing intimate details about the
clients illness. Which of the following actions 2. Suctioning the clients tracheostomy and
is best for the nurse to take? reporting back to the nurse.
1. Go over and tell the nursing assistants that 3. Changing the clients postoperative (post-op)
their actions are inappropriate, especially in surgical dressing then describing it to the
a public place. nurse.
4. Testing urine with a reactant strip, and
2. Wait and tell the assistants later that they
recording and reporting the results.
were overheard discussing the client.
Otherwise, they might be embarrassed. 24. The nurse is making the assignment for the
3. Tell the nursing assistants supervisor about floor. There is one LPN and three RNs. Which
the incident. It is the supervisors of the following clients should the LPN be
responsibility to address the issue. assigned to?
4. Say nothing. It is not the nurses job or 1. A client who is intubated and a newly
responsibility for the assistants actions. diagnosed diabetic.
2. A recent ICU transfer and a person with
20. The nurse is about to medicate a woman for AIDS.
breast cancer lumpectomy. The client says, Ill 3. A client awaiting a nursing home bed and a
be glad when the surgery is over. It will eliminate client 1 day post-hernia repair.
all the cancer from my body. Which of the
4. A new admission for cholecystectomy and a
following is the best action for the nurse to take?
client 1 day post-op mastectomy.
1. Medicate the client and tell the physician.
2. Correct the clients misconceptions. 25. Which of the following clients should the nurse
3. Call the doctor without medicating the client. provide care to first?
4. Give the medication to the client and note 1. A client who needs her dressing changed.
her comment in the chart. 2. A client who needs to be suctioned.
3. A client who needs to be medicated for
21. A client on your medical-surgical unit has a incisional pain.
cousin who is a physician and wants to see the
4. A client who is incontinent and needs to be
chart. Which of the following is the best
cleaned.
response for the nurse to take?
1. Hand the cousin the clients chart to review. 26. Which of the following clients should the nurse
2. Ask the client to sign an authorization, and see first?
have someone review the chart with the cousin. 1. A client who has just returned from the OR.
3. Call the attending physician and have the 2. A client whose call light is not working.
doctor speak with the cousin. 3. A client with Alzheimers disease.
4. Tell the cousin that the request cannot be 4. A client who is receiving a heating pad

3
granted. treatment.

UNIVERSAL PRINCIPLES OF NURSING CARE MANAGEMENT 133


53155_03_Ch03_p111-142.qxd 2/21/09 11:35 AM Page 134

27. Four clients have signaled with their call bells 32. Which action by the CNA demonstrates the best
for the nurse. Who should be seen first? understanding of the use of restraints?
1. A client who needs to use the toilet. 1. Placing all clients in bed with the siderails up.
2. A client who does not have his glasses or 2. Applying a jacket restraint for the client who
hearing aid. pulls out IV lines.
3. A client who has just been given morphine. 3. Fastens the ends of the restraint(s) to the
4. A client in a geri chair with a restraint siderails.
vest on. 4. Fastens the restraints with a half bow knot to
an area the client cannot reach.
28. An adult who is in the terminal stages of AIDS is
admitted to the floor. During the admission 33. An adult has had both wrists restrained because
assessment, the nurse would ask her if she she is agitated and pulls out her IV lines. Which of
brought with her which of the following? the following would the nurse observe if the client
1. A will. is not suffering any ill effects from the restraints?
2. Funeral instructions. 1. She cannot reach her water pitcher.
3. An organ donation card. 2. She is sleeping with her hands by her side.
4. Health care proxy. 3. Her capillary refill is less than 2 seconds.
4. Her feet restraints are tied to the bed.
29. The nurse enters a room and finds a fire. Which
34. An adult is to be placed in a knee-chest position
is the best initial action?
for an exam by a new staff member. Which of the
1. Evacuate any people in the room, beginning following should the nurse observe?
with the most ambulatory and ending with
1. The arms are at the clients side.
the least mobile.
2. The head and upper chest are supported with
2. Activate the fire alarm or call the operator,
a pillow.
depending on the institutions system.
3. The lower legs are supported with a pillow.
3. Get a fire extinguisher and put out the fire.
4. The back supports the clients weight.
4. Close all the windows and doors, and turn off
any oxygen or electrical appliances. 35. An adult has been placed in Sims position by
the CNA. Which of the following should the
30. The nurse is unfamiliar with a new piece of OR nurse observe?
equipment that is scheduled to be used today.
1. The right arm is flat under the hip.
What is the best course of action?
2. The left leg is flexed at the hip and knee.
1. Ask another nurse for instructions on how to
use it. 3. The right leg is flexed at the hip and knee.
2. Wait until she has attended a class on using 4. A pillow under lower legs to reduce plantar
the equipment before using it. flexion.
3. Get another nurse who is familiar with the 36. The nurse is evaluating whether the CNAs are
equipment to operate it. correctly log rolling an adult in bed. Which action
4. Read the instructions provided with the by the CNA should be observed by the nurse?
equipment. 1. Use a draw sheet to aid the turning.
2. Do not place a pillow behind the head.
31. It is the first home care visit to an adult who is
in an electric hospital bed with an oxygen tank 3. Do not put a pillow between the clients legs.
behind it. The beds three-prong, grounded 4. Place the bed in the lowest position.
electric cord is connected to a frayed, two-prong
extension cord. What is the most appropriate 37. An adult is supine. Which of the following can the
action for the nurse to take? nurse do to prevent external rotation of the legs?
1. Turn off the oxygen supply, so as not to 1. Put a pillow under the clients lower legs.
accelerate any spark into a fire. 2. Place a pillow directly under the clients
2. Turn off the electricity, so as to maintain the knees.
oxygen supply to the client. 3. Use a trochanter roll alongside the clients
3. Tell the family to replace the extension cord upper thighs.
as soon as possible. 4. Lower the clients legs so that they are below

3
4. Unplug the bed after turning off the power. the hips.

134 NCLEX-RN Review


53155_03_Ch03_p111-142.qxd 2/21/09 11:35 AM Page 135

38. A C4 quadriplegic has slid down in the bed. 44. A man who has been in a motor vehicle
Which of the following is the best method for accident is going into shock. Before placing
the nurse to use to reposition him? the client in a modified Trendelenburg
1. One nurse lifting under his buttocks while he position, what problem would the nurse
uses the trapeze. assess for first?
2. Two people lifting him up in bed with a draw 1. Long bone fractures.
sheet. 2. Air embolus.
3. Two people log rolling the client from one 3. Head injury.
side to the other. 4. Thrombophlebitis.
4. One nurse lifting him under his shoulders
from behind. 45. The client has been placed in the Trendelenburg
position. The nurse knows the effects of this
39. A woman is to have a pelvic exam. Which of the position on the client include which of the
following should the nurse have the client do first? following?
1. Remove all her clothes and her socks and 1. Increased blood flow to the feet.
shoes. 2. Decreased blood pressure.
2. Go to the bathroom and void, saving a sample. 3. Increased pressure on the diaphragm.
3. Assume a lithotomy position on the exam table. 4. Decreased intracranial pressure.
4. Have the client sign the consent form.
46. What is the difference between the left lateral
40. An adult had a left, above-the-knee amputation and the Sims position?
2 weeks ago. For what reason should a nurse place 1. Sims position is semiprone, halfway
the client in a prone position three times a day? between lateral and prone.
1. Prevents pressure ulcers on the sacrum. 2. Lateral position places the clients weight
2. Helps the prosthesis to fit correctly. on the anterior upper chest and the left
3. Prevents flexion contractures. shoulder.
4. Allows better blood flow to the heart. 3. Sims position places the weight on the right
shoulder and hip.
41. An adult has a chest tube placed and is in a 4. Lateral position places the weight on the
semi-Fowlers position. Why would the nurse right hip and shoulder.
place the client in this position?
1. It is necessary to prevent pulmonary emboli. 47. A woman needs to be placed in position
2. It allows the nurse to have access to the chest for a pelvic exam. Which of the following
tube. describes how the nurse should position
the client?
3. It promotes comfort and drainage.
1. Supine with knees and hips bent and thighs
4. It is the only position a chest tube will work in.
abducted.
42. An adult is to have a rectal examination. In 2. Lying on her back, extremities moderately
which of the following positions should the flexed.
nurse position the client? 3. Kneeling with arms, upper chest, and head
1. Supine. resting on a pillow.
2. Prone. 4. Lying on her left side with right knee and
3. Sims. thigh flexed toward her chest.
4. Right lateral. 48. An adult is bedridden. The nurse knows which
of the following should be included in the plan
43. An adult has just returned to the unit from the
of care?
OR where he spent more than 2 hours in the
lithotomy position. Which of the following 1. Asking the client about comfort prior to
assessments should the nurse make because of positioning.
the positioning during the surgery? 2. Instituting a 4-hour turning schedule.
1. Lower extremity pulses, paresthesias, and pain. 3. Planning range of motion exercises every
2. The presence of bowel sounds. 2 hours.
3. Upper extremity pulses, paresthesias, and pain. 4. Using support devices to maintain
alignment.

3
4. Ability to walk.

UNIVERSAL PRINCIPLES OF NURSING CARE MANAGEMENT 135


53155_03_Ch03_p111-142.qxd 2/21/09 11:35 AM Page 136

49. The nurse of a bedridden woman is evaluating 54. Which of the following techniques would the
whether the family members understand how to nurse in a nursing home use to transfer a C4
position the client correctly. Which of the quadriplegic from bed to wheelchair?
following should the nurse observe? 1. One nurse dangling the client, then using a
1. Lower arm and leg are always supported in transfer belt.
the lateral positions. 2. Two people, one at the clients knees, the
2. The extremities should always be extended to other under his arms.
prevent contractures. 3. Two nurses using a mechanical lifting device
3. The spine should have maximal lordosis in (Hoya).
almost all positions. 4. Two nurses, one on either side, lifting the
4. The family should change the position at client with a sheet.
least every 2 hours.
55. The nurse will be dangling an adult prior to
50. A victim of a motor vehicle accident is brought transferring her from the bed to a wheelchair.
to the emergency room via ambulance in Which of the following actions is essential for
hypovolemic shock. When placing the client in a the nurse to make before moving the client?
modified Trendelenburg position, how will the 1. Assess blood pressure and heart rate.
nurse place the client?
2. Ensuring that the bed is in the highest position.
1. Legs out straight and elevated
3. Assessing the clients height and range of
approximately 208.
motion.
2. Supine, with the head of the bed lowered.
4. Enlisting the help of another nurse or a CNA.
3. Prone, with the head of the bed elevated.
4. Supine, tilting the bed so the head is above 56. An adult has just been admitted for acute asthma
the heart. exacerbation and placed in a high Fowlers
position. For what reason does the nurse know
51. A bedridden woman is positioned on her right that this is the best position?
side. There is a pillow beneath her head. Her
1. Facilitates maximal ventilation.
right arm is extended near her hip. Her left leg is
extended and parallel with the right leg. Which 2. Is required for the aerosol treatments to work.
of the following is correct? 3. Allows for chest physiotherapy.
1. The clients right leg should be flexed at the 4. Is the position for the chest X-ray.
hip and knee.
57. An older adult is to go home with her family.
2. The clients right arm should be flexed at the
The nurse is evaluating that the family members
shoulder and elbow.
can correctly move the client from the bed to a
3. There should not be a pillow under her head. chair. Which of the following should be seen?
4. She should be semiprone with the weight on 1. The transfer belt is placed loosely around the
her upper chest. waist.
52. The nurse uses a wide stance when moving a 2. There is no pause while the client is standing.
heavy box of supplies. Which of the following is 3. The family member leans forward from the
the best reason the nurse would do this? waist.
1. Avoids back strain. 4. The client and family member have one foot
2. Contracts the muscles. slightly in front of the other.
3. Lowers the center of gravity.
58. An adult suffered a stroke and has right-sided
4. Increases stability. hemiparesis. The nurse is going to transfer her
53. A woman who is brought in after a motor from bed to wheelchair. Which of the following
vehicle accident has suffered a head injury and is the best method?
possible spinal injury. What action should the 1. Have the client put her arms around the
nurse perform when moving her from the nurses neck.
stretcher to the bed? 2. Position the wheelchair closer to the weaker
1. Have the client move segmentally. foot.
2. Sit the woman up and transfer her to the bed. 3. Place the wheelchair about a foot away from
3. Move the woman with a draw sheet. the bed on the right side.
4. Log roll the client. 4. Put the wheelchair at a 458 angle to the bed

3
on the left side.

136 NCLEX-RN Review


53155_03_Ch03_p111-142.qxd 2/21/09 11:35 AM Page 137

59. The nurse knows which of the following is the indicates that the daughters technique is done
proper technique for medical asepsis? correctly?
1. Gloving for all client contact. 1. She uses only sterile gloves to remove the old
2. Changing hospital linen weekly. dressing.
3. Using your hands to turn off the faucet after 2. She irrigates the wound from the bottom up.
handwashing. 3. She places the forceps used to remove the old
4. Gowning to care for a 1-year-old child with dressing on the sterile field.
infectious diarrhea. 4. She washes her hands before each gloving
and after the procedure is done.
60. The nurse is conducting a class on aseptic
technique and standard precautions. Which of 64. A woman is transferred to a skilled nursing
the following statements is correct and should facility from the hospital because she is unable
be included in the discussion? to ambulate due to a left femoral fracture. Which
1. Standard precautions destroy the number of client description gives a greater risk factor for
potentially infectious agents. developing a pressure ulcer?
2. Medical asepsis is designed to decrease 1. 5 ft 4 in tall, 130 lb, and eats more than half
exposure to bloodborne pathogens. of most meals.
3. Medical asepsis is designed to confine 2. Apathetic but oriented to person, place, and time.
microorganisms to a specific area, limiting 3. Slightly limited mobility and needs
the number, growth, and transmission of assistance to move from bed to chair.
microorganisms. 4. Good skin turgor, no edema, and her
4. The term standard precautions is capillary refill is less than 3 seconds.
synonymous with disease or category-specific
isolation precautions. 65. An elderly male client is transferred to a skilled
nursing facility from the hospital because he is
61. The nurse is to open a sterile package from unable to ambulate due to a left femoral fracture.
central supply. Which is the correct direction to When doing a skin assessment, the nurse notices
open the first flap? a 3-cm, round area partial thickness skin loss
that looks like a blister on the clients sacrum.
1. Toward the nurse.
Which stage is apparent?
2. Away from the nurse.
1. Stage I pressure ulcer.
3. To the nurses left or right.
2. Stage II pressure ulcer.
4. It does not matter as long as the nurse only
3. Stage III pressure ulcer.
touches the outside edge.
4. Stage IV pressure ulcer.
62. For which procedure would the nurse use
66. When planning for the care of a client with a
aseptic technique and which would require the
pressure ulcer on the sacrum, the nurse would
nurse to use sterile technique?
include which of the following?
1. Aseptic technique for changing the clients
1. Positioning the client with a donut around
linen and sterile technique for placing a
the area to relieve pressure on the ulcer.
central line.
2. Massaging the sacrum, concentrating on the
2. Aseptic technique for urinary catheterization
bony prominences and reddened areas.
in the hospital and sterile technique for
cleaning surgical wounds. 3. Using a heat lamp twice a day to dry the
wound.
3. Aseptic technique for a spinal tap and sterile
technique for surgery. 4. Having a pressure-relieving device such as an
air mattress or gel flotation pad.
4. Aseptic technique for food preparation
and sterile technique for starting an 67. The nurse is to apply a dressing to a stage II
IV line. pressure ulcer. Which of the following dressings
is best?
63. An adult has a draining pressure ulcer on
1. Dry gauze dressing.
her sacrum and is to be discharged to her
daughters care. The nurse has taught the 2. Wet gauze dressing.
clients daughter to perform dressing 3. Wet to dry dressing.
changes. Which observation by the nurse 4. Moisture-vapor permeable dressing.

UNIVERSAL PRINCIPLES OF NURSING CARE MANAGEMENT

3 137
53155_03_Ch03_p111-142.qxd 2/21/09 11:35 AM Page 138

68. A client with a hip fracture has a sacral pressure 3. Throughout the treatment, the water remains
ulcer. Which of the following would indicate the at approximately the same temperature.
best response to treatment? 4. The clients baseline and after-treatment
1. The clients nutritional status including: temperature.
adequate protein; carbohydrates; fats;
vitamins A, B, C, and K; and minerals, 73. An adult has chronic lower back pain and
including copper, iron, and zinc. receives hot packs three times a week. The nurse
knows the treatment is given for which of the
2. The clients skin status, including length,
following reasons?
width, depth, condition of the wound
margins, and stage of the ulcer as well as the 1. To help remove debris from the wound.
integrity of the surrounding skin. 2. To keep the client warm and raise his
3. Increased mobility including the ability to temperature.
reposition self in bed or wheelchair and 3. To improve the clients general circulation.
walking with assistance. 4. To relieve muscle spasm and promote muscle
4. Absence of clinical signs of infection relaxation.
including redness, warmth, swelling, pain,
odor, and exudate. 74. While giving an adult a tepid sponge bath to
reduce his temperature, the nurse notes that the
69. An adult who has a disorder of the hypothalamus client is shivering. How does the nurse interpret
is on a hypothermia blanket. The nurse should this action?
make which of the following assessments? 1. Sponge bath is being given too slowly.
1. Document the clients ability to sweat. 2. Client has a decreased metabolic demand.
2. Ensure the clients skin is warm and dry. 3. Body is trying to warm itself.
3. Record baseline vital signs, neurologic status, 4. Temperature of the water is below 908F (328C).
and skin integrity.
4. Confirm that the client is alert and oriented. 75. A caregiver is giving a tepid sponge bath to her
invalid mother who has a fever. When
70. The nurse notices that a Jewish client did not eat evaluating the caregiver to ensure the procedure
any of their food on the meal tray. What would is being given correctly, the nurse would note
be the nurses first best action? the caregiver performing which of the following?
1. Request the clients family to bring food in for 1. Tests the water temperature on the inside of
the client. her wrist.
2. Request a kosher meal from the dietary 2. Rubs each area with the wet sponge.
department. 3. Sponges one part of the body, and then another.
3. Instruct the client that food will facilitate the 4. Rubs her mothers skin dry after each area is
healing process. sponged.
4. Ask the client why the food has not been eaten.
76. An adult is to have a tepid sponge bath to lower
71. The physicians orders for an adult include his fever. What temperature should the nurse
warm compresses to the left leg three times a make the water?
day for treatment of an open wound. What 1. 658F (1888C).
action will the nurse perform? 2. 908F (328C).
1. Use medical aseptic techniques throughout 3. 1108F (438C).
the procedure.
4. 1058F (40.58C).
2. Wet the compress and apply it directly to the area.
3. Place both a dry covering and waterproof 77. A man has sprained his ankle. Why would the
material over the compress. nurse apply cold therapy to the injured area?
4. Remove the compress after about 5 minutes. 1. Reduce the bodys temperature.
2. Increase circulation to the area.
72. An adult is receiving a hot soak to her right arm.
3. Aid in reabsorbing the edema.
What assessment will the nurse make?
4. Relieve pain and control bleeding.
1. The water temperature at the start of the
treatment is 1208F (488C). 78. An adult is going home from the emergency
2. That the water basin is placed at shoulder room with directions to apply a cold pack to his

3
height. ankle sprain. He asks how he will know if the

138 NCLEX-RN Review


53155_03_Ch03_p111-142.qxd 2/21/09 11:35 AM Page 139

cold pack has worked. What information would 2. 2. This provides the client with expert care.
the nurse provide to the client? Standard 5b provides for client participation
1. After the first application, the swelling will in gaining knowledge and for promotion of
be decreased. health.
2. He will notice the red-blue bruises will turn 3. 3. Nurses cannot obtain consent. They may legally
purple. witness consent to medical procedures. When the
3. There should be less pain after applying the consent is for a research study, the research team
cold pack. is responsible for obtaining consent.
4. That the skin will be blanched and numb
afterward. 4. 2. Autonomy is the ethical right to decide what
treatment you will or will not receive. Informed
79. The nurse is caring for a client who has consent can be withdrawn; it includes the right
recently immigrated from India. Which action to know and competence.
is most appropriate when developing the
nursing care plan? 5. 1. Place the wheelchair beside the bed, on the
1. Ask the client if any special needs are clients strongest side, so that it faces the foot of
present. the bed.
2. Order a diet with no pork products. 6. 3. Objects should be pushed or pulled instead of
3. Assign the client to an east-facing room. lifted. Using the body weight to push or pull
4. Perform a cultural needs assessment. prevents strain to muscles and joints.

80. An unconscious adult is admitted to the 7. 2. A pillow should be placed between the knees/
emergency department in hypovolemic shock. legs for support while the client is being turned.
The clients spouse says that the client is a
Jehovahs Witness and should not receive 8. 4. Assessment of the skin, protection of the skin
a blood transfusion. The physician orders a surfaces with oil, and repositioning are all vital
transfusion. What should the nurse do? to prevent skin breakdown.
1. Inform the physician of the familys request 9. 4. Heat is not well tolerated in clients with
and encourage exploration of other volume circulatory impairment. If topical heat application
expander options. is to be carried out in a client with circulatory
2. Call the hospital attorney to get an impairment, the nurse should assess the site
authorization to administer the transfusion. frequently for signs of tissue damage.
3. Discuss the urgent need for a transfusion
with the clients spouse. 10. 3. Wet cloths should be applied to forehead, ankles,
4. Give the emergency transfusion as ordered. wrists, axilla, and groin. These are the areas where
blood circulates closest to the skin surface.
81. A client of the Muslim faith is admitted with
insulin-dependent diabetes mellitus and 11. 1. Promoting lifting rather than dragging when
pneumonia. Which aspects of the clients care turning or moving the client will reduce friction
would be of greatest concern to the nurse? Select and shearing. This will assist in preventing
all that apply. pressure sores.
1. Well-done roast beef on the lunch tray. 12. 2. Stage I pressure ulcers show discoloration of
2. Order for porcine insulin. skin to a deep pink, red, or mottled appearance.
3. Chicken for Fridays meal.
13. 2. In a wet to dry dressing, the wet gauze dressing
4. Medication in a capsule.
either covers the wound or is packed into the
5. Elixir of terpin hydrate. wound and covered with a dry dressing. The dry
layer creates a wick and pulls moisture (drainage)
from the wound, debriding slough and bacteria.

Answers and Rationales 14. 3. After opening a sterile bottle, the edge of the
bottle is considered to be contaminated. The
nurse should pour a little solution out first to
1. 4. A nurse is legally obliged to protect a clients wash away organisms on the lip of the opening
right to privacy. The second point in the ANA and then pour from the same side of the bottle
Code is the ethical obligation. into the sterile container on the sterile field.

UNIVERSAL PRINCIPLES OF NURSING CARE MANAGEMENT

3 139
53155_03_Ch03_p111-142.qxd 2/21/09 11:35 AM Page 140

15. 4. A competent client may decide which 30. 3. Only those with knowledge of the equipment
treatments and procedures to accept or refuse. should operate it.

16. 3. It is the physicians responsibility to obtain 31. 4. Because any electrical (or gas) appliance is a
the consent and to ensure that the signer is hazard around oxygen, it is better to unplug the
competent. A medicated client generally is not dangerous cord after turning off the power.
deemed competent and the surgery may have to
be postponed. 32. 4. The half bow knot is a secure knot that will
not loosen but can be easily released by the
17. 3. Endorsement (reciprocity) from one state to nurse in an emergency.
another is usually done when the nurse is licensed.
33. 3. A normal capillary refill is less than 3 seconds,
18. 2. The nurse has been negligent and can be liable which would indicate good circulation.
for malpractice. Answers 1 and 2 are expected, not ill
effects.
19. 1. The client has a right to confidentiality and her
case should not be discussed in a public place. 34. 2. A pillow can be placed under the head or
chest.
20. 3. The client does not clearly understand the
procedure. Medicating the client can cloud her 35. 3. The correct position is with the right leg
judgment and should be withheld. The doctor is flexed, left arm extended at side, right arm and
the person to clarify the misconceptions. head on pillow.

21. 2. The client must agree to and sign an 36. 1. A draw sheet helps to maintain tension along
authorization before others can review the chart, the back and allows the body to be turned as one.
including insurance companies. Most institutions
require someone on staff to review the chart with 37. 3. A blanket roll along the side of the hips
the client or client representative. down to the midthighs helps to prevent external
rotation.
22. 2. Actions that a reasonable, prudent person with
the same level of skill and training would have 38. 2. A draw sheet is the easiest and most effective
provided are covered, but gross negligence is not. method to lift a quadriplegic client up in bed.

23. 4. Testing urine via reactant strips (Dip stix) and 39. 2. The client should have an empty bladder,
recording the results is usually within the scope reserving a sample for analysis if needed.
of a CNAs training. The CNA should also report
40. 3. Flexion contractures can be prevented by
the results to the nurse, especially if they are
placing the client in a prone position and by
abnormal.
exercising.
24. 3. These clients are the least sick and require the
41. 3. This position facilitates drainage and is
least amount of highly skilled nursing care.
generally most comfortable.
25. 2. Any client with a potential compromise of the
42. 3. The rectum is easily accessed when the hip is
airway should be dealt with first.
bent at a right angle.
26. 1. A client who has just returned from the OR is
43. 1. The lithotomy position places pressure on the
at highest risk for potential problems.
nerves and blood vessels of the legs.
27. 3. An adverse reaction to any drug can be life-
44. 3. Head injuries and chest injuries are
threatening and should be dealt with first.
contraindications for the Trendelenburg
28. 4. A living will, durable power of attorney for position.
health care, or a health care proxy is an
45. 3. The chest cavity is pushed by the pressure
important part of an admission assessment,
from the abdominal contents.
especially for a terminally ill client.
46. 1. The Sims position is halfway between the left
29. 1. Rescue and evacuate any people in the room
lateral position and the prone position.
first. Begin with those who are able to walk, then
those in wheelchairs, finally those who are 47. 1. The dorsal lithotomy position is used for most
nonambulatory in stretchers or beds.

3
pelvic exams.

140 NCLEX-RN Review


53155_03_Ch03_p111-142.qxd 2/21/09 11:35 AM Page 141

48. 4. Support devices such as pillows, special to sterile gloves, and after the procedure. This
mattresses, trochanter rolls, and foot boards prevents the spread of microorganisms.
help to maintain alignment and prevent
contractures. 64. 3. The fact that the client is chair-bound has the
greatest impact on her developing pressure
49. 4. Position changes should occur at least every ulcers.
2 hours, more often if needed.
65. 2. A stage II pressure ulcer may look like a
50. 1. The modified Trendelenburg position raises blister, abrasion, or shallow crater and only
the legs only. involve a partial thickness skin loss of the
epidermis and/or dermis.
51. 2. The lower arm should be flexed, so the body
does not rest on it. 66. 4. Any supportive device that protects bony
prominences aids in relieving pressure. This
52. 4. The greater the stability, the less chance of can include gel flotation devices, sheepskins,
injury. When increasing the base of support, the alternating pressure mattresses, and various air
nurse helps to maintain balance. loss beds.

53. 4. Log rolling a client would protect the spinal 67. 4. Moisture-vapor permeable dressings help
column and keep the body in alignment. stage II ulcers heal faster than saline
dressings.
54. 3. A mechanical lifting device (Hoya, Hoyer)
helps to transfer clients and prevents back injury 68. 2. The best clinical indicator of healing is
to the nurses. observation of the skin and evaluation of the
pressure ulcer.
55. 1. The client may experience a drop in blood
pressure and should be assessed before and 69. 3. Baseline vital sign assessment is necessary to
after dangling, especially if standing will be document against those taken during and after
included. the treatment.

56. 1. A high Fowlers position allows maximal 70. 4. Assessment should be performed first to
chest expansion and decreases hypoxia. determine why the client is not eating, which
may be due to illness, medication, or cultural
57. 4. Both the family member and the client beliefs.
should have one foot slightly in front of the
71. 3. The layers act as insulators and prevent moisture
other. This allows for a greater base of support
loss. Some nurses prefer placing the waterproof
and helps when rocking to achieve a standing
layer next to the compress and then covering with a
position.
dry cover, whereas others reverse the order, putting
58. 4. This position is best for clients who have the waterproof layer on the outside.
difficulty walking. The client can pivot into the
72. 3. The nurse should check the temperature every
chair and lessen the amount of body rotation.
5 minutes or so, and replace some of the water
The chairs should be on the strong side.
with a hotter solution. Care should be taken to
59. 4. Gowns should be worn when the nurses stir the basin while adding the additional water
clothing is likely to be soiled by infected so as not to burn the client.
material.
73. 4. Most people with chronic lower back pain
60. 3. Medical asepsis should be practiced everywhere. find relief with applications of heat.
It includes such things as handwashing.
74. 3. Shivering indicates that the body is trying to
61. 2. This allows for the least possible potential for warm itself and conserve heat.
contamination while opening the package. 75. 3. Each area is sponged slowly and gently. The
face and forehead, the neck, arms, and legs for
62. 1. Changing linen should be done with aseptic
35 minutes, and the back for 10 minutes.
technique, whereas putting in central lines
requires sterile technique. 76. 2. Unlike a cooling sponge bath where the
temperature begins at this point and gradually is
63. 4. Handwashing should occur before donning the

3
lowered to 658F (188C) at the end, this is the
nonsterile gloves, when changing from nonsterile

UNIVERSAL PRINCIPLES OF NURSING CARE MANAGEMENT 141


53155_03_Ch03_p111-142.qxd 2/21/09 11:35 AM Page 142

temperature that the water begins and ends for a REFERENCES AND SUGGESTED READINGS
tepid sponge bath.
Daniels, R., Nosek, L., & Nicoll, L. (2007). Contemporary
77. 4. Cold will produce an anesthetic effect and medical-surgical nursing. Clifton Park, NY: Delmar Cengage
help to reduce pain as well as control bleeding Learning.
Delane, S., & Ladner, P. (2006). Fundamentals of nursing 3E.
by constricting blood vessels.
Clifton Park, NY: Delmar Cengage Learning.
Federal requirements for Medicare/Medicaid interpretive
78. 3. Cold produces an anesthetic effect and can
guidelines. (2003). Washington, DC: Department of Health
relieve pain. and Human Services, Health Care Financing
Administration.
79. 4. The nurse should perform a cultural needs Maslow, A. H. (1970). Motivation and personality (2nd ed.).
assessment. Just because the client is a recent New York: Harper & Row.
immigrant from India does not mean that the Mazanec, P., & Tyler, M. (2003). Cultural considerations in end-
client belongs to a particular religion. of-life care. American Journal of Nursing, 103(3), 5059.
McNeill, L., & Schanne, L. (2003, May). Combating the effects of
80. 1. The clients next of kin has stated that the immobility. Nursing Spectrum, 22.
client should not get a transfusion. Jehovahs National Pressure Ulcer Advisory Panel Fifth National
Conference. (2007). Washington, D.C: Author.
Witnesses prohibit blood transfusions. The
Nursesbooks.org. American Nurses Association (2004). Nursing:
clients family has a right to refuse the Scope and Standards of Practice. Silver Spring MD: American
treatment. There are other volume expanders Nurses Association.
that could be tried. Purnell, L., & Paulanka, B. (2003). Transcultural health care.
(2nd, ed) Philadelphia: F. A. Davis.
81. 2, 4, 5. Muslims do not put pork products in
their bodies. Porcine insulin is a pork product.
Gelatin capsules may come from pork. An elixir
is made of alcohol. A devout Muslim does not
drink alcohol.

3 142 NCLEX-RN Review


53155_04_Ch04a_p143-263.qxd 2/26/09 6:50 PM Page 143

U N I T 4

A D U LT N U R S I N G

Nursing care of the adult client in todays changing health care UNIT OUTLINE
environment is a challenge to the skill and knowledge of the 144 Multisystem Stressors
professional nurse. Holistic care requires that nurses not only meet 173 Aging
a clients physical needs through technical skills and sound clinical 182 Perioperative Nursing
judgment, they also must be aware of a clients psychosocial 191 Oncologic Nursing
needs. The role of client advocate puts the nurse in a unique 199 The Neurosensory System
position to help clients achieve the highest level of wellness. 236 The Cardiovascular System
This unit presents a comprehensive review of nursing care of 264 The Hematologic System
adult clients with specific health problems. It begins with a 283 The Respiratory System
section on multisystem stressors (such as infection, pain, and 307 The Gastrointestinal System
surgery). These stressors are common to many areas of nursing 341 The Genitourinary System
practice and may be applied to clients with various levels of 362 The Musculoskeletal System
health care needs. Issues related to aging are also presented. 382 The Endocrine System
407 The Integumentary System
The unit is further divided according to specific body systems.
414 Complementary and
For each system there is a review of anatomy and physiology.
Alternative Medicine (CAM)
Each step of the nursing process (assessment, analysis,
planning, intervention, and evaluation) is then reviewed for the
system, followed by consideration of the major health problems
of that system. A discussion of complementary and alternative
therapies is also included at the end of this unit. Congenital
disorders will be discussed in Unit 5.

143
53155_04_Ch04a_p143-263.qxd 2/26/09 6:50 PM Page 144

Multisystem Stressors

STRESS AND ADAPTATION 4. Use relaxation methods and techniques: such


as deep breathing, guided imagery, progressive
muscle relaxation, relaxation response,
Definitions meditation, yoga, biofeedback.
A. Stress: tension resulting from changes in the 5. Engage in a social support system.
internal or external environment; either
physiologic, psychologic, developmental, or social
B. Stressors: agents or forces threatening an INFLAMMATORY RESPONSE
individuals ability to meet his or her needs
C. Adaptation: an individuals (or the bodys) reaction A reaction of the body at the cellular level in response
to and attempt to deal with stress to injury or noxious stimuli.

General Characteristics Causes


A. A certain amount of stress is necessary for life and A. Physical irritants (e.g., trauma or a foreign body)
growth, but excessive and continuous stress can be B. Chemical irritants (e.g., strong acids or alkalis)
detrimental. C. Microorganisms (e.g., bacteria and viruses)
B. Success of adaptation depends on perception of
stressor(s), the individuals coping mechanisms, Components
and biologic adaptive resources.
C. Types of stressors: physical, chemical, A. Vascular response: transitory period of localized
microbiologic, psychologic, social, and age-related vasoconstriction, followed by vasodilation,
growth and development. increased capillary permeability, and blood stasis
B. Formation of inflammatory exudate
1. Composition: water, colloids, ions, and
General Adaptation Syndrome defensive cells
Response to Stress 2. Functions: dilution of toxins, transportation of
nutrients to area of injury for tissue repair,
A. Caused by release of certain adaptive hormones transportation of protective cells that
B. Three stages phagocytize and destroy bacteria
1. Alarm reaction C. Defense cell response: migration of leukocytes to
a. Sympathetic nervous system is activated affected area for phagocytosis of foreign bodies and
(fight-or-flight response) dead cells
b. Results in increased heart rate, blood D. Healing: resolution of inflammation and
pressure, and respirations; dilated pupils; regeneration of tissue or replacement with scar
increased state of alertness; increased tissue
blood sugar and coagulability; increased
tension of skeletal muscles
2. Resistance: body adapts to stressor; uses Assessment Findings
physical, physiologic, and psychologic coping A. Local: pain, swelling, heat, redness, and impaired
mechanisms. function of part (five cardinal signs of
3. Exhaustion: adaptive resources are depleted, inflammation)
overwhelmed, or insufficient; if stress is B. Systemic (appear with moderate to severe
excessive and continues, death will occur response): fever, leukocytosis, chills, sweating,
without support. anorexia, weight loss, general malaise

Stress Management/Nursing Responsibilities


A. Instruct the client concerning ways to manage
IMMUNE RESPONSE
stress The essence of the immune system response is
1. Eat a well-balanced diet. recognition, neutralization, elimination, and
2. Get sufficient amount of rest. metabolism of foreign substance with or without
3. Exercise regularly. injury to the bodys own tissue.

4 144 NCLEX-RN Review


53155_04_Ch04a_p143-263.qxd 2/26/09 6:50 PM Page 145

Functions of the Immune System passive. Active immunity results when the
body produces its own antibodies in response
A. Defense: protection against antigens. An antigen is to an antigen. Passive immunity results when
a protein or protein complex recognized as an antibody is transferred artificially.
nonself. a. Naturally acquired active immunity:
B. Homeostasis: removal of worn out or damaged results from having the disease and
components (e.g., dead cells). recovering successfully
C. Surveillance: ability to perceive or destroy b. Naturally acquired passive immunity:
mutated cells or nonself cells. antibodies obtained through placenta or
breast milk
Alterations in Immune Functioning c. Artificially acquired active immunity:
conferred by immunization with an
See Table 4-1. antigen
d. Artificially acquired passive immunity:
Types of Immunity antibodies transferred from sensitized
person (e.g., immune serum globulin
There are two major types of immunity: natural (or [gamma globulin])
innate) and acquired.
A. Natural (innate) immunity: immune responses that
exist without prior exposure to an Components of Immune Response
immunologically active substance. Genetically A. Located throughout the body
acquired immunity is natural immunity. B. Organs include thymus, bone marrow, lymph
B. Acquired immunity nodes, spleen, tonsils, appendix, Peyers patches of
1. Immune responses that develop during the small intestine.
course of a persons lifetime. C. Main cell types are WBCs (especially lymphocytes,
2. Acquired immunity may be further classified plasma cells, and macrophages); all originate from
as naturally or artificially acquired, active or the same stem cell in bone marrow, then
differentiate into separate types.
1. Granulocytes
a. Eosinophils: increase with allergies and
parasites
Table 4-1 Alterations in Immune Functioning b. Basophils: contain histamine and increase
with allergy and anaphylaxis
Immune c. Neutrophils: involved in phagocytosis
Function Hypofunction Hyperfunction 2. Monocytes (macrophages) (e.g., histiocytes,
Kupffer cells): involved in phagocytosis
Defense Immunosuppression Inappropriate and
3. Lymphocytes (T cells and B cells): involved in
with increased abnormal
cellular and humoral immunity
susceptibility to response to
infection; includes external antigens;
disorders such as an allergy. Classification of Immune Responses
neutropenia, AIDS,
immunosuppression
Cellular Immunity
secondary to A. Mediated by T cells: persist in tissues for months
drugs and hypo- or or years
agammaglobulinemia. B. Functions: transplant rejection, delayed
Homeostasis No known effect Abnormal hypersensitivity, tuberculin reactions, tumor
response where surveillance/destruction, intracellular infections
antibodies react
against normal Humoral Immunity
tissues and cells;
A. Mediated by B cells
an autoimmune
1. Production of circulating antibodies (gamma
disease.
globulin)
Surveillance Inability of the immune No known effect. 2. Only survive for days
system to perceive B. Functions: bacterial phagocytosis, bacterial lysis,
and respond to virus and toxin neutralization, anaphylaxis,
mutated cells, allergic hay fever and asthma
suspected mechanism
in cancer.

ADULT NURSING

4 145
53155_04_Ch04a_p143-263.qxd 2/26/09 6:50 PM Page 146

NUTRITION 3. Necessary for the complete oxidation of fats


(to prevent ketosis)
D. Dietary sources: grains, fruits, vegetables, nuts,
Basic Concepts milk, sugars (empty calories, contain few
Principles nutrients)

A. Essential nutrients: carbohydrates, fats, proteins, Lipids (Fats)


minerals, vitamins, and water that must be
supplied to the body in specified amounts. A. Most concentrated source of energy in foods;
B. Foods: the sources of nutrients, provide energy to 9 kcal/g; contain carbon, hydrogen, oxygen
help build, repair, and maintain tissue and B. Include fats, oils, resins, waxes, and fatlike
regulate body processes. substances such as glycerides, phospholipids,
C. Malnutrition: results from deficiency, excess, or sterols, and lipoproteins
imbalance of required nutrients. C. Fatty acids
1. Saturated fatty acids: usually solid at room
Carbohydrates (Sugars and Starches) temperature; predominantly present in animal
fats
A. Major source of food energy; 4 kcal/g; composed of 2. Monounsaturated fatty acids: present in oleic
carbon, hydrogen, and oxygen acid found in olive oil, peanut oil
B. Classification 3. Polyunsaturated fatty acids: usually liquid at
1. Monosaccharides: simplest form of room temperature; predominantly present in
carbohydrate plant fats and fish
a. Glucose (dextrose): found chiefly in fruits 4. Essential fatty acids: cannot be manufactured
and vegetables; oxidized for immediate by the body (e.g., linoleic fatty acid)
energy 5. Nonessential fatty acids: can be synthesized by
b. Fructose: found in honey and fruits the body
c. Galactose: not free in nature; part of milk D. Functions of lipids
sugar 1. Most concentrated source of energy
2. Disaccharides: double sugars 2. Insulation and padding of body organs
a. Sucrose: found in table sugar, syrups, and 3. Component of the cell membrane
some fruits and vegetables 4. Carrier of the fat-soluble vitamins A, D, E, K
b. Lactose: found in milk 5. Help maintain body temperature
c. Maltose: intermediate product in the E. Dietary sources: oil from seeds of grains, nuts,
hydrolysis of starch vegetables; milk fat, butter, cream cheese; fat in
3. Polysaccharides: composed of many glucose meat; lard, bacon fat; fish oil; egg yolk
molecules F. Cholesterol: essential constituent of body tissues
a. Starch: found in cereal grains, potatoes, 1. A component of cell membranes
root vegetables, and legumes 2. A precursor of steroid hormones
b. Glycogen: synthesized and stored in the 3. Can be manufactured in the body
liver and skeletal muscles 4. Present in animal fats
c. Cellulose, hemicellulose, pectins, gums, 5. Dietary sources: egg yolk, brains, liver, butter,
and mucilages: indigestible cream, cheese, shellfish
polysaccharides G. Indications for low-fat diet
4. Dietary fiber: includes several polysaccharides 1. Cardiovascular disease
plus other substances that are not digestible by 2. Gallbladder disease
GI enzymes. 3. Malabsorption syndromes, cystic fibrosis,
a. Dietary fiber (roughage) holds water so that pancreatitis
stools are soft and bulky; increases motility
of the small and large intestine and Proteins
decreases transit time; reduces
intraluminal pressure in the colon. A. Organic compounds that may be composed
b. Sources: wheat bran, unrefined cereals, of hundreds of amino acids; 4 kcal/g; contain
whole wheat, raw fruits and vegetables, nitrogen in addition to carbon, hydrogen, and
dried fruits. oxygen
C. Functions of carbohydrates B. Classification
1. Cheapest and most abundant source of energy; 1. Complete protein: contains all the essential
only source of energy for central nervous amino acids; usually from animal food
system sources.
2. To spare protein for tissue building when 2. Incomplete protein: lacks one or more

4
sufficient carbohydrate is present essential amino acids; usually from plant food
sources.

146 NCLEX-RN Review


53155_04_Ch04a_p143-263.qxd 2/26/09 6:50 PM Page 147

C. Amino acids I. Nursing interventions for clients needing low-


1. Essential amino acids: eight amino acids that protein diet
cannot be synthesized in the body and must be 1. Increase carbohydrates so energy needs will be
taken in food. met by carbohydrates, not by breakdown of
2. Nonessential amino acids: 12 amino acids that proteins
can be synthesized in the body. 2. Protein intake that is allowed will be complete
D. Functions of proteins proteins (animal sources)
1. Necessary for growth and continuous
replacement of cells throughout life Energy Metabolism
2. Play a role in the immune processes
3. Participate in regulating body processes such A. Measurement of energy expressed in terms of heat
as fluid balance, muscle contraction, mineral units called kilocalories (kcal): amount of heat
balance, iron transport, buffer actions required to raise 1 kg water by 1C
4. Provide energy if necessary B. Energy expenditure
E. Dietary sources: meat, fish, eggs, milk, cheese, 1. Basal metabolism
poultry, grains, nuts, legumes (soybeans, lentils, a. Amount of energy expended to carry on
peanuts, peanut butter) the involuntary work of the body while at
F. Deficiencies rest
1. Conditions b. Factors influencing basal metabolic rate
a. Kwashiorkor (protein depletion that (BMR): body surface area, sex, age, body
develops over short period of time) temperature, hormones, pregnancy, fasting,
b. Marasmus (severe tissue wasting from malnutrition
inadequate intake of calories and proteins) 2. Physical activity: amount of energy expended
2. Manifestations depends upon the type of activity, the length
a. Generalized weakness of time involved, and the weight of the person
b. Weight loss C. Factors determining total energy needs
c. Lowered resistance to infection 1. Amount necessary for BMR
d. Slow wound healing and prolonged 2. Amount required for physical activity
recovery from illness 3. Specific dynamic action of food ingested
e. Growth failure 4. Growth
f. Brain damage to fetus or infant 5. Climate
g. Edema due to decreased albumin in blood
h. Anemia in severe deficiency Minerals
i. Fatty infiltration of liver and liver damage
3. Risk factors Inorganic compounds that yield no energy; essential
a. Chronically ill structural components involved in many body
b. Elderly on fixed incomes processes (see Table 4-2).
c. Low-income families
d. Strict vegetarians Vitamins
G. Indications for high-protein diet
Organic compounds necessary in small quantities for
1. Burns, massive wounds when tissue building
cellular functions of the body; do not give energy;
desired
necessary in many enzyme systems (see Table 4-3).
2. Mild to moderate liver disease for organ repair
A. Fat-soluble vitamins (A, D, E, K): can be stored in
when liver is still functioning
body; toxic in large amounts.
3. Malabsorption syndromes such as cystic
B. Water-soluble vitamins (B1 [thiamin];
fibrosis
B2 [riboflavin]; B6 [pyridoxine];
4. Undernutrition
B12 [hydroxycobalamin]; C [ascorbic acid]; folacin;
5. Pregnancy to meet needs of mother and
niacin): cannot be stored in body so must be
developing fetus
ingested daily; dissolves in cooking water, toxicity
6. Pregnancy-induced hypertension to replace
unlikely.
protein lost in urine
7. Nephrosis to replace protein lost in urine
8. Deficiencies Water
H. Indications for low-protein diet A. Distribution: present in all body tissues; accounts
1. Liver failure (liver does not metabolize protein for 5060% total body weight in adults and
causing nitrogen toxicity to brain) 7075% in infants.
2. Kidney failure (kidneys can no longer excrete 1. Intracellular fluid: exists within the cells.
nitrogenous waste products causing toxic 2. Extracellular fluid: includes plasma fluid,
nitrogen levels in the brain) interstitial fluid, lymph, and secretions.

ADULT NURSING

4 147
53155_04_Ch04a_p143-263.qxd 2/26/09 6:50 PM Page 148

Table 4-2 Minerals

Mineral Functions Deficiency Syndrome Food Sources Comments


Calcium Development of bones and teeth Rickets, osteoporosis, Dairy products; Needs vitamin D,
Transmission of nerve impulses osteomalacia, stunted dark green leafy magnesium, and
Muscle contraction growth, fragile bones, vegetables; parathormone for
Permeability of cell membrane tetany, occurs when broccoli; canned- utilization. Acid,
Catalyze thrombin formation parathyroids removed fish with bones lactose, and vitamin D
Maintenance of normal heart (sardines, salmon) favor absorption.
rhythm clams, oysters; Inverse relationship to
cooked dried phosphorus.
beans; peas
Phosphorus Development of bones and teeth Rickets, stunted growth, Milk, cheese, meat, Factors that affect
Transfer of energy in cells (ATP) poor bone mineralization, fish, poultry, calcium absorption
Cell permeability muscle weakness eggs, legumes, also affect phosphorus.
Buffer salts nuts, whole-grain Inverse relationship to
Component in phospholipids cereals calcium.
Magnesium Constituent of bones and teeth Tremor observed in severe Milk, cheese, meat, Absorption similar
Cation in intracellular fluid alcoholism, diabetic nuts, legumes, to calcium.
Muscle and nerve irritability acidosis, severe renal green leafy
Activate enzymes in disease vegetables, whole-
carbohydrate metabolism grain cereals,
seafood
Sulfur Constituent of keratin in hair, None Protein foods, Diet adequate in protein
skin, and nails eggs, meat, fish, provides sufficient
Detoxification reactions poultry, milk, sulfur.
Constituent of thiamin, biotin, cheese, nuts
insulin, coenzyme A,
melanin, glutathione
Iron Constituent of hemoglobin, Anemia Liver, organ meats, Ascorbic acid enhances
myoglobin, oxidative meat, poultry, absorption.
enzymes egg yolk, whole-
grain cereals,
legumes, dark
green vegetables,
dried fruit
Iodine Constituent of thyroxine Simple goiter, creatinism, Iodized salt, Allergies to iodine-rich
Regulate rate of energy myxedema seafood foods may indicate
metabolism allergy to iodine dyes
used in diagnostic
tests.
Sodium Principle cation of extracellular Rare, seen in persons Table salt, Diet usually provides
fluid with SIADH processed excess.
Osmotic pressure meats, Increase in clients with
Fluid balance canned soups cystic fibrosis and
Regulate nerve irritability and and vegetables persons taking lithium.
muscle contraction Decrease intake in clients
Pump for active transport of with hypertension,
glucose congestive heart
failure, renal failure,
and edema.
(continues)

4 148 NCLEX-RN Review


53155_04_Ch04a_p143-263.qxd 2/26/09 6:50 PM Page 149

Table 4-2 Minerals (continued)

Mineral Functions Deficiency Syndrome Food Sources Comments


Potassium Principal cation of intracellular Muscle weakness, Oranges, bananas, Readily absorbed.
fluid arrhythmias dried fruits, Increase intake in clients
Osmotic pressure Deficiency may occur melons, apricots, taking thiazide and
Fluid balance with diabetic acidosis most fruits and loop diuretics.
Acid-base balance Deficiency may occur vegetables, Decrease intake for
Regular heart rhythm with thiazide and whole-grain clients in renal failure.
Nerve irritability and loop diuretics cereals
muscle contraction
Chlorine Chief anion of extracellular fluid Seen only after Table salt, Rapidly absorbed.
Constituent of gastric juice prolonged processed meats,
Acid-base balance vomiting fish, fruits
Activate salivary amylase (dates, bananas)

Table 4-3 Vitamins

Vitamin Functions Deficiency Syndrome Food Sources Comments


Fat Soluble
Vitamin A Maintenance of mucous Night blindness, Fish liver oils, liver, Bile necessary for
(retinol) membranes xerophthalmia, butter, cream, absorption.
Visual acuity in dim light, keratinization of whole milk, egg Large amounts
growth and bone epithelium, poor yolk, dark green are toxic.
development bone and tooth vegetables, yellow
development vegetables, yellow
fruits, fortified
margarine
Vitamin D Increase absorption of Rickets, osteomalacia, Fish liver oils, Synthesized in skin by
(cholecalci- calcium and phosphorus enlarged joints, fortified milk activity of ultraviolet
ferol) Bone mineralization muscle spasms, light.
delayed dentition Large amounts are toxic.
Vitamin E Reduces oxidation of Hemolysis of red Vegetable oils, wheat Not toxic.
(tocopherol) vitamin A, phospholipids, blood cells, germ, nuts, legumes,
and polyunsaturated deficiency not green leafy vegetables
fatty acids likely
Vitamin K Formation of prothrombin Prolonged clotting time, Green leafy vegetables, Bile necessary for
(phyllo- and other clotting proteins hemorrhagic disease in cabbage, liver, alfalfa absorption; injectable
quinone) newborn and liver form may be given in
disease gallbladder and liver
disease. Large
amounts are toxic.
Water Soluble
Vitamin B1 Involved in carbohydrate Beriberi, mental Enriched cereals, whole Very little storage.
(thiamine) metabolism depression, grains, meat, organ
Thiamine pyrophosphate polyneuritis, cardiac meats, pork, fish,
(TPP) failure poultry, legumes, nuts
Vitamin B2 Coenzyme for transfer and Cheilosis, photophobia, Milk, eggs, organ meats, Limited storage.
(riboflavin) removal of hydrogen burning and itching of green leafy vegetables
Flavin adenine dinucleotide eyes, sore tongue and
(FAD) mouth

(continues)

ADULT NURSING

4 149
53155_04_Ch04a_p143-263.qxd 2/26/09 6:50 PM Page 150

Table 4-3 Vitamins (continued)

Vitamin Functions Deficiency Syndrome Food Sources Comments


Vitamin B6 Coenzyme for Convulsions, dermatitis, Meat, poultry, fish, Converts glycogen
(pyridoxine, transamination, nervous irritability vegetables, potatoes to glucose.
pyridoxal, transsulfuration, and Given with isoniazid
pyridoxamine) decarboxylation (INH) to prevent INH
side effect of
peripheral neuropathy.
Vitamin B12 Formation of mature red Pernicious anemia, Animal foods only Intrinsic factor is
(hydroxy- blood cells neurologic degeneration, necessary for
cobalamin) Synthesis of DNA and RNA macrocytic anemia absorption.
Vitamin C Synthesis of collagen Scurvy, bleeding gums, Citrus fruits, tomatoes, Most easily destroyed
(ascorbic Formation of intercellular poor wound healing, melon, raw cabbage, vitamin. Very little
acid) cement cutaneous hemorrhage, broccoli, strawberries storage in body.
Facilitation of iron absorption capillary fragility
Folacin (folic Maturation of red blood cells, Megaloblastic anemia, Organ meats, muscle Ascorbic acid necessary
acid) interrelated with vitamin B12 tropical sprue meats, poultry, fish, for utilization.
eggs, green leafy
vegetables
Niacin Coenzyme to accept and Pellagra, dermatitis, Meat, poultry, fish, Amino acid tryptophan
(nicotinamide) transfer hydrogen, neurologic degeneration, whole grains, enriched is a precursor.
coenzyme for glycolysis glossitis, diarrhea breads, nuts, legumes

B. Functions: the medium of all body fluids B. Recommended daily allowances: established by
1. Necessary for many biologic reactions. the Food and Nutrition Board of the National
2. Acts as a solvent. Academy of Science; recommended nutrient
3. Transports nutrients to cells and eliminates intake is provided for infants, children, men,
waste. women, pregnant and lactating women;
4. Body lubricant. recommendations are stated for protein, kcal,
5. Regulates body temperature. and most vitamins and minerals.
C. Sources C. Food composition tables: helpful in calculating the
1. Ingestion of water and other beverages nutritive value of the daily diet; list nutrient
2. Water content of food eaten content of foods.
3. Water resulting from food oxidation D. Height and weight charts: ideal or desirable body
D. Recommended daily intake weight for both men and women at specified
1. Replacement of losses through the kidneys, heights with a small, medium, or large frame.
lungs, skin, and bowel E. Exchange lists for meal planning
2. Thirst usually a good guide 1. Foods are separated into six exchange lists.
3. Approximately 48 oz/day of water from all 2. Specific foods on each list are approximately
sources is adequate; requirement is higher if equal in carbohydrate, protein, fat, and kcal
physical activity is strenuous or if sweating is content.
profuse. 3. Individual foods on the same list may be
exchanged for each other at the same meals.
Dietary Guides 4. Food lists are helpful in planning diets for
weight control or diabetes.
A. Food pyramid
1. Foods are grouped by composition and
nutrient value: grains; vegetable group; fruit Nutritional Assessment
group; meat, poultry, fish, dry beans, eggs and Health History
nut group; and milk, yogurt and cheese group.
2. Greater emphasis on fruits and vegetables with A. Presenting problem
less emphasis on meats and fats than with 1. Weight changes
basic four. a. Usual body weight 20% above or below

4
3. Recommends using fats and sweets sparingly. normal standards.

150 NCLEX-RN Review


53155_04_Ch04a_p143-263.qxd 2/26/09 6:50 PM Page 151

b. Recent loss or gain of 10% of usual body count, blood sugar, total cholesterol, high-density
weight. lipids, low-density lipids, triglycerides, serum
2. Appetite changes: may increase or decrease electrolytes
from usual. B. Urine studies, urinalysis, glucose, ketones,
3. Food intolerances: allergies, fluids, fat, salt, albumin, 24-hour creatinine
seafood C. Nitrogen balance studies
4. Difficulty swallowing D. Feces, hair
5. Dyspepsia or indigestion E. Intradermal delayed hypersensitivity testing
6. Bowel dysfunction: record frequency,
consistency, color of stools.
a. Constipation
Analysis
b. Diarrhea Nursing diagnoses for the client with a nutritional
B. Lifestyle: eating behaviors such as fast foods, junk dysfunction may include:
foods, and skipping meals; cultural/religious A. Imbalanced nutrition: less than body requirements
concerns (vegetarian, kosher foods, exclusion of B. Imbalanced nutrition: more than body
certain food groups); alcohol, socioeconomic requirements
status, living conditions (alone or with family). C. Risk for imbalanced nutrition: more than body
C. Use of medications: vitamin supplements, requirements
antacids, antidiarrheals, laxatives, diuretics, D. Impaired oral mucous membrane
antihypertensives, immunosuppressants, oral E. Self-care deficit, feeding
contraceptives, antibiotics, antidepressants, F. Disturbed sensory perceptions
digitalis, anti-inflammatory agents, catabolic G. Risk for impaired skin integrity
steroids. H. Impaired swallowing
D. Medical history: gastrointestinal diseases; I. Impaired tissue integrity
endocrine diseases; hyperlipidemia; coronary J. Activity intolerance
artery disease; malabsorption syndrome; K. Disturbed body image
circulatory problems or heart failure; cancer; L. Constipation
radiation therapy; chronic lung, renal, or liver M. Diarrhea
disease; food allergies; recent major surgery; eating N. Deficient fluid volume
disorders; obesity. O. Excess fluid volume
E. Family history: obesity, allergies, cardiovascular P. Delayed growth and development
diseases, diabetes, thyroid disease. Q. Risk for infection
F. Dietary history: evaluation of the nutritional R. Deficient knowledge
adequacy of diet S. Noncompliance
1. 24-hour recall
2. Food diary for a given number of days
Planning and Implementation
Physical Examination Goals
A. Assess for alertness and responsiveness A. Normal weight will be achieved and maintained.
B. Record weight in relation to height, body build, B. Integrity of oral cavity will be maintained.
and age C. Client will feed self or receive help with feeding.
C. Inspect posture, muscle tone, skeleton for D. Normal skin integrity will be
deformities achieved/maintained.
D. Elicit reflexes E. Client will not aspirate.
E. Auscultate heart rate, rhythm; blood pressure F. Normal tissue integrity will be
F. Inspect hair, skin, nails, oral mucosa, tongue, teeth achieved/maintained.
G. Inspect for swelling of legs or feet G. Client will be able to exercise normally.
H. Anthropometric measurements: indicators of H. Client will maintain/develop satisfactory
available stores in muscle and fat compartments self-image.
of body I. Normal bowel functioning will be maintained.
1. Height/weight ratio (Body Mass Index [BMI]) J. Fluid and electrolyte balance will be
2. Midarm muscle circumference achieved/maintained.
3. Skinfold thickness (triceps, biceps, K. Client will have normal growth and development
subscapular, abdominal, hip, pectoral, or calf) patterns.
L. Client will not develop infection.
Laboratory/Diagnostic Tests M. Client will demonstrate knowledge of special
dietary needs/prescriptions.
A. Blood studies: serum albumin, iron-binding N. Client will comply with special diet.

4
capacity, hemoglobin, hematocrit, lymphocyte

ADULT NURSING 151


53155_04_Ch04a_p143-263.qxd 2/26/09 6:50 PM Page 152

Interventions C. Principles
1. Distribution of kcal: protein 1220%;
Care of the Client on a Special Diet carbohydrates 5560%; fats (unsaturated)
2025%.
A. General information: therapeutic diets involve
2. Daily distribution of kcal: equally divided
modifications of nutritional components
among breakfast, lunch, supper, snacks.
necessitated by a clients disease state or
3. Use foods high in fiber and complex
nutritional status or to prepare a client for a
carbohydrates.
procedure.
4. Avoid simple sugars, jams, honey, syrup,
B. Nursing care in relation to special diets
frosting.
1. Assess clients mental, emotional, physical,
D. Teach client to utilize exchange lists.
and economic status; appropriateness of diet
E. New recommendations include low-fat, high fiber
to clients condition; and ability to understand
diet.
diet and comply with it.
2. Maintain appropriate diet and teach client.
3. Changing diet means changing lifelong patterns. Low-Sodium Diet (No-added-salt diet)
4. Teach client importance of adhering to special A. Purpose is to restrict sodium intake to less than
diets that are long term. 2300 mg of sodium per day for clients with
hypertension or cardiac disease.
Weight Control Diets B. One method is the DASH (Dietary Approaches to
Stop Hypertension) Eating Plan.
A. Underweight: 10% or more below individuals
C. Food choices
ideal weight
1. Choose and prepare food with little salt.
1. Causes: failure to ingest enough kcal, excess
2. Continue to meet potassium requirement of
energy expenditure, irregular eating habits, GI
4700 mg/day.
disturbances, mouth sores, cancer, endocrine
3. Avoid table salt, processed meats, canned
disorders, emotional disturbances, lack of
soups, snack food containing salt.
education, economic problems.
4. Teach client to read labels of prepared food.
2. Treatment: diet counseling, correction of
underlying disease, nutritional supplements,
behavioral therapy, social service referral. Protein-Modified Diets
B. Overweight: 10% or more above individuals ideal A. Gluten-free diet
weight 1. Purpose is to eliminate gluten (a protein) from
C. Obesity: 20% or more above individuals ideal the diet.
weight 2. Indicated in malabsorption syndromes such as
1. Causes: overeating, underactivity, genetic sprue and celiac disease.
factors, fat cell theory, alteration in 3. Eliminate all barley, rye, oats, and wheat
hypothalamic function, endocrine disorders, (BROW).
emotional disturbances. 4. Avoid: cream sauces, breaded foods, cakes,
2. Treatment: diet counseling, nutritionally breads, muffins.
balanced diet, behavior modification, 5. Allow corn, rice, and soy flour.
increased physical activity, medical treatment 6. Teach client to read labels of prepared foods.
of any underlying disease, appropriate B. PKU (Phenylketonuria) diet
referrals. 1. Purpose is to control intake of phenylalanine,
D. Nursing care an amino acid that cannot be metabolized.
1. Explain dietary instructions. 2. Diet will be prescribed until at least age 6 to
a. Reducing fats and empty calories prevent brain damage and mental retardation.
reduces caloric intake without sacrificing 3. Avoid: breads, meat, fish, poultry, cheeses,
nutritional intake legumes, nuts, eggs.
b. Increasing exercise increases metabolism 4. Give Lofenalac formula.
2. Caution against fad diets that may be 5. Teach family to use low-protein flour for
nutritionally inadequate. baking.
3. Encourage support groups if indicated. 6. Sugar substitutes such as Nutrasweet contain
phenylalanine and must not be used.
Diabetic Diet (Consistent carbohydrates) C. Low-purine diet
1. Indicated for gout, uric acid kidney stones,
A. Prescribed for clients with diabetes mellitus.
and uric acid retention.
B. Purposes include: attain or maintain ideal body
2. Purpose is to decrease the amount of purine, a
weight, ensure normal growth, maintain plasma
precursor to uric acid.
glucose levels as close to normal as possible.

4 152 NCLEX-RN Review


53155_04_Ch04a_p143-263.qxd 2/26/09 6:50 PM Page 153

3. Teach client to avoid: organ meats, other C. Soft diet


meats, fowl, fish and lobster, lentils, dried 1. Used as a transition diet between full liquid
peas and beans, nuts, oatmeal, whole wheat. and regular diet.
4. Eggs are not high in purine. 2. Indications include postoperatively, mild GI
disturbances, chewing difficulties from lack of
Fat-Restricted Diets teeth or oral surgery.
3. Foods allowed: foods low in fiber, connective
Purpose is to restrict amount of fats ingested for tissue and fat (full liquid diet, pureed
clients with chronic pancreatitis, malabsorption vegetables, eggs cooked any way except fried,
syndromes, gallbladder disease, cystic fibrosis, and tender meat, potatoes, cooked fruit).
hyperlipidemia, and to control weight. 4. Nutritionally adequate.
D. Bland diet
Bariatric Diet 1. Promotes healing of the gastric mucosa and is
chemically and mechanically nonstimulating.
A. Prescribed for clients after bariatric weight loss 2. Foods allowed: soft diet without spices.
surgeries for obesity. E. Low-residue diet
B. After surgery, small stomach will hold about 1 oz. 1. Residue is the indigestible substances left in
1. First week: nutritious liquids; Second week: digestive tract after food has been digested.
Pureed, high-protein foods 2. Indications include colon, rectal, or perineal
2. Avoid high carbohydrates surgery to reduce pressure on the operative
3. Possible complication: Dumping Syndrome site; prior to examination of the lower bowel to
(nausea, hypotension, hypoglycemia) enhance visualization; internal radiation for
4. Client is educated before surgery about cancer of the cervix; Crohns disease or
post-operative diet. regional enteritis; ulcerative colitis to reduce
irritation of the large bowel; and diarrhea.
Renal Diet 3. Teach client to avoid foods high in fiber, foods
having skins and seeds, and milk and milk
A. Prescribed for clients with end-stage renal disease
products.
(ESRD).
B. Principles
1. Prevent accumulation of protein waste Evaluation
between dialysis treatments. A. Clients weight is within normal limits.
2. Potassium is restricted to 30004000 mg/day. B. No lesions in oral cavity.
(Restrict milk intake to 12 cup/day due to high C. Client feeds self or receives needed assistance with
potassium content.) feeding.
3. Limit sodium to 3 grams/day (No-added-salt diet). D. Skin and tissue integrity is maintained.
4. Teach client to measure proper food choices E. Client demonstrates ability to exercise.
and to measure fluid intake and output. F. Client makes positive statements about self-image.
G. Clients bowel functioning is normal.
Consistency Modifications H. Serum electrolytes are within normal limits.
I. Client will exhibit growth and development
A. Clear liquid diet
patterns appropriate for age.
1. Purpose is to rest GI tract and maintain fluid
J. Client shows no evidence of infection.
balance.
K. Client states reason for special diet.
2. Indications include difficulty chewing or
L. Client describes foods allowed and not allowed on
swallowing; before certain diagnostic tests to
prescribed diet.
reduce fecal material; immediate postoperative
M. Client adheres to prescribed diet.
period (until bowel sounds have returned) to
maintain electrolyte balance; and nausea,
vomiting, and diarrhea. Enteral Nutrition
3. Foods allowed: see-through foods include Preferred method for nutritional support for the
water, tea, broth, jello, apple juice, clear malnourished client whose GI system is intact.
carbonated beverages, and frozen ice pops.
4. Not nutritionally adequate. Oral Feeding
B. Full liquid diet
1. Used as a transition diet between clear liquid A. Always the first choice.
and soft diet; usually short term. B. Oral formula supplements may be used between
2. Foods allowed: clear liquids, milk and milk meals to provide added kcal and nutrients.
products, all fruit juices, cooked and strained 1. Offer small quantities several times a day.
cereals. 2. Vary flavors, avoid taste fatigue.

4
3. Can be nutritionally adequate. 3. Chill and serve over ice.

ADULT NURSING 153


53155_04_Ch04a_p143-263.qxd 2/26/09 6:50 PM Page 154

Tube Feeding a. Check for correct solution and additives


against physicians order.
A. Used for clients who have a functioning GI tract b. Check expiration date.
but cannot ingest food orally c. Observe fluid for cloudiness or floating
1. Feeding tubes particulate matter.
a. Short term: nasogastric tube 3. Control flow rate of solution.
b. Long term: esophagostomy, gastrostomy, or a. Verify order and monitor flow rate.
enterostomy tube b. Administration via pump is required.
2. Formulas: nutritionally adequate, tolerated by c. Tubing with in-line filter is required.
client, easily prepared, easily digested, usual d. Never attempt to speed up or slow down
concentration 1 kcal/mL infusion rate.
3. Feeding schedules 1) Speeding up infusion causes large
a. Intermittent: usually 46 times per day, amounts of glucose to enter body,
volumes up to 400 mL, by slow gravity causing hyperosmolar state.
drip over 3060 minutes 2) Slowing down infusion can cause
b. Continuous: usually administered by hypoglycemic state, as it takes time for
pump through a duodenal or proximal the pancreas to adjust to reduced
jejunostomy feeding tube glucose level.
4. Nursing responsibilities 4. Monitor fluid balance.
a. Administer formulas at room temperature 5. Assess client for signs and symptoms of
(refrigerate unused portion). infection (fever, chills, elevated WBC count).
b. Gradually increase rate and concentration 6. Obtain fractional urines or Accu-Chek every
until desired amount is attained if there are 6 hours.
no signs of intolerance (e.g., gastric 7. Administer sliding scale insulin for
residual greater than 120 mL, nausea, hyperglycemia, as ordered.
vomiting, diarrhea, distention, diaphoresis, 8. Provide psychological support.
increased pulse, glycosuria, aspiration). 9. Encourage exercise regimen.
c. Check tube placement and elevate head of
bed (see also Nasogastric Tubes). IV Lipid Emulsions
d. Monitor I&O, serum electrolytes, fractional
urines, serum glucose, daily weights; keep A. May be given through a central vein or
a stool record as well as an ongoing peripherally in order to prevent essential fatty acid
assessment of tolerance. deficiency in long-term TPN clients, or to provide
supplemental kcal IV.
B. Nursing care
Parenteral Nutrition 1. Protect the stability of the emulsion.
Nutrients are infused directly into a vein for clients who a. Administer in its own separate IV bottle
are unable to eat or digest food through the GI tract, who and IV tubing, and piggyback the emulsion
refuse to eat, or who have inadequate oral intake. into the Y connector closest to the catheter
insertion. Follow hospital policy and
Total Parenteral Nutrition (TPN) manufacturers recommendations for
specific products. Some hospital facilities
A. Involves the infusion of nutrients through a central combine TPN and lipid into one bag.
vein catheter. A central vein is needed because its b. Inspect solution for evidence of separation
larger caliber and higher blood flow will quickly of oil, frothiness, inconsistency, particulate
dilute the hypertonic hyperalimentation solution matter; discard solution if any of these
to isotonic concentrations. signs of instability occur.
B. Hyperalimentation solutions c. Do not shake the bottle; this might cause
1. Hypertonic glucose of 2070%, amino acids, aggregation of fat globules.
water, vitamins, and minerals with lipid d. Discard partially used bottles.
emulsions given in a separate solution. 2. Control the infusion rate accurately and safely.
2. Three-in-one solutions a. If using gravity method, lipid emulsion
a. Lipids mixed with dextrose and amino must hang higher than hyperalimentation
acids in pharmacy. to prevent backflow.
b. Prepared by pharmacy in a 3-liter b. Pump is preferred but may not be possible
container and administered over 24 hours. due to viscous nature of emulsion.
C. Nursing responsibilities 3. Prevent and assess for adverse reactions.
1. For details of nursing care of the client with a a. Administer slowly according to package
central venous line, see IV Therapy. insert over first 30 minutes; if no adverse

4
2. Inspect solution before hanging. reactions, increase rate to complete

154 NCLEX-RN Review


53155_04_Ch04a_p143-263.qxd 2/26/09 6:50 PM Page 155

infusion over the specified number of C. Disinfect and handle wastes and contaminated
hours. materials properly.
b. Obtain baseline vital signs; repeat after first D. Prevent transmission of infectious droplets.
30 minutes, and then every 4 hours until 1. Teach clients to cover mouth and nose when
completion. sneezing or coughing.
c. Acute reactions may include: fever, chills, 2. Place contaminated tissues and articles in
dyspnea, nausea, vomiting, headache, paper bag before disposing.
lethargy, syncope, chest or back pain, E. Institute proper isolation techniques as required by
hypercoagulability, thrombocytopenia. specific disease
4. Evaluate tolerance and client response. F. Use surgical aseptic technique when appropriate:
caring for open wounds, irrigating, or entering
Peripheral Vein Parenteral Nutrition (PPN) sterile cavities.
G. Practice standard precautions when caring for all
A. Can be used for short-term support, when the clients regardless of their diagnosis in order to
central vein is not available, and as a supplemental minimize contact with blood and body fluids and
means of obtaining nutrients. Client must be able prevent the transmission of specific infections
to tolerate a relatively high fluid volume. such as hepatitis B and human immunodeficiency
B. Solution contains the same components as central virus (HIV).
vein therapy, but lower concentrations (less than 1. Hands must always be washed before and after
20% glucose). contact with clients even when gloves have
C. Care is the same as for the client receiving been used.
hyperalimentation centrally. 2. If hands come in contact with blood, body
D. Phlebitis and thrombosis are common and IV sites fluids, or human tissue they should be
will need frequent changing. immediately washed with soap and water.
3. Gloves should be worn before touching blood
or body fluids, mucous membranes, or
INFECTION nonintact skin.
4. Gloves should be changed between each client
Infection is an invasion of the body by pathogenic contact and as soon as possible if torn.
organisms that multiply and produce injurious effects. 5. Wear masks and protective eyewear during
Communicable disease is an infectious disease that procedures that are likely to generate splashes
may be transmitted from one person to another. of blood or other body fluids.
6. Wear gowns during procedures that are likely
Chain of Events to generate splashes of blood or other body
fluids and when cleaning spills from
A. Causative agent: invading organism (e.g., bacteria, incontinent clients or changing soiled linen.
virus) 7. Disposable masks should be used when
B. Reservoir: environment in which the invading performing CPR.
organism lives and multiplies 8. Dispose of used needles properly. They should
C. Portal of exit: mode of escape from reservoir be promptly placed in a puncture-resistant
(e.g., respiratory tract, GI tract) container (i.e., sharps container). They should
D. Mode of transmission: method by which invading not be recapped, bent, broken, or removed
organism is transported to new host (e.g., direct from syringes.
contact, air, food)
E. Portal of entry: means by which organism enters
new host (e.g., respiratory tract, broken skin) PAIN
F. Susceptible host: susceptibility determined by
factors such as number of invading organisms, Pain is an unpleasant sensation, entirely subjective,
duration of exposure, age, state of health, that produces discomfort, distress, or suffering. Pain
nutritional status is what the person says it is and exists when the
person says it does. It is considered the fifth vital
Nursing Responsibilities in Prevention sign and is included in the routine patient
assessment (Daniels, et al.)
of Spread of Infection
A. Maintain an environment that is clean, dry, and Gate Control Theory
well ventilated.
B. Use proper handwashing before and after client A. Substantia gelatinosa in the dorsal horn of the spinal
contact and after contact with contaminated cord acts as a gate mechanism that can close to keep
material. pain impulses from reaching the brain, or can open

4
to allow pain impulses to ascend to the brain.

ADULT NURSING 155


53155_04_Ch04a_p143-263.qxd 2/26/09 6:50 PM Page 156

B. Most pain impulses are conducted over small- 2. Respect the clients attitudes and behavioral
diameter nerve fibers; if predominant nerve responses to pain using a standardized pain
message is pain, the gate opens and allows pain scale appropriate to age and condition.
impulses to reach the brain. 3. Document effectiveness of interventions in a
C. The gate can be closed by conflicting impulses timely manner.
from the skin conducted over large-diameter nerve B. Assess characteristics of pain and evaluate clients
fibers, by impulses from the reticular formation in response to interventions.
the brainstem, or by impulses from the entire C. Promote rest and relaxation.
cerebral cortex or thalamus. 1. Prevent fatigue.
2. Teach relaxation techniques, e.g., slow,
Acute Pain and Chronic Pain rhythmic breathing, guided imagery.
D. Institute comfort measures.
A. Acute pain 1. Positioning: support body parts.
1. Short duration; may last from split second to 2. Decrease noxious stimuli such as noise or
about 6 months. bright lights.
2. Serves the purpose of warning the client that E. Provide cutaneous stimulation: massage, pressure,
damage or injury has occurred in the body that baths, vibration, heat, cold packs; increased input
requires treatment. of large-diameter fibers closes gate.
3. Subsides as healing occurs. F. Relieve anxiety and fears.
4. Usually associated with autonomic nervous 1. Spend time with client.
system symptoms, e.g., increased pulse and 2. Offer reassurance, explanations.
blood pressure, sweating, pallor. G. Provide distraction and diversion, e.g., music,
B. Chronic pain puzzles.
1. Prolonged duration; lasts for 6 months or H. Administer pain medication as needed.
longer. 1. Administer pain medication in early stages
2. Serves no useful purpose. before pain becomes severe.
3. Persists long after injury has healed. 2. Administer pain medication prior to
4. Rarely accompanied by autonomic nervous procedure that produces discomfort.
system activity. 3. If pain is present most of the day, a
preventative approach may be used, e.g., an
Assessment of Pain around-the-clock schedule may be ordered in
place of a prn schedule.
See Table 4-4.
4. Document effectiveness of intervention.
I. Teach client about pain and pain control measures,
General Nursing Interventions e.g., relaxation techniques, cutaneous stimulation.
A. Establish nurse-client relationship.
1. Let the client know that you believe that his Specific Medical and Surgical
pain is real.
Therapies for Pain
Table 4-4 Pain Assessment See also Narcotic and Nonnarcotic Analgesics in
Unit 2.
Influencing factors
Past experience with pain Nonnarcotic Analgesics
Age (tolerance generally increases with age)
Culture and religious beliefs A. Salicylates (ASA, aspirin [Ecotrin] choline
Level of anxiety magnesium trisalicylate [Trillisate], diflunisal
Physical state (fatigue or chronic illness may decrease [Dolobid], salsalate [Disalcid])
tolerance) B. Acetaminophen (Datril, Tylenol)
C. Nonsteroidal anti-inflammatory drugs (NSAIDs:
Characteristics of pain
ibuprofen [Motrin], indomethacin [Indocin],
Location
piroxicam [Feldene]) (See Table 2-5).
Quality
Intensity
Timing and duration
Adjuvants
Precipitating factors A. Includes several classes of drugs that may either:
Aggravating factors 1. Potentiate the effects of narcotic or
Alleviating factors nonnarcotic analgesics, e.g., hydroxyzine
Interference with activities of daily living (Vistaril, Atarax)
Patterns of response

4 156 NCLEX-RN Review


53155_04_Ch04a_p143-263.qxd 2/26/09 6:50 PM Page 157

2. Have independent analgesic properties in E. Nursing interventions


certain situations, e.g., tricyclic 1. Monitor client closely for respiratory
antidepressants such as amitriptyline (Elavil) depression especially during initiation of
for neuropathic pain treatment. (May be reversed with naloxone
3. Help control signs and symptoms associated [Narcan]).
with pain, e.g., anxiety, depression, nausea, 2. Assess for other side effects:
and insomnia a. Urinary retention: Foley catheter may be
used in post-op client until infusion is
Patient-Controlled Analgesia (PCA) discontinued
b. Pruritus: may be treated with
A. Type of intravenous pump that allows the client to antihistamine or medication rate reduction
administer narcotic analgesic (e.g., morphine) on c. Nausea and vomiting
demand within preset dose and frequency limits. 3. Check insertion site frequently for signs of
B. Goal is to achieve more constant level of analgesia infection.
as compared to prn IM injections; also, in general,
causes less sedation and lower risk of respiratory Electrical Stimulation Techniques
depression.
for Pain Control
C. Used most often for postoperative pain
management; also used for intractable pain in A. Transcutaneous electrical nerve stimulator (TENS)
terminal illness. 1. Noninvasive alternative to traditional methods
D. PCA pump may be used solely on PCA mode or of pain relief
may be combined with a continuous basal mode 2. Used in treating acute pain (e.g., post-op pain)
where client is receiving continuous infusion of and chronic pain (e.g., chronic low back pain)
narcotic in addition to self-administered bolus 3. Consists of impulse generator connected by
injections. wires to electrodes on skin; produces tingling,
E. The dose of the analgesic bolus and the time interval buzzing sensation in the area.
between boluses (lockout period) is preset on the 4. Mechanism based on gate-control theory:
pump by the RN according to physicians orders. electrical impulse stimulates large diameter
F. Nursing Interventions nerve fibers to close the gate.
1. Instruct client in use of PCA pump 5. Nursing responsibilities
a. Demonstrate how to push control button. a. Do not place electrodes over incision site,
b. Explain concept of client-controlled broken skin, carotid sinus, eyes, laryngeal
analgesia. or pharyngeal muscles.
2. Assess clients level of consciousness, b. Do not use in client with cardiac
respiratory rate, and degree of pain relief pacemaker.
frequently. c. Provide skin care.
3. Keep control button within client reach. 1) Remove electrodes once a day; wash
4. Educate the family to contact the nurse if area with soap and water and air dry.
clients pain is not controlled, instead of 2) Wipe area with skin prep pad before
family members pushing the button. reapplying electrode.
3) Assess area for signs of redness;
Intraspinal Narcotic Infusion reposition electrodes if redness
persists for more than 30 minutes.
A. Involves intraspinal infusion of narcotics or local B. Dorsal column stimulator
anesthetic agents for relief of acute or chronic pain. 1. Used in selected clients for whom
B. Medication is infused through catheter placed in conventional methods of pain relief have not
the subarachnoid (intrathecal) or epidural space in been effective.
the thoracic or lumbar area. 2. Electrode is surgically placed over the dorsal
C. Repeated injections of narcotics produce analgesia column of the spinal cord via laminectomy;
without many of the side effects associated with connected by wires to a transmitter that may
systemic narcotics (e.g., sedation). be worn externally or be implanted
D. Indications subcutaneously.
1. Temporary intraspinal narcotic therapy is used
most frequently for postoperative pain.
Neurosurgical Procedures for Pain Control
2. For chronic pain, e.g., management of chronic
cancer pain, the catheter may be tunneled A. Performed for persistent intractable pain of high
under the skin and implanted subcutaneously intensity
in the abdomen; an implantable infusion B. Involves surgical destruction of nerve pathways to
device may be used to provide continuous block transmission of pain

4
narcotic infusion.

ADULT NURSING 157


53155_04_Ch04a_p143-263.qxd 2/26/09 6:50 PM Page 158

C. Types B. Mechanism is thought to be related to positive


1. Neurectomy: interruption of cranial or suggestions that alter clients perception of pain.
peripheral nerves by incision or injection
2. Rhizotomy: interruption of posterior nerve Behavioral Techniques
root close to the spinal cord
a. Laminectomy is necessary. A. Types
b. Results in permanent loss of sensation and 1. Operant conditioning: based on decreasing
position sense in affected parts. positive reinforcement for pain behaviors
3. Chordotomy: interruption of pain-conducting 2. Biofeedback: teaches clients to control
pathways within the spinal cord physiologic responses to pain (e.g., muscle
a. Laminectomy usually required. tension, heart rate, blood pressure) and to
b. May be done by percutaneous needle replace them with a state of relaxation.
insertion. B. Work best in conjunction with other types of pain
c. Interrupts conduction of pain and management and stress reduction techniques.
temperature sense in affected parts.
4. Sympathectomy: interruption of afferent
pathways in the sympathetic division of the FLUIDS AND ELECTROLYTES
autonomic nervous system; used to control pain
from causalgia and peripheral vascular disease. Basic Principles
D. Nursing responsibilities
1. Provide pre- and post-op care for a Fluids
laminectomy.
A. Water constitutes over 50% of individuals weight;
2. Assess extremities for sensation (e.g., touch,
largest single component.
pain, temperature, pressure, position sense)
B. Body water divided into two compartments
and movement.
1. Intracellular: within cells
3. Provide safety measures to protect client from
2. Extracellular: outside cells, further divided
injury and carefully monitor skin for signs of
into interstitial and intravascular fluid
damage or pressure.
C. Fluids in two compartments move among cells,
4. Teach client ways to compensate for loss of
tissue spaces, and plasma.
sensation in affected parts.
a. Visually inspect skin for signs of injury or
pressure. Electrolytes
b. Check temperature of bath water. A. Salts or minerals in extracellular or intracellular
c. Avoid use of hot water bottles, heating body fluids
pads. B. If positively charged, called cations; if negatively
d. Avoid extremes of temperature. charged, called anions
C. Common electrolytes and normal blood values
Acupuncture 1. Sodium (Na)135148 mEq/liter
2. Potassium (K)3.55 mEq/liter
A. A Chinese technique of pain control by insertion
3. Calcium (Ca)8.510.5 mg/dL
of fine needles at various points on the body to
4. Magnesium (Mg)1.82.7 mEq/liter
promote the flow of chi (life energy).
5. Chloride (Cl)98106 mEq/liter
B. Based on Eastern philosophy where insertion of
needles is thought to block energy flow and restore
the bodys harmony Movement of Fluids and Electrolytes
C. Mechanism of action: two theories
A. Diffusion: movement of particles from an area of
1. Trigger points: the needles stimulate
greater concentration to an area of lesser
hypersensitive areas in muscle that produce
concentration as part of random activity
local and referred pain. Extinction of the
B. Active transport: movement across cell membranes
trigger point alleviates the referred pain.
requiring energy from an outside source
2. Endorphin system: needle insertion activates
C. Osmosis: movement of water through a
production of endorphins (bodys natural
semipermeable membrane
opiates).
D. Osmolality: concentration of body fluids
D. Acupressure: a less invasive variation; uses finger
pressure and massage
Fluid and Electrolyte Imbalances
Hypnosis See Table 4-5.
A. Has been used in dental procedures, labor and A. Hypovolemia: extracellular fluid volume deficit
B. Hypervolemia: extracellular fluid volume excess

4
delivery, pain control in cancer.

158 NCLEX-RN Review


53155_04_Ch04a_p143-263.qxd 2/26/09 6:50 PM Page 159

Table 4-5 Fluid and Electrolyte Imbalances

Imbalance Causes Assessment Findings Nursing Interventions


Hypovolemia Hemorrhage, diarrhea, vomiting, Nausea and vomiting, weakness, Measure I&O.
(extracellular kidney disease, diaphoresis, weight loss, anorexia, longitudinal Weigh daily.
fluid volume burns, fever, draining fistulas, wrinkles of the tongue, dry skin Monitor closely and regulate
deficit) sequestration of fluids and mucous membranes, isotonic IV infusion.
(peritonitis, edema associated decreased fullness of neck veins, Monitor blood pressure
with burns) postural hypotension, oliguria (determine lying down, sitting,
to anuria, shock and standing).
Report urine output less than
30 mL/hr.
Carefully assess skin and
mucous membranes.
Monitor for signs of shock.
Hypervolemia Excess or too rapid Weight gain, pitting edema, dyspnea, Weigh daily.
(extracellular administration of any isotonic cough, diaphoresis, frothy or Measure I&O.
fluid volume solution; side effect of pink-tinged sputum, edema of Regulate IV fluids/administration
excess) corticosteroid administration; the eyelids, distended neck veins, of diuretics strictly and monitor
cardiac, liver, or renal disease; elevated blood pressure, moist carefully.
cerebral damage; stress rales (crackles) Monitor abdominal girth.
Assess for pitting edema.
Restrict sodium and water intake.
Water excess Excessive intake of water, inability Polyuria (in absence of renal Measure I&O.
syndromes to excrete water due to kidney disease), oliguria (with renal Weigh daily.
or brain damage, excessive disease), twitching, hyper Restrict oral and IV intake.
administration of electrolyte- irritability, disorientation, Replace fluid losses with isotonic
free solutions, poor salt intake, coma, convulsions, abdominal solutions.
use of diuretics, irrigation of cramps Use normal saline solution for
nasogastric tube with plain nasogastric tube irrigation.
water, administration of
excessive amount of ice chips
to a vomiting client or one
with a nasogastric tube
Water deficit Increased water output due to Thirst, poor skin turgor, dry skin and Measure I&O.
syndromes watery diarrhea, diabetic mucous membranes, dry Weigh daily.
acidosis, excess TPN; dysphagia; furrowed tongue, sunken eyeballs, Assess skin frequently.
impaired thirst mechanism; weight loss, elevated temperature, Ensure that clients with a high
coma; general debility; apprehension, oliguria to anuria solute intake receive adequate
diaphoresis; excess protein water.
intake without sufficient water Assess vital signs frequently,
intake particularly temperature.
Monitor TPN infusions
accurately.
Hyperkalemia Renal insufficiency, adrenocortical Thready, slow pulse; shallow Administer Kayexalate as ordered.
insufficiency, cellulose damage breathing; nausea and vomiting; Administer/monitor IV infusion of
(burns), infection, acidotic diarrhea; intestinal colic, irritability; glucose and insulin.
states, rapid infusion of IV muscle weakness, numbness, Control infection.
solutions with potassium, flaccid paralysis; tingling; difficulty Provide adequate calories and
overzealous administration of with phonation, respiration carbohydrates.
potassium-conserving diuretics Discontinue IV or oral sources of
potassium.
(continues)

ADULT NURSING

4 159
53155_04_Ch04a_p143-263.qxd 2/26/09 6:50 PM Page 160

Table 4-5 Fluid and Electrolyte Imbalances (continued)

Imbalance Causes Assessment Findings Nursing Interventions


Hypokalemia Anorexia, alcoholism, gastric and Thready, rapid, weak pulse; faint Be especially cautious if
intestinal suction, GI surgery, heart sounds; decreased blood administering drugs that are
vomiting, diarrhea, laxative pressure; skeletal muscle weakness; not potassium sparing.
abuse, thiazide diuretics, decreased or absent reflexes; Administer potassium
steroid therapy, stress, alkalotic shallow respirations; malaise; supplements to replace losses.
states apathy; lethargy; loss of orientation; Monitor acid-base balance.
anorexia, vomiting, weight loss, Monitor pulse, blood pressure,
gaseous intestinal distention and ECG.
Hypernatremia Excessive/rapid IV administration Dry, sticky mucous membranes; Weigh daily.
of normal saline solution, flushed skin; rough, dry tongue; Assess degree of edema
inadequate water intake, kidney firm skin turgor; intense thirst; frequently.
disease edema; oliguria to anuria Measure I&O.
Assess skin frequently and
institute nursing measures to
prevent breakdown.
Encourage sodium-restricted diet.
Hyponatremia Decreased sodium intake, Nausea and vomiting; abdominal Provide foods high in sodium.
increased sodium excretion cramps; weight loss; cold, clammy Administer normal saline
through diaphoresis or GI skin; decreased skin turgor; solution IV.
suctioning, adrenal fingerprinting over the sternum; Assess blood pressure frequently
insufficiency shrunken tongue; apprehension; (measure lying down, sitting,
headache; convulsions; confusion; and standing).
weakness; fatigue; postural
hypotension; rapid, thready pulse
Hypercalcemia Hyperparathyroidism, immobility, Nausea and vomiting, anorexia, Encourage mobilization.
increased vitamin D intake, constipation, headache, Limit vitamin D and calcium
osteoporosis and osteomalacia confusion, lethargy, stupor, intake.
(early stages) decreased muscle tone, deep Administer diuretics.
bone and/or flank pain Protect from injury.
Hypocalcemia Acute pancreatitis, diarrhea, Painful tonic muscle spasms, facial Administer oral calcium lactate or
hypoparathyroidism, lack of spasms, fatigue, laryngospasm, IV calcium chloride or
vitamin D in diet, long-term positive Trousseaus and Chvosteks gluconate.
steroid therapy signs, convulsions, dyspnea Provide safety by padding
side rails.
Administer dietary sources
of calcium.
Provide quiet environment.
Hypermagnesemia Renal insufficiency, dehydration, Lethargy, somnolence, confusion, Withhold magnesium-containing
excessive use of magnesium- nausea and vomiting, muscle drugs/foods.
containing antacids or laxatives weakness, depressed reflexes, Increase fluid intake (unless
decreased pulse and respirations contraindicated).
Hypomagnesemia Low intake of magnesium in diet, Paresthesias, confusion, Provide good dietary sources
prolonged diarrhea, massive hallucinations, convulsions, ataxia, of magnesium.
diuresis, hypoparathyroidism tremors, hyperactive deep reflexes,
muscle spasm, flushing of the
face, diaphoresis

4 160 NCLEX-RN Review


53155_04_Ch04a_p143-263.qxd 2/26/09 6:50 PM Page 161

C. Water excess: hypo-osmolar imbalances; water excrete bicarbonate and the respiratory system
intoxication or solute deficit retains carbonic acid.
D. Water deficit: hyperosmolar imbalances; water See Table 4-6.
depletion or solute excess
E. Hyperkalemia: potassium excess, serum potassium
above 5.5 mEq/liter Table 4-6 Normal Blood Gas Values
F. Hypokalemia: potassium deficit, serum potassium
pH PaCO2 PaO2 HCO3
below 3.5 mEq/liter
G. Hypernatremia: sodium excess, serum sodium 7.357.45 3545 mm Hg 80100 mm Hg 2226 mEq/L
level above 148 mEq/liter
H. Hyponatremia: sodium deficit, serum sodium level
below 135 mEq/liter
I. Hypercalcemia: calcium excess, serum calcium Acid-Base Imbalances
level above 10.5 mg/dL
See Table 4-7.
J. Hypocalcemia: calcium deficit, serum calcium
A. Metabolic acidosis: a primary deficit in the
level below 8.5 mg/dL
concentration of base bicarbonate in the
K. Hypermagnesemia: magnesium excess, serum
extracellular fluid; decreased pH and bicarbonate,
magnesium level above 2.7 mEq/liter
decreased pCO2 (if respiratory compensation)
L. Hypomagnesemia: magnesium deficit, serum
B. Metabolic alkalosis: a primary excess of base
magnesium level below 1.8 mEq/liter
bicarbonate in the extracellular fluid; elevated pH
and bicarbonate, elevated pCO2 (if respiratory
compensation)
ACID-BASE BALANCE C. Respiratory acidosis: a primary excess of carbonic
acid in the extracellular fluid; decreased pH,
Basic Principles elevated pCO2 and bicarbonate (if renal
compensation)
A. Normal pH of the body is 7.357.45.
D. Respiratory alkalosis: a primary deficit of carbonic
B. Buffer or control systems maintain normal pH.
acid in the extracellular fluid; elevated pH,
In acidic state, kidneys excrete acids and reabsorb
decreased pCO2 and bicarbonate (if renal
bicarbonate while the respiratory system gives off
compensation)
carbon dioxide. In alkalotic states, the kidneys

Table 4-7 Acid-Base Imbalances

Imbalance Causes Assessment Findings Nursing Interventions


Metabolic Diabetic ketoacidosis, uremia, Headache, nausea and vomiting, Administer sodium bicarbonate
acidosis starvation, diarrhea, severe weakness, lethargy, disorientation, as ordered and monitor for
infections, renal tubular tremors, convulsions, coma signs of excess.
acidosis Monitor for signs of hyperkalemia.
Provide alkaline mouthwash
(baking soda and water) to
neutralize acids.
Lubricate lips to prevent dryness
from hyperventilation.
Measure I&O.
Institute seizure precautions.
Monitor arterial blood gases and
electrolytes.
Metabolic Severe vomiting, nasogastric Nausea and vomiting, diarrhea, Replace fluid and electrolyte
alkalosis suctioning, diuretic therapy, numbness and tingling of losses (potassium and chloride).
excessive ingestion of sodium extremities, tetany, bradycardia, Institute seizure precautions.
bicarbonate, biliary drainage decreased respirations Measure I&O.
Assess for signs of hypokalemia.
Monitor arterial blood gases and
electrolytes.

(continues)

ADULT NURSING

4 161
53155_04_Ch04a_p143-263.qxd 2/26/09 6:50 PM Page 162

Table 4-7 Acid-Base Imbalances (continued)

Imbalance Causes Assessment Findings Nursing Interventions


Respiratory COPD, barbiturate or sedative Headache, weakness, visual Place in semi-Fowlers position.
acidosis overdose, acute airway disturbances, rapid respirations, Maintain patent airway.
obstruction, weakness of confusion, drowsiness, Turn, cough, and deep breathe.
respiratory muscles tachycardia, coma Perform postural drainage.
Administer fluids to help liquefy
secretions (unless
contraindicated).
Administer low-concentration
oxygen therapy.
Monitor arterial blood gases and
electrolytes.
Administer prophylactic
antibiotics as ordered.
Respiratory Hyperventilation, mechanical Numbness and tingling of mouth Offer reassurance.
alkalosis overventilation, encephalitis and extremities, inability to Encourage breathing into a paper
concentrate, rapid respirations, bag or voluntary breath
dry mouth, coma holding.
Ensure adequate rest.
Provide sedation as ordered.
Monitor mechanical ventilation,
arterial blood gases, and
electrolytes.

E. Compensation Nursing Interventions


1. If the metabolic system is causing the problem,
the respiratory system attempts to correct the A. Select correct solution after checking physicians
problem or compensate. order.
a. Compensation for metabolic acidosis is B. Note clarity of solution.
known as Kussmaul or air hunger C. Calculate flow rate (Intravenous Calculations, Unit 2).
respirations in which client breathes Time-tape bag to assist in monitoring flow rate.
deeply to blow off acid (CO2). D. Assess infusion rate and site at least hourly.
b. Compensation for metabolic alkalosis is E. Use infusion pump if administering medications
slow breathing to conserve CO2. (e.g., aminophylline, heparin, insulin).
2. If the respiratory system is causing the F. Maintain I&O record.
problem, the metabolic system attempts to G. Provide tubing change and IV site change
correct the problem or compensate. according to hospital policy. Intravenous Nurses
a. Compensation for respiratory acidosis is a Society standards recommend IV site and tubing
rise in HCO3. change every 48 hours.
b. Respiratory alkalosis is such a rapid H. Discontinue IV if complications occur.
process there might not be significant
compensation. Complications of Intravenous Therapy
See Table 4-8.
INTRAVENOUS THERAPY
Central Lines
Purposes Uses
A. Maintenance of fluid and electrolyte balance A. Administration of TPN
B. Replacement of fluid and electrolyte loss B. Measurement of central venous pressure (CVP)
C. Provision of nutrients C. IV therapy when suitable peripheral veins are not
D. Provision of a route for medications available
D. Long-term antibiotic therapy

4
E. Chemotherapy

162 NCLEX-RN Review


53155_04_Ch04a_p143-263.qxd 2/26/09 6:50 PM Page 163

infection with long-term use; catheter tip is located


Table 4-8 Complications of IV Therapy in the superior vena cava.
1. Hickman/Broviac catheters: single- or double-
Nursing
lumen catheters with external presentation;
Complication Manifestation Interventions
need to be flushed daily with a heparinized
Infiltration Blanching of skin, Discontinue IV. saline solution and must be clamped when not
swelling, pain at Restart in a new in use; repair kit available.
site; cool to touch; site. May apply 2. Groshong catheters: similar to Hickman/
decreased infusion warm compresses Broviac; difference is in valve at closed distal
rate to increase fluid end of catheter that opens when used and
absorption. remains closed at other times, preventing
blood backup into catheter; no clamping is
Extravasation Infusion of a vesicant Discontinue IV.
necessary; flushing is done daily with saline in
into the Restart in a new
a vigorous manner.
surrounding tissue; site. May apply
3. Implantable ports (portacath): totally internal
redness, heat, warm compress.
device consists of subcutaneous self-sealing
pain at site
injection port and a tunneled catheter;
Phlebitis Redness, heat, and Discontinue IV. flushing is done with a heparinized saline
swelling at site; Restart in new solution every 28 days; access must be with a
possible pain and site. Apply warm special noncoring needle.
red line along compresses to 4. Peripherally inserted central catheter (PICC):
course of vein site. short-term long lines that can be inserted by
Pyrogenic Fever, chills, Discontinue qualified nurses; inserted via a vessel in the
reaction general malaise, infusion antecubital fossa (median or cephalic);
nausea, vomiting, immediately. flushing is with a heparinized saline solution.
headache, Monitor vital
backache signs and notify Care of the Client with a Central
physician. Retain
IV equipment
Venous Line (CVL)
for culture/lab A. Assist physician with placement; catheters should
study. initially be flushed with saline. Have fluids or cap
Air embolism Dyspnea, cyanosis, Stop infusion and flush available (heparin or saline).
hypotension, immediately. B. Confirm placement in superior vena cava by X-ray
tachycardia, Turn client on prior to catheter use.
loss of left side with his C. Institute nursing measures to prevent infection
consciousness head down. (particularly important with TPN because high
Administer concentration of glucose encourages growth of
oxygen. Notify bacteria).
physician. 1. Change dressings
a. Usually 3 times per week and as needed
Circulatory Apprehension, Slow down IV rate. (e.g., when loose or wet) but agency
overload shortness of Monitor vital policies may vary.
breath, coughing, signs. Notify b. Use sterile technique and apply sterile
frothy sputum, physician. occlusive dressing.
crackles, engorged 2. Monitor for signs of infection: redness,
neck veins, drainage, odor at site, or elevated temperature.
increased blood 3. Do not piggyback anything into a TPN infusion
pressure and pulse line except intralipids.
D. Monitor for infiltration: check for swelling of neck,
face, and shoulder, and pain in upper arm.
Types E. Prevent catheter occlusion.
A. Nontunneled catheters: inserted into subclavian 1. Keep infusion continuous.
vein for short-term access 2. Use infusion pump.
1. Subclavian catheters: single lumen 3. Check for kinks in tubing.
2. Multilumen catheters: double, triple, or 4. Evaluate for catheter migration or
quadruple lumens for simultaneous infusion of dislodgment.
fluids or for blood drawing with fluid infusion. F. Prevent air embolism.
B. Tunneled catheters: long, silicone catheter 1. Tighten and tape all tubing connections to
threaded through subcutaneous layer to prevent prevent accidental disconnection.

ADULT NURSING

4 163
53155_04_Ch04a_p143-263.qxd 2/26/09 6:50 PM Page 164

2. Clamp catheter (except Groshong) and instruct Blood Transfusions


client to perform Valsalva maneuver when
changing or detaching tubing. See Blood Transfusions.
3. Check tubing for cracks or perforations.
G. Maintain proper infusion rate.
1. Monitor rate closely to prevent clotting, fluid SHOCK
depletion, or fluid overload.
2. Never attempt to speed up or slow down An abnormal physiologic state in which an imbalance
infusion. between the amount of circulating blood volume and
H. With a multilumen catheter, flush ports not being the size of the vascular bed results in circulatory
used to prevent clotting (per agencys protocol). failure and oxygen and nutrient deprivation of tissues.
I. With Hickman/Broviac catheters, Groshongs, See Table 4-9 for classification of shock.
PICCs, and implanted port provide other specific
care according to agency protocol. Bodys Response to Shock
J. If clotting occurs, try to aspirate or add a declotting
agent according to agency protocol. Do not irrigate. A. Hyperventilation leading to respiratory alkalosis
Do not use force to flush. B. Vasoconstriction: shunts blood to heart and brain
1. Streptokinase requires 1 hour waiting time to C. Tachycardia
achieve results. D. Fluid shifts: intracellular to extracellular shift to
2. Urokinase requires 10 minutes waiting time to maintain circulating blood volume
achieve results. E. Impaired metabolism: tissue anoxia leads to
K. When drawing blood specimens, discard initial anaerobic metabolism causing increased capillary
sample of 10 mL prior to drawing required volume permeability and lactic acid buildup, resulting in
for specimens. Flush with saline prior to flushing metabolic acidosis
with heparinized saline solution or continuing fluids. F. Impaired organ function

Table 4-9 Classification of Shock

Type Characteristics Causes


Hypovolemic Decreased circulating blood volume Blood loss
Plasma loss (e.g., burns)
Fluid loss (e.g., excessive vomiting or diarrhea)
Cardiogenic Failure of the heart to pump properly Myocardial infarction
Congestive heart failure
Cardiac arrhythmias
Pericardial tamponade
Tension pneumothorax
Septic Factors favoring septic shock: Release of bacterial toxins that act directly on the
Development of antibiotic-resistant organisms blood vessels producing massive vasodilation and
Invasive procedures such as urinary tract pooling of blood; results most frequently from
instrumentation gram-negative septicemia.
Immunosuppression and old age
Trauma: presence of blood in peritoneal
cavity greatly increases likelihood of peritonitis
Neurogenic Failure of arteriolar resistance, leading to massive Interruption of sympathetic impulses from:
vasodilation and pooling of blood Exposure to unpleasant circumstances
Extreme pain
Spinal cord injury
High spinal anesthesia
Vasomotor depression
Head injury
Anaphylactic Massive vasodilation resulting from allergic Allergic reaction to:
reaction causing release of histamine and Insect venom or snake venom
related substances Medications or food
Dyes used in radiologic studies

4 164 NCLEX-RN Review


53155_04_Ch04a_p143-263.qxd 2/26/09 6:50 PM Page 165

1. Kidney: decreased perfusion can result in absorption; when circulation improves,


renal failure. client may get overdose.
2. Lung: shock lung (adult respiratory distress 5. Keep client warm.
syndrome [ARDS]) E. Maintain continuous assessment of the client.
1. Check vital signs frequently.
2. Monitor urine output: report urine output of
Assessment Findings less than 30 mL/hour.
A. Skin 3. Observe color and temperature of skin.
1. Cool, pale, moist in hypovolemic and 4. Monitor CVP.
cardiogenic shock 5. Monitor ECG.
2. Warm, dry, pink in septic and neurogenic shock 6. Check lab studies: CBC, electrolytes, BUN,
B. Pulse creatinine, blood gases.
1. Tachycardia, due to increased sympathetic 7. Monitor other parameters such as arterial blood
stimulation pressures, cardiac output, pulmonary artery
2. Weak and thready pressures, pulmonary artery wedge pressures.
C. Blood pressure F. Provide psychologic support: reassure client to
1. Early stages: may be normal due to relieve apprehension, and keep family advised.
compensatory mechanisms
2. Later stages: systolic and diastolic blood
pressure drops Table 4-10 Drugs Used to Treat Shock
D. Respirations: rapid and shallow, due to tissue
anoxia and excessive amounts of CO2 (from Generic (Trade)
metabolic acidosis) Name Action
E. Level of consciousness: restlessness and Dopamine (Intropin) Low dosage: Dilates renal, mesenteric,
apprehension, progressing to coma and splanchnic vessels, which in turn
F. Urinary output: decreases due to impaired renal increases perfusion of kidneys and
perfusion urine output.
G. Temperature: decreases in severe shock (except High dosage: Increases cardiac
septic shock). contractility; causes vasoconstriction
(often given with nitroprusside
Nursing Interventions [Nipride]).
Dobutamine Increases myocardial contractility;
A. Maintain patent airway and adequate ventilation.
(Dobutrex) vasodilator.
1. Establish and maintain airway.
2. Administer oxygen as ordered. Isoproterenol Increases myocardial contractility;
3. Monitor respiratory status, blood gases. (Isuprel) decreases peripheral resistance
4. Start resuscitative procedures as necessary. by dilating peripheral vascular bed;
B. Promote restoration of blood volume; administer usefulness is limited by the
fluid and blood replacement as ordered. tachycardia it produces.
1. Crystalloid solutions: Ringers lactate, normal Norepinephrine Improves cardiac contractility and
saline (Levophed) cardiac output; potent
2. Colloid solutions: albumin, plasmanate, dextran vasoconstrictor.
3. Blood products: whole blood, packed red Sodium Vasodilator; decreases peripheral
blood cells, fresh frozen plasma nitroprusside resistance and workload of heart,
C. Administer drugs as ordered (see Table 4-10). (Nipride) thereby increasing cardiac output;
D. Minimize factors contributing to shock. used in cardiogenic shock and
1. Elevate lower extremities to 45 to promote hypertensive emergencies.
venous return to heart, thereby improving
cardiac output. Digitalis preparations Improves cardiac performance.
2. Avoid Trendelenburgs position: increases Corticosteroids Used especially in septic shock;
respiratory impairment. helps to protect cell membranes
3. Promote rest by using energy-conservation and decreases the inflammatory
measures and maintaining as quiet an response to stress.
environment as possible. Antibiotics Used in treating infectious processes
4. Relieve pain by cautious use of narcotics. related to septic shock.
a. Because narcotics interfere with
vasoconstriction, give only if absolutely Note: Vasopressors such as Levophed can cause almost
necessary, IV and in small doses. complete occlusion of arterioles, causing a decrease of blood
b. If given IM or subcutaneously, flow to larger tissue areas. Therefore, if blood pressure is

4
vasoconstriction may cause incomplete adequate, a vasodilator such as Nipride could probably be
given as well, to modify the vasoconstrictor effects.

ADULT NURSING 165


53155_04_Ch04a_p143-263.qxd 2/26/09 6:50 PM Page 166

MULTIPLE TRAUMA C. Inspect abdomen for injuries.


D. Auscultate bowel sounds.
E. Do not palpate the abdomen (could aggravate
Assessment and Emergency Care possible internal injuries).
Airway F. Prepare client for peritoneal lavage if indicated.
G. Insert Foley catheter.
A. Assess, establish, and maintain an adequate airway. 1. Measure urine output every 15 minutes.
1. Do not hyperextend the neck in a client with 2. Assess for hematuria.
suspected cervical spine injury.
2. Use jaw thrust instead. Musculoskeletal Injuries
B. Administer artificial resuscitation if necessary.
C. Observe for chest trauma such as open sucking A. Observe for sign of fracture: pain, swelling,
wounds or flail chest (see Chest Trauma). tenderness, ecchymosis, crepitation (grating
D. Administer high-flow oxygen 85100% to achieve sound), loss of function, exposed bone fragments.
maximum cellular oxygenation. Monitor for CO2 B. Cover open fracture with sterile dressing to
retention in clients with COPD. prevent infection.
E. Draw blood samples for ABGs. C. Immobilize any suspected fractures by splinting
the joint above and below the injury.
Hemorrhage and Shock D. Perform neurovascular check of area distal to
fracture: assess for color, temperature, capillary
A. Deep wounds with pulsating blood flow refill, sensation, movement, pulses.
1. Apply firm pressure over the wound with a
sterile dressing.
2. If wound is on a limb, elevate the extremity.
3. Apply pressure with three fingers over Sample Questions
appropriate pressure point.
4. Once bleeding is controlled, apply a pressure
dressing. 1. A client is admitted for treatment of Crohns
5. Tourniquets should be used only when all disease. Which information is most significant
other methods have failed. when the nurse assesses nutritional health?
B. Venous bleeding: apply direct pressure to bleeding 1. Anthropometric measurements.
site. 2. Bleeding gums.
C. Never remove any foreign object, such as a knife,
from the client; immobilize the object with 3. Dry skin.
packing. 4. Facial rubor.
D. Assess for and treat shock (see Shock).
E. Administer tetanus booster as ordered. 2. Total parenteral nutrition (TPN) is ordered for an
adult client. Which statement is true regarding
Neurologic Injuries TPN?
1. It contains 2070% glucose.
A. Establish a baseline level of consciousness and
2. The RN will mix the lipids into the TPN bag
reassess frequently.
before administration.
B. Inspect the scalp, head, face, and neck for
abrasions, hematomas, and lacerations. 3. When the clients blood glucose runs high,
C. Gently palpate the head for any injuries. decrease the TPN rate.
D. Inspect the nose and ears for leakage of 4. Check the clients potassium level every 6 hours.
cerebrospinal fluid.
E. Evaluate pupillary size, shape, equality, and 3. The nurse has administered analgesia, but the
reaction to light. client continues to complain of severe pain.
F. Assess for sensation and motor abilities. Which of the following actions should be most
G. Observe for signs of increased intracranial pressure. appropriate for the nurse to take first?
H. For additional details of care (see Head Injury and 1. Contact the physician to request additional
Spinal Cord Injury). medication.
2. Assist the client to a comfortable position
Abdominal Injuries and help her relax.
A. Keep client NPO. 3. Administer a narcotic analgesic.
B. Assist with insertion of nasogastric tube (for 4. Inform the client that she may request
assessment of stomach bleeding and aspiration of additional pain medication in 3 hours.
stomach contents, which prevents vomiting).

4 166 NCLEX-RN Review


53155_04_Ch04a_p143-263.qxd 2/26/09 6:50 PM Page 167

4. Acetylsalicylic acid is being administered to 4. After a bolus is administered, there is a


an adult client. What is the most common mandatory waiting period before another
mechanism of action for nonnarcotic analgesics? dose is given.
1. To inhibit prostaglandin synthesis.
10. A nurse is the first professional to arrive at the
2. To alter pain perception in the cerebellum.
scene of a multivehicle accident. An adult was
3. To directly affect the central nervous system. riding a motorcycle. Upon impact, he fell off the
4. To target the pain-producing effect of kinins. bike and it fell back on his legs. What should be
the RNs priority action?
5. An adult has been taking acetylsalicylic acid
1. Assessing blood loss.
(ASA) 650 mg four times a day for chronic back
pain. What complication is possible due to this 2. Monitoring respiratory status.
level of ASA ingestion? 3. Obtaining vital signs.
1. Liver failure. 4. Removing the bike off him.
2. Paralytic ileus.
11. A nurse stops to help at a motorcycle accident in
3. Gastrointestinal bleeding. which an adult is bleeding profusely from a 4-inch
4. Retinal detachment. gash on the left knee. Which of the following is the best
approach for the nurse to take to stop the bleeding?
6. Ibuprofen (Motrin) is prescribed for an adult
1. Apply direct pressure to the wound.
with chronic pain. The nurse must teach the
client to observe which dietary precaution while 2. Move the motorcycle off his legs.
taking ibuprofen? 3. Raise the extremity.
1. Eat a high-fiber diet. 4. Wrap a tourniquet above the wound.
2. Drink citrus juices daily.
12. The nurse is caring for a client who is receiving
3. Take the medication with milk. IV fluids. Which observation by the nurse
4. Omit spinach and other green leafy indicates the IV has infiltrated?
vegetables from the clients diet. 1. Pain at the site.
7. A post-op surgical client is ordered patient- 2. A change in flow rate.
controlled analgesia (PCA). Which statement by 3. Coldness around the insertion site.
the RN will help reduce anxiety about receiving 4. Redness around the insertion site.
adequate pain relief?
1. A PCA is almost always effective. 13. The nurse is caring for a client whose arterial
blood gases indicate metabolic alkalosis. Which
2. Your comfort will be assessed frequently.
of the following is most likely to cause metabolic
3. Have your family push the button for you. alkalosis?
4. We will have Narcan available if you get too 1. Nasogastric suctioning.
much.
2. Diabetic ketoacidosis.
8. Preoperative teaching for an adult who is to have 3. COPD.
patient controlled analgesia (PCA) following 4. Renal failure.
surgery includes telling the client:
1. You will not be drowsy. 14. An adult client is admitted with metabolic
acidosis. Which set of arterial blood gases
2. You will experience no pain.
should the nurse expect to find in a client with
3. Pain control will be adequate. metabolic acidosis?
4. You will not have incisional pain but you 1. pH 7.28; PCO255; HCO326.
may have muscle pain.
2. pH 7.50; PCO240; HCO331.
9. The clients family expresses concern that the 3. pH 7.48; PCO230; HCO322.
client could overdose with a PCA. What protective 4. pH 7.30; PCO236; HCO318.
mechanism prevents drug overdose with a PCA?
1. The nurse controls the amount administered 15. A elderly client is hospitalized for the treatment
with each dose. of gastroenteritis complicated by dehydration
and hyponatremia. What is an early symptom of
2. Extensive client teaching precedes its use.
hyponatremia that the nurse would expect from
3. The client can stop drug administration but this client?
not initiate it.

4
1. Ataxia.
2. Hunger.

ADULT NURSING 167


53155_04_Ch04a_p143-263.qxd 2/26/09 6:50 PM Page 168

3. Thirst. 4. That the formula is used directly from the


4. Weakness. refrigerator.

16. An adult has just been brought in by ambulance 22. Which food choice contains the highest
after a motor vehicle accident and has moderate kilocalorie?
anxiety. When assessing the client, the nurse 1. Apple.
would expect which of the following from 2. Bacon.
sympathetic nervous system stimulation? 3. Chicken.
1. A rapid pulse and increased respiratory rate. 4. Bread.
2. Decreased physiologic functioning.
3. Rigid posture and altered perceptual focus. 23. The nurse knows that a client understands a low
4. Increased awareness and attending. residue diet when he selects which of the
following from a menu?
17. An adult has received an injection of immunoglo- 1. Rice and lean chicken.
bulin. The nurse knows that the client will develop 2. Strawberry pie.
which of the following types of immunity? 3. Pasta with vegetables.
1. Active natural immunity. 4. Tuna casserole.
2. Active artificial immunity.
3. Passive natural immunity. 24. An adult is receiving total parenteral nutrition
4. Passive artificial immunity. (TPN). The nurse knows which of the following
assessments is essential?
18. The nurse knows which of the following is true 1. Evaluation of the peripheral intravenous (IV)
about immunity? site.
1. Antibody-mediated defense occurs through 2. Confirmation that the tube is in the stomach.
the T-cell system. 3. Assessment of the GI tract, including bowel
2. Cellular immunity is mediated by antibodies sounds.
produced by the B-cells. 4. Fluid and electrolyte monitoring.
3. Antibodies are produced by the B-cells.
4. Lymphocytes increase with an allergic response. 25. The nurse knows which of the following
statements about TPN and peripheral parenteral
19. An adult is on a clear liquid diet. Which food nutrition (PPN) is true?
item can be offered? 1. TPN is usually indicated for clients needing
1. Milk. short-term (less than 3 weeks) nutritional
2. Jello. support, whereas PPN is for long-term
maintenance.
3. Orange juice.
2. A client needing more than 3000 calories
4. Ice cream.
would receive PPN, whereas TPN is given to
20. An adult is being taught about a healthy diet. those requiring less than 3000 calories.
How can the food pyramid help guide the client 3. TPN is often given to those with fluid
on his diet? restrictions, whereas PPN is used for those
1. By indicating exactly how many servings of without constraints on their fluid intake.
each group to eat. 4. TPN is given to those who need to augment
2. By calculating how many calories the client oral feedings, whereas PPN is used for those
should have. who are nothing by mouth (NPO).
3. By suggesting daily food choices. 26. What is an important consideration regarding
4. By dividing the food into four basic groups. TPN administration?
1. IV site is kept aseptic while infusing the
21. Before administering a tube feeding the nurse
solution.
knows to perform which of the following
assessments? 2. Feeding is poured into a pouch and then
infused.
1. The gastrointestinal (GI) tract, including
bowel sounds, last BM, and distention. 3. Solution is only hung for a maximum of
8 hours at a time.
2. The clients neurologic status, especially gag
reflex. 4. New formula is added as needed so the line

4
does not run dry.
3. The amount of air in the stomach.

168 NCLEX-RN Review


53155_04_Ch04a_p143-263.qxd 2/26/09 6:50 PM Page 169

27. An adult has been treated for pulmonary 32. An adult is to receive narcotic analgesic via a
tuberculosis (TB) and is being discharged home patient-controlled analgesia (PCA). The nurse is
with his wife and two young children. His wife evaluating the clients understanding of the
asks how TB is passed from one person to procedure. Which of the following statements by
another so she can prevent anyone else from the client indicates that she understands PCA?
catching it. How should the nurse respond? 1. When I press this button the machine will
1. You should wear gloves when handling his always give me more medicine.
linen and bedding. 2. I will press the button whenever I begin to
2. You should keep the windows and doors experience pain.
closed so as not to spread the droplets. 3. I should press this button every hour so the
3. He must be careful to cough into a pain doesnt come back.
handkerchief that is washed in hot water or 4. With this machine I will experience no more
discarded. pain.
4. Make sure to boil all water before drinking
or using it. 33. An adult suffered second- and third-degree
burns over 20% of his body 2 days ago. What is
28. The nurse evaluates a certified nursing assistant the best way to assess the clients fluid balance?
(CNA). Which of the following actions by the 1. Maintain strict records of intake and output.
CNA demonstrates understanding of standard 2. Weigh the client daily.
precautions?
3. Monitor skin turgor.
1. Wears gloves during all client contact.
4. Check for edema.
2. Cleans blood spills with soap and water.
3. Pours bulk blood and other secretions down a 34. A 78-year-old male has been working on his lawn
drain connected to a sanitary sewer. for 2 days, although the temperature has been
4. Carries blood sample to the lab in an open above 90F. He has been on thiazide diuretics for
basket. hypertension. His lab values are: K 3.7 mEq/L,
Na 129 mEq/L, Ca 9.9 mg/dL, and Cl 95 mEq/L.
29. An adult is on long-term aspirin therapy and is What would be a priority action for this man?
experiencing tinnitus. What is the best 1. Make sure he drinks eight glasses of water a
interpretation of this occurrence? day.
1. The aspirin is working correctly. 2. Monitor for fatigue, muscle weakness,
2. The client has a metal taste in their mouth. restlessness, and flushed skin.
3. The client has an upper GI bleed. 3. Look for signs of hyperchloremia.
4. The client is experiencing a mild overdosage. 4. Observe for neurologic changes.

30. An adult is receiving a nonsteroidal anti- 35. An adult who has gastroenteritis and is on
inflammatory drug (NSAID). Which of the digitalis has lab values of: K 3.2 mEq/L,
following would the nurse include in the Na 136 mEq/L, Ca 8.8 mg/dL, and Cl 98 mEq/L.
teaching about this medication? The nurse puts which of the following on the
1. Take the NSAID with ASA for full effect. clients plan of care?
2. Take the NSAID with meals. 1. Stop digitalis therapy.
3. Orange juice will help potential the action of 2. Avoid foods rich in potassium.
the medication. 3. Observe for digitalis toxicity.
4. The NSAID will coat the stomach lining. 4. Observe for Trousseaus and Chvosteks signs.

31. An adult is to receive an intramuscular (IM) 36. A client on hemodialysis is complaining of


injection of morphine for post-op pain. Which of muscle weakness and numbness in his legs.
the following is necessary for the nurse to assess His lab results are: Na 136 mEq/L, K 5.9 mEq/L,
prior to giving a narcotic analgesic? Cl 100 mEq/L, Ca 8.5 mg/dL. Which electrolyte
1. The clients level of alertness and respiratory imbalance is the client suffering from?
rate. 1. Hyperkalemia.
2. The last time the client ate or drank something. 2. Hypernatremia.
3. The clients bowel habits and last bowel 3. Hypocalcemia.
movement. 4. Hypochloremia.

4
4. The clients history of addictions.

ADULT NURSING 169


53155_04_Ch04a_p143-263.qxd 2/26/09 6:50 PM Page 170

37. A client has cancer that has metastasized to her 1. The potassium bag is piggybacked into the
bones. She is complaining of increased thirst, dextrose at 75 mL/h.
polyuria, and decreased muscle tone in the legs. 2. The clamp should be closed below the D5 12
Her lab values are: Na 139 mEq/L, K 4.0 mEq/L, NS bag.
Cl 103 mEq/L, and Ca 8.0 mg/dL. What 3. Potassium is on the secondary line.
electrolyte imbalance is present?
4. 75 mL will infuse in 1 hour.
1. Hypocalcemia.
2. Hypercalcemia. 43. An adult has a central venous line. Which of the
3. Hyperkalemia. following should the nurse include in the care
4. Hypochloremia. plan?
1. Complete blood count (CBC) and electrolytes.
38. An adult who is anxious and hyperventilating 2. Regular serial chest X-rays to ensure proper
has blood gases of: pH 7.47, PaCO2 33. What is placement of the central line.
the best initial action for the nurse to take? 3. Continuous infusion of a solution at a keep
1. Try to have the client breathe slower or into vein open rate.
a paper bag. 4. Any signs of infection, air embolus, and
2. Monitor the clients fluid balance. leakage or puncture.
3. Give O2 via nasal cannula.
4. Administer sodium bicarbonate. 44. An adult has a Hickman-type central venous
catheter and needs to have blood drawn from it.
39. An adult has had gastroenteritis with vomiting Which of the following is the nurse going to do
for 3 days. He has taken baking soda without first?
relief. His blood gases are as follows: pH 7.49, 1. Use sterile technique to assemble the
PaCO2 45, and HCO3 30. The nurse would expect supplies needed.
which of the following to be included in the 2. Aspirate and discard the first 10 mL of the
plan of care? blood.
1. Have the client drink at least eight glasses of 3. First flush the catheter with heparinized
water in the first day. solution, then withdraw the blood.
2. Administer NaHCO3 IV as per physicians 4. Remove the cap on the catheter and replace it
orders. with a new one.
3. Continue sodium bicarbonate for nausea.
4. Monitor electrolytes for hypokalemia and 45. An adult has a central line in his right
hypocalcemia. subclavian vein. The nurse is to change the
tubing. Which of the following should be done?
40. An adults blood gas results are: pH 7.31, PaCO2 1. Use the present solution with the new tubing.
49, and HCO3 24. What does the nurse interpret 2. Connect the new tubing to the hub prior to
this as? running any fluid through the tubing.
1. Respiratory acidosis. 3. Close the roller clamp on the new tubing after
2. Respiratory alkalosis. priming it.
3. Metabolic acidosis. 4. Have the client roll to the right side to
4. Metabolic alkalosis. prevent an air embolus.

41. An adult who has diabetes has infectious 46. An adult suffered a diving accident and is being
diarrhea. His arterial blood gases are: pH 7.30, brought in by an ambulance intubated and on a
PaCO2 35, and HCO3 of 19. The nurse would backboard with a cervical collar. What is the first
monitor the client for which of the following? action the nurse would take on arrival in the
1. Trousseaus sign. hospital?
2. Hypokalemia. 1. Take the clients vital signs.
3. Hypoglycemia. 2. Insert a large bore IV line.
4. Respiratory changes. 3. Check the lungs for equal breath sounds
bilaterally.
42. An adult has an IV line in the right forearm 4. Perform a neurologic check using the
infusing D5 12 NS with 20 mEq of potassium at Glasgow scale.
75 mL/h. Which statement would be a correct

4
report from the RN?

170 NCLEX-RN Review


53155_04_Ch04a_p143-263.qxd 2/26/09 6:50 PM Page 171

47. An adult has been shot. His vital signs are blood 3. 2. Medication is usually more effective with
pressure (BP) 90/60, pulse (P) 120 weak and relieving techniques. Many basic nursing
thready, respirations (R) 20. During the initial measures reduce or eliminate discomfort.
assessment, he is placed in a modified Administering analgesia alone does not replace
Trendelenburg position. What desired effect thoughtful, comprehensive pain management.
should the position have on the client?
1. An increase in the clients blood pressure. 4. 1. Nonnarcotic analgesics inhibit prostaglandin
synthesis. Prostaglandins increase the sensitivity
2. An increase in the clients heart rate.
of peripheral pain receptors to endogenous pain-
3. An increase in the clients respiratory rate. producing substances.
4. A decrease in blood loss.
5. 3. High doses of aspirin are associated with GI
48. An adult has been stung by a bee and is in bleeding.
anaphylactic shock. An epinephrine (adrenaline)
injection has been given. The nurse would 6. 3. NSAIDs such as ibuprofen are very irritating to
expect which of the following if the injection the GI tract and should always be taken with milk
has been effective? or food to minimize the possibility of bleeding.
1. The clients breathing will become easier.
7. 2. Pain is an individual experience. It is
2. The clients blood pressure will decrease. important to reassure the client that assessments
3. There will be an increase in angioedema. will be made frequently and that drug dosages
4. There will be a decrease in the clients level will be adjusted according to the amount of pain
of consciousness. the client perceives.

49. An adult who was in a motor vehicle accident, 8. 3. Clients should be told that they will be able to
has been brought to the emergency department. control their pain.
She has a 4-in laceration on her forehead that is
bleeding profusely. Her left ankle has an obvious 9. 4. Immediately after a bolus dose of medication
deformity and is splinted. Her vital signs are BP is administered the device enters a mandatory
100/60, P 110, and R 16. What is the first action lockout mode where no other boluses of
the nurse should take? medication can be delivered.
1. Start an IV line for fluids. 10. 2. In the presence of multiple traumas,
2. Place a Foley catheter. maintenance of a patent airway must always be
3. Get an ECG. the priority in the sequence of care delivery.
4. Check her neurologic status.
11. 1. Direct pressure to the wound will aid in the
50. An adult is brought in by ambulance after a development of a blood clot, which is the first step
motor vehicle accident. He is unconscious, on a in wound healing and will prevent hemorrhage.
backboard with his neck immobilized. He is
bleeding profusely from a large gash on his right 12. 3. Coldness, pallor, and swelling around the
thigh. What is the first action the nurse should take? insertion site are the best indicators that the
fluid has infiltrated into subcutaneous tissue.
1. Stop the bleeding.
2. Check his airway. 13. 1. One cause of metabolic alkalosis is removal of
3. Take his vital signs. H+ and Cl- from the stomach through emesis or
4. Find out what happened from eyewitnesses. gastric suction, resulting in an excess of base.

14. 4. The pH is below the normal range of 7.357.45.


The PCO2 is within the normal range of 3545,
and the HCO3 is below the normal limits of
Answers and Rationales 2128. These values indicate a metabolic problem
because the PCO2 is within normal limits. It is the
low HCO3 that is causing acidosis.
1. 1. Anthropometric measurements are the prime
parameters used to evaluate fat and muscle 15. 3. Thirst is the bodys attempt to restore blood
stores in the body. volume depletion that occurs in hyponatremia.

2. 1. The concentration for dextrose in TPN can 16. 1. The sympathetic nervous system during

4
range from 2070%. moderate anxiety will increase the pulse and
respirations.

ADULT NURSING 171


53155_04_Ch04a_p143-263.qxd 2/26/09 6:50 PM Page 172

17. 4. Passive artificial immunity occurs when should be taken prior to giving the medication
antibodies are produced by another person or for baseline purposes.
animal and injected into the recipient.
32. 2. PCA allows the client to administer more
18. 3. Antibodies or immunoglobulins are produced analgesic before the pain becomes severe, thus
by the B cells and are part of the bodys plasma allowing better pain control.
proteins.
33. 2. This is the best way to assess fluid balance,
19. 2. Plain gelatins can be given on a clear liquid especially acute changes in those with large
diet, as well as tea, coffee, ginger ale, or losses or acutely ill.
lemon-lime soda.
34. 4. Neurologic changes can occur from
20. 3. The pyramid helps to guide the client in hyponatremia. They include confusion,
choosing a variety of foods to obtain the disorientation, lethargy, seizures, and coma.
nutrients needed. It also aids in eating more of
some groups (bread, cereal, rice, and pasta) and 35. 3. Hypokalemia enhances digitalis toxicity and
less of others (fats, oils, and sweets). must be observed for carefully.
21. 1. The GI tract should be assessed before each 36. 1. Potassium is normally 3.55.5 mEq/L. Clients
feeding to ensure functioning and minimal with renal failure are prone to hyperkalemia.
problems.
37. 1. The clients calcium is low. The normal values
22. 2. Bacon contains the highest kilocalorie, as it is
are 8.510.5 mg/dL. Hypocalcemia is common
from the fat group. Fats yield 9 kcal/g, whereas
among those with bone cancer.
the other choices, from the carbohydrate and
protein groups only yield 4 kcal/g. 38. 1. The client is in respiratory alkalosis and needs
to increase the carbon dioxide. The easiest way
23. 1. A low-residue diet includes rice, lean meats,
to do this is to try and calm the client and/or
and eggs.
have him breathe in and out of a paper bag, thus
24. 4. Clients receiving TPN can experience inhaling the exhaled carbon dioxide.
electrolyte imbalances, hypo- or hyperglycemia,
as well as difficulties with fluid balance. 39. 4. Hypokalemia and hypocalcemia are both
common with metabolic alkalosis as a result of
25. 3. TPN can provide a greater concentration of cellular buffering.
calories than PPN. Therefore, TPN is given to
those with fluid restrictions. 40. 1. A low pH indicates acidosis, whereas the high
PaCO2 indicates the problem is respiratory
26. 1. The IV site is kept aseptic by an occlusive rather than metabolic.
dressing. It is a central line and the TPN with its
high concentration of glucose provides an ideal 41. 4. The client is in metabolic acidosis and the
medium for pathogens. body will try to compensate through the
respiratory system (with deep breaths), although
27. 3. TB is spread through residue of evaporated it cannot completely correct the problem.
droplets and may remain in the air for long
periods of time. Thus care should be given when 42. 4. The IV fluids will infuse 75 mL/h, as the rate
coughing or sneezing. states. The potassium has already been mixed in
the bag from pharmacy and infuses from one bag.
28. 3. Bulk blood and other secretions like suctioned
fluids are carefully poured down a drain 43. 4. All of these are potential problems for those
connected to a sanitary sewer. with a central line, which the nurse needs to be
observant for.
29. 4. Tinnitus is a classic sign of aspirin
overdosages, either from too much ingestion or 44. 2. The first 10 mL are drawn off and discarded. Lab
limited excretion. values can be altered by the solution remaining in
the catheter from the infusion or flush.
30. 2. NSAIDs should be taken with food, milk, or
antacid to prevent nausea or vomiting. 45. 3. The roller clamp should be closed after
priming, otherwise the fluid will continue to
31. 1. A decreasing level of alertness can signal early
flow. Open the roller clamp after inserting the
respiratory depression and a significant drop in
tubing into the hub.

4
the respiratory rate is a warning sign. Both

172 NCLEX-RN Review


53155_04_Ch04a_p143-263.qxd 2/26/09 6:50 PM Page 173

46. 3. The airway is provided by the endotracheal 48. 1. The epinephrine would help to ease the
tube. The nurse should assess breathing, the clients respiratory distress.
next step in the ABCs.
49. 1. Her vital signs indicate that she is probably
47. 1. The Trendelenburg position increases the going into shock. Fluids are the first action to
blood return from the legs, thereby raising the do after assessing ABCs.
blood pressure.
50. 2. Airway is the first step of ABCs.

Aging

GENERAL INFORMATION 2. Life expectancies among family members is


similar, i.e., if the parents died over the age of 80,
Aging is a normal developmental process occurring the children are more likely to live to that age.
throughout the human life span that causes a mild F. Neuroendocrine theories
progressive decline in body system functioning. 1. Anterior pituitary hormones are thought to
The older client is generally regarded as one who is contribute to the aging process.
65 years of age or older. 2. An imbalance of certain chemicals in the brain
may contribute to altered cell division within
the body.
Biologic Theories of Aging
A. Immune system theory Age-Related Changes
1. The two primary immune organs, the thymus
and bone marrow, are affected by the aging See Table 4-11.
process, which contributes to a decline in
T-cell production and stem cell efficiency. Psychosocial Changes
2. Increase of infections, autoimmune disease,
and cancer with aging. in the Older Adult
B. Cross-linking theory A. A successful aging process includes physical,
1. Cross-linking is a chemical reaction that binds psychological, social, and cultural factors.
glucose to protein, which causes abnormal 1. Some cultures have a great respect for older
division of DNA, interfering with normal cell persons.
functioning and intracellular transport over a 2. In the United States, much value is placed on
lifetime. youth.
2. Eventually causes tissue and organ failure. 3. Be aware of ageism (discrimination against the
C. Free radical theory older adult simply because of age).
1. Molecules that are highly reactive as a result B. Developmental tasks for the older adult
of oxygen metabolism in the body. 1. Ego integrity vs. despair (Erikson)
2. Over time, cause physical decline by a. With ego integrity, the persons life is felt
damaging proteins, enzymes, and DNA. to have meaning and accomplishment.
3. Beta-carotene and vitamins C and E are b. With despair, there are feelings of
naturally occurring antioxidants that worthlessness for a life not well lived.
counteract the free radicals. 2. Other possible developmental tasks
D. Stress theory (wear and tear) a. Successfully adjusting to retirement.
1. The body, like any machine, will eventually b. Making safe and satisfactory living
wear out secondary to repetitive usage, arrangements.
damage, and stress. c. Adjusting to reduced income.
2. While this theory is seen as having some d. Keeping socially active.
merit, individuals react differently to stress e. Maintaining contact with friends and
(positive and negative), causing controversy family.
over the concept. f. Making safe decisions about driving a car.
E. Genetics theory g. Adjusting to death of spouse/significant
1. Preprogrammed life expectancy. Cells can

4
other.
only divide a specific number of times. h. Adjusting to idea of ones own death.

ADULT NURSING 173


53155_04_Ch04a_p143-263.qxd 2/26/09 6:50 PM Page 174

Table 4-11 Age-Related Changes

System Change Nursing Interventions


Special Senses
Vision Presbyopia: decreased ability to focus on near Provide increased illumination without glare.
objects (often requiring reading glasses) Provide safe environment by orienting client to
Decreased lacrimal secretionschronic dry eye, surroundings and removing potential hazards.
scratchiness
Pupils: smallerdecreased peripheral vision and ability Use sunglasses outdoors.
to adapt to the dark; increased sensitivity to glare Use large-print books.
Lens: larger, more rigid, and discolored (yellow Avoid night driving.
opacity)decreased depth perception and ability
to focus
Colors distorted, especially blue/green
Red/orange more pleasing
Hearing Presbycusis: decreased ability to hear pitch or level Look directly at the client when speaking and speak
of sound (first high, then low and background) clearly and slowly; low-pitched voice heard best.
Tinnitusringing in the ear; results from decreased Decrease background noise.
blood supply to neurosensory receptors in ear;
problems distinguishing horns/sirens
Taste/Smell Olfactory fibers atrophydecreased sense of Provide attractive meals in comfortable social setting.
smell, decreased appetite/ability to enjoy foods Vary taste, textures, and colors of foods.
Be alert for difficulty chewing or swallowing when
selecting foods.
Touch Sensory nerve receptors less acuterequires stronger Protect skin from injury.
stimuli, increased pain tolerance, skin tears, more Lower bath water temperature to 100105F.
difficult to distinguish hot, cold, or pressure Provide for safety around hot liquids at mealtimes.
Fine discrimination abilities impaired, especially
hands and feet
Nervous Overall intellect remains the same Promote independence in daily activities.
Fewer neurons and reduced blood flow to brain Allow ample time for completion of tasks.
some short-term memory loss and learning Offer back rub or warm milk at bedtime.
ability slowed Provide recreational and diversional activities.
Myelin sheath degeneratesdecreased reaction Maintain environmental stability, minimize
time, reduced deep tendon reflexes, and frequency of transfers.
increased time to respond to stimuli
Change in sleep patterns
Integumentary Sweat glands diminishdecreased thermoregulation Provide adequate warmth.
Collagen and subcutaneous fat decreases Maintain adequate hydration.
(subcutaneous medications absorb more Avoid overexposure to the sun.
slowly)wrinkles, poor turgor (poor estimate Provide adequate heat and humidity in
of hydration) environment.
Hair follicles decrease/produce less melanin Keep skin clean, dry, lubricated, and pressure free.
baldness/gray hair Decrease frequency of baths.
Vascular supply to nailbeds reduceddull, brittle Refer to podiatrist.
nailshard to cut!
Delayed wound healing
Musculoskeletal Muscle fibers decrease and muscles atrophy Encourage exercise program.
decreased strength and endurance Promote optimum physical activity within level of
Bone density decreasesosteoporosis, increased ability.
fractures Maintain optimum nutrition, especially intake of
Ligaments and tendons lose elasticitydecreased protein, calcium, and vitamins.
ROM in joints Encourage use of appropriate adaptive or assistive
Intervertebral disks narrowincreased spine devices to enhance mobility.
curves, balance diminishes (center of gravity)

4 174 NCLEX-RN Review


(continues)
53155_04_Ch04a_p143-263.qxd 2/26/09 6:50 PM Page 175

Table 4-11 Age-Related Changes (continued)

System Change Nursing Interventions


Cardiovascular Increased peripheral resistance/increased blood Assess symptoms and make appropriate
pressure, especially systolic modifications in care.
Baroreceptors less sensitivedecreased sensitivity Teach client to change positions slowly to avoid falls.
to change in positions (orthostatic hypotension) Minimize edema and fatigue with rest periods and
Decreased venous valve competencyincreased elevation of legs.
dependent edema Teach energy conservation methods in daily
Mitral/aortic valves thicker and more rigidmore activities.
murmurs without disease
Decreased stroke volume and cardiac output
Decreased pacemaker cellspossible dysrhythmias
Respiratory Muscles weaken and atrophy, rib cage calcifies, Manipulate environment to enhance ventilation.
barrel-shaped chestincreased energy to expand Position client to promote optimum ventilation.
lungs, harder to cough and deep breathe Encourage exercises and prescribed pulmonary
Less tidal volume and increased residual volume exercises.
secondary to cell fibrosis Encourage annual influenza vaccines and one-time
Alveoli decrease and thickenless sensitive to pneumococcal vaccine.
hypoxia and hypercapnia
Atrophy of ciliaslowed cough reflex, increased
risk of infection
Gastrointestinal Decreased smooth muscle tone, difficulty in Assess condition of teeth and mouth, fit and
swallowing (minor) and decreased peristalsis comfort of dentures, and ability to chew.
decreased esophageal motility, increased Encourage fluids and foods high in fiber.
heartburn and constipation Encourage optimal activity.
Decrease in digestive enzymesaltered absorption Promote independence and privacy in use of
of fats, protein, B12, folic acid, calcium, iron, bathroom.
medications Keep stool record and observe for constipation.
Decreased saliva, loss of teeth
Decreased sphincter toneimpactions, incontinence
Renal Decreased GFR secondary to decreased kidney size Assess voiding patterns.
and number of nephrons and decreased renal Provide adequate fluids.
blood flowability to concentrate/dilute urine Establish a bladder program to promote continence
decreased, decreased excretion of medications (assist to bathroom or offer bedpan every
Decreased bladder capacity and weakened bladder 23 hours).
and pelvic musclesfrequency, urgency, nocturia, Avoid catheterization unless comatose, skin
incontinence, retention, infections breakdown, or bladder outlet obstruction.
Prostate enlargement/obstruction (retention),
dribbling, overflow incontinence
Reproductive
Female Diminished vaginal secretions secondary to Promote good perineal care, treat with prescribed
decreased estrogenpainful intercourse, infections creams (e.g., estrogen).
Use vaginal lubricant as needed.
Male Slower erections and ejaculations secondary to Provide encouragement and discuss modifications
sclerosis of penile veins and arteriesdecrease of sexual expression as necessary; rest before
in sexual activity and after sexual activity.

Psychologic/Social Theories of Aging 2. As life roles or physical capacity are lost, the
older adult will substitute new roles or
A. Activity theory intellectual activities.
1. Maintaining a level of active involvement in B. Continuity or developmental theory
life helps the older adult stay psychologically 1. Adjustment to old age is impacted by

4
and socially healthy. individual personality, and the older adult

ADULT NURSING 175


53155_04_Ch04a_p143-263.qxd 2/26/09 6:50 PM Page 176

will exhibit similar choices and decisions to


younger years. Table 4-12 Chronic Disease vs. Acute Disease
2. This theory allows for great variation in
successful aging, as individual habits and Characteristic Chronic Diseases Acute Diseases
preferences are unique. Cause Multiple causes; often Specific etiologies
C. Disengagement theory related to lifestyle
1. Gradual mutual withdrawal between the
individual and society as the aging process Onset Slow, insidious Rapid
continues. Duration Indeterminate; Short
2. While this theory was a major milestone in remissions and
aging research, it is now felt to be flawed, as exacerbations
many older adults remain engaged in
Understanding Often difficult because Simpler because
psychosocial aspects of life.
of disease of indeterminate symptoms
course, remissions, more overt
PATTERNS OF HEALTH AND DISEASE and exacerbations
IN THE OLDER ADULT Outcomes Somewhat predictable Symptoms resolve
but often debilitating with cure of
A. Diseases that occur to varying degrees in most
and associated with disease
older adults
long periods of Outcomes usually
1. Cataracts
illness favorable; cures
2. Arteriosclerosis
Management of Health care
3. Benign prostatic hypertrophy (males)
condition provider directs
B. Diseases with increased incidence with advancing
Lifestyle changes care and cure
age
required
1. Neoplastic disease
Individual with disease
2. Diabetes mellitus
must assume control
3. Dementia disorders
of disease
C. Diseases that have more serious consequences in
the elderly and make homeostasis more difficult to
maintain
1. Pneumonia
2. Influenza 2. Depending on the clients stability, the
3. Trauma interview may take more than one session.
D. Chronic disease very common B. Presenting problem
1. Seventy-nine percent of noninstitutionalized 1. Assess client systematically depending upon
persons over age 70 have at least one chronic the presenting problem.
disease. 2. Typical presentations of disease may change
2. Most common chronic diseases: arthritis, with age (i.e., client may not exhibit chest pain
hypertension, heart disease, cancer. with a myocardial infarction).
3. Most hospital visits for persons over 65 are for 3. The problem is likely to have multiple
chronic diseases. contributing factors and affect the clients
E. Functional disability (inability to perform functional abilities.
activities of daily living [ADL]) C. Mental status and mental health
1. Thirty-two percent of persons over 65 have 1. It is important to obtain a baseline for
some limitation of functions. orientation, memory, level of alertness, and
2. Twenty-five percent of persons over 65 require decision-making capabilities.
help with at least one ADL or IADL 2. Assess the client for quality of life issues,
(instrumental activities of daily living; e.g., mood, affect, and anxiety.
shopping, paying bills). D. Lifestyle and function
F. Chronic vs. acute diseases (see Table 4-12). 1. Often, there is little correlation between
diseases and functional abilities.
2. The functional assessment provides a clearer
picture of physical, psychologic, and social
ASSESSMENT health.
3. Use the clients own baseline from previous
Health History and Gerontologic Focus assessments to determine any changes in
A. Assessment of the older adult client is complex function.
1. Allow sufficient time to conduct a thorough 4. Have the client demonstrate function wherever
health history interview. possible (i.e., observe gait and balance,

4
drinking a glass of water, dressing self).

176 NCLEX-RN Review


53155_04_Ch04a_p143-263.qxd 2/26/09 6:50 PM Page 177

E. Medication usage G. Impaired memory


1. Ask for information about all types of H. Impaired physical mobility
medications that the client is taking, including I. Impaired oral mucous membrane
prescription medications, nonprescription J. Imbalanced nutrition: less or more than body
medications (especially analgesics and requirements
laxatives), vitamin supplements, and herbal K. Ineffective airway clearance or breathing pattern,
medications. or impaired gas exchange
2. Be sure the client understands the purpose, L. Self-care deficits: feeding, bathing/hygiene,
dosage, side effects, and any special dressing/grooming, toileting
considerations or interactions for all M. Disturbed body image or ineffective role
medications. performance
3. Discuss the clients abilities to obtain N. Disturbed sensory perception
medications (i.e., renewing prescriptions, O. Sexual dysfunction
paying for medications). P. Impaired skin integrity
4. Polypharmacy is often present. Average older Q. Disturbed sleep pattern
adult takes 11 prescription medications per day. R. Disturbed thought processes
F. Nutrition and hydration S. Ineffective tissue perfusion
1. Obtain food/fluid intake profile (either T. Impaired urinary elimination
24 hours or 3 days). U. Deficient diversional activity
2. Determine any difficulties ingesting V. Wandering
food/fluids (chewing, salivation, swallowing, W. Impaired social interaction
manual dexterity, tremors). X. Risk for other-directed violence
3. Any foods the client is unable to eat (dairy Y. Risk for falls or injury
products, sodium, sugar) or foods the client Z. Relocation stress syndrome
should eat (potassium- or calcium-rich AA. Impaired home maintenance
foods/fluids).
4. Inquire if an adequate amount of water is
taken daily to stay hydrated. PLANNING AND
5. Inquire if able to afford/purchase/prepare food.
IMPLEMENTATION
G. Past medical history
1. Inquire about all chronic diseases and
conditions. Be aware that the client may not Goals
even consider certain conditions treatable and Client will maintain:
therefore does not mention them, e.g., urinary A. Maximum functional independence
incontinence or pain from arthritis. B. Normal bowel and bladder elimination patterns
2. Obtain information about previous illnesses, C. Sufficient communication skills
hospitalizations, and surgeries. D. Positive self-concept
E. Freedom from injury and infection
Physical Examination F. Optimal cognitive functioning
G. Adequate nutritional status and fluid balance
A. Assess body systems as indicated.
H. A restful sleep pattern
B. Note physical changes in the older adult (see
I. Social contacts and interpersonal needs
Table 4-11).
J. Treatment regimens as prescribed

Laboratory/Diagnostic Tests
A. Laboratory tests as indicated according to INTERVENTIONS
symptoms of individual client.
B. Interpret lab test results with aging changes in Pharmacotherapy in the Older Adult
mind.
A. General information
1. Decreased body weight, dehydration,
ANALYSIS alterations in fat to muscle ratio, and slowed
Nursing diagnoses for older adult clients may include: organ functioning may cause accumulation of
A. Activity intolerance a drug in the body due to higher
B. Bowel incontinence, constipation, diarrhea concentrations in the tissues and slowed
C. Acute or chronic pain metabolism and excretion of the drug.
D. Anxiety or death anxiety 2. Multiple chronic diseases affecting older
E. Deficient fluid volume adults may also cause changes in the

4
F. Risk for infection metabolism and excretion of medications.

ADULT NURSING 177


53155_04_Ch04a_p143-263.qxd 2/26/09 6:50 PM Page 178

3. Medication errors among older community- CONDITIONS


dwelling adults are estimated to be 2550%.
4. Drug-drug interactions are increased
secondary to older adults often having Senile Dementia, Alzheimers Disease
more than one prescribing health care See also Unit 7.
provider. A. General information
B. Nursing care 1. In dementia, the elderly client is alert with a
1. Conduct a brown bag evaluation to assess all progressive decline in memory and cognition
prescription, over-the-counter, and herbal accompanied by personality and behavioral
medications the client may be taking. changes.
2. Assess the clients understanding of the 2. Alzheimers disease accounts for 6075% of
reasons for the drug therapy. all dementias and is the number one reason for
3. Assess the clients vision, memory, judgment, institutionalization of the elderly.
reading level, hand dexterity, and motivation 3. Other types of dementias include:
to determine ability to self-medicate. a. Vascular: small infarctions in the brain
4. Provide instructions in large-print, result in dementia. Risk factors include
premeasured syringes, memory aids, and daily hyperlipidemia, hypertension, and
drug dose containers to enhance self- smoking.
medicating abilities. b. Frontoparietal: atrophy of neurons in the
5. Check with the pharmacist for any drug-drug frontal lobes of the brain. Early symptoms
interactions if unsure. are behavioral rather than cognitive
6. Before beginning a medication, obtain baseline abnormalities. (Picks disease is most
vital signs, mental status, vision, and common.)
bowel/bladder function. B. Medical management
7. Drug-induced side effects may present as 1. Rule out other conditions that might be
confusion, incontinence, falls, or immobility. causing symptoms. A definitive diagnosis of
8. Assess the clients ability to pay for the Alzheimers disease can only be made upon
prescriptions. autopsy.
9. If the client requires assistance in taking 2. Medications for treatment include tacrine
medications, teach family members. Proper (Cognex), donepezil (Aricept), rivastigmine
techniques for administering oral medications (Exelon), or galantamine (Reminyl).
include: position head forward with neck a. Used in mild to moderate stages of the
slightly flexed to facilitate swallowing and disease: 25% of clients show symptom
avoid risk of aspiration. improvement, and 80% of clients show a
10. If client has swallowing difficulties, obtain delay in decline of cognition.
liquid forms of oral medications wherever b. Nursing considerations: Monitor for
possible. bradycardia and GI side effects, including
11. Assess client for effectiveness of medications anorexia and weight loss.
and any adverse reactions. 3. Treatment goals are to minimize behavioral
symptoms, i.e., agitation, and maximize
EVALUATION quality of life.
C. Assessment findings
A. Client performs self-care activities or caregiver 1. Early in the disease process, may become
provides assistance as needed. depressed or anxious. Increased risk of suicide
B. Client is continent of bowel and bladder; voids in at this time.
adequate amounts and has regular bowel 2. See Table 4-13, for stages of Alzheimers
movements. disease.
C. Client is able to successfully communicate needs D. Nursing interventions
and concerns. 1. Provide a safe environment, e.g., bed in lowest
D. Client makes positive statements about self. position; check on client frequently; avoid
E. Client/caregiver modifies environment to support restraints (increases agitation); use night-
safety. lights.
F. Client is alert, calm, and oriented, if possible. 2. Provide structured environment and simple
G. Skin is intact without pressure ulcers. routines, e.g., lowered noise levels;
H. Client eats a nutritionally balanced diet and appropriate lighting; confined area for
maintains a stable weight. wandering.
I. Client maintains friends, social interactions, and 3. Enlist caregivers assistance in assessing
sexual function. routine and establishing plan of care.
J. Client describes and adheres to treatment plan.

4 178 NCLEX-RN Review


53155_04_Ch04a_p143-263.qxd 2/26/09 6:50 PM Page 179

Table 4-13 Stages of Alzheimers Disease

Stage Timeframe Clinical Manifestations


Early 24 years Forgetfulness; poor memory (may compensate by using notes)
Declining interest in people, environment, and current events
Impaired acquisition of new information
Impaired judgment; mild cognitive changes
job performance declines; may lose job
May be apathetic, irritable, or show signs of depression
Normal EEG and CT
Middle 212 years Progressive memory loss
Disorientation to time, place, and events
Impaired ability to follow simple directions or simple math
Significant impairment of cognitive function and judgment
Wanders at night due to sleep-wake cycle disturbance
Neglects personal hygiene and activities of daily living
Becomes increasingly irritable, evasive, and anxious
May experience episodes of violent behavior
EEGslowing; CTnormal or dilated ventricles
Late 812 years Extreme weight loss secondary to lack of eating
Severe impairment of all cognitive functions
Unable to communicate (verbal or written)
Dependent on others for activities of daily living
Incontinent of urine and feces
Pathological reflexes can be elicited
Motor skills are lost; becomes bedridden
EEGdiffuse slowing
CTdilated ventricles and sulcal enlargement

4. Use touch and a calm, relaxed manner in ELDER MISTREATMENT


approaching the client.
5. Facilitate effective communication. A. General information
a. Face the client and speak clearly using 1. Elder mistreatment can take the form of abuse
simple words and short sentences. or neglect.
b. Keep directions simple and choices to a 2. Abuse may be actual injury inflicted or verbal
minimum (e.g., Do you want to eat insults, the use of physical or chemical
chicken or fish?). restraints, financial abuse of money or assets,
c. Allow ample time for responses. withholding food/fluids/medications, sexual
6. Encourage orientation with use of calendars abuse, or abandonment.
and clocks; constantly reorienting the client is 3. The most common type of neglect is self-
frustrating to all; consider validation therapy. neglect in which the older adult is unable or
7. Have family bring items that stimulate unwilling to provide for him/herself. The
memory, e.g., photographs and personal responsible caregiver may also neglect by not
belongings, to the room. providing necessary services or care.
8. Encourage mobility and provide opportunities 4. A range of 410% of the older population
for exercise, including walking and range of experience elder mistreatment.
motion. B. Assessment findings
9. Avoid isolating the client; provide some 1. Identify individuals at risk.
stimulation such as soft music or television. a. Women over 75 years old who live with
10. Provide nutritious, high-fiber foods and relatives and are physically, socially,
adequate fluids to maintain weight and and/or financially dependent.
hydration. Finger foods are a good choice. b. Persons whose primary caregivers express
11. Promote bowel and bladder continence by resentment, anger, or frustration with the
toileting at regular intervals. older adult.
12. Provide a simple bedtime routine that 2. Clues to mistreatment
facilitates sleep, and encourage daytime a. Signs and symptoms may include poor

4
activities to avoid excess napping. hygiene and grooming, failure to thrive

ADULT NURSING 179


53155_04_Ch04a_p143-263.qxd 2/26/09 6:50 PM Page 180

(malnutrition), oversedation, depression, 53. An elderly client reports using more salt in food
and/or fearfulness. to enhance flavor. What explains the reason for
b. Skin provides objective evidence. Look for this action?
bruises, burns, lacerations, or pressure ulcers. 1. A decreased number of taste buds.
3. When assessing for mistreatment, the nurse
2. Confusion because of advanced age.
should consider:
a. Is the person in immediate danger of 3. A need for more sodium to ensure renal
bodily harm? function.
b. Is the person competent to make decisions 4. An attempt to compensate for lost fluids.
regarding self care?
c. What is the degree and significance of the 54. Which assessment finding in the elderly is
persons functional impairments? caused by decreased vessel elasticity and
d. What specific services might help to meet increased peripheral resistance?
the unmet needs? 1. Confusion and disorientation.
e. Who in the family is involved and to what
2. An irregular peripheral pulse rate.
extent?
f. Are the client and family willing to accept 3. An increase in blood pressure.
interventions? 4. Wide QRS complexes on the ECG.
C. Nursing interventions
1. Report suspected mistreatment to adult 55. Which action by the CNA would the nurse
protective services. correct in the care of the older client with a
2. Obtain clients consent for treatment. hearing problem?
3. Document nursing assessments of clients 1. Facing the client, speaking slowly and clearly.
physical and emotional status. 2. Examining the ear for cerumen accumulation.
3. Assisting the client with hearing aid placement.
OSTEOPOROSIS (See Unit 6) 4. Speaking loudly when talking to the client.

CEREBRAL VASCULAR ACCIDENT 56. An elderly client repeatedly talks about how he
(See Cerebrovascular Accident [CVA]) wishes he was as strong and energetic as he was
when he was younger. In planning care for this
BENIGN PROSTATIC HYPERTROPHY client, the nurse should include which of the
(See Disorder of the Male Reproductive System) following?
1. Use of the intervention reminiscence.
CATARACTS (See Disorders of the Eye) 2. Confrontation of the client about being so grim.
GLAUCOMA (See Disorders of the Eye) 3. Changing the topic whenever he brings it up.
4. Incorporation of a humorous view of the
normal loss of strength.

57. An elderly woman is hospitalized for


dehydration. During the admission interview,
Sample Questions she admits to the nurse that she is depressed.
The nurse would expect this client to exhibit
which of the following symptoms?
51. What pulmonary function is common in elderly 1. Increased energy level.
clients?
2. Increased anxiety.
1. Reduction in vital capacity.
3. Increased autonomy.
2. A decrease in residual volume.
4. Increased socialization.
3. An increase in functional alveoli.
4. Blood gases that reflect mild acidosis. 58. Knowing the difference between normal age-
related changes and pathologic findings, which
52. Which is a normal sign of aging in the renal system? finding should the nurse identify as pathologic
1. Intermittent incontinence. in a 74-year-old client?
2. Concentrated urine. 1. Increase in residual lung volume.
3. Microscopic hematuria. 2. Decrease in sphincter control of the bladder.
4. A decreased glomerular filtration rate. 3. Increase in diastolic blood pressure.

4
4. Decreased response to touch, heat, and pain.

180 NCLEX-RN Review


53155_04_Ch04a_p143-263.qxd 2/26/09 6:50 PM Page 181

59. A sexually active 63-year-old client complains of 63. A 76-year-old man who is a resident in an
painful intercourse secondary to vaginal dryness. extended care facility is in the late stages of
Which information is most important for the Alzheimers disease. He tells his nurse that he
nurse to include in a teaching plan for this client? has sore back muscles from all the construction
1. Discuss with the client all the medications work he has been doing all day. Which response
being taken, including over-the-counter by the nurse is most appropriate?
drugs, in order to determine a possible 1. You know you dont work in construction
etiology for the dryness. anymore.
2. Teach the client alternative methods of 2. What type of motion did you do to
intimacy in the form of touch. precipitate this soreness?
3. Instruct the client to use an artificial water- 3. Youre 76 years old and youve been here all
based lubricant in the vagina to decrease the day. You dont work in construction anymore.
discomfort of intercourse. 4. Would you like me to rub your back for you?
4. Prepare the client for a vascular work-up
because the dryness is often related to 64. An 86-year-old male with senile dementia has
vascular deficiencies. been physically abused and neglected for the
past two years by his live-in caregiver. He has
60. An older client, who has medically controlled since moved and is living with his son and
manic-depression and asthma has been daughter-in-law. Which response by the clients
prescribed cardiac medications for congestive son would cause the nurse great concern?
heart failure. He complains to the home care 1. How can we obtain reliable help to assist us
nurse that he is nauseated. It would be justifiable in taking care of Dad? We cant do it alone.
for the nurse to reach which of the following 2. Dad used to beat us kids all the time.
conclusions as to the cause of the clients nausea? I wonder if he remembered that when it
1. The reaction between the new medication happened to him?
regime and the foods caused the nausea. 3. Im not sure how to deal with Dads constant
2. The problem of polypharmacy may exist as repetition of words.
the client symptomatology may be a result of 4. I plan to ask my sister and brother to help
multiple drug interactions. my wife and me with Dad on the weekends.
3. The nausea could be psychosomatic and
related to the clients depression over having 65. An alert and oriented 84-year-old client is
to take new medications. receiving home care services following a
cerebrovascular accident (CVA) that has left her
4. The client may be taking too much of his new
with right-sided hemiparesis. She lives with her
medications, which may contribute to his
middle-aged daughter and son-in-law. The nurse
symptoms.
suspects she is being physically abused by her
61. An older client has several medications ordered daughter. To elicit information effectively, the
and has difficulty swallowing them. What nurse should do which of the following?
strategy should the nurse use to administer these 1. Directly ask the client if she has been
medications? physically struck or hurt by anyone.
1. Hide the medication by placing them in meat. 2. Wait until enough trust has been developed to
2. Crush the medication and mix them with soft enable the client to approach the nurse first.
foods. 3. Confront the daughter with the suspicions.
3. Substitute injectable medications. 4. Interview the son-in-law to gain his
4. Dissolve medications in liquid. perspective of the situation.

62. Which of the following measures is necessary to


incorporate into a plan of care for a client who is
diagnosed with senile dementia? Answers and Rationales
1. Because these clients are easily bored, they
need to be challenged with new activities.
2. Environmental stimuli need to be eliminated. 51. 1. Muscles weaken reducing capability to cough or
deep breathe, which reduces total lung capacity.
3. Communicate in simple words, short
sentences, and a calm tone of voice. 52. 4. The glomerular filtration rate is decreased
4. Schedule more demanding activities later in dramatically in the elderly due to changes in the

4
the day. renal tubules.

ADULT NURSING 181


53155_04_Ch04a_p143-263.qxd 2/26/09 6:50 PM Page 182

53. 1. The taste buds begin to atrophy at age 40, and 60. 2. Polypharmacy is the prescription, use, or
insensitivity to taste qualities occurs after age 60. administration of five or more medications.
Studies related to diminished taste indicate that If not coordinated, different physicians, each
there are changes in the salt threshold for some focusing on a specific disease process,
elderly individuals. contribute to polypharmacy.

54. 3. The blood pressure increases in response to 61. 2. Medications, crushed and mixed with soft
the thickening of vessels and less distensible foods, are easier to digest for persons who have
arteries and veins. There is also impedance to difficulty swallowing.
blood flow and increased systemic vascular
resistance, contributing to hypertension. 62. 3. Keep communications simple and concrete.
Close-ended questions are more beneficial than
55. 4. Raising the voice to speak loudly only open-ended questions, which may require
increases the emission of higher frequency complex answers that serve only to confuse the
sounds, which the elderly client with presbycusis client. Even if the client isnt able to fully
(a progressive bilateral perceptive loss of hearing comprehend communications, a calm tone of
in the older individual that occurs with the aging voice may alleviate any stress.
process) will have difficulty hearing.
63. 4. In the late stages of Alzheimers disease, it is
56. 1. Assisting the older adult in reminiscing, better to go along with the clients reality rather
or engaging in a life review process, is one than confront him with logic and reasoning.
way to assist the individual to accomplish Asking close-ended, simple questions that relate
developmental tasks. One such task, adjusting to his reality is nonthreatening and calming.
to decreasing physical strength, needs to be Note that the nurse responds in a way that is
met to establish and preserve ego integrity. congruent with his main concern, which is his
sore back.
57. 2. Many psychosocial symptoms occur with
depression, including feelings of hopelessness, 64. 2. This statement is a cause for concern. Abusive
helplessness, and increased anxiety, which patterns are highly likely to be passed from
contributes to despair rather than ego integrity. parents to children. When children grow up and
move into positions where they are caring for
58. 3. A modest increase in systolic blood pressure,
their aged parents (role reversal), the abusive
not diastolic blood pressure, is an expected age-
behavior can surface.
related change due to an increase in vascular
resistance and vessel rigidity. An increase in 65. 1. Direct questioning, in an open and accepting
diastolic blood pressure, however, is not an manner, is important. Abused elders are often
expected age-related change. It is pathologic and reluctant to report abuse and will not volunteer
needs to be monitored. the information on their own. Clients need to
59. 3. The decrease in vaginal secretions, which feel free to indicate the existence of an activity
contributes to vaginal dryness and subsequent about which they may feel embarrassment and
painful intercourse, is a normal age-related shame.
change. Using a lubricant will decrease or
eliminate this discomfort.

Perioperative Nursing

OVERVIEW B. Resistance to infection is lowered due to surgical


incision.
C. Vascular system is disturbed due to severing of
Effects of Surgery on the Client blood vessels and blood loss.
Physical Effects D. Organ function may be altered due to
manipulation.
A. Stress response (neuroendocrine response) is

4
activated.

182 NCLEX-RN Review


53155_04_Ch04a_p143-263.qxd 2/26/09 6:50 PM Page 183

Psychologic Effects C. Answer questions, clarify and reinforce


explanations given by surgeon.
Common fears: pain, anesthesia, loss of control, D. Explain routine pre- and post-op procedures and
disfigurement, separation from loved ones, alterations any special equipment to be used.
in roles or lifestyle. E. Teach coughing and deep-breathing exercises,
splinting of incision, turning side to side in bed,
Factors Influencing Surgical Risk and leg exercises; explain their importance in
preventing complications; provide opportunity for
A. Age: very young and elderly are at increased risk. return demonstration.
B. Nutrition: malnutrition and obesity increase risk of F. Assure client that pain medication will be
complications. available post-op.
C. Fluid and electrolyte balance: dehydration,
hypovolemia, and electrolyte imbalances can pose
problems during surgery. Physical Preparation
D. General health status: infection, cardiovascular A. Obtain history of past medical conditions, surgical
disease, pulmonary problems, liver dysfunction, procedures, allergies, dietary restrictions, and
renal insufficiency, or metabolic disorders create medications.
increased risk. B. Perform baseline head-to-toe assessment,
E. Medications including vital signs, height, and weight.
1. Anticoagulants (including aspirin and NSAIDs) C. Ensure that diagnostic procedures are performed
predispose to hemorrhage; discontinue use as ordered. Common tests are:
according to physicians orders. 1. CBC (complete blood count)
2. Tranquilizers (e.g., phenothiazines) may cause 2. Electrolytes
hypotension and potentiate shock. 3. PT/PTT (prothrombin time; partial
3. Antibiotics: aminoglycosides may intensify thromboplastin time)
neuromuscular blockade of anesthesia with 4. Urinalysis
resultant respiratory paralysis. 5. ECG (electrocardiogram)
4. Diuretics: may cause electrolyte imbalances. 6. Type and crossmatch
5. Antihypertensives: can cause hypotension and 7. Chest X-ray
contribute to shock. D. Prepare clients skin.
6. Long-term steroid therapy: causes 1. Shower with antibacterial soap to cleanse skin
adrenocortical suppression; may need if ordered; client may do this at home the
increased dosage during perioperative period. night before surgery if outpatient admission.
F. Type of surgery planned: major surgery (e.g., 2. Skin prep if ordered: shave or clip hairs and
thoracotomy) poses greater risk than minor surgery cleanse appropriate areas to reduce bacteria on
(e.g., dental extraction). skin and minimize chance of infection.
G. Psychologic status of client: excessive fear or E. Administer enema if ordered (usually for surgery
anxiety may have adverse effect on surgery. on GI tract, gynecologic surgery).
F. Promote adequate rest and sleep.
1. Provide back rub, clean linens.
PREOPERATIVE PERIOD 2. Administer bedtime sedation.
G. Instruct client to remain NPO after midnight to
Psychologic Support prevent vomiting and aspiration during surgery.

A. Assess clients fears, anxieties, support systems,


and patterns of coping. Legal Responsibilities
B. Establish trusting relationship with client and A. Surgeon obtains operative permit (informed
significant others. consent).
C. Explain routine procedures, encourage 1. Surgical procedure, alternatives, possible
verbalization of fears, and allow client to ask complications, disfigurements, or removal of
questions. body parts are explained.
D. Demonstrate confidence in surgeon and staff. 2. It is part of the nurses role as client advocate
E. Provide for spiritual care if appropriate. to confirm that the client understands
information given.
Preoperative Teaching B. Informed consent is necessary for each operation
performed, however minor. It is also necessary for
A. Frequently done on an outclient basis. major diagnostic procedures, e.g., bronchoscopy,
B. Assess clients level of understanding of surgical thoracentesis, etc., where a major body cavity is
procedure and its implications. entered.

ADULT NURSING

4 183
53155_04_Ch04a_p143-263.qxd 2/26/09 6:50 PM Page 184

C. Adult client (over 18 years of age) signs own 2. Sedatives (secobarbital sodium [Seconal]),
permit unless unconscious or mentally sodium pentobarbital [Nembutal] decrease
incompetent. anxiety and promote relaxation and sleep.
1. If unable to sign, relative (spouse or next of 3. Anticholinergics (atropine sulfate,
kin) or guardian will sign. scopolamine [Hyoscine]) and glycopyrrolate
2. In an emergency, permission via telephone or (Robinul) decrease tracheobronchial secretions
telegram is acceptable; have a second nurse to minimize danger of aspirating secretions in
verify by phone the telephone permission; lungs, decrease vagal response to inhibit
both nurses will sign the consent form. undesirable effects of general anesthesia
3. Consents are not needed for emergency care if (bradycardia).
all four of the following criteria are met. 4. Droperidol, fentanyl, or a combination may be
a. There is an immediate threat to life. ordered; should not be given with sedatives
b. Experts agree that it is an emergency. because of danger of respiratory depression;
c. Client is unable to consent. also helpful in control of postoperative nausea
d. A legally authorized person cannot be and vomiting.
reached. H. Elevate side rails and provide quiet environment.
D. Minors (under 18) must have consent signed by an I. Prepare clients chart for OR, including operative
adult (i.e., parent or legal guardian). An permit and complete pre-op check list.
emancipated minor (married, college student J. Stay with client after pre-op medications have
living away from home, in military service, any been given and assist with bedpan for toileting
pregnant female or any who has given birth) may needs.
sign own consent.
E. Witness to informed consent may be nurse,
another physician, or other authorized person. INTRAOPERATIVE PERIOD
F. If nurse witnesses informed consent, specify
whether witnessing explanation of surgery or just
signature of client.
Anesthesia
General Anesthesia
Preparation Immediately A. General information
before Surgery 1. Drug-induced depression of CNS; produces
decreased muscle reflex activity and loss of
A. Obtain baseline vital signs; report elevated consciousness.
temperature or blood pressure. 2. Balanced anesthesia: combination of several
B. Provide oral hygiene and remove dentures. anesthetic drugs to provide smooth induction,
C. Remove clients clothing and dress in clean gown. appropriate depth and duration of anesthesia,
D. Remove nail polish, cosmetics, hair pins, contact sufficient muscle relaxation, and minimal
lenses, prostheses, and any body jewelry. complications.
E. Instruct client to empty bladder. B. Stages of general anesthesia: induction,
F. Check identification band. excitement, surgical anesthesia, and danger stage
G. Administer pre-op medications as ordered. (see Table 4-14).
1. Narcotic analgesics (meperidine [Demerol], C. Agents for general anesthesia
morphine sulfate) relax client, reduce anxiety, 1. Inhalation agents may be gas or liquid.
and enhance effectiveness of general 2. IV anesthetics: used as induction agents
anesthesia. because they produce rapid, smooth

Table 4-14 Stages of Anesthesia

Stage From To Client Status


Stage I (induction) Beginning administration Loss of consciousness May appear euphoric, drowsy, dizzy.
of anesthetic agent
Stage II (delirium Loss of consciousness Relaxation Breathing irregular; may appear excited; very
or excitement) susceptible to external stimuli.
Stage III (surgical Relaxation Loss of reflexes and Regular breathing pattern; corneal reflexes
anesthesia) depression of vital functions absent; papillary constriction.
Stage IV Vital functions depressed Respiratory arrest; possible No respirations; absent or minimal
(danger stage) cardiac arrest heartbeat; dilated pupils.

4 184 NCLEX-RN Review


53155_04_Ch04a_p143-263.qxd 2/26/09 6:50 PM Page 185

induction; may be used alone in short


procedures such as endoscopies. Table 4-15 Regional Anesthesia
a. Common IV anesthetics: methohexital
(Brevital), sodium thiopental (Pentathol),
Types Method
midazolam hydrochloride (Versed) Topical Cream, spray, drops, or ointment applied
b. Disadvantages: poor relaxation; respiratory externally, directly to area to be
and myocardial depression in high doses; anesthetized.
bronchospasm, laryngospasm; Local infiltration Injected into subcutaneous tissue of
hypotension, respiratory depression block surgical area.
3. Dissociative agents: produce state of profound
analgesia, amnesia, and lack of awareness Field block Area surrounding the surgical site
without loss of consciousness; used alone in injected with anesthetic.
short surgical and diagnostic procedures or for Nerve block Injection into a nerve plexus to
induction prior to administration of more anesthetize part of body.
potent general anesthetics. Spinal Anesthetic introduced into subarachnoid
a. Agent: ketamine (Ketalar) space of spinal cord producing
b. Side effects: tachycardia, hypertension, anesthesia below level of diaphragm.
respiratory depression, hallucinations,
Epidural Anesthetic injected extradurally to
delirium
produce anesthesia below level of
c. Precautions: decrease verbal, tactile, and
diaphragm; used in obstetrics.
visual stimulation during recovery period
4. Neuroleptics: produce state of neuroleptic Caudal Variation of epidural block; produces
analgesia characterized by reduced motor anesthesia of perineum and
activity, decreased anxiety, and analgesia occasionally lower abdomen;
without loss of consciousness; used alone for commonly used in obstetrics.
short surgical and diagnostic procedures, as Saddle block Similar to spinal, but anesthetized area is
premedication or in combination with other more limited; commonly used in
anesthetics for longer anesthesia. obstetrics.
a. Agent: fentanyl citrate with droperidol
(Innovar)
b. Side effects: hypotension, bradycardia, 2. Sedative and analgesic medications are used to
respiratory depression, skeletal muscle achieve an altered state of consciousness with
rigidity, twitching minimal risk, relief of anxiety, an amnestic
c. Precautions: reduce narcotic doses by state, and pain relief from noxious stimuli.
1
2 to 13 for at least 8 hours postanesthesia as 3. These agents may include a combination of
ordered to prevent respiratory depression. a benzodiazepine (midazolam, diazepam) and
D. Adjuncts to general anesthesia: neuromuscular a narcotic (fentanyl, morphine).
blocking agents: used with general anesthetics to 4. This provides a safe and effective option for
enhance skeletal muscle relaxation. clients undergoing minor surgical and
1. Agents: pancuronium (Pavulon), diagnostic procedures such as, but not limited
succinylcholine (Anectine), tubocurarine, to, endoscopic procedures, breast biopsy,
atracurium besylate (Tracrium), vecuronium dental surgery, and plastic surgery.
bromide (Norcuron) 5. Conscious sedation is extremely safe when
2. Precaution: monitor clients respirations for at administered by qualified providers.
least 1 hour after drugs effect has worn off. a. Certified registered nurse anesthetists
(CRNAs), anesthesiologists, dentists, oral
Regional Anesthesia surgeons, and other physicians are
qualified providers of conscious sedation.
A. General information (see also Table 4-15). b. Specifically trained registered nurses may
1. Produces loss of painful sensation in one area of assist in the administration of conscious
the body; does not produce loss of consciousness sedation.
2. Uses: biopsies, excision of moles and cysts, 6. Verbal communication with the client can be
endoscopies, surgery on extremities; childbirth maintained throughout the procedure, client
3. Agents: lidocaine (Xylocaine), procaine may have a brief period of amnesia which may
(Novocain), tetracaine (Pontocaine) erase any memory of the procedure.
7. This will provide reassurance to a cooperative
Conscious Sedation client and help monitor intact neurological
status.
A. General information 8. Supplemental oxygen should always be given

4
1. Intravenous conscious sedation is induced by to a client receiving conscious sedation.
pharmacologic agents.

ADULT NURSING 185


53155_04_Ch04a_p143-263.qxd 2/26/09 6:50 PM Page 186

9. Constant vigilance in monitoring of 6. Assess respiratory status and auscultate lungs


cardiorespiratory status (heart rate and every 4 hours; be alert for any signs of
rhythm, blood pressure, respiratory rate, and respiratory complications.
pulse oximetry) is crucial. B. Monitor cardiovascular status and avoid post-op
10. The provider who monitors the client receiving complications.
conscious sedation should have no other 1. Encourage leg exercises every 2 hours while in
responsibilities during the procedure and bed.
should never abandon that client for any reason. 2. Encourage early ambulation.
3. Apply antiembolism stockings as ordered.
4. Assess vital signs, color and temperature of
POSTOPERATIVE PERIOD skin every 4 hours.
C. Promote adequate fluid and electrolyte balance.
1. Monitor IV and ensure adequate intake.
Postoperative Care 2. Measure I&O.
Recovery Room (Immediate 3. Irrigate NG tube properly, using normal saline
Postoperative Care) solution.
4. Observe for signs of fluid and electrolyte
A. Assess for and maintain patent airway. imbalances.
1. Position unconscious or semiconscious client D. Promote optimum nutrition.
on side (unless contraindicated) or on back 1. Maintain IV infusion as ordered.
with head to side and chin extended forward. 2. Assess for return of peristalsis (presence of
2. Check for presence/absence of gag reflex. bowel sounds and flatus); peristalsis may take
3. Maintain artificial airway in place until gag hours to return.
and swallow reflex have returned. 3. Add progressively to diet as ordered and note
B. Administer oxygen as ordered. tolerance.
C. Assess rate, depth, and quality of respirations. E. Monitor and promote return of urinary function.
D. Check vital signs every 15 minutes until stable, 1. Measure I&O.
then every 30 minutes. 2. Assess clients ability to void.
E. Note level of consciousness; reorient client to time, 3. Report to surgeon if client has not voided
place, and situation. within 8 hours after surgery.
F. Assess color and temperature of skin, color of 4. Check for bladder distention.
nailbeds, and lips. 5. Use measures to promote urination (e.g., assist
G. Monitor IV infusions: condition of site, type, and male to sit on side of bed, pour warm water
amount of fluid being infused and flow rate. over females perineum).
H. Check all drainage tubes and connect to suction or F. Promote bowel elimination.
gravity drainage as ordered; note color, amount, 1. Encourage ambulation.
and odor of drainage. 2. Provide adequate food and fluid intake when
I. Assess dressings for intactness, drainage, tolerated.
hemorrhage. 3. Keep stool record and note any difficulties
J. Monitor and maintain clients temperature; may with bowel elimination.
need extra blankets. G. Administer post-op analgesics as ordered; provide
K. Encourage client to cough and deep breathe after additional comfort measures.
artificial airway is removed. H. Encourage optimal activity, turning in bed every
L. If spinal anesthesia used, maintain flat position 2 hours, early ambulation if allowed (generally
and check for sensation and movement in lower client will be out of bed within 24 hours; have
extremities. client dangle legs before getting out of bed).
I. Provide wound care.
Care on Surgical Floor 1. Check dressings frequently to ensure they are
clean, dry, and intact.
A. Monitor respiratory status and promote optimal 2. Observe aseptic technique when changing
functioning. dressings.
1. Encourage client to cough (if not 3. Encourage diet high in protein and vitamin C.
contraindicated) and deep breathe every 4. Report any signs of infection: redness,
12 hours. drainage, odor, fever.
2. Instruct client to splint incision while J. Provide adequate psychologic support to
coughing. client/significant others.
3. Assist client to turn in bed every 2 hours. K. Provide appropriate discharge teaching: dietary
4. Encourage early ambulation. restrictions, medication regimen, activity
5. Encourage use of incentive spirometer every limitations, wound care, and possible

4
2 hours: causes sustained, maximal inspiration complications.
that inflates the alveoli.

186 NCLEX-RN Review


53155_04_Ch04a_p143-263.qxd 2/26/09 6:50 PM Page 187

Postoperative Complications C. Post-op urinary tract infections are most


commonly caused by catheterization; prevention
Respiratory System consists of using strict sterile technique when
inserting a catheter, and appropriate catheter care
Common post-op complications of respiratory tract are
(every 8 hours or according to agency protocol).
atelectasis and pneumonia.
A. Predisposing factors
1. Type of surgery (e.g., thoracic or high abdomen
Gastrointestinal System
surgery) An important GI post-op complication is paralytic
2. Previous history of respiratory problems ileus (paralysis of intestinal peristalsis).
3. Age: greater risk over age 40 A. Predisposing factors
4. Obesity 1. Temporary: anesthesia, manipulation of bowel
5. Smoking during abdominal surgery
6. Respiratory depression caused by narcotics 2. Prolonged: electrolyte imbalance, wound
7. Severe post-op pain infection, pneumonia
8. Prolonged post-op immobility B. Assessment findings
B. Prevention: see Care on Surgical Floor. 1. Absent bowel sounds
2. No passage of flatus
Cardiovascular System 3. Abdominal distention
C. Nursing interventions
Common post-op complications of the cardiovascular 1. Assist with insertion of nasogastric or intestinal
system are deep vein thrombosis, pulmonary tube with application of suction as ordered.
embolism, and shock. 2. Keep client NPO.
A. Predisposing factors to deep venous thrombosis 3. Maintain IV therapy as ordered.
(DVT) 4. Assess for bowel sounds every 4 hours; check
1. Lower abdominal surgery or septic diseases for abdominal distention, passage of flatus.
(e.g., peritonitis) 5. Encourage ambulation if appropriate.
2. Injury to vein by tight leg straps during surgery
3. Previous history of venous problems Wound Complications
4. Increased blood coagulability due to
dehydration, fluid loss A. Wound infection
5. Venous stasis in the extremity due to 1. Predisposing factors
decreased movement during surgery a. Obesity
6. Prolonged post-op immobilization b. Diabetes mellitus
B. Predisposing factors to pulmonary embolism: may c. Malnutrition
occur as a complication of DVT. d. Elderly clients
C. Most common causes of shock during post-op e. Steroids and immunosuppressive agents
period f. Lowered resistance to infection, as found
1. Hemorrhage in clients with cancer
2. Sepsis 2. Assessment findings: redness, tenderness,
3. Myocardial infarction and cardiac arrest drainage, heat in incisional area; fever; usually
4. Drug reactions occurs 35 days after surgery.
5. Transfusion reactions 3. Prevention: see Care on Surgical Floor.
6. Pulmonary embolism 4. Nursing interventions
7. Adrenal failure a. Obtain culture and sensitivity of wound
D. Prevention of DVT, pulmonary embolism, and drainage (S. aureus most frequently
shock: see Care on Surgical Floor. cultured).
b. Perform cleansing and irrigation of wound
Genitourinary System as ordered.
c. Administer antibiotic therapy as ordered.
Post-op complications of the genitourinary system B. Wound dehiscence and evisceration
often include urinary retention and urinary tract 1. Dehiscence: opening of wound edges
infection. 2. Evisceration: protrusion of loops of bowel
A. Predisposing factors to urinary retention include: through incision; usually accompanied by
1. Anxiety sudden escape of profuse, pink serous drainage
2. Pain 3. Predisposing factors to wound dehiscence and
3. Lack of privacy evisceration
4. Narcotics and certain anesthetics that a. Wound infection
diminish clients sense of a full bladder b. Faulty wound closure
B. Prevention and nursing interventions for urinary c. Severe abdominal stretching

4
retention: see Care on Surgical Floor. (e.g., coughing, retching)

ADULT NURSING 187


53155_04_Ch04a_p143-263.qxd 2/26/09 6:50 PM Page 188

4. Nursing interventions for wound dehiscence local anesthesia. What is a major advantage to the
a. Apply Steri-Strips to incision. client for having regional anesthesia?
b. Notify physician. 1. Retains all reflexes.
c. Promote wound healing.
2. Remains conscious.
5. Nursing interventions for wound evisceration
a. Place client in supine position. 3. Has retroactive amnesia.
b. Cover protruding intestinal loops with 4. Is in the OR for a short period of time.
sterile moist normal saline soaks.
c. Notify physician. 71. An adult male is scheduled for surgery and the
d. Check vital signs. nurse is assessing for risk factors. Which of the
e. Observe for signs of shock. following are the greatest risk factors?
f. Start IV line. 1. He is 5 ft 4 in tall and weighs 125 lb.
g. Prepare client for OR for surgical closure 2. He expresses a fear of pain in the post-op period.
of wound.
3. He is 5 ft 4 in tall, weighs 360 lb, and has
diabetes.
4. He expresses a fear of the unknown.
Sample Questions
72. The nurse in an outclient department is
interviewing an adult 1 week prior to her
66. An adult man is in the postanesthesia care unit scheduled elective surgery. In planning for the
(PACU) following a hemicolectomy. How often surgery, which of the following should the nurse
will the nurse monitor the vital signs? include in her teaching?
1. Continuously. 1. Detailed information about the procedure.
2. Every 5 minutes. 2. Limitations of oral intake the day of the
3. Every 15 minutes. procedure.
4. On a prn basis. 3. Writing a list for postoperative
complications.
67. An adult who has had general anesthesia for 4. The client should not take any of her routine
major surgery is in the PACU. Which of the medications the morning of the surgery.
following indicates the artificial airway should
be removed? 73. The nurse enters a womans room to administer
1. Gagging. the ordered pre-op medication for her
2. Restlessness. hysterectomy. During the conversation, the
client tells the nurse that she and her husband
3. An increase in pain.
are planning to have another child in the coming
4. Clear lungs on auscultation. year. The best action for the nurse to take is
which of the following?
68. An adult is 6 days post abdominal surgery.
Which sign alerts the nurse to wound 1. Do not administer the pre-op medication,
evisceration? notify the nursing supervisor and the
physician.
1. Acute bleeding.
2. Go ahead and administer the medication as
2. Protruding intestines.
ordered.
3. Purple drainage.
3. Check to see if the client has signed a surgical
4. Severe pain. consent.
69. An adult clients wound has eviscerated. Why 4. Send the client to the operating room (OR)
would the respiratory status need to be assessed? without the medication.
1. Dehiscence elevates the diaphragm. 74. The nurse administers 10 mg intramuscular (IM)
2. Coughing increases intestine protrusion. morphine as a pre-op medication, and then
3. Respiratory arrest commonly accompanies discovers that there is no signed operative permit.
wound dehiscence. What is the best action for the nurse to take?
4. Splinting the wound will compromise 1. Send the client to surgery as scheduled.
respiratory status. 2. Notify the nursing supervisor, the OR, and
the physician.
70. An adult client has acute leukemia and is
3. Cancel the surgery immediately.
scheduled for a Hickman catheter insertion under

4
4. Obtain the needed consent.

188 NCLEX-RN Review


53155_04_Ch04a_p143-263.qxd 2/26/09 6:50 PM Page 189

75. An adult received atropine sulfate (Atropine) as 2. Assess for amount of urinary output and the
a pre-op medication 30 minutes ago and is now presence of any distention.
complaining of dry mouth and her pulse rate is 3. Allow the family to visit with the client to
higher than before the medication was decrease the anxiety of the client.
administered. What is the nurses best 4. Take vital signs, assessing first for a patent
interpretation of these finding? airway and the quality of respirations.
1. The client is having an allergic reaction to the
drug. 80. An adult is receiving morphine via a PCA pump
2. The client needs a higher dose of this drug. after her surgery. What statement by the nurse
3. This is a normal side effect of Atropine. would best evaluate the level of pain being
experienced?
4. The client is anxious about the upcoming
surgery. 1. Please rate your pain on a scale of 110.
2. Is the morphine working for you?
76. An adult who has chronic obstructive 3. Are you feeling any pain?
pulmonary disease (COPD) is scheduled for 4. Do you need the morphine level increased?
surgery and the physician has recommended an
epidural anesthetic. Why would an epidural 81. A 58-year-old smoker underwent major
anesthetic be used instead of general anesthesia? abdominal surgery 2 days ago. During the
1. There is too high a risk for pressure sores respiratory assessment, the nurse notes he is
developing. taking shallow breaths and breath sounds are
2. There is less effect on the respiratory system decreased in the bases. What is the best
with epidural anesthesia. interpretation for these findings?
3. Central nervous system control of vascular 1. Pneumonia.
constriction would be affected with general 2. Atelectasis.
anesthesia. 3. Hemorrhage.
4. There is too high a risk of lacerations to the 4. Thromboembolism.
mouth, bruising of lips, and damage to teeth.
82. To prevent thromboembolism in the post-op
77. An adult had a bunion removed under an client the nurse should include which of the
epidural block. In the immediate post-op period, following in the plan of care?
the nurse plans to assess the client for side 1. Place a pillow under the knees and restrict
effects of the epidural block that include which fluids.
of the following?
2. Use strict aseptic technique including
1. Headache. handwashing and sterile dressing technique.
2. Hypotension, bradycardia, nausea, and 3. Assess bowel sounds in all four quadrants on
vomiting. every shift and avoid early ambulation.
3. Hypertension, muscular rigidity, fever, and 4. Assess for Homans signs on every shift,
tachypnea. encourage early ambulation, and maintain
4. Urinary retention. adequate hydration.
78. An adult received droperidol and fentanyl 83. It is 2200 and the nurse notes that an adult male
(Innovar) during surgery. In planning who returned from the PACU at 1400 has not
postoperative care, the nurse will need to voided. The client has an out of bed order, but
monitor for which of the following during the has not been up yet. What is the best action for
immediate post-op period? the nurse to take?
1. Restlessness and anxiety. 1. Insert a Foley catheter into the client.
2. Delirium. 2. Straight-catheterize the client.
3. Dysrhythmias. 3. Assist the client to stand at the side of his
4. Respiratory depression. bed and attempt to void into a urinal.
4. Encourage the client to lie on his left side in
79. An adult has just arrived on the general surgery bed and attempt to void into a urinal.
unit from the postanesthesia care unit (PACU).
Which of the following needs to be the initial 84. When assessing a post-op client, the nurse notes
intervention the nurse takes? a nasogastric tube to low constant suction, the

4
1. Assess the surgical site, noting the amount absence of a bowel movement since surgery, and
and character of drainage.

ADULT NURSING 189


53155_04_Ch04a_p143-263.qxd 2/26/09 6:50 PM Page 190

no bowel sounds. Based on these findings, what Even if the consent form is signed, the nurse
would be the most appropriate plan of action? should withhold sedating medication. This
1. Increase the clients mobility and ensure he is client clearly does not understand the planned
receiving adequate pain relief. procedure.
2. Increase coughing, turning, and deep
74. 2. If a narcotic, sedative, or tranquilizing drug
breathing exercises.
has been administered before signing of the
3. Discontinue the nasogastric tube as the client consent, the drugs effects must be allowed to
does not need it any more. wear off before consent can be given.
4. Assess for bladder pain and distention.
75. 3. These are normal side effects of an
85. Preoperatively, the clients blood pressure was anticholinergic drug; adverse side effects would
110/70. In PACU, the vitals signs are assessed include ECG changes, constipation, and urinary
and the blood pressure is now 150/90. The client retention.
is complaining of severe pain. What is the nurse
aware of due to this finding? 76. 2. Epidural anesthesia does not cause respiratory
1. Pain does not affect the blood pressure. depression, but general anesthesia can,
especially in a client with COPD.
2. The blood pressure elevation is an indication
of hypovolemic shock. 77. 2. These are all symptoms of sympathetic
3. Pain may cause elevated blood pressure. nervous system blockade, so the client should be
4. The client needs a medication to lower the closely monitored for these.
blood pressure.
78. 4. Depression of respiratory rate has been reported
and tends to last longer than the analgesic effect
when Innovar is used during surgery.

Answers and Rationales 79. 4. A specific assessment priority is the


evaluation of a patent airway and respiratory
and circulatory adequacy.
66. 3. While in the postanesthesia care unit (PACU) the
clients vital signs are assessed every 15 minutes. 80. 1. The client should obtain relief from pain, and
using a scale to assess this is a more objective
67. 1. The return of the gag reflex usually indicates measure.
that the client is able to manage his own
secretions and maintain a patent airway. 81. 2. Atelectasis occurs commonly after abdominal
surgery, especially in smokers. This occurs when
68. 2. Evisceration is the actual intestinal contents mucus blocks the bronchioles and causes
protruding through the abdominal wall. decreased breath sounds and shallow breathing.

69. 2. Coughing increases intra-abdominal pressure, 82. 4. Thromboembolism can be related to


which could force loops of bowel out through dehydration and immobility. These measures
the open wound. help prevent hypovolemia and subsequent
sludging of cells. A positive Homans sign is
70. 2. The client receiving regional anesthesia has often associated with thromboembolism.
nerve impulses blocked but does not lose
consciousness. 83. 3. Nursing interventions to facilitate voiding
include ambulation and normal positioning for
71. 3. Obesity and diabetes are major risk factors voiding. The normal voiding position for the
with the potential for complications related to male is standing.
anesthesia.
84. 1. Paralytic ileus can be related to immobility
72. 2. Instructions should be given to the client and inadequate pain medication as well as
regarding limitations of oral intake to avoid bowel manipulation and the anesthetic used
nausea and vomiting from the anesthesia. during surgery.
73. 1. No client should be administered the pre-op 85. 3. Physical pain may increase circulating
medication until the informed consent has been catecholamines, resulting in hypertension. The
obtained. Informed consent means that the client nurse should assess the pain, provide comfort

4
understands the information about the surgery. measures and analgesia, and then reassess the
blood pressure.

190 NCLEX-RN Review


53155_04_Ch04a_p143-263.qxd 2/26/09 6:50 PM Page 191

Oncologic Nursing

PATHOPHYSIOLOGY AND Spread (Invasion and Metastasis)


ETIOLOGY OF CANCER A. Cancer cells are less adhesive than normal cells,
more easily dissociated from their location.
Evolution of Cancer Cells B. Lack of adhesion and loss of contact inhibition
make it possible for a cancer to spread to distant
A. All cells constantly change through growth, parts of the body (metastasis).
degeneration, repair, and adaptation. Normal cells C. Malignant tumors are not encapsulated and
must divide and multiply to meet the needs of the expand into surrounding tissue (invasion).
organism as a whole, and this cycle of cell growth
and destruction is an integral part of life processes.
The activities of the normal cells in the human
Etiology (Carcinogenesis)
body are all coordinated to meet the needs of the Actual cause of cancer is unknown but there are a
organism as a whole, but when the regulatory number of theories; it is currently thought that there
control mechanisms of normal cells fail, and are probably multiple etiologies.
growth continues in excess of the bodys needs,
neoplasia results. Environmental Factors
B. The term neoplasia refers to both benign and
malignant growths, but malignant cells behave A. Majority (over 80%) of human cancers related to
very differently from normal cells and have environmental carcinogens
special features characteristic of the cancer B. Types
process. 1. Physical
C. Because the growth control mechanism of normal a. Radiation: X-rays, radium, nuclear
cells is not entirely understood, it is not clear what explosion or waste, ultraviolet
allows the uncontrolled growth, therefore no b. Trauma or chronic irritation
definitive cure has been found. 2. Chemical
a. Nitrites and food additives, polycyclic
hydrocarbons, dyes, alkylating agents
Characteristics of Malignant Cells b. Drugs: arsenicals, stilbestrol, urethane
Differentiation c. Cigarette smoke
d. Hormones
A. Cancer cells are mutated stem cells that have
undergone structural changes so that they are Genetics
unable to perform the normal functions of
specialized tissue (un- or dedifferentiation). A. Some cancers show familial pattern.
B. They may function in a disorderly way or cease B. May be caused by inherited genetic defects.
normal function completely, only functioning for
their own survival and growth. Viral Theory
C. The most undifferentiated cells are also called
anaplastic. A. Viruses have been shown to be the cause of certain
tumors in animals.
Rate of Growth B. Oncoviruses (RNA-type viruses) thought to be
culprit.
A. Cancer cells have uncontrolled growth or cell C. Viruses (HTLV-I, Epstein-Barr, Human Papilloma
division. Virus) linked to human tumors.
B. Rate at which a tumor grows involves both
increased cell division and increased survival time Immunologic Factors
of cells.
C. Malignant cells do not form orderly layers, but A. Failure of the immune system to respond to and
pile on top of each other to eventually form eradicate cancer cells
tumors. B. Immunosuppressed individuals more susceptible
to cancer

ADULT NURSING

4 191
53155_04_Ch04a_p143-263.qxd 2/26/09 6:50 PM Page 192

DIAGNOSIS OF CANCER Table 4-16 Seven Warning Signs of Cancer (Caution)

Classification and Staging C Change in bowel or bladder habits


A A sore that doesnt heal
Tissue of Origin U Unusual bleeding or discharge
T Thickening or lump in breast (or elsewhere)
A. Carcinoma: arises from surface, glandular, or
I Indigestion or dysphagia
parenchymal epithelium.
O Obvious change in wart or mole
1. Squamous cell carcinoma: surface epithelium
N Nagging cough or hoarseness
2. Adenocarcinoma: glandular or parenchymal
tissue
B. Sarcoma: arises from connective tissue.
C. Leukemia: starts in blood-forming tissues Most testicular cancers are found by men
(i.e., bone marrow) themselves, by accident or when doing TSE.
D. Lymphoma and multiple myeloma: starts in cells 2. Testicular self-examination: Ideally, should be
of immune system performed monthly, after a warm shower or
bath, when the skin of the scrotum is relaxed.
Stages of Tumor Growth Standing in front of a mirror, the man should
A. Several staging systems, important in selection of gently roll each testicle between the thumb
therapy and fingers of both hands. The testes are
1. TNM system: uses letters and numbers to smooth, oval-shaped, and rather firm.
designate the extent of the tumor. 3. Warning signs that men should look for:
a. T: stands for primary growth; 14 with a. Painless swelling
increasing size. T1S indicates carcinoma in b. Feeling of heaviness
situ. c. Hard lump (size of a pea)
b. N: stands for lymph node involvement; 04 d. Sudden collection of fluid in the scrotum
indicates progressively advancing nodal e. Dull ache in the lower abdomen or in the
disease. groin
c. M: stands for metastasis; 0 indicates no f. Pain in a testicle or in the scrotum
distant metastases, 1 indicates presence of g. Enlargement or tenderness of the breasts
metastases.
2. Stages 0-IV: all cancers divided into five stages
incorporating size, nodal involvement, and TREATMENT OF CANCER
spread.
B. Cytologic diagnosis of cancer (e.g., Pap smear) Chemotherapy
1. Involves study of shed cells
2. Classified by degree of cellular abnormality Principles
a. Normal A. Based on ability of drug to kill cancer cells; normal
b. Probably normal (slight changes) cells may also be damaged, producing side effects.
c. Doubtful (more severe changes) Effect is greatest on rapidly dividing cells, such as
d. Probably cancer or precancerous bone marrow cells, the GI tract, and hair.
e. Definitely cancer B. Different drugs act on tumor cells in different
stages of the cell growth cycle.
Client Factors
Early detection of cancer is crucial in reducing Types of Chemotherapeutic Drugs
morbidity and mortality. Clients need to be taught about:
See Unit 2.
A. Seven warning signs of cancer (see Table 4-16).
A. Antimetabolites: foster cancer cell death by
B. Breast self-examination (BSE).
interfering with cellular metabolic process.
C. Importance of rectal exam for those over age 40
B. Alkylating agents: act with DNA to hinder cell
D. Hazards of smoking
growth and division.
E. Oral self-examination as well as annual exam of
C. Plant alkaloids: obtained from the periwinkle
mouth and teeth
plant; makes the hosts body a less favorable
F. Hazards of excess sun exposure
environment for the growth of cancer cells.
G. Importance of Pap smear
D. Antitumor antibiotics: affect RNA to make
H. Physical exam with lab work-up: every 3 years
environment less favorable for cancer growth.
ages 2040; yearly age 40 and over
E. Steroids and sex hormones: alter the endocrine
I. Testicular self-examination (TSE)
environment to make it less conducive to growth
1. Testicular cancer: Most common cancer in

4
of cancer cells.
young men between the ages of 15 and 34.

192 NCLEX-RN Review


53155_04_Ch04a_p143-263.qxd 2/26/09 6:50 PM Page 193

Major Side Effects and Nursing Interventions D. Renal System


1. May cause direct damage to kidney by
A. GI System excretion of metabolites; encourage fluids and
1. Nausea and vomiting frequent voiding to prevent accumulation of
a. Administer antiemetics routinely every metabolites in bladder.
46 hours as well as prophylactically 2. Increased excretion of uric acid may damage
before chemotherapy is initiated. kidneys.
b. Withhold foods/fluids 46 hours before 3. Administer allopurinol (Zyloprim) as ordered
chemotherapy. to prevent uric acid formation; encourage
c. Provide bland foods in small amounts after fluids when administering allopurinol.
treatments. E. Reproductive System
2. Diarrhea 1. Damage may occur to both men and women
a. Administer antidiarrheals. resulting in infertility and/or mutagenic
b. Maintain good perineal care. damage to chromosomes.
c. Give clear liquids as tolerated. 2. Banking sperm often recommended for men
d. Monitor potassium, sodium, and chloride before chemotherapy.
levels. 3. Clients and partners advised to use reliable
3. Stomatitis methods of contraception during
a. Provide and teach the client good oral chemotherapy.
hygiene, including avoidance of F. Neurologic System
commercial mouthwashes. 1. Plant alkaloids (vincristine) cause neurologic
b. Rinse with viscous lidocaine before meals damage with repeated doses.
to provide an analgesic effect. 2. Peripheral neuropathies, hearing loss, loss of
c. Perform a cleansing rinse with plain water deep tendon reflexes, and paralytic ileus may
or dilute a water-soluble lubricant such as occur.
hydrogen peroxide after meals.
d. Apply water-soluble lubricant such as K-Y
jelly to lubricate cracked lips. Radiation Therapy
e. Advise client to suck on Popsicles to Principles
provide moisture.
B. Hematologic System A. Radiation therapy uses ionizing radiation to kill or
1. Thrombocytopenia limit the growth of cancer cells, may be internal or
a. Teach client the importance of avoiding external.
bumping or bruising the skin. B. It not only injures the cell membrane, but destroys
b. Protect client from physical injury. or alters DNA so that the cells cannot reproduce.
c. Avoid aspirin or aspirin products. C. Like chemotherapy, effect cannot be limited to
d. Avoid giving IM injections. cancer cells only; all exposed cells, including
e. Monitor blood counts carefully. normal ones, will be injured, causing side effects.
f. Assess for and teach signs of increased Localized effects are related to area of body being
bleeding tendencies (epistaxis, petechiae, treated; generalized effects may be related to
ecchymoses). cellular breakdown products.
2. Leukopenia D. Types of energy emitted:
a. Use careful handwashing technique. 1. Alpha: particles cannot pass through skin,
b. Maintain reverse isolation if white blood rarely used
cell count drops below 1000/mm3. 2. Beta: particles cannot pass through skin,
c. Assess for signs of respiratory infection. somewhat more penetrating than alpha,
d. Instruct client to avoid crowds/persons generally emitted from radioactive isotopes,
with known infection. used for internal source
3. Anemia 3. Gamma rays (electromagnetic or X-rays):
a. Provide for adequate rest periods. penetrate deeper areas of body, most common
b. Monitor hemoglobin and hematocrit. form of external radiotherapy
c. Protect client from injury.
d. Administer oxygen as necessary. Methods of Delivery
C. Integumentary SystemAlopecia
1. Explain that hair loss is not permanent. A. External radiation therapy: beams high-energy rays
2. Offer support and encouragement. directly to the affected area.
3. Scalp tourniquets or scalp hypothermia via ice B. Internal radiation therapy: radioactive material is
pack may be ordered to minimize hair loss injected or implanted in the clients body for a
with some agents. designated period of time.
1. Sealed implants: a radioisotope enclosed

4
4. Advise client to obtain a wig before initiating
treatments. in a container so it does not circulate in

ADULT NURSING 193


53155_04_Ch04a_p143-263.qxd 2/26/09 6:50 PM Page 194

the body; clients body fluids should not D. Anemia, leukopenia, and thrombocytopenia
become contaminated with radiation. 1. Isolate from those with known infections.
2. Unsealed sources: a radioisotope that is not 2. Provide frequent rest periods.
encased in a container and does circulate in 3. Encourage high-protein diet.
the body and contaminates body fluids. 4. Instruct client to avoid injury.
5. Assess for bleeding.
Factors Controlling Exposure 6. Monitor CBC, leukocytes, and platelets.

A. Half-life: time required for half of radioactive


atoms to decay Bone Marrow Transplant
1. Each radioisotope has a different half-life. A. General information
2. At the end of the half-life, the danger from 1. Treatment alternative for a variety of
exposure decreases. diseases
B. Time: the shorter the duration, the less the exposure a. Malignancies including several types of
C. Distance: the greater the distance from the leukemias
radiation source the less the exposure b. Blood disorders including severe aplastic
D. Shielding: all radiation can be blocked; rubber anemia, thalassemia
gloves stop alpha and usually beta rays; thick lead c. Solid tumors such as breast cancer and
or concrete stops gamma rays brain tumors; treatment for these diseases
E. These factors affect health care workers exposure frequently causes bone marrow
as well as clients. destruction; autologous bone marrow
1. Health care worker at greater risk from internal transplant may be indicated (Bone marrow
than external sources harvested before chemotherapy or
2. Film badge can measure the amount of radiation destroys it and infused after
exposure received therapy completed)
3. No pregnant nurses or visitors permitted near d. Other conditions including malignant
radiation source infantile osteopetrosis, some inherited
metabolic disorders
Side Effects of Radiation Therapy 2. Types
and Nursing Interventions a. Autologous: client transplant with own
harvested marrow
A. Skin: itching, redness, burning, oozing, sloughing b. Syngeneic: transplant between identical
1. Keep skin free from foreign substances. twins
2. Avoid use of medicated solutions, ointments, c. Allogeneic: transplant from a genetically
or powders that contain heavy metals such as nonidentical donor
zinc oxide. 1) Most common transplant type
3. Avoid pressure, trauma, infection to skin; use 2) Sibling most common donor
bed cradle. 3. Procedure
4. Wash affected areas with plain water and pat a. Donor suitability determined through
dry; avoid soap. tissue antigen typing; includes human
5. Use cornstarch, olive oil for itching; avoid leukocyte antigen (HLA) and mixed
talcum powder. leukocyte culture (MLC) typing.
6. If sloughing occurs, use a sterile dressing with b. Donor bone marrow is aspirated from
micropore tape. multiple sites along the iliac crests under
7. Teach client to avoid exposing skin to heat, general anesthesia.
cold, or sunlight and to avoid constricting or c. Donor marrow is infused IV into the
irritating clothing. recipient.
B. Anorexia, nausea, and vomiting 4. Early evidence of engraftment seen during the
1. Arrange mealtimes so they do not directly second week post-transplant; hematologic
precede or follow therapy. reconstitution takes 46 weeks; immunologic
2. Encourage bland foods. reconstitution takes months.
3. Provide small, attractive meals. 5. Hospitalization of 2 or 3 months required.
4. Avoid extremes of temperature. 6. Prognosis is highly variable depending on
5. Administer antiemetics as ordered before meals. indication for use.
C. Diarrhea B. Complications
1. Encourage low-residue, bland, high-protein foods. 1. Failure of engraftment
2. Administer antidiarrheal drugs as ordered. 2. Infection: highest risk in first 34 weeks
3. Provide good perineal care. 3. Pneumonia: nonbacterial or interstitial
4. Monitor electrolytes, particularly sodium, pneumonias are principal cause of death

4
potassium, and chloride. during first 3 months post-transplant

194 NCLEX-RN Review


53155_04_Ch04a_p143-263.qxd 2/26/09 6:50 PM Page 195

4. Graft vs. host disease (GVHD): principal b. Provide mouth rinses, viscous lidocaine,
complication; caused by an immunologic and antibiotic rinses.
reaction of engrafted lymphoid cells against c. Do not use lemon and glycerin swabs.
the tissues of the recipient d. Administer parenteral narcotics as ordered
a. Acute GVHD: develops within first if necessary to control pain.
100 days post-transplant and affects e. Provide care every 2 hours or as needed.
skin, gut, liver, marrow, and lymphoid 3. Provide skin care: skin breakdown may result
tissue from profuse diarrhea from the TBI.
b. Chronic GVHD: develops 100400 days 4. Monitor carefully for bleeding.
post-transplant; manifested by multiorgan a. Check for occult blood in emesis and
involvement stools.
5. Recurrent malignancy b. Observe for easy bruising, petechiae on
6. Late complications such as cataracts, skin, mucous membranes.
endocrine abnormalities c. Monitor changes in vital signs.
C. Nursing care: pretransplant d. Check platelet count daily.
1. Recipient immunosuppression attained with e. Replace blood products as ordered (all
total body irradiation (TBI) and chemotherapy blood products should be irradiated).
to eradicate existing disease and create space 5. Maintain fluid and electrolyte balance and
in host marrow to allow transplanted cells to promote nutrition.
grow. a. Measure I&O carefully.
2. Provide protected environment. b. Provide adequate fluid, protein, and
a. Client should be in a laminar airflow room caloric intake.
or on strict reverse isolation; surveillance c. Weigh daily.
cultures done twice a week. d. Administer fluid replacement as ordered.
b. Objects must be sterilized before being e. Monitor hydration status: check skin
brought into the room. turgor, moisture of mucous membranes,
c. When working with children introduce urine output.
new people where they can be seen, but f. Check electrolytes daily.
outside childs room so child can see what g. Check urine for glucose, ketones, protein.
they look like without isolation garb. h. Administer antidiarrheal agents as needed.
3. Monitor central lines frequently; check 6. Provide client teaching and discharge
patency and observe for signs of infection planning concerning:
(fever, redness around site). a. Home environment (e.g., cleaning, pets,
4. Provide care for the client receiving visitors)
chemotherapy and radiation therapy to induce b. Diet modifications
immunosuppression. c. Medication regimen: schedule, dosages,
a. Administer chemotherapy as ordered, effects, and side effects
assist with radiation therapy if required. d. Communicable diseases and
b. Monitor side effects and keep client as immunizations
comfortable as possible. e. Daily hygiene and skin care
c. Monitor carefully for potential infection. f. Fever
d. Client will become very ill; prepare client g. Activity
and family.
D. Nursing care: post-transplant
1. Prevent infection.
a. Maintain protective environment. Sample Questions
b. Administer antibiotics as ordered.
c. Assess all mucous membranes, wounds,
catheter sites for swelling, redness, 86. A woman is undergoing chemotherapy treatment
tenderness, pain. for uterine cancer. She asks the nurse how
d. Monitor vital signs frequently (every chemotherapeutic drugs work. The most
14 hours as needed). accurate explanation would include which
e. Collect specimens for cultures as needed statement?
and twice a week. 1. They affect all rapidly dividing cells.
f. Change IV setups every 24 hours.
2. Molecular structure of the DNA segment is
2. Provide mouth care for stomatitis and
altered.
mucositis (severe mucositis develops about
5 days after irradiation). 3. Chemotherapy only kills cancer cells.
a. Note tissue sloughing, bleeding, changes in 4. The cancer cells are sensitive to drug

4
color. toxins.

ADULT NURSING 195


53155_04_Ch04a_p143-263.qxd 2/26/09 6:50 PM Page 196

87. An adult experiences severe vomiting from 93. A man says to the nurse, I dont understand
cancer chemotherapy drugs. Which acid-base how my wife could have come down with
imbalance should the nurse anticipate? cancer. She doesnt smoke or drink. How do
1. Ketoacidosis. people get cancer? Which of the following
2. Metabolic acidosis. should be included in the nurses response?
Select the one or all that apply.
3. Metabolic alkalosis.
___ Bacteria.
4. Respiratory alkalosis.
___ Viruses.
88. A woman loses most of her hair as a result of ___ Dietary factors.
cancer chemotherapy. The nurse understands ___ Genetic factors.
that which of the following is true about
chemotherapy-induced alopecia? 94. A woman has breast cancer. Her physician has
1. New hair will be gray. just told her that her cancer has been staged as
2. Avoid the use of wigs. T2, N1, M0, and the client asks the nurse what
this means. What is the nurses best response?
3. The hair loss is temporary.
1. The primary tumor is 2 cm in diameter, she
4. Pre-chemo hair texture will return.
has one positive lymph node, and no
89. An adult is diagnosed with Hodgkins disease metastasis.
Stage 1A. He is being treated with radiation 2. There are two primary tumors, one involved
therapy. To minimize skin damage from lymph node chain, and no metastasis.
radiation therapy, the nursing care plan should 3. The primary tumor is between 2 cm and
include which of the following? 5 cm in size, she has metastasis to one movable
1. Avoid washing with water. lymph node, and no distant metastasis.
2. Apply a heating pad to the site. 4. There is carcinoma in situ, no regional
3. Cover the area with an airtight dressing. lymph node metastasis, and the presence of
distant metastasis cannot be assessed.
4. Avoid applying creams and powders to the
area. 95. The nurse at a senior citizen center is teaching
a class on the early warning signals of cancer.
90. An adult develops a second-degree or second-
Which of the following will be a part of the
level skin reaction from radiation therapy. When
teaching plan for this class?
evaluating his symptoms, which of the following
would not be present? 1. Reduction in the amount of dietary fat.
1. Scaly skin. 2. Stop cigarette smoking.
2. An itchy feeling. 3. Avoid overexposure to the sun.
3. Dry desquamation. 4. Practice monthly breast self-exam (BSE).
4. Reddening of the skin. 96. Which statement tells the nurse that a man
needs further information about testicular
91. The nurse is teaching the client about signs of
self-examination (TSE)?
radiation-induced thrombocytopenia. Which
symptom would be included in the teaching? 1. The best time to perform TSE is
immediately before sexual intercourse.
1. Fatigue.
2. Its normal to find one testis lower than the
2. Shortness of breath.
other.
3. Elevated temperature.
3. I should have my doctor examine any lumps
4. A tendency to bruise easily. I find, even though they might be benign.
92. The nurse is caring for a client who is receiving 4. That cord-like thing that I feel on the top
radiation therapy. Which activity by the client and back of the testicle is not something to be
indicates further instruction on the side effects worried about.
of radiation therapy?
97. Which of the following actions is vital for the
1. Using an electric razor. nurse to perform when assessing a client
2. Eating a high-protein diet. receiving chemotherapy?
3. Taking his children to see Santa at the mall. 1. Checking complete vital signs every 8 hours.
4. Calling the doctor for a temperature of 101F 2. Taking rectal temperatures every 4 hours to

4
(38.3C). check for infection.

196 NCLEX-RN Review


53155_04_Ch04a_p143-263.qxd 2/26/09 6:50 PM Page 197

3. Testing emesis for blood. (Hgb): 7.6 gm/dl; RBC 3,000,000/mm3;


4. Avoiding fresh fruits and vegetables if hematocrit (Hct) 22.3%; platelets 20,000/mm3.
absolute white blood count (WBC) is less The nurse knows that the clients plan of care
than 1000/mm3. will include which of the following
interventions?
98. An adult asks the nurse how the chemotherapy 1 Insertion of two extra IV lines for blood
that she is receiving for her lung cancer works. administration.
What is the nurses best response?
2. Cutting fingernails and toenails to prevent
1. Block the sodium-potassium pump in the cell scratching that could lead to bleeding or
wall and cause cellular death due to an infection.
excess of intracellular potassium.
3. Providing the client with a low-residue diet.
2. Prevent the entry of oxygen into the cell and
4. Using a soft toothbrush and avoiding dental
cause cellular death due to cellular anoxia.
floss.
3. Shrink the size of the existing tumor, which
causes the release of antitumor metabolites 102. An adult who is receiving external radiation
that are toxic to further tumor cell growth. therapy for Hodgkins disease makes all of the
4. Destroy enough of the tumor so that the following statements to her nurse. Which
bodys immune system can eradicate the statement tells the nurse that the client needs
remaining cells. further teaching about the care she requires
because of her radiation therapy?
99. An adult, diagnosed with multiple myeloma, is 1. I will check my mouth frequently for signs
receiving cyclophosphamide (Cytoxan). The nurse of irritation.
must include which of the following interventions
2. I know that if I get tired easily, it may be
in the nursing care plan of this client?
from the radiation and doesnt necessarily
1. High-flow oxygen delivery to combat mean my Hodgkins disease is getting worse.
interstitial pneumonitis, which routinely
3. I will use a good quality lotion on my skin
develops with cyclophosphamide therapy.
to keep the radiation from burning it.
2. Encouraging the client to empty his bladder
4. I may lose some of my hair during radiation
every 2 to 3 hours to prevent development of
and foods may not taste right.
hemorrhagic cystitis.
3. Application of an ice cap to reduce or 103. A client asks the nurse to explain how the
prevent alopecia. radiation therapy she will be receiving for her
4. Antiemetic therapy for 710 days after neck cancer is effective. What is the nurses best
cyclophosphamide administration, or until response?
blood studies show nadir is reached. 1. Radiation causes breakage in the strands of
the DNA helix, which leads to cell death.
100. An adult is receiving cancer chemotherapy and
2. Radiation is antagonistic to glucose, which
demonstrates alteration in her oral mucous
cells need for energy and replication;
membranes. Which of the following should be
radiation prevents glucose from entering
included in her plan of care?
cells, leading to cell death.
1. Brushing teeth and flossing after every meal
3. Cell walls are broken down by gamma rays
and at bedtime.
during radiation therapy, leading to cell death.
2. Using normal saline mouth rinses every
4. Oxygen cannot enter cells that have been
2 hours while awake.
irradiated, so the cell converts to anaerobic
3. No use of dentures until mucous membranes metabolism that causes its death.
have healed.
4. Bland, mechanical, soft diet until mucous 104. The nurse manager on the oncology unit is
membranes have healed. assessing the knowledge level of the staff in
regard to safety requirements for the client
101. Ten days ago, a client received chemotherapy for receiving internal radiation therapy. Which
his non-Hodgkins lymphoma, stage IV. Drugs observation by the nurse manager indicates that
administered through his vascular access device further instruction is necessary?
(Port-a-cath) included doxorubicin, 1. The physical therapist is ambulating the
cyclophosphamide, vincristine, and prednisone client in the hall.
(CHOP protocol). This morning, his blood work
2. The nurse uses rubber gloves when emptying

4
is as follows: WBC: 1500/mm3; hemoglobin
the bedpan.

ADULT NURSING 197


53155_04_Ch04a_p143-263.qxd 2/26/09 6:50 PM Page 198

3. The dietitian provides a low-residue diet for 87. 3. Severe vomiting results in a loss of
the client. hydrochloric acid and acids from extracellular
4. The housekeeping staff calls for Radiation fluids, leading to metabolic alkalosis.
Safety personnel to inspect the room before
the client is discharged. 88. 3. Alopecia from chemotherapy is only temporary.

105. A woman is receiving internal radiation therapy 89. 4. Creams and powders, many of which contain
for cancer of the cervix. Which statement heavy metals, will further irritate skin sensitized
indicates to the nurse that the client understands by radiation therapy and reduce the effectiveness
precautions necessary during her treatment? of therapy by blocking radiation.
1. I should get out of bed and walk around in 90. 4. Reddening of the skin will not be seen in
my room at least every other hour. a second-level or second-degree reaction. A
2. My seven-year-old twins should not come to second-degree skin reaction would be evidenced
visit me while Im receiving treatment. by scaly skin, an itchy feeling, and dry
3. I will try not to cough, because the force desquamation.
might make me expel the applicator.
91. 4. Clients with decreased platelet count
4. I know that my primary nurse has to wear one
(thrombocytopenia) bleed easily. Thrombocytes
of those badges like the people in the X-ray
are clotting cells.
department wear, but they arent necessary for
anyone else who comes in here. 92. 3. People being treated with radiation therapy
should avoid crowds because of the increased
106. An adult is receiving internal radiation therapy
risk of infection. Crowds at Christmastime can
for cancer of the cervix. Her radiation source, a
be very large and children are frequent carriers
rod, becomes dislodged. What will be the nurses
of infection.
first action?
1. Notify the Radiation Safety personnel at once 93. Bacteria should be marked. Helicobacter pylori,
and await further information. which causes stomach ulcers, has been linked to
2. Use long-handled forceps to remove the rod stomach cancer.
and place in a lead-lined container. Viruses should be marked. Viruses are thought
3. Apply two sets of rubber gloves and pick up to insinuate themselves into the genetic structure
the rod; place it in a white plastic of cells, thereby altering future generations of
biohazard bucket and call Radiation Safety that cell. Epstein-Barr is strongly implicated in
personnel for a special pick-up. the development of Burkitts lymphoma. Some
types of human papilloma virus, which causes
4. Use long-handled forceps to pick up rod; clean
genital warts, cause cancer of the cervix.
with normal saline, and reinsert into clients
Dietary factors should be marked.
vagina, stopping when the rod meets resistance.
Approximately 4060% of all environmental
This indicates that it is against the cervix.
cancers are thought to have links to dietary factors
107. In caring for the client receiving external such as fats, alcohol, foods containing
radiation therapy, the nurse assesses for which nitrates/nitrites, and salt-cured and smoked meats.
of the following side effects? Genetic factors should be marked. Genetics are
1. Extravasation injury at the IV site used for involved in cancer cell development. Damage to
contrast media injection. the DNA in certain populations of cells may lead
to mutant cells being transmitted to future
2. Generalized or local edema.
generations. Examples of cancers associated with
3. Infection and bleeding. familial inheritance include breast, colon, and
4. Allergic reactions, particularly anaphylaxis. rectal cancers.

94. 3. In the TNM staging classification system,


T refers to the primary tumor, and T2 is between
Answers and Rationales 2 cm and 5 cm without extension to chest wall
or skin. The N refers to regional lymph node
involvement, with N1 indicating spread to an
86. 1. There are numerous mechanisms of action ipsilateral movable node. N0 indicates no regional
for cancer chemotherapeutic drugs, but most lymph node spread; N2 indicates metastasis to an
affect rapidly dividing cells. The drugs interfere ipsilateral axillary node fixed to another node or

4
with cell division and prevent rapid division of other structure. M refers to distant metastasis,
cells.

198 NCLEX-RN Review


53155_04_Ch04a_p143-263.qxd 2/26/09 6:50 PM Page 199

with MX indicating that metastasis cannot be 100. 2. The client will use normal saline mouth rinses
assessed, M0 that there is no distant spread, and every 2 hours while awake and every 6 hours at
M1 that there is spread present. night to aid in the removal of thick secretions,
debris, and bacteria.
95. 4. Only this answer, practicing breast self-exam,
will yield a warning signal of cancer (i.e., a 101. 4. By using a soft toothbrush and avoiding dental
breast lump). Be certain that your response floss, the client promotes a healthy oral cavity
answers the question, not just that it contains without risking bleeding or disruption of skin
factual information. integrity, which could lead to infection.

96. 1. The man is mistaken (and needs more teaching) 102. 3. It is important to protect skin from irritation,
if he says that testicular self-exam should be and lotions, creams, powders, and ointments can
performed immediately prior to sexual intercourse. all contribute to skin problems. Clients are
The best time to do TSE is when the scrotum is advised to consult their radiation oncologists for
relaxed, such as after a warm bath or shower. troublesome skin problems and should be advised
that after treatment, reepithelialization will occur.
97. 3. Because of the bleeding disorders common in
clients receiving chemotherapy, all body 103. 1. There are two types of ionizing radiation:
secretions, including emesis, should be assessed electromagnetic rays and particulate radiation.
for obvious and occult blood. Either of these can cause tissue disruption, the
most harmful of which is the alteration in the
98. 4. Each time the tumor is exposed to the structure of the cells DNA molecules; this will
chemotherapeutic drug, a certain percentage of lead to cell death.
cells are killed. (The exact percentage is
determined by the drug dosage used.) Because a 104. 1. The client will be restricted to her room to
percentage of tumor is killed, a part of tumor minimize exposure of staff, visitors, and other
will remain after therapy. It is up to the bodys clients to the radiation source.
immune system to destroy the remaining tumor,
which an intact immune system may be able to 105. 2. Visitors younger than 18 years of age, and
do if the tumor is made small enough. pregnant visitors, are not allowed during
internal radiation therapy.
99. 2. If the metabolites of cyclophosphamide are
allowed to accumulate in the bladder, the 106. 2. Long-handled forceps and a lead-lined
subsequent irritation of the bladder wall container (sometimes called a lead pig) must be
capillaries will cause hemorrhagic cystitis. This kept in the room of any client receiving internal
condition is preventable; if it develops, one of its radiation therapy for this very occurrence.
serious sequelae is bladder fibrosis. In addition to
107. 3. If bone marrow-producing sites are included
monitoring BUN and creatinine prior to
in the field being irradiated, anemia, leukopenia
administration, the nurse must promote hydration
(low white blood cells), and thrombocytopenia
of at least 3 liters a day and frequent voiding.
(low platelets) may occur; these may lead to
infection and/or bleeding.

The Neurosensory System

OVERVIEW OF ANATOMY and spinal cord, and the peripheral nervous system
(PNS), which includes the cranial nerves and the spinal
AND PHYSIOLOGY nerves. The autonomic nervous system (ANS) is a
subdivision of the PNS that automatically controls body
The Nervous System functions such as breathing and heartbeat. It is further
divided into the sympathetic and parasympathetic
The functional unit of the nervous system is the nerve nervous systems. The special senses of vision and
cell, or neuron. The nervous system consists of the

4
hearing are also covered in this section.
central nervous system (CNS), which includes the brain

ADULT NURSING 199


53155_04_Ch04a_p143-263.qxd 2/26/09 6:50 PM Page 200

Neuron
A. Primary component of the nervous system;
composed of cell body (gray matter), axon, and
dendrites
B. Axon: elongated process or fiber extending from
the cell body; transmits impulses (messages) away
from the cell body to dendrites or directly to the
cell bodies of other neurons; neuron usually has
only one axon.
C. Dendrites: short, branching fibers that receive
impulses and conduct them toward the nerve cell
body. Neurons may have many dendrites.
D. Synapse: junction between neurons where an
impulse is transmitted
E. Neurotransmitters: chemical agents (e.g.,
acetylcholine, norepinephrine) involved in the
transmission of impulse across synapse
F. Myelin sheath: a wrapping of myelin (a whitish, fatty Figure 4-1 Side view of the brain, showing principal
material) that protects and insulates nerve fibers and functional areas
enhances the speed of impulse conduction
1. Both axons and dendrites may or may not have
a myelin sheath (myelinated/unmyelinated) 3. Corpus callosum: large fiber tract that connects
2. Most axons leaving the CNS are heavily the two cerebral hemispheres
myelinated by Schwann cells 4. Basal ganglia: islands of gray matter within
white matter of cerebrum
Functional Classification a. Regulate and integrate motor activity
A. Afferent (sensory) neurons: transmit impulses from originating in the cerebral cortex
peripheral receptors to the CNS b. Part of extrapyramidal system
B. Efferent (motor) neurons: conduct impulses from B. Diencephalon: connecting part of the brain,
CNS to muscles and glands between the cerebrum and the brain stem.
C. Internuncial neurons (interneurons): connecting Contains several small structures; the thalamus
links between afferent and efferent neurons and hypothalamus are most important.
1. Thalamus
a. Relay station for discrimination of sensory
Central Nervous System: signals (e.g., pain, temperature, touch).
Brain and Spinal Cord b. Controls primitive emotional responses
Brain (e.g., rage, fear).
2. Hypothalamus
A. Cerebrum: outermost area (cerebral cortex) is gray a. Found immediately beneath the thalamus.
matter; deeper area is composed of white matter b. Plays major role in regulation of vital
1. Two hemispheres: right and left functions such as blood pressure, sleep,
2. Each hemisphere divided into four lobes; many food and water intake, and body
of the functional areas of the cerebrum have temperature.
been located in these lobes (see Figure 4-1). c. Acts as control center for pituitary gland
a. Frontal lobe and affects both divisions of the autonomic
1) Personality, behavior nervous system.
2) Higher intellectual functioning C. Brain stem
3) Precentral gyrus: motor function 1. Contains midbrain, pons, and medulla
4) Brocas area: specialized motor speech oblongata.
area 2. Extends from the cerebral hemispheres to the
b. Parietal lobe foramen magnum at the base of the skull.
1) Postcentral gyrus: registers general 3. Contains nuclei of the cranial nerves and the
sensation (e.g., touch, pressure) long ascending and descending tracts
2) Integrates sensory information connecting the cerebrum and the spinal cord.
c. Temporal lobe 4. Contains vital centers of respiratory,
1) Hearing, taste, smell vasomotor, and cardiac functions.
2) Wernickes area: sensory speech area D. Cerebellum: coordinates muscle tone and
(understanding/formulation of language) movements and maintains position in space

4
d. Occipital lobe: vision (equilibrium).

200 NCLEX-RN Review


53155_04_Ch04a_p143-263.qxd 2/26/09 6:50 PM Page 201

Spinal Cord B. Spinal column


A. Serves as a connecting link between the brain and 1. Consists of 7 cervical, 12 thoracic, and
the periphery. 5 lumbar vertebrae, as well as sacrum and
B. Extends from foramen magnum to second lumbar coccyx.
vertebra. 2. Supports the head and protects the spinal cord.
C. H-shaped gray matter in the center (cell bodies) C. Meninges
surrounded by white matter (nerve tracts and fibers). 1. Membranes between the skull and brain and
D. Gray matter the vertebral column and spinal cord.
1. Anterior horns: contain cell bodies giving rise 2. Layers
to efferent (motor) fibers a. Dura mater: outermost layer, tough,
2. Posterior horns: contain cell bodies connecting leathery
with afferent (sensory) fibers from dorsal root b. Arachnoid mater: middle layer, weblike
ganglion c. Pia mater: innermost layer, delicate, clings
3. Lateral horns: in thoracic region, contain cells to surface of brain
giving rise to autonomic fibers of sympathetic 3. Area between arachnoid and pia mater is
nervous system called subarachnoid space.
E. White matter D. Ventricles
1. Ascending tracts (sensory pathways) 1. Four fluid-filled cavities connecting with one
a. Posterior columns: carry impulses another and the spinal canal.
concerned with touch, pressure, vibration, 2. Produce and circulate cerebrospinal fluid.
and position sense. E. Cerebrospinal fluid (CSF)
b. Spinocerebellar: carry impulses concerned 1. Surrounds brain and spinal cord.
with muscle tension and position sense to 2. Offers protection by functioning as a shock
cerebellum. absorber.
c. Lateral spinothalamic: carry impulses 3. Allows fluid shifts from the cranial cavity to
resulting in pain and temperature sensations. the spinal cavity.
d. Anterior spinothalamic: carry impulses 4. Carries nutrients to and waste products away
concerned with crude touch and pressure. from nerve cells.
2. Descending tracts (motor pathways) F. Vascular supply
a. Corticospinal (pyramidal, upper motor 1. Two internal carotid arteries anteriorly.
neuron): conduct motor impulses from 2. Two vertebral arteries leading to basilar artery
motor cortex to anterior horn cells (cross in posteriorly.
the medulla). 3. These arteries communicate at the base of the
b. Extrapyramidal: help to maintain muscle brain through the circle of Willis.
tone and to control body movement, 4. Anterior, middle, and posterior cerebral
especially gross automatic movements arteries are the main arteries for distributing
such as walking. blood to each hemisphere of the brain.
F. Reflex arc 5. Brain stem and cerebellum are supplied by
1. Reflex consists of an involuntary response to a branches of the vertebral and basilar arteries.
stimulus occurring over a neural pathway 6. Venous blood drains into dural sinuses and
called a reflex arc. then into internal jugular veins.
2. Not relayed to and from brain; takes place at G. Blood-brain barrier: protective barrier preventing
cord levels. harmful agents from entering the capillaries of the
3. Components CNS; protects brain and spinal cord.
a. Sensory receptor: receives/reacts to a
stimulus. Peripheral Nervous System
b. Afferent pathway: transmits impulses to
Spinal Nerves
spinal cord.
c. Interneuron: synapses with a motor neuron A. 31 pairs: carry impulses to and from spinal cord.
(anterior horn cell). B. Each segment of the spinal cord contains a pair of
d. Efferent pathway: transmits impulses from spinal nerves (one for each side of the body).
motor neuron to effector. C. Each nerve is attached to the spinal cord by two
e. Effector: muscle or organ that responds to roots.
stimulus. 1. Dorsal (posterior) root: contains afferent
(sensory) nerve whose cell body is in the
Supporting Structures dorsal root ganglion.
2. Ventral (anterior) root: contains efferent
A. Skull (motor) nerve whose nerve fibers originate in
1. Rigid; numerous bones fused together. the anterior horn cell of the spinal cord (lower

4
2. Protects and supports the brain. motor neuron).

ADULT NURSING 201


53155_04_Ch04a_p143-263.qxd 2/26/09 6:50 PM Page 202

Table 4-17 Cranial Nerves Vision


External Structures of Eye
Name and
Number Function A. Eyelids (palpebrae) and eyelashes: protect the eye
from foreign particles
Olfactory: Sensory: carries impulses for sense of
B. Conjunctiva
cranial nerve I smell
1. Palpebral conjunctiva: pink; lines inner
Optic: Sensory: carries impulses for vision
surface of eyelids.
cranial nerve II
2. Bulbar conjunctiva: white with small blood
Oculomotor: Motor: muscles for pupillary
vessels, covers anterior sclera.
cranial nerve III constriction, elevation of upper eyelid;
C. Lacrimal apparatus (lacrimal gland and its ducts
4 out of 6 extraocular movements
and passages): produces tears to lubricate the eye
Trochlear: Motor: muscles for downward, inward
and moisten the cornea; tears drain into the
cranial nerve IV movement of eye
nasolacrimal duct, which empties into nasal
Trigeminal: Mixed: impulses from face, surface of
cavity.
cranial nerve V eyes (corneal reflex); muscles
D. Movement of the eye is controlled by six
controlling mastication
extraocular muscles.
Abducens: Motor: muscles for lateral deviation
cranial nerve VI of eye
Facial: Mixed: impulses for taste from anterior
Internal Structures of Eye
cranial nerve VII tongue; muscles for facial movement
Acoustic: Sensory: impulses for hearing (cochlear A. Three layers of the eyeball
cranial nerve VIII division) and balance (vestibular 1. Outer layer
division) a. Sclera: tough, white connective tissue
Glossopharyngeal: Mixed: impulses for sensation to (white of the eye); located anteriorly and
cranial nerve IX posterior tongue and pharynx; posteriorly
muscles for movement of pharynx b. Cornea: transparent tissue through
(elevation) and swallowing which light enters the eye; located
Vagus: Mixed: impulses for sensation to lower anteriorly
cranial nerve X pharynx and larynx; muscles for 2. Middle layer
movement of soft palate, pharynx, a. Choroid: highly vascular layer, nourishes
and larynx; parasympathetic retina; located posteriorly
Spinal accessory: Motor: movement of sternomastoid b. Ciliary body: anterior to choroid, secretes
cranial nerve XI: muscles and upper part of trapezius aqueous humor; muscles change shape of
muscles lens
Hypoglossal: Motor: movement of tongue c. Iris: pigmented membrane behind cornea,
cranial nerve XII gives color to eye; located anteriorly. Pupil
is a circular opening in the middle of the
iris that constricts or dilates to regulate
amount of light entering eye.
Cranial Nerves 3. Inner layer: retina
A. 12 pairs: carry impulses to and from brain (see a. Light-sensitive layer composed of rods and
Table 4-17). cones (visual cells)
B. May have sensory, motor, or mixed functions. 1) Cones: specialized for fine
discrimination and color vision
Autonomic Nervous System 2) Rods: more sensitive to light than
cones, aid in peripheral vision
A. Part of the peripheral nervous system b. Optic disk: area in retina for entrance of
B. Includes those peripheral nerves (both cranial and optic nerve, has no photoreceptors
spinal) that regulate functions occurring B. Lens: transparent body that focuses image on retina
automatically in the body; ANS regulates smooth C. Fluids of the eye
muscle, cardiac muscle, and glands. 1. Aqueous humor: clear, watery fluid in anterior
C. Components and posterior chambers in anterior part of eye;
1. Sympathetic nervous system: generally serves as refracting medium and provides
accelerates some body functions in response to nutrients to lens and cornea; contributes to
stress; fight or flight maintenance of intraocular pressure.
2. Parasympathetic nervous system: controls 2. Vitreous humor: clear, gelatinous material that
normal body functioning fills posterior cavity of eye; maintains
D. Effects of ANS activity: see Table 4-18. transparency and form of eye.

4 202 NCLEX-RN Review


53155_04_Ch04a_p143-263.qxd 2/26/09 6:50 PM Page 203

Table 4-18 Effects of Autonomic Nervous System Activity Hearing


External Ear
Sympathetic Parasympathetic
Effector (Adrenergic) Effects (Cholinergic) Effects A. Auricle (pinna): outer projection of ear composed
of cartilage and covered by skin; collects sound
Eye Dilates pupil Constricts pupil
waves.
(mydriasis) (miosis)
B. External auditory canal: lined with skin; glands
Glands secrete cerumen (wax), providing protection;
of head transmits sound waves to tympanic membrane.
Lacrimal No effect Stimulates secretion C. Tympanic membrane (eardrum): at end of external
Salivary Scanty thick, viscous Copious thin, watery canal; vibrates in response to sound and transmits
secretions, dry mouth secretions vibrations to middle ear.
Heart Increases rate and Decreases rate
force of contraction Middle Ear
Blood Constricts smooth No effect A. Ossicles
vessels muscles of skin, 1. 3 small bones: malleus (hammer) attached to
abdominal blood tympanic membrane, incus (anvil), stapes
vessels, and (stirrup)
cutaneous blood 2. Ossicles are set in motion by sound waves
vessels from tympanic membrane.
Dilates smooth muscle 3. Sound waves are conducted by vibration to
of bronchioles, the footplate of the stapes in the oval window
blood vessels of (an opening between the middle ear and the
heart, and skeletal inner ear).
muscles B. Eustachian tube: connects nasopharynx and
middle ear; brings air into middle ear, thus
Lungs Bronchodilation Bronchoconstriction
equalizing pressure on both sides of eardrum.
GI tract Decreases motility Increases motility
Constricts sphincters Relaxes sphincters Inner Ear
Possibly inhibits Stimulates secretion
secretions Stimulates activity of A. Cochlea
Inhibits activity of gallbladder and 1. Contains organ of Corti, the receptor end-organ
gallbladder and ducts for hearing.
ducts 2. Transmits sound waves from the oval window
Inhibits glycogenolysis and initiates nerve impulses carried by cranial
in liver nerve VIII (acoustic branch) to the brain
(temporal lobe of cerebrum).
Adrenal Stimulates secretion No effects
B. Vestibular apparatus
gland of epinephrine and
1. Organ of balance.
norepinephrine
2. Composed of three semicircular canals and the
Urinary Relaxes detrusor Contracts detrusor utricle.
tract muscle muscle
Contracts trigone Relaxes trigone
sphincter (prevents sphincter (allows ASSESSMENT
voiding) voiding)
Health History
Visual Pathways Nervous System
A. Retina (rods and cones) translates light waves into A. Presenting problem: symptoms may include
neural impulses that travel over the optic nerves. behavior changes, memory loss, mood changes,
B. Optic nerves for each eye meet at the optic chiasm. nervousness or anxiety, headache, seizures, syncope,
1. Fibers from median halves of the retinas cross vertigo, loss of consciousness; problems with
here and travel to the opposite side of the brain. speech, vision, or smell; motor problems (paralysis,
2. Fibers from lateral halves of retinas remain tremor); sensory problems (pain, paresthesias)
uncrossed. B. Lifestyle: drug and alcohol intake, exposure to
C. Optic nerves continue from optic chiasm as optic toxins, recent travel, employment, stressors
tracts and travel to the cerebrum (occipital lobe), C. Use of medications: prescribed and over-the-

4
where visual impulses are perceived and interpreted. counter (OTC)

ADULT NURSING 203


53155_04_Ch04a_p143-263.qxd 2/26/09 6:50 PM Page 204

D. Past medical history


1. Perinatal exposure to toxic agents, X-rays;
difficult labor and delivery
2. Childhood and adult: history of systemic
diseases; seizures; loss of consciousness; head
trauma
E. Family history: may uncover diseases with
hereditary or congenital background

Eye
A. Presenting problem: symptoms may include
blurred vision, decreased vision, or blind spots;
pain, redness, excessive tearing; double vision
(diplopia); drainage
B. Use of eyeglasses, contact lenses; date of last eye
exam
C. Lifestyle: occupation (exposure to fumes, smoke, Figure 4-2 Pathologic reflex (Babinski)
or eye irritant); use of safety glasses
D. Use of medications: cortisone preparations may 3) Dysarthria: difficult speech due to
contribute to formation of glaucoma and cataracts impairment of muscles involved with
E. Past medical history: systemic diseases; previous production of speech
childhood or adult eye disorders, eye trauma 2. Cranial nerves (see Table 4-17)
F. Family history: many eye disorders may be 3. Cerebellar function: posture, gait, balance,
inherited coordination
4. Motor function: muscle size, tone, strength;
Ear abnormal or involuntary movements
5. Sensory function: light touch, superficial pain,
A. Presenting problem: symptoms may include
temperature, vibration, and position sense
hearing loss, tinnitus (ringing in ear), dizziness or
6. Reflexes
vertigo, pain, drainage
a. Deep tendon: grade from 0 (no response) to
B. Lifestyle: occupation (exposure to excessive noise
4 (hyperactive); 2 is normal
levels), swimming habits
b. Superficial
C. Use of medications: ototoxic drugs; aspirin
c. Pathologic: Babinskis reflex (dorsiflexion
(tinnitus)
of great toe with fanning of other toes)
D. Past medical history
indicates damage to corticospinal tracts
1. Perinatal: rubella in first trimester of
(see Figure 4-2)
pregnancy
B. Neuro check
2. Childhood and adult: otitis media, perforated
1. Level of consciousness (LOC)
eardrum, measles, mumps, allergies,
a. Orientation to time, place, and person
tonsillectomy, and adenoidectomy
b. Speech: clear, garbled, rambling
E. Family history: hearing loss in family members
c. Ability to follow commands
d. If client does not respond to verbal stimuli,
Physical Examination apply a painful stimulus (e.g., pressure on
nailbeds, squeeze trapezius muscle); note
Nervous System response to pain:
A. Neurologic examination 1) Appropriate: withdrawal, moaning
1. Mental status exam (cerebral function); see 2) Inappropriate: nonpurposeful
also Unit 7. e. Abnormal posturing (may occur
a. General appearance and behavior spontaneously or in response to stimulus)
b. Level of consciousness; see Neuro Check. 1) Decorticate posturing: extension of
c. Intellectual function: memory (recent and legs, internal rotation and adduction of
remote), attention span, cognitive skills arms with flexion of elbows, wrists,
d. Emotional status and fingers (damage to corticospinal
e. Thought content tracts; cerebral hemispheres)
f. Language/speech 2) Decerebrate posturing: back arched,
1) Expressive aphasia: inability to speak rigid extension of all four extremities
2) Receptive aphasia: inability to with hyperpronation of arms and
understand spoken words plantar flexion of feet (damage to

4
upper brain stem, midbrain, or pons)

204 NCLEX-RN Review


53155_04_Ch04a_p143-263.qxd 2/26/09 6:50 PM Page 205

of 25/minute) with normal blood oxygen


levels; usually due to brain stem
dysfunction.
c. Apneustic breathing: prolonged inspiratory
phase, followed by a 2- to 3-second pause;
usually indicates dysfunction of
respiratory center in pons.
d. Cluster breathing: clusters of irregular
breathing, irregularly followed by periods
of apnea; usually caused by a lesion in
upper medulla and lower pons.
e. Ataxic breathing: breathing pattern
completely irregular; indicates damage to
respiratory centers of the medulla.

Eye
A. Visual acuity: Snellen chart
B. Visual fields (peripheral vision)
1. Confrontation method
2. Perimetry: more precise method
C. External structures
1. Position and alignment of eyes
2. Eyebrows, eyelids, lacrimal apparatus,
conjunctiva, sclera, cornea, iris, pupils (size,
Figure 4-3 Glasgow Coma Scale shape, equality, and reaction to light)
D. Extraocular movements; note paralysis, nystagmus
(rapid, abnormal movement of the eyeball)
2. Glasgow coma scale (see Figure 4-3) E. Corneal reflex
a. Objective evaluation of LOC, motor/verbal
response; a standardized system for Ear
assessing the degree of neurologic
impairment in critically ill clients. A. Inspection and palpation of auricle, preauricular
b. Cannot replace a complete neurologic area, and mastoid area
check, but can be used as an aid in B. Hearing acuity
evaluation and to eliminate ambiguous 1. Whispered voice or ticking watch tests: gross
terms such as stupor and lethargy. estimation
c. A score of 15 indicates client is awake and 2. Audiometry: more precise method
oriented; the lowest score, 3, is deep coma; C. Tuning fork tests distinguish between
a score of 7 or below is considered coma. sensorineural and conductive hearing loss.
3. Pupillary reaction and eye movements 1. Conductive hearing loss: secondary to problem
a. Observe size, shape, and equality of pupils in external or middle ear; transmission of
(note size in millimeters) sound waves to inner ear impaired
b. Reaction to light: pupillary constriction 2. Sensorineural (perceptive) hearing loss:
c. Corneal reflex: blink reflex in response to disease of inner ear or cranial nerve VIII
light stroking of cornea (acoustic branch)
d. Oculocephalic reflex (dolls eyes): present 3. Webers test: handle of vibrating tuning fork
in unconscious client with intact brain stem placed on midline of clients skull, sound
4. Motor function should be heard equally in midline or in both
a. Movement of extremities (paralysis) ears; in conductive hearing loss, sound is
b. Muscle strength louder in poorer ear; in sensorineural hearing
5. Vital signs: respiratory patterns (may help loss, sound is louder in better ear.
localize possible lesion) 4. Rinnes test: tuning fork placed on mastoid
a. Cheyne-Stokes respiration: regular, process (bone conduction) until sound no
rhythmic alternating between longer heard, then placed in front of the ear
hyperventilation and apnea; may be caused (air conduction); sound should be heard longer
by structural cerebral dysfunction or by (almost twice as long) with air conduction
metabolic problems, such as diabetic coma. than with bone conduction; bone conduction
b. Central neurogenic hyperventilation: greater than air conduction indicates

4
sustained, rapid, regular respirations (rate conductive hearing deficit.

ADULT NURSING 205


53155_04_Ch04a_p143-263.qxd 2/26/09 6:50 PM Page 206

Laboratory/Diagnostic Tests 2. Computer-drawn, detailed pictures of


structures of the body through use of large
Nervous System magnet, radio waves
3. Used to detect intracranial and spinal
A. Lumbar puncture (LP)
abnormalities associated with disorders such
1. A hollow spinal needle introduced into
as cerebrovascular disease, tumors, abscesses,
subarachnoid space of spinal canal between
cerebral edema, hydrocephalus, multiple
L4/L5 for diagnostic or therapeutic reasons
sclerosis
2. Purposes
4. Nursing care
a. Measures CSF pressure (normal opening
a. Instruct client to remove jewelry, hairpins,
pressure 60150 mm H2O)
glasses, wigs (with metal clips), and other
b. Obtain specimens for lab analysis (protein
metallic objects.
[normally not present], sugar [normally
b. Be aware that this test cannot be performed
present], cytology, C&S)
on anyone with orthopedic hardware,
c. Check color of CSF (normally clear) and
intrauterine devices, pacemaker, internal
check for blood
surgical clips, or other fixed metallic
d. Inject air, dye, or drugs into the spinal
objects in the body.
canal
c. Inform client of need to remain still
3. Nursing care: pretest
while completely enclosed in scanner
a. Have client empty bladder.
throughout the procedure, which lasts
b. Position client in lateral recumbent
4560 minutes.
position with head and neck flexed onto
d. Teach relaxation techniques to assist client
the chest and knees pulled up.
to remain still and to help prevent
c. Explain the need to remain still during the
claustrophobia.
procedure.
e. Warn client of normal audible humming
4. Nursing care: posttest
and thumping noises from the scanner
a. Ensure labeling of CSF specimens in
during test.
proper sequence.
f. Have client void before test.
b. Keep client flat for 1224 hours as ordered.
g. Sedate client if ordered.
c. Force fluids.
E. Brain scan
d. Check puncture site for bleeding, leakage
1. Injection of radioactive isotope, followed by
of CSF.
scanning of head; isotopes will accumulate in
e. Assess sensation and movement in lower
abnormal lesions and be recorded by the
extremities.
scanner.
f. Monitor vital signs.
2. Used to detect intracranial masses, vascular
g. Administer analgesics for headache as
lesions, infarcts, hemorrhage
ordered.
3. Nursing care: check for allergy to iodine.
B. X-rays of skull and spine
F. Myelography
1. Used to detect atrophy, erosion, or fractures of
G. Cerebral angiography
bones; calcifications
1. Injection of radiopaque substance into the
2. Pretest nursing care: remove hairpins, glasses,
cerebral circulation via carotid, vertebral,
hearing aids.
femoral, or brachial artery followed by
C. Computerized tomography (CT scan)
X-rays
1. Skull/spinal cord are scanned in successive
2. Used to visualize cerebral vessels and detect
layers by a narrow beam of X-rays; computer
tumors, aneurysms, occlusions, hematomas, or
uses information obtained to construct a
abscesses
picture of the internal structure of the brain;
3. Nursing care: pretest
contrast medium may or may not be used.
a. Explain that client may have warm,
2. Used to detect intracranial and spinal cord
flushed feeling and salty or metallic taste
lesions and monitor effects of surgery or other
in mouth during procedure.
therapy.
b. Check for allergy to iodine.
3. Nursing care
c. Keep NPO after midnight or offer clear
a. Explain appearance of scanner.
liquid breakfast only.
b. Instruct client to lie still during procedure.
d. Take baseline vital signs and neuro check.
c. Check for allergy to iodine if contrast
e. Administer sedation if ordered.
material is used.
4. Nursing care: posttest
d. Remove hairpins, etc.
a. Maintain pressure dressing over site if
D. Magnetic resonance imaging (MRI)
femoral or brachial artery used; apply ice
1. Also known as nuclear magnetic resonance
as ordered.
(NMR)

4 206 NCLEX-RN Review


53155_04_Ch04a_p143-263.qxd 2/26/09 6:50 PM Page 207

b. Maintain bed rest until next morning or as I. Ineffective airway clearance


ordered. J. Ineffective breathing pattern
c. Monitor vital signs and neuro checks K. Risk for injury
frequently; report any changes L. Risk for aspiration
immediately. M. Risk for disuse syndrome
d. Check site frequently for bleeding or N. Risk for impaired skin integrity
hematoma; if carotid artery used, assess for O. Impaired verbal communication
swelling of neck, difficulty swallowing or P. Sexual dysfunction
breathing. Q. Impaired physical mobility
e. Check pulse, color, and temperature of R. Feeding self-care deficit
extremity distal to site used. S. Impaired swallowing
f. Keep extremity extended and avoid flexion. T. Bathing/hygiene self-care deficit
H. Echoencephalography: use of ultrasound to detect U. Dressing/grooming self-care deficit
midline shift of intracranial contents due to brain V. Toileting self-care deficit
tumors, hematomas. W. Disturbed sensory perception: visual, auditory,
I. Electroencephalography (EEG) kinesthetic, gustatory, tactile, olfactory
1. Graphic recording of electrical activity of the X. Unilateral neglect
brain by several small electrodes placed on the Y. Disturbed thought processes
scalp
2. Used to detect focus or foci of seizure activity
and to quantitatively evaluate level of brain PLANNING AND
function (determine brain death)
3. Pretest nursing care: withhold sedatives, IMPLEMENTATION
tranquilizers, stimulants for 23 days.
4. Posttest nursing care: remove electrode paste Goals
with acetone and shampoo hair.
A. Nutritional state will be optimal.
B. Normal body temperature will be maintained.
Eye
C. Complications will be recognized early and treated
A. Ophthalmoscopic exam promptly.
B. Refraction: detects refractive errors and provides D. Adequate bowel and bladder elimination will be
information for prescription of eyeglasses and maintained.
contact lenses E. Cerebral perfusion will be improved.
C. Perimetry: assesses peripheral vision, visual fields F. Adequate respiratory function will be maintained.
D. Tonometry: measures intraocular pressure (normal: G. Client will remain free from any injury resulting
1220 mm Hg) from neurosensory deficits.
H. Clients skin integrity will be maintained.
Ear I. Clients ability to communicate will be improved.
J. Sexual health will return to optimal level.
A. Otoscopic exam K. Mobility will be restored to optimal level.
B. Audiometry: screening test for hearing loss and L. Maximum independence in self-care activities will
diagnostic test to determine degree and type of be attained.
hearing loss M. Sensory perception will be improved.
C. Vestibular function N. Optimal cognitive functioning will be attained.
1. Caloric test
2. Electronystagmography (ENG)
Interventions
Care of the Unconscious Client
ANALYSIS
A. Maintain a clear, patent airway.
Nursing diagnoses for clients with disorders of the 1. Place client in a side-lying or three-quarters
neurosensory system may include: prone position to prevent tongue from
A. Imbalanced nutrition: less than body requirements obstructing airway.
B. Ineffective thermoregulation 2. If tongue is obstructing, insert oral airway.
C. Autonomic dysreflexia 3. Prepare for insertion of a cuffed endotracheal
D. Constipation or tracheostomy tube as the clients condition
E. Bowel incontinence requires.
F. Impaired urinary elimination 4. Suction as needed.
G. Urinary retention 5. Check respiratory rate, depth, and quality

4
H. Ineffective tissue perfusion: cerebral every 12 hours and as needed.

ADULT NURSING 207


53155_04_Ch04a_p143-263.qxd 2/26/09 6:50 PM Page 208

6. Auscultate breath sounds for crackles (rales), B. Assessment findings


rhonchi, or absent breath sounds every 4 hours 1. Earliest sign: decrease in LOC; progresses from
and before and after suctioning. restlessness to confusion and disorientation to
B. Take vital signs and perform neuro checks at lethargy and coma
specified intervals as ordered; report any 2. Changes in vital signs (may be a late sign)
significant changes immediately. a. Systolic blood pressure rises while
C. Maintain fluid and electrolyte balance and ensure diastolic pressure remains the same
adequate nutrition. (widening pulse presence)
1. Administer IV fluids, nasogastric tube feedings b. Pulse slows
as ordered. c. Abnormal respiratory patterns
2. Maintain accurate I&O. (e.g., Cheyne-Stokes respirations)
3. Assess clients hydration status: skin turgor, d. Elevated temperature
check for dry mucous membranes. 3. Pupillary changes
4. Provide mouth care to keep mucous a. Ipsilateral (same side) dilation of pupil
membranes clean, moist, and intact. with sluggish reaction to light from
D. Provide for clients safety. compression of cranial nerve III
1. Keep side rails up at all times. b. Pupil eventually becomes fixed and dilated.
2. Avoid restraints if at all possible. 4. Motor abnormalities
3. Observe client carefully for seizures and a. Contralateral (opposite side) hemiparesis
intervene to avoid precipitating factors: fever, from compression of corticospinal tracts
hypoxia, electrolyte imbalance. b. Decorticate or decerebrate rigidity
4. Protect client if seizure occurs. 5. Headache, projectile vomiting, papilledema
5. Speak softly and use clients name during (edema of the optic disc)
nursing care. C. Nursing care
6. Touch client as gently as possible. 1. Maintain patent airway and adequate
7. Protect clients eyes from corneal irritation. ventilation.
a. Check for corneal reflex. a. Prevention of hypoxia and hypercarbia
b. Instill artificial tears as ordered; patch (increased CO2) important: hypoxia may
eye. cause brain swelling and hypercarbia
E. Prevent complications of immobility. causes cerebral vasodilation, which
1. Keep skin clean, dry, and pressure free. increases ICP.
2. Turn and reposition client every 2 hours. b. Before and after suctioning, hyperventilate
3. Perform passive range-of-motion (ROM) the client with a resuscitator bag connected
exercises every 4 hours. to 100% oxygen. Limit suctioning to
4. Use nursing measures to prevent deformities: 1015 seconds.
footboard/high-topped sneakers to prevent c. Assist with mechanical hyperventilation as
footdrop, splint to prevent wrist drop. indicated: produces hypocarbia (decreased
F. Maintain adequate bladder and bowel CO2) causing cerebral vasoconstriction and
elimination. decreased ICP.
1. Urinary: indwelling catheter (may use external 2. Monitor vital signs and neuro checks
device in male) frequently to detect rises in ICP.
2. Bowel: stool softeners and suppositories as 3. Maintain fluid balance: fluid restriction to
ordered 12001500 mL/day may be ordered.
4. Position client with head of bed elevated to
3045 and neck in neutral position unless
Care of the Client with Increased contraindicated (improves venous drainage
Intracranial Pressure (ICP) from brain).
5. Prevent further increases in ICP.
A. General information
a. Maintain quiet, comfortable environment.
1. An increase in intracranial bulk due to an
b. Avoid use of restraints.
increase in any of the major intracranial
c. Prevent straining at stool; administer
components: brain tissue, CSF, or blood.
stool softeners and mild laxatives as
2. Increased ICP may be caused by tumors,
ordered.
abscesses, hemorrhage, edema, hydrocephalus,
d. Prevent vomiting; administer antiemetics
inflammation.
as ordered.
3. Untreated increased ICP can lead to
e. Prevent excessive coughing.
displacement of brain tissue (herniation).
f. Avoid clustering nursing care activities
4. Presents life-threatening situation because of
together.
pressure on vital structures in the brain stem,
6. Prevent complications of immobility.
nerve tracts, and cranial nerves.

4
7. Administer medications as ordered.

208 NCLEX-RN Review


53155_04_Ch04a_p143-263.qxd 2/26/09 6:50 PM Page 209

a. Hyperosmotic agents (mannitol [Osmitrol]) e. EEG monitoring as necessary.


to reduce cerebral edema; monitor urine f. Provide appropriate nursing care for the
output every hour (should increase). client on a ventilator.
b. Corticosteroids (dexamethasone 10. Observe for hyperthermia secondary to
[Decadron]); anti-inflammatory effect hypothalamus damage.
reduces cerebral edema
c. Diuretics (furosemide [Lasix]) to reduce Care of the Client with Hyperthermia
cerebral edema.
d. Anticonvulsants (phenytoin [Dilantin]) to A. General information
prevent seizures. 1. Abnormal elevation of body temperature to
e. Analgesics for headache as needed 41C (106F) or above
1) Small doses of codeine 2. Caused by dysfunction of hypothalamus
2) Stronger opiates may be (temperature regulating center) from edema,
contraindicated since they potentiate head injury, hemorrhage, CVA, brain tumor, or
respiratory depression, alter LOC, and intracranial surgery
cause pupillary changes. 3. Hyperthermia increases cerebral metabolism;
8. Assist with ICP monitoring when indicated. predisposes to seizures; may cause neurologic
a. ICP monitoring records the pressure damage if prolonged
exerted within the cranial cavity by the B. Nursing care
brain, cerebral blood, and CSF. 1. Remove blankets and excess clothing if
b. Types of monitoring devices temperature rises above 38.4C (101F).
1) Intraventricular catheter: inserted in 2. Maintain room temperature at 21.1C (70F).
lateral ventricle to give direct 3. Administer antipyretic drugs (acetaminophen
measurement of ICP; also allows for [Tylenol]) orally or rectally every 4 hours as
drainage of CSF if needed ordered.
2) Subarachnoid screw (bolt): inserted 4. Increase fluid intake to 3000 mL/day unless
through skull and dura mater into contraindicated (in increased ICP).
subarachnoid space 5. Monitor vital signs, especially temperature,
3) Epidural sensor: least invasive every 24 hours (more often if hypothermia is
method; placed in space between used).
skull and dura mater for indirect 6. Monitor urine output and urine specific
measurement of ICP gravity and assess for signs of dehydration.
c. Monitor ICP pressure readings frequently 7. Observe for seizure activity and protect client
and prevent complications. if seizures occur.
1) Normal ICP reading is 015 mm Hg; a 8. Change linen frequently if client is diaphoretic
sustained increase above 15 mm Hg is (sweating profusely).
considered abnormal. 9. Apply methods for inducing hypothermia as
2) Use strict aseptic technique when ordered: cool or tepid sponge baths, fans,
handling any part of the monitoring hypothermia blanket. (See also Unit 3.)
system. 10. Provide special care for the client with a
3) Check insertion site for signs of hypothermia blanket. (See also Unit 3.)
infection; monitor temperature. a. Reduce temperature gradually to prevent
4) Assess system for CSF leakage, loose shivering and serious dysrhythmias;
connections, air bubbles in lines, and chlorpromazine (Thorazine) may be given
occluded tubing. for shivering.
9. Provide intensive nursing care for client b. Provide frequent skin care to prevent
treated with barbiturate therapy or breakdown.
administration of paralyzing agents. 1) Check every hour for signs of tissue
a. Intravenous administration of barbiturates damage or frostbite.
may be ordered to induce coma artificially 2) Apply lotion to skin to prevent
in the client who has not responded to drying.
conventional treatment. 3) Turn every 2 hours if not
b. Paralytic agents such as vecuronium contraindicated because of increased
bromide (Norcuron) may be administered intracranial pressure.
to paralyze the client. c. Monitor core body temperature.
c. Reduces cellular metabolic demands that
may protect the brain from further injury. Care of the Client with Diminished Eyesight
d. Constant monitoring of the clients ICP,
arterial blood pressures, pulmonary A. Always speak and identify yourself upon entering

4
pressures, arterial blood gases, serum the room to prevent startling the client.
barbiturate levels, and ECG is necessary.

ADULT NURSING 209


53155_04_Ch04a_p143-263.qxd 2/26/09 6:50 PM Page 210

B. Orient the client to his surroundings. EVALUATION


1. Walk the client around the room and have him
touch the objects in the room, e.g., table, chair. A. Client maintains normal weight; no evidence of
2. Keep personal belongings and objects in the malnutrition.
room in the same place in order to increase B. Clients temperature is maintained within normal
clients independence and sense of security. limits.
3. Explain noises or other activities going on in C. Dysreflexia will be prevented or recognized early
the room. and treated promptly.
C. Provide safety measures. D. Client has regular bowel movements.
1. Keep call bell nearby. E. Client has adequate patterns of urinary elimination.
2. Keep at least one side rail up. F. Neuro checks are within normal limits.
3. Keep the room orderly and free of clutter. G. Client maintains patent airway and has effective
D. Assist the client in walking by having him take respiratory patterns.
your arm; walk a half step in front of the client. H. Client remains free from injuries.
E. Offer explanations to the client and tell him what I. Client remains free from aspiration and
to expect next. complications of immobility.
F. Provide mental stimulation and prevent sensory J. Clients skin remains clear and intact.
deprivation by providing frequent contacts with K. Client communicates effectively, responds
the staff, visitors, use of radio, TV, etc. appropriately to others.
L. Client experiences satisfying sexual
Communicating with the Client activity/expression.
with Impaired Hearing M. No contractures or limitations in motor function
have occurred or loss of mobility has been kept to
A. Attract the clients attention by raising an arm or a minimum.
hand. N. Client attains independence in self-care activities;
B. Face the client directly when speaking. uses assistive devices as necessary.
C. Do not obscure the clients view of your mouth in O. Sensory dysfunction is corrected or compensated for.
any way. P. Client is oriented to time, place, and person;
D. Initially state the topic or subject of your memory is intact; able to evaluate reality.
conversation to give the client clues as to what you
are going to say.
E. Speak slowly and distinctly, but do not DISORDERS OF THE
overaccentuate words.
F. Speak in a normal tone of voice; do not shout. NERVOUS SYSTEM
G. Verify that the client has understood you, if necessary.
Headache
Irrigation of the Ear A. General information
A. Introduction of fluid into external auditory canal 1. Diffuse pain in different parts of the head
for cleansing purposes; may be used to apply 2. Types
antiseptic solutions. a. Functional
B. Nursing care 1) Tension (muscle contraction):
1. Explain procedure to the client. associated with tension or anxiety
2. Prepare supplies needed: irrigating solution 2) Migraine: recurrent throbbing
(about 500 mL normal saline at body headache
temperature), irrigating syringe, basin, towel, a) Often starts in adolescence
cotton-tipped applicators, cotton balls. b) Affects women more than men
3. Assist client to a sitting or lying position with c) Vascular origin: vasoconstriction
head tilted toward the affected ear. or spasm of cerebral blood vessels
4. Straighten ear canal by pulling auricle upward (producing an aura) then
and backward (down and backward on a child vasodilation
under 3 years). 3) Cluster: similar to migraine (vascular
5. Insert tip of syringe into auditory meatus and origin); recur several times a day over
direct the solution gently upward toward the a period of weeks followed by
top of the canal. remission lasting for weeks or months
6. Collect returning fluid in basin. b. Organic: secondary to intracranial or systemic
7. Dry the outer ear with cotton balls. disease (e.g., brain tumor, sinus disease)
8. Instruct client to lie on affected side to B. Assessment findings
encourage drainage of solution. 1. Tension headache: pain usually bilateral, often
occurring in the back of the neck and

4
9. Record the procedure and results.
extending diffusely over top of head

210 NCLEX-RN Review


53155_04_Ch04a_p143-263.qxd 2/26/09 6:50 PM Page 211

2. Migraine headache: severe, throbbing pain, 2. Caused by bacteria, viruses, or other


often in temporal or supraorbital area, lasting microorganisms
several hours to days; may be an aura (e.g., 3. May reach CNS
visual disturbance) preceding the pain; nausea a. Via the blood, CSF, lymph
and vomiting; pallor; sweating; irritability b. By direct extension from adjacent cranial
3. Cluster headache: intense, throbbing pain, structures (nasal sinuses, mastoid bone,
usually affecting only one side of face and ear, skull fracture)
head; abrupt onset, lasts 3090 minutes; eye c. By oral or nasopharyngeal route
and nose water on side of pain; skin reddens 4. Most common organisms: meningococcus,
4. Diagnostic tests may be used to rule out pneumococcus, H. influenzae, streptococcus
organic causes. B. Assessment findings
C. Nursing interventions 1. Headache, photophobia, malaise, irritability
1. Carefully assess details regarding the 2. Chills and fever
headache. 3. Signs of meningeal irritation
2. Provide quiet, dark environment. a. Nuchal rigidity: stiff neck
3. Administer medications as ordered. b. Kernigs sign: contraction or pain in the
a. Symptomatic during acute attack hamstring muscle when attempting to
1) Nonnarcotic analgesics (aspirin, extend the leg when the hip is flexed
acetaminophen [Tylenol]) c. Opisthotonos: head and heels bent
2) Fiorinal (analgesic- backward and body arched forward
sedative/tranquilizer combination) d. Brudzinskis sign: flexion at the hip and knee
3) For migraines, ergotamine tartrate in response to forward flexion of the neck
(Gynergen) or ergotamine with caffeine 4. Vomiting
(Cafergot); vasoconstrictors given 5. Possible seizures and decreasing LOC
during aura may prevent the headache 6. Diagnostic test: lumbar puncture
4) Midrin (vasoconstrictor and sedative) (measurement and analysis of CSF shows
5) Sumatriptan (Imitrex) causes increased pressure, elevated WBC and protein,
vasoconstriction in cerebral arteries; decreased glucose and culture positive for
given via cutaneous injection. specific microorganism)
b. Prophylactic to prevent migraine attacks C. Nursing interventions
1) Methysergide maleate (Sansert): after 1. Administer large doses of antibiotics IV as
6 months use, drug should be ordered.
discontinued for a 2-month period 2. Enforce respiratory isolation for 24 hours after
before resuming initiation of antibiotic therapy for some types
2) Propranolol (Inderal) and of meningitis (consult hospitals infection
amytriptyline (Elavil): have also been control manual for specific directions).
used in migraine prevention 3. Provide nursing care for increased ICP,
4. Provide additional nursing interventions for seizures, and hyperthermia if they occur.
pain. 4. Provide nursing care for delirious or
5. Provide client teaching and discharge unconscious client as needed.
planning concerning: 5. Provide bed rest; keep room quiet and dark if
a. Identification of factors including diet that client has headache or photophobia.
appear to precipitate attacks 6. Administer analgesics for headache as
b. Examination of lifestyle, identification of ordered.
stressors, and development of more 7. Maintain fluid and electrolyte balance.
positive coping behaviors 8. Prevent complications of immobility.
c. Importance of daily exercise and relaxation 9. Monitor vital signs and neuro checks
periods frequently.
d. Relaxation techniques 10. Provide client teaching and discharge
e. Use and side effects of prescribed planning concerning:
medications a. Importance of good diet: high protein, high
f. Alternative ways of handling the pain of calorie with small, frequent feedings.
headache: meditation, relaxation, self- b. Rehabilitation program for residual deficits.
hypnosis, yoga
Encephalitis
Meningitis A. General information
A. General information 1. Inflammation of the brain caused by a virus,
1. Inflammation of the meninges of the brain and e.g., herpes simplex (type I) or arbovirus

4
spinal cord (transmitted by mosquito or tick)

ADULT NURSING 211


53155_04_Ch04a_p143-263.qxd 2/26/09 6:50 PM Page 212

2. May occur as a sequela of other diseases such 1. Headache: worse in the morning and with
as measles, mumps, chickenpox. straining and stooping
B. Assessment findings 2. Vomiting
1. Headache 3. Papilledema
2. Fever, chills, vomiting 4. Seizures (focal or generalized)
3. Signs of meningeal irritation 5. Changes in mental status
4. Possibly seizures 6. Focal neurologic deficits (e.g., aphasia,
5. Alterations in LOC hemiparesis, sensory problems)
C. Nursing interventions 7. Diagnostic tests
1. Monitor vital signs and neuro checks a. Skull X-ray, CT scan, MRI, brain scan:
frequently. reveal presence of tumor
2. Provide nursing measures for increased ICP, b. Abnormal EEG
seizures, hyperthermia if they occur. c. Brain biopsy
3. Provide nursing care for confused or D. Nursing interventions
unconscious client as needed. 1. Monitor vital signs and neuro checks; observe
4. Provide client teaching and discharge for signs and symptoms of increased ICP.
planning: same as for meningitis. 2. Administer medications as ordered.
a. Drugs to decrease ICP, e.g.,
Brain Abscess dextromethasone (Decadron)
b. Anticonvulsants, e.g., phenytoin (Dilantin)
A. General information c. Analgesics for headache, e.g.,
1. Collection of free or encapsulated pus within acetaminophen (Tylenol)
the brain tissue 3. Provide supportive care for any neurologic
2. Usually follows an infectious process deficit (see Cerebrovascular Accident).
elsewhere in the body (ear, sinuses, mastoid 4. Prepare client for surgery (see Craniotomy).
bone) 5. Provide care for effects of radiation therapy or
B. Assessment findings chemotherapy (see Oncologic Nursing).
1. Headache, malaise, anorexia 6. Provide psychologic support to
2. Vomiting client/significant others.
3. Signs of increased ICP 7. Provide client teaching and discharge
4. Focal neurologic deficits (hemiparesis, planning concerning
seizures) a. Use and side effects of prescribed
C. Nursing interventions medications.
1. Administer large doses of antibiotics as b. Rehabilitation program for residual deficits.
ordered.
2. Monitor vital signs and neuro checks.
3. Provide symptomatic and supportive care. Cerebrovascular Accident (CVA)
4. Prepare client for surgery if indicated (see A. General information
Craniotomy). 1. Destruction (infarction) of brain cells caused by
a reduction in cerebral blood flow and oxygen
Brain Tumors 2. Affects men more than women; incidence
increases with age
A. General information 3. Caused by thrombosis, embolism, hemorrhage
1. Tumor within the cranial cavity; may be 4. Risk factors
benign or malignant a. Hypertension, diabetes mellitus,
2. Types arteriosclerosis/atherosclerosis, cardiac
a. Primary: originates in brain tissue (e.g., disease (valvular disease/replacement,
glioma, meningioma) chronic atrial fibrillation, myocardial
b. Secondary: metastasizes from tumor infarction)
elsewhere in the body (e.g., lung, breast) b. Lifestyle: obesity, smoking, inactivity,
B. Medical management stress, use of oral contraceptives
1. Craniotomy: to remove the tumor when 5. Pathophysiology
possible a. Interruption of cerebral blood flow for
2. Radiation therapy and chemotherapy: may 5 minutes or more causes death of neurons
follow surgery; also for inaccessible tumors in affected area with irreversible loss of
and metastatic tumors function
3. Drug therapy: hyperosmotic agents, b. Modifying factors
corticosteroids, diuretics to manage increased 1) Cerebral edema: develops around
ICP affected area causing further

4
C. Assessment findings impairment

212 NCLEX-RN Review


53155_04_Ch04a_p143-263.qxd 2/26/09 6:50 PM Page 213

2) Vasospasm: constriction of cerebral b. Administer stool softeners and


blood vessel may occur, causing further suppositories as ordered to prevent
decrease in blood flow constipation and fecal impaction.
3) Collateral circulation: may help to 8. Provide a quiet, restful environment.
maintain cerebral blood flow when there 9. Establish a means of communicating with the
is compromise of main blood supply client.
6. Stages of development 10. Administer medications as ordered.
a. Transient ischemic attack (TIA) a. Hyperosmotic agents, corticosteroids to
1) Warning sign of impending CVA decrease cerebral edema
2) Brief period of neurologic deficit: b. Anticonvulsants to prevent or treat
visual loss, hemiparesis, slurred seizures
speech, aphasia, vertigo c. Thrombolytics given to dissolve clot
3) May last less than 30 seconds, but no (hemorrhage must be ruled out)
more than 24 hours with complete 1) Tissue plasminogen activator (tPA,
resolution of symptoms Alteplase)
b. Stroke in evolution: progressive 2) Streptokinase, urokinase
development of stroke symptoms over a 3) Must be given within 3 hours of episode
period of hours to days d. Anticoagulants for stroke in evolution or
c. Completed stroke: neurologic deficit embolic stroke (hemorrhage must be ruled
remains unchanged for a 2- to 3-day out)
period. 1) Heparin
B. Assessment findings 2) Warfarin (Coumadin) for long-term
1. Headache therapy
2. Generalized signs: vomiting, seizures, 3) Aspirin and dipyridamole (Persantine)
confusion, disorientation, decreased LOC, to inhibit platelet aggregation in
nuchal rigidity, fever, hypertension, slow treating TIAs
bounding pulse, Cheyne-Stokes respirations e. Antihypertensives if indicated for elevated
3. Focal signs (related to site of infarction): blood pressure
hemiplegia, sensory loss, aphasia, D. Nursing interventions: rehabilitation
homonymous hemianopsia 1. Hemiplegia: results from injury to cells in the
4. Diagnostic tests cerebral motor cortex or to corticospinal tracts
a. CT and brain scan: reveal lesion (causes contralateral hemiplegia because tracts
b. EEG: abnormal changes cross in medulla)
c. Cerebral arteriography: may show a. Turn every 2 hours (20 minutes only on
occlusion or malformation of blood vessels affected side).
C. Nursing interventions: acute stage b. Use proper positioning and repositioning
1. Maintain patent airway and adequate to prevent deformities (foot drop, external
ventilation. rotation of hip, flexion of fingers, wrist
2. Monitor vital signs and neuro checks and drop, abduction of shoulder and arm).
observe for signs of increased ICP, shock, c. Support paralyzed arm on pillow or use
hyperthermia, and seizures. sling while out of bed to prevent
3. Provide complete bed rest as ordered. subluxation of shoulder.
4. Maintain fluid and electrolyte balance and d. Elevate extremities to prevent dependent
ensure adequate nutrition. edema.
a. IV therapy for the first few days e. Provide active and passive ROM exercises
b. Nasogastric tube feedings if client unable every 4 hours.
to swallow 2. Susceptibility to hazards
c. Fluid restriction as ordered to decrease a. Keep side rails up at all times.
cerebral edema b. Institute safety measures.
5. Maintain proper positioning and body alignment. c. Inspect body parts frequently for signs of
a. Head of bed may be elevated 3045 to injury.
decrease ICP 3. Dysphagia (difficulty swallowing)
b. Turn and reposition every 2 hours (only a. Check gag reflex before feeding client.
20 minutes on the affected side) b. Maintain a calm, unhurried approach.
c. Passive ROM exercises every 4 hours. c. Place client in upright position.
6. Promote optimum skin integrity: turn client d. Place food in unaffected side of mouth.
and apply lotion every 2 hours. e. Offer soft foods.
7. Maintain adequate elimination. f. Give mouth care before and after meals.
a. Offer bedpan or urinal every 2 hours, 4. Homonymous hemianopsia: loss of right or left

4
catheterize only if absolutely necessary. half of each visual field

ADULT NURSING 213


53155_04_Ch04a_p143-263.qxd 2/26/09 6:50 PM Page 214

a. Approach client on unaffected side. Cerebral Aneurysm


b. Place personal belongings, food, etc., on
unaffected side. A. General information
c. Gradually teach client to compensate by 1. Dilation of the walls of a cerebral artery,
scanning, i.e., turning the head to see resulting in a sac-like outpouching of vessel
things on affected side. 2. Caused by congenital weakness in the vessel,
5. Emotional lability: mood swings, frustration trauma, arteriosclerosis, hypertension
a. Create a quiet, restful environment with a 3. Pathophysiology
reduction in excessive sensory stimuli. a. Aneurysm compresses nearby cranial nerves
b. Maintain a calm, nonthreatening manner. or brain substance, producing dysfunction.
c. Explain to family that the clients behavior b. Aneurysm may rupture, causing
is not purposeful. subarachnoid hemorrhage or intracerebral
6. Aphasia: most common in right hemiplegics; hemorrhage.
may be receptive/expressive c. Initially a clot forms at the site of rupture,
a. Receptive aphasia but fibrinolysis (dissolution of the clot)
1) Give simple, slow directions. tends to occur within 710 days and may
2) Give one command at a time; gradually cause rebleeding.
shift topics. B. Assessment findings
3) Use nonverbal techniques of 1. Severe headache and pain in the eyes
communication (e.g., pantomime, 2. Diplopia, tinnitus, dizziness
demonstration). 3. Nuchal rigidity, ptosis, decreasing LOC,
b. Expressive aphasia hemiparesis, seizures
1) Listen and watch very carefully when C. Nursing interventions
the client attempts to speak. 1. Maintain a patent airway and adequate
2) Anticipate clients needs to decrease ventilation.
frustration and feelings of a. Instruct client to take deep breaths but to
helplessness. avoid coughing.
3) Allow sufficient time for client to b. Suction only with a specific order.
answer. 2. Monitor vital signs and neuro checks and
7. Sensory/perceptual deficits: more common in observe for signs of vasospasm, increased ICP,
left hemiplegics; characterized by hypertension, seizures, and hyperthermia.
impulsiveness, unawareness of disabilities, 3. Enforce strict bed rest and provide complete care.
visual neglect (neglect of affected side and 4. Keep head of bed flat or elevated to 2030 as
visual space on affected side) ordered.
a. Assist with self-care. 5. Maintain a quiet, darkened environment.
b. Provide safety measures. 6. Avoid taking temperature rectally and instruct
c. Initially arrange objects in environment on client to avoid sneezing, coughing, and
unaffected side. straining at stool.
d. Gradually teach client to take care of the 7. Enforce fluid restriction as ordered; maintain
affected side and to turn frequently and accurate I&O.
look at affected side. 8. Administer medications as ordered.
8. Apraxia: loss of ability to perform purposeful, a. Antihypertensive agents to maintain
skilled acts normotensive levels
a. Guide client through intended movement b. Corticosteroids to prevent increased ICP
(e.g., take object such as washcloth and c. Anticonvulsants to prevent seizures
guide client through movement of d. Stool softeners to prevent straining
washing). e. Aminocaproic acid (Amicar) to decrease
b. Keep repeating the movement. fibrinolysis of the clot (administered IV)
9. Generalizations about clients with left 9. Prevent complications of immobility.
hemiplegia versus right hemiplegia and 10. Institute seizure precautions.
nursing care 11. Provide nursing care for the unconscious
a. Left hemiplegia client if needed.
1) Perceptual, sensory deficits; quick and 12. Prepare the client for surgery if indicated (see
impulsive behavior Craniotomy).
2) Use safety measures, verbal cues,
simplicity in all areas of care Parkinsons Disease
b. Right hemiplegia
1) Speech-language deficits; slow and A. General information
cautious behavior 1. A progressive disorder with degeneration of

4
2) Use pantomime and demonstration the nerve cells in the basal ganglia resulting in

214 NCLEX-RN Review


53155_04_Ch04a_p143-263.qxd 2/26/09 6:50 PM Page 215

generalized decline in muscular function; C. Nursing interventions


disorder of the extrapyramidal system 1. Administer medications as ordered
2. Usually occurs in the older population a. Levodopa (L-dopa)
3. Cause unknown; predominantly idiopathic, 1) Increases level of dopamine in the
but sometimes disorder is postencephalitic, brain; relieves tremor, rigidity, and
toxic, arteriosclerotic, traumatic, or drug bradykinesia.
induced (reserpine, methyldopa [Aldomet], 2) Side effects: anorexia; nausea and
haloperidol [Haldol], phenothiazines) vomiting; postural hypotension;
4. Pathophysiology mental changes such as confusion,
a. Disorder causes degeneration of the agitation, and hallucinations; cardiac
dopamine-producing neurons in the arrhythmias; dyskinesias.
substantia nigra in the midbrain. 3) Contraindications: narrow-angle
b. Dopamine influences purposeful movement. glaucoma; clients taking MAO
c. Depletion of dopamine results in inhibitors, reserpine, guanethidine,
degeneration of the basal ganglia. methyldopa, antipsychotics; acute
B. Assessment findings psychoses.
1. Tremor: mainly of the upper limbs, pill-rolling, 4) Avoid multiple vitamin preparations
resting tremor; most common initial symptom containing vitamin B6 (pyridoxine)
2. Rigidity: cogwheel type and foods high in vitamin B6.
3. Bradykinesia: slowness of movement 5) Be aware of any worsening of
4. Fatigue symptoms with prolonged high-dose
5. Stooped posture; shuffling, propulsive gait therapy: on-off syndrome.
(see Figure 4-4) 6) Administer with food or snack to
6. Difficulty rising from sitting position decrease GI irritation.
7. Masklike face with decreased blinking of eyes 7) Inform client that urine and sweat may
8. Quiet, monotone speech be darkened.
9. Emotional lability, depression b. Carbidopa-levodopa (Sinemet): prevents
10. Increased salivation, drooling breakdown of dopamine in the periphery
11. Cramped, small handwriting and causes fewer side effects.
12. Autonomic symptoms: excessive sweating, c. Amantadine (Symmetrel): used in mild
seborrhea, lacrimation, constipation; cases or in combination with L-dopa to
decreased sexual capacity reduce rigidity, tremor, and bradykinesia.

Figure 4-4 The shuffling gait and early postural changes of Parkinsons disease
shown in (A). (B) and (C) show an advanced stage of the disease with head held
forward, mouth open, and inability to stand.

ADULT NURSING

4 215
53155_04_Ch04a_p143-263.qxd 2/26/09 6:50 PM Page 216

d. Anticholinergic drugs: benztropine c. Importance of daily exercise: walking,


mesylate (Cogentin), procyclidine swimming, gardening as tolerated;
(Kemadrin), trihexyphenidyl (Artane) balanced activity and rest
1) Inhibit action of acetylcholine d. Activities/methods to limit postural
2) Used in mild cases or in combination deformities: firm mattress with a small
with L-dopa pillow; keep head and neck as erect as
3) Relieve tremor and rigidity possible; use broad-based gait; raise feet
4) Side effects: dry mouth, blurred vision, while walking
constipation, urinary retention, e. Promotion of active participation in
confusion, hallucinations, tachycardia self-care activities
e. Antihistamines: diphenhydramine
(Benadryl)
1) Decrease tremor and anxiety
Multiple Sclerosis (MS)
2) Side effect: drowsiness A. General information
f. Bromocriptine (Parlodel) 1. Chronic, intermittently progressive disease of
1) Stimulates release of dopamine in the the CNS, characterized by scattered patches of
substantia nigra. demyelination within the brain and spinal cord
2) Often employed when L-dopa loses 2. Incidence
effectiveness. a. Affects women more than men
g. Eldepryl (Selegilene) a MAO inhibitor b. Usually occurs from 2040 years of age
inhibits dopamine breakdown and slows c. More frequent in cool or temperate
progression of disease climates
h. Tricyclic antidepressants given to treat 3. Cause unknown; may be a slow-growing virus
depression commonly seen in Parkinsons or possibly of autoimmune origin
disease 4. Signs and symptoms are varied and multiple,
2. Provide a safe environment. reflecting the location of demyelination within
a. Side rails on bed; rails and handlebars in the CNS
toilet, bathtub, and hallways; no scatter 5. Characterized by remissions and exacerbations
rugs B. Assessment findings
b. Hard-back or spring-loaded chair to make 1. Visual disturbances: blurred vision, scotomas
getting up easier (blind spots), diplopia
3. Provide measures to increase mobility. 2. Impaired sensation: touch, pain, temperature,
a. Physical therapy: active and passive ROM or position sense; paresthesias such as
exercises; stretching exercises; warm baths numbness, tingling
b. Assistive devices 3. Euphoria or mood swings
c. If client freezes, suggest thinking of 4. Impaired motor function: weakness, paralysis,
something to walk over. spasticity
4. Encourage independence in self-care activities: 5. Impaired cerebellar function: scanning speech,
alter clothing for ease in dressing; use assistive ataxic gait, nystagmus, dysarthria, intention
devices; do not rush client. tremor
5. Improve communication abilities: instruct 6. Bladder: retention or incontinence
client to practice reading aloud, to listen to 7. Constipation
own voice, and enunciate each syllable clearly. 8. Sexual impotence in the male
6. Refer for speech therapy when indicated. 9. Diagnostic tests:
7. Maintain adequate nutrition. a. CSF studies: increased protein and IgG
a. Cut food into bite-sized pieces. (immunoglobulin)
b. Provide small, frequent feedings. b. Visual evoked response (VER) determined
c. Allow sufficient time for meals, use by EEG: may be delayed
warming tray. c. CT scan: increased density of white matter
8. Avoid constipation and maintain adequate d. MRI: shows areas of demyelination
bowel elimination. C. Nursing interventions
9. Provide psychologic support to 1. Assess the client for specific deficits related to
client/significant others; depression is location of demyelinization.
common due to changes in body image and 2. Promote optimum mobility.
self-concept. a. Muscle-stretching and strengthening
10. Provide client teaching and discharge exercises
planning concerning b. Walking exercises to improve gait: use
a. Nature of the disease wide-based gait
b. Use of prescribed medications and side c. Assistive devices: canes, walker, rails,

4
effects wheelchair as necessary

216 NCLEX-RN Review


53155_04_Ch04a_p143-263.qxd 2/26/09 6:50 PM Page 217

3. Administer medications as ordered. Myasthenia Gravis


a. For acute exacerbations: corticosteroids
(ACTH [IV], prednisone) to reduce edema A. General information
at sites of demyelinization 1. A neuromuscular disorder in which there is
b. For spasticity: baclofen (Lioresal), a disturbance in the transmission of impulses
dantrolene (Dantrium), diazepam from nerve to muscle cells at the
(Valium) neuromuscular junction, causing extreme
c. Beta interferon (Betaseron) to alter immune muscle weakness
response 2. Incidence
4. Encourage independence in self-care a. Highest between ages 15 and 35 for
activities. women, over 40 for men.
5. Prevent complications of immobility. b. Affects women more than men
6. Institute bowel program. 3. Cause: thought to be autoimmune disorder
7. Maintain urinary elimination. whereby antibodies destroy acetylcholine
a. Urinary retention receptor sites on the postsynaptic membrane
1) Administer bethanecol chloride of the neuromuscular junction.
(Urecholine) as ordered. 4. Voluntary muscles are affected, especially
2) Perform intermittent catheterization as those muscles innervated by the cranial
ordered. nerves.
b. Urinary incontinence B. Medical management
1) Establish voiding schedule. 1. Drug therapy
2) Administer propantheline bromide a. Anticholinesterase drugs: ambenonium
(Pro-Banthine) if ordered. (Mytelase), neostigmine (Prostigmin),
c. Force fluids to 3000 mL/day. pyridostigmine (Mestinon)
d. Promote use of acid-ash foods like 1) Block action of cholinesterase and
cranberry or grape juice. increase levels of acetylcholine at the
8. Prevent injury related to sensory problems. neuromuscular junction
a. Test bath water with thermometer. 2) Side effects: excessive salivation and
b. Avoid heating pads, hot-water bottles. sweating, abdominal cramps, nausea
c. Inspect body parts frequently for injury. and vomiting, diarrhea, fasciculations
d. Make frequent position changes. (muscle twitching)
9. Prepare client for plasma exchange (to remove b. Corticosteroids: prednisone
antibodies) if indicated. 1) Used if other drugs are not effective
10. Provide psychologic support to 2) Suppress autoimmune response
client/significant others. 2. Surgery (thymectomy)
a. Encourage positive attitude and assist a. Surgical removal of the thymus gland
client in setting realistic goals. (thought to be involved in the production
b. Provide compassion in helping client of acetylcholine receptor antibodies)
adapt to changes in body image and b. May cause remission in some clients
self-concept. especially if performed early in the
c. Do not encourage false hopes during disease
remission. 3. Plasma exchange
d. Refer to multiple sclerosis societies and a. Removes circulating acetylcholine receptor
community agencies. antibodies
11. Provide client teaching and discharge b. Use in clients who do not respond to other
planning concerning types of therapy
a. General measures to ensure optimum C. Assessment findings
health 1. Diplopia, dysphagia
1) Balance between activity and rest 2. Extreme muscle weakness, increased with
2) Regular exercise such as walking, activity and reduced with rest
swimming, biking in mild cases 3. Ptosis, masklike facial expression
3) Use of energy conservation techniques 4. Weak voice, hoarseness
4) Well-balanced diet 5. Diagnostic tests
5) Fresh air and sunshine a. Tensilon test: IV injection of Tensilon
6) Avoiding fatigue, overheating or provides spontaneous relief of symptoms
chilling, stress, infection (lasts 510 minutes)
b. Use of medications and side effects b. Electromyography (EMG): amplitude of
c. Alternative methods for sexual evoked potentials decreases rapidly
gratification; refer for sexual counseling c. Presence of antiacetylcholine receptor

4
if indicated. antibodies in the serum

ADULT NURSING 217


53155_04_Ch04a_p143-263.qxd 2/26/09 6:50 PM Page 218

D. Nursing interventions 2) Monitor arterial blood gases and vital


1. Administer anticholinesterase drugs as ordered. capacities.
a. Give medication exactly on time. 3) Administer medications as ordered.
b. Give with milk and crackers to decrease a) Myasthenic crisis: increase doses
GI upset. of anticholinesterase drugs as
c. Monitor effectiveness of drugs: assess ordered.
muscle strength and vital capacity before b) Cholinergic crisis: discontinue
and after medication. anticholinesterase drugs as ordered
d. Avoid use of the following drugs: until the client recovers.
morphine and strong sedatives (respiratory 4) Establish a method of communication.
depressant effect), quinine, curare, 5) Provide support and reassurance.
procainamide, neomycin, streptomycin, 6. Provide nursing care for the client with a
kanamycin and other aminoglycosides thymectomy.
(skeletal muscle blocking effects). 7. Provide client teaching and discharge
e. Observe for side effects. planning concerning
2. Promote optimal nutrition. a. Nature of the disease
a. Mealtimes should coincide with the peak b. Use of prescribed medications, their side
effects of the drugs: give medications effects and signs of toxicity
30 minutes before meals. c. Importance of checking with physician
b. Check gag reflex and swallowing ability before taking any new medications
before feeding. including OTC drugs
c. Provide a mechanical soft diet. d. Importance of planning activities to take
d. If the client has difficulty chewing and advantage of energy peaks and of
swallowing, do not leave alone at scheduling frequent rest periods
mealtimes; keep emergency airway and e. Need to avoid fatigue, stress, people with
suction equipment nearby. upper-respiratory infections
3. Monitor respiratory status frequently: rate, f. Use of eye patch for diplopia (alternate
depth; vital capacity; ability to deep breathe eyes)
and cough. g. Need to wear Medic-Alert bracelet
4. Assess muscle strength frequently; plan h. Myasthenia Gravis Foundation and other
activity to take advantage of energy peaks and community agencies
provide frequent rest periods.
5. Observe for signs of myasthenic or cholinergic
crisis.
Epilepsy
a. Myasthenic crisis See Unit 5.
1) Abrupt onset of severe, generalized
muscle weakness with inability to
swallow, speak, or maintain
Head Injury
respirations A. General information
2) Caused by undermedication, physical 1. Usually caused by car accidents, falls, assaults
or emotional stress, infection 2. Types
3) Symptoms will improve temporarily a. Concussion: severe blow to the head jostles
with Tensilon test brain, causing it to strike the skull; results
b. Cholinergic crisis in temporary neural dysfunction
1) Symptoms similar to myasthenic crisis b. Contusion: results from more severe blow
and, in addition, the side effects of that bruises the brain and disrupts neural
anticholinesterase drugs (e.g., function
excessive salivation and sweating, c. Hemorrhage
abdominal cramps, nausea and 1) Epidural hematoma: accumulation of
vomiting, diarrhea, fasciculations) blood between the dura mater and
2) Caused by overmedication with the skull; commonly results from
cholinergic (anticholinesterase) drugs laceration of middle meningeal artery
3) Symptoms worsen with Tensilon test; during skull fracture; blood
keep atropine sulfate and emergency accumulates rapidly
equipment on hand 2) Subdural hematoma: accumulation of
c. Nursing care in crisis blood between the dura and arachnoid;
1) Maintain tracheostomy or venous bleeding that forms slowly;
endotracheal tube with mechanical may be acute, subacute, or chronic
ventilation as indicated (see 3) Subarachnoid hematoma: bleeding in

4
Mechanical Ventilation). subarachnoid space

218 NCLEX-RN Review


53155_04_Ch04a_p143-263.qxd 2/26/09 6:50 PM Page 219

4) Intracerebral hematoma: accumulation 5. Prevent complications of immobility.


of blood within the cerebrum 6. Prepare the client for surgery if indicated.
d. Fractures: linear, depressed, comminuted, a. Depressed skull fracture: surgical removal
compound or elevation of splintered bone;
B. Assessment findings (depend on type of injury) debridement and cleansing of area; repair
1. Concussion: headache, transient loss of of dural tear if present; cranioplasty (if
consciousness, retrograde or posttraumatic necessitated for large cranial defect)
amnesia, nausea, dizziness, irritability b. Epidural or subdural hematoma:
2. Contusion: neurologic deficits depend on the evacuation of the hematoma
site and extent of damage; include decreased 7. Provide psychologic support to
LOC, aphasia, hemiplegia, sensory deficits client/significant others.
3. Hemorrhages 8. Observe for hemiplegia, aphasia, and sensory
a. Epidural hematoma: brief loss of problems, and plan care accordingly (see
consciousness followed by lucid interval; Cerebrovascular Accident).
progresses to severe headache, vomiting, 9. Provide client teaching and discharge
rapidly deteriorating LOC, possible planning concerning rehabilitation for
seizures, ipsilateral pupillary dilation neurologic deficits; note availability of
b. Subdural hematoma: alterations in LOC, community agencies.
headache, focal neurologic deficits,
personality changes, ipsilateral pupillary
dilation
Intracranial Surgery
c. Intracerebral hematoma: headache, A. Types
decreased LOC, hemiplegia, ipsilateral 1. Craniotomy: surgical opening of skull to gain
pupillary dilation access to intracranial structures; used to
4. Fractures remove a tumor, evacuate blood clot, control
a. Headache, pain over fracture site hemorrhage, relieve increased ICP
b. Compound fractures: rhinorrhea (leakage 2. Craniectomy: excision of a portion of the skull;
of CSF from nose); otorrhea (leakage of CSF sometimes used for decompression
from ear) 3. Cranioplasty: repair of a cranial defect with a
5. Diagnostic tests metal or plastic plate
a. Skull X-ray: reveals skull fracture or B. Nursing interventions: preoperative
intracranial shift 1. Routine pre-op care (see Perioperative Nursing).
b. CT scan: reveals hemorrhage 2. Provide emotional support; explain post-op
C. Nursing interventions (see also Care of the procedures and that clients head will be
Unconscious Client and Care of the Client with shaved, there will be a large bandage on head,
Increased ICP) possibly temporary swelling and discoloration
1. Maintain a patent airway and adequate around the eye on the affected side, and
ventilation. possible headache.
2. Monitor vital signs and neuro checks; observe 3. Shampoo the scalp and check for signs of
for changes in neurologic status, signs of infection.
increased ICP, shock, seizures, and 4. Shave hair.
hyperthermia. 5. Evaluate and record baseline vital signs and
3. Observe for CSF leakage. neuro checks.
a. Check discharge for positive Testape or 6. Avoid enemas unless directed (straining
Dextrostix reaction for glucose; bloody spot increases ICP).
encircled by watery, pale ring halo on 7. Give pre-op steroids as ordered to decrease
pillowcase or sheet. brain swelling.
b. Never attempt to clean the ears or nose of a 8. Insert Foley catheter as ordered.
head-injured client or use nasal suction C. Nursing interventions: postoperative
unless cleared by physician. 1. Provide nursing care for the unconscious client.
4. If a CSF leak is present: 2. Maintain a patent airway and adequate
a. Instruct client not to blow nose. ventilation.
b. Elevate head of bed 30 as ordered. a. Supratentorial incision: elevate head of
c. Observe for signs of meningitis and bed 1545 as ordered; position on back
administer antibiotics to prevent (if intubated or conscious) or on unaffected
meningitis as ordered. side; turn every 2 hours to facilitate
d. Place a cotton ball in the ear to absorb breathing and venous return.
otorrhea; replace frequently. b. Infratentorial incision: keep head of bed
e. Gently place a sterile gauze pad at the bottom flat or elevate 2030 as ordered; do not

4
of the nose for rhinorrhea; replace frequently. flex head on chest; turn side to side every

ADULT NURSING 219


53155_04_Ch04a_p143-263.qxd 2/26/09 6:50 PM Page 220

2 hours using a turning sheet; check 3) May be to the spinal cord and its roots:
respirations closely and report any signs of concussion, contusion, compression or
respiratory distress. laceration by fracture/dislocation or
c. Instruct the conscious client to breathe penetrating missiles
deeply but not to cough; avoid vigorous b. Level of injury: cervical, thoracic, lumbar
suctioning. c. Mechanisms of injury
3. Check vital signs and neuro checks frequently; 1) Hyperflexion
observe for decreasing LOC, increased ICP, 2) Hyperextension
seizures, hyperthermia. 3) Axial loading (force exerted straight up
4. Monitor fluid and electrolyte status. or down spinal column as in a diving
a. Maintain accurate I&O. accident)
b. Restrict fluids to 1500 mL/day or as 4) Penetrating wounds
ordered to decrease cerebral edema. 5. Pathophysiology: hemorrhage and edema
c. Avoid overly rapid infusions. cause ischemia, leading to necrosis and
d. Watch for signs of diabetes insipidus destruction of the cord
(severe thirst, polyuria, dehydration) and B. Medical management: immobilization and
inappropriate ADH secretion (decreased maintenance of normal spinal alignment to
urine output, hunger, thirst, irritability, promote fracture healing
decreased LOC, muscle weakness). 1. Horizontal turning frames (Stryker frame)
e. For infratentorial surgery: may be NPO for 2. Skeletal traction: to immobilize the fracture
24 hours due to possible impaired and maintain alignment of the cervical spine
swallowing and gag reflexes. a. Cervical tongs (Crutchfield, Gardner-
5. Assess dressings frequently and report any Wells, Vinke): inserted through burr holes;
abnormalities. traction is provided by a rope extended
a. Reinforce as needed with sterile dressings. from the center of tongs over a pulley with
b. Check dressings for excessive drainage, weights attached at the end.
CSF, infection, displacement and report to b. Halo traction
physician. 1) Stainless steel halo ring fits around the
c. If surgical drain is in place, note color, head and is attached to the skull with
amount, and odor of drainage. four pins; halo is attached to plastic
6. Administer medications as ordered. body cast or plastic vest
a. Corticosteroids: to decrease cerebral edema 2) Permits early mobilization, decreased
b. Anticonvulsants: to prevent seizures period of hospitalization and reduces
c. Stool softeners: to prevent straining complications of immobility
d. Mild analgesics 3. Surgery: decompression laminectomy, spinal
7. Apply ice to swollen eyelids; lubricate lids fusion
and areas around eyes with petrolatum jelly. a. Depends on type of injury and the
8. Refer client for rehabilitation for residual preference of the surgeon
deficits. b. Indications: unstable fracture, cord
compression, progression of neurologic
deficits
Spinal Cord Injuries C. Assessment findings
A. General information 1. Spinal shock
1. Occurs most commonly in young adult males a. Occurs immediately after the injury as a
between ages 15 and 25 result of the insult to the CNS
2. Common traumatic causes: motor vehicle b. Temporary condition lasting from several
accidents, diving in shallow water, falls, days to 3 months
industrial accidents, sports injuries, gunshot c. Characterized by absence of reflexes below
or stab wounds the level of the lesion, flaccid paralysis,
3. Nontraumatic causes: tumors, hematomas, lack of temperature control in affected
aneurysms, congenital defects (spina bifida) parts, hypotension with bradycardia,
4. Classified by extent, level, and mechanism of retention of urine and feces
injury 2. Symptoms depend on the level and the extent
a. Extent of injury of the injury.
1) May affect the vertebral column: a. Level of injury
fracture, fracture/dislocation 1) Quadriplegia: cervical injuries (C1C8)
2) May affect anterior or posterior cause paralysis of all four extremities;
ligaments, causing compression of respiratory paralysis occurs in lesions
spinal cord above C6 due to lack of innervation to

4 220 NCLEX-RN Review


53155_04_Ch04a_p143-263.qxd 2/26/09 6:50 PM Page 221

the diaphragm; (phrenic nerves at the c. Change position slowly and gradually
C4C5 level). elevate the head of the bed to prevent
2) Paraplegia: thoraco/lumbar injuries postural hypotension.
(T1L4) cause paralysis of the lower d. Observe for signs of deep-vein thrombosis.
half of the body involving both legs 3. Maintain fluid and electrolyte balance and
b. Extent of injury nutrition.
1) Complete cord transection a. Nasogastric tube may be inserted until
a) Loss of all voluntary movement bowel sounds return.
and sensation below the level of b. Maintain IV therapy as ordered; avoid
the injury; reflex activity below the overhydration (can aggravate cord edema).
level of the lesion may return after c. Check bowel sounds before feeding client
spinal shock resolves. (paralytic ileus is common).
b) Lesions in the conus medullaris or d. Progress slowly from clear liquid to regular
cauda equina result in permanent diet.
flaccid paralysis and areflexia. e. Provide diet high in protein,
2) Incomplete lesions: varying degrees of carbohydrates, calories.
motor or sensory loss below the level 4. Maintain immobilization and spinal alignment
of the lesion depending on which always.
neurologic tracts are damaged and a. Turn every hour on turning frame.
which are spared. b. Maintain cervical traction at all times if
3. Diagnostic test: spinal X-rays may reveal fracture. indicated.
D. Nursing interventions: emergency care 5. Prevent complications of immobility; use
1. Assess airway, breathing, circulation footboard/high-topped sneakers to prevent
a. Do not move the client during assessment. footdrop; provide splint for quadriplegic client
b. If airway obstruction or inadequate to prevent wrist drop.
ventilation exists: do not hyperextend neck 6. Maintain urinary elimination.
to open airway, use jaw thrust instead. a. Provide intermittent catheterization or
2. Perform a quick head-to-toe assessment: check maintain indwelling catheter as ordered.
for LOC, signs of trauma to the head or neck, b. Increase fluids to 3000 mL/day.
leakage of clear fluid from ears or nose, signs c. Provide acid-ash foods/fluids to acidify
of motor or sensory impairment. urine and prevent infection.
3. Immobilize the client in the position found 7. Maintain bowel elimination: administer stool
until help arrives. softeners and suppositories to prevent
4. Once emergency help arrives, assist in impaction as ordered.
immobilizing the head and neck with a 8. Monitor temperature control.
cervical collar and place the client on a spinal a. Check temperature every 4 hours.
board; avoid any movement during transfer, b. Regulate environment closely.
especially flexion of the spinal column. c. Avoid excessive covering or exposure.
5. Have suction available to clear the airway and 9. Observe for and prevent infection.
prevent aspiration if the client vomits; client a. Observe tongs or pin site for redness,
may be turned slightly to the side if secured to drainage.
a board. b. Provide tong- or pin-site care. Cleanse with
6. Evaluate respiration and observe for weak or antiseptic solution according to agency
labored respirations. policy.
E. Nursing interventions: acute care c. Observe for signs of respiratory or urinary
1. Maintain optimum respiratory function. infection.
a. Observe for weak or labored respirations; 10. Observe for and prevent stress ulcers.
monitor arterial blood gases. a. Assess for epigastic or shoulder pain.
b. Prevent pneumonia and atelectasis: turn b. If corticosteroids are ordered, give with
every 2 hours; cough and deep breathe food or antacids; administer H2 blocker as
every hour; use incentive spirometry every ordered.
2 hours. c. Check nasogastric tube contents and stools
c. Tracheostomy and mechanical ventilation for blood.
may be necessary if respiratory F. Nursing interventions: chronic care
insufficiency occurs. 1. Neurogenic bladder
2. Maintain optimal cardiovascular function. a. Reflex or upper motor neuron bladder;
a. Monitor vital signs; observe for reflex activity of the bladder may occur
bradycardia, arrhythmias, hypotension. after spinal shock resolves; the bladder is
b. Apply thigh-high elastic stockings or Ace unable to store urine very long and empties

4
bandages. involuntarily

ADULT NURSING 221


53155_04_Ch04a_p143-263.qxd 2/26/09 6:50 PM Page 222

b. Nonreflexive or lower motor neuron b. Assist client to adjust to effects of


bladder: reflex arc is disrupted and no injury.
reflex activity of the bladder occurs, c. Encourage independence.
resulting in urine retention with overflow d. Involve the client in decision making.
c. Management of reflex bladder 2. Provide sexual counseling.
1) Intermittent catheterization every a. Work with the client and partner.
4 hours and gradually progress to b. Explore alternative methods of sexual
every 6 hours. gratification.
2) Regulate fluid intake to 18002000 3. Initiate rehabilitation program.
mL/day. a. Physical therapy
3) Bladder taps or stimulating trigger b. Vocational rehabilitation
points to cause reflex emptying of the c. Psychologic counseling
bladder. d. Use of braces, electronic wheelchair, and
d. Management of nonreflexive bladder other assistance devices to maximize
1) Intermittent catheterization every independence
6 hours.
2) Cred maneuver or rectal stretch.
3) Regulate intake to 18002000 mL/day
Specific Disorders of the Peripheral
to prevent overdistention of bladder. Nervous System
e. Management depends on lifestyle, age, sex,
home care, and availability of caregiver.
Trigeminal Neuralgia (Tic Douloureux)
2. Spasticity A. General information
a. Return of reflex activity may occur after 1. Disorder of cranial nerve V causing disabling
spinal shock resolves; severe spasticity and recurring attacks of severe pain along the
may be detrimental sensory distribution of one or more branches
b. Drug therapy: baclofen (Lioresal), of the trigeminal nerve
dantrolene (Dantrium), diazepam (Valium) 2. Incidence increased in elderly women
c. Physical therapy: stretching exercises, 3. Cause unknown
warm tub baths, whirlpool B. Medical management
d. Surgery: chordotomy 1. Anticonvulsant drugs: carbamazepine
3. Autonomic dysreflexia (Tegretol), phenytoin (Dilantin)
a. Rise in blood pressure, sometimes to fatal 2. Nerve block: injection of alcohol or phenol
levels into one or more branches of the trigeminal
b. Occurs in clients with cord lesions above nerve; temporary effect, lasts 618 months
T6 and most commonly in clients with 3. Surgery
cervical injuries a. Peripheral: avulsion of peripheral branches
c. Reflex response to stimulation of the of trigeminal nerve
sympathetic nervous system b. Intracranial
d. Stimulus may be overdistended bladder or 1) Retrogasserian rhizotomy: total
bowel, decubitus ulcer, chilling, pressure severance of the sensory root of the
from bedclothes trigeminal nerve intracranially; results
e. Symptoms: severe headache, hypertension, in permanent anesthesia, numbness,
bradycardia, sweating, goose bumps, nasal heaviness, and stiffness in affected
congestion, blurred vision, convulsions part; loss of corneal reflex
f. Interventions 2) Microsurgery: uses more precise
1) Raise client to sitting position to cutting and may preserve facial
decrease BP. sensation and corneal reflex
2) Check for source of stimulus (bladder, 3) Percutaneous radio-frequency
bowel, skin). trigeminal gangliolysis: current
3) Remove offending stimulus (e.g., surgical procedure of choice; thermally
catheterize client, digitally remove destroys the trigeminal nerve in the
impacted feces, reposition client). area of the ganglion; provides
4) Monitor blood pressure. permanent pain relief with
5) Administer antihypertensives (e.g., preservation of sense of touch,
hydralazine HCl [Apresoline]) as proprioception, and corneal reflex;
ordered. done under local anesthesia
G. Nursing interventions: general rehabilitative care 4) Microvascular decompression of
1. Provide psychologic support to trigeminal nerve: decompresses the
client/significant others. trigeminal nerve; craniotomy

4
a. Support during grieving process.

222 NCLEX-RN Review


53155_04_Ch04a_p143-263.qxd 2/26/09 6:50 PM Page 223

necessary; provides permanent pain B. Assessment findings


relief while preserving facial sensation 1. Loss of taste over anterior two-thirds of tongue
C. Assessment findings on affected side
1. Sudden paroxysms of extremely severe 2. Complete paralysis of one side of face
shooting pain in one side of the face 3. Loss of expression, displacement of mouth
2. Attacks may be triggered by a cold breeze, toward unaffected side, and inability to close
foods/fluids of extreme temperature, eyelid (all on affected side)
toothbrushing, chewing, talking, or touching 4. Pain behind the ear
the face C. Nursing interventions
3. During attack: twitching, grimacing, and 1. Assess facial nerve function regularly (see
frequent blinking/tearing of the eye Table 4-16).
4. Poor eating and hygiene habits 2. Administer medications as ordered.
5. Withdrawal from interactions with others a. Corticosteroids: to decrease edema and
6. Diagnostic tests: X-rays of the skull, teeth, and pain
sinuses may identify dental or sinus infection b. Mild analgesics as necessary
as an aggravating factor. 3. Provide soft diet with supplementary feedings
D. Nursing interventions as indicated.
1. Assess characteristics of the pain including 4. Instruct to chew on unaffected side, avoid hot
triggering factors, trigger points, and pain fluids/foods, and perform mouth care after
management techniques. each meal.
2. Administer medications as ordered; monitor 5. Provide special eye care to protect the cornea.
response. a. Dark glasses (cosmetic and protective
3. Maintain room at an even, moderate reasons) or eyeshield
temperature, free from drafts. b. Artificial tears to prevent drying of the
4. Provide small, frequent feedings of lukewarm, cornea
semiliquid, or soft foods that are easily c. Ointment and eye patch at night to keep
chewed. eyelid closed
5. Provide the client with a soft washcloth and 6. Provide support and reassurance.
lukewarm water and perform hygiene during
periods when pain is decreased. Guillain-Barr Syndrome
6. Prepare the client for surgery if indicated.
7. Provide client teaching and discharge A. General information
planning concerning 1. Symmetrical, bilateral, peripheral polyneuritis
a. Need to avoid outdoor activities during characterized by ascending paralysis
cold, windy, or rainy weather 2. Can occur at any age; affects women and men
b. Importance of good nutrition and hygiene equally
c. Use of medications, side effects, and signs 3. Cause unknown; may be an autoimmune
of toxicity process
d. Specific instructions following surgery for 4. Precipitating factors: antecedent viral
residual effects of anesthesia and loss of infection, immunization
corneal reflex 5. Progression of disease is highly individual;
1) Protective eye care 90% of clients stop progression in 4 weeks;
2) Chew on unaffected side only recovery is usually from 36 months; may
3) Avoid hot fluids/foods have residual deficits
4) Mouth care after meals to remove B. Medical management
particles 1. Mechanical ventilation if respiratory problems
5) Good oral hygiene; visit dentist every present
6 months 2. Plasmapheresis to reduce circulating
6) Protect the face during extremes of antibodies
temperature 3. Continuous ECG monitoring to detect
alteration in heart rate and rhythm
Bells Palsy 4. Propranolol to prevent tachycardia
5. Atropine may be given to prevent episodes of
A. General information bradycardia during endotracheal suctioning
1. Disorder of cranial nerve VII resulting in the and physical therapy
loss of ability to move the muscles on one side C. Assessment findings
of the face 1. Mild sensory changes; in some clients severe
2. Cause unknown; may be viral or autoimmune misinterpretation of sensory stimuli resulting
3. Complete recovery in 35 weeks in majority of in extreme discomfort

4
clients 2. Clumsiness: usually first symptom

ADULT NURSING 223


53155_04_Ch04a_p143-263.qxd 2/26/09 6:50 PM Page 224

3. Progressive motor weakness in more than one 2.Dysarthria, dysphagia


limb (classically is ascending and symmetrical) 3.Fasciculations
4. Cranial nerve involvement (dysphagia) 4.Respiratory insufficiency
5. Ventilatory insufficiency if paralysis ascends 5.Diagnostic tests: EMG and muscle biopsy can
to respiratory muscles rule out other diseases
6. Absence of deep tendon reflexes C. Nursing interventions
7. Autonomic dysfunction 1. Provide nursing measures for muscle
8. Diagnostic tests weakness and dysphagia.
a. CSF studies: increased protein 2. Promote adequate ventilatory function.
b. EMG: slowed nerve conduction 3. Prevent complications of immobility.
D. Nursing interventions 4. Encourage diversional activities; spend time
1. Maintain adequate ventilation. with the client.
a. Monitor rate and depth of respirations; 5. Provide compassion and intensive support to
serial vital capacities. client/significant others.
b. Observe for ventilatory insufficiency. 6. Provide or refer for physical therapy as indicated.
c. Maintain mechanical ventilation as 7. Promote independence for as long as possible.
needed; keep airway free of secretions and
prevent pneumonia.
2. Check individual muscle groups every 2 hours DISORDERS OF THE EYE
in acute phase to check for progression of
muscle weakness.
3. Assess cranial nerve function: check gag reflex
Cataracts
and swallowing ability; ability to handle A. General information
secretions; voice. 1. Opacity of the ocular lens
4. Monitor vital signs and observe for signs of 2. Incidence increases with age
autonomic dysfunction such as acute periods 3. May be caused by changes associated with
of hypertension fluctuating with hypotension, aging (senile cataract); may be congenital; or
tachycardia, arrhythmias. may develop secondary to trauma, radiation,
5. Administer corticosteroids to suppress infection, certain drugs (corticosteroids)
immune reaction as ordered. B. Assessment findings
6. Administer antiarrhythmic agents as ordered. 1. Blurred vision
7. Prevent complications of immobility. 2. Progressive decrease in vision
8. Promote comfort (especially in clients with 3. Glare in bright lights
sensory changes): foot cradle, sheepskin, 4. Pupil may develop milky-white appearance
guided imagery, relaxation techniques. 5. Diagnostic test: ophthalmoscopic exam
9. Promote optimum nutrition. confirms presence of cataract
a. Check gag reflex before feeding. C. Nursing interventions: prepare client for cataract
b. Start with pureed foods. surgery.
c. Assess need for nasogastric tube feedings if
unable to swallow. Cataract Surgery
10. Provide psychologic support and
encouragement to client/significant others. A. General information
11. Refer for rehabilitation to regain strength and 1. Performed when client can no longer remain
to treat any residual deficits. independent because of reduced vision
2. Surgery performed on one eye at a time;
Amyotrophic Lateral Sclerosis usually in a same-day surgery unit
3. Local anesthesia and intravenous sedation
(Lou Gehrigs Disease)
usually used
A. General information 4. Types
1. Progressive motor neuron disease, which a. Extracapsular extraction: lens capsule is
usually leads to death in 26 years excised and the lens is expressed; posterior
2. Onset usually between ages 40 and 70; affects capsule is left in place (may be used to
men more than women support new artificial lens implant).
3. Cause unknown b. Phacoemulsification: a type of
4. There is no cure or specific treatment; death extracapsular extraction; a hollow needle
usually occurs as a result of respiratory capable of ultrasonic vibration is inserted
infection secondary to respiratory insufficiency into lens, vibrations emulsify the lens,
B. Assessment findings which is then aspirated.
1. Progressive weakness and atrophy of the c. Intracapsular extraction: lens is totally

4
muscles of the arms, trunk, or legs removed within its capsule, may be

224 NCLEX-RN Review


53155_04_Ch04a_p143-263.qxd 2/26/09 6:50 PM Page 225

delivered from eye by cryoextraction (lens e. Cataract glasses/contact lenses


is frozen with a metal probe and removed). 1) If a lens implant has not been
5. Peripheral iridectomy may be performed at the performed, the client will need glasses
time of surgery; small hole cut in iris to or contact lenses.
prevent development of secondary glaucoma 2) Temporary glasses are worn for
6. Intraocular lens implant often performed at the 14 weeks, then permanent glasses
time of surgery. fitted.
B. Nursing interventions: preoperative (see also 3) Cataract glasses magnify objects by 13
Perioperative Nursing) and distort peripheral vision; have
1. Assess vision in the unaffected eye since the client practice manual coordination
affected eye will be patched post-op. with assistance until new spatial
2. Provide pre-op teaching regarding measures to relationships become familiar; have
prevent increased intraocular pressure post-op. client practice walking, using stairs,
3. Administer medications as ordered. reaching for articles.
a. Topical mydriatics and cycloplegics to 4) Contact lenses cause less distortion of
dilate the pupil vision; prescribed at 1 month.
b. Topical antibiotics to prevent infection
c. Acetazolamide (Diamox) and osmotic
agents (oral glycerin or IV mannitol) to
Glaucoma
decrease intraocular pressure to provide a A. General information
soft eyeball for surgery 1. Characterized by increased intraocular pressure
C. Nursing interventions: postoperative resulting in progressive loss of vision; may
1. Reorient the client to surroundings. cause blindness if not recognized and treated
2. Provide safety measures: elevate side rails, 2. Risk factors: age over 40, diabetes,
provide call bell, assist with ambulation when hypertension, heredity; history of previous eye
fully recovered from anesthesia. surgery, trauma, or inflammation
3. Prevent increased intraocular pressure and 3. Types
stress on the suture line. a. Chronic (open-angle) glaucoma: most
a. Elevate head of bed 3040. common form, due to obstruction of the
b. Have client lie on back or unaffected side. outflow of aqueous humor, in trabecular
c. Avoid having client cough, sneeze, bend meshwork or canal of Schlemm
over, or move head too rapidly. b. Acute (closed-angle) glaucoma: due to
d. Treat nausea with antiemetics as ordered to forward displacement of the iris against the
prevent vomiting. cornea, obstructing the outflow of the
e. Give stool softeners as ordered to prevent aqueous humor; occurs suddenly and is an
straining. emergency situation; if untreated,
f. Observe for and report signs of increased blindness will result
intraocular pressure: severe eye pain, c. Chronic (closed-angle) glaucoma: similar to
restlessness, increased pulse. acute (closed-angle) glaucoma, with the
4. Protect eye from injury. potential for an acute attack
a. Dressings are usually removed the day 4. Early detection is very important; regular eye
after surgery. exams including tonometry for persons over
b. Eyeglasses or eye shield used during the age 40 is recommended.
day. B. Medical management
c. Always use eye shield during the night. 1. Chronic (open-angle) glaucoma
5. Administer medications as ordered. a. Drug therapy: one or a combination of the
a. Topical mydriatics and cycloplegics to following
decrease spasm of ciliary body and relieve 1) Miotic eyedrops (pilocarpine) to
pain increase outflow of aqueous humor
b. Topical antibiotics and corticosteroids 2) Epinephrine eyedrops to decrease
c. Mild analgesics as needed aqueous humor production and
6. Provide client teaching and discharge increase outflow
planning concerning 3) Acetazolamide (Diamox): carbonic
a. Technique of eyedrop administration anhydrase inhibitor to decrease
b. Use of eye shield at night aqueous humor production
c. No bending, stooping, or lifting 4) Timolol maleate (Timoptic): topical
d. Report signs or symptoms of complications beta-adrenergic blocker to decrease
immediately to physician: severe eye pain, intraocular pressure
decreased vision, excessive drainage, b. Surgery (if no improvement with drugs)

4
swelling of eyelid.

ADULT NURSING 225


53155_04_Ch04a_p143-263.qxd 2/26/09 6:50 PM Page 226

1) Filtering procedure (trabeculectomy, 3. Maintain accurate I&O with the use of osmotic
trephining) to create artificial openings agents.
for the outflow of aqueous humor 4. Prepare the client for surgery if indicated.
2) Laser trabeculoplasty: noninvasive 5. Provide post-op care (see Cataract Surgery).
procedure performed with argon laser 6. Provide client teaching and discharge
that can be done on an outclient basis; planning concerning
produces similar results as a. Self-administration of eyedrops
trabeculectomy b. Need to avoid stooping, heavy lifting, or
2. Acute (closed-angle) glaucoma pushing, emotional upsets, excessive fluid
a. Drug therapy (before surgery) intake, constrictive clothing around the
1) Miotic eyedrops (e.g., pilocarpine) to neck
cause pupil to contract and draw iris c. Need to avoid the use of antihistamines or
away from cornea sympathomimetic drugs (found in cold
2) Osmotic agents (e.g., glycerin [oral], preparations) in closed-angle glaucoma
mannitol [IV]) to decrease intraocular because they may cause mydriasis
pressure d. Importance of follow-up care
3) Narcotic analgesics for pain e. Need to wear Medic-Alert tag
b. Surgery
1) Peripheral iridectomy: portion of the
iris is excised to facilitate outflow of
Detached Retina
aqueous humor A. General information
2) Argon laser beam surgery: noninvasive 1. Detachment of the sensory retina from the
procedure using laser that produces pigment epithelium of the retina
same effect as iridectomy; done on an 2. Caused by trauma, aging process, severe
outclient basis myopia, postcataract extraction, severe
3) Iridectomy usually performed on diabetic retinopathy
second eye later because a large 3. Pathophysiology: tear in the retina allows
number of clients have an acute attack vitreous humor to seep behind the sensory
in the other eye retina and separate it from the pigment
3. Chronic (closed-angle) glaucoma epithelium
a. Drug therapy: miotics (pilocarpine) B. Medical management
b. Surgery: bilateral peripheral iridectomy to 1. Bed rest with eyes patched and detached areas
prevent acute attacks dependent to prevent further detachment
C. Assessment findings 2. Surgery: necessary to repair detachment
1. Chronic (open-angle) glaucoma: symptoms a. Photocoagulation: light beam (argon laser)
develop slowly; impaired peripheral vision through dilated pupil creates an
(tunnel vision); loss of central vision if inflammatory reaction and scarring to heal
unarrested; mild discomfort in the eyes; halos the area
around lights b. Cryosurgery or diathermy: application of
2. Acute (closed-angle) glaucoma: severe eye extreme cold or heat to the external globe;
pain; blurred, cloudy vision; halos around inflammatory reaction causes scarring and
lights; nausea and vomiting; steamy cornea; healing of area
moderate pupillary dilation c. Scleral buckling: shortening of sclera to
3. Chronic (closed-angle) glaucoma: transient force pigment epithelium close to retina
blurred vision; slight eye pain; halos around C. Assessment findings
lights 1. Flashes of light, floaters
4. Diagnostic tests 2. Visual field loss, veil-like curtain coming
a. Visual acuity: reduced across field of vision
b. Tonometry: reading of 2432 mm Hg 3. Diagnostic test: ophthalmoscopic examination
suggests glaucoma; may be 50 mm Hg or confirms diagnosis
more in acute (closed-angle) glaucoma D. Nursing interventions: preoperative
c. Ophthalmoscopic exam: reveals narrowing 1. Maintain bed rest as ordered with head of bed
of small vessels of optic disk, cupping of flat and detached area in a dependent position.
optic disk 2. Use bilateral eye patches as ordered; elevate
d. Perimetry: reveals defects in visual fields side rails to prevent injury.
e. Gonioscopy: examine angle of anterior 3. Identify yourself when entering the room.
chamber 4. Orient the client frequently to time, date, and
D. Nursing interventions surroundings; explain procedures.
1. Administer medications as ordered. 5. Provide diversional activities to provide

4
2. Provide quiet, dark environment. sensory stimulation.

226 NCLEX-RN Review


53155_04_Ch04a_p143-263.qxd 2/26/09 6:50 PM Page 227

E. Nursing interventions: postoperative (see also DISORDERS OF THE EAR


Cataract Surgery)
1. Check orders for positioning and activity level.
a. May be on bed rest for 12 days. Otosclerosis
b. May need to position client so that A. General information
detached area is in dependent position. 1. Formation of new spongy bone in the
2. Administer medications as ordered: topical labyrinth of the ear causing fixation of the
mydriatics, analgesics as needed. stapes in the oval window; this prevents
3. Provide client teaching and discharge transmission of auditory vibration to the
planning concerning inner ear
a. Technique of eyedrop administration 2. Found more often in females
b. Use of eye shield at night 3. Cause unknown, but there is a familial
c. No bending from waist; no heavy work or tendency
lifting for 6 weeks B. Medical management: stapedectomy is the
d. Restriction of reading for 3 weeks or more procedure of choice.
e. May watch television C. Assessment findings
f. Need to check with physician regarding 1. Progressive loss of hearing
combing and shampooing hair and shaving 2. Tinnitus
g. Need to report complications such as 3. Diagnostic tests
recurrence of detachment a. Audiometry: reveals conductive hearing
loss
Eye Injuries/Emergency Care b. Webers and Rinnes tests: show bone
conduction is greater than air conduction
A. Removal of loose foreign body from conjunctiva D. Nursing interventions: see Stapedectomy.
1. Foreign bodies, e.g., sand, dust, may cause
pain and lacrimation.
2. Instruct client to look upward. Stapedectomy
3. Evert lower lid to expose the conjunctival sac. A. General information
4. Gently remove the particle with a cotton 1. Removal of diseased portion of stapes and
applicator dipped in sterile normal saline replacement with a prosthesis to conduct
using a twisting motion. vibrations from the middle ear to inner ear;
5. If particle is not found, examine the upper lid. usually performed under local anesthesia
6. Place cotton applicator stick or tongue blade 2. Used to treat otosclerosis
horizontally on outer surface of upper lid; B. Nursing interventions: preoperative
grasp under eyelashes with fingers of other 1. Provide general pre-op nursing care, including
hand and pull the upper lid outward and an explanation of post-op expectations.
upward over the applicator stick. 2. Explain to the client that hearing may improve
7. Gently remove the particle as previously during surgery and then decrease due to
described. edema and packing.
B. Penetrating injuries to the eye C. Nursing interventions: postoperative
1. Examples: darts, scissors, flying metal. 1. Position the client according to the surgeons
2. Do not attempt to remove object. orders (possibly with operative ear uppermost
3. Do not allow client to apply pressure to the eye. to prevent displacement of the graft).
4. Cover eye lightly with sterile eye patch for 2. Have client deep breathe every 2 hours while
embedded objects, e.g., metal; apply protective in bed, but no coughing.
shield, e.g., paper cup, for impaled objects 3. Elevate side rails; assist the client with
such as darts. ambulation and move slowly (may have some
5. Cover uninjured eye to prevent excessive vertigo).
movement of injured eye. 4. Administer medications as ordered:
6. Refer the client to an emergency room analgesics, antibiotics, antiemetics, anti-
immediately. motion-sickness drugs.
C. Chemical burns 5. Check dressings frequently for excessive
1. Flush eye immediately with copious amounts drainage or bleeding.
of water for 1520 minutes. 6. Assess facial nerve function, i.e., ask client to
a. Have client hold head under faucet to let wrinkle forehead, close eyelids, puff out
water run over eye to thoroughly wash it cheeks, smile and show teeth; check for any
out; may need to forcibly separate eyelids asymmetry.
during flush 7. Question client about pain, headache, vertigo,
b. If available, flush eye with syringe and unusual sensations in the ear; report

4
2. After flushing, refer client to an emergency existence to physician.
room immediately.

ADULT NURSING 227


53155_04_Ch04a_p143-263.qxd 2/26/09 6:50 PM Page 228

8. Provide client teaching and discharge 4. Monitor IV therapy; maintain accurate I&O.
planning concerning 5. Assist with ambulation when the attack is over.
a. Warnings against blowing nose or 6. Administer medications as ordered.
coughing; sneeze with the mouth open 7. Prepare the client for surgery as indicated
b. Need to keep ear dry in the shower; no (post-op care includes using above measures).
shampooing until allowed 8. Provide client teaching and discharge
c. No flying for 6 months, especially if an planning concerning
upper respiratory tract infection is present a. Use of medication and side effects
d. Placement of cotton ball in auditory b. Low-sodium diet and decreased fluid intake
meatus after packing is removed; change c. Importance of eliminating smoking
twice a day

Mnires Disease Sample Questions


A. General information
1. Disease of the inner ear resulting from dilation
of the endolymphatic system and increased 108. An adult has a medical diagnosis of increased
volume of endolymph; characterized by intracranial pressure and is being cared for on
recurrent and usually progressive triad of the neurology unit. The nursing care plan
symptoms: vertigo, tinnitus, and hearing loss includes elevating the head of the bed and
2. Incidence highest between ages 30 and 60 positioning the clients head in proper
3. Cause unknown; theories include allergy, alignment. What is the reason for these actions?
toxicity, localized ischemia, hemorrhage, viral 1. Makes it easier for the client to breathe.
infection, or edema
2. Prevents a Valsalva maneuver.
B. Medical management
1. Acute: atropine (decreases autonomic nervous 3. Promotes venous drainage.
system activity), diazepam (Valium), fentanyl, 4. Reduces pain.
and droperidol (Innovar)
2. Chronic 109. A client begins to have Cheyne-Stokes respirations.
a. Drug therapy: vasodilators (nicotinic acid), What is the correct explanation for this occurrence?
diuretics, mild sedatives or tranquilizers 1. Completely irregular breathing pattern with
(diazepam [Valium]), antihistamines random deep and shallow respirations.
(diphenhydramine [Benadryl], meclizine 2. Prolonged inspirations with inspiratory
[Antivert])
and/or expiratory pauses.
b. Low-sodium diet, restricted fluid intake,
restrict caffeine and nicotine. 3. Rhythmic waxing and waning of both rate
3. Surgery and depth of respiration with brief periods of
a. Surgical destruction of labyrinth causing interspersed apnea.
loss of vestibular and cochlear function (if 4. Sustained, regular, rapid respirations of
disease is unilateral) increased depth.
b. Intracranial division of vestibular portion
of cranial nerve VIII 110. Which of the following reduces cerebral edema
c. Endolymphatic sac decompression or by constricting cerebral veins?
shunt to equalize pressure in 1. Dexamethasone (Decadron).
endolymphatic space 2. Mechanical hyperventilation.
C. Assessment findings
1. Sudden attacks of vertigo lasting hours or 3. Mannitol (Osmitrol).
days; attacks occur several times a year 4. Ventriculostomy.
2. Nausea, tinnitus, progressive hearing loss
3. Vomiting, nystagmus 111. The nurse is caring for an adult client who was
4. Diagnostic tests admitted unconscious. The initial assessment
a. Audiometry: reveals sensorineural hearing utilized the Glasgow Coma Scale. The nurse
loss knows that of the following, which are included
b. Vestibular tests: reveal decreased function when assessing a client using the Glasgow Coma
D. Nursing interventions Scale? Select all that apply.
1. Maintain bed rest in a quiet, darkened room in ____ Eye opening.
position of choice; elevate side rails as needed. ____ Motor response.
2. Only move the client for essential care (bath
may not be essential). ____ Pupillary reaction

4
3. Provide an emesis basin for vomiting. ____ Verbal performance.

228 NCLEX-RN Review


53155_04_Ch04a_p143-263.qxd 2/26/09 6:50 PM Page 229

112. Utilizing the Glasgow Coma Scale, which score 118. The nurse is assessing reflexes on a client.
would be indicative of coma? Which of the following correctly describes this
1. 0 reflex?
2. 2 1. Extension of the elbow and contraction of the
3. 6 triceps tendon.
4. 10 2. Supination and flexion of the forearm.
3. Dorsiflexion of the great toe with fanning of
113. When the nurse tested an unconscious client for the other toes.
noxious stimuli, the client responded with 4. Flexion of the arm at the antecubital fossa
decorticate rigidity or posturing. What is the best and contraction of the biceps.
description for this action?
1. Flexion of the upper and lower extremities 119. The nurse is assessing the optic nerve of a client.
into a fetal-like position. Which of the following is a correct method to
2. Rigid extension of the upper and lower evaluate cranial nerve (CN) II, the optic nerve?
extremities and plantar flexion. 1. Inspect the pupils for reaction.
3. Complete flaccidity of both upper and lower 2. Test extraocular movements.
extremities and hyperextension of the neck. 3. Use of a Snellen chart.
4. Flexion of the upper extremities, extension of 4. Test for a corneal reflex.
the lower extremities, and plantar flexion.
120. Which of the following tests or tools could the
114. An adult male is receiving cryotherapy for repair nurse use to assess CN VIII, the acoustic nerve?
of a detached retina. When taking a history from 1. Romberg.
him, which symptom should the nurse expect 2. Rosenbaum chart.
him to have?
3. Inspection of pupils.
1. Diplopia.
4. Audiometry.
2. Severe eye pain.
3. Sudden blindness. 121. A nurse is obtaining a Glasgow Coma Score on a
4. Bright flashes of light. client. The score is as follows:
Best eye opening 3
115. An adult who has a detached retina asks the Best motor response 6
nurse what may have contributed to the Best verbal response 4
development of his detached retina. What is a How would the nurse interpret this score?
risk factor associated with this condition? 1. Opens eyes to speech, obeys verbal
1. Hypertension. commands, and is confused.
2. Nearsightedness. 2. Opens eyes to pain, decoricates to pain, and
3. Cranial tumors. does not speak.
4. Sinusitis. 3. Opens eyes to pain, no motor response, and
has inappropriate speech.
116. The nurse is explaining cryotherapy to a client 4. Opens eyes spontaneously, obeys verbal
who has a detached retina. What would be a commands, and is oriented 3.
major purpose for the procedure?
1. Create a scar that promotes healing. 122. A nurse is preparing a client for an MRI. Which
factor would exclude the client from the test?
2. Disintegrate debris in the eye.
1. Wearing jewelry.
3. Freeze small blood vessels.
2. Cardiac pacemaker.
4. Halt secretions of the lacrimal duct.
3. Claustrophobia.
117. An adult client has a stapedectomy. Which of 4. Allergy to iodine.
the following is most important for the nurse to
include in the post-op care plan? 123. A nurse is assessing a client who has returned
1. Checking the gag reflex. from a cerebral arteriogram. The left carotid was
the site punctured. Which of the following
2. Encouraging independence.
indicates complications?
3. Instructing the client not to blow the nose.
1. Difficulty in swallowing.
4. Positioning the client on the operative side.
2. Puncture site is dry and red.

ADULT NURSING

4 229
53155_04_Ch04a_p143-263.qxd 2/26/09 6:50 PM Page 230

3. BP 120/82, HR 86, RR 22. 130. A client presents with symptoms of increased


4. No swelling or hematoma at the site. intracranial pressure, papilledema, and
headache. No history of trauma is found. Vital
124. A client with a closed head injury is confused, signs are: BP 110/60, HR 80, T 98.9F, RR 24.
drowsy, and has unequal pupils. Which of the What condition does the nurse suspect?
following nursing diagnosis is most important at 1. Brain tumor.
this time? 2. Meningitis.
1. Altered level of cognitive function. 3. Skull fracture.
2. High risk for injury. 4. Encephalitis.
3. Altered cerebral tissue perfusion.
4. Sensory perceptual alteration. 131. What should the nurse include in the plan of
care for a newly admitted client with an
125. A client is admitted with a head injury. To infratentorial craniotomy for a brain tumor?
monitor hypothalamic function, the nurse 1. Keep the head of the bed elevated 3045;
should monitor what parameters? and a large pillow under the clients head and
1. Temperature and urinary output. shoulder.
2. Gastric aspirate and BP. 2. Keep the head of the bed flat with a small
3. Heart rate and pupillary responses. pillow under the nape of the neck.
4. Respiratory rate and skin integrity. 3. Assess vital signs and pupils every four hours.
4. Flex neck every 2 hours to prevent stiffness.
126. Which of the following is the best way for the
nurse to assist a blind client in ambulation? 132. The home health nurse observes an aide who is
1. Allow client to take nurses arm with the transferring a client with hemiplegia from a
nurse walking slightly ahead of the client. sitting position in the bed to the wheelchair.
2. Allow client to walk beside the nurse with Which action by the aide requires correction?
the nurses hand on the clients back. 1. Grasping the clients arms to pull the client to
3. Allow client to walk down the hall with his a standing position.
or her hand along the wall. 2. Reminding the client to lean forward before
4. Push the client in a wheelchair. rising.
3. Moving the client toward the unaffected side.
127. Which of the following is the best way for the nurse 4. Bracing the affected knee and foot to assist
to communicate with the hearing impaired client? the client to stand.
1. Talk directly into the impaired ear.
2. Speak directly and clearly facing the person. 133. When comparing a cerebrovascular accident
(CVA) to a transient ischemic attack (TIA), what
3. Shout into the good ear.
is unique about the TIA?
4. Write out all communication.
1. It has permanent long-term focal deficits.
128. A nurse is assessing a client who is unable to 2. It is intermittent with spontaneous resolution
extend the legs without pain, has a temperature of the neurologic deficit.
of 103F, and on flexion of the neck also flexes 3. It is intermittent with permanent motor and
the hip and knee. Based on this assessment, sensory deficits.
What condition does the nurse suspect? 4. It has permanent long-term neurologic deficits.
1. Meningitis.
2. Brain abscess. 134. The nurse is teaching a client with transient
ischemic attacks about aspirin therapy. Which
3. Brain tumor.
statement by the client indicates understanding
4. Epilepsy. of the reason for his aspirin therapy?
129. A client has a fever of 103F, nuchal rigidity, 1. I must take the aspirin regularly to prevent
pain on extension of the legs, and opisthotonos. the headache that many people have with
What would be the priority nursing diagnosis? this disorder.
1. Acute pain. 2. If I take aspirin, I am less likely to develop
bleeding in my brain.
2. Ineffective tissue perfusion
3. The aspirin will help to prevent me from
3. Anxiety.
having a stroke.

4
4. Risk for injury.
4. Taking aspirin regularly will reduce my
chances of having a heart attack.

230 NCLEX-RN Review


53155_04_Ch04a_p143-263.qxd 2/26/09 6:50 PM Page 231

135. A client with Parkinsons disease is receiving 1. Measure the pH of the fluid.
combination therapy with levodopa (L-dopa) 2. Measure the specific gravity of the fluid.
and carbidopa (Sinemet). Which of the following 3. Test for glucose.
manifestations indicates to the nurse that an
4. Test for chloride.
adverse drug reaction is occurring?
1. Involuntary head movement. 141. The nurse is caring for a confused client who
2. Bradykinesia. sustained a head injury resulting in a subdural
3. Shuffling gait. hematoma. The clients blood pressure is
4. Depression. 100/60 mm Hg and he is unresponsive. Select
the most effective position for the client as the
136. A nurse is teaching the family of a client with nurse transports him to the operating room.
Parkinsons disease. Which of the following 1. Semi-Fowlers.
statements by the family reflects a need for more 2. Trendelenburg.
education? 3. High-Fowlers.
1. We can buy lots of soups for Dad. 4. Supine.
2. We are teaching Dad posture exercises.
3. Dad is going to do his range of motion 142. A client who has been treated in the emergency
(ROM) exercises three times a day. room for a head injury is preparing for
4. The bath bars will be installed before Dad discharge. The nurse is teaching the family
comes home. about the signs of complications that may occur
within the first 24 hours and the appropriate
137. A 36-year-old female reports double vision, action to take if a complication is suspected.
visual loss, muscular weakness, numbness of the Which statement by the clients spouse would
hands, fatigue, tremors, and incontinence. Based require further teaching by the nurse?
on this report, what does the nurse suspect? 1. Im not looking forward to checking on my
1. Parkinsons disease. husband all night long.
2. Myasthenia gravis (MG). 2. If he can just get a long nap, Im sure that
3. Amyotrophic lateral sclerosis (ALS). my husband will be fine.
4. Multiple sclerosis (MS). 3. Ill call the doctor immediately if my
husband starts to vomit.
138. Which nursing diagnosis is of the highest 4. If my husband has trouble talking, Ill bring
priority when caring for a client with him to the hospital.
myasthenia gravis (MG)?
1. Pain. 143. A client is admitted postcraniotomy. Decadron
4 mg IV is ordered every 6 hours. What is the
2. Risk for injury.
purpose for this medication?
3. Ineffective coping.
1. Stabilize the blood sugar.
4. Ineffective airway clearance.
2. Decrease cerebral edema.
139. The nurse has explained the use of neostigmine 3. Prevent seizures.
methylsulfate (Prostigmin) to a client with 4. Maintain the integrity of the gastric
myasthenia gravis. Which comment by the client mucosa.
indicates the need for further instruction?
1. I need to take the medication regularly, even 144. A client is admitted with a C7 complete
when I feel strong. transection. What must the nurse plan for in the
immediate post-injury period?
2. I should take the medication once daily at
bedtime. 1. Bladder and bowel training.
3. If I take too much medication, I can become 2. Possible ventilatory support.
weak and have breathing problems. 3. Complications of autonomic dysreflexia.
4. I may have difficulty swallowing my saliva 4. Diaphragmatic pacing.
if I take too much medication.
145. A client fell backward over a stair rail to the
140. A nurse is assessing a client with a head injury. floor below and is not breathing. After calling for
The client has clear drainage from the nose and assistance, how should the nurse proceed?
ears. How can the nurse determine if the 1. Initiate rescue breathing by performing a chin

4
drainage is cerebrospinal fluid (CSF)? tilt maneuver and administering two breaths.

ADULT NURSING 231


53155_04_Ch04a_p143-263.qxd 2/26/09 6:50 PM Page 232

2. After determining absence of breathing, 4. Because you cannot blink the affected eye, it
administer 15 chest compressions at the rate can become dry and irritated.
of 60 per minute.
3. Initiate rescue breathing by performing a jaw 150. A nurse is caring for a client with Guillain-Barr
thrust maneuver and administering two syndrome. Which of the following strategies is of
breaths. the most importance in the plan of care?
4. After determining pulselessness, administer 1. Range of motion exercises three to four times
five chest compressions at the rate of 60 per per day.
minute. 2. Frequent measurement of vital capacity.
3. Use of artificial tears.
146. A client with a cervical spine injury was placed 4. Starting an enteral feeding.
in Halo traction yesterday. When the client
complains of discomfort around the pins, what 151. The nurse has presented information about
action should the nurse take? amyotrophic lateral sclerosis (ALS) to a newly
1. Carefully loosen the pins and notify the diagnosed client. Which question by the client
physician immediately. indicates that he understands the nature of the
2. Cleanse the skin around the pin sites and dry disease?
the area thoroughly. 1. How can I avoid infecting my family with
3. Give the ordered analgesic and reassure the the virus?
client that the pain is temporary. 2. How can I execute a living will?
4. Loosen the pins immediately and maintain 3. How can I prevent an exacerbation of the
the head in a neutral position. disease?
4. How many people achieve remission with
147. A client with a C6 spinal cord injury 2 months chemotherapy?
ago now complains of a pounding headache.
The pulse is 64 and the blood pressure is 152. A client reports gradual painless blurring of
220/110 mm Hg. Which of the following vision. On assessment, the nurse notes a cloudy
actions should the nurse take first? opaque lens. What condition does the nurse
1. Give the analgesic as ordered. suspect?
2. Check the clients output. 1. Glaucoma.
3. Elevate the clients head and lower the legs. 2. Cataracts.
4. Notify the physician. 3. Retinal detachment.
4. Diabetic retinopathy.
148. The nurse is evaluating the ability of a client
with trigeminal neuralgia to implement the 153. Which of the following risk factors would the
treatment that has been suggested. Which of the nurse assess for in a client with glaucoma?
following behaviors by the client will be most 1. Family history, increased intraocular
effective in controlling manifestations? pressure, and age of 4565.
1. Exercise the facial muscles at least twice daily. 2. History of diabetes and age greater than 50.
2. Put the affected arm through full range of 3. Female gender, cigarette smoking, age greater
motion daily. than 65.
3. Avoid extremes in temperature of food and 4. Myopia, history of diabetes, and sudden
drink. severe physical exertion.
4. Use proper body mechanics in sitting and
bending. 154. The nurse has been planning for home care with
the family of a client who will undergo
149. A client with Bells palsy asks the nurse why extracapsular lens extraction with an intraocular
artificial tears were ordered by the physician. lens implant. Because the client and family speak
Select the best reply by the nurse. very little English, the nurse takes extra care to
1. When your affected eye fails to make tears, evaluate their understanding. Which behavior by
the eye can become irritated and ulcerated. the client and/or family shows progress in
2. Because your eye remains closed, foreign understanding post-op home care instructions?
matter can be trapped beneath the lid. 1. Using a chart showing various sleeping
3. Artificial tears will remove the purulent positions, the client points to a person lying
on the affected side.

4
drainage from your eye, which speeds
healing.

232 NCLEX-RN Review


53155_04_Ch04a_p143-263.qxd 2/26/09 6:50 PM Page 233

2. The family demonstrates that the eye should favors bone conduction. What condition does
be cleaned with a washcloth, soap, and water. the nurse suspect?
3. The client demonstrates medication 1. Cholesteatoma.
instillation by carefully dropping the solution 2. Actinic keratosis.
on the cornea. 3. External otitis.
4. The family shows the nurse the sunglasses 4. Otosclerosis.
they have purchased for the client to wear
post-op. 160. The nurse is teaching a post-op stapedectomy
client. What should be included in the teaching?
155. A nurse is admitting a client who reports vision
1. Work can be resumed the next day.
loss. What additional information will be
reported for the nurse to suspect glaucoma? 2. Gently sneeze or cough with the mouth
closed.
1. Seeing floating spots.
3. Avoid airline flight for 6 months.
2. Eye pain.
4. Resume exercise in 1 week.
3. Seeing flashing lights.
4. Sudden loss of vision. 161. A client reports very loud, overpowering ringing
in the ears, fluctuating hearing loss on the right
156. Which of the following techniques should the side with severe vertigo accompanied by nausea
nurse use to evaluate a clients understanding of and vomiting, What condition does the nurse
self-care for chronic (primary) open-angle suspect?
glaucoma?
1. Mnires disease.
1. The nurse asks for the clients weekly blood
2. Acoustic neuroma.
pressure readings.
3. Otosclerosis.
2. The nurse asks if the client avoids watching
television. 4. Cholesteatoma.
3. The nurse observes the clients technique for 162. What is the priority nursing diagnosis for a
monitoring blood glucose. client with very loud overpowering ringing in
4. The nurse observes the clients his ears, fluctuating hearing loss on the right
administration of eye drops. side with severe vertigo accompanied by nausea
and vomiting and a feeling of fullness in the
157. A client is admitted with a detached retina of right ear?
the left eye. The nurse patches both eyes. What
1. Knowledge deficit related to the disease
is the rationale for patching both eyes?
process.
1. To prevent eye infections.
2. Anxiety.
2. To decrease eye movement.
3. Impaired physical mobility.
3. To prevent photophobia.
4. Pain.
4. To prevent nystagmus.

158. A neighbor splashes chlorine bleach in her eyes


and calls the nurse for immediate help. What
should be the nurses first action? Answers and Rationales
1. Lift the upper lid over the lower lid of each
eye.
2. Close and patch both eyes with a loose 108. 3. It has been demonstrated that positioning the
bandage. client with the head elevated to 30 decreases
ICP by promoting venous drainage from the head
3. Continuously flush the eyes with tap water
by gravity. Pronounced angulation of the head
for 20 minutes.
can obstruct venous return and increase ICP.
4. Instill an over-the-counter anti-irritant
solution, such as Visine. 109. 3. Cheynes-Stokes respirations are a pattern of
breathing in which phases of hyperapnea
159. A client reports bilateral hearing loss. On regularly alternate with apnea. The pattern
assessment of the ear, the nurse observes chalky waxes (crescendo) and wanes (decrescendo).
white plaques on the eardrum and the eardrum
appears pinkish orange in color. The Rinne test

ADULT NURSING

4 233
53155_04_Ch04a_p143-263.qxd 2/26/09 6:50 PM Page 234

110. 2. Mechanical hyperventilation to reduce CO2 122. 2. Pacemakers and cerebral aneurysm clips are
levels to 25 mm Hg produces cerebral the exclusions for an MRI.
vasoconstriction and thereby decreases ICP.
123. 1. Difficulty swallowing occurs from a
111. Eye opening should be selected. The Glasgow hematoma developing and pushing on the
Coma Scale is a practical scale that trachea.
independently evaluates three features: eye
opening, motor response in the upper limb, and 124. 3. The client is manifesting symptoms of
verbal performance. increased intracranial pressure.
Motor response should be selected. The Glasgow
125. 1. Increased intracranial pressure causes
Coma Scale is a practical scale that
hypothalamic dysfunction creating
independently evaluates three features: eye
hypo/hyperthermia, SIADH, and diabetes
opening, motor response in the upper limb, and
insipidus. The hypothalamus regulates body
verbal performance.
temperature, osmolality of body fluids, hunger,
Verbal performance should be selected. The
and satiety.
Glasgow Coma Scale is a practical scale that
independently evaluates three features: eye 126. 1. This method allows the client to have a
opening, motor response in the upper limb, and feeling of control.
verbal performance.
127. 2. Facing the person and speaking clearly is the
112. 3. A score of 7 or less defines coma. The lowest best way to communicate with the hearing
achievable score is 3, which indicates deep impaired.
coma. Fifteen is a perfect score.
128. 1. These are some of the symptoms of
113. 4. Decorticate rigidity or posturing is best meningitis.
described as an abnormal flexor response in the
arm with extension and plantar flexion in the 129. 2. The ineffective tissue perfusion is related to
lower extremities. the increased intracranial pressure and
inflammatory process.
114. 4. Momentary bright flashes of light are a
common symptom of retinal detachment. 130. 1. These findings are consistent with a brain
tumor.
115. 2. Myopia or nearsightedness is a predisposing
factor in the development of a retinal tear. 131. 2. This is the correct position for an
infratentorial approach.
116. 1. Cryotherapy is used to produce a chorioretinal
adhesion or scar that allows the retina to return 132. 1. Pulling the clients paralyzed arm can result in
to its normal position. shoulder subluxation and pain. The clients
unaffected hand must be free to reach for the
117. 3. The client should be taught to avoid blowing arm of the wheelchair.
the nose because this action could increase the
pressure in the eustachian tube and dislodge the 133. 2. A TIA is a temporary loss of function due to
surgical graft. cerebral ischemia.

118. 3. The response describes the Babinski reflex. 134. 3. Platelet-inhibiting drugs such as aspirin are
It is abnormal in an adult, signifying an upper taken prophylactically to prevent cerebral
motor neuron lesion. infarction secondary to embolism and
thrombosis.
119. 3. To correctly test cranial nerve II, the optic
nerve, use a Snellen chart to assess visual acuity. 135. 4. Depression, confusion, and hallucinations are
adverse effects that can occur after prolonged
120. 4. An audiometry test tests different pitches and use of L-dopa. A drug holiday under medical
sounds. supervision may restore drug effectiveness.
121. 1. Three points are given for opening eyes to 136. 1. The client should have semisolid, thickened
speech, 6 points are given for obeying verbal food. Soup is thin in texture and could be
commands related to motor response, and aspirated by the client.
4 points are given for best verbal response
when client is confused.

4 234 NCLEX-RN Review


53155_04_Ch04a_p143-263.qxd 2/26/09 6:50 PM Page 235

137. 4. These are the symptoms of MS, which is more The eye may not close completely. These
common in women ages 2040. problems render the eye susceptible to drying
and irritation from dust or other debris.
138. 4. Clients with MG have respiratory muscle
failure. 150. 2. Clients with Guillain-Barr have respiratory
muscle weakness and respiratory failure.
139. 2. The client is confused about the timing
of medication administration. The 151. 2. Clients with ALS often experience respiratory
anticholinesterase medication should be taken failure as the disease progresses and need to
30 minutes prior to meals to enhance the muscle communicate their wishes regarding ventilator
strength needed for chewing and swallowing. support. The nurse should explore the clients
wishes and facilitate discussion within the
140. 3. Cerebrospinal fluid is positive for glucose. family. Arranging for the client to sign a living
will, if the client wishes to do so, is also a
141. 1. The clients head should be elevated about 30
nursing responsibility.
to lower the intracranial pressure, which may be
dangerously elevated in a subdural hematoma. 152. 2. These are the assessment findings of cataracts.
The venous blood pressure begins to decline as
intracranial pressure rises. 153. 1. These are common risk factors for glaucoma.

142. 2. The wife may not understand that she must 154. 4. Sunglasses should be worn post-op for
interrupt the clients sleep to detect early signs comfort and protection when outdoors.
of increased intracranial pressure caused by
contusion or hematoma development. 155. 2. Eye pain is present with open- and narrow-
angle glaucoma, but not with a detached retina.
143. 2. Cerebral edema is common after surgery.
Decadron (a corticosteroid) is given to decrease 156. 4. Glaucoma is usually treated with eye drops,
the edema. such as betaxolol (Timoptic), a beta-adrenergic
antagonist. The eye can be damaged when eye
144. 2. Edema above the area of the lesion can cause drops are used incorrectly.
respiratory depression and arrest.
157. 2. Eye movements can increase the amount of
145. 3. When initiating rescue breathing for a client detachment.
with a suspected spinal injury, the jaw thrust
maneuver is used with rescue breathing at the 158. 3. Immediate irrigation with copious amounts of
rate of 12 breaths per minute. water or normal saline solution may reduce
alkaline burns of the cornea and conjunctiva.
146. 3. Discomfort at the pin sites is expected for Any delay in initiating the irrigation can result
several days after application of the Halo device. in serious damage to eye structures.
The pain can be controlled with mild analgesic
medication. The client can benefit from the 159. 4. These are classic signs of otosclerosis.
reassurance that the pain will not continue for
160. 3. The client should avoid flying to prevent
the weeks that the traction will be in place.
pressure changes in the ear at higher altitudes.
147. 3. The client is showing signs of autonomic
161. 1. These are classic signs of Mnires disease.
dysreflexia. Placing the client in a sitting
position will allow blood to pool in the legs, 162. 1. This client most likely has Mnires disease.
which should lower the blood pressure and In Mnires disease, client education is
prevent possible hypertensive hemorrhage. paramount. The client needs to be taught that
with the increased volume of hydrolymph,
148. 3. Extremes of temperature of food or drink can
excessive fluid intake increases the volume even
trigger paroxysms of severe facial pain along the
more and exacerbates the disease. They should
pathways of the trigeminal nerve. Meals are
also be taught not to ambulate or make extreme
better tolerated if served at room temperature.
movements during the acute attacks.
149. 4. Bells palsy may cause paralysis of the eyelid
and loss of the blink reflex on the affected side.

ADULT NURSING

4
235
53155_04_Ch04a_p143-263.qxd 2/26/09 6:50 PM Page 236

The Cardiovascular System

OVERVIEW OF ANATOMY Valves


AND PHYSIOLOGY See Figure 4-5.
A. Atrioventricular (AV) valves
The cardiovascular system consists of the heart, 1. Mitral valve: located between the left atrium
arteries, veins, and capillaries. The major functions and left ventricle; contains two leaflets
are circulation of blood, delivery of oxygen and other attached to the chordae tendinae.
nutrients to the tissues of the body, and removal of 2. Tricuspid valve: located between the right
carbon dioxide and other products of cellular atrium and right ventricle; contains three
metabolism. leaflets attached to the chordae tendinae.
3. Functions
Heart a. Permit unidirectional flow of blood from
specific atrium to specific ventricle during
The heart is a muscular pump that propels blood into ventricular diastole.
the arterial system and receives blood from the venous b. Prevent reflux flow during ventricular
system. systole.
c. Valve leaflets open during ventricular
Heart Wall diastole and close during ventricular
systole; valve closure produces first heart
A. Pericardium: composed of fibrous (outermost sound (S1).
layer) and serous pericardium (parietal and B. Semilunar valves
visceral); a sac that functions to protect the heart 1. Pulmonary valve: located between right
from friction. ventricle and pulmonary artery
B. Epicardium: covers surface of heart, becomes 2. Aortic valve: located between left ventricle
continuous with visceral layer of serous and aorta
pericardium. 3. Functions
C. Myocardium: middle, muscular layer. a. Permit unidirectional flow of blood from
D. Endocardium: thin, inner membranous layer specific ventricle to arterial vessel during
lining the chambers of the heart. ventricular systole.
E. Papillary muscles: arise from the endocardial and
myocardial surface of the ventricles and attach to
the chordae tendinae.
F. Chordae tendinae: attach to the tricuspid and
mitral valves and prevent eversion during systole.

Chambers
A. Atria: two chambers, function as receiving
LA
chambers, lie above the ventricles
1. Right atrium: receives systemic venous blood
through the superior vena cava, inferior vena Aortic valve
cava, and coronary sinus.
Mitral valve
2. Left atrium: receives oxygenated blood Pulmonic valve
returning to the heart from the lungs through RA
LV
the pulmonary veins. Tricuspid valve
B. Ventricles: two thick-walled chambers; major RV
responsibility for forcing blood out of the heart;
lie below the atria.
1. Right ventricle: contracts and propels
deoxygenated blood into the pulmonary
circulation via the pulmonary artery.
2. Left ventricle: propels blood into the systemic
circulation via the aorta during ventricular Figure 4-5 The valves of the heart; arrows indicate
systole. the direction of blood flow.

4 236 NCLEX-RN Review


53155_04_Ch04a_p143-263.qxd 2/26/09 6:50 PM Page 237

Figure 4-7 Coronary circulation

Coronary Circulation
Figure 4-6 Conduction system of the heart See Figure 4-7.
A. Coronary arteries: branch off at the base of the
aorta and supply blood to the myocardium and the
conduction system; two main coronary arteries are
b. Prevent reflux blood flow during right and left.
ventricular diastole. B. Coronary veins: return blood from the myocardium
c. Valves open when ventricles contract and back to the right atrium via the coronary sinus.
close during ventricular diastole; valve
closure produces second heart sound (S2).
Vascular System
Conduction System The major function of the blood vessels is to supply
the tissues with blood, remove wastes, and carry
See Figure 4-6. unoxygenated blood back to the heart.
A. Sinoatrial (SA) node: the pacemaker of the heart;
initiates the cardiac impulse, which spreads across
the atria and into AV node.
Types of Blood Vessels
B. Atrioventricular (AV) node: delays the impulse A. Arteries: elastic-walled vessels that can stretch
from the atria while the ventricles fill. during systole and recoil during diastole; they
C. Bundle of His: arises from the AV node and carry blood away from the heart and distribute
conducts impulse to the bundle branch system. oxygenated blood throughout the body.
1. Right bundle branch: divided into anterior, B. Arterioles: small arteries that distribute blood to
lateral, and posterior; transmits impulses the capillaries and function in controlling systemic
down the right side of the interventricular vascular resistance and, therefore, arterial pressure.
septum toward the right ventricular C. Capillaries: the following exchanges occur in the
myocardium capillaries
2. Left bundle branch: divided into anterior and 1. Oxygen and carbon dioxide
posterior 2. Solutes between the blood and tissues
a. Anterior portion transmits impulses to the 3. Fluid volume transfer between the plasma and
anterior endocardial surface of the left interstitial spaces
ventricle. D. Venules: small veins that receive blood from the
b. Posterior portion transmits impulses over capillaries and function as collecting channels
the posterior and inferior endocardial between the capillaries and veins.
surfaces of the left ventricle. E. Veins: low-pressure vessels with thin walls and
D. Purkinje fibers: transmit impulses to the ventricles less muscle than arteries; most contain valves that
and provide for depolarization after ventricular prevent retrograde blood flow; they carry
contraction. deoxygenated blood back to the heart. When
E. Electrical activity of heart can be visualized by skeletal muscles surrounding veins contract, the
attaching electrodes to the skin and recording veins are compressed, promoting movement of

4
activity by electrocardiograph. blood back to the heart.

ADULT NURSING 237


53155_04_Ch04a_p143-263.qxd 2/26/09 6:50 PM Page 238

ASSESSMENT B. Peripheral pulses: palpate and rate all arterial


pulses (temporal, carotid, brachial, radial, femoral,
popliteal, dorsalis pedis, and posterior tibial) on
Health History scale of: 0 5 absent, 1 5 palpable, 2 5 normal,
A. Presenting problem 3 5 full, 4 5 full and bounding.
1. Nonspecific symptoms may include fatigue, C. Assess for arterial insufficiency and venous
shortness of breath, cough, dizziness, syncope, impairment.
headache, palpitations, weight loss/gain, D. Measure and record blood pressure.
anorexia, difficulty sleeping. E. Inspect and palpate the neck vessels.
2. Specific signs and symptoms 1. Jugular veins: note location, characteristics;
a. Chest pain: note character, quality, measure jugular venous pressure.
location, radiation, frequency, and whether 2. Carotid arteries: note location, characteristics
it is associated with precipitating factors F. Precordium
(exertion, eating, excitement). 1. Inspect and palpate sternoclavicular, aortic,
b. Dyspnea (shortness of breath): note kind pulmonic, Erbs point, tricuspid, apical,
and extent of precipitating activities. epigastric sites.
c. Orthopnea (form of dyspnea that develops 2. Note point of maximum impulse (PMI),
when client lies down): determine how pulsations, thrills.
many pillows are used when sleeping; note G. Auscultate aortic, pulmonic, Erbs point, tricuspid,
any paroxysmal nocturnal dyspnea (PND) mitral or apical, xiphoid areas; note heart rate and
(client awakens suddenly in the night, rhythm (see Figure 4-8).
breathing with difficulty). 1. Normal heart sounds (S1 and S2): note location,
d. Palpitations (awareness of heartbeat, intensity, splitting
fluttering feeling): assess precipitating 2. Abnormal heart sounds (S3, S4): note location,
factors (anxiety, caffeine, nicotine, stress); occurrence in cardiac cycle
ask client to tap out the rhythm. 3. Murmurs: note location, occurrence in cardiac
e. Edema (abnormal accumulation of fluid cycle
in tissues): note whether unilateral/ 4. Friction rubs
bilateral, location, time of day when most
apparent. Laboratory/Diagnostic Tests
f. Cyanosis (dusky, bluish coloration to the
skin): note whether peripheral or central. A. Blood chemistry and electrolyte analysis
B. Lifestyle: occupation, hobbies, financial status, 1. Serial cardiac enzymes (protein assays) will be
stressors, unusual life patterns, relaxation time, evaluated with symptoms of acute coronary
exercise, living conditions, smoking, sleep habits
C. Use of medications: OTC drugs, cardiac drugs, oral
contraceptives, or estrogen replacement therapy
D. Personality profile: Type A, manic-depressive,
anxieties
E. Nutrition: dietary habits; calorie, cholesterol, salt
intake; alcohol consumption
F. Past medical history
1. Heart murmurs, rheumatic fever, sexually
transmitted diseases, angina, myocardial
infarction (MI), hypertension, CVA,
alcoholism, obesity, hyperlipidemia, varicose
veins, claudication
2. Pregnancies, contraceptive use
G. Family history: heart disease (congenital, acute,
chronic); risk factors (diabetes, hypertension,
obesity)

Physical Examination
A. Skin and mucous membranes: note color/texture,
temperature, hair distribution on extremities,
atrophy or edema, venous pattern, petechiae, Figure 4-8 Heart valves and areas of auscultation:
lesions, ulcerations or gangrene; examine (1) aortic area; (2) pulmonic area; (3) Erbs point;
nails. (4) tricuspid area; (5) mitral area

4 238 NCLEX-RN Review


53155_04_Ch04a_p143-263.qxd 2/26/09 6:50 PM Page 239

syndrome, chest pain/ischemia with and E. Stress tests may show heart disease when resting
without infection, congestive heart failure, ECG does not. Stress test types:
post cardiac surgical intervention, and post 1. Exercise: treadmill or bicycle
chest trauma. 2. Chemical: Persantine, Dobutamine, Adenosine
a. Nonspecific enzymes, elevated in F. Cardiac nuclear scan: Radionucleotide imaging to
myocardial injury and with other systems: identify ischemic/infracted tissue.
1) Creatine kinase (CK); normally G. Phonocardiogram: noninvasive device to amplify
50325 million units/mL and record heart sounds and murmurs.
2) Myoglobin H. Echocardiogram: noninvasive recording of the
3) LDH cardiac structures using ultrasound.
4) AST (SGOT); normally 740 units/mL I. Cardiac catheterization: invasive, but often
b. Specific cardiac isoenzymes, elevated in definitive test for diagnosis of cardiac disease.
myocardial injury: 1. A catheter is inserted into the right or left side
1) Creatine kinase-MB (CKMB); normally 0% of the heart to obtain information.
2) LDH1 and LDH2 a. Right-sided catheterization: the catheter is
3) Troponin I or cardiac troponin inserted into an antecubital vein and
T (currently used in place of LDH advanced into the vena cava, right atrium,
isoenzymes) and right ventricle with further insertion
c. Specific enzymes, elevations correlated into the pulmonary artery.
with vascular inflammation, irritability of b. Left-sided catheterization: performed by
atherosclerotic plaque, and future coronary inserting the catheter into a brachial or
risk: femoral artery; the catheter is passed
1) Ischemic modified albumin (IMA) retrograde up the aorta and into the left
2) Serum lipids (HDLs, LDLs, VDRLs) ventricle.
3) C-reactive protein (CRP) 2. Purpose: to measure intracardiac pressures
4) Lipoprotein phospholipase A2 (PLAQ and oxygen levels in various parts of the heart;
test) with injection of a dye, it allows visualization
d. Specific cardiac proteins, elevated in of the heart chambers, blood vessels, and
congestive heart failure: course of blood flow (angiography).
1) B-type natriuretic peptide and 3. Nursing care: pretest
N-terminal proB-type natriuretic a. Confirm that informed consent has been
peptide (BNP) signed.
B. Hematologic studies b. Ask about allergies, particularly to iodine,
1. CBC (see Hematologic system for values) if dye being used.
2. Coagulation time: 515 min.; increased levels c. Keep client NPO for 8&ndash;12 hours
indicate bleeding tendency, used to monitor prior to test.
heparin therapy d. Temporarily suspend metformin for dye
3. Prothrombin time (PT) 9.512 sec.; INR 1.0, and surgical procedures; do not restart
increased levels indicate bleeding tendency, until oral intake has resumed and renal
used to monitor warfarin therapy function is normal.
4. Activated partial thromboplastin time (APTT) e. Record height and weight, take baseline
2045 sec., increased levels indicate bleeding vital signs, and monitor peripheral pulses.
tendency, used to monitor heparin therapy f. Inform client that a feeling of warmth and
5. Erythrocyte sedimentation rate (ESR) < 20 fluttering sensation as catheter is passed is
mm/hr; increased level indicate inflammatory common.
process 4. Nursing care: posttest
C. Urine studies: routine urinalysis a. Assess circulation to the extremity used for
D. Electrocardiogram (ECG or EKG) catheter insertion.
1. Noninvasive test that produces a graphic b. Check peripheral pulses, color, sensation
record of the electrical activity of the heart. In of affected extremity every 15 minutes for
addition to determining cardiac rhythm, 4 hours.
pattern variations may reveal pathologic c. If protocol requires, keep affected
processes (MI and ischemia, electrolyte and extremity straight for approximately 8
acid-base imbalance, chamber enlargement, hours.
block of the right or left bundle branch); see d. Observe catheter insertion site for swelling
also Cardiac Monitoring. and bleeding; a sandbag or pressure
2. Portable recorder (Holter monitor) provides dressing may be placed over insertion
continuous recording of ECG for up to 24 site.
hours; client keeps a diary noting presence of e. Assess vital signs and report significant

4
symptoms or any unusual activities. changes from baseline.

ADULT NURSING 239


53155_04_Ch04a_p143-263.qxd 2/26/09 6:50 PM Page 240

J. Aortography F. Pain in the chest or in the affected extremity will


1. Injection of radiopaque contrast medium into be diminished.
the aorta to visualize the aorta, valve leaflets, G. Client will use effective coping techniques.
and major vessels on a movie film. H. Clients level of fear and anxiety will be
2. Purpose: to determine and diagnose aortic decreased.
valve incompetence, aneurysms of the
ascending aorta, abnormalities of major
branches of the aorta.
Interventions
3. Nursing care: pretest Cardiac Monitoring
a. Confirm that informed consent has been
signed. A. The cardiac monitor provides continuous
b. Inform client that a dye will be injected information regarding the cardiac rhythm and
and to report any dyspnea, numbness, or rate (ECG). Constant surveillance and
tingling. understanding of the basic electrocardiographic
4. Nursing care: posttest system is imperative to avoid/treat arrhythmias
a. Assess the puncture site frequently for (see Figure 4-9).
bleeding or inflammation. 1. ECG strip: each small square represents
b. Assess peripheral pulses distal to the 0.04 second, each large square 0.2 second.
injection site every hour for 4&ndash;8 2. P wave: produced by atrial depolarization;
hours posttest. indicates SA node function.
K. Coronary arteriography 3. P-R interval
1. Visualization of coronary arteries by injection a. Indicates atrioventricular conduction time
of radiopaque contrast dye and recording on a or the time it takes an impulse to travel
movie film. from the atria down and through the
2. Purpose: evaluation of heart disease and AV node
angina, location of areas of infarction b. Measured from beginning of P wave to
and extent of lesions, ruling out coronary beginning of QRS complex
artery disease in clients with myocardial c. Normal: 0.120.20 second.
disease. 4. QRS complex
3. Nursing care: same as for Aortography. a. Indicates ventricular depolarization
b. Measured from onset of Q wave to end of
S wave
ANALYSIS c. Normal: 0.060.10 seconds
5. ST segment
Nursing diagnoses for the client with a cardiovascular a. Indicates time interval between complete
dysfunction may include: depolarization of ventricles and
A. Excess fluid volume repolarization of ventricles
B. Decreased cardiac output b. Measured after QRS complex to beginning
C. Ineffective tissue perfusion of T wave
D. Impairment of skin integrity 6. T wave
E. Activity intolerance a. Represents ventricular repolarization
F. Pain b. Follows ST segment
G. Ineffective individual coping
H. Fear
I. Anxiety

PLANNING AND
IMPLEMENTATION
Goals
A. Fluid imbalance will be resolved, edema line
minimized.
B. Cardiac output will be improved.
C. Cardiopulmonary and peripheral tissue perfusion Figure 4-9 A typical ECG; all beats appear as a similar
will be improved.
pattern, equally spaced, and have three major units:
D. Adequate skin integrity will be maintained.
P wave, QRS complex, and T wave

4
E. Activity tolerance will progressively increase.

240 NCLEX-RN Review


53155_04_Ch04a_p143-263.qxd 2/26/09 6:50 PM Page 241

Hemodynamic Monitoring 3. Provides an IV route for drawing blood


(Swan-Ganz Catheter) samples, administering fluids or medication,
and possibly inserting a pacing catheter
A. A pulmonary artery (PA) catheter with a C. Normal range is 410 cm H2O or 26 mm Hg;
balloon tip that is advanced through the elevation indicates hypervolemia, decreased level
superior vena cava into the right atrium, right indicates hypovolemia.
ventricle, and pulmonary artery. When it is D. Nursing care
wedged it is in the distal arterial branch of the 1. Ensure client is relaxed.
pulmonary artery. 2. Maintain zero point of manometer always at
B. Purposes level of right atrium (midaxillary line).
1. Proximal port: measures right atrial 3. Determine patency of catheter by opening IV
pressure infusion line.
2. Distal port 4. Turn stopcock to allow IV solution to run into
a. Measures pulmonary artery (PA) pressure manometer to a level of 1020 cm above
(reflects left and right heart pressures) and expected pressure reading.
pulmonary capillary wedge pressure 5. Turn stopcock to allow IV solution to flow
(PCWP) (reflects left atrial and left from manometer into catheter; fluid level in
ventricular end diastolic pressure) manometer fluctuates with respiration.
b. Normal values: PA systolic 1530 mm Hg 6. Stop ventilatory assistance during
and diastolic 412 mm Hg; PCWP measurement of CVP.
612 mm Hg 7. After CVP reading, return stopcock to IV
3. Balloon port: inflated with 11.5 cc air to infusion position.
obtain PCWP 8. Record CVP reading and position of client.
4. Thermistor lumen: used to measure cardiac 9. Electronic transducer
output if ordered. a. Ensure client is relaxed.
C. Nursing care b. Make sure transducer is zeroed and
1. A sterile dry dressing should be applied to site calibrated.
and changed every 24 hours; inspect site daily c. If CVP line is in use, turn stopcock to allow
and report signs of infection. pressure measurement from transducer,
2. If catheter is inserted via an extremity, which will temporarily stop infusion.
immobilize extremity to prevent catheter d. Ensure that infusion is reinitiated upon
dislodgment or trauma. completion of reading.
3. Observe catheter site for leakage. e. Record CVP.
4. Ensure that balloon is deflated with a
syringe attached, except when PCWP is
read. EVALUATION
5. Continuously monitor PA systolic and
diastolic pressures and report significant A. Resolution of peripheral edema and neck vein
variations. distention; weight stable; lungs clear.
6. Maintain client in same position for each B. Capillary refill is less than 3 seconds; balanced
reading. I&O with urine output at least 30 mL/hour;
7. Maintain pressure bag at 300 mm Hg. hemodynamic measurements within normal range;
8. Record PA systolic and diastolic readings at usual mental status.
least every hour and PCWP as ordered, noting C. Stable vital signs; skin warm and dry; peripheral
position of client. pulses present, equal, and strong; absence of
edema; increased tolerance to activity; usual
Central Venous Pressure (CVP) mentation; absence of pain.
D. Clients skin warm and dry, shows absence of
A. Obtained by inserting a catheter into the external redness and irritation; healing of lesions.
jugular, antecubital, or femoral vein and threading E. Progressive increase in tolerance for activity with
it into the vena cava. The catheter is attached to an heart rate and blood pressure stable; absence of
IV infusion and H2O manometer by a three-way pain.
stopcock or electronic transducer. F. Client expresses relief from pain; relaxed facial
B. Purposes expression; stable vital signs; progressive increase
1. Reveals right atrial pressure, reflecting in activity tolerance.
alterations in the right ventricular G. Demonstrates the use of effective coping skills and
pressure problem-solving techniques.
2. Provides information concerning blood H. Verbalizes awareness of feelings of fear and
volume and adequacy of central venous anxiety. Client reports fear/anxiety as

4
return reduced/controlled.

ADULT NURSING 241


53155_04_Ch04a_p143-263.qxd 2/26/09 6:50 PM Page 242

DISORDERS OF THE C. Assessment findings


1. Pain: substernal with possible radiation to the
CARDIOVASCULAR SYSTEM neck, jaw, back, and arms; may be relieved by
rest
The Heart 2. Palpitations, tachycardia
3. Dyspnea
Coronary Artery Disease (CAD) 4. Diaphoresis
5. Increased serum lipid levels
A. General information 6. Diagnostic tests
1. CAD refers to a variety of pathologic a. ECG may reveal ST segment depression
conditions that cause narrowing or obstruction and T-wave inversion during chest pain.
of the coronary arteries, resulting in decreased b. Stress test may reveal an abnormal ECG
blood supply to the myocardium. during exercise.
2. Atherosclerosis (deposits of cholesterol and D. Nursing interventions
lipids within the walls of the artery) is the 1. Administer oxygen.
major causative factor. 2. Give prompt pain relief with nitrates or
3. Occurs most often between ages 30 and 50; narcotic analgesics as ordered.
men affected more often than women; 3. Monitor vital signs, status of cardiopulmonary
nonwhites have higher mortality rates. function.
4. May manifest as angina pectoris or MI. 4. Monitor ECG.
5. Risk factors: family history of CAD, elevated 5. Place client in semi- to high-Fowlers position.
serum lipoproteins, cigarette smoking, 6. Provide emotional support.
diabetes mellitus, hypertension, obesity, 7. Provide client teaching and discharge
sedentary and/or stressful/competitive planning concerning
lifestyle, elevated serum uric acid levels. a. Proper use of nitrates
B. Medical management, assessment findings, and 1) Nitroglycerin tablets (sublingual)
nursing interventions: see Angina Pectoris and a) Allow tablet to dissolve.
Myocardial Infarction. b) Relax for 15 minutes after taking
tablet to prevent dizziness.
Angina Pectoris c) If no relief with 1 tablet, take
additional tablets at 5-minute
A. General information
intervals, but no more than
1. Transient, paroxysmal chest pain produced by
3 tablets within a 15-minute
insufficient blood flow to the myocardium
period, reassess blood pressure
resulting in myocardial ischemia
after each tablet dissolved.
2. Risk factors: CAD, atherosclerosis,
d) Know that transient headache is a
hypertension, diabetes mellitus,
frequent side effect.
thromboangiitis obliterans, severe anemia,
e) Keep pills in original bottle, tightly
aortic insufficiency
capped and prevent exposure to
3. Precipitating factors: physical exertion,
air, light, and heat.
consumption of a heavy meal, extremely cold
f) Ensure tablets are within reach at
weather, strong emotions, cigarette smoking,
all times.
sexual activity
g) Check shelf life, expiration date of
4. Two main types:
tablets.
a. Stable angina: chest pain occurs with
2) Nitroglycerin ointment (topical)
increased oxygen demand; relieved when
a) Rotate sites to prevent dermal
precipitating factor is removed or with
inflammation.
nitroglycerin.
b) Remove previously applied ointment.
b. Unstable angina chest pain increases in
c) Avoid massaging/rubbing as this
frequency, duration, and intensity at low
increases absorption and interferes
levels of activity or at rest; often a
with the drugs sustained action.
precursor to MI.
b. Ways to minimize precipitating events
B. Medical management
1) Reduce stress and anxiety (relaxation
1. Drug therapy: nitrates, beta-adrenergic blocking
techniques, guided imagery)
agents, and/or calcium-blocking agents, lipid
2) Avoid overexertion and smoking
reducing drugs if cholesterol elevated
3) Maintain low-cholesterol, low-
2. Modification of diet and other risk factors
saturated fat diet and eat small,
3. Surgery: coronary artery bypass surgery
frequent meals
4. Percutaneous transluminal coronary
4) Avoid extremes of temperature
angioplasty (PTCA)

4
5) Dress warmly in cold weather

242 NCLEX-RN Review


53155_04_Ch04a_p143-263.qxd 2/26/09 6:50 PM Page 243

c. Gradual increase in activities and exercise meningitis, myxedema, cardiac fibrosis; a


1) Participate in regular exercise program normal variation of the heart rate in well-
2) Space exercise periods and allow for trained athletes
rest periods B. Assessment findings
8. Instruct client to notify physician immediately 1. Rate: less than 60 beats/minute
if pain occurs and persists, despite rest and 2. Rhythm: regular
medication administration. 3. P wave: precedes each QRS with a normal
contour
Dysrhythmias 4. P-R interval: normal
5. QRS complex: normal
A dysrhythmia, often called an arrhythmia, is a C. Treatment: usually not needed; if cardiac output is
disruption in the normal events of the cardiac cycle. inadequate, atropine and isoproterenol (Isuprel)
It may take a variety of forms. Treatment varies are usually prescribed; if drugs are not effective, a
depending on the type of dysrhythmia; commonly pacemaker may need to be inserted.
used drugs are summarized in Table 2-14.
Atrial Tachycardia
Sinus Tachycardia
A. General information
A. General information 1. A heart rate above 160250, originates in the
1. A heart rate of over 100 beats/minute, SA node
originating in the SA node 2. May be drug induced (including substance
2. May be caused by fever, apprehension, abuse), caused by fever, severe blood loss,
physical activity, anemia, hyperthyroidism, thyroid storm, electrolyte imbalances, severe
drugs (epinephrine, theophylline), myocardial hypoxia
ischemia, caffeine B. Assessment findings
B. Assessment findings 1. Rate: 160250 beats/minute
1. Rate: 100160 beats/minute 2. Rhythm: regular
2. Rhythm: regular 3. P Wave: precedes each QRS complex with
3. P wave: precedes each QRS complex with normal contour
normal contour 4. P-R interval: normal (0.08 second)
4. P-R interval: normal (0.08 second) 5. QRS complex: normal (0.06 second)
5. QRS complex: normal (0.06 second) C. Treatment: correction of underlying problem, beta-
C. Treatment: correction of underlying cause, blockers, calcium channel blocker, amniodarone
elimination of stimulants; sedatives, propranolol
(Inderal). Atrial Flutter
A. General information
Premature Atrial Complex (PAC)
1. Atrial rate between 250 and 400, ventricular
A. General information rate between 75 and 150
1. Physical appearance: single ECG complex that 2. May be idiopathic, associated with advanced
occurs early age, valvular disease, HTN, cardiomyopathy,
2. Causes: nicotine, alcohol, anxiety, low pulmonary disease, hyperthyroidism,
potassium level, hypovolemia, myocardial moderate-to-heavy alcohol consumption
ischemia B. Assessment findings
B. Assessment findings 1. Rate: 250400 beats/minute
1. Ventricular and atrial rate dependent on 2. Rhythm: irregular
underlying rhythm 3. P Wave: varies to QRS
2. Rhythm; irregular due to premature complexes 4. P-R interval: difficult to distinguish due to rate
3. QRS shape; usually normal 5. QRS complex: normal or abnormal
4. P wave: morphology may be the same, C. Treatment: correction of underlying problem, beta-
different, or absent blockers, calcium channel blocker, amniodarone,
5. P-R interval: may be shorter but within limits digitalis
(0.120.20 second)
6. P-QRS: 1:1 Atrial Fibrillation
A. General information
Sinus Bradycardia
1. An arrhythmia in which ectopic foci in the
A. General information atria cause rapid, irregular contractions of the
1. A slowed heart rate initiated by SA node heart
2. Caused by excessive vagal or decreased 2. Commonly seen in clients with rheumatic
sympathetic tone, MI, intracranial tumors,

4
mitral stenosis, thyrotoxicosis,

ADULT NURSING 243


53155_04_Ch04a_p143-263.qxd 2/26/09 6:50 PM Page 244

cardiomyopathy, hypertensive heart disease, 2. Procainamide via IV infusion of


pericarditis, and coronary heart disease 26 mg/minute
B. Assessment findings 3. Direct-current cardioversion
1. Rate 4. Bretylium, propranolol (Inderal)
a. Atrial: 350600 beats/minute
b. Ventricular: varies between 100160 Ventricular Fibrillation
beats/minute
A. General information
2. Rhythm: atrial and ventricular regularly
1. Rapid and disorganized rhythm caused by
irregular
quivering of the ventricles
3. P wave: no definite P wave; rapid undulations
2. No atrial activity is seen
called fibrillatory (f) waves
3. May be caused by idiopathic sudden death,
4. P-R interval: not measurable
electrical shock
5. QRS complex: generally normal
B. Assessment findings
C. Treatment: digitalis preparations, propranolol,
1. Ventricular rate: greater than 300
verapamil in conjunction with digitalis; direct-
2. Ventricular rhythm irregular, without specific
current cardioversion
pattern
3. QRS shape and duration: irregular, undulating
Premature Ventricular Contractions (PVCs) waves without recognizable QRS pattern
A. General information C. Treatment: counter-shock (defibrillation)
1. Irritable impulses originate in the ventricles
2. Caused by electrolyte imbalance
(hypokalemia); digitalis drug therapy;
Myocardial Infarction (MI)
myocardial disease; stimulants (caffeine, A. General information
epinephrine, isoproterenol); hypoxia; 1. The death of myocardial cells from inadequate
congestive heart failure oxygenation, often caused by a sudden
B. Assessment findings complete blockage of a coronary artery;
1. Rate: varies according to number of PVCs characterized by localized formation of
2. Rhythm: irregular because of PVCs necrosis (tissue destruction) with subsequent
3. P wave: normal; however, often lost in QRS healing by scar formation and fibrosis.
complex 2. Risk factors: atherosclerotic CAD, thrombus
4. P-R interval: often not measurable formation, hypertension, diabetes mellitus,
5. QRS complex: wide and distorted in shape, hyperlipidemia, and genetic predisposition
greater than 0.12 second B. Assessment findings (see also Angina Pectoris)
C. Treatment 1. Pain usually substernal with radiation to the
1. IV push of lidocaine (50100 mg) followed by neck, arm, jaw, or back; severe, crushing,
IV drip of lidocaine at rate of 14 mg/minute viselike with sudden onset; unrelieved by rest
2. Procainamide (Pronestyl), quinidine or nitrates
3. Treatment of underlying cause 2. Nausea and vomiting
3. Dyspnea
Ventricular Tachycardia 4. Skin: cool, clammy, ashen
5. Elevated temperature
A. General information
6. Initial increase in blood pressure and pulse,
1. A run of three or more consecutive PVCs;
with gradual drop in blood pressure
occurs from repetitive firing of an ectopic
7. Restlessness
focus in the ventricles
8. Occasional findings: rales or crackles;
2. Caused by acute MI, CAD, digitalis
presence of S4; pericardial friction rub; split
intoxication, hypokalemia
S1, S2
B. Assessment findings
9. Diagnostic tests
1. Rate
a. Elevated WBC
a. Atrial: 60100 beats/minute
b. Elevated CPK and CPK-MB
b. Ventricular: 110250 beats/minute
c. Elevated SGOT or AST
2. Rhythm: atrial (regular), ventricular
d. Elevated LDH, LDH1, and LDH2
(occasionally irregular)
e. Elevated troponin levels
3. P wave: often lost in QRS complex
f. ECG changes (specific changes dependent
4. P-R interval: usually not measurable
on location of myocardial damage and
5. QRS complex: greater than 0.12 second, wide
phase of the MI; inverted T wave and ST
C. Treatment
segment changes seen with myocardial
1. IV push of lidocaine (1 mg/kg for a dose
ischemia
of 50100 mg), then IV drip of lidocaine
g. Increased ESR, elevated serum cholesterol

4
14 mg/minute

244 NCLEX-RN Review


53155_04_Ch04a_p143-263.qxd 2/26/09 6:50 PM Page 245

C. Nursing interventions 2. The aim of PTCA is to revascularize the


1. Establish a patent IV line myocardium, decrease angina, and increase
2. Provide pain relief; morphine sulfate IV (given survival.
IV because after an infarction there is poor 3. PTCA is performed in the cardiac
peripheral perfusion and because serum catheterization lab and is accomplished by
enzymes would be affected by IM injections) insertion of a balloon-tipped catheter into the
as ordered. stenotic, diseased coronary artery. The balloon
3. Administer oxygen as ordered to relieve is inflated with a controlled pressure and
dyspnea and prevent arrhythmias. thereby decreases the stenosis of the vessel.
4. Provide bed rest with semi-Fowlers position B. Nursing interventions: preoperative and
to decrease cardiac workload. postoperative care is similar to the care of the
5. Monitor ECG and hemodynamic procedures. client undergoing cardiac catheterization.
6. Administer antiarrhythmias as ordered.
7. Perform complete lung/cardiovascular Coronary Artery Bypass Surgery (CABG)
assessment.
8. Monitor urinary output and report output of A. General information
less than 30 mL/hour; indicates decreased 1. A coronary artery bypass graft is the surgery of
cardiac output. choice for clients with severe CAD.
9. Maintain full liquid diet with gradual increase 2. New supply of blood brought to
to soft; low sodium. diseased/occluded coronary artery by
10. Maintain quiet environment. bypassing the obstruction with a graft that is
11. Administer stool softeners as ordered to attached to the aorta proximally and to the
facilitate bowel evacuation and prevent coronary artery distally.
straining. 3. Several bypasses can be performed depending
12. Relieve anxiety associated with coronary care on the location and extent of the blockage.
unit (CCU) environment. 4. Procedure frequently requires use of
13. Administer anticoagulants, as ordered. extracorporeal circulation (heart-lung
14. Administer thrombolytics (tissue-type machine, cardiopulmonary bypass). Some
plasminogen activator or t-pa and streptokinase) clients may be candidates for off pump
and monitor for side effects (bleeding). coronary artery bypass (OPCAB).
15. Provide client teaching and discharge B. Nursing interventions: preoperative
planning concerning 1. Explain anatomy of the heart, function of
a. Effects of MI, healing process, and coronary arteries, effects of CAD.
treatment regimen 2. Explain events of the day of surgery: length of
b. Medication regimen including name, time in surgery, length of time until able to see
purpose, schedule, dosage, side effects family.
c. Risk factors, with necessary lifestyle 3. Orient to the critical and coronary care units
modifications and introduce to staff.
d. Dietary restrictions: low sodium, low 4. Explain equipment to be used (monitors,
cholesterol, avoidance of caffeine hemodynamic procedures, ventilator,
e. Importance of participation in a endotracheal tube, drainage tubes).
progressive activity program 5. Demonstrate activity and exercises (turning
f. Resumption of sexual activity according to from side to side, dangling, sitting in a chair,
physicians orders (usually 46 weeks) ROM exercises for arms and legs, effective
g. Need to report the following symptoms: deep breathing, and coughing).
increased persistent chest pain, dyspnea, 6. Reassure client that pain medication is
weakness, fatigue, persistent palpitations, available.
light-headedness C. Nursing interventions: postoperative
h. Enrollment of client in a cardiac 1. Maintain patent airway.
rehabilitation program 2. Promote lung reexpansion.
a. Monitor drainage from chest/mediastinal
Percutaneous Transluminal tubes, and check patency of chest drainage
system.
Coronary Angioplasty
b. Assist client with turning, coughing, and
A. General information deep breathing.
1. Percutaneous transluminal coronary 3. Monitor cardiac status.
angioplasty (PTCA), with or without a. Monitor vital signs and cardiac rhythm and
placement of a stent, can be performed as an report significant changes, particularly
alternative to coronary artery bypass graft temperature elevation.

4
surgery (CABG). b. Perform peripheral pulse checks.

ADULT NURSING 245


53155_04_Ch04a_p143-263.qxd 2/26/09 6:50 PM Page 246

c. Carry out hemodynamic monitoring. e. Wound cleansing daily with mild soap and
d. Administer anticoagulants as ordered H2O and report signs of infection
and monitor hematologic test results f. Symptoms to be reported: fever, dyspnea,
carefully. chest pain with minimal exertion
4. Maintain fluid and electrolyte balance.
a. Maintain accurate I&O with hourly Heart Failure (HF)
outputs; report if less than 30 mL/hour
urine. A. General information: inability of the heart to pump
b. Assess color, character, and specific gravity an adequate supply of blood to meet the metabolic
of urine. needs of the body.
c. Daily weights. B. Types
d. Assess lab values, particularly BUN, 1. Left-sided heart failure
creatinine, sodium, and potassium levels. a. Left ventricular damage causes blood to
5. Maintain adequate cerebral circulation: back up through the left atrium and into
frequent neuro checks. the pulmonary veins. Increased pressure
6. Provide pain relief. causes transudation into the interstitial
a. Administer narcotics cautiously and tissues of the lungs with resultant
monitor effects. pulmonary congestion.
b. Assist with positioning for maximum b. Caused by left ventricular damage (usually
comfort. due to an MI), hypertension, ischemic
c. Teach relaxation techniques. heart disease, aortic valve disease, mitral
7. Prevent abdominal distension. stenosis
a. Monitor nasogastric drainage and maintain c. Assessment findings
patency of system. 1) Dyspnea, orthopnea, PND, tiredness,
b. Assess for bowel sounds every 24 hours. muscle weakness, cough
c. Measure abdominal girths if necessary. 2) Tachycardia, PMI displaced laterally,
8. Monitor for and prevent the following possible S3, bronchial wheezing, rales
complications. or crackles, cyanosis, pallor
a. Thrombophlebitis/pulmonary embolism 3) Decreased pO2, increased pCO2
b. Cardiac tamponade 4) Diagnostic tests
c. Arrhythmias a) Chest X-ray: shows cardiac
1) Maintain continuous ECG monitoring hypertrophy
and report changes. b) PAP and PCWP usually increased;
2) Assess electrolyte levels daily and however, this is dependent on the
report significant changes, particularly degree of heart failure
potassium. 5) Echocardiography: shows increased
3) Administer antiarrhythmics as size of cardiac chambers
ordered. 2. Right-sided heart failure
d. Heart failure a. Weakened right ventricle is unable to
9. Provide client teaching and discharge pump blood into the pulmonary system;
planning concerning systemic venous congestion occurs as
a. Limitation with progressive increase in pressure builds up.
activities b. Caused by left-sided heart failure, right
1) Encourage daily walking with gradual ventricular infarction, atherosclerotic heart
increase in distance weekly disease, COPD, pulmonic stenosis,
2) Avoid heavy lifting and activities that pulmonary embolism.
require continuous arm movements c. Assessment findings
(vacuuming, playing golf, bowling) 1) Anorexia, nausea, weight gain
3) Avoid driving a car until physician 2) Dependent pitting edema, jugular
permits venous distension, bounding pulses,
b. Sexual intercourse: can usually be resumed hepatomegaly, cool extremities,
by third or fourth week post-op; avoid oliguria
sexual positions in which the client would 3) Elevated CVP, decreased pO2,
be supporting weight increased ALT (SGPT)
c. Medical regimen: ensure client/family are 4) Diagnostic tests
aware of drugs, dosages, proper times of a) Chest X-ray: reveals cardiac
administration, and side effects hypertrophy
d. Meal planning with prescribed b) Echocardiography: indicates
modifications (decreased sodium, increased size of cardiac

4
cholesterol, and possibly carbohydrates) chambers

246 NCLEX-RN Review


53155_04_Ch04a_p143-263.qxd 2/26/09 6:50 PM Page 247

3. High-output failure c. Prescribed dietary plan (low sodium;


a. Cardiac output is adequate but exceeded small, frequent meals)
by the metabolic needs of the tissues; d. Need to avoid fatigue and plan for rest
the exorbitant demands made on the periods
heart eventually cause ventricular
failure. Pulmonary Edema
b. Caused by hyperthyroidism, anemia,
AV fistula, pregnancy A. General information
C. Medical management (all types) 1. A medical emergency that occurs when the
1. Determination and elimination/control of capillary pressure within the lungs becomes so
underlying cause great that fluid moves from the intravascular
2. Drug therapy: digitalis preparations, diuretics, space into the alveoli, bronchi, and
vasodilators bronchioles. Death occurs by suffocation if this
3. Sodium-restricted diet to decrease fluid condition is untreated.
retention 2. Caused by left-sided heart failure, rapid
4. If medical therapies unsuccessful, mechanical administration of IV fluids.
assist devices (intra-aortic balloon pump), B. Medical management
cardiac transplantation, or mechanical hearts 1. Oxygen therapy
may be employed. 2. Endotracheal/nasotracheal intubation
D. Nursing interventions (possible)
1. Monitor respiratory status and provide 3. Drug therapy
adequate ventilation (when HF progresses to a. Morphine sulfate to induce vasodilation
pulmonary edema). and decrease anxiety; 5 mg IV, administer
a. Administer oxygen therapy. slowly
b. Maintain client in semi- or high-Fowlers b. Digitalis to improve cardiac output
position. c. Diuretics (furosemide [Lasix] is drug of
c. Monitor ABGs. choice) to relieve fluid retention
d. Assess for breath sounds, noting any d. Aminophylline to relieve bronchospasm
changes. and increase cardiac output; 250500 mg
2. Provide physical and emotional rest. IV, administer slowly
a. Constantly assess level of anxiety. e. Vasodilators (nitroglycerin, isosorbide
b. Maintain bed rest with limited activity. dinitrate) to dilate the vessels, thereby
c. Maintain quiet, relaxed environment. reducing amount of blood returned to the
d. Organize nursing care around rest periods. heart
3. Increase cardiac output. 4. Rotating tourniquets or phlebotomy
a. Administer digitalis as ordered and C. Assessment findings
monitor effects. 1. Dyspnea
b. Monitor ECG and hemodynamic 2. Cough with large amounts of blood-tinged
monitoring. sputum
c. Administer vasodilators as ordered. 3. Tachycardia, pallor, wheezing, rales or
d. Monitor vital signs. crackles, diaphoresis
4. Reduce/eliminate edema. 4. Restlessness, fear/anxiety
a. Administer diuretics as ordered. 5. Jugular vein distension
b. Daily weights. 6. Decreased pO2, increased pCO2, elevated CVP
c. Maintain accurate I&O. D. Nursing interventions
d. Assess for peripheral edema. 1. Assist with intubation (if necessary) and
e. Measure abdominal girths daily. monitor mechanical ventilation.
f. Monitor electrolyte levels. 2. Administer oxygen by mask in high
g. Monitor CVP and Swan-Ganz readings. concentrations (4060%) if not intubated.
h. Provide sodium-restricted diet as ordered. 3. Place client in semi-Fowlers position or over
i. Provide meticulous skin care. bedside table to ease dyspnea.
5. Provide client teaching and discharge 4. Administer medications as ordered.
planning concerning 5. Assist with phlebotomy (removal of
a. Need to monitor self daily for signs and 300500 mL of blood from a peripheral vein)
symptoms of HF (pedal edema, weight gain if performed.
of 12 kg in a 2-day period, dyspnea, loss 6. CVP/hemodynamic monitoring.
of appetite, cough) 7. Provide client teaching and discharge
b. Medication regimen including name, planning concerning
purpose, dosage, frequency, and side a. Prescribed medications, including name,

4
effects (digitalis, diuretics) purpose, schedule, dosage, and side effects

ADULT NURSING 247


53155_04_Ch04a_p143-263.qxd 2/26/09 6:50 PM Page 248

b. Dietary restrictions: low sodium, low b. Observe the presence of pacemaker spikes on
cholesterol ECG tracing or cardiac monitor; spike before
c. Importance of adhering to planned rest P wave with atrial pacemaker; spike before
periods with gradual progressive increase QRS complex with ventricular pacemaker
in activities c. Assess for signs of pacemaker malfunction,
d. Daily weights such as weakness, fainting, dizziness, or
e. Need to report the following symptoms to hypotension.
physician immediately: dyspnea, persistent 2. Maintain the integrity of the system
productive cough, pedal edema, restlessness a. Ensure that catheter terminals are attached
securely to the pulse generator (temporary
Pacemakers pacemaker)
b. Attach pulse generator to client securely to
A. General information prevent accidental dislodgment (temporary
1. A pacemaker is an electronic device that pacemaker)
provides repetitive electrical stimulation to the 3. Provide safety and comfort
cardiac musculature to control the heart rate. a. Provide safe environment by properly
2. Artificial pacing system consists of a battery- grounding all equipment in the room.
powered generator and a pacing wire that b. Monitor electrolyte level periodically,
delivers the stimulus to the heart. particularly potassium.
B. Indications for use 4. Prevent infection
1. Adams-Stokes attack a. Assess vital signs, particularly temperature
2. Acute MI with Mobitz II AV block changes.
3. Third-degree AV block with slow ventricular rate b. Assess catheter insertion site daily for
4. Right bundle branch block signs of infection.
5. New left bundle branch block c. Maintain sterile dressing over catheter
6. Symptomatic sinus bradycardia insertion site.
7. Sick sinus syndrome F. Provide client teaching and discharge planning
8. Arrhythmias (during or after cardiac surgery) concerning
9. Drug-resistant tachyarrhythmia 1. Fundamental concepts of cardiac physiology
C. Modes of pacing 2. Daily pulse check for 1 minute
1. Fixed rate: pacemaker fires electrical stimuli at 3. Need to report immediately any sudden
preset rate, regardless of the clients rate and slowing or increase in pulse rate
rhythm. 4. Importance of adhering to weekly monitoring
2. Demand: pacemaker produces electrical schedule during first month after implantation
stimuli only when the clients own heart rate and when battery depletion is anticipated
drops below the preset rate per minute on the (depending on type of battery)
generator. 5. Wear loose-fitting clothing around the area of
D. Types of pacemakers the pacemaker for comfort
1. Temporary 6. Notify physician of any pain or redness over
a. Used in emergency situations and incision site
performed via an endocardial 7. Avoid trauma to area of pulse generator
(transvenous) or transthoracic approach to 8. Avoid heavy contact sports
the myocardium. 9. Carry an identification card/bracelet that
b. Performed at bedside or using fluoroscopy. indicates physicians name, type and model
2. Permanent number of pacemaker, manufacturers name,
a. Endocardial or transvenous procedure pacemaker rate
involves passing endocardial lead into 10. Display identification card and request
right ventricle with subcutaneous scanning by hand scanner when going through
implantation of pulse generator into right weapons detector at airport
or left subclavian areas. Usually done 11. Remember that periodic hospitalization is
under local anesthesia. necessary for battery changes/pacemaker unit
b. Epicardial or myocardial method involves replacement
passing the electrode transthoracically to
the myocardium where it is sutured in Cardiac Arrest
place. The pulse generator is implanted
into the abdominal wall. A. General information: sudden, unexpected
E. Nursing interventions cessation of breathing and adequate circulation
1. Assess pacemaker function of blood by the heart
a. Monitor heart rate, noting deviations from B. Medical management

4
the preset rate. 1. Cardiopulmonary resuscitation (CPR)

248 NCLEX-RN Review


53155_04_Ch04a_p143-263.qxd 2/26/09 6:50 PM Page 249

2. Drug therapy 7. According to the American Heart Association


a. Lidocaine, procainamide, verapamil for in March 2008, new guidelines were issued
ventricular tachycardia that stressed the importance of chest
b. Dopamine (Intropin), isoproterenol compressions, especially if the bystander was
(Isuprel), norepinephrine (Levophed): see not trained to perform rescue breathing.
also Drugs Used to Treat Shock, Table 4-9
c. Epinephrine to enhance myocardial Endocarditis
automaticity, excitability, conductivity, and
contractility A. General information
d. Atropine sulfate to reduce vagus nerves 1. Inflammation of the endocardium; platelets
control over the heart, thus increasing the and fibrin deposit on the mitral and/or aortic
heart rate valves causing deformity, insufficiency, or
e. Sodium bicarbonate to correct respiratory stenosis.
and metabolic acidosis 2. Caused by bacterial infection: commonly
f. Calcium chloride: calcium ions help the S. aureus, S. viridans, B-hemolytic
heart beat more effectively by enhancing streptococcus, gonococcus
the myocardiums contractile force 3. Precipitating factors: rheumatic heart disease,
3. Defibrillation (electrical countershock) open-heart surgery procedures, GU/Ob-Gyn
C. Assessment findings: unresponsiveness, cessation instrumentation/surgery, dental extractions,
of respiration, pallor, cyanosis, absence of heart invasive monitoring, septic thrombophlebitis
sounds/blood pressure/palpable pulses, dilation of B. Medical management
pupils, ventricular fibrillation or asystole (if client 1. Drug therapy
on a monitor) a. Antibiotics specific to sensitivity of
D. Nursing interventions: monitored arrest caused by organism cultured
ventricular fibrillation b. Penicillin G and streptomycin if organism
1. CPR until defibrillation possible. not known
2. If defibrillation unsuccessful, continue CPR c. Antipyretics
and assist with administration of and monitor 2. Cardiac surgery to replace affected valve
effects of additional emergency drugs. C. Assessment findings
3. If defibrillation successful, monitor client 1. Fever, malaise, fatigue, dyspnea and cough
status. (if extensive valvular damage), acute upper
quadrant pain (if splenic involvement), joint
Cardiopulmonary Resuscitation (CPR) pain
2. Petechiae, murmurs, edema (if extensive
A. General information: process of externally valvular damage), splenomegaly, hemiplegia
supporting the circulation and respiration of a and confusion (if cerebral infarction),
person who has had a cardiac arrest hematuria (if renal infarction)
B. Nursing interventions: unwitnessed cardiac arrest 3. Elevated WBC and ESR, decreased Hgb and
1. Assess LOC. Hct
a. Shake victims shoulder and shout. 4. Diagnostic tests: positive blood culture for
b. If no response, summon help. causative organism
2. Position victim supine on a firm surface. D. Nursing interventions
3. Open airway. 1. Administer antibiotics as ordered to control
a. Use head tilt, chin lift maneuver. the infectious process.
b. Place ear over nose and mouth. 2. Control temperature elevation by
1) Look to see if chest is moving. administration of antipyretics.
2) Listen for escape of air. 3. Assess for vascular complications (see
3) Feel for movement of air against face. Thrombophlebitis and Pulmonary Embolism).
c. If no respiration, proceed to #4. 4. Provide client teaching and discharge
4. Ventilate twice, allowing for deflation between planning concerning
breaths. a. Types of procedures/treatments (e.g., tooth
5. Assess circulation: in adults palpate for extractions, GU instrumentation) that
carotid pulse; if not present, proceed to #6. increase the chances of recurrences
6. Initiate external cardiac compressions b. Antibiotic therapy, including name,
a. Proper placement of hands: lower half of purpose, dose, frequency, side effects
the sternum c. Signs and symptoms of recurrent
b. Depth of compressions: 1122 inches for endocarditis (persistent fever, fatigue,
adults chills, anorexia, joint pain)
c. 30 compressions (at rate of 80100 per d. Avoidance of individuals with known
minute) with 2 ventilations

4
infections

ADULT NURSING 249


53155_04_Ch04a_p143-263.qxd 2/26/09 6:50 PM Page 250

Pericarditis into the pericardial sac to aspirate fluid/blood and


relieve the pressure on the heart)
A. General information C. Assessment findings
1. An inflammation of the visceral and parietal 1. Chest pain
pericardium 2. Hypotension, distended neck veins,
2. Caused by a bacterial, viral, or fungal infection; tachycardia, muffled or distant heart sounds,
collagen diseases; trauma; acute MI; neoplasms; paradoxical pulse, pericardial friction rub
uremia; radiation therapy; drugs (procainamide, 3. Elevated CVP, decreased Hgb and Hct if
hydralazine, doxorubicin HCl [Adriamycin]) massive hemorrhage
B. Medical management 4. Diagnostic test: chest X-ray reveals enlarged
1. Determination and elimination/control of heart and widened mediastinum.
underlying cause D. Nursing interventions
2. Drug therapy 1. Administer oxygen therapy
a. Medication for pain relief 2. Monitor CVP/IVs closely
b. Corticosteroids, salicylates (aspirin), and 3. Assist with pericardiocentesis
indomethacin (Indocin) to reduce a. Monitor ECG, blood pressure, and pulse.
inflammation b. Assess aspirated fluid for color,
c. Specific antibiotic therapy against the consistency.
causative organism may be indicated. c. Send specimen to lab immediately.
C. Assessment findings
1. Chest pain with deep inspiration (relieved by Cardiogenic Shock
sitting up), cough, hemoptysis, malaise
2. Tachycardia, fever, pleural friction rub, See Table 4-9.
cyanosis or pallor, accentuated component of
S2, pulsus paradoxus, jugular vein distension The Blood Vessels
3. Elevated WBC and ESR, normal or elevated
AST (SGOT) Hypertension
4. Diagnostic tests A. General information
a. Chest X-ray may show increased heart size 1. According to the World Health Organization,
if effusion occurs hypertension is a persistent elevation of the
b. ECG changes: ST elevation (precordial systolic blood pressure above 140 mm Hg and
leads and 2- or 3-limb heads), T wave of the diastolic above 90 mm Hg.
inversion 2. Types
D. Nursing interventions a. Essential (primary, idiopathic): marked by
1. Ensure comfort: bed rest with semi- or loss of elastic tissue and arteriosclerotic
high-Fowlers position. changes in the aorta and larger vessels
2. Monitor hemodynamic parameters carefully. coupled with decreased caliber of the
3. Administer medications as ordered and arterioles
monitor effects. b. Benign: a moderate rise in blood pressure
4. Provide client teaching and discharge marked by a gradual onset and prolonged
planning concerning: course
a. Signs and symptoms of pericarditis c. Malignant: characterized by a rapid onset
indicative of a recurrence (chest pain that and short dramatic course with a diastolic
is intensified by inspiration and position blood pressure of more than 150 mm Hg
changes, fever, cough) d. Secondary: elevation of the blood pressure
b. Medication regimen including name, as a result of another disease such as renal
purpose, dosage, frequency, side effects parenchymal disease, Cushings disease,
pheochromocytoma, primary
Cardiac Tamponade aldosteronism, coarctation of the aorta
3. Essential hypertension usually occurs between
A. General information
ages 30 and 50; more common in men over 35,
1. An accumulation of fluid/blood in the
women over 45; African-Americans affected
pericardium that prevents adequate
twice as often as white Americans
ventricular filling; without emergency
4. Risk factors for essential hypertension include
treatment client will die.
positive family history, obesity, stress, cigarette
2. Caused by blunt or penetrating chest trauma,
smoking, hypercholesteremia, increased
malignant pericardial effusion; can be a
sodium intake
complication of cardiac surgery.
B. Medical management
B. Medical management: emergency treatment of
1. Diet and weight reduction (restricted sodium,

4
choice is pericardiocentesis (insertion of a needle
kcal, cholesterol)

250 NCLEX-RN Review


53155_04_Ch04a_p143-263.qxd 2/26/09 6:50 PM Page 251

2. Lifestyle changes: alcohol moderation, with resultant ischemia usually affects the
exercise regimen, cessation of smoking femoral, popliteal, aortal, and iliac arteries.
3. Antihypertensive drug therapy (see Table 2-17) 2. Occurs most often in men ages 5060
C. Assessment findings 3. Caused by atherosclerosis
1. Pain similar to anginal pain; pain in calves of 4. Risk factors: cigarette smoking, hyperlipidemia,
legs after ambulation or exercise (intermittent hypertension, diabetes mellitus
claudication); severe occipital headaches, B. Medical management
particularly in the morning; polyuria; nocturia; 1. Drug therapy
fatigue; dizziness; epistaxis; dyspnea on exertion a. Vasodilators: papaverine, isoxsuprine HCl
2. Blood pressure consistently above 140/90, (Vasodilan), nylidrin HCl (Arlidin),
retinal hemorrhages and exudates, edema of nicotinyl alcohol (Roniacol), cyclandelate
extremities (indicative of right-sided heart (Cyclospasmol), tolazoline HCl (Priscoline)
failure) to improve arterial circulation;
3. Rise in systolic blood pressure from supine to effectiveness questionable
standing position (indicative of essential b. Analgesics to relieve ischemic pain
hypertension) c. Anticoagulants to prevent thrombus
4. Diagnostic tests; elevated serum uric acid, formation
sodium, cholesterol levels d. Lipid-reducing drug: cholestyramine
D. Nursing interventions (Questran), colestipol HCl (Cholestid),
1. Record baseline blood pressure in three dextrothyroxine sodium (Choloxin),
positions (lying, sitting, standing: also known clofibrate (Atromid-S), gemfibrozil (Lopid),
as orthostatics) and in both arms. niacin, lovastatin (Mevacor) (see Unit 2)
2. Continuously assess blood pressure and report 2. Surgery: bypass grafting, endarterectomy,
any variables that relate to changes in blood balloon catheter dilation; lumbar
pressure (positioning, restlessness). sympathectomy (to increase blood flow),
3. Administer antihypertensive agents as amputation may be necessary
ordered; monitor closely and assess for side C. Assessment findings
effects. 1. Pain, both intermittent claudication and rest
4. Monitor intake and hourly outputs. pain, numbness or tingling of the toes
5. Provide client teaching and discharge 2. Pallor after 12 minutes of elevating feet, and
planning concerning dependent hyperemia/rubor; diminished or
a. Risk factor identification and absent dorsalis pedis, posterior tibial and
development/implementation of methods femoral pulses; trophic changes; shiny, taut
to modify them skin with hair loss on lower legs
b. Restricted sodium, kcal, cholesterol diet; 3. Diagnostic tests
include family in teaching a. Oscillometry may reveal decrease in pulse
c. Antihypertensive drug regimen (include volume
family); see Table 2-17 b. Doppler ultrasound reveals decreased
1) Names, actions, dosages, and side blood flow through affected vessels
effects of prescribed medications c. Angiography reveals location and extent of
2) Take drugs at regular times and avoid obstructive process
omission of any doses 4. Elevated serum triglycerides; sodium
3) Never abruptly discontinue the drug D. Nursing interventions
therapy 1. Encourage slow, progressive physical activity
4) Supplement diet with potassium-rich (out of bed at least 34 times per day, walking
foods if taking potassium-wasting 2 times per day).
diuretics 2. Administer medications as ordered.
5) Avoid hot baths, alcohol, or strenuous 3. Assist with Buerger-Allen exercises 4 times a
exercise within 3 hours of taking day.
medications that cause vasodilation a. Client lies with legs elevated above heart
d. Development of a graduated exercise for 23 minutes
program b. Client sits on edge of bed with legs and feet
e. Importance of routine follow-up care dependent and exercises feet and toes
upward and downward, inward and
Arteriosclerosis Obliterans outwardfor 3 minutes
c. Client lies flat with legs at heart level for
A. General information 5 minutes
1. A chronic occlusive arterial disease that may 4. Assess for sensory function and trophic
affect the abdominal aorta or the lower changes.

4
extremities. The obstruction to blood flow 5. Protect client from injury.

ADULT NURSING 251


53155_04_Ch04a_p143-263.qxd 2/26/09 6:50 PM Page 252

6. Provide client teaching and discharge 4. Predisposing factors: collagen diseases


planning concerning: (systemic lupus erythematosus, rheumatoid
a. Restricted kcal, low-saturated-fat diet; arthritis), trauma (e.g., from typing, piano
include family playing, operating a chain saw)
b. Importance of continuing with established B. Medical management: vasodilators, catecholamine-
exercise program depleting antihypertensive drugs (reserpine,
c. Measures to reduce stress (relaxation guanethidine monosulfate [Ismelin])
techniques, biofeedback) C. Assessment findings
d. Importance of avoiding smoking, 1. Coldness, numbness, tingling in one or more
constrictive clothing, standing in any digits; pain (usually precipitated by exposure
position for a long time, injury to cold, emotional upsets, tobacco use)
e. Importance of foot care, immediately 2. Intermittent color changes (pallor, cyanosis,
taking care of cuts, wounds, injuries rubor); small ulcerations and gangrene at tips
7. Prepare client for surgery if necessary. of digits (advanced)
D. Nursing interventions
Thromboangiitis Obliterans 1. Provide client teaching concerning:
(Buergers Disease) a. Importance of stopping smoking
b. Need to maintain warmth, especially in
A. General information cold weather
1. Acute, inflammatory disorder affecting c. Need to use gloves when handling cold
medium/smaller arteries and veins of the lower objects/opening freezer or refrigerator door
extremities. Occurs as focal, obstructive process; d. Drug regimen
results in occlusion of a vessel with subsequent
development of collateral circulation. Aneurysms
2. Most often affects men ages 2540.
3. Disease is idiopathic; high incidence among An aneurysm is a sac formed by dilation of an artery
smokers. secondary to weakness and stretching of the arterial
B. Medical management: see Arteriosclerosis wall. The dilation may involve one or all layers of the
Obliterans, only really effective treatment is arterial wall.
cessation of smoking.
C. Assessment findings Classification
1. Intermittent claudication, sensitivity to cold A. Fusiform: uniform spindle shape involving the
(skin of extremity may at first be white, entire circumference of the artery
changing to blue, then red) B. Saccular: outpouching on one side only, affecting
2. Decreased or absent peripheral pulses only part of the arterial circumference
(posterior tibial and dorsalis pedis), trophic C. Dissecting: separation of the arterial wall layers to
changes, ulceration and gangrene (advanced) form a cavity that fills with blood
3. Diagnostic tests: same as in Arteriosclerosis D. False: the vessel wall is disrupted, blood escapes
Obliterans except no elevation in serum into surrounding area but is held in place by
triglycerides surrounding tissue
D. Nursing interventions
1. Prepare client for surgery.
Thoracic Aortic Aneurysm
2. Provide client teaching and discharge
planning concerning: A. General information
a. Drug regimen (vasodilators, anticoagulants, 1. An aneurysm, usually fusiform or dissecting,
analgesics) to include names, dosages, in the descending, ascending, or transverse
frequency, and side effects section of the thoracic aorta
b. Need to avoid trauma to the affected 2. Usually occurs in men ages 5070
extremity 3. Caused by arteriosclerosis, infection, syphilis,
c. Need to maintain warmth, especially in hypertension
cold weather B. Medical management
d. Importance of stopping smoking. 1. Control of underlying hypertension
2. Surgery: resection of the aneurysm and
Raynauds Phenomenon replacement with a Teflon/Dacron graft; clients
will need extracorporeal circulation (heart-
A. General information lung machine).
1. Intermittent episode of arterial spasms, most C. Assessment findings
frequently involving the fingers 1. Often asymptomatic
2. Most often affects women between the teenage 2. Deep, diffuse chest pain; hoarseness;
years and age 40

4
dysphagia; dyspnea
3. Cause unknown

252 NCLEX-RN Review


53155_04_Ch04a_p143-263.qxd 2/26/09 6:50 PM Page 253

3. Pallor, diaphoresis, distended neck veins, 6. Prevent thrombophlebitis.


edema of head and arms a. Encourage client to dorsiflex foot while
4. Diagnostic tests in bed.
a. Aortography shows exact location of the b. Use elastic stockings or sequential
aneurysm compression devices as ordered.
b. X-rays: chest film reveals abnormal c. Assess for signs and symptoms (see
widening of aorta; abdominal film may Thrombophlebitis).
show calcification within walls of aneurysm 7. Provide client teaching and discharge
D. Nursing interventions: see Cardiac Surgery. planning concerning
a. Importance of changes in
Abdominal Aortic Aneurysm color/temperature of extremities
b. Avoidance of prolonged sitting, standing,
A. General information
and smoking
1. Most aneurysms of this type are saccular or
c. Need for a gradual progressive activity
dissecting and develop just below the renal
regimen
arteries but above the iliac bifurcation
d. Adherence to low-cholesterol, low-
2. Occur most often in men over age 60
saturated fat diet
3. Caused by atherosclerosis, hypertension, trauma,
e. Adherence to activity restrictions and
syphilis, other types of infectious processes
avoid lifting heavy objects (limit 15 to
B. Medical management: surgical resection of the
20 pounds).
lesion and replacement with a graft (extracorporeal
circulation not needed)
C. Assessment findings Femoral-Popliteal Bypass Surgery
1. Severe mid- to low-abdominal pain, low-back A. General information
pain 1. Most common type of surgery to correct
2. Mass in the periumbilical area or slightly to arterial obstructions of the lower extremities
the left of the midline with bruits heard over 2. Procedure involves bypassing the occluded
the mass vessel with a graft, such as Teflon, Dacron, or
3. Pulsating abdominal mass an autogenous artery or vein (saphenous).
4. Diminished femoral pulses B. Nursing interventions: preoperative
5. Diagnostic tests: same as for thoracic 1. Provide routine pre-op care.
aneurysms 2. Monitor and correct potassium imbalances to
D. Nursing interventions: preoperative prevent cardiac arrhythmias.
1. Prepare client for surgery: routine pre-op care. 3. Assess for focus of infection (infected tooth) or
2. Assess rate, rhythm, character of the infectious processes (urinary tract infections).
peripheral pulses and mark all distal pulses. 4. Mark distal peripheral pulses.
E. Nursing interventions: postoperative C. Nursing interventions: postoperative
1. Provide routine post-op care. 1. Provide routine post-op care.
2. Monitor the following parameters: 2. Assess the following
a. Hourly circulation checks noting rate, a. Circulation, noting rate, rhythm, and
rhythm, character of all pulses distal to the quality of peripheral pulses distal to the
graft graft; color; temperature; and sensation
b. CVP/PAP/PCWP b. Signs and symptoms of thrombophlebitis
c. Hourly outputs through Foley catheter c. Neuro checks
(report less than 30 mL/hour) d. Hourly outputs
d. Daily BUN/creatinine/electrolyte levels e. CVP
e. Presence of back pain (may indicate f. Wound drainage, noting amount, color, and
retroperitoneal hemorrhage) characteristics
f. IV fluids 3. Elevate legs above the level of the heart.
g. Neuro status including LOC, pupil size and 4. Encourage turning, coughing, and deep
response to light, hand grasp, movement of breathing while splinting incision.
extremities
h. Heart rate and rhythm via monitor Venous Stasis Ulcers
3. Maintain client flat in bed without sharp
flexion of hip/knee (avoid pressure on A. General information
femoral/popliteal arteries). 1. Usually a complication of thrombophlebitis
4. Auscultate lungs and encourage turning, and varicose veins.
coughing, and deep breathing. 2. Ulcers result from incompetent valves in the
5. Assess for signs and symptoms of paralytic veins, causing high pressure with rupture of
ileus. small skin veins and venules.

ADULT NURSING

4 253
53155_04_Ch04a_p143-263.qxd 2/26/09 6:51 PM Page 254

B. Medical management 2) Side effects


1. Antibiotic therapy (specific to organism a) GI: anorexia, nausea and vomiting,
cultured); topical bacteriocidal solutions diarrhea, stomatitis
2. Skin grafting b) Hypersensitivity: dermatitis,
3. Enzymatic or surgical debridement urticaria, pruritus, fever
C. Assessment findings c) Other: transient hair loss, burning
1. Pain in the limb in dependent position or sensation of feet, bleeding
during ambulation complications
2. Skin of leathery texture, brownish pigment 2. Surgery
around ankles; positive pulses but edema a. Vein ligation and stripping
makes palpation difficult. b. Venous thrombectomy: removal of a clot in
D. Nursing interventions the iliofemoral region
1. Provide bed rest, elevating extremity. c. Plication of the inferior vena cava:
2. Provide a balanced diet with added protein insertion of an umbrella-like prosthesis
and vitamin supplements. into the lumen of the vena cava to filter
3. Administer antibiotics as ordered to control incoming clots. (See Figure 4-10).
infection. C. Assessment findings
4. Promote healing by cleansing ulcer with 1. Pain in the affected extremity
prescribed agents. 2. Superficial vein: tenderness, redness,
5. Provide client teaching and discharge induration along course of the vein
planning concerning 3. Deep vein: swelling, venous distension of
a. Importance of avoiding trauma to affected limb, tenderness over involved vein, positive
limb Homans sign, cyanosis
b. Skin care regimen
c. Use of elastic support stockings (after ulcer
is healed)
cm Renal
d. Need for planned rest periods with
elevation of the extremities Veins
1
6. Adherence to balanced diet with vitamin
supplements.
Filter
2
Thrombophlebitis
A. General information
3
1. Inflammation of the vessel wall with formation
of a clot (thrombus); may affect superficial or Vena
deep veins. Cava
4
2. Most frequent veins affected are the
saphenous, femoral, and popliteal.
3. Can result in damage to the surrounding
tissues, ischemia, and necrosis. 5
4. Risk factors: obesity, HF, prolonged
immobility, MI, pregnancy, oral A.
contraceptives, trauma, sepsis, cigarette
smoking, dehydration, severe anemias, venous
cannulation, complication of surgery.
B. Medical management
1. Anticoagulant therapy
a. Heparin
1) Blocks conversion of prothrombin to
thrombin and reduces formation or
extension of thrombus
2) Side effects: spontaneous bleeding,
injection site reactions, ecchymoses, B.
tissue irritation and sloughing,
reversible transient alopecia, cyanosis,
pain in arms or legs, thrombocytopenia Figure 4-10 Vena caval filters to prevent embolus
b. Warfarin (Coumadin) from traveling to the lungs, heart, or brain.
1) Blocks prothrombin synthesis by
(A) Greenfield; (B) Umbrella

4
interfering with vitamin K synthesis

254 NCLEX-RN Review


53155_04_Ch04a_p143-263.qxd 2/26/09 6:51 PM Page 255

4. Elevated WBC and ESR 5) Have antidote (vitamin K) available.


5. Diagnostic tests 6) Alert client to factors that may affect
a. Venography (phlebography): increased the anticoagulant response (high-fat
uptake of radioactive material diet or sudden increases in vitamin K-
b. Doppler ultrasonography: impairment of rich foods).
blood flow ahead of thrombus 7) Instruct client to wear Medic-Alert
c. Venous pressure measurements: high in bracelet.
affected limb until collateral circulation is 4. Assess vital signs every 4 hours.
developed 5. Monitor for chest pain or shortness of breath
D. Nursing interventions (possible pulmonary embolism).
1. Provide bed rest, elevating involved extremity 6. Measure thighs, calves, ankles, and instep
to increase venous return and decrease edema. every morning.
2. Apply continuous warm, moist soaks to 7. Provide client teaching and discharge
decrease lymphatic congestion. planning concerning:
3. Administer anticoagulants as ordered a. Need to avoid standing, sitting for long
a. Heparin periods; constrictive clothing; crossing
1) Monitor PTT; dosage should be legs at the knees; smoking; oral
adjusted to keep PTT between 1.52.5 contraceptives
times normal control level. b. Importance of adequate hydration to
2) Use infusion pump to administer IV prevent hypercoagulability
heparin. c. Use of elastic stockings when ambulatory
3) Ensure proper injection technique. d. Importance of planned rest periods with
a) Use 26- or 27-gauge syringe with elevation of the feet
1
258-inch needle, inject into fatty e. Drug regimen
layer of abdomen above iliac crest. f. Plan for exercise/activity
b) Avoid injecting within 2 inches of 1) Begin with dorsiflexion of the feet
umbilicus. while sitting or lying down
c) Insert needle at 4590 to skin. 2) Swim several times weekly
d) Do not withdraw plunger to assess 3) Gradually increase walking distance
blood return. g. Importance of weight reduction if obese
e) Apply gentle pressure after
removal of needle, avoid massage. Pulmonary Embolism
4) Assess for increased bleeding
tendencies (hematuria; hematemesis; A. General information
bleeding gums; petechiae of soft 1. Most pulmonary emboli arise as detached
palate, conjunctiva, retina; portions of venous thrombi formed in the deep
ecchymoses, epistaxis, bloody sputum, veins of the legs, right side of the heart, or
melena) and instruct client to observe pelvic area.
for and report these. 2. Distribution of emboli is related to blood flow;
5) Have antidote (protamine sulfate) emboli involve the lower lobes of the lung
available. because of higher blood flow.
6) Instruct client to avoid aspirin, 3. Embolic obstruction to blood flow increases
antihistamines, and cough venous pressure in the pulmonary artery and
preparations containing glyceryl pulmonary hypertension.
guaiacolate, and to obtain physicians 4. Risk factors: venous thrombosis, immobility,
permission before using other OTC pre- and post-op states, trauma, pregnancy, HF,
drugs. use of oral contraceptives, obesity
b. Warfarin (Coumadin) B. Medical management
1) Assess PT daily; dosage should be 1. Drug therapy
adjusted to maintain PT at 1.52.5 a. Anticoagulants (see Thrombophlebitis)
times normal control level; INR of 2. b. Thrombolytics: streptokinase or urokinase
2) Obtain careful medication history c. Dextran 70 to decrease blood viscosity and
(there are many drug-drug aggregation of blood cells
interactions). d. Narcotics for pain relief
3) Advise client to withhold dose and e. Vasopressors (in the presence of shock)
notify physician immediately if 2. Surgery: embolectomy (surgical removal of an
bleeding or signs of bleeding occur embolus from the pulmonary arteries)
(see Heparin). C. Assessment findings
4) Instruct client to use a soft toothbrush 1. Chest pain (pleuritic), severe dyspnea, feeling

4
and to floss gently. of impending doom

ADULT NURSING 255


53155_04_Ch04a_p143-263.qxd 2/26/09 6:51 PM Page 256

2. Tachypnea, tachycardia, anxiety, hemoptysis, 2) Apply elastic bandages and stockings,


shock symptoms (if massive) which should be removed every
3. Decreased pCO2; increased pH (due to 8 hours for short periods and reapplied.
hyperventilation) 3) Assist out of bed within 24 hours,
4. Increased temperature ensuring that elastic stockings are
5. Intensified pulmonic S2; rales or crackles applied.
6. Diagnostic tests 4) Assess for increased bleeding,
a. Pulmonary angiography: reveals particularly in the groin area.
location/extent of embolism 6. Provide client teaching and discharge
b. Lung scan reveals adequacy/inadequacy planning: same as for Thrombophlebitis.
of pulmonary circulation
D. Nursing interventions Amputation
1. Administer medications as ordered; monitor
effects and side effects. A. General information
2. Administer oxygen therapy to correct hypoxemia. 1. Surgical procedure done for peripheral
3. Assist with turning, coughing, deep breathing, vascular disease if medical management is
and passive ROM exercises. ineffective and the symptoms become worse.
4. Provide adequate hydration to prevent 2. The level of amputation is determined by the
hypercoagulability. extent of the disease process.
5. Offer support/reassurance to client/family. a. Above knee (AKA): performed between the
6. Elevate head of bed to relieve dyspnea. lower third to the middle of the thigh
7. Provide client teaching and discharge b. Below knee (BKA): usually done in middle
planning: same as for thrombophlebitis. third of leg, leaving a stump of 12.517.5 cm
B. Nursing interventions: preoperative
Varicose Veins 1. Provide routine pre-op care.
2. Offer support/encouragement and accept
A. General information clients response of anger/grief.
1. Dilated veins that occur most often in the 3. Discuss
lower extremities and trunk. As the vessel a. Rehabilitation program and use of
dilates, the valves become stretched and prosthesis
incompetent with resultant venous b. Upper extremity exercises such as
pooling/edema push-ups in bed
2. Most common between ages 30 and 50 c. Crutch walking
3. Predisposing factor: congenital weakness of d. Amputation dressings/cast
the veins, thrombophlebitis, pregnancy, e. Phantom limb sensation as a normal
obesity, heart disease occurrence
B. Medical management: vein ligation (involves C. Nursing interventions: postoperative
ligating the saphenous vein where it joins the 1. Provide routine post-op care.
femoral vein and stripping the saphenous vein 2. Prevent hip/knee contractures.
system from groin to ankle) 3. Avoid letting client sit in chair with hips
C. Assessment findings flexed for long periods of time.
1. Pain after prolonged standing (relieved by 4. Have client assume prone position several
elevation) times a day and position hip in extension
2. Swollen, dilated, tortuous skin veins (unless otherwise ordered).
3. Diagnostic tests 5. Avoid elevation of the stump after 1224 hours.
a. Trendelenburg test: varicose veins distend 6. Observe stump dressing for signs of
very quickly (less than 35 seconds) hemorrhage and mark outside of dressing so
b. Doppler ultrasound: decreased or no blood rate of bleeding can be assessed.
flow heard after calf or thigh compression 7. Administer pain medication as ordered.
D. Nursing interventions 8. Ensure that stump bandages fit tightly and are
1. Elevate legs above heart level. applied properly to enhance prosthesis fitting.
2. Measure circumference of ankle and calf daily. 9. Initiate active ROM exercises of all joints
3. Apply knee-length elastic stockings. (when medically advised), crutch walking, and
4. Provide adequate rest. arm/shoulder exercises.
5. Prepare client for vein ligation, if necessary. 10. Provide stump care.
a. Provide routine pre-op care; usually a. Inspect daily for signs of skin irritation.
outpatient surgery. b. Wash thoroughly daily with warm water and
b. In addition to routine post-op care: bacteriostatic soap; rinse and dry thoroughly.
1) Keep affected extremity elevated above c. Avoid use of irritating substances such as

4
the level of the heart to prevent edema. lotions, alcohol, powders.

256 NCLEX-RN Review


53155_04_Ch04a_p143-263.qxd 2/26/09 6:51 PM Page 257

168. The nurse reading an ECG rhythm strip notes


Sample Questions that there are 8 QRS complexes in a 6-second
strip. What would be the heart rate?
1. 48.
163. An adult is admitted to the coronary care unit
2. 64.
to rule out a myocardial infarction. The client
states, I am not sure if it is just angina, and 3. 80.
I cannot understand the difference between 4. 120.
angina and heart attack pain. Which response
is most appropriate for the nurse to make? 169. A 57-year-old is being treated in the clinic for
hypertension. His blood pressure is 170/92 and
1. Anginal pain usually stops after resting.
he is complaining of fatigue and lassitude. He
2. Anginal pain produces clenching of the fists has been taking propranolol (Inderal) 80 mg bid.
over the chest while acute MI pain does not. What action by the client will assure the nurse
3. Anginal pain requires morphine for relief. that teaching has been successful?
4. Anginal pain radiates to the left arm while 1. Checks his pulse for bradycardia.
acute MI pain does not. 2. Makes an appointment as soon as he notices
fatigue.
164. An adult woman is admitted to the cardiac care
unit with a myocardial infarction. The morning 3. Stops the drug when he experiences chest
after admission she and her husband tell the pain.
nurse that she must be home tonight to care for 4. Takes the drug with breakfast and dinner.
the children when her husband goes to work.
What nursing diagnosis describes this problem? 170. A 56-year-old obese man is recovering from a
bowel resection for cancer of the colon. On his
1. Anxiety related to physical limitations.
third post-op day he complains that the area
2. Alteration in cardiac output. around the calf of his leg is warm and tender.
3. Knowledge deficit related to disease process. What assessment will the nurse observe that will
4. Impaired home maintenance. suggest the client has a deep vein thrombosis?
1. Absence of a pulse distal to the clot.
165. The nurse is caring for an adult admitted to the
2. Cyanosis distal to the clot.
coronary care unit with a myocardial infarction.
During the second night in the CCU, the client 3. Pain on dorsiflexion.
develops heart failure. What is the basis for 4. Reddened area around the clot.
insertion of a pulmonary artery catheter?
171. A client with a history of venous insufficiency
1. It provides information about pulmonary
came to the medical clinic complaining of severe
resistance.
pain in the legs. They were swollen and covered
2. It measures myocardial oxygen consumption. with deep, draining, foul-smelling ulcers. What
3. It increases renal blood flow. would the nurse suspect as an underlying cause
4. It controls afterload. of the venous insufficiency?
1. Congestive heart failure.
166. The nurse is planning care for a client who is in
2. Hypertrophied leg muscles.
heart failure. Which of the following goals are
appropriate? Select all that apply. 3. Decreased hemoglobin levels.
____ An increase in cardiac output. 4. Poor blood return to the heart.
____ An elevation in renal blood flow. 172. A 65-year-old man is admitted with venous
____ A reduction in the hearts workload. stasis ulcers and chronic venous insufficiency.
____ A decrease in myocardial contractility. To help control the swelling, what intervention
should the nurse teach the client to perform?
167. The nurse is caring for an adult who is being 1. Exercise vigorously.
treated for a myocardial infarction. Oxygen is
2. Restrict fluid intake.
ordered. Administering oxygen to this client is
related to which of the following client problems? 3. Promote gravity drainage.
1. Anxiety. 4. Eat a high-protein, low-salt diet.
2. Chest pains. 173. Which are some of the common symptoms
3. Ineffective myocardial perfusion. associated with cardiovascular disease?

4
4. Alteration in heart rate, rhythm, or conduction.

ADULT NURSING 257


53155_04_Ch04a_p143-263.qxd 2/26/09 6:51 PM Page 258

1. Shortness of breath, chest discomfort, How should the nurse interpret this rhythm?
palpitations. 1. Ventricular tachycardia.
2. Dyspnea, chest discomfort, sputum 2. Ventricular fibrillation.
production. 3. Sinus tachycardia.
3. Fatigue, weight changes, mood swings. 4. Supraventricular tachycardia.
4. Mood swings, headaches, fainting.
179. A client has the following rhythm. The client
174. Which of the following assessment findings by has no pulse or blood pressure.
the nurse is abnormal?
1. S1 heard at the fourth-fifth left intercostal
space in a 35-year-old man.
2. S2 heard at the second-third left intercostal
space in a 40-year-old female.
3. S4 heard at the apex in an 80-year-old male.
How should the nurse interpret this rhythm?
4. S3 heard at the apex in a 15-year-old female.
1. Ventricular tachycardia.
175. Which of the following instructions should the 2. Ventricular fibrillation.
nurse give to a client prior to an exercise 3. Sinus tachycardia.
electrocardiogram?
4. Supraventricular tachycardia.
1. Avoid coffee, tea, and alcohol the day of the
test. 180. A client had a myocardial infarction yesterday.
2. Smoking is permitted up to the time of the His cardiac monitor shows 6 to 8 PVCs per
test. minute, with occasional couplets. What is the
3. Allow only 3 hours of sleep the night prior to nurses best action?
the test. 1. Monitor the client for development of
4. Take all medications as prescribed prior to ventricular tachycardia.
the test. 2. Administer the ordered prn dose of lidocaine.
3. Perform a precordial thump.
176. To prevent possible complication, which of the
4. Initiate manual chest compressions.
following questions should a nurse ask a client
prior to a cardiac catheterization? 181. A client is admitted in cardiogenic shock. What
1. Have you ever had a cardiac catheterization will be inserted to best evaluate the hearts
before? hemodynamic performance?
2. Can you eat shellfish? 1. Intra-arterial line.
3. Have you ever had general anesthesia 2. Pulmonary artery catheter.
before? 3. Intra-aortic balloon pump (IABP).
4. Have you ever had a heart attack? 4. Triple lumen catheter.
177. Which of the following should the nurse include 182. Which of the following statements by a client to
in the plan of care for a post-op coronary the nurse indicates a risk factor for coronary
arteriogram client? artery disease?
1. Assess pedal pulses. 1. I exercise four times a week.
2. Assess lung sounds. 2. No one in my family has heart problems.
3. Provide early ambulation. 3. My cholesterol is 189.
4. Monitor vital signs every 8 hours. 4. I smoke 112 packs of cigarettes per day.
178. A client has the following rhythm. The client 183. An adult female has a history of coronary artery
has no pulse or blood pressure. disease and angina pectoris. After walking to the
bathroom, she complains of aching substernal
pain that radiates to her left shoulder. What is
the nurses best action?
1. Provide a warm shoulder massage.
2. Administer a prn dose of nitroglycerin
sublingually.

4 258 NCLEX-RN Review


53155_04_Ch04a_p143-263.qxd 2/26/09 6:51 PM Page 259

3. Use pillows to support and immobilize the 3. Encourage the man to void.
left shoulder. 4. Assist him to a supine position.
4. Administer a prn dose of aspirin or
acetaminophen (Tylenol). 189. An adult male is being discharged from the
hospital following a myocardial infarction.
184. A nitroglycerin transdermal patch was Knowing he will wait 46 weeks before having
prescribed 6 weeks ago for an adult to treat sexual activity, what statement demonstrates the
angina pectoris. How will the nurse know the client understands the guidelines taught?
patch is effective? 1. Bedtime is the best time to have intercourse.
1. The clients serum cholesterol level has 2. He should exercise for 1015 minutes before
decreased. intercourse, to warm up.
2. The clients pressure is within normal limits. 3. He should take a nitroglycerin before
3. The client reports no episodes of chest pain. intercourse to prevent chest pain.
4. Pulse oximetry shows the clients oxygen 4. It is best to avoid having intercourse when
saturation is improved. the stomach is empty.

185. An adult has developed angina pectoris 190. An adult is scheduled for a percutaneous
secondary to coronary artery disease. A low-fat, transluminal coronary angioplasty (PTCA). The
low-cholesterol diet is prescribed for the client. adult asks the nurse, Can you tell me again
The nurse should praise the client for a wise what the doctor is going to do? What is the
choice if which of the following was selected for nurses best response?
an evening snack? 1. A clot dissolving drug is administered
1. Cheese cubes and crackers. through a catheter into the blocked section of
2. Tuna salad sandwich. your artery.
3. No-added sugar ice cream. 2. A piece of vein from your leg is used to
4. Jello mold with fruit slices. bypass the blocked section of your artery.
3. A tiny rotating blade is used to scrape off
186. Lidocaine is mixed 2 g in 500 mL D5W. The the plaque that is blocking your artery.
nurse prepared to start an infusion at 2 mg/h 4. A balloon is placed next to the plaque
using a 60-drop tubing. Which of the following is blocking your artery, then the balloon is
the correct rate to start the infusion on a pump? inflated to crush the plaque.
1. 15 mL.
2. 30 mL. 191. A man is being discharged following coronary
artery bypass graft surgery (CABG). The nurse
3. 45 mL.
recognizes that he needs additional teaching if
4. 60 mL. he makes which of the following statements?
187. An adult male is transferred to the step-down 1. Ill be going to a support group to help me
unit on the third day after a myocardial quit smoking.
infarction. Which of the following should the 2. I will take a walk twice a week.
nurse include in his care plan at this time? 3. I should bake or broil my chicken instead of
1. Enforcing complete bed rest. frying it.
2. Supervising short walks in the hallway. 4. Ive learned a breathing exercise to help me
3. Performing passive range of motion exercises. calm down if I get upset.
4. Having him sit on the side of the bed and 192. Which of the following assessment findings by
dangle his legs. the nurse indicates right ventricular failure in a
client?
188. A 55-year-old man with a history of angina
pectoris complains of chest pain radiating to the 1. Pink frothy sputum.
jaw. After taking three nitroglycerin gr 1/150 2. Paroxysmal nocturnal dyspnea.
tablets he is still having the chest pain. His skin 3. Jugular venous distention.
is cool and pale and he is diaphoretic and 4. Crackles.
mildly short of breath. What is the nurses
priority action? 193. A nurse is assessing a client with fatigue,
1. Auscultate heart and lung sounds. tachycardia, crackles, and pink frothy sputum.
Which nursing diagnosis is of the most importance?

4
2. Administer another nitroglycerin tablet.

ADULT NURSING 259


53155_04_Ch04a_p143-263.qxd 2/26/09 6:51 PM Page 260

1. Impaired skin integrity. 1. A verbal and visual check of all clear.


2. Impaired gas exchange. 2. No lubricant on the paddles.
3. Potential for injury. 3. Placing paddles lightly on the chest.
4. Anxiety. 4. Standing in alignment with the bed while
administering the shock.
194. An adult is admitted with an acute exacerbation
of congestive heart failure. Assessment reveals 199. An adult client has experienced a cardiac arrest
tachycardia, hypertension, and crackles in the and the nurse is performing CPR. What is the
bases of the lung fields. The medications of correct hand position on the clients chest?
0.5 mg of digoxin (Lanoxin) IV and 40 mg of 1. Over the upper half of the sternum.
furosemide (Lasix) IV are administered 2. Two finger widths below the sternal notch.
immediately. How would the nurse be aware
3. Two finger widths above the xiphoid process.
that the medications are having a therapeutic
effect? 4. Over the xiphoid process.
1. The clients pulse rate decreases below 100. 200. A client with a history of a myocardial infarction
2. The client has an increased urine specific 2 days ago reports chest pain that is worse on
gravity. inspiration but is relieved on sitting forward.
3. The client expectorates frothy sputum. Based on this finding, what does the nurse
4. The clients lungs are clear to auscultation. suspect the client is experiencing?
1. Endocarditis.
195. A client is admitted with pulmonary edema. 2. Angina pectoris.
The nurse is preparing to administer morphine
3. Pericarditis.
sulfate. What beneficial effect does morphine
have in pulmonary edema? 4. Recurrent myocardial infarction.
1. Decreases anxiety, work of breathing, and 201. An adult has prazocin hydrochloride
vasodilates. (Minipress) prescribed to treat hypertension.
2. Decreases respiratory rate. What instructions should the nurse provide to
3. Provides an analgesic and sedative effect. the client?
4. Decreases anxiety and vasoconstricts. 1. Take the medication with meals.
2. Take the first dose at bedtime.
196. An adult female is being discharged after having
3. Report a pulse rate below 50 to the physician.
a ventricular demand pacemaker inserted. The
nurse should include which of the following in 4. Check the ankles daily for edema.
the teaching plan for this client?
202. An adult has essential hypertension. She is
1. She should not use remote control devices being treated with a thiazide diuretic and dietary
(e.g., TV channel selector). and lifestyle modifications. The nurse knows
2. She must leave the room while a microwave that she understands the treatment if she makes
oven is in operation. which of the following statements?
3. She will need to avoid air travel. 1. I will use soy sauce or mustard instead of
4. She should not pass through metal detectors. salt on my food.
2. I need to cut back to two, 4-ounce glasses of
197. An adult has a ventricular demand pacemaker wine a day.
that is set at 72 beats per minute. The nurse
3. I will stop riding my bike because vigorous
knows that the clients pacemaker is functioning
exercise will raise my blood pressure.
correctly if which of the following appears on
the ECG? 4. Smoking helped cause my hypertension, but
quitting wont reverse the damage.
1. Pacemaker spikes instead of QRS complexes.
2. Pacemaker spikes followed by QRS 203. Hydrochlorothiazide is prescribed to treat high
complexes. blood pressure. Which dietary modifications
3. Pacemaker spikes before each P wave. will the nurse instruct the client to increase?
4. Pacemaker spikes appearing only if the heart 1. Fresh oranges.
rate is over 72. 2. Cold cereals.
3. Cola drinks.
198. Which strategy by the nurse provides safety
4. Cranberry juice.

4
during a defibrillation attempt?

260 NCLEX-RN Review


53155_04_Ch04a_p143-263.qxd 2/26/09 6:51 PM Page 261

204. A client reports an aching pain and cramping 210. The client has a large, venous stasis ulcer on her
sensation that occurs while walking. The pain left ankle. Wound care is performed three times
disappears after cessation of walking. What a week by a home health nurse. What instruction
condition does the nurse suspect is occurring? should the nurse include in the teaching?
1. Deep venous thrombosis (DVT). 1. Dangle the legs for 510 minutes several
2. Raynauds disease. times a day.
3. Arteriosclerosis obliterans. 2. Wear heavy cotton or wool socks when going
4. Thrombophlebitis. outdoors.
3. Soak the feet in tepid water three or four
205. An adult has severe arteriosclerosis obliterans times daily.
and complains of intermittent claudication after 4. Take frequent rest periods with her legs
walking 20 feet. How should the nurse plan to elevated.
position the client when she is in bed?
1. Supine with legs elevated. 211. The nurse assesses signs of bleeding in a client
2. In semi-Fowlers position with knees extended. taking Coumadin and notifies the physician.
Which of the following would the nurse expect
3. In reverse Trendelenburg position.
to administer?
4. In Trendelenburg position.
1. Packed RBCs.
206. An adult female experiences painful arterial 2. Plasma protein.
spasms in her hands due to Raynauds 3. Platelets.
phenomenon. Which of the following should the 4. Vitamin K.
nurse include in the teaching plan for her?
1. Drink a hot beverage, such as tea or coffee, to 212. A client is being discharged after treatment of deep
relieve spasms. venous thrombosis. Coumadin (warfarin) 2.5 mg
2. Reduce intake of high fat or high cholesterol daily is prescribed. The nurse recognizes that
foods. which of the following statements indicates that
the client understands the effects of Coumadin?
3. Raise the hands above the head to relieve
spasms. 1. Ill use an electric razor to shave my legs.
4. Wear gloves when handling refrigerated foods. 2. This will prevent me from having future DVTs.
3. I need to eat more salads and fresh fruits.
207. Which assessment finding by the nurse would 4. I will take aspirin instead of Tylenol for
indicate an abdominal aortic aneurysm? headaches.
1. Knifelike pain in the back.
2. Pulsatile mass in the abdomen. 213. An adult female, who was admitted 4 hours ago
with thrombophlebitis in the left leg, suddenly
3. Unequal femoral pulses.
becomes confused and dyspneic. She begins
4. Boardlike rigid abdomen. coughing up blood-streaked sputum and
complains of chest pain that worsens on
208. A nurse is assessing a post-op femoral popliteal
inspiration. What is the nurses best response?
bypass client. Which of the following
assessment findings indicates a complication? 1. Apply soft restraints to prevent excessive
movement.
1. BP 110/80, HR 86, RR 20.
2. Perform a Heimlich maneuver.
2. Small amount of dark-red blood on dressing.
3. Place her in bed in semi-Fowlers position.
3. A decrease in pulse quality in the operated leg.
4. Place her in Trendelenburg position on her
4. Swelling of the operative leg.
left side.
209. An adult has just returned to the surgical unit after
214. A client is admitted to rule out pulmonary
a femoral-popliteal bypass on the right leg. The
embolism (PE) from a deep venous thrombosis.
nurse should place the client in what position?
A Dextran 70 infusion is ordered for the client.
1. Fowlers position with the right leg extended. What is the action of Dextran?
2. Supine with the right knee flexed 45. 1. Increase blood viscosity.
3. Supine with the right leg extended and flat 2. Decrease platelet adhesion.
on the bed.
3. Decrease plasma volume.
4. Semi-Fowlers position with the right leg
4. Increase the hemoglobin.

4
elevated on two pillows.

ADULT NURSING 261


53155_04_Ch04a_p143-263.qxd 2/26/09 6:51 PM Page 262

215. A client reports aching, heaviness, itching, and blood flow will increase as cardiac output
moderate swelling of the legs. On assessment, increases.
the nurse notes dilated tortuous skin veins. A reduction in the hearts workload should be
What condition does the nurse suspect? checked. Reducing the venous return or the
1. Thrombophlebitis. cardiac workload is an appropriate goal.
2. Venous thrombosis.
167. 3. With acute myocardial infarction there is
3. Varicose veins.
ineffective myocardial perfusion, resulting in a
4. Chronic venous insufficiency. decrease in the amount of oxygen available for
tissue perfusion. Oxygen is administered to
216. An adult had an above-the-knee amputation of
improve tissue perfusion in these clients.
the left leg 2 days ago. The nurse should include
which of the following in the care plan? 168. 3. A regular heart rate is calculated by
1. Resting in a prone or supine position with multiplying the number of QRS complexes in
the stump extended several times a day. 6 seconds (8 QRS complexes) by 10 (because
2. Using a rolled towel or small pillow to there are 60 seconds in 1 minute). The heart rate
elevate the stump at all times. is 80. This method is not accurate if the clients
3. Applying warm soaks to the stump to reduce heart rate is irregular.
phantom limb pain.
169. 1. A common side effect of propranolol is
4. Avoiding turning to the left side until the
slowed pulse rate because the drug is a beta
stump has healed completely.
blocker.
217. An adult male had a below-the-knee amputation
170. 3. Pain on dorsiflexion is a common
of the right foot 2 days ago. He is complaining of
manifestation of deep vein thrombosis.
pain in his right foot. What is the best response
by the nurse? 171. 4. Venous insufficiency is stasis of venous blood
1. Explain to him that this is a common flow or poor blood return to the heart. The leg
sensation after amputation. muscles that normally compress the veins to
2. Remind him that that foot was amputated force blood upward are not effective.
and therefore cannot have pain.
172. 3. Swelling is minimized by promoting gravity
3. Apply an ice pack to the stump.
drainage. This could be accomplished by
4. Show him the stump so he will realize his elevating the extremities.
right foot is gone.
173. 1. Some of the most common clinical
manifestations of cardiovascular disease are
shortness of breath, chest pain or discomfort,
Answers and Rationales dyspnea, palpitations, fainting, and peripheral
skin changes such as edema.
163. 1. Anginal pain is of short duration and is 174. 3. S4 is an abnormal heart sound. It is indicative
usually relieved by rest. of decreased ventricular compliance.
164. 3. The nurse should assess the couples 175. 1. Avoid any stimulants such as coffee, tea, or a
understanding of the disease process and depressant such as alcohol.
rehabilitation, so that they can make rational
decisions. 176. 2. Shellfish contains iodine, which is also in the
contrast media used during a catheterization. It
165. 1. The Swan-Ganz catheter measures pulmonary is imperative to obtain information regarding
artery and capillary wedge pressures, which are iodine allergies.
good indicators of increase in pulmonary
pressure caused by increase in left ventricular 177. 1. Assessment of pedal pulses is imperative after
pressure. a cardiac catheterization. Evaluation of presence
and quality of pulses indicates blood flow to the
166. An increase in cardiac output should be catheterized extremity.
checked. This is an appropriate goal.
An elevation in renal blood flow should be 178. 1. The above rhythm is ventricular tachycardia.
checked. This is an appropriate goal. Renal

4
179. 2. The above rhythm is ventricular fibrillation.

262 NCLEX-RN Review


53155_04_Ch04a_p143-263.qxd 2/26/09 6:51 PM Page 263

180. 2. Lidocaine, a class I antidysrhythmic drug, is further progression of his disease. Riding instead
indicated when the client has six or more PVCs of walking would not provide aerobic exercise.
per minute, multifocal PVCs, couplets or Therefore, this statement shows that the client
triplets, or PVCs occurring on the downslope of needs further teaching.
the T wave. Any of these situations is likely to
progress to the more dangerous ventricular 192. 3. Jugular venous distention is seen in right
tachycardia or ventricular fibrillation if not ventricular failure as volume overload occurs. This
treated immediately. overload is reflected upward into the jugulars.

181. 2. A pulmonary artery catheter will show all 193. 2. With left ventricular heart failure, carbon
right and left heart hemodynamic pressures and dioxide and oxygen exchange is impaired due
provide for cardiac output measurements. to fluid overload and leads to hypoxia.

182. 4. Smoking has been determined to increase the 194. 4. Crackles in the lungs are a sign of pulmonary
risk of coronary heart disease. edema due to HF. Improved cardiac output due
to digoxin and reduced extracellular fluid
183. 2. Nitroglycerin dilates peripheral veins, reducing volume due to furosemide should result in
venous return to the heart. This immediately reduction of pulmonary edema.
decreases cardiac workload, relieving ischemia
and chest pain. It also dilates coronary arteries, 195. 1. This is the beneficial effect of morphine in
improving oxygen supply to the heart. pulmonary edema.

184. 3. Nitroglycerin reduces cardiac workload and 196. 4. Metal detectors generate strong magnetic
improves myocardial oxygenation. This prevents fields that can alter pacemaker settings or
episodes of anginal pain. produce interference that causes malfunction.

185. 4. Most fruits and vegetables are low in fat and 197. 2. The ventricular pacemaker stimulates the
cholesterol-free. Jello also has no fat or cholesterol. ventricle if no atrial impulse is transmitted
through the AV node. The appearance of the
186. 2. 30 mL is 2 mg/h. QRS complex shows that the ventricle has
1000 mg 5 1 g. 2 g is 200 mg. responded to the stimulus.
2000 mg:500 mL::2 mg: x mL
2000x 5 1000 198. 1. The nurse must make sure both verbally and
x 5 0.5 mL/hr visually that all health care providers are clear.
60 drops 5 1 mL 60 drops 3 0.5 mL 5 30 mL/hr.
199. 3. This hand position would depress the lower
187. 2. To improve activity tolerance, supervised half of the sternum, which would compress the
walks for gradually increasing distances are heart effectively.
encouraged when the client is transferred out of
200. 3. The pain of pericarditis is exacerbated with
the coronary care unit.
respirations. Rotating the trunk and sitting up
188. 1. Assessment is important to identify the frequently relieves the pain.
probable cause of the pain so that definitive
201. 2. First dose syncope occurs with prazocin. To
intervention can be planned. Dysrhythmias are a
reduce the risk of fainting, the client should take
common complication of MI. Crackles in the
the first dose at bedtime.
lungs and an S3 gallop may indicate heart failure.
202. 2. Moderation in alcohol intake is an important
189. 3. Nitroglycerin is used prophylactically before
lifestyle change for controlling high blood
activities that are known to cause chest pain,
pressure. Alcohol adds empty calories to the diet
including sexual intercourse.
and elevates arterial blood pressure.
190. 4. PTCA is also called balloon angioplasty
203. 1. Oranges are high in potassium. Thiazide
because a balloon-tipped catheter is used. When
diuretics, such as hydrochlorothiazide, deplete
the balloon is inflated, the plaque is compressed,
body potassium by increasing urinary excretion,
leaving the artery unobstructed.
so potassium intake should be increased.
191. 2. Aerobic exercise, such as walking, helps to
204. 3. Intermittent claudication is the main
slow formation of atherosclerotic plaques in
symptom of narrowing of the arteries
coronary artery disease. The client needs to

4
(arteriosclerosis).
make the necessary lifestyle changes to prevent

ADULT NURSING 263


53155_04_Ch04b_p264-377.qxd 2/26/09 7:32 AM Page 264

205. 3. Gravity facilitates improved arterial blood 212. 1. Warfarin is an anticoagulant, which increases
flow. The reverse Trendelenburg position, in the risk of bleeding from any injury. Use of an
which the feet are below heart level, is used to electric razor reduces the risk of a cut, which
improve circulation to the lower extremities. might bleed excessively.

206. 4. Cold induces arterial spasms. When the hands 213. 3. Her symptoms suggest that the client has
will be exposed to cold, warm gloves or mittens pulmonary emboli. Her activity should be
should be worn. limited to prevent further embolization, and her
head should be elevated to promote lung
207. 2. A pulsating abdominal mass is a common expansion and ease dyspnea.
finding of an abdominal aortic aneurysm.
214. 2. Dextran coats the platelet surface to decrease
208. 3. A decrease in pulse quality signifies a adhesion. In doing so, the plasma volume
decrease in the patency of the artery. expands, and viscosity is decreased.
209. 3. The best position for the affected leg is 215. 3. This describes varicose veins.
extended and flat in the bed. Elevating the leg
would allow gravity to impede circulation. 216. 1. It is essential to prevent contractures of the
Having the leg dependent would promote hip joint so that the client will be able to walk
development of edema, which could also impair with a prosthesis. Lying supine or prone with
circulation. the stump extended helps to prevent hip
contractures.
210. 4. Elevating the legs improves venous drainage
and reduces edema, which will promote wound 217. 1. Phantom limb pain is common after
healing. amputation. It is a real sensation and needs to be
acknowledged by the nurse.
211. 4. An injection of vitamin K will increase the
synthesis of prothrombin and balance clotting
time, thereby decreasing the chance of bleeding.

The Hematologic System

OVERVIEW OF ANATOMY B. Primary function is hematopoiesis (the formation


of blood cells)
AND PHYSIOLOGY C. Two kinds of bone marrow, red and yellow
1. Red (functioning) marrow
The structures of the hematologic or hematopoietic a. Carries out hematopoiesis; production site
system include the blood, blood vessels, and blood- of erythroid, myeloid, and thrombocytic
forming organs (bone marrow, spleen, liver, lymph components of blood; one source of
nodes, and thymus gland). The major function of blood lymphocytes and macrophages
is to carry necessary materials (oxygen, nutrients) to b. Found in ribs, vertebral column, other flat
cells and to remove carbon dioxide and metabolic bones
waste products. The hematologic system also plays an 2. Yellow marrow: red marrow that has changed
important role in hormone transport, the inflammatory to fat; found in long bones; does not contribute
and immune responses, temperature regulation, fluid- to hematopoiesis
electrolyte balance, and acid-base balance. D. All blood cells start as stem cells in the bone
marrow; these mature into the different, specific
Bone Marrow types of cells, collectively referred to as formed
elements of blood or blood components:
A. Contained inside all bones, occupies interior of erythrocytes, leukocytes, and thrombocytes.
spongy bones and center of long bones;
collectively one of the largest organs of the body
(45% of total body weight) Blood

4
A. Composed of plasma (55%) and cellular
components (45%); see Figure 4-11.

264 NCLEX-RN Review


53155_04_Ch04b_p264-377.qxd 2/26/09 7:32 AM Page 265

2. Serum globulins: alpha, beta, gamma


a. Alpha: role in transport of steroids, lipids,
bilirubin
b. Beta: role in transport of iron and copper
c. Gamma: role in immune response, function
of antibodies
3. Fibrinogen, prothrombin, plasminogen (see
Blood Coagulation)

Cellular Components
Cellular components or formed elements of blood are
erythrocytes (red blood cells [RBCs]), which are
responsible for oxygen transport; leukocytes (white
blood cells [WBCs]), which play a major role in
defense against microorganisms; and thrombocytes
(platelets), which function in hemostasis.
A. Erythrocytes
1. Bioconcave disc shape, no nucleus, chiefly
sacs of hemoglobin
2. Cell membrane is highly diffusible to O2 and
CO2
3. RBCs are responsible for oxygen transport via
hemoglobin (Hgb)
a. Two portions: iron carried on heme
Figure 4-11 Components of blood portion; second portion is protein
b. Normal blood contains 1218 g Hgb/
100 mL blood; higher (1418 g) in men
than in women (1214 g)
B. Hematocrit 4. Production
1. Reflects portion of blood composed of red a. Start in bone marrow as stem cells,
blood cells released as reticulocytes (immature cells),
2. Centrifugation of blood results in separation mature into erythrocytes
into top layer of plasma, middle layer of b. Erythropoietin stimulates differentiation;
leukocytes and platelets, and bottom layer of produced by kidneys and stimulated by
erythrocytes. hypoxia
3. Majority of formed elements is erythrocytes; c. Iron, vitamin B12, folic acid, pyridoxine
volume of leukocytes and platelets is (vitamin B6), and other factors required for
negligible. erythropoiesis
C. Distribution 5. Hemolysis (destruction)
1. 1300 mL in pulmonary circulation a. Average life span 120 days
a. 400 mL arterial b. Immature RBCs destroyed in either bone
b. 60 mL capillary marrow or other reticuloendothelial organs
c. 840 mL venous (blood, connective tissue, spleen, liver,
2. 3000 mL in systemic circulation lungs, and lymph nodes)
a. 550 mL arterial c. Mature cells removed chiefly by liver and
b. 300 mL capillary spleen
c. 2150 mL venous d. Bilirubin: by-product of Hgb released when
RBCs destroyed, excreted in bile
Plasma e. Iron: freed from Hgb during bilirubin
formation; transported to bone marrow via
A. Liquid part of blood; yellow in color because of transferrin and reclaimed for new Hgb
pigments production
B. Consists of serum (liquid portion of plasma) and f. Premature destruction: may be caused by
fibrinogen RBC membrane abnormalities, Hgb
C. Contains plasma proteins such as albumin, serum abnormalities, extrinsic physical factors
globulins, fibrinogen, prothrombin, plasminogen (such as the enzyme defects found in G6PD)
1. Albumin: largest of plasma proteins, involved g. Normal age RBCs may be destroyed by
in regulation of intravascular plasma volume gross damage as in trauma or extravascular
and maintenance of osmotic pressure

4
hemolysis (in spleen, liver, bone marrow)

ADULT NURSING 265


53155_04_Ch04b_p264-377.qxd 2/26/09 7:32 AM Page 266

B. Leukocytes: granulocytes and mononuclear cells: f. Antibodies are automatically formed


involved in protection from bacteria and other against the ABO antigens not on persons
foreign substances own RBCs; transfusion with mismatched or
1. Granulocytes: eosinophils, basophils, and incompatible blood results in a transfusion
neutrophils reaction (see Table 4-19).
a. Eosinophils: involved in phagocytosis and 2. Rh typing
allergic reactions a. Identifies presence or absence of Rh
b. Basophils: involved in prevention of antigen (Rh positive or Rh negative).
clotting in microcirculation and allergic b. Anti-Rh antibodies not automatically
reactions formed in Rh-negative person, but if
c. Eosinophils and basophils are reservoirs of Rh-positive blood is given, antibody
histamine, serotonin, and heparin formation starts and a second exposure
d. Neutrophils: involved in short-term to Rh antigen will trigger a transfusion
phagocytosis reaction.
1) mature neutrophils: c. Important for Rh-negative woman carrying
polymorphonuclear leukocytes Rh-positive baby; first pregnancy not
2) immature neutrophils: band cells affected, but in a subsequent pregnancy with
(bacterial infection usually produces an Rh-positive baby, mothers antibodies
increased numbers of band cells) attack babys RBCs (see Unit 6).
2. Mononuclear cells: monocytes and
lymphocytes: large nucleated cells Blood Coagulation
a. Monocytes: involved in long-term
phagocytosis; play a role in immune Conversion of fluid blood into a solid clot to reduce
response blood loss when blood vessels are ruptured.
1) largest leukocyte A. Systems that initiate clotting
2) produced by bone marrow: give rise to 1. Intrinsic system: initiated by contact activation
histiocytes (Kupffer cells of liver), following endothelial injury (intrinsic to
macrophages, and other components of vessel itself)
reticuloendothelial system a. Factor XII initiates as contact made
b. Lymphocytes: immune cells; produce between damaged vessel and plasma
substances against foreign cells; produced protein
primarily in lymph tissue (B cells) and b. Factors VIII, IX, and XI activated
thymus (T cells) (see also Immune 2. Extrinsic system
Response). a. Initiated by tissue thromboplastins,
C. Thrombocytes (platelets) released from injured vessels (extrinsic
1. Fragments of megakaryocytes formed in bone to vessel)
marrow b. Factor VII activated
2. Production regulated by thrombopoietin B. Common pathway: activated by either intrinsic or
3. Essential factor in coagulation via adhesion, extrinsic pathways
aggregation, and plug formation 1. Platelet factor 3 (PF3) and calcium react with
4. Release substances involved in coagulation factors X and V.
2. Prothrombin converted to thrombin via
Blood Groups thromboplastin.
3. Thrombin acts on fibrinogen, forming soluble
A. Erythrocytes carry antigens, which determine the fibrin.
different blood groups. 4. Soluble fibrin polymerized by factor XIII to
B. Blood-typing systems are based on the many produce a stable, insoluble fibrin clot.
possible antigens, but the most important are the C. Clot resolution: takes place via fibrinolytic system
antigens of the ABO and Rh blood groups because by plasmin and proteolytic enzymes; clot dissolves
they are most likely to be involved in transfusion as tissue repairs.
reactions.
1. ABO typing
a. Antigens of system are labelled A and B.
Spleen
b. Absence of both antigens results in type O A. Largest lymphatic organ: functions as blood
blood. filtration system and reservoir
c. Presence of both antigens is type AB. B. Vascular, bean shaped; lies beneath the diaphragm,
d. Presence of either A or B results in type A behind and to the left of the stomach; composed of
and type B, respectively. a fibrous tissue capsule surrounding a network of
e. Nearly half the population is type O, the fiber

4
universal donor. C. Contains two types of pulp

266 NCLEX-RN Review


53155_04_Ch04b_p264-377.qxd 2/26/09 7:32 AM Page 267

Table 4-19 Complications of Blood Transfusion

Type Causes Mechanism Occurrence Signs and Symptoms Intervention


Hemolytic ABO Antibodies in Acute: first Headache, lumbar or Stop transfusion.
incompatibility; recipient plasma 5 min after sternal pain, nausea, Continue saline
Rh react with antigen completion vomiting, diarrhea, fever, IV. Send blood
incompatibility; in donor cells. of transfusion chills, flushing, heat unit and client
use of dextrose Agglutinated cells Delayed: days along vein, restlessness, blood sample to
solutions; wide block capillary to 2 weeks anemia, jaundice, lab. Watch for
temperature blood flow to after dyspnea, signs of hemoglobinuria.
fluctuations organs. Hemolysis shock, renal Treat or prevent
(Hgb into plasma shutdown, DIC shock, DIC, and
and urine). renal shutdown.
Allergic Transfer of an Immune sensitivity Within 30 min Urticaria, laryngeal Stop transfusion.
antigen or to foreign serum of start of edema, wheezing, Administer
antibody from protein transfusion dyspnea, antihistamine
donor to bronchospasm, and/or
recipient; headache, epinephrine. Treat
allergic anaphylaxis life-threatening
donors reactions.
Pyrogenic Recipient Leukocyte Within 1590 Fever, chills, flushing, Stop transfusion.
possesses agglutination min after palpitations, Treat temperature.
antibodies Bacterial organisms initiation of tachycardia, Transfuse with
directed transfusion occasional leukocyte-poor
against WBCs; lumbar pain blood or washed
bacterial RBCs.
contamination; Administer
multitransfused antibiotics prn.
clients;
multiparous
clients
Circulatory Too rapid infusion Fluid volume overload During and after Dyspnea, tachycardia, Slow infusion rate.
overload in susceptible transfusion orthopnea, increased Use packed cells
clients blood pressure, instead of whole
cyanosis, anxiety blood.
Monitor CVP through
a separate line.
Air embolism Blood given under Bolus of air blocks Anytime Dyspnea, increased Clamp tubing.
air pressure pulmonary artery pulse, wheezing, chest Turn client on
following severe outflow pain, decreased blood left side.
blood loss pressure, apprehension
Thrombo- Use of large Platelets deteriorate When large Abnormal bleeding Assess for signs of
cytopenia amounts of rapidly in amounts of bleeding.
banked blood stored blood blood given Initiate bleeding
over 24 hr precautions.
Use fresh blood.
Citrate Large amounts of Citrate binds ionic After large Neuromuscular irritability Monitor/treat
intoxication citrated blood calcium amounts Bleeding due to hypocalcemia.
in clients with of banked decreased calcium Avoid large
decreased blood amounts of
liver citrated blood.
function Monitor liver function.
Hyperkalemia Potassium levels Release of potassium In clients with Nausea, colic, diarrhea, Administer blood
increase in into plasma with renal muscle spasms, ECG less than 57 days
stored blood red cell lysis insufficiency changes (tall peaked old in clients with
T-wave, short Q-T impaired potassium
segment) excretion.

ADULT NURSING

4 267
53155_04_Ch04b_p264-377.qxd 2/26/09 7:32 AM Page 268

1. Red pulp: located between the fibrous strands, i. CNS: confusion, headache, paresthesias,
composed of RBCs, WBCs, and macrophages syncope
2. White pulp: scattered throughout the red pulp, j. Musculoskeletal: joint, back, or bone pain
produces lymphocytes and sequesters B. Lifestyle: exposure to chemicals, occupational
lymphocytes, macrophages, and antigens exposure to radiation
D. 12% of red cell mass or 200 mL blood/minute C. Use of medications
stored in spleen; blood comes via the splenic 1. Iron, vitamins (B6, B12, folic acid)
artery to the pulp for cleansing, then passes into 2. Corticosteroids
splenic venules that are lined with phagocytic 3. Anticoagulants
cells, and finally to the splenic vein to the liver. 4. Antibiotics
E. Important hematopoietic site in fetus; postnatally 5. Aspirin or aspirin-containing compounds
produces lymphocytes and monocytes 6. Cold or allergy preparations
F. Important in phagocytosis; removes misshapen 7. Antiarrhythmics
erythrocytes, unwanted parts of erythrocytes 8. Blood transfusions (cryoprecipitates)
G. Also involved in antibody production by plasma 9. Cancer chemotherapy drugs
cells and iron metabolism (iron released from Hgb 10. Immunosuppressant drugs
portion of destroyed erythrocytes returned to bone D. Medical history
marrow) 1. Surgery: splenectomy, tumor resection, cardiac
H. In the adult, functions of the spleen can be taken valve replacement, GI tract resection
over by the reticuloendothelial system. 2. Allergies: multiple transfusions with whole
blood or blood products, other known allergies
3. Mononucleosis; radiation therapy; recurrent
Liver infections; malabsorption syndrome; anemia;
See also Gastrointestinal Tract. delayed wound healing; thrombophlebitis,
A. Involved in bile production (via erythrocyte pulmonary embolism, deep venous thrombosis
destruction and bilirubin production) and (DVT); liver disease, ETOH abuse, vitamin K
erythropoiesis (during fetal life and when bone deficiency; angina pectoris, atrial fibrillation
marrow production is insufficient). E. Family history; jaundice, anemia, bleeding
B. Kupffer cells of liver have reticuloendothelial disorders (hemophilia, polycythemia),
function as histiocytes; phagocytic activity and malignancies, congenital blood dyscrasias
iron storage.
C. Liver also involved in synthesis of clotting factors,
synthesis of antithrombins.
Physical Examination
A. Auscultate for heart murmurs; bruits (cerebral,
cardiac, carotid); pericardial or pleural friction
ASSESSMENT rubs; bowel sounds.
B. Inspect for
1. Flush or pallor of mucous membranes, nail
Health History beds, palms, soles of feet
A. Presenting problem 2. Infection or pallor of sclera, conjunctiva
1. Nonspecific symptoms may include chills, 3. Cyanosis
fatigue, fever, weakness, weight loss, night 4. Jaundice of skin, mucous membranes,
sweats, delayed wound healing, malaise, conjunctiva
lethargy, depression, cold/heat intolerance 5. Signs of bleeding, petechiae, ecchymoses, oral
2. Note specific signs and symptoms mucosal bleeding (especially gums), epistaxis,
a. Skin: prolonged bleeding, petechiae, hemorrhage from any orifice
jaundice, ecchymosis, pruritus, pallor 6. Ulcerations or lesions
b. Eyes: visual disturbance, yellowed sclera 7. Swelling or erythema
c. Ears: vertigo, tinnitus 8. Neurologic changes: pain and touch, position
d. Mouth and nose: epistaxis; gingival and vibratory sense, superficial and deep
bleeding, ulceration, pain; dysphagia, tendon reflexes
hoarseness C. Palpate lymph nodes; note location, size, texture,
e. Neck: nuchal rigidity, lymphadenopathy sensation, fixation; palpate the ribs for sternal,
f. Respiratory: dyspnea, orthopnea, bone tenderness.
palpitations, chest discomfort or pain, D. Evaluate joint range of motion and tenderness.
cough (productive or dry), hemoptysis E. Percuss for lung excursion, splenomegaly,
g. GI: melena, abdominal pain, change in hepatomegaly.
bowel habits
h. GU: hematuria, recurrent infection,

4
amenorrhea, menorrhagia

268 NCLEX-RN Review


53155_04_Ch04b_p264-377.qxd 2/26/09 7:32 AM Page 269

Laboratory/Diagnostic Tests Table 4-20 Normal Adult Values


A. Blood
1. Complete blood count (CBC) with differential Hematocrit
and peripheral smear Male 41.550.4%
a. White blood cell count (WBC) with Female 35.944.6%
differential Hemoglobin
b. Hgb and HCT Male 14.017.5 g/dL
c. Platelet and reticulocyte count Female 12.315.3 g/dL
d. Red blood cell count (RBC) with peripheral Red cell count 4.76.1 M/mL
smear White cell count 4.810.8 K/mL
2. Coagulation studies Mean corpuscular volume (MCV) 8199 fL
a. Prothrombin time (PT) Mean corpuscular hemoglobin (MCH) 2734 pg
b. Partial thromboplastin time (PTT) Mean corpuscular hemoglobin
c. Fibrin split products (FSP) concentration (MCHC) 3236 g/dL
d. Lee-White clotting time (whole blood Platelet count 150400 K/mL
clotting time) Neutrophils 3570%
3. Blood chemistry Lymphocytes 2545%
a. Blood urea nitrogen (BUN) Monocytes 012%
b. Creatinine Eosinophils 07%
c. Bilirubin: direct and indirect Basophils 02%
d. Uric acid
4. Miscellaneous
a. Erythrocyte sedimentation rate (ESR)
b. Serum protein electrophoresis ANALYSIS
c. Serum iron and total iron-binding
capacity Nursing diagnoses for clients with disorders of the
d. Plasma protein assays hematologic system may include:
e. Direct and indirect Coombs tests A. Imbalanced nutrition
B. Urine and stool B. Risk for infection
1. Urinalysis C. Ineffective tissue perfusion: cerebral, peripheral
2. Hematest D. Impaired gas exchange
3. Bence-Jones protein assay (urine) E. Ineffective protection
C. Radiologic F. Risk for impaired skin integrity
1. Chest or other X-ray as indicated by history G. Impaired oral mucous membrane
and physical exam H. Risk for activity intolerance
2. Radionuclide scans (e.g., bone scan) I. Acute pain
3. Lymphangiography J. Anxiety
D. Bone marrow aspiration and biopsy
1. Puncture of iliac crest (preferred site),
vertebrae body, sternum, or tibia (in infants) to PLANNING AND
collect tissue from bone marrow
2. Purpose: study cells involved in blood
IMPLEMENTATION
production
3. Nursing care Goals
a. Confirm that consent form has been A. Optimal nutrition will be maintained.
signed. B. Client will be free from infection.
b. Allay client anxiety; prepare client for a C. Adequate cerebral and peripheral tissue perfusion
sharp, brief pain when bone marrow is will be maintained.
aspirated into syringe. D. Client will maintain optimal respiratory function.
c. Position client and assist physician to E. Client will maintain adequate protective
maintain sterile field. mechanisms.
d. Immediately after the aspiration, apply F. Optimal skin integrity will be maintained.
pressure to the site for at least 5 minutes G. Client maintains optimal health of oral mucous
and longer, if necessary. membranes.
e. Check the site frequently for signs of H. Client will have increased strength and endurance.
bleeding or infection. I. Clients pain will be relieved/controlled.
f. Send specimen to laboratory. J. Clients anxiety will be relieved/reduced.
E. See Table 4-20

ADULT NURSING

4 269
53155_04_Ch04b_p264-377.qxd 2/26/09 7:32 AM Page 270

Interventions of meperidine IV until rigors are


diminished, and resumption of transfusion
Blood Transfusion and Component Therapy when symptoms relieved
7. Volume expanders: albumin; percentage
A. Purpose: improve oxygen transport (RBCs); volume
concentration varies (50100 mL/unit);
expansion (whole blood, plasma, albumin);
hyperosmolar solutions should not be used in
provision of proteins (fresh frozen plasma,
dehydrated clients.
albumin, plasma protein fraction); provision of
C. Nursing care
coagulation factors (cryoprecipitate, fresh frozen
1. Assess client for history of previous blood
plasma, fresh whole blood); provision of platelets
transfusions and any adverse reactions.
(platelet concentrate, fresh whole blood)
2. Ensure that the adult client has an 1820 gauge
B. Blood and blood products
IV catheter in place.
1. Whole blood; provides all components
3. Use 0.9% sodium chloride.
a. Large volume can cause difficulty:
4. At least two nurses should verify the ABO
1224 hours for Hgb and HCT to rise
group, Rh type, client and blood numbers, and
b. Complications: volume overload,
expiration date.
transmission of hepatitis or AIDS,
5. Take baseline vital signs before initiating
transfusion reaction, infusion of excess
transfusion.
potassium and sodium, infusion of
6. Start transfusion slowly (2 mL/minute).
anticoagulant (citrate) used to keep stored
7. Stay with the client during the first 15 minutes
blood from clotting, calcium binding and
of the transfusion and take vital signs
depletion (citrate) in massive transfusion
frequently.
therapy
8. Maintain the prescribed transfusion rate.
2. Red blood cells
a. Whole blood: approximately 34 hours
a. Provide twice the amount of Hgb as an
b. RBCs: approximately 24 hours
equivalent amount of whole blood
c. Fresh frozen plasma: as quickly as possible
b. Indicated in cases of blood loss, pre- and
d. Platelets: as quickly as possible
post-op clients, and those with incipient
e. Cryoprecipitate: rapid infusion
congestive failure
f. Granulocytes: usually over 2 hours
c. Complications: transfusion reaction (less
g. Volume expanders: volume-dependent rate
common than with whole blood due to
9. Monitor for adverse reactions (see Table 4-19).
removal of plasma proteins)
10. Document the following:
3. Fresh frozen plasma
a. Blood component unit number (apply
a. Contains all coagulation factors including
sticker if available)
V and VIII
b. Date infusion starts and ends
b. Can be stored frozen for 12 months; takes
c. Type of component and amount transfused
20 minutes to thaw
d. Client reaction and vital signs
c. Hang immediately upon arrival to unit
e. Signature of transfusionist
(loses its coagulation factors rapidly)
f. If a reaction occurs, follow facility protocol
4. Platelets
for blood packaging and assessing client.
a. Will raise recipients platelet count by
10,000/mm3
b. Pooled from 48 units of whole blood
c. Single-donor platelet transfusions may be
EVALUATION
necessary for clients who have developed
A. Client maintains normal weight; no evidence of
antibodies; compatibility testing may be
malnutrition.
necessary
B. Clients temperature is within normal range; no
5. Factor VIII fractions (cryoprecipitate): contains
signs of infection.
Factors VIII, fibrinogen, and XIII
C. Client neuro status is within normal limits.
6. Granulocytes
D. Client demonstrates adequate peripheral capillary
a. Do not increase WBC; increase marginal pool
refill, sensation, and movement; palpable
(at tissue level) rather than circulating pool
peripheral pulses; skin warm, dry, and usual color.
b. Premedication with steroids,
E. Clients respirations are of normal rate, rhythm,
antihistamines, and acetaminophen
and depth; lungs clear to auscultation.
c. Respiratory distress with shortness of
F. Client verbalizes signs and symptoms of infection
breath, cyanosis, and chest pain may
and preventive measures; reports signs and
occur; requires cessation of transfusion and
symptoms of infection immediately.
immediate attention
G. Clients skin remains clear and intact.
d. Shaking chills or rigors common, require
H. Clients oral mucous membranes are healthy and
brief cessation of therapy, administration

4
intact.

270 NCLEX-RN Review


53155_04_Ch04b_p264-377.qxd 2/26/09 7:32 AM Page 271

I. Client experiences increased strength and possible bleeding (helps allay anxiety and
endurance. ensure cooperation).
J. Client reports relief/control of pain. 4. Administer iron preparations as ordered.
K. Client expresses relief/reduction in anxiety. a. Oral iron preparations: route of choice
1) Give following meals or a snack.
2) Dilute liquid preparations well and
DISORDERS OF THE administer using a straw to prevent
staining teeth.
HEMATOLOGIC SYSTEM 3) When possible administer with orange
juice as vitamin C (ascorbic acid)
Anemias enhances iron absorption.
4) Warn clients that iron preparations
Iron-Deficiency Anemia will change stool color and
A. General information consistency (dark and tarry) and may
1. Chronic microcytic, hypochromic anemia cause constipation.
caused by either inadequate absorption or 5) Antacid ingestion will decrease oral
excessive loss of iron iron effectiveness; milk products and
2. Acute or chronic bleeding principal cause in eggs inhibit absorption.
adults (chiefly from trauma, dysfunctional b. Parenteral: used in clients intolerant to oral
uterine bleeding, and GI bleeding) preparations, who are noncompliant with
3. May also be caused by inadequate intake of therapy, or who have continuing blood
iron-rich foods or by inadequate absorption of losses.
iron (from chronic diarrhea, malabsorption 1) Use one needle to withdraw and
syndromes, high cereal-product intake with another to administer iron
low animal protein ingestion, partial or preparations as tissue staining and
complete gastrectomy, pica) irritation are a problem.
4. Incidence related to geographic location, 2) Use the Z-track injection technique to
economic class, age group, and sex prevent leakage into tissues
a. More common in developing countries and 3) Do not massage injection site but
tropical zones (blood-sucking parasites) encourage ambulation as this will
b. Women between ages 15 and 45 and enhance absorption; advise against
children affected more frequently, as are vigorous exercise and constricting
the poor garments.
5. In iron-deficiency states, iron stores are 4) Observe for local signs of
depleted first, followed by a reduction in Hgb complications: pain at the injection
formation. site, development of sterile abscesses,
B. Assessment findings lymphadenitis as well as fever,
1. Mild cases usually asymptomatic headache, urticaria, hypotension, or
2. Palpitations, dizziness, and cold sensitivity anaphylactic shock.
3. Brittleness of hair and nails; pallor 5. Provide dietary teaching regarding food high
4. Dysphagia, stomatitis, and atrophic glossitis in iron (meats, fortified cereals, nuts, seeds,
5. Dyspnea, weakness dried beans, dried fruit).
6. Laboratory findings 6. Encourage ingestion of roughage and increase
a. RBCs small (microcytic) and pale fluid intake to prevent constipation if oral iron
(hypochromic) preparations are being taken.
b. Hgb markedly decreased
c. HCT moderately decreased Pernicious Anemia
d. Serum iron markedly decreased
A. General information
e. Hemosiderin absent from bone marrow
1. Chronic progressive, macrocytic anemia
f. Serum ferritin decreased
caused by a deficiency of intrinsic factor; the
g. Reticulocyte count decreased
result is abnormally large erythrocytes and
C. Nursing interventions
hypochlorhydria (a deficiency of hydrochloric
1. Monitor for signs and symptoms of bleeding
acid in gastric secretions)
through hematest of all elimination including
2. Characterized by neurologic and GI symptoms;
stool, urine, and gastric contents.
death usually results if untreated
2. Provide for adequate rest: plan activities so as
3. Lack of intrinsic factor is caused by gastric
not to overtire.
mucosal atrophy (possibly due to heredity,
3. Provide a thorough explanation of all
prolonged iron deficiency, or an autoimmune
diagnostic tests used to determine sources of

4
disorder); can also result in clients who have

ADULT NURSING 271


53155_04_Ch04b_p264-377.qxd 2/26/09 7:32 AM Page 272

had a total gastrectomy if vitamin B12 not nonradioactive vitamin B12 IM to


administered saturate tissue binding sites and to
4. Usually occurs in men and women over age permit some excretion of radioactive
50, with an increase in blue-eyed persons of vitamin B12 in the urine if it is absorbed
Scandinavian descent 5) 2448 hour urine collection is
5. Pathophysiology obtained; client is encouraged to drink
a. Intrinsic factor is necessary for the fluids
absorption of vitamin B12 into the small 6) If indicated, second stage Schilling test
intestine. performed 1 week after first stage.
b. B12 deficiency diminishes DNA synthesis, Fasting client is given radioactive
which results in defective maturation of vitamin B12 combined with human
cells (particularly rapidly dividing cells intrinsic factor and test is repeated.
such as blood cells and GI tract cells). g. Gastric analysis: decreased free
c. B12 deficiency can alter structure and hydrochloric acid
function of peripheral nerves, spinal cord, h. Large numbers of reticulocytes in the blood
and the brain. following parenteral vitamin B12
B. Medical management administration
1. Drug therapy D. Nursing interventions
a. Vitamin B12 injections: monthly 1. Provide a nutritious diet high in iron, protein,
maintenance and vitamins (fish, meat, milk/milk products,
b. Iron preparations (if Hgb level inadequate and eggs).
to meet increased numbers of erythrocytes) 2. Avoid highly seasoned, coarse, or very hot
c. Folic acid foods if client has mouth sores.
1) Controversial 3. Provide mouth care before and after meals
2) Reverses anemia and GI symptoms but using a soft toothbrush and nonirritating
may intensify neurologic symptoms rinses.
3) May be safe if given in small amounts 4. Bed rest may be necessary if anemia is severe.
in addition to vitamin B12 5. Provide safety when ambulating (especially if
2. Transfusion therapy carrying hot items, etc.).
C. Assessment findings 6. Provide client teaching and discharge
1. Anemia, weakness, pallor, dyspnea, planning concerning:
palpitations, fatigue a. Dietary instruction
2. GI symptoms: sore mouth; smooth, beefy, red b. Importance of lifelong vitamin B12 therapy
tongue; weight loss; dyspepsia; constipation or c. Rehabilitation and physical therapy for
diarrhea; jaundice neurologic deficits, as well as instruction
3. CNS symptoms; tingling, paresthesias of hands regarding safety
and feet, paralysis, depression, psychosis
4. Laboratory tests Aplastic Anemia
a. Erythrocyte count decreased
b. Blood smear: oval, macrocytic erythrocytes A. General information
with a proportionate amount of Hgb 1. Pancytopenia or depression of granulocyte,
c. Bone marrow platelet, and erythrocyte production due to
1) Increased megaloblasts (abnormal fatty replacement of the bone marrow
erythrocytes) 2. Bone marrow destruction may be idiopathic or
2) Few normoblasts or maturing secondary
erythrocytes 3. Secondary aplastic anemia may be caused by
3) Defective leukocyte maturation a. Chemical toxins (e.g., benzene)
d. Bilirubin (indirect): elevated unconjugated b. Drugs (e.g., chloramphenicol, cytotoxic
fraction drugs)
e. Serum LDH elevated c. Radiation
f. Positive Schilling test d. Immunologic injury
1) Measures absorption of radioactive B. Medical management
vitamin B12 both before and after 1. Blood transfusions: key to therapy until
parenteral administration of intrinsic clients own marrow begins to produce blood
factor cells
2) Definitive test for pernicious anemia 2. Aggressive treatment of infections
3) Used to detect lack of intrinsic factor 3. Bone marrow transplantation
4) Fasting client is given radioactive 4. Drug therapy
vitamin B12 by mouth and a. Corticosteroids and/or androgens to

4
stimulate bone marrow function and to

272 NCLEX-RN Review


53155_04_Ch04b_p264-377.qxd 2/26/09 7:32 AM Page 273

increase capillary resistance (effective in and a corresponding compensatory effort by


children but usually not in adults) bone marrow to increase production.
b. Estrogen and/or progesterone to prevent B. Medical management
amenorrhea in female clients 1. Identify and eliminate (if possible) causative
5. Identification and withdrawal of offending factors
agent or drug 2. Drug therapy
C. Assessment findings a. Corticosteroids in autoimmune types of
1. Fatigue, dyspnea, pallor anemia
2. Increased susceptibility to infection b. Folic acid supplements
3. Bleeding tendencies and hemorrhage 3. Blood transfusion therapy
4. Laboratory findings: normocytic anemia, 4. Splenectomy
granulocytopenia, thrombocytopenia C. Assessment findings
5. Bone marrow biopsy: marrow is fatty and 1. Clinical manifestations vary depending on
contains very few developing cells severity of anemia and the rate of onset (acute
D. Nursing interventions vs. chronic)
1. Administer blood transfusions as ordered. 2. Pallor, scleral icterus, and slight jaundice
2. Provide nursing care for client with bone (chronic)
marrow transplantation. 3. Chills, fever, irritability, precordial spasm, and
3. Administer medications as ordered. pain (acute)
4. Monitor for signs of infection and provide care 4. Abdominal pain and nausea, vomiting,
to minimize risk. diarrhea, melena
a. Maintain neutropenic precautions. 5. Hematuria, marked jaundice, and dyspnea
b. Encourage high-protein, high-vitamin diet 6. Splenomegaly and symptoms of cholelithiasis,
to help reduce incidence of infection. hepatomegaly
c. Provide mouth care before and after meals. 7. Laboratory tests
5. Monitor for signs of bleeding and provide a. Hgb and HCT decreased
measures to minimize risk. b. Reticulocyte count elevated
a. Use a soft toothbrush and electric razor. (compensatory)
b. Avoid intramuscular injections. c. Coombs test (direct): positive if
c. Hematest urine and stool. autoimmune features present
d. Observe for oozing from gums, petechiae, d. Bilirubin (indirect): elevated unconjugated
or ecchymoses. fraction
6. Provide client teaching and discharge D. Nursing interventions
planning concerning 1. Monitor for signs and symptoms of hypoxia
a. Self-care regimen including confusion, cyanosis, shortness of
b. Identification of offending agent and breath, tachycardia, and palpitations.
importance of avoiding it (if possible) in 2. Note that the presence of jaundice may make
future assessment of skin color in hypoxia unreliable.
3. If jaundice and associated pruritus are present,
Hemolytic Anemia avoid soap during bathing and use cool or
tepid water.
A. General information 4. Frequent turning and meticulous skin care are
1. A category of diseases in which there is an important as skin friability is increased.
increased rate of RBC destruction. 5. Teach clients about the nature of the disease
2. May be congenital or acquired. and identification of factors that predispose to
a. Congenital: includes hereditary episodes of hemolytic crisis.
spherocytosis, G6PD deficiency, sickle cell
anemia, thalassemia Splenectomy
b. Acquired: includes transfusion
incompatibilities, thrombotic A. General information
thrombocytopenic purpura, disseminated 1. Indications
intravascular clotting, spur cell anemia a. Rupture of the spleen caused by trauma,
3. Cause often unknown, but erythrocyte life span accidental tearing during surgery, diseases
is shortened and hemolysis occurs at a rate that causing softening or damage (e.g.,
the bone marrow cannot compensate for. infectious mononucleosis)
4. The degree of anemia is determined by the lag b. Hypersplenism: excessive splenic damage
between erythrocyte hemolysis and the rate of of cellular blood components
bone marrow erythropoiesis. c. As the spleen is a major source of antibody
5. Diagnosis is based on laboratory evidence of formation in children, splenectomy is not

4
an increased rate of erythrocyte destruction recommended during the early years of

ADULT NURSING 273


53155_04_Ch04b_p264-377.qxd 2/26/09 7:32 AM Page 274

life; if absolutely necessary, client should a. Underlying disease (e.g., toxemia of


receive prophylactic antibiotics post-op pregnancy, cancer) causes release of
2. Primary hypersplenism can be alleviated with thromboplastic substances that promote
splenectomy; procedure is palliative only in the deposition of fibrin throughout the
secondary hypersplenism microcirculation.
B. Nursing interventions: postoperatively b. Microthrombi form in many organs,
1. Provide routine preoperative care and explain causing microinfarcts and tissue necrosis.
what to expect postoperatively. c. RBCs are trapped in fibrin strands and are
2. Administer pneumococcal vaccine as ordered hemolysed.
since client will be at increased risk for d. Platelets, prothrombin, and other clotting
pneumococcal infections for several years after factors are destroyed, leading to bleeding.
splenectomy. e. Excessive clotting activates the fibrinolytic
C. Nursing interventions: postoperatively system, which inhibits platelet function,
1. Be aware that it is crucial to monitor carefully causing further bleeding.
for hemorrhage and shock as clients with pre- 7. Mortality rate is high, usually because
op bleeding tendencies will remain at risk underlying disease cannot be corrected.
post-op. B. Medical management
2. Monitor post-op temperature elevation: fever 1. Identification and control of underlying
may not be the best indicator of post-op disease is key
complications such as pneumonia or urinary 2. Blood transfusions: include whole blood,
tract infection, as fever without concomitant packed RBCs, platelets, plasma,
infection is common following splenectomy. cryoprecipitates, and volume expanders
3. Observe for abdominal distension and 3. Heparin administration
discomfort secondary to expansion of the a. Somewhat controversial
intestines and stomach; an abdominal binder b. Inhibits thrombin thus preventing further
may reduce distension. clot formation, allowing coagulation
4. Know that postoperative infection in a child is factors to accumulate
considered life threatening; administer C. Assessment findings
prophylactic antibiotics as ordered. 1. Petechiae and ecchymoses on the skin, mucous
5. Ambulate early and provide chest physical membranes, heart, lungs, and other organs
therapy as location of the incision makes post- 2. Prolonged bleeding from breaks in the skin
op atelectasis or pneumonia a risk. (e.g., IV or venipuncture sites)
6. Emphasize to client the need to report even 3. Severe and uncontrollable hemorrhage during
minor signs or symptoms of infection childbirth or surgical procedures
immediately to the physician. 4. Mental status changes
5. Oliguria and acute renal failure
Sickle Cell Anemia 6. Convulsions, coma, death
7. Laboratory findings
See Unit 5 a. PT prolonged
b. PTT usually prolonged
Disorders of Platelets and c. Thrombin time usually prolonged
d. Fibrinogen level usually decreased
Clotting Mechanism e. Platelet count usually depressed
Disseminated Intravascular f. Fibrin split products elevated
g. Protamine sulfate test strongly positive
Coagulation (DIC)
h. Factor assays (II, V, VII) depressed
A. General information D. Nursing interventions
1. Diffuse fibrin deposition within arterioles and 1. Monitor blood loss and attempt to quantify.
capillaries with widespread coagulation all 2. Observe for signs of additional bleeding or
over the body and subsequent depletion of thrombus formation.
clotting factors. 3. Monitor appropriate laboratory data.
2. Hemorrhage from kidneys, brain, adrenals, 4. Prevent further injury.
heart, and other organs. a. Avoid IM injections.
3. Cause unknown. b. Apply pressure to bleeding sites.
4. Clients are usually critically ill with an c. Turn and position client frequently and
obstetric, surgical, hemolytic, or neoplastic gently.
disease. d. Provide frequent nontraumatic mouth care
5. May be linked with entry of thromboplastic (e.g., soft toothbrush or gauze sponge).
substances into the blood. 5. Provide emotional support to client and

4
6. Pathophysiology significant others.

274 NCLEX-RN Review


53155_04_Ch04b_p264-377.qxd 2/26/09 7:32 AM Page 275

6. Administer blood transfusions and 3. Drugs used to treat Pneumocystis carinii


medications as ordered. pneumonia (PCP) include:
7. Teach client the importance of avoiding a. PO or IV trimethoprim-sulfamethoxazole
aspirin or aspirin-containing compounds. (Bactrim, Septra); side effects include rash,
leukopenia, fever
Hemophilia b. IM or IV pentamidine (Pentam 300); side
effects include hepatotoxicity,
See Unit 5. nephrotoxicity, blood sugar imbalances,
abscess or necroses at IM injection site,
Idiopathic Thrombocytopenic Purpura hypotension
c. Corticosteroids are also frequently used.
See Unit 5. C. Assessment findings (see Table 4-22)
1. Fatigue, weakness, anorexia, weight loss,
Immunologic Disorders diarrhea, pallor, fever, night sweats
2. Shortness of breath, dyspnea, cough, chest
Acquired Immune Deficiency pain, and progressive hypoxemia secondary to
Syndrome (AIDS) infection (pneumonia)
3. Progressive weight loss secondary to anorexia,
A. General information
nausea, vomiting, diarrhea, and a general
1. Characterized by severe deficits in cellular
wasting syndrome; fatigue, malaise
immune function; manifested clinically by
4. Temperature elevations (persistent or
opportunistic infections and/or unusual
intermittent); night sweats
neoplasms
5. Neurologic dysfunction secondary to acute
2. Etiologic factors
meningitis, progressive dementia,
a. Results from infection with human
encephalopathy, encephalitis
immunodeficiency virus (HIV), a retrovirus
6. Presence of opportunistic infection, for example
that preferentially infects helper
a. Pneumocystis carinii pneumonia
T-lymphocytes (T4 cells)
b. Herpes simplex, cytomegalovirus, and
b. Transmissible through sexual contact,
Epstein-Barr viruses
contaminated blood or blood products, and
from infected woman to child in utero or
possibly through breast-feeding
Table 4-21 Antiretroviral Drug Classifications
c. HIV is present in an infected persons
blood, semen, and other body fluids Nucleoside Reverse Transcriptase Inhibitors (NRTIs)
3. Epidemiology is similar to that of hepatitis B; Zidovudine (AZT, ZDV, Retrovir)the first developed drug
increased incidence in populations in which Didanosine (ddl, Videx)
sexual promiscuity is common and in IV drug Stavudine (d4T, Zerit)
abusers Lamivudine (3TC, Epivir)
4. Proposed strategies for prevention Abacavir (Ziagen)
a. Early detection; include HIV testing as
routine part of medical care Nucleotide Reverse Transcriptase Inhibitor
b. Expand opportunities for testing outside of Tenofovir DF (viread)
the medical settings Nonnucleoside Reverse Transcriptase
c. Behavior modification with persons Inhibitors (NNRTIs)
diagnosed with HIV and partners Nevirapine (Viramune)
d. Reduce viral load in pregnant woman with Delavirdne (Rescriptor)
HIV; reduce perinatal transmission of HIV Efavirenz (Sustival)
disease to newborn
B. Medical management Protease Inhibitors (PIs)
1. No effective cure for AIDS at present; several Saquinavir (Fortovase)
categories of antiretroviral drugs are available Indinavir (Crixivan)
(see Table 4-21). Ritonavir (Norvir)
2. Highly active antiretroviral therapy (HAART) Nelfinavir (Viracept)
refers to combination of antiretroviral drugs to Amprenavir (Agenerase)
avoid development of viral resistance to drugs; Kaletra (combination of lopinavir and ritonvir)
drug therapy is initiated as early as possible to Fusion Inhibitor (Entry Inhibitor)
reduce HIV RNA levels in blood and to Enfuvirtide (Fuzeon)
maintain or increase CD4+ levels to greater
Adapted from Broyles, B.E., Reiss, B.S., & Evans, M.E. (2007).
than 200 cells/L
Pharmacological aspects of nursing care (7th ed.). New York:

4
Thomson Delmar Learning.

ADULT NURSING 275


53155_04_Ch04b_p264-377.qxd 2/26/09 7:32 AM Page 276

Table 4-22 Classification System for HIV Infection

A B C
CD4 + T-cell Symptomatic, not (A) or
categories Asymptomatic, acute HIV or PGL (C) conditions AIDS-indicator conditions
(1) 500/uL A1 B1 C1
(2) 200499/uL A2 B2 C2
(3) < 200/uL A3 B3 C3
Clinical Category A Clinical Category B Clinical Category C
1 or more of the following, confirmed Candidiasis (oral or vaginal), frequent Candidiasis of bronchi, trachea, or lungs
HIV infection, and without conditions or poorly resistant to therapy Cervical cancer, invasive
in B and C Cervical dysplasia/cervical carcinoma Coccidiomycosis
Asymptomatic HIV infection in situ Cryptosporidiosis
Persistent Generalized Fever or diarrhea exceeding 1 month Cytomegalovirus
Lymphadenopathy (PGL) Hairy leukoplakia, oral Encephalopathy
Acute (primary) HIV infection with Herpes zoster, involving 2 episodes Herpes simplex: chronic ulcerexceeding
accompanying illness or history or more than one dermatome 1 month duration
of acute HIV infection ITP Histoplasmosis
PID Kaposis sarcoma
Peripheral neuropathy Lymphoma
Mycobacteriumavium complex
Mycobacterium tuberculosis
Pneumocystis carinii pneumonia
Salmonella
Toxoplasmosis of brain
Wasting syndrome due to HIV
NOTE: Adapted from 1993 Revised Classification System for HIV Infections and Expanded Surveillance Case Definition for AIDS
Among Adolescents and Adults, by Centers for Disease Control and Prevention, U.S. Department of Health and Human Services,
1993, Atlanta, GA: Author.

c. Candidiasis: oral or esophageal 3. Assess neurological status; reorient client as


d. Mycobacterium-avium complex needed; provide safety measures for the
7. Neoplasms confused/disoriented client.
a. Kaposis sarcoma 4. Assess for signs and symptoms of fluid and
b. CNS lymphoma electrolyte imbalances; monitor lab studies;
c. Burkitts lymphoma ensure adequate hydration.
d. Diffuse undifferentiated non-Hodgkins 5. Monitor clients nutritional intake; provide
lymphoma supplements, total parenteral nutrition, etc., as
8. Laboratory findings: diagnosis based on ordered.
clinical criteria and positive HIV antibody 6. Assess skin daily (especially perianal area) for
testELISA (enzyme-linked immunosorbent signs of breakdown; keep skin clean and dry;
assay) confirmed by Western blot assay. Other turn q4 h while in bed.
lab findings may include 7. Inspect oral cavity daily for ulcerations, signs
a. Leukopenia with profound lymphopenia of infection; instruct client to rinse mouth
b. Anemia with normal saline and hydrogen peroxide or
c. Thrombocytopenia normal saline and sodium bicarbonate rinses.
d. Decreased circulatory CD4 lymphocyte 8. Observe for signs and symptoms of infection;
cells report immediately if any occur.
e. Low CD4:CD8 lymphocyte ratio 9. If severe leukopenia develops, institute
D. Nursing interventions neutropenic precautions
1. Administer medications as ordered for a. Prevent trauma to skin and mucous
concomitant disease; monitor for signs of membranes, e.g., avoid enemas, rectal
medication toxicity. temperatures; minimize all parenteral
2. Monitor respiratory status; provide care infections
as appropriate for respiratory problems, b. Do not place client in a room with clients

4
e.g., pneumonia. having infections

276 NCLEX-RN Review


53155_04_Ch04b_p264-377.qxd 2/26/09 7:32 AM Page 277

c. Screen visitors for colds, infections, etc. B. Medical management


d. Do not allow fresh fruits, vegetables, or 1. Drug therapy
plants in clients room. a. Analgesics for bone pain
e. Mask client when leaving room for walks, b. Chemotherapy (melphalan [Alkeran] and
X-rays, etc. cyclophosphamide [Cytoxan]) to reduce
10. Institute blood and body fluid precautions tumor mass; may intensify the
11. Provide emotional support for pancytopenia to which these clients are
client/significant others; help to decrease prone; requires careful monitoring of
sense of isolation laboratory studies
12. Provide client teaching and discharge c. Antibiotics to treat infections
planning concerning d. Gammaglobulin for infection prophylaxis
a. Importance of observing for signs of e. Corticosteroids and mithramycin for severe
infections and notifying physician hypercalcemia
immediately if any occur 2. Radiation therapy to reduce tumor mass and
b. Ways to reduce chance of infection for palliation of bone pain
1) Clean kitchen and bathroom surfaces 3. Transfusion therapy
regularly with disinfectants. C. Assessment findings
2) Avoid direct contact with pets litter 1. Headache and bone pain increasing with
boxes or stool, bird cage droppings, activity
and water in fish tanks. 2. Pathologic fractures
3) Avoid contact with people with 3. Skeletal deformities of sternum and ribs
infections, e.g., cold, flu. 4. Loss of height (spinal column shortening)
4) Importance of balancing activity with 5. Osteoporosis
rest. 6. Renal calculi
5) Need to eat a well-balanced diet with 7. Anemia, hemorrhagic tendencies, and
plenty of fluids. increased susceptibility to infection
c. Prevention of disease transmission 8. Hypercalcemia
1) Use safer sex practices, e.g., condoms 9. Renal dysfunction secondary to obstruction of
for sexual intercourse. convoluted tubules by coagulated protein
2) Do not donate blood, semen, organs. particles
3) Do not share razors, toothbrushes, or 10. Neurologic dysfunction: spinal cord
other items that may draw blood. compression and paraplegia
4) Inform all physicians, dentists, sexual 11. Laboratory tests
partners of diagnosis. a. Radiologic: diffuse bone lesions,
d. Resources include Public Health Service, widespread demineralization,
National Gay Task Force, American Red osteoporosis, osteolytic lesions of skull
Cross, local support groups b. Bone marrow; many immature plasma
cells; depletion of other cell types
c. CBC: reduced Hgb, WBC, and platelet
Malignancies counts
Multiple Myeloma d. Serum globulins elevated
e. Bence-Jones protein: positive (abnormal
A. General information globulin that appears in the urine of clients
1. A neoplastic condition characterized by the with multiple myeloma and other bone
abnormal proliferation of plasma cells in the tumors)
bone marrow, causing the development of single D. Nursing interventions
or multiple tumors composed of abnormal 1. Provide comfort measures to help alleviate
plasma cells. Disease disseminates into lymph bone pain.
nodes, liver, spleen, and kidneys and causes 2. Encourage ambulation to slow
bone destruction throughout the body. demineralization process.
2. Cause unknown, but environmental factors 3. Promote safety as clients are prone to
thought to be involved pathologic and other fractures.
3. Disease occurs after age 40; affects men twice 4. Encourage fluids: 30004000 mL/day to
as often as women counteract calcium overload and to prevent
4. Pathophysiology protein from precipitating in the renal tubules.
a. Bone demineralization and destruction 5. Provide nursing care for clients with bleeding
with osteoporosis and a negative calcium tendencies and susceptibility to infection.
balance 6. Provide a supportive atmosphere to enhance
b. Disruption of erythrocyte, leukocyte, and communication and reduce anxiety.

4
thrombocyte production

ADULT NURSING 277


53155_04_Ch04b_p264-377.qxd 2/26/09 7:32 AM Page 278

7. Provide client teaching and discharge d. Bilirubin (indirect): increase in


planning concerning unconjugated fraction
a. Crucial importance of long-term hydration e. Liver enzymes may be increased
to prevent urolithiasis and renal f. Uric acid increased
obstruction g. Hematuria and melena possible
b. Safety measures vital to decrease the risk C. Nursing interventions
of injury 1. Monitor for signs and symptoms of bleeding
c. Avoidance of crowds or sources of complications.
infection if leukopenic 2. Force fluids and record I&O.
3. Prevent development of DVT.
Polycythemia Vera 4. Monitor for signs and symptoms of CHF.
5. Provide care for the client having a phlebotomy.
A. General information 6. Prevent/provide care for bleeding or infection
1. An increase in both the number of circulating complications.
erythrocytes and the concentration of Hgb 7. Administer medications as ordered.
within the blood a. Radioactive phosphorus (32P): reduction of
2. Three forms: polycythemia vera, secondary erythrocyte production, produces a
polycythemia, and relative polycythemia remission of 6 months to 2 years
3. Classified as a myeloproliferative disorder b. Nitrogen mustard, busulfan (Myleran),
(bone marrow overgrowth) chlorambucil, cyclophosphamide to effect
4. Cause unknown, but thought to be a form of myelosuppression
malignancy similar to leukemia c. Antigout and peptic ulcer drugs as needed.
5. Usually develops in middle age, common in 8. Provide client teaching and discharge
Jewish men planning concerning
6. Pathophysiology a. Decrease in activity tolerance, need to
a. A pronounced increase in the production of space activity with periods of rest
erythrocytes accompanied by an increase in b. Phlebotomy regimens: outclient frequency
the production of myelocytes (leukocytes is determined by HCT; importance of long-
within bone marrow) and thrombocytes. term therapy
b. The consequences of this overproduction c. High fluid intake
are an increase in blood viscosity, an d. Avoidance of iron-rich foods to avoid
increase in total blood volume (23 times counteracting the therapeutic effects of
greater than normal), and severe congestion phlebotomy
of all tissues and organs with blood. e. Recognition and reporting of bleeding
B. Assessment findings f. Need to avoid persons with infections,
1. Ruddy complexion and duskiness of mucosa especially in leukopenic clients.
secondary to capillary congestion in the skin
and mucous membranes Leukemia
2. Hypertension associated with vertigo,
headache, and fullness in the head See Unit 5.
secondary to increased blood volume
3. Symptoms of HF secondary to overwork of the Hodgkins and Non-Hodgkins Lymphoma
heart
4. Thrombus formation: CVA, MI, gangrene of the See Unit 5.
extremities, DVT, and pulmonary embolism
can occur
5. Bleeding and hemorrhage secondary to
congestion and overdistension of capillaries Sample Questions
and venules
6. Hepatomegaly and splenomegaly
7. Peptic ulcer secondary to increased gastric 218. A client is admitted to the hospital with a bleeding
secretions ulcer and is to receive 4 units of packed cells.
8. Gout secondary to increased uric acid released Which nursing intervention is of PRIMARY
by nucleoprotein breakdown importance in the administration of blood?
9. Laboratory tests 1. Checking the flow rate.
a. CBC: increase in all mature cell forms 2. Identifying the client.
(erythrocytes, leukocytes, and platelets)
3. Monitoring the vital signs.
b. HCT: increased
c. Bone marrow: increase in immature cell 4. Maintaining blood temperature.

4
forms

278 NCLEX-RN Review


53155_04_Ch04b_p264-377.qxd 2/26/09 7:32 AM Page 279

219. Within 20 minutes of the start of transfusion, the 225. A client has the following blood lab values:
client develops a sudden fever. What is the Platelets 50,000/uL
nurses first action? RBCs 3.5 (3 106)
1. Force fluids. Hemoglobin 10 g/dL
2. Continue to monitor the vital signs. Hematocrit 30%
3. Increase the flow rate of IV fluids. WBCs 10,000/uL
4. Stop the transfusion. Which nursing instruction should be included
in the teaching plan?
220. A male client who is HIV positive is admitted to the
hospital with a diagnosis of Pneumocystis carinii 1. Bleeding precautions.
pneumonia. His live-in partner has accompanied 2. Seizure precautions.
him. During the history interview, the nurse is 3. Isolation to prevent infection.
aware of feeling a negative attitude about the clients 4. Control of pain with analgesics.
lifestyle, what action is most appropriate?
1. Share these feelings with the client. 226. A hospitalized client has the following blood lab
2. Develop a written interview form. values:
3. Avoid eye contact with the client. WBC 3,000/uL
4. Discuss the negative feelings with the charge RBC 5.0 (3 106)
nurse. platelets 300,000
What would be a priority nursing intervention?
221. What should the client at risk for developing
1. Preventing infection.
AIDS be advised to do?
2. Controlling blood loss.
1. Abstain from anal intercourse.
3. Alleviating pain.
2. Have an ELISA test for antibodies.
4. Monitoring blood transfusion reactions.
3. Have a semen analysis done.
4. Inform all sexual contacts. 227. A mans blood type is AB and he requires a
blood transfusion. To prevent complications of
222. A client who is HIV positive should have the blood incompatibilities, which blood type(s)
mouth examined for which oral problem may the client receive?
common associated with AIDS?
1. Type A or B blood only.
1. Halitosis.
2. Type AB blood only.
2. Carious teeth.
3. Type O blood only.
3. Creamy white patches.
4. Either type A, B, AB, or O blood.
4. Swollen lips.
228. Which nursing intervention is appropriate for
223. The nurse is caring for a client who is HIV the nurse to take when setting up supplies for a
positive. To prevent the spread of the HIV virus, client who requires a blood transfusion?
what do the Centers for Disease Control and
1. Add any needed IV medication in the blood
Prevention (CDC) recommend?
bag within one-half hour of planned infusion.
1. Universal blood and body fluid precautions.
2. Obtain blood bag from laboratory and leave at
2. Laminar flow rooms during active infection. room temperature for at least one hour prior
3. Body systems isolation. to infusion.
4. Needle and syringe precautions. 3. Prime tubing of blood administration set with
0.9% NS solution, completely filling filter.
224. An adult has been diagnosed with some type of
4. Use a small-bore catheter to prevent rapid
anemia. The results of his blood tests showed:
infusion of blood products that may lead to a
decreased WBC, normal RBC, decreased HCT,
reaction.
decreased Hgb. Based on these data, which of
the following nursing diagnoses should the 229. A client who is receiving a blood transfusion
nurse prioritize as being the most important? begins to experience chills, shortness of breath,
1. Potential for infection. nausea, excessive perspiration, and a vague sense
2. Alteration in nutrition. of uneasiness. What is the nurses first best action?
3. Self-care deficit. 1. Report the signs and symptoms to the physician.
2. Stop the transfusion.

4
4. Fluid volume excess.

ADULT NURSING 279


53155_04_Ch04b_p264-377.qxd 2/26/09 7:32 AM Page 280

3. Monitor the clients vital signs. 235. Which of the following lab value profiles should
4. Assess respiratory status. the nurse know to be consistent with hemolytic
anemia?
230. A client with iron deficiency anemia is ordered 1. Increased RBC, decreased bilirubin,
parenteral iron to be given intramuscularly. decreased hemoglobin and hematocrit,
Which of the following actions should the nurse increased reticulocytes.
take in the preparation/administration of this 2. Decreased RBC, increased bilirubin,
medication? decreased hemoglobin and hematocrit,
1. Use the same large (1920) gauge needle for increased reticulocytes.
drawing up the medication and injecting it. 3. Decreased RBC, decreased bilirubin,
2. Inject medication into the upper arm muscle. increased hemoglobin and hematocrit,
3. Use a 1-inch needle to administer the decreased reticulocytes.
medication. 4. Increased RBC, increased bilirubin, increased
4. Use the Z-track technique to administer the hemoglobin and hematocrit, decreased
medication. reticulocytes.

231. The nurse has been teaching an adult who has 236. A client is admitted for a splenectomy.
iron deficiency anemia about those foods that What problem is the nurse aware that could
she needs to include in her meal plans. Which of develop?
the following, if selected, would indicate to the 1. Infection.
nurse that the client understands the dietary 2. Congestive heart failure.
instructions?
3. Urinary retention.
1. Citrus fruits and green leafy vegetables.
4. Viral hepatitis.
2. Bananas and nuts.
3. Coffee and tea. 237. An adult is diagnosed with disseminated
4. Dairy products. intramuscular coagulation (DIC). The nurse
should identify that the client is at risk for
232. In assessing clients for pernicious anemia, the which of the following nursing diagnoses?
nurse should be alert for which of the following 1. Risk for increased cardiac output related to
risk factors? fluid volume excess.
1. Positive family history. 2. Disturbed sensory perception related to
2. Acute or chronic blood loss. bleeding into tissues.
3. Infectious agents or toxins. 3. Alteration in tissue perfusion related to
4. Inadequate dietary intake. bleeding and diminished blood flow.
4. Risk for aspiration related to constriction of
233. A client has been scheduled for a Schilling test. the respiratory musculature.
What instruction will the nurse give the client?
1. Take nothing by mouth for 12 hours prior to 238. A client diagnosed with DIC is ordered heparin.
the test. What is the reason for this medication?
2. Collect his urine for 12 hours. 1. Prevent clot formation.
3. Administer a fleets enema the evening before 2. Increase blood flow to target organs.
the test. 3. Increase clot formation.
4. Empty his bladder immediately before the 4. Decrease blood flow to target organs.
test.
239. A 34-year-old client is diagnosed with AIDS.
234. A 40-year-old woman with aplastic anemia is His pharmacologic management includes
prescribed estrogen with progesterone. The zidovudine (AZT). During a home visit, the
nurse can expect that these medications are client states, I dont understand how this
given for which of the following reasons? medication works. Will it stop the infection?
1. To stimulate bone growth. What is the nurses best response?
2. To regulate fluid balance. 1. The medication helps to slow the disease
3. To enhance sodium and potassium process, but it wont cure or stop it totally.
absorption. 2. The medication blocks reverse transcriptase,
4. To promote utilization and storage of fluids. the enzyme required for HIV replication.

4 280 NCLEX-RN Review


53155_04_Ch04b_p264-377.qxd 2/26/09 7:32 AM Page 281

3. Dont you know? There arent any


medications to stop or cure HIV. Answers and Rationales
4. No, it wont stop the infection. In fact,
sometimes the HIV can become immune to
the drug itself. 218. 2. The most important consideration in
transfusion therapy is to give the correct blood
240. Which statement, from a participant attending product to the correct client.
a class on AIDS prevention, indicates an
understanding of how to reduce transmission 219. 4. Sudden development of fever during a blood
of HIV? transfusion may be indicative of a pyrogenic
reaction. The most appropriate nursing action is
1. Mothers who are HIV-positive should
to discontinue the blood flow to prevent a more
still be encouraged to breastfeed their
severe reaction.
babies because breast milk is superior to
cows milk. 220. 4. When performing the interview, it is
2. I think a needle exchange program, important to communicate an attitude of trust.
where clean needles are exchanged for If the nurse senses negative feelings with a
dirty needles, should be offered in every client, it is necessary to share this information
city. with the charge nurse, who will obtain a more
3. Females taking birth control pills are accurate assessment. This strategy also provides
protected from getting HIV. the opportunity for the nurse to seek the counsel
4. Its okay to use natural skin condoms since of others.
they offer the same protection as the latex
221. 2. The ELISA test detects the presence of
condoms.
antibodies to the AIDS virus and is a useful
241. What should be included in the teaching plan to screening tool. It is not a definitive diagnostic
young adults about the spread of AIDS? test but provides additional information for the
1. Heterosexual transmission of HIV is on the high-risk individual.
rise. 222. 3. Creamy white patches indicating an
2. The increase of HIV in children is opportunistic infection (candidiasis) are often
primarily attributed to the rise in sexual seen in the client with AIDS.
abuse.
3. Herpes zoster is a form of the HIV virus. 223. 1. Universal blood and body fluid precautions,
4. Transmission of HIV by IV drug users is general infection control outlines, and disease-
prominent even when sterile equipment specific measures are recommended by the CDC
is used. for AIDS and AIDS-related infections.

242. What orders would likely be included for a 224. 1. These blood values are consistent with a
client diagnosed with multiple myeloma? diagnosis of aplastic anemia in which the
nurses primary goals are to prevent
1. Bed rest.
complications from infection and hemorrhage.
2. Corticosteroid therapy. Whenever WBC blood levels are low, this should
3. Fluid restrictions. cue the nurse to recognize that the clients
4. Calcium replacement therapy. immune system is weakened and the potential
for infection is great.
243. Which client statement would indicate to the
nurse that the client with polycythemia vera 225. 1. The RBCs are decreased (normal 4.55.0),
is in need of further instruction? which is associated with either the decreased
1. Ill be flying overseas to see my son and production of RBCs, increased destruction of
grandchildren for the holidays. RBCs, or blood loss. Both hemoglobin (normal
2. I plan to do my leg exercises at least three 1218) and hematocrit (normal 3854) values are
times a week. subsequently decreased. Low platelets (normal
150,000400,000) are most frequently associated
3. Im going to be walking in the mall every
with a tendency to bleed. These factors support
day to build up my strength.
the need for the nurse to monitor the client
4. At night when I sleep, I like to use two closely for bleeding problems.
pillows to raise my head up.

ADULT NURSING

4 281
53155_04_Ch04b_p264-377.qxd 2/26/09 7:32 AM Page 282

226. 1. The WBC is very low (normal 4,00011,000). destruction, there is a subsequent decrease in
This indicates that the clients immune system both hemoglobin and hematocrit. In addition,
is deficient and the client is subject to this increased destruction causes an elevation in
infection. bilirubin levels. The reticulocyte count is high
because the numbers of immature RBCs are
227. 4. Persons with type AB blood, because they are increased when RBCs are being destroyed. This
universal blood recipients, are able to receive count reflects the bone marrow activity, which is
either type A, B, AB, or O blood. People with active in producing RBCs to compensate for the
any blood type other than AB, are restricted as destruction.
to the type of blood they can receive.
236. 1. Following a splenectomy, immunologic
228. 3. The tubing is primed with 0.9% NS solution. deficiencies may develop, and vulnerability
If the filter is not completely primed, debris will to infection is greatly increased. The
coagulate in the filter and the transfusion will be postsplenectomy client is highly susceptible
slowed. In addition, saline is prepared to infuse to infection from organisms such as
in case a transfusion reaction occurs. Pneumococcus. A preventive measure is
immunization with Pneumovax.
229. 2. The signs and symptoms the client is
experiencing are indicative of a transfusion 237. 3. Cerebral, cardiopulmonary, and peripheral
reaction and the transfusion must first be tissue perfusion is affected by DIC. The many
discontinued. clots can cause obstruction to blood flow and
tissue damage can subsequently occur.
230. 4. A Z-track injection technique should be used
to prevent leakage of the iron to subcutaneous 238. 1. In DIC, the paradoxical events of hemorrhage
tissues. and clotting occur. Although it may seem
counterproductive to administer heparin while a
231. 1. Dark, leafy green vegetables (as well as meats,
client is bleeding, it is necessary to prevent the
eggs, legumes, and whole-grain or enriched
clotting that is simultaneously occurring in the
breads and cereals) are rich in iron. In addition,
microcirculation. It is critical that the client be
both citrus foods and green leafy vegetables
monitored closely.
are high in vitamin C, which aids in iron
absorption. 239. 1. This statement answers the clients question
in a simple, matter of fact manner that is truthful
232. 1. There is a familial predisposition for
and to the point.
pernicious anemia, and although the disease
cannot be prevented, it can be controlled if 240. 2. Although needle exchange programs are very
detected and treated early. Pernicious anemia controversial, it is evident the transmission of
occurs as a result of the lack of the protein HIV can be significantly reduced when needle
intrinsic factor that is secreted by the gastric exchange programs are introduced.
mucosa.
241. 1. Heterosexual transmission of HIV is a
233. 1. The client is to fast for 12 hours prior to the concern, especially in this age group. It is on the
test. No food or drink is permitted. Following rise and this is often overlooked because the
administration of the vitamin B12 dose, food is more known transmissions take place among
delayed for 3 hours. homosexuals and IV drug abusers.
234. 1. In aplastic anemia, the bone marrow elements 242. 2. Corticosteroids may be added to the
(erythrocytes, leukocytes, and platelets) are chemotherapy regime because of their antitumor
suppressed. Treatments include, but are not effect. In addition, they assist in the excretion of
limited to, bone marrow transplantation, calcium, which helps to treat the hypercalcemia
transfusions to reduce symptomatology, and that occurs in clients who have multiple
drugs to stimulate bone marrow function. myeloma.
Drugs like estrogen and progesterone work to
stimulate bone growth. Estrogen and 243. 1. With polycythemia vera, maintaining
progesterone also stop menstruation so there oxygenation is critical. High altitudes can
is less blood loss. precipitate hypoxia. This client needs further
instruction.
235. 2. Decreased RBCs are a result of the excessive

4
destruction of the red blood cells. With this

282 NCLEX-RN Review


53155_04_Ch04b_p264-377.qxd 2/26/09 7:32 AM Page 283

The Respiratory System

OVERVIEW OF ANATOMY 1. Nasopharynx: located above the soft palate of


the mouth, contains the adenoids and
AND PHYSIOLOGY openings to the eustachian tubes.
2. Oropharynx: located directly behind the mouth
Upper Respiratory Tract and tongue, contains the palatine tonsils; air
and food enter body through oropharynx.
Structures of the respiratory system, primarily an air 3. Laryngopharynx: extends from the epiglottis to
conduction system, include the nose, pharynx, and the sixth cervical level.
larynx. Air is filtered, warmed, and humidified in
the upper airway before passing to lower airway Larynx
(see Figure 4-12).
A. Sometimes called voice box, connects upper and
Nose lower airways; framework is formed by the hyoid
bone, epiglottis, and thyroid, cricoid, and
A. External nose is a framework of bone and cartilage, arytenoid cartilages. The opening of the larynx is
internally divided into two passages or nares called the glottis.
(nasal cavities) by the septum; air enters the B. Larynx opens to allow respiration and closes to
system through the nares. prevent aspiration when food passes through the
B. The septum is covered with a mucous membrane, pharynx.
where the olfactory receptors are located. C. Vocal cords of larynx permit speech and are
Turbinates, located internally, assist in warming involved in the cough reflex.
and moistening the air.
C. The major functions of the nose are warming,
moistening, and filtering the air. Lower Respiratory Tract
Consists of the trachea, bronchi and branches, and the
Pharynx lungs and associated structures (see Figure 4-12).
A. Muscular passageway commonly called the throat.
B. Air passes through the nose to the pharynx, Trachea
composed of three sections A. Air moves from the pharynx to larynx to trachea
(length 1113 cm, diameter 1.52.5 cm in adult).
Paranasal B. Extends from the larynx to the second costal
sinus cartilage, where it bifurcates and is supported by
1620 C-shaped cartilage rings.
Nasal cavity C. The area where the trachea divides into two
Nose branches is called the carina.

Bronchi
Larynx
A. Formed by the division of the trachea into two
branches (bronchi)
Trachea 1. Right mainstem bronchus: larger and straighter
than the left; further divides into three lobar
branches (upper, middle, and lower lobar
bronchi) to supply the three lobes of right
lung. If passed too far, endotracheal tube might
enter right mainstem bronchus; only right lung
is then intubated.
2. Left mainstem bronchus: divides into the
upper and lower lobar bronchi, to supply two
lobes of left lung.
B. At the point a bronchus reaches about 1 mm in
diameter it no longer has a connective tissue
Figure 4-12 The respiratory system

4
sheath and is called a bronchiole.

ADULT NURSING 283


53155_04_Ch04b_p264-377.qxd 2/26/09 7:32 AM Page 284

Bronchioles B. Pulmonary arteries arise from the right ventricle of


the heart and continue to the bronchi and alveoli,
A. In the bronchioles, airway patency is primarily gradually decreasing in size to capillaries.
dependent upon elastic recoil formed by network C. The capillaries, after contact with the gas-exchange
of smooth muscles. surface of the alveoli, reform to form the
B. The tracheobronchial tree ends at the terminal pulmonary veins.
bronchioles. Distal to the terminal bronchioles the D. The two pulmonary veins, superior and inferior,
major function is no longer air conduction, but gas empty into the left atrium.
exchange between blood and alveolar air. The
respiratory bronchioles serve as the transition to
the alveolar epithelium. Gas Exchange
Alveolar Ducts and Alveoli
Lungs (Right and Left)
A. Alveolar ducts arise from the respiratory
A. Main organs of respiration, lie within the thoracic bronchioles and lead to the alveoli.
cavity on either side of the heart. B. Alveoli are the functional cellular units of the
B. Broad area of lung resting on diaphragm is called lungs; about half arise directly from the alveolar
the base; the narrow, superior portion is the apex. ducts and are responsible for about 35% of
C. Each lung is divided into lobes: three in the right alveolar gas exchange.
lung, two in the left. C. Alveoli produce surfactant, a phospholipid
D. Pleura: serous membrane covering the lungs; substance found in the fluid lining the alveolar
continuous with the parietal pleura that lines the epithelium. Surfactant reduces surface tension and
chest wall. increases the stability of the alveoli and prevents
E. Lungs and associated structures are protected by their collapse.
the chest wall. D. Alveolar sacs form the last part of the airway;
functionally the same as the alveolar ducts, they
Chest Wall are surrounded by alveoli and are responsible for
65% of the alveolar gas exchange.
A. Includes the rib cage, intercostal muscles, and
diaphragm.
B. Parietal pleura lines the chest wall and secretes
ASSESSMENT
small amounts of lubricating fluid into the
intrapleural space (space between the visceral and
Health History
parietal pleura). This fluid holds the lung and A. Presenting problem
chest wall together as a single unit while allowing 1. Nose/nasal sinuses: symptoms may include
them to move separately. colds, discharge, epistaxis, sinus problems
C. The chest is shaped and supported by 12 pairs of (swelling, pain)
ribs and costal cartilages; the ribs have several 2. Throat: symptoms may include sore throat,
attached muscles. hoarseness, difficulty swallowing, strep throat
1. Contraction of the external intercostal muscles 3. Lungs: symptoms may include
raises the rib cage during inspiration and helps a. Cough: note duration; frequency; type (dry,
increase the size of the thoracic cavity. hacking, bubbly, barky, hoarse, congested);
2. The internal intercostal muscles tend to pull sputum (productive vs nonproductive);
ribs down and in and play a role in forced circumstances related to cough (time of
expiration. day, positions, talking, anxiety); treatment.
D. The diaphragm is the major muscle of ventilation b. Dyspnea: note onset, severity, duration,
(the exchange of air between the atmosphere and the efforts to treat, whether associated with
alveoli). Contraction of muscle fibers causes the radiation, if accompanied by cough or
dome of the diaphragm to descend, thereby diaphoresis, time of day when it most likely
increasing the volume of the thoracic cavity. As occurs, interference with ADL, whether
exertion increases, additional chest muscles or even precipitated by any specific activities,
abdominal muscles may be employed in moving the whether accompanied by cyanosis.
thoracic cage. c. Wheezing
d. Chest pain
Pulmonary Circulation e. Hemoptysis
B. Lifestyle: smoking (note type of tobacco, duration,
A. Provides for reoxygenation of blood and release of number per day, number of years of smoking,
CO2; gas transfer occurs in the pulmonary capillary inhalation, related cough, desire to quit);
bed. occupation (work conditions that could irritate

4 284 NCLEX-RN Review


53155_04_Ch04b_p264-377.qxd 2/26/09 7:32 AM Page 285

respiratory system [asbestos, chemical irritants,


dry-cleaning fumes] and monitoring or protection
of exposure conditions); geographical location
(environmental conditions that could irritate
respiratory system [chemical plants/industrial
pollutants]); type and frequency of
exercise/recreation.
C. Nutrition/diet: fluid intake per 24-hour period;
intake of vitamins
D. Past medical history: immunizations (yearly
immunizations for colds/flu; frequency and results
of tuberculin skin testing); allergies (foods, drugs,
contact or inhalant allergens, precipitating factors,
specific treatment, desensitization)

Physical Examination
A. Inspect for configuration of the chest (kyphosis,
scoliosis, barrel chest) and cyanosis.
B. Determine rate and pattern of breathing (normal
rate 1218/minute); note tachypnea,
hyperventilation, or labored breathing pattern. Figure 4-13 Locations for hearing normal breath
C. Palpate skin, subcutaneous structures, and muscles sounds
for texture, temperature, and degree of development.
D. Palpate for tracheal position, respiratory excursion
(symmetric or asymmetric movement of the chest),
and for fremitus. B. Pulmonary function studies
1. Fremitus is normally increased in intensity at 1. Evaluation of lung volume and capacities by
second intercostal spaces at sternal border and spirometry: tidal volume (TV), vital capacity
interscapular spaces only. (VC), inspiratory and expiratory reserve
2. Increased intensity elsewhere may indicate volume (IRV and ERV), residual volume (RV),
pneumonia, pulmonary fibrosis, or tumor. inspiratory capacity (IC), functional residual
3. Decreased intensity may indicate capacity (FRC)
pneumothorax, pleural effusion, COPD. 2. Involves use of a spirometer to diagram
E. Percuss lung fields (should find resonance over movement of air as client performs various
normal lung tissue, note hyperresonance or respiratory maneuvers; shows restriction or
dullness) and for diaphragmatic excursion (normal obstruction to airflow, or both
distance between levels of dullness on full 3. Nursing care
expiration and full inspiration is 612 cm). a. Carefully explaining procedure will help
F. Auscultate for normal (vesicular, bronchial, allay anxiety and ensure cooperation.
bronchovesicular) and adventitious (rales or b. Perform tests before meals.
crackles, rhonchi, pleural friction rub) breath c. Withhold medication that may alter
sounds (see Figure 4-13). respiratory function unless otherwise
ordered.
d. After procedure assess pulse and provide
Laboratory/Diagnostic Tests for rest period.
A. Arterial blood gases (ABGs) C. Hematologic studies (ESR, Hgb and hct, WBC)
1. Measure base excess/deficit, blood pH, CO2, D. Sputum culture and sensitivity
total CO2, O2 content, O2 saturation (SaO2), 1. Culture: isolation and identification of specific
pCO2 (partial pressure of carbon dioxide), pO2 microorganism from a specimen
(partial pressure of oxygen) 2. Sensitivity: determination of antibiotic agent
2. Nursing care effective against organism (sensitive or resistant)
a. If drawn by arterial stick, place a 4 3 4 3. Nursing care
bandage over puncture site after withdrawal a. Explain necessity of effective coughing.
of needle and maintain pressure with two b. If client unable to cough, heated aerosol
fingers for at least 2 minutes. will assist with obtaining a specimen.
b. Gently rotate sample in test tube to mix c. Collect specimen in a sterile container that
heparin with the blood. can be capped afterwards.
c. Place sample in ice-water container until it d. Volume need not exceed 13 mL.

4
can be analyzed. e. Deliver specimen to lab rapidly.

ADULT NURSING 285


53155_04_Ch04b_p264-377.qxd 2/26/09 7:32 AM Page 286

E. Tuberculin skin test b. Explain procedure; instruct client not to


1. Intradermal test done to detect tuberculosis cough or talk during procedure.
infection; does not differentiate active from c. Position client at side of bed, with upper
dormant infections torso supported on overbed table, feet and
2. Purified protein derivative (PPD) tuberculin legs well supported. (See Figure 4-14)
administered to determine any previous d. Assess vital signs.
sensitization to tubercle bacillus 3. Nursing care: following procedure
3. Several methods of administration a. Observe for signs and symptoms of
a. Mantoux test: 0.1 mL solution containing pneumothorax, shock, leakage at puncture
0.5 tuberculin units of PPD-tuberculin is site.
injected into the forearm. b. Auscultate chest to ascertain breath
b. Tine test: a stainless steel disc with 4 tines sounds.
impregnated with PPD-tuberculin is G. Bronchoscopy
pressed into the skin. 1. Insertion of a fiber-optic scope into the bronchi
4. Results: read within 4872 hours; inspect skin for diagnosis, biopsy, specimen collection,
and circle zone of induration with a pencil; examination of structures/tissues, removal of
measure diameter in mm foreign bodies
a. Negative: zone diameter less than 5 mm 2. Nursing care: prior to procedure
b. Doubtful or probable: zone diameter a. Confirm that a signed permit has been
510 mm obtained.
c. Positive: zone diameter 10 mm or more b. Explain procedure, remove dentures, and
F. Thoracentesis provide good oral hygiene.
1. Insertion of a needle through the chest wall c. Keep client NPO 612 hours pretest.
into the pleural space to obtain a specimen for 3. Nursing care: following procedure
diagnostic evaluation, removal of pleural fluid a. Position client on side or in semi-Fowlers.
accumulation, or to instill medication into the b. Keep NPO until return of gag reflex.
pleural space c. Assess for and report frank bleeding.
2. Nursing care: prior to procedure d. Apply ice bags to throat for comfort;
a. Confirm that a signed permit has been discourage talking, coughing, smoking for a
obtained. few hours to decrease irritation.

ANALYSIS
Nursing diagnoses for the client with a respiratory
dysfunction may include:
A. Impaired gas exchange
B. Ineffective airway clearance
C. Ineffective breathing pattern
D. Impaired verbal communication
E. Activity intolerance
F. Anxiety
G. Imbalanced nutrition: less than body
requirements
H. Risk for infection

PLANNING AND IMPLEMENTATION


Goals
A. Adequate ventilation will be maintained.
B. Maintenance of patent airway.
C. Effective breathing patterns will be maintained.
D. Client will communicate in an effective manner.
E. Client will demonstrate increased tolerance for
activity.
F. Anxiety will be reduced.
Figure 4-14 Client position for thoracentesis G. Adequate nutritional status will be maintained.

4
H. Client remains free from infection.

286 NCLEX-RN Review


53155_04_Ch04b_p264-377.qxd 2/26/09 7:32 AM Page 287

Interventions b. The third bottle controls the amount of


pressure in the system. The suction-control
Chest Drainage Systems bottle has three tubes inserted in the
stopper, two short and one long. One short
A. Insertion of a catheter into the intrapleural space
tube is joined with the tubing to the former
to maintain constant negative pressure when
air vent of the water-seal bottle; the second
air/fluid have accumulated.
short tube is connected to suction. The
B. Chest tube is attached to underwater drainage to
third (long) tube (or suction-control tube)
allow for the escape of air/fluid and to prevent
is located between the short tubes and has
reflux of air into the chest.
one end open to the atmosphere and the
C. For evacuation of air, chest tube is placed in the
other below the water level.
second or third intercostal space, anterior or
c. The depth to which the suction-control
midaxillary line (air rises to the upper chest).
tube is immersed controls the amount of
D. For drainage of fluid, chest tube is placed in the
pressure within the system. The pressure is
eighth or ninth intercostal space, midaxillary line.
determined by the physician.
E. Chest tube is connected to tubing for the collection
G. Dry-suction water-seal drainage; also referred to as
system; the distal end of the collection tubing must
dry suction; has three chambers (collection
be placed below the water level in order to prevent
chamber, water seal chamber, and wet suction
atmospheric air from entering the pleural space.
control chamber) (see Figure 4-15E).
F. Traditional water-seal drainage; also known as wet
H. Dry-suction one-way valve system (also known as
suction; can be set up using one, two, or three
Heimlich Flutter Valve); allows unidirectional flow
bottles. No longer used in the United States
of air and fluid from pleural space, prevents reflux,
because it has been replaced by the newer
sets up quickly in emergency, works when
technology of dry-suction water-seal drainage. For
knocked over, works well for ambulatory clients.
explanatory purposes it is included here. Also may
I. Nursing care: without suction
be used in other countries.
1. Prepare the unit for use and connect the chest
1. One-bottle system (Figure 4-15A)
catheter to the drainage tubing.
a. Operates by gravity, not suction; the bottle
2. Examine the entire system to ensure
serves as both collection chamber and
airtightness and absence of obstruction from
water seal.
kinks or dependent loops of tubing.
b. Two hollow tubes (glass rods) are inserted
3. Note oscillation of the fluid level within the
into the stopper of the bottle; the drainage
water-seal tube. It will rise on inspiration and
tube is connected to the glass rod that is
fall on expiration due to changes in the
submerged approximately 2 cm below the
intrapleural pressure. If oscillation stops and
water level; the second glass tube allows
system is intact, notify physician.
for the escape of air.
4. Check the amount, color, and characteristics of
c. If considerable drainage accumulates it is
the drainage. If drainage ceases and system is
difficult for the client to expel air and fluid
not blocked, assess for signs of respiratory
from the pleural space. If this occurs, the
distress from fluid/air accumulation.
glass rod may be pulled up or a new
5. Always keep drainage system lower than the
drainage bottle may be set up (according to
level of the clients chest.
physicians orders).
6. Keep Vaseline gauze at bedside at all times in
2. Two-bottle system (see Figure 4-15B,C)
case chest tube falls out.
a. One bottle serves as a drainage collection
7. Keep clamps at bedside in case of accidental
chamber, the other as a water seal.
disconnection. Clamp, troubleshoot, and
b. The first bottle is the drainage collection
immediately reconnect.
chamber and has two short tubes in the
8. Encourage coughing and deep breathing to
rubber stopper. One of these tubes is
facilitate removal of air and drainage from
attached to the drainage tubing coming
pleural cavity.
from the client; the other is attached to the
9. Provide ROM exercises.
underwater tube of the second bottle (the
J. Nursing care: with suction
water-seal bottle). The air vent of the
1. Attach suction tubing to suction apparatus,
water-seal bottle must be left open to
and chest catheter to drainage tubing.
atmospheric air. If suction is used, the first
2. Open suction slowly until a stream of bubbles
bottle serves as drainage collection and
is seen in the suction chamber. There should
water-seal chamber, and the second bottle
be continuous bubbling in this chamber and
serves as the suction chamber.
intermittent bubbling in the water seal. Check
3. Three-bottle system (Figure 4-15D)
for an air leak in the system if bubbling in
a. This system has a drainage collection, a
water seal is constant; notify physician if air
water-seal, and a suction-control bottle.

4
leak.

ADULT NURSING 287


53155_04_Ch04b_p264-377.qxd 2/26/09 7:32 AM Page 288

From client From client From client To suction

Collection and Collection Water seal Collection and Suction


water seal water seal control
(A) (B) (C)

From client To suction From air vent From client

Suction

Collection Water seal Suction


(D) control
Suction
control
Chest
Water Drainage drainage
(E) seal collection
chambers

Figure 4-15 Water-seal drainage systems: (A) one-bottle system; (B) two-bottle system; (C) two-bottle system
with suction; (D) three-bottle system; (E) underwater seal chest drainage device

3. Check drainage, keep drainage system below 3. If the water-seal bottle should break,
level of clients chest, keep Vaseline gauze at immediately obtain some type of fluid-filled
bedside, encourage coughing and deep container to create an emergency water seal
breathing, and provide ROM exercises. until a new unit can be obtained.
K. General principles of chest tube management:
1. Never clamp chest tubes over an extended Heimlich Flutter Valve
period of time unless a specific order is
written by the physician. Clamping the chest A. This disposable valve allows a unidirectional flow
tubes of a client with air in the pleural space of air and fluid from the pleural space into a
will cause increased pressure buildup and drainage bag and prevents any reflux of air or fluid.
possible tension pneumothorax. A water-seal drainage system is not necessary.
2. Removal of the chest tube: instruct the client B. Controlled suction can be attached if ordered.
to perform Valsalva maneuver; apply a C. The valve is encased in clear plastic, which
Vaseline or gauze dressing to the site (per eliminates the possibility of kinks. Its small size,

4
hospital protocol). approximately 7 inches, permits greater mobility.

288 NCLEX-RN Review


53155_04_Ch04b_p264-377.qxd 2/26/09 7:32 AM Page 289

Chest Physiotherapy 11. Repeat the same procedure in all designated


positions.
A. General information 12. After procedure, assist client to comfortable
1. Used for individuals with increased production position and provide good oral hygiene.
of secretions or thick, sticky secretions, and for
clients with impaired removal of secretions or Mechanical Ventilation
with ineffective cough. May also be used as a
preventive measure for clients with weakness of A. General information
the muscles of respiration or a predisposition to 1. Ventilation is performed by mechanical means
increased production or thickness of secretions. in individuals who are unable to maintain
2. Includes the techniques of postural drainage, normal levels of oxygen and carbon dioxide in
percussion, and vibration. the blood.
a. Postural drainage: uses gravity and various 2. Indicated in clients with COPD, obesity,
positions to stimulate the movement of neuromuscular disease, severe neurologic
secretions. depression, thoracic trauma, ARDS; clients
1) Postural drainage positions are who have undergone thoracic or open-heart
determined by the areas of involved surgery are likely to be maintained on
lung, assessed by chest X-ray and mechanical ventilation post-op.
physical assessment findings. B. Types (positive pressure ventilators)
2) Careful positioning is required to help 1. Positive pressure-cycled ventilator: pushes air
secretions flow from smaller airways into the lungs until a predetermined pressure
into the segmental bronchus and larger is reached within the tracheobronchial tree;
airways where secretions can be expiration occurs by passive relaxation of the
coughed up. diaphragm.
b. Percussion: involves clapping with cupped 2. Volume-cycled ventilator: most popular type
hands on the chest wall over the segment for intubated adults and older children;
to be drained. delivers air into the lungs until a certain
1) The hand is cupped by holding the predetermined tidal volume is reached before
fingers together so that the shape of the terminating inspiration.
hand conforms with chest wall. 3. Time-cycled ventilator: terminates inspiration
2) Clapping should be vigorous but not after a preset time; tidal volume is regulated by
painful. adjusting length of inspiration and flow rate of
c. Vibration: in this technique the hand is pressurized gas.
pressed firmly over the appropriate segment C. Modes of mechanical ventilation
of chest wall, and muscles of upper arm and 1. Assist/control mode: clients inspiratory effort
shoulder are tensed (isometric contraction); triggers ventilator, which then delivers breath;
done with flattened, not cupped hand; may be set to deliver breath automatically if
mechanical vibration is performed by a client does not trigger it. The same tidal
hand-held instrument. volume is delivered with each breath.
B. Nursing care 2. Intermittent mandatory ventilation (IMV):
1. Perform procedure before or 3 hours after client may breathe at own rate. IMV breaths
meals. are delivered under positive pressure;
2. Administer bronchodilators about 20 minutes however, all other respirations taken by the
before procedure. client are delivered at ambient pressure and
3. Remove all tight/constricting clothing. tidal volume is of clients own determination.
4. Have all equipment available (tissues, emesis 3. Positive end expiratory pressure (PEEP):
basin, towel, paper bag). ventilator delivers additional positive pressure
5. Assist client to correct prescribed position for at the end of expiration, which maintains the
postural drainage (client to assume each alveoli in an expanded state.
postural drainage position for approximately 4. Controlled mandatory ventilation (CMV): all
35 minutes). breaths initiated by ventilator as there is no
6. Place towel over area to be percussed. pressure sensed by the machine. Same tidal
7. Instruct client to take several deep breaths. volume is delivered with each breath.
8. Percuss designated area for approximately 3 5. Continuous positive airway pressure (CPAP):
minutes during inspiration and expiration. achieves the same results as PEEP, except CPAP
9. Vibrate same designated area during is used on adult clients who are on a T-piece.
exhalations of 45 deep breaths. 6. Pressure support (PS): clients inspiratory efforts
10. Assist client with coughing when in postural trigger each breath; every breath ventilator
drainage position; some clients may need to sit assisted without dyssynchrony, which can occur

4
upright to produce a cough. with SIMV. Often used prior to extubation.

ADULT NURSING 289


53155_04_Ch04b_p264-377.qxd 2/26/09 7:32 AM Page 290

D. Nursing care c) Monitor carefully in clients who


1. Assess for decreased cardiac output and are prone to develop obstructed
administer appropriate nursing care. airways.
2. Monitor for positive water balance. Pressure d) Replace mask with nasal cannula
breathing may cause increase in antidiuretic during meals and reposition mask
hormone (ADH) and retention of water. immediately after eating.
a. Maintain accurate I&O. c. Nonrebreathing mask
b. Assess daily weights. 1) Standard mask with a reservoir bag
c. Take PCWP readings as ordered. designed to deliver 90100% oxygen; a
d. Palpate for peripheral edema. one-way valve between reservoir bag
e. Auscultate chest for altered breath sounds. and mask allows the client to inhale
3. Monitor for barotrauma (see Tension only from the reservoir bag and exhale
Pneumothorax). through separate valves on the side of
a. Assess ventilator settings every 4 hours. the mask; ranges: 615 liters/minute;
b. Auscultate breath sounds every 2 hours. FiO2: 6090%.
c. Monitor ABGs. 2) Nursing care
d. Perform complete pulmonary physical a) Instruct client to breathe through
assessment every shift. the nose.
4. Monitor for GI problems (stress ulcer). b) Ensure that bag does not collapse
5. Administer muscle relaxants, tranquilizers, completely with each inspiration.
analgesics, or paralyzing agents as ordered to c) Remove and clean mask every
increase client-machine synchrony by relaxing 23 hours.
the client. 2. High-flow system: client receives entire
inspired gas from the apparatus, flow rates
Oxygen Therapy must exceed the volume of air required for a
persons minute ventilation; Venturi mask
A. Most common therapy for clients with respiratory commonly used.
disease a. Provides precise delivery of oxygen
B. Indications include arterial hypoxemia; COPD; concentrations of 2450%.
ARDS; tissue, cellular, and circulatory hypoxia b. Nursing care
C. Delivery systems 1) Provide supplemental oxygen by
1. Low-flow system: delivers oxygen at variable cannula during meals and other
liter flows designed to add to clients inspired activities where mask interferes.
air. 2) Remove and clean mask every 23 hours.
a. Nasal cannula
1) Most common mode of oxygen Tracheobronchial Suctioning
delivery; consists of delivering 100%
oxygen through two prongs inserted A. Suction removal of secretions from the
1 cm into each nostril; general flow tracheobronchial tree using a sterile catheter
rates of 14 liters/minute are used inserted into the airway.
with desired FiO2 range of 2440%. B. Catheters may be inserted through various routes:
2) Nursing care nasopharyngeal, oropharyngeal, or via an artificial
a) Instruct client to breathe through airway.
the nose. C. Purposes
b) Remove cannula and clean nares 1. Maintain a patent airway through removal of
every 8 hours. secretions
c) Provide mouth care every 23 hours. 2. Promote adequate exchange of oxygen/carbon
d) Use gauze pads behind ears to dioxide
decrease irritation. 3. Substitute for effective coughing
e) Assess arterial pO2 frequently. 4. Obtain a specimen for analysis
b. Standard mask D. Procedure
1) Simple face mask that covers the nose 1. Gather suctioning equipment (receptacle for
and mouth and provides an additional secretions, sterile catheter, sterile gloves, and
area for oxygen collection; ranges: container of sterile normal saline), or use
612 liters/minute; FiO2: 4065%. inline device for endotracheal tube suctioning.
2) Nursing care 2. Turn vacuum on and test suction system.
a) Instruct client to breathe through 3. Place client in semi- to high-Fowlers position.
the nose. 4. Apply sterile glove, fill sterile cup with
b) Remove and clean mask every solution, and attach sterile catheter to

4
23 hours. connecting tube.

290 NCLEX-RN Review


53155_04_Ch04b_p264-377.qxd 2/26/09 7:32 AM Page 291

5. Increase inspired oxygen concentration to 3) Use each sponge or applicator only


highest point and hyperinflate the lungs before once.
and after each catheter insertion by using self- 4) Allow area to dry and apply a new
inflating bag; have client deep breathe if able. sterile dressing (free of lint and
6. Use gloved hand to insert catheter. fibers).
a. Oral route 5) Change tracheostomy ties as needed.
1) If oral airway in place, slide the b. For a double-cannula tube (nondisposable
catheter alongside it and back to the inner cannula)
pharynx; if no oral airway in place, 1) Disconnect ventilator or
have client protrude the tongue and humidification device and unlock the
guide the catheter into the oropharynx. inner cannula of trach tube using
2) Insert during inspiration until cough is ungloved hand.
stimulated or secretion obtained. 2) Place inner cannula in basin containing
b. Nasal route: advance catheter along the H2O2 to remove encrustations.
floor of the nares or pass it through an 3) If client on a ventilator, insert another
artificial nasal airway (nasal trumpet) until inner cannula while old one is being
cough is stimulated or secretions obtained. cleaned and reconnect client to
c. Artificial airway: insert the catheter into ventilator.
the artificial airway until cough is 4) Cleanse stomal area and trach tube
stimulated or secretions obtained. flanges with presoaked gauze sponges.
7. Do not cover the thumb control and do not 5) Clean inner cannula.
apply suction during insertion of the catheter. 6) Remove excess liquid by gentle
8. During withdrawal, rotate the catheter while shaking.
applying intermittent suction. 7) If client not on a ventilator, gently
9. Whole suctioning procedure including reinsert inner cannula into
insertion and removal of the catheter should tracheostomy tube and lock in place.
not exceed 10 seconds. 8) Allow area to dry, apply dressing and
10. If it is necessary to continue the suctioning new tracheostomy ties as described
process, hyperinflate the lungs, allow the above.
client to rest briefly, and repeat the process. c. For a double-cannula tube (disposable
11. Discard catheter, glove, and cup; record inner cannula)
amount, color, characteristics of the secretions 1) Disconnect ventilator tube.
obtained; note clients tolerance of procedure. 2) Remove inner cannula.
12. Auscultate for changes in breath sounds. 3) Insert replacement cannula.
4) Cleanse stomal area and trach tube
Tracheostomy Care flanges with presoaked gauze sponges.
5) Allow area to dry, apply dressing and
A. Performed to avoid bacterial contamination and new tracheostomy ties as described
obstruction of tracheostomy tube; frequency varies above.
depending on amount of secretions
B. Procedure
1. Explain procedure and provide reassurance to EVALUATION
the client.
2. If not contraindicated, place client in semi- A. Client demonstrates ABGs or O2 saturation within
Fowlers position to promote lung normal limits; absence of dyspnea and cyanosis;
reexpansion. usual or improved breath sounds and usual
3. Disconnect ventilator or humidification device. mentation.
4. Suction trachea to clear secretions. B. Client demonstrates effective coughing with
5. Reconnect ventilator or humidifier. expectoration of secretions; absence of dyspnea;
6. Remove all tracheostomy dressing. rate and depth of ventilation within normal range;
7. Assemble equipment (trach care kit). improved breath sounds.
8. Set up sterile field and put on sterile glove. C. ABGs or O2 saturation within normal range; lungs
a. For a single-cannula tube clear to auscultation; rate and depth of respirations
1) With sterile gloved hand, wipe clients within clients normal range; effective use of
neck under trach tube flanges with muscles of respiration.
presoaked sterile sponge. D. Client identifies plans for appropriate alternate
2) Wipe skin around tracheostomy with a speech methods.
second sponge until cleansed E. Client demonstrates increased activity tolerance
thoroughly (may use wet cotton-tipped with absence of dyspnea and excessive fatigue and

4
applicators to cleanse around stoma). vital signs within normal limits.

ADULT NURSING 291


53155_04_Ch04b_p264-377.qxd 2/26/09 7:32 AM Page 292

F. Improved rest/sleep patterns; respiratory rate and b. Antimicrobials: tetracycline, ampicillin to


rhythm within clients normal range; demonstrates treat bacterial infections
effective problem-solving abilities. c. Corticosteroids: prednisone
G. Client demonstrates behaviors/lifestyle changes to 2. Facilitate removal of secretions.
regain and maintain appropriate body weight. a. Ensure fluid intake of at least 3 liters/day.
Verbalizes importance of nutrition to general well- b. Provide (and teach client) chest physical
being. Stable weight; improved anthropometric therapy, coughing and deep breathing, and
measurements. use of hand nebulizers.
H. Vital signs within clients normal range; client c. Suction as needed.
verbalizes understanding of causative/risk factors d. Provide oral hygiene after expectoration of
and utilizes techniques to promote a safe sputum.
environment. 3. Improve ventilation.
a. Position client in semi- or high-Fowlers.
b. Instruct client to use diaphragmatic muscle
DISORDERS OF THE to breathe.
c. Encourage productive coughing after all
RESPIRATORY SYSTEM treatments (splint abdomen to help
produce more expulsive cough).
Chronic Obstructive Pulmonary d. Employ pursed-lip breathing techniques
(prolonged, slow relaxed expiration against
Disease (COPD) pursed lips).
Refers to respiratory conditions that produce e. Oxygen delivery is used with caution,
obstruction of airflow; includes emphysema, usually via nasal cannula keeping pulse
bronchitis, and bronchiectasis. Asthma is now oximetry at or above 90% (keep oxygen
considered a separate disorder because it is often flow rate low to prevent suppressing
reversible. respiratory drive).
4. Provide client teaching and discharge
Emphysema planning concerning
a. Prevention of recurrent infections
A. General information 1) Avoid crowds and individuals with
1. Enlargement and destruction of the alveolar, known infection.
bronchial, and bronchiolar tissue with 2) Adhere to high-protein, high-
resultant loss of recoil, air trapping, thoracic carbohydrate, increased vitamin C diet.
overdistension, sputum accumulation, and 3) Receive immunizations for influenza
loss of diaphragmatic muscle tone and pneumonia.
2. These changes cause a state of carbon dioxide 4) Report changes in characteristics and
retention, hypoxia, and respiratory acidosis. color of sputum immediately.
3. Caused by cigarette smoking, infection, 5) Report worsening of symptoms
inhaled irritants, heredity, allergic factors, (increased tightness of chest, fatigue,
aging increased dyspnea).
B. Assessment findings b. Control of environment
1. Anorexia, fatigue, weight loss, clubbing 1) Use home humidifier at 3050%
(bulbous enlargement of distal fingers/nails) humidity.
2. Feeling of breathlessness, cough, sputum 2) Wear scarf over nose and mouth in
production, flaring of the nostrils, use of cold weather to prevent
accessory muscles of respiration, increased bronchospasm.
rate and depth of breathing, dyspnea 3) Avoid smoking and contact with
3. Decreased respiratory excursion, resonance to environmental smoke.
hyperresonance, decreased breath sounds with 4) Avoid abrupt changes in temperature.
prolonged expiration, normal or decreased c. Avoidance of inhaled irritants
fremitus 1) Stay indoors if pollution levels are high.
4. Diagnostic tests: pCO2 elevated or normal; pO2 2) Use air conditioner with high-
normal or slightly decreased efficiency particulate air filter to
C. Nursing interventions remove particles from air.
1. Administer medications as ordered. d. Increasing activity tolerance
a. Bronchiodilators: aminophylline, 1) Start with mild exercises, such as
isoproterenol (Isuprel), terbutaline walking, and gradually increase
(Brethine), metaproterenol (Alupent), amount and duration.
theophylline, isoetharine (Bronkosol); used 2) Use breathing techniques (pursed lip,

4
in treatment of bronchospasm diaphragmatic) during

292 NCLEX-RN Review


53155_04_Ch04b_p264-377.qxd 2/26/09 7:32 AM Page 293

activities/exercises to control Pulmonary Tuberculosis


breathing.
3) Have oxygen available as needed to A. General information
assist with activities. 1. Bacterial infectious disease caused by
4) Plan activities that require low M. tuberculosis and spread via airborne
amounts of energy. droplets when infected persons cough, sneeze,
5) Plan rest periods before and after or laugh.
activities. 2. Once inhaled, the organisms implant
themselves in the lung and begin dividing
Bronchitis slowly, causing inflammation, development of
the primary tubercle, and eventual caseation
A. General information and fibrosis.
1. Excessive production of mucus in the 3. Infection spreads via the lymph and
bronchi with accompanying persistent circulatory systems.
cough. 4. The CDC (2006) reports that the highest
2. Characteristic changes include incidence is in foreign-born persons and racial/
hypertrophy/hyperplasia of the mucus- ethnic minority populations (Hispanics, African
secreting glands in the bronchi, decreased Americans, Asians). Socially and economically
ciliary activity, chronic inflammation, and disadvantaged, alcoholic, and malnourished
narrowing of the small airways. individuals are affected more often.
3. Caused by the same factors that cause 5. The causative agent, M. tuberculosis, is an
emphysema. acid-fast bacillus spread via droplet nuclei
B. Medical management: drug therapy includes from infected persons.
bronchodilators, antimicrobials, expectorants B. Assessment findings
(e.g., Robitussin) 1. Cough (yellow mucoid sputum), dyspnea,
C. Assessment findings hemoptysis, rales or crackles
1. Productive (copious) cough, dyspnea on 2. Anorexia, malaise, weight loss, afternoon low-
exertion, use of accessory muscles of grade fever, pallor, pain, fatigue, night sweats
respiration, scattered rales and rhonchi 3. Diagnostic tests
2. Feeling of epigastric fullness, slight cyanosis, a. Chest X-ray indicates presence and extent
distended neck veins, ankle edema of disease process but cannot differentiate
3. Diagnostic tests: increased pCO2, decreased active from inactive form
pO2 b. Skin test (PPD) positive; area of induration
D. Nursing interventions: same as for emphysema 10 mm or more in diameter after 48 hours
c. Sputum positive for acid-fast bacillus
Bronchiectasis (three samples is diagnostic for disease)
d. Culture positive
A. General information e. WBC and ESR increased
1. Permanent abnormal dilation of the bronchi C. Nursing interventions
with destruction of muscular and elastic 1. Administer medications as ordered (see Unit 2).
structure of the bronchial wall 2. Prevent transmission.
2. Caused by bacterial infection; recurrent lower a. Strict isolation not required if
respiratory tract infections; congenital defects client/significant others adhere to special
(altered bronchial structures); lung tumors; respiratory precautions for tuberculosis.
thick, tenacious secretions b. Client should be in a well-ventilated
B. Medical management: same as for emphysema private room, with the door kept closed at
C. Assessment findings all times.
1. Chronic cough with production of c. All visitors and staff should wear masks
mucopurulent sputum, hemoptysis, exertional when in contact with the client and should
dyspnea, wheezing discard the used masks before leaving the
2. Anorexia, fatigue, weight loss room; client should wear a mask when
3. Diagnostic tests leaving the room for tests.
a. Bronchoscopy reveals sources and sites of d. All specimens should be labeled AFB
secretions precautions.
b. Possible elevation of WBC e. Handwashing is required after direct contact
D. Nursing interventions: same as for emphysema with the client or contaminated articles.
3. Promote adequate nutrition.
Asthma a. Make ongoing assessments of clients
appetite and do kcal counts for 3 days;
See Unit 5.

4
consult dietitian for diet guidelines.

ADULT NURSING 293


53155_04_Ch04b_p264-377.qxd 2/26/09 7:32 AM Page 294

b. Offer small, frequent feedings and a. Cough


nutritional supplements; assist client with b. Fever
menu selection stressing balanced nutrition. c. Joint pain
c. Weigh client at least twice a week. d. Malaise
d. Encourage activity as tolerated to increase 2. Sometimes asymptomatic
appetite. 3. Diagnostic tests
4. Prevent social isolation. a. Chest X-ray (often appears similar to
a. Impart a comfortable, confident attitude tuberculosis)
when caring for the client. b. Histoplasmin skin test (read the same as
b. Explain the nature of the disease to the PPD)
client, significant others, and visitors in D. Nursing interventions
simple terms. 1. Monitor respiratory status
c. Stress that visits are important, but 2. Administer medications as ordered; observe
isolation precautions must be followed. for severe side effects of Amphotericin B: fever
5. Vary the clients routine to prevent boredom. (acetaminophen given prophylactically),
6. Discuss the clients feelings and assess for anaphylactic reaction (Benadryl and steroids
boredom, depression, anxiety, fatigue, or given prophylactically), abnormal renal
apathy; provide support and encourage function with hypokalemia and azotemia.
expression of concerns.
7. Provide client teaching and discharge
planning concerning:
Chest Trauma
a. Medication regimen: prepare a sheet with Fractured Ribs
each drug name, dosage, time due, and
major side effects; stress importance A. General information
of following medication schedule for 1. Common chest injury resulting from blunt trauma.
prescribed period of time (usually 2. Ribs 48 are most commonly fractured because
9 months); include significant others they are least protected by chest muscles.
b. Transmission prevention: client should Splintered or displaced fractured ribs may
cover mouth when coughing, expectorate penetrate the pleura and lungs.
into a tissue and place it in a paper bag; B. Medical management: drug therapy consists of
client should also wash hands after narcotics, intercostal nerve block (injection of
coughing or sneezing; stress importance intercostal nerves above and below the injury with
of plenty of fresh air; include significant an anesthetic agent) for pain relief.
others C. Assessment findings
c. Importance of notifying physician at the 1. Pain, especially on inspiration
first sign of persistent cough, fever, or 2. Point tenderness and bruising at injury site,
hemoptysis (may indicate recurrence) splinting with shallow respirations,
d. Need for follow-up care including physical apprehensiveness
exam, sputum cultures, and chest X-rays 3. Diagnostic tests
e. Availability of community health services a. Chest X-ray reveals area and degree of
f. Importance of high-protein, high- fracture
carbohydrate diet with inclusion of b. pCO2 elevated; pO2 decreased (later)
supplemental vitamins D. Nursing interventions
1. Provide pain relief/control.
a. Administer ordered narcotics and
Histoplasmosis analgesics cautiously and monitor effects.
A. General information: a systemic fungal disease b. Place client in semi- or high-Fowlers
caused by inhalation of dust contaminated by position to ease pain associated with
Histoplasma capsulatum; fungus has been found in breathing.
poultry house litter, caves, areas harboring bats, 2. Monitor client closely for complications.
and in bird roosts. a. Assess for bloody sputum (indicative of
B. Medical management: antifungal agent lung penetration).
Amphotericin B b. Observe for signs and symptoms of
1. Very toxic: toxicity includes anorexia, chills, pneumothorax or hemothorax.
fever, headache, and renal failure
2. Acetaminophen, Benadryl, and steroids given Flail Chest
with Amphotericin B to prevent reactions
A. General information
C. Assessment findings
1. Fracture of several ribs and resultant
1. Symptoms similar to tuberculosis or
instability of the affected chest wall.

4
pneumonia

294 NCLEX-RN Review


53155_04_Ch04b_p264-377.qxd 2/26/09 7:32 AM Page 295

2. Chest wall is no longer able to provide the d. Hemothorax: accumulation of blood in the
bony structure necessary to maintain adequate pleural space; frequently found with an
ventilation; consequently, the flail portion and open pneumothorax resulting in a
underlying tissue move paradoxically (in hemopneumothorax.
opposition) to the rest of the chest cage and B. Assessment findings
lungs. 1. Sudden sharp pain in the chest, dyspnea,
3. The flail portion is sucked in on inspiration diminished or absent breath sounds on affected
and bulges out on expiration. side, decreased respiratory excursion on
4. Result is hypoxia, hypercarbia, and increased affected side, hyperresonance on percussion,
retained secretions. decreased vocal fremitus, tracheal shift to the
5. Caused by trauma (sternal rib fracture with opposite side (tension pneumothorax
possible costochondral separations). accompanied by mediastinal shift)
B. Medical management 2. Weak, rapid pulse; anxiety; diaphoresis
1. Internal stabilization with a volume-cycled 3. Diagnostic tests
ventilator a. Chest X-ray reveals area and degree of
2. Drug therapy (narcotics, sedatives) pneumothorax
C. Assessment findings b. pCO2 elevated
1. Severe dyspnea; rapid, shallow, grunty c. pO2, pH decreased
breathing; paradoxical chest motion C. Nursing interventions
2. Cyanosis, possible neck vein distension, 1. Provide nursing care for the client with an
tachycardia, hypotension endotracheal tube: suction secretions, vomitus,
3. Diagnostic tests blood from nose, mouth, throat, or via
a. pO2 decreased endotracheal tube; monitor mechanical
b. pCO2 elevated ventilation.
c. pH decreased 2. Restore/promote adequate respiratory function.
D. Nursing interventions a. Assist with thoracentesis and provide
1. Maintain an open airway: suction appropriate nursing care.
secretions/blood from nose, throat, mouth, and b. Assist with insertion of a chest tube to
via endotracheal tube; note changes in water-seal drainage and provide
amount, color, characteristics. appropriate nursing care.
2. Monitor mechanical ventilation. c. Continuously evaluate respiratory patterns
3. Encourage turning, coughing, and deep and report any changes.
breathing. 3. Provide relief/control of pain.
4. Monitor for signs of shock. a. Administer narcotics/analgesics/sedatives
as ordered and monitor effects.
Pneumothorax/Hemothorax b. Position client in high-Fowlers position.

A. General information
1. Partial or complete collapse of the lung due to Atelectasis
an accumulation of air or fluid in the pleural A. General information
space 1. Collapse of part or all of a lung due to
2. Types bronchial obstruction
a. Spontaneous pneumothorax: the most 2. May be caused by intrabronchial obstruction
common type of closed pneumothorax; air (secretions, tumors, bronchospasm, foreign
accumulates within the pleural space bodies); extrabronchial compression (tumors,
without an obvious cause. Rupture of a small enlarged lymph nodes); or endobronchial
bleb on the visceral pleura most frequently disease (bronchogenic carcinoma,
produces this type of pneumothorax. Occurs inflammatory structures)
most often among tall, thin men between the B. Assessment findings
ages of 20 and 40 years. 1. Signs and symptoms may be absent depending
b. Open pneumothorax: air enters the pleural upon degree of collapse and rapidity with
space through an opening in the chest which bronchial obstruction occurs
wall; usually caused by stabbing or 2. Dyspnea, decreased breath sounds on affected
gunshot wound. side, decreased respiratory excursion, dullness
c. Tension pneumothorax: air enters the to flatness upon percussion over affected area
pleural space with each inspiration but 3. Cyanosis, tachycardia, tachypnea, elevated
cannot escape; causes increased temperature, weakness, pain over affected area
intrathoracic pressure and shifting of the 4. Diagnostic tests
mediastinal contents to the unaffected side a. Bronchoscopy: may or may not reveal an

4
(mediastinal shift). obstruction

ADULT NURSING 295


53155_04_Ch04b_p264-377.qxd 2/26/09 7:32 AM Page 296

b. Chest X-ray shows diminished size of c. Thoracentesis may contain blood if cause
affected lung and lack of radiance over is cancer, pulmonary infarction, or
atelectic area tuberculosis; positive for specific organism
c. pO2 decreased in empyema
C. Nursing interventions (prevention of atelectasis in D. Nursing interventions: vary depending on etiology
hospitalized clients is an important nursing 1. Assist with repeated thoracentesis.
responsibility) 2. Administer narcotics/sedatives as ordered to
1. Turn and reposition every 12 hours while decrease pain.
client is bedridden or obtunded. 3. Assist with instillation of medication into
2. Encourage mobility (if permitted). pleural space (reposition client every 15 minutes
3. Promote liquification and removal of to distribute the drug within the pleurae).
secretions. 4. Place client in high-Fowlers position to
4. Avoid administration of large doses of promote ventilation.
sedatives and opiates that depress respiration
and cough reflex.
5. Prevent abdominal distension.
Pneumonia
6. Administer prophylactic antibiotics as ordered A. General information
to prevent respiratory infection. 1. An inflammation of the alveolar spaces of the
lung, resulting in consolidation of lung tissue
as the alveoli fill with exudate.
Pleural Effusion 2. The various types of pneumonias are classified
A. General information according to the offending organism.
1. Collection of fluid in the pleural space 3. Bacterial pneumonia accounts for 10% of all
2. A symptom, not a disease; may be produced hospital admissions; affects infants and elderly
by numerous conditions most often, and most often occurs in winter
3. Classification and early spring.
a. Transudative: accumulation of protein- 4. Caused by various organisms: D. pneumoniae,
poor, cell-poor fluid S. aureus, E. coli, H. influenzae.
b. Suppurative (empyema): accumulation of B. Assessment findings
pus 1. Cough with greenish to rust-colored sputum
4. May be found in clients with liver/kidney production; rapid, shallow respirations with
disease, pneumonia, tuberculosis, lung an expiratory grunt; nasal flaring; intercostal
abscess, bronchial carcinoma, leukemia, rib retraction; use of accessory muscles of
trauma, pulmonary edema, systemic infection, respiration; dullness to flatness upon
disseminated lupus erythematosus, percussion; possible pleural friction rub; high-
polyarteritis nodosa pitched bronchial breath sounds; rales or
B. Medical management crackles (early) progressing to coarse (later)
1. Identification and treatment of the underlying 2. Fever, chills, chest pain, weakness,
cause generalized malaise
2. Thoracentesis 3. Tachycardia, cyanosis, profuse perspiration,
3. Drug therapy abdominal distension
a. Antibiotics: either systemic or inserted 4. Diagnostic tests
directly into pleural space a. Chest X-ray shows consolidation over
b. Fibrinolytic enzymes: trypsin, affected areas
streptokinase-streptodornase to decrease b. WBC increased
thickness of pus and dissolve fibrin clots c. pO2 decreased
4. Closed chest drainage d. Sputum specimens reveal particular
5. Surgery: open drainage causative organism
C. Assessment findings C. Nursing interventions
1. Dyspnea, dullness over affected area upon 1. Facilitate adequate ventilation.
percussion, absent or decreased breath sounds a. Administer oxygen as needed and assess
over affected area, pleural pain, dry cough, its effectiveness.
pleural friction rub b. Place client in semi-Fowlers position.
2. Pallor, fatigue, fever, and night sweats (with c. Turn and reposition frequently clients who
empyema) are immobilized/obtunded.
3. Diagnostic tests d. Administer analgesics as ordered to relieve
a. Chest X-ray positive if greater than 250 mL pain associated with breathing (codeine is
pleural fluid drug of choice).
b. Pleural biopsy may reveal bronchogenic e. Auscultate breath sounds every 24 hours.

4
carcinoma f. Monitor ABGs.

296 NCLEX-RN Review


53155_04_Ch04b_p264-377.qxd 2/26/09 7:32 AM Page 297

2. Facilitate removal of secretions (general of mucus, desquamation of cells, hyperplasia,


hydration, deep breathing and coughing, and obstruction.
tracheobronchial suctioning as needed, 3. Metastasis occurs primarily by direct
expectorants as ordered, aerosol treatments via extension and via the circulatory or lymphatic
nebulizer, humidification of inhaled air, chest system.
physical therapy). 4. Men over age 40 affected most often; 1 out of
3. Observe color, characteristics of sputum and every 10 heavy smokers; affects right lung
report any changes; encourage client to more often than left.
perform good oral hygiene after expectoration. 5. Caused by inhaled carcinogens (primarily
4. Provide adequate rest and relief/control of pain. cigarette smoke but also asbestos, nickel, iron
a. Provide bed rest with limited physical oxides, air silicone pollution; preexisting
activity. pulmonary disorders [TB, COPD]).
b. Limit visits and minimize conversations. B. Medical management: depends on cell type, stage
c. Plan for uninterrupted rest periods. of disease, and condition of client; may include
d. Institute nursing care in blocks to ensure 1. Radiation therapy
periods of rest. 2. Chemotherapy: usually includes
e. Maintain pleasant and restful environment. cyclophosphamide, methotrexate, vincristine,
5. Administer antibiotics as ordered, monitor doxorubicin, and procarbazine; concurrently
effects and possible toxicity. in some combination
6. Prevent transmission (respiratory isolation 3. Surgery: when entire tumor can be removed
may be required for clients with C. Assessment findings
staphylococcal pneumonia). 1. Persistent cough (may be productive or blood
7. Control fever and chills: monitor temperature tinged), chest pain, dyspnea, unilateral
and administer antipyretics as ordered, wheezing, friction rub, possible unilateral
maintain increased fluid intake, provide paralysis of the diaphragm
frequent clothing and linen changes. 2. Fatigue, anorexia, nausea, vomiting, pallor
8. Provide client teaching and discharge 3. Diagnostic tests
planning concerning prevention of recurrence. a. Chest X-ray may show presence of tumor
a. Medication regimen/antibiotic therapy or evidence of metastasis to surrounding
b. Need for adequate rest, limited activity, structures
good nutrition with adequate fluid intake, b. Sputum for cytology reveals malignant cells
and good ventilation c. Bronchoscopy: biopsy reveals malignancy
c. Need to continue deep breathing and d. Thoracentesis: pleural fluid contains
coughing for at least 68 weeks after malignant cells
discharge e. Biopsy of scalene lymph nodes may reveal
d. Availability of vaccines (pneumonococcal metastasis
pneumonia, influenza) D. Nursing interventions
e. Techniques that prevent transmission (use 1. Provide support and guidance to client as
of tissues when coughing, adequate needed.
disposal of secretions) 2. Provide relief/control of pain.
f. Avoidance of persons with known 3. Administer medications as ordered and
respiratory infections monitor effects/side effects.
g. Need to report signs and symptoms of 4. Control nausea: administer medications as
respiratory infection (persistent or ordered, provide good oral hygiene, provide
recurrent fever; changes in characteristics, small and more frequent feedings.
color of sputum; chills; increased pain; 5. Provide nursing care for a client with a
difficulty breathing; weight loss; persistent thoracotomy.
fatigue) 6. Provide client teaching and discharge
h. Need for follow-up medical care and planning concerning:
evaluation a. Disease process, diagnostic and therapeutic
interventions
b. Side effects of radiation and chemotherapy
Bronchogenic Carcinoma c. Realistic information about prognosis
A. General information
1. The majority of primary pulmonary tumors
arise from the bronchial epithelium and are
Thoracic Surgery
therefore referred to as bronchogenic A. General information
carcinomas. 1. Types
2. Characteristic pathologic changes include a. Exploratory thoracotomy: anterior or

4
nonspecific inflammation with hypersecretion posterolateral incision through the fourth,

ADULT NURSING 297


53155_04_Ch04b_p264-377.qxd 2/26/09 7:32 AM Page 298

fifth, sixth, or seventh intercostal spaces to 5. Provide client teaching and discharge
expose and examine the pleura and lung planning concerning
b. Lobectomy: removal of one lobe of a lung; a. Need to continue with coughing/deep
treatment for bronchiectasis, bronchogenic breathing for 68 weeks post-op and to
carcinoma, emphysematous blebs, lung continue ROM exercises
abscesses b. Importance of adequate rest with gradual
c. Pneumonectomy: removal of an entire increases in activity levels
lung; most commonly done as treatment c. High-protein diet with inclusion of
for bronchogenic carcinoma adequate fluids (at least 2 liters/day)
d. Segmental resection: removal of one or d. Chest physical therapy
more segments of lung; most often done as e. Good oral hygiene
treatment for bronchiectasis f. Need to avoid persons with known upper
e. Wedge resection: removal of lesions that respiratory infections
occupy only part of a segment of lung g. Adverse signs and symptoms (recurrent
tissue; for excision of small nodules or to fever; anorexia; weight loss; dyspnea;
obtain a biopsy increased pain; difficulty swallowing;
2. Nature and extent of disease and condition of shortness of breath; changes in color,
client determine type of pulmonary resection. characteristics of sputum) and importance
B. Nursing interventions: preoperative of reporting to physician
1. Provide routine pre-op care. h. Avoidance of crowds and poorly ventilated
2. Perform a complete physical assessment of the areas
lungs to obtain baseline data.
3. Explain expected post-op measures: care of Acute Respiratory Distress
incision site, oxygen, suctioning, chest tubes
(except if pneumonectomy performed). Syndrome (ARDS)
4. Teach client adequate splinting of incision A. General information
with hands or pillow for turning, coughing, 1. A form of pulmonary insufficiency more
and deep breathing. commonly encountered in adults with no
5. Demonstrate ROM exercises for affected side. previous lung disorders than in those with
6. Provide chest physical therapy to help remove existing lung disease.
secretions. 2. Initial damage to the alveolar-capillary
C. Nursing interventions: postoperative membrane with subsequent leakage of fluid into
1. Provide routine postoperative care. the interstitial spaces and alveoli, resulting in
2. Promote adequate ventilation. pulmonary edema and impaired gas exchange.
a. Perform complete physical assessment of 3. There is cell damage, decreased surfactant
lungs and compare with preoperative production, and atelectasis, which in turn
findings. produce hypoxemia, decreased compliance,
b. Auscultate lung fields every 12 hours. and increased work of breathing.
c. Encourage turning, coughing, and deep 4. Predisposing conditions include shock,
breathing every 12 hours after pain relief trauma, infection, fluid overload, aspiration,
obtained. oxygen toxicity, smoke inhalation, pneumonia,
d. Perform tracheobronchial suctioning DIC, drug allergies, drug overdoses, neurologic
if needed. injuries, fat emboli.
e. Assess for proper maintenance of chest 5. Has also been called shock lung.
drainage system (except after B. Assessment findings
pneumonectomy). 1. Dyspnea, cough, tachypnea with
f. Monitor ABGs and report significant intercostal/suprasternal retraction, scattered to
changes. diffuse rales/rhonchi
g. Place client in semi-Fowlers position 2. Changes in orientation, tachycardia, cyanosis
(if pneumonectomy performed, follow (rare)
surgeons orders about positioning, often 3. Diagnostic tests
on back or operative side, but not turned a. pCO2 increased and pO2 decreased
to unoperative side). b. Hypoxemia
3. Provide pain relief. c. Hgb and hct possibly decreased
a. Administer narcotics/analgesics prior to d. pO2 and O2 saturation not reflective of high
turning, coughing, and deep breathing. O2 administration
b. Assist with splinting while turning, C. Nursing interventions
coughing, deep breathing. 1. Promote optimal ventilatory status.
4. Prevent impaired mobility of the upper a. Perform ongoing assessment of lungs with

4
extremities by doing ROM exercises; passive auscultation every 12 hours.
day of surgery, then active.

298 NCLEX-RN Review


53155_04_Ch04b_p264-377.qxd 2/26/09 7:32 AM Page 299

b. Elevate head and chest. C. Assessment findings


c. Administer/monitor mechanical 1. Supraglottic: localized throat pain; burning
ventilation with PEEP. when drinking hot liquids or orange juice;
d. Assist with chest physical therapy as lump in the neck; eventual dysphagia;
ordered. dyspnea; weight loss; debility; cough;
e. Encourage coughing and deep breathing hemoptysis; muffled voice
every hour. 2. Glottic: progressive hoarseness (more than
f. Monitor ABGs, O2 saturation, and report 2-week duration), eventual dyspnea
significant changes. 3. Enlarged cervical lymph nodes
2. Promote rest by spacing activities and D. Nursing interventions: provide care for the client
treatments. with a laryngectomy.
3. Maintain fluid and electrolyte balance.
4. Treat cause. Total Laryngectomy
A. General information: consists of removal of the
Cancer of the Larynx entire larynx, hyoid bone, pre-epiglottic space,
A. General information cricoid cartilage, and 34 rings of trachea. The
1. Most common upper respiratory malignancy. pharyngeal opening to the trachea is closed and
2. The majority of laryngeal malignancies are remaining trachea brought out to the neck to form
squamous cell carcinomas. a permanent tracheostomy. The result is loss of
3. Types normal speech and breathing and loss of olfaction.
a. Supraglottic (also called extrinsic laryngeal B. Nursing care: preoperative
cancer): involves the epiglottis and false 1. Provide routine pre-op care.
cords and is likely to produce no 2. Explain expected procedures after surgery
symptoms until advanced stages. including suctioning, humidification,
b. Glottic (also referred to as intrinsic coughing and deep breathing, IV fluids,
laryngeal cancer): affects the true vocal nasogastric tube feedings, tracheostomy or
cords; the most frequently occurring laryngectomy tube.
laryngeal cancer; produces early 3. Reinforce physicians teaching regarding loss
symptoms. of normal speech, breathing patterns, and
4. Occurs most often in white men in middle or sense of smell.
later life 4. Encourage client/significant others to talk
5. Caused by cigarette smoking, excessive alcohol about fears and hopes following surgery.
consumption, chronic laryngitis, vocal abuse, 5. Introduce client to changes in modes of
family predisposition to cancer of larynx communication (esophageal speech, artificial
B. Medical management larynx).
1. Radiation therapy: may be effective in cases of 6. Establish method of communication to be used
localized disease, affecting only one vocal cord immediately post-op (Magic Slate, gestures).
2. Chemotherapy: used as adjuvant therapy to C. Nursing interventions: postoperative
help shrink tumor and eradicate metastases 1. Promote optimum ventilatory status.
(experimental) 2. Suction nose frequently because of rhinitis.
3. Surgery 3. Assess function of tracheostomy/laryngectomy
a. Partial laryngectomy: a lesion on the true tube and suction as needed.
cord on one side is removed along with 4. Promote pain relief.
adjoining tissue. Useful in early, intrinsic a. Elevate head of bed to decrease pressure on
lesions. Client is able to talk and has a suture line.
normal airway post-op. b. Administer analgesics as needed and
b. Total laryngectomy monitor effects.
c. Radical neck dissection c. Assist with moving head and turning by
1) Performed when metastasis from supporting back of neck with hands.
cancer of the larynx is suspected 5. Promote wound drainage.
2) Includes removal of entire larynx, a. Elevate head of bed to promote lymphatic
lymph nodes, sternocleidomastoid drainage from head.
muscle, internal jugular vein, and b. Monitor amount, characteristics of
spinal accessory nerve drainage.
3) May also involve removal of the 6. Promote nutrition.
mandible, submaxillary gland, part of a. Institute and monitor tube feedings as
the thyroid and parathyroid gland ordered.
4) Nursing care: same as for total b. Increase fluid intake as tolerated to

4
laryngectomy, below improve hydration.

ADULT NURSING 299


53155_04_Ch04b_p264-377.qxd 2/26/09 7:32 AM Page 300

c. Encourage self-feeding. f. Need to use only electric razors for


d. Advance to normal diet as soon as client 6 months after surgery as facial area will
able to tolerate. be numb
7. Prevent infection. g. Need to lean forward when expectorating
a. Assess WBC and report significant secretions and to cover stoma when
increases. coughing or sneezing
b. Take temperature every 4 hours. h. Snorkle devices to enable swimming
c. Maintain sterile technique when (caution is advised since drowning can
suctioning and performing tracheostomy occur rapidly in these clients)
care. i. Need to wear an identification bracelet
d. Observe stoma/suture lines for signs of indicating that client is a neck breather
infection. j. Types of stoma guards available
e. Provide frequent oral hygiene. k. Necessity of installing smoke detectors
f. Monitor sputum and drainage for changes since sense of smell is lost
in color, odor, characteristics. l. Information about prosthetic devices,
8. Enhance communication. speech therapy, and reconstructive surgery
a. Carry out modes of communication
determined pre-op.
b. Assess nonverbal behavior.
c. Allow client time to ask questions and do Sample Questions
not anticipate answers.
d. Arrange for volunteer laryngectomy client
to visit client and assist with esophageal 244. A male client is admitted following an
speech, artificial larynx, and living with a automobile accident. He is very anxious,
laryngectomy. dyspneic, and in severe pain. The left chest wall
e. Consult with speech therapist if needed. moves in during inspiration and balloons out
f. Progress to normal diet as soon as possible when he exhales. What condition are these
to regain muscle tone of throat and symptoms most suggestive of?
abdomen. 1. Hemothorax.
9. Support client during adaptation to altered
2. Flail chest.
physical status.
a. Encourage client to discuss feelings about 3. Atelectasis.
changes in appearance, body functioning, 4. Pleural effusion.
and lifestyle; be aware of nonverbal
responses to the changes. 245. A young man is admitted with a flail chest
b. Assist to identify and use coping following a car accident. He is intubated with an
techniques that have been helpful in past. endotracheal tube and is placed on a mechanical
c. Suggest flattering clothing styles that dont ventilator (control mode, positive pressure).
emphasize chest or neck configuration. Which physical finding alerts the nurse to an
d. Monitor for and support behaviors additional problem in respiratory function?
indicative of positive adaptation to 1. Dullness to percussion in the third to fifth
changes (e.g., interest in appearance). intercostal space, midclavicular line.
10. Assess for respiratory complications (dyspnea,
cyanosis, tachycardia, tachypnea, 2. Decreased paradoxical motion.
restlessness). 3. Louder breath sounds on the right chest.
11. Provide client teaching and discharge 4. pH of 7.36 in arterial blood gases.
planning concerning
a. Tracheostomy/laryngectomy and stomal 246. The nurse is caring for a client who has had a
care chest tube inserted and connected to a portable
b. Proper administration of nasogastric tube water-seal drainage. The nurse determines the
feedings and maintenance of nasogastric drainage system is functioning correctly when
tube which of the following is observed?
c. Control of dryness/crusting of tongue by 1. Continuous bubbling in the water-seal
brushing tongue regularly with soft chamber.
toothbrush and toothpaste and using
mouthwashes 2. Fluctuation in the water-seal (U-tube)
d. Need for humidified air at home chamber.
e. Importance of protecting the stoma with a 3. Suction tubing attached to a wall unit.
shield or towel while showering, directing 4. Vesicular breath sounds throughout the lung

4
shower nozzle away from stoma fields.

300 NCLEX-RN Review


53155_04_Ch04b_p264-377.qxd 2/26/09 7:32 AM Page 301

247. The nurse is caring for a client who has just had 252. The treatment plan for a client newly diagnosed
a chest tube attached to a portable water-seal with tuberculosis is likely to include which of
drainage system. the following medications as initial treatment?
1. Observe for intermittent bubbling in the 1. Ethambutol (Myambutol) and isoniazid (INH).
water-seal chamber. 2. Streptomycin and penicillin G (Crysticillin).
2. Flush the chest tubes with 30 to 60 mL of 3. Tetracycline and thioridazine (Mellaril).
NSS q4 to 6 hours. 4. Pyridoxine (Beesix) and tetracycline.
3. Maintain the client in an extreme lateral
position. 253. A 64-year-old has been smoking since he was
4. Strip the chest tubes in the direction of the 11 years old. He has a long history of
client. emphysema and is admitted to the hospital
because of a respiratory infection that has not
248. The nurse enters the room of a client who has a improved with outpatient therapy. Which
chest tube attached to a water-seal drainage finding would the nurse expect to observe
system and notices the chest tube is dislodged during the clients nursing assessment?
from the chest. What is the most appropriate 1. Electrocardiogram changes.
nursing intervention? 2. Increased anterior-posterior chest diameter.
1. Notify the physician. 3. Slow, labored respiratory pattern.
2. Insert a new chest tube. 4. Weight-height relationship indicating obesity.
3. Cover the insertion site with petroleum
gauze. 254. Supplemental low-flow oxygen therapy is
4. Instruct the client to breathe deeply until prescribed for a man with emphysema. Which is
help arrives. the most essential action for the nurse to initiate?
1. Anticipate the need for humidification.
249. An adult is ordered oxygen via nasal prongs. 2. Notify the physician that this order is
What is true of administering oxygen this contraindicated.
way?
3. Place the client in an upright position.
1. Mixes room air with oxygen.
4. Schedule frequent pulse oximeter checks.
2. Delivers a precise concentration of oxygen.
3. Requires humidity during delivery. 255. When auscultating the lung fields, a sound
4. Is less traumatic to the respiratory tract. described as a rustling, like the wind in the
trees, is heard. What is the correct term for this
250. An adult is receiving oxygen by nasal prongs. occurrence?
Which statement by the client indicates that 1. Crackles.
client teaching regarding oxygen therapy has 2. Rhonchi.
been effective?
3. Wheezes.
1. I was feeling fine so I removed my nasal
4. Vesicular.
prongs.
2. It will be good to rest from taking deep 256. The nurses assessment of a client with lung
breaths now that my oxygen is on. cancer reveals the following: copious secretions,
3. Dont forget to come back quickly when you dyspnea, and cough. Based on these finding,
get me out of bed; I dont like to be without what is the most appropriate nursing diagnosis?
my oxygen for too long. 1. Impaired gas exchange.
4. My family was angry when I told them they 2. Ineffective airway clearance.
could not smoke in my room. 3. Pain.
251. The client diagnosed with tuberculosis is taught 4. Altered tissue perfusion.
prevention of disease transmission. Which
257. A client has just had arterial blood gases drawn.
correct answer will the client state is a means of
What will the nurse do with the specimen
transmission?
collected?
1. Hands.
1. Gently shake the syringe.
2. Droplet nuclei.
2. Place the sample in a syringe of warm water.
3. Milk products.
3. Aspirate 0.5 mL of heparin into the syringe.
4. Eating utensils.

4
4. Have the specimen analyzed immediately.

ADULT NURSING 301


53155_04_Ch04b_p264-377.qxd 2/26/09 7:32 AM Page 302

258. The nurse is to obtain a sputum specimen from a 1. Take the clients blood pressure.
client. Select the correct set of statements 2. Suction the tracheostomy tube.
instructing the client in the proper technique for 3. Drain water from the O2 tubing.
obtaining a sputum specimen.
4. Change the tracheostomy tube.
1. Collect the specimen right before bed. Spit
carefully into the container. 263. The nurse will perform chest physiotherapy
2. Brush your teeth, then cough into the (CPT) on a client every 4 hours. What is the
container. Do this first thing in the morning. appropriate action by the nurse?
3. Right after lunch, cough and spit into the 1. Gently slap the chest wall.
container. 2. Use vibration techniques to move secretions
4. Spit into the container, then add two from affected lung areas during the
tablespoons of water. inspiration phase.
3. Perform CPT at least 2 hours after meals.
259. The nurse is checking tuberculin skin test
4. Plan apical drainage at the beginning of the
results at a health clinic. One client has an area
CPT session.
of induration measuring 12 mm in diameter.
What does this finding indicate? 264. A client is on a ventilator. The ventilator alarm
1. This finding is a normal reading. goes off. The nurse assesses the client and
2. This finding indicates active TB. observes increased respiratory rate, use of
3. This is a positive reaction and can indicate accessory muscles, and agitation. What should
exposure to TB. be the nurses first action?
4. This client needs to come back in two more 1. Remove the client from the ventilator and
days and let the nurse look at the area of ambu bag the client, while continuing to
induration again. assess to determine the cause of the clients
distress.
260. An adult has undergone a bronchoscopy. Which 2. Call respiratory therapy to check the
assessment findings indicate to the nurse that he ventilator.
is ready for discharge? 3. Notify the physician.
1. Use of accessory muscles for breathing, 4. Turn off the alarm.
decreased lung sounds.
2. Stable vital signs, return of gag and cough 265. A client with respiratory failure is on a
reflex. ventilator. The alarm goes off. What should be
3. Hemoptysis, rhonchi. the nurses first action?
4. Development of tachycardia with occasional 1. Notify the physician.
PVCs, able to eat and drink. 2. Assess the client to determine the cause of
the alarm.
261. An adult has a chest tube to a Pleur-evac 3. Turn off the alarm.
drainage system attached to a wall suction. An
4. Disconnect the client and use the ambu bag
order to ambulate the client has been received.
to ventilate the client.
How should the nurse ambulate the client safely?
1. Clamp the chest tube and carefully ambulate 266. A nurse is setting up oxygen for an adult male.
the client a short distance. He is to receive oxygen at 2 liters per nasal
2. Question the order to ambulate the client. cannula. What should be included for this
3. Carefully ambulate the client, keeping the treatment?
Pleur-evac lower than the clients chest. 1. Adjust the flow rate to keep the reservoir bag
4. Disconnect the Pleur-evac from the clients inflated 23 full during inspiration.
chest tube, leave it attached to the bed, 2. Monitor the client carefully for risk of
ambulate the client, and then reconnect the aspiration.
chest tube when he is returned to bed. 3. Make sure the valves and rubber flaps are
patent, functional, and not stuck.
262. Approximately 10 minutes after a client returns
4. Remind the client not to use Vaseline lip balm.
from surgery with a tracheostomy tube the nurse
assesses increasing noisy respiration and an 267. A long-term COPD client is receiving oxygen
increased pulse. What action should be taken at 1 liter/minute. A family member decides she
immediately?

4
doesnt look too good and increases her

302 NCLEX-RN Review


53155_04_Ch04b_p264-377.qxd 2/26/09 7:32 AM Page 303

oxygen to 7 liter/minute. What should the 3. Rapid shallow breathing, prolonged labored
nurses initial action be? expiration, stridor.
1. Thank the clients cousin and continue to 4. Dyspnea, hypoxemia, decreased pulmonary
observe the client. compliance.
2. Immediately decrease the oxygen.
273. An adult is being admitted to the nursing unit
3. Notify the physician.
with a diagnosis of pneumonia. She has a
4. Add humidity to the oxygen. history of arrested TB. What will be the nurses
initial action?
268. An adult is receiving oxygen per face mask at
40%. The nurse should include which of the 1. Place the client in respiratory isolation.
following in her plan of care? 2. Encourage cough and deep breathing.
1. Provide good skin care, making sure the mask 3. Force fluids.
fits well. 4. Administer O2.
2. Keep all visitors out of the room.
274. An adult is being followed in the outpatient clinic
3. Turn off the CPAP during the day.
for a diagnosis of active TB. She is receiving
4. Keep the bag inflated at all time. isoniazid 200 mg po daily, rifampin 500 mg po
daily, and streptomycin 1500 mg IM twice weekly.
269. An adult has a new tracheostomy in place. He
Which statement by the client best indicates she
has a small amount of thin, white secretions.
understands her therapeutic regime?
The stoma is pink with no drainage noted. How
often should the nurse perform trach care? 1. Im glad I only have to take these drugs for a
couple of weeks.
1. 4 hours.
2. I need to take these two drugs every day and
2. 8 hours.
come back to the clinic once a week for the
3. 24 hours. shot.
4. Every hour. 3. It may work best to take these pills in the
evening right before bed.
270. A female client is admitted to the hospital. She
has smoked two packs per day for 30 years. 4. Im glad my birth control pills arent
While providing her history, she becomes affected by these drugsthe doctor told me
breathless, pauses frequently between words, not to get pregnant!
and appears very anxious. She has a cough with
275. The nurse is counseling the family of an 18-year-
thick white sputum production. Her chest is
old with active TB, about measures to prevent
barrel shaped. Based on the data, on what
transmission of the disease. Which statement by
condition will the nurse develop a plan of care?
the family best indicates understanding of these
1. Pneumonia. instructions?
2. Chronic obstructive pulmonary disease. 1. I wont let her and her sister share clothes.
3. Tuberculosis. 2. We will have to keep her in her room.
4. Asthma. 3. We all need to wash our hands carefully, but
especially our daughter.
271. A 68-year-old male is being admitted to the
hospital for an exacerbation of his COPD. What 4. We cannot get TB from exposure to her
will most likely be included in the plan of care? sputum.
1. Placed on 10 liters of oxygen per nasal 276. An elderly client has fallen and broken her
cannula. eighth rib on her left side. The nurse should
2. Placed in respiratory isolation. include which of the following when developing
3. Require frequent rest periods throughout the the plan of care?
day. 1. Bind the clients chest with a 6-inch Ace
4. Placed on fluid restriction. bandage.
2. Keep the client on bed rest for 3 days.
272. A client with suspected tuberculosis will most
3. Encourage the client to use her incentive
likely relate which clinical manifestations?
spirometer and cough and deep breathe.
1. Fatigue, weight loss, low-grade fevers, night
4. Administer large doses of narcotic analgesic
sweats.
so that the client will be able to more fully
2. Asymmetrical chest expansion.

4
participate in pulmonary care.

ADULT NURSING 303


53155_04_Ch04b_p264-377.qxd 2/26/09 7:32 AM Page 304

277. A man is injured in an industrial accident. The 2. I guess I will need to eat more green and
industrial nurse assesses him and observes use yellow vegetables.
of accessory muscles, severe chest pain, 3. Just because I have COPD doesnt mean that
agitation, shortness of breath. The nurse also I have a higher risk.
notices one side of his chest moving differently 4. Ive worked with asbestos all my life and
than the other. The nurse suspects flail chest. have never had any problems.
What will be the nurses initial action?
1. Apply a sandbag to the flail side of his chest. 282. An adult male was diagnosed with lung cancer
2. Prepare for intubation and mechanical 18 months ago. He is now in the terminal stages
ventilation. and is experiencing severe generalized pain. He
3. Prepare for chest tube placement. has ordered morphine sulfate 10 mg IM q 46 h
prn. What is the most appropriate action by the
4. Administer pain medication.
nurse?
278. What manifestation would the client with 1. Teach him that the pain medicine prescribed
pleural effusion display? will take away all his pain and he will have
1. Pain. no discomfort.
2. Swelling. 2. Counsel him about the addictive qualities of
his prescribed narcotic.
3. Dyspnea.
3. Inform him that he may only ask for the pain
4. Increased sputum production.
medicine every 4 hours and there is nothing
279. An 86-year-old female was admitted to the else you can offer in between medication
hospital two days ago with pneumonia. She now times.
has an order to be up in the chair as much as 4. Encourage him to ask for the pain medicine
possible. How will the nurse plan the clients before the pain becomes too severe.
morning care?
283. A client is admitted to the nursing unit from the
1. Get her up before breakfast. Have her eat in
recovery room following a left pneumonectomy.
the chair, then bathe while still up.
What will the nurse expect in the plan of care?
2. Allow her to eat breakfast in bed, rest for
1. Have a chest tube to water seal.
30 minutes, get up in the chair, and rest for
a few minutes. Allow her to wash her hands 2. Have a chest tube to suction.
and facenurse to complete bath. 3. Be monitored closely for respiratory and
3. Allow her to eat in bed, get her up, and provide cardiac complications.
her with a pan of water for her to bathe. 4. Have his left arm maintained in a sling to
4. Get her up before breakfast, have her bathe prevent pain and discomfort.
before breakfast, eat in the chair, then a rest
284. An adult who has had a right thoracotomy for a
in the chair.
wedge resection of his lung repeatedly refuses to
280. A client has been admitted to the hospital. Lung do breathing or arm exercises because of the
assessment reveals the following: bronchial pain. What should the nurse include on the
breath sounds over (L) lower lobe, diminished clients plan of care?
breath sounds (L) lower lobe, tactile fremitus 1. Offer the client pain medication immediately
present, percussion dulled in this area. Based on after arm exercises are completed.
the assessment findings, what condition does 2. Offer the client sips of ice water prior to a
the nurse suspect? deep breathing and coughing session.
1. Pneumonia. 3. Schedule the clients activity 3045 minutes
2. Asthma. after his IM injection of pain medication.
3. Emphysema. 4. Have the client hold a pillow against his
4. Early left-sided heart failure. abdomen for support.

281. A nurse is teaching a class in a community 285. The nurse may expect a client with suspected
center about lung cancer. Which statement best early ARDS to exhibit which of the following?
demonstrates the clients understanding of the 1. PaO2 of 90, PaCO2 of 45, X-ray showing
risk factors for lung cancer? enlarged heart, bradycardia.
1. My husband smokes, but I dont! So, I really 2. Thick green sputum production, PaO2 of 75,
pH 7.45.

4
dont need to worry about getting lung cancer.

304 NCLEX-RN Review


53155_04_Ch04b_p264-377.qxd 2/26/09 7:32 AM Page 305

3. Restlessness, suprasternal retractions, PaO2 of 65. when there are multiple rib fractures due to
4. Wheezes, slow deep respirations, PaCO2 of trauma.
55, pH of 7.25.
245. 3. Louder breath sounds on the right side of the
286. The client had a removal of the larynx and a chest indicate that the endotracheal tube may be
permanent opening made into the trachea. What misplaced and is aerating only one lung.
is the correct name of this procedure?
246. 2. Fluctuation in the water-seal chamber
1. Total laryngectomy.
demonstrates that the tubing system is patent.
2. Tracheostomy.
3. Radical neck dissection. 247. 1. Intermittent bubbling in the water-seal chamber
4. Partial laryngectomy. indicates that air is leaving the thoracic cavity. If
there is no bubbling in the water-seal chamber, it
287. An adult will undergo a total laryngectomy indicates either obstruction of the tubing or
tomorrow. She is concerned about reexpansion of the lung. Reexpansion of the lung
communicating post-op. The nurse should plan is unlikely, as the tube has just been inserted.
for her to communicate by which method the
first 2448 hours after surgery? 248. 3. Covering the insertion site with petroleum
gauze is a priority nursing measure that prevents
1. Using the artificial larynx.
air from entering the chest cavity.
2. Writing or pointing on a communication board.
3. Using esophageal speech. 249. 1. Low-flow oxygen systems provide an oxygen
4. Using a voice button. concentration that is determined by the amount
of air drawn into the system and the dilution of
288. An adult has had a total laryngectomy. The oxygen with room air.
nurse is discussing options for verbal
communication with the client. Which 250. 4. Oxygen is a flammable gas and smoking is not
statement indicates the client understands the permitted in the area.
available options for verbal communication?
251. 2. The most frequent means of transmission of the
1. Because of the arthritis in my hands, I think the tubercle bacillus is by droplet nuclei. The bacillus
voice button method would be easiest to use. is present in the air as a result of coughing,
2. By the time I leave the hospital, I will be sneezing, and expectorating by infected persons.
able to talk.
3. If I use the esophageal speech, my voice will 252. 1. Ethambutol, isoniazid, streptomycin, and
be high pitched and soft. rifampin are first-line drugs used in the
4. Using an artificial larynx will make me treatment of tuberculosis.
sound sort of monotone.
253. 2. An increased anterior-posterior chest
289. An adult is ready for discharge after undergoing diameter, commonly referred to as barrel
a total laryngectomy. The nurse is discussing chest, is seen in clients with emphysema as a
safety aspects of his home care. Which statement result of chronic hyperinflation of the lungs.
by the client best indicates that he understands
254. 4. The stimulus to breathe in a client with
the safety aspects of his care at home?
emphysema becomes low oxygen levels rather
1. It is okay to swim as long as Im careful. than rising CO2 levels. Frequent pulse oximeter
2. I should use paper tissues to cover my stoma checks are necessary to see how the client
when Im coughing. handles low-flow oxygen administration.
3. I should not wear anything to cover my stoma.
255. 4. This is a description of the normal vesicular
4. I will need to use a humidifier in my home.
breath sounds. They are low pitched, soft
sounds heard over the peripheral lung fields
where air flows through smaller bronchioles.
Answers and Rationales 256. 2. A client with lung cancer demonstrating the
assessment findings provided would indicate a
244. 2. Paradoxical breathing movements (opposite nursing diagnosis of ineffective airway
the normal) are characteristic of flail chest. The clearance. The goal is this client will breathe
flail portion is sucked in on inspiration and without dyspnea or discomfort and maintain a

4
bulges out on expiration. Flail chest occurs patent airway.

ADULT NURSING 305


53155_04_Ch04b_p264-377.qxd 2/26/09 7:32 AM Page 306

257. 4. The sample must be analyzed within and aortic arch bodies become the major stimuli
20 minutes, or if the client has leukocytosis for breathing. When the client with COPD
immediately, to ensure accurate results. receives high levels of O2, the hypoxic drive to
breathe is eliminated. The client experiences
258. 2. Teeth are brushed to reduce contamination, respiratory depression that may lead to apnea.
then the client coughs into the container.
Sputum is best collected in the morning when it 268. 1. The mask must fit properly, as a poor-fitting
is more plentiful. mask reduces the amount of oxygen delivered.
The mask may also cause skin breakdown, so it
259. 3. A positive reaction is present when the is very important to provide skin care. Loosen
induration is greater than 10 mm in diameter. The the strap holding the mask frequently and assess
positive reaction indicates exposure to TB or the the skin.
presence of inactive disease, not active disease.
269. 2. Trach care should be provided once every
260. 2. Vital signs are taken frequently. Nothing is 8 hours.
given by mouth until the cough and swallow
reflexes have returned. Both are important 270. 2. These are signs and symptoms of COPD. The
criteria for discharge. nurse would also need to evaluate breathing
rate/pattern, use of accessory muscles for
261. 3. The Pleur-evac must not be raised above chest breathing, cyanosis, capillary refill, and clubbing
level because it can cause backflow of the fluid of fingernails.
into the pleural space precipitating collapse of
the lung or mediastinal shift. The Pleur-evac 271. 3. A major goal for the COPD client is that the
must remain upright and the chest tube should client will use a breathing pattern that does not
not have traction on it. lead to tiring and to plan activities so that the
client does not become overtired. Care should
262. 2. Noisy, increasing respirations and increasing be spaced, allowing frequent rest periods, to
pulse are signs that the client requires prevent fatigue.
suctioning.
272. 1. Typically, the client with TB will present with
263. 3. Chest physiotherapy should be performed at fatigue, lethargy, nausea, anorexia, weight loss,
least 2 hours after meals to reduce the risk of low-grade fever, and night sweats.
vomiting and aspiration.
273. 1. The client should be placed in respiratory
264. 1. The nurses best initial action should be to isolation until active TB is ruled out. TB is
remove the client from the ventilator, ventilating spread by droplet infection, thus her sputum
the client with an ambu bag. Obviously, the should be handled according to respiratory
client is experiencing respiratory distress and is isolation protocol.
not receiving adequate ventilation. The nurse
should continue to closely assess to determine 274. 3. These medications frequently cause nausea.
the cause and determine if the respiratory The nausea may be decreased if the medications
distress is related to ventilator malfunction or are taken at bedtime.
change in client status.
275. 3. Handwashing is the best tool for prevention of
265. 2. It is important for the nurse to quickly assess the infection. The client should wash her hands
client and determine the cause of the alarm. Once very carefully after any contact with body
the cause has been determined, the nurse must substances, masks, or soiled tissues. The family
intervene promptly to prevent complications. should also use good handwashing techniques.

266. 4. Oxygen supports combustion. Smoking is not 276. 3. Pulmonary care is a vital part of the
permitted in the room while O2 is set up or management of this type of client. Measures are
being administered. A sign should be posted to taken to prevent stasis of secretions and promote
that effect. chest expansion, preventing complications such
as atelectasis and pneumonia.
267. 2. The COPD clients drive to breathe is hypoxia.
In COPD the CO2 level gradually rises over time 277. 2. Based on the clients symptoms the nurse
and central chemoreceptors are no longer suspects impending respiratory failure and
sensitive to high CO2 levels. Instead, the should prepare for intubation and mechanical
peripheral chemoreceptors found in the carotid ventilation.

4 306 NCLEX-RN Review


53155_04_Ch04b_p264-377.qxd 2/26/09 7:32 AM Page 307

278. 3. With pleural effusions, lung expansion may be 284. 3. Thirty or 45 minutes after the administration
restricted and the client will experience of IM pain medication is the time when the pain
dyspnea, primarily on exertion. medication is most effective. Thus, this is the
best time to schedule coughing and deep
279. 2. This plan allows frequent rest periods for the breathing and arm exercises.
client. The client should not rush through
morning care activities as rushing will increase 285. 3. It is common for the client to have
hypoxemia, dyspnea, and fatigue. suprasternal and intercostal retractions as the
client loses lung capacity. The nurse should
280. 1. In a client with pneumonia, bronchial breath anticipate restlessness, apprehension, agitation,
sounds are heard over areas of density or sluggishness, disorientation, and tachycardia.
consolidation. Breath sounds are diminished
when the airflow is decreased as is typical with 286. 1. A total laryngectomy is the removal of the
pneumonia. Tactile fremitus is increased over larynx and formation of the tracheostomy. The
the affected area. Percussion is dulled. In esophagus remains attached to the pharynx. No
pneumonia, the alveoli fill with fluid, red cells, air will enter through the nose. The client will
and white cells creating consolidation. breathe through the tracheostomy. The
procedure is indicated for large glottic tumors
281. 2. Research has shown that there may be a with fixation of vocal cords.
correlation between vitamin A deficiency in the
diet and the development of lung cancer. Daily 287. 2. For the first few days after surgery, the client
consumption of green and yellow vegetables is should communicate by writing. If the client
encouraged. is very tired, a communication board may
be used allowing the client to point to
282. 4. A preventive approach to pain control statements.
provides a more consistent level of relief and
reduces client anxiety, which in turn can reduce 288. 4. An artificial larynx is an electronic device
discomfort and pain. held along the neck and vibration produces
mechanical speech. The speech quality is
283. 3. Post-op respiratory insufficiency may result monotone and artificial.
from an altered level of consciousness related to
anesthesia, pain medications, decreased 289. 4. To substitute for the nose and pharynx, where
respiratory effort secondary to pain, or air is usually warmed and humidified, a
inadequate airway clearance. So the client must humidifier, pans of water, or houseplants
be monitored very closely with frequent vital should be used to increase the humidity in
sign checks and respiratory assessments. the home.

The Gastrointestinal System

OVERVIEW OF ANATOMY Mouth


AND PHYSIOLOGY A. Consists of the lips and oral cavity: provides
entrance and initial processing for nutrients and
The organs of the digestive system are grouped into sensory data, such as taste, texture, and
the alimentary canal (GI tract), consisting of the temperature.
mouth, esophagus, stomach, and small and large B. Oral cavity contains the teeth, used for
intestine; and the accessory digestive organs, mastication, and the tongue, which assists in
including the liver, pancreas, gallbladder, and ductal deglutition, taste sensation, and mastication.
system (Figure 4-16). The primary functions of this C. The salivary glands, located in the mouth, produce
system are movement of food, digestion, absorption, secretions containing ptyalin for starch digestion
elimination, and provision of a continuous supply of and mucus for lubrication.
nutrients, electrolytes, and water.

ADULT NURSING

4 307
53155_04_Ch04b_p264-377.qxd 2/26/09 7:32 AM Page 308

C. Two sphincters control the rate of food passage.


1. Cardiac sphincter: located at the opening
between the esophagus and stomach
2. Pyloric sphincter: located between the
stomach and duodenum
D. Three anatomic divisions: fundus, body, and
antrum.
E. Gastric secretions
1. Pepsinogen: secreted by chief cells, located in
fundus, aids in protein digestion
2. Hydrochloric acid: secreted by parietal cells,
functions in protein digestion, released in
response to gastrin
3. Intrinsic factor: secreted by parietal cells,
promotes absorption of vitamin B12
4. Mucoid secretions: coat stomach wall and
prevent autodigestion

Small Intestine
A. Composed of the duodenum, jejunum, and ileum
B. Extends from the pylorus to the ileocecal valve,
which regulates flow into the large intestine and
prevents reflux into the small intestine.
C. Major functions of the small intestine are
digestion and absorption of the end products
of digestion.
D. Structural features
Figure 4-16 Anterior view of the structures 1. Villi (functional units of the small intestine):
of the GI tract fingerlike projections located in the mucous
membrane; contain goblet cells that secrete
mucus and absorptive cells that absorb
D. The pharynx aids in swallowing and functions in digested foodstuffs.
ingestion by providing a route for food to pass 2. Crypts of Lieberkuhn: produce secretions
from the mouth to the esophagus. containing digestive enzymes.
3. Brunners glands: found in the submucosa of
the duodenum, secrete mucus.
Esophagus
Muscular tube that receives food from the pharynx Large Intestine
and propels it into the stomach by peristalsis.
A. Divided into four parts: cecum (with appendix),
colon (ascending, transverse, descending,
Stomach sigmoid), rectum, and anus.
A. Located on the left side of the abdominal cavity, B. Serves as a reservoir for fecal material until
occupying the hypochondriac, epigastric, and defecation occurs; functions to absorb water and
umbilical regions. electrolytes.
B. Stores and mixes food with gastric juices and C. Microorganisms present in the large intestine are
mucus, producing chemical and mechanical responsible for a small amount of further
changes in the bolus of food. breakdown and also make some vitamins.
1. The secretion of digestive juices is stimulated 1. Amino acids are deaminated by bacteria,
by smelling, tasting, and chewing food, which resulting in ammonia, which is converted to
is known as the cephalic phase of digestion. urea in the liver.
2. The gastric phase is stimulated by the 2. Bacteria in the large intestine aid in the
presence of food in the stomach; regulated by synthesis of vitamin K and some of the
neural stimulation via the PNS and hormonal vitamin B groups.
stimulation through secretions of gastrin by D. Feces (solid waste) leave the body via the rectum
the gastric mucosa. and anus.
3. After processing in the stomach, the food 1. Anus contains internal sphincter (under
bolus, called chyme, is released into the small involuntary control) and external sphincter

4
intestine through the duodenum. (voluntary control)

308 NCLEX-RN Review


53155_04_Ch04b_p264-377.qxd 2/26/09 7:32 AM Page 309

2. Fecal matter usually 75% water and 25% solid 1. Bile is formed in the liver and excreted into
wastes (roughage, dead bacteria, fat, protein, the hepatic duct.
inorganic matter) 2. Hepatic duct joins with the cystic duct (which
drains the gallbladder) to form the common
bile duct.
Liver 3. If sphincter of Oddi is relaxed, bile enters the
A. Largest internal organ; located in the right duodenum. If contracted, bile is stored in
hypochondriac and epigastric regions of the gallbladder.
abdomen.
B. Liver lobules: functional units of the liver,
composed of hepatic cells. Pancreas
C. Hepatic sinusoids (capillaries) are lined with A. Positioned transversely in the upper abdominal
Kupffer cells, which carry out the process of cavity.
phagocytosis. B. Consists of a head, body, and tail along with a
D. Portal circulation brings blood to the liver from the pancreatic duct, which extends along the gland
stomach, spleen, pancreas, and intestines. and enters the duodenum via the common bile
E. Functions duct.
1. Metabolism of fats, carbohydrates, and C. Has both exocrine and endocrine functions;
proteins; oxidizes these nutrients for energy function in GI system is exocrine.
and produces compounds that can be stored 1. Exocrine cells in the pancreas secrete
2. Production of bile trypsinogen and chymotrypsin for protein
3. Conjugation and excretion (in the form of digestion, amylase to break down starch
glycogen, fatty acids, minerals, fat-soluble and to disaccharides, and lipase for fat
water-soluble vitamins) of bilirubin digestion.
4. Storage of vitamins A, D, B12, and iron 2. Endocrine function is related to islets of
5. Synthesis of coagulation factors Langerhans.
6. Detoxification of many drugs and conjugation
of sex hormones
Physiology of Digestion
Biliary System and Absorption
Consists of the gallbladder and associated ductal A. Digestion: physical and chemical breakdown of
system (bile ducts), see Figure 4-17. food into absorptive substances
A. Gallbladder: lies on the undersurface of the liver, 1. Initiated in the mouth where food mixes with
functions to concentrate and store bile. saliva and starch is broken down.
B. Ductal system: provides a route for bile to reach 2. Food then passes into the esophagus where it
the intestines. is propelled into the stomach.
3. In the stomach, food is processed by gastric
secretions into a substance called chyme.
4. In the small intestine, carbohydrates are
hydrolyzed to monosaccharides, fats to
glycerol, and fatty acids and proteins to
amino acids to complete the digestive
process.
a. When chyme enters the duodenum, mucus
is secreted to neutralize hydrochloric acid;
in response to release of secretin, pancreas
releases bicarbonate to neutralize acid
chyme.
b. Cholecystokinin and pancreozymin
(CCK-PZ) are also produced by the
duodenal mucosa; stimulate contraction of
the gallbladder along with relaxation of the
sphincter of Oddi (to allow bile to flow
from the common bile duct into the
duodenum), and stimulate release of
Figure 4-17 Gallbladder and ductal system pancreatic enzymes.

ADULT NURSING

4 309
53155_04_Ch04b_p264-377.qxd 2/26/09 7:32 AM Page 310

ASSESSMENT 5. Hepatic/biliary problems: symptoms may


include
a. Jaundice: note location, duration, notable
Health History increase/decrease in degree.
A. Presenting problem b. Pruritus: note location, distribution, onset.
1. Mouth: symptoms may include dental caries, c. Urine changes: note color, onset, notable
bleeding gums, dryness or increased increase or decrease in color change,
salivation, odors, difficulty chewing (note use associated symptoms (pain).
of dentures) d. Clay-colored stools: note onset,
2. Ingestion: symptoms may include: number/day, associated symptoms (pain,
a. Changes in appetite: anorexia or problems with ingestion/digestion).
hyperorexia; note food e. Increased bleeding: note ecchymoses,
preferences/dislikes. purpura, bleeding gums, hematuria.
b. Food intolerances: allergies, fluid, fatty B. Lifestyle: eating behaviors (rapid ingestion,
foods. skipping meals, snacking), cultural/religious
c. Weight gain/loss: note values (vegetarian, kosher foods), ingestion of
symptoms/situations that might interfere alcohol, smoking
with appetite (stress, deliberate weight C. Use of medications: note use of antacids,
reduction, dental problems); note average antiemetics, antiflatulents, vitamin supplements;
weight and percent gain/loss within past aspirin and anti-inflammatory agents
29 months. D. Past medical history: childhood, adult, psychiatric
d. Dysphagia: note level of sensation where illness; surgery; bleeding disorders; menstrual
problem occurs, whether it occurs with history; exposure to infectious agents; allergies
foods/fluids.
e. Nausea: note onset and duration, existence Physical Examination
of associated symptoms (weakness,
headache, vomiting), occurrence before or A. Mouth: inspect/palpate
after meals. 1. Outer/inner lips: color, texture, moisture
f. Vomiting: note onset and duration; 2. Buccal mucosa: color, texture, lesions,
foods/fluids that can be maintained; ulcerations
associated symptoms (fever, diarrhea). 3. Teeth/gums: missing teeth, cavities,
g. Regurgitation (reflux): note whether occurs tenderness, swelling
with ingestion of certain foods, any 4. Tongue: protrusion without deviation, texture,
associated symptoms (vomiting), color, moisture
occurrence with certain positions (supine, 5. Palates (hard and soft): color
recumbent). B. Abdomen: divided into regions and quadrants (see
3. Digestion/absorption: symptoms may include Figure 4-18); note specific location of any
a. Dyspepsia (indigestion): note location of abnormality.
discomfort, whether associated with 1. Inspect skin: color, scars, striae, pigmentation,
certain foods, time of day/night of lesions, vascularity.
occurrence, associated symptoms 2. Inspect architecture: contour, symmetry,
(vomiting). distension, umbilicus.
b. Heartburn (pyrosis): note location, whether 3. Inspect movement: peristalsis, pulsations.
pain radiates, whether it occurs before or 4. Auscultate peristaltic sounds.
after meals, time of day when discomfort is a. Normal: bubbling, gurgling,
most noticeable, foods that aggravate or 530 times/minute
eliminate symptoms. b. Increased (hyperactive): may indicate
c. Pain: character, frequency, location, diarrhea, gastroenteritis, early intestinal
duration, distribution, aggravating or obstruction
alleviating factors. c. Decreased (hypoactive): may indicate
4. Bowel habits: symptoms may include constipation, late intestinal obstruction,
a. Constipation: note number of stools/day or use of anticholinergics, post-op anesthesia
week, changes in size or color of stool, 5. Auscultate arterial sounds: note presence or
alterations in food/fluid intake, presence of absence of bruits in aorta/renal arteries.
tenesmus, painful defecation, associated 6. Percuss for tenderness/masses; determine
symptoms (abdominal pain, cramps) distribution of tympany and dullness
b. Diarrhea: note number of stools/day, a. Liver span: normal 612 cm dullness at the
consistency, quantity, odor, interference midclavicular line; determine shifting
with ADL, associated symptoms (nausea, dullness (ascites)

4
vomiting, flatus, abdominal distension) b. Stomach: normal tympany

310 NCLEX-RN Review


53155_04_Ch04b_p264-377.qxd 2/26/09 7:32 AM Page 311

2. For fat or infectious organisms collect three


separate specimens and label day #1, day #2,
day #3.
F. Upper GI series (barium swallow)
1. Fluoroscopic examination of upper GI tract to
determine structural problems and gastric
emptying time; client must swallow barium
sulfate or other contrast medium; sequential
films taken as it moves through the system.
2. Nursing care: pretest
a. Keep NPO after midnight or 68 hours
pretest.
b. Explain that the barium will taste chalky.
3. Nursing care: posttest: administer laxatives to
enhance elimination of barium and prevent
obstruction or impaction.
G. Lower GI series (barium enema)
1. Barium is instilled into the colon by enema;
client retains the contrast medium while
X-rays are taken to identify structural
abnormalities of the large intestine or colon.
2. Nursing care: pretest
a. Keep NPO for 8 hours pretest.
b. Give enemas until clear the morning of
test.
c. Administer laxative or suppository.
d. Explain that cramping may be experienced
during the procedure.
3. Nursing care: posttest: administer laxatives
and fluids to assist in expelling barium.
Figure 4-18 Abdominal quadrants (broken lines) and H. Endoscopy (esophagogastroduodenoscopy)
regions (solid lines) 1. Direct visualization of the esophagus, stomach,
and duodenum by insertion of a lighted
fiberscope
2. Used to observe structures, ulcerations,
c. Spleen: normal tympany, dullness only if inflammation, tumors; may include a biopsy
enlarged 3. Nursing care: pretest
d. Small/large intestine: normal tympany a. Keep NPO for 68 hours.
e. Bladder: normal tympany, dullness if full b. Ensure consent form has been signed.
7. Palpate to depth of 1 cm (light palpation) to c. Explain that a local anesthetic will be used
determine areas of tenderness, muscle to ease discomfort and that speaking
guarding, and masses during the procedure will not be possible;
8. Palpate to a depth of 48 cm (deep palpation) the client should expect hoarseness and a
to identify rigidity, masses, ascites, tenderness, sore throat for several days.
liver margins, spleen 4. Nursing care: posttest
a. Keep NPO until return of gag reflex.
b. Assess vital signs and for pain, dysphagia,
Laboratory/Diagnostic Tests bleeding.
A. Blood chemistry and electrolyte analysis: albumin, c. Administer warm normal saline gargles for
alkaline phosphatase, ammonia, amylase, relief of sore throat.
bilirubin, chloride, LDH, lipase, potassium, SGOT I. Colonoscopy
or AST, serum glutamic pyruvic transaminase 1. Endoscopic visualization of the large intestine:
(SGPT or ALT), sodium may include biopsy and removal of foreign
B. Hematologic studies: Hgb and HCT, PT, WBC substances.
C. Serologic studies: carcinoembryonic antigen (CEA), 2. Nursing care: pretest
hepatitis-associated antigens, Helicobacter pylori a. Keep NPO for 8 hours pretest.
D. Urine studies: amylase, bilirubin b. Administer laxatives for 13 days before
E. Fecal studies: for blood, fat, infectious organisms the exam, and sometimes enemas until
1. A freshly passed, warm stool is the best clear the night before the test.

4
specimen. c. Ensure a consent form has been signed.

ADULT NURSING 311


53155_04_Ch04b_p264-377.qxd 2/26/09 7:32 AM Page 312

d. Explain to client that when the instrument b. Place client on right side for a few hours
is inserted into the rectum a feeling of with a pillow against the abdomen to
pressure might be experienced. provide pressure on the liver.
3. Nursing care: posttest c. Observe puncture site for hemorrhage.
a. Observe for rectal bleeding and signs of d. Assess for complications of shock and
perforation. pneumothorax.
b. Schedule planned rest periods for the client. N. Endoscopic Retrograde Cholangiopancreatography
J. Sigmoidoscopy (ERCP)
1. Endoscopic visualization of the sigmoid colon 1. Flexible fiber-optic endoscope permits direct
2. Used to identify inflammation or lesions, or visualization of biliary structures and
remove foreign bodies. pancreas.
3. Nursing care: pretest 2. Consent and NPO status required.
a. Offer a light supper and light breakfast. 3. Moderate sedation used.
b. Do bowel prep.
c. Explain to client that the sensation of an
urge to defecate or abdominal cramping ANALYSIS
might be experienced.
4. Nursing care: posttest: assess for signs of Nursing diagnoses for the client with a disorder of the
bowel perforation. digestive system may include:
K. Gastric analysis A. Risk for deficient fluid volume
1. Insertion of a nasogastric tube to examine B. Disturbed body image
fasting gastric contents for acidity and C. Imbalanced nutrition: less than body requirements
volume D. Diarrhea
2. Nursing care: pretest E. Constipation
a. Keep NPO 68 hours pretest. F. Pain
b. Advise client about no smoking, G. Ineffective breathing pattern
anticholinergic medications, antacids for H. Impaired verbal communication
24 hours prior to test. I. Impaired skin integrity
c. Inform client that tube will be inserted into
the stomach via the nose, and instruct to
expectorate saliva to prevent buffering of
secretions. PLANNING AND
3. Nursing care: posttest: provide frequent mouth IMPLEMENTATION
care.
L. Oral cholecystogram
1. Injection of a radiopaque dye and X-ray Goals
examination to visualize the gallbladder A. Restoration of fluid and electrolyte balance.
2. Used to determine the gallbladders ability to B. Client will express feelings of self-worth.
concentrate and store the dye and to assess C. Adequate nutritional status will be maintained.
patency of the biliary duct system D. Client will experience decreased frequency of
3. Nursing care: pretest regular bowel habits.
a. Offer a low-fat meal the evening before the E. Client will establish regular bowel habits of
test and black coffee, tea, or water the appropriate amount and consistency.
morning of the exam. F. Client will be free from pain.
b. Check for iodine sensitivity and administer G. Effective breathing patterns will be maintained.
six dye tablets (Telepaque) as ordered. H. Effective communication methods will be
4. Nursing care: posttest: observe for side effects established.
of the dye (nausea, vomiting, diarrhea). I. Skin integrity will be restored/maintained.
M. Liver biopsy (closed needle)
1. Invasive procedure where a specially designed
needle is inserted into the liver to remove a Interventions
small piece of tissue for study Enemas
2. Nursing care: pretest
a. Ensure client has signed consent form. A. General information
b. Keep NPO 68 hours pretest. 1. Instillation of fluid into the rectum, usually for
c. Instruct client to hold breath during the the purpose of stimulating defecation. The
biopsy. various types include:
3. Nursing care: posttest a. Cleansing enema (tap water, normal saline,
a. Assess vital signs every hour for or soap): used to treat constipation or feces

4
812 hours. impaction, as bowel cleansing prior to

312 NCLEX-RN Review


53155_04_Ch04b_p264-377.qxd 2/26/09 7:32 AM Page 313

diagnostic procedures or surgery, to help 3. Promote adequate nutrition.


establish regular bowel functions. a. Administer feeding with client in high-
b. Retention enema (mineral oil, olive oil, Fowlers and keep head of bed elevated for
cottonseed oil): usually administered to 30 minutes after meals to prevent
lubricate or soften a hard fecal mass to regurgitation.
facilitate defecation. b. Maintain feeding at room temperature.
B. Nursing care for a cleansing enema c. Ensure that prescribed amount of feeding
1. Explain procedure and that breathing through be given within prescribed amount of time.
the mouth relaxes abdominal musculature and d. Weigh client daily.
helps to avoid cramps; explain the need to e. Monitor I&O until feedings are well
take adequate time to defecate. tolerated.
2. Assemble equipment: prepare solution at f. Monitor for signs of dehydration.
105110 and have bedpan, commode, or
nearby bathroom ready for use. Nasogastric (NG) Tubes
3. Position client and drape adequately.
4. Place waterproof pad under buttocks. A. General information
5. Lubricate tube and allow solution to fill the 1. Soft rubber or plastic tube inserted through a
tubing, displacing air. nostril and into the stomach for gastric
6. Insert rectal tube 45 inches without using force; decompression, feeding, or obtaining
request that client take several deep breaths. specimens for analysis of stomach contents
7. Administer 5001000 mL of solution over 2. Types
510 minutes; if cramping occurs slow the a. Levin: single-lumen, nonvented
speed of instillation. b. Salem: a tube within a tube; vented to
8. After administration, have the client retain provide constant inflow of atmospheric air
solution until the urge to defecate becomes B. Nursing care
strong. 1. Insertion of the tube
9. Document amount, color, characteristics of a. Explain the purpose of the tube and the
stool, and clients reaction during procedure. procedure for insertion.
10. Assess for dizziness, light-headedness, b. Measure the tube: distance on the tube
abdominal cramps, nausea. from the tip of the nose to the ear lobe plus
11. Monitor electrolyte levels if client is to receive the distance from the ear lobe to the tip of
repeated enemas. the xiphoid.
C. Nursing care for a retention enema: same as for a c. Instruct client to bend head forward if
cleansing enema except: possible during insertion.
1. Oil is used instead of water (comes prepared 2. Monitor functioning of system and ensure
in commercial kits and is given at body patency of the NG tube: abdominal
temperature). discomfort/distension, nausea and vomiting,
2. Administer 150200 mL of prepared solution. and little or no drainage in collection bottle
3. Instruct client to retain oil for at least are all signs that system is not functioning
30 minutes in order for it to take effect. properly.
a. Assess the position: aspirate gastric
Gastrostomy contents to confirm that tube is in stomach;
inject 10 mL air through tube and
A. General information auscultate for rapid influx.
1. Insertion of a catheter through an abdominal b. Check that tubing is free of kinks; irrigate
incision into the stomach where it is secured as per physician order.
with sutures or by balloon fixation. c. Record amount, color, and odor of
2. Used as an alternative method of feeding, drainage.
either temporary or permanent, for clients who 3. Provide measures to ensure maximal comfort.
have problems with swallowing, ingestion, a. Apply water-soluble lubricant to lips to
and digestion. prevent dryness.
B. Nursing care b. Keep nares free from secretions.
1. Maintain skin integrity: inspect and cleanse c. Provide periodic warm saline gargles to
skin around stoma frequently; keep deep area prevent dryness.
dry to avoid excoriation. d. Provide frequent mouth care with
2. Maintain patency of the gastrostomy tube. toothbrush/toothpaste or flavored
a. Assess for residual before each feeding (check mouthwashes.
orders concerning withholding feeding). e. If allowed, give client hard candy or gum
b. Irrigate tube before and after meals. to stimulate the flow of saliva and prevent

4
c. Measure/record any drainage. dryness.

ADULT NURSING 313


53155_04_Ch04b_p264-377.qxd 2/26/09 7:32 AM Page 314

f. Elevate head and chest during and for 12 F. Relaxed facial expression; decreased abdominal
hours after feedings to prevent reflux (most distension; healed mouth ulcers.
comfortable position when suction is used). G. Improved respiratory rate, depth, and rhythm;
4. Monitor/maintain fluid and electrolyte lungs clear to auscultation; effective use of
balance. muscles of respiration.
a. Assess for signs of metabolic alkalosis H. Client effectively uses artificial means of
(suctioning causes excessive loss of communication (artificial larynx, sign language, or
hydrochloric acid and potassium). esophageal speech).
b. Administer IV fluids as ordered. I. No redness, irritation, or breakdown; client
c. If suction used, irrigate NG tube with demonstrates techniques to prevent skin
normal saline to decrease sodium loss. breakdown.
d. Keep accurate I&O.
e. If suction used provide ice chips sparingly
(if allowed) to avoid dilution of DISORDERS OF THE
electrolytes.
f. Monitor lab values and electrolytes GASTROINTESTINAL SYSTEM
frequently.
Nausea and Vomiting
Intestinal Tubes A. General information
A. General information 1. Nausea: a feeling of discomfort in the
1. Tube is inserted via a nostril through the epigastrium with a conscious desire to vomit;
stomach and into the intestine for occurs in association with and prior to
decompression proximal to an obstruction, vomiting.
relief of an obstruction, decompression of 2. Vomiting: forceful ejection of stomach
post-op edema at the surgical site. contents from the upper GI tract. Emetic center
2. Types in medulla is stimulated (e.g., by local
a. Cantor tube: single lumen irritation of intestine or stomach or
b. Harris tube: single lumen disturbance of equilibrium), causing the
c. Miller-Abbott: double lumen vomiting reflex.
B. Nursing care 3. Nausea and vomiting are the two most
1. Facilitate placement of the tube. common manifestations of GI disease.
a. Position client in high-Fowlers while tube 4. Contributing factors
is being passed from the nose to the a. GI disease
stomach; then place client on right side to b. CNS disorders (meningitis, CNS lesions)
aid in advancing the tube from the stomach c. Circulatory problems (HF)
to duodenum. d. Metabolic disorders (uremia)
b. Continuously monitor tube markings. e. Side effects of certain drugs
c. Tape tube in place only after placement in (chemotherapy, antibiotics)
duodenum is confirmed by X-ray. f. Pain
2. Provide measures for maximal comfort, as for g. Psychic trauma
NG tube. h. Response to motion
B. Assessment findings
1. Weakness, fatigue, pallor, possible lethargy
EVALUATION 2. Dry mucous membrane and poor skin
turgor/mobility (if prolonged with
A. Adequate urine output; stable vital signs; moist dehydration)
mucous membranes; adequate skin turgor and 3. Serum sodium, calcium, potassium decreased
mobility; electrolyte levels within normal range. 4. BUN elevated (if severe vomiting and
B. Client expresses interest in personal well-being; dehydration)
actively participates in ADL, treatments, and care. C. Nursing interventions
C. Stable weight; improved anthropometric 1. Maintain NPO until client able to tolerate oral
measurements; laboratory values within normal intake.
limits; client verbalizes types of foods that should 2. Administer medications as ordered and
be included or eliminated from prescribed diet. monitor effects/side effects.
D. Client reports reduction in frequency of stools and a. Phenothiazines: chlorpromazine
return to more normal stool consistency; (Thorazine), perphenazine (Trilafon),
laboratory values within normal range. prochlorperazine (Compazine),
E. Client reports increased frequency with improved Promethazine (Phenergan), trifluoperazine

4
consistency of stool. (Stelazine)

314 NCLEX-RN Review


53155_04_Ch04b_p264-377.qxd 2/26/09 7:32 AM Page 315

b. Antihistamines: benzquinamide (Emete- d. Biliary tract disorders


con), dimenhydrinate (Dramimine), e. Hyperthyroidism
diphenhydramine (Benadryl), hydroxyzine f. Saline laxatives
(Atarax, Vistaril), cyclizine (Marezine), g. Magnesium-based antacids
meclizine (Antivert), promethazine h. Stress
(Phenergan) i. Antibiotics
c. Other drugs to help control nausea and j. Neoplasms
vomiting: thiethylperazine (Torecan), k. Highly seasoned foods
trimethobenzamide (Tigan), B. Assessment findings
metoclopramide (Reglan) 1. Abdominal cramps/distension, foul-smelling
3. Notify physician if vomiting pattern changes. watery stools, increased peristalsis
4. Maintain fluid and electrolyte balance. 2. Anorexia, thirst, tenesmus, anxiety
a. Administer IV fluids as ordered, keep 3. Decreased potassium and sodium if severe
accurate record of I&O. C. Nursing interventions
b. Record amount/frequency of vomitus. 1. Administer antidiarrheals: diphenoxylate with
c. Assess skin tone/turgor for degree of atropine (Lomotil), paregoric, loperamide
hydration. (Imodium), Kaopectate as ordered; monitor
d. Monitor laboratory/electrolyte values. effects.
e. Test NG tube drainage or vomitus for 2. Control fluid/food intake.
blood, bile; monitor pH. a. Avoid milk and milk products.
5. Provide measures for maximum comfort. b. Provide liquids with gradual introduction
a. Institute frequent mouth care with tepid of bland, high-protein, high-calorie, low-
water/saline mouthwashes. fat, low-bulk foods.
b. Remove encrustations around nares. 3. Monitor and maintain fluid and electrolyte
c. Keep head of bed elevated and avoid status; record number, characteristics, and
sudden changes in position. amount of each stool.
d. Eliminate noxious stimuli from 4. Prevent anal excoriation.
environment. a. Cleanse rectal area after each bowel
e. Keep emesis basin clean. movement with mild soap and water and
f. Maintain quiet environment and avoid pat dry.
unnecessary procedures. b. Apply A and D ointment or Desitin to
6. When vomiting subsides provide clear fluids promote healing.
(ginger ale, warm tea) in small amounts, c. Use a local anesthetic as needed.
gradually introduce solid foods (toast, 5. Provide client teaching and discharge
crackers), and progress to bland foods (baked planning concerning
potato), in small amounts. a. Medication regimen
7. Provide client teaching and discharge b. Adherence to prescribed diet and
planning concerning: avoidance of foods that are known to
a. Avoidance of situations, foods, or liquids produce diarrhea
that precipitate nausea and vomiting c. Importance of perineal hygiene and care
b. Need for planned, uninterrupted rest periods and daily assessment of skin changes
c. Medication regimen, including side effects d. Importance of good handwashing
d. Signs of dehydration techniques after each stool
e. Need for daily weights with frequent e. Need to report worsening of symptoms
anthropometric measurements (increased abdominal cramps, increased
frequency or amount of stool)
f. Need to assess daily weights with frequent
Anorexia/Eating Disorders anthropometric measurements
See Unit 7.
Constipation
Diarrhea A. General information
A. General information 1. Lengthening of normal (for individual) time
1. Increase in peristaltic motility, producing period between bowel movements; small
watery or loosely formed stools. Diarrhea is a volume of dry, hard stool; results from
symptom of other pathologic processes. decreased motility of the colon or from
2. Causes retention of feces in the colon or rectum
a. Chronic bowel disorders 2. Causes
b. Malabsorption problems a. Inadequate bulk/liquids in the diet

4
c. Intestinal infections b. Lack of physical activity

ADULT NURSING 315


53155_04_Ch04b_p264-377.qxd 2/26/09 7:32 AM Page 316

c. Retention of barium after radiographic 3. Surgery: type depends on location and extent
exam of the tumor
d. Prolonged use of constipation medications a. Mandibulectomy: removal of the mandible
(aluminum-based antacids, anticholinergics, b. Hemiglossectomy: removal of half the
antihistamines, antidepressants, tongue
phenothiazines, calcium, iron) c. Glossectomy: removal of the entire tongue
B. Assessment findings d. Radical neck dissection
1. Feeling of abdominal fullness, pressure in the C. Assessment findings
rectum; abdominal distension, dyschezia; 1. Ulcerations (often painless) on the lip, tongue,
increased flatus or buccal mucosa
2. Hardened stool upon digital examination 2. Pain or soreness of the tongue upon eating hot
C. Nursing interventions or highly seasoned foods
1. Promote adequate intake of fluids/foods and 3. Erythroplakia, leukoplakia
dietary modification: increase fluid intake to at 4. Difficulty chewing/speaking, dysphagia
least 3000 mL/day; include high-fiber foods in 5. Positive oral exfoliative cytology
diet. 6. Positive toluidine blue test
2. Administer medications as ordered D. Nursing interventions
a. Cathartics: milk of magnesia, castor oil, 1. Provide nursing care for the client receiving
cascara sagrada, senna (Senokot), bisacodyl radiation therapy
(Dulcolax), psyllium (Metamucil) 2. Prepare client for surgery: in addition to
b. Stool softeners: docusate calcium (Surfak), routine pre-op care
docusate sodium (Colace) a. Inform client of expected changes post-op.
3. Prevent accumulation of stool in the b. Provide explanation of anticipated post-op
colon/rectum. suctioning, NG tube, drains.
a. Instruct client not to suppress urge to 3. In addition to routine post-op care
defecate. a. Promote drainage.
b. Gently massage abdomen to promote 1) Place in side-lying position initially,
stimulation and movement of feces. then Fowlers.
4. Provide client teaching and discharge 2) Suction mouth (except for lip surgery).
planning concerning: 3) Maintain patency of drainage tubes.
a. Need to establish and maintain a regular b. Promote oral hygiene/comfort.
time to defecate 1) Provide mouth irrigations with sterile
b. Diet modification water, diluted peroxide, normal saline,
c. Medication regimen or sodium bicarbonate.
d. Need to assume position of comfort when 2) Avoid use of commercial
sitting on toilet mouthwashes, lemon and glycerine
swabs.
c. Monitor/promote optimum nutritional
Cancer of the Mouth status.
A. General information 1) Provide tube feedings following a
1. Cancer of the mouth may occur on the lips or hemiglossectomy.
within the mouth (tongue, floor of mouth, 2) Place oral fluids in back of the throat
buccal mucosa, hard/soft palate, pharynx, with an asepto syringe.
tonsils). 3) Provide foods/fluids that are
2. Most common type of oral tumor is squamous nonirritating and facilitate swallowing
cell carcinoma; most malignancies occur on (yogurt, puddings).
the lower lip. d. Monitor for signs and symptoms of facial
3. More common in men. nerve damage (drooping, uneven smile,
4. Caused by circumoral numbness or tingling).
a. Excessive sun exposure
b. Tobacco (cigar, pipe, cigarette, snuff)
c. Excessive alcohol intake
Cancer of the Esophagus
d. Constant irritation (dental caries) A. General information
5. Early detection is very important; most 1. Malignant tumors of the esophagus usually
discovered by dentists in routine checkups. appear as ulcerated lesions, most often in
B. Medical management middle and lower portions of the esophagus.
1. Radiation therapy: both primary lesion and 2. Penetration of the muscular layers with
affected lymph nodes; radioactive implants extension to the outer wall of the esophagus is
2. Chemotherapy: sometimes indicated, not used commonly found. Metastases may cause

4
as often as radiation therapy and surgery eventual esophageal obstruction.

316 NCLEX-RN Review


53155_04_Ch04b_p264-377.qxd 2/26/09 7:32 AM Page 317

3. More common in men than in women (4:1); Esophageal Varices


usually between ages 5070.
4. Cause unknown; contributing factors include See Disorders of the Liver.
cigarette smoking, excessive alcohol intake,
trauma, poor oral hygiene, achalasia, Hiatal Hernia
diverticula, and lye burns.
B. Medical management A. General information
1. Radiation therapy: used for inoperable tumors, 1. Sliding hiatal hernia occurs when a portion of
has been found to alleviate symptoms the stomach and vagus nerve slide upward
2. Chemotherapy: not found effective into the thorax through an enlarged hiatus in
3. Surgery the diaphragm.
a. Esophagectomy: removal of part or all of 2. Result is reflux of gastric juices and inflammation
the esophagus using a Dacron graft to of the lower portion of the esophagus.
replace the resected portion 3. Occurs more often in women ages 4070.
b. Esophagogastrostomy: resection of a 4. May be caused by congenital weakening of the
portion of the esophagus (usually middle muscles in the diaphragm around the
third) and anastomosis of the remaining esophagogastric opening; increased intra-
portion of the stomach abdominal pressure (obesity, pregnancy,
c. Esophagoenterostomy: resection of portion ascites); trauma.
of the esophagus and anastomosis of a B. Medical management
segment of colon to the remaining portion 1. Drug therapy: antacids to reduce acidity and
d. Palliative gastrostomy: done for the relieve discomfort, cholinergics
purpose of feeding the client 2. Modification of diet: elimination of spicy
C. Assessment findings foods and caffeine
1. Substernal burning after drinking hot fluids 3. Surgery: reduction of the hiatal hernia via an
2. Pain located in the substernal and epigastric abdominal or thoracic approach
areas; usually intensified with swallowing C. Assessment findings
3. Weight loss 1. Heartburn, especially after meals, at night, or
4. Barium swallow reveals narrowing of the with position changes (particularly
esophagus at the area of the tumor recumbent), dysphagia, regurgitation several
5. Diagnostic test: esophagoscopy with a biopsy hours after meals without vomiting
reveals malignant cells 2. Barium swallow displays protrusion of the
D. Nursing interventions gastric mucosa through a hiatus
1. Provide nursing care for the client receiving 3. Esophagoscopy reveals an incompetent cardiac
radiation therapy. sphincter
2. Prepare client for surgery: in addition to D. Nursing interventions
routine pre-op care 1. Provide a bland diet with six small
a. Provide meticulous oral hygiene including feedings/day, as ordered.
teeth, gums, tongue, and mouth. 2. Administer medications as ordered.
b. Explain that client may have a chest tube if 3. Prepare client for surgery: in addition to
thoracic approach is used. routine pre-op care
c. Prepare client for feedings through a a. Inform client about chest tubes (if thoracic
gastrostomy. approach to be used).
3. In addition to routine post-op care b. Provide information regarding NG
a. Monitor NG tube: expect bloody drainage intubation.
for approximately 12 hours with gradual 4. In addition to routine post-op care
change to green, then to yellow. a. Decrease/avoid gastric distension.
b. Prevent gastric reflux: place client in semi- b. Promote pulmonary expansion: chest tubes if
Fowlers position; maintain upright a thoracic approach; semi-Fowlers position.
position for 2 hours after meals when 5. Provide client teaching and discharge
client is able to take fluids/food orally. planning concerning:
4. Provide emotional support to client/significant a. Modification of diet
others; prognosis is grave. b. Sitting up for meals and for 2 hours after
5. Provide client teaching and discharge meals will help reduce gastric acid reflux
planning concerning c. Use of antacids
a. Gastrostomy and proper dietary measures d. Eating small, frequent meals slowly to help
b. Importance of cessation of smoking and prevent gastric distension
elimination of alcohol consumption e. Need to avoid carbonated beverages and
c. Maintain good oral hygiene. anticholinergic drugs (and OTC

4
d. Maintain a high-calorie, high-protein diet medications that contain them)

ADULT NURSING 317


53155_04_Ch04b_p264-377.qxd 2/26/09 7:32 AM Page 318

f. Avoidance of heavy lifting (to prevent 6. Predisposing factors include smoking, alcohol
intra-abdominal pressure); bend, kneel, or abuse, emotional tension, and drugs
stoop instead (salicylates, steroids, Butazolidin)
g. Importance of treating persistent cough 7. Caused by bacterial infection (Helicobacter
h. Adherence to weight-reduction plan if obese pylori)
B. Medical management
1. Supportive: rest, bland diet, stress
Gastritis management
A. General information 2. Drug therapy: antacids, histamine (H2)
1. An acute inflammatory condition that causes a receptor antagonists, anticholinergics,
breakdown of the normal gastric protective omeprazole (Prilosec), sucralfate (Carafate);
barriers with subsequent diffusion of also metronidazole and amoxacillin for ulcers
hydrochloric acid into the gastric lumen caused by H. pylori
2. Results in hemorrhage, ulceration, and 3. Surgery: various combinations of gastric
adhesions of the gastric mucosa resections and anastomosis
3. Present in some form (mild to severe) in 50% C. Assessment findings
of all adults 1. Pain located in left epigastrium, with possible
4. Caused by excessive ingestion of certain drugs radiation to the back; usually occurs 12 hours
(salicylates, steroids, Butazolidin), alcohol; after meals
food poisoning; large quantities of spicy, 2. Weight loss
irritating foods in diet 3. Hgb and HCT decreased (if anemic)
B. Assessment findings 4. Endoscopy reveals ulceration; differentiates
1. Anorexia, nausea and vomiting, hematemesis, ulcers from gastric cancer
epigastric fullness/discomfort, epigastric 5. Gastric analysis: normal gastric acidity in
tenderness gastric ulcer, increased in duodenal ulcer
2. Decreased Hgb and HCT (if anemic) 6. Upper GI series: presence of ulcer
3. Endoscopy: inflammation and ulceration of confirmed
gastric mucosa D. Nursing interventions
4. Gastric analysis: hydrochloric acid usually 1. Administer medications as ordered
increased, except in atrophic gastritis (see Table 4-23).
C. Nursing interventions 2. Provide nursing care for the client with ulcer
1. Monitor and maintain fluid and electrolyte surgery.
balances. 3. Provide client teaching and discharge
2. Control nausea and vomiting (NPO until able planning concerning
to tolerate foods, then bland diet). a. Medical regimen
3. Administer medications as ordered: 1) Take medications at prescribed
antiemetics, antacids, sedatives. times.
4. Maintain patency of NG tube. 2) Have antacids available at all
5. Provide client teaching and discharge times.
planning concerning avoidance of 3) Recognize situations that would
foods/medications such as coffee, spicy foods, increase the need for antacids.
alcohol, salicylates, ibuprofen, steroids. 4) Avoid ulcerogenic drugs (salicylates,
steroids).
5) Know proper dosage, action, and side
Peptic Ulcer Disease effects.
Gastric Ulcers b. Proper diet
1) Bland diet consisting of six small
A. General information meals/day.
1. Ulceration of the mucosal lining of the 2) Eat meals slowly.
stomach; most commonly found in the antrum 3) Avoid acid-producing substances
2. Gastric secretions and stomach emptying rate (caffeine, alcohol, highly seasoned
usually normal foods).
3. Rapid diffusion of gastric acid from the gastric 4) Avoid stressful situations at
lumen into gastric mucosa, however, causes an mealtime.
inflammatory reaction with tissue breakdown 5) Plan for rest periods after meals.
4. Also characterized by reflux into the stomach 6) Avoid late bedtime snacks.
of bile containing duodenal contents c. Avoidance of stress-producing situations
5. Occurs more often in men, in unskilled and development of stress-reduction
laborers, and in lower socioeconomic groups; methods (relaxation techniques, exercises,

4
peak age 4055 years biofeedback).

318 NCLEX-RN Review


53155_04_Ch04b_p264-377.qxd 2/26/09 7:32 AM Page 319

Table 4-23 Drug Therapy for Peptic Ulcer

Drug Type Action Side Effects Nursing Implications


TRANQUILIZERS
Combination drug Decrease vagal activity and Sedation, headache, mental Contraindicated with other CNS
(Librax); reduce anxiety depression, blurred vision, depressants, antidepressants; avoid
chlordiazepoxide nausea, vomiting, diarrhea, alcohol use, narcotic analgesics.
(Librium) and physical/psychological
clidinium bromide dependence
(Quarzan)
ANTICHOLINERGICS
Belladonna tincture Decreases acetylcholine, Dry mouth, constipation Contraindicated in narrow-angle
block cholinergic receptors glaucoma, myasthenia gravis,
paralytic ileus, urinary retention.
Pirenzepine Blocks muscarinic receptors No severe anticholinergic
(Gastrozepine) that regulate gastric acid side effects
secretion
Propantheline bromine Decreases gastric secretions; Standard anticholinergic
(Pro-Banthine) used in irritable bowel effects: dry mouth,
syndrome, pancreatitis, decreased secretions,
urinary bladder spasm tachycardia, urinary
retention

Tridihexethyl chloride Decreases gastric secretions


(Pathilon)
ANTACIDS
Aluminum hydroxide, Neutralize gastric acid and Prolonged use may cause
aluminum carbonate reduce pepsin activity hypophosphatemia
(Amphogel, Alternagel)

Magnesium hydroxide, Hypermagnesemia Contraindicated in impaired renal


magnesium trisilicate, function.
magnesium phosphate
(Maalox, Gaviscon)
HISTAMINE (H2)
BLOCKERS
Cimetadine (Tagamet) Block H2 receptor sites of Headaches, dizziness, Do not give within 1 hour of antacids.
Rantadine (Zantac) parietal cells of stomach constipation, pruritus, Cimetidine may enhance effects of
Famotidine (Pepcid) skin rash, gynecomastia, oral anticoagulants, theophylline,
Nizatidine (Axid) decreased libido, caffeine, phenytoin, diazepam,
impotence propranolol, phenobarbital,
calcium channel blockers;
rantadine and famotidine have
fewer side effects.
PROTON PUMP
INHIBITORS (PPIs)
Esomeprazole Inhibit gastric secretion
magnesium (Nexium) regardless of
Lansoprazole (Prevacid) acetylcholine or
Omeprazole (Prilosec) histamine release; used
Pantoprazole (Protonix) in treatment of erosive
Rabeprazole (Aciphex) esophagitis/GERD,
gastric and duodenal
ulcers, H. pylori
(continues)

ADULT NURSING

4 319
53155_04_Ch04b_p264-377.qxd 2/26/09 7:32 AM Page 320

Table 4-23 Drug Therapy for Peptic Ulcer (continued)


Drug Type Action Side Effects Nursing Implications
PEPSIN INHIBITOR
Sucralfate (Carafate) Reacts with acid to form a Dizziness, nausea, Mucosal protective drug; must be
paste that binds to constipation, dry mouth, given 30 minutes before meals and
ulcerated tissue to prevent rash, pruritus, back pain, at bedtime.
further destruction by sleepiness
digestive enzyme pepsin
PROSTAGLANDIN E1
ANALOGUE
Misoprostol (Cytotec) Synthetic prostaglandin Diarrhea, abdominal pain,
replacement product that flatulence, nausea,
blocks secretion of excess vomiting, constipation,
acid and protects stomach menstrual spotting
mucosa; used adjunctively
with long-term NSAIDS
or ASA therapy
ANTI-INFECTIVES
Metronidazole Used in dual, triple,
hydrochloride (Flagyl, quadruple combination
Protostat) therapy for treatment of
Amoxicillin (Amoxil) H. pylori
Clarithromycin (Biaxin)
Tetracycline

Duodenal Ulcers b. Antrectomy: removal of the antrum of the


stomach to eliminate the gastric phase of
A. General information digestion
1. Most commonly found in the first 2 cm of the c. Pyloroplasty: enlargement of the pyloric
duodenum sphincter with acceleration of gastric
2. Occur more frequently than gastric ulcers emptying
3. Characterized by gastric hyperacidity and a d. Gastroduodenostomy (Billroth I): removal
significant increased rate of gastric emptying of the lower portion of the stomach with
4. Occur more often in younger men; more women anastomosis of the remaining portion of
affected after menopause; peak age 3545 years the duodenum
5. Predisposing factors include smoking, alcohol e. Gastrojejunostomy (Billroth II): removal
abuse, psychological stress, bacterial infection of the antrum and distal portion of the
(H. pylori) stomach and duodenum with anastomosis
B. Medical management: same as for gastric ulcers of the remaining portion of the stomach to
C. Assessment findings the jejunum
1. Pain located in midepigastrium and described f. Gastrectomy: removal of 6080% of the
as burning, cramping; usually occurs 24 stomach
hours after meals and is relieved by food. g. Esophagojejunostomy (total gastrectomy):
2. Diagnostic tests: same as for gastric ulcer. removal of the entire stomach with a
D. Nursing interventions: same as for gastric ulcer loop of jejunum anastomosed to the
esophagus
Gastric Surgery 3. Dumping syndrome
a. Abrupt emptying of stomach contents into
A. General information the intestine
1. Surgery is performed when peptic ulcer b. Common complications of gastric
disease does not respond to medical surgery
management or for gastric cancer c. Associated with the presence of
2. Types hyperosmolar chyme in the jejunum,
a. Vagotomy: severing of part of the vagus which draws fluid by osmosis from the
nerve innervating the stomach to decrease extracellular fluid into the bowel.

4
gastric acid secretion Decreased plasma volume and distension

320 NCLEX-RN Review


53155_04_Ch04b_p264-377.qxd 2/26/09 7:32 AM Page 321

of the bowel stimulates increased intestinal into adjacent tissues, lymphatics, regional
motility. lymph nodes, and other abdominal organs, or
d. Signs and symptoms include weakness, through the bloodstream to the lungs and
faintness, palpitations, diaphoresis, feeling bones.
of fullness, or discomfort, nausea, and 2. Affects men twice as often as women; more
occasionally diarrhea; appear frequent in African Americans and Asians;
1530 minutes after meals and last most commonly occurs between ages 50
for 2060 minutes. and 70
B. Nursing interventions: routine preoperative 3. Causes
care a. Excessive intake of highly salted or
C. Nursing interventions: postoperative smoked foods
1. Provide routine post-op care. b. Diet low in quantity of vegetables and
2. Ensure adequate function of NG tube. fruits
a. Measure drainage accurately to determine c. Atrophic gastritis
necessity for fluid and electrolyte d. Achlorhydria
replacement; notify physician if there is e. Helicobacter pylori infection
no drainage. B. Medical management
b. Anticipate frank, red bleeding for 1. Chemotherapy
1224 hours. 2. Radiation therapy
3. Promote adequate pulmonary ventilation. 3. Treatment for anemia, gastric decompression,
a. Place client in mid- or high-Fowlers nutritional support, fluid and electrolyte
position to promote chest expansion. maintenance
b. Teach client to splint high upper 4. Surgery: type depends on location and extent
abdominal incision before turning, of lesion.
coughing, and deep breathing. a. Subtotal gastrectomy (Billroth I or II)
4. Promote adequate nutrition. b. Total gastrectomy
a. After removal of NG tube, provide clear C. Assessment findings
liquids with gradual introduction of small 1. Fatigue, weakness, dizziness, shortness of
amounts of bland food at frequent breath, nausea and vomiting, hematemesis,
intervals. weight loss, indigestion, epigastric fullness,
b. Monitor weight daily. feeling of early satiety when eating, epigastric
c. Assess for regurgitation; if present, instruct pain (later)
client to eat smaller amounts of food at a 2. Pallor, lethargy, poor skin turgor and mobility,
slower pace. palpable epigastric mass
5. Provide client teaching and discharge 3. Diagnostic tests
planning concerning: a. Stool for occult blood positive
a. Gradually increasing food intake until able b. CEA positive
to tolerate three meals/day c. Hgb and HCT decreased
b. Daily monitoring of weight d. SGOT (AST), SGPT (ALT), LDH, serum
c. Stress-reduction measures amylase elevated (if liver and pancreatic
d. Need to report signs of complications to involvement)
physician immediately (hematemesis, e. Gastric analysis reveals histologic changes
vomiting, diarrhea, pain, melena, D. Nursing interventions
weakness, feeling of abdominal 1. Give consistent nutritional assessment and
fullness/distension) support.
e. Methods of controlling symptoms 2. Provide care for the client receiving
associated with dumping syndrome chemotherapy.
1) Avoidance of concentrated sweets 3. Provide care for the client with gastric surgery
2) Adherence to six, small, dry meals/day (see Gastric Surgery).
3) Maintenance of modified diet
4) Refraining from taking fluids during
meals but rather 2 hours after meals
Hernias
5) Assuming a recumbent position for A. General information
1
2 hour after meals 1. Protrusion of a viscus from its normal cavity
through an abnormal opening/weakened area
2. Occurs anywhere but most often in the
Cancer of the Stomach abdominal cavity
A. General information 3. Types
1. Most often develops in the distal third and a. Reducible: can be manually placed back

4
may spread through the walls of the stomach into the abdominal cavity.

ADULT NURSING 321


53155_04_Ch04b_p264-377.qxd 2/26/09 7:32 AM Page 322

b. Irreducible: cannot be placed back into the 2. Paralytic ileus (neurogenic or adynamic ileus):
abdominal cavity. interference with the nerve supply to the
c. Inguinal: occurs when there is weakness in intestine resulting in decreased or absent
the abdominal wall where the spermatic peristalsis; caused by abdominal surgery,
cord in men and round ligament in women peritonitis, pancreatic toxic conditions, shock,
emerge. spinal cord injuries, electrolyte imbalances
d. Femoral: protrusion through the femoral (especially hypokalemia)
ring; more common in females. 3. Vascular obstructions: interference with the
e. Incisional: occurs at the site of a previous blood supply to a portion of the intestine,
surgical incision as a result of inadequate resulting in ischemia and gangrene of the
healing postoperatively. bowel; caused by an embolus, atherosclerosis
f. Umbilical: most commonly found in B. Assessment findings
children. 1. Small intestine: nonfecal vomiting; colicky
g. Strangulated: irreducible, with obstruction intermittent abdominal pain
to intestinal flow and blood supply. 2. Large intestine: cramplike abdominal pain,
B. Medical management occasional fecal-type vomitus; client will be
1. Manual reduction, use of a truss (firm support) unable to pass stools or flatus
2. Bowel surgery if strangulated 3. Abdominal distension, rigidity, high-pitched
3. Herniorrhaphy: surgical repair of the hernia by bowel sounds above the level of the
suturing the defect obstruction, decreased or absent bowel sounds
C. Assessment findings distal to obstruction
1. Vomiting, protrusion of involved area (more 4. Diagnostic tests
obvious after coughing), and discomfort at site a. Flat-plate (X-ray) of the abdomen reveals
of protrusion the presence of gas/fluid
2. Crampy abdominal pain and abdominal b. HCT increased
distention (if strangulated with a bowel c. Serum sodium, potassium, chloride
obstruction) decreased
D. Nursing interventions d. BUN increased
1. Observe client for complications such as C. Nursing interventions
strangulation. 1. Monitor fluid and electrolyte balance, prevent
2. Prepare client for herniorrhaphy, provide further imbalance; keep client NPO and
routine pre-op care. administer IV fluids as ordered.
3. In addition to routine post-op care 2. Accurately measure drainage from
a. Assess for possible distended bladder, NG/intestinal tube.
particularly with inguinal hernia repair. 3. Place client in Fowlers position to alleviate
b. Discourage coughing, but deep breathing pressure on the diaphragm and encourage
and turning should be done. nasal breathing to minimize swallowing of air
c. Assist to splint incision when coughing or and further abdominal distension.
sneezing. 4. Institute comfort measures associated with NG
d. Apply ice bags to scrotal area (if inguinal intubation and intestinal decompression.
repair) to decrease edema. 5. Prevent complications.
e. Scrotal (athletic) or abdominal binder a. Measure abdominal girth daily to assess for
support may be ordered in some cases. increasing abdominal distension.
4. Provide client teaching and discharge b. Assess for signs and symptoms of
planning concerning: peritonitis.
a. Need to avoid strenuous physical activities c. Monitor urinary output.
(e.g., heavy lifting, pulling, pushing) for at
least 6 weeks.
b. Need to report any difficulty with urination.
Chronic Inflammatory Bowel Disorders
Regional Enteritis (Crohns Disease)
Intestinal Obstructions A. General information
A. General information 1. Chronic inflammatory bowel disease that can
1. Mechanical intestinal obstruction: physical affect both the large and small intestine;
blockage of the passage of intestinal contents terminal ileum, cecum, and ascending colon
with subsequent distension by fluid and gas; most often affected
caused by adhesions, hernias, volvulus, 2. Characterized by granulomas that may affect
intussusception, inflammatory bowel disease, all the bowel wall layers with resultant
foreign bodies, strictures, neoplasms, fecal thickening, narrowing, and scarring of the

4
impaction intestinal wall

322 NCLEX-RN Review


53155_04_Ch04b_p264-377.qxd 2/26/09 7:32 AM Page 323

3. Both sexes affected equally; more common that starts in the rectosigmoid area and spreads
in the Jewish population; two age peaks: upward. The mucosa of the bowel becomes
2030 years and 4060 years edematous, thickened with eventual scar
4. Cause unknown; contributing factors include formation. The colon consequently loses its
food allergies, autoimmune reaction, elasticity and absorptive capabilities.
psychologic disorders 2. Occurs more often in women and the Jewish
B. Medical management population, usually between ages 15 and 40.
1. Diet: high calorie, high vitamin, high protein, 3. Cause unknown; contributing factors include
low residue, milk free; supplementary iron autoimmune factors, viral infection, allergies,
preparations emotional stress, insecurity.
2. Drug therapy: antimicrobials (especially B. Medical management
sulfasalazine) to prevent or control infection, 1. Mild to moderate form
corticosteroids, antidiarrheals, anticholinergics a. Low-roughage diet with no milk products
3. Supplemental parenteral nutrition b. Drug therapy (antimicrobials,
4. Surgery: resection of diseased portion of bowel corticosteroids, anticholinergics,
and temporary or permanent ileostomy antidiarrheals, immunosuppressives,
C. Assessment findings hematinic agents)
1. Right, lower quadrant tenderness and pain; 2. Severe form: client kept NPO with IVs and
abdominal distension electrolyte replacements, NG tube with
2. Nausea and vomiting, 34 semisoft stools/day suction, blood transfusions, surgery
with mucus and pus C. Assessment findings
3. Decreased skin turgor, dry mucus membranes 1. Severe diarrhea (1520 liquid stools/day
4. Increased peristalsis containing blood, mucus, and pus); severe
5. Pallor tenesmus, weight loss, anorexia, weakness,
6. Diagnostic tests crampy discomfort
a. Hgb and HCT (if anemic) decreased 2. Decreased skin turgor, dry mucous membranes
b. Sigmoidoscopy negative or reveals 3. Low-grade fever, abdominal tenderness over
scattered ulcers the colon
c. Barium enema shows narrowing with areas 4. Diagnostic tests
of strictures separated by segments of a. Sigmoidoscopy reveals mucosa that bleeds
normal bowel easily with ulcer development
D. Nursing interventions b. Hgb and HCT decreased
1. Provide appropriate nutrition while reducing D. Nursing interventions: same as for Crohns disease
bowel motility.
a. Administer/monitor TPN. Diverticulosis/Diverticulitis
b. Provide high-protein, high-calorie, low-
residue diet with no milk products (if able A. General information
to tolerate oral foods/fluids). 1. A diverticulum is an outpouching of the
c. Weigh daily, monitor kcal counts, and take intestinal mucosa, most commonly found in
periodic anthropometric measurements. the sigmoid colon.
d. Record number and characteristics of 2. Diverticulosis: multiple diverticula of the colon
stools daily. 3. Diverticulitis: inflammation of the diverticula
e. Administer antidiarrheals, antispasmodics, 4. Men affected more often than women, more
and anticholinergics as ordered. common in obese individuals; usually occurs
f. Provide tepid fluids to avoid stimulation of between ages 40 and 45
the bowel. 5. Caused by stress, congenital weakening of
g. Omit gas-producing foods/fluids from diet. muscular fibers of intestine, and dietary
h. Administer/monitor enteral tube feedings deficiency of roughage and fiber
as ordered. B. Medical management
2. Promote comfort/rest: provide good perineal 1. High-residue diet with no seeds for
care with frequent washing and adequate diverticulosis; low-residue diet for
drying after each bowel movement; apply diverticulitis
analgesic or protective ointment as needed; 2. Drug therapy: bulk laxatives, stool softeners,
provide sitz baths as needed. anticholinergics, antibiotics
3. Provide care for the client with bowel surgery. 3. Surgery (rare): resection of diseased portion of
colon with temporary colostomy may be
Ulcerative Colitis indicated
C. Assessment findings
A. General information 1. Intermittent lower left quadrant pain and

4
1. Inflammatory disorder of the bowel tenderness over rectosigmoid area
characterized by inflammation and ulceration

ADULT NURSING 323


53155_04_Ch04b_p264-377.qxd 2/26/09 7:32 AM Page 324

2. Alternating constipation and diarrhea with B. Types: see Table 4-24.


blood and mucus C. Nursing interventions common to all bowel
3. Diagnostic tests surgery
a. Barium enema indicates an inflammatory 1. In addition to routine pre-op care:
process a. Ensure adherence to dietary restrictions.
b. Hgb and HCT decreased (if anemic) 1) Offer clear liquids only on day before
D. Nursing interventions surgery.
1. Administer medications as ordered. 2) Provide high-calorie, low-residue diet
2. Provide nursing care for the client with bowel 35 days before surgery.
surgery.
3. Provide client teaching and discharge
planning concerning: Table 4-24 Bowel Surgeries
a. Importance of adhering to dietary regimen.
Type Procedures
b. Prevention of increased intraabdominal
pressure. Abdominoperineal Distal sigmoid colon, rectum, and anus
c. Signs and symptoms of peritonitis and resection are removed through a perineal
need to notify physician immediately if incision and a permanent colostomy
they occur. is created. Surgery of choice for
cancer of the colon/rectum.
Cancer of the Colon/Rectum Ileostomy Opening of the ileum onto the
abdominal surface; most frequently
A. General information done for treatment of ulcerative
1. Adenocarcinoma is the most common type of colitis, but may also be done for
colon cancer and may spread by direct Crohns disease.
extension through the walls of the intestine or
Continent ileostomy An intra-abdominal reservoir with a
through the lymphatic or circulatory system.
(Kocks pouch) nipple valve is formed from the distal
Metastasis is most often to the liver.
ileum. The pouch acts as a reservoir
2. Second most common site for cancer in men
for fecal material and is cleaned at
and women; usually occurs between ages
regular intervals by insertion of a
50 and 60
catheter.
3. May be caused by diverticulosis, chronic
ulcerative colitis, familial polyposis Cecostomy An opening between the cecum and the
B. Medical management: chemotherapy, radiation abdominal base temporarily diverts
therapy, bowel surgery the fecal flow to rest the distal
C. Assessment findings portion of the colon after some
1. Alternating diarrhea/constipation, lower types of surgery.
abdominal cramps, abdominal distension Temporary Usually located in the ascending or
2. Weakness, anorexia, weight loss, pallor, colostomy transverse colon; most often done to
dyspnea rest the bowel.
3. Diagnostic tests Double-barreled The colon is resected and both ends
a. Stool for occult blood positive colostomy are brought through the abdominal
b. Hgb and HCT decreased wall creating two stomas, a proximal
c. CEA positive and a distal; done most often for an
d. Sigmoidoscopy reveals a mass obstruction or tumor in the
e. Barium enema shows a colon mass descending or transverse colon.
f. Digital rectal exam indicates a palpable mass Loop colostomy Often a temporary procedure whereby
D. Nursing interventions a loop of bowel is brought out above
1. Administer chemotherapy agents as ordered, the skin surface and held in place by
provide care for the client receiving a glass rod. There is one stoma but
chemotherapy. two openings, a proximal and a
2. Provide care for the client receiving radiation distal.
therapy. Permanent Consists of a single stoma made when
3. Provide care for the client with bowel surgery. colostomy the distal portion of the bowel is
removed; most often located in the
Bowel Surgery sigmoid or descending colon.
Resection with Diseased part of the bowel is removed
A. General information: type of surgery varies
anastomosis and remaining portions
depending on location and extent of lesion; may be
anastomosed, allowing elimination
indicated in Crohns disease, ulcerative colitis,
through the rectum.

4
intestinal obstructions, colon/rectal cancer.

324 NCLEX-RN Review


53155_04_Ch04b_p264-377.qxd 2/26/09 7:32 AM Page 325

b. Assist with bowel preparation. e. Gently dilate stoma and insert the
1) Administer antibiotics 35 days pre-op irrigation catheter or cone snugly.
to decrease bacteria in intestine. f. Open tubing and allow fluid to enter the
2) Administer enemas (possibly with bowel.
added antibiotics) to further cleanse g. Remove catheter or cone and allow fecal
the bowel. contents to drain.
c. Administer vitamins C and K (decreased h. Observe and record amount and character
by bowel cleansing) to prevent post-op of fecal return.
complications. 5. Promote adequate nutrition.
2. In addition to routine post-op care: a. Assess return of peristalsis.
a. Promote elimination. b. Advance diet as tolerated, add new foods
1) Assess for signs of returning gradually.
peristalsis. c. Avoid constipating foods.
2) Monitor characteristics of initial 6. Provide at least 2500 mL liquid/day.
stools. 7. Encourage client to discuss concerns and
b. Monitor and maintain fluid and electrolyte feelings about surgery.
balance. 8. Provide client teaching and discharge
D. Additional nursing interventions specific to planning concerning:
abdominoperineal resection a. Recognition of complications and need to
1. Reinforce and change perineal dressings as report immediately
needed. 1) Changes in odor, consistency, and
2. Record type, amount, color of drainage. color of stools
3. Irrigate with normal saline or hydrogen 2) Bleeding from the stoma
peroxide. 3) Persistent constipation or diarrhea
4. Provide warm sitz baths 4 times per day. 4) Changes in the contour of the stoma
5. Cover wound with dry dressing. 5) Persistent leakage around the stoma
E. Additional nursing interventions specific to 6) Skin irritation despite treatment
colostomy b. Proper procedure for colostomy
1. Prevent skin breakdown. irrigation.
a. Cleanse skin around stoma with mild soap
and water and pat dry.
b. Use a skin barrier to protect skin around
Peritonitis
the stoma. A. General information
c. Assess skin regularly for irritation. 1. Local or generalized inflammation of part or
d. Avoid the use of adhesives on irritated all of the parietal and visceral surfaces of the
skin. abdominal cavity
2. Control odor, maintain pleasant environment. 2. Initial response: edema, vascular congestion,
a. Change pouch/seal whenever necessary. hypermotility of the bowel and outpouring of
b. Empty or clean bag frequently, and provide plasmalike fluid from the extracellular,
ventilation afterwards; use deodorizer in vascular, and interstitial compartments into
bag/room if needed. the peritoneal space
c. Avoid gas-producing foods. 3. Later response: abdominal distension leading
3. Promote adequate stomal drainage. to respiratory compromise, hypovolemia
a. Assess stoma for color and intactness. results in decreased urinary output
b. Expect mucoid/serosanguinous drainage 4. Intestinal motility gradually decreases and
during the first 24 hours, then liquid type. progresses to paralytic ileus
c. Assess for flatus indicating return of 5. Caused by trauma (blunt or penetrating),
intestinal function. inflammation (ulcerative colitis, diverticulitis),
d. Monitor for changing consistency of fecal volvulus, intestinal ischemia, or intestinal
drainage. obstruction
4. Irrigate colostomy as needed. B. Medical management
a. Position client on toilet or in high-Fowlers 1. NPO with fluid replacement
if client on bed rest. 2. Drug therapy: antibiotics to combat infection,
b. Fill irrigation bag with desired amount of analgesics for pain
water (5001000 mL) and hang bag so the 3. Surgery
bottom is at shoulder height. a. Laparotomy: opening made through the
c. Remove air from tubing and lubricate the abdominal wall into the peritoneal cavity
tip of the catheter or cone. to determine the cause of peritonitis
d. Remove old pouch and clean skin and b. Depending on cause, bowel resection may

4
stoma with water. be necessary

ADULT NURSING 325


53155_04_Ch04b_p264-377.qxd 2/26/09 7:32 AM Page 326

C. Assessment findings D. Nursing interventions: preoperative


1. Severe abdominal pain, rebound tenderness, 1. Prepare client for hemorrhoidectomy.
muscle rigidity, absent bowel sounds, 2. In addition to routine pre-op care, provide
abdominal distension (particularly if large laxatives/enemas to promote cleansing of the
bowel obstruction) bowel.
2. Anorexia, nausea, and vomiting E. Nursing interventions: postoperative
3. Shallow respirations; decreased urinary 1. Provide routine post-op care.
output; weak, rapid pulse; elevated 2. Assess for rectal bleeding: inspect rectal
temperature area/dressings every 23 hours and report
4. Diagnostic tests significant increases in bloody drainage.
a. WBC elevated 3. Promote comfort.
b. HCT elevated (if hemoconcentration) a. Assist client to side-lying or prone position,
D. Nursing interventions provide flotation pad when sitting.
1. Assess respiratory status for possible b. Administer analgesics as ordered and
distress. monitor effects.
2. Assess characteristics of abdominal pain and 4. Promote elimination: administer stool
changes over time. softeners as ordered and, if possible,
3. Administer medications as ordered. administer analgesic before first post-op bowel
4. Perform frequent abdominal assessment. movement.
5. Monitor and maintain fluid and electrolyte 5. Provide client teaching and discharge
balance; monitor for signs of septic shock. planning concerning:
6. Maintain patency of NG or intestinal tubes. a. Dietary modifications (high fiber and
7. Place client in Fowlers position to localize ingestion of at least 2000 mL/day)
peritoneal contents. b. Need to defecate when urge is felt
8. Provide routine pre- and post-op care if c. Use of stool softeners as needed until
surgery ordered. healing occurs
d. Sitz baths after each bowel movement for
at least 2 weeks after surgery
Hemorrhoids e. Perineal care
A. General information f. Recognition and reporting immediately to
1. Congestion and dilation of the veins of the physician of the following signs and
rectum and anus; usually result from symptoms
impairment of flow of blood through the 1) Rectal bleeding
venous plexus 2) Continued pain on defecation
2. May be internal (above the anal sphincter) or 3) Puslike drainage from rectal area
external (outside anal sphincter)
3. Most commonly occur between ages 20 and 50
4. Predisposing conditions include occupations DISORDERS OF THE LIVER
requiring long periods of standing; increased
intra-abdominal pressure caused by prolonged
constipation, pregnancy, heavy lifting, obesity,
Hepatitis
straining at defecation; portal hypertension A. General information
B. Medical management 1. Widespread inflammation of the liver tissue
1. Stool softeners, local anesthetics, or anti- with liver cell damage due to hepatic cell
inflammatory creams degeneration and necrosis; proliferation and
2. Diet modification: high fiber, adequate liquids enlargement of the Kupffer cells; inflammation
3. Hemorrhoidectomy: surgical excision of of the periportal areas (may cause interruption
hemorrhoids indicated when there is prolapse, of bile flow)
severe pain, and excessive bleeding 2. Hepatitis A
C. Assessment findings a. Incubation period: 1545 days
1. Bleeding with defecation, hard stools with b. Transmitted by fecal/oral route: often
streaks of blood occurs in crowded living conditions; with
2. Pain with defecation, sitting, or walking poor personal hygiene; or from
3. Protrusion of external hemorrhoids upon contaminated food, milk, water, or shellfish
inspection 3. Hepatitis B
4. Diagnostic tests a. Incubation period: 50180 days
a. Proctoscopy reveals presence of internal b. Transmitted by blood and body fluids
hemorrhoids (saliva, semen, vaginal secretions): often
b. Hgb and HCT decreased if bleeding from contaminated needles among IV drug

4
excessive, prolonged abusers; intimate/sexual contact

326 NCLEX-RN Review


53155_04_Ch04b_p264-377.qxd 2/26/09 7:32 AM Page 327

4. Hepatitis C 2) If ELISA test is positive and ALT is


a. Incubation period: 750 days normal then recombinant immunoblot
b. Transmitted by parenteral route: through assay (RIBA) done
blood and blood products, needles, 3) Reverse transcription polymerase
syringes chain reaction (RT-PCR) test can pick
5. Hepatitis D up virus 23 weeks after exposure
a. Incubation period: 1456 days e. Hepatitis D: rise in hepatitis D virus
b. Transmitted by blood and body fluids; seen antibodies (anti-HDV) titer
in persons who have hepatitis B f. Hepatitis E: testing usually done only for
6. Hepatitis E symptomatic persons who have traveled to
a. Incubation period: 1564 days high-risk areas; hepatitis E antibodies (anti-
b. Transmitted by fecal/oral route; usually HEV) present
water-borne; seen in travelers returning C. Nursing interventions
from underdeveloped countries 1. Promote adequate nutrition.
B. Assessment findings a. Administer antiemetics as ordered,
1. Preicteric stage 30 minutes before meals to decrease
a. Anorexia, nausea and vomiting, fatigue, occurrence of nausea and vomiting.
constipation or diarrhea, weight loss b. Provide small, frequent meals of a high-
b. Right upper quadrant discomfort, carbohydrate, moderate- to high-protein,
hepatomegaly, splenomegaly, high-vitamin, high-calorie diet.
lymphadenopathy c. Avoid very hot or very cold foods.
2. Icteric stage 2. Ensure rest/relaxation: plan schedule for rest
a. Fatigue, weight loss, light-colored stools, and activity periods, organize nursing care to
dark urine minimize interruption.
b. Continued hepatomegaly with tenderness, 3. Monitor/relieve pruritus (see Cirrhosis of the
lymphadenopathy, splenomegaly Liver).
c. Jaundice, pruritus 4. Administer corticosteroids as ordered.
3. Posticteric stage 5. Institute isolation procedures as required; pay
a. Fatigue, but an increased sense of well- special attention to good hand-washing
being technique and adequate sanitation.
b. Hepatomegaly gradually decreasing 6. In hepatitis A administer immune serum
4. Diagnostic tests globulin (ISG) early to exposed individuals as
a. Hepatitis A, B, C ordered.
1) SGPT (ALT), SGOT (AST), alkaline 7. In hepatitis B
phosphatase, bilirubin, ESR: all a. Screen blood donors for HBsAg.
increased (preicteric) b. Use disposable needles and syringes.
2) Leukocytes, lymphocytes, neutrophils: c. Instruct client/others to avoid sexual
all decreased (pericteric) intercourse while disease is active.
3) Prolonged PT d. Administer ISG to exposed individuals as
b. Hepatitis A ordered.
1) Hepatitis A virus (HAV) in stool before e. Administer hepatitis B immunoglobulin
onset of disease (HBIG) as ordered to provide temporary
2) Anti-HAV (IgG) appears soon after and passive immunity to exposed
onset of jaundice; peaks in 12 months individuals.
and persists indefinitely f. To produce active immunity, administer
3) Anti-HAV (IgM): positive in acute hepatitis B vaccine to those individuals at
infection; lasts 46 weeks high risk.
c. Hepatitis B 8. In non-A, non-B: use disposable needles and
1) HBsAg (surface antigen): positive, syringes; ensure adequate sanitation.
develops 412 weeks after infection 9. In hepatitis C: Medications used are interferon
2) Anti-HBsAG: negative in 80% of cases combined with ribavirin. Side effects may
3) Anti-HBc: associated with infectivity, include flu-like symptoms, nausea; hair loss;
develops 216 weeks after infection emotional changes.
4) HBeAg: associated with infectivity and 10. Provide client teaching and discharge
disappears before jaundice planning concerning:
5) Anti-HBe: present in carriers, a. Importance of avoiding alcohol
represents low infectivity b. Avoidance of persons with known
d. Hepatitis C infections
1) Initial screening test enzyme-linked c. Balance of activity and rest periods

4
immunoabsorbent assay (ELISA) test d. Importance of not donating blood

ADULT NURSING 327


53155_04_Ch04b_p264-377.qxd 2/26/09 7:32 AM Page 328

e. Dietary modifications a. Provide bed rest with bathroom privileges.


f. Recognition and reporting of signs of b. Encourage gradual, progressive, increasing
inadequate convalescence: anorexia, activity with planned rest periods.
jaundice, increasing liver c. Institute measures to relieve pruritus.
tenderness/discomfort 1) Do not use soaps and detergents.
g. Techniques/importance of good personal 2) Bathe in tepid water followed by
hygiene application of an emollient lotion.
3) Provide cool, light, nonrestrictive
clothing.
Cirrhosis of the Liver 4) Keep nails short to avoid skin
A. General information excoriation from scratching.
1. Chronic, progressive disease characterized by 5) Apply cool, moist compresses to
inflammation, fibrosis, and degeneration of the pruritic areas.
liver parenchymal cells 2. Promote nutritional intake.
2. Destroyed liver cells are replaced by scar a. Encourage small frequent feedings.
tissue, resulting in architectural changes and b. Promote a high-calorie, low- to moderate-
malfunction of the liver protein, high-carbohydrate, low-fat diet,
3. Types with supplemental vitamin therapy
a. Lannecs cirrhosis: associated with (vitamins A, B-complex, C, D, K, and folic
alcohol abuse and malnutrition; acid).
characterized by an accumulation of fat in 3. Prevent infection.
the liver cells, progressing to widespread a. Prevent skin breakdown by frequent
scar formation. turning and skin care.
b. Postnecrotic cirrhosis: results in severe b. Provide reverse isolation for clients with
inflammation with massive necrosis as a severe leukopenia; pay special attention to
complication of viral hepatitis. handwashing technique.
c. Cardiac cirrhosis: occurs as a consequence c. Monitor WBC.
of right-sided heart failure; manifested by 4. Monitor/prevent bleeding.
hepatomegaly with some fibrosis. 5. Administer diuretics as ordered.
d. Biliary cirrhosis: associated with biliary 6. Provide client teaching and discharge
obstruction, usually in the common bile planning concerning:
duct; results in chronic impairment of bile a. Avoidance of agents that may be
excretion. hepatotoxic (sedatives, opiates, or OTC
4. Occurs twice as often in men as in women; drugs detoxified by the liver)
ages 4060 b. How to assess for weight gain and
B. Assessment findings increased abdominal girth
1. Fatigue, anorexia, nausea and vomiting, c. Avoidance of persons with upper
indigestion, weight loss, flatulence, irregular respiratory infections
bowel habits d. Recognition and reporting of signs of
2. Early symptoms: hepatomegaly; pain in right recurring illness (liver tenderness,
upper quadrant. Late symptoms: hard nodular increased jaundice, increased fatigue,
liver upon palpitation; atrophy of liver anorexia)
3. Changes in mood, alertness, and mental e. Avoidance of all alcohol
ability; sensory deficits; gynecomastia, f. Avoidance of straining at stool, vigorous
decreased axillary and pubic hair in males; blowing of nose and coughing, to decrease
amenorrhea in young females the incidence of bleeding
4. Jaundice of the skin, sclera, and mucous
membranes; pruritus
5. Easy bruising, spider angiomas, palmar
Ascites
erythema A. General information
6. Muscle atrophy 1. Accumulation of free fluid in the abdominal
7. Diagnostic tests cavity
a. SGOT (AST), SGPT (ALT), LDH alkaline 2. Most frequently caused by cirrhotic liver
phosphatase increased damage, which produces hypoalbuminemia,
b. Serum bilirubin increased increased portal venous pressure, and
c. PT prolonged hyperaldosteronism
d. Serum albumin decreased 3. May also be caused by HF
e. Hgb and HCT decreased B. Medical management
C. Nursing interventions 1. Supportive: modify diet, bed rest, salt-poor

4
1. Provide sufficient rest and comfort. albumin

328 NCLEX-RN Review


53155_04_Ch04b_p264-377.qxd 2/26/09 7:32 AM Page 329

2. Diuretic therapy (see Unit 2) 4. Promote comfort: place client in mid- to


3. Surgery high-Fowlers and reposition frequently.
a. Paracentesis: insertion of a needle into the 5. Provide nursing care for the client undergoing
peritoneal cavity through the abdomen to paracentesis.
remove abnormally large amounts of a. Confirm that client has signed a consent
peritoneal fluid. form.
1) Peritoneal fluid assessed for cell count, b. Instruct client to empty bladder before the
specific gravity, protein, and procedure to prevent inadvertent puncture
microorganisms. of the bladder during insertion of trocar.
2) Used in clients with acute respiratory or c. Inform client that a local anesthetic will be
abdominal distress secondary to ascites. provided to decrease pain.
b. LeVeen shunt (peritoneal-venous shunt): d. Place in sitting position to facilitate the
used in chronic, unmanageable ascites flow of fluid by gravity.
1) Permits continuous reinfusion of e. Measure abdominal girth and weight
ascitic fluid back into the venous before and after the procedure.
system through a silicone catheter with f. Record color, amount, and consistency of
a one-way pressure-sensitive valve. fluid withdrawn and client tolerance
2) One end of the catheter is implanted during procedure.
into the peritoneal cavity and is g. Assess insertion site for leakage.
channeled through the subcutaneous 6. Provide routine pre- and post-op care for the
tissue to the superior vena cava, where client with LeVeen shunt.
the other end of the catheter is
implanted; the valve opens when
pressure in the peritoneal cavity
Esophageal Varices
is 35 cm of water higher than in A. General information
superior vena cava, thereby allowing 1. Dilation of the veins of the esophagus, caused
ascitic fluid to flow into the venous by portal hypertension from resistance to
system. normal venous drainage of the liver into the
C. Assessment findings portal vein
1. Anorexia, nausea and vomiting, fatigue, 2. Causes blood to be shunted to the
weakness, changes in mental functioning esophagogastric veins, resulting in distension,
2. Positive fluid wave and shifting dullness on hypertrophy, and increased fragility.
percussion, flat or protruding umbilicus, 3. Caused by portal hypertension, which may be
abdominal distension/tautness with striae and secondary to cirrhosis of the liver (alcohol
prominent veins, abdominal pain abuse), swallowing poorly masticated food,
3. Peripheral edema, shortness of breath increased intra-abdominal pressure
4. Diagnostic tests B. Medical management
a. Potassium and serum albumin decreased 1. Iced normal saline lavage
b. PT prolonged 2. Transfusions with fresh whole blood
c. LDH, SGOT (AST), SGPT (ALT), BUN, 3. Vitamin K therapy
sodium increased 4. Sengstaken-Blakemore tube: a three-lumen
D. Nursing interventions tube used to control bleeding by applying
1. Monitor nutritional status/provide adequate pressure on the cardiac portion of the stomach
nutrition with modified diet. and against bleeding esophageal varices. One
a. Restrict sodium to 200500 mg/day. lumen serves as NG suction, a second lumen is
b. Restrict fluids to 10001500 mL/day. used to inflate the gastric balloon, the third to
c. Promote high-calorie foods/snacks. inflate the esophageal balloon.
2. Monitor/prevent increasing edema. 5. Intra-arterial or IV vasopressin
a. Administer diuretics as ordered and 6. Injection sclerotherapy
monitor for effects. 7. Surgery for portal hypertension (decompresses
b. Measure I&O. esophageal varices and helps to maintain
c. Monitor peripheral pulses. optimal portal perfusion)
d. Measure abdominal girth. a. Ligation of esophageal and gastric veins to
e. Inspect/palpate extremities, sacrum. stop acute bleeding
f. Administer salt-poor albumin to replace b. Portacaval shunt: end-to-side or side-to-
vascular volume. side anastomosis of the portal vein to the
3. Monitor/promote skin integrity. inferior vena cava
a. Reposition frequently. c. Splenorenal shunt: end-to-side or side-to-
b. Apply lotions to stretched areas. side anastomosis of the splenic vein to the

4
c. Assess for redness, breakdown. left renal vein

ADULT NURSING 329


53155_04_Ch04b_p264-377.qxd 2/26/09 7:32 AM Page 330

d. Mesocaval shunt: end-to-side or use of a 2. Diseased liver is unable to convert ammonia to


graft to anastomose the inferior vena cava urea, so that large quantities remain in the
to the side of the superior mesenteric vein systemic circulation and cross the blood/brain
C. Assessment findings barrier, producing neurologic toxic symptoms.
1. Anorexia, nausea and vomiting, hematemesis, 3. Caused by cirrhosis, GI hemorrhage,
fatigue, weakness hyperbilirubinemia, transfusions (particularly
2. Splenomegaly, increased splenic dullness, with stored blood), thiazide diuretics, uremia,
ascites, caput medusae, peripheral edema, bruits dehydration
3. Diagnostic tests B. Assessment findings
a. PT prolonged 1. Early in course of disease: changes in mental
b. Hematest of vomitus positive functioning (irritability); insomnia, slowed
c. Serum albumin, RBC, Hgb, and HCT affect; slow slurred speech; impaired
decreased judgment; slight tremor; Babinskis reflex,
d. LDH, SGOT (AST), SGPT (ALT), BUN, hyperactive reflexes
increased 2. Progressive disease: asterixis, disorientation,
D. Nursing interventions apraxia, tremors, fetor hepaticus, facial
1. Monitor/provide care for client with grimacing
Sengstaken-Blakemore tube. 3. Late in disease: coma, absent reflexes
a. Facilitate placement of the tube: check and 4. Diagnostic tests
lubricate tip and elevate head of bed. a. Serum ammonia levels increased
b. Prevent dislodgment of the tube by placing (particularly later)
client in semi-Fowlers position; maintain b. PT prolonged
traction by securing the tube to a piece of c. Hgb and HCT decreased
sponge or foam rubber placed on the nose. C. Nursing interventions
c. Keep scissors at bedside at all times. 1. Conduct ongoing neurologic assessment and
d. Monitor respiratory status; assess for signs report deteriorations.
of distress and if respiratory distress occurs 2. Restrict protein in diet; provide high
cut the tubing to deflate the balloons and carbohydrate intake and vitamin K
remove tubing immediately. supplements.
e. Label each lumen to avoid confusion; 3. Administer enemas, cathartics, intestinal
maintain prescribed amount of pressure on antibiotics, and lactulose as ordered to reduce
esophageal balloon and deflate balloon as ammonia levels.
ordered to avoid necrosis. 4. Protect client from injury: keep side rails up;
f. Observe nares for skin breakdown and provide eye care with use of artificial tears/eye
provide mouth and nasal care every patch.
12 hours (encourage client to expectorate 5. Avoid administration of drugs detoxified in
secretions, suction gently if unable). liver (phenothiazines, gold compounds,
2. Promote comfort: place client in semi-Fowlers methyldopa, acetaminophen).
position (if not in shock); provide mouth care. 6. Maintain client on bed rest to decrease
3. Monitor for further bleeding and for signs and metabolic demands on liver.
symptoms of shock; hematest all secretions.
4. Administer vasopressin as ordered and
monitor effects.
Cancer of the Liver
5. Provide routine pre- and post-op care if the A. General information
client has portasystemic or portacaval shunt. 1. Primary cancer of the liver is extremely rare,
6. Provide client teaching and discharge but it is a common site for metastasis because
planning concerning: of livers large blood supply and portal
a. Minimizing esophageal irritation drainage. Primary cancers of the colon,
(avoidance of salicylates, alcohol; use of rectum, stomach, pancreas, esophagus, breast,
antacids as needed; importance of chewing lung, and melanomas frequently metastasize to
food thoroughly) the liver.
b. Avoidance of increased abdominal, 2. Enlargement, hemorrhage, and necrosis are
thoracic, and portal pressure common occurrences; primary liver tumors
c. Recognition and reporting of signs of often metastasize to the lung.
hemorrhage 3. Higher incidence in men.
4. Prognosis poor; disease well advanced before
clinical signs evident.
Hepatic Encephalopathy B. Medical management
A. General information 1. Chemotherapy and radiotherapy (palliative) to

4
1. Frequent terminal complication in liver disease decrease tumor size and pain

330 NCLEX-RN Review


53155_04_Ch04b_p264-377.qxd 2/26/09 7:32 AM Page 331

2. Resection of liver segment or lobe if tumor is 5. Acute cholecystitis usually follows stone
localized impaction, adhesions; neoplasms may also be
C. Assessment findings implicated.
1. Weakness, anorexia, nausea and vomiting, B. Medical management
weight loss, slight increase in temperature 1. Supportive treatment: NPO with NG
2. Right upper quadrant discomfort/tenderness, intubation and IV fluids
hepatomegaly, blood-tinged ascites, friction 2. Diet modification with administration of fat-
rub over liver, peripheral edema, jaundice soluble vitamins
3. Diagnostic tests: same as cirrhosis of the liver 3. Drug therapy
plus: a. Narcotic analgesics (Demerol is drug of
a. Blood sugar decreased choice) for pain. Morphine sulfate is
b. Alpha fetoprotein increased contraindicated because it causes spasms
c. Abdominal X-ray, liver scan, liver biopsy of the sphincter of Oddi.
all positive b. Anticholinergics (atropine) for pain.
D. Nursing interventions: same as for cirrhosis of the (Anticholinergics relax smooth muscle and
liver plus: open bile ducts.)
1. Provide emotional support for c. Antiemetics
client/significant others regarding poor 4. Surgery: cholecystectomy/choledochostomy
prognosis. C. Assessment findings
2. Provide care of the client receiving radiation 1. Epigastric or right upper quadrant pain,
therapy or chemotherapy. precipitated by a heavy meal or occurring at
3. Provide care of client with abdominal surgery night
plus: 2. Intolerance for fatty foods (nausea, vomiting,
a. Preoperative sensation of fullness)
1) Perform bowel prep to decrease 3. Pruritus, easy bruising, jaundice, dark amber
ammonium intoxication. urine, steatorrhea
2) Administer vitamin K to decrease risk 4. Diagnostic tests
of bleeding. a. Direct bilirubin transaminase, alkaline
b. Postoperative phosphatase, WBC, amylase, lipase: all
1) Administer 10% glucose for first 48 increased
hours to avoid rapid blood sugar drop. b. Oral cholecystogram (gallbladder series):
2) Monitor for hyper/hypoglycemia. positive for gallstone
3) Assess for bleeding (hemorrhage is D. Nursing interventions
most threatening complication). 1. Administer pain medications as ordered and
4) Assess for signs of hepatic monitor for effects.
encephalopathy. 2. Administer IV fluids as ordered.
3. Provide small, frequent meals of modified diet
(if oral intake allowed).
DISORDERS OF THE GALLBLADDER 4. Provide care to relieve pruritus.
5. Provide care for the client with a
cholecystectomy or choledochostomy.
Cholecystitis/Cholelithiasis
A. General information
1. Cholecystitis: acute or chronic inflammation of
Cholecystectomy/Choledochostomy
the gallbladder, most commonly associated A. General information
with gallstones. Inflammation occurs within 1. Cholecystectomy: removal of the gallbladder
the walls of the gallbladder and creates a with insertion of a T-tube into the common
thickening accompanied by edema. bile duct if common bile duct exploration is
Consequently, there is impaired circulation, performed
ischemia, and eventual necrosis. 2. Choledochostomy: opening of common
2. Cholelithiasis: formation of gallstones, duct, removal of stone, and insertion of a
cholesterol stones most common variety T-tube
3. Most often occurs in women after age 40, in 3. Cholecystectomy performed via laparoscopy
postmenopausal women on estrogen therapy, for uncomplicated cases when client has not
in women taking oral contraceptives, and in had previous abdominal surgery
the obese; Caucasians and Native Americans B. Nursing interventions: routine preoperative care
are also more commonly affected. C. Nursing interventions: postoperative
4. Stone formation may be caused by genetic 1. Provide routine post-op care.
defect of bile composition, gallbladder/bile 2. Position client in semi-Fowlers or side-lying

4
stasis, infection. positions; reposition frequently.

ADULT NURSING 331


53155_04_Ch04b_p264-377.qxd 2/26/09 7:32 AM Page 332

3. Splint incision when turning, coughing, and which will cause exacerbation of
deep breathing. symptoms.
4. Maintain/monitor functioning of T-tube. b. Smooth-muscle relaxants (papaverine,
a. Ensure that T-tube is connected to closed nitroglycerin) to relieve pain
gravity drainage. c. Anticholinergics (atropine, propantheline
b. Avoid kinks, clamping, or pulling of the bromide [Pro-Banthine]) to decrease
tube. pancreatic stimulation
c. Measure and record drainage every shift. d. Antacids to decrease pancreatic
d. Expect 300500 mL bile-colored drainage stimulation
first 24 hours, then 200 mL/24 hours for e. H2 antagonists, vasodilators, calcium
34 days. gluconate
e. Monitor color of urine and stools (stools 2. Diet modification
will be light colored if bile is flowing 3. NPO (usually)
through T-tube but normal color should 4. Peritoneal lavage
reappear as drainage diminishes). 5. Dialysis
f. Assess for signs of peritonitis. C. Assessment findings
g. Assess skin around T-tube; cleanse 1. Pain located in left upper quadrant with
frequently and keep dry. radiation to back, flank, or substernal area;
5. Provide client teaching and discharge may be accompanied by difficulty breathing
planning concerning and is aggravated by eating
a. Adherence to dietary restrictions 2. Vomiting, shallow respirations (with pain),
b. Resumption of ADL (avoid heavy lifting for tachycardia, decreased or absent bowel
at least 6 weeks; resume sexual activity as sounds, abdominal tenderness with muscle
desired unless ordered otherwise by guarding, positive Grey Turners spots
physician); clients having laparoscopy (ecchymoses on flanks) and positive Cullens
cholecystectomy usually resume normal sign (ecchymoses of periumbilical area)
activity within 2 weeks. 3. Diagnostic tests
c. Recognition and reporting of signs of a. Serum amylase and lipase, urinary amylase,
complications (fever, jaundice, pain, dark blood sugar, lipid levels: all increased
urine, pale stools, pruritus) b. Serum calcium decreased
c. CT scan shows enlargement of the pancreas
D. Nursing interventions
Appendicitis 1. Administer analgesics, antacids,
See Unit 5. anticholinergics as ordered, monitor effects.
2. Withhold food/fluid and eliminate odor and
sight of food from environment to decrease
DISORDERS OF THE PANCREAS pancreatic stimulations.
3. Maintain NG tube and assess for drainage.
4. Institute nonpharmacologic measures to
Pancreatitis decrease pain.
A. General information a. Assist client to positions of comfort (knee-
1. An inflammatory process with varying degrees chest; fetal position).
of pancreatic edema, fat necrosis, or b. Teach relaxation techniques and provide a
hemorrhage quiet, restful environment.
2. Proteolytic and lipolytic pancreatic enzymes 5. Provide client teaching and discharge
are activated in the pancreas rather than in the planning concerning
duodenum, resulting in tissue damage and a. Dietary regimen when oral intake permitted
autodigestion of the pancreas 1) High-carbohydrate, high-protein,
3. Occurs most often in the middle aged low-fat diet
4. Caused by alcoholism, biliary tract disease, 2) Eating small, frequent meals instead of
trauma, viral infection, penetrating duodenal three large ones
ulcer, abscesses, drugs (steroids, thiazide 3) Avoiding caffeine products
diuretics, and oral contraceptives), metabolic 4) Eliminating alcohol consumption
disorders (hyperparathyroidism, 5) Maintaining relaxed atmosphere after
hyperlipidemia) meals
B. Medical management b. Recognition and reporting of signs of
1. Drug therapy complications
a. Analgesics to relieve pain. Note: Morphine 1) Continued nausea and vomiting
is contraindicated due to the spasmodic 2) Abdominal distension with increasing

4
effects of opiates on the sphincter of Oddi, fullness

332 NCLEX-RN Review


53155_04_Ch04b_p264-377.qxd 2/26/09 7:32 AM Page 333

3) Persistent weight loss


4) Severe epigastric or back pain Sample Questions
5) Frothy/foul-smelling bowel
movements
6) Irritability, confusion, persistent 290. An adult client has a nasogastric tube in place to
elevation of temperature (2 days) maintain gastric decompression. Which nursing
action will relieve discomfort in the nostril with
Cancer of the Pancreas the NG tube?
1. Remove any tape and loosely pin the tube to
A. General information
his gown.
1. Most pancreatic tumors are adenocarcinomas
and half occur in the head of the 2. Lubricate the NG tube with viscous
pancreas xylocaine.
2. Tumor growth results in common bile duct 3. Loop the NG tube to avoid pressure on the
obstruction with jaundice nares.
3. Occurs more often in men and in the African- 4. Replace the NG tube with a smaller diameter
American and Jewish populations; ages tube.
4565
4. Contributing factors: chemical carcinogens, 291. An adult client has just returned to his room
cigarette smoking, high-fat diet, diabetes following a bowel resection and end-to-end
mellitus anastomosis. What type of drainage will the
5. Prognosis generally poor nurse expect in the early post-op period?
B. Medical management
1. Clear.
1. Radiation therapy
2. Whipples procedure 2. Mucoid.
(pancreatoduodenectomy): resection of the 3. Scant.
proximal pancreas, adjoining duodenum, 4. Discolored.
distal portion of the stomach, and distal
segment of the common bile duct 292. A client with a long history of ulcerative colitis
3. Drug therapy is experiencing an exacerbation of the disease
a. Pancreatic enzymes; oral hypoglycemic and is admitted with severe diarrhea, electrolyte
agents or insulin, bile salts necessary after disturbances, and severe abdominal pain. He
surgery questions the nurse about his prognosis. What is
b. Chemotherapy may also be used the nurses best response?
C. Assessment findings
1. You should ask your physician about this.
1. Anorexia; rapid, progressive weight loss;
dull abdominal pain located in upper 2. Dont worry, colitis is rarely fatal.
abdomen or left hypochondriacal region 3. It depends on the form of the disease.
with radiation to the back, related to eating; 4. Tell me what you know about this disease.
jaundice
2. Diagnostic tests 293. The nurse questions a client with hepatitis B.
a. Increased serum lipase (early) How long ago could the client have been
b. Increased bilirubin (conjugated) exposed to hepatitis B?
c. Increased serum amylase a. 37 days.
D. Nursing interventions
1. See Pancreatitis. b. 714 days.
2. Provide care for the client receiving radiation c. 4050 days.
therapy or chemotherapy. d. 60160 days.
3. Routine pre- and post-op care (for clients
undergoing Whipples procedure). 294. A client has a history of peptic ulcer disease. He
4. Provide emotional support to client/significant has had numerous bleeding episodes in the past
others. and is admitted to the hospital for evaluation.
5. Provide client teaching and discharge His physician has prescribed cimetidine
planning concerning (Tagamet). What is the primary reason for the
a. Need to eat small frequent meals of a client to take Tagamet?
low-fat, high-calorie diet with vitamin 1. Blocks the secretion of gastric hydrochloric
supplements. acid.
b. Importance of adhering to medication
regimen after surgery. 2. Coats the gastric mucosa with a protective

4
membrane.

ADULT NURSING 333


53155_04_Ch04b_p264-377.qxd 2/26/09 7:32 AM Page 334

3. Increases the sensitivity of H2 receptors. 1. Keep the client positioned on his left side for
4. Releases basal gastric acid. 810 hours.
2. Assess for a gag reflex before offering the
295. An adult has a Billroth II procedure and does client anything to eat or drink.
well postoperatively. The nurse knows the client
3. Provide throat lozenges for complaints of a
understands discharge teaching when the client
sore throat.
recognizes that symptoms of dizziness, sweating,
and weakness in the weeks following the surgery 4. Position the client in high Fowlers until he is
are usually associated with what condition? fully awake and alert.
1. Afferent loop syndrome. 301. A client is being evaluated for cancer of the
2. Dumping syndrome. colon. In preparing the client for a barium
3. Pernicious anemia. enema, which intervention will be included that
4. Marginal ulcers. pertains to the procedure?
1. Placement on a low-residue diet 1 to 2 days
296. A client has had a significant problem with before the study.
alcohol abuse for the past 15 years. His wife
2. Given an oil retention enema the morning of
brings him to the emergency department because
the study.
he is increasingly confused and is coughing
blood. His medical diagnosis is cirrhosis of the 3. Instruction to swallow six radiopaque tablets
liver. He has ascites and esophageal varices. the evening before the study.
Which symptom is the client least likely to have? 4. Positioning in a high Fowlers position
1. Bulging flanks. immediately following the procedure.
2. Protruding umbilicus. 302. A client complains of excessive weight loss and
3. Abdominal distension. anorexia. Laboratory studies show that he is
4. Bluish discoloration of the umbilicus. anemic. Hepatocellular carcinoma is suspected.
A liver biopsy is performed at the bedside. What
297. What is the major dietary treatment for ascites? intervention will be expected after the
1. High protein. procedure?
2. Increased potassium. 1. Encourage to ambulate to prevent the
3. Restricted fluids. formation of venous thrombosis.
4. Restricted sodium. 2. Ask to turn, cough, and deep breathe every
2 hours for the next 8 hours.
298. Which laboratory value would the nurse expect
3. Place in a high Fowlers position to maximize
to find in a client as a result of liver failure?
thoracic expansion.
1. Decreased serum creatinine.
4. Position on his right side with a pillow under
2. Decreased sodium. the costal margin, and immobile for several
3. Increased ammonia. hours.
4. Increased calcium.
303. A client has a fecal impaction. The physician
299. A man is admitted with bleeding esophageal orders an oil-retention enema followed by a
varices. A Sengstaken-Blakemore tube is cleansing enema. What is the reason for
inserted in an effort to stop the bleeding. After administering an oil-retention enema to the
the Sengstaken-Blakemore tube is inserted, the client?
client has difficulty breathing. Based on this 1. Lubricate the walls of the intestinal tract.
information, what is the first action the nurse 2. Soften the fecal mass and lubricate the walls
should take? of the rectum and colon.
1. Deflate the esophageal balloon. 3. Reduce bacterial content of the fecal mass.
2. Encourage him to take deep breaths. 4. Coat the walls of the intestines to prevent
3. Monitor his vital signs. irritation by the hardened fecal mass.
4. Notify the physician.
304. A client has amyotrophic lateral sclerosis. His
300. A client is scheduled for a esophagoduodenoscopy. neurologic status has continued to deteriorate.
In planning for the post-procedural care, what is He is receiving enteral feedings through a
the most effective nursing action to prevent gastrostomy tube. What priority assessment
respiratory complications?

4 334 NCLEX-RN Review


53155_04_Ch04b_p264-377.qxd 2/26/09 7:32 AM Page 335

should be performed before administering a 1. Reducing fluid intake to encourage bulk


bolus feeding? formation in the intestinal lumen.
1. Check the expiration date of the prepared 2. Use of laxatives daily to establish a regular
enteral feeding. elimination pattern.
2. Confirm the presence of a gag reflex. 3. A regimen of exercises directed at toning the
3. Check placement of feeding tube. abdominal muscles.
4. Review laboratory studies for indications of 4. Setting a routine for bowel elimination just
electrolyte imbalances. before bedtime.

305. An adult is 8 hours post-op a Billroth II (gastric 309. A client is scheduled for a resection of the lower
resection) for an intractable gastric ulcer. The thoracic esophagus to remove a malignant
drainage from his nasogastric decompression tumor. What intervention would be included in
tube is thickened and the volume of secretions the postoperative care?
has dramatically reduced in the last 2 hours. The 1. Keep the client in a supine position to
client complains that he feels like he is going to encourage thoracic expansion.
vomit. What is the most appropriate nursing 2. Carefully advance the nasogastric tube past
action? the anastomosis site.
1. Reposition the nasogastric tube by advancing 3. Frequently assess the clients breath sounds.
it gently. 4. Provide a regular diet high in protein.
2. Notify the physician of your findings.
3. Irrigate the nasogastric tube with 50 mL of 310. A client has been experiencing frequent
sterile normal saline. episodes of heartburn and regurgitation of
4. Discontinue the low-intermittent suctioning. acrid, sour-tasting fluid. These episodes tend to
occur especially after a heavy meal. The client is
306. A client is receiving chemotherapy for cancer of diagnosed with a hiatal hernia. Which statement
the liver. Her physician has prescribed by the client shows a good understanding of her
metoclopramide for the nausea and vomiting treatment regimen?
associated with the chemotherapy. 1. I will elevate my legs when sleeping.
Metoclopramide has anticholinergic and 2. I will increase the roughage in my diet.
extrapyramidal side effects. Due to the side 3. I will drink more fluid with my meals.
effects of this mediation, which nursing
4. I will avoid caffeine, alcohol, and chocolate.
diagnosis is the client at high risk for?
1. Hyperglycemia related to increased gastric 311. A client stockbroker has recently been diagnosed
emptying. with peptic ulcer disease. Diagnostic studies
2. Injury related to decreased visual acuity and confirm the presence of the gram-negative
ataxia. bacteria Helicobacter pylori in his
3. Decreased cardiac output related to reduced gastrointestinal tract. If the client has a duodenal
heart rate. ulceration, how would the nurse expect the
4. Fluid volume deficit related to frequent ulcer pain to be described by the client?
episodes of diarrhea. 1. Located in the upper right epigastric area
radiating to his right shoulder or back.
307. An adult develops diarrhea secondary to 2. Relieved by vomiting.
hyperosmolar enteral therapy. The care plan 3. Occurring 2 to 3 hours after a meal, often
now includes giving the client water every 4 to 6 awakening him between 1:00 and 2:00 A.M.
hours and after feedings. Which of the following
4. Worsening with the ingestion of food.
findings would indicate that fluid therapy was
effective? 312. A client has been diagnosed with peptic ulcer
1. Dry mucous membranes. disease. Her medication regimen includes
2. Hyperactive bowel sounds. misoprostol. What therapeutic effect will be
3. Increased urinary output. performed by misoprostol?
4. Hypokalemia. 1. Neutralizing excess gastric acid.
2. Inhibiting gastric acid production.
308. An elderly client complains of frequent episodes 3. Increasing mucous production and
of constipation. What is an effective strategy for bicarbonate levels.
preventing constipation?

4
4. Increasing gastric emptying time.

ADULT NURSING 335


53155_04_Ch04b_p264-377.qxd 2/26/09 7:32 AM Page 336

313. A client has Billroth II (gastrojejunostomy) for 318. The nurse is caring for a client recently diagnosed
intractable peptic ulcer disease. The nurse is with ulcerative colitis. The nurse has been giving
instructing the client concerning the potential dietary instructions to help prevent exacerbation
complication of dumping syndrome. What of his inflammatory bowel disease. Which dietary
would be included in the clients dietary and choice indicates that the client understands the
activity instructions? dietary instructions?
1. A high-carbohydrate diet. 1. Apple.
2. Exercise after mealtime to promote the 2. Celery.
digestive process. 3. Refined cereals.
3. Limit drinking fluids with meals. 4. Hard cheeses.
4. A protein-restricted diet.
319. When a client is diagnosed with ulcerative
314. A client underwent a total gastrectomy for colitis, What complication would the nurse be
gastric cancer. The nurse has been giving the on alert for?
client post-op instructions about his diet, 1. Intestinal obstruction.
activities, and medications. Which of the 2. Toxic megacolon.
following statements indicates that he 3. Malnutrition from malabsorption.
understands his post-op care?
4. Fistula formation.
1. I should take a walk after meals to aid my
digestion. 320. A client with diverticulosis is admitted to the
2. Drinking more water with my meals will hospital. What type of diet would be ordered for
prevent indigestion. this client?
3. I need more carbohydrates in my diet for an 1. A bland, low residue diet.
extra energy source. 2. A low protein, high carbohydrate diet.
4. The visiting nurse will come monthly to 3. A soft, but high fiber diet.
give an injection of vitamin B12. 4. Saline cathartics to increase intestinal
peristalsis.
315. A client has a direct inguinal hernia. For
what symptoms should the nurse be on alert 321. An adult has been diagnosed with colon cancer.
for? What would the nursing assessment most likely
1. Hypoactive bowel sounds. reveal?
2. Passage of semi-liquid, brown stools. 1. Epigastric pain that intensifies when the
3. Vomiting of bile-stained gastric contents. stomach is empty.
4. Complaints of constant, localized abdominal 2. Stools that are fatty and foul-smelling.
pain. 3. Alternating episodes of diarrhea and
constipation.
316. An adult has developed peritonitis related to a 4. A rigid, board-like abdomen.
perforated duodenal ulceration. What would the
nurse expect to find during the assessment? 322. The nurse has been instructing a client regarding
1. Decreased or absent bowel sounds. identifying and alleviating the risk factors
associated with colon cancer. Which statement by
2. Colicky abdominal pain.
the client demonstrates a good understanding of
3. High-pitched bowel sounds. the means to reduce the chances of colon cancer?
4. Alternating episodes of constipation and 1. I will exercise daily.
diarrhea.
2. I will include more red meat in my diet.
317. What would be an appropriate nursing diagnosis 3. I will have an annual chest X-ray.
for a client with ulcerative colitis? 4. I will include more fresh fruits and
1. Abdominal pain, related to decreased vegetables in my diet.
peristalsis.
323. An adult has a sigmoid colostomy. The nurse is
2. Diarrhea related to hyperosmolar intestinal performing peristomal skin care and changing
contents. the stoma pouch. What is the most appropriate
3. Excess fluid volume related to increased nursing action?
water absorption by intestinal mucosa. 1. Empty the ostomy pouch when it is full.
4. Activity intolerance related to fatigue. 2. Pull flange and pouch off together to prevent

4
spillage of stomach pouch contents.

336 NCLEX-RN Review


53155_04_Ch04b_p264-377.qxd 2/26/09 7:32 AM Page 337

3. Leave 14 inch of skin exposed around stoma 328. A client visits her physician with flu-like
when determining size to cut new skin barrier. symptoms that have persisted for nearly a
4. Apply liquid deodorant to mucous membrane month. She complains of headaches, malaise,
of protruding stoma. anorexia, and fever. She is a childcare worker at
a local daycare center with children ranging in
324. An adult has a double-barreled, transverse ages from 6 months to 5 years. Based on the
colostomy. The nurse has formulated the nursing associated risk factors and mode of transmission,
diagnosis: risk for impaired skin integrity related which condition is the client most likely
to irritation of the peristomal skin by the experiencing?
effluent. What is the most appropriate nursing 1. Hepatitis A.
action relevant to this nursing diagnosis?
2. Hepatitis B.
1. Strict measurement and recording of I&O.
3. Hepatitis C.
2. Assessing for bowel sounds when changing
4. Hepatitis D.
ostomy appliance.
3. Wash peristomal skin with an astringent 329. A client that works in a community health clinic
solution to reduce bacterial contamination. has been experiencing fatigue, headaches,
4. Apply skin barrier before applying flange and diminished appetite, and a yellowish
ostomy pouch. discoloration of sclera for the past 2 months. He
is diagnosed with hepatitis B and asks the nurse
325. An adult is brought to the emergency room with how he contracted hepatitis. What is the nurses
severe, constant, localized abdominal pain. most appropriate response?
Abdominal muscles are rigid and rebound 1. Airborne droplets carry the infectious
tenderness is present. Peritonitis is suspected. hepatitis B virus.
The client is hypotensive and tachycardiac.
2. The hepatitis B virus is transmitted
What is the nursing diagnosis most appropriate
parenterally and through intimate contact.
to the signs/symptoms presented?
3. An individual may contract hepatitis B by
1. Deficient fluid volume related to depletion of
using contaminated eating utensils.
intravascular volume.
4. Hepatitis B may be transmitted through eating
2. Disturbed thought process related to toxic
shellfish from contaminated water sources.
effects of elevated ammonia levels.
3. Abdominal pain related to increased 330. A client with hepatic cirrhosis related to 10-year
intestinal peristalsis. history of alcohol abuse is at risk for injury
4. Imbalanced nutrition, less than body related to portal hypertension. What is the most
requirements, related to malabsorption. appropriate nursing action to decrease his risk of
injury?
326. A client has had a hemorrhoidectomy. Which 1. Keep his fingernails short.
activity by the client will demonstrate the client
2. Offer small, frequent feedings.
has good understanding of post-op discharge
instruction? 3. Observe stools for color and consistency.
1. Reduce her fluid intake for several weeks 4. Assess for jaundice of skin and sclera.
after her surgery. 331. A client is diagnosed with Lannecs cirrhosis.
2. Include more fresh fruits and vegetables in He has massive ascites formation. His
her diet. respirations are rapid and shallow. The
3. Vigorously clean her perianal area with soap physician decides to perform a paracentesis.
and water after every bowel movement. Which activity does the nurse give the highest
4. Limit her activities to bed rest for at least 6 priority to during the procedure?
hours a day. 1. Gathering the appropriate sterile equipment.
2. Labeling samples of abdominal fluid and
327. The client with hepatitis may be anicteric and
sending them to the laboratory.
symptomless. What sign/symptom is most likely
present in the early presentation of hepatic 3. Positioning the client upright on the edge of
inflammatory disorder? the bed.
1. Dark urine. 4. Measuring and recording blood pressure and
pulse frequently during the procedure.
2. Ascites.
3. Occult blood in stools. 332. An adult who has a 7-year history of hepatic

4
4. Anorexia. cirrhosis was brought to the emergency room

ADULT NURSING 337


53155_04_Ch04b_p264-377.qxd 2/26/09 7:32 AM Page 338

because he began vomiting large amounts of 2. Depression of GI peristalsis may cause


dark-red blood. A Sengstaken-Blakemore tube constipation.
was inserted to tamponade the bleeding 3. Spasms of the sphincter of Oddi may occur.
esophageal varices. While the balloon tamponade 4. It is rapidly metabolized by the liver.
is in place, what is given the highest priority?
1. Assessing his stools for occult blood. 337. A client has a 5-year history of alcohol abuse. He
2. Evaluating capillary refill in extremities. appears acutely ill. He is vomiting bile-stained
3. Performing frequent mouth care. emesis. What would be documented as signs of
severe, hemorrhagic pancreatitis?
4. Auscultating breath sounds.
1. The presence of a positive fluid wave in the
333. A client is experiencing advanced hepatic abdominal area.
cirrhosis complicated by hepatic 2. A yellowish color of the sclera and skin.
encephalopathy. He is confused, restless, and 3. Ecchymosis in the flank and around the
demonstrating asterixis. In developing a two- umbilical area.
part nursing diagnosis for this condition, what 4. Bloody, foul-smelling stools.
would be the second part following Disturbed
Thought Processes? 338. A client with a 10-year history of alcohol abuse
1. Massive ascites formation. has recently been diagnosed with chronic
2. Increased serum ammonia levels. pancreatitis. The physician has prescribed
3. Fluid volume excess. pancrelipase, a pancreatic enzyme replacement.
Which statement by the client demonstrates
4. Altered clotting mechanism.
understanding of his medication regimen?
334. A client has choledocholithiasis. During the 1. I will take this medication with a glass of milk.
nursing admission, the nurse notes that the 2. If my stools appear yellowish with a foul
clients sclera and skin are jaundiced. When odor, I will call my doctor.
complaining of abdominal distention and pain, 3. I will take my medication with some antacid
how is the client most likely to describe this so it will not bother my stomach.
condition? 4. I will be sure to thoroughly chew the
1. An intermittent, colicky pain in his left flank. capsule contents before swallowing the
2. Pain which awakens him during the night, medication.
and is relieved by eating.
3. A vise-like pressure over his sternum. 339. A client has been diagnosed with a malignant
tumor of the head of the pancreas. A
4. Right upper quadrant pain that often radiates
pancreatoduodenectomy (Whipples procedure)
to his right shoulder.
was done to resect the tumor. The nurse
335. A client is admitted to the hospital for acute recognizes that hemorrhage is a potentially
cholecystitis. She is now 6 hours post-op major complication of a Whipple procedure.
abdominal cholecystectomy with a Which of the following assessment findings
choledochostomy and has a T-tube in place. suggest this complication?
What is the proper management of the T-tube? 1. Jaundice of the skin and sclera.
1. Hanging the T-tube drainage below the bed. 2. Hyperglycemia.
2. Notifying the physician if T-tube drainage is 3. Oliguria.
75 mL for the first 24 hours after surgery. 4. Bradycardia.
3. Irrigating the T-tube with sterile normal
saline q 2 h to prevent obstruction.
4. Clamping the T-tube if the client develops Answers and Rationales
sudden, severe abdominal pain.

336. A client has experienced repeated episodes of 290. 3. Looping the NG tube will prevent pressure on
acute pancreatitis. He has continued to consume the nares that can cause pain and eventual
alcohol. The nurse observes that he is doubled- necrosis.
over, rocking back-and-forth in pain. Why is
morphine derivative contraindicated for the pain 291. 4. The drainage following abdominal surgery is
associated with acute pancreatitis? discolored as it is evacuating stomach and

4
1. It causes severe respiratory depression. intestinal contents, not mucoid material.

338 NCLEX-RN Review


53155_04_Ch04b_p264-377.qxd 2/26/09 7:32 AM Page 339

292. 4. The nurse should explore the clients 304. 3. A client with altered central nervous system
motivation for posing the question and establish functioning is at high risk for aspiration.
this current knowledge. Checking for the placement of the feeding tube
using several different methods, i.e., aspiration
293. 4. The onset of hepatitis B is long and insidious, of gastric contents for residual volume,
lasting from 60 to 160 days. determining pH of aspirated gastric contents,
and auscultating for gurgling sounds with
294. 1. Cimetidine (Tagemet) is a histamine
injection of air bolus, is the priority nursing
antagonist that blocks the secretion of
assessment to ensure client safety.
hydrochloric acid.
305. 2. The nasogastric tube for gastric decompression
295. 2. Signs of dumping syndrome include vertigo,
after a gastric resection is never irrigated without
pallor, sweating, palpitations, and weakness.
a specific order from the physician. Irrigating the
Dumping syndrome occurs after a gastric
nasogastric tube may rupture the suture line and
resection because ingested foods rapidly enter
hemorrhaging may occur.
the jejunum without proper mixing and without
the normal duodenal processing. It subsides in 306. 2. Metoclopramide blocks dopamine receptors
612 months. in the chemoreceptor trigger zone (CTZ). This
action results in the extrapyramidal and
296. 4. Bluish discoloration of the umbilicus
anticholinergic side effects that include
(Cullens sign) is present in massive
sedation, dilated pupils and parkinsonian
gastrointestinal hemorrhage resulting from free
effects.
blood present in the abdomen. This is not
consistent with cirrhosis of the liver. 307. 3. Increased urinary output is related to the
resolved dehydration state. Adding fluid to the
297. 4. Sodium restriction is most important for a
enteral feedings reduced the osmolarity of the
client with cirrhosis because fluid retention
gastrointestinal contents.
contributes to ascites.
308. 3. Exercises to strengthen the abdominal muscles
298. 3. Increased ammonia levels could be seen
are appropriate to aiding the defecation process.
because ammonia is a by-product of protein
metabolism, and a diseased liver is unable to 309. 3. Surgical resection of the esophagus has a
convert ammonia into urea to be excreted in the relatively high mortality rate related to
urine. pulmonary complications.
299. 1. If the clients airway is obstructed by the 310. 4. These substances aggravate the episodes of
Sengstaken-Blakemore tube, the esophageal heartburn (pyrosis) and gastroesophageal reflux.
balloon must be deflated so the client can
breathe. 311. 3. Duodenal ulcer pain characteristically occurs
2 to 3 hours after a meal, often awakening the
300. 2. The clients throat is sprayed with a local client in the very early morning hours.
anesthetic agent. Until the anesthetic agent
wears off, the client is at high risk for aspiration. 312. 3. Misoprostol, a synthetic prostaglandin, is a
cytoprotective agent. By increasing mucous
301. 1. A low-residue diet 1 to 2 days before the production and bicarbonate levels, the mucosal
study aids in evacuating the lower intestinal barrier better resists the erosive action of the
tract of all fecal matter. gastric acid-pepsin complex.
302. 4. The client experiencing a liver biopsy is at risk 313. 3. Fluids with meals cause rapid emptying of the
for bleeding or hemorrhage related to penetration gastric contents. Fluids with meals should be
of the liver capsule. Positioning on the right side limited.
acts as a tamponade against the puncture site
discouraging bleeding from the site. 314. 4. A total gastrectomy results in a loss of
intrinsic factor, which is necessary for the
303. 2. Oil retention enemas are given to soften the absorption of vitamin B12.
hardened fecal mass and lubricate the walls of
the rectum and colon. Cleansing enemas 315. 3. A direct inguinal hernia is most likely to
stimulate intestinal peristalsis, thus eliminating cause a small bowel obstruction. Therefore, the
the softened fecal mass. nurse must monitor closely for the

ADULT NURSING

4 339
53155_04_Ch04b_p264-377.qxd 2/26/09 7:32 AM Page 340

signs/symptoms of a small bowel mechanical 327. 4. Anorexia is often an early and severe
obstruction, including vomiting of bile-stained symptom of hepatitis.
gastric contents from reverse peristalsis.
328. 1. Hepatitis A is transmitted through the fecal-
316. 1. A paralytic ileus is related to disturbance of oral route. Childcare workers are in a high risk
the neural stimulation of the bowel. There is group because of potentially poor
decreased or absence of bowel sounds. hygiene/sanitation practices.

317. 4. Anorexia, weight loss, fever, vomiting, and 329. 2. Hepatitis B is transmitted parenterally or
blood loss are conditions that will cause the through intimate sexual contact with a carrier.
client to become easily fatigued. Activities are
planned or restricted to conserve energy. 330. 3. Portal hypertension puts the client at risk
for injury related to bleeding/hemorrhaging
318. 3. When grain is refined, most of the original esophageal varices. Monitoring stools permits
fiber is removed, then vitamins and additives are early detection of bleeding in the GI tract.
added to compensate, thus producing a low-
residue product. 331. 4. A serious complication of a paracentesis is
hypovolemic shock or vascular collapse. Early
319. 2. Toxic megacolon is a serious complication detection of this cardiovascular complication
of ulcerative colitis. Excessive dilation of the through monitoring blood pressure and pulse is
colon may lead to intestinal perforation and a nursing priority intervention.
death.
332. 4. Airway obstruction and aspiration of
320. 3. A soft, high-fiber diet is indicated to increase
gastric contents are potential serious
the bulk of the stool, thereby promoting
complications of balloon tamponade. Frequent
defecation. Fluid intake of 2 liters/day is
assessment of the clients respiratory status is
recommended unless otherwise contraindicated.
the priority.
Seeds are not allowed.
333. 2. Hepatic cirrhosis leads to elevated serum
321. 3. A change in bowel habits such as alternating
ammonia levels, which have an adverse toxic
episodes of diarrhea and constipation is a
effect on cerebral metabolism.
common manifestation of colon cancer.

322. 4. A diet low in fiber is a major risk factor for 334. 4. Pain related to gallstones in the common duct
colon cancer. Fresh fruits and vegetables is located in the right upper quadrant and often
increase the fiber content of the diet, thereby radiates to the right shoulder or back.
reducing the risk of colon cancer.
335. 2. The T-tube usually drains 200500 mL in the
323. 3. Leaving 4 inch of skin exposed around stoma
1 first 24 hours. Decreased bile drainage may
when determining size to cut skin barrier indicate an obstruction to bile flow or bile may
prevents trauma to stoma. be leaking into the peritoneum.

324. 4. A skin barrier applied helps prevent 336. 3. Morphine sulfate causes spasms of the
enzymatic activity, which is a risk for peristomal sphincter of Oddi, which will exacerbate the
skin breakdown. episode of acute pancreatitis.

325. 1. Hypovolemia occurs because massive 337. 3. Cullens sign and Turners sign reveal
amounts of fluid and electrolytes move from discoloration and occur with intra-abdominal
intestinal lumen into peritoneal cavity and bleeding.
deplete intravascular volume. Hypotension and
tachycardia are manifestations of this massive 338. 2. The presence of steatorrhea indicates that
fluid shift. the dosage of pancrelipase needs to be
adjusted.
326. 2. Post-hemorrhoidectomy diet is modified to
include increased fluid and fiber intake. This 339. 3. Oliguria is a primary sign of hypovolemic
promotes regular bowel elimination and reduces shock related to hemorrhage.
the occurrence of constipation.

4 340 NCLEX-RN Review


53155_04_Ch04b_p264-377.qxd 2/26/09 7:32 AM Page 341

The Genitourinary System

OVERVIEW OF ANATOMY D. Nephron: the functional unit of the kidney


(see Figure 4-20)
AND PHYSIOLOGY 1. Renal corpuscle (vascular system of nephron)
a. Bowmans capsule: a portion of the
The genitourinary system includes the kidneys, ureters, proximal tubule, surrounds the glomerulus
urinary bladder, urethra, and the male and female b. Glomerulus: a capillary network permeable
genitalia. This section discusses only the male genitalia; to water, electrolytes, nutrients, and
the female reproductive system is covered in Unit 6. wastes; impermeable to large protein
molecules
Urinary System 2. Renal tubule: divided into proximal
convoluted tubule, descending loop of Henle,
Kidneys ascending loop of Henle, distal convoluted
tubule, and collecting duct
See Figure 4-19.
A. Two bean-shaped organs that lie in the
retroperitoneal space on either side of the
vertebral column; adrenal glands located on top of
each kidney
B. Renal parenchyma
1. Cortex: outermost layer; site of glomeruli and
proximal and distal tubules of nephron
2. Medulla: middle layer; formed by collecting
tubules and ducts
C. Renal sinus and pelvis
1. Papillae: projections of renal tissues located at
the tips of the renal pyramids
2. Calices
a. Minor calyx: collects urine flow from
collecting duct
b. Major calyx: directs urine from renal sinus
to renal pelvis
3. Urine flows from renal pelvis to ureters

Figure 4-19 Anatomy of the kidney Figure 4-20 The nephron

ADULT NURSING

4 341
53155_04_Ch04b_p264-377.qxd 2/26/09 7:32 AM Page 342

Ureters B. Tubular function: the tubules and collecting ducts


carry out the functions of reabsorption, secretion,
A. Two tubes approximately 2535 cm long and excretion. Reabsorption of water and
B. Extend from the renal pelvis to the pelvic cavity, electrolytes is controlled by antidiuretic hormone
where they enter the bladder, convey urine from (ADH), released by the pituitary, and aldosterone,
the kidneys to the bladder secreted by the adrenal glands (see Table 4-25).
C. Ureterovesical valve prevents backflow of urine 1. Proximal convoluted tubule: reabsorption of
into ureters certain constituents of the glomerular filtrate:
80% of electrolytes and H2O, all glucose and
Bladder amino acids, and bicarbonate; secretes organic
substances and wastes.
A. Located behind the symphysis pubis; composed of 2. Loop of Henle: reabsorption of sodium and
muscular, elastic tissue that makes it distensible chloride in the ascending limb; reabsorption of
B. Serves as a reservoir of urine (capable of holding water in the descending limb;
10001800 mL; moderately full bladder usually concentrates/dilutes urine.
holds about 500 mL) 3. Distal convoluted tubule: secretes potassium,
C. Internal and external urethral sphincters control the hydrogen ions, and ammonia; reabsorbs H2O
flow of urine; urge to void stimulated by passage of (regulated by ADH) and bicarbonate; regulates
urine past the internal sphincter (involuntary) to the calcium and phosphate concentrations.
upper part of the urethra. Relaxation of external 4. Collecting ducts: receive urine from distal
sphincter (voluntary) produces emptying of the convoluted tubules and reabsorb water
bladder (voiding, micturition). (regulated by ADH).
C. Normal adult produces 1 liter/day of urine.
Urethra
A. A small tube that extends from the bladder to the Blood Pressure Control
exterior of the body A. Kidneys regulate blood pressure partly through
B. In females, located behind the symphysis pubis and maintenance of volume (formation/excretion of
anterior to the vagina; approximately 35 cm long urine).
C. In males, extends the entire length of the penis; B. The renin-angiotensin system is the other kidney-
approximately 20 cm long controlled mechanism that can contribute to rise
in blood pressure. When blood pressure drops, the
Regulatory Functions of Kidney cells of the glomerulus release renin, which then
activates angiotensin to cause vasoconstriction.
Kidneys and urinary system play a major role in
maintenance of homeostatic control of the body.
Kidneys remove nitrogenous wastes and regulate fluid
and electrolyte balance and acid-base balance. Urine is Male Reproductive System
the end product of these mechanisms. See Figure 4-21.

Formation of Urine Penis


A. Glomerular filtration
A. An external structure that serves as a passageway
1. Ultrafiltration of blood by the glomerulus;
for urine and semen
beginning of urine formation
B. Capable of distension during sexual excitement
a. Requires hydrostatic pressure (supplied by
C. Distal portion, glans penis, is covered by a prepuce
the heart and assisted by vascular resistance
or foreskin that may or may not be removed
[glomerular hydrostatic pressure]) and
(circumcised)
sufficient circulating volume.
b. Pressure in Bowmans capsule opposes
hydrostatic pressure and filtration; if Scrotum
glomerular pressure insufficient to force A. Saclike structure that hangs from the root of the
substances out of the blood into the penis
tubules, filtrate formation stops. B. Contains the testes and epididymis, and helps to
2. Glomerular filtration rate (GFR): amount of regulate temperature conducive to sperm production
blood filtrated by the glomeruli in a given
time; normal is 125 mL/min
3. Filtrate formed has essentially same
Testes
composition as blood plasma without the A. Small oval structures suspended in the scrotum
proteins; blood cells and proteins are usually B. Produce sperm (exocrine function) and male

4
too large to pass the glomerular membrane. hormones (endocrine function, see Table 4-25).

342 NCLEX-RN Review


53155_04_Ch04b_p264-377.qxd 2/26/09 7:32 AM Page 343

Figure 4-21 The male reproductive system

Ductal System 2. Secretes a milky fluid that aids in the passage


of spermatozoa and helps keep them viable
A. Epididymis: first part of ductal system B. Cowpers glands: lie on each side of the urethra
1. Soft cordlike structure that lies along the and just below the prostate; secrete a small amount
posterolateral surface of each testis of lubricating fluid.
2. Head is attached to the top of the testis, tail is C. Seminal vesicles: paired structures parallel to the
continuous with vas deferens; stores bladder; secrete a portion of the ejaculate and may
spermatozoa while they mature. contribute to nutrition and activation of sperm.
B. Spermatic cord: consists of vas deferens, arteries,
veins, nerves, and lymphatic vessels. Vas deferens
joins the duct of the seminal vesicles to become ASSESSMENT
the ejaculatory duct.

Accessory Glands Health History


A. Presenting problem: symptoms may include:
A. Prostate: located below the bladder and in front of 1. Pain in flank, groin; dysuria
the rectum; approximately 46 cm long 2. Changes in urination patterns: frequency,
1. Enclosed in firm, fibrous capsule; connected to nocturia, hesitancy of stream, urgency,
the urethra and ejaculatory ducts dribbling, incontinence, retention

ADULT NURSING

4 343
53155_04_Ch04b_p264-377.qxd 2/26/09 7:32 AM Page 344

3. Changes in urinary output: polyuria, oliguria, B. Urine collection methods: nursing care
anuria 1. Routine urinalysis: wash perineal area if
4. Changes in color/consistency of urine: dilute, soiled, obtain first voided morning specimen;
concentrated, malodorous; hematuria, pyuria send to lab immediately (should be examined
B. Lifestyle: occupation (type of employment, within 1 hour of voiding).
exposure to chemicals such as carbon tetrachloride, 2. Clean catch (midstream) specimen for urine
ethylene glycol); level of activity, exercise culture.
C. Nutrition/diet: water, calcium, dairy product a. Cleanse perineal area.
intake 1) Females: spread labia and cleanse
D. Past medical history: hypertension; diabetes meatus front to back using antiseptic
mellitus; gout; cystitis; kidney infections; sponges.
connective tissue diseases (systemic lupus 2) Males: retract foreskin
erythematosus); infectious diseases; drug use (if uncircumsized) and cleanse
(prescribed/OTC); previous catheterizations, glans with antiseptic sponges.
hospitalizations, or surgery for renal problems b. Have client initiate urine stream then stop.
E. Family history: hypertension, diabetes mellitus, c. Collect specimen in a sterile container.
renal disease, gout, connective tissue disorders, d. Have client complete urination but not in
urinary tract infections (UTIs), renal calculi specimen container.
3. 24-hour specimen (preferred method for
Physical Examination creatinine clearance test)
a. Have client void and discard specimen;
A. Inspect skin for color, turgor, and mobility; note time.
purpuric lesions; integrity. b. Collect all subsequent urine specimens for
B. Inspect mouth for color, moisture, odor, 24 hours.
ulcerations. c. If specimen is accidentally discarded, the
C. Inspect face for edema, particularly periorbital test must be restarted.
edema. d. Record exact start and finish of collection;
D. Inspect abdomen and palpate bladder for include date and times.
distension; percuss bladder for tympany or C. Blood studies
dullness (if full). 1. Bicarbonate
E. Inspect extremities for edema. 2. BUN: measures renal ability to excrete urea
F. Determine rate, rhythm, and depth of respirations. nitrogen
G. Inspect muscles for tremors, atrophy. 3. Calcium
H. Palpate right and left kidneys for tenderness, pain, 4. Serum creatinine: specific test for renal disorders;
enlargement; percuss costovertebral angles for reflects ability of kidneys to excrete creatinine
tenderness/pain; fist percuss kidneys for 5. Phosphorus
tenderness/pain. 6. Potassium
I. Palpate flank area for pain and prostate for size, 7. Sodium
shape, consistency. 8. Prostate-specific antigen (PSA)
J. Auscultate aorta and renal arteries for bruits. D. KUB/plain film: an abdominal flat-plate X-ray
showing the kidneys, ureters, and bladder; may
Laboratory/Diagnostic Tests identify the number and size of kidneys with
A. Urine studies tumors, malformations, and calculi
1. Urinalysis: examination to assess the nature of E. Intravenous pyelogram (IVP)
the urine produced 1. Fluoroscopic visualization of the urinary tract
a. Evaluates color, pH, and specific gravity. after injection with a radiopaque dye
b. Determines presence of glucose 2. Nursing care: pretest
(glycosuria), protein, blood (hematuria), a. Assess for iodine sensitivity.
ketones (ketonuria). b. Inform client he will lie on a table
c. Analyzes sediment for cells (presence of throughout procedure.
WBC called pyuria), casts, bacteria, crystals. c. Administer cathartic or enema the night
2. Urine culture and sensitivity: diagnoses before.
bacterial infections of the urinary tract d. Keep client NPO for 8 hours pretest.
3. Residual urine: amount of urine left in bladder 3. Nursing care: posttest: force fluids
after voiding measured via catheter F. Cystoscopy
(permanent or temporary) in bladder 1. Use of a lighted scope (cystoscope) to inspect
4. Creatinine clearance: determines amount of the bladder
creatinine (waste product of protein a. Inserted into the bladder via the urethra
breakdown) in the urine over 24 hours, b. May be used to remove tumors, stones, or

4
measures overall renal function other foreign material (use of electrical

344 NCLEX-RN Review


53155_04_Ch04b_p264-377.qxd 2/26/09 7:32 AM Page 345

current to remove tumors is called Interventions


fulguration); or to implant radium, place
catheters in ureters Urinary Catheterization
2. Nursing care: pretest
A. General information
a. Explain to client that procedure will be
1. Insertion of a catheter through the external
done under general or local anesthesia.
meatus and the urethra into the bladder
b. Confirm consent form is signed.
2. Purposes include relief from urinary retention,
c. Administer sedatives 1 hour before test, as
bladder decompression, prevention of bladder
ordered.
obstruction, instillation of medications into
d. General anesthesia: keep client NPO.
the bladder, splinting the bladder, and output
e. Local anesthesia: offer liquid breakfast.
monitoring.
f. Give enemas as ordered.
B. Nursing care: insertion
3. Nursing care: posttest
1. Explain procedure to client and collect
a. Provide warm sitz baths, mild analgesics to
necessary equipment (catheter set).
relieve discomfort after test.
2. Wash hands and position client.
b. Monitor I&O and vital signs (especially
3. Use sterile technique while inserting catheter.
temperature, as elevation may indicate
4. Observe for urine return and obtain specimen.
infection).
5. Connect drainage tubing to catheter
c. Expect mild hematuria at first; urine will
(indwelling) and tape.
be pink tinged, subsiding over 2448
C. Nursing care: indwelling catheter
hours; monitor for large clots.
1. Maintain catheter patency: place drainage
d. Advise client that burning on urination is
tubing properly to avoid kinking or pinching.
normal and will subside.
2. Observe for signs of obstruction (e.g.,
e. Force fluids.
decreased urine in collection bag, voiding
around the catheter, abdominal discomfort,
bladder distension).
ANALYSIS 3. Irrigate catheter as necessary.
4. Ensure comfort and safety: relieve bladder
Nursing diagnoses for the client with a disorder of the
spasms by administering belladonna
genitourinary system may include:
suppositories (if ordered); ensure adequate
A. Deficient fluid volume
fluid intake and provide perineal care.
B. Fluid volume excess
5. Prevent infection: maintain a closed drainage
C. Fatigue
system and prevent backflow of urine by
D. Risk for injury
keeping drainage system below level of bladder.
E. Disturbed thought processes
6. Empty collection bag at least every 8 hours.
F. Impaired oral mucous membrane
7. Promote acidification of the urine with acid-
G. Imbalanced nutrition: less than body requirements
ash diet and ascorbic acid.
H. Risk for infection
8. Change catheter/drainage system only when
I. Impaired skin integrity
necessary.
J. Urinary retention
K. Sexual dysfunction
Dialysis
A. General information
PLANNING AND 1. Removal by artificial means of metabolic
IMPLEMENTATION wastes, excess electrolytes, and excess fluid
from clients with renal failure
2. Principles
Goals a. Diffusion: movement of particles from an
A. Fluid imbalance will be resolved. area of high concentration to one of low
B. Client will exhibit improved sense of energy. concentration across a semipermeable
C. Client will not exhibit unusual bleeding. membrane
D. Thought processes will improve. b. Osmosis: movement of water through a
E. Integrity of mucous membranes will be semipermeable membrane from an area of
maintained. lesser concentration of particles to one of
F. Adequate nutritional status will be maintained. greater concentration
G. Client will remain free from infection. B. Purposes
H. Adequate skin integrity will be maintained. 1. Remove the endproducts of protein
I. Client will demonstrate restored urinary flow. metabolism from blood
2. Maintain safe levels of electrolytes

4
J. Changes in sexual functioning will be accepted.

ADULT NURSING 345


53155_04_Ch04b_p264-377.qxd 2/26/09 7:32 AM Page 346

3. Correct acidosis and replenish blood d. Graft: piece of bovine artery or vein, Gore-
bicarbonate system Tex material, or saphenous vein sutured to
4. Remove excess fluid from the blood clients own vessel; used for clients with
C. Types: hemodialysis and peritoneal dialysis compromised vascular systems; provides a
segment in which to place dialysis needles;
Hemodialysis ready for use in 23 weeks.
A. General information B. Nursing care: external AV shunt
1. Shunting of blood from the clients vascular 1. Auscultate for a bruit and palpate for a thrill to
system through an artificial dialyzing system, ensure patency.
and return of dialyzed blood to the clients 2. Assess for clotting (color change of blood,
circulation absence of pulsations in tubing).
2. Dialysis coil acts as the semipermeable 3. Change sterile dressing over shunt daily.
membrane; the dialysate is a specially 4. Avoid performing venipuncture, administering
prepared solution. IV infusions, giving injections, or taking a
3. Access routes (see Figure 4-22) blood pressure with a cuff on the shunt arm.
a. External AV shunt: one cannula inserted C. Nursing care: AV fistula.
into an artery and the other into a vein; 1. Auscultate for a bruit and palpate for a thrill to
both are brought out to the skin surface ensure patency.
and connected by a U-shaped shunt. 2. Report bleeding, skin discoloration, drainage,
b. AV fistula: internal anastomosis of an and pain.
artery to an adjacent vein in a sideways 3. Avoid restrictive clothing/dressings over site.
position; fistula is accessed for 4. Avoid administration of IV infusions, giving
hemodialysis by venipuncture; takes injections, or taking blood pressure with a cuff
46 weeks to be ready for use. on the fistula extremity.
c. Femoral/subclavian cannulation: insertion D. Nursing care: femoral/subclavian cannulation
of a catheter into one of these large veins 1. Palpate peripheral pulses in cannulized
for easy access to circulation; procedure is extremity.
similar to insertion of a CVP line; 2. Observe for bleeding/hematoma formation.
temporary, but can be used immediately; 3. Position catheter properly to avoid
associated with more clotting problems. dislodgment during dialysis.
E. Nursing care: before and during hemodialysis
1. Have client void.
2. Chart clients weight.
Nearest adjacent vein

Arteriovenous fistula Artery


Edges of incision in artery and vein are
sutured together to form a common opening.

Venous cannula
installed in vein

Arteriovenous shunt
Arterial cannula
installed in artery

Arteriovenous vein graft

Ends of natural or synthetic graft sutured


A. into an artery and a vein. B. FEMORAL VEIN CATHETERIZATION C. SUBCLAVIAN VEIN CATHETERIZATION

Figure 4-22 Dialysis vascular access sites. (A) Arteriovenous access; (B) femoral access; (C) subclavian
access catheter

4 346 NCLEX-RN Review


53155_04_Ch04b_p264-377.qxd 2/26/09 7:32 AM Page 347

3. Assess vital signs before and every 30 minutes b. Dialysis disequilibrium syndrome (urea is
during procedure. removed more rapidly from the blood than
4. Withhold antihypertensives, sedatives, and from the brain): assess for nausea, vomiting,
vasodilators to prevent hypotensive episode elevated blood pressure, disorientation, leg
(unless ordered otherwise). cramps, and peripheral paresthesias.
5. Ensure bed rest with frequent position changes
for comfort. Peritoneal Dialysis
6. Inform client that headache and nausea may A. General information: introduction of a specially
occur. prepared dialysate solution into the abdominal
7. Monitor closely for signs of bleeding because cavity, where the peritoneum acts as a
blood has been heparinized for procedure. semipermeable membrane between the dialysate and
F. Nursing care: postdialysis blood in the abdominal vessels (see Figure 4-23).
1. Chart clients weight. B. Nursing care
2. Assess for complications. 1. Chart clients weight.
a. Hypovolemic shock: may occur as a result 2. Assess vital signs before, every 15 minutes
of rapid removal or ultrafiltration of fluid during first exchange, and every hour
from the intravascular compartment. thereafter.
3. Assemble specially prepared dialysate
solution with added medications.
4. Have client void.
5. Warm dialysate solution to body temperature.
6. Assist physician with trocar insertion.
7. Inflow: allow dialysate to flow unrestricted
into peritoneal cavity (1020 minutes).
8. Dwell: allow fluid to remain in peritoneal
cavity for prescribed period (3045 minutes).
Fresh
dialysate
9. Drain: unclamp outflow tube and allow to flow
solution by gravity.
10. Observe characteristics of dialysate outflow.
a. Clear pale yellow: normal
b. Cloudy: infection, peritonitis
c. Brownish: bowel perforation
d. Bloody: common during first few
Peritoneal
exchanges; abnormal if continues
cavity 11. Monitor total I&O and maintain records.
Tenchkoff
peritoneal 12. Assess for complications.
catheter a. Peritonitis resulting from contamination of
solution or tubing during exchange
b. Respiratory difficulty: may occur from
upward displacement of diaphragm due to
increased pressure in the peritoneal cavity;
assess for signs and symptoms of
Adapter atelectasis, pneumonia, and bronchitis
c. Protein loss: most serum proteins pass
through the peritoneal membrane and are
lost in the dialysate fluid; monitor serum
protein levels closely

Continuous Ambulatory Peritoneal Dialysis


A. General information
Used 1. A continuous type of peritoneal dialysis
dialysate
solution performed at home by the client or significant
others.
2. Dialysate is delivered from flexible plastic
containers through a permanent peritoneal
catheter.
Figure 4-23 Peritoneal dialysis removes wastes 3. Following infusion of the dialysate into the
through a fluid exchange in the peritoneal cavity peritoneal cavity, the bag is folded and tucked

4
away during the dwell period.

ADULT NURSING 347


53155_04_Ch04b_p264-377.qxd 2/26/09 7:32 AM Page 348

B. Provide client teaching and discharge planning B. Assessment


concerning: 1. Abdominal or flank pain/tenderness,
1. Need to assess the permanent peritoneal frequency and urgency of urination, pain on
catheter for complications voiding, nocturia
a. Dialysate leak 2. Fever
b. Exit site infection 3. Diagnostic tests: urine culture and sensitivity
c. Bacterial/fungal contamination reveals specific organism (80% E. coli)
d. Obstruction C. Nursing interventions
2. Adherence to high-protein (if indicated), well- 1. Force fluids (3000 mL/day).
balanced diet 2. Provide warm sitz baths for comfort.
3. Importance of periodic blood chemistries 3. Assess urine for odor, hematuria, sediment.
4. Daily weights 4. Administer medications as ordered and
monitor effects.
EVALUATION a. Systemic antibiotics: ampicillin,
cephalosporins, aminoglycosides
A. Adequate urinary output with specific gravity/ b. Sulfonamides: sulfisoxazole (Gantrisin),
laboratory studies within clients normal range; sulfamethoxazole (Gantanol),
stable weight; absence of edema; pulmonary trimethoprim-sulfamethoxazole
congestion. (Bactrim)
B. Client verbalizes increased tolerance for c. Antibacterials: nitrofurantoin
activities. (Macrodantin), methenamine mandelate
C. Skin and mucous membranes free from ecchymoses/ (Mandelamine), nalidixic acid (NegGram)
bleeding; improved laboratory values (CBC, platelet d. Urinary tract analgesic: pyridium
count; clotting factors); no signs of bleeding. 5. Provide client teaching and discharge
D. Client identifies ways to compensate for cognitive planning concerning:
impairment; demonstrates improved problem- a. Importance of adequate hydration
solving skills. b. Frequent voiding to avoid stagnation
E. Oral mucosa pink, moist, and intact; no ulcerations; c. Proper personal hygiene; women to cleanse
saliva consistency normal; verbalizes interventions from front to back
to promote/maintain healthy oral mucosa. d. Voiding after sexual intercourse
F. Stable weight gain; laboratory values within e. Acidification of the urine to decrease
clients normal range; improved anthropometric bacterial multiplication (acid-ash diet,
measurements. vitamin C)
G. Vital signs within normal range; client identifies f. Need for follow-up urine cultures
measures to prevent/reduce the risk of infection.
H. Skin warm and dry; absence of redness and
Bladder Cancer
irritation.
I. Voiding in adequate amounts with no palpable A. General information
bladder distention; postvoid residuals less than 1. Most common site of cancer of the urinary
50 mL; absence of dribbling/overflow. tract
J. Client identifies acceptable sexual practices and 2. Occurs in men 3 times more often than
explores alternate methods. women; peak age 5070 years
K. Client integrates treatment regimens into ADL; 3. Predisposing factors include exposure to
shows increased interest in appearance; actively chemicals (especially aniline dyes), cigarette
participates in treatments. smoking, chronic bladder infections
B. Medical management: dependent on the staging of
cell type; includes:
DISORDERS OF THE 1. Radiation therapy, usually in combination
GENITOURINARY SYSTEM with surgery
2. Chemotherapy: considerable research on both
agents and methods of administration
Disorders of the Urinary Tract a. Methods include direct bladder
Cystitis instillations, intra-arterial infusions,
IV infusion, oral ingestion
A. General information b. Agents include 5-fluorouracil,
1. Inflammation of the bladder due to bacterial methotrexate, bleomycin, mitomycin-C,
invasion hydroxyurea, doxorubicin,
2. More common in women cyclophosphamide, cisplatin; results
3. Predisposing factors include stagnation of variable

4
urine, obstruction, sexual intercourse, high 3. Surgery: see Bladder Surgery.
estrogen levels

348 NCLEX-RN Review


53155_04_Ch04b_p264-377.qxd 2/26/09 7:32 AM Page 349

C. Assessment findings b. Patch test all adhesives, sprays, and skin


1. Intermittent painless hematuria, dysuria, barriers before use.
frequent urination c. Change appliance only when necessary
2. Diagnostic tests and when production of urine is slowest
a. Cytoscopy with biopsy reveals malignancy (early morning).
b. Cytologic exam of the urine reveals d. Place wick (rolled gauze pad) on stomal
malignant cells opening when appliance is off.
D. Nursing interventions: provide care for the client e. Cleanse peristomal skin with mild soap
receiving radiation therapy or chemotherapy, and and water.
for the client with bladder surgery. f. Remove alkaline encrustations by applying
vinegar and water solution to peristomal area.
Bladder Surgery g. Implement measures to maintain urine
acidity (acid-ash foods, vitamin C therapy,
A. General information
omission of milk/dairy products).
1. Cystectomy (removal of the urinary bladder) with
4. Provide care for the client with an NG tube;
one of the various types of urinary diversions is
will be in place until bowel motility returns.
the surgical procedure done for bladder cancer
5. Assist client to identify strengths and qualities
2. Types of urinary diversions
that have a positive effect on self-concept.
a. Ureterosigmoidostomy: ureters are excised
6. Provide client teaching and discharge
from the bladder and implanted into
planning concerning
sigmoid colon; urine flows through the
a. Maintenance of stomal/peristomal skin
colon and is excreted via the rectum
integrity
b. Ileal conduit: ureters are implanted into a
b. Proper application of appliance
segment of the ileum that has been
c. Recommended method of cleaning
resected from the intestinal tract with
reusable ostomy equipment
formation of an abdominal stoma; most
(manufacturers recommendations)
common type of urinary diversion
d. Information regarding prevention of UTIs
c. Cutaneous ureterostomy: ureters are excised
(adequate fluids; empty pouch when half full;
from the bladder and brought through
change to bedside collection bag at night)
abdominal wall with creation of a stoma
e. Control of odor (adequate fluids; avoid
d. Nephrostomy: insertion of a catheter into
foods with strong odor; place small amount
the renal pelvis via an incision into the
of vinegar or deodorizer in pouch)
flank or by percutaneous catheter
f. Reporting signs and symptoms of UTIs (see
placement into the kidney
Cystitis).
B. Nursing interventions: preoperative
1. Provide routine pre-op care.
2. Assess clients ability to learn prior to starting Nephrolithiasis/Urolithiasis
a teaching program. A. General information
3. Discuss social aspects of living with a stoma 1. Presence of stones anywhere in the urinary
(sexuality, changes in body image). tract; frequent composition of stones: calcium,
4. Assess understanding and emotional response oxalate, and uric acid
of client/significant others. 2. Most often occurs in men age 2055; more
5. Perform pre-op bowel preparation for common in the summer
procedures involving the ileum or colon. 3. Predisposing factors
6. Inform client of post-op procedures. a. Diet: large amounts of calcium, oxalate
C. Nursing interventions: postoperative b. Increased uric acid levels
1. Provide routine post-op care. c. Sedentary lifestyle, immobility
2. Maintain integrity of the stoma. d. Family history of gout or calculi;
a. Monitor for and report signs of impaired hyperparathyroidism
stomal healing (pale, dark red, or blue- B. Medical management
black color; increased stomal height, 1. Surgery
edema, bleeding). a. Cystoscopic stone removal: basket
b. Maintain stomal circulation by using extraction is invasive but effective for
properly fitted faceplate. stones even in kidney.
c. Monitor for signs and symptoms of stomal b. Percutaneous nephrostomy: tube is
obstruction (sudden decrease in urine inserted through skin and underlying
output, increased abdominal tenderness tissues into renal pelvis to remove calculi.
and distension). c. Percutaneous nephrostolithotomy: delivers
3. Prevent skin irritation and breakdown. ultrasound waves through a probe placed
a. Inspect skin areas for signs of breakdown

4
on the calculus.
daily.

ADULT NURSING 349


53155_04_Ch04b_p264-377.qxd 2/26/09 7:32 AM Page 350

2. Extracorporeal shock-wave lithotripsy: 2. Acute: infection usually ascends from lower


delivers shock waves from outside the body to urinary tract
the stone, causing pulverization 3. Chronic: thought to be a combination of
3. Pain management and diet modification structural alterations along with infection,
C. Assessment findings major cause is ureterovesical reflux, with
1. Abdominal or flank pain; renal colic; hematuria infected urine backing up into ureters and
2. Cool, moist skin renal pelvises; result of recurrent infections is
3. Diagnostic tests eventual renal parenchymal deterioration and
a. KUB: pinpoints location, number, and size possible renal failure
of stones B. Medical management
b. IVP: identifies site of obstruction and 1. Acute: antibiotics, antispasmodics, surgical
presence of nonradiopaque stones removal of any obstruction
c. Urinalysis: indicates presence of bacteria, 2. Chronic: antibiotics and urinary antiseptics
increased protein, increased WBC and RBC (sulfanomides, nitrofurantoin); surgical
D. Nursing interventions correction of structural abnormality if possible
1. Strain all urine through gauze to detect stones C. Assessment findings
and crush all clots. 1. Acute: fever, chills, nausea and vomiting;
2. Force fluids (30004000 mL/day). severe flank pain or dull ache
3. Encourage ambulation to prevent stasis. 2. Chronic: client usually unaware of disease;
4. Relieve pain by administration of analgesics may have bladder irritability, chronic fatigue,
as ordered and application of moist heat to or slight dull ache over kidneys; eventually
flank area. develops hypertension, atrophy of kidneys
5. Monitor I&O. D. Nursing interventions: acute pyelonephritis
6. Provide modified diet, depending upon stone 1. Provide adequate comfort and rest.
consistency. 2. Monitor I&O.
a. Calcium stones: limit milk/dairy products; 3. Administer antibiotics as ordered.
provide acid-ash diet to acidify urine 4. Provide client teaching and discharge
(cranberry or prune juice, meat, eggs, poultry, planning concerning:
fish, grapes, whole grains); take vitamin C. a. Medication regimen
b. Oxalate stones: avoid excess intake of b. Follow-up cultures
foods/fluids high in oxalate (tea, chocolate, c. Signs and symptoms of recurrence and
rhubarb, spinach); maintain alkaline-ash diet need to report
to alkalinize urine (milk; vegetables; fruits E. Nursing interventions: chronic pyelonephritis
except prunes, cranberries, and plums). 1. Administer medications as ordered.
c. Uric acid stones: reduce foods high in 2. Provide adequate fluid intake and nutrition.
purine (liver, brains, kidneys, venison, 3. Support client/significant others and explain
shellfish, meat soups, gravies, legumes); possibility of dialysis, transplant options if
maintain alkaline urine. significant renal deterioration.
7. Administer allopurinol (Zyloprim) as ordered,
to decrease uric acid production; push fluids Glomerulonephritis
when giving allopurinol.
8. Provide client teaching and discharge See Unit 5.
planning concerning:
a. Prevention of urinary stasis by maintaining Nephrosis
increased fluid intake especially in hot
weather and during illness; mobility; See Unit 5.
voiding whenever the urge is felt and at
least twice during the night Acute Renal Failure
b. Adherence to prescribed diet
c. Need for routine urinalysis (at least every A. General information
34 months) 1. Sudden inability of the kidneys to regulate
d. Need to recognize and report fluid and electrolyte balance and remove toxic
signs/symptoms of recurrence (hematuria, products from the body
flank pain) 2. Causes
a. Prerenal: factors interfering with perfusion
Pyelonephritis and resulting in decreased blood flow and
glomerular filtrate, ischemia, and oliguria;
A. General information include CHF, cardiogenic shock, acute
1. Inflammation of the renal pelvis; may be vasoconstriction, hemorrhage, burns,

4
unilateral or bilateral, acute or chronic septicemia, hypotension

350 NCLEX-RN Review


53155_04_Ch04b_p264-377.qxd 2/26/09 7:32 AM Page 351

b. Intrarenal: conditions that cause damage to e. Assess every hour for hypovolemia;
the nephrons; include acute tubular replace fluids as ordered.
necrosis (ATN), endocarditis, diabetes f. Monitor ECG and auscultate heart as
mellitus, malignant hypertension, acute needed.
glomerulonephritis, tumors, blood g. Check urine, serum osmolality/osmolarity,
transfusion reactions, hypercalcemia, and urine specific gravity as ordered.
nephrotoxins (certain antibiotics, X-ray 3. Promote optimal nutritional status.
dyes, pesticides, anesthetics) a. Weigh daily.
c. Postrenal: mechanical obstruction b. Maintain strict I&O.
anywhere from the tubules to the urethra; c. Administer TPN as ordered.
include calculi, BPH, tumors, strictures, d. With enteral feedings, check for residual
blood clots, trauma, anatomic and notify physician if residual volume
malformation increases.
B. Assessment findings e. Restrict protein intake.
1. Oliguric phase (caused by reduction in 4. Prevent complications from impaired mobility
glomerular filtration rate) (pulmonary embolism, skin breakdown,
a. Urine output less than 400 mL/24 hours; contractures, atelectasis).
duration 12 weeks 5. Prevent fever/infection.
b. Manifested by hypernatremia, a. Take rectal temperature and obtain orders
hyperkalemia, hyperphosphatemia, for cooling blanket/antipyretics as needed.
hypocalcemia, hypermagnesemia, and b. Assess for signs of infection.
metabolic acidosis c. Use strict aseptic technique for wound and
c. Diagnostic tests: BUN and creatinine elevated catheter care.
2. Diuretic phase (slow, gradual increase in daily 6. Support client/significant others and
urine output) reduce/relieve anxiety.
a. Diuresis may occur (output 35 liters/day) a. Explain pathophysiology and relationship
due to partially regenerated tubules to symptoms.
inability to concentrate urine b. Explain all procedures and answer all
b. Duration: 23 weeks; manifested by questions in easy-to-understand terms.
hyponatremia, hypokalemia, and c. Refer to counseling services as
hypovolemia needed.
c. Diagnostic tests: BUN and creatinine 7. Provide care for the client receiving dialysis if
elevated used.
3. Recovery or convalescent phase: renal 8. Provide client teaching and discharge
function stabilizes with gradual improvement planning concerning:
over next 312 months a. Adherence to prescribed dietary regime
C. Nursing interventions b. Signs and symptoms of recurrent renal
1. Monitor/maintain fluid and electrolyte balance. disease
a. Obtain baseline data on usual appearance c. Importance of planned rest periods
and amount of clients urine. d. Use of prescribed drugs only
b. Measure I&O every hour; note excessive e. Signs and symptoms of UTI or respiratory
losses. infection, need to report to physician
c. Administer IV fluids and electrolyte immediately
supplements as ordered.
d. Weigh daily and report gains. Chronic Renal Failure
e. Monitor lab values; assess/treat fluid and
electrolyte and acid-base imbalances as A. General information
needed. 1. Progressive, irreversible destruction of the
2. Monitor alteration in fluid volume. kidneys that continues until nephrons are
a. Monitor vital signs, PAP, PCWP, CVP as replaced by scar tissue; loss of renal function
needed. gradual
b. Weigh client daily. 2. Predisposing factors: recurrent infections,
c. Maintain strict I&O records. exacerbations of nephritis, urinary tract
d. Assess every hour for hypervolemia; obstructions, diabetes mellitus, hypertension
provide nursing care as needed. B. Medical management
1) Maintain adequate ventilation. 1. Diet restrictions
2) Decrease fluid intake as ordered. 2. Multivitamins
3) Administer diuretics, cardiac 3. Hematinics
glycosides, and antihypertensives as 4. Aluminum hydroxide gels

4
ordered; monitor effects. 5. Antihypertensives

ADULT NURSING 351


53155_04_Ch04b_p264-377.qxd 2/26/09 7:32 AM Page 352

C. Assessment findings 1. Transplantation of a kidney from a donor to


1. Nausea, vomiting; diarrhea or constipation; recipient to prolong the life of person with
decreased urinary output; dyspnea renal failure
2. Stomatitis, hypotension (early), hypertension 2. Sources of donor selection
(later), lethargy, convulsions, memory a. Living relative or potential with compatible
impairment, pericardial friction rub, HF serum and tissue studies, free from
3. Diagnostic tests: urinalysis systemic infection, and emotionally stable
a. Protein, sodium, and WBC elevated b. Cadavers with good serum and tissue
b. Specific gravity, platelets, and calcium crossmatching; free from renal disease,
decreased neoplasms, and sepsis; absence of
D. Nursing interventions ischemia/trauma
1. Prevent neurologic complications. B. Nursing interventions: preoperative
a. Assess every hour for signs of uremia 1. Provide routine pre-op care.
(fatigue, loss of appetite, decreased urine 2. Discuss the possibility of post-op
output, apathy, confusion, elevated blood dialysis/immunosuppressive drug therapy
pressure, edema of face and feet, itchy with client and significant others.
skin, restlessness, seizures). C. Nursing interventions: postoperative
b. Assess for changes in mental functioning. 1. Provide routine post-op care.
c. Orient confused client to time, place, date, 2. Monitor fluid and electrolyte balance
and persons; institute safety measures to carefully.
protect client from falling out of bed. a. Monitor I&O hourly and adjust IV fluid
d. Monitor serum electrolytes, BUN, and administration accordingly.
creatinine as ordered. b. Anticipate possible massive diuresis.
2. Promote optimal GI function. 3. Encourage frequent and early
a. Assess/provide care for stomatitis ambulation.
b. Monitor nausea, vomiting, anorexia; 4. Monitor vital signs, especially temperature;
administer antiemetics as ordered. report significant changes.
c. Assess for signs of GI bleeding. 5. Provide mouth care and nystatin (Mycostatin)
3. Monitor/prevent alteration in fluid and mouthwashes for candidiasis.
electrolyte balance. 6. Administer immunosuppressive agents as
4. Assess for hyperphosphatemia (paresthesias, ordered.
muscle cramps, seizures, abnormal reflexes), a. Cyclosporine (Sandimmune): does not
and administer aluminum hydroxide gels cause significant bone marrow
(Amphojel, AlternaGEL) as ordered. depression. Assess for hypertension;
5. Promote maintenance of skin integrity. blood chemistry alterations
a. Assess/provide care for pruritus. (hypermagnesemia, hyperkalemia,
b. Assess for uremic frost (urea decreased sodium bicarbonate);
crystallization on the skin) and bathe in neurologic functioning.
plain water. b. Azathioprine (Imuran): assess for
6. Monitor for bleeding complications, prevent manifestations of anemia, leukopenia,
injury to client. thrombocytopenia, oral lesions.
a. Monitor Hgb, HCT, platelets, RBC. c. Cyclophosphamide (Cytoxan): assess for
b. Hematest all secretions. alopecia, hypertension, kidney/liver
c. Administer hematinics as ordered. toxicity, leukopenia.
d. Avoid IM injections. d. Antilymphocytic globulin (ALG),
7. Promote/maintain maximal cardiovascular antithymocytic globulin (ATG): assess for
function. fever, chills, anaphylactic shock,
a. Monitor blood pressure and report hypertension, rash, headache.
significant changes. e. Corticosteroids (prednisone,
b. Auscultate for pericardial friction rub. methylprednisolone sodium succinate
c. Perform circulation checks routinely. [Solu-Medrol]): assess for peptic ulcer and
d. Administer diuretics as ordered and GI bleeding, sodium/water retention,
monitor output. muscle weakness, delayed healing, mood
e. Modify digitalis dose as ordered (digitalis alterations, hyperglycemia, acne.
is excreted in kidneys). 7. Assess for signs of rejection. Include decreased
8. Provide care for client receiving dialysis. urinary output, fever, pain/tenderness over
transplant site, edema, sudden weight gain,
Kidney Transplantation increasing blood pressure, generalized
malaise, rise in serum creatinine, and decrease

4
A. General information in creatinine clearance.

352 NCLEX-RN Review


53155_04_Ch04b_p264-377.qxd 2/26/09 7:32 AM Page 353

8. Provide client teaching and discharge g. Medication regimen and avoidance of OTC
planning concerning: drugs that may be nephrotoxic (except
a. Medication regimen: names, dosages, with physician approval)
frequency, and side effects
b. Signs and symptoms of rejection and the
need to report immediately
Disorders of the Male
c. Dietary restrictions: restricted sodium and Reproductive System
calories, increased protein
d. Daily weights
Epididymitis
e. Daily measurement of I&O A. General information
f. Resumption of activity and avoidance of 1. Inflammation of epididymis, one of the most
contact sports in which the transplanted common intrascrotal infections
kidney may be injured 2. May be sexually transmitted, usually caused
by N. gonorrhoeae, C. trachomatis; also caused
Nephrectomy by GU instrumentation, urinary reflux
A. General information B. Assessment findings
1. Surgical removal of an entire kidney 1. Sudden scrotal pain, scrotal edema, tenderness
2. Indications include renal tumor, massive over the spermatic cord
trauma, removal for a donor, polycystic 2. Diagnostic test: urine culture reveals specific
kidneys organism
B. Nursing interventions: preoperative care C. Nursing interventions
1. Provide routine pre-op care. 1. Administer antibiotics and analgesics as
2. Ensure adequate fluid intake. ordered.
3. Assess electrolyte values and correct any 2. Provide bed rest with elevation of the scrotum.
imbalances before surgery. 3. Apply ice packs to scrotal area to decrease edema.
4. Avoid nephrotoxic agents in any diagnostic tests.
5. Advise client to expect flank pain after surgery Prostatitis
if retroperitoneal approach (flank incision) is
A. General information
used.
1. Inflammatory condition that affects the
6. Explain that client will have chest tube if a
prostate gland
thoracic approach is used.
2. Several forms: acute bacterial prostatitis,
C. Nursing interventions: postoperative
chronic bacterial prostatitis, or abacterial
1. Provide routine post-op care.
chronic prostatitis
2. Assess urine output every hour; should be
3. Acute and chronic bacterial prostatitis usually
3050 mL/hour.
caused by E. coli, N. gonorrhoeae,
3. Observe urinary drainage on dressing and
Enterobacter or Proteus species, and group D
estimate amount.
streptococci
4. Weigh daily.
4. Most important predisposing factor: lower
5. Maintain adequate functioning of chest
UTIs
drainage system; ensure adequate oxygenation
B. Assessment findings
and prevent pulmonary complications.
1. Acute: fever, chills, dysuria, urethral
6. Administer analgesics as ordered.
discharge, prostatic tenderness, copious
7. Encourage early ambulation.
purulent urethral discharge upon palpation
8. Teach client to splint incision while turning,
2. Chronic: backache; perineal pain; mild
coughing, deep breathing.
dysuria; frequency; enlarged, firm, and slightly
9. Provide client teaching and discharge
tender prostate upon palpation
planning concerning:
3. Diagnostic tests
a. Prevention of urinary stasis
a. WBC elevated
b. Maintenance of acidic urine
b. Bacteria in initial urinalysis specimens
c. Avoidance of activities that might cause
C. Nursing interventions
trauma to the remaining kidney (contact
1. Administer antibiotics, analgesics, and stool
sports, horseback riding)
softeners as ordered.
d. No lifting heavy objects for at least
2. Provide increased fluid intake.
6 months
3. Provide sitz baths/rest to relieve discomfort.
e. Need to report unexplained weight gain,
4. Provide client teaching and discharge
decreased urine output, flank pain on
planning concerning:
unoperative side, hematuria
a. Importance of maintaining adequate
f. Need to notify physician if cold or other
hydration
infection present for more than 3 days

ADULT NURSING

4 353
53155_04_Ch04b_p264-377.qxd 2/26/09 7:32 AM Page 354

b. Antibiotic therapy regimen (may need to B. Medical management


remain on medication for several months) 1. Drug therapy: estrogens, chemotherapeutic
c. Activities that drain the prostate agents
(masturbation, sexual intercourse, prostatic 2. Radiation therapy
massage) 3. Surgery: radical prostatectomy
4. Watchful waiting: tumor is small and slow-
Benign Prostatic Hypertrophy (BPH) growing without symptoms; decide with
physician to monitor condition without surgery.
A. General information
C. Assessment findings: same as for BPH but
1. Mild to moderate glandular enlargement,
diagnostic test results are:
hyperplasia, and overgrowth of the smooth
1. Elevated acid phosphatase (distant metastasis)
muscles and connective tissue
and alkaline phosphatase (bone metastasis)
2. As the gland enlarges, it compresses the
2. Bone scan: abnormal in metastatic areas
urethra, resulting in urinary retention.
D. Nursing interventions
3. Most common problem of the male
1. Administer medications as ordered and
reproductive system; occurs in 50% of men
provide care for the client receiving
over age 50; 75% of men over age 75
chemotherapy.
4. Cause unknown; may be related to hormonal
2. Provide care for the client receiving radiation
mechanism
therapy.
B. Assessment findings
3. Provide care for the client with a
1. Nocturia, frequency, decreased force and
prostatectomy.
amount of urinary stream, hesitancy (more
difficult to start voiding), hematuria
2. Enlargement of prostate gland upon palpation Prostatic Surgery
by digital rectal exam A. General information
3. Diagnostic tests 1. Indicated for benign prostatic hypertrophy and
a. Urinalysis: alkalinity increased; specific prostatic cancer.
gravity normal or elevated 2. Types
b. BUN and creatinine elevated (if a. Transurethral resection (TUR or TURP):
longstanding BPH) insertion of a resectoscope into the urethra
c. Prostate-specific antigen (PSA) elevated to excise prostatic tissue; good for poor
(normal is 4 ng/mL) surgical risks, does not require an incision;
d. Cystoscopy reveals enlargement of gland most common type of surgery or BPH
and obstruction of urine flow b. Suprapubic prostatectomy: the prostate is
C. Nursing interventions approached by a low abdominal incision
1. Administer antibiotics as ordered. into the bladder to the anterior aspect of
2. Provide client teaching concerning the prostate; for large tumors obstructing
medications the urethra
a. Terazosin (Hytrin) relaxes bladder c. Retropubic prostatectomy: to remove a
sphincter and makes it easier to large mass high in the pelvic area; involves
urinate. May cause hypotension and a low midline incision below the bladder
dizziness. and into the prostatic capsule
b. Finasteride (Proscar) shrinks enlarged d. Perineal prostatectomy: often used for
prostate. prostatic cancer; the incision is made
3. Force fluids. through the perineum, which facilitates
4. Provide care for the catheterized client. radical surgery if a malignancy is found
5. Provide care for the client with prostatic surgery. e. da vinci Prostatectomy: removal of
prostate using robotic system with few
Cancer of the Prostate small incisions and great precision; more
A. General information benefits than traditional surgery (less pain,
1. Second most common cause of cancer deaths nerve damage, scarring, blood loss,
in American males over age 55 hospital stay).
2. Usually an adenocarcinoma; growth related to B. Nursing interventions: preoperative
the presence of androgens 1. Provide routine pre-op care.
3. Spreads from the prostate to the seminal 2. Institute and maintain urinary drainage.
vesicles, urethral mucosa, bladder wall, 3. Force fluids; administer antibiotics, acid-ash
external sphincter, and lymphatic system diet to eradicate UTI.
4. Highest incidence is in African-American men 4. Reinforce what surgeon has told
age 60 or over client/significant others regarding effects of

4
5. Cause is unknown surgery on sexual function.

354 NCLEX-RN Review


53155_04_Ch04b_p264-377.qxd 2/26/09 7:32 AM Page 355

C. Nursing interventions: postoperative


1. Provide routine post-op care. Sample Questions
2. Ensure patency of 3-way Foley.
3. Monitor continuous bladder irrigations with
sterile saline solution (removes clotted blood 340. A client is hospitalized for bladder cancer. He
from bladder), and control rate to keep urine is scheduled for ileal loop surgery to create a
light pink changing to clear. urostomy. Which information is most important
4. Expect hematuria for 23 days. for the nurse to include in a teaching plan for
5. Irrigate catheter with normal saline as ordered. this client when learning to change his urostomy
6. Control/treat bladder spasms; encourage appliance?
short, frequent walks; decrease rate of 1. Change the appliance before going to bed.
continuous bladder irrigations (if urine is not
red and is without clots); administer 2. Cut the wafer one inch larger than the stoma.
anticholinergics (propantheline bromide [Pro- 3. Cleanse the peristomal skin with mild soap
Banthine]) or antispasmodics (B&O and water.
suppositories) as ordered. 4. Use firm pressure to attach the wafer to the
7. Prevent hemorrhage: administer stool softeners skin.
to discourage straining at stool; avoid rectal
temperatures and enemas; monitor Hgb and 341. Which nursing intervention best prevents
HCT. urinary tract infections in a person who has an
8. Report bright red, thick blood in the catheter; ileal conduit?
persistent clots, persistent drainage on 1. Allowing the bag to fill completely.
dressings. 2. Attaching a larger bag at night.
9. Provide for bladder retraining after Foley
3. Restricting fluids to less than 1000 mL daily.
removal.
a. Instruct client to perform perineal 4. Changing the appliance every 8 hours.
exercises (stopping and starting stream
342. An elderly client has just returned to the nursing
during voiding; pressing buttocks together
unit after a transurethral resection. He has a
then relaxing muscles) to improve
sphincter control. 3-way indwelling urinary catheter for continuous
b. Limit liquid intake in evening. bladder irrigation connected to straight drainage.
c. Restrict caffeine-containing beverages. Immediately after surgery, what color of urine
d. Withhold anticholinergics and does the nurse expect in the catheter bag?
antispasmodics (these drugs relax bladder 1. Clear.
and increase chance of incontinence) if 2. Light yellow.
permitted. 3. Pink or dark red.
10. Provide client teaching and discharge
4. Bright red.
planning concerning:
a. Client and/or family education on going 343. An elderly man has just returned to the nursing
home with a smaller leg-bag catheter until care unit following a transurethral resection. He
physician office visit. has a three-way indwelling catheter with
b. Continued increased fluid intake continuous bladder irrigation. He tells the nurse
c. Signs of UTI and need to report he has to void. What is the most appropriate
them nursing action?
d. Continued perineal exercises
1. Allow him to void around the catheter.
e. Avoidance of heavy lifting, straining
during defecation, and prolonged travel 2. Irrigate the catheter.
(at least 812 weeks) 3. Notify the physician.
f. Measures that promote urinary continence 4. Remove the catheter.
g. Possible impotence (more common after
perineal resection) 344. A young adult is admitted to the hospital with a
1) Discuss ways of expressing sexuality diagnosis of acute renal failure. She is oliguric
(massage, cuddling) and has proteinuria. She asks the nurse, How
2) Suggest alternative methods of long will it be until I start to make urine again?
sexual gratification and use of 1. 12 days.
assistive aids 2. 37 days.
3) Discuss possibility of penile prosthesis
3. 12 weeks.
with physician

4
h. Need for annual and self-exams 4. 34 weeks.

ADULT NURSING 355


53155_04_Ch04b_p264-377.qxd 2/26/09 7:32 AM Page 356

345. A client who is in acute renal failure develops the toilet. Stop, and finally continue to void
pulmonary edema. Nursing interventions for this into the sterile container.
person should include which of the following? 4. Retract the foreskin, clean with soap and
Check all that apply. water, and then start to void. Stop, and finally
____ Administering oxygen. continue to void into the sterile container.
____ Encouraging coughing and deep breathing.
350. The nurse is to collect a urine culture specimen
____ Placing the client in a semi-sitting position.
from a catheterized client. Which one of the
____ Replacing lost fluids. following statements describes the nurses
actions for this procedure?
346. A client is admitted to the floor from PACU with
continuous bladder irrigation after undergoing a 1. With a sterile syringe the nurse aspirates
TURP. The nurse knows to increase the rate of 50 mL of urine from the silicone catheter
flow of the irrigation if what condition of the tubing.
urine return is present? 2. With a sterile syringe, the nurse aspirates
1. There is no return. 13 mL from the sampling port of the
catheter after first cleaning with alcohol.
2. The return is slow.
3. With a sterile syringe, the nurse aspirates
3. The return is yellow and cloudy.
13 mL from the distal end of the catheter
4. The return becomes brighter red. after first cleaning the sampling port with
soap and water.
347. A young man is admitted in chronic renal failure
and placed on hemodialysis three times a week. 4. The nurse disconnects the catheter from the
Which is an attainable short-term goal for this tubing and allows a small volume of urine to
person when he is placed on hemodialysis? drain into a sterile container.
1. Understanding the treatment and its 351. The nurse is ordered to perform a urinary
implications. catheterization for post-void residual volume
2. Independence in the care of the AV shunt. (PVR) on a client with urinary incontinence.
3. Self-monitoring during dialysis. Several minutes after the client voids, the nurse
4. Recording dialysate composition and obtains a residual urine of 30 mL. How does the
temperature. nurse interpret this result?
1. Adequate bladder emptying.
348. The nurse is caring for a client who is on 2. Inadequate bladder emptying.
hemodialysis and has an arteriovenous fistula.
3. Decreased urethral pressure.
Which finding is expected when assessing the
fistula? 4. Increased urethral pressure.
1. Ecchymotic area. 352. Post cystoscopy, which one of the following
2. Enlarged veins. assessment findings would the nurse expect to
3. Pulselessness. find?
4. Redness. 1. Gross hematuria and pain.
2. Pink-tinged urine and burning on voiding.
349. A female client is to have a urine culture
3. Colicky pain and bladder distention.
collected. What are the correct instructions the
nurse will give the client for collecting a clean 4. Flank pain and bladder distention.
catch urine specimen?
353. A post-op client is unable to void and is ordered
1. Separate the labia, clean from front to back to have an indwelling catheter inserted
with the three wipes impregnated with the immediately. What is the nurses greatest
cleaning solution, and then start to void in concern?
the toilet. Stop, and finally continue to void
1. Teaching the client deep breathing
into the sterile container.
techniques to decrease post-op pain,
2. Retract the foreskin, cleanse with the three preprocedure.
cleansing sponges, and start to void. Stop,
2. Maintaining strict aseptic technique.
and finally continue to void into the sterile
container. 3. Medicating the client for pain, before the
procedure.
3. Separate the labia, clean from back to front
with the three wipes impregnated with the 4. Teaching the client the signs and symptoms

4
cleaning solution, and then start to void in of urinary tract infection.

356 NCLEX-RN Review


53155_04_Ch04b_p264-377.qxd 2/26/09 7:32 AM Page 357

354. The nurse is assessing a client with an exchange by draining the dialysate and notices
indwelling catheter and finds the catheter is not the dialysate is cloudy. What is the nurses
draining and the clients bladder is distended. interpretation of this finding?
What is the nurses first best action? 1. The normal appearance of draining dialysate.
1. Notify the physician. 2. A sign of infection.
2. Assess the catheter tubing for kinks and 3. An indication of an impending lower back
position so downhill flow is initiated. problem.
3. Change the catheter. 4. A sign of a vascular access occlusion.
4. Aspirate urine for culture.
359. A client is on continuous ambulatory peritoneal
355. The nurse is teaching a client about the concept dialysis (CAPD). Which statement by the client
of dialysis and how it works for the body. What demonstrates understanding of the treatment?
statement best describes the nurses 1. I must increase my carbohydrate intake daily.
understanding of dialysis? 2. I must maintain a positive nitrogen balance
1. It will move blood through a semipermeable by decreasing proteins.
membrane into a dialysate that is used to 3. I must take prophylactic antibiotics to
remove waste products as well as correct prevent infection.
fluid and electrolyte imbalances.
4. I must be aware of the signs and symptoms
2. It will add electrolytes and water to the blood of peritonitis.
when passing through a semipermeable
membrane to correct electrolyte imbalances. 360. A woman presents to the urgent care center with
3. It will increase potassium to the blood when dysuria and hematuria. The woman reveals that
passing through a semipermeable membrane she has a history of cystitis. The nurse should
to correct electrolyte imbalances. also assess for which of the following clinical
4. It allows the nurse to choose to use either manifestations suggesting cystitis?
diffusion osmosis or ultrafiltration to correct 1. Frequency and urgency of urination, flank
the clients fluid and electrolyte imbalance. pain, nausea, and vomiting.
2. Abscess formation and flank pain.
356. A client with end-stage renal disease (ESRD) 3. Frequency and urgency of urination,
receives hemodialysis three times a week. Which suprapubic pain, and foul smelling urine.
statement demonstrates that dialysis is effective?
4. Fever, nausea, vomiting, and flank pain.
1. The client does not have a large weight gain.
2. The client has no signs and symptoms of 361. A 3-day post-op client for a ureterosigmoidostomy
infection. is complaining of cramping in lower extremities
3. The client expresses he or she can catch up and occasional dizziness. What intervention
on rest while on dialysis. should be given the highest priority?
4. The client is able to return to employment. 1. Assessing for electrolyte imbalance.
2. Assessing for cardiac dysrhythmias.
357. The nurse is caring for a client going to 3. Observing the clients response to surgery.
hemodialysis three times a week. The client
4. Verifying the temperature of the clients
receives the following medications every morning:
lower extremities.
hydrochlorothiazide (Hydrodiuril), nitroglycerin
patch (Minitran), vancomycin, and allopurinol 362. What teaching by the nurse should be given to
(Zyloprim). The nurse expects to withhold which the client with a Kocks pouch?
of the above medications until after hemodialysis? 1. Decreasing the clients sexual encounters.
1. Hydrochlorothiazide (Hydrodiuril) and 2. Adhering to catheterization schedules.
vancomycin.
3. Decreasing food intake to avoid embarrassing
2. Hydrochlorothiazide (Hydrodiuril) and situations.
nitroglycerin patch (Minitran).
4. Decreasing fluid intake to manage the urinary
3. Nitroglycerin (Minitran) and allopurinol diversion.
(Zyloprim).
4. Vancomycin and allopurinol (Zyloprim). 363. A 35-year-old male presents to the ER with
hematuria, flank pain, fever, nausea, and vomiting.
358. The nurse is caring for a client receiving He is admitted and passes a stone. The stone is

4
peritoneal dialysis. The nurse is completing the sent to the laboratory and is found to be composed

ADULT NURSING 357


53155_04_Ch04b_p264-377.qxd 2/26/09 7:32 AM Page 358

of uric acid. The client is placed on allopurinol recognizes this finding as uremic frost and takes
(Zyloprim). What is the action for this medication? which of the following nursing actions?
1. Decrease the clients serum creatinine. 1. Administers an antihistamine because the
2. Reduce the urinary concentration of uric acid. doctor would prescribe one to relieve itching.
3. Acidify the urine. 2. Increases fluids to prevent crystal formation
4. Bind oxalate in the gastrointestinal tract. and decrease itching.
3. Provides skin care with tepid water and
364. The nurse is caring for a client who has just been applies lotion on the skin to relieve itching.
given discharge instruction for kidney stones. 4. Permits the client to soak in a bathtub to
Which statement by the client indicates a need remove crystals.
for further instruction?
1. I will decrease my intake of all foods on the 369. The nurse has been working with a client with
list you gave me that are high in purine, chronic renal failure. Which of the following
calcium, or oxalate. behaviors would indicate to the nurse that the
2. I will decrease my fluid intake. client understands his dietary regimen?
3. I will take my medication daily. 1. He reports eating two bananas for breakfast,
rice and beans for lunch, and fruit salad, green
4. I will return to my doctor in one week for
beans, and an 8 oz. T-bone steak for dinner.
follow-up.
2. He reports eating bacon and eggs for breakfast,
365. Medication will be used in the management of a hot dogs and sauerkraut for lunch, and baked
client with urolithiasis. Based on knowledge of canned ham with green beans for dinner.
urolithiasis, the nurse should include which of the 3. He reports eating an apple and oatmeal for
following in planning nursing care for the client? breakfast, homemade tomato soup for lunch,
1. Place the client in bed with upper rails up, and pasta with fish for dinner.
call bell within reach, and instructions to call 4. He reports eating half a honeydew melon and
to get out of bed. three eggs for breakfast, a baked potato with
2. Keep the client NPO so there will be no processed cheese spread and broccoli for
experience of nausea with medication lunch, and chicken, yams, pinto beans,
administration. squash, and 8 oz. of pecans for dinner.
3. Increase intake of purine-, calcium-, and
370. A client who has been in intensive care, for
oxalate-rich food.
cardiogenic shock related to a myocardial
4. Add Probenecid to the narcotic to prevent infarction, is recovering. He is transferred to the
renal tubular excretion of the narcotic. renal unit in renal failure. The clients spouse
asks the nurse Is this acute or chronic renal
366. The nurse is performing discharge teaching for a
failure? What is the nurses best response?
client who was admitted with pyelonephritis. The
client asks the nurse, What is pyelonephritis? 1. Dont worry; this is an excellent renal unit, so
What is the nurses best response? we can treat either acute or chronic failure.
1. Pyelonephritis is an inflammation of the 2. Acute renal failure always progresses to
bladder. chronic renal failure.
2. Pyelonephritis is a rupture of the bladder. 3. Acute renal failure is glomerular
degeneration whereas chronic renal failure is
3. Pyelonephritis is an infection of the kidney.
the result of cardiovascular collapse.
4. Pyelonephritis is an infection of the lower
4. Acute renal failure generally results from
urinary tract.
decreased blood to the kidneys, nephrotoxicity,
367. On a medical-surgical unit, a client is admitted or muscle injury. The myocardial infarction
with acute renal failure. What problem must the caused extensive heart muscle damage
nurse assess for continually? decreasing blood to the kidneys.
1. Hyponatremia and hyperkalemia. 371. What should be assessed immediately post
2. Decreased BUN and creatinine. kidney transplant?
3. Alkalosis. 1. Fluid and electrolyte imbalances.
4. Hypercalcemia. 2. Infection.
3. Hepatotoxicity.
368. The client with chronic renal failure complains

4
of irritating white crystals on his skin. The nurse 4. Cardiomegaly.

358 NCLEX-RN Review


53155_04_Ch04b_p264-377.qxd 2/26/09 7:32 AM Page 359

372. An adult had a renal transplant, as a result of 377. A 68-year-old client, 48 hours post-transurethral
glomerulonephritis, and is at the physicians resection prostatectomy asks How will my sex
office for a follow-up visit. The client tells the life be affected? The nurses best response
office nurse I am not worried about rejection. would be,
I am not going to come here weekly. What 1. I will get the physician to determine if your
defense mechanism is the client expressing? sex life was affected during surgery.
1. Projection. 2. Only your doctor can answer that. Why
2. Intellectualization. dont you ask him prior to discharge.
3. Denial. 3. A transurethral prostatectomy does not
4. Regression. usually result in erectile dysfunction.
4. Dont you remember, before surgery you
373. The nurse will complete which one of the were told that you would not be able to
following initial assessments on the client engage in sexual intercourse but you can
immediately post-op nephrectomy? express your love for your spouse by
1. Performing cardiovascular assessment. alternate acts such as cuddling.
2. Ordering laboratory studies monitoring renal
functions and electrolytes. 378. Following a prostatectomy, the client has a
3-way, indwelling catheter for continuous
3. Inspecting the incision site for bleeding.
bladder irrigation. During evening shift, 2400 mL
4. Obtaining a urine culture. of irrigant was instilled. At the end of the shift,
the drainage bag was drained of 2900 mL of
374. The nurse is completing an admission
fluid. What is the total urine output for the shift?
assessment on a client with benign prostatic
hyperplasia (BPH). What in-depth assessment 1. 5300 mL.
should the nurse obtain? 2. 2900 mL.
1. Laboratory studies. 3. 240 mL.
2. Urinary patterns. 4. 500 mL.
3. Electrocardiograms.
4. Internal bleeding.

375. A client with BPH is at the clinic for follow-up.


Answers and Rationales
Which of the following statements indicates to
the nurse his understanding of management of 340. 3. Cleansing the peristomal skin is critical to
his condition? maintenance of skin integrity.
1. As soon as I finish this visit I wont ever
have to worry about BPH again. 341. 2. Attaching a larger bag at night helps to
2. I dont know how I am going to get used prevent reflux of urine into the stoma during a
to voiding every 2 to 3 hours. period when the bag is emptied less frequently.
3. I will wear an athletic supporter while 342. 3. The urine is expected to be pink or dark
I am awake. red for up to 36 hours after a transurethral
4. I am going to avoid fluids while at work to resection.
prevent dribbling.
343. 2. Blood clots obstructing the catheter can
376. A client who is 8 hours post-transurethral produce the sensation of needing to void.
resection prostatectomy (TURP) asks the Irrigating the catheter will remove any blood
nurse Why are there blood clots in my clots, allowing the urine to drain freely.
bag? How does the nurse interpret this
occurrence? 344. 3. The oliguric period in acute renal failure is
1. After all surgery bleeding is normal. usually 12 weeks.
2. It is common for blood clots to be irrigated 345. Administered oxygen should be checked.
from the bladder for a day or so. Oxygenation is seriously compromised in
3. The physician needs to be called as the client pulmonary edema.
is hemorrhaging. Encouraging coughing and deep breathing
4. The client is tugging on the catheter causing should be checked. Coughing and deep
irritation to the bladder mucosa. breathing may help with oxygenation.

ADULT NURSING

4 359
53155_04_Ch04b_p264-377.qxd 2/26/09 7:32 AM Page 360

Placing the client in a semi-sitting position be performed when all other means fail as they
should be checked. This position facilitates are associated with a high potential for infection.
breathing.
355. 1. Dialysis allows substances to move from the
346. 4. If return becomes brighter red, the solution blood through a semipermeable membrane into a
rate should be increased to flush the irrigation dialysis solution (dialysate) to correct fluid and
tube of clots. electrolyte imbalances as well as remove waste
products that accumulate when the client is in
347. 1. Prior to the start of dialysis the client should renal failure. The principles of dialysis include
fully comprehend its meaning and the changes diffusion, osmosis, and ultrafiltration.
in lifestyle required.
356. 4. By the client returning to employment, it
348. 2. Leaking of arterial blood into an AV fistula helps to maintain a positive self-image and to
causes the veins to enlarge so they are easier to continue to be a productive member of society. It
access for hemodialysis. will be an ongoing assessment of the clients
fluid status and sign/symptoms of infection.
349. 1. Women should separate the labia, clean from
front to back, and then proceed to void into the 357. 2. The morning of dialysis antihypertensives,
toilet. Stop, and finally continue to void into the nitrates, and sedatives are usually withheld as
sterile container. they may precipitate hypotensive episodes.
350. 2. Several milliliters of urine for culture can be 358. 2. Peritonitis is usually caused by
aspirated with a 21-gauge needle and 3-mL Staphylococcus. The first indication of
syringe after the sampling port or the distal peritonitis is cloudy dialysate.
catheter has been swabbed with alcohol or
iodine swabs. The urinary catheter and drainage 359. 4. Peritonitis is a life-threatening complication
system should remain a closed system to prevent of CAPD, which is manifested by abdominal
infection. pain and distention, diarrhea, vomiting, and
fever. Clients are given antibiotics orally or
351. 1. Measurement of post-void residual volume parenterally as necessary, not prophylactically.
(PVR) should be performed for all clients with
urinary incontinence. Catheterization is 360. 3. The signs and symptoms of cystitis are
performed several minutes after the client voids. frequency and urgency of urination, suprapubic
A residual of less than 50 mL signifies adequate pain, dysuria, foul-smelling urination, and
bladder emptying. sometimes pyuria. Some clients with cystitis
may be asymptomatic.
352. 2. Pink-tinged urine and burning on voiding for
a day or two following the procedure are 361. 1. In this surgical procedure, the clients ureters
expected. are anastomosed to the sigmoid colon. This
results in the client having drainage from the
353. 2. Strict aseptic technique is vital to prevent rectum, which often leads to acidosis and
urinary tract infection. The client is positioned electrolyte imbalance involving potassium,
on the back with heels flat on the bed with legs chloride, and magnesium.
separated. The meatus is cleansed with an
iodine solution. The catheter is lubricated with a 362. 2. The client with a Kocks pouch should
water-soluble jelly and is inserted through the be taught about living with a stoma, how to
urethra into the bladder until urine starts to self-catheterize and irrigate the appliance,
flow. The balloon is inflated and the catheter is increasing fluid intake to dilute urine to prevent
taped securely to the leg. irritation of the stoma, and lastly, stoma care.
The client will need to self-catheterize at regular
354. 2. Possible signs of indwelling catheter intervals.
obstruction can be pain, distention, and no
urinary output. Possible causes of obstruction 363. 2. Allopurinol (Zyloprim) reduces the urinary
include blood clots, mineral sediment, or concentration of uric acid to decrease the
mucous plugs in the catheter or tubing. The recurrence of uric acid stones.
most effective strategies to promote drainage are
to place the tubing so downhill flow is 364. 2. A high fluid intake of at least 3000 mL/day is
unobstructed and to empty the collection system needed to remove minerals prior to
precipitation.

4
regularly. Irrigation and catheter changes should

360 NCLEX-RN Review


53155_04_Ch04b_p264-377.qxd 2/26/09 7:32 AM Page 361

365. 1. Nursing care priorities for the client with 371. 1. The immediate assessments to be performed
urolithiasis include pain relief and prevention for a kidney recipient are fluid and electrolyte
of urinary tract obstruction and recurrence status, intake and output, and blood pressure.
of stones. The nurse can expect to administer
narcotics and maintain client safety. 372. 3. Denial is disowning intolerable thoughts. The
client is denying feelings of anxiety and the
366. 3. Pyelonephritis is an inflammation or infection seriousness of potential rejection of the organ.
of the kidney or kidney pelvis.
373. 3. The renal system is highly vascular; the client
367. 1. The most common findings in acute renal is at risk for post-op bleeding.
failure include elevations in BUN and
374. 2. Benign prostatic hyperplasia (BPH) is the
creatinine, metabolic acidosis, hyponatremia,
growth of new cells in the prostate gland, resulting
hyperkalemia, hypocalcemia, and
in urinary obstruction; therefore, assessment of the
hypophosphatemia.
obstructive symptoms are: decrease in the force of
368. 3. Skin care should be provided for the client the urinary stream; hesitancy in initiation of urine;
by bathing with tepid water and oils to reduce dribbling; urinary retention; incomplete bladder
dryness and itching. emptying; nocturia; dysuria; and urgency.

369. 3. A client with chronic renal failure needs to 375. 2. Clients with BPH should void every 2 to 3
adhere to a low-protein, low-sodium, and low- hours to flush the urinary tract.
potassium diet. This meal plan would fall into
376. 2. Blood clots are normal after a prostatectomy
these restrictions.
for the first 36 hours. Large quantities of bright
370. 4. A myocardial infarction causes decreased red blood may indicate hemorrhage.
cardiac output, which may cause acute renal
377. 3. Prior to surgery, the client should be informed
failure. The other mechanisms responsible for
that his sexual functioning will not be hampered
acute renal failure are nephrotoxicity, trauma,
other than retrograde ejaculation, which is not
burns, sepsis, and mismatched blood. Chronic
physically harmful.
renal failure results from irreversible damage
to the nephrons and glomeruli. Diseases 378. 4. Urine output is calculated by subtracting the
commonly responsible for chronic renal failure amount of irrigant instilled from the total fluid
are diabetes, hypertension, and kidney removed from the drainage bag (2900 mL
infections. drainage 2400 mL irrigant 5 500 mL urine).

The Musculoskeletal System

OVERVIEW OF ANATOMY 3. Protect vital organs and soft tissues


4. Manufacture RBCs in the red bone marrow
AND PHYSIOLOGY (hematopoiesis)
5. Provide site for storage of calcium and
The musculoskeletal system consists of the bones, phosphorus
muscles, joints, cartilage, tendons, ligaments, and B. Types of bones
bursae. Its major function is to provide a structural 1. Long: central shaft (diaphysis) made of compact
framework for the body and to provide a means for bone and two ends (epiphyses) composed of
movement. cancellous bone (e.g., femur and humerus)
2. Short: cancellous bone covered by thin layer of
Bones compact bone (e.g., carpals and tarsals)
3. Flat: two layers of compact bones separated
A. Functions by a layer of cancellous bone (e.g., skull and
1. Provide support to skeletal framework ribs)
2. Assist in movement by acting as levers for 4. Irregular: sizes and shapes vary (e.g., vertebrae
muscles and mandible)

ADULT NURSING

4 361
53155_04_Ch04b_p264-377.qxd 2/26/09 7:32 AM Page 362

Joints 2. Bones and joints: symptoms may include


stiffness, swelling, pain, redness, heat,
A. Articulation of bones occurs at joints; movable limitation of movement
joints provide stabilization and permit a variety of B. Lifestyle: usual patterns of activity and exercise
movements (limitations in ADL, use of assistive devices such
B. Classification (according to degree of movement) as canes or walkers), nutrition (obesity) and diet,
1. Synarthroses: immovable joints occupation (sedentary, heavy lifting, or pushing)
2. Amphiarthroses: partially movable joints C. Use of medications: drugs taken for
3. Diarthroses (synovial): freely movable joints musculoskeletal problems
a. Have a joint cavity (synovial cavity) D. Past medical history: congenital defects, trauma,
between the articulating bone surfaces inflammations, fractures, back pain
b. Articular cartilage covers the ends of the E. Family history: arthritis, gout
bones
c. A fibrous capsule encloses the joint
d. Capsule is lined with synovial membrane Physical Examination
that secretes synovial fluid to lubricate the A. Inspect for overall body build, posture, and gait.
joint and reduce friction B. Inspect and palpate joints for swelling, deformity,
masses, movement, tenderness, crepitations.
Muscles C. Inspect and palpate muscles for size, symmetry,
tone, strength.
A. Functions
1. Provide shape to the body
2. Protect the bones Laboratory/Diagnostic Tests
3. Maintain posture A. Hematologic studies
4. Cause movement of body parts by contraction 1. Muscle enzymes: CPK, aldolase, SGOT (AST)
B. Types of muscles 2. Erythrocyte sedimentation rate (ESR)
1. Cardiac: involuntary; found only in heart 3. Rheumatoid factor
2. Smooth: involuntary; found in walls of hollow 4. Complement fixation
structures (e.g., intestines) 5. Lupus erythematosus cells (LE prep)
3. Striated (skeletal): voluntary 6. Antinuclear antibodies (ANA)
C. Characteristics of skeletal muscles 7. Anti-DNA
1. Muscles are attached to the skeleton at the point 8. C-reactive protein
of origin and to bones at the point of insertion. 9. Uric acid
2. Have properties of contraction and extension, as B. X-rays: detect injury to or tumors of bone or soft
well as elasticity, to permit isotonic (shortening tissue
and thickening of the muscle) and isometric C. Bone scan
(increased muscle tension) movement. 1. Measures radioactivity in bones 2 hours after
3. Contraction is innervated by nerve stimulation. IV injection of a radioisotope; detects bone
tumors, osteomyelitis.
Cartilage 2. Nursing care
a. Have client void immediately before the
A. A form of connective tissue procedure.
B. Major functions are to cushion bony prominences b. Explain that client must remain still during
and offer protection where resiliency is required the scan itself.
D. Arthroscopy
Tendons and Ligaments 1. Insertion of fiber-optic endoscope
(arthroscope) into a joint to visualize it,
A. Composed of dense, fibrous connective tissue perform biopsies, or remove loose bodies from
B. Functions the joint
1. Ligaments attach bone to bone 2. Performed in OR using aseptic technique
2. Tendons attach muscle to bone 3. Nursing care
a. Maintain pressure dressing for 24 hours.
b. Advise client to limit activity for several
ASSESSMENT days.
E. Arthrocentesis: insertion of a needle into the joint
Health History to aspirate synovial fluid for diagnostic purposes
or to remove excess fluid
A. Presenting problem F. Myelography
1. Muscles: symptoms may include pain, 1. Lumbar puncture used to withdraw a small

4
cramping, weakness amount of CSF, which is replaced with a

362 NCLEX-RN Review


53155_04_Ch04b_p264-377.qxd 2/26/09 7:32 AM Page 363

radiopaque dye; used to detect tumors or Interventions


herniated intravertebral discs
2. Nursing care: pretest Preventing Complications of Immobility
a. Keep NPO after liquid breakfast.
b. Check for iodine allergy. Range-of-Motion (ROM) Exercises
c. Confirm that consent form has been A. Movement of joint through its full ROM to prevent
signed and explain procedure to contractures and increase or maintain muscle
client. tone/strength
3. Nursing care: posttest (see also Lumbar B. Types
Puncture) 1. Active: carried out by client; increases and
a. If oil-based dye (e.g., iophendylate maintains muscle tone; maintains joint
[Pantopaque]) was used, keep client flat for mobility
12 hours. 2. Passive: carried out by nurse without
b. If water-based dye (e.g., metrizamide assistance from client; maintains joint mobility
[Amipaque]) was used only; body part not to be moved beyond its
1) Elevate head of bed 3045; to prevent existing ROM
upward displacement of dye, which 3. Active assistive: client moves body part as far
may cause meningeal irritation and as possible and nurse completes remainder of
possibly seizures. movement
2) Institute seizure precautions and do 4. Active resistive: contraction of muscles against
not administer any phenothiazine an opposing force; increases muscle size and
drugs to client, e.g., prochlorperazine strength
(Compazine).
G. Electromyography Isometric Exercises
1. Measures and records activity of contracting A. Active exercise through contraction/relaxation of
muscles in response to electrical stimulation; muscle; no joint movement; length of muscle does
helps differentiate muscle disease from motor not change.
neuron dysfunction B. Client increases tension in muscle for several
2. Nursing care: explain procedure to the client seconds and then relaxes.
and advise that some discomfort may occur C. Maintains muscle strength and size.
due to needle insertion
Assistive Devices for Walking
A. Cane
ANALYSIS 1. Types: single, straight-legged cane; tripod
cane; quad cane.
Nursing diagnoses for clients with disorders of the
2. Nursing care: teach client to hold cane in hand
musculoskeletal system may include:
opposite affected extremity and to advance
A. Risk for injury
cane at the same time the affected leg is moved
B. Risk for disuse syndrome
forward.
C. Impaired physical mobility
B. Walker
D. Bathing/hygiene self-care deficit
1. Mechanical device with four legs for
E. Dressing/grooming self-care deficit
support.
F. Toileting self-care deficit
2. Nursing care: teach client to hold upper bars of
G. Body-image disturbance
walker at each side, then to move the walker
H. Pain
forward and step into it.
C. Crutches: teaching the client proper use of
crutches is an important nursing responsibility.
PLANNING AND 1. Ensure proper length
IMPLEMENTATION a. When client assumes erect position the top
of crutch is 2 inches below the axilla, and
the tip of each crutch is 6 inches in front
Goals and to the side of the feet.
Client will: b. Clients elbows should be slightly flexed
A. Be free from injury. when hand is on hand grip.
B. Be free from complications of immobility. c. Weight should not be borne by the axillae.
C. Attain optimal level of mobility. 2. Crutch gaits
D. Perform self-care activities at optimal level. a. Four-point gait: used when weight bearing
E. Adapt to alterations in body image. is allowed on both extremities
1) Advance right crutch.

4
F. Achieve maximum comfort level.

ADULT NURSING 363


53155_04_Ch04b_p264-377.qxd 2/26/09 7:32 AM Page 364

2) Step forward with left foot. 3. Turn client every 2 hours to reduce pressure
3) Advance left crutch. and promote drying.
4) Step forward with right foot. 4. Do not cover the cast until it is dry (may use
b. Two-point gait: typical walking pattern, an fan to facilitate drying).
acceleration of four-point gait 5. Do not use heat lamp or hair dryer on plaster
1) Step forward moving both right crutch cast.
and left leg simultaneously. D. Assessment
2) Step forward moving both left crutch 1. Perform neurovascular checks to area distal to
and right leg simultaneously. cast.
c. Three-point gait: used when weight a. Report absent or diminished pulse,
bearing is permitted on one extremity cyanosis or blanching, coldness, lack of
only sensation, inability to move fingers or toes,
1) Advance both crutches and affected excessive swelling.
extremity several inches, maintaining b. Report complaints of burning, tingling, or
good balance. numbness.
2) Advance the unaffected leg to the 2. Note any odor from the cast that may indicate
crutches, supporting the weight of the infection.
body on the hands. 3. Note any bleeding on cast in a surgical client.
d. Swing-to gait: used for clients with 4. Check for hot spots that may indicate
paralysis of both lower extremities who are inflammation under cast.
unable to lift feet from floor 5. Compartment syndrome: report of pain due to
1) Both crutches are placed forward. inadequate space for tissue swelling; treatment
2) Client swings forward to the crutches. may include removing case and fasciotomy
e. Swing-through gait: same indications as for (surgical opening of the fascia); non-treatment
swing-to gait could result in permanent nerve damage and
1) Both crutches are placed forward. deformity.
2) Client swings body through the E. General care
crutches. 1. Instruct client to wiggle toes or fingers to
improve circulation.
Care of the Client with a Cast 2. Elevate affected extremity above heart level to
reduce swelling.
A. Types of casts: long arm, short arm, long leg, short 3. Apply ice bags to each side of the cast if ordered.
leg, walking cast with rubber heel, body cast, F. Provide client teaching and discharge planning
shoulder spica, hip spica concerning:
B. Casting materials 1. Isometric exercises when cleared with physician
1. Plaster of paristraditional cast 2. Reinforcement of instructions given on crutch
a. Takes 2472 hours to dry. walking
b. Precautions must be taken until cast is dry 3. Do not get cast wet; wrap cast in plastic bag
to prevent dents, which may cause when bathing or take sponge bath
pressure areas. 4. If a cast that has already dried and hardened
c. Signs of a dry cast: shiny white, hard, does become wet, may use blow-dryer on low
resistant. setting over wet spot; if large area of plaster
d. Must be kept dry because water can ruin a cast becomes wet, call physician
plaster cast. 5. Do not scratch or insert foreign bodies under
2. Synthetic casts, e.g., fiberglass cast; may direct cool air from blow-dryer
a. Strong, lightweight; sets in about 20 under cast for itching
minutes. 6. Recognize and report signs of impaired
b. Can be dried using cast dryer or hair blow- circulation or of infection
dryer on cool setting; some synthetic casts 7. Cast cleaning
need special lamp to harden. a. Clean surface soil on plaster cast with a
c. Water-resistant; however, if cast becomes slightly damp cloth; mild soap may be
wet, must be dried thoroughly to prevent used for synthetic cast
skin problems under cast. b. To brighten a plaster cast, apply white shoe
C. Cast dryingplaster cast polish sparingly
1. Use palms of hands, not fingertips, to support
cast when moving or lifting client. Care of the Client in Traction
2. Support cast on rubber- or plastic-protected
pillows with cloth pillowcase along length of A. A pulling force exerted on bones to reduce and/or
cast until dry. immobilize fractures, reduce muscle spasm,

4
correct or prevent deformities

364 NCLEX-RN Review


53155_04_Ch04b_p264-377.qxd 2/26/09 7:32 AM Page 365

B. Types 2) Generally used to stabilize fractures of


1. Skin traction: weights are attached to a the femoral shaft while client is
moleskin or adhesive strip secured by elastic awaiting surgery
bandage or other special device (e.g., foam 3) Elevating foot of bed slightly provides
rubber boot) used to cover the affected limb. countertraction
a. Bucks extension (See Figure 4-24). 4) Head of bed should remain flat
1) Exerts straight pull on affected extremity 5) Foot of bed usually elevated by
2) Generally used to temporarily shock blocks to provide
immobilize the leg in a client with a countertraction
fractured hip c. Cervical traction (See Figure 4-24).
3) Shock blocks at the foot of the bed 1) Cervical head halter attached to
produce countertraction and prevent weights that hang over head of bed
the client from sliding down in bed 2) Used for soft tissue damage or
b. Russell traction (See Figure 4-24). degenerative disc disease of cervical
1) Knee is suspended in a sling attached spine to reduce muscle spasm and
to a rope and pulley on a Balkan maintain alignment
frame, creating upward pull from the 3) Usually intermittent traction
knee; weights are attached to foot of 4) Elevate head of bed to provide
bed (as in Bucks extension) creating a countertraction
horizontal force exerted on the tibia
and fibula

C.

A.

D.

B.

Figure 4-24 Types of traction. (A) Bucks extension traction; (B) Russells traction; (C) Cervical traction; (D) Pelvic traction

ADULT NURSING

4 365
53155_04_Ch04b_p264-377.qxd 2/26/09 7:32 AM Page 366

d. Pelvic traction (See Figure 4-24). 8. Assist with ADL; provide overhead trapeze to
1) Pelvic girdle with extension straps facilitate moving, using bedpan, etc.
attached to ropes and weights 9. Prevent complications of immobility.
2) Used for low back pain to reduce 10. Encourage active ROM exercises to unaffected
muscle spasm and maintain alignment extremities.
3) Usually intermittent traction 11. Check carefully for orders about turning.
4) Client in semi-Fowlers position with a. Bucks extension: client may turn to
knee bent unaffected side (place pillows between legs
5) Secure pelvic girdle around iliac crests before turning).
2. Skeletal traction: traction applied directly to b. Russell traction and balanced suspension
the bones using pins, wires, or tongs (e.g., traction: client may turn slightly from side to
Crutchfield tongs) that are surgically inserted; side without turning body below the waist.
used for fractured femur, tibia, humerus, c. May need to make bed from head to foot.
cervical spine
3. Balanced suspension traction: produced by a
counterforce other than the clients weight; EVALUATION
extremity floats or balances in the traction
apparatus; client may change position without A. Client remains free from injury.
disturbing the line of traction B. Client is free from complications of immobility.
4. Thomas splint and Pearson attachment 1. Maintains clear, intact skin.
(usually used with skeletal traction in 2. Has regular bowel movements.
fractures of the femur) 3. Is free from urinary tract
a. Hip should be flexed at 20 infection/retention/calculi.
b. Use footplate to prevent foot drop 4. Has clear breath sounds; normal rate, rhythm,
C. Nursing care and depth of respiration.
1. Check traction apparatus frequently to ensure 5. Demonstrates adequate peripheral circulation.
that: 6. Maintains joint mobility and muscle tone.
a. Ropes are aligned and weights are hanging 7. Remains oriented to time, place, and person.
freely. 8. Is active in decision making regarding own care.
b. Bed is in proper position. C. Optimum level of mobility is attained.
c. Line of traction is within the long axis of D. Client attains independence in self-care activities;
the bone. uses assistive devices as necessary.
2. Maintain client in proper alignment. E. Client successfully adjusts to alterations in body
a. Align in center of bed. image; exhibits increased self-esteem.
b. Do not rest affected limb against foot of bed. F. Pain is relieved or is more manageable.
3. Perform neurovascular checks to affected
extremity. DISORDERS OF THE
4. Observe for and prevent foot drop.
a. Provide footplate. MUSCULOSKELETAL SYSTEM
b. Encourage plantarflexion and dorsiflexion
exercises. Rheumatoid Arthritis (RA)
5. Observe for and prevent deep venous
thrombosis (especially in Russell traction due A. General information
to pressure on popliteal space). 1. Chronic systemic disease characterized by
6. Observe for and prevent skin irritation and inflammatory changes in joints and related
breakdown (especially over bony prominences structures.
and traction application sites). 2. Occurs in women more often than men (3:1);
a. Russell traction: check popliteal area peak incidence between ages 3545.
frequently and pad the sling with felt 3. Cause unknown, but may be an autoimmune
covered by stockinette or ABDs. process; genetic factors may also play a role.
b. Thomas splint: pad top of splint with same 4. Predisposing factors include fatigue, cold,
material as in Russell traction. emotional stress, infection.
c. Cervical traction: pad chin area and protect 5. Joint distribution is symmetric (bilateral); most
ears. commonly affects smaller peripheral joints of
7. Provide pin care for clients in skeletal traction. hands and also commonly involves wrists,
a. Usually consists of cleansing and applying elbows, shoulders, knees, hips, ankles, and jaw.
antibiotic ointment, but individual agency 6. If unarrested, affected joints progress through
policies may vary. four stages of deterioration: synovitis, pannus
b. Observe for any redness, drainage, odor. formation, fibrous ankylosis, and bony

4
ankylosis.

366 NCLEX-RN Review


53155_04_Ch04b_p264-377.qxd 2/26/09 7:32 AM Page 367

B. Medical management 2. Promote maintenance of joint mobility and


1. Drug therapy muscle strength.
a. Aspirin: mainstay of treatment, has both a. Perform ROM exercises several times a
analgesic and anti-inflammatory effect. day; use of heat prior to exercise may
b. Nonsteroidal anti-inflammatory drugs decrease discomfort; stop exercise at the
(NSAIDs): ibuprofen (Motrin), indomethacin point of pain.
(Indocin), fenoprofen (Nalfon), mefenamic b. Use isometric or other exercise to
acid (Ponstel), phenylbutazone strengthen muscles.
(Butazolidin), piroxicam (Feldene), 3. Change position frequently; alternate sitting,
naproxen (Naprosyn), sulindac (Clinoril); standing, lying.
relieve pain and inflammation by inhibiting 4. Promote comfort and relief/control of pain.
the synthesis of prostaglandins a. Ensure balance between activity and rest.
c. Gold compounds (chrysotherapy) b. Provide 12 scheduled rest periods
1) Injectable form: sodium thiomalate throughout day.
(Myochrysine); aurothioglucose c. Rest and support inflamed joints; if splints
(Solganal); given IM once a week; take used, remove 12 times per day for gentle
36 months to become effective; side ROM exercises.
effects include proteinuria, mouth 5. Ensure bed rest if ordered for acute
ulcers, skin rash, aplastic anemia; exacerbations.
monitor blood studies and urinalysis a. Provide firm mattress.
frequently. b. Maintain proper body alignment.
2) Oral form: auranofin (Ridaura); smaller c. Have client lie prone for 12 hour twice a day.
doses are effective; take 36 months to d. Avoid pillows under knees.
become effective; diarrhea also a side e. Keep joints mainly in extension, not flexion.
effect with oral form; blood and urine f. Prevent complications of immobility.
studies should also be monitored. 6. Provide heat treatments (warm bath, shower,
d. Corticosteroids or whirlpool; warm, moist compresses;
1) Intra-articular injections temporarily paraffin dips) as ordered.
suppress inflammation in specific joints. a. May be more effective in chronic pain.
2) Systemic administration used only b. Reduce stiffness, pain, and muscle spasm.
when client does not respond to less 7. Provide cold treatments as ordered; most
potent anti-inflammatory drugs. effective during acute episodes.
e. Methotrexate, Cytoxan given to suppress 8. Provide psychologic support and encourage
immune response; side effects include client to express feelings.
bone marrow suppression 9. Assist client in setting realistic goals; focus on
2. Physical therapy to minimize joint deformities client strengths.
3. Surgery to remove severely damaged joints 10. Provide client teaching and discharge
(e.g., total hip replacement; knee replacement) planning concerning
C. Assessment findings a. Use of prescribed medications and side effects
1. Fatigue, anorexia, malaise, weight loss, slight b. Self-help devices to assist in ADL and to
elevation in temperature. increase independence
2. Joints are painful, warm, swollen, limited in c. Importance of maintaining a balance
motion, stiff in morning and after periods of between activity and rest
inactivity, and may show crippling deformity d. Energy conservation methods
in long-standing disease. e. Performance of ROM, isometric, and
3. Muscle weakness secondary to inactivity. prescribed exercises
4. History of remissions and exacerbations. f. Maintenance of well-balanced diet
5. Some clients have additional extra-articular g. Application of resting splints as ordered
manifestations: subcutaneous nodules; eye, h. Avoidance of undue physical or emotional
vascular, lung, or cardiac problems. stress
6. Diagnostic tests i. Importance of follow-up care
a. X-rays show various stages of joint disease
b. CBC: anemia is common
c. ESR elevated
Osteoarthritis
d. Rheumatoid factor positive A. General information
e. ANA may be positive 1. Chronic, nonsystemic disorder of joints
f. C-reactive protein elevated characterized by degeneration of articular
D. Nursing interventions cartilage
1. Assess joints for pain, swelling, tenderness, 2. Women and men affected equally; incidence

4
limitation of motion. increases with age

ADULT NURSING 367


53155_04_Ch04b_p264-377.qxd 2/26/09 7:32 AM Page 368

3. Cause unknown; most important factor in 2. Inflammation of the joints caused by


development is aging (wear and tear on joints); deposition of urate crystals in articular tissue
others include obesity, joint trauma 3. Occurs most often in males
4. Weight-bearing joints (spine, knees, hips) and 4. Familial tendency
terminal interphalangeal joints of fingers most B. Medical management
commonly affected 1. Drug therapy
B. Assessment findings a. Acute attack: Colchicine IV or PO
1. Pain (aggravated by use and relieved by rest) (discontinue if diarrhea occurs); NSAIDs
and stiffness of joints such as indomethacin (Indocin), naproxen
2. Heberdens nodes: bony overgrowths at (Naprosyn), phenylbutazone (Butazolidin)
terminal interphalangeal joints b. Prevention of attacks
3. Decreased ROM, possible crepitation (grating 1) Uricosuric agents (probenecid
sound when moving joint) [Benemid], sulfinpyrazone [Anturane])
4. Diagnostic tests increase renal excretion of uric acid
a. X-rays show joint deformity as disease 2) Allopurinal (Zyloprim) inhibits uric
progresses acid formation
b. ESR may be slightly elevated when disease 2. Low-purine diet may be recommended
is inflammatory 3. Joint rest and protection
C. Nursing interventions 4. Heat or cold therapy
1. Assess joints for pain and ROM. C. Assessment findings
2. Relieve strain and prevent further trauma to 1. Joint pain, redness, heat, swelling; joints of
joints. foot (especially great toe) and ankle most
a. Encourage rest periods throughout day. commonly affected (acute gouty arthritis stage)
b. Use cane or walker when indicated. 2. Headache, malaise, anorexia
c. Ensure proper posture and body 3. Tachycardia; fever; tophi in outer ear, hands,
mechanics. and feet (chronic tophaceous stage)
d. Promote weight reduction if obese. 4. Diagnostic test: uric acid elevated
e. Avoid excessive weight-bearing activities D. Nursing interventions
and continuous standing. 1. Assess joints for pain, motion, appearance.
3. Maintain joint mobility and muscle strength. 2. Provide bed rest and joint immobilization as
a. Provide ROM and isometric exercises. ordered.
b. Ensure proper body alignment. 3. Administer antigout medications as ordered.
c. Change clients position frequently. 4. Administer analgesics for pain as ordered.
4. Promote comfort/relief of pain. 5. Increase fluid intake to 20003000 mL/day to
a. Administer medications as ordered: aspirin prevent formation of renal calculi.
and NSAIDs most commonly used; intra- 6. Apply local heat or cold as ordered.
articular injections of corticosteroids relieve 7. Apply bed cradle to keep pressure of sheets off
pain and improve mobility; calcitonin joints.
(Miacalcin) PO or nasal spray to help 8. Provide client teaching and discharge
preservation of bone mass in lumbar spine planning concerning:
b. Apply heat as ordered (e.g., warm baths, a. Medications and their side effects
compresses, hot packs) or ice to reduce pain. b. Modifications for low-purine diet:
5. Prepare client for joint replacement surgery if avoidance of shellfish, liver, kidney,
necessary. brains, sweetbreads, sardines, anchovies
6. Provide client teaching and discharge c. Limitation of alcohol use
planning concerning: d. Increase in fluid intake
a. Use of prescribed medications and side e. Weight reduction if necessary
effects f. Importance of regular exercise
b. Importance of rest periods
c. Measures to relieve strain on joints
d. ROM and isometric exercises
Systemic Lupus Erythematosus (SLE)
e. Maintenance of a well-balanced diet A. General information
f. Use of heat/ice as ordered 1. Chronic connective tissue disease involving
multiple organ systems
2. Occurs most frequently in young women
Gout 3. Cause unknown; immune, genetic, and viral
A. General information factors have all been suggested
1. A disorder of purine metabolism; causes high 4. Pathophysiology
levels of uric acid in the blood and the a. A defect in bodys immunologic

4
precipitation of urate crystals in the joints mechanisms produces autoantibodies in

368 NCLEX-RN Review


53155_04_Ch04b_p264-377.qxd 2/26/09 7:32 AM Page 369

the serum directed against components of a. Disease process and relationship to


the clients own cell nuclei. symptoms
b. Affects cells throughout the body resulting b. Medication regimen and side effects
in involvement of many organs, including c. Importance of adequate rest
joints, skin, kidney, CNS, and d. Use of daily heat and exercises as
cardiopulmonary system. prescribed for arthritis
B. Medical management e. Need to avoid physical or emotional stress
1. Drug therapy f. Maintenance of a well-balanced diet
a. Aspirin and NSAIDs to relieve mild g. Need to avoid direct exposure to sunlight
symptoms such as fever and arthritis (wear hat and other protective clothing)
b. Corticosteroids to suppress the h. Need to avoid exposure to persons with
inflammatory response in acute infections
exacerbations or severe disease i. Importance of regular medical follow-up
c. Immunosuppressive agents such as j. Availability of community agencies
azathioprine (Imuran), cyclophosphamide
(Cytoxan) to suppress the immune
response when client unresponsive to
Osteomyelitis
more conservative therapy A. General information
2. Plasma exchange to provide temporary 1. Infection of the bone and surrounding soft
reduction in amount of circulating antibodies tissues, most commonly caused by S. aureus.
3. Supportive therapy as organ systems become 2. Infection may reach bone through open wound
involved (compound fracture or surgery), through the
C. Assessment findings bloodstream, or by direct extension from
1. Fatigue, fever, anorexia, weight loss, malaise, infected adjacent structures.
history of remissions and exacerbations 3. Infections can be acute or chronic; both cause
2. Joint pain, morning stiffness bone destruction.
3. Skin lesions B. Assessment findings
a. Erythematous rash on face, neck, or 1. Malaise, fever
extremities may occur 2. Pain and tenderness of bone, redness and
b. Butterfly rash over bridge of nose and cheeks swelling over bone, difficulty with weight
c. Photosensitivity with rash in areas exposed bearing; drainage from wound site may be
to sun present
4. Oral or nasopharyngeal ulcerations 3. Diagnostic tests
5. Alopecia a. CBC: WBC elevated
6. Renal system involvement (proteinuria, b. Blood cultures may be positive
hematuria, renal failure) c. ESR may be elevated
7. CNS involvement (peripheral neuritis, C. Nursing interventions
seizures, organic brain syndrome, psychosis) 1. Administer analgesics and antibiotics as
8. Cardiopulmonary system involvement ordered.
(pericarditis, pleurisy) 2. Use sterile technique during dressing changes.
9. Increase susceptibility to infection 3. Maintain proper body alignment and change
10. Diagnostic tests position frequently to prevent deformities.
a. ESR elevated 4. Provide immobilization of affected part as
b. CBC; anemia; WBC and platelet counts ordered.
decreased 5. Provide psychologic support and diversional
c. ANA positive activities (depression may result from
d. LE prep positive prolonged hospitalization).
e. Anti-DNA positive 6. Prepare client for surgery if indicated.
f. Chronic false-positive test for syphilis a. Incision and drainage of bone abscess
D. Nursing interventions b. Sequestrectomy: removal of dead, infected
1. Assess symptoms to determine systems bone and cartilage
involved. c. Bone grafting after repeated infections
2. Monitor vital signs, I&O, daily weights. d. Leg amputation
3. Administer medications as ordered. 7. Provide client teaching and discharge
4. Institute seizure precautions and safety planning concerning:
measures with CNS involvement. a. Use of prescribed oral antibiotic therapy
5. Provide psychologic support to and side effects
client/significant others. b. Importance of recognizing and reporting
6. Provide client teaching and discharge signs of complications (deformity, fracture)

4
planning concerning: or recurrence

ADULT NURSING 369


53155_04_Ch04b_p264-377.qxd 2/26/09 7:32 AM Page 370

Fractures c. Signs of complications and need to report


them
A. General information
1. A break in the continuity of bone, usually Open Reduction and Internal Fixation (ORIF)
caused by trauma
2. Pathologic fractures: spontaneous bone break, A. General information
found in certain diseases or conditions 1. Open reduction of fractures requires surgery to
(osteoporosis, osteomyelitis, multiple realign bones; may include internal fixation
myeloma, bone tumors) with pins, screws, wires, plates, rods, or nails
3. Types 2. Indications include:
a. Complete: separation of bone into two parts a. Compound fractures
1) Transverse b. Fractures accompanied by serious
2) Oblique neurovascular injuries
3) Spiral c. Fractures with widely separated fragments
b. Incomplete (partial): fracture does not go d. Comminuted fractures
all the way through the bone, only part of e. Fractures of the femur
the bone is broken. f. Fractures of joints
c. Comminuted: bone is broken or splintered B. Nursing interventions: preoperative
into pieces. 1. Provide routine pre-op care.
d. Closed or simple: bone is broken without 2. Provide meticulous skin preparation to
break in skin. prevent infection.
e. Open or compound: break in skin with or C. Nursing interventions: postoperative
without protrusion of bone. 1. Provide routine post-op care.
B. Medical management 2. Maintain affected limb in proper alignment.
1. Traction 3. Perform neurovascular checks to affected
2. Reduction extremity.
a. Closed reduction through manual manipu- 4. Observe for post-op infection.
lation followed by application of cast
b. Open reduction Fractured Hip
3. Application of a cast
C. Assessment findings A. General information
1. Pain, aggravated by motion; tenderness 1. Fracture of the head, neck (intracapsular
2. Loss of motion; edema, crepitus (grating fracture), or trochanteric area (extracapsular
sound), ecchymosis fracture) of the femur
3. Diagnostic test: X-ray reveals break in bone 2. Occurs most often in elderly women
D. Nursing interventions 3. Predisposing factors include osteoporosis and
1. Provide emergency care of fractures. degenerative changes of bone
2. Perform neurovascular checks on affected B. Medical management
extremity. 1. Bucks or Russell traction as temporary
3. Observe for signs of compartment syndrome measures to maintain alignment of affected
(swelling causes an increase within muscle limb and reduce the pain of muscle spasm
compartment which causes edema and more 2. Surgery
pressure; irreversible neuromuscular damage a. Open reduction and internal fixation with
can occur within 4 to 6 hours); signs include pins, nails, and/or plates
weak pulse, pallor followed by cyanosis, b. Hemiarthroplasty: insertion of prosthesis
paresthesias, and severe pain. (e.g., Austin-Moore) to replace head of
4. Observe for signs of fat emboli (respiratory femur
distress, mental disturbances, fever, petechiae) C. Assessment findings
especially in the client with multiple long- 1. Pain in affected limb
bone fractures. 2. Affected limb appears shorter, external
5. Encourage diet high in protein and vitamins to rotation
promote healing. 3. Diagnostic test: X-ray reveals hip fracture
6. Encourage fluids to prevent constipation, renal D. Nursing interventions
calculi, and UTIs. 1. Provide general care for the client with a
7. Provide care for the client in traction, with a fracture.
cast, or with open reduction. 2. Provide care for the client with Bucks or
8. Provide client teaching and discharge Russell traction.
planning concerning 3. Monitor for disorientation and confusion in
a. Cast care if indicated the elderly client; reorient frequently and

4
b. Crutch walking if necessary provide safety measures.

370 NCLEX-RN Review


53155_04_Ch04b_p264-377.qxd 2/26/09 7:32 AM Page 371

4. Perform neurovascular checks to affected d. Turn only to unoperative side if ordered; use
extremity. abductor splint or two pillows between
5. Prevent complications of immobility. knees while turning and when lying on side.
6. Encourage use of trapeze to facilitate movement. e. Assist client in getting out of bed when
7. Administer analgesics as ordered for pain. ordered.
8. In addition to routine post-op care for the client 1) PT usually ordered to get client out of
with open reduction and internal fixation: bed day of surgery or first day post-op,
a. Check dressings for bleeding, drainage, and every day thereafter.
infection: empty Hemovac and note output; 2) Avoid weight bearing until allowed.
keep compressed to facilitate drainage. 3) Avoid adduction and hip flexion; do
b. Assess clients LOC. not use low chair.
c. Reorient the confused client frequently. 3. Provide client teaching and discharge
d. Avoid oversedating the elderly client. planning concerning:
e. Turn client every 2 hours. a. Prevention of adduction of affected limb
f. Turn to unoperative side only. and hip flexion
g. Place two pillows between legs while 1) Do not cross legs.
turning and when lying on side. 2) Use raised toilet set.
h. Institute measures to prevent thrombus 3) Do not bend down to put on shoes or
formation. socks.
1) Apply elastic stockings. 4) Do not sit in low chairs.
2) Encourage plantarflexion and b. Signs of wound infection
dorsiflexion foot exercises. c. Exercise program as ordered
3) Administer anticoagulants such as d. Partial weight bearing only until full
aspirin if ordered. weight bearing allowed
i. Encourage quadriceps setting and gluteal
setting exercises when allowed.
j. Observe for adequate bowel and bladder
Herniated Nucleus Pulposus (HNP)
function. A. General information
k. Assist client in getting out of bed, usually 1. Protrusion of nucleus pulposus (central part of
on first or second post-op day. intervertebral disc) into spinal canal causing
l. Pivot or lift into chair as ordered. compression of spinal nerve roots
m. Avoid weight bearing until allowed. 2. Occurs more often in men
9. Provide care for the client with a hip 3. Herniation most commonly occurs at the
prosthesis if necessary (similar to care for fourth and fifth intervertebral spaces in the
client with total hip replacement). lumbar region
4. Predisposing factors include heavy lifting or
pulling and trauma
Total Hip Replacement B. Medical management
A. General information 1. Conservative treatment
1. Replacement of both acetabulum and head of a. Bed rest
femur with prostheses b. Traction
2. Indications 1) Lumbosacral disc: pelvic traction
a. Rheumatoid arthritis or osteoarthritis 2) Cervical disc: cervical traction
causing severe disability and intolerable pain c. Drug therapy
b. Fractured hip with nonunion 1) Anti-inflammatory agents
B. Nursing interventions 2) Muscle relaxants
1. Provide routine pre-op care. 3) Analgesics
2. In addition to routine post-op care for the d. Local application of heat and diathermy
client with hip surgery e. Corset for lumbosacral disc
a. Maintain abduction of affected limb at all f. Cervical collar for cervical disc
times with abductor splint or two pillows g. Epidural injections of corticosteroids
between legs. 2. Surgery
b. Prevent external rotation (may vary a. Discectomy with or without spinal fusion
depending on type of prosthesis and b. Chemonucleolysis
method of insertion) by placing trochanter 1) Injection of chymopapain (derivative of
rolls along leg. papaya plant) into disc to reduce size
c. Prevent hip flexion. and pressure on affected nerve root
1) Keep head of bed flat if ordered. 2) Used as alternative to laminectomy in
2) May raise bed to 45 for meals if selected cases

4
allowed.

ADULT NURSING 371


53155_04_Ch04b_p264-377.qxd 2/26/09 7:32 AM Page 372

C. Assessment findings Discectomy/Laminectomy


1. Lumbosacral disc
a. Back pain radiating across buttock and A. General information
down leg (along sciatic nerve) 1. Discectomy: excision of herniated fragments of
b. Weakness of leg and foot on affected intervertebral disc.
side 2. Laminectomy: excision of lamina to reduce
c. Numbness and tingling in toes and foot pressure on the spinal cord, spinal nerves, or
d. Positive straight-leg raise test: pain on to provide access for removing the disc.
raising leg 3. Indications
e. Depressed or absent Achilles reflex a. Most commonly used for herniated
f. Muscle spasm in lumbar region nucleus pulposus not responsive to
2. Cervical disc conservative therapy or with evidence of
a. Shoulder pain radiating down arm decreasing sensory or motor status
to hand b. Also indicated for spinal decompression as
b. Weakness of affected upper extremity with spinal cord injury, to remove
c. Paresthesias and sensory disturbances fragments of broken bone, or to remove
3. Diagnostic tests: myelogram localizes site of spinal neoplasm or abscess
herniation 4. Spinal fusion may be done at the same time if
D. Nursing interventions spine is unstable
1. Ensure bed rest on a firm mattress with bed B. Nursing interventions: preoperative
board. 1. Provide routine pre-op care.
2. Assist client in applying pelvic or cervical 2. Teach client log rolling (turning body as a unit
traction as ordered. while maintaining alignment of spinal
3. Maintain proper body alignment. column) and use of bedpan.
4. Administer medications as ordered. C. Nursing interventions: postoperative
5. Prevent complications of immobility. 1. Provide routine post-op care.
6. Provide additional comfort measures to relieve 2. Position client as ordered.
pain. a. Lower spinal surgery: generally flat
7. Provide pre-op care for client receiving b. Cervical spinal surgery: slight elevation of
chemonucleolysis. head of bed to prevent edema around airway
a. Administer cimetidine (Tagamet) and 3. Maintain proper body alignment; with cervical
diphenhydramine HCl (Benadryl) every spinal surgery avoid neck flexion and apply
6 hours as ordered to reduce possibility of cervical collar as ordered.
allergic reaction. 4. Turn client every 2 hours.
b. Possibly administer corticosteroids before a. Use log-rolling technique and turning
procedure. sheet.
8. Provide post-op care for client receiving b. Place pillows between legs while on side.
chemonucleolysis. 5. Assess for complications.
a. Observe for anaphylaxis. a. Monitor sensory and motor status every
b. Observe for less serious allergic reaction 24 hours.
(e.g., rash, itching, rhinitis, difficulty in b. With cervical spinal surgery client may
breathing). have difficulty swallowing and coughing.
c. Monitor for neurologic deficits (numbness 1) Monitor for respiratory distress.
or tingling in extremities or inability to 2) Keep suction and tracheostomy set
void). available.
9. Provide client teaching and discharge 6. Check dressings for hemorrhage, CSF leakage,
planning concerning: infection.
a. Back-strengthening exercises as 7. Promote comfort.
prescribed a. Administer analgesics as ordered.
b. Maintenance of good posture b. Provide additional comfort measures and
c. Use of proper body mechanics, how to lift positioning.
heavy objects correctly 8. Assess for adequate bladder and bowel
1) Maintain straight spine. function.
2) Flex knees and hips while stooping. a. Monitor every 24 hours for bladder
3) Keep load close to body. distension.
d. Prescribed medications and side effects b. Assess bowel sounds.
e. Proper application of corset or cervical c. Prevent constipation.
collar 9. Prevent complications of immobility.
f. Weight reduction if needed 10. Assist with ambulation.

4
a. Usually out of bed day after surgery.

372 NCLEX-RN Review


53155_04_Ch04b_p264-377.qxd 2/26/09 7:32 AM Page 373

b. Apply brace or corset if ordered.


c. If client allowed to sit, use straight-back Sample Questions
chair and keep feet flat on floor.
D. Provide client teaching and discharge planning
concerning: 379. An adult has been diagnosed with rheumatoid
1. Wound care arthritis for the past 8 years. Her condition is
2. Maintenance of good posture and proper body deteriorating despite conservative treatment, and
mechanics intramuscular gold is prescribed by the
3. Activity level as ordered physician. When teaching about gold
4. Recognition and reporting of signs of (chrysotherapy), what important teaching should
complications such as wound infection, the nurse emphasize?
sensory or motor deficits
1. Cushings syndrome is common.
2. Improvement may not occur for 36 months.
Spinal Fusion 3. Side effects are rare.
A. General information 4. The need to take this drug daily.
1. Fusion of spinous processes with bone graft
from iliac crest to provide stabilization of spine 380. A woman who has had rheumatoid arthritis for
2. Performed in conjunction with laminectomy several years is admitted to the hospital. Upon
or discectomy physical examination of the client, what should
B. Nursing interventions the nurse expect to find?
1. Provide pre-op care as for laminectomy. 1. Asymmetric joint involvement.
2. In addition to post-op care for laminectomy:
2. Heberdens nodes.
a. Position client correctly.
1) Lumbar spinal fusion: keep bed flat for 3. Obesity.
first 12 hours, then may elevate head 4. Small joint involvement.
of bed 2030, keep off back for first
48 hours. 381. An elderly client is admitted to the orthopedic
2) Cervical spinal fusion: elevate head of unit with a diagnosis of a right intracapsular hip
bed slightly. fracture. Bucks extension traction is employed
b. Assist with ambulation. prior to surgery. She complains of numbness in
1) Time varies with surgeon and extent of the right foot. After the nurse notes the fraction
fusion. tapes are lengthwise on the opposite sides of the
2) Usually out of bed 34 days post-op. limb, what would be the nurses best response?
3) Apply brace before getting client out 1. How long has your foot been numb?
of bed. 2. I can adjust it for your comfort.
4) Apply special cervical collar for
cervical spinal fusion. 3. Ill call your doctor about it.
c. Promote comfort: client may experience 4. There is nothing wrong with the traction.
considerable pain from graft site.
382. An elderly woman had an Austin-Moore
3. In addition to client teaching and discharge
prosthesis inserted following an intracapsular
planning for laminectomy, advise client that:
a. Brace will be needed for 4 months and hip fracture. Which statement by the client
lighter corset for 1 year after surgery. indicates the client has understood instruction
b. It takes 1 year until graft becomes stable. about maintaining the hip in proper position?
c. No bending, lifting, stooping, or sitting for 1. I shouldnt bend my knees.
prolonged periods for 4 months. 2. Put a pillow between my legs when you turn
d. Walking without excessive tiring is me.
healthful exercise. 3. I will be sure to put my shoes on when I go
e. Diet modification will help prevent weight
for a walk.
gain resulting from decreased activity.
4. Put the head of my bed way up so I can eat
breakfast in my normal position.
Harrington Rod Insertion
383. The nurse is caring for an elderly woman who
See Unit 5.
has had a fractured hip repaired. In the first few
days following the surgical repair which of the
Amputation of a Limb following nursing measures will best facilitate
See Amputation. the resumption of activities for this client?

4
1. Arranging for a wheelchair.

ADULT NURSING 373


53155_04_Ch04b_p264-377.qxd 2/26/09 7:32 AM Page 374

2. Asking her family to visit. 1. Pain and spasm are not expected and
3. Assisting her to sit out of bed in a chair qid. therefore there will be minimal need for pain
4. Encouraging the use of an overhead trapeze. medication.
2. Pain and spasm are expected and pain
384. A 90-year-old woman is preparing for transfer to medication will be provided as needed.
an extended care facility to continue recovery 3. Pain and spasm are expected but pain
following repair of a fractured hip. She begins to medication will interfere with a neurological
cry and says, When youre young these things assessment and will therefore be given
dont happen. Why did I break my hip at this sparingly.
age? Which response by the nurse indicates the 4. Pain and spasm are expected but pain
best understanding of risk factors for the elderly? medication will be limited as client tolerance
1. As you age you become less aware of your to the medication is feared.
surroundings and careless about safety.
2. Nothing works as well when we are older. 389. A client attends a class on osteoporosis. Which
3. There are no known specific reasons why hip statement by the client needs further teaching
fractures occur more often in your age group. about the relationship between exercise and
maintenance of bone mass?
4. Your age and sex are factors in the loss of
minerals from your bones, making them more 1. I will begin jogging.
likely to break. 2. I will begin jumping rope.
3. I will begin swimming.
385. An adult is admitted to the hospital. X-rays
4. I will begin walking.
reveal a fractured tibia and a cast is applied. Of
the following, which nursing action would be 390. In preparing a teaching plan for an adult who
most important after the cast is in place? has had an arthroscopy, the nurse will include
1. Assessing for capillary refill. which of the following?
2. Arranging for physical therapy. 1. Client should check extremity for color,
3. Discussing cast care with the client. mobility, and sensation at least every 2 hours
4. Helping the client to ambulate. after procedure.
2. Client may return to regular activities
386. A client is admitted to the floor after having immediately after procedure.
a laminectomy with spinal fusion. Of the
3. Remove compression dressing 6 to 8 hours
following maneuvers the client may use to
after procedure.
avoid pain, which is unsafe?
4. Keep extremity in flexion for 24 hours after
1. Log rolling.
procedure.
2. Asking for pain medication.
3. Placing pillows between her legs. 391. The nursing care plan for an adult who has had a
4. Sitting in semi-Fowlers. myelogram using an oil-based contrast medium
should include which intervention by the nurse?
387. A client has suffered low-back pain and sciatica 1. Give the client a light meal immediately
for over 2 years. Why is it important for the before the myelogram, to help prevent nausea
nurse to conduct a thorough assessment of his or lightheadedness.
level of discomfort from low-back pain?
2. Restrict fluids for 12 hours after the
1. This will provide a baseline for later myelogram.
comparison.
3. Keep the client in a recumbent position for
2. This is a method for identifying clients with 1224 hours after the myelogram.
low back neurosis.
4. Assure the client that stiff neck or
3. Clients who have pain localized to the back photophobia are expected side effects of the
and radiating to one extremity are probably contrast medium used during the myelogram.
not candidates for surgery.
4. Surgery is contraindicated for clients who 392. Which statement by the family tells the nurse that
have had pain for less than 2 years. they understand how to perform passive range-of-
motion exercises on a bed-bound family member?
388. An adult man is scheduled for a lumbar 1. We should put each joint through a full
laminectomy. Preoperative teaching regarding series of exercises until Mother tells us shes
postoperative pain management should include fatigued.

4
which of the following explanations?

374 NCLEX-RN Review


53155_04_Ch04b_p264-377.qxd 2/26/09 7:32 AM Page 375

2. Every day, we should try to move all of her the hand piece. Measure from palm to sole to
joints a degree or two further than they determine length of lower part of crutch.
naturally go. 4. Subtract 24 inches from clients height to
3. If Mother has a muscle spasm, we should determine length of crutch from top to tip.
stop exercising that limb for a day or two.
396. The nurse is teaching a client with a broken left
4. To exercise Mothers elbow, we would hold
ankle how to go up stairs when using crutches.
her upper arm still, and move her forearm.
Which statement by the nurse is correct?
393. An adult is learning how to use a cane. The 1. Place both crutches on the next step, stand
nurse knows that the person can use the cane on the right foot and place the left foot on the
safely when observing which of the following? step next to the crutches.
1. The cane is held on the unaffected side; the 2. Place the left crutch and right foot on the
cane and affected leg are moved forward, next step and push off with both arms then
then the unaffected leg comes forward. lift the left foot up to the step.
2. The cane is held on the affected side; the 3. Place the right foot on the next step, then
cane is moved forward, then the unaffected move the crutches and the left foot onto the
leg, then the affected leg. step.
3. The cane is held on the unaffected side; the 4. Place the right crutch and left foot on the
cane is moved forward, then the unaffected next step; move the right crutch up onto the
leg, then the affected leg. step, then swing the right foot up.
4. The cane is held on the affected side; the 397. What is one major disadvantage of a fiberglass
cane and unaffected leg are moved forward, cast?
then the affected leg comes forward.
1. It is heavy.
394. An adult who has had a total hip replacement is 2. It must remain dry.
learning how to walk with a standard (not 3. It may cause skin irritation.
reciprocal) walker. Which description below tells 4. It must be replaced frequently.
the nurse that he is using the walker correctly?
1. One side of the walker is simultaneously 398. Which of the following findings would alert the
advanced with the opposite foot; the process nurse to notify the physician of a serious
is repeated on the other side. complication for the client with a cast on his leg?
2. Each time he steps on his nonaffected side, 1. Itching under the cast.
the client advances the walker; when moving 2. Poor capillary refill of the toes.
his affected side, he steps into the walker and 3. Ability of client to move toes without
lifts his nonaffected foot. difficulty.
3. The client balances on both feet, most weight 4. Pain relieved by application of ice bag to cast.
on his nonaffected side, and lifts the walker
399. Which intervention below would be appropriate
forward; he then balances on the walker and
for the nurse to teach the client with a cast on
swings both feet forward into the walker.
his left arm?
4. The client lifts the walker in front while
1. Cover your plaster cast with plastic before
balancing on both feet, then walks into the
taking a long bath or shower.
walker, supporting his body weight on his
hands while advancing his affected side. 2. Repair breaks in the cast with super-glue or
epoxy.
395. A man has sprained his knee and the emergency 3. Remove surface dirt on your cast with a
nurse is fitting him with crutches. If the man is damp cloth.
measured while he is lying down, how does the 4. If your fiberglass cast gets wet, dry it with
nurse ensure the correct crutch length? the warm setting on your blowdryer.
1. Measure client from anterior axillary fold to
sole of foot and add 2 inches. 400. An adult is in Russell traction. It is appropriate
for the nurse to make which of the following
2. Add 6 inches to the length of the clients foot
assessments because of the clients treatment
and measure the distance from that point to
modality?
the clients axilla.
1. Make sure sling under the affected knee is
3. Measure from the clients axilla to his palm to
smooth and doesnt apply pressure in the
get the length from the top of the crutch to

4
popliteal space.

ADULT NURSING 375


53155_04_Ch04b_p264-377.qxd 2/26/09 7:32 AM Page 376

2. Ensure that both buttocks clear the mattress. 405. An adult is admitted to the medical unit with an
3. Check that the leg in traction is on the acute exacerbation of rheumatoid arthritis.
mattress, not elevated. Which of the following will the nurse include on
4. Assess for numbness and tingling of one or his nursing care plan?
more fingers, suggesting radial, ulnar, or 1. Administer analgesics for pain when systolic
median nerve pressure. blood pressure increases 20 mm Hg or more
or pulse increases 20% or more.
401. A clients family asks why the client has been 2. Develop plan with client to meet self-care
put into pelvic traction for low-back pain. What needs.
is the nurses best response? 3. Instruct client to stop taking iron
1. He really needs bed rest; the traction will supplements that lead to constipation.
force him to stay in bed. 4. Schedule hygiene activities together in one
2. By pulling on either side of the pelvis, the block to provide longer rest periods before
lower back muscles are stretched and this and after care.
gives relief from the crampy back muscles.
3. Traction helps to relieve compression of the 406. An adult has rheumatoid arthritis and is taking
roots of the nerves. prednisone. In creating a teaching plan, the
4. By holding the pelvis still, the back muscles nurse will be certain to include which of the
can relax and start to heal. following?
1. You should expect to be on corticosteroids
402. An adults left leg is in Bucks extension traction. for the rest of your life.
She complains of burning under the traction 2. It will take 3 to 6 months for you to notice
boot and the toes on that foot are cool. What is any effect from this medication.
the nurses first best action? 3. Notify your physician of any stomach upset
1. Ask, What do you mean by burning? you may have.
2. Notify the physician at once. 4. Avoid bananas and spinach while you are
3. Remove the boot, then reapply and reassess. taking this drug.
4. Apply an ice pack to the boot for 15 minutes.
407. Which statement by an adult with osteoarthritis
403. A client whose left leg is in balanced suspension indicates to the nurse that she understands her
traction for a femur fracture needs to be moved to therapeutic regimen?
a new bed. What is the best way to do this safely? 1. I will wait until my pain is very bad before I
1. All weights are removed from the ends of the take my pain medication, or else further on in
traction ropes so the leg moves freely before my disease, the medication wont help at all.
the move is attempted. 2. Jogging for short distances is better for my
2. The left leg is kept above the level of the heart. arthritis than walking for longer distances.
3. Sufficient time is given to the client to move 3. It would probably be a good idea for me to
himself to the new bed at his own rate of lose the 30 pounds my doctor recommended
tolerance. I lose.
4. The line of pull is maintained on the left leg. 4. I should do all my house cleaning on one day,
so I can rest for the remainder of the week.
404. Which statement best describes the nurses
assessment of the client with rheumatoid 408. In preparing a teaching plan for the client with
arthritis? osteoarthritis, the nurse would include which of
1. Assessment is done of the musculoskeletal, the following?
cardiac, pulmonary, and renal systems. 1. Application of cold packs to affected joints to
2. Pain is best assessed by monitoring the decrease swelling.
clients facial expression during exam and by 2. Client education regarding self-
observing limitations in the clients own administration of medications.
movement. 3. Progressively increasing activity to point of
3. Vital signs are an adequate assessment of the muscle fatigue to build muscle bulk and
acuity of the clients level of pain. improve rate of metabolism.
4. The clients health history is not nearly as 4. Teaching client that degenerative changes are
important as the nurses findings on physical progressive and that pain is a natural sequela

4
examination. of age.

376 NCLEX-RN Review


53155_04_Ch04b_p264-377.qxd 2/26/09 7:32 AM Page 377

409. The nurse, assessing a client with systemic 414. A 20-year-old was brought to the emergency
lupus erythematosus can expect to find which of department after an auto accident. There is a
the following? strong scent of alcohol about her, and she states
1. Dysphagia. she had three beers over 3 hours. Her only injury
2. Decreased visual acuity or blindness. is an open fracture of the left humerus. Which
assessment finding by the emergency nurse is
3. Dryness or itching of genitalia.
critical?
4. Abnormal lung sounds.
1. Status of clients tetanus immunization.
410. A client with systemic lupus erythematosus is 2. Current blood alcohol level.
taking gold. Which of the following 3. Support systems available at home to assist
interventions would the nurse include in the with care.
teaching plan for this client? 4. Last time client voided.
1. Stop taking your anti-inflammatory
medication as long as you are taking gold 415. A firefighter fell off a roof while fighting a house
preparations. fire and fractured his femur. Approximately
2. You will give yourself intramuscular 24 hours after the incident, the nurse finds him
injections of gold preparations every day for dyspneic, tachypneic, with scattered crackles in
2 to 4 weeks, then taper down to one his lung fields; he is coughing up large amounts
injection every 2 months. of thick, white sputum. What nursing diagnosis
would be formulated?
3. You will be taking a large dose when you
start taking gold capsules, and will taper 1. Respiratory compromise related to inhalation
down to a smaller dose as the therapy of smoke.
becomes effective. 2. Pneumonia related to prolonged bed rest.
4. Stay away from crowds during flu season 3. Fat embolism syndrome related to femur
and have your blood tested after every other fracture.
gold injection. 4. Hypovolemic shock related to multiple
trauma.
411. In assessing the client with osteomyelitis, the
nurse would expect to find which of the 416. An adult has had a total right hip replacement.
following? Why does the operative hip need to be kept in
1. Pale, cool, tender skin at site. the extension and abduction position?
2. Decreased white blood cell count. 1. Reduces the risk for the development of
3. Positive wound cultures. thromboemboli.
4. Decreased erythrocyte sedimentation rate. 2. Promotes circulation to the operative site,
reducing the risk of avascular necrosis.
412. An adult has a fractured left radius, which has 3. Helps to prevent dislocation of the hip
been casted. While performing an assessment of prosthesis.
this client, the nurse will correctly identify 4. Facilitates the drainage of blood and fluid at
which of these findings as emergent? the operative site.
1. Pain at the fracture site.
2. Swelling of fingers of left hand. 417. An adult who has had a total right hip
replacement asks the nurse about moving
3. Diminished capillary refill of fingers of left
around in this bed. What is the nurses best
hand.
response?
4. Warm, dry fingers of left hand.
1. The client should remain supine for 48 hours
413. Which intervention by the emergency nurse is after surgery, with affected leg in a slightly
critical in caring for the client with a fractured inward-rotation position.
tibia and fibula? 2. Although the client must remain supine, she
1. Cutting away clothing on the injured leg. can cross her legs to change position for
comfort.
2. Palpation of the dorsalis pedis pulses.
3. A side-lying position is undesirable, but the
3. Administration of analgesic medications as
head of the bed can be elevated 6075 to
ordered.
shift weight off of back and buttocks.
4. Initiating two, large-bore IV catheters and
4. The client will be repositioned using an
warmed normal saline at a fast rate.

4
abductor pillow between the legs.

ADULT NURSING 377


53155_04_Ch04c_p378-420.qxd 2/26/09 7:33 AM Page 378

418. Which statement by a client who has had an client will most likely be maintained in what
open reduction/internal fixation of her fractured position?
left hip indicates to the nurse that the client 1. Left lateral decubitus with neck flexed to 30.
understands her care? 2. Supine, with no pillows under the head.
1. My nephew will move my bed down to the 3. Semi-Fowlers.
first floor so I wont have to go upstairs when
4. Modified Trendelenburg, with a soft cervical
I get home.
collar in place.
2. I should expect my surgical site to be swollen
and red for a week or two after I get home. 423. An adult is being discharged after a lumbar
3. The night nurse will take off these thigh- laminectomy. Which statement indicates to the
high stockings at bedtime, and the day nurse nurse that the client understands her discharge
will put them back on at breakfast time. teaching?
4. I need to limit my fluid intake so I wont be 1. I cant wait to sit in my own recliner and
getting on and off the bedpan so often; its not rest while I watch my soaps!
good for my hip. 2. Ill be able to man the refreshment stand at
my nephews baseball game next weekend,
419. The nurse teaches an adult woman that because wont I?
she has osteoporosis, she must take safety
3. My friend is getting me a footstool for in
precautions to prevent falls, because you could
front of my sink.
break a hip. When the client asks what one has
to do with the other, what is the nurses best 4. I have to buy a soft mattress so my spine
response? wont be subjected to any extra pressure.
1. Osteoporosis yields brittle bones which break 424. The nurse is planning post-op care for a client
easily. undergoing a laminectomy. Why does the nurse
2. Osteoporosis causes changes in balance, need to know whether the client will be having a
which makes the client more susceptible to spinal fusion also?
falls that could lead to hip fractures. 1. The contrast medium used to check the
3. Hips are the primary sites of calcium loss in fusion site for grafting could cause an allergic
osteoporosis, making them more susceptible reaction.
to fracture. 2. The client whose laminectomy is performed
4. Both osteoporosis and hip fractures are with a spinal fusion will be on bed rest
common in elderly women. longer than the client who does not undergo
spinal fusion.
420. An adult is diagnosed with a herniated nucleus
3. Clients undergoing spinal fusion will be in
pulposus at the C5-C6 interspace and a second at
long torso casts for 6 to 8 weeks after surgery.
the C6-C7 interspace. Which of the following
findings would the nurse expect to discover 4. The client whose laminectomy is performed
during the assessment? with a spinal fusion is at greater risk for
spontaneous pneumothorax than the client
1. Constant, throbbing headaches.
who does not undergo spinal fusion.
2. Numbness of the face.
3. Clonus in the lower extremities.
4. Pain in the scapular region.

421. An adult has undergone a cervical laminectomy. Answers and Rationales


Because of the potential complications
associated with this procedure, the nurse must
perform which of the following assessments? 379. 2. Chrysotherapy often requires a 36 month
period before effects are seen.
1. Assess for wheezes and stridor.
2. Check pupils for response to direct and 380. 4. Small joint involvement is common in
consensual light. rheumatoid arthritis. All of the other symptoms
3. Assess gag reflex. are seen in osteoarthritis but not rheumatoid
4. Assess shoulder shrug and neck strength. arthritis.

422. The nurse is caring for a person who just had a 381. 1. Numbness is symptomatic of circulatory or
cervical laminectomy. The nurse knows that the nerve impairment to the extremity. It is

4 378 NCLEX-RN Review


53155_04_Ch04c_p378-420.qxd 2/26/09 7:33 AM Page 379

important to know the length of time the client remain in bed, with the head elevated 1530 (to
has been experiencing this sensation. minimize the upward migration of the medium),
but some physicians may allow these clients to
382. 2. A pillow placed between the clients legs will ambulate.
keep the affected leg abducted and in good
alignment while the client is being turned. 392. 4. To perform passive range of motion, the joint
is supported, the bones above the joint are
383. 4. Exercise is important to keep the joints and stabilized, and the body part distal to the joint is
muscles functioning and to prevent secondary exercised through the range of motion. The
complications. Use of the overhead trapeze familys description of how to maneuver the
prevents hazards of immobility by permitting elbow illustrates this well.
movement in bed and strengthening of the upper
extremities in preparation for ambulation. 393. 1. The cane, held on the unaffected side, will
provide a wider base of support for the affected
384. 4. Elderly females are prone to hip fractures side while the unaffected limb is moving. The
because the cessation of estrogen production client should keep the cane close to the body to
after menopause contributes to demineralization prevent leaning.
of bone.
394. 4. The sequence for using a walker is balance on
385. 1. Good capillary refill indicates that the cast has both feet, lift the walker and place in front of
not caused a circulatory problem in the extremity. you, walk into the walker (using it for support
Assessing circulation is a priority of action. when standing on affected limb) and then
balance on both feet before repeating the
386. 4. The client returning from a lumbar spinal
sequence.
fusion should be kept flat in bed.
395. 1. Although measuring the client while he is
387. 1. The importance of an accurate history cannot
lying down is not the preferred method of fitting
be overemphasized in assessing the character
crutches, this formula may be used successfully.
and location of the pain. A baseline assessment
of neurological signs is made so that deviation 396. 3. The unaffected limb is advanced to the next
from the database can be noted. Once a pain step, then the crutches and the affected limb
assessment is complete a plan for pain move to that step (weight stays on crutches or
management can be developed. foot of unaffected side). A handy mnemonic for
clients is, Up with the good leg, down with the
388. 2. Clients should be told that they may
bad, meaning the good leg is used first when
experience pain and spasm in the early
going up stairs, and the crutches and bad leg
postoperative period and that pain medication
go to the new step first when going down stairs.
will be provided.
397. 3. Although fiberglass casts have other
389. 3. Physical compression of weight-bearing joints
advantages, the particles of fiberglass may
stimulates osteoblastic deposition of calcium.
scratch and cause a skin reaction.
Swimming does not involve weight bearing and
physical compression of joints. 398. 2. Poor capillary refill (a pinking up of the toes
after the nailbeds are blanched by compression,
390. 1. Because the joint is distended with saline and
which takes more than 3 seconds) is indicative of a
the arthroscope is introduced into the joint area,
circulatory compromise. In this scenario, the likely
the potential for neurovascular damage exists.
cause is compartment syndrome: an increase of
Color (indicating adequate vascular perfusion),
pressure within the cast (compartment). Other
sensation, and mobility (indicating intact
signs/symptoms include pain unrelieved by usual
neurologic status) should be assessed, although
modalities, disproportional swelling, and inability
mobility assessment will likely be limited to
to move digits.
wiggling the digits.
399. 3. A damp, not wet, cloth can be used to remove
391. 3. If an oil-based contrast medium is used, the
superficial dirt. Stained areas can be covered
client will be kept in a recumbent position for
with a thin layer of white shoe polish.
1224 hours to reduce cerebrospinal fluid
leakage, and thus decrease the likelihood of 400. 1. Russell traction is a modification of Buck
developing a postprocedure headache. If a water- extension, used for a femur fracture that is not
soluble medium is used, the client will usually appropriate for internal fixation. The

ADULT NURSING

4 379
53155_04_Ch04c_p378-420.qxd 2/26/09 7:33 AM Page 380

modification occurs when a sling is placed 408. 2. Anti-inflammatory medications including


under the affected knee, giving more comfort to salicylates and NSAIDs will be taken by the client
the client, and preventing some of the rotation indefinitely. The client must understand the
tendencies. regimen; ways to monitor for (and when possible,
diminish) adverse effects must also be taught.
401. 3. Although this information may be a bit
technical for the lay members of the clients 409. 4. Abnormal lung sounds are indicative of
family, it is the only answer that provides respiratory insufficiency from pleural effusions
correct information. Increased lumbar flexion or infiltrations. Pleural effusions may occur with
relieves compression of the lumbar nerve roots, myocarditis, which might manifest as a
which is why the head of the bed is elevated 30 pericardial friction rub assessed during the
and the knees are flexed; 1530 lb of traction cardiovascular exam.
increases lumbar flexion and facilitates relief.
410. 4. Signs of gold toxicity include bone marrow
402. 2. These are signs of potential neurovascular suppression and hematuria, proteinuria,
compromise, an orthopedic emergency. diarrhea, and stomatitis. The client receiving
Additionally, the nurse would check the parenteral gold should expect to have blood and
capillary refill of the toes, any peripheral pulses urine monitored after every other injection.
present, and the sensation and mobility of the
foot. In addition, the nurse would compare 411. 3. Positive wound cultures are used to help
findings on the affected side with those on the determine the causative organisms and indicate
nonaffected side. which antibiotic therapy is appropriate. Blood
cultures may also be positive.
403. 4. A vertical transfer is permitted, as long as
manual traction is applied to maintain the line 412. 3. Diminished capillary refill suggests vascular
of pull, that is, the direction of the traction, or compromise, an emergency condition.
pull, which the balanced suspension device
413. 2. Neurovascular compromise is possible with
supplied.
fracture; distal pulses should be palpated to
404. 1. Rheumatoid arthritis is a condition with ensure circulation is adequate; the dorsalis pedis
multisystem effects. The cardiopulmonary and pulses in both feet should be assessed for
renal systems must be assessed as well as the comparison purposes. Likewise color, sensation,
obvious assessment of the musculoskeletal and mobility should be assessed. These
system. Because some of the medications used interventions are also to be repeated at frequent
in treating rheumatoid arthritis can have serious intervals during the emergent phase, and
systemic effects (e.g., gold therapy, especially after any intervention (i.e., splinting,
corticosteroids), this is especially true of the casting).
client on aggressive pharmacotherapy.
414. 1. There is a strong risk of infection with an
405. 2. Although this is an exacerbation of an existing open fracture (a fracture with an open wound
problem, it is always appropriate to formulate a through skin surface to bone injury), and the
planwith the clients inputas how best to nurse will expect the client to be given
meet his self-care needs. The plan will need to antibiotics prophylactically; in addition, tetanus
address protection of joints, conservation of immunization status must be assessed, and
energy, and methods to simplify work tasks. tetanus prophylaxis given if needed, or if the
Correct use of assistive devices may also be status cannot be determined.
involved in the plan.
415. 3. These are classic observations of the client
406. 3. High dosage or long-term use of with fat embolism syndrome, seen within
corticosteroids is associated with the 48 hours of a long-bone fracture. Onset of
development of gastric ulcers. Other adverse symptoms is rapid, and often fatal. Any
effects associated with this treatment include respiratory difficulties, personality changes,
hypertension, hyperglycemia, infection or chest pain in clients with recent long-bone
susceptibility, and psychiatric disorders. fractures must be assessed with fat embolism
syndrome in mind.
407. 3. Weight reduction can reduce stress on weight-
bearing joints; because the clients physician has 416. 3. Positioning the client in extension and
recommended it, we can believe that she will abduction helps to ensure that the femoral head

4
benefit from weight loss. part of the prosthesis remains in the acetabular

380 NCLEX-RN Review


53155_04_Ch04c_p378-420.qxd 2/26/09 7:33 AM Page 381

cup. The use of wedge pillows or abduction Stiffness, paresthesias, or numbness in upper
splints helps in maintaining correct position. extremities is also possible.

417. 4. To maintain the femoral component of the 421. 1. The client is assessed for respiratory distress
prosthesis in the acetabular cup, an abductor that would be caused by cord edema. The nurse
pillow may be used to keep the legs separate; would also look for signs of interstitial edema
this must be maintained in all repositioning and/or hematoma.
activities. The client is always encouraged to
assist with repositioning, as long as the integrity 422. 2. This position will maintain spinal alignment.
of the hip position is maintained. Occasionally, the physician will approve the use
of a pillow under the head, or the head of the
418. 1. The client will need to refrain from climbing bed raised 30 if a soft cervical collar is used,
stairs in the early recovery phase, except as and if the surgical approach was posterior.
guided during physical therapy sessions.
Moving to the first floor is a prudent decision. 423. 3. When standing (for example, while washing
dishes at the sink), the client should place one
419. 1. Osteoporosis is a disorder in which bone foot on a stool, thus alternating the weight
formation is slower than bone resorption. The between the feet.
outcome of this disequilibrium is bones that are
increasingly porous, brittle, and fragile. Any 424. 2. Because the bone graft used for fusion is taken
bone can be broken, but the trauma associated from the iliac crest or fibula, the client will have
with falls frequently leads to hip fractures in pain and the potential for complications at those
clients with osteoporosis. sites as well as complications from the
laminectomy. Bed rest may be maintained for
420. 4. Sometimes misinterpreted by the client as a longer periods of time; lumbar support may be
heart attack or bursitis, pain with cervical disc used once the client is ambulating, and pain
herniation at this level may occur between the relief must be directed at the graft site as well as
scapulae, in the neck or the top of the shoulders. the laminectomy site.

The Endocrine System

OVERVIEW OF ANATOMY B. Negative feedback mechanisms: major means of


regulating hormone levels
AND PHYSIOLOGY 1. Decreased concentration of a circulating
hormone triggers production of a stimulating
The endocrine system is composed of an interrelated hormone from the pituitary gland; this
complex of glands (pituitary, adrenals, thyroid, hormone in turn stimulates its target organ to
parathyroids, islets of Langerhans of the pancreas, produce hormones.
ovaries, and testes) that secrete a variety of hormones 2. Increased concentration of a hormone inhibits
directly into the bloodstream. Its major function, together production of the stimulating hormone,
with the nervous system, is to regulate body functions. resulting in decreased secretion of the target
organ hormone.
Hormone Regulation C. Some hormones are controlled by changing blood
levels of specific substances (e.g., calcium,
A. Hormones: chemical substances that act as glucose).
messengers to specific cells and organs (target D. Certain hormones (e.g., cortisol or female
organs), stimulating and inhibiting various reproductive hormones) follow rhythmic patterns
processes; two major categories of secretion.
1. Local: hormones with specific effect in the E. Autonomic and CNS control (pituitary-
area of secretion (e.g., secretin, hypothalamic axis): hypothalamus controls release
cholecystokinin-pancreozymin [CCK-PZ]) of the hormones of the anterior pituitary gland
2. General: hormones transported in the blood through releasing and inhibiting factors that
to distant sites where they exert their effect

4
stimulate or inhibit hormone secretion.
(e.g., cortisol)

ADULT NURSING 381


53155_04_Ch04c_p378-420.qxd 2/26/09 7:33 AM Page 382

Structure and Function 1. Anterior lobe (adenohypophysis)


a. Secretes tropic hormones (hormones that
of Endocrine Glands stimulate target glands to produce their
See Table 4-25 and Figure 4-25. hormone): adrenocorticotropic hormone
(ACTH), thyroid-stimulating hormone
(TSH), follicle-stimulating hormone (FSH),
Pituitary Gland (Hypophysis)
luteinizing hormone (LH)
A. Located in sella turcica at the base of the brain b. Also secretes hormones that have direct
B. Master gland of the body, composed of three effect on tissues: somatotropic or growth
lobes hormone, prolactin

Table 4-25 Hormone Functions

Endocrine Gland Hormones Functions


Pituitary
Anterior lobe TSH Stimulates thyroid gland to release thyroid hormones.
ACTH Stimulates adrenal cortex to produce and release adrenocorticoids.
FSH, LH Stimulate growth, maturation, and function of primary and
secondary sex organs.
GH or somatotropin Stimulates growth of body tissues and bones.
Prolactin or LTH Stimulates development of mammary glands and lactation.
Posterior lobe
Regulates water metabolism; released during stress or in
ADH response to an increase in plasma osmolality to stimulate
reabsorption of water and decrease urine output.
Stimulates uterine contractions during delivery and the
Oxytocin release of milk in lactation.
Intermediate lobe MSH Affects skin pigmentation.
Adrenal
Adrenal cortex Mineralocorticoids Regulate fluid and electrolyte balance; stimulate reabsorption of
(e.g., aldosterone) sodium, chloride, and water; stimulate potassium excretion.
Glucocorticoids (e.g., cortisol, Increase blood glucose levels by increasing rate of glyconeogenesis;
corticosterone) increase protein catabolism, increase mobilization of fatty acids;
promote sodium and water retention; anti-inflammatory effect;
aid body in coping with stress.
Sex hormones (androgens, Influence development of secondary sex characteristics.
estrogen, progesterone)
Adrenal medulla Epinephrine, norepinephrine Function in acute stress; increase heart rate, blood pressure; dilate
bronchioles; convert glycogen to glucose when needed by
muscles for energy.
Thyroid T3, T4 Regulate metabolic rate; carbohydrate, fat, and protein metabolism;
aid in regulating physical and mental growth and development.
Thyrocalcitonin Lowers serum calcium by increasing bone deposition.
Parathyroid PTH Regulates serum calcium and phosphate levels.
Pancreas (Islets
of Langerhans)
Beta cells Insulin Allows glucose to diffuse across cell membrane; converts glucose
to glycogen.
Alpha cells Glucagon Increases blood glucose by causing glyconeogenesis and
glycogenolysis in the liver; secreted in response to low blood sugar.
Ovaries Estrogen, progesterone Development of secondary sex characteristics in the female,
maturation of sex organs, sexual functioning, maintenance of
pregnancy.
Testes Testosterone Development of secondary sex characteristics in the male,
maturation of sex organs, sexual functioning.

4 382 NCLEX-RN Review


53155_04_Ch04c_p378-420.qxd 2/26/09 7:33 AM Page 383

Figure 4-25 Hormones secreted by the anterior pituitary gland

c. Regulated by hypothalamic releasing and Pancreas


inhibiting factors and by negative feedback
system A. Located behind the stomach
2. Posterior lobe (neurohypophysis): does not B. Has both endocrine and exocrine functions
produce hormones; stores and releases (See Gastrointestinal System).
antidiuretic hormone (ADH) and oxytocin, C. Islets of Langerhans (alpha and beta cells) involved
produced by the hypothalamus in endocrine function
3. Intermediate lobe: secretes melanocyte- 1. Beta cells: produce insulin
stimulating hormone (MSH) 2. Alpha cells: produce glucagon

Adrenal Glands Gonads


A. Two small glands, one above each kidney A. Ovaries: located in pelvic cavity, produce estrogen
B. Consist of two sections and progesterone
1. Adrenal cortex (outer portion): produces B. Testes: located in scrotum, produce
mineralocorticoids, glucocorticoids, sex hormones testosterone
2. Adrenal medulla (inner portion): produces
epinephrine, norepinephrine

Thyroid Gland ASSESSMENT


A. Located in anterior portion of the neck Health History
B. Consists of two lobes connected by a narrow isthmus
C. Produces thyroxine (T4), triiodothyronine (T3), A. Presenting problem: symptoms may include:
thyrocalcitonin 1. Change in appearance: hair, nails, skin (change
in texture or pigmentation); change in size,
Parathyroid Glands shape, or symmetry of head, neck, face, eyes,
or tongue
A. Four small glands located in pairs behind the
2. Change in energy level
thyroid gland
3. Temperature intolerance

4
B. Produce parathormone (PTH)

ADULT NURSING 383


53155_04_Ch04c_p378-420.qxd 2/26/09 7:33 AM Page 384

4. Development of abnormal secondary sexual 3. Nursing care


characteristics; change in sexual function a. Take thorough history; thyroid medication
5. Change in emotional state, thought pattern, or must be discontinued 710 days prior to
intellectual functioning test; medications containing iodine, cough
6. Signs of increased activity of sympathetic preparations, excess intake of iodine-rich
nervous system (e.g., nervousness, foods, and tests using iodine (e.g., IVP) can
palpitations, tremors, sweating) invalidate this test.
7. Change in bowel habits, appetite, or weight; b. Assure client that no radiation precautions
excessive hunger or thirst are necessary.
8. Change in urinary pattern C. Thyroid scan
B. Lifestyle: any increased stress 1. Administration of radioactive isotope (orally
C. Past medical history: growth and development or IV) and visualization by a scanner of the
(any delayed or excessive growth); diabetes, distribution of radioactivity in the gland
thyroid disease, hypertension, obesity, infertility 2. Performed to determine location, size, shape,
D. Family history: endocrine diseases, growth and anatomic function of thyroid gland;
problems, obesity, mental illness identifies areas of increased or decreased
uptake; valuable in evaluating thyroid nodules
3. Nursing care: same as RAIU
Physical Examination
A. Check height, weight, body stature, and body Pancreatic Function
proportions.
B. Observe distribution of muscle mass, fat A. Fasting blood sugar: measures serum glucose
distribution, any muscle wasting. levels; client fasts from midnight before the test
C. Inspect for hair growth and distribution. B. Two-hour postprandial blood sugar: measurement
D. Check condition and pigmentation of skin; of blood glucose 2 hours after a meal is ingested
presence of striae. 1. Fast from midnight before test
E. Inspect eyes for any bulging. 2. Client eats a meal consisting of at least 75 g
F. Observe for enlargement in neck area and quality carbohydrate or ingests 100 g glucose
of voice. 3. Blood drawn 2 hours after the meal
G. Observe development of secondary sex C. Oral glucose tolerance test: most specific and
characteristics. sensitive test for diabetes mellitus
H. Palpate thyroid gland (normally cannot be 1. Fast from midnight before test
palpated): note size, shape, symmetry, any 2. Fasting blood glucose and urine glucose
tenderness, presence of any lumps or nodules. specimens obtained
3. Client ingests 100 g glucose; blood sugars are
drawn at 30 and 60 minutes and then hourly
Laboratory/Diagnostic Tests for 35 hours; urine specimens may also be
A variety of tests may be performed to measure the collected
amounts of hormones present in the serum or urine in 4. Diet for 3 days prior to test should include
assessing pituitary, adrenal, and parathyroid functions; 200 g carbohydrate and at least 1500 kcal/day
these tests will be referred to when appropriate under 5. During test, assess the client for reactions such
specific disorders of the endocrine system. as dizziness, sweating, and weakness
D. Glycosylated hemoglobin (hemoglobin A1c) reflects
Thyroid Function the average blood sugar level for the previous
100120 days. Glucose attaches to a minor
A. Serum studies: nonfasting blood studies (no hemoglobin (A1c). This attachment is irreversible.
special preparation necessary) 1. Fasting is not necessary.
1. Serum T4 level: measures total serum level of 2. Excellent method to evaluate long-term control
thyroxine of blood sugar.
2. Serum T3 level: measures serum
triiodothyronine level
3. TSH: measurement differentiates primary from ANALYSIS
secondary hypothyroidism
B. Radioactive iodine uptake (RAIU) Nursing diagnoses for the client with a disorder of the
1. Administration of 123I or 131I orally; measurement endocrine system may include:
by a counter of the amount of radioactive iodine A. Imbalanced nutrition: more or less than body
taken up by the gland after 24 hours requirements
2. Performed to determine thyroid function; B. Risk for infection
increased uptake indicates hyperactivity; C. Impaired urinary elimination

4
minimal uptake may indicate hypothyroidism D. Deficient fluid volume

384 NCLEX-RN Review


53155_04_Ch04c_p378-420.qxd 2/26/09 7:33 AM Page 385

E. Risk for impaired skin integrity d. Endocrine: impaired glucose metabolism,


F. Sexual dysfunction hyperglycemia, menstrual dysfunction,
G. Deficient knowledge growth retardation
H. Ineffective individual coping e. Musculoskeletal: muscle weakness,
I. Disturbed sleep pattern osteoporosis
J. Disturbed body image f. Neurologic: personality changes, headache,
syncope, vertigo, irritability, insomnia,
seizures
PLANNING AND g. Ophthalmologic: cataract formation,
glaucoma
IMPLEMENTATION h. Other: hypokalemia, thrombophlebitis,
masking of signs of infection, increased
Goals susceptibility to infection
i. Sudden withdrawal may precipitate acute
Client will: adrenal insufficiency
A. Regain optimal nutritional status. B. Nursing care
B. Be free from infection. 1. Administer with food or milk; instruct client
C. Have adequate urinary elimination and fluid to report gastric distress (antacids may be
volume. necessary).
D. Maintain skin integrity. 2. Give in a single daily dose, preferably before
E. Experience optimum sexual health. 9 A.M. (cortisol level is at highest peak between
F. Demonstrate and use knowledge of disease 6 and 8 A.M.).
process, prescribed medications, treatments, and 3. Instruct client to avoid infections and to report
complications in order to maintain optimal health. immediately if one is suspected.
G. Use positive coping behaviors in dealing with the 4. Instruct client never to withdraw the drug
effects of acute and chronic illness. abruptly, as this may cause acute adrenal
H. Attain an optimal balance of rest and activity. insufficiency.
5. Observe client for any mental changes
Interventions (e.g., irritability, mood swings, euphoria,
depression).
Care of the Client on Corticosteroid 6. Alert women that menstrual irregularity may
Therapy develop.
7. Monitor blood pressure, I&O, weight, blood
A. General information
glucose, and serum potassium.
1. Types of preparations include cortisone,
8. Advise client to restrict salt intake.
hydrocortisone, prednisone, dexamethasone
9. Encourage intake of foods high in potassium.
(Decadron)
2. Indications
a. Replacement therapy in primary and
secondary adrenocortical insufficiency EVALUATION
b. Symptomatic treatment for anti-
A. Client maintains normal weight; no evidence of
inflammatory effect of numerous
malnutrition.
inflammatory, allergic, or immunoreactive
B. Clients temperature is within normal limits; no
disorders (e.g., arthritis, SLE, bronchial
signs of infection.
asthma, skin diseases, ocular disorders,
C. Client has adequate patterns of urinary
allergic diseases, inflammatory bowel
elimination.
disorders, cerebral edema and increased
D. Peripheral edema is reduced.
ICP, shock, nephrotic syndrome,
E. Blood pressure and urine output are within normal
malignancies, myasthenia gravis, multiple
limits; no signs of dehydration.
sclerosis)
F. Skin is intact and free from irritation.
3. Common side effects: salt and water retention,
G. Client verbalizes satisfying sexual
sweating, increased appetite
activity/expression.
4. Adverse reactions
H. Client demonstrates and uses knowledge of disease
a. Cardiovascular: hypertension, CHF
process, prescribed medications, and treatments;
b. GI: peptic ulcer, ulcerative esophagitis
reports any complications.
c. Integumentary: petechiae, ecchymoses,
I. Client uses effective coping behaviors to
purpura, hirsutism, acne, thinning of skin,
successfully adapt to effects of illness, changes in
striae, redistribution of body fat in
body image, and loss of function.
subcutaneous tissue, abnormal
J. Client maintains balance between activity and rest.
pigmentation, poor wound healing

4
K. Client demonstrates increased self-esteem.

ADULT NURSING 385


53155_04_Ch04c_p378-420.qxd 2/26/09 7:33 AM Page 386

DISORDERS OF THE B. Medical management: surgical removal or


irradiation of the gland
ENDOCRINE SYSTEM C. Assessment findings
1. Tumor: bitemporal hemianopia; headache
Specific Disorders 2. Hormonal disturbances depending on which
hormones are being excreted in excess
of the Pituitary Gland 3. Acromegaly caused by oversecretion of growth
hormones: transverse enlargement of bones,
Hypopituitarism
especially noticeable in skull and in bones of
A. General information hands and feet; features become coarse and
1. Hypofunction of the anterior pituitary gland heavy; lips become heavier; tongue enlarged
resulting in deficiencies of both the hormones 4. Diagnostic tests
secreted by the anterior pituitary gland and a. Skull X-ray, CT scan reveal pituitary tumor
those secreted by the target glands b. Plasma hormone levels reveal increased
2. May be caused by tumor, trauma, surgical growth hormone, oversecretion of other
removal, or irradiation of the gland; or may be hormones
congenital (pituitary dwarfism) D. Nursing interventions
B. Medical management: specific treatment depends 1. Monitor for hyperglycemia and cardiovascular
on cause problems (hypertension, angina, HF) and
1. Tumor: surgical removal or irradiation of the modify care accordingly.
gland 2. Provide psychologic support and acceptance
2. Regardless of cause, treatment will include for alterations in body image.
replacement of deficient hormones: e.g., 3. Provide care for the client undergoing
corticosteroids, thyroid hormone, sex hypophysectomy or radiation therapy if
hormones, gonadotropins (may be used to indicated.
restore fertility).
C. Assessment findings Hypophysectomy
1. Tumor: bitemporal hemianopia, headache
2. Varying signs of hormonal disturbances A. General information
depending on which hormones are being 1. Partial or complete removal of the pituitary
undersecreted (e.g., menstrual dysfunction, gland
hypothyroidism, adrenal insufficiency) 2. Indications: pituitary tumors, diabetic
3. Retardation of growth if condition occurs retinopathy, metastatic cancer of the breast or
before epiphyseal closure prostate, which may be endocrine dependent
4. Diagnostic tests 3. Surgical approaches
a. Skull X-ray, CT scan may reveal pituitary a. Craniotomy: usually transfrontal
tumor b. Transphenoidal: incision made in inner
b. Plasma hormone levels may be decreased aspect of upper lip and gingiva; sella
depending on specific hormones turcica is entered through the floor of the
undersecreted nose and sphenoid sinuses
D. Nursing interventions B. Nursing care
1. Provide care for the client undergoing 1. In addition to pre-op care of the craniotomy
hypophysectomy or radiation therapy if client, explain post-op expectations.
indicated. 2. In addition to post-op care of the craniotomy
2. Provide client teaching and discharge client, observe for signs of target gland
planning concerning: deficiencies (diabetes insipidus, adrenal
a. Hormone replacement therapy insufficiency, hypothyroidism) due to total
b. Importance of follow-up care removal of the gland or to post-op edema.
a. Perform hourly urine outputs and specific
gravities; alert physician if urine output is
Hyperpituitarism
greater than 800900 mL/2 hours or if
A. General information specific gravity is less than 1.004.
1. Hyperfunction of the anterior pituitary gland b. Administer cortisone replacement as
resulting in oversecretion of one or more of the ordered.
anterior pituitary hormones 3. If transphenoidal approach used:
2. Overproduction of the growth hormone a. Elevate the head of the bed to 30 to
produces acromegaly in adults and gigantism decrease headache and pressure on the
in children (if hypersecretion occurs before sella turcica.
epiphyseal closure). b. Administer mild analgesics for headache

4
3. Usually caused by a benign pituitary adenoma as ordered.

386 NCLEX-RN Review


53155_04_Ch04c_p378-420.qxd 2/26/09 7:33 AM Page 387

c. Perform frequent oral hygiene with soft 3. Tachycardia, eventual shock if fluids not replaced
swabs to cleanse the teeth and mouth 4. Diagnostic tests
rinses; no toothbrushing. a. Urine specific gravity less than 1.004
d. Observe for and prevent CSF leak from b. Water deprivation test reveals inability to
surgical site. concentrate urine
1) Warn the client not to cough, sneeze, C. Nursing interventions
or blow nose. 1. Maintain fluid and electrolyte balance.
2) Observe for clear drainage from nose a. Keep accurate I&O.
or postnasal drip (constant b. Weigh daily.
swallowing); check drainage for c. Administer IV/oral fluids as ordered to
glucose; positive results indicate that replace fluid losses.
drainage is CSF. 2. Monitor vital signs and observe for signs of
3) If leakage does occur: dehydration and hypovolemia.
a) Elevate head of bed and call the 3. Administer hormone replacement as ordered.
physician. a. Vasopressin (Pitressin) given IV and SC;
b) Most leaks will resolve in 72 hours desmopressin, given PO or intranasal.
with bed rest and elevation. 1) Warm to body temperature before giving.
c) May do daily spinal taps to 2) Shake tannate suspension to ensure
decrease CSF pressure. uniform dispersion.
d) Administer antibiotics as ordered b. Lypressin (Diapid): nasal spray
to prevent meningitis. 4. Provide client teaching and discharge
4. Provide client teaching and discharge planning concerning:
planning concerning: a. Lifelong hormone replacement; lypressin
a. Hormone therapy as needed to control polyuria and
1) If gland is completely removed, client polydipsia
will have permanent diabetes b. Need to wear Medic-Alert bracelet
insipidus
2) Cortisone and thyroid hormone Syndrome of Inappropriate Antidiuretic
replacement Hormone Secretion (S I ADH)
3) Replacement of sex hormones
a) Testosterone: may be given for A. General information
impotence in men 1. Hypersection of ADH from the posterior
b) Estrogen: may be given for atropy pituitary gland even when the client has
of the vaginal mucosa in women abnormal serum osmolality.
c) Human pituitary gonadotropins: 2. SIADH may occur in persons with
may restore fertility in some bronchogenic carcinoma or other
women nonendocrine conditions.
b. Need for lifelong follow-up and hormone B. Medical management
replacement 1. Treat underlying cause if possible
c. Need to wear Medic-Alert bracelet 2. Diuretics and fluid restriction
d. If transphenoidal approach was used: C. Assessment findings
1) Avoid bending and straining at stool 1. Persons with SIADH cannot excrete a dilute urine
for 2 months post-op 2. Fluid retention and sodium deficiency.
2) No toothbrushing until sutures are D. Nursing interventions
removed and incision heals (about 1. Administer diuretics (furosemide [Lasix]) as
10 days) ordered.
2. Restrict fluids to promote fluid loss and
Diabetes Insipidus gradual increase in serum sodium.
3. Monitor serum electrolytes and blood
A. General information chemistries carefully.
1. Hypofunction of the posterior pituitary gland 4. Careful intake and output, daily weight.
resulting in deficiency of ADH 5. Monitor neurologic status.
2. Characterized by excessive thirst and urination
3. Caused by tumor, trauma, inflammation,
pituitary surgery
Disorders of the Adrenal Gland
B. Assessment findings Addisons Disease
1. Polydipsia (excessive thirst) and severe
polyuria with low specific gravity A. General information
2. Fatigue, muscle weakness, irritability, weight 1. Primary adrenocortical insufficiency;

4
loss, signs of dehydration hypofunction of the adrenal cortex causes

ADULT NURSING 387


53155_04_Ch04c_p378-420.qxd 2/26/09 7:33 AM Page 388

decreased secretion of the mineralocorticoids, Addisonian Crisis


glucocorticoids, and sex hormones A. General information
2. Relatively rare disease caused by: 1. Severe exacerbation of Addisons disease
a. Idiopathic atrophy of the adrenal cortex caused by acute adrenal insufficiency
possibly due to an autoimmune process 2. Precipitating factors
b. Destruction of the gland secondary a. Overexertion, infection, trauma, stess,
to tuberculosis (rare, due to early failure to take prescribed medications
TB treatment available) or fungal b. Iatrogenic: surgery on pituitary or adrenal
infection glands, rapid withdrawal of exogenous
B. Assessment findings steroids in a client on long-term steroid
1. Fatigue, muscle weakness therapy
2. Anorexia, nausea, vomiting, abdominal pain, B. Assessment findings: severe generalized muscle
weight loss weakness, severe hypotension, hypovolemia,
3. History of frequent hypoglycemic reactions shock (vascular collapse)
4. Hypotension, weak pulse C. Nursing interventions
5. Bronzelike pigmentation of the skin 1. Administer IV fluids (5% dextrose in saline,
6. Decreased capacity to deal with stress plasma) as ordered to treat vascular collapse.
7. Diagnostic tests: low cortisol levels, 2. Administer IV glucocorticoids (hydrocortisone
hyponatremia, hyperkalemia, hypoglycemia [Solu-Cortef]) and vasopressors as ordered.
C. Nursing interventions 3. If crisis precipitated by infection, administer
1. Administer hormone replacement therapy as antibiotics as ordered.
ordered. 4. Maintain strict bed rest and eliminate all forms
a. Glucocorticoids (cortisone, of stressful stimuli.
hydrocortisone): to stimulate diurnal 5. Monitor vital signs, I&O, daily weights.
rhythm of cortisol release, give 23 of dose 6. Protect client from infection.
in early morning and 13 of dose in 7. Provide client teaching and discharge
afternoon planning: same as for Addisons disease.
b. Mineralocorticoids: fludrocortisone acetate
(Florinef)
2. Monitor vital signs. Cushings Syndrome
3. Decrease stress in the environment.
4. Prevent exposure to infection. A. General information
5. Provide rest periods; prevent fatigue. 1. Condition resulting from excessive secretion of
6. Monitor I&O. corticosteroids, particularly the glucocorticoid
7. Weigh daily. cortisol
8. Provide small, frequent feedings of diet high in 2. Occurs most frequently in females between
carbohydrates, sodium, and protein to prevent ages 3060
hypoglycemia and hyponatremia and to 3. Primary Cushings syndrome caused by
provide proper nutrition. adrenocortical tumors or hyperplasia
9. Provide client teaching and discharge 4. Secondary Cushings syndrome (also called
planning concerning: Cushings disease): caused by functioning
a. Disease process; signs of adrenal pituitary or nonpituitary neoplasm secreting
insufficiency ACTH, causing increased secretion of
b. Use of prescribed medications for lifelong glucocorticoids
replacement therapy; never omit 5. Iatrogenic: caused by prolonged use of
medications corticosteroids
c. Need to avoid stress, trauma, and B. Assessment findings
infections, and to notify physician if these 1. Muscle weakness, fatigue, obese trunk with
occur as medication dosage may need to be thin arms and legs, muscle wasting
adjusted 2. Irritability, depression, frequent mood swings
d. Stress management techniques 3. Moon face, buffalo hump, pendulous
e. Diet modification (high in protein, abdomen
carbohydrates, and sodium) 4. Purple striae on trunk, acne, thin skin
f. Use of salt tablets (if prescribed) or 5. Signs of masculinization in women; menstrual
ingestion of salty foods (potato chips) if dysfunction, decreased libido
experiencing increased sweating 6. Osteoporosis, decreased resistance to infection
g. Importance of alternating regular exercise 7. Hypertension, edema
with rest periods 8. Diagnostic tests: cortisol levels increased,
h. Avoidance of strenuous exercise especially slight hypernatremia, hypokalemia,

4
in hot weather hyperglycemia

388 NCLEX-RN Review


53155_04_Ch04c_p378-420.qxd 2/26/09 7:33 AM Page 389

C. Nursing interventions 5. Provide client teaching and discharge


1. Maintain muscle tone. planning concerning
a. Provide ROM exercises. a. Use and side effects of medication if
b. Assist with ambulation. the client is being maintained on
2. Prevent accidents or falls and provide spironolactone therapy
adequate rest. b. Signs of symptoms of
3. Protect client from exposure to infection. hypo/hyperaldosteronism
4. Maintain skin integrity. c. Need for frequent blood pressure checks
a. Provide meticulous skin care. and follow-up care
b. Prevent tearing of skin: use paper tape
if necessary. Pheochromocytoma
5. Minimize stress in the environment.
6. Monitor vital signs; observe for hypertension, A. General information
edema. 1. Functioning tumor of the adrenal medulla that
7. Measure I&O and daily weights. secretes excessive amounts of epinephrine and
8. Provide diet low in calories and sodium and norepinephrine
high in protein, potassium, calcium, and 2. Occurs most commonly between ages 2550
vitamin D. 3. May be hereditary in some cases
9. Monitor urine for glucose and acetone; B. Assessment findings
administer insulin if ordered. 1. Severe headache, apprehension, palpitations,
10. Provide psychologic support and acceptance. profuse sweating, nausea
11. Prepare client for hypophysectomy or 2. Hypertension, tachycardia, vomiting,
radiation if condition is caused by a pituitary hyperglycemia, dilation of pupils, cold
tumor. extremities
12. Prepare client for an adrenalectomy if 3. Diagnostic tests
condition is caused by an adrenal tumor or a. Increased plasma levels of catecholamines;
hyperplasia. elevated blood sugar; glycosuria
13. Provide client teaching and discharge b. Elevated urinary catecholamines and
planning concerning: urinary vanillylmandelic acid (VMA)
a. Diet modifications levels
b. Importance of adequate rest c. Presence of tumor on X-ray
c. Need to avoid stress and infection C. Nursing interventions
d. Change in medication regimen (alternate 1. Monitor vital signs, especially blood pressure.
day therapy or reduced dosage) if cause of 2. Administer medications as ordered to control
the condition is prolonged corticosteroid hypertension.
therapy 3. Promote rest; decrease stressful stimuli.
4. Monitor urine tests for glucose and acetone.
Primary Aldosteronism (Conns Syndrome) 5. Provide high-calorie, well-balanced diet; avoid
stimulants such as coffee, tea.
A. General information 6. Provide care for the client with an
1. Excessive aldosterone secretion from the adrenalectomy as ordered; observe
adrenal cortex postadrenelectomy client carefully for shock
2. Seen more frequently in women, usually due to drastic drop in catecholamine level.
between ages 3050 7. Provide client teaching and discharge
3. Caused by tumor or hyperplasia of adrenal gland planning: same as for adrenalectomy.
B. Assessment findings
1. Headache, hypertension Adrenalectomy
2. Muscle weakness, polyuria, polydipsia,
metabolic alkalosis, cardiac arrhythmias A. General information
(due to hypokalemia) 1. Removal of one or both adrenal glands
3. Diagnostic tests 2. Indications
a. Serum potassium decreased, alkalosis a. Tumors of adrenal cortex (Cushings
b. Urinary aldosterone levels elevated syndrome, hyperaldosteronism) or medulla
C. Nursing interventions (pheochromocytoma)
1. Monitor vital signs, I&O, daily weights. b. Metastatic cancer of the breast or prostate
2. Maintain sodium restriction as ordered. B. Nursing interventions: preoperative
3. Administer spironolactone (Aldactone) and 1. Provide routine pre-op care.
potassium supplements as ordered. 2. Correct metabolic/cardiovascular problems.
4. Prepare the client for an adrenelectomy if a. Pheochromocytoma: stabilize blood

4
indicated. pressure.

ADULT NURSING 389


53155_04_Ch04c_p378-420.qxd 2/26/09 7:33 AM Page 390

b. Cushings syndrome: treat hyperglycemia 3) Genetic defects that prevent synthesis


and protein deficits. of thyroid hormone
c. Primary hyperaldosteronism: treat 3. Low levels of thyroid hormone stimulate
hypertension and hypokalemia. increased secretion of TSH by pituitary; under
3. Administer glucocorticoid preparation on the TSH stimulation the thyroid increases in size
morning of surgery as ordered to prevent acute to compensate and produces more thyroid
adrenal insufficiency. hormone.
C. Nursing interventions: postoperative B. Medical management
1. Provide routine post-op care. 1. Drug therapy
2. Observe for hemorrhage and shock. a. Hormone replacement with levothyroxine
a. Monitor vital signs, I&O. (Synthroid) (T4), dessicated thyroid, or
b. Administer IV therapy and vasopressors as liothyronine (Cytomel) (T3)
ordered. b. Small doses of iodine (Lugols or
3. Prevent infections (suppression of immune potassium iodide solution) for goiter
system makes clients especially susceptible). resulting from iodine deficiency
a. Encourage coughing and deep breathing to 2. Avoidance of goitrogenic foods or drugs in
prevent respiratory infection. sporadic goiter
b. Use meticulous aseptic technique during 3. Surgery: subtotal thyroidectomy (if goiter is
dressing changes. large) to relieve pressure symptoms and for
4. Administer cortisone or hydrocortisone as cosmetic reasons
ordered to maintain cortisol levels. C. Assessment findings
5. Provide general care for the client with 1. Dysphagia, enlarged thyroid, respiratory
abdominal surgery. distress
D. Provide client teaching and discharge planning 2. Diagnostic tests
concerning: a. Serum T4 level low-normal or normal
1. Self-administration of replacement hormones b. RAIU uptake normal or increased
a. Bilateral adrenalectomy: lifelong D. Nursing interventions
replacement of glucocorticoids and 1. Administer replacement therapy as
mineralocorticoids ordered.
b. Unilateral adrenalectomy: replacement 2. Provide care for client with subtotal
therapy for 612 months until the thyroidectomy if indicated.
remaining adrenal gland begins to function 3. Provide client teaching and discharge
normally planning concerning
2. Signs and symptoms of adrenal insufficiency a. Use of iodized salt in preventing and
3. Importance of follow-up care treating endemic goiter
b. Thyroid hormone replacement
Specific Disorders of the Thyroid Gland
Hypothyroidism (Myxedema)
Simple Goiter
A. General information
A. General information 1. Slowing of metabolic processes caused by
1. Enlargement of the thyroid gland not caused hypofunction of the thyroid gland with
by inflammation or neoplasm decreased thyroid hormone secretion; causes
2. Types myxedema in adults and cretinism in children.
a. Endemic: caused by nutritional iodine 2. Occurs more often in women between ages
deficiency, most common in the goiter 30 and 60
belt (midwest, northwest, and Great Lakes 3. Primary hypothyroidism: atrophy of the gland
regions), areas where soil and water are possibly caused by an autoimmune process
deficient in iodine; occurs most frequently 4. Secondary hypothyroidism: caused by
during adolescence and pregnancy decreased stimulation from pituitary TSH
b. Sporadic: caused by: 5. Iatrogenic: surgical removal of the gland or
1) Ingestion of large amounts of goitrogenic overtreatment of hyperthyroidism with drugs
foods (contain agents that decrease or radioactive iodine
thyroxine production): e.g., cabbage, 6. In severe or untreated cases, myxedema coma
soybeans, rutabagas, peanuts, peaches, may occur
peas, strawberries, spinach, radishes a. Characterized by intensification of signs
2) Use of goitrogenic drugs: and symptoms of hypothyroidism and
propylthiouracil, large doses of iodine, neurologic impairment leading to coma
phenylbutazone, para-amino salicylic b. Mortality rate high; prompt recognition

4
acid, cobalt, lithium and treatment essential

390 NCLEX-RN Review


53155_04_Ch04c_p378-420.qxd 2/26/09 7:33 AM Page 391

c. Precipitating factors: failure to take 9. Provide client teaching and discharge


prescribed medications; infection; trauma, planning concerning:
exposure to cold; use of sedatives, a. Thyroid hormone replacement
narcotics, or anesthetics 1) Take daily dose in the morning to
B. Medical management prevent insomnia.
1. Drug therapy: levothyroxine (Synthroid), 2) Self-monitor for signs of
thyroglobulin (Proloid), dessicated thyroid, thyrotoxicosis.
liothyronine (Cytomel) b. Importance of regular follow-up care
2. Myxedema coma is a medical emergency. c. Need for additional protection in cold
a. IV thyroid hormones weather
b. Correction of hypothermia d. Measures to prevent constipation
c. Maintenance of vital functions
d. Treatment of precipitating causes Hyperthyroidism (Graves Disease)
C. Assessment findings
1. Fatigue; lethargy; slowed mental processes; A. General information
dull look; slow, clumsy movements 1. Secretion of excessive amounts of thyroid
2. Anorexia, weight gain, constipation hormone in the blood causes an increase in
3. Intolerance to cold; dry, scaly skin; dry, sparse metabolic processes
hair; brittle nails 2. Overactivity and changes in the thyroid gland
4. Menstrual irregularities; generalized may be present
interstitial nonpitting edema 3. Most often seen in women between ages 3050
5. Bradycardia, cardiac complications (CAD, 4. Cause unknown, but may be an autoimmune
angina pectoris, MI, CHF) process
6. Increased sensitivity to sedatives, narcotics, 5. Symptomatic hyperthyroidism may also be
and anesthetics called thyrotoxicosis
7. Exaggeration of these findings in myxedema B. Medical management
coma: weakness, lethargy, syncope, 1. Drug therapy
bradycardia, hypotension, hypoventilation, a. Antithyroid drugs (propylthiouracil and
subnormal body temperature methimazole ([Tapazole]): block synthesis
8. Diagnostic tests of thyroid hormone; toxic effects include
a. Serum T3 and T4 level low agranulocytosis
b. Serum cholesterol level elevated b. Adrenergic blocking agents (commonly
c. RAIU decreased propranolol [Inderal]): used to decrease
D. Nursing interventions sympathetic activity and alleviate
1. Monitor vital signs, I&O, daily weights; symptoms such as tachycardia
observe for edema and signs of cardiovascular 2. Radioactive iodine therapy
complications. a. Radioactive isotope of iodine (e.g., 131I)
2. Administer thyroid hormone replacement given to destroy the thyroid gland, thereby
therapy as ordered and monitor effects. decreasing production of thyroid hormone
a. Observe for signs of thyrotoxicosis b. Used in middle-aged or older clients who
(tachycardia, palpitations, nausea, are resistant to, or develop toxicity from,
vomiting, diarrhea, sweating, tremors, drug therapy
agitation, dyspnea). c. Hypothyroidism is a potential
b. Increase dosage gradually, especially in complication
clients with cardiac complications. 3. Surgery: thyroidectomy performed in younger
3. Provide a comfortable, warm environment. clients for whom drug therapy has not been
4. Provide a low-calorie diet. effective
5. Avoid the use of sedatives; reduce the dose of C. Assessment findings
any sedative, narcotic, or anesthetic agent by 1. Irritability, agitation, restlessness, hyperactive
half as ordered. movements, tremor, sweating, insomnia
6. Institute measures to prevent skin breakdown. 2. Increased appetite, hyperphagia, weight loss,
7. Provide increased fluids and foods high in diarrhea, intolerance to heat
fiber to prevent constipation; administer stool 3. Exophthalmos (protrusion of the eyeballs),
softeners as ordered. goiter
8. Observe for signs of myxedema coma; provide 4. Warm, smooth skin; fine, soft hair; pliable nails
appropriate nursing care. 5. Tachycardia, increased systolic blood pressure,
a. Administer medications as ordered. palpitations
b. Maintain vital functions: correct 6. Diagnostic tests
hypothermia, maintain adequate a. Serum T3 and T4 levels elevated

4
ventilation. b. RAIU increased

ADULT NURSING 391


53155_04_Ch04c_p378-420.qxd 2/26/09 7:33 AM Page 392

D. Nursing interventions B. Nursing interventions: preoperative


1. Monitor vital signs, daily weights. 1. Ensure that the client is adequately prepared
2. Administer antithyroid medications as for surgery.
ordered. a. Cardiac status is stable.
3. Provide for periods of uninterrupted rest. b. Weight and nutritional status are normal.
a. Assign to a private room away from 2. Administer antithyroid drugs as ordered to
excessive activity. suppress the production and secretion of
b. Administer medications to promote sleep thyroid hormone and to prevent thyroid storm.
as ordered. 3. Administer iodine preparations (Lugols or
4. Provide a cool environment. potassium iodide solution) to reduce the size
5. Minimize stress in the environment. and vascularity of the gland and prevent
6. Encourage quiet, relaxing diversional hemorrhage.
activities. C. Nursing interventions: postoperative
7. Provide a diet high in carbohydrates, protein, 1. Monitor vital signs and I&O.
calories, vitamins, and minerals with 2. Check dressings for signs of hemorrhage;
supplemental feedings between meals and at check for wetness behind neck.
bedtime; omit stimulants. 3. Place client in semi-Fowlers position and
8. Observe for and prevent complications. support head with pillows.
a. Exophthalmos: protect eyes with dark 4. Observe for respiratory distress secondary to
glasses and artificial tears as ordered. hemorrhage, edema of the glottis, laryngeal
b. Thyroid storm: see Thyroid Storm. nerve damage, or tetany; keep tracheostomy
9. Provide client teaching and discharge set, oxygen, and suction nearby.
planning concerning: 5. Assess for signs of tetany due to hypocalcemia
a. Need to recognize and report signs and secondary to accidental removal of
symptoms of agranulocytosis (fever, sore parathyroid glands; keep calcium gluconate
throat, skin rash) if taking antithyroid available (see Hypoparathyroidism).
drugs 6. Encourage the client to rest voice.
b. Signs and symptoms of hyper/ a. Some hoarseness is common.
hypothyroidism b. Check every 3060 minutes for extreme
hoarseness or any accompanying
Thyroid Storm respiratory distress.
7. Observe for thyroid storm due to release of
A. General information excessive amounts of thyroid hormone during
1. Uncontrolled and potentially life-threatening surgery.
hyperthyroidism caused by sudden and 8. Administer IV fluids as ordered until the
excessive release of thyroid hormone into the client is tolerating fluids by mouth.
bloodstream 9. Administer analgesics as ordered for incisional
2. Precipitating factors: stress, infection, pain.
unprepared thyroid surgery 10. Relieve discomfort from sore throat.
3. Now quite rare a. Cool mist humidifier to thin secretions.
B. Assessment findings b. Administer analgesic throat lozenges
1. Apprehension, restlessness before meals and prn as ordered.
2. Extremely high temperature (up to 106F c. Encourage fluids.
[40.7C]), tachycardia, HF, respiratory distress, 11. Encourage coughing and deep breathing every
delirium, coma hour.
C. Nursing interventions (see also Hyperthermia) 12. Assist the client with ambulation: instruct the
1. Maintain a patent airway and adequate client to place hands behind neck to decrease
ventilation; administer oxygen as ordered. stress on suture line if added support
2. Administer IV therapy as ordered. necessary.
3. Administer medications as ordered: 13. Provide client teaching and discharge
antithyroid drugs, corticosteroids, sedatives, planning concerning:
cardiac drugs. a. Signs and symptoms of
hypo/hyperthyroidism
Thyroidectomy b. Self-administration of thyroid hormones
if total thyroidectomy performed
A. General information c. Application of lubricant to the incision
1. Partial or total removal of the thyroid gland once sutures are removed
2. Indications d. Performance of ROM neck exercises
a. Subtotal thyroidectomy: hyperthyroidism 34 times a day

4
b. Total thyroidectomy: thyroid cancer e. Importance of regular follow-up care

392 NCLEX-RN Review


53155_04_Ch04c_p378-420.qxd 2/26/09 7:33 AM Page 393

Specific Disorders of the 9. Provide high-calcium, low-phosphorus diet;


milk and egg yolks are restricted because of
Parathyroid Glands high levels of phosphorus.
Hypoparathyroidism 10. Provide client teaching and discharge
planning concerning:
A. General information a. Medication regimen; oral calcium
1. Disorder characterized by hypocalcemia preparations and vitamin D to be taken
resulting from a deficiency of parathormone with meals to increase absorption
(PTH) production b. Need to recognize and report signs and
2. May be hereditary, idiopathic, or caused symptoms of hypo/hypercalcemia
by accidental damage to or removal of c. Importance of follow-up care with periodic
parathyroid glands during surgery, e.g., serum calcium levels
thyroidectomy
B. Assessment findings Hyperparathyroidism
1. Acute hypocalcemia (tetany)
a. Tingling of fingers and around lips, painful A. General information
muscle spasms, dysphagia, laryngospasm, 1. Increased secretion of PTH that results in an
seizures, cardiac arrhythmias altered state of calcium, phosphate, and bone
b. Chvosteks sign: sharp tapping over facial metabolism
nerve causes twitching of mouth, nose, 2. Most commonly affects women between ages
and eye 3565
c. Trousseaus sign: carpopedal spasm 3. Primary hyperparathyroidism: caused by
induced by application of blood pressure tumor or hyperplasia of parathyroid glands
cuff for 3 minutes 4. Secondary hyperparathyroidism: caused by
2. Chronic hypocalcemia compensatory oversecretion of PTH in
a. Fatigue, weakness, muscle cramps response to hypocalcemia from chronic renal
b. Personality changes, irritability, memory disease, rickets, malabsorption syndrome,
impairment osteomalacia
c. Dry, scaly skin; hair loss; loss of tooth B. Assessment findings
enamel 1. Bone pain (especially at back), bone
d. Tremor, cardiac arrhythmias, cataract demineralization, pathologic fractures
formation 2. Renal colic, kidney stones, polyuria,
e. Diagnostic tests polydipsia
1) Serum calcium levels decreased 3. Anorexia, nausea, vomiting, gastric ulcers,
2) Serum phosphate levels elevated constipation
3) Skeletal X-rays reveal increased bone 4. Muscle weakness, fatigue
density 5. Irritability, personality changes, depression
C. Nursing interventions 6. Cardiac arrhythmias, hypertension
1. Administer calcium gluconate by slow IV drip 7. Diagnostic tests
as ordered for acute hypocalcemia. a. Serum calcium levels elevated
2. Administer medications for chronic b. Serum phosphate levels decreased
hypocalcemia. c. Skeletal X-rays reveal bone
a. Oral calcium preparations: calcium demineralization
gluconate, lactate, carbonate (Os-Cal) C. Nursing interventions
b. Large doses of vitamin D (Calciferol) to 1. Administer IV infusions of normal saline
help absorption of calcium solution and give diuretics as ordered; monitor
c. Aluminum hydroxide gel (Amphogel) or I&O and observe for fluid overload and
aluminum carbonate gel, basic (Basaljel) to electrolyte imbalances.
decrease phosphate levels 2. Assist client with self-care: provide careful
3. Institute seizure and safety precautions. handling, moving, and ambulation to prevent
4. Provide quiet environment free from excessive pathologic fractures.
stimuli. 3. Monitor vital signs; report irregularities.
5. Monitor for signs of hoarseness or stridor; 4. Force fluids; provide acid-ash juices, e.g.,
check for Chvosteks and Trousseaus signs. cranberry juice.
6. Keep emergency equipment (tracheostomy set, 5. Strain urine for stones.
injectable calcium gluconate) at bedside. 6. Provide low-calcium, high-phosphorus
7. For tetany or generalized muscle cramps, may diet.
use rebreathing bag to produce mild 7. Provide care for the client undergoing
respiratory acidosis. parathyroidectomy (see Thyroidectomy).

4
8. Monitor serum calcium and phosphate levels.

ADULT NURSING 393


53155_04_Ch04c_p378-420.qxd 2/26/09 7:33 AM Page 394

8. Provide client teaching and discharge d. Polyphagia (hunger and increased appetite)
planning concerning results from cellular starvation.
a. Need to engage in progressive ambulatory e. The body turns to fats and protein for
activities energy; but in the absence of glucose in the
b. Increased intake of fluids cell, fats cannot be completely metabolized
c. Use of calcium preparations and and ketones (intermediate products of fat
importance of high-calcium diet following metabolism) are produced.
a parathyroidectomy f. This leads to ketonemia, ketonuria
(contributes to osmotic diuresis), and
metabolic acidosis (ketones are acid bodies).
Specific Disorders of the Pancreas g. Ketones act as CNS depressants and can
Diabetes Mellitus cause coma.
h. Excess loss of fluids and electrolytes leads
A. General information to hypovolemia, hypotension, renal failure,
1. Diabetes mellitus represents a heterogenous and decreased blood flow to the brain
group of chronic disorders characterized by resulting in coma and death unless treated.
hyperglycemia. 8. Acute complications of diabetes include
2. Hyperglycemia is due to total or partial insulin diabetic ketoacidosis (see Ketoacidosis),
deficiency or insensitivity of the cells to insulin reaction (see Insulin Reaction/
insulin. Hypoglycemia), hyperglycemic hyperosmolar
3. Characterized by disorders in the metabolism nonketotic coma (see Hyperglycemic
of carbohydrate, fat, and protein, as well as Hypersmolar Coma (HHNK)).
changes in the structure and function of blood B. Medical management
vessels. 1. Type 1: insulin, diet, exercise
4. Most common endocrine problem; affects over 2. Type 2: ideally managed by diet and exercise;
20 million people in the United States. may need oral hypoglycemics or occasionally
5. Exact etiology unknown; causative factors may insulin if diet and exercise are not effective in
include: controlling hyperglycemia; insulin needed for
a. Genetics, viruses, and/or autoimmune acute stresses, e.g., surgery, infection
response in Type 1 3. Diet Exchange (see Appendix)
b. Genetics and obesity in Type 2 a. Type 1: consistency is imperative to avoid
6. Types hypoglycemia
a. Type 1 (insulin-dependent diabetes b. Type 2: weight loss is important because it
mellitus [IDDM]) decreases insulin resistance
1) Secondary to destruction of beta cells c. High-fiber, low-fat diet also recommended
in the islets of Langerhans in the d. Utilize Exchange list as recommended
pancreas resulting in little or no insulin from American Diabetes Association.
production; requires insulin injections. 4. Drug therapy
2) Usually occurs in children (see Unit 5) a. Insulin: used for Type 1 diabetes (also
or in nonobese adults. occasionally used in Type 2 diabetes)
b. Type 2 (non-insulin-dependent diabetes 1) Types (Table 4-25)
mellitus [NIDDM]) a) Short acting: used in treating
1) May result from a partial deficiency of ketoacidosis; during surgery,
insulin production and/or an infection, trauma; management of
insensitivity of the cells to insulin. poorly controlled diabetes; to
2) Usually occurs in obese adults over 40. supplement longer-acting insulins
c. Diabetes associated with other conditions or b) Intermediate: used for
syndromes, e.g., pancreatic disease, Cushings maintenance therapy
syndrome, use of certain drugs (steroids, c) Long acting: used for maintenance
thiazide diuretics, oral contraceptives). therapy in clients who experience
7. Pathophysiology hyperglycemia during the night
a. Lack of insulin causes hyperglycemia with intermediate-acting insulin
(insulin is necessary for the transport of 2) Various preparations of short-,
glucose across the cell membrane). intermediate-, and long-acting insulins
b. Hyperglycemia leads to osmotic diuresis as are available (see Table 4-26)
large amounts of glucose pass through the 3) Insulin preparations can consist of a
kidney; results in polyuria and glycosuria. mixture of beef and pork insulin, pure
c. Diuresis leads to cellular dehydration and beef, pure pork, or human insulin.
fluid and electrolyte depletion causing Human insulin is the purest insulin

4
polydipsia (excessive thirst). and has the lowest antigenic effect.

394 NCLEX-RN Review


53155_04_Ch04c_p378-420.qxd 2/26/09 7:33 AM Page 395

Table 4-26 Types of Insulins

TYPE ONSET PEAK DURATION


Humalog Less than 30-90 4 hours
(Lispro) 15 minutes minutes
Regular 30-60 2-4 hours 5-7 hours
minutes
NPH 3-4 hours 6-12 hours 18-28 hours
Lente 1-3 hours 8-12 hours 18-28 hours
Ultra Lente 4-6 hours 18-24 hours 36-hours
70/30 15-30 2-3 hours and 18-24 hours
minutes 8-12 hours
Insulin 1.1 hour 5 hours 24 hours
glargine
Adapted from, Broyles, B. E. Reiss, B.S., & Evans, M.E. (2007).
Pharmacological aspects of nursing care (7th ed.). New York:
Thomson Delmar Learning.

4) Human insulin is recommended for all


newly diagnosed Type 1 diabetics,
Type 2 diabetics who need short-term Figure 4-26 Insulin infusion pump
insulin therapy, the pregnant client,
and diabetic clients with insulin
allergy or severe insulin resistance.
5) Insulin pumps are small, externally 3. Type 2: obesity; frequently no other symptoms
worn devices that closely mimic 4. Diagnostic tests
normal pancreatic functioning. Insulin a. Fasting blood sugar
pumps contain a 3-mL syringe attached 1) A level of 126 mg/dL or greater on at
to a long (42-inch), narrow-lumen tube least two occasions confirms diabetes
with a needle or Teflon catheter at the mellitus
end. The needle or Teflon catheter is 2) May be normal in Type 2 diabetes
inserted into the subcutaneous tissue b. Postprandial blood sugar: elevated greater
(usually on the abdomen) and secured than 200 mg/dL 2 hours after a meal.
with tape or a transparent dressing. c. Oral glucose tolerance test (most sensitive
The needle or catheter is changed at test): elevated greater than 200 mg/dL
least every 3 days. The pump is worn d. Glycosolated hemoglobin (hemoglobin
either on a belt or in a pocket (see A1c) elevated; (normal range 46%)
Figure 4-26). The pump uses only D. Nursing interventions
regular insulin. Insulin can be 1. Administer insulin or oral hypoglycemic
administered via the basal rate (usually agents as ordered; monitor for hypoglycemia,
0.52.0 units/hr) and by a bolus dose especially during period of drugs peak action.
(which is activated by a series of 2. Provide special diet as ordered.
button pushes) prior to each meal. a. Ensure that the client is eating all meals.
b. Oral hypoglycemic agents (Table 4-27) b. If all food is not ingested, provide
1) Used for Type 2 diabetics who are not appropriate substitutes according to the
controlled by diet and exercise exchange lists or give measured amount of
2) Increase the ability of islet cells of the orange juice to substitute for leftover food;
pancreas to secrete insulin; may have provide snack later in the day.
some effect on cell receptors to 3. Monitor urine sugar and acetone (freshly
decrease resistance to insulin voided specimen).
5. Exercise: helpful adjunct to therapy as exercise 4. Perform finger sticks to monitor blood glucose
decreases the bodys need for insulin. levels as ordered (more accurate than urine tests).
C. Assessment findings 5. Observe for signs of hypo/hyperglycemia.
1. All types: polyuria, polydipsia, polyphagia, 6. Provide meticulous skin care and prevent
fatigue, blurred vision, susceptibility to infection injury.

4
2. Type 1: anorexia, nausea, vomiting, weight loss 7. Maintain I&O; weigh daily.

ADULT NURSING 395


53155_04_Ch04c_p378-420.qxd 2/26/09 7:33 AM Page 396

Table 4-27 Oral Hypoglycemic Agents

Onset of Peak Action Duration of


Drug Action (hrs) (hrs) Action (hrs) Comments
Oral Sulfonylureas
Acetohexamide (Dymelor) 1 46 1224
Chlorpropamide (Diabinase) 1 46 4060
Glyburide (Micronase 0.251 hr 28 1024
Diabeta)
Oral Biguanides
Metformin (Glucophage) 22.5 1016 Decreases glucose production in liver;
decreases intestinal absorption of
glucose and improves insulin
sensitivity.
Oral Alpha-glucosidose Inhibitor
Acarbose (Precose) Unknown 1 Unknown Delay glucose absorption and
Miglitol (Glyset) 23 digestion of carbohydrates,
Troglitazone (Rezulin) Rapid 23 Unknown lowering blood sugar. Reduces
plasma glucose and insulin. Exact
mechanism is unknown. Potentiates
action of insulin in skeletal muscle
and decreases glucose production
in liver.

8. Provide emotional support; assist client in b) Gently roll vial between palms of
adapting to change in lifestyle and body image. hands.
9. Observe for chronic complications and plan c) Draw up insulin using sterile
care accordingly. technique.
a. Macrovascular: changes in large vessels d) If mixing insulins, draw up clear
1) Atherosclerosis: increased plaque insulin before cloudy insulin.
formation (decrease lipid level) 2) Injection technique
2) Cardiovascular, cerebral, peripheral a) Systematically rotate sites to
vascular diseases (modify lifestyle- prevent lipodystrophy
obesity, smoking, sedentary lifestyle, (hypertrophy or atrophy of tissue).
hypertension) b) Insert needle at a 45 or 90 angle
b. Microvascular: thickening of capillaries depending on amount of adipose
and arterioles tissue.
1) Diabetic retinopathy: premature 3) May store current vial of insulin at
cataracts (prevent/control elevated room temperature; refrigerate extra
blood glucose) supplies.
2) Diabetic nephropathy: renal disease 4) Provide many opportunities for return
(control blood glucose) demonstration.
3) Diabetic neuropathies: peripheral, d. Oral hypoglycemic agents
autonomic spinal nerves affected 1) Stress importance of taking the drug
(control blood glucose) regularly.
10. Provide client teaching and discharge 2) Avoid alcohol intake while on
planning concerning medication.
a. Disease process e. Urine testing (not very accurate reflection
b. Diet of blood glucose level)
1) Client should be able to plan meals 1) May be satisfactory for Type 2
using exchange lists before discharge diabetics since they are more stable.
2) Emphasize importance of regularity of 2) Use Clinitest, Tes-Tape, Diastix for
meals; never skip meals glucose testing.
c. Insulin 3) Perform tests before meals and at
1) How to draw up into syringe bedtime.
a) Use insulin at room temperature. 4) Use freshly voided specimen.

4 396 NCLEX-RN Review


53155_04_Ch04c_p378-420.qxd 2/26/09 7:33 AM Page 397

5) Be consistent in brand of urine test used. j. Complications


6) Report results in percentages. 1) Learn to recognize signs and
7) Report results to physician if results symptoms of hypo/hyperglycemia.
are greater than 1%, especially if 2) Eat candy or drink orange juice with
experiencing symptoms of sugar added for insulin reaction
hyperglycemia. (hypoglycemia).
8) Urine testing for ketones should be k. Need to wear a Medic-Alert bracelet.
done by Type 1 diabetics when there is
persistent glycosuria, increased blood
Ketoacidosis (DKA)
glucose levels, or if the client is not
feeling well (Acetest, Ketostix). A. General information
f. Blood glucose monitoring 1. Acute complication of diabetes mellitus
1) Use for Type 1 diabetics since it gives characterized by hyperglycemia and
exact blood glucose level and also accumulation of ketones in the body; causes
detects hypoglycemia. metabolic acidosis
2) Use for Type 2 diabetics to monitor 2. Occurs in insulin-dependent diabetic clients
effectiveness of oral and/or insulin 3. Precipitating factors: undiagnosed diabetes,
treatment. neglect of treatment; infection, cardiovascular
3) Instruct client in finger-stick disorder; other physical or emotional stress
technique, use of monitor device (if 4. Onset slow, may be hours to days
used), and recording and utilization B. Assessment findings
of test results. 1. Polydipsia, polyphagia, polyuria
g. General care 2. Nausea, vomiting, abdominal pain
1) Perform good oral hygiene and have 3. Skin warm, dry, and flushed
regular dental exams. 4. Dry mucous membranes; soft eyeballs
2) Have regular eye exams. 5. Kussmauls respirations or tachypnea; acetone
3) Care for sick days (e.g., cold or flu) breath
a) Do not omit insulin or oral 6. Alterations in LOC
hypoglycemic agents because 7. Hypotension, tachycardia
infection causes increased blood 8. Diagnostic tests
sugar. a. Serum glucose and ketones elevated
b) Notify physician. (glucose greater than 250 mg/dL)
c) Monitor urine or blood glucose b. BUN, creatinine, hct elevated (due to
levels and urine ketones dehydration)
frequently. c. Serum sodium decreased, potassium
d) If nausea and/or vomiting occurs, (may be normal or elevated at first)
sip on clear liquids with simple d. ABGs: metabolic acidosis with
sugars. compensatory respiratory alkalosis
h. Foot care C. Nursing interventions
1) Wash feet with mild soap and water 1. Maintain a patent airway.
and pat dry. 2. Maintain fluid and electrolyte balance.
2) Apply lanolin to feet to prevent drying a. Administer IV therapy as ordered.
and cracking. 1) Normal saline (0.9% NaCl), then
3) Cut toenails straight across. hypotonic (0.45% NaCl) sodium
4) Avoid constricting garments such as chloride
garters. 2) When blood sugar drops to 250 mg/dL,
5) Wear clean, absorbent socks (cotton may add 5% dextrose to IV.
or wool). 3) Potassium will be added when the
6) Purchase properly fitting shoes and urine output is adequate.
break new shoes in gradually. b. Observe for fluid and electrolyte
7) Never go barefoot. imbalances, especially fluid overload,
8) Inspect feet daily and notify physician hypokalemia, and hyperkalemia.
if cuts, blisters, or breaks in skin occur. 3. Administer insulin as ordered.
i. Exercise a. Regular insulin IV (drip or push) and/or
1) Undertake regular exercise; avoid subcutaneously (SC).
sporadic, vigorous exercise. b. If given IV drip, give with small amounts of
2) Food intake may need to be increased albumin since insulin adheres to IV tubing.
before exercising. c. Monitor blood glucose levels frequently.
3) Exercise is best performed after meals 4. Check urine output every hour.

4
when the blood sugar is rising. 5. Monitor vital signs.

ADULT NURSING 397


53155_04_Ch04c_p378-420.qxd 2/26/09 7:33 AM Page 398

6. Assist client with self-care.


7. Provide care for the unconscious client if in a Sample Questions
coma.
8. Discuss with client the reasons ketosis
developed and provide additional diabetic 425. A client was admitted to the hospital for
teaching if indicated. uncontrolled diabetes. Lab studies reveal a
fasting blood sugar of 310 mg/dL. The client
Insulin Reaction/Hypoglycemia is informed that Type 1 diabetes has a
characteristic feature. What is it?
A. General information
1. It is associated with the destruction of beta
1. Abnormally low blood sugar, usually below
50 mg/dL cells.
2. Usually caused by insulin overdosage, too 2. It usually causes complete fat metabolism.
little food, nutritional and fluid imbalances 3. It often occurs in obese individuals.
from nausea and vomiting, excessive exercise 4. It is rarely controlled.
3. Onset rapid; may develop in minutes to hours
B. Assessment findings 426. A diabetic client who is taking regular and NPH
1. Headache, dizziness, difficulty with problem insulin asks why she must mix the two insulins.
solving, restlessness, hunger, visual What would be the explanation for mixing
disturbances regular and NPH insulin?
2. Slurred speech; alterations in gait; decreasing
1. Immediate onset of the regular insulin.
LOC; pallor, cold, clammy skin; diaphoresis
3. Diagnostic test: serum glucose level 2. Onset of the regular insulin within 2 hours.
5060 mg/dL or lower 3. A peak action of the NPH insulin at 2 hours.
C. Nursing interventions 4. A total duration of action of 24 hours.
1. Administer oral sugar in the form of candy or
orange juice with sugar added if the client is alert. 427. A female client has a simple goiter. She is being
2. If the client is unconscious, administer seen by the community health nurse for teaching
2050 mL 50% dextrose IV push, or 1 mg and follow-up regarding nutritional deficiencies
glucagon IM, IV, or SC, as ordered. related to her goiter. The clients problems are
3. Explore with client reasons for hypoglycemia most likely associated with which nutritional
and provide additional diabetic teaching as deficiency?
indicated.
1. Calcium.
2. Iodine.
Hyperglycemic Hyperosmolar 3. Iron.
Nonketotic Coma (HHNK)
4. Sodium.
A. General information
1. Complication of diabetes, characterized by 428. The nurse is teaching a woman who has a
hyperglycemia and a hyperosmolar state simple goiter. The nurse teaches the client that
without ketosis to enhance glandular function, she should
2. Occurs in non-insulin-dependent diabetics or eliminate which of the following foods?
nondiabetic persons (typically elderly clients) 1. Corn.
3. Precipitating factors: undiagnosed diabetes;
infections or other stress; certain medications 2. Milk.
(e.g., Dilantin, thiazide diuretics); dialysis; 3. Turnips.
hyperalimentation; major burns; pancreatic 4. Watermelon.
disease
B. Assessment findings 429. A client is admitted to the hospital with
1. Similar to ketoacidosis but without Kussmaul Addisons disease. He has a respiratory
respirations and acetone breath infection. When the clients vital signs are
2. Laboratory tests assessed, his blood pressure is 90/40. Why
a. Blood glucose level extremely elevated should the nurse notify the physician
(greater than 600 mg/dL) immediately?
b. BUN, creatine, hct elevated (due to 1. Blood gases need to be drawn.
dehydration)
c. Urine positive for glucose 2. Seizure activity is imminent.
C. Nursing interventions: treatment and nursing care 3. Shock may be developing.
is similar to DKA, excluding measures to treat 4. The reading is atypical.

4
ketosis and metabolic acidosis.

398 NCLEX-RN Review


53155_04_Ch04c_p378-420.qxd 2/26/09 7:33 AM Page 399

430. A client who is diagnosed with Addisons 435. A client is to have the following diagnostic
disease is admitted to the hospital. Which of the procedures: serum T3 and T4, carotid
following would the nurse expect to find when arteriogram, and thyroid scan. In what order
assessing the client? should the nurse schedule the tests?
1. Acne. 1. Arteriogram, serum T3 and T4, scan.
2. Hyperpigmentation. 2. Serum T3 and T4, scan, arteriogram.
3. Moon face. 3. Arteriogram, scan, serum T3 and T4.
4. Supraclavicular fat pads. 4. Serum T3 and T4, arteriogram, scan.

431. A client who is diagnosed as having Addisons 436. After reading about the procedure for his
disease is receiving teaching about his disease upcoming thyroid scan, a client expresses
from the nurse. Which statement the client concern about the dangers of being radioactive
makes indicates to the nurse that he understands after the test. Which understanding about the
the teaching? test should guide the nurses response?
1. I should avoid strenuous exercise during hot 1. There is no danger because the thyroid scan
weather. no longer involves the use of a radioactive
2. I should not eat salty foods. isotope.
3. I need to take medication only when I am 2. The radioactive isotope is only a tracer dose,
having symptoms. which is not harmful to the client or others
4. I should eat foods such as bananas and close to him.
oranges several times daily. 3. The client must avoid close contact with
others for 5 days following the test.
432. The nurse is teaching a person who has 4. Wearing a lead shield during the test will
Addisons disease about drug therapy for his protect the client from radioactivity.
condition. In evaluating the effectiveness of
teaching regarding drug therapy, what should 437. The nurse is explaining to a client about a
the client know and be able to verbalize? radioactive iodine uptake test. Which of the
1. To avoid antibiotics. following OTC medications should the nurse
2. For lifelong therapy. advise the client to avoid prior to the test?
3. To taper the steroid dose. 1. Antiflatulents.
4. To receive alternate-day therapy. 2. Poison ivy remedies.
3. Cough syrups.
433. A client is newly diagnosed with Type 1 4. Antifungal agents.
diabetes. She is hospitalized for insulin dose
stabilization and is being taught insulin 438. Select the most accurate explanation by the nurse
administration and self-monitoring of blood to a client who is to have an oral glucose tolerance
glucose (SMBG) levels. What is the major benefit test and needs to understand the procedure.
of self-monitoring of blood glucose levels? 1. You will go to the laboratory and your blood
1. Blood glucose is maintained at close to will be drawn.
normal levels. 2. After you drink a concentrated glucose
2. Materials and laboratory expenses are cost solution, you cannot eat or drink anything
efficient. until your blood is drawn.
3. Dependence on the health care system is 3. You will eat a large meal and your blood
reduced. will be drawn 2 hours later.
4. Larger but fewer doses of insulin are required. 4. Your blood will be drawn, you will drink
a concentrated glucose solution, and your
434. The nurse is teaching an adult client who has blood will be drawn again.
Type 1 diabetes mellitus about ketoacidosis.
What is the primary cause for the development 439. An adult is suffering from adrenocortical
of ketoacidosis? insufficiency and is placed on glucocorticoid
1. A GI disturbance. therapy. The nurse plans to include which of the
2. An insulin overdosage. following administration directions?
3. Omitted meals. 1. You will need to take the large dose of the
medication at bedtime and the smaller dose in
4. Not taking insulin regularly.

4
the morning until the prescription is finished.

ADULT NURSING 399


53155_04_Ch04c_p378-420.qxd 2/26/09 7:33 AM Page 400

2. You will need to take the medication at 1. Breath sounds.


bedtime for life. 2. Capillary refill.
3. You will need to take the medication in the 3. Blood pressure.
morning until the prescription is finished. 4. Skin integrity.
4. You will need to take the large dose of the
medication in the morning and the smaller 445. An adult is readmitted to the medical/surgical
dose in the afternoon for life. care unit in addisonian crisis. He is exhibiting
signs of tachycardia, dehydration, hyponatremia,
440. Which assessment is most important for the hyperkalemia, and hypoglycemia. What will the
nurse to make when monitoring a client with a initial orders for this client include?
pituitary tumor that secretes ACTH? 1. Administration of oxygen via 100%
1. Height. nonrebreathing mask.
2. Blood pressure. 2. Starting an IV solution of saline and dextrose.
3. Pulse rate. 3. Administering potassium chloride.
4. Output. 4. Preparing for an emergency tracheostomy.
441. The nurse is caring for a client who underwent 446. A female client is suffering from Cushings
surgical hypophysectomy. Which of the syndrome. She is constantly lashing out at her
following assessments is most essential for the coworkers and family. Her husband informs the
nurse to make immediately post-op? nurse of this behavior. What condition does the
1. Blood pressure. nurse interpret from this behavior?
2. Serum calcium levels. 1. Mineralocorticoid excess.
3. Patent catheter. 2. Glucocorticoid excess.
4. Bowel sounds. 3. Activity intolerance.
4. Sensory-perceptual alterations.
442. An adult has had a hypophysectomy with a
complete removal of the pituitary gland. Which 447. A nurse at a weight loss center assesses a client
of the following statements represents to the who has a large abdomen and a rounded face.
nurse the most complete understanding of Which additional assessment finding would lead
follow-up care? the nurse to suspect that the client has Cushings
1. I will need to wear a Medic-Alert bracelet. syndrome rather than obesity from imbalance of
2. I will need to take hormone replacements food intake and body need?
for the next 2 months. 1. Large thighs and upper arms.
3. I will need to wear a Medic-Alert bracelet and 2. Pendulous abdomen and large hips.
take hormone replacements for the next year. 3. Abdominal striae and ankle enlargement.
4. I will need to have lifelong follow-up, to 4. Posterior neck fat pad and thin extremities.
take hormone replacement therapy for the
rest of my life, and to wear a Medic-Alert 448. A client has primary aldosteronism. Which
bracelet. assessment findings would the nurse expect to
find initially?
443. A client is admitted with a posterior pituitary 1. Decreased serum sodium and potassium.
tumor and is experiencing diabetes insipidus, a
2. Decreased blood glucose and elevated
complication of that tumor. Which nursing
temperature.
diagnosis is most appropriate for this client?
3. Tachycardia and albuminuria.
1. Fluid volume excess.
4. Hypertension and decreased serum potassium.
2. Deficient fluid volume.
3. Bowel incontinence. 449. A client who is suspected of having a
4. Diarrhea. pheochromocytoma complains of sweating,
palpitations, and headache. Which assessment is
444. A client who has been taking prednisone to treat essential for the nurse to make first?
lupus erythematosus has discontinued the 1. Pupil reaction.
medication because of lack of funds to buy the
2. Hand grips.
drug. When the nurse becomes aware of the
situation, which assessment is most important 3. Blood pressure.
4. Blood glucose.

4
for the nurse to make first?

400 NCLEX-RN Review


53155_04_Ch04c_p378-420.qxd 2/26/09 7:33 AM Page 401

450. An adult is to have a bilateral adrenalectomy. 455. An adult who is newly diagnosed with Graves
The nurse is performing preoperative teaching. disease asks the nurse Why do I need to take
The client asks the nurse What will I look like propranolol (Inderal)? Based on the nurses
after surgery? What is the nurses best response? understanding of the medication and Graves
1. Dont worry about that now. You need to disease, what would be the best response?
concentrate on the surgery. 1. The medication will limit thyroid hormone
2. You will only have a small incision. secretion.
3. I know you are worried, maybe we should 2. The medication will inhibit synthesis of
resume the education session later. thyroid hormones.
4. Youre appearance wont change 3. The medication will relieve the symptoms
immediately after surgery. of Graves disease.
4. The medication will increase the synthesis
451. An adult has undergone a bilateral of thyroid hormones.
adrenalectomy. Which of the following
demonstrates to the nurse the best 456. The nurse is preparing a room to receive a client
understanding of long-term care needs? immediately post-thyroidectomy. The nurse
1. When I run out of the medication the doctor should be sure that which of the following
gave me, I can stop taking the hormones. equipment is available at the bedside?
2. I can take the steroid replacement therapy 1. Nasogastric tray.
once every 3 days. 2. Central venous tray set-up.
3. I need to take the steroid replacement therapy 3. Tracheostomy tray.
every day. I should not alter the dose or stop 4. Lumbar puncture tray.
taking it.
4. I can take the dose of the medication when 457. An adult had a total thyroidectomy. Which
I feel stressed. statement by the client demonstrates to the
nurse an adequate understanding of long-term
452. An adult is admitted to the hospital for removal care?
of a simple goiter. What is the cause of a simple 1. I will need to take replacement hormones
goiter? for the rest of my life.
1. Low intake of fat-free foods. 2. I should try to avoid stress and be alert for
2. Excessive thyroid-stimulating hormone (TSH) signs of recurrent hyperthyroidism.
stimulation. 3. Thank goodness this is over, I will never
3. Excessive adrenocorticotropic hormone have to worry about weight problems again!
(ACTH) stimulation. 4. I should increase my caloric intake to
4. Low intake of goitrogenic foods. replace what I lost during the surgery.
453. An adult is currently being treated at the clinic for 458. The nurse is caring for a client who is status
Graves disease. It is essential for the nurse to assess post-thyroidectomy. The client is exhibiting
for which of the following signs immediately? hyperreflexia, muscle twitching, and spasms.
1. Goiter. What is the first action the nurse should
2. Tachycardia. perform?
3. Constipation. 1. Assess for additional signs of tetany.
4. Hypothermia. 2. Prepare to send a blood sample to the
laboratory for a calcium level.
454. A 35-year-old female visits her managed care 3. Place the client in semi-Fowlers position.
physician for an annual physical examination.
4. Administer post-op pain medication.
Routine laboratory studies reveal thyroxine (T4)
and triiodothyronine (T3) levels are elevated, 459. An adult who has Graves disease just received a
whereas the thyroid-stimulating hormone (TSH) dose of sodium 131I. Which of the following
level was undetectable. What condition would statements made to the nurse best demonstrates
the nurse suspect? an understanding of immediate care needs?
1. Hypothyroidism. 1. I should be able to go home after about
2. Addisonian crisis. 2 hours if I dont have any vomiting.
3. Hypoparathyroidism. 2. I have my belongings with me to stay in the

4
4. Hyperthyroidism. isolation room for the next 24 hours.

ADULT NURSING 401


53155_04_Ch04c_p378-420.qxd 2/26/09 7:33 AM Page 402

3. My daughter is pregnant, so I told her I will 464. The nurse is attending a bridal shower for a
not be able to see her for the next month. friend when another guest starts to tremble and
4. I brought my antithyroid drug with me so complains of dizziness. The nurse notices a
I will not miss a dose. medical alert bracelet for diabetes. What will be
the nurses best action?
460. An adult has had hypoparathyroidism for 20 years. 1. Encourage the guest to eat some baked ziti.
The client has come in to the center for a check- 2. Call the guests personal physician.
up. For what condition should the nurse assess?
3. Offer the guest a peppermint.
1. Hypothermia.
4. Give the guest a glass of orange juice.
2. Hyperthermia.
3. Tetany. 465. A woman usually administers her NPH insulin
4. Hypertension. at 0600. but she plans to attend a banquet and
fashion show next week, at which lunch will be
461. A client who is newly diagnosed with Type 1 served at 1400. rather than noon when she
diabetes asks the nurse Why cant I take a pill usually eats lunch. Which of the following
for my diabetes like my neighbor? Select the statements demonstrates to the nurse an
statement that states the primary difference understanding of peak action of NPH and risk
between Type 1 and Type 2 diabetes. for hypoglycemia?
1. Type 1 diabetes and Type 2 diabetes can be 1. I will administer the insulin at my regular
controlled with injections of antibodies. time, it is important to adhere to my
2. Type 1 diabetes is the result of autoimmune schedule.
destruction of beta cell function in the 2. I will take the insulin at 0800. that day, as
pancreas, whereas Type 2 diabetes is the the insulin peaks in 612 hours.
result of the lack of responsiveness of beta 3. I will not take any insulin until they serve
cells to insulin. the lunch at the banquet.
3. Type 1 diabetes insulin production is a 4. I will take the insulin at 1000. that day as
circadian function, whereas in Type 2 the peak action of NPH is 4 hours after
diabetes, insulin production depends on administered.
serum glucose levels.
4. Type 1 diabetes has a complication known as 466. A man is hospitalized for an infected foot ulcer. At
hyperglycemic hyperosmolar nonketosis, 1100 his blood glucose is 460 mg/dL and he has
whereas Type 2 diabetes has a complication been up to the bathroom seven times this morning
known as diabetic ketoacidosis. to urinate. What will be the nurses best action?
1. Administer regular insulin according to the
462. The nurse administers the clients morning dose physicians sliding scale order.
of regular insulin at 0730. The nurse should 2. Administer NPH insulin according to the
anticipate to observe the client for a hypoglycemic physicians sliding scale order.
reaction at which of the following times?
3. Notify the physician.
1. Immediately.
4. Make sure the clients urinal is close to the
2. 1000. bed so he does not have to keep getting up.
3. 1300.
4. 1930. 467. A client with diabetes is displaying signs of
irritability and irrational behavior during an
463. The nurse is planning an education session for a office visit. The nurse observes visible tremors
client newly diagnosed with diabetes. Which in the clients hands. Based on the clients
concept is essential to include when developing history, how are these findings interpreted?
the plan of care? 1. Hyperglycemia.
1. All diabetic teaching needs to be accomplished 2. Diabetic ketoacidosis (DKA).
within 20 hours before discharge. 3. Hyperglycemic hyperosmolar nonketosis
2. Insulin injection sites should be cleaned with (HHNK).
iodine prior to injection. 4. Hypoglycemia.
3. Snacks should be ingested prior to physical
exercise. 468. Which statement by a woman newly diagnosed
4. Urine sugar levels should be checked prior to with NIDDM demonstrates to the nurse an
adequate understanding of dietary needs?

4
insulin administration.

402 NCLEX-RN Review


53155_04_Ch04c_p378-420.qxd 2/26/09 7:33 AM Page 403

1. I will increase my intake of fat and 3. I used to take a shower every other night but
carbohydrates. now I am going to wash and examine my feet
2. Having diabetes makes it harder for my every night.
system to digest food. 4. I have a corn on my left foot, so I am going
3. I met with the dietician who said to eat to go to the pharmacy to get something for it
carbohydrates, protein, and fats together. right away.
4. When will I start the TPN medication to
473. A client took her usual NPH insulin dose at
control my sugar?
0600, her lunch is delayed until 1300, and she
469. The client came to the diabetic clinic for follow- begins to feel weak. Which of the following
up teaching on the complications of diabetes. actions by the client demonstrates an
What is a correct explanation for the result of understanding of her condition?
neuropathy? 1. Administers an extra 4 units of regular insulin.
1. Microangiopathies or metabolic defects that 2. Administers an additional 4 units of NPH
cause by-products to accumulate in the nerve insulin.
tissues. 3. Takes a nap.
2. Microvascular damage to the retina. 4. Drinks a cup of milk and then eats her lunch.
3. Macroangiopathy in the extremities.
474. The nurse is teaching a client about diabetic
4. End-stage renal disease.
management. The client asks the nurse What is
470. The nurse is teaching a client with Type 2, non- a hemoglobin A1c test? What is the most
insulin-dependent diabetes about the acute appropriate answer by the nurse?
metabolic complications. Why does a client with 1. The hemoglobin A1c is a blood test that
Type 2 diabetes usually not develop diabetic evaluates glycemic control over a 3-month
ketoacidosis? time period by measuring the glucose
1. There is no insulin available for the state of attached to hemoglobin.
hyperglycemia. 2. The hemoglobin A1c is a blood test that
2. The client with Type 2 diabetes has no measures the glucose attached to hemoglobin
protein or fat reserves. molecule over the last 7 days.
3. There is sufficient insulin to prevent the 3. The hemoglobin A1c test is a kidney test
breakdown of protein and fatty acid for that measures protein to evaluate glucose
metabolic needs. control over the last 7 days.
4. There is insufficient serum glucose 4. The hemoglobin A1c test is a urine test that
concentrations. measures protein to evaluate glucose control
over the last few months.
471. The primary caretaker for a man who was
recently started on an oral hypoglycemic agent is
his wife. The wife should know to watch for
which of the following symptoms of
hypoglycemia? Answers and Rationales
1. High blood sugar readings (greater than
250 mg/dL). 425. 1. Type 1 diabetes mellitus, also know as
2. Presence of ketones in the urine. insulin-dependent diabetes, occurs in
3. Significant increase in urine output. individuals in whom the beta islet cells of the
4. Cold sweats, weakness, and trembling. pancreas do not make insulin.

472. Which of the following statements by a person 426. 4. NPH insulin is an intermediate-acting insulin.
who has diabetes mellitus shows the nurse that Regular insulin is a rapid-acting insulin. Mixing
he has an adequate understanding of special foot the two gives insulin over a 24-hour period,
care needs? requiring fewer injections for the client.
1. I am looking forward to the summer when 427. 2. Lack of iodine in the diet is a primary
I can go barefoot in my house and at the contributor to the development of simple goiter.
beach.
2. I like to use a heating pad at night as 428. 3. Turnips belong to a classification of foods
called exogenous goitrogens. Goitrogens are

4
I always have cold feet.

ADULT NURSING 403


53155_04_Ch04c_p378-420.qxd 2/26/09 7:33 AM Page 404

thyroid-inhibiting substances and therefore 439. 4. Adrenal insufficiency requires lifelong


should be avoided. Other goitrogens include glucocorticoid replacement therapy.
rutabagas, cabbage, soybeans, peanuts, peaches, Glucocorticoids are given in divided doses, two-
peas, strawberries, spinach, and radishes. thirds in the morning and one-third in the
afternoon to reflect the bodys own circadian
429. 3. Any emotional or physical stress, such as rhythm, which decreases the side effects of
infection, may precipitate acute adrenal crisis therapy.
with vascular collapse and shock.
440. 2. ACTH-secreting tumors can cause Cushings
430. 2. Addisons disease is characterized by bilateral syndrome, which can elevate the blood pressure
hypofunction of the adrenal cortex, resulting in to dangerously high levels.
insufficient production of adrenal steroids
including cortisol and aldosterone. There is 441. 1. Hypophysectomy (removal of the pituitary
increased production of melanocyte stimulating gland) interferes with the secretion of both
hormone (MSH), which stimulates production glucocorticoids and antidiuretic hormone, both
of melanin, a dark pigment. Persons with of which are essential to maintain fluid balance
Addisons disease note that their skin is and blood pressure. Careful monitoring of blood
darker. pressure is essential to ensure that hormone
replacement therapy is adequate.
431. 1. If the person with Addisons perspires heavily
he will lose sodium and fluid and become 442. 4. Hormone replacement and follow-up care are
hyponatremic and hypovolemic. He should needed for the rest of the clients life because the
avoid strenuous exercise during hot weather. pituitary gland has been removed. This is the
master gland that secretes trophic hormones that
432. 2. Addisons disease cannot be cured but it is stimulate target glands to produce their
controlled with lifelong hormone replacement. hormones. A Medic-Alert bracelet is needed to
alert others of the clients condition.
433. 1. Self-monitoring of blood glucose has become
an important part of diabetes management. 443. 2. Diabetes insipidus is characterized by
Maintaining good control of blood sugar reduces polydipsia and polyuria. It occurs with lesions
complications. This is the best method to keep of the hypothalamus and pituitary. Because
close control. antidiuretic hormone synthesis is affected, the
client is at high risk for dehydration, which is
434. 4. In order to meet metabolic demand insulin
life-threatening.
must be taken on a regular basis. Ketoacidosis
occurs when there is a lack of insulin in relation 444. 3. Withdrawal from glucocorticoid therapy can
to metabolic demands. precipitate addisonian crisis, which is
characterized by circulatory collapse and shock.
435. 2. The blood work can be done quickly without
Hypotension is a major manifestation of
preparation, while the remaining tests must be
addisonian crisis and must be treated vigorously.
scheduled in advance. The scan must precede
the arteriogram, because the arteriogram dye 445. 2. Management of addisonian crisis includes
contains iodine that will interfere with the scan. glucocorticoid management. Intravenous
replacement of sodium and dextrose is also
436. 2. The tracer dose used is much smaller than a
necessary.
therapeutic dose and is not harmful. The client
can resume contact with others immediately 446. 1. Cushings syndrome is caused by excessive
following the test. corticosteroids. Excessive mineralocorticoid
produces hypertension, acne, changes in
437. 3. Many cough syrups contain iodine, which
secondary sex characteristics, and mood
interferes with the test. Other OTC drugs to be
disturbances such as irritability, anxiety,
avoided include salicylates, multivitamins, and
euphoria, insomnia, irrationality, and
some iodine-containing preparations found in
psychosis.
health food stores.
447. 4. Clients with Cushings syndrome exhibit
438. 4. A fasting blood glucose is drawn before
central obesity, with a buffalo hump, a heavy
Glucola is administered. The client can drink
trunk, and thin extremities. The accumulation of
only water as blood samples are obtained at
fat on the cheeks makes the face moon-shaped,
designated intervals.

4
or rounded.

404 NCLEX-RN Review


53155_04_Ch04c_p378-420.qxd 2/26/09 7:33 AM Page 405

448. 4. Hypertension and hyperkalemia are the 460. 3. The signs and symptoms of
classic manifestations of primary aldosteronism, hypoparathyroidism are due to low serum
in which the adrenal cortex secretes excessive calcium levels. A decrease in serum calcium
mineralocorticoid. may produce tetany. Tetany produces tingling in
lips, fingers, and feet. Severe tetany is associated
449. 3. Clients with pheochromocytoma can experience with muscle spasms.
episodes of life-threatening hypertension when
the adrenal tumor secretes catecholamines that 461. 2. Type 1 diabetes arises from the destruction
stimulate the sympathetic nervous system. These of the beta cells, which results in little or no
attacks are often accompanied by sweating, insulin production. Type 2 diabetes is the result
palpitations, and headache. of tissues being unresponsive to insulin, which
eventually exhausts the production of insulin.
450. 4. The client will have to take corticosteroids Type 2 diabetics tend to be older than 35 years
life-long, but the physical changes they may and overweight.
cause take time to present. Explaining all
treatment and procedures can reduce the 462. 2. The peak action for regular insulin occurs in
patients stress and anxiety level for now. 2 to 4 hours after administration. If regular
insulin is administered at 0730, then the client
451. 3. Without adrenal glands there is a lifelong need should be observed for hypoglycemia between
for a constant dose for replacement therapy 0930 and 1130.
daily.
463. 3. Snacks should be eaten prior to any exercise
452. 2. A simple goiter is an enlargement of the so glucose is readily available for the bodys use.
thyroid gland caused by excess thyroid-
stimulating hormone (TSH) stimulation, growth- 464. 4. A conscious client experiencing hypoglycemia
stimulating hormones, or excessive intake of needs 520 grams of simple carbohydrates
goitrogenic foods. immediately. A 46 oz glass of orange juice
would provide enough glucose to counteract
453. 2. The client with Graves disease is at risk for hypoglycemia.
tachycardia, shock, hyperthermia, weight loss,
and nervousness. 465. 2. The peak action for NPH is 6 hours after
administration; therefore, delaying the
454. 4. Thyroxine and triiodothyronine levels are administration 2 hours in the morning will allow
usually elevated and thyroid-stimulating the client to safely eat lunch at 1400.
hormone levels may be normal or undetectable
in hyperthyroidism. 466. 1. At the first sign of diabetic ketoacidosis
(elevated blood glucose and frequent urination)
455. 3. Propranolol (Inderal) is a beta-adrenergic the nurse should administer insulin per
blocker that will relieve the symptoms of Graves physicians order to stabilize the blood glucose
disease caused by increased circulating thyroid level.
hormone. The symptoms are heat intolerance,
palpitations, nervousness, tachycardia, and 467. 4. Hypoglycemia or low blood glucose occurs
tremors. when there is more insulin than glucose in the
serum or when blood glucose levels drop too
456. 3. Oxygen, suction equipment, and a rapidly. The signs of hypoglycemia include
tracheostomy tray should be available in case irritability, irrational behavior, dizziness,
airway obstruction occurs. tremors, or loss of consciousness.
457. 1. The administration of thyroid hormone is 468. 3. The client states the recommendation by the
needed after surgery because there is no thyroid American Diabetes Association and may also
gland to perform the usual functions. use the Exchange List, knowing that eating the
different groups will help keep the blood
458. 2. During a thyroidectomy it is possible for the
glucose from going up too quickly.
parathyroid glands to be removed or damaged.
If the parathyroid glands are disturbed, 469. 1. Neuropathy is one of the most common
hypocalcemia may result. complications of diabetes caused by
microangiopathies or metabolic defects that
459. 1. The client remains in the outpatient department
cause waste to build up in the nerves, resulting
for about 2 hours to be monitored for vomiting.
in demyelinization.

ADULT NURSING

4 405
53155_04_Ch04c_p378-420.qxd 2/26/09 7:33 AM Page 406

470. 3. A client with Type 2 diabetes is more likely to 473. 4. The client is experiencing low blood sugar or
have hyperglycemic hyperosmolar nonketosis hypoglycemia; ingesting a quick-acting
because there is sufficient insulin to prevent the carbohydrate source is the best action.
metabolism of protein and fats for basic energy
needs. 474. 1. Glycosylated hemoglobin, also known as the
hemoglobin A1c test, is used to determine
471. 4. The cardinal signs of hypoglycemia are cold glycemic control over time. Glucose attaches
sweats, weakness, and trembling. Additional to hemoglobin and remains there for the life
signs include nervousness, irritability, pallor, of the blood cell (120 days); therefore, the
increased heart rate, confusion, and fatigue. test indicates the overall glucose level of
120 days.
472. 3. The client with diabetes should wash the feet
daily and examine for cuts, blisters, swelling,
and any red, tender spots.

The Integumentary System

OVERVIEW OF ANATOMY
AND PHYSIOLOGY
The integumentary system consists of the skin and its
appendages, such as hair, nails, and various glands.
The integumentary system not only provides a barrier
against the external environment, it also plays a role in
maintenance of the bodys internal environment.

Skin
A. Functions
1. Protection: barrier to noxious agents
(microorganisms, parasites, chemical
substances) and to loss of water and electrolytes
2. Thermoregulation: radiant cooling,
evaporation
3. Sensory perception: touch, temperature,
pressure, pain
4. Metabolism: excretion of water and sodium,
production of vitamin D, wound repair
B. Layers (see Figure 4-27)
1. Epidermis
a. The avascular outermost layer
b. Stratified into several layers
c. Composed mainly of keratinocytes and
melanocytes
1) Keratinocytes produce keratin,
responsible for formation of hair and
nails
2) Melanocytes produce melanin, a
pigment that gives color to skin and hair Figure 4-27 Cross-section of skin: observe the
d. Appendages (hair and nails, eccrine sweat skin layers and note the location of the glands in the
glands, sebaceous glands, apocrine sweat dermal layer

4
glands) all derived from epidermis

406 NCLEX-RN Review


53155_04_Ch04c_p378-420.qxd 2/26/09 7:33 AM Page 407

2. Dermis: layer beneath the epidermis composed skin exposure (duration of exposure to sun,
of connective tissue that contains lymphatics, irritants [occupational], cold weather)
nerves, and blood vessels; elasticity of skin C. Nutrition/diet: intake of vitamins, essential
results from presence of collagen, elastin, and nutrients, water; food allergies
reticular fibers in the dermis, which also D. Use of medications: steroids, vitamin use,
nourish the epidermis hormones, antibiotics, chemotherapeutic
3. Subcutaneous layer: layer beneath the dermis agents
composed of loose connective tissue and fat E. Past medical history: renal, hepatic, or collagen
cells; stores fat; important in temperature diseases; trauma or surgery; food, drug, or contact
regulation allergies
F. Family history: diabetes mellitus, allergic
disorders, blood dyscrasias, specific dermatologic
Hair problems, cancer
A. Covers most of the body surface (except palms of
hands, soles of feet, lips, nipples, and parts of
external genitalia).
Physical Examination
B. Hair follicles: tube-like structures, derived from A. Color: note areas of uniform color; pigmentation;
epidermis, from which hair grows. redness, jaundice, cyanosis.
C. Hair functions as protection from external B. Vascular changes
elements and from trauma. 1. Purpuric lesions: note ecchymoses,
D. Hair growth is controlled by hormonal influences petechiae.
and by blood supply. 2. Vascular lesions: note angiomas,
E. Loss of body hair is called alopecia. hemangiomas, venous stars.
C. Lesions: note color, type, size, distribution,
location, consistency, grouping (annular, circular,
Nails linear, or clustered).
A. Dense layer of flat, dead cells; filled with keratin D. Edema: differentiate pitting from nonpitting.
B. Systemic illnesses may be reflected by changes E. Moisture content: note dryness, clamminess.
in the nail or its bed; common changes include: F. Temperature: note whether increased or decreased,
1. Clubbing: enlargement of fingers and toes, distribution of temperature changes.
nail becomes convex; caused by chronic G. Texture: note smoothness, coarseness.
pulmonary or cardiovascular disease H. Mobility/turgor: note whether increased or
2. Beaus line; transverse groove caused by decreased.
temporary halt in nail growth because of
systemic disorder
Laboratory/Diagnostic Studies
A. Blood chemistry/electrolytes: calcium, chloride,
Glands magnesium, potassium, sodium
A. Eccrine sweat glands: located all over the body; B. Hematologic studies: Hbg, HCT, RBC, WBC
participate in heat regulation C. Biopsy
B. Apocrine sweat glands: odiferous glands, found 1. Removal of a small piece of skin for
primarily in axillary, nipple, anal, and pubic areas; examination to determine diagnosis
bacterial decomposition of excretions causes body 2. Nursing care: instruct client to keep biopsied
odor area dry until healing occurs
C. Sebaceous glands: oil glands, located all over the D. Skin testing
body except for palms of hands and soles of feet 1. Administration of allergens or antigens on the
(abundant on face, scalp, upper chest, and back); surface of or into the dermis to determine
produce sebum hypersensitivity
2. Three types: patch, scratch, and intradermal
ASSESSMENT
ANALYSIS
Health History Nursing diagnoses for clients with a disorder of the
integumentary system may include:
A. Presenting problem: symptoms may include A. Impaired skin integrity
changes in color or texture of skin, hair, nails; B. Pain
pruritus; infections; tumors; lesions; dermatitis; C. Disturbed body image
ecchymoses; rashes; dryness D. Risk for infection
B. Lifestyle: hygienic practices (skin-cleansing E. Ineffective airway clearance

4
measures, use of cosmetics [type, brand names]); F. Ineffective peripheral tissue perfusion

ADULT NURSING 407


53155_04_Ch04c_p378-420.qxd 2/26/09 7:33 AM Page 408

PLANNING AND IMPLEMENTATION 2. Protecting grafted skin from direct sunlight for
at least 6 months
3. Protecting graft from physical injury
Goals 4. Need to report changes in graft (fluid
A. Restoration of skin integrity. accumulation, pain, hematoma)
B. Client will experience absence of pain. 5. Possible alteration in pigmentation and hair
C. Client will adapt to changes in appearance. growth; ability to sweat lost in most grafts
D. Client will be free from infection. 6. Sensations may or may not return
E. Maintenance of effective airway clearance.
F. Maintenance of adequate peripheral tissue perfusion.
EVALUATION
Interventions A. Healing of burned areas; absence of drainage,
Skin Grafts edema, and pain over graft sites.
B. Relaxed facial expression/body posture; achieves
A. Replacement of damaged skin with healthy skin effective rest patterns; participates in daily
to provide protection of underlying structures or activities without pain.
to reconstruct areas for cosmetic or functional C. Incorporates changes into self-concept without
purposes negating self-esteem; verbalizes about changes that
B. Graft sources occurred; demonstrates interest in physical
1. Autograft: clients own skin appearance.
2. Isograft: skin from a genetically identical D. Achieves wound healing; vital signs within normal
person (identical twin) range; lungs clear; laboratory studies within
3. Homograft or allograft: cadaver of same species normal range.
4. Heterograft or xenograft: skin from another E. Lungs clear to auscultation; respiratory rate and
species (such as a porcine graft) depth within normal limits; free of dyspnea.
5. Human amniotic membrane F. Palpable peripheral pulses of equal quality;
C. Nursing care: preoperative adequate capillary refill; skin color normal in
1. Donor site: cleanse with antiseptic soap the uninjured areas.
night before and morning of surgery as ordered.
2. Recipient site: apply warm compresses and
topical antibiotics as ordered. DISORDERS OF THE
D. Nursing care: postoperative
1. Donor site INTEGUMENTARY SYSTEM
a. Keep area covered for 2448 hours.
b. Use bed cradle to prevent pressure and Primary Lesions of the Skin
provide greater air circulation
c. Outer dressing may be removed A. Macule: a flat, circumscribed area of color change
2472 hours postsurgery; maintain fine in the skin without surface elevation, up to 2 cm
mesh gauze (innermost dressing) until in diameter
it falls off spontaneously. B. Papule: a circumscribed solid and elevated lesion,
d. Trim loose edges of gauze as it loosens up to 1 cm in size
with healing. C. Nodule: a solid, elevated lesion extending deeper
e. Administer analgesics as ordered (more into the dermis, 12 cm in diameter
painful than recipient site). D. Wheal: a slightly irregular, transient superficial
2. Recipient site elevation of the skin with a palpable margin
a. Elevate site when possible. (e.g., hive)
b. Protect from pressure (use bed cradle). E. Vesicle: circumscribed elevated lesion filled with
c. Apply warm compresses as ordered. serous fluid, less than 1 cm in diameter
d. Assess for hematoma, fluid accumulation F. Bulla: a vesicle larger than 1 cm in diameter
under graft. G. Pustule: a vesicle or bulla containing purulent
e. Monitor circulation distal to graft. exudate
3. Provide emotional support and monitor
behavioral adjustments; refer for counseling if Contact Dermatitis
needed.
E. Provide client teaching and discharge planning A. General information
concerning 1. An irritation of the skin from a specific
1. Applying lubricating lotion to maintain substance or from a hypersensitivity
moisture on surfaces of healed graft for at least immune reaction from contact with a

4
612 months specific antigen

408 NCLEX-RN Review


53155_04_Ch04c_p378-420.qxd 2/26/09 7:33 AM Page 409

2. Caused by irritants (mechanical, chemical, a. Feelings about changes in appearance of


biologic); allergens skin (encourage client to cover arms and
B. Assessment findings legs with clothing if sensitive about
1. Pruritus appearance)
2. Erythema; localized edema; vesicles (oozing, b. Importance of adhering to prescribed
crusting, and scaling [later]) treatment and avoidance of commercially
3. Diagnostic test: skin testing reveals advertised products
hypersensitivity to specific antigen
C. Nursing interventions
1. Apply wet dressings of Burrows solution for
Acne
20 minutes 4 times a day to help clear oozing See Unit 5.
lesions.
2. Provide relief from pruritus (see Cirrhosis of
the Liver).
Pediculosis
3. Administer topical steroids and antibiotics as See Unit 5.
ordered.
4. Provide client teaching and discharge
planning concerning
Skin Cancer
a. Avoidance of causative agent A. General information
b. Preventing skin dryness 1. Types of skin cancers
1) Use mild soaps (Ivory). a. Basal cell epithelioma: most common type
2) Soak in plain water for 2030 minutes. of skin cancer; locally invasive and rarely
3) Apply prescribed steroid cream metastasizes; most frequently located
immediately after bath. between the hairline and upper lip
4) Avoid extremes of heat and cold. b. Squamous cell carcinoma (epidermoid):
c. Allowing crusts and scales to drop off skin grows more rapidly than basal cell
naturally as healing occurs carcinoma and can metastasize; frequently
d. Avoidance of wool, nylon, or fur fibers on seen on mucous membranes, lower lip,
sensitive skin neck, and dorsum of the hands
e. Need to use gloves if handling irritant or c. Malignant melanoma: least frequent of skin
allergenic substances cancers, but most serious; capable of
invasion and metastasis to other organs
2. Precancerous lesions
Psoriasis a. Leukoplakia: white, shiny patches in the
A. General information mouth and on the lip
1. Chronic type of dermatitis that involves b. Nevi (moles): junctional nevus may
accelerated turnover rate of the epidermal cells become malignant (signs include a color
2. Predisposing factors include stress, trauma, change to black, bleeding, and irritation);
infection; changes in climate may produce compound and dermal nevi unlikely to
exacerbations; familial predisposition to the become cancerous
disease c. Senile keratoses: brown, scalelike spots on
B. Medical management older individuals
1. Topical corticosteroids 3. Contributing factors include hereditary
2. Coal tar preparations predisposition (fair, blue-eyed people;
3. Ultraviolet light redheads and blondes); irritation (chemicals or
4. Antimetabolites (methotrexate) ultraviolet rays)
C. Assessment findings 4. Occurs more often in those with outdoor
1. Mild pruritus occupations who are exposed to more sunlight
2. Sharply circumscribed scaling placques that B. Medical management: varies depending on type of
are mostly present on the scalp, elbows, and cancer; surgical excision with or without radiation
knees; yellow discoloration of nails therapy most common; chemotherapy and
D. Nursing interventions immunotherapy for melanoma
1. Apply occlusive wraps over prescribed topical C. Assessment findings: characteristics depend on
steroids. specific type of lesion; biopsy reveals malignant
2. Protect areas treated with coal tar preparations cells
from direct sunlight for 24 hours. D. Nursing interventions: provide client teaching
3. Administer methotrexate as ordered, assess concerning:
for side effects. 1. Limitation of contact with chemical irritants
4. Provide client teaching and discharge 2. Protection against ultraviolet radiation from

4
planning concerning sun

ADULT NURSING 409


53155_04_Ch04c_p378-420.qxd 2/26/09 7:33 AM Page 410

a. Wear thin layer of clothing. superheated air) causes respiratory tissue


b. Use sun block or lotion containing damage
para-amino benzoic acid (PABA). 3. Chemical: caused by tissue contact, ingestion
3. Need to report lesions that change or inhalation of acids, alkalies, or vesicants
characteristics and/or those that do not heal. 4. Electrical: injury occurs from direct damage to
nerves and vessels when an electric current
passes through the body.
Herpes Zoster (Shingles) B. Classification
A. General information 1. Partial thickness
1. Acute viral infection of the nervous system a. Superficial partial-thickness (first degree)
2. The virus causes an inflammatory reaction in 1) Depth: epidermis only
isolated spinal and cranial sensory ganglia and 2) Causes: sunburn, splashes of hot liquid
the posterior gray matter of the spinal cord 3) Sensation: painful
3. Contagious to anyone who has not had 4) Characteristics: erythema, blanching
varicella or who is immunosuppressed on pressure, no vesicles
4. Caused by activation of latent varicella-zoster b. Deep partial thickness (second degree)
virus 1) Depth: epidermis and dermis
B. Medical management 2) Causes: flash, scalding, or flame burn
1. Analgesics 3) Sensation: very painful
2. Corticosteroids 4) Characteristics: fluid-filled vesicles;
3. Acetic acid compresses red, shiny, wet after vesicles rupture
4. Acyclovir (Zovirax) 2. Full thickness (third and fourth degree)
C. Assessment findings a. Depth: all skin layers and nerve
1. Neuralgic pain, malaise, itching, burning endings; may involve muscles, tendons,
2. Cluster of skin vesicles along course of and bones
peripheral sensory nerves, usually unilateral b. Causes: flame, chemicals, scalding, electric
and primarily on trunk, thorax, or face current
D. Nursing interventions c. Sensation: little or no pain
1. Apply acetic acid compresses or white d. Characteristics: wound is dry, white,
petrolatum to lesions. leathery, or hard
2. Administer medications as ordered. C. Medical management
a. Analgesics for pain. 1. Supportive therapy: fluid management (IVs),
b. Systemic corticosteroids: monitor for side catheterization
effects of steroid therapy. 2. Wound care: hydrotherapy, debridement
c. Acyclovir (Zovivax): antivalagent reduces (enzymatic or surgical)
severity when given early in illness. 3. Drug therapy
a. Topical antibiotics: mafenide (Sulfamylon),
silver sulfadiazine (Silvadene), silver
Herpes Simplex Virus, Type I nitrate, povidone-iodine (Betadine)
A. General information solution
1. Causes cold sores or fever blisters, canker b. Systemic antibiotics: gentamicin
sores, and herpetic whitlow c. Tetanus toxoid or hyperimmune human
2. Common disorder, frequently seen in women tetanus globulin (burn wound good
3. Primary infection occurs in children, medium for anaerobic growth)
recurrences in adults d. Analgesics
4. Self-limiting virus 4. Surgery: excision and grafting
B. Assessment findings: clusters of vesicles, may D. Assessment
ulcerate or crust; burning, itching, tingling; usually 1. Extent of burn injury by rule of nines: head
appears on lip or cheek and neck (9%); each arm (9%), each leg (18%),
C. Nursing interventions: keep lesions dry; apply trunk (36%), genitalia (1%) (see Figure 4-28)
topical antibiotics or anesthetic as ordered; 2. Lund and Browder method determines the
antivirals (valacyclovir) may be ordered 13 days. extent of the burn injury by using clients age
in proportion to relative body-part size.
3. Severity of burn
Burns a. Major: partial thickness greater than 25%;
A. Types full thickness greater than or equal to 10%
1. Thermal: most common type; caused by flame, b. Moderate: partial thickness 1525%, full
flash, scalding, and contact (hot metals, grease) thickness less than 10%
2. Smoke inhalation: occurs when smoke c. Minor: partial thickness less than 15%; full

4
(particular products of a fire, gases, and thickness less than 2%

410 NCLEX-RN Review


53155_04_Ch04c_p378-420.qxd 2/26/09 7:33 AM Page 411

Figure 4-28 Body proportions change with growth. Shown here, in percentage, is the
relationship of the body area to the whole body surface area at various ages. This method
of determining the extent of the burned area is attributed to Lund and Browder

E. Stages 3. Fluid remobilization or diuretic phase


1. Emergent phase (25 days postburn)
a. Remove person from source of burn. a. Interstitial fluid returns to the vascular
1) Thermal: smother burn beginning with compartment.
the head. b. Assessment findings
2) Smoke inhalation: ensure patent airway. 1) Elevated blood pressure, increased
3) Chemical: remove clothing that urine output
contains chemical; lavage area with 2) Diagnostic tests: hypokalemia,
copious amounts of water. hyponatremia, metabolic acidosis
4) Electrical: note victim position, identify 4. Convalescent (rehabilitation) phase
entry/exit routes, maintain airway. a. Starts when diuresis is completed and
b. Wrap in dry, clean sheet or blanket to wound healing and coverage begin.
prevent further contamination of wound b. Assessment findings
and provide warmth. 1) Dry, waxy-white appearance of full
c. Assess how and when burn occurred. thickness burn changing to dark
d. Provide IV route if possible. brown; wet, shiny, and serous exudate
e. Transport immediately. in partial thickness
2. Shock phase (first 2448 hours) 2) Diagnostic test: hyponatremia
a. Plasma to interstitial fluid shift causing F. Nursing interventions
hypovolemia; fluid also moves to areas 1. Provide relief/control of pain.
that normally have little or no fluid (third- a. Administer morphine sulfate IV and
spacing). monitor vital signs closely.
b. Assessment findings b. Administer analgesics/narcotics
1) Dehydration, decreased blood 30 minutes before wound care.
pressure, elevated pulse, decreased c. Position burned areas in proper alignment.
urine output, thirst 2. Monitor alterations in fluid and electrolyte
2) Diagnostic tests: hyperkalemia, balance.
hyponatremia, elevated HCT, a. Assess for fluid shifts and electrolyte

4
metabolic acidosis alteration (see Table 4-5).

ADULT NURSING 411


53155_04_Ch04c_p378-420.qxd 2/26/09 7:33 AM Page 412

Table 4-28 Guidelines and Formulas for Fluid Replacement for Burns

Consensus Formula Evans Formula Brooke Army Formula Parkland/Baxter Formula

Lactated Ringers: 1. Colloids: 1 mL 3 wt. 1. Colloids: 0.5 mL 3 wt. Lactated Ringers: 4 mL 3 wt.
24 mL 3 wt. kg 3 % BSA burned kg 3 % BSA burned kg 3 % BSA burned.
in kg 3 % body 2. Electrolytes (saline): 2. Electrolytes (lactated Day 1: Half to be given in first 8 hr;
surface area (BSA) 1 mL 3 wt. kg 3 % BSA Ringers): 1.5 mL 3 wt. half to be given over next 16 hr.
burned. Half to be burned kg 3 % BSA burned Day 2: Varies; colloid is added.
given in first 8 hr 3. Glucose (5% in water): 3. Glucose (5% in water):
after burn; remaining 2000 mL for insensible loss 2000 mL for insensible
fluid to be given over Day 1: Half to be given in first loss
next 16 hr. 8 hr; remaining half over Day 1: Half to be given in
next 16 hr. first 8 hr; remaining
Day 2: Half of previous days half over next 16 hr.
colloids and electrolytes; all of Day 2: Half of colloids,
insensible fluid replacement. half of electrolytes, all
Maximum of 10,000 mL over of insensible fluid
24 hr. replacement.
Second- and third-degree Second- and third-degree
burns exceeding 50% BSA burns exceeding 50%
calculated on basis of 50% BSA calculated on basis
BSA. of 50% BSA.

b. Administer IV fluids as ordered (see 2) Observe for and report hypersensitivity


Table 4-28) reactions (rash, itching, burning
c. Monitor Foley catheter output hourly sensation in unburned areas).
(30 mL/hour desired). 3) Store drug away from heat.
d. Weigh daily. f. Apply silver nitrate as ordered.
e. Monitor circulation status regularly. 1) Handle carefully; solution leaves a
f. Administer/monitor gray or black stain on skin, clothing,
crystalloids/colloids/H2O solutions. and utensils.
3. Promote maximal nutritional status. 2) Administer analgesic before application.
a. Monitor tube feedings/TPN if ordered. 3) Keep dressings wet with solution;
b. When oral intake permitted, provide high- dryness increases the concentration
calorie, high-protein, high-carbohydrate diet and causes precipitation of silver salts
with vitamin and mineral supplements. in the wound.
c. Serve small portions. g. Apply povidone-iodine (Betadine) solution
d. Schedule wound care and other treatments as ordered.
at least 1 hour before meals. 1) Administer analgesics before
4. Prevent wound infection. application.
a. Place client in controlled sterile 2) Assess for metabolic acidosis/renal
environment. function studies.
b. Use hydrotherapy for no more than h. Administer gentamicin as ordered: assess
30 minutes to prevent electrolyte loss. vestibular/auditory and renal functions at
c. Observe wound for separation of eschar regular intervals.
and cellulitis. 5. Prevent GI complications.
d. Apply mafenide (Sulfamylon) as ordered. a. Assess for signs and symptoms of paralytic
1) Administer analgesics 30 minutes ileus.
before application. b. Assist with insertion of NG tube to
2) Monitor acid-base status and renal prevent/control Curlings/stress ulcer;
function studies. monitor patency/drainage.
3) Provide daily tubbing for removal of c. Administer prophylactic antacids through
previously applied cream. NG tube and/or IV cimetidine (Tagamet) or
e. Apply silver sulfadiazine (Silvadene) as ranitidine (Zantac) (to prevent gastric pH
ordered. of less than 5).
1) Administer analgesics 30 minutes d. Monitor bowel sounds.

4
before application. e. Test stools for occult blood.

412 NCLEX-RN Review


53155_04_Ch04c_p378-420.qxd 2/26/09 7:33 AM Page 413

6. Provide client teaching and discharge 2) Avoid use of fabric softeners or harsh
planning concerning detergents (might cause irritation).
a. Care of healed burn wound 3) Avoid constrictive clothing over burn
1) Assess daily for changes. wound.
2) Wash hands frequently during dressing c. Adherence to prescribed diet
change. d. Importance of reporting formation of
3) Wash area with prescribed solution or blisters, opening of healed area, increased
mild soap and rinse well with H2O; or foul-smelling drainage from wound,
dry with clean towel. other signs of infection
4) Apply sterile dressing. e. Methods of coping and resocialization
b. Prevention of injury to burn wound
1) Avoid trauma to area.

Complementary and Alternative Medicine (CAM)

OVERVIEW available in hospitals in the United States or


other Western societies (see Table 4-29).
A. Definitions B. Patterns of CAM use in the United States
1. Alternative medicine is a holistic approach to 1. Most clients do not tell their physician they
health and focuses on balancing the body to are using CAM unless specifically asked.
achieve optimal wellness. 2. Estimates that one in three adults aged 35 to
2. Alternative medical systems are built upon a 49 years is using at least one CAM therapy.
complex system of theory and practice and 3. Higher incidence in incomes over $50,000.
have often evolved apart from, and earlier 4. 42% of CAM use is for treatment of a specific
than, the conventional medicine that is disease or illness, while 58% of use is, at least
practiced in the United States. in part, to prevent future illness from
3. Modalities that are generally not taught widely occurring or to maintain health and vitality.
in Western medical schools nor generally

Table 4-29 Complementary and Alternative Modalities


Modality What It Is Common Uses
Ayurvedic Medicine Ayurvedic tradition believes that all illnesses, physical or mental, Acute and chronic illness,
are a state of disharmony or imbalance in the bodys systems. promotion of health
Diagnosis is made by feeling the pulse and looking for and wellness
indicators of disharmony on the tongue. Utilizes a wide variety
of modalities such as medication, herbals, and massage.
Nutrition and food groups play a key role in balancing the
body systems. Practiced in India for more than 5000 years.
Aromatherapy The use of essential oils, distilled from plants, aromatherapy Abdominal pain, stress and
treats. Oils are massaged into the skin in diluted form, anxiety, insomnia, as an
inhaled, or placed in water for diffusion. Aromatherapy is antiseptic, skin
often used in conjunction with massage therapy, acupuncture, conditions
reflexology, herbology, chiropractic, and other holistic treatments.
Massage Manipulation of muscles and other soft tissue. Improves Back and neck pain, stress,
circulation, muscle fatigue, and promotes well-being. Multiple insomnia, headache, promotion
techniques including deep tissue, vibration, effleurage, of well-being
percussion. Developed in both Western and Eastern cultures.

4
(continues)

ADULT NURSING 413


53155_04_Ch04c_p378-420.qxd 2/26/09 7:33 AM Page 414

Table 4-29 Complementary and Alternative Modalities (continued)

Modality What It Is Common Uses


Chiropractic Manipulation of the spine to promote balance and health. Back and neck pain, stress, insomnia,
Misalignments of the spine thought to cause pressure on headache
the spinal nerve roots. Pressure from trauma or poor posture
leads to decreased function and wellness. Adjustment or
manipulation of the spine corrects the misalignment and
results in a more balanced state of health.
Acupuncture Based on the concept of chi, or life-force energy. Very fine Pain relief, smoking cessation, treatment
needles are placed along energy lines to disperse congestion, of acute and chronic illness; also used
increase energy flow, and allow the body to heal and balance. extensively with pets
Biofeedback Use of a sensor unit (TENS) allows individuals to monitor their Stress-related conditions such as asthma,
own bodies response to stimuli. Usually a light and sound migraines, insomnia, and high blood
device. A method of monitoring minute metabolic changes pressure
in ones own body with the aid of sensitive machines.
Individuals use imagery, breathing techniques, and
visualization to control their heart rate, breathing, and
thought process. These changes are audible and visible
on the biofeedback unit.
Hypnosis A method of focusing on the unconscious mind and accessing Anxiety, weight loss, pain relief, phobias,
the subconscious to access repressed emotions, forgotten chronic pain
events, and memories that may assist the client to heal.
Trained hypnotherapist creates a calm, safe environment
for the client. Client remains aware but focused on the
subconscious mind rather than the present.
Homeopathy A healing system that uses minute amounts of natural Arthritis, abdominal complaints, allergies,
substances, called remedies, to stimulate the individuals fatigue, menstrual irregularities
immune system. These substances, given in larger dose,
mimic the symptoms the client is experiencing. Based
on the philosophy like cures like.
Herbal Medicine The use of plant products for treatment of a wide range of Acute and chronic illness including, but
chronic and acute illnesses. Plants, and their derivatives, are not limited to, asthma, arthritis,
used in a variety of forms, including dried, distilled, tincture, abdominal pain, menstrual
pills, caplets, elixirs, and topical preparations. One of the irregularities, cuts and bruises; used
oldest complementary therapies, it is used widely throughout to enhance the functioning of the
the world today as the primary source of medicinal products. bodys systems
Not regulated by the FDA in the United States.
Reflexology Designed to stimulate chi, or life-force energy, in the body Respiratory symptoms, stress, headache,
by applying pressure to meridians, or contact points, on the fatigue, pain
feet and hands. Pressure on the reflexology points stimulates
flow of chi, allowing the client to restore balance in the body
and promote healing. The practitioner uses the fingers to
apply pressure to the reflexology points.
Reiki Use of the practitioners hands to channel the universal life-force Pain, digestive problems, stress and
energy to the client to promote healing. Tibetan healing system. stress-related disorders; assist the
Hands may hover above the client or lightly touch. Use light recipient in achieving spiritual focus
hand placements to channel healing energies to the recipient. and clarity
Therapeutic Touch Practitioner senses clients energy field through auditory- Headache, musculoskeletal pain,
visual-kinetic or intuitive systems. Using the hands, the emotional distress
practitioner facilitates a symmetrical and rhythmical flow
of energy and redirects it, allowing the individuals energy
to flow freely. Treatment is complete when the practitioner
senses a symmetrical state and rhythmic order.

4 414 NCLEX-RN Review


53155_04_Ch04c_p378-420.qxd 2/26/09 7:33 AM Page 415

What would an acceptable range for hourly urine


Sample Questions output during the first 2 days post-burn?
1. 20 mL.
2. 3050 mL.
475. A client is seen in the outpatient clinic for
treatment of psoriasis. The nurse should anticipate 3. 100150 mL.
which of the following findings in this man? 4. 150200 mL.
1. Intense pain.
481. The nurse is to perform a scratch test for allergy.
2. Discolored nails. Which statement best describes this procedure?
3. Abdominal lesions. 1. The antigen is directly applied to the skin
4. Hyperpigmented skin. and covered with a gauze dressing.
2. The allergen is applied superficially to a
476. The physician prescribes coal tar preparations as
small cut of the outer layer of skin.
part of the treatment plan for a client who has
psoriasis. The nurse should include which 3. A small amount of allergen is injected into
statement in the teaching plan for this client? the intradermal layer of skin.
1. Avoid sunlight immediately after the 4. Suspected food allergy items are scratched
treatment. from the diet, one at a time, until all allergy
symptoms are no longer present.
2. Ingest the coal tar with liberal amounts of
water. 482. An adult has undergone a skin graft from his left
3. Eat a high-carbohydrate diet. buttock to his right upper thigh. What
4. Restrict activity for 24 hours. intervention will the nurse expect to perform
when caring for the recipient site?
477. An adults shirt catches on fire and is now in 1. Apply silver sulfadiazine to promote rapid
flames. He panics and runs into his neighbors healing.
yard. Which of the following interventions is
2. Assess for bleeding and large amounts of
appropriate? Select all that apply.
fluid accumulation beneath the graft.
______ Dousing the flames with water.
3. Encourage the client to ambulate and do leg
______ Removing his burned clothing. lifts on return from the OR.
______ Removing his jewelry. 4. Encourage the client to take frequent soaking
______ Rolling him on the ground. baths to relieve his soreness and discomfort.
478. The nurse is caring for a man admitted with 483. A client with a skin graft has undergone a full-
severe burns sustained when his clothing caught thickness skin graft from her right upper thigh
fire while he was burning leaves. During the to her upper chest area. What is the most
acute burn phase, which intervention will be appropriate nursing action in caring for the
absent from the nursing care plan? donor site?
1. Strict aseptic technique. 1. Keep the fine mesh gauze dressing on her
2. Proper alignment of all joints. chest soaked with normal saline.
3. Maintenance of fluid and electrolyte balance. 2. Completely immobilize her right upper thigh
4. Frequent and routine administration of area.
narcotics. 3. Maintain the compression bandage on her
right upper thigh for several days.
479. The nurse is planning care for an adult man who 4. Remove the nylon fabric adhered to the
is admitted with severe flame burns. During the donor site no later than 2 to 3 days after
first 2448 hours of a post-burn client, which grafting has taken place.
body changes can the nurse expect to occur?
1. An increase in the total volume of 484. A client has undergone a skin graft. Which
intravascular plasma. finding most likely indicates that complications
2. Excessive renal perfusion with diuresis. with the recipient site may exist?
3. Fluid shift from interstitial spaces to plasma. 1. Small amounts of blood beneath the graft.
4. Fluid shift from plasma to interstitial spaces. 2. Small amounts of serum beneath the graft.
3. A meshed pattern in the graft.
480. The nurse is caring for an adult who was admitted 4. Continuous bleeding beneath the graft.

4
following severe burns sustained in a house fire.

ADULT NURSING 415


53155_04_Ch04c_p378-420.qxd 2/26/09 7:33 AM Page 416

485. A female client has been diagnosed with atopic would be a priority when developing a plan of
dermatitis. She has severe pruritis. Which care for the client?
interventions are most appropriate to include 1. Pain, related to herpes simplex.
in the plan of care? 2. Pain, related to herpes zoster.
1. Soak in a hot water bath at least once a day 3. Pain, related to herpetic whitlow.
for 1520 minutes.
4. Pain, related to staphylococcal cellulitis.
2. Avoid use of air conditioning when possible.
3. When symptoms are worse, decrease bathing. 491. What should the nurse include in the plan of
4. Use a moisturizing soap. care for a client with herpes zoster?
1. Teaching the client to avoid sexual contact
486. The client is ordered soaks with Burrows during outbreaks.
solution due to severe poison ivy. What is the 2. Administering analgesics and evaluating the
primary reason for the soaks? efficacy.
1. To clean out the wounds. 3. Informing the client that people who have
2. To help dry the oozing lesions. not had chickenpox will not develop them
3. To stop the pruritus. from exposure to the client.
4. To stop the pain from the lesions. 4. Scheduling several diagnostic tests to
confirm the presence of herpes zoster.
487. The nurse has been giving instructions to a
white female about preventing skin cancer. 492. Which client statement best indicates the client
Which statement best indicates understanding of understands how herpes simplex is transmitted?
skin cancer risk factors? 1. It is okay to share towels as long as it is a
1. I guess because I am dark complected I will family member.
be more prone to developing skin cancer. 2. I really dont need to use a condom, unless
2. I used to lie in the sun all the timenow I have a sore.
I just go to the tanning bed. 3. Once Im over this spell, I wont need to
3. My father was treated for melanoma, but my worry about it again.
mom says not to worry. 4. I shouldnt have sex if some of those sores
4. I really need to use sunscreeneven in are around.
winter.
493. A client is admitted with severe flame burns
488. A client presents with a diagnosis of basal cell resulting from smoking in bed. The nurse would
epithelioma. What is the best description for the expect the room environment to include which
lesion? of the following?
1. Dome shaped, shiny, with a well-defined 1. Strict isolation techniques and policies.
border. 2. A semi-private room.
2. Poorly marginated, flat red area. 3. Liberal unrestricted visiting.
3. Red, dark blue, or purple macules. 4. Equipment shared between the client and the
4. Erythema, edema, and blisters. other burn client in the unit.

489. While providing a nursing history, a client with 494. An adult client was burned severely in an
suspected malignant melanoma will most often industrial accident. He has second-degree burns
relate which of the following? on his right leg and arm and on his back. He has
1. A history of intense sunlight exposure. third-degree burns on his left arm. The triage
2. Complaints of frequently occurring nurse, using the rule of nines, estimates the
irregularly shaped, flat macules with extent of the clients burns as _____%.
overlying hard scale.
495. An adult was burned in a house fire 16 hours
3. Consistent use of sunscreen agents. ago. She suffered second- and third-degree burns
4. Complaints of several lesions with a raised over 65% of her body. She is receiving lactated
border and flattened center. Ringers at 200 mL/h. Which intervention is a
priority at this time?
490. An adult presents with the following symptoms:
1. Monitoring hourly urine output.
clusters of vesicles on the right flank; constant,
severe pain; burning; itching and discomfort in 2. Assessing for signs and symptoms of

4
the flank area. Which of the following diagnoses infection.

416 NCLEX-RN Review


53155_04_Ch04c_p378-420.qxd 2/26/09 7:33 AM Page 417

3. Performing range of motion q 12 h. across the power line. What is the nurses best
4. Meeting the high caloric needs of the client. initial action?
1. Move the person away from the power line
496. A nurse is providing care for a severely burned using a wooden pole.
client during the shock phase of the burn injury. 2. Cover the person with a blanket.
Which assessment findings would indicate that
3. Grab the person and pull him away from the
the client is receiving adequate fluid volume
power lines.
replacement?
4. Flush the wound with copious amounts of
1. Urine output 20 mL/h, CVP 3, weak pulses,
water.
K+ level of 5.3.
2. Urine output 50 mL/h, BP 100/60, oriented to
person and place.
3. Weak thready pulses, BP 70/40, pulse 130,
HCT 52%.
4. Restlessness, confusion, urine output
Answers and Rationales
15 mL/h, rapidly increasing weight.
475. 2. A yellow discoloration of the nails is
497. A client with severe burns is receiving IV
frequently seen in psoriasis.
Zantac. Which statement best explains the
reason for administration of this medication in 476. 1. Sunlight should be avoided after a coal tar
this situation? treatment.
1. The client was treated for gastritis several
years ago. 477. Dousing the flames should be selected. This is
2. The medication will reduce hypoxemia in an appropriate way to smother the flames.
burn clients. Removing his jewelry should be selected. Hot
3. The medication is an H2 receptor antagonist metal jewelry could increase burning. Rings
and will decrease acid secretion. should be removed before edema occurs.
Rolling him on the ground should be selected
4. The medication will aid in removal of
because it will smother flames.
pulmonary secretions.

498. An 18-year-old was burned 6 weeks ago. She is 478. 4. Narcotics should be given only after careful
now ready for discharge. Select the statement assessment in this phase due to the danger of
best reflecting understanding of discharge care. shock and respiratory depression.
1. I will be so glad to get home so that I dont 479. 4. The initial fluid alteration following a severe
have to wear this pressure thing anymore. burn is a plasma-to-interstitial fluid and
2. I will need to call my doctor if my electrolyte balance, which is a nursing priority.
temperature goes up or this burn area starts
draining and oozing. 480. 2. A safe range for the hourly urine output post-
3. I really need to stick to a low-calorie, burn is 3050 mL. Less than this amount would
low-protein diet. indicate severely decreased renal arterial perfusion.
4. To prevent that area of new skin from
481. 2. The allergen is applied to a small superficial
feeling so tight, I can rub ice and baby
scratch that cuts the outer layer of the skin.
oil on it.

499. A client has suffered a chemical burn. What is 482. 2. Bleeding and large amounts of fluid
the nurses best initial action? accumulation beneath the graft may prevent
successful adherence of the graft.
1. Roll the client in a blanket.
2. Secure lead-lined gloves and move the client 483. 3. Compression bandages are often applied in
away from the chemical. the operating room on top of a synthetic,
3. Flush the area with copious amounts of water semipermeable polyurethane film. This dressing
or normal saline. allows the polyurethane film to adhere to the
4. If the chemical is an acid, neutralize with a base. donor site, reducing accumulation of fluid.

500. An electrical worker has come in contact with a 484. 4. Continuous bleeding beneath a graft may
live power line. He is unconscious and is lying prevent adherence of the graft.

ADULT NURSING

4 417
53155_04_Ch04c_p378-420.qxd 2/26/09 7:33 AM Page 418

485. 4. A moisturizing soap will help decrease the adequate fluid volume replacement is occurring.
pruritus, as it will be less drying. Pulses should also be easily palpable.

486. 2. The solution helps dry up oozing lesions such 497. 3. Burn clients are very susceptible to
as poison ivy. It does not debride wounds, development of stress ulcers. Routinely they
prevent pain, or stop itching. receive Zantac to help prevent this
complication.
487. 4. Almost all cases of basal and squamous
cell skin cancer diagnosed each year in the 498. 2. This statement demonstrates that the client
United States are considered to be sun-related. realizes she must be alert to the signs and
symptoms of infection and notify her physician
488. 1. The most common presentation of BSE is a if they do occur.
nodular lesion that is dome-shaped papules with
well-defined borders. The lesions can have a 499. 3. Water will neutralize most chemicals while
pearly or shiny appearance because it does not decreasing the heat reaction.
keratinize.
500. 1. Emergency treatment starts with separating
489. 1. The majority of malignant melanoma appears the client from the power source. It is important
to be associated with the intensity of sunlight to use nonconductive implements such as
exposure rather than the duration. wooden poles to prevent injury to the rescuer.

490. 2. Pain is a significant problem with herpes


zoster. Topical solutions, cooling soaks, and use
of analgesics are usually incorporated into the
REFERENCES AND SUGGESTED READINGS
plan of care. Adams, J., & Apple, F. (2004). New blood tests for detecting heart
disease. Cardiology Patient Page, 1214. (Retrieved from
491. 2. Pain is usually present and can be quite Circulation).
severe. It is important to help the client obtain Alternative therapies in health and medicine. Boulder, CO.
relief. Cool compresses, analgesics, and topical InnoVision Communications, LLC. Retrieved on August 21,
antipruritic preparations may be used. 2008 from http://www.alternative-therapies.com
American Association of Nurse Anesthetists. Conscious
492. 4. Sexual contact should be avoided during the sedation: What patients should expect. Retrieved August 21,
2008, from http://www.aana.com/forpatients.aspx?
initial and recurring infections. The client
ucNavMenu_TSMenuTargetID=68&ucNavMenu_TSMenuTarget
should also avoid touching the infected area, as Type=4&ucNavMenu_TSMenuID=6&id=298&terms=conscious
it may be transferred to the eyes or face. Good +sedation
handwashing is vital. Aschenbrenner, D., Cleveland, L., & Venable, S. (2007). Drug
therapy in nursing (2nd ed.). Philadelphia: Lippincott
493. 1. Isolation is thought by some clinicians to Williams & Wilkins.
reduce the incidence of cross contamination Barash, P., Cullen, B., Stoelting, R., Cullen, B. (2005). Handbook
significantly. However, methods vary drastically of clinical anesthesia (5th ed.). Philadelphia: Lippincott
Williams & Wilkins.
from one center to another. The single most
Barnes, P. M., Powell-Griner, E., McFann, K., & Nahin, R. L.
effective technique to prevent transmission of (2004). Complementary and alternative medicine use among
infection is handwashing! adults: United States, 2002. Advance data from vital & health
statistics. Hyattsville, MD: National Center for Health
494. The rule of nines is a quick assessment scale Statistics.
used to estimate the extent of burn injury. The Bickley, L., & Szilagyi, P. (2005). Bates guide to physical
basis of the rule is to divide the body into areas examination and history taking (9th ed.). Philadelphia:
each representing 9% or a multiple of 9% of the Lippincott Williams & Wilkins.
total body surface area. This clients injuries Black, J., & Hawks, J. (2008). Medical-surgical nursing: Clinical
management for positive outcomes (8th ed.). Philadelphia:
were assigned the following percentages: R arm Saunders.
9%, L arm 9%, R leg 18%, back 18%, total 54%. Board on Health Promotion and Disease Prevention and Institute
of Medicine. Institute of Medicine of the National Academies.
495. 1. Urine output is the most readily available and (2005). Complementary and alternative medicine in the
reliable indicator for determining the adequacy United States. The National Academies Press: Washington,
of fluid resuscitation. Urine output should be D.C.
monitored every hour and maintained between Brown, K. (2003). Emergency dysrhythmias & ECG injury
30 and 50 mL/h. patterns. Clifton Park, NY: Thomson Delmar Learning.
Carpenito, L. (2007). Nursing diagnosis: Application to clinical
496. 2. 50 mL/h of urine is adequate, BP is stable, practice (12th ed.). Philadelphia: Lippincott Williams &

4
Wilkins.
clear sensorium is another positive sign that

418 NCLEX-RN Review


53155_04_Ch04c_p378-420.qxd 2/26/09 7:33 AM Page 419

Centers for Disease Control and Prevention. (2006).Trends in Linton, A., & Lach, H. (2006). Matteson & McConnells
tuberculosis incidenceUnited States, 2006 (March 23, gerontological nursing: concepts and practice (3rd ed.).
2007/56(11);245-250). Atlanta, GA. St. Louis, MO: Elsevier, Health Science Division.
Daniels, R., Nosek, L., & Nicoll, L. (2007). Contemporary Mayhew, M., Edmunds, M., & Wendel, V. (2005). Gerontological
medical-surgical nursing. Clifton Park, NY: Thomson Delmar nurse practitioner: Review and resource manual. Washington,
Cengage Learning. DC: The American Nurses Credentialing Center.
Doenges, M., & Moorehouse, M. (2008). Application of nursing Micozzi, M. S. (2005). Fundamentals of complementary and
process and nursing diagnosis (5th ed.). Philadelphia: F. A. Davis. alternative medicine. (3rd ed.). New York: Churchill-
Eliopoulos, C. (2004). Gerontological nursing (6th ed.). Livingston.
Philadelphia: Lippincott Williams & Wilkins. Miller, C. (2003). Nursing for wellness in older adults: Theory &
Estes, M. (2010). Health assessment and physical examination practice (4th ed.). Philadelphia: Lippincott Williams &
(4th ed.). Clifton Park, NY: Delmar Cengage Learning. Wilkins.
Fischbach, F. (2008). A manual of laboratory and diagnostic tests Miller, J. (2003). Delmars NCLEX-PN review. Clifton Park,
(8th ed.). Philadelphia: Lippincott Williams & Wilkins. NY: Thomson Delmar Learning.
Fischbach, F., & Dunning III, M. (2005). Nurses quick reference NANDA International. (2007). Nursing diagnosis: Definitions &
to common laboratory and diagnostic tests (4th ed.). classification 20072008. NANDA International, Philadelphia.
Philadelphia: Lippincott Williams & Wilkins. Author.
Guyton, A., & Hall, J. (2006). Textbook of medical physiology Phipps, W., Long, B., Woods, N., & Cass Meyer, V. (2006).
(11th ed.). Philadelphia: Saunders. Medical-surgical nursing concepts and clinical practice
Hudack, C., Morton, P., & Gallo, B. (2004). Critical care nursing: (8th ed.). St. Louis, MO: Elsevier, Health Science
A holistic approach (8th ed.). Philadelphia: Lippincott Division.
Williams & Wilkins. Reuben, D. B., Pacala, J., & Herr, K. (2004). Geriatrics at your
Ignatavicius, D., & Workman, M. (2006). Medical-surgical fingertips (6th ed.). Malden, MA: Blackwell.
nursing: Critical thinking for collaborative care (5th ed.). Seeley, R., Stephens, T., & Tate, P. (2006). Anatomy & physiology
Philadelphia: W. B. Saunders. (7th ed.). New York: McGraw Hill.
Kost, M. (2002). Administration of conscious sedation. Smeltzer, S., Bare, B., & Boyer, M. (2008). Brunner and
http://nsweb.nursingspectrum.com/ce/ce159.htm Suddarths textbook of medical surgical nursing (11th ed.).
Lemone, P., & Burke, K. (2008). Medical-surgical nursing: Critical Philadelphia: Lippincott Williams & Wilkins.
thinking in client care (4th ed.). Upper Saddle River, NJ: Spratto, G. R., & Woods, A. L. (2009). Delmar Nurses drug
Prentice Hall. handbook 2009 Edition. Clifton Park, NY: Delmar Cengage
Lewis, S., Bucher, L., Heitkemper, M., Dirksen, S. OBrien, P., & Learning.
Bucher, L. (2007). Medical-surgical nursing: Assessment and
management of clinical problems (7th ed.) St. Louis, MO:
Elsevier, Health Science Division.

ADULT NURSING

4 419
53155_04_Ch04c_p378-420.qxd 2/26/09 7:33 AM Page 420
53155_05_Ch05_p421-532.qxd 2/27/09 1:32 PM Page 421

U N I T 5

P E D I AT R I C
NURSING

Todays pediatric nurse faces an array of challenges in providing UNIT OUTLINE


care for children and their families. A nurse requires competent 422 Growth and Development
skills from a wide spectrum of both technological and 441 Multisystem Stressors
psychosocial disciplines. To effectively care for children and 450 The Neurosensory System
families in a variety of settings, the nurse must have a thorough 464 The Cardiovascular System
understanding of disease processes, as well as knowledge of 475 The Hematologic System
emotional, social, cultural, and developmental needs. 482 The Respiratory System
To provide this essential knowledge base, this unit begins 492 The Gastrointestinal System
with a review of growth and development, which is basic to 501 The Genitourinary System
understanding the behavior of children and the influences of 506 The Musculoskeletal System
illness. The next section, multisystem stressors, emphasizes such 515 The Endocrine System
topics as fluid, electrolyte, and acid-base imbalance, which are 517 The Integumentary System
applicable to many pediatric health care problems. 521 Pediatric Oncology

The unit is further divided into specific body systems. For each
system, there is an initial review of aspects of anatomy and
physiology unique to the child. Each step of the nursing process
is then reviewed, followed by discussions of the major health
problems of that system.

Throughout the unit, only information specific to children is


presented. In many cases, for instance, when lab tests or nursing
care do not differ from those for the adult, the content is not
repeated. Refer to Unit 4 for additional background information
when needed.
421
53155_05_Ch05_p421-532.qxd 2/27/09 1:32 PM Page 422

Growth and Development

GENERAL PRINCIPLES C. Fetal period and infancy: the head and neurologic
tissue grow faster than other tissues.
D. Toddler and preschool periods: the trunk grows
Definition of Terms more rapidly than other tissue.
A. Growth: increase in size of a structure. Human E. The limbs grow most during school-age period.
growth is orderly and predictable, but not even; it F. The trunk grows faster than other tissue during
follows a cyclical pattern. adolescence.
B. Development: increased complexity in thought,
behavior, skill, or function. Development includes Child Development Theorists
growth and is a process that continues over time.
C. Maturation: genetically determined pattern for Also see Unit 7.
growth and development.
D. Gephalocaudal: head-to-toe progression of growth Sigmund Freud (Psychosexual Theory)
and development.
E. Proximodistal: trunk-to-periphery (fingers and See Table 5-1.
toes) progression of growth and development.
F. Phylogeny: development or evolution of a species Erik Erikson (Psychosocial Theory)
or group; a pattern of development for a species.
See Table 5-2.
G. Ontogeny: development of an individual within a
species.
H. Critical period: specific time period during which Jean Piaget (Cognitive Theory)
certain environmental events or stimuli have See Table 5-3.
greatest effect on a childs development.
I. Developmental task: skill or competency unique to
a stage of development.
ASSESSMENT
Rates of Development Developmental Tasks
Growth and development are not synonymous but are Developmental tasks are skills or competencies
closely interrelated processes directed by both genetic normally occurring at one stage and having an effect
and environmental factors. Although changes in on the development of subsequent stages; fall into
growth and development are more obvious in some three categories
periods than others, they are important in all periods. A. Physical tasks (e.g., learning to sit, crawl, walk;
A. Infancy and adolescence: fast growth periods toileting)
B. Toddler through school-age: slow growth periods B. Psychologic tasks (e.g., learning trust, self-esteem)

Table 5-1 Stages of Freuds Psychosexual Development

Stage Age Characteristics


Oral Birth to 1 year Receives satisfaction from oral needs being met; attachment to
mother important because she usually meets infants needs
Anal 1 to 3 years Learns to control bodily functions, especially toileting
Phallic 3 to 6 years Fascinated with gender differences, childbirth; Oedipus or
Electra complex
Latency 6 to 11 years Sexual drives submerged; appropriate gender roles adopted;
learning about society
Genital 12 years and older Sexual desires directed toward opposite gender; learns how to
form loving relationships and manage sexual urges in societally
appropriate ways

5 422 NCLEX-RN Review


53155_05_Ch05_p421-532.qxd 2/27/09 1:32 PM Page 423

Table 5-2 Stages of Eriksons Psychosocial Theory of Development

Stage Age Characteristics


Trust versus mistrust 1 month to 112 years Learns world is good and can be trusted as basic
needs are met
Autonomy versus shame and 112 to 3 years Learns independent behaviors regarding toileting, bathing,
doubt feeding, dressing; exerts self; exercises choices
Initiative versus guilt 3 to 6 years Goal directed, competitive, exploratory behavior; imaginative
Industry versus inferiority 6 to 11 years Learns self-worth as gains mastery of psychosocial,
physiological, and cognitive skills; becomes society or
peer focused
Identity versus role confusion 12 to 18 years Develops sense of who I am; gains independence from
parents; peers important

Table 5-3 Stages of Piagets Theory of Cognitive Development

Stage/Phase Age Characteristics


Sensiomotor Birth to 2 years
Reflexive Birth to 1 month Predictable, innate survival reflexes
Primary circular reactions 1 to 4 months Responds purposefully to stimuli; initiates , respects
satisfying behaviors
Secondary circular reactions 4 to 8 months Learns from intentional behavior; motor skills/ vision
coordinated; recognizes familiar objects
Coordination of secondary schemes 8 to 12 months Develops object permanence; anticipates others actions;
differentiates familiar/unfamiliar
Tertiary circular reactions 12 to 18 months Interested in novelty, repetition, understands causality;
solicits help from others
Mental combinations 18 to 24 months Simple problem solving; imitates

Preoperational 2 to 7 years
Preconceptual 2 to 4 years Egocentric thought; mental imagery; increasing language
Intuitive 4 to 7 years Sophisticated language; decreasing egocentric thought;
reality-based play

Concrete Operations 7 to 11 years Understands relationships, classification, conservation,


seriation, reversibility; logical reasoning limited, less
egocentric thought

Formal Operation 11 years and older Capable of systemic, abstract thought

C. Cognitive tasks (e.g., acquiring concepts of time 2. Results from at least two separate testing
and space, abstract thought) sessions needed before determination of
See Table 5-4. cognitive level is made
3. Uses toys and language based on mental rather
than chronologic age
Measurement Tools C. Denver II (Revision and restandardized from
There are a number of different assessment tools for Denver Developmental Screening Test (DDST) and
measuring the progress of growth and development. its revision, the DDST-R).
A. Chronologic age: assessment of developmental 1. Generalized assessment tool; measures gross
tasks related to birth date motor, fine motor, language, and personal-
B. Mental age: assessment of cognitive development social development from newborn-6 years
1. Measured by variety of standardized 2. Does not measure intelligence

5
intelligence tests (IQ), such as the Stanford
Binet Intelligence Scale

PEDIATRIC NURSING 423


53155_05_Ch05_p421-532.qxd 2/27/09 1:32 PM Page 424

Table 5-4 Stages, Age Ranges, and Characteristics of Human Development Related to Pediatric Nursing

Stage Age Characteristics


Infant Birth to 1 year Period of rapid growth and change; attachments to family members
and other caregivers are formed; trust develops.
Toddler 1 to 3 years Motor ability, coordination, sensory skills developing; basic feelings,
emotions, a sense of self, and being independent becomes
important.
Preschooler 3 to 6 years Continued physiological, psychological, and cognitive growth; better
able to care for selves; interested in playing with other children;
beginning to develop a concept of who they are.
School age 6 to 12 years Interested in achievement; ability to read, write, and complete
academic work advances; understanding of the world
broadens.
Adolescent 12 to 19 years, or later Transition period between childhood and adulthood; physiological
maturation occurs, formal operational thought begins; preparation
for becoming an adult takes place.

D. Growth parameters 5) Weak neck muscles result in poor head


1. Bone age: X-ray of tarsals and carpals control
determines degree of ossification d. Vital signs
2. Growth charts: norms are expressed as 1) Pulse: 110160 and irregular; count for
percentile of height, weight, head a full minute apically
circumference, and body mass index (BMI) 2) Respirations: 3260 and irregular;
for age; any child who crosses over multiple count for full minute; neonates are
percentile lines or is above the 95th or abdominal breathers and obligate nose
below the 5th percentile needs further breathers
evaluation. 3) Blood pressure: 75/49 mm Hg
E. Correction for prematurity 4) Poor development of sweating and
1. Subtract time premature from chronological age shivering mechanisms; impaired
2. Use corrected age for developmental temperature control
assessment until age 2 e. Motor development
1) Behavior is reflex controlled
2) Flexed extremities
Developmental Stages 3) Can lift head slightly off bed when prone
Infant (Birth through 12 months) f. Sensory development
1) Hearing and touch well developed at
A. Physical tasks birth
1. Neonate (Birth to 1 month) 2) Sight not fully developed until 6 years
a. Weight: 68 lb (27503629 g); gains 57 oz a) Differentiates light and dark at
(142198 g) weekly for first 6 months birth
b. Length: 20 inches (50 cm); grows 1 inch b) Rapidly develops clarity of vision
(2.5 cm) monthly for first 6 months within 1 foot
c. Head growth c) Fixates on moving objects
1) Head circumference 3335.5 cm d) Strabismus due to lack of
(1314 inches) binocular vision
2) Head circumference slightly larger 2. 14 months
than chest a. Head growth: posterior fontanel closes
3) Increases by 12 inch (1.25 cm) monthly b. Motor development
for first 6 months 1) Reflexes begin to fade (e.g., Moro,
4) Brain growth related to myelinization tonic neck)
of nerve fibers; increase in size of 2) Gains head control; balances head
brain reflects this process, reaches 23 in sitting position
adult size at 1 year; 90% adult size at 3) Rolls from back to side
2 years 4) Begins voluntary hand-to-mouth activity

5 424 NCLEX-RN Review


53155_05_Ch05_p421-532.qxd 2/27/09 1:32 PM Page 425

c. Sensory development 5. 1012 months


1) Begins to be able to coordinate stimuli a. Weight: birth weight tripled
from various sense organs b. Length: 50% increase over birth length
2) Hearing: locates sounds by turning c. Head and chest circumference equal
head and visually searching d. Teething
3) Vision 1) Lower lateral incisors erupt
a) Binocular vision developing; less 2) Average of eight deciduous teeth
strabismus e. Motor development
b) Beginning hand-eye coordination 1) Creeps with abdomen off floor
c) Prefers human face 2) Walks with help or cruises
d) Follows objects 180 3) May attempt to stand alone
e) Ability to accommodate is equal to 4) Can sit down from upright position
adult 5) Weans from bottle to cup
3. 56 months f. Sensory development: vision
a. Weight: birth weight doubles; gains 35 oz 1) Able to discriminate simple geometric
(84140 g) weekly for next 6 months forms
b. Length: gains 12 inch (1.25 cm) for next 2) Able to follow rapidly moving objects
6 months 3) Visual acuity 20/50 or better
c. Eruption of teeth begins 4) Binocularity well established; if not,
1) Lower incisors first amblyopia may develop
2) Causes increased saliva and drooling B. Psychosocial tasks
3) Enzyme released with teething 1. Neonatal period
causes mild diarrhea, facial skin a. Cries to express displeasure
irritation b. Smiles indiscriminately
4) Slight fever may be associated with c. Receives gratification through sucking
teething, but not a high fever or seizures d. Makes throaty sounds
d. Motor development 2. 14 months
1) Intentional rolling over a. Crying becomes differentiated at 1 month
2) Supports weight on arms 1) Decreases during awake periods
3) Creeping; pushes backward with 2) Ceases when parent in view
hands b. Vocalization distinct from crying at 1 month
4) Can grasp and let go voluntarily 1) Squeals to show pleasure at 3 months
5) Transfers toys from one hand to 2) Coos, babbles, laughs; vocalizes when
another smiling
6) Sits with support c. Socialization
e. Sensory development 1) Stares at parents faces when talking at
1) Hearing: can localize sounds above 1 month
and below ear 2) Smiles socially at 2 months
2) Vision: smiles at own mirror image 3) Shows excitement when happy at
and responds to facial expressions of 4 months
others 4) Demands attention, enjoys social
3) Taste: sucking needs have decreased interaction with people at 4 months
and cup weaning can begin; chewing, 3. 56 months
biting, and taste preferences begin to a. Vocalization: begins to imitate sounds
develop b. Socialization: recognizes parents, stranger
4. 79 months anxiety begins to develop; comfort habits
a. Teething continues begin
1) 7 months: upper central incisors 4. 79 months
2) 9 months: upper lateral incisors a. Vocalization: verbalizes all vowels and
b. Motor development most consonants
1) Sits unsupported; goes from prone to b. Socialization
sitting upright 1) Shows increased stranger anxiety and
2) Crawls; may go backwards initially anxiety over separation from parent
3) Pulls self to standing position 2) Exhibits aggressiveness by biting at
4) Develops finger-thumb opposition times
(pincer grasp) 3) Understands the word no
5) Preference for dominant hand evident 5. 1012 months
c. Sensory development: vision a. Vocalization: imitates animal sounds, can
1) Can fixate on small objects say only 45 words but understands many

5
2) Beginning to develop depth perception more (ma, da)

PEDIATRIC NURSING 425


53155_05_Ch05_p421-532.qxd 2/27/09 1:32 PM Page 426

b. Socialization f. Chopped table food or junior food can be


1) Begins to explore surroundings introduced by 12 months.
2) Plays games such as pat-a-cake, g. Weaning from breast or bottle to cup
peek-a-boo should be gradual during second 6 months.
3) Shows emotions such as jealousy, h. Breastfeeding can continue beyond
affection, anger, fear (especially in new 12 months.
situations) i. No honey, nuts, egg whites until 12 months.
C. Cognitive tasks E. Safety
1. Neonatal period: reflexive behavior only 1. Birth to 4 months
2. 14 months a. Use car seat properly. Infants up to 9 kg
a. Recognizes familiar faces (20 lb) and younger than 1 year should
b. Is interested in surroundings face rear.
c. Discovers own body parts b. Ensure crib mattress fits snugly; do not use
3. 56 months a pillow or comforters in the crib.
a. Begins to imitate c. Keep side rails of crib up.
b. Can find partially hidden objects d. Position infant supine for sleep until infant
4. 79 months is able to turn over. Prone position may
a. Begins to understand object permanence; increase risk for sudden infant death
searches for dropped objects syndrome (SIDS).
b. Reacts to adult anger; cries when scolded e. Do not leave infant unattended on bed,
c. Imitates simple acts and noises couch, table.
d. Responds to simple commands f. Do not tie pacifier on string around infants
5. 1012 months neck; remove bib before sleep.
a. Recognizes objects by name g. Remove small objects that infant could
b. Looks at and follows pictures in book choke on.
c. Shows more goal-directed actions h. Check temperature of bath water and
D. Nutrition warmed formula or food.
1. Birth to 6 months i. Use cool mist vaporizer.
a. Breast milk is a complete and healthful 2. 57 months
diet; supplementation may include 0.25 mg a. Restrain in high chair or infant seat.
fluoride, 400 International Units vitamin D, b. Do not feed hard candy, nuts, food with pits.
and iron after 4 months. c. Inspect toys for small removable parts.
b. Commercial iron-fortified formula is d. Be sure paint on furniture does not contain
acceptable alternative; supplementation lead.
may include 0.25 mg fluoride if water e. Keep phone number of Poison Control
supply is not fluoridated. Center readily available.
c. No solid foods before 5 months; too early 3. 812 months
exposure may lead to food allergies, and a. Keep crib away from other furniture and
extrusion reflex will cause food to be windows.
pushed out of mouth. b. Keep gates across stairways.
d. Juices may be introduced at 56 months, c. Keep safety plugs in electrical outlets.
diluted 1:1 and preferably given by cup. d. Remove hanging electrical wires and
2. 612 months tablecloths.
a. Breast milk or formula continues to be e. Use child protective caps and cabinet locks.
primary source of nutrition. f. Place cleaning solutions and medications
b. Introduction of solid foods starts with out of reach.
cereal (usually rice cereal), which is g. Do not let child use fork to self-feed.
continued until 18 months. h. Do not leave alone in bathtub.
c. Introduction of other food is arbitrary; F. Play (Solitary)
most common sequence is fruits, 1. Birth to 4 months
vegetables, meats. a. Provide variety of brightly colored objects,
1) Introduce only one new food at a time. different sizes and textures.
2) Separate new foods by minimum of b. Hang mobiles within 810 inches of
34 days. infants face.
3) Decrease amount of formula to about c. Expose to various environmental sounds;
30 oz. as foods are added. use rattles, musical toys.
d. Iron supplementation can be stopped. 2. 57 months
e. Finger foods such as cheese, meat, carrots a. Provide brightly colored toys to hold and
can be started around 10 months. squeeze.

5 426 NCLEX-RN Review


53155_05_Ch05_p421-532.qxd 2/27/09 1:32 PM Page 427

b. Allow infant to splash in bath. 9. Toilet training usually completed by 3 years.


c. Provide crib mirror. a. Demonstrates readiness for toilet training
3. 812 months between 18 and 24 months
a. Provide toys with movable parts and b. Indicators of readiness: walks, sits, and
noisemakers; stack toys, blocks; pots, pans, squats well, has voluntary control of bowel
drums to bang on; stationary activity center and urinary function, regular bowel
and push-pull toys. movements, can communicate wetness or
b. Plays games: hide and seek, pat-a-cake. bowel movement, can remove clothes,
G. Fears wants to please caregivers, imitates
1. Separation from parents c. Daytime bladder control by 23 years
a. Searches for parents with eyes. d. Nighttime bladder control by 34 years
b. Shows preference for parents. C. Cognitive tasks
c. Develops stranger anxiety around 6 months. 1. Follows simple directions by 2 years.
2. Pain 2. Begins to use short sentences at 18 months to
a. Reacts with generalized body movement 2 years.
and loud crying. 3. Can remember and repeat 3 numbers by 3 years.
b. Can be distracted with talking, sucking 4. Knows own name by 12 months; refers to self,
opportunities. gives first name by 24 months; gives full name
by 3 years.
Toddler (12 months to 3 years) 5. Able to identify geometric forms by 18 months.
6. Achieves object permanence; is aware that
A. Physical tasks: this is a period of slow growth objects exist even if not in view.
1. Weight: gain of approximately 11 lb (5 kg) 7. Uses magical thinking; believes own feelings
during this time; birth weight quadrupled by affect events (e.g., anger causes rain).
212 years 8. Uses ritualistic behavior; repeats skills to
2. Height: grows 20.3 cm (8 inches); adult height master them and to decrease anxiety.
about 2 times height at 2 years 9. May develop dependency on transitional
3. Head circumference: 191220 inches object such as blanket or stuffed animal.
(4950 cm) by 2 years; anterior fontanel D. Nutrition
closes by 18 months 1. Caloric requirement is approximately
4. Pulse 110; respirations 26; blood pressure 99/64 100 calories/kg/day.
5. Primary dentition (20 teeth) completed by 2. Increased need for calcium, iron, and
212 years phosphorus.
6. Develops sphincter control necessary for 3. Needs 1624 oz milk/day.
bowel and bladder control 4. Appetite decreases.
7. Mobility 5. Able to feed self.
a. Walks alone by 18 months. 6. Negativism may interfere with eating.
b. Climbs stairs and furniture by 18 months. 7. Initial dental examination at 3 years.
c. Runs fairly well by 2 years. E. Safety
d. Jumps from chair or step by 212 years. 1. Turn pot handles toward back of stove.
e. Balances on one foot momentarily by 2. Teach swimming and water safety; supervise
212 years. near water.
f. Rides tricycle by 3 years. 3. Supervise play outdoors.
B. Psychosocial tasks 4. Avoid large chunks of meat, particularly hot dogs.
1. Increases independence; better able to tolerate 5. Do not allow child to walk around with objects
separation from primary caregiver. such as lollipops in mouth.
2. Less likely to fear strangers. 6. Know when and how to use ipecac.
3. Able to help with dressing/undressing at 7. Car seat safety: children sit in forward facing
18 months; dresses self at 24 months. car seat only after age is greater than 1 year
4. Has sustained attention span. and weight is greater than 20 lb. All car seats
5. May have temper tantrums during this period; placed in rear seat of car. No car seats should
should decrease by 212 years. be placed in front of the passenger side air bag.
6. Vocabulary increases from about 1020 words F. Play
to over 900 words by 3 years. 1. Predominantly parallel play period.
7. Has beginning awareness of ownership 2. Imitation of adults often part of play.
(my, mine) at 18 months; shows proper use 3. Begins imaginative and make-believe play.
of pronouns (I, me, you) by 3 years. 4. Provide toys appropriate for increased
8. Moves from hoarding and possessiveness locomotive skills: push toys, rocking horse,
at 18 months to sharing with peers by riding toys or tricycles; swings and slide.

5
3 years.

PEDIATRIC NURSING 427


53155_05_Ch05_p421-532.qxd 2/27/09 1:32 PM Page 428

5. Give toys to provide outlet for aggressive 3) Copies a diamond and triangle, prints
feelings: work bench, toy hammer and nails, letters and numbers by 5 years.
drums, pots, pans. e. Handles scissors well by 5 years.
6. Provide toys to help develop fine motor skills, B. Psychosocial tasks
problem-solving abilities: puzzles, blocks; 1. Becomes independent
finger paints, crayons. a. Feeds self completely.
G. Fears: separation anxiety b. Dresses self.
1. Learning to tolerate and master brief periods c. Takes increased responsibility for actions.
of separation is important developmental task. 2. Aggressiveness and impatience peak at 4 years
2. Increasing understanding of object then abate; by 5 years child is eager to please
permanence helps toddler overcome this fear. and manners become more evident.
3. Potential patterns of response to separation 3. Gender-specific behavior is evident by 5 years.
a. Protest: screams and cries when mother 4. Egocentricity changes to awareness of others;
leaves; attempts to call her back. rules become important; understands sharing.
b. Despair: whimpers, clutches transitional C. Cognitive development
object, curls up in bed, decreased activity, 1. Focuses on one idea at a time; cannot look at
rocking. entire perspective.
c. Denial: resumes normal activity but does 2. Awareness of racial and sexual differences begins.
not form psychosocial relationships; when a. Prejudice may develop based on values of
mother returns, child ignores her. parents.
4. Bedtime may represent desertion. b. Manifests sexual curiosity.
c. Sexual education begins.
Preschooler (3 to 5 years) d. Beginning body awareness.
3. Has beginning concept of causality.
A. Physical tasks 4. Understanding of time develops during this
1. Slower growth rate continues period.
a. Weight: increases 46 lb (1.82.7 kg) a year a. Learns sequence of daily events.
b. Height: increases 212 inches (56.25 cm) b. Is able to understand meaning of some
a year time-oriented words (day of week, month,
c. Birth length doubled by 4 years etc.) by 5 years.
2. Vital signs decrease slightly 5. Has 2000-word vocabulary by 5 years.
a. Pulse 90100 6. Can name four or more colors by 5 years.
b. Respirations 2425/minute 7. Is very inquisitive.
c. Blood pressure: systolic 85100 mm Hg: D. Nutrition
diastolic 6070 mm Hg 1. Caloric requirement is approximately
3. Permanent teeth may appear late in preschool 90 calories/kg/day.
period; first permanent teeth are molars, 2. May demonstrate strong taste preferences.
behind last temporary teeth. 3. More likely to taste new foods if child can
4. Gross motor development assist in the preparation.
a. Walks up stairs using alternate feet by E. Safety
3 years. 1. Safety issues similar to toddler
b. Walks down stairs using alternate feet by 2. Education of children concerning potential
4 years. dangers important during this period
c. Rides tricycle by 3 years. 3. Car safety: children 2040 lb and younger than
d. Stands on 1 foot by 3 years. age 4 should ride in car safety seat. Children
e. Hops on 1 foot by 4 years. over 40 lb and between ages 4 and 8 should
f. Skips and hops on alternate feet by 5 years. ride in a booster seat in the rear of the car.
g. Balances on 1 foot with eyes closed by F. Play
5 years. 1. Predominantly associative play period.
h. Throws and catches ball by 5 years. 2. Enjoys imitative and dramatic play.
i. Jumps off 1 step by 3 years. a. Imitates same-sex role functions in play.
j. Jumps rope by 5 years. b. Enjoys dressing up, dollhouses, trucks,
5. Fine motor development cars, telephones, doctor and nurse kits.
a. Hand dominance is established by 5 years. 3. Provide toys to help develop gross motor
b. Builds a tower of blocks by 3 years. skills: tricycles, wagons, outdoor gym;
c. Ties shoes by 5 years. sandbox, wading pool.
d. Ability to draw changes over this time 4. Provide toys to encourage fine motor skills,
1) Copies circles, may add facial features self-expression, and cognitive development:
by 3 years. construction sets, blocks, carpentry tools; flash

5
2) Copies a square, traces a diamond by cards, illustrated books, puzzles; paints,
4 years. crayons, clay, simple sewing sets.

428 NCLEX-RN Review


53155_05_Ch05_p421-532.qxd 2/27/09 1:32 PM Page 429

5. Television, when supervised, can provide a 3. Peer relationships


quiet activity; some programs have a. Child makes first real friends during this
educational content. period.
6. Imaginary playmates common during this period. b. Is able to understand concepts of
a. More prevalent in bright children cooperation and compromise (assist in
b. Help child deal with loneliness and acquiring attitudes and values); learns fair
fears play vs competition.
c. Abandoned by school age c. Help child develop self-concept.
G. Fears d. Provide feeling of belonging.
1. Greatest number of imagined and real fears of 4. Enjoys family activities.
childhood during this period. 5. Has some ability to evaluate own strengths and
2. Fears concerning body integrity are common. weaknesses.
a. Child is able to imagine an event without 6. Has increased self-direction.
experiencing it. 7. Is aware of own body; compares self to others;
b. Observing injuries or pain in others can modesty develops.
precipitate fear. C. Cognitive development
c. Magical and animistic thinking allows 1. Period of industry
children to develop many illogical fears a. Is interested in exploration and adventure.
(fear of inanimate objects, the dark, ghosts). b. Likes to accomplish or produce.
3. Exposing child to feared object in a safe c. Develops confidence.
situation may provide a degree of conditioning; 2. Concept of time and space develops.
child should progress at own rate. a. Understands causality.
b. Masters concept of conservation:
School-Age (6 to 12 years) permanence of mass and volume; concept
of reversibility.
A. Physical tasks c. Develops classification skills: understands
1. Slow growth continues. relational terms; may collect things.
a. Height: 2 inches (5 cm) per year d. Masters arithmetic and reading.
b. Weight: doubles over this period D. Nutrition
c. At age 9, both sexes same size; age 12, girls 1. Caloric needs diminish in relation to body
bigger than boys size: 85 kcal/kg/day.
2. Dentition 2. Junk food may become a problem; excess
a. Loses first primary teeth at about 6 years. sugar, starches, fat.
b. By 12 years, has all permanent teeth except 3. Obesity is a risk in this age group.
final molars. 4. Nutrition education should be integrated into
3. Bone growth faster than muscle and ligament school program.
development; very limber but susceptible to E. Safety
bone fractures during this time. 1. Incidence of accidents is decreased when
4. Vision is completely mature; hand-eye compared with younger children.
coordination develops completely. 2. Motor vehicle accidents most common cause
5. Gross motor skills: predominantly involving of severe injury and death.
large muscles; children are very energetic, develop 3. Other common activities associated with
greater strength, coordination, and stamina. injuries include sports (skateboarding,
6. Develops smoothness and speed in fine motor rollerskating, etc.).
control. 4. Education and supervision are key elements
B. Psychosocial tasks in prevention.
1. School occupies half of waking hours; has a. Proper use of equipment
cognitive and social impact. b. Risk-taking behavior
a. Readiness includes emotional (attention 5. Car safety: children weighing over 40 lb and
span), physical (hearing and vision), and younger than age 8 should ride in a booster
intellectual components. seat placed in the rear of the car. Children over
b. Teacher may be parent substitute, causing age 8 can use shoulder/lap belt combination in
parents to lose some authority. rear seat of the car. Children younger than age
2. Morality develops 12 should not sit in the front passenger seat or
a. Before age 9 moral realism predominates: in front of an air bag.
strict superego, rule dominance; things are F. Play
black or white, right or wrong. 1. Rules and ritual dominate play; individuality
b. After age 9 autonomous morality develops: not tolerated by peers; knowing rules provides
recognizes differing points of view, sees sense of belonging; cooperative play.

5
gray areas.

PEDIATRIC NURSING 429


53155_05_Ch05_p421-532.qxd 2/27/09 1:32 PM Page 430

2. Team play: games or sports 5. Sexual development: boys


a. Help learn value of individual skills and a. Development of secondary sex
team accomplishments. characteristics, sex organs and function
b. Help learn nature of competition. under hormonal control (see Table 4-24).
3. Quiet games and activities: board games, b. Enlargement of testes is first sign of sexual
collections, books, television, painting maturation; occurs at approximately age
4. Athletic activities: swimming, hiking, 13, about 1 year before growth spurt.
bicycling, skating c. Scrotum and penis increase in size until
G. Fears: more realistic fears than younger children; age 18.
include death, disease or bodily injury, d. Reaches reproductive maturity about age
punishment; school phobia may develop, resulting 17, with viable sperm.
in psychosomatic illness. e. Nocturnal emission: a physiologic reflex to
ejaculate buildup of semen; natural and
Adolescent (12 to 19 years) normal; occurs during sleep (child should
not be made to feel guilty; needs to
A. Physical tasks understand that this is not enuresis).
1. Fast period of growth f. Masturbation increases (also a normal way
2. Vital signs approach adult norms to release semen).
3. Puberty g. Pubic hair continues to grow and spread
a. Follows same pattern for all races and until mid 20s.
cultures. h. Facial hair; appears first on upper lip.
b. Is related to hormonal changes. i. Voice changes due to growth of laryngeal
c. Results in growth spurt, change in body cartilage.
structure, development of secondary sex j. Gynecomastia: slight hypertrophy of
characteristics, and reproductive maturity. breasts due to estrogen production; will
d. Girls: height increases approximately pass within months but causes
3 inches/year; slows at menarche; stops embarrassment.
around age 16. B. Psychosocial tasks
e. Boys: growth spurt starts around age 1. Early adolescence: ages 1214 years
13; height increases 4 inches/year; slows a. Starts with puberty.
in late teens. b. Physical body changes result in an altered
f. Boys double weight between 12 and self-concept.
18, related to increased muscle mass. c. Tends to compare own body to others.
g. Body shape changes d. Early and late developers have anxiety
1) Boys become leaner with broader regarding fear of rejection.
chest. e. Fantasy life, daydreams, crushes are all
2) Girls have fat deposited in thighs, normal, help in role play of varying social
hips, and breasts; pelvis broadens. situations.
h. Apocrine glands cause increased body f. Is prone to mood swings.
odor. g. Needs limits and consistent discipline.
i. Increased production of sebum and 2. Middle adolescence: ages 1516 years
plugging of sebaceous ducts causes acne a. Is separate from parents (except financially).
4. Sexual development: girls b. Can identify own values.
a. Menarche c. Can define self (self-concept, strengths and
1) Onset about 2 years after first of weaknesses).
pubescent changes d. Involved with peer group; conforms to
2) Average age 1212 years values/fads.
3) First 12 years: menses irregular, e. Has increased heterosexual interest;
infertile communicates with opposite sex; may
b. Menstrual cycle: controlled by complex form love relationship.
interaction of hormones f. Sex education continues.
c. Development of secondary sex 3. Late adolescence: ages 1719 years
characteristics and sexual functioning a. Achieves greater independence.
under hormonal control (see Table 4-24). b. Chooses a vocation.
d. Breast development is first visible sign of c. Participates in society.
puberty. d. Finds an identity.
1) Bud stage: areola around nipple is e. Finds a mate.
protuberant. f. Develops own morality.
2) Breast development is complete g. Completes physical and emotional

5
around the time of first menses. maturity.

430 NCLEX-RN Review


53155_05_Ch05_p421-532.qxd 2/27/09 1:32 PM Page 431

C. Cognitive development PLANNING AND


1. Develops abstract thinking abilities.
2. Is often unrealistic. IMPLEMENTATION
3. Is capable of scientific reasoning and formal
logic. Goals
4. Enjoys intellectual abilities.
5. Is able to view problems comprehensively. A. Child will achieve appropriate developmental
D. Nutrition level for age.
1. Nutritional requirements peak during years B. Family/child will adapt successfully to
of maximum growth: age 1012 in girls, developmental changes.
2 years later in boys. C. Family/child will cope successfully with crises of
2. Appetite increases. illness and hospitalization.
3. Inadequate diet can retard growth and delay D. Family/child will cope successfully with issues
sexual maturation. related to death and dying.
4. Food intake needs to be balanced with energy
expenditure. Interventions for the Ill
5. Increased needs include calcium for skeletal
growth; iron for increased muscle mass and or Hospitalized Child
blood cell development; zinc for development
Communicating with Children
of skeletal and muscle tissue and sexual
maturation. A. Speak in quiet, pleasant tones.
E. Safety B. Bend down to meet child on own level.
1. Accidents are leading cause of death: motor C. Use words appropriate to age/communication
vehicle accidents, sports injuries, firearms ability; do not use clichs.
accidents. D. Do not explain more than is necessary.
2. Safety measures include education about E. Always explain what you are going to do and give
proper use of equipment and caution the reason for it.
concerning risk taking. F. Be honest; do not lie about whether something will
3. Drug and alcohol use may be a serious hurt.
problem during this period. G. Do not make a promise you know you cannot keep.
4. Adolescent characteristics of poor impulse H. Observe nonverbal behavior for clues to level of
control and recklessness make prevention understanding.
complex. I. Do not threaten; and when necessary, punish the act,
F. Activities: group activities predominate (sports are not the child (I like you, but not what you did.).
important); activities involving opposite sex by J. Never shame a child by using terms like baby or
middle adolescence. sissy.
G. Fears K. Allow child to show feelings (hurt and anger);
1. Threats to body image: acne, obesity provide therapeutic play, pounding or throwing
2. Injury or death toys; allow child to cry; encourage drawing and
3. The unknown creative writing.
L. Provide time to talk; encourage a trusting
ANALYSIS environment where the child can talk without
embarrassment and confide without fear.
Nursing diagnoses for problems of growth and M. Provide support to child and parents/family.
development may include: N. Teach parents to anticipate next stage of
A. Altered family process development.
B. Altered health maintenance O. If teaching with a child is interrupted, start over
C. Altered parenting from the beginning.
D. Altered thought processes P. Promote independence; allow the child to perform
E. Delayed growth and development as many self-care activities as possible.
F. Disturbance in self-esteem Q. Do not compare childs progress to that of anyone else.
G. Disturbed body image R. Provide praise at every opportunity.
H. High risk for violence S. Instead of asking what something is, ask child to
I. Impaired dentition give it a name or tell you about it.
J. Ineffective family coping T. Allow choices where possible, but do not use
K. Knowledge deficit (specify) yes/no questions unless you can accept a no
L. Risk for delayed development answer (It is time for your medication now; do
M. Risk for disproportionate growth you want it with milk or juice? versus Do you
N. Social isolation want your medication now?).

5
U. Involve parents in childs care.

PEDIATRIC NURSING 431


53155_05_Ch05_p421-532.qxd 2/27/09 1:32 PM Page 432

V. Keep routines as much like home as possible (on J. Inadequate preparation leads to heightened
admission, ask parents about routines such as anxiety that may result in regressive behavior,
toileting, eating, sleeping, and names for bowel uncooperativeness, or acting out.
movements and urination).
W. Allow parents time and opportunities to ask
questions and express themselves. EVALUATION
X. If parents cannot stay with child, encourage them to
bring in a favorite toy, pictures of family members, A. Child maintains normal developmental level
or to make a tape to be played for the child. during hospitalization.
B. Parents participate in care of child during
Play hospitalization.

A. Play is a way to solve problems, become


enculturated, express creativity, decrease stress in GROWTH AND DEVELOPMENT
the environment, prepare for different situations,
sublimate sensations, enhance fine and gross ISSUES
motor development as well as social development.
B. Make play appropriate for mental age and Health Promotion
physical/disease state (e.g., appropriate for oxygen
tents, isolation, hearing or vision defects). A. Immunization schedule (see Figure 5-1)
C. Use multisensory stimulation. B. Types of immunity
D. Provide toys safe for mental age (no points, sharp C. Considerations concerning immunization schedule:
edges, small parts, loud noises, propelled objects). 1. If the immunization schedule is interrupted it
E. Offer play specific to age group. is not necessary to reinstitute the entire series.
1. Toddler: enjoys repetition; solitary play, Immunization should occur on the next visit
parallel play. as if the usual interval has elapsed.
2. Preschooler: likes to role play and make 2. If immunization status is unknown, children
believe; associative play. should be considered susceptible and
3. School-age: likes group, organized activities appropriate immunizations administered
(to enhance sharing); cooperative play, group 3. For children not immunized during the first
goals with interaction. year of life and who are less than 7 years old
the same immunizations are given but
Preparation for Procedures following different time schedule.
4. For children 7 years old and older who are not
A. Allow child to play with equipment to be used. immunized, Td rather than DTaP is administered.
B. Demonstrate procedure first on a doll. 5. Preterm infants are immunized according to
C. Teach child skills that will be needed after the chronological, not corrected age.
procedure and provide time to practice (crutches, 6. Minor illnesses are not contraindications to
blow bottles). immunization.
D. Show the child pictures of staff garb, special D. Contraindications for immunization
treatment room, special machines to be used, etc., 1. Severe allergic reaction to a vaccine
before the procedure. contraindicates further doses of that vaccine.
E. Describe sensations the child may experience 2. Anaphylactic reaction to a vaccine additive
during or after the procedure and what child will contraindicates the use of vaccines containing
have to do. that substance (e.g., eggs, neomycin,
F. Listen carefully to child to detect misconceptions streptomycin).
or fantasies. 3. Immunocompromised persons should not
G. With younger children, the preparatory receive live vaccines.
information should be simple and as close to the 4. Immunizations should be delayed after recent
time of the procedure as possible. transfusion with passive immunity agents
H. Parents can often be helpful in preparing child for (e.g., gamma globulin).
procedures, but need to be prepared as well. E. Tuberculin testing
1. May need different explanation, away from child. 1. The tuberculin skin test is the only practical
2. Should have opportunity to ask questions tool for diagnosing tuberculosis infection.
about what will happen to child. 2. Tuberculin testing may be done at the same
I. School-age children and adolescents may not wish visit at which an immunization is being given.
parents to be present during procedure. 3. Routine testing is no longer recommended.
1. Childs desires should be confirmed. Testing is always indicated for individuals
2. Parents need to be assured that this is not with known contact with a person with

5
rejection by child. tuberculosis disease.

432 NCLEX-RN Review


53155_05_Ch05_p421-532.qxd 2/27/09 1:32 PM Page 433

Figure 5-1 Recommended childhood immunization schedule, 2009 (Courtesy of U.S. Centers for Disease
Control and Prevention. Retrieved February 24, 2009 from cdc.gov)

PEDIATRIC NURSING

5
433
53155_05_Ch05_p421-532.qxd 2/27/09 1:32 PM Page 434

4. Positive reaction signifies infection with b. In sexual abuse, 80% of children know
Mycobacterium tuberculosis. their abuser.
5. Positive reaction indicates need for further c. Problem usually related to parents limited
evaluation. capacity to cope with, provide for, or relate
6. Children from other countries who have to a child and/or to each other.
received BCG vaccine against tuberculosis d. Adults who abuse were often themselves
may show positive skin test. victims of child abuse; although abuser
F. Common childhood communicable diseases may care about child, pattern of response to
(see Table 5-5). frustration and discipline is to be abusive.
e. Occurs in all socioeconomic groups.
f. Only 10% of abusers have serious
Challenges of Parenting psychologic disturbances, but most have
A. Failure to thrive (FTT) low self-esteem, little confidence, low
1. General information tolerance for frustration.
a. A condition in which a child fails to gain g. Abuse is most common among toddlers as
weight and is persistently less than the 5th they exercise autonomy and parents may
percentile on growth charts. sense loss of power.
b. When related to nonorganic cause, it is 2. Assessment findings
usually due to a disrupted maternal-child a. History may be indicative of child abuse.
relationship. 1) History inconsistent with injury
c. Other pathology (especially absorption 2) Delay in seeking medical attention
problems and hormonal dysfunction) must 3) History changes with repetition
be ruled out before a disorder can be 4) No explanation for injury
diagnosed as FTT. b. Skin injuries (bruises, lacerations, burns)
d. Growth and developmental delay usually are most common; may show outline of
improve with appropriate stimulation. instrument used and may be in varying
2. Assessment findings stages of healing.
a. Sleep disturbances; rumination (voluntary c. Musculoskeletal injuries, fractures
regurgitation and reswallowing) (especially chip or spiral fractures),
b. History of parental isolation and social sprains, dislocations are also common;
crisis with inadequate support systems X-rays may show multiple old fractures.
c. Physical exam reveals delayed growth and d. Signs of central nervous system (CNS)
development (decreased vocalization, low injuries include subdural hematoma, retinal
interest in environment) and characteristic hemorrhage (shaken baby syndrome).
postures (child is stiff or floppy, resists e. Abdominal injuries may include lacerated
cuddling) liver, ruptured spleen.
d. Disturbed maternal-infant interaction may f. Observation of parents and child may
be demonstrated in feeding techniques, reveal interactional problems.
amount of stimulation provided by mother, 1) Does parent respond to childs cues?
ability of mother to respond to infants cues 2) Does parent comfort child?
3. Nursing interventions 3) Does child respond to parent with fear?
a. Provide consistent care. 3. Nursing interventions
b. Teach parents positive feeding techniques. a. In emergency room: tend to physical needs
1) Provide quiet environment. of child first; determination of existence of
2) Follow childs rhythm of feeding. abuse must wait until childs condition is
3) Maintain face-to-face posture with child. stable.
4) Talk to child encouragingly during b. Report suspected child abuse to
feeding. appropriate agency.
c. Involve parents in care. c. Provide a role model for parents in terms
1) Provide supportive environment. of communication, stimulation, feeding,
2) Give positive feedback. and daily care of child.
3) Demonstrate and reinforce responding d. Encourage parents to be involved in childs
to childs cues. care.
d. Refer to appropriate community agencies. e. Encourage parents to express feelings
B. Child abuse concerning abuse, hospitalization, and
1. General information home situation.
a. Physical, emotional, or sexual abuse of 1) Feelings of fear and guilt should be
children: may result from intentional and acknowledged.
nonaccidental actions; or may be from 2) Provide reassurance.

5
intentional and nonaccidental acts of
omission (neglect).

434 NCLEX-RN Review


53155_05_Ch05_p421-532.qxd 2/27/09 1:32 PM Page 435

Table 5-5 Communicable Childhood Diseases

Disease Characteristics Immunization


Diphtheria A respiratory disease caused by bacteria. Included in DTaP up to 6 years, then in Td,
Bacteria forms a pseudomembrane across the trachea repeated every 10 years throughout life.
causing respiratory distress; also produces an
exotoxin that causes myocarditis and neurologic
problems.
Pertussis Respiratory disease caused by bacteria; life threatening Included in DTaP; not given after 6 years
(whooping in young children. because of risk from associated side effects.
cough) Severe paroxysmal cough results in severe respiratory Do not give pertussis vaccine if child has active
distress; complications include seizures, pneumonia, neurologic disorder.
encephalopathy, and death.
Tetanus (lockjaw) Neurologic disorder caused by bacterial exotoxin Included in DTaP up to 6 years; included in Td
affects motor neurons, causing rigidity and spastic every 10 years throughout life.
muscles; first symptom is stiffness of the jaw May be given with a puncture wound if the
(trismus). wound is dirty and no immunization has
No immunity is conferred after having the disease; been given in 5 years, or if wound is clean
associated mortality 2550%. but more than 10 years have elapsed since
previous immunization.
Measles (rubeola) Viral infection producing harsh cough, maculopapular Maternal antibodies last for at least a year,
rash, photophobia, and Koplik spots; complications then included in MMR given at 12 to
may include pneumonia, bacterial superinfections, 15 months.
and encephalitis. Do not give to pregnant women or
Incubation period is 8 to 12 days. immunocompromised persons.
Care includes keeping room darkened and providing
antipruritic measures.
German measles Viral infection causing lymphadenopathy and pink Included in MMR
(rubella) maculopapular rash; very mild disease, no specific
care needed; complications may include arthralgia or
arthritis, especially if occurring in young adults.
Greatest danger is if pregnant woman contracts the
disease; causes serious congenital anomalies.
Mumps (parotitis) Viral infection causing swelling of the salivary glands Included in MMR
with painful swallowing.
Ice collar may help relieve discomfort.
Complications include orchitis (usually unilateral) if
disease occurs after puberty, aseptic meningitis,
encephalitis.
Poliomyelitis Viral infection, 95% of infected clients have no IPV
(polio) symptoms.
Virus multiplies in the GI tract and enters the
bloodstream to affect the CNS, resulting in paralysis
in less than 2% of infected.
Chickenpox Most common communicable childhood disease, Short-term protection from maternal
(varicella) caused by the varicella zoster virus. antibodies.
Causes rash that starts on the trunk and spreads. Varicella vaccine.
Rash starts as vesicles, which then erupt and crust over.
Highly contagious from 2 days prior to rash to 6 days
after rash erupts; incubation period 21 days.
Once lesions have crusted or scabbed over they are no
longer contagious.
Care is directed at comfort measures.

PEDIATRIC NURSING

5 435
53155_05_Ch05_p421-532.qxd 2/27/09 1:32 PM Page 436

f. Provide family education concerning child b. Cannot be predicted; cannot be prevented


care, especially safety and nutrition needs, (unexpected and unexplained)
discipline, and age-appropriate stimulation. c. Peak age: 3 months; 95% by 6 months
g. Initiate referrals for long-term follow-up d. Usually occurs during sleep; there is no
(community agencies, pediatric and mental struggle and death is silent
health clinics, self-help groups). e. Diagnosis made at autopsy
C. Learning disabilities f. Although cause of death is not known,
1. General information suffocation and DTaP reactions are not
a. A heterogeneous group of disorders causes of SIDS
manifested by significant difficulties in 2. Assessment findings
acquisition and use of listening, speaking, a. Factors associated with increased SIDS
writing, reasoning, or math skills risk: prematurity, low birth weight,
b. Presumed to be due to CNS dysfunction multiple births, siblings of SIDS victims,
c. Children of average or above-average IQ maternal substance abuse
d. Affects all aspects of learning, not just b. Infants with neurologic problems and
academics abnormal respiratory function at higher risk
e. Boys affected 6 times more often than girls c. Co-sleeping with parents, prone sleep
f. Categories include: position, soft bedding associated with
1) Receptive/sensory: perceptual problem higher risk
(dyslexia, visual misperception) 3. Nursing interventions
2) Integrative: difficulty processing a. Nursing care is directed at supporting
information (analysis, organization, parents/family; parents usually arrive at
sequencing, abstract thought) emergency department.
3) Expressive: motor dysfunction (aphasia, b. Provide a room for the family to be alone if
writing or drawing difficulties, possible, stay with them; prepare them for
difficulty in sports or games) how infant will look and feel (baby will be
4) Diffuse: combination of above bruised and blanched due to pooling of
2. Assessment findings blood until death was discovered; also will
a. Poor attention span be cold).
b. Poor grades, normal IQ c. Let parents say good-bye to baby (hold, rock).
c. Low general information scores on d. Reinforce that death was not their fault.
standardized IQ tests e. Provide appropriate support referrals:
d. Decreased participation in extracurricular clergy, notification of significant others,
activities or hobbies local SIDS program, visiting nurse.
e. Low self-esteem due to multiple failures f. Explain how parents can receive autopsy
f. Diagnostic tests: specific testing to confirm results.
diagnosis and determine type of defect g. Notify family physician or pediatrician.
3. Medical management: psychostimulants may
be prescribed to reduce hyperactivity and
frustration and to increase attention span and DEATH AND DYING
self-control; side effects include anorexia.
4. Nursing interventions
a. Environmental manipulation for behavior
Overview
management Parental Response to Death
1) Limit external stimuli.
2) Maintain predictable routines. A. Major life stress event
3) Enforce limits on behavior. B. Initially parents experience grief in response to
b. Teaching strategies tailored for childs potential loss of child
specific defects 1. Acknowledgment of terminal disease is a
1) Repeat directions often. struggle between hope and despair with
2) Elicit feedback from child. resultant awareness of inevitable death.
3) Give time to ask questions. 2. Parents will be at different stages of grief at
4) Keep teaching sessions short. different times and constantly changing.
5) Do not give nonessential information. C. Parental response is related to age of child, cause
D. Sudden infant death syndrome (SIDS) of death, available social support, and degree of
1. General information uncertainty; response might include denial, shock,
a. Sudden death of any young child that is disbelief, guilt.
unexpected by history and in which D. Parents often confronted with major decisions such as
thorough postmortem examination fails to home care versus hospital care, use of investigational

5
demonstrate adequate cause of death drugs, and continuation of life supports.

436 NCLEX-RN Review


53155_05_Ch05_p421-532.qxd 2/27/09 1:32 PM Page 437

E. May have long-term disruptive effects on family Care Guidelines at Impending Death
system
1. Stress may result in divorce. A. Do not leave child alone.
2. May contribute to behavioral problems or B. Do not whisper in the room (increases fears).
psychosomatic symptoms in siblings. C. Know that touching child is important.
F. Bereaved parents experience intense grief of long D. Let the child and family talk and cry.
duration. E. Continue to read favorite stories to child or play
favorite music.
Childs Response to Death F. Let parents participate in care as far as they are
emotionally capable.
A. Childs concept of death depends on mental age. G. Be aware of the needs of siblings who are in the
1. Infants and toddlers room with the family.
a. Live only in present.
b. Are concerned only with separation from
mother and being alone and abandoned.
c. Can sense sadness in others and may feel Sample Questions
guilty (due to magical thinking).
d. Do not understand life without themselves.
e. Can sense they are getting weaker. 1. The nurse has assessed four children of varying
f. Healthy toddlers may insist on seeing a ages; which one requires further evaluation?
significant other long after that persons death. 1. A 7-month-old who is afraid of strangers.
2. Preschoolers 2. A 4-year-old who talks to an imaginary
a. See death as temporary; a type of sleep or playmate.
separation.
b. See life as concrete; they know the word 3. A 9-year-old with enuresis.
dead but do not understand the finality. 4. A 16-year-old male who had nocturnal
c. Fear separation from parents; want to emissions.
know who will take care of them when
they are dead. 2. The nurse is caring for a 5-year-old child who
d. Dying children may regress in their behavior. has leukemia and is now out of remission and
3. School-age not expected to survive. The child says to his
a. Have a concept of time, causality, and mother, Will you take care of me when I am
irreversibility (but still question it). dead the way you do now? The childs mother
b. Fear pain, mutilation, and abandonment. asks the nurse how to answer her child. The
c. Will ask directly if they are dying. nurses response should be based on which of
d. See death as a period of immobility. the following understandings of the childs
e. Interested in the death ceremony; may behavior?
make requests for own ceremony. 1. The child is denying that he has a terminal
f. Feel death is punishment. illness.
g. May personify death (bogey man, angel of
death). 2. The child may be hallucinating.
h. May know they are going to die but feel 3. Children of this age do not understand the
comforted by having parents and loved finality of death.
ones with them. 4. Most 5-year-old children have a great fear of
4. Adolescents mutilation.
a. Are thinking about the future and knowing
they will not participate. 3. The nurse is talking with the mother of a 1-year-
b. May express anger at impending death. old child in well-baby clinic. Which statement
c. May find it difficult to talk about death. the mother makes indicates a need for more
d. Have an accurate understanding of death. instruction in keeping the child safe?
e. May wish to write something for friends and 1. I have some syrup of ipecac at home in case
family, make things to leave, or make a tape.
my child ever needs it.
f. May wish to plan own funeral.
2. I put all the medicines on the highest shelf
in the kitchen.
Nursing Implications 3. We have moved all the valuable vases and
Communicating with Dying Child figurines out of the family room.
4. My husband put the gates up at the top and
A. Use the childs own language.
bottom of the stairs.
B. Do not use euphemisms.

5
C. Do not expect an immediate response.
D. Never give up hope.

PEDIATRIC NURSING 437


53155_05_Ch05_p421-532.qxd 2/27/09 1:32 PM Page 438

4. A baby was born 6 weeks prematurely and is 10. The nurse is testing reflexes in a 4-month-old
now 2 months old, and her mother brings her to infant as part of the neurologic assessment.
the clinic for her checkup. What will determine Which of the following findings would indicate
if the baby can receive the DTaP? an abnormal reflex pattern and an area of
1. The presence of sufficient muscle mass. concern in a 4-month-old infant?
2. Whether the vaccines are live or inactive. 1. Closes hand tightly when palm is touched.
3. The Denver Developmental Screening results. 2. Begins strong sucking movements when
4. Calculating her age by subtracting 6 weeks mouth area is stimulated.
from the due date. 3. Hyperextends toes in response to stroking
sole of foot upward.
5. A 12-month-old is brought in for her well-child 4. Does not extend and abduct extremities in
checkup. All of the immunizations are up to response to loud noise.
date. The childs mother asks the nurse what
immunizations her child will receive today. 11. The mother of a 4-month-old infant asks the
What will be the nurses best response? nurse when she can start feeding her baby solid
1. First dose of MMR. food. Which of the following should the nurse
2. Second dose of Hib. include in teaching this mother about the
nutritional needs of infants?
3. Third dose of DtaP.
1. Infant cereal can be introduced by spoon
4. Final dose of IPV.
when the extrusion reflex fades.
6. The presence of what condition would 2. Solid foods should be given as soon as the
necessitate a change in the standard infants first tooth erupts.
immunization schedule for a child? 3. Pureed food can be offered when the infant
1. Allergy to eggs. has tripled his birth weight.
2. Immunosuppression. 4. Infant formula or breast milk provides
3. Congenital defects. adequate nutrients for the first year.
4. Mental retardation. 12. The nurse is assessing a 6-month-old infant
during a well-child visit. The nurse makes all of
7. A 2-year-old is brought to the pediatric clinic
the following observations. Which of the
with an upper respiratory infection. After
following assessments made by the nurse is an
assessing the child, the nurse suspects this child
area of concern indicating a need for further
may be a victim of child abuse. What physical
evaluation?
signs usually indicate child abuse?
1. Absence of Moro reflex.
1. Diaper rash.
2. Closed posterior fontanel.
2. Bruises on the lower legs.
3. Three-pound weight gain in 2 months.
3. Scraped and scabbed knees.
4. Moderate head lag when pulled to sitting
4. Welts or bruises in various stages of healing.
position.
8. Which action by a parent-child interaction does
13. The nurse is giving anticipatory guidance
NOT warrant further assessment when child
regarding safety and injury prevention to the
abuse is suspected?
parents of an 18-month-old toddler. Which of
1. Appears tired and disheveled. the following actions by the parents indicates
2. Is hypercritical of the child. understanding of the safety needs of a toddler?
3. Pushes the frightened child away. 1. Supervise the child in outdoor, fenced play
4. Expresses far more concern than the situation areas.
warrants. 2. Teach the child swimming and water safety.
3. Use automobile booster seat with lap belt.
9. When child abuse is suspected, what
recognizable factor will be present? 4. Allow child to cross the street with 4-year-
old sibling.
1. Have a number of scars.
2. Have identifiable shapes. 14. The community health nurse is making a
3. Display an erratic pattern. newborn follow-up home visit. During the visit
4. Be on one side of the body. the 2-year-old sibling has a temper tantrum. The
parent asks the nurse for guidance in dealing

5 438 NCLEX-RN Review


53155_05_Ch05_p421-532.qxd 2/27/09 1:32 PM Page 439

with the toddlers temper tantrums. Which of the 3. Refers to self as being too dumb and too small
following is the most appropriate nursing during the exam.
action? 4. Has lost three deciduous teeth and has the
1. Help the child understand the rules. central and lateral incisors.
2. Leave the child alone in his bedroom.
19. The nurse is performing a neurologic assessment
3. Suggest that the parent ignore the childs
on an 8-year-old child. As part of this neurologic
behavior.
assessment the nurse is assessing how the child
4. Explain that the toddler is zealous of the new thinks. Which of the following abilities best
baby. illustrates that the child is developing concrete
operational thought?
15. The parent of a 3-year-old child brings the child
to the clinic for a well-child checkup. The 1. Able to make change from a dollar bill.
history and assessment reveals the following 2. Describes a ball as both red and round.
findings. Which of these assessment findings 3. Tells time in terms of after breakfast and
made by the nurse is an area of concern and before lunch.
requires further investigation? 4. Able to substitute letters for numbers in
1. Unable to ride a tricycle. simple problems.
2. Has ability to hop on one foot.
20. The nurse is caring for a 10-year-old child
3. Uses gestures to indicate wants.
during the acute phase of rheumatic fever. Bed
4. Weight gain of 4 pounds in last year. rest is part of the childs plan of care. Which of
the following diversional activities is
16. The parents of a 4-year-old child tell the nurse
developmentally appropriate and meets the
that the child has an invisible friend named
health needs of this child in the acute phase of
Felix. The child blames Felix for any
rheumatic fever?
misbehavior and is often heard scolding Felix,
calling him a bad boy. The nurse understands 1. Using handheld computer video games.
that the best interpretation of this behavior is 2. Sorting and organizing baseball cards in a
which of the following? notebook.
1. A delay in moral development. 3. Playing basketball with a hoop suspended
2. Impaired parent-child relationship. from the bed.
3. A way for the child to assume control. 4. Using art supplies to make drawings about
the hospital experience.
4. Inconsistent parental discipline strategies.
21. The nurse is caring for a 13-year-old who has
17. The nurse is caring for a 5-year-old child who is
been casted following spinal instrumentation
in the terminal stages of acute leukemia. The
surgery to correct idiopathic scoliosis. The nurse
child refuses to go to sleep and is afraid that his
is helping the teen and family plan diversional
parents will leave. The nurse recognizes that the
activities while the teen is in the cast. Which of
child suspects he is dying and is afraid. Which
the following activities would be most
of the following questions about death is most
appropriate to support adolescent development
likely to be made by a 5-year-old child?
while the teen is casted?
1. What does it feel like when you die?
1. Take the teen shopping at the mall in a
2. Who will take care of me when I die? wheelchair.
3. What will my friends do when I die? 2. Plan family evenings playing a variety of
4. Why do children die if theyre not old? board games.
3. Have teen regularly attend special school
18. The parents of an 8-year-old child bring the
activities for own class.
child into the clinic for a school physical. The
nurse makes all of the following assessments. 4. Encourage siblings to spend time with teen
Which assessment finding is an area of concern watching television and movies.
and needs further investigation?
22. A 2-month-old infant is in the clinic for a well-
1. Complains of a stomach ache on test days at baby visit. Which of the following
school. immunizations can the nurse expect to
2. Has many evening rituals and resists going to administer?
bed at night. 1. TD, Varicella, IPV.

PEDIATRIC NURSING

5 439
53155_05_Ch05_p421-532.qxd 2/27/09 1:32 PM Page 440

2. DTaP, Varicella. the highest shelf is not sufficient. All medicines


3. DTaP, MMR, Menomune. should be put in a locked cabinet.
4. DTaP, Hib, IPV, HBV.
4. 1. DtaP is given intramuscularly; therefore,
23. An 18-month-old child with a history of falling administration is dependent on the presence of
out of his crib has been brought to the sufficient muscle mass, which may not be
emergency room by the parents. Examination of present in the infant who was born prematurely.
the child reveals a skull fracture and multiple
5. 1. Current recommendations call for measles,
bruises on the childs body. Which of the
mumps, and rubella combined vaccine (MMR)
following findings obtained by the nurse is most
to be given at 12-15 months.
suggestive of child abuse?
1. Poor personal hygiene of the child. 6. 2. Immunosuppressed clients may need
2. Parents want to comfort child. alteration in immunization protection as live
3. Conflicting explanations about the accident virus vaccines may overwhelm them.
from the parents.
7. 4. Injuries at various stages of healing are
4. Cuts and bruises on the childs lower legs in symptomatic of child abuse.
various stages of healing.
8. 1. Being tired and disheveled gives no
24. The nurse is discussing the risk of sudden infant information about the quality of the parent-child
death syndrome (SIDS) in infants with the interaction. It may be a normal state for a busy
parents whose first baby died of SIDS 6 months parent.
ago. The parents express fear that other children
will die from SIDS since they have already had 9. 2. Burns typical of child abuse have symmetrical
one baby die. Which of the following statements shapes and resemble the shape of the item used
made by the parents indicate their to burn the child.
understanding of the relationship of future
children and the risk of SIDS? 10. 1. The palmar grasp is present at birth. The
1. Any new baby will be on home monitoring palmar grasp lessens by age 3 months and is no
for one year to prevent SIDS. longer reflexive. The infant is able to close hand
voluntarily.
2. There is a 99% chance that we will not have
another baby die of SIDS. 11. 1. Infant cereal is generally introduced first
3. Genetic testing is available to determine the because of its high iron content. The infant is
likelihood of another baby dying from SIDS. able to accept spoon feeding at around 4 to
4. There is medicine that can be used to 5 months when the tongue thrust or extrusion
stimulate the heart rate while the baby is reflex fades.
sleeping.
12. 4. By 4 to 6 months, head control is well
established. There should be no head lag when
infant is pulled to a sitting position by the age
of 6 months.
Answers and Rationales
13. 1. The child has great curiosity and has the
mobility to explore. Toddlers need to be
1. 3. A 9-year-old should not be wetting the bed. supervised in play areas. Play areas with soft
This child may have physiologic or psychologic ground cover and safe equipment need to be
problems. selected.

2. 3. Preschool children do not understand the 14. 3. The best approach toward extinguishing
finality of death. They often view it as a long attention-seeking behavior is to ignore it as long
sleep. It is common for preschoolers to ask who as the behavior is not inflicting injury.
will take care of them when they die. Preschool
children may know the word dead but do not 15. 3. This behavior indicates a delay in language
really comprehend what it means. and speech development. The child may not be
able to hear. The child should have a vocabulary
3. 2. At 1 year of age babies are, or soon will be, of about 900 words and use complete sentences
climbing on everything. Putting medicines on of three to four words.

5 440 NCLEX-RN Review


53155_05_Ch05_p421-532.qxd 2/27/09 1:32 PM Page 441

16. 3. Imaginary friends are a normal part of and neatly organized in scrapbooks. This quiet
development for many preschool children. These activity supports the development of industry
imaginary friends often have many faults. The and concrete operational thought as well as the
child plays the role of the parent with the physical restrictions related to the rheumatic
imaginary friend. This becomes a way of assuming fever.
control and authority in a safe situation.
21. 3. Early adolescents have a strong need to fit in
17. 2. The greatest fear of preschool children is and be accepted by their peers. Attending school
being left alone and abandoned. Preschool activities helps the teen continue peer
children still think as though they are alive and relationships and develop a sense of belonging.
need to be taken care of.
22. 4. Healthy infants at 2 months of age receive
18. 3. The school-age years are very important in the diphtheria, tetanus, and pertussis (DTaP);
development of a healthy self-esteem. These hemophilus influenza (Hib); polio vaccine (IPV);
statements by the 8-year-old child indicate a risk and hepatitis B virus (HBV).
for development of a sense of inferiority and
need further assessment. 23. 3. Incompatibility between the history and the
injury is probably the most important criterion
19. 1. This ability illustrates the concept of on which to base the decision to report
conservation, which is one of the major suspected abuse.
cognitive tasks of school-age children.
24. 2. Whether subsequent siblings of the SIDS
20. 2. The middle childhood years are times for infant are at risk is unclear. Even if the increased
collections. The collections of middle to late risk is correct, families have a 99% chance that
school-age children become orderly, selective, their subsequent child will not die of SIDS.

Multisystem Stressors

GENETIC DISORDERS a. Mendels law: for each hereditary property


we receive 2 genes, 1 from each parent; 1 is
A. Genes are functional units of heredity, capable of dominant and expressed; 1 is recessive and
replication, mutation, and expression. not expressed.
B. Teratology is the branch of embryology that deals b. Homozygous: alleles for characteristic are
with the study of abnormal development and identical; both dominant (DD) or both
congenital malformations. recessive (dd).
1. Congenital disorders: present at birth, c. Heterozygous: alleles are different (Dd).
although may not be noticeable until later; D. Normal cell division
may be caused by genetic factors, nongenetic 1. Meiosis: cell division that produces gametes,
factors, or a combination. each with a haploid set of chromosomes (one-
2. Genetic disorders: caused by a single aberrant half the number of the parent cell); this is
gene or a deviation in chromosome structure reductional division, occurs in the ova and
or number. sperm.
C. In humans there are normally 46 chromosomes 2. Mitosis: cell division that produces two cells
(23 pairs) that contain the genes. (daughter cells), each with a full complement
1. Genotype: the gene constitution of an of chromosomes, identical to the composition
individual of the parent cell.
2. Phenotype: the outward visible physical
appearance/expression of a persons genes Principles of Inheritance
(color, size, allergies)
3. Karyotype: the number and pattern of Traits that are controlled by genes located on
chromosomes in a cell autosomes are inherited according to dominant or
4. Allele: one or two or more forms of a gene that recessive patterns. Most cases of autosomal inheritance

5
controls expression of specific characteristic in humans involve traits controlled by one gene.
(e.g., genes for eye color)

PEDIATRIC NURSING 441


53155_05_Ch05_p421-532.qxd 2/27/09 1:32 PM Page 442

Autosomal Dominant a. There is a 50% chance with each


pregnancy that her son will have the
A. General information disorder.
1. Allele responsible for the trait (or disease) is b. There is a 50% chance her daughter will
dominant. become a carrier.
2. Only one parent needs to pass on the gene 2. If a man has a sex-linked disorder, all his
(child may be heterozygous for trait). daughters will be carriers but none will
3. Examples of inherited diseases; Huntingtons manifest the disease.
chorea, myotonic muscular dystrophy, night 3. For sex-linked disorders, there are no carrier
blindness, osteogenesis imperfecta, states in males.
neurofibromatosis.
B. Genetic counseling: advise parents that if one of Chromosome Alterations
them has a disease inherited through autosomal
dominant pattern, there is a 50% chance with each A. General information: deviations from normal
pregnancy that the child will have the chromosome complement may be numeric or
disease/disorder. structural.
1. Mutation: spontaneous alteration in genetic
Autosomal Recessive material not present in previous generation
2. Nondisjunction: failure of a pair of
A. General information chromosomes to separate during meiosis;
1. Allele responsible for trait (or disease) will not results in numeric change called trisomy
result in expression if the other allele in the (47 chromosomes); can be passed on by
pair is dominant. either parent (one parent would pass
2. Both parents must pass on the gene(s) (child is 24 chromosomes)
homozygous for trait). 3. Translocation: the transfer of all or part of a
3. Examples of inherited diseases: cystic fibrosis, chromosome to another location on the same
PKU, sickle cell anemia, albinism, Tay-Sachs. chromosome or to a different chromosome
B. Genetic counseling: advise parents that if both are after chromosome breakage
heterozygous for the trait then: 4. Mosaicism: the presence in the same
1. There is a 25% chance with each pregnancy of individual of two or more genotypically
having a child with the disease/disorder. different cell lines
2. There is a 50% chance with each pregnancy B. Genetic counseling: varies depending on the origin
of having a child who is a carrier of the disease of the alteration
but who will not have the symptoms. 1. Random risk: chromosome alterations caused
3. There is a 25% chance with each pregnancy by environmental agents are not likely to occur
of having a child who will neither have the in subsequent pregnancies. Therefore, the risk
disease nor be a carrier. of the same defect recurring is no more than
for any person in the general population.
Sex-Linked (X-Linked) Inheritance 2. High risk: at least one parent carries a
chromosomal aberration or mutant gene and
A. General information passes it on to the offspring (e.g., if a parent is
1. Inheritance of characteristics located on X and a balanced translocation carrier, the risk of a
Y chromosomes child being affected is 1 in 4)
2. Only known genetic locus on the Y 3. Moderate risk: largest group; includes
chromosome is associated with determination multifactorial disorders. Risk recurrence in
of male sex. these disorders is empiric, based not on
3. X chromosome carries other traits in addition genetic theory but on prior experience and
to determination of female sex. observation.
4. Sex-linked inheritance in males: even a
recessive defective gene on X chromosome can
manifest itself in males because there is no Assessment
opposing normal gene on Y chromosome.
History
5. Sex-linked inheritance in females: recessive
defective gene can be masked by a normal A. Careful, detailed history is the basis of genetic
dominant gene. counseling; can help confirm diagnosis and
6. Examples of sex-linked inherited disorders: establish recurrence risk in multifactorial
color blindness, baldness, hemophilia A and disorders.
B, Duchennes muscular dystrophy. B. Family history (pattern of affected family
B. Genetic counseling members) is recorded in form of pedigree chart
1. If a woman is a carrier for a sex-linked disorder or family tree.

5
and her partner does not have the disorder:

442 NCLEX-RN Review


53155_05_Ch05_p421-532.qxd 2/27/09 1:32 PM Page 443

1. Information about affected child and immediate Evaluation


family: history of this pregnancy and all
previous pregnancies, including stillbirths and A. Optimum level of motor and cognitive
abortions; information about siblings development is attained.
2. Information about maternal relatives B. Self-care is performed at satisfactory level.
a. Mothers siblings C. Family is able to function appropriately.
b. Outcome of maternal grandmothers D. Parents demonstrate ability to meet childs
pregnancies physical and developmental needs.
c. Health status of maternal relatives E. Parents derive comfort/satisfaction from parenting
3. Information about paternal relatives: same as affected child.
maternal
Chromosome Disorder
Laboratory/Diagnostic Tests
Down Syndrome
A. Amniocentesis
1. Examination of amniotic fluid to screen for: A. General information
a. Some inborn errors of metabolism 1. One of the most common causes of mental
b. Chromosomal abnormalities retardation; incidence: about 1 in 600 live
c. Some CNS disorders (spina bifida) births
d. Sex of infant in sex-related disorders 2. Caused by an extra chromosome 21 (total 47)
2. Indications a. Most cases associated with nondisjunction;
a. One parent is chromosome mosaic or has incidence increased with maternal, and to
balanced translocation. some degree paternal, age; incidence in
b. Mother is age 35 or older. women over age 35 markedly increased
c. Both parents are heterozygous for an b. Also associated with translocation;
autosomal recessive disorder. hereditary type, incidence not increased
d. Mother is carrier of an X-linked disorder. with parental age
e. Couple already has an affected child. B. Assessment findings
B. Karyotyping (chromosomal analysis) 1. Head and face
1. Confirms or refutes probable diagnosis of a. Small head, flat facial profile, broad flat
chromosomal abnormality. nose
2. Identifies whether individual is a carrier of b. Small mouth, normal-size protruding
chromosomal abnormality. tongue
3. Determines infants sex if necessary. c. Upward slanting palpebral fissure
C. Determination of fetal status d. Low-set ears
2. Extremities
a. Short, thick fingers and hands
Analysis b. Simian creases (single crease across palms)
Nursing diagnoses for the family/individual with a c. Muscle weakness, lax joints
genetic disorder may include: 3. Associated anomalies and disorders
A. Disturbed thought processes a. Congenital heart defects (40% incidence)
B. Deficient knowledge (specify) b. GI structural defects
C. Risk for altered parenting c. Increased incidence of leukemia
D. Grief (anticipatory) d. Increased incidence of respiratory
infection
e. Visual defects: strabismus, myopia,
Planning and Implementation nystagmus, cataracts
Goals f. Obesity in older children
4. Retardation usually moderate, IQ 5070
A. Child will achieve maximum potential for C. Nursing interventions: see also Mental Retardation.
cognitive and motor development. 1. Provide parent education concerning
B. Family will develop effective coping strategies. a. Increased susceptibility to respiratory
infection
Interventions b. Nutritional needs, feeding techniques
c. Medication administration if necessary
A. Provide community health agency referral. d. Protection from injury due to hypotonia,
B. Support family in identification of appropriate atlanto-axial instability (weak neck)
stimulation programs for child. e. Needs of siblings
C. Communicate with and provide support for parents. 2. Promote developmental progress
D. Offer genetic counseling (see Principles of 3. Provide genetic counseling.

5
Inheritance).

PEDIATRIC NURSING 443


53155_05_Ch05_p421-532.qxd 2/27/09 1:32 PM Page 444

FLUID AND ELECTROLYTE, D. Head and face


1. Inspect fontanels, eye orbits.
ACID-BASE BALANCES 2. Ascertain presence of tears, saliva.
E. Abdomen/genitalia
General Principles and Variations 1. Auscultate for bowel sounds, peristaltic
waves.
from Adult 2. Inspect for diaper rash, urine stream.
A. Percent body water compared to total body weight
1. Premature infant: 90% water Laboratory/Diagnostic Tests
2. Infant: 7580% water
A. Blood studies
3. Child: 64% water
1. Hbg and HCT
B. Infant also has a higher percentage of water in
2. Sodium, potassium, chloride, calcium,
extracellular fluid compared to adults (therefore,
magnesium
infant has less fluid reserve).
3. pCO2 and CO2
C. Renal function
4. pH
1. Concentrating ability of kidney does not reach
B. Urine studies: specific gravity, glucose, ketones,
adult levels until approximately age 2; specific
osmolarity, pH
gravity of infants urine is 1.003.
C. Stool studies: culture, reducing substances, blood,
2. Glomerular filtration rate does not reach adult
sodium, potassium, pH
levels until approximately age 2.
3. Average urine output
a. Infant: 510 mL/hour Analysis
b. 110 years: 1025 mL/hour
Nursing diagnoses for children with fluid/electrolyte
c. 35 mL/hour thereafter
and acid-base imbalances may include:
D. Metabolic rate in children is 23 times that of
A. Diarrhea
adults; children therefore have an increased need
B. Pain
for nutrients and fluids, and an increased amount
C. Deficient fluid volume
of waste to excrete.
D. Imbalanced nutrition
E. Fluid is not conserved; there is less reserve, and
E. Impaired oral mucous membrane
children more prone to fluid volume deficit than
F. Impaired skin integrity
adults; 12 of infants extracellular fluid is
G. Ineffective tissue perfusion
exchanged each day compared with only 15 of
H. Impaired urinary elimination
adults.
F. Children have faster respiratory rate than adults,
causing more water loss through breathing. Planning and Implementation
G. Infants have a greater body surface area per kg
body weight than adults, therefore fluid loss
Goals
through skin (evaporation) is greater in children. A. Child will have normal hydration status.
B. Parents will demonstrate knowledge of childs
Assessment disorder, prescribed treatment, and prevention of
complications.
Health History
A. Ascertain age, recent weight, usual feeding Interventions
habits/patterns, amount and type of daily intake. A. Maintain strict I&O.
B. Determine usual voiding/stooling habits and 1. Weigh diapers.
volume of urine/stool output. 2. Monitor urine specific gravity.
C. Identify any recent illnesses or medications taken. 3. Hematest stools.
D. Ascertain usual activity level. B. Take daily weights.
C. Keep NPO for bowel rest.
Physical Examination D. Administer IV fluids.
1. Maintenance plus replacement
A. Measure present weight and vital signs.
2. Generally use hypotonic solutions (0.25 NSS,
B. Observe general appearance.
0.33 NSS, or 0.45 NSS)
1. Muscle tone
E. Provide pacifier for infant.
2. Reflex responses
F. Reintroduce oral fluids slowly.
3. Activity level
1. In children under 2 years use Pedialyte, a
C. Inspect skin and mucous membranes for turgor,
balanced electrolyte solution.
color, temperature.
2. In children over 2 use weak tea, flat soda.

5 444 NCLEX-RN Review


53155_05_Ch05_p421-532.qxd 2/27/09 1:32 PM Page 445

3. Do not use C. Nursing interventions


a. Broth (high sodium) 1. Teach caregivers to watch for signs/symptoms
b. Milk/formula (high solute) of dehydration:
c. Water, glucose water, Jell-O (no a. A decrease in wet diapers
electrolytes) b. Mouth dry inside
c. No tears present with crying if older than
24 months
Evaluation d. Irritable or lethargic
A. Child has adequate hydration status. e. Sunken fontanel
1. Adequate I&O f. Pale, gray, mottled skin color
2. Normal stooling pattern 2. Correct dehydration with oral solutions
3. Good skin turgor that should contain glucose, sodium,
4. Normal vital signs potassium, and bicarbonate (Pedialyte,
5. Normal serum laboratory values Lytren, Infalyte).
B. Parents participate in care, demonstrate 3. Administer IV fluid therapy as ordered.
understanding of signs and symptoms of disorder 4. Assess daily weight, vital signs, I&O.
and treatment. 5. Educate family on signs of dehydration and to
monitor childs urine output.
Disorders
Diarrhea
Dehydration
A. General information
A. The most common fluid and electrolyte 1. A change in consistency and frequency of
disturbance in infants and children stool
B. Osmotic factors, particularly sodium, control the 2. Very common in young children
movement of fluid between extracellular and 3. Caused by bacteria and viruses, parasites,
intracellular compartments and influence the poisons, inflammation, malabsorption,
types of dehydration allergies, abnormal bowel motility, and
1. Isotonic dehydration: most common anatomic alterations
a. Plasma sodium level is normal 4. Infants can easily lose 5% of their body weight
(130150 mEq/liter). in 1 day
b. Water and electrolyte lost in proportionate 5. Intestinal fluids are alkaline; large loss causes
amounts; net loss is isotonic. metabolic acidosis
c. Major loss is from extracellular fluid; loss 6. Also causes bicarbonate and potassium loss
of circulating blood volume. 7. Body may use its own fat for energy, leading to
d. Shock may develop if losses are severe. ketosis
2. Hypotonic dehydration B. Assessment findings
a. Plasma sodium level is less than 130 1. History of frequent stools; child may complain
mEq/liter. of or indicate abdominal cramping by
b. Electrolyte deficit exceeds water deficit. guarding, weight loss; child may be lethargic
c. May occur when fluid and electrolyte losses and irritable
are replaced with plain or glucose water. 2. Decreased urine output, decreased tears and
d. Fluid lost from extracellular compartment; saliva, dry mucous membranes, dry skin with
also moves from extracellular to poor tissue turgor
intracellular compartment. 3. In children less than 18 months, may find
e. Signs of decreased fluid appear sooner depressed anterior fontanel
with smaller fluid losses than in isotonic 4. Soft eyeballs with sunken appearance
dehydration. 5. Ashen skin color; cold extremities
f. Shock is a frequent finding. 6. High-pitched cry
3. Hypertonic dehydration 7. Increased pulse rate and decreased blood
a. Plasma sodium exceeds 150 mEq/liter. pressure
b. Water loss exceeds electrolyte loss; may 8. Diagnostic tests
occur when fluid and electrolyte losses are a. HCT elevated if dehydrated
replaced with large amount of solute b. Serum sodium and potassium decreased
(hypertonic formula). c. BUN elevated if renal circulation is
c. Fluid shifts from the intracellular to the decreased
extracellular compartments. d. CBC will show increased bands if caused
d. Physical signs of dehydration may be less by bacterial infection
apparent. e. Low pH and positive sugar with

5
e. Neurologic symptoms (e.g., seizures) may disaccharide intolerance
occur.

PEDIATRIC NURSING 445


53155_05_Ch05_p421-532.qxd 2/27/09 1:32 PM Page 446

f. Stool culture will identify specific D. Methods of prevention


microorganism 1. Childproofing the environment
g. Leukocytes in stool if caused by 2. Educating parents and child
enteroinvasive organisms 3. Legislation (e.g., seat belts, safe toys)
C. Nursing interventions 4. Anticipatory guidance
1. Keep NPO to rest bowel, if ordered. 5. Understanding and applying growth and
2. Administer IV fluid therapy as ordered. development principles
3. Resume oral feedings slowly; regular diet is a. Infant: totally dependent on adults for
recommended. maintenance of safe environment
4. Provide skin care to prevent/treat excoriation b. Toddler: more mobile, impatient; urge to
of diaper area. investigate and imitate; climbing, running,
5. Test all stool and chart results. jumping
6. Isolation may be ordered with infectious cause. c. Preschooler: very curious, exploring
neighborhood, running, climbing, riding
Vomiting bikes; can accept and respond to teaching
but still needs protection
A. General information d. School-age and adolescent: taking dares,
1. Common symptom during childhood; usually sports injuries, peer pressure, learning to
not a cause for concern drive
2. Differs from spitting up (dribbling of E. Precipitating factors
undigested formula, often with burping) 1. Arguments or tension in the home
3. If prolonged may result in metabolic alkalosis 2. Change in routines
or aspiration 3. Tired child/tired parents
B. Assessment findings: in addition to vomiting, child 4. Inadequate babysitting
may have fever, abdominal pain, and distension. 5. Hungry child
C. Nursing interventions 6. Illness in immediate family member
1. Assist with identification and treatment of F. Potential outcomes include temporary
underlying cause. incapacitation, permanent disfigurement, and death
a. Assess for accompanying diarrhea that may
indicate gastroenteritis.
b. Determine whether others in Specific Disorders
family/school/etc., are also sick; may Pediatric Poisonings
indicate food poisoning.
c. Assess for history of anxiety-producing life A. General information
events. 1. Toddlers and suicidal adolescents most often
d. Assess amount and force of vomitus; involved
forceful, projectile vomiting may indicate 2. Deaths have declined due to continued efforts
pyloric stenosis in prevention and the establishment of Poison
e. Determine frequency and character of Control Centers
vomitus (color, whether formula or food, 3. Modes of exposure: ocular, skin, ingestion
presence of bile or blood), relationship to (vast majority)
feeding (new foods, overeating). 4. Types of substances ingested: drugs,
2. Prevent complications: monitor household products (cleaning agents), garden
fluid/electrolyte and acid-base status supplies, plants and berries
3. Administer antiemetics if ordered; trimetho- 5. Most ingestions are acute in nature and
benzamide HCl (Tigan) and promethazine HCl accompanied by a history of invasion of the
(Phenergan) recommended for children. medicine chest or cabinet where household
cleaners are kept
6. Chronic ingestions result in accumulation of
ACCIDENTS, POISONINGS, toxic substance, such as lead
B. General assessment findings
AND INGESTION 1. Signs vary depending on substance ingested
2. May evidence bradycardia; tachycardia;
A. Accidents are the main cause of death in children tachypnea; slow depressed respiration;
over the age of 1 year. hypotension or hypertension; hypothermia or
B. 90% of accidents are preventable. hyperpyrexia
C. Interaction among host (the child), agent (the 3. Confusion, disorientation, coma, ataxia,
principal cause), and environment (23 of accidents seizures
occur in the home); safeguard the host while the 4. Miosis, mydriasis, nystagmus
agent and environment are made safer.

5
5. Jaundice, cyanosis

446 NCLEX-RN Review


53155_05_Ch05_p421-532.qxd 2/27/09 1:32 PM Page 447

6. Child may have a distinctive odor: Salicylate Poisoning


hydrocarbons, alcohol, garlic, sweat
C. General interventions A. General information
1. Resuscitate child and stabilize condition: 1. Toxicity begins at doses of 150200 mg/kg.
establish patent airway and provide measures 2. Products include not only aspirin but oil of
to restore circulation as indicated. wintergreen and analgesic cold medicines.
2. Prevent absorption. 3. Peak effect of aspirin is 24 hours and effects
a. Determine what, when, and how much was may last 8 hours.
ingested; will frequently not be able to 4. Ingestion may be accidental or due to
identify substance. therapeutic overdosing.
b. Gastric lavage; may be used to prevent 5. Salicylate ingestion causes
absorption a. Acid-base alterations
1) Use largest nasogastric (NG) or b. Respiratory alkalosis
orogastric tube possible. c. Metabolic acidosis
2) Recommended within 1 to 2 hours d. Impaired glucose metabolism
after ingestion. e. An inhibition of prothrombin formation
3) Indicated when ingested substance is B. Assessment findings
highly toxic or not absorbed by activated 1. Hyperventilation, confusion, loss of
charcoal (i.e., lithium, iron, lead). consciousness
4) Contraindicated in ingestion of 2. Hyperpnea, hyperpyrexia, dehydration
corrosives. 3. CNS depression, vomiting, lethargy
5) Complications include aspiration and 4. Coma, respiratory failure, circulatory collapse
perforation of esophagus. C. Nursing interventions
c. Activated charcoal: minimizes amount of 1. Assist with emergency management
toxic agent for absorption. 2. Administer fluid therapy
1) May be given alone or following 3. Monitor vital signs, BP, urine specific gravity, I&O
gastric lavage. 4. Monitor temperature, provide tepid sponging
2) Used if ingestion has occurred within or cooling mattress
2 hours prior to treatment 5. Provide emotional support to child and family
3) If liquid form, shake bottle, administer, 6. Administer vitamin K
then rinse bottle with small amount of
water, shake again and administer to Acetaminophen Ingestion
obtain full dose.
4) Complications include aspiration and A. General information
pneumothorax. 1. Has become a commonly used analgesic-
d. Cathartic: may be used after emesis or antipyretic
lavage to speed elimination of ingested 2. Little associated morbidity or mortality in
substance; recommended agents are accidental ingestion
sodium or magnesium sulfate. 3. Major risk is severe hepatic damage
3. Provide treatment and prevention information B. Assessment findings
to parents. 1. Vague and nonspecific initially; nausea,
a. Parents should always be instructed to vomiting, anorexia, sweating
save container, vomitus, spills on clothing 2. Jaundice, liver tenderness, increase in liver
for analysis. enzymes, abdominal pain
b. Teach parents about safety practices that 3. Progression to hepatic failure
will decrease chances of accidental C. Nursing interventions
poisonings; educate as to use of drugs, 1. Emesis or lavage; do not use activated charcoal
labeling, storage, and handling of (will bind antidote).
household products, importance of child- 2. Administer antidote (acetylcysteine
resistant safety packaging. [Mucomyst]): lessens hepatic damage if given
c. Advise parents to keep Poison Control within 16 hours of ingestion.
Center phone number readily available
(1-800-222-1222). Lead Poisoning
d. Incorporate anticipatory guidance related
A. General information
to developmental stage of child.
1. Increased blood lead levels resulting from
e. Discuss general first aid measures with
ingestion and absorption of lead-containing
parents.
substances
f. Syrup of ipecac is no longer recommended
2. Most common source: candy and spices from
or sold OTC and only used by healthcare
other countries not approved by the FDA;

5
personnel.

PEDIATRIC NURSING 447


53155_05_Ch05_p421-532.qxd 2/27/09 1:32 PM Page 448

found in lead-based paint (used in houses prior B. Medical management


to 1950); toys made in China recalled in 2007 1. Blood tests available, but lack sensitivity and
due to lead paint used specificity, therefore diagnosis is often based
3. Toddlers and preschoolers most often affected; many on clinical and epidemiologic data.
have pica (tendency to eat nonfood substances) 2. Antibiotic treatment: if early infection, given
4. Lead value of more than 10 g/dL is for 1021 days. If neurologic or arthritic
considered a health hazard symptoms occur, combined treatment for
5. Acute symptoms usually appear once level is longer duration may be necessary.
70 g/dL C. Assessment findings
6. Lead is absorbed through the GI tract and 1. Divided into stages on the basis of chronologic
pulmonary system; it is then deposited in relationship to the tick bite
bone, soft tissue, and blood; excretion occurs a. Stage I: Skin rash (erythema migrans) starting
via urine, feces, and sweat; toxic effects are 332 days past tick bite and lasting about
due to enzyme inhibition. 3 weeks. Most common on thighs, buttock,
7. Low dietary iron and calcium may enhance axilla. Systemic symptoms of malaise,
toxic effects. fatigue, headache, stiff neck, fever, arthralgia.
8. Widespread screening programs have b. Stage II: Symptoms of late disease may
diminished severe effects. occur months to years after the initial
B. Assessment findings disease. Includes neurologic symptoms
1. Abdominal complaints including colicky pain, such as facial palsies, sensory losses, focal
constipation, anorexia, vomiting, weight loss weaknesses, cardiac rhythm abnormalities,
2. Pallor, listlessness, fatigue and increased arthritis complaints
3. Clumsiness, irritability, loss of coordination, involving multiple joints.
ataxia, seizures D. Nursing interventions
4. Encephalopathy 1. Medication administration
5. Identification of lead in the blood 2. Prevention
6. Erythrocyte protoporphyrin (EP) levels increased a. Avoid high-risk areas such as woody,
C. Nursing interventions grassy areas
1. Prevention b. If walking in such areas, wear long pants,
a. Nutrition: adequate iron and calcium in diet long-sleeved shirt, high socks, and sneakers
b. Environment: damp mop and damp wipe c. Use insect repellents for skin and clothing
floors, windowsills, cover over flaking d. After every potential exposure, check
paint, handwashing before meals, baths carefully for ticks
2. Chelating agents e. Remove tick by pulling straight out with
a. Succimer (chemet)-oral tweezers
b. Dimercaprol (BAL)-IM
c. Disodium calcium edetate (calcium
EDTA)-IV or IM
d. IM agents-multiple injections-pain Sample Questions
1) Rotate sites
2) Warm soaks
3) Topical analgesia (EMLA cream) 25. A mother of an 8-year-old child shows the
e. All chelating agents physician a funny red circle on the childs leg
1) Maintain hydration (chelation is toxic and states the child went camping last weekend.
to kidneys). The physician draws blood to rule out Lyme
2) Measure I&O. disease and prescribes doxycycline for the child.
3) Monitor lead levels. The childs mother asks why an antibiotic is
3. Provide nutritional counseling. prescribed for a tick bite. The nurses response is
4. Aid in eliminating environmental conditions based on which of the following
that led to lead ingestion. understandings?
1. Lyme disease weakens the person so they are
Lyme Disease susceptible to infections.
A. General information 2. Lyme disease is caused by a spirochete that is
1. Caused by the spirochete Borrelia burgdorferi sensitive to doxycycline.
2. Transmitted by a deer tick, requires 24-hour 3. Doxycycline will kill the tick, which may
tick attachment still be in the child.
3. Most prevalent during summer and early fall
4. Antibiotics are given to cure the infection at
4. Symptoms usually involve the skin, nervous
the site of the tick bite.

5
system, and joints
5. Incubation period is 3 to 32 days

448 NCLEX-RN Review


53155_05_Ch05_p421-532.qxd 2/27/09 1:32 PM Page 449

26. The mother of a 3-year-old child calls her nurse 32. A 10-month-old weighs 10 kg and has voided
neighbor in a panic state, saying that the child 100 mL in the past 4 hours. The nurse is aware
swallowed most of a bottle of aspirin. The nurse that _________ is normal urine output.
determines that the child is still alert. What 1. 12 mL/kg/hour.
instruction should the nurse give to the mother? 2. 35 mL/kg/hour.
1. Induce vomiting in the child. 3. 79 mL/kg/hour.
2. Observe the child carefully until the 4. 10 mL/kg/hour.
ambulance arrives.
3. Call the Poison Control Center 33. A 3-month-old is NPO for surgery. What would
be an appropriate method for the nurse to
4. Give the child lots of milk to drink.
comfort him?
27. An 8-month-old infant was admitted to the 1. Administering acetaminophen.
hospital with severe diarrhea and dehydration. 2. Encouraging parents to leave so the child can
Fluid replacement therapy was initiated. Which rest.
observation the nurse makes indicates an 3. Offering pacifier.
improvement in the infants status? 4. Giving 10 cc Pedialyte.
1. Fontanels are depressed.
34. An 11-year-old is admitted for treatment of lead
2. Infant has gained 3 oz since yesterday.
poisoning. The nurse includes which of the
3. Skin remains pulled together after being following in the plan of care?
gently pinched and released.
1. Oxygen.
4. The infants hematocrit is greater today than
2. Strict I&O.
yesterday.
3. Heme-occult stool testing.
28. A 17-year-old has Down syndrome. He is 57 4. Calorie counts.
inches tall and weighs 155 pounds. In planning
35. A 2-month-old is admitted with diarrhea. What is
his care, what is the most important fact for the
the best room assignment for the nurse to make?
nurse to consider?
1. Semi-private room with no roommate.
1. His mental age.
2. Private room with no bathroom.
2. His chronologic age.
3. Semi-private room with 10-year-old who has
3. His bone age.
acute lymphocytic leukemia.
4. Growth chart percentiles.
4. Open ward.
29. What is the cause of Down syndrome? 36. The nurse is discussing safety measures to
1. An autosomal recessive defect. prevent poisoning with the mother of a 1-year-
2. An extra chromosome. old. Which statement by the mother demonstrates
understanding of safety precautions?
3. A sex-linked defect.
1. I have child protection locks on my cabinet
4. A dominant gene.
under the sink.
30. A 10-day-old baby is admitted with 5% 2. My child is not potty-trained, so the
dehydration. The nurse is most likely to note bathroom is safe.
which of the following signs? 3. I keep all poisons and cleaners above the
1. Tachycardia. fridge.
2. Bradycardia. 4. I dont think I have any poisons in my house.
3. Hypothermia. 37. The home health nurse observes a new mother
4. Hyperthermia. providing care for her 1 month old infant. What
information would the nurse give the mother to
31. The nurse is asked why infants are more prone help prevent hepatitis A to herself or the infant?
to fluid imbalances than adults. What would be
1. Avoid sexual relations for 3 months since
the nurses best response?
birthing occurred.
1. Adults have a greater body surface area.
2. Wear gloves when changing diapers with
2. Adults have a greater metabolic rate. loose stool.
3. Infants have functionally immature kidneys. 3. Clean contaminated household surfaces with
4. Infants ingest a lesser amount of fluid per a solution of 12 alcohol and 12 water.

5
kilogram. 4. Restrict visitors from holding the infant.

PEDIATRIC NURSING 449


53155_05_Ch05_p421-532.qxd 2/27/09 1:32 PM Page 450

38. A 16-year-old admits to her mother that she tried 29. 2. In Down syndrome there is an extra
to commit suicide by swallowing a bottle of chromosome on the 21st pair, which is why the
Tylenol (acetaminophen) 16 hours ago. Her disease is also called trisomy 21.
mother brings the girl to the ER. Which is the
treatment of choice for this occurrence? 30. 1. Tachycardia is associated with dehydration.
1. Ipecac syrup. 31. 3. Infant kidneys are unable to concentrate or
2. Activated charcoal. dilute urine, to conserve or secrete sodium, or to
3. Mucomyst. acidify urine.
4. Milk and observation.
32. 1. Normal urine output is 12 mL/kg/hour.
39. The nurse would include which of the following
nursing diagnoses for a 10-year-old client with 33. 3. Non-nutritive sucking will help console and
stage I Lyme disease? pacify him.
1. Decreased cardiac output. 34. 2. CaNaEDTA (treatment for lead poisoning)
2. Impaired mobility. is nephrotoxic and strict I&O records need
3. Altered cerebral tissue perfusion. to be kept.
4. Alteration in skin integrity.
35. 2. A bathroom is irrelevant with an infant in
diapers. A private room is necessary.

36. 3. All cleaners and poisons should be kept in


Answers and Rationales high locked cabinets.

37. 2. The mode of transmission for hepatitis A is


25. 2. Lyme disease is caused by a spirochete that is fecal-oral route or poor hygiene. Using gloves
sensitive to antibiotics. will prevent stool on the hands, as the mother
also should wash the hands afterwards. Sexual
26. 3. Because the child is still alert, the mother activity is related to hepatitis B, a proper
should call the Poison Control Center for cleaning solution would be with bleach and
guidance. Transport to the hospital will occur, water and visitors may hold the infant after
followed by gastric lavage and activated charcoal. washing hands to prevent infection to the infant.

27. 2. A weight gain would suggest greater 38. 3. Mucomyst is the treatment of choice to bind
circulating volume. Blood has weight. with acetaminophen and help reduce levels.

28. 1. Children with Down syndrome have some 39. 4. Stage I consists of tick bite followed by small
degree of mental retardation and care must be erythematous papules that may be described as
geared to their mental age. burning.

The Neurosensory System

VARIATIONS FROM THE ADULT B. Head growth results from development of nerve
tracts within the brain and an increase in nerve
fibers, not an increase in the number of neurons.
Brain and Spinal Cord C. Infants skull is not a rigid structure.
Size and Structure 1. Bones of skull are not fused until 18 months.
2. Head circumference will increase with
A. Rapid head growth in early childhood: brain is increase in intracranial volume in infants.
25% of adult weight at birth, 75% at 2 years, and 3. Sutures may separate if there is significant
90% at 6 years. gradual increase in intracranial volume up to
age 12.

5 450 NCLEX-RN Review


53155_05_Ch05_p421-532.qxd 2/27/09 1:32 PM Page 451

D. Fontanels (soft spots): areas of head not covered d. 3 years: 20/30


by skull e. 4 years: nearly 20/20
1. Anterior fontanel
a. Diamond-shaped opening at junction of
parietal and frontal bones
Ear and Hearing
b. Closes between 9 and 18 months A. Hearing is fully developed at birth.
2. Posterior fontanel B. Abnormal physical structure of ears may indicate
a. Triangular-shaped opening at junction of genetic problems (low-set ears often associated
occipital and parietal bones with renal problems or mental retardation).
b. Closes by 2 months
3. Should feel flat and firm
4. May be sunken with severe dehydration ASSESSMENT
5. Will bulge with increased intracranial
pressure (ICP)
History
Function A. Most important part of neurologic evaluation
B. Family history: seizure disorders, degenerative
A. Cortical functions (e.g., fine motor coordination) neurologic diseases, mental retardation, sensory
are incompletely developed at birth. defects
B. The autonomic nervous system (ANS) is intact but C. History of pregnancy: maternal illness, placental
immature. dysfunction, fetal movements, nuchal cord,
1. Infant has limited ability to control body intrapartal fetal distress, prematurity, meconium
temperature staining, Apgar scores
2. Infants heart rate very sensitive to D. Childs health history: delayed motor or speech
parasympathetic stimulation development, hypotonia, seizures, childhood
C. Infants behavior primarily reflexive illnesses
1. Neurologic exam consists of evaluating E. Parental concerns: development, vision, hearing
reflexes
2. Babinskis reflex normal in infant; disappears
after child begins to walk Physical Examination
D. Peripheral neurons not myelinated at birth A. Inspect size and shape of head: note fontanels in
1. Myelination occurs in later infancy infants, chart head circumference on growth chart.
2. Motor skill development depends on B. Observe posture and activity: note flexed posture
myelinization versus hypotonia or opisthotonos, symmetry of
E. Infant usually demonstrates a dominance of flexor movement of extremities, excessive tremors or
muscles; extremities will be flexed even when twitching, abnormal eye movements, ineffective
infant is sleeping suck or swallow, high-pitched cry.
F. Small tremors are normal findings during first C. Observe respiratory pattern: note apnea, ataxic
few months of life: not considered seizure activity breathing, asymmetric or paradoxic chest movement.
if occurring in response to environmental stimuli, D. Determine developmental level with Denver II.
if they are not accompanied by abnormal eye E. Vision tests
movements, and if movements cease with passive 1. Binocularity
flexion a. Corneal light reflex test: performed by
shining a light at the bridge of the nose as
Eye and Vision the child looks straight ahead; light reflex
should fall at the same point in both
A. Vision changes as the eye and eye muscles pupils; deviation indicates strabismus
undergo physiologic change. b. Cover/uncover test: Ask child to fix on an
B. Visual function becomes more organized. object. Cover one eye, assess uncovered
1. Binocular vision developed by 4 months eye movement. Uncover eye, assess that
2. Maturation of eye muscles by 1 year eye for movement. Repeat by covering the
a. Nystagmus common in infant other eye. Normal responseno movement
b. Strabismus (eyes out of alignment when of either eye in response to cover/uncover
fixating on an object): due to imbalance in maneuver.
extraocular muscles, common up to 2. Visual acuity
6 months, abnormal after 6 months a. Snellen E chart or Blackbird cards for
3. Visual acuity changes preschoolers
a. 16 weeks: 20/50 to 20/100 b. Snellen alphabet chart for older children
b. 1 year: 20/50+ 3. Peripheral vision

5
c. 2 years: 20/40 4. Color vision

PEDIATRIC NURSING 451


53155_05_Ch05_p421-532.qxd 2/27/09 1:32 PM Page 452

F. Auditory tests c. Increased cerebrospinal fluid (CSF) volume


1. Audiometry: perception of sound (hydrocephalus)
2. Tympanometry: conduction of sound in 2. Herniation of brain tissue: most serious
middle ear complication of increased ICP; may result in
3. Crib-o-gram: neonatal motor response to sound life-threatening deterioration of vital functions
4. Conduction tests B. Medical management
5. Newborn hearing screening: auditory evoked 1. Directed toward reducing intracranial volume
response and controlling underlying disorder
2. Drug therapy
a. Osmotic diuretics (mannitol, glycerol) to
Laboratory/Diagnostic Tests reduce acute brain edema; for short-term
A. Same neurologic tests that are used in adults are use only
used in children. b. Corticosteroids (dexamethasone) to reduce
B. Child should be carefully prepared and informed brain swelling
of what to expect during the test. 1) May be used for longer periods than
C. Sedation may be necessary for tests requiring child osmotic diuretics
to be immobile for extended period. 2) Antacids may be given concomitantly
D. Positioning and immobilization is crucial to the to prevent gastric irritation
success of lumbar puncture. 3. Fluid restriction, hyperventilation,
temperature regulation may all be used to
control increased ICP
ANALYSIS 4. Surgery: if increased ICP caused by
obstruction to CSF, shunt procedures may be
Nursing diagnoses for the child with a disorder of the performed
nervous system may include: C. Assessment findings
A. Impaired physical mobility 1. Infants
B. Disturbed thought process a. Lethargy, poor feeding, anorexia, vomiting,
C. Disturbed sensory perceptions or irritability
D. Deficient knowledge b. High-pitched cry
E. Pain c. Tense, bulging fontanel; increased head
F. Compromised or disabled family coping circumference; separation of cranial
G. Risk for injury sutures
H. Impaired verbal communication 2. Children
I. Risk for impaired skin integrity a. Anorexia, nausea, vomiting, irritability, or
lethargy
b. Headache, blurred vision, papilledema
c. Separation of cranial sutures
PLANNING AND IMPLEMENTATION 3. Late signs
a. Altered level of consciousness
Goals b. Pupil dilation and sluggish response to
light
A. Child will be protected from injury.
c. Tachycardia then bradycardia
B. Child will be free from signs and symptoms of
d. Altered respiratory rate then apnea
increased intracranial pressure.
e. Elevation in BP, increased pulse pressure
C. Normal respiratory function will be maintained.
f. Unstable temperature
D. Optimum developmental level will be achieved.
D. Nursing interventions
E. Family will be able to care for child at home.
1. Administer medications as ordered.
a. With osmotic diuretics, monitor fluid and
Interventions electrolyte balance carefully.
b. With corticosteroids, monitor for signs of
Care of the Child with Increased gastric bleeding.
Intracranial Pressure 2. Monitor hydration status carefully.
A. General information a. Administer IV fluids as ordered, assess
1. Intracranial volume and pressure can increase carefully for fluid overload.
as a result of: b. Assess for fluid and electrolyte imbalances.
a. Increased brain volume (cerebral edema, c. Monitor for hypovolemic shock if on strict
tumor) fluid restriction.
b. Increased cerebral blood volume 3. Assist with hyperventilation if ordered;
(hematoma or hemorrhage) monitor arterial blood gases (ABGs)

5 452 NCLEX-RN Review


53155_05_Ch05_p421-532.qxd 2/27/09 1:32 PM Page 453

4. Assist with reduction of body temperature as 2. Older children: changes in head size less
needed. common; signs of increased ICP (vomiting,
a. Administer antipyretics as ordered. ataxia, headache) common; alteration in
b. Use sponge baths, hypothermia pads as consciousness and papilledema late signs
necessary. 3. Diagnostic tests
5. Monitor LOC and behavioral/mental changes a. Serial transilluminations detect increases
carefully. in light areas
6. Elevate head of bed 3045 unless b. CT scan shows dilated ventricles as well as
contraindicated (e.g., possible spinal injury); presence of mass; with dye injection shows
keep neck in neutral alignment and avoid course of CSF flow
flexion. C. Nursing interventions: provide care for the child
7. Arrange nursing care activities to minimize with increased ICP and for the child undergoing
stimulation and keep environment as quiet as shunt procedures.
possible.
8. Prepare for shunt surgery if needed. Shunts
A. General information (See Figure 52)
1. Insertion of a flexible tube into the lateral
EVALUATION ventricle of the brain
A. Head growth progresses normally, fontanels are 2. Catheter is then threaded under the skin and
flat, and seizure activity is controlled. the distal end positioned in the peritoneum
B. Child maintains an appropriate activity level. (most common type) or the right atrium; a
C. Child is placed in an appropriate special program subcutaneous pump may be attached to ensure
or school, if needed. patency
D. Parents demonstrate ability to perform treatments 3. Shunt drains excess CSF from the lateral
and administer appropriate medications. ventricles of the brain in communicating or
noncommunicating hydrocephalus; fluid is

DISORDERS OF THE
NERVOUS SYSTEM
Disorders of the Brain and Spinal Cord

Hydrocephalus
A. General information
1. Increased amount of CSF within the ventricles
of the brain
2. May be caused by obstruction of CSF flow or
by overproduction or inadequate reabsorption
of CSF
3. May result from congenital malformation or be
secondary to injury, infection, or tumor
4. Classification
a. Noncommunicating: flow of CSF from
ventricles to subarachnoid space is
obstructed.
b. Communicating: flow is not obstructed, but
CSF is inadequately reabsorbed in
subarachnoid space, or excess CSF is
produced.
B. Assessment findings: depend on age at onset,
amount of CSF in brain
1. Infant to 2 years: enlarging head size; bulging,
nonpulsating fontanels; downward rotation of
eyes; separation of cranial sutures; poor
feeding, vomiting, lethargy, irritability; high- Figure 5-2 Ventriculoperitoneal shunt
pitched cry and abnormal muscle tone

PEDIATRIC NURSING

5 453
53155_05_Ch05_p421-532.qxd 2/27/09 1:32 PM Page 454

then absorbed by the peritoneum or enters the


general circulation via the right atrium
B. Nursing interventions
1. Provide routine pre-op care with special Meninges
attention to monitoring neurologic status.
2. Provide post-op care.
a. Maintain patency of the shunt.
1) Position child off the operative site.
2) Pump the shunt as ordered.
3) Observe for signs of infection of the
incision. Spinal nerves
4) Observe for signs of increased ICP.
5) Position the child with head slightly
elevated or as ordered.
3. Instruct parents regarding:
a. Wound care, positioning of infant, and
how to pump the shunt
b. Signs of infection
c. Signs of increased ICP
d. Need for repeated shunt revisions as child
grows or if shunt becomes blocked or
infected
e. Expected level of developmental Figure 5-3 Illustration of meningomyelocele
functioning
f. Availability of support groups and
community agencies 1) Sac (meninges) filled with spinal fluid
protrudes through opening in spinal
canal; sac is covered with thin skin
Spina Bifida (Myelodysplasia)
2) No nerves in sac
A. General information 3) No motor or sensory loss
1. Failure of posterior vertebral arches to fuse 4) Good prognosis after surgery
during embryologic development b. Myelomeningocele/meningomyelocele
2. Incidence: 2 in 1,000 infants in the United 1) Same as meningocele except there are
States spinal nerves in the sac (herniation of
3. Although actual cause is unknown, frequency dura and meninges).
of the defect is increased if a sibling has had a 2) Child will have sensory/motor deficit
neural tube defect; radiation, viral, and below site of the lesion.
environmental factors; and maternal folic acid 3) 80% of these children have multiple
deficiency have been suggested as causative. handicaps.
4. Site of the defect varies C. Medical management
a. Approximately 85% of the defects in the 1. Surgery
spine involve the lower thoracic lumbar or a. Closure of the sac within 48 hours of birth to
sacral area. prevent infection and preserve neural tissue
b. Defects in the upper thoracic and cervical b. Shunt procedure if accompanying
regions make up the remaining 15%. hydrocephalus
5. Folic acid supplementation can decrease risk. c. Orthopedic procedures to correct defects of
B. Types hips, knees, or feet
1. Spina bifida occulta 2. Drug therapy
a. Spinal cord and meninges remain in the a. Antibiotics for prevention of infections.
normal anatomic position. b. Anticholinergic drugs to increase bladder
b. Defect may not be visible, or may be capacity and lower intravesicular pressure.
identified by a dimple or a tuft of hair on 3. Immobilization (casts, braces, traction) for
the spine. defects of the hips, knees, or feet.
c. Child is asymptomatic or may have slight D. Assessment findings
neuromuscular deficit. 1. Examine the defect for size, level, tissue
d. No treatment needed if asymptomatic; covering, and CSF leakage.
otherwise treatment aimed at specific 2. Motor/sensory involvement may include:
symptoms. a. Voluntary movement of lower extremities
2. Spina bifida cystica b. Withdrawal of lower extremities or crying

5
a. Meningocele (see Figure 5-3) after pinprick

454 NCLEX-RN Review


53155_05_Ch05_p421-532.qxd 2/27/09 1:32 PM Page 455

c. Paralysis of lower extremities d. Medication regimen: schedule, dosage,


d. Joint deformities effects, and side effects
e. Hydrocephalus e. Feeding, diapering, positioning
f. Evaluate bowel and bladder function. f. Availability of appropriate support
Neurogenic bowel and bladder occur in up groups/community agencies/genetic
to 90% of the children. counseling
3. Diagnostic tests
a. Prenatal Meningitis
1) Ultrasound image of the pregnant
uterus shows fetal spinal defect and sac See Unit 4.
2) Amniocentesis: increased
alphafetoprotein (AFP) level prior to Encephalitis
18th week of gestation
b. Postbirth See Unit 4.
1) X-ray of spine shows vertebral defect;
CT scan of skull may show Reyes Syndrome
hydrocephalus
A. General information
2) Myelogram shows extent of neural
1. An acute encephalopathy with fatty
defect
degeneration of the liver
3) Encephalogram may show
2. Reyes syndrome is a true pediatric emergency:
hydrocephalus
cerebral complication may reach an
4) Urinalysis, culture and sensitivity (C&S)
irreversible state
may identify organism and indicate
3. Increased ICP secondary to cerebral edema is
appropriate antibacterial therapy
major factor contributing to morbidity and
5) BUN may be increased
mortality
6) Creatinine clearance rate may be
4. Early recognition and prompt management
decreased
reducing mortality
E. Nursing interventions
5. Etiology unknown; links with aspirin have
1. Prevent trauma to the sac.
been suspected but not proven
a. Cover with sterile dressing soaked with
B. Medical management
normal saline.
1. Proper initial staging essential.
b. Position infant prone or side-lying.
2. Treatment is supportive, based on stage of
c. Keep the area free from contamination by
coma and level of blood ammonia.
urine or feces. A protective barrier drape
3. Treatment should take place in a pediatric
may be necessary.
intensive care unit.
d. Inspect the sac for intactness or signs of
C. Assessment findings
infection.
1. Child appears to be recovering from a viral
e. Administer antibiotics as ordered.
illness, such as influenza or chickenpox,
2. Prevent complications.
during which salicylates have been
a. Observe for signs of hydrocephalus,
administered; symptoms then appear that
meningitis, joint deformities.
follow a definite pattern, which has led to
b. Clean intermittent urinary catheterization
clinical staging.
to manage neurogenic bladder.
a. Stage I: sudden onset of persistent
c. Administer medications to prevent urinary
vomiting, fatigue, listlessness
complications as ordered.
b. Stage II: personality and behavior changes,
d. Perform passive ROM exercises to lower
disorientation, confusion, hyperreflexia
extremities.
c. Stage III: coma, decorticate posturing
3. Provide adequate nutrition: adapt diet and
d. Stage IV: deeper coma, decerebrate rigidity
feeding techniques according to the childs
e. Stage V: seizures, absent deep tendon
position.
reflexes, respiratory reflexes, flaccid
4. Provide sensory stimulation.
paralysis
a. Adjust objects for visual stimulation
2. Pathophysiologic changes include
according to childs position.
a. Increased free fatty acid level
b. Provide stimulation for other senses.
b. Hyperammonemia due to reduction of
5. Provide emotional support to parents/family.
enzyme that converts ammonia to urea
6. Provide client teaching and discharge
c. Impaired liver function
planning to parents concerning
d. Structural changes of mitochondria in
a. Wound care
muscle and brain tissue
b. Physical therapy, range of motion exercises
e. Significant swelling of the brain

5
c. Signs of complications

PEDIATRIC NURSING 455


53155_05_Ch05_p421-532.qxd 2/27/09 1:32 PM Page 456

D. Nursing interventions (depend on stage) particular area of the brain; the energy
1. Stage I: assess hydration status: monitor skin generated overcomes the inhibitory
turgor, mucous membranes, I&O, urine specific feedback mechanism
gravity; maintain IV therapy. 7. Classification (Table 5-6)
2. Stages I-V: assess neurologic status: monitor a. Generalized: initial onset in both
LOC, pupils, motor coordination, extremity hemispheres, usually involves loss of
movement, orientation, posturing, seizure consciousness and bilateral motor
activity. activity
3. Stages II-V b. Partial: begins in focal area of brain and
a. Assess respiratory status: note changing symptoms are related to a dysfunction of
rate and pattern, presence of circumoral that area; may progress into a generalized
cyanosis, restlessness, agitation. seizure, further subdivided into simple
b. Assess circulatory status: frequent vital partial or complex partial
signs, note neck vein distension, skin color B. Medical management
and temperature, abnormal heart sounds. 1. Drug therapy (refer to Anticonvulsants)
c. Support child/family. a. Phenytoin (Dilantin)
1) Explain all treatments and procedures. 1) Often used with phenobarbital for its
2) Incorporate family members in potentiating effect
treatment as applicable. 2) Inhibits spread of electrical discharge
3) Organize regular family and client-care 3) Side effects include gum hyperplasia,
conferences. hirsutism, ataxia, gastric distress,
4) Use support services as needed. nystagmus, anemia, sedation
5) Educate family on over-the-counter b. Phenobarbital: elevates the seizure
medications containing aspirin (i.e., threshold and inhibits the spread of
Alka-Seltzer, Bufferin, Pepto-Bismol). electrical discharge
d. Provide additional parental and 2. Surgery: to remove the tumor, hematoma, or
community education to ensure early epileptic focus
recognition and treatment. C. Assessment findings
1. Clinical picture varies with type of seizure (see
Seizure Disorders Table 5-6)
2. Diagnostic tests
A. General information a. Blood studies to rule out lead poisoning,
1. Seizures: recurrent sudden changes in hypoglycemia, infection, or electrolyte
consciousness, behavior, sensations, and/or imbalances
muscular activities beyond voluntary control b. Lumbar puncture to rule out infection or
that are produced by excess neuronal discharge trauma
2. Epilepsy: chronic recurrent seizures c. Skull X-rays, CT scan, or ultrasound of the
3. Incidence higher in those with family history head, brain scan, arteriogram, or
of idiopathic seizures pneumoencephalogram to detect any
4. Cause unknown in 75% of epilepsy cases pathologic defects
5. Seizures may be symptomatic or acquired, d. EEG may detect abnormal wave patterns
caused by: characteristic of different types of seizures
a. Structural or space-occupying lesion 1) Child may be awake or asleep;
(tumors, subdural hematomas) sedation is ordered and child may be
b. Metabolic abnormalities (hypoglycemia, sleep deprived the night before the test
hypocalcemia, hyponatremia) 2) Evocative stimulation: flashing strobe
c. Infection (meningitis, encephalitis) light, clicking sounds,
d. Encephalopathy (lead poisoning, pertussis, hyperventilation
Reyes syndrome) D. Nursing interventions
e. Degenerative diseases (Tay-Sachs) 1. During seizure activity
f. Congenital CNS defects (hydrocephalus) a. Protect from injury.
g. Vascular problems (intracranial 1) Prevent falling, gently support head.
hemorrhage) 2) Decrease external stimuli; do not
6. Pathophysiology restrain.
a. Normally neurons send out messages in 3) Do not use tongue blades (they add
electrical impulses periodically, and the additional stimuli).
firing of individual neurons is regulated by 4) Loosen tight clothing.
an inhibitory feedback loop mechanism b. Keep airway open.
b. With seizures, many more neurons than 1) Place in side-lying position.

5
normal fire in a synchronous fashion in a 2) Suction excess mucus.

456 NCLEX-RN Review


53155_05_Ch05_p421-532.qxd 2/27/09 1:32 PM Page 457

Table 5-6 Types of Seizures

Type of Seizure Clinical Findings


Generalized seizures
Major motor seizure May be preceded by aura; tonic and clonic phases. Tonic phase: limbs contract or stiffen;
(grand mal) pupils dilate and eyes roll up and to one side; glottis closes, causing noise on exhalation; may
be incontinent; occurs at same time as loss of consciousness; lasts 2040 seconds. Clonic
phase: repetitive movements, increased mucus production; slowly tapers. Seizure ends with
postictal period of confusion, drowsiness.
Absence seizure (petit mal) Usually nonorganic brain damage (petit mal) present; must be differentiated from
daydreaming. Sudden onset, with twitching or rolling of eyes; lasts a few seconds.
Myoclonic seizure Associated with brain damage, may be precipitated by tactile or visual sensations.
May be generalized or local. Brief flexor muscle spasm; may have arm extension, trunk
flexion. Single group of muscles affected; involuntary muscle contractions; myoclonic jerks.
Akinetic seizure (tonic) Related to organic brain damage. Sudden brief loss of postural tone, and temporary loss
of consciousness.
Febrile seizure Common in 5% of population under 5, familial, nonprogressive; does not generally result
in brain damage.
Seizure occurs only when fever is rising. EEG is normal 2 weeks after seizure.
Partial seizure
Psychomotor seizure May follow trauma, hypoxia, drug use. Purposeful but inappropriate, repetitive motor acts.
Aura present; dreamlike state.
Simple partial seizure Seizure confined to one hemisphere of brain.
No loss of consciousness.
May be motor, sensory, or autonomic symptoms.
Complex partial seizure Begins in focal area but spreads to both hemispheres.
Impairs consciousness.
May be preceded by an aura.
Status epilepticus Usually refers to generalized grand mal seizures.
Seizure is prolonged (or there are repeated seizures without regaining consciousness)
and unresponsive to treatment.
Can result in decreased oxygen supply and possible cardiac arrest.

c. Observe and record seizure. f. Availability of support groups/community


1) Note any preictal aura. agencies
a) Affective signs: fear, anxiety g. How to assist the child in explaining
b) Psychosensory signs: hallucinations disorder to peers
c) Cognitive signs: dj-vu h. Inform parents that Ketogenic diet has had
symptoms success (diet limits intake of proteins and
2) Note nature of the ictal phase. carbohydrates)
a) Symmetry of movement
b) Response to stimuli; LOC Cerebral Palsy (CP)
c) Respiratory pattern
3) Note postictal response: amount of A. General information
time it takes child to orient to time and 1. Neuromuscular disorder resulting from
place; sleepiness. damage to or altered structure of the part of the
2. Provide client teaching and discharge brain responsible for controlling motor
planning concerning: function
a. Care during a seizure 2. Incidence: 1.55 in 1,000 live births
b. Need to continue drug therapy 3. May be caused by a variety of factors resulting
c. Safety precautions/activity limitations in damage to the CNS; possible causes include:
d. Need to wear Medic-Alert identification a. Prenatally: genetic, altered neurologic
bracelet or carry identification card development, or infection, trauma, or
e. Potential behavioral changes and school anoxia to mother (toxemia, rubella,
problems accidents, chorioamnionitis)

PEDIATRIC NURSING

5 457
53155_05_Ch05_p421-532.qxd 2/27/09 1:32 PM Page 458

b. Perinatally: during the birth process (drugs a. Occurs with extrapyramidal tract (basal
at delivery, precipitate delivery, fetal ganglia) lesion
distress, breech deliveries with delay) b. Found in 5% of all CP
c. Postnatally: kernicterus or head trauma c. Diminished or absent reflexes
(child falls out of crib or is hit by a car) d. Potential for severe contractures
B. Medical management 6. Associated problems
1. Drug therapy a. Mental retardation: the majority of CP
a. Antianxiety agents clients are of normal or higher than average
b. Skeletal muscle relaxants intelligence, but are unable to demonstrate
c. Local nerve blocks it on standardized tests; 1850% have
2. Physical/occupational therapy some form of mental retardation
3. Speech/audiology therapy b. Hearing loss in 13% of CP clients
4. Surgery: muscle- and tendon-releasing c. Defective speech in 75% of CP clients
procedures d. Dental anomalies (from muscle
C. Assessment findings: disease itself does not contractures)
progress once established; progressive e. Orthopedic problems from contractures or
complications, however, cause changes in signs inability to mobilize
and symptoms f. Visual disabilities in 28% due to poor
1. Spasticity: exaggerated hyperactive reflexes muscle control
(increased muscle tone, increase in stretch g. Seizures
reflex, scissoring of legs, poorly coordinated h. Disturbances of body image, touch,
body movements for voluntary activities) perception
a. Occurs with pyramidal tract lesion i. Feelings of worthlessness
b. Found in 40% of all CP D. Nursing interventions
c. Results in contractures 1. Obtain a careful pregnancy, birth, and
d. Also affects ability to speak: altered quality childhood history.
and articulation 2. Observe the childs behavior in various
e. Loud noise or sudden movement causes situations.
reaction with increased spasm 3. Assist with activities of daily living (ADL),
f. No parachute reflex to protect self when help child to learn as many self-care activities
falling as possible; CP clients cannot do any task
2. Athetosis: constant involuntary, purposeless, unless they are consciously aware of each step
slow, writhing motions in the task; careful teaching and demonstration
a. Occurs with extrapyramidal tract (basal is essential.
ganglia) lesion 4. Provide a safe environment (safety helmet,
b. Found in 40% of all CP padded crib).
c. Athetosis disappears during sleep; 5. Provide physical therapy to prevent
therefore, contractures do not develop contractures and assist in mobility (braces if
d. Movements increase with increase in necessary).
physical or emotional stress 6. Provide client teaching and discharge
e. Also affects facial muscles planning concerning:
3. Ataxia: disturbance in equilibrium; a. Nature of disease: CP is a nonfatal,
diminished righting reflex (lack of balance, noncurable disorder
poor coordination, dizziness, hypotonia) b. Need for continued physical, occupational,
a. Occurs with extrapyramidal tract and speech therapy
(cerebellar) lesion c. Care of orthopedic devices
b. Found in 10% of all CP d. Provision for childs enrollment or return
c. Muscles and reflexes are normal to school
4. Tremor: repetitive rhythmic involuntary e. Availability of support groups/community
contractions of flexor and extensor muscles agencies
a. Occurs with extrapyramidal tract (basal
ganglia) lesion Tay-Sachs Disease
b. Found in 5% of all CP
c. Interferes with performance of precise A. General information
movements 1. Degenerative brain disease, caused by absence
d. Often a mild disability of hexosaminidase A from all body tissues
5. Rigidity: resistance to flexion and extension 2. Autosomal recessive inheritance
resulting from simultaneous contraction of 3. Occurs predominantly in children of Eastern
both agonist and antagonist muscle groups European Jewish ancestry

5
4. A fatal disease; death usually occurs before age 4

458 NCLEX-RN Review


53155_05_Ch05_p421-532.qxd 2/27/09 1:32 PM Page 459

B. Assessment findings 5. Do not rearrange furniture without first telling


1. Progressive lethargy in a previously healthy child.
2- to 6-month-old infant 6. For a partially sighted child
2. Loss of developmental accomplishments a. Encourage child to sit in front of
3. Loss of visual acuity classroom.
4. Hyperreflexia, decerebrate posturing, b. Speak directly to childs face; do not look
dysphagia, malnutrition, seizures down or turn back.
5. Diagnosis confirmed by classic cherry-red spot c. Use large print and provide adequate
on the macula and by enzyme measurements nonglare lighting.
in serum, amniotic fluid, or white cells d. Use contrasting colors to help locate areas.
C. Nursing interventions 7. Provide client teaching and discharge
1. Support parents at time of diagnosis; help planning concerning:
them cope with feelings of anger and guilt. a. General child care, with adaptations for
2. Assist parents in planning long-term care for safety and developmental/functional level
the child. b. Availability of support groups/community
3. Provide genetic counseling and psychologic agencies
follow-up as needed. c. Special education programs
d. Interaction with peers: assist child as
necessary
Disorders of the Eye
Blindness Conjunctivitis
A. Causes A. General information: infection of membrane
1. Genetic disorders: Tay-Sachs disease, inborn covering anterior surface of eye globe and inner
errors of metabolism surface of eyelid due to multiple causes (bacterial,
2. Maternal infections during pregnancy: TORCH viral, allergic)
syndrome B. Medical management: ophthalmic antibiotics,
3. Perinatal: prematurity, retrolental fibroplasia steroids, anesthetics
4. Postnatal: trauma, childhood infections C. Assessment findings: weeping eye, reddened
B. Medical management: treatment of causative conjunctiva, sensitivity to light, eyelid stuck shut
disorders with exudate
C. Assessment findings D. Nursing interventions
1. Vacant stare; obvious failure to look at objects 1. Administer medications as ordered: apply
2. Rubbing eyes, tilting head, examining objects ophthalmic antibiotic ointments from inner to
very close to the eyes outer canthus (do not let container touch eye).
3. Does not reach for objects (over 4 months) 2. Provide client teaching and discharge
4. Does not smile when mother smiles (over planning concerning measures to prevent
3 months) but does smile in response to spread of infection
mothers voice a. Very contagious if bacterial or viral; no
5. Crawls or walks into furniture (over 12 months) school until antibiotics have been taken for
6. Does not respond to the motions of others 2448 hours
7. No concept of the look of an object, no concept b. Should not share pillows, tissues, toys
of color or reflection of self c. Good hand-washing technique
8. Other senses become more keenly developed d. Medication regimen: schedule, dosage,
to compensate desired and side effects
9. Unable to copy the actions of others; delayed
motor milestones in accomplishing tasks but
are not mentally handicapped
Disorders of the Ear
10. Various degrees (20/200 O.U. and worse) Deafness
D. Nursing interventions
1. For hospitalized child, find out parents usual A. Causes
method of care. 1. Conductive: interference in transmission from
2. Encourage infant to be active; use outer to middle ear from chronic otitis media,
multisensory stimulation (rocking, water play, foreign bodies
musical toys, touch). 2. Sensorineural: dysfunction of the inner ear;
3. From ages 25 arrange environment for damage to cranial nerve VIII (from rubella,
maximum autonomy and safety (e.g., avoid meningitis, drugs)
foods with seeds and bones). B. Medical management
4. Speak before you touch the child, announce 1. Treatment of causative disorders
2. Speech/auditory therapy

5
what you plan to do.

PEDIATRIC NURSING 459


53155_05_Ch05_p421-532.qxd 2/27/09 1:32 PM Page 460

3. Hearing aids Otitis Media


4. Surgery, depending on the cause
a. Cochlear implant for neural deafness A. General information
C. Assessment findings 1. Bacterial or viral infection of the middle ear
1. Infant 2. More common in infants and preschoolers as
a. Fails to react to loud noises (does have the ear canal is shorter and more horizontal
a Moro reflex, but not to noise) than in older children; also found in children
b. Makes no attempt to locate sound with cleft lip/palate
c. Remains in babbling stage or ceases to babble 3. Blockage of eustachian tube causes
d. Fails to develop speech lymphedema and accumulation of fluid in the
e. Startled by sudden appearances middle ear
2. All children B. Medical management
a. Respond only when speakers lips are visible 1. Drug therapy
b. Cannot concentrate for long on visual a. Systemic and otic antibiotics
images; constantly scan the surroundings b. Analgesics/antipyretics
for change 2. Surgery: myringotomy, with or without insertion
c. May have slow motor development of tubes (incision into the tympanic membrane
d. Appear puzzled or withdrawn, or strain to to relieve the pressure and drain the fluid)
hear C. Assessment findings
e. Use high volume on TV/radio 1. Dysfunction of eustachian tube
3. Audiologic testing 2. Ear infection usually related to respiratory
a. Slight hearing deficit: difficulty hearing infection
faint sounds, very little interference in 3. Increased middle ear pressure; bulging
school, no speech defect, benefits from tympanic membrane
favorable seating 4. Pain; infant pulls or touches ear frequently
b. Mild hearing deficit: can understand 5. Irritability; cough; nasal congestion
conversational speech at 35 feet when 6. Diagnostic tests: C&S of fluid reveals causative
facing the other person, decreased organism
vocabulary, may miss half of class D. Nursing interventions
discussions 1. Administer antibiotics as ordered, for a full
c. Marked hearing deficit: misses most of 10-day course. When administering ear drops
conversation, hears loud noises, needs pull earlobe up and back for children older
special education for language skills than 3 years and down and back if younger.
D. Nursing interventions 2. Administer acetaminophen for fever and
1. Speak slowly, not more loudly. discomfort.
2. Face child. 3. Administer decongestants to relieve
3. Get childs attention before talking; let child eustachian tube obstruction as ordered.
see you before performing any care. 4. Provide care for child with a myringotomy
4. Get feedback from child to make sure child has tube insertion (day surgery)
understood. a. Child should wear earplugs when
5. Decrease outside noises that could interfere with swimming, showering, or having hair
childs ability to discern what you are saying. washed; do not permit diving.
6. Be careful not to cover your mouth with hands. b. Be aware that tubes may fall out for no
7. Teach language through visual cues, touch, reason.
and kinesthetics. 5. Provide client teaching and discharge
8. Use body demonstrations or use doll play. planning concerning:
9. Provide appropriate stimulation (puppets and a. Medication administration
musical toys are inappropriate). b. Post-op care, depending on the type of
10. Provide client teaching and discharge surgery
planning concerning:
a. General child care, with adaptation for
safety and developmental/functional levels
b. Availability of support groups/community Sample Questions
agencies
c. Special education programs
d. Care and use of hearing aids, cochlear 40. An infant who was born with a
implant equipment myelomeningocele with accompanying
e. Interaction with peers: assist child as hydrocephalus has had a shunt procedure to
needed alleviate the hydrocephalus. The baby should be

5
placed in which of the following positions?

460 NCLEX-RN Review


53155_05_Ch05_p421-532.qxd 2/27/09 1:32 PM Page 461

1. Trendelenburg. 1. Engaging a tutor to assist with learning


2. Supine. problems.
3. Lithotomy. 2. Administering the prescribed antibiotic.
4. Prone. 3. Notifying the physician if the childs fever or
headache persists more than a few days after
41. The nurse is caring for an infant who is born discharge.
with hydrocephalus and has a shunt inserted. 4. Encouraging the child to resume normal
Which of the following signs indicates that the activities immediately.
shunt is functioning properly?
1. The sunset sign. 47. A 6-year-old is brought to the emergency
2. A bulging anterior fontanel. department unconscious after being hit by a car.
What information will be most helpful for the
3. Decreasing daily head circumference.
nurse performing the neurological examination
4. Widened suture lines. of the child?
42. A 13-year-old has been diagnosed as having 1. Normal growth and development.
epilepsy. What would be a positive sign that the 2. The childs usual behavior and status.
child is taking his Dilantin properly? 3. The childs past medical history.
1. Hair growth on his upper lip. 4. The childs growth and developmental
2. Absence of seizures. progress during infancy.
3. Lowered hemoglobin and hematocrit.
48. The nurse is assessing a child who has a head
4. Drowsiness. injury for the occulocephalic reflex (dolls eyes).
What will the nurse observe about the childs
43. A 3-year-old is admitted with a diagnosis of viral
eyes if this condition is present?
meningitis. During an initial assessment, what
would the nurse expect to find? 1. Move in the same direction in which his
head is turned.
1. Headache, fever, and petechiae.
2. Move in the direction opposite to which his
2. Seizures, lethargy, and hypothermia.
head is turned.
3. Pallor, anorexia, and bulging fontanels.
3. Remain midline when his head is turned.
4. Fever, irritability, and nuchal rigidity.
4. Move to the medial aspect of the orbit when
44. To meet the sensory need of a child with viral his head is turned.
meningitis, what should be included in the
49. The pupils of a child with a head injury are
nursing strategies?
dilated and react sluggishly. What occurrence is
1. Minimizing bright lights and noise. this indicative of?
2. Promoting active range of motion. 1. Barbiturate overdosage.
3. Increasing environmental stimuli. 2. Damage to the diencephalon.
4. Avoiding physical contact with family 3. Damage to the sympathetic system.
members.
4. Damage to the parasympathetic system.
45. When addressing the emotional needs of the
50. A 4-year-old female is admitted to the pediatric
parents of a young child with meningitis, what
intensive care unit (PICU) after suffering a severe
should be the primary focus?
closed head injury following a car accident. An
1. Assuming all responsibility for physical care intracranial pressure (ICP) monitor is in place
of the child. and reveals an ICP of 40 mm Hg. In an effort to
2. Providing reassurance that the symptoms will lower the ICP, what position will the nurse know
resolve within the week. is the best position for the client?
3. Reinforcing information about the childs 1. Supine with the head turned to the right.
condition and plan of treatment. 2. Supine with the head turned to the left.
4. Explaining the importance of an optimistic 3. Supine with the head midline.
outlook when interacting with their child.
4. Side-lying on the right with the head turned
46. A child who had meningitis is being discharged. to the left.
What should be included in the discharge
51. A 5-year-old female is admitted to the PICU after
teaching?

5
being hit by a car while riding her bike. She

PEDIATRIC NURSING 461


53155_05_Ch05_p421-532.qxd 2/27/09 1:32 PM Page 462

sustained a severe closed head injury and has an 1. Pepto-Bismol.


ICP monitor in place. Her ICP is 40 mm Hg and 2. Acetaminophen (Tylenol).
mannitol is ordered. What is the rationale for 3. Childrens aspirin.
administering mannitol for a child with an
4. Ritalin.
increased ICP?
1. It will produce a rise in the intravascular 55. The nurse is caring for a 5-year-old boy with a
osmolality, resulting in a shift of free water known seizure disorder. On entering his room,
from the interstitial and cellular spaces to the the nurse sees that he is experiencing a
intravascular space, thus decreasing the ICP. generalized, tonic-clonic seizure. What would be
2. It will produce a decrease in the intravascular the nurses first response?
osmolality, resulting in a shift of free water 1. Immediately leave the room to retrieve
from the interstitial and cellular spaces to the intravenous (IV) phenobarbital.
intravascular space, thus decreasing the ICP. 2. Place a metal spoon between his teeth to
3. It will produce a rise in the intravascular prevent him from biting his tongue.
osmolality, resulting in a shift of free water 3. Position him on his side to maintain a patent
from the intravascular space to the cellular airway.
space, thus decreasing the ICP.
4. Try to hold the client down.
4. It will produce a decrease in the intravascular
osmolality, resulting in a shift of free water 56. When assessing a client who is taking hydantoin
from the interstitial space to the cellular (Dilantin), the nurse would recognize which of
space, thus decreasing the ICP. the following findings as side effects?
1. Drowsiness and irritability.
52. An infant who has a ventriculoperitoneal (VP)
shunt in place for treatment of hydrocephalus is 2. Slurred speech and increased salivation.
hospitalized for potential shunt malfunction. 3. Hair loss and tremor.
When developing the plan of care, which of the 4. Gum hyperplasia and nystagmus.
following assessment findings would the nurse
list as a positive sign of shunt malfunction? 57. A 12-year-old girl with cerebral palsy has severe
1. Overriding sutures. language deficits and poor muscle coordination.
However, she can voluntarily turn her head from
2. Bulging, tense anterior fontanel.
side to side and her mother reports that she has
3. Flat, soft anterior fontanel. normal intelligence. The nurse is concerned
4. Consistent head circumference. with the childs ability to call when help is
needed. Considering the childs abilities, which
53. A male newborn was just admitted to the pediatric of the following would be the best way for her to
floor with a myelomeningocele. When developing call the nurse?
the preoperative plan of care, the nurse lists high
1. Kick the side rails of the bed.
risk for infection of and trauma to the
nonepithelialized lesion as the diagnosis of most 2. Scream loudly.
concern. What would be the most effective strategy 3. Press the call bell with her fingers.
to prevent infection and trauma to the lesion? 4. Press a large, padded call bell with her cheek.
1. Leave the lesion uncovered and open to the
air and place the baby supine. 58. The nurse is teaching a child how to prevent the
spread of conjunctivitis. Which statement by the
2. Cover the lesion with sterile, saline-soaked
child would indicate further instruction is
gauze and place the baby prone.
necessary?
3. Apply lotion to the lesion and place the baby
1. It is important that I wash my hands
on his side.
regularly.
4. Cover the lesion with dry sterile gauze and
2. I can use a tissue to clean my eyes, but must
place the baby supine.
throw it away immediately.
54. The nurse has been giving instructions to parents 3. I need to use my own washcloth and towel,
on measures to prevent Reyes syndrome. When not my sisters.
questioning the parents on a safe medication to 4. My Dad said he would carry a handkerchief
provide to their child during a viral illness, with him so I could wipe my eyes with it
which choice indicates that they understand during the day.
steps toward Reyes syndrome prevention?

5 462 NCLEX-RN Review


53155_05_Ch05_p421-532.qxd 2/27/09 1:32 PM Page 463

59. A woman brings her daughter to the pediatric information about his pretrauma neurological
clinic because she is concerned that the child condition.
has otitis media. On examination, the nurse
would recognize which of the following findings 48. 2. The occulocephalic reflex occurs if, when the
as the most common positive sign of otitis head of an unconscious child is turned rapidly
media? in one direction, the eyes move in the opposite
1. Temperature of 39C (102.2F) and loss of direction.
appetite. 49. 4. When dilated pupils react sluggishly to light
2. Pearly gray tympanic membrane and or are nonreactive, it is an indication that there
rhinorrhea. has been damage to the parasympathetic nervous
3. Pain on pressure on the tragus and edema system, which controls the pupillary
within the canal. constriction response.
4. Feeling of fullness in the ear and a popping
50. 3. The clients head must be kept in midline to
sensation during swallowing.
facilitate venous return. Clients with a severe
closed head injury have low intracranial
compliance and turning of the head may result
in an increase of ICP of 1015 mm Hg. The head
Answers and Rationales of bed (i.e., 30) should be determined
individually for each client based on the ICP and
cerebral perfusion pressure (CPP) as well as the
40. 4. Pressure must be kept off the spinal sac. clinical appearance.

41. 3. With improved draining of the CSF, the head 51. 1. A shift in fluid from the interstitial and
circumference should become smaller. cellular space to the intravascular space will
occur with a rise in intravascular osmolality, the
42. 2. Phenytoin (Dilantin) is an antiepileptic drug that fluid will then be diuresed resulting in a
controls seizures. Absence of seizures indicates the decreased ICP.
client is taking the medication properly.
52. 2. This is a common sign of shunt malfunction.
43. 4. The clinical symptoms of viral (aseptic) The best way to assess an infants fontanel is
meningitis include fever, irritability, and when the infant is upright and calm. In this
stiffness of the neck (nucchal rigidity). Other position, a fontanel that is bulging and firm to
symptoms include drowsiness, photophobia, light palpation is considered abnormal.
weakness, painful extremities, and sometimes
seizures. Aseptic meningitis usually resolves 53. 2. The lesion must be kept moist with sterile,
within 2 weeks. saline-soaked gauze. The prone position should
be maintained preoperatively to prevent tension
44. 1. Photophobia and hypersensitivity to on the lesion and minimize trauma.
environmental stimuli are common clinical
manifestations of meningeal irritation and 54. 2. Acetaminophen does not contain salicylates,
infection. Comfort measures include providing which have been suspected as an ingredient that
an environment that is quiet and has minimal can lead to Reyes syndrome.
stressful stimuli. 55. 3. The first priority is to maintain a patent
airway. The best position for the client during a
45. 3. Successful coping in times of anxiety and
seizure is on his side.
stress requires that the nurse be available to
provide information that validates parental right 56. 4. Hydantoin (Dilantin) may cause gum
to know and participation in their childs care. hyperplasia and nystagmus. Other side effects
include hirsutism, ataxia, diplopia, anorexia,
46. 3. Parents should be instructed to contact the
nausea, nervousness, and folate deficiency.
physician if the childs symptoms worsen or
persist. The child recovering from viral 57. 4. She is able to control her head movements
meningitis should show signs of feeling better a voluntarily. A large padded call bell could easily
week after discharge. be pressed when she turns her head to the side.

47. 2. The childs usual behavior and level of 58. 4. The eye should be wiped with disposable
development is what provides critical baseline tissues after a single use and no other

PEDIATRIC NURSING

5 463
53155_05_Ch05_p421-532.qxd 2/27/09 1:32 PM Page 464

individual should be exposed to items that come vomiting, diarrhea, loss of appetite, and red
in contact with the infected eye. tympanic membrane. Infants become irritable,
hold their ears, and roll their head from side to
59. 1. Common signs of otitis media include fever side. Young children verbally complain of pain.
(as high as 40C [104F]), postauricular and A concurrent respiratory or pharyngeal infection
cervical lymph gland enlargement, rhinorrhea, may also be present.

The Cardiovascular System

VARIATIONS FROM THE ADULT Normal Circulatory Changes at Birth


A. When the umbilical cord is clamped or severed,
Fetal Circulation the blood supply from the placenta is cut off, and
A. Fetal circulation differs from adult circulation in oxygenation must then take place in the newborns
several ways and is designed to ensure a high- lungs.
oxygen blood supply to the brain and B. As the lungs expand with air, the pulmonary artery
myocardium. pressure decreases and circulation to lungs
B. Characteristics increases.
1. Placenta is the source of oxygen for the fetus. C. Structural changes
2. Fetal lungs receive less than 10% of the blood 1. Ductus venosus: after the umbilical cord is
volume; lungs do not exchange gas. severed, flow through the ductus venosus
3. Right atrium of fetal heart is the chamber with decreases and eventually ceases; it constricts
the highest oxygen concentration. within 37 days after birth and eventually
C. Pattern of altered blood flow and facilitating becomes ligamentum venosum.
structures 2. Foramen ovale
1. Blood is carried from the placenta through the a. Functional closure of this valvelike
umbilical vein and enters the inferior vena opening occurs when pressure in the left
cava through the ductus venosus. atrium exceeds pressure in the right.
2. This permits most of the highly oxygenated b. Expansion of the pulmonary artery causes
blood to go directly to the right atrium, a drop in pulmonary artery pressure and in
bypassing the liver. right atrial and ventricular pressure.
3. This right atrial blood flows directly into the c. At the same time there is increased
left atrium through the foramen ovale, an pulmonary blood flow to the left atrium
opening between the right and left atria. and increased aortic pressure (from
4. From the left atrium, blood flows into the left clamping of the umbilical cord), which in
ventricle and aorta, through the subclavian turn raises left ventricular and left atrial
arteries, to the cerebral and coronary arteries, pressure.
resulting in the brain and heart receiving the d. Anatomic closure of the foramen ovale
most highly oxygenated blood. occurs within the first weeks after birth
5. Deoxygenated blood returns from the head with the deposit of fibrin.
and arms through the superior vena cava, 3. Ductus arteriosus
enters the right atrium, and passes into the a. Increase in aortic blood flow increases
right ventricle. aortic pressure and decreases right-to-left
6. Blood from the right ventricle flows into the shunt through the ductus arteriosus; shunt
pulmonary artery, but because fetal lungs are becomes bidirectional.
collapsed, the pressure in the pulmonary b. Increased pulmonary blood flow increases
artery is very high. arterial oxygen, causing vasoconstriction of
7. Because pulmonary resistance is high, most of ductus arteriosus within hours of birth.
the blood passes into the distal aorta through c. Functional closure occurs when this
the ductus arteriosus, which connects the constriction causes cessation of blood flow,
pulmonary artery and the aorta distal to the usually 24 hours after birth.
origin of the subclavian arteries. d. Anatomic closure occurs when there is
8. From the aorta, blood flows to the rest of the growth of fibrous tissue in the lumen of the

5
body. ductus arteriosus, by 13 weeks.

464 NCLEX-RN Review


53155_05_Ch05_p421-532.qxd 2/27/09 1:32 PM Page 465

Abnormal Circulatory Patterns after Birth B. Inspect for presence of cyanosis: lips, mucous
membranes, extremities.
A. Normal blood flow in the child may be disrupted C. Inspect for clubbing of fingers (thought to be
as a result of abnormal openings between the caused by increased capillary formation and soft
pulmonary and systemic circulations. tissue fibrosis).
B. Any time there is a defect connecting systemic and D. Observe for distended veins.
pulmonary circulation, blood will go from high to E. Palpate/percuss quality and symmetry of pulses,
low pressure (the path of least resistance). size of liver and spleen, presence of thrill over
1. Normally pressure is higher in the systemic heart during expiration.
circulation, so blood will be shunted from F. Auscultate for heart rate and rhythm.
systemic to pulmonary (left to right). G. Auscultate for abnormal heart sounds and
2. An obstruction to pulmonary blood flow, murmurs; murmurs are caused by abnormal flow of
however, may cause increased pressure blood between chambers or vessels; classified as:
proximal to the site of the obstruction. 1. Innocent: no anatomic or physiologic
3. With an obstruction to pulmonary blood flow, abnormality
as well as an opening between ventricles, the 2. Functional: no anatomic defect, but may be
blood flow may be right to left (if right-sided caused by a physiologic abnormality
pressure exceeds left-sided pressure). 3. Organic: caused by a structural abnormality
H. Measure blood pressure in both arm and thigh.
ASSESSMENT 1. In infants under 1 year, arm and thigh blood
pressure should be the same.
Overview 2. In children over 1 year, systolic pressure in leg
is usually higher by 1040 mm Hg.
A. Approximately 40,000 babies are born with congenital
3. A wide pulse pressure (greater than 50 mm Hg)
heart disease (CHD) in the United States yearly.
or a narrow pulse pressure (less than 10 mm Hg)
B. One third of these babies will be seriously ill at
may be associated with a heart defect.
birth, one third will have problems detected
I. Select proper blood pressure cuff size.
during childhood or later, and one third never
1. Too small a cuff can give a falsely elevated BP
have problems.
reading
C. Etiology is multifactional.
2. Bladder inside the cuff should be two thirds
the length of the upper arm
History
A. Family history: parental history of CHD, congenital
Laboratory/Diagnostic Tests
defects in siblings, history of genetic problems in A. Chest X-ray
family. B. Cardiac fluoroscopy
B. History of pregnancy: rubella, viral infections, C. Magnetic resonance imaging (MRI)
medications, X-ray exposure, alcohol ingestion, D. Electrocardiogram
cigarette smoking. E. Echocardiography
C. Childs health history F. Hematologic testing: polycythemia is often
1. Presenting problem: symptoms may include: associated with cyanotic heart defects
a. Feeding problems: fatigue, irritability, G. Cardiac catheterization
tachypnea, profuse sweating 1. Femoral vein often used for access
b. Failure to thrive 2. Catheter threaded into right side of the heart
c. Respiratory difficulties: tachypnea, difficulty since septal defects permit entry into the left
breathing, frequent respiratory infections side
d. Color changes: pallor, cyanosis (persistent 3. Nursing care: pretest
or intermittent) a. Childs preparation should be based on
e. Activity intolerance developmental level, level of
f. All presenting symptoms must be explored understanding, and past experience.
within a developmental framework b. Use doll play and pictures as appropriate.
2. Past medical history: rheumatic fever; c. Describe sensations child will feel in
associated chromosomal abnormalities (e.g., simple terms.
Down syndrome) d. Administer medications as ordered.
4. Nursing care: posttest
a. Check extremity distal to the
Physical Examination catheterization site for color, temperature,
A. Plot height and weight on growth chart; measure pulse, capillary refill.
respiratory rate and rhythm; inspect for chest b. Keep extremity distal to the catheterization
enlargement or asymmetry. site extended for 6 hours.

PEDIATRIC NURSING

5 465
53155_05_Ch05_p421-532.qxd 2/27/09 1:32 PM Page 466

c. Check pressure dressing over d. Potassium supplement if diuretic is


catheterization site for bleeding. potassium depleting
d. Monitor heart rate for signs of bradycardia, 3. High-caloric formula or nasogastric feedings
tachycardia, and dysrhythmia. may be required to meet nutritional needs
e. Monitor for transient temperature elevation C. Assessment findings
due to physiologic dehydration (NPO, 1. Tachycardia, gallop rhythm, cardiomegaly,
contrast media). decreased peripheral pulses, and mottling of
f. Monitor urine output and BP. the extremities
2. Tachypnea, retractions, grunting, nasal flaring,
cough, cyanosis, orthopnea
ANALYSIS 3. Hepatomegaly, edema, distended neck and
peripheral veins, decreased urine output
Nursing diagnoses for the child with a disorder of the 4. Failure to thrive, decreased exercise tolerance
cardiovascular system may include: D. Nursing interventions
A. Delayed growth and development 1. Decrease energy expenditure
B. Risk for injury: physiologic a. Frequent rest periods
C. Imbalanced nutrition: less than body requirements b. Small, frequent feedings
D. Fear/anxiety c. Minimize crying
E. Risk for infection d. Prevent cold stress
F. Deficient knowledge 2. Provide adequate nutrition
G. Decreased cardiac output a. Estimate daily caloric requirement
H. Excess fluid volume b. Use soft nipple
c. Consider gavage feeding if necessary
3. Monitor fluid status
PLANNING AND a. I&O, specific gravity
b. Daily weight
IMPLEMENTATION 4. Administer medications as ordered
a. Digoxin
Goals 1) Check dosage with another RN
2) Give 1 hour before feeding or 2 hours
A. Tissue will be adequately oxygenated. after feeding
B. Child will achieve normal growth and 3) Take apical pulse for 1 minute; if
development milestones. bradycardia is present, hold dose and
C. Child will be free from symptoms of complications contact physician
of heart disease. 4) Monitor serum potassium levels; if less
D. Parents will understand childs condition. than 3.5 mEq/liter, may be
E. Parents will be able to care for child at home. contraindicated
5) Monitor therapeutic effects;
Interventions therapeutic serum digoxin levels range
from 0.8 to 2.0 ng/mL.
Care of the Child with Heart Failure (HF) 6) Monitor for toxicity: nausea, anorexia,
A. General information vomiting, lethargy, bradycardia
1. Usually due to a surgically correctable 7) Parent/child teaching
structural abnormality of the heart that results b. ACE inhibitorsalso monitor BP
in increased blood volume and pressure or c. Diuretic (see Table 2-17)
increased pulmonary blood flow 1) I&O
2. A symptom complex reflecting the hearts 2) Daily weight
inability to meet the metabolic demands of the 3) Monitor side effects: dehydration,
body electrolyte imbalance especially
B. Medical management hypokalemia (potentiates digoxin and
1. Directed toward improvement of cardiac may lead to toxicity)
function and energy conservation 4) Parent/child teaching
2. Drug therapy 5. Provide adequate rest
a. Digitalis to improve myocardial 6. Prevent infections
contractility and slow the heart rate 7. Promote growth and development
b. ACE inhibitors to decrease cardiac 8. Reduce respiratory distress
afterload a. Position in semi- or high-Fowlers
c. Diuretics to decrease total body water and position
to increase urine output b. Knee-chest position for children with

5
tetralogy of Fallot

466 NCLEX-RN Review


53155_05_Ch05_p421-532.qxd 2/27/09 1:32 PM Page 467

EVALUATION DISORDERS OF THE


A. Child demonstrates optimal cardiac status. CARDIOVASCULAR SYSTEM
1. Normal color
2. No respiratory distress Congenital Heart Defects
3. Increased exercise tolerance
4. Satisfactory growth See Figure 5-4.
B. Child has no evidence of complications.
C. Parents demonstrate ability to care for child, Classification
perform necessary treatments, and administer
A. Defects associated with increased pulmonary
prescribed medications.
blood flow

Atrial Septal Defect Ventricular Septal Defect

Transposition of
Patent Ductus Arteriosus Coarctation of the Aorta Great Arteries

Truncus Arteriosus Tetralogy of Fallot

Figure 5-4 Congenital heart abnormalities

PEDIATRIC NURSING

5 467
53155_05_Ch05_p421-532.qxd 2/27/09 1:32 PM Page 468

1. Left-to-right shunting of blood across a septal d. Dyspnea, tachycardia, growth failure, HF


defect or blood vessel (higher left side heart may be present. Systolic pulmonary
pressure) ejection murmur present.
2. Pulmonary overcirculation and increased work e. Surgical repair includes patching of
of ventricles, possible right ventricular defectopen heart/cardiopulmonary
hypertrophy bypass procedure. Some defects plugged
3. Risk for heart failure during cardiac catheterization.
4. Usually acyanotic 2. Ventricular septal defect (VSD): opening in
5. Examples: atrial septal defect, ventricular ventricular septum with left-to-right shunting
septal defect, patent ductus arteriosus, of blood
atrioventricular canal (also called endocardial a. 25% of congenital heart defects.
cushion defect) b. Manifestations dependent on age, size of
B. Defects associated with decreased pulmonary defect, and degree of pulmonary vascular
blood flow resistance. Usually found in infancy.
1. Right-to-left shunting of blood due to presence c. Small lesions may be asymptomatic; may
of a defect and obstruction of pulmonary blood close spontaneously.
flow (obstructed pulmonary flow leads to d. With large lesions, higher pressure in
higher right side heart pressure) ventricles results in high degree of
2. Some or most blood does not enter the shunting.
pulmonary circulation and does not pick up e. Risk for right ventricular hypertrophy, HF,
oxygen in the lungs; instead, blood is shunted bacterial endocarditis, pulmonary
to the left side of the heart problems.
3. Deoxygenated as well as oxygenated blood f. Dyspnea, tachycardia, growth failure, HF,
circulated to the body frequent respiratory infections common.
4. Cyanosis and hypoxemia present Harsh systolic murmur at lower left sternal
5. Example: tetralogy of Fallot border present.
C. Defects causing obstruction to cardiac chamber g. Surgical repair includes suturing or
outflow patching of defect using open
1. Narrowing of outflow tract from heart to blood heart/cardiopulmonary bypass procedure,
vessels usually done in infancy. Primary
2. Increased work of heart as it strains to push (complete) repair is preferred.
blood out h. Occasionally, surgical palliation with
3. Risk for heart failure and poor cardiac output pulmonary artery banding for severely ill
4. Examples: coarctation of the aorta, pulmonic infants, with complete repair when infant
stenosis, aortic stenosis is more stable. Banding decreases blood
D. Defects associated with mixing of saturated and flow through pulmonary artery: decreases
desaturated blood pressure difference between right and left
1. Oxygenated and deoxygenated blood mixes in ventricles to decrease left-to-right shunting
heart chambers of blood across defect.
2. Increased pulmonary blood flow due to defect 3. Patent ductus arteriosus (PDA): failure of fetal
3. Hypoxemia and cyanosis present, often severe ductus arteriosus to close after birth
4. Risk for poor cardiac output and risk for heart a. 10% of congenital heart defects in term
failure (HF). infants, more common in preterm infants.
5. Examples: transposition of the great vessels b. Blood vessel connecting pulmonary artery
(also called transposition of the great arteries), and aorta.
truncus arteriosus, hypoplastic left heart c. Higher pressure in aorta results in left-to-
syndrome right shunting of blood from aorta to
pulmonary circulation.
Increased Pulmonary Blood Flow d. Manifestations depend on size of defect.
Small lesions may be asymptomatic.
A. General information and medical management e. Risk for HF.
1. Atrial septal defect (ASD): opening between f. May have bounding pulses and visible
right and left atria with left-to-right shunting precordial pulsations (especially preterm
of blood infants). Continuous machine-like murmur
a. 15% of congenital heart defects at upper left sternal border.
b. Manifestations dependent on age and size g. Administration of indomethacin may close
and location of defect. defect in preterm infants.
c. Small lesions may be asymptomatic until h. If indomethacin fails, or if not a preterm
childhood. infant, surgical ligation of vessel (closed

5 468 NCLEX-RN Review


53155_05_Ch05_p421-532.qxd 2/27/09 1:32 PM Page 469

heart procedure) or mechanical occlusion dental, genitourinary tract, and surgical


of vessel. procedures. Will be required throughout life.
4. Atrioventricular canal (AV canal): combination 10. Teach care to family.
septal defect resulting in large opening
between right and left atria and ventricles and Decreased Pulmonary Blood Flow
defects of valves.
a. Most common cardiac defect in children A. General information and medical management
with Down syndrome. 1. Tetralogy of Fallot (TOF)
b. High degree of left-to-right shunting of blood. a. Most common congenital heart defect
c. HF commonly develops. Pulmonary flow causing cyanosis and hypoxemia; 10% of
murmurs and valvular murmurs present. all congenital heart defect.
d. Open heart/cardiopulmonary bypass b. Four associated defects: pulmonary
surgical repair. stenosis, VSD, overriding aorta (also called
B. Assessment findings in conditions with increased dextropositioned aorta), right ventricular
pulmonary blood flow hypertrophy.
1. Poor feeding, anorexia c. Pulmonary stenosis creates obstruction to
2. Growth failure, poor weight gain outflow of blood from right ventricle to
3. Respiratory difficulties: tachypnea, dyspnea, pulmonary artery, causing decreased
orthopnea, coughing, wheezing, hoarseness, pulmonary blood flow. Increased right
grunting, nasal flaring, retractions, frequent ventricle pressure creates right-to-left
respiratory infections shunt. Right-shifted aorta sits over VSD so
4. Exercise intolerance, fatigue, lethargy, blood from both right and left ventricles
excessive sweating with feeding or activity flows into aorta.
5. Signs/symptoms of heart failure d. Aorta carries mixed oxygenated and
6. Cardiac murmur deoxygenated blood to body.
C. Nursing interventions e. Manifestations include low oxygen
1. Prepare child/family for diagnostic studies, saturation, cyanosis, polycythemia, activity
surgery. intolerance, fatigue, poor feeding, poor
2. Administer medications as ordered. See HF. growth, harsh systolic murmur along left
3. Ensure adequate nutrition. sternal border, hypercyanotic spells (also
a. Anticipate infant hunger to prevent crying called TET spells), also signs of chronic
and increased oxygen demands. hypoxia.
b. Small, frequent feedings/small frequent, f. Hypercyanotic spells: occur when oxygen
nutritious meals if child. demand exceeds supply.
c. Feed infants in semi-upright position. 1) Transient obstruction of pulmonary
d. Soft nipple to decrease fatigue during blood flow.
infant feedings. Gavage feedings may be 2) Increasing cyanosis, tachypnea, poor
necessary. muscle tone, loss of consciousness.
e. Burp frequently during bottle and breast May progress to seizures, CVA, death.
feedings. 3) Often precipitated by crying, feeding,
f. Observe for tolerance of feedings if high- defecation.
calorie formula or breast milk fortifier 4) Treat by placing in knee-chest
used: vomiting, diarrhea. position, give oxygen, morphine,
g. Assist breastfeeding mothers. occasionally propranolol.
h. Monitor growth. g. Surgical repair includes open heart/
4. Monitor vital signs. cardiopulmonary bypass procedure to
5. Provide rest. patch VSD and relieve stenosis. Palliative
a. Quiet age-appropriate play if HF present. surgery to increase pulmonary blood flow
b. Cluster care to provide periods of includes anastomosis of right or left
undisturbed rest. subclavian artery to pulmonary artery
c. Anticipate needs; prevent crying. (Blalock-Taussig shunt).
6. Position semi-upright if HF or respiratory B. Assessment findings in conditions with decreased
difficulty present. pulmonary blood flow
7. Prevent infections. 1. Low oxygen saturation
8. Meet age-appropriate developmental needs. 2. Cyanosis
9. Bacterial endocarditis antibiotic prophylaxis 3. Polycythemia (chronic hypoxemia results in
for unrepaired ASD, VSD, PDA, and all other increased production of RBCs)
congenital heart defect before or after repair. 4. Clubbing of digits (chronic hypoxia)
Give prescribed antibiotic 1 hour before 5. Poor feeding, anorexia, fatigue, activity

5
intolerance, growth failure, weak cry

PEDIATRIC NURSING 469


53155_05_Ch05_p421-532.qxd 2/27/09 1:32 PM Page 470

6. Squatting: increases systemic vascular 3. Aortic stenosis: narrowed aortic valve


resistance and improves pulmonary blood a. Increased resistance to left ventricular
flow (not seen frequently due to early repair of blood outflow into aorta.
cardiac defects) b. Leads to left ventricular hypertrophy, left-
7. Hypercyanotic (TET) spells sided heart failure.
8. Tachycardia, tachypnea, dyspnea c. Manifestations include faint pulses,
9. Cardiac murmur tachycardia, hypotension, poor feeding,
10. Risk for emboli, bacterial endocarditis exercise intolerance, aortic murmur.
C. Nursing interventions d. Repair: balloon angioplasty or valvotomy.
1. Squatting: observe only. No other intervention B. Assessment findings: HF, cardiac murmur; also see
needed unless distress develops. individual defects
2. Cluster care. Provide age-appropriate quiet C. Nursing interventions
activities. Promote uninterrupted rest. 1. Monitor for hypotension.
3. Provide oxygen as needed. 2. Monitor for HF.
4. Prevent crying; anticipate needs. 3. Monitor for cyanosis and hypoxemia in
5. Monitor vital signs. children with pulmonic stenosis.
6. Support nutrition (see interventions for 4. Prepare child and family for diagnostic/
increased pulmonary blood flow). therapeutic procedures and surgery.
7. Prepare child/family for diagnostic tests and 5. Support nutrition as noted earlier.
surgery. 6. Promote rest as noted earlier.
8. Administer medications as ordered. 7. Administer medications as ordered.
9. Bacterial endocarditis prophylaxis as noted 8. Bacterial endocarditis prophylaxis as noted
earlier. earlier.
10. For hypercyanotic spells, place in knee-chest 9. Meet age-appropriate developmental needs.
position, administer oxygen, prepare to 10. Teach care to parents.
administer morphine.
11. Meet age-appropriate developmental needs. Lesions with Mixing of Saturated
12. Teach care to parents. and Desaturated Blood
Obstruction to Outflow A. General information and medical management
1. Transposition of the great vessels
A. General information and medical management (transposition of the great arteries): aorta
1. Coarctation of the aorta: narrowing of a portion emerges from right ventricle and pulmonary
of aorta, usually near aortic arch beyond left artery emerges from left ventricle.
subclavian artery a. Essentially two independent circulations:
a. Decreased blood flow to lower part of body, 1) Unoxygenated blood from right
more blood shunted to arms and head. ventricle exits to aorta, goes to body
b. Manifestations dependent on degree of and returns to right atrium without
narrowing, include arm blood and pulse flowing to lungs.
pressures greater than in legs, strong 2) Oxygenated blood exits left ventricle
brachial and diminished femoral pulses, to pulmonary arteries, goes to lungs,
lower body cooler than upper. In older and returns to left atrium.
children, dizziness, headaches, fainting, b. Incompatible with life unless there is
epistaxis occur. communication between left and right
c. Risk for heart failure, hypertension, sides of heart (usually through foramen
rupture of aorta, CVA. ovale or PDA).
d. Vascular surgery to remove narrowed c. Manifestations include neonatal cyanosis,
portion or repair with graft. hypoxemia, systolic murmur.
2. Pulmonic stenosis: narrowed pulmonic valve d. Treatment includes administration of
opening Prostaglandin E to maintain patency of
a. Minor to moderate narrowing may be ductus arteriosus, balloon atrial
asymptomatic. septostomy (also called Rashkind
b. Severe narrowing causes increased work of procedure) during cardiac catheterization
right ventricle and ventricular to improve mixing of blood in atria.
hypertrophy. e. Surgical repair: arterial switching
c. Manifestations: cyanosis, systolic thrill, procedure in newborn period or
systolic ejection murmur at upper left installation of atrial baffle to direct venous
sternal border. blood to left ventricle and oxygenated
d. Repair includes balloon angioplasty to blood to right ventricle (Senning and

5
dilate stenosed area or surgical valvotomy. Mustard procedures) in older children
(rarely used).

470 NCLEX-RN Review


53155_05_Ch05_p421-532.qxd 2/27/09 1:32 PM Page 471

2. Truncus arteriosus: failure of embryonic blood 3. Monitor infants receiving Prostaglandin E for
vessel to divide into aorta and pulmonary apnea, hypotension, hypothermia.
artery results in one large vessel positioned 4. Cluster care to provide periods of
over both ventricles. uninterrupted rest.
a. Has associated large VSD. 5. Provide oxygen as ordered.
b. Both oxygenated and deoxygenated blood 6. Prevent crying; anticipate needs.
flow to systemic circulation; blood flow 7. Support nutrition (see interventions for
and pressure in pulmonary circulation are increased pulmonary blood flow).
increased. 8. Bacterial endocarditis prophylaxis as noted
c. Manifestations include cyanosis, growth earlier.
failure, activity intolerance, HF. 9. Meet age-appropriate developmental needs.
d. Treatment includes digoxin and diuretics 10. Teach care to parents.
for HF.
e. Surgical repair includes open heart/ Cardiac Surgery
cardiopulmonary bypass procedure to
close VSD, incorporate trunk into left A. General information
ventricle, grafting of right and left 1. Surgical correction of congenital defects
pulmonary arteries to right ventricle. within the heart, or surgery of the great vessels
3. Hypoplastic left heart syndrome: poorly in the immediate area surrounding the heart
developed left side of heart, including 2. Open-heart surgery (uses cardiopulmonary
hypoplastic left ventricle, aortic valve atresia bypass): provides a relatively blood-free
or mitral valve atresia, narrowed ascending operative site; heart-lung machine maintains
aorta and aortic arch. gas exchange during surgery
a. Some oxygenated blood flows from left 3. Closed-heart surgery does not use
atrium across foramen ovale to right cardiopulmonary bypass machine; indicated
atrium, enters pulmonary circulation, and for ligation of a patent ductus arteriosus or
flows across PDA into aorta. coarctation of the aorta
b. Clinical manifestations include progressive B. Nursing interventions: preoperative
cyanosis, pallor, weak or absent pulses, HF, 1. Determine the childs level of understanding;
shock. have child draw a picture, tell you a story, or
c. Treatment includes administration of use doll play.
Prostaglandin E to maintain PDA, 2. Correct misunderstandings/teach the child
administration of medications to support about the surgery using diagrams and play
blood pressure and cardiac function. therapy; use terms appropriate to
d. Usually fatal without surgery or heart developmental level.
transplantation. 3. Accompany the child to the operating and
1) Norwood procedure (palliative): connect recovery rooms and the intensive care unit,
pulmonary artery and aorta, create ASD, explaining the various equipment; allow child
allows mixed blood to get to tissues. to handle/experience it, if possible, and
2) Repair includes intracardiac redirection introduce staff and clients, depending on
of blood flow (Fontan procedure) childs developmental/emotional levels.
involving open heart/cardiopulmonary 4. Have child practice post-op procedures
bypass technique. (turning, coughing, deep breathing, etc.).
3) Heart transplant may be performed. 5. Include parents in teaching sessions, but have
B. Assessment findings in conditions with mixing of separate sessions for the parents only.
oxygenated and deoxygenated blood 6. Establish pre-op baseline data for vital signs,
1. Cyanosis and hypoxemia activity/sleep patterns, I&O.
2. Tachycardia, dyspnea, tachypnea C. Nursing interventions: postoperative
3. Cardiac murmur 1. Prevent injury/complications.
4. Poor feeding, growth failure, activity a. Monitor vital signs and circulatory
intolerance, weak cry, lethargy pressure readings frequently until stable.
5. Varying degrees of HF Monitor ECG.
6. Polycythemia b. Assess neurologic status frequently.
7. Clubbing of digits c. Observe surgical site for intactness/
8. Risk for bacterial endocarditis, emboli, stroke drainage.
C. Nursing interventions 2. Promote gas exchange (client may be on
1. Prepare child/family for diagnostic procedures mechanical ventilation).
and surgery. a. Position as ordered.
2. Assess vital signs and assess for poor cardiac b. Administer oxygen at prescribed rate.

5
output. c. Provide humidification.

PEDIATRIC NURSING 471


53155_05_Ch05_p421-532.qxd 2/27/09 1:32 PM Page 472

d. Suction as necessary. C. Assessment findings


e. Perform postural drainage and chest 1. Major symptoms (Jones criteria)
percussion as ordered. a. Carditis
f. Turn, cough, and deep breathe hourly. 1) Seen in 50% of clients
g. Perform routine care of chest tubes and 2) Aschoff nodules (areas of
drainage system, depending on the type of inflammation and degeneration around
surgery. heart valves, pericardium, and
3. Monitor I&O. myocardium)
4. Provide nutrition as ordered. 3) Valvular insufficiency of mitral and
5. Provide alternative means of communication if aortic valves possible
mechanical ventilation is used, e.g., picture 4) Cardiomegaly
cards. 5) Shortness of breath, hepatomegaly,
6. Provide psychologic support of the edema
child/family. b. Polyarthritis
7. Allow activity as tolerated. 1) Migratory, therefore no contractures
8. Provide client teaching and discharge develop
planning concerning: 2) Most common in large joints, which
a. Need for child/family to express become red, swollen, painful
feelings/fears 3) Synovial fluid is sterile
b. Resumption of ADL 4) No arthralgia
c. Assisting child in dealing with c. Chorea (Sydenhams chorea, St. Vitus
peers/returning to school dance): CNS disorder characterized by
d. Referral for parents to support abrupt, purposeless, involuntary muscular
groups/community agencies movements
1) Gradual, insidious onset: starts with
personality change or clumsiness
Acquired Heart Disease 2) Mostly seen in prepubertal girls
Rheumatic Fever (RF) 3) May appear months after strep
infection
A. General information 4) Movements increase with excitement
1. An inflammatory disorder that may involve 5) Lasts 13 months
the heart, joints, connective tissue, and the d. Subcutaneous nodules
CNS 1) Usually a sign of severe disease
2. Peaks in school-age children; linked to 2) Occur with active carditis
environmental factors and family history of 3) Firm, nontender nodes on bony
disorder prominences of joints
3. Thought to be an autoimmune disorder 4) Lasts for weeks
a. Preceded by an infection of group A beta- e. Erythema marginatum: transient,
hemolytic streptococcus (usually a strep nonpruritic rash starting with central red
throat); the heart itself is not infected, patches that expand; results in series of
however. irregular patches with red, raised margins
b. Antigenic markers for strep toxin closely and pale centers (resemble giraffe spots)
resemble markers for heart valves; this 2. Minor symptoms
resemblance causes antibodies made a. Reliable history of RF, fever
against the strep to also attack heart b. Recent history of strep infection
valves. c. Diagnostic tests: erythrocyte sedimentation
4. Prognosis depends on degree of heart damage rate (ESR) and antistreptolysin O (ASO)
B. Medical management titer increased; changes on ECG
1. Drug therapy D. Nursing interventions
a. Penicillin 1. Carditis
1) Used in the acute phase a. Administer penicillin as ordered.
2) Used prophylactically for several years 1) Used prophylactically to prevent
after the attack future attacks of strep and further
3) Erythromycin substituted if child is damage to the heart
sensitive to penicillin 2) To be taken until age 20 or for 5 years
b. Salicylates: for analgesic, anti- after attack, whichever is longer
inflammatory, antipyretic effect b. Promote bed rest until ESR returns to
c. Steroids: for anti-inflammatory effect normal.
2. Decrease cardiac workload: bed rest until lab 2. Arthritis: administer aspirin as ordered,

5
studies return to normal change childs position in bed frequently.

472 NCLEX-RN Review


53155_05_Ch05_p421-532.qxd 2/27/09 1:32 PM Page 473

3. Chorea 3. At the fifth intercostal space to the right of


a. Decrease stimulation. the midclavicular line.
b. Provide a safe environment: no forks with 4. In the aortic area.
meals, assistance with mobility.
c. Provide small, frequent meals; increased 63. A 2-week-old infant has a patent ductus
muscle activity causes increased kcal arteriosus. Prior to administering digoxin, which
requirements. action by the nurse would be correct?
4. Nodules and rash: none.
1. Take the apical pulse for 30 seconds and
5. Alleviate childs anxiety about the ability of
multiply by 2.
heart to continue to function.
6. Prevent recurrent infection. 2. Give the medication if his pulse is 92, but
7. Minimize boredom with age-appropriate notify the physician.
sedentary play. 3. Take the radial pulse for 1 full minute.
8. Provide client teaching and discharge 4. Give the medication after finding that the
planning concerning: apical pulse is 135 beats/minute.
a. Adaptation of home environment to
promote bed rest (commode, call bell, 64. The nurse is planning care for a 2-week-old
diversional activities) infant who has a congenital heart defect. Which
b. Importance of prophylactic medication of the following actions are appropriate? Select
regimen all that apply.
c. Diet modification in relation to decreased
_____ Using a soft preemie nipple for feedings.
activity/cardiac demands
d. Avoidance of reinfections _____ Providing passive stimulation.
e. Home-bound education _____ Allowing him to cry to promote increased
f. Availability of community agencies oxygenation.
_____ Placing him in orthopneic position.

65. A 10-year-old has been hospitalized for 2 weeks


Sample Questions with rheumatic fever. The childs mother
questions whether her other children can catch
the rheumatic fever. What is the nurses best
60. A 4-year-old with tetralogy of Fallot is seen in a response?
squatting position near his bed. What would be
1. The fact that you brought your child to the
the nurses response?
hospital early enough will decrease the
1. Administer oxygen. chance of your other children getting it.
2. Take no action if he looks comfortable but 2. It is caused by an autoimmune reaction and
continue to observe him. is not contagious.
3. Pick him up and place him in 3. You appear concerned that your childs
Trendelenburgs position in bed. disease is contagious.
4. Have him stand up and walk around the 4. Your other children should be taking
room. antibiotics to prevent them from catching
61. A 2-month-old is suspected of having rheumatic fever.
coarctation of the aorta. What is a cardinal sign 66. A 10-year-old child is admitted with rheumatic
of this defect? fever. In addition to carditis, for what other
1. Clubbing of the digits and circumoral problem would the nurse assess?
cyanosis. 1. Arthritis.
2. Pedal edema and portal congestion. 2. Bronchitis.
3. Systolic ejection murmur. 3. Malabsorption.
4. Upper extremity hypertension. 4. Oliguria.
62. When assessing the apical heart rate in infants 67. An infants blood pressure is reported to be very
and toddlers, where is the point of maximal high. What is the most appropriate nursing
impulse (PMI) located? action to take?
1. Between the third and fourth left intercostal 1. Take it again in 20 minutes.
space.
2. Call the nursing supervisor.
2. Between the fourth and fifth left intercostal

5
space.

PEDIATRIC NURSING 473


53155_05_Ch05_p421-532.qxd 2/27/09 1:32 PM Page 474

3. Measure the cuff width to the infants arm. 74. A 9-year-old boy has been transferred back to the
4. Prepare to give an antihypertensive. floor after cardiac surgery. Which of the
following does the nurse need to include in the
68. Prior to discharge from the newborn nursery at plan of care to evaluate that the fluid needs are
48 hours old, the nurse knows that murmurs are being appropriately met?
frequently assessed and are most often due to 1. Call if the heart rate falls below 60 per
which factor? minute.
1. A ventricular septal defect. 2. Place a Foley catheter.
2. Heart disease of the newborn period. 3. Prepare to assist with an arterial line to
3. Transition from fetal to pulmonic circulation. monitor blood pressure.
4. Cyanotic heart disease. 4. Calculate the daily maintenance fluid
69. A 10-year-old with a ventricular septal defect requirements and ensure correct delivery.
(VSD) is going to have a cardiac catheterization. 75. A 9-year-old girl with rheumatic fever is asking
Which of the following needs to be a high to play. Which diversional activity is the nurse
priority for the nurse to assess? likely to offer?
1. Capillary refill. 1. Walking to the gift store.
2. Breath sounds. 2. Coloring books and crayons.
3. Arrhythmias. 3. A 300-piece puzzle.
4. Pedal pulses. 4. A dancing contest.
70. An infant with heart failure (HF) is admitted to
76. A 10-year-old girl has been diagnosed with
the hospital. Which goal has the highest priority
rheumatic fever and is now being discharged.
when planning nursing care?
What statement made by the parents shows an
1. The infant will maintain an adequate fluid understanding of long-term care?
balance.
1. She will need penicillin each day.
2. The infant will have digoxin at the bedside.
2. She will need antibiotic prophylaxis when
3. Skin integrity will be addressed. she has dental work.
4. Administer medications on time. 3. We will have yearly checkups.
71. An infant on the ward is receiving digoxin and 4. The murmur will always go away by
diuretic therapy. The nurse knows that which of adolescence.
the following choices indicates no toxicity?
1. Heart rate less than 100, no dysrhythmias.
2. Heart rate of 80100.
3. Heart rate greater than 100, no dysrhythmias. Answers and Rationales
4. Vomiting.
72. An infant with cardiac disease has been 60. 2. Squatting is a normal response in a child who
admitted to the nursery from the delivery room. has tetralogy of Fallot. This position increases
Which finding helps the nurse to differentiate pulmonary blood flow because it changes the
between a cyanotic and an acyanotic defect? relationship between systemic and pulmonary
1. Infants with cyanotic heart disease feed poorly. vascular resistance.
2. The pulse oximeter does not read above 93%.
61. 4. Coarctation of the aorta is characterized by
3. Infants with cyanotic heart disease usually go
upper extremity hypertension and diminished
directly to the operating room.
pulses in the extremities.
4. Cyanotic heart disease causes high fevers.
62. 1. The heartbeat is most easily counted at the
73. A child with tetralogy of Fallot has been
point of maximum impulse. From birth through
admitted. What equipment is most important to
toddlerhood it is located between the third and
have at the bedside?
fourth left intercostal space.
1. Morphine.
2. A blood pressure cuff. 63. 4. The apical pulse is taken for one full minute
3. A thermometer. and the medication is withheld if the pulse is
less than 100 beats/minute.
4. An oxygen setup.

5 474 NCLEX-RN Review


53155_05_Ch05_p421-532.qxd 2/27/09 1:32 PM Page 475

64. Using a soft preemie nipple for feedings 69. 4. The nurse needs to know the baseline pedal
should be selected. This will help to reduce pulses.
energy expenditure.
Providing passive stimulation should be 70. 1. This is a major priority for HF clients.
selected. This will help to reduce energy 71. 3. Infants heart rates need to be greater than
expenditure. 100, with no rhythm disturbances.
Placing the child in orthopneic position should
be selected. This will help promote oxygenation. 72. 2. Cyanotic heart disease is unlikely to produce
a reading above 93%.
65. 2. Rheumatic fever is an autoimmune reaction to
a streptococcal infection and is limited to the 73. 4. This is used emergently in a TET spell.
person having the reaction. It is not a contagious
disease. 74. 4. It is vital for pediatric nurses to know exactly
how much fluid should be delivered each
66. 1. A major symptom of rheumatic fever is arthritis. 24 hours to evaluate proper fluid needs.

67. 3. The cuff should be approximately two thirds 75. 3. This will be quiet, yet stimulating.
the length of the humerus.
76. 2. This will be necessary for many years.
68. 3. As the transition occurs, the murmurs may
become loud, and then resolve.

The Hematologic System

VARIATIONS FROM THE ADULT 4. Ingestions: lead-based paint; drugs


5. Ability to participate in age-appropriate
A. In the young child all the bone marrow is involved activities
in blood cell formation.
B. By puberty, only the sternum, ribs, pelvis, Physical Examination
vertebrae, skull, and proximal epiphyses of femur
and humerus are involved. A. General appearance
C. During the first 6 months of life, fetal hemoglobin 1. Skin: note whether cyanotic, pale, ruddy,
is gradually replaced by adult hemoglobin, and it jaundiced; note bruises or petechiae, other
is only after this that hemoglobin disorders can be evidences of hemorrhage; pain, swelling
diagnosed. around joints.
2. Neurologic status: note listlessness or fatigue,
irritability, dizziness, or lightheadedness.
ASSESSMENT B. Measure vital signs; note tachycardia or tachypnea.
C. Plot height and weight on growth chart.
History D. Inspect and palpate abdomen; note enlargement of
A. Family history: genetic hematologic disorders, liver and spleen, pain, or tenderness on palpation.
anemia, or jaundice
B. History of pregnancy: parents blood types,
anemia, infection or drug ingestion, course of ANALYSIS
labor and delivery
C. Childs health history Nursing diagnoses for the child with a disorder of the
1. Neonatal course: occurrence, duration, and hematologic system may include:
treatment of jaundice; bleeding episodes; A. Activity intolerance
blood transfusions B. Pain
2. Accidents, operations, hospitalizations (any C. Impaired gas exchange
blood transfusions or unusual bleeding) D. Ineffective tissue perfusion: cardiopulmonary
3. Nutrition: dietary intake of iron and vitamin E. Imbalanced nutrition

5
B12; history of pica

PEDIATRIC NURSING 475


53155_05_Ch05_p421-532.qxd 2/27/09 1:32 PM Page 476

PLANNING AND F. Parents are able to administer medications and


participate in childs care.
IMPLEMENTATION
Goals DISORDERS OF THE
A. Child will have adequate tissue oxygenation. HEMATOLOGIC SYSTEM
B. Child will be free from complications associated
with hematologic diseases. Anemias
C. Child will be free from pain, or have pain
controlled. Iron Deficiency Anemia
D. Optimal growth and developmental level will be
A. General information: iron deficiency is most
achieved.
common cause of anemia in children; children
E. Parents will participate in care of child.
whose diet consists mainly of cows milk, which is
low in absorbable iron, are especially vulnerable.
Interventions B. Assessment findings
See Unit 4. 1. Pallor, fatigue, irritability
2. History of iron-deficient diet
3. Diagnostic tests
EVALUATION a. RBC normal or slightly reduced
b. Hgb below normal range for child
A. Serum values of hematologic components are c. HCT below normal
normal. C. Nursing interventions
B. Child is free from signs or symptoms of infection. 1. Add iron to formula, food, or by vitamins by
C. Child has no abnormal bleeding episodes. age 46 months.
D. Normal activity level is maintained without pain a. Oral iron
or fatigue. 1) Give iron with citrus juice and on
E. Parents are able to describe symptoms of disease empty stomach (iron is best absorbed
and complications. in an acidic environment).
Sickle-cell anemia

Abnormal
hemoglobin
molecule

Sickling of
red blood
cells
Rapid Accumulation
destruction of red blood
of sickled cells in
cells Clumping of spleen
red blood cells
interferes with blood
circulation

Joint and
Heart Kidney Brain Lung Spleen
Anemia muscle
problem damage damage damage damage
damage

Overactive Dilation
bone marrow of heart

Tower Physical Heart Kidney Spleen


Rheumatism Paralysis Pneumonia
skull fatigue failure failure fibrosis

Figure 5-5 A series of damages and effects caused by sickle-cell anemia

5 476 NCLEX-RN Review


53155_05_Ch05_p421-532.qxd 2/27/09 1:32 PM Page 477

2) Have child use straw if possible, 11. Death often occurs in early adulthood due to
because iron stains teeth and skin. occlusion or infection.
b. Administer IM iron if ordered. Use z-track 12. Sickle cell crisis
method. a. Vaso-occlusive (thrombocytic) crisis: most
2. Provide iron-rich foods: meats, nuts, dried common type
beans/legumes, dried fruit, dark-green leafy 1) Crescent-shaped RBCs clump together;
vegetables, whole grains, egg yolk, potatoes, agglutination causes blockage of small
shellfish. blood vessels.
D. Also see Unit 4. 2) Blockage causes the blood viscosity to
increase, producing sludging and
Sickle-Cell Anemia resulting in further hypoxia and
increased sickling.
A. General information (see Figure 5-5) b. Splenic sequestration: often seen in
1. Most common inherited disorder in U.S. toddler/preschooler
African American population; sickle cell trait 1) Sickled cells block outflow tract
found in 10% of African Americans. resulting in sudden and massive
2. Autosomal recessive inheritance pattern. collection of sickled cells in spleen.
3. Individuals who are homozygous for the sickle 2) Blockage leads to hypovolemia and
cell gene have the disease (more than 80% of severe decrease in hemoglobin and
their hemoglobin is abnormal [HgbS]). blood pressure, leading to shock.
4. Those who are heterozygous for the gene have B. Medical management: sickle cell crisis
sickle cell trait (normal hemoglobin 1. Drug therapy
predominates, may have 2550% HgbS). a. Urea: interferes with hydrophobic bonds of
Although sickle cell trait is not a disease, the HgbS molecules
carriers may exhibit symptoms under periods b. Analgesics/narcotics to control pain
of severe anoxia or dehydration. c. Antibiotics to control infection
5. In this disease, the structure of hemoglobin is 2. Exchange transfusions
changed; the sixth rung of the beta chain 3. Hydration: oral and IV
changes glutamine for valine. 4. Bed rest
6. HgbS (abnormal Hgb), which has reduced 5. Surgery: splenectomy
oxygen-carrying capacity, replaces all or part C. Assessment findings
of the hemoglobin in the RBCs. 1. First sign in infancy may be colic due to
7. When oxygen is released, the shape of the abdominal pain (abdominal infarct)
RBCs changes from round and pliable to 2. Infants may have dactylitis (hand-foot
crescent-shaped, rigid, and inflexible syndrome): symmetrical painful soft tissue
(see Figure 5-6). swelling of hands and feet in absence of
8. Local hypoxia and continued sickling lead to trauma (aseptic, self-limiting)
plugging of vessels. 3. Splenomegaly: initially due to hemolysis and
9. Sickled RBCs live for 620 days instead of 120, phagocytosis; later due to fibrosis from
causing hemolytic anemia. repeated infarct to spleen
10. Usually no symptoms prior to age 6 months; 4. Weak bones or spinal defects due to
presence of increased level of fetal hemoglobin hyperplasia of marrow and osteoporosis
tends to inhibit sickling. 5. Frequent infections, especially with
H. influenzae and D. pneumoniae
6. Leg ulcers, especially in adolescents, due to
blockage of blood supply to skin of legs
7. Delayed growth and development, especially
delay in sexual development
8. CVA/infarct in the CNS
9. Renal failure: difficulty concentrating urine
due to infarcts; enuresis
10. Heart failure due to hemosiderosis
11. Priapism: may result in impotence
Normal RBC 12. Pain wherever vaso-occlusive crisis occurs
Sickled RBC 13. Development of collateral circulation
14. Diagnostic tests
a. Hgb indicates anemia, usually 69 g/dL
b. Sickling tests
Figure 5-6 Regular and sickled RBCs 1) Sickle cell test: deoxygenation of a

5
drop of blood on a slide with

PEDIATRIC NURSING 477


53155_05_Ch05_p421-532.qxd 2/27/09 1:32 PM Page 478

a cover slip; takes several hours for 4. The spleen is the site for destruction of
results to be read; false negatives for platelets; spleen is not enlarged
the trait possible. B. Medical management
2) Sickledex: a drop of blood from 1. Drug therapy: steroids, immunosuppressive
a finger stick is mixed with a solution; agents, anti-D antibody
mixture turns cloudy in presence 2. Platelet transfusion
of HgbS; results available within a 3. Surgery: splenectomy
few minutes; false negatives in C. Assessment findings
anemia clients or young infants 1. Petechiae: spider-web appearance of bleeding
possible. under skin due to small size of platelets
c. Hgb electrophoresis: diagnostic for the 2. Ecchymosis
disease and the trait; provides accurate, 3. Blood in any body secretions, bleeding from
fast results. mucous membranes, nosebleeds
D. Nursing interventions: sickle cell crisis 4. Diagnostic tests: platelet count decreases,
1. Keep child well hydrated and oxygenated. anemia
2. Avoid tight clothing that could impair D. Nursing interventions
circulation. 1. Control bleeding
3. Keep wounds clean and dry. a. Administer platelet transfusions as
4. Provide bed rest to decrease energy ordered.
expenditure and oxygen use. b. Apply pressure to bleeding sites as needed.
5. Correct metabolic acidosis. c. Position bleeding part above heart level if
6. Administer medications as ordered. possible.
a. Analgesics: acetaminophen, Ketoralac 2. Prevent bruising.
(NSAID), morphine (avoid aspirin as it 3. Provide support to client and be sensitive to
enhances acidosis, which promotes change in body image.
sickling). 4. Protect from infection.
b. Avoid anticoagulants (sludging is not due 5. Measure normal circumference of extremities
to clotting) for baseline.
c. Antibiotics 6. Administer medications orally, rectally, or IV,
7. Administer blood transfusions as ordered. rather than IM; if administering
8. Keep arms and legs from becoming cold. immunizations, give subcutaneously (SC) and
9. Decrease emotional stress. hold pressure on site for 5 minutes.
10. Provide good skin care, especially to legs. 7. Administer analgesics (acetaminophen) as
11. Test siblings for presence of sickle cell ordered; avoid aspirin.
trait/disease. 8. Provide care for the client with a splenectomy.
12. Provide client teaching and discharge 9. Provide client teaching and discharge
planning concerning: planning concerning:
a. Pre-op teaching for splenectomy if a. Pad crib and playpen, use rugs wherever
needed possible.
b. Genetic counseling b. Provide soft toys.
c. Need to avoid activities that interfere with c. Sew pads in knees and elbows of clothing.
oxygenation, such as mountain climbing, d. Provide protective headgear during
flying in unpressurized planes toddlerhood.
e. Use soft Toothettes instead of bristle
toothbrushes.
Disorders of Platelets or f. Keep weight to low normal to decrease
Clotting Mechanism extra stress on joints.
g. Use stool softeners to prevent straining.
Immune Thrombocytopenic Purpura (ITP) h. Avoid contact sports; suggest swimming,
biking, golf, billiards.
A. General information
1. Formerly known as idiopathic
thrombocytopenic purpura Hemophilia
2. Increased destruction of platelets with A. General information
resultant platelet count of less than 1. A group of bleeding disorders in which there
100,000/mm3 characterized by petechiae and is a deficit of one of several factors in clotting
ecchymoses of the skin mechanism
3. Autoimmune disorder; onset sudden, often 2. Sex-linked, inherited disorder; classic form
preceded by a viral illness affects males only

5 478 NCLEX-RN Review


53155_05_Ch05_p421-532.qxd 2/27/09 1:32 PM Page 479

3. Types 2. Provide care for hemarthrosis.


a. Hemophilia A: factor VIII deficiency (75% a. Immobilize joint and control acute
of all hemophilia) bleeding.
b. Hemophilia B (Christmas disease): factor b. Elevate joint in a slightly flexed position.
IX deficiency (1012% of all hemophilia) c. Avoid excessive handling of joint.
c. Hemophilia C: factor XI deficiency d. Administer analgesics as ordered; pain
(autosomal recessive, affects both sexes) relief will minimize increases in pulse rate
4. Only the intrinsic system is involved; platelets and blood loss.
are not affected, but fibrin clot does not always e. Aspirin should not be given because it
form; bleeding from minor cuts may be inhibits platelet function.
stopped by platelets. f. Instruct to avoid weight bearing for
5. If individual has less than 2030% of factor 48 hours after bleeding episode if
VIII or IX, there is an impairment of clotting bleeding is in lower extremities.
and clot is jelly-like. g. Provide active or passive ROM exercises
6. Bleeding in neck, mouth, and thorax requires after bleeding has been controlled
immediate professional care. (48 hours), as long as exercises do not
B. Assessment findings cause pain or irritate trauma site.
1. Prolonged bleeding after minor injury 3. Administer factor VIII concentrate, or DDAVP
a. At birth after cutting of cord as ordered
b. Following circumcision 4. Provide client teaching and discharge
c. Following IM immunizations planning concerning
d. Following loss of baby teeth a. Prevention of trauma (see Immune
e. Increased bruising as child learns to crawl Thrombocytopenic Purpura)
and walk b. Genetic counseling
2. Bruising and hematomas but no petechiae 1) When mother is carrier: 50% chance
3. Peripheral neuropathies (due to bleeding near with each pregnancy for sons to have
peripheral nerves): pain, paresthesias, muscle hemophilia, 50% chance with each
atrophy pregnancy for daughters to be carriers
4. Hemarthrosis 2) When father has hemophilia, mother is
a. Repeated bleeding into a joint results in a normal: no chance for children to have
swollen and painful joint with limited disease, but all daughters will be
mobility carriers
b. May result in contractures and possible c. Availability of support/counseling agencies
degeneration of joint
c. Knees, ankles, elbows, wrists most often
affected
Disorder of White Blood Cells
5. Diagnostic tests Infectious Mononucleosis
a. Platelet count normal
b. Prolonged coagulation time: PTT increased A. General information
c. Anemia 1. Viral infection that causes hyperplasia of
C. Nursing interventions lymphoid tissue and a characteristic change in
1. Control acute bleeding episode. mononuclear cells of the blood
a. Apply ice compress for vasoconstriction. 2. Affects adolescents and young adults most
b. Immobilize area to prevent clots from commonly
being dislodged. 3. Caused by the Epstein-Barr virus, which is not
c. Elevate affected extremity above heart highly contagious but is transmitted by saliva
level. (the kissing disease)
d. Provide manual pressure or pressure 4. Incubation period 26 weeks
dressing for 15 minutes; do not keep lifting 5. Pathophysiology: mononuclear infiltration of
dressing to check for bleeding status. lymph nodes and other body tissue
e. Maintain calm environment to decrease B. Assessment findings
pulse. 1. Lethargy
f. Avoid sutures, cauterization, aspirin: all 2. Sore throat/tonsilitis
exacerbate bleeding. 3. Lymphadenopathy; enlarged spleen, liver
g. Administer hemostatic agents as ordered. involvement
1) Fibrin foam 4. Diagnostic tests
2) Topical application of a. Atypical WBCs increased
adrenalin/epinephrine to promote b. Heterophil antibody and Monospot tests
vasoconstriction positive

PEDIATRIC NURSING

5 479
53155_05_Ch05_p421-532.qxd 2/27/09 1:32 PM Page 480

blood cells do not have as long a life expectancy


Sample Questions as normal red cells. Which answer would be
correct for the nurse to tell the mother regarding
how long a sickled blood cell survives?
77. The mother of a child with classic hemophilia 1. 5 days.
asks the nurse what her chances are of having
2. 15 days.
another child with hemophilia. What is the
nurses best response? 3. 30 days.
1. All of your daughters will be carriers of the 4. 60 days.
disease.
82. What symptom would the child with sickle cell
2. If you have another son, there is almost a exhibit?
100% chance he will have hemophilia.
1. Vitiligo.
3. If you have a son, there is a 50% chance he
2. Hyperactivity.
will have hemophilia but none of your
daughters are likely to have it. 3. Mild mental retardation.
4. There is a 25% chance of having another 4. Delayed physical development.
child with hemophilia. 83. A child who has sickle cell anemia has
78. A 4-year-old male has been diagnosed as having developed stasis ulcers on her lower extremities.
iron deficiency anemia. A liquid iron What is the cause of the ulcers?
preparation has been prescribed. What 1. Poor range of motion.
intervention will the nurse include in the 2. Ruptured blood vessels.
medication administration? 3. Impaired venous circulation.
1. Ask the child if he wants to take his medicine. 4. Hypertrophy of muscular tissue.
2. Mix the medication in his milk bottle and
give it to him at nap time. 84. Which complication is associated with sickle
3. Allow him to sip the medication through a cell anemia?
straw. 1. Constipation.
4. Give the medication after lunch with a sweet 2. Hypothyroidism.
dessert to disguise the taste. 3. Addisons disease.
4. Cerebrovascular accidents.
79. A 10-year-old has hemophilia A and is admitted
to the hospital for hemarthrosis of the right knee. 85. Both parents carry the sickle cell anemia trait.
He is in a great deal of pain. Which of the Their 8-month-old child contracted chickenpox
following interventions would aggravate his from his brother and now is very weak, febrile,
condition? and anorexic, and cries with pain when his
1. Applying an ice bag to the affected knee. wrists and elbows are moved. He is admitted to
2. Administering childrens aspirin for pain relief. the hospital with a diagnosis of sickle cell crisis.
3. Elevating the right leg above the level of his The childs mother asks the nurse why he has
heart. not been symptomatic before now. What would
be the nurses best response?
4. Keeping the right leg immobilized.
1. High fetal hemoglobin protected him against
80. Which is a correct statement regarding a client sickling.
with the sickle cell trait? 2. His red blood cell levels remained normal.
1. The client has a chronic form of sickle cell 3. Maternal antibodies protected him against
anemia. sickling.
2. The client has the most lethal form of the 4. Sickle cell hemoglobin was not present until
disease. about 1 year of life.
3. The client will transmit the disease to all
children. 86. In planning care for a child with newly
4. The client has some normal and some diagnosed sickle cell anemia, what should his
abnormal hemoglobin cells. mother be taught to prevent vaso-occlusive
crises?
81. The mother of a child with sickle cell anemia 1. Prophylactic administration of acetaminophen.
tells the nurse that she learned that sickled 2. Eating food with a high iron content.

5 480 NCLEX-RN Review


53155_05_Ch05_p421-532.qxd 2/27/09 1:32 PM Page 481

3. Exercising regularly. 3. Elevating and immobilizing the affected joint.


4. Promoting hydration. 4. Encouraging active range of motion of
affected joint.
87. How could the nurse best evaluate whether
parents are giving their child with iron
deficiency anemia iron as prescribed?
1. Parents state they offer orange juice when Answers and Rationales
they give the medication.
2. Parents state the child has greenish black
stools. 77. 3. Classic hemophilia is inherited as an X-linked
3. Parents state the child experiences nausea recessive trait. If this family has another son,
with the iron preparation. there is a 50% chance that he will have the
4. Parents state they are giving the iron as disease. If they have a daughter she is very
prescribed. unlikely to have the disease but there is a 50%
chance she will be a carrier.
88. Parents of a child who has sickle-cell anemia
want to know why their child did not have the 78. 3. Iron is given with a straw to prevent staining
first episode until he was approximately a year the teeth.
old. What would be the best reply from the nurse? 79. 2. Aspirin is an anticoagulant. The child has a
1. Are you sure your child has sickle-cell clotting disorder and has been bleeding into his
anemia and not sickle-cell trait? knee joint. He should not receive an
2. Affected children can be asymptomatic in anticoagulant.
infancy because of high levels of fetal
hemoglobin that inhibit sickling. 80. 4. Clients with sickle cell trait inherit only one
3. Have you asked your doctor about this? defective gene. They can synthesize both normal
and abnormal hemoglobin chains.
4. Your child probably had a crisis and you did
not realize it. 81. 2. The life span of a sickled cell is 6 to 20 days
as opposed to 120 days for a normal red blood
89. A 5-year-old is admitted to the nursing care unit
cell.
in vaso-occlusive crisis from sickle cell anemia.
What is the priority nursing intervention? 82. 4. Children with sickle cell disease usually
1. Teaching the family about sickle cell anemia manifest growth impairment.
and home care needs.
2. Managing the childs pain. 83. 3. The tissues of a client with sickle cell disease
are constantly vulnerable to microcirculatory
3. Encouraging a high-protein, high-calorie diet.
interruptions.
4. Administering oxygen via nasal cannula.
84. 4. The sudden appearance of a stroke in sickle
90. A 3-year-old with a recent history of chickenpox cell anemia is related to the microcirculatory
is admitted to the unit with immune interruptions that are caused by the sickled
thrombocytopenic purpura. His platelet count is cell.
15,000 mm3/dL. His lesions are enlarging.
Which of the following nursing actions best 85. 1. High levels of fetal hemoglobin inhibit
provides for the childs safety? sickling of red cells prior to the age of 6 months.
1. Supervised outdoor play.
86. 4. Promoting good hydration is a major factor in
2. Set times of rest periods.
maintaining the blood viscosity needed to
3. Only allowing him to have soft stuffed toys to maximize the circulation of red blood cells.
play with. Dehydration causes sickling of red blood cells.
4. Keeping him on complete bed rest.
87. 2. When an adequate dosage of iron is reached,
91. A child is admitted to the pediatric unit with the stools usually turn a greenish black. Absence
hemarthrosis secondary to hemophilia. What of this color stool usually gives a clue to poor
would be the most appropriate nursing compliance.
intervention?
1. Daily bleeding times. 88. 2. Children with sickle-cell anemia are often
asymptomatic until at least 4 to 6 months of age.

5
2. Prophylactic antibiotic therapy.

PEDIATRIC NURSING 481


53155_05_Ch05_p421-532.qxd 2/27/09 1:32 PM Page 482

A crisis is usually precipitated by an acute 90. 4. Initially when his platelets are below 20,000
upper respiratory or gastrointestinal infection. mm3/dL and he is experiencing active bleeding
or progression of lesions, activity is restricted.
89. 4. During a vaso-occlusive crisis, tissue hypoxia
and ischemia cause pain. By delivering oxygen 91. 3. During bleeding episodes, hemarthrosis is
at the prescribed rate, further tissue hypoxia can managed by elevating and immobilizing the joint
be avoided. and applying ice packs.

The Respiratory System

VARIATIONS FROM THE ADULT 3. Pectus excavatum (funnel chest): lower part of
sternum is depressed; usually does not
A. Infants are obligatory nose breathers and produce symptoms; may impair cardiac
diaphragmatic breathers. function.
B. Number and size of alveoli continue to increase B. Note pattern of respirations.
until age 8 years. 1. Rate
C. Until age 5, structures of the respiratory tract have 2. Regularity
a narrower lumen and children are more a. Periodic respirations (periods of rapid
susceptible to obstruction/distress from respirations, separated by periods of slow
inflammation. breathing or short periods of no
D. Normal respiratory rate in children is faster than respirations) normal in young infants
in adults. b. Apnea episodes (cessation of breathing for
1. Infants: 4060/minute 20 seconds or more accompanied by color
2. 1 year: 2040/minute change or bradycardia) an abnormal
3. 24 years: 2030/minute finding
4. 510 years: 2025/minute 3. Respiratory effort
5. 1015 years: 1722/minute a. Nasal flaring: attempt to widen airway and
6. 15 and older: 1520/minute decrease resistance
E. Most episodes of acute illness in young children b. Open-mouth breathing: chin drops with
involve the respiratory system due to frequent each inhalation
exposure to infection and a general lack of c. Retractions: from use of accessory
immunity. muscles
C. Observe skin color and temperature, particularly
mucous membranes and peripheral extremities.
ASSESSMENT D. Note behavior: position of comfort, signs of
irritability or lethargy, facial expression (anxiety).
History E. Note speech abnormalities: hoarseness or muffled
A. Presenting problem: symptoms may include speech.
cough, wheezing, dyspnea F. Observe presence and quality of cough:
B. Medical history: incidence of infections, productive; paroxysmal, with inspiratory whoop
respiratory allergies or asthma, prescribed and characteristic of pertussis.
OTC medications, recent immunizations G. Auscultate for abnormal breath sounds
C. Exposure to other children with respiratory (auscultation may be more difficult in infants and
infections or other communicable diseases young children because of shallowness of
respirations).
1. Grunting on expiration
Physical Examination 2. Stridor: harsh inspiratory sound associated
A. Inspect shape of chest; note: with obstruction or edema
1. Barrel chest: occurs with chronic respiratory 3. Wheezing: whistling noise during inspiration
disease. or expiration due to narrowed airways,
2. Pectus carinatum (pigeon breast): sternum common in asthma
protrudes outward, producing increased AP 4. Snoring: noisy breathing associated with
diameter; usually not significant. nasal obstruction

5 482 NCLEX-RN Review


53155_05_Ch05_p421-532.qxd 2/27/09 1:32 PM Page 483

Laboratory/Diagnostic Tests 5. Mist is usually prescribed in addition to


oxygen
A. Pulmonary function testing is usually not done a. Keep reservoir for humidification filled.
under age 6 years because children have difficulty b. Do not allow condensation on tent walls to
following directions. obstruct view of child.
B. Chest X-rays: avoid unnecessary exposure; protect c. Keep clothes and bedding dry to avoid
gonads and thyroid. chilling.
6. Provide safety measures.
a. Keep plastic away from childs face.
ANALYSIS b. Avoid toys that produce spark or friction,
such as mechanical toys.
Nursing diagnoses for the child with a disorder of the c. Avoid stuffed toys because of tendency to
respiratory system may include: absorb moisture.
A. Activity intolerance d. Encourage use of one or two favorite toys
B. Altered respiratory functions: ineffective airway or transitional object in tent; other toys
clearance, ineffective breathing pattern, impaired may be kept outside of tent in childs
gas exchange view.
C. Anxiety
D. Fatigue Vaporizers
E. Impaired oral mucous membrane
F. Altered nutrition A. General information
G. Disturbed sleep pattern 1. Same principle as oxygen tent
H. Acute pain 2. Used at home; placed at bedside; mist directed
I. Deficient knowledge (caregiver) (specify) into room around child
3. Usually cool mist
B. Nursing responsibilities: teach parents to clean
PLANNING AND frequently because bacteria that grow in vaporizer
can be dispersed into air.
IMPLEMENTATION
Chest Physical Therapy
Goals
A. Postural drainage: infants and young children do
A. Child will have patent airway and satisfactory not have enough rib cage for a lower front position
oxygenation. 1. Combine side and lower front positions
B. Child will be free from symptoms of respiratory 2. Five positions: upper front, upper back, lower
distress. back, right side and front, and left side and
C. Child will have improved ability to tolerate front
exercise, conserve energy. 3. 25 minutes per position
D. Parents will participate in caring for child. B. Percussion
1. Do not use with clients with an acute attack of
Interventions asthma or croup (dislodged mucus may cause
plugging because of bronchial edema).
Oxygen Tent (Croup Tent, Mist Tent, 2. Use percussion 30 minutes before meals to
Oxygen Canopy) clear mucus before eating, thus enhancing
intake.
A. General information 3. If aerosol medications are being used,
1. Used when desired oxygen concentration is administer immediately before percussion.
40% or less as oxygen concentration can be 4. Percussion is done with cupped hand, never
difficult to control on bare skin, over the rib cage only
2. Primarily used for croup, when mist is to be a. Use an undershirt, gown, or diaper over
delivered skin.
B. Nursing care b. If infants chest is too small for nurses
1. Keep sides of plastic down and tucked in. hand, a small face mask can be substituted.
2. If tent has been opened for awhile, increase c. Be careful to avoid spine and neck during
oxygen flow to raise concentration quickly. percussion.
3. If child has been out of the tent, return oxygen d. Infants can be percussed while being held
concentration to ordered percent before on nurses lap.
returning child to tent. e. If child is unable to cough during and after
4. If tent enclosure gets too warm, add ice to percussion, suction as needed.
cooling chamber as needed.

5
C. Vibration: performed on expiration.

PEDIATRIC NURSING 483


53155_05_Ch05_p421-532.qxd 2/27/09 1:32 PM Page 484

Suctioning DISORDERS OF THE


A. Bulb syringes can be used to clear nasal stuffiness. RESPIRATORY SYSTEM
B. For nasotracheal and nasopharyngeal suction, use
low pressure.
C. Assess response, and for improved respiratory
Tonsillitis
status. A. General information
1. Inflammation of tonsils often as a result of a
Deep Breathing Exercises viral or bacterial pharyngitis
2. 1015% caused by group A beta-hemolytic
A. Encourage deep breathing by making exercises into streptococci
games (e.g., touch toes, sit-ups, jumping jacks, B. Medical management
blowing out flashlight, ping-pong ball games 1. Comfort measures and symptomatic relief
using blowing). 2. Antibiotics for bacterial infection, usually
B. Encourage use of toys that require blowing penicillin or erythromycin
(harmonica, bubbles). 3. Surgery: removal of tonsils/adenoids if
C. Laughing and crying also stimulate coughing and necessary
deep breathing. C. Assessment findings
1. Enlarged, red tonsils; fever
Apnea Monitor 2. Sore throat, difficulty swallowing, mouth
breathing, snoring
A. General information: monitors often use same 3. White patches of exudate on tonsillar pillars,
three chest leads for simultaneous cardiac and enlarged cervical lymph nodes
respiratory rate monitoring D. Nursing interventions
1. Lead placement will differ from that usually 1. Provide soft or liquid diet.
prescribed for cardiac monitoring if apnea 2. Use cool-mist vaporizer.
monitoring is also required. 3. Administer salt water gargles, throat lozenges.
2. To monitor respiratory rate, chest leads will 4. Administer analgesics (acetaminophen) as
need to be where chest moves during ordered.
inspiration. 5. Administer antibiotics as ordered; stress to
3. As chest wall movement rather than air entry parents importance of completing entire
is monitored, obstruction and dyspnea may course of medication.
not be recognized early.
4. Most useful for early recognition of cessation
of breathing. Tonsillectomy
B. Nursing care: when alarm sounds A. General information
1. Note whether cardiac or respiratory rate has 1. One of the most common operations
triggered alarm. performed on children
2. Assess childs color, activity, and presence of 2. Indications for tonsillectomy include recurrent
respiratory effort. tonsillitis, peritonsillar abscess, airway or
3. Auscultate cardiac and respiratory sounds. esophageal obstruction
4. If no physical distress, check lead placement. 3. Indications for adenoidectomy include nasal
5. If apneic, gently stimulate lower extremities. obstruction due to hypertrophy
May need oxygen. B. Nursing interventions: preoperative
6. If no improvement with stimulation and 1. Make pre-op preparation age-appropriate;
oxygen, assess need for cardiopulmonary child enters the hospital feeling well and will
resuscitation (CPR). leave with a very sore throat.
2. Obtain baseline bleeding and clotting times.
3. Check for any loose teeth.
EVALUATION C. Nursing interventions: postoperative
1. Position on side or abdomen to facilitate
A. Child is satisfactorily oxygenated. drainage of secretions.
1. Absence of respiratory distress 2. Avoid suctioning if possible; if not, be
2. Normal color and activity especially careful to avoid trauma to surgical
3. Decreased need for supplementary oxygen site.
therapy 3. Provide ice collar/analgesia for pain.
B. Parents are able to care for child at home. 4. Observe for hemorrhage; signs may include
1. Identify symptoms of increased oxygen need frequent swallowing, increased pulse,
2. Perform prescribed treatments vomiting bright red blood (vomiting old dried
3. Have obtained and demonstrated use of

5
blood or pink-tinged emesis is normal).
necessary equipment

484 NCLEX-RN Review


53155_05_Ch05_p421-532.qxd 2/27/09 1:32 PM Page 485

5. Offer clear, cool, noncitrus, nonred fluids several days with upper respiratory
when awake and alert. infection
6. Provide client teaching and discharge 7. Usually treated on outpatient basis;
planning concerning: indications for admission include dehydration
a. Need to maintain adequate fluid and food and respiratory compromise
intake and to avoid spicy and irritating foods B. Medical management
b. Quiet activity for a few days 1. Drug therapy
c. Need to avoid coughing, mouth gargles a. Aerosolized racemic epinephrine
d. Chewing gum (but not Aspergum): can b. Antibiotics only if secondary bacterial
help relieve pain and difficulty swallowing infection present
and aids in diminishing bad breath c. Steroids
e. Mild analgesics for pain 2. Oxygen therapy: low concentrations to relieve
f. Signs and symptoms of bleeding and need mild hypoxia
to report to physician 3. Oral or nasotracheal intubation for moderate
hypoxia
Acute Spasmodic Laryngitis (Croup) 4. IV fluids to maintain hydration
C. Assessment findings
A. General information 1. Fever, coryza, inspiratory stridor, barking
1. Respiratory distress characterized by cough, tachycardia, tachypnea, retractions
paroxysmal attacks of laryngeal obstruction 2. May have difficulty taking fluids
2. Etiology unclear but familial predisposition, 3. WBC normal
allergy, viruses, psychologic factors, and D. Nursing interventions
anxious temperament have been implicated 1. Instruct parents to take child into steamy
3. Common in children ages 13 years bathroom for acute distress.
4. Attacks occur mostly at night; onset sudden 2. Keep child calm.
and usually preceded by a mild upper 3. After distress subsides, use cool mist vaporizer
respiratory infection in bedroom.
5. Respiratory symptoms last several hours; may 4. Child may vomit large amounts of mucus after
occur in a milder form on a few subsequent the episode; reassure parents that this is normal.
nights 5. For hospitalized child
B. Assessment findings a. Monitor vital signs, I&O, skin color, and
1. Inspiratory stridor, hoarseness, barking cough, respiratory effort.
anxiety, retractions b. Maintain hydration.
2. Afebrile, skin cool c. Provide care for the intubated child.
C. Nursing interventions d. Plan care to disturb the child as little as
1. Instruct parents to take the child into the possible.
bathroom, close the door, turn on the hot
water, and sit on floor of the steamy bathroom
with child.
Epiglottitis
2. If the laryngeal spasm does not subside the A. General information
child should be taken to the emergency 1. Life-threatening bacterial infection of
department. epiglottis and surrounding structures
3. After the spasm subsides, provide cool mist 2. Primary organism: H. influenzae, type B
with a vaporizer. 3. Often preceded by upper respiratory infection
4. Provide clear fluids. 4. Rapid progression of swelling causes
5. Try to keep child calm and quiet. reduction in airway diameter; may lead to
6. Assure parents this is self-limiting. sudden respiratory arrest
5. Affects children ages 37 years
B. Assessment findings
Laryngotracheobronchitis 1. Fever, tachycardia, inspiratory stridor
A. General information (possibly), labored respirations with retractions,
1. Viral infection of the larynx that may extend sore throat, dysphagia, drooling, muffled
into trachea and bronchi voice
2. Most common cause for stridor in febrile child 2. Irritability, restlessness, anxious-looking, quiet
3. Parainfluenza viruses most common cause 3. Position: sitting upright, head forward and
4. Infection causes endothelial insult, increased jaw thrust out
mucus production, edema, low-grade fever 4. Diagnostic tests
5. Affects children less than 5 years of age a. WBC increased
6. Onset more gradual than with croup, takes b. Lateral neck X-ray reveals characteristic

5
longer to resolve; usually develops over findings

PEDIATRIC NURSING 485


53155_05_Ch05_p421-532.qxd 2/27/09 1:32 PM Page 486

C. Nursing interventions 2. Offer small, frequent feedings; clear fluids if


1. Provide mist tent with oxygen. trouble with secretions.
2. Administer IV antibiotics as ordered. 3. Provide adequate rest.
3. Provide tracheostomy or endotracheal tube 4. Administer antipyretics as ordered to control
care; note the following: fever.
a. Restlessness, fatigue, dyspnea, cyanosis,
pallor, tachycardia, tachypnea, diminished
breath sounds, adventitious lung sounds.
Asthma
b. Need for suctioning to remove secretions; A. General information (See Figure 5-7.)
note amount, color, consistency. 1. Obstructive disease of the lower respiratory
4. Reassure child through touch, sound, and tract
physically being present. 2. Most common chronic respiratory disease in
5. Involve parents in all aspects of care. children, in younger children affects twice as
6. Avoid direct examination of the epiglottis as it many boys as girls; incidence equal by
may precipitate spasm and obstruction. adolescence
7. Remember this is extremely frightening 3. Often caused by an allergic reaction to an
experience for child and parents; explain environmental allergen, may be seasonal or
procedures and findings; reinforce year-round
explanations of physician. 4. Immunologic/allergic reaction results in
histamine release, which produces three main
airway responses
Bronchiolitis a. Edema of mucous membranes
A. General information b. Spasm of the smooth muscle of bronchi
1. Pulmonary viral infection characterized by and bronchioles
wheezing c. Accumulation of tenacious secretions
2. Usually caused by respiratory syncytial virus 5. Status asthmaticus occurs when there is little
3. Virus invades epithelial cells of nasopharynx response to treatment and symptoms persist
and spreads to lower respiratory tract, causing
increased mucus production, decreased
diameter of bronchi, hyperinflation, and
Normal
possible atelectasis
4. Affects infants ages 28 months
5. Increased incidence of asthma as child grows
older
B. Medical management
1. Nebulized bronchodilators (e.g., Albuterol)
2. Steroids
3. Ribavirinantiviral, given by aerosol (SPAG)
through hood, tent, mask, or ventilatorfor
severe symptoms
4. Humidity, oxygen, fluids
5. Prevention: RSV antibodies
Asthma
a. RSV-IG (immune globulin)-given IV
monthly
b. Palivizumab (Synagis)-given IM monthly
C. Assessment findings
1. Difficulty feeding, fever
Mucosal edema
2. Cough, coryza
3. Wheezing, prolonged expiratory phase,
tachypnea, nasal flaring, retractions Mucus in airway
(intercostal more pronounced than Contracted
supraclavicular retractions) hypertrophied
4. Diagnostic tests muscle
a. WBC normal
Enlarged
b. X-ray reveals hyperaeration mucous glands
D. Nursing interventions
1. Provide high-humidity environment, with
oxygen in some cases (instruct parents to take Figure 5-7 Pathophysiology of asthma
child into steamy bathroom if at home).

5 486 NCLEX-RN Review


53155_05_Ch05_p421-532.qxd 2/27/09 1:32 PM Page 487

B. Medical management
1. Drug therapy: Bronchodilators
a. Beta-adrenergic agonists
1) Metered dose inhaler (MDI)most
children will need spacers
2) Nebulizerinfants and toddlers
3) Rescue drugs for acute attacks
b. Corticosteroids
1) Inhaled by MDI or nebulizer
2) Oral for persistent wheezing
3) IV in hospital
c. Nonsteroid anti-inflammatory agents
1) Cromolyn sodium
2) Nedocromil
3) Leukotriene inhibitors and receptor-
antagonists
4) Used for maintenance, not rescue
d. Xanthine-derivatives
1) Theophylline (oral)
2) Aminophylline (IV)
3) Used for status asthmaticus
e. Procedure for use of oral inhaler.
See Figure 5-8.
2. Physical therapy
3. Hyposensitization
4. Exercise
C. Assessment findings
1. Family history of allergies
1. Attach metered dose inhaler canister to
2. Client history of eczema
3. Respiratory distress: shortness of breath, mouthpiece to spacer.
expiratory wheeze, prolonged expiratory 2. Shake to increase pressure in canister.
phase, air trapping (barrel chest if chronic), 3. Blow out air.
use of accessory muscles, irritability (from 4. Place mouthpiece in mouth and make a seal
hypoxia), diaphoresis, change in sensorium with lips.
if severe attack
5. Activate the canister.
4. Diagnostic tests: ABGs indicate respiratory
acidosis 6. Breathe in slowly to total lung capacity.
D. Nursing interventions 7. Hold breath for 510 seconds, then breathe
1. Place client in high-Fowlers position. normally. (For infants and small children, a
2. Administer oxygen as ordered. mask should be used and remain in place
3. Administer medications as ordered. until they have taken 56 breaths.)
4. Provide humidification/hydration to loosen 8. Wait 60 seconds.
secretions.
5. Provide chest percussion and postural 9. Repeat steps 27.
drainage when bronchodilation improves. 10. Rinse mouth and equipment following use
6. Monitor for respiratory distress. to prevent fungal infections.
7. Provide client teaching and discharge
planning concerning: Figure 5-8 Instructions for use of an oral inhaler
a. Modification of environment with spacer
1) Ensure room is well ventilated.
2) Stay indoors during grass cutting or
when pollen count is high.
Aspiration of a Foreign Object
3) Use damp dusting. A. General information
4) Avoid rugs, draperies or curtains, 1. Relatively common airway problem.
stuffed animals. 2. Severity depends on object (e.g., pins, coins,
5) Avoid natural fibers (wool and feathers). nuts, buttons, parts of toys) aspirated and the
b. Importance of moderate exercise degree of obstruction.
(swimming is excellent) 3. Depending on object aspirated, symptoms will
c. Purpose of breathing exercises (to increase increase over hours or weeks.

5
the end expiratory pressure of each 4. The curious toddler is most frequently
respiration) affected.

PEDIATRIC NURSING 487


53155_05_Ch05_p421-532.qxd 2/27/09 1:32 PM Page 488

5. If object does not pass trachea immediately, b. Respiratory tract: 99.9% of CF clients have
respiratory distress will be evident. respiratory involvement
6. If object moves beyond tracheal region, it will 1) Increased production of secretions
pass into one of the main stem bronchi; causes increased obstruction of airway,
symptoms will be vague, insidious. air trapping, and atelectasis
7. Causes 400 deaths per year in children under 2) Pulmonary congestion leads to cor
age 4. pulmonale
B. Medical management 3) Eventually death occurs by drowning
1. Objects in upper airway require immediate in own secretions
removal. c. Reproductive system
2. Lower airway obstruction is less urgent 1) Males are sterile
(bronchoscopy or laryngoscopy). 2) Females can conceive, but increased
C. Assessment findings mucus in vaginal tract makes
1. Sudden onset of coughing, dyspnea, wheezing, conception more difficult
stridor, apnea (upper airway) 3) Pregnancy causes increased stress on
2. Persistent or recurrent pneumonia, persistent respiratory system of mother
croupy cough or wheeze d. Liver: one third of clients have
3. Object not always visible on X-ray cirrhosis/portal hypertension
4. Secondary infection 9. 95% of deaths are from abnormal mucus
D. Nursing interventions secretion and fibrosis in the lungs; shortened
1. Perform Heimlich maneuver if indicated. life span
2. Reassure the scared toddler. B. Medical management
3. After removal, place child in high-humidity 1. Pancreatic involvement: aimed at promoting
environment and treat secondary infection absorption of nutrients
if applicable. a. Diet modification
4. Counsel parents regarding age-appropriate 1) Infant: predigested formula
behavior and safety precautions. 2) Older children: may require high-
calorie, high-protein, or low/limited-
fat diet, but many CF clients tolerate
Cystic Fibrosis (CF) normal diet
A. General information b. Pancreatic enzyme supplementation:
1. Disorder characterized by dysfunction of the enzyme capsules, tablets, or powders
exocrine glands (mucus-producing glands of (Pancrease, Cotazym, Viokase) given with
the respiratory tract, GI tract, pancreas, sweat meals and snacks
glands, salivary glands) 2. Respiratory involvement: goals are to maintain
2. Transmitted as an autosomal recessive trait airway patency and to prevent lung infection
3. Incidence: According to Cystic Fibrosis a. Chest physiotherapy
Foundation: 30,000 Americans, 3000 b. Antibiotics for infection
Canadians, and 20,000 Europeans. C. Assessment findings: symptoms vary greatly in
4. Most common lethal genetic disease among severity and extent
Caucasians in United States and Europe 1. Pancreatic involvement
5. Prenatal diagnosis of CF is not reliable a. Growth failure; failure to thrive
6. Secretions from mucous glands are thick, b. Stools are foul smelling, large, frequent,
causing obstruction and fibrosis of tissue foamy, fatty (steatorrhea), contain
7. Sweat and saliva have characteristic high undigested food
levels of sodium chloride c. Meconium ileus (meconium gets stuck in
8. Affected organs bowel due to lack of enzymes) in newborns
a. Pancreas: 85% of CF clients have d. Rectal prolapse is possible
pancreatic involvement e. Voracious appetite
1) Obstruction of pancreatic ducts and f. Characteristic protruding abdomen with
eventual fibrosis and atrophy of the atrophy of extremities and buttocks
pancreas leads to little or no release g. Symptoms associated with deficiencies in
of enzymes (lipase [fats], amylase the fat-soluble vitamins
[starch], and trypsin [protein]) h. Anemia
2) Absence of enzymes causes i. Diagnostic tests
malabsorption of fats and proteins 1) Trypsin decreased or absent in
3) Unabsorbed food fractions excreted in aspiration of duodenal contents
the stool produce steatorrhea 2) Fecal fat in stool specimen increased
4) Loss of nutrients and inability to 2. Respiratory involvement

5
absorb fat-soluble vitamins causes a. Signs of respiratory distress
failure to thrive b. Barrel chest due to air trapping

488 NCLEX-RN Review


53155_05_Ch05_p421-532.qxd 2/27/09 1:32 PM Page 489

c. Clubbing of digits b. Promotion of childs independence


d. Decreased exercise tolerance due to c. Avoidance of cigarette smoking in the
distress house
e. Frequent productive cough d. Availability of support groups/community
f. Frequent pseudomonas infections agencies
g. Diagnostic tests e. Alternative school education during
1) Chest X-ray reveals atelectasis, extended hospitalization/home recovery
infiltrations, emphysemic changes
2) Pulmonary function studies
abnormal
3) ABGs show respiratory acidosis Sample Questions
3. Electrolyte involvement
a. Hyponatremia/heat exhaustion in hot
weather 92. The nurse is caring for a child who had a
b. Salty taste to sweat tonsillectomy performed 4 hours ago. Which of
c. Diagnostic tests the following is an abnormal finding and a cause
1) Pilocarpine iontophoresis sweat test: for concern?
indicates 25 times normal amount of 1. An emesis of dried blood.
sodium and chloride in the sweat
2. Increased swallowing.
2) Fecal fat elevated
3) Fecal trypsin absent or decreased 3. Pink-tinged mucus.
D. Nursing interventions 4. The child complains of a very sore throat.
1. Pancreatic involvement
a. Administer pancreatic enzymes with 93. A 7-year-old has been diagnosed as having
meals as ordered: do not mix enzymes cystic fibrosis. Chest physiotherapy has been
until ready to use them; best to mix in ordered. When should chest percussion be
applesauce. performed?
b. Provide a high-calorie, high-carbohydrate 1. Before postural drainage.
(no empty-calorie foods), high-protein, 2. 12 hour before meals.
normal-fat diet.
c. Provide a double dose of multivitamins 3. Before an aerosol treatment.
per day, especially fat-soluble vitamins 4. After suctioning.
(A, D, E, K), in water-soluble form.
d. If low-fat diet required, MCT (medium- 94. The nurse is performing chest physiotherapy on
chain triglycerides) oil may be used. a 6-year-old child who has congestion in his left
2. Respiratory involvement lower lobe. In which position should the nurse
a. Administer antibiotics as ordered (all place the child?
antibiotics for pseudomonas are given IV; 1. Left side in semi-Fowlers position.
doses may be above recommended levels 2. Right side in semi-Fowlers position.
(for virulent organisms).
3. Left side in Trendelenburg position.
b. Administer expectorants, mucolytics
(rarely used) as ordered. 4. Right side in Trendelenburg position.
c. Avoid cough suppressants and
antihistamines. 95. An infant is being evaluated for possible cystic
d. Encourage breathing exercises. fibrosis. The sweat test will show an elevation of
e. Provide percussion and postural drainage which electrolyte?
4 times a day. 1. Chloride.
f. Provide aerosol treatments as needed; 2. Fluoride.
handheld nebulizers, mask, intermittent 3. Potassium.
positive pressure breathing (IPPB), mist
tent. 4. Calcium.
3. Electrolyte involvement
96. A 2-year-old is admitted to the hospital with
a. Add salt to all meals, especially in
cystic fibrosis. He is small for his age. What
summer.
dietary suggestions can the nurse recommend to
b. Give salty snacks (pretzels).
4. Provide appropriate long-term support to the childs mother to enhance his growth?
child and family. 1. Low-fat, low-residue, and high-potassium diet.
5. Provide client teaching and discharge 2. Low-carbohydrate, soft diet with no sugar
planning concerning: products.

5
a. Genetic counseling

PEDIATRIC NURSING 489


53155_05_Ch05_p421-532.qxd 2/27/09 1:32 PM Page 490

3. High-carbohydrate, high-fat diet with extra 101. A 12-month-old is hospitalized for a severe case
water between meals. of croup and has been placed in an oxygen tent.
4. High-protein, high-calorie meals with skim- Today the oxygen order has been reduced from
milk milkshakes between meals. 35% to 25%. His blood gases are normal. The
child refuses to stay in the oxygen tent. Attempts
97. The nurse is caring for a 2-year-old who has to placate him only cause him to become more
cystic fibrosis. His mother asks why the child upset. What would be an appropriate action for
developed cystic fibrosis. What explanation will the nurse to perform?
the nurse provide? 1. Restrain him in the tent and notify the
1. It develops due to meconium ileus at physician.
birth. 2. Take him out of the tent and notify the
2. It is an autosomal recessive genetic physician.
defect. 3. Take him out of the tent and let him sit in the
3. It occurs during embryologic development. playroom.
4. It results from chromosomal nondysjunction 4. Tell him it will please his mother if he stays
that occurred at conception. in the tent.

98. A 2-year-old is admitted to the hospital and 102. The nurse should recognize which of the
will need to stay for several days. The childs following respiratory findings as normal in a
mother is unable to stay overnight because 10-month-old infant?
there is no one to care for her other children. 1. Respiratory rate of 60 at rest.
What should the nurse recommend the 2. Use of accessory muscles to assist in
mother do? respiratory effort.
1. Leave something of hers with the child and 3. Respiratory rate of 32 at rest.
tell him shell be back in the morning.
4. Diaphoresis with shallow respirations.
2. Leave while he is in the playroom.
3. Leave after he has fallen asleep. 103. An 18-month-old presents with nasal flaring,
4. Tell him shell be back in a few minutes after intercostal and substernal retractions, and a
she has dinner. respiratory rate of 50. What is the most
appropriate nursing diagnosis?
99. The mother of a 2-year-old who has cystic 1. Knowledge deficit.
fibrosis tells the nurse that the family is 2. Ineffective breathing pattern.
planning their first summer vacation. She
3. Ineffective individual coping.
wants to know if there are any special
precautions needed because he has cystic 4. High risk for altered body temperature:
fibrosis. What condition will the nurse state hyperthermia.
that children with cystic fibrosis are
104. An 11-month-old is admitted to the hospital
particularly susceptible?
with bronchiolitis. He is currently in a croup
1. Severe sunburn. tent with supplemental oxygen. Which toy is
2. Infectious diarrhea. most appropriate for the nurse to recommend
3. Heat exhaystion. to the childs parents?
4. Respiratory allergies. 1. A stuffed animal made from a washable
fabric.
100. A 4-year-old is admitted to the hospital for the 2. A soft plastic stacking toy with multicolored
treatment of an acute asthma attack. She rings.
received nebulized albuterol (Proventil) in the
3. A set of wooden blocks.
emergency department and was transferred to
the pediatric unit with an aminophylline 4. A pull toy.
infusion. What significant finding will
105. Which of the following statements best assures
inform the nurse that the treatment is
the nurse that the parents understand the
effective?
safety concerns related to use of a vaporizer at
1. A decrease in mucus production. home?
2. A decrease in wheezing. 1. I have a high dresser in the bedroom on
3. An increase in blood pressure. which to place the vaporizer. The cord will

5
4. A sleeping child. be concealed behind the dresser.

490 NCLEX-RN Review


53155_05_Ch05_p421-532.qxd 2/27/09 1:32 PM Page 491

2. I plan to put the vaporizer on a stool next to mother suggests that she understands the care
the bed so that my child will get the most requirements?
benefit from the cool mist. 1. I plan to take her back to her play group
3. I purchased a warm mist vaporizer because tomorrow. I know she wont want to stay
I dont want my child to get chilled from the home.
mist in her face. 2. I have bought popsicles to give her later
4. I thought I could just set the vaporizer on today.
the floor next to the bed. 3. I will give her aspirin if she gets irritable.
4. She is just waiting for the ice cream we
106. A 4-year-old is experiencing an acute asthma
promised her before she came to the
attack. Why should the nurse avoid chest
hospital.
percussion with this child?
1. Chest percussion may lead to increased 110. A 3-year-old boy presents in the ER with
bronchospasm and more respiratory dysphagia, drooling, and respiratory difficulty
distress. that has increased significantly over the past
2. Chest percussion may cause mucous 6 hours. The nurse should know that these
plugging of the alveoli. findings are suggestive of which of the following
3. Chest percussion is useful in removing conditions?
airway secretions and should be used. 1. Croup.
4. Chest percussion will produce increased 2. Pneumonia.
coughing and thereby enhance respiratory 3. Bronchopulmonary dysplasia.
distress. 4. Epiglottitis.
107. A 5-month-old has severe nasal congestion. 111. A 2-year-old presents to an urgent care center
What is the best way for the nurse to clear his with respiratory distress and cyanosis. Parents
nasal passages? report an initial episode of choking. What is the
1. Administer saline nose drops and use a best initial action for the nurse to take?
bulb syringe to clear passages. 1. Call 911 and have parents wait for an
2. Ask him to blow his nose and keep tissues ambulance to transport the child to a
handy. pediatric hospital.
3. Place him in a mist tent with 40% oxygen. 2. Administer oxygen by face mask and call the
4. Administer vasoconstrictive nose drops childs pediatrician.
before meals and at bedtime. 3. Perform abdominal thrusts as described in
the Heimlich maneuver.
108. A 30-week gestation infant who had apnea of
4. Start CPR after the child loses consciousness.
prematurity is ready for discharge and will be
going home on apnea monitoring. What should
the nurse teach the parents for proper use of the
monitor?
1. The monitor is only used when the child is Answers and Rationales
awake. It is not indicated at night or during
naps.
2. The alarms on the monitor should be turned 92. 2. Increased swallowing could be a sign of
off when an attendant is with the infant. hemorrhage from the surgical site.
3. The monitor should be kept on at all times 93. 2. Chest percussion is done between meals to
except when the infant is being bathed. prevent vomiting, which might occur if done
Careful attention to skin integrity and following meals.
hygiene is important.
4. It is best for the parents to have 24-hour 94. 4. The affected lobe must be uppermost to be
home health supervision to watch the drained by gravity.
infant while monitoring is required.
95. 1. There is increased excretion of chloride in the
109. A 3-year-old underwent a tonsillectomy this sweat of children with cystic fibrosis. A chloride
morning. As the nurse giving discharge level of over 60 mEq/liter is diagnostic for the
instructions, which comment by the childs disease.

PEDIATRIC NURSING

5 491
53155_05_Ch05_p421-532.qxd 2/27/09 1:32 PM Page 492

96. 4. A person with cystic fibrosis lacks pancreatic 104. 2. Stacking toys with bright, large, colored
enzymes necessary for fat absorption. A diet high plastic rings provide age-appropriate activity
in protein and calories is necessary to meet the that is safe within the croup tent environment.
childs growth needs. Between-meal snacks, The large rings can be held or stacked. They can
milkshakes made with skim milk may be given to be wiped down if damp. The size of the objects
provide additional protein, vitamins, and calories. prevent them from creating any environmental
hazards if the child is not continuously
97. 2. Cystic fibrosis is an autosomal recessive genetic supervised.
disease. If both parents have the cystic fibrosis
trait, each child has a 25% chance of developing 105. 1. It is best to keep the vaporizer out of the
the disease, a 50% chance of being a carrier, and a childs way. Concealing the cord and placing
25% chance of not having the disease. the appliance on a high surface is preferable.

98. 1. Leaving something of his mothers with the 106. 1. During the course of an acute asthma attack,
child and telling him that she will be back in the bronchospasm is a significant problem. Chest
morning is the best approach in developing trust percussion can enhance the bronchospasm,
between the mother and her child. leading to more pronounced respiratory distress.

99. 3. Persons with cystic fibrosis are prone to 107. 1. Saline nose drops will help loosen secretions.
electrolyte imbalances due to increased loss of The bulb syringe is necessary because the
sodium and potassium in their sweat. The child is not old enough to effectively blow
mother should avoid having her child become his nose.
overheated and should frequently replenish
body fluids with water or fruit juices. 108. 3. Although apneic episodes are most common
during sleep, they can occur at other times. Initially,
100. 2. Aminophylline is a bronchodilator. As it it is particularly advisable to use the monitor
exerts its effects, wheezing will decrease. continually except when bathing the infant.

101. 2. The energy expended by the child in resisting 109. 2. Clear liquids are a good choice during the first
the oxygen tent is causing increased respiratory 24 hours after surgery. Popsicles are appealing to
effort. The child should be removed from the children while providing fluids. They are less
tent and closely monitored to be sure that he likely to irritate the surgical site than juices.
handles being in room air. The physician should
be notified because the oxygen content of room 110. 4. Epiglottitis is a medical emergency. The
air is only 20%, which is less than that ordered. drooling and dysphagia are most often
diagnostic of this condition.
102. 3. Rates of 2040 breaths per minute are normal
at this age. 111. 3. The reported episode of choking and the
childs condition suggest foreign body aspiration.
103. 2. The findings on assessment suggest The Heimlich maneuver should be attempted as
respiratory distress. Ineffective breathing pattern an initial action to remove the object.
is an appropriate diagnosis with the information
now available.

The Gastrointestinal System

VARIATIONS FROM THE ADULT 3. Peristalsis increased, faster emptying time,


more prone to diarrhea
A. Mechanical functions of digestion are immature at 4. Relaxed cardiac sphincter contributes to
birth. tendency to regurgitate food
1. No voluntary control over swallowing until 6 B. Liver functions (glyconeogenesis and storage of
weeks vitamins) are immature throughout infancy.
C. Production of mucosal-lining antibodies is decreased.

5
2. Stomach capacity decreased

492 NCLEX-RN Review


53155_05_Ch05_p421-532.qxd 2/27/09 1:32 PM Page 493

D. Gastric acidity is low in infants, slowly rises until D. Imbalanced nutrition: less than body requirements
age 10, and then increases again during E. Impaired oral mucous membrane
adolescence to reach adult levels. F. Risk for impaired skin integrity
E. Secretory cells are functional at birth, but efficiency G. Ineffective tissue perfusion
of enzymes impaired by lower gastric pH. H. Interrupted family processes
F. Infant has decreased saliva, which causes
decreased ability to digest starches.
G. Digestive processes are mature by toddlerhood. PLANNING AND
H. Completion of myelinization of spinal cord allows
voluntary control of elimination. IMPLEMENTATION
Goals
ASSESSMENT A. Child will maintain adequate nutritional intake.
B. Child will be free from complications of
History inadequate nutritional intake.
A. Presenting problem: symptoms may include: C. Pain will be relieved/controlled.
1. Vomiting: type, color, amount, relationship D. Child will reach optimal developmental level.
to eating or other events E. Parents will be able to care for child at home.
2. Abnormal bowel habits: diarrhea,
constipation, bleeding Interventions
3. Weight loss or growth failure
4. Pain: location; relationship to meals or other
events; effect on sleep, play, appetite Nasogastric Tube Feeding
5. Any other parental concerns
A. Provide continuous NG tube feedings when child
B. Diet/nutrition history: appetite, daily caloric
needs high-calorie intake.
intake, food intolerances, feeding schedule,
B. Use infusion pump to ensure sustained intake.
nutritional deficits
C. Check tube placement every 4 hours.
D. Check residuals and refeed every 4 hours.
Physical Examination
A. General appearance Gastrostomy
1. Plot height and weight on growth chart. A. Used for clients at high risk for aspiration.
2. Measure midarm circumference and tricep B. Regulate height of tube so feeding flows in over
skinfold thickness. 2030 minutes.
3. Observe color: jaundiced or pale.
B. Mouth
Parenteral Nutrition
1. Note level of dentition, presence of dental caries.
2. Observe mucosal integrity. A. Use central venous line for high dextrose solutions
C. Abdomen (greater than 10%).
1. Observe skin integrity. B. Check infusion rate and amount every 30 minutes.
2. Note abdominal distension or visible C. Monitor urine sugar and acetone every 4 hours for
peristaltic waves (seen in pyloric stenosis). 24 hours after a solution change, then every 8 hours.
3. Inspect for hernias (umbilical, inguinal). D. Monitor for signs of hyperglycemia (nausea,
4. Auscultate for bowel sounds (a sound every vomiting, dehydration).
1030 seconds is normal). E. Provide sterile care for insertion site.
5. Palpate for tenderness. 1. Change solution and tubing every 24 hours.
6. Palpate for liver (inferior edge normally 2. Change dressing every 13 days.
palpated 12 cm below right costal margin). 3. Apply restraints (if needed) to prevent
7. Palpate for spleen (may be felt on inspiration dislodgment of central line.
12 cm below left costal margin). F. Provide infants who are not receiving oral feedings
D. Vital signs: note presence of fever. with a pacifier to satisfy sucking needs.

ANALYSIS
EVALUATION
Nursing diagnoses for the child with a disorder of the
gastrointestinal system may include: A. Child is receiving adequate nourishment as
A. Constipation or diarrhea evidenced by normal growth and development.
B. Pain B. Skin is intact, free from signs of redness or

5
C. Risk for deficient fluid volume inflammation.

PEDIATRIC NURSING 493


53155_05_Ch05_p421-532.qxd 2/27/09 1:32 PM Page 494

C. Child is free from infection, diarrhea, or vomiting. C. Assessment findings


D. Child is free from pain. 1. Facial abnormality visible at birth: cleft lip or
1. Relaxed facial expression palate or both, unilateral or bilateral, partial or
2. Level of activity complete
3. No guarding of abdomen 2. Difficulty sucking, inability to form airtight
E. Parents participate in care of child. seal around nipple (size of defect may
F. Child participates in normal daily activities with preclude breastfeeding)
family and peers. 3. Formula/milk escapes through nose in infants
with cleft palate
4. Predisposition to infection because of free
DISORDERS OF THE communication between mouth and nose
5. Possible difficulty swallowing
GASTROINTESTINAL SYSTEM 6. Abdominal distension due to swallowed air
D. Nursing interventions: preoperative cleft lip repair
Congenital Disorders 1. Feed in upright position to decrease chance of
aspiration and decrease amount of air
Cleft Lip and Palate swallowed.
A. General information 2. Burp frequently; increased swallowed air
1. Nonunion of the tissue and bone of the upper causes abdominal distension and discomfort.
lip and hard/soft palate during embryologic 3. Use a large-holed nipple; press cleft lip
development together with fingers to encourage sucking and
2. Familial disorder, often associated with other to strengthen muscles needed later for speech.
congenital abnormalities; incidence higher in 4. If infant unable to suck, use a rubber-tipped
Caucasians syringe and drip formula into side of mouth.
a. Cleft lip or palate: 1.5 in 1000 births 5. Administer gavage feeding as ordered if
b. Cleft lip with or without cleft palate affects necessary.
more boys; cleft palate affects more girls 6. Finish feeding with water to wash away
3. With cleft palate, the failure of the bone and formula in palate area.
tissues to fuse results in a communication 7. Provide small, frequent feedings.
between the mouth and nose 8. Provide emotional support for parents/family.
B. Medical management: team approach for therapy a. Demonstrate benefits of surgery by
1. Speech therapist showing before and after pictures.
2. Dentist and orthodontist b. Reinforce that disorder is not their fault
3. Audiologist, otolaryngologist, pediatrician and that it will not affect childs life span
(these children are prone to otitis media and or mental ability.
possible hearing loss) E. Nursing care: postoperative cleft lip repair
4. Surgical correction 1. Maintain patent airway (child may appear to
a. Timing varies with severity of defect; early have respiratory distress because of closure of
correction helps to avoid speech defects. previously open space; adaptation occurs
b. Cheiloplasty: correction of cleft lip quickly).
1) Goal is to unite edges to allow lips to 2. Assess color; monitor amount of swallowing to
be both functional and cosmetically detect hemorrhage.
attractive. 3. Do not place in prone position or with
2) Usually performed approximately age pressure on cheeks; avoid any pressure or
2 months (to prepare gums for tension on suture line.
eruption of teeth) when child is free 4. Avoid straining on suture line by anticipating
from respiratory infection. childs needs.
3) Steri-Strips or Logan bow usually used a. Prevent crying.
to take tension off suture line. b. Keep child comfortable and content.
c. Cleft palate repair is usually not done until 5. Use elbow restraints or pin sleeves of shirt to
age 18 months in anticipation of speech diaper to keep childs hands away from suture
development. line.
1) Between lip and palate repair child is 6. Resume feedings as ordered.
maintained on normal nutritional and 7. Keep suture line clean; clean after each
respiratory status; also maintains feeding with saline, peroxide, or water to
normal immunization schedule. remove crusts and prevent scarring.
2) Child should be weaned and able to 8. Provide pain control/relief.
take liquids from a cup before palate F. Nursing interventions: preoperative cleft palate
repair. repair

5
1. Prepare parents to care for child after surgery.

494 NCLEX-RN Review


53155_05_Ch05_p421-532.qxd 2/27/09 1:32 PM Page 495

2. Instruct concerning feeding methods and B. Medical management


positioning. 1. Drug therapy: antibiotics for respiratory
G. Nursing interventions: postoperative cleft palate infections
repair 2. Surgery
1. Position on side for drainage of blood/mucus. a. Palliative
2. Have suction available but use only in 1) Gastrostomy for placement of a feeding
emergency. tube
3. Prevent injury or trauma to suture line. 2) Esophagostomy to drain secretions
a. Use cups only for liquids; no bottles. b. Corrective
b. Avoid straws, utensils, Popsicle sticks, 1) End-to-end anastomosis to correct the
chewing gum. defect and restore normal anatomy
c. Provide soft toys. 2) Colon transplant for defects where
d. Use elbow restraints. there is insufficient tissue for an end-
e. Provide liquid diet initially, then progress to-end anastomosis
to soft before returning to normal. C. Assessment findings
f. Give water after each feeding to clean 1. Esophageal atresia
suture line. a. History of polyhydramnios in mother (from
g. Hold and cuddle these babies to help infants inability to swallow and excrete
distract them. amniotic fluid)
b. Inability to pass an NG tube
Altered Connections between Trachea, c. Increased drooling and salivation
Esophagus, and Stomach d. Immediate regurgitation of undigested
formula/milk when fed
A. General information e. Intermittent cyanosis from choking on
1. Types (Figure 5-9) aspirated secretions
a. Esophageal atresia: esophagus ends in a 2. TEF
blind pouch; no entry route to stomach a. Normal swallowing but some food/mucus
b. Tracheoesophageal fistula (TEF): open crosses fistula, causing choking and
connection between trachea and esophagus intermittent cyanosis
c. Esophageal atresia with TEF: esophagus b. Distended abdomen from inhaled air
ends in a blind pouch; stomach end of crossing fistula into stomach
esophagus connects with trachea c. Aspiration pneumonia from reflux of
2. These deformities are found more often in gastric secretions into the trachea
low-birth-weight or premature infants, and are
associated with polyhydramnios in the mother
and multiple congenital anomalies.

A B C D E

Figure 5-9 Types of esophageal atresia and tracheoesophageal fistula. (A) Esophageal atresia with distal
tracheoesophageal fistula; (B) isolated or pure esophageal atresia; (C) tracheoesophageal fistula without
esophageal atresia; (D) esophageal atresia with proximal tracheoesophageal fistula; and (E) esophageal atresia
with proximal and distal tracheoesophageal fistula.

PEDIATRIC NURSING

5 495
53155_05_Ch05_p421-532.qxd 2/27/09 1:32 PM Page 496

3. Esophageal atresia with TEF 3. Provide client teaching and discharge planning:
a. All findings for esophageal atresia teach parents how to position and feed infant.
b. Abdominal distension and aspiration
pneumonia from gas and reflux of gastric Pyloric Stenosis
acids into trachea
4. Diagnostic tests: fluoroscopy with contrast A. General information
material reveals type of defect 1. Hypertrophy (thickening) of the pyloric
D. Nursing interventions: preoperative sphincter causing stenosis and obstruction
1. Maintain patent airway. 2. Incidence: 5 in 1000 births; more common in
a. Position according to type of defect Caucasian, firstborn, full-term boys
(usually 30 head elevation). 3. Cause unknown; possibly familial
b. Provide continuous or prn nasal suctioning. B. Medical management
2. Keep NPO. 1. Correction of fluid electrolyte abnormalities
3. Administer IV fluids as ordered. 2. Surgery: pyloromyotomy (Fredet-Ramstedt
E. Nursing interventions: postoperative procedure)
1. Provide nutrition. C. Assessment findings
a. Provide gastrostomy tube feedings until the 1. Olive-size bulge under right rib cage
anastomosis site has healed. 2. Vomiting
b. Start oral feedings when infant can a. As obstruction increases, vomiting
swallow well. becomes more forceful and projectile.
c. Progress from glucose water to small, b. Vomitus does not contain bile (bile duct is
frequent formula feedings. distal to the pylorus).
2. Promote respiratory function. 3. Peristaltic waves during and after feeding
a. Position properly. (look like rolling balls under abdominal wall)
b. Suction as needed. 4. Failure to thrive, even though infant appears
c. Provide chest tube care. hungry after vomiting
3. Provide client teaching and discharge 5. Dehydration: sunken fontanels, poor skin
planning concerning: turgor, decreased urinary output
a. Alternative feeding methods 6. Diagnostic tests
b. Signs of respiratory distress and suctioning a. Upper GI series reveals narrowing of the
technique diameter of the pylorus
b. Sodium, potassium, chloride decreased
Gastroesophageal Reflux (Chalasia) c. HCT increased
d. Metabolic alkalosis
A. General information D. Nursing interventions: preoperative
1. Reversal of flow of stomach contents into 1. Administer replacement fluids and
lower portion of esophagus electrolytes as ordered.
2. More common in premature infants due to 2. Prevent vomiting.
hypotonia a. May be NPO with NG tube to suction.
3. Caused by relaxed cardiac sphincter or b. Keep in high-Fowlers position.
overdistension of stomach by gas or c. Place on right side after feedings.
overfeeding d. Minimize handling.
4. Results in local irritation of the lining of the e. Record strict I&O, daily weights, and urine
esophagus from backflow of acidic gastric specific gravity.
contents; sometimes causes aspiration 3. Observe for symptoms of aspiration of vomitus.
pneumonia E. Nursing interventions: postoperative
B. Assessment findings 1. Advance diet as tolerated.
1. Irritability 2. Place on right side after feeding. Elevate head.
2. Spitting up (versus vomiting or projectile 3. Monitor strict I&O, daily weights.
vomiting); note relationship to feedings 4. Observe incision for signs of infection.
3. Diagnostic tests 5. Provide client teaching and discharge
a. Muscle tone of cardiac sphincter reduced planning concerning feeding and positioning
b. Esophageal pH: contents acidic of infant.
c. Fluoroscopy: presence of refluxed contrast
material not quickly cleared or repeated Intussusception
reflux
C. Nursing interventions A. General information
1. Position with head elevated 3045. 1. Telescoping of bowel into itself (usually at the
2. Give small, frequent feedings with adequate ileocecal valve) causing edema, obstruction,

5
burping. and possible necrosis of the bowel

496 NCLEX-RN Review


53155_05_Ch05_p421-532.qxd 2/27/09 1:32 PM Page 497

2. Most common at about age 6 months; occurs 9. Diagnostic tests: rectal biopsy confirms
more often in boys than in girls; associated presence of aganglionic cells
with cystic fibrosis and celiac disease D. Nursing interventions
3. Cause unknown 1. Administer enemas as ordered.
B. Medical management a. Use mineral oil or isotonic saline.
1. Barium or contrast medium enema to reduce b. Do not use tap water or soap suds enemas
telescoping by hydrostatic pressure in infants because of danger of water
2. Surgery if barium enema unsuccessful or if intoxication.
signs of peritonitis c. Use volume appropriate to weight of child.
C. Assessment findings 1) Infants: 150200 mL
1. Piercing cry 2) Children: 250500 mL
2. Severe abdominal pain (pulls legs up) 2. Do not treat the loose stools; the child actually
3. Vomiting of bile-stained fluid is constipated.
4. Bloody mucus in stool 3. Administer TPN as ordered.
5. Currant-jelly stool 4. Provide a low-residue diet.
D. Nursing interventions 5. Provide client teaching and discharge
1. Provide routine pre- and post-op care for planning concerning colostomy care and low-
abdominal surgery. residue diet.
2. Monitor for fluid and electrolyte imbalance
and intervene as needed. Imperforate Anus
3. Monitor for peritonitis and intervene as needed.
4. Monitor stools. Report changes. A. General information
1. Congenital malformation caused by abnormal
Hirschsprungs Disease fetal development
2. Many variations; anal agenesis most frequent
(Aganglionic Megacolon)
3. Often associated with fistula formation to
A. General information rectum or vagina and other congenital
1. Absence of autonomic parasympathetic anomalies
ganglion cells in a portion of the large colon 4. Surgical correction performed in stages with
(usually occurs 425 cm proximally from anus), completion at about age 1 year
resulting in decreased motility in that portion 5. May need temporary colostomy
of the colon and signs of functional obstruction B. Medical management
2. Usually diagnosed in infancy 1. Manual dilatation
3. Familial disease; more common in boys than 2. Surgery: anoplasty (reconstruction of anus)
girls; associated with Down syndrome 3. Prophylactic antibiotics
4. When stool enters the affected part of the C. Assessment findings
colon, lack of peristalsis causes it to remain 1. No stool passage within 24 hours of birth
there until additional stool pushes it through; 2. Meconium stool from inappropriate orifice
colon dilates as stool is impacted. 3. Inability to insert thermometer
B. Medical management D. Nursing interventions
1. Drug therapy: stool softeners 1. If suspected, do not take rectal temperature
2. Isotonic enemas because of risk of perforating wall and causing
3. Diet therapy: low residue peritonitis.
4. Surgery 2. Perform manual dilatation as ordered; instruct
a. Palliative: loop or double-barrel colostomy parents in proper technique.
b. Corrective: abdominal-perineal pull 3. After surgery prevent infection; keep anal
through; bowel containing ganglia is pulled incisional area as clean as possible.
down and anastomosed to the rectum. 4. After surgery use side-lying position, or have
C. Assessment findings child lie prone with hips elevated.
1. Failure or delay in passing meconium
2. Abdominal distension; failure to pass stool Acquired Gastrointestinal Disorders
3. Temporary relief following digital rectal exam
4. Loose stools; only liquid can get around Celiac Disease
impaction (may also be a ribbonlike stool)
5. Nausea, anorexia, lethargy A. General information
6. Possibly bile-stained or fecal vomiting 1. Malabsorption syndrome characterized by
7. Loss of weight, failure to grow intolerance of gluten, found in rye, oats,
8. Volvulus (bowel twists upon itself, causing wheat, and barley
obstruction and necrosis) and enterocolitis due 2. Familial disease, found more commonly in
Caucasians

5
to fecal stagnation

PEDIATRIC NURSING 497


53155_05_Ch05_p421-532.qxd 2/27/09 1:32 PM Page 498

3. Cause unknown; thought to be an inborn error 4. Decreased bowel sounds


of metabolism or an immunologic disorder 5. Fever
4. Characterized by flat mucosal surface and 6. Diagnostic tests
atrophy of villi of the intestine; reduced a. WBC increased
absorptive surface causes marked b. Elevated acetone in urine
malabsorption of fats C. Nursing interventions
B. Medical management: diet therapy main 1. Administer antibiotics/antipyretics as ordered
intervention; gluten-free diet, TPN in children who 2. Prevent perforation of the appendix; do not
are severely malnourished give enemas or cathartics or use heating pad
C. Assessment findings 3. In addition to routine pre-op care for
1. Steatorrhea: frothy, pale, bulky, foul-smelling, appendectomy:
greasy stools a. Give support to parents if seeking
2. Chronic diarrhea during late infancy and treatment was delayed.
throughout toddlerhood b. Explain necessity of obtaining lab work
3. Failure to thrive prior to surgery.
4. Grossly distended abdomen; muscle wasting of 4. In addition to routine post-op care:
limbs and buttocks a. Monitor NG tube (usually with low
5. Abdominal pain, irritability, listlessness, suction).
vomiting b. Monitor Penrose drains.
6. Symptoms of vitamin A, D, E, and K c. Position in semi-Fowlers or lying on right
deficiency side to facilitate drainage.
7. Diagnostic tests d. Administer antibiotics as ordered.
a. Pancreatic enzymes and sweat chloride test
normal (performed to rule out the Parasitic Worms
possibility of cystic fibrosis)
b. Jejunal and duodenal biopsies show A. General information
characteristic atrophy of the mucosa 1. A parasite is an organism that lives in, on, or
D. Nursing interventions at the expense of the host.
1. Monitor gluten-free diet (no wheat, barley, 2. Common human GI parasites are pinworms
oats, and rye products) and roundworms.
2. Provide supplemental fat-soluble vitamins in 3. Medication varies depending on type of
water-soluble form parasite.
3. Provide client teaching and discharge B. Assessment findings
planning concerning: 1. Pinworms: anal irritation, itching, disturbed
a. Gluten-free diet; stress allowed foods and sleep
importance of reading labels carefully 2. Roundworm: colic, abdominal pain, lack of
b. Avoidance of infection appetite, weight loss
c. Assisting child to feel like a normal peer C. Nursing interventions
d. Importance of adhering to diet even though 1. Obtain stool culture.
symptoms are controlled 2. Observe for worms in all excreta (Scotch tape
e. Importance of long-term follow-up test for stool).
management 3. Instruct parents to change clothing, bed linens,
towels and launder in hot water.
Appendicitis 4. Clean toilets with disinfectant.
5. Instruct all family members to scrub hands
A. General information and fingernails prior to eating and after using
1. Inflammation of the appendix that prevents toilet.
mucus from passing into the cecum; if 6. Follow specific medication and hygiene orders
untreated, ischemia, gangrene, rupture, and given by physician.
peritonitis occur
2. Most common in school-age children Giardiasis
3. May be caused by mechanical obstruction
(fecaliths, intestinal parasites) or anatomic A. General information
defect; may be related to decreased fiber in 1. Common cause of diarrhea
the diet 2. Protozoan Giardia lamblia
B. Assessment findings 3. Common in daycare centers
1. Diffuse pain, localizes in lower right quadrant 4. Cysts ingested, mature in GI tract, cysts
2. Nausea/vomiting excreted in stools, and complete maturation
3. Guarding of abdomen, rebound tenderness, 5. Multiple stool cultures required as all stools

5
walks stooped over dont contain cysts

498 NCLEX-RN Review


53155_05_Ch05_p421-532.qxd 2/27/09 1:32 PM Page 499

6. Usually fecal-oral transmission, also 113. An 8-year-old has celiac disease. She had an
contaminated water and animals emergency appendectomy. She is progressing
B. Assessment findings well and is having her first real meal. Which
1. Diarrhea food should the nurse remove from her tray?
2. Vomiting, anorexia 1. Chicken rice soup.
3. Failure to thrive
4. Abdominal cramps 2. Crackers.
C. Medical management 3. Hamburger patty.
1. Metronidazole (Flagyl) 4. Fresh fruit cup.
2. Furazolidone (Furoxone)
D. Nursing interventions 114. A 10-month-old is brought to the clinic for a
1. Hygiene, especially with diaper changes checkup and his MMR immunization. While
2. Handwashing talking to the nurse, the mother reports that her
3. Instructions about drug therapy teenage babysitter has just come down with
rubeola. What is the most appropriate plan of
Constipation treatment for the child?
1. Administer immune serum globulin.
A. General information
1. Decrease in number of bowel movements with 2. Administer prophylactic penicillin.
large, hard stools 3. Vaccinate him now with MMR.
2. May be caused by high fat and protein and low 4. Allow him to catch measles from the
fluid in diet babysitter in order to develop active
3. May cause bowel obstruction if severe immunity.
B. Medical management
1. Drug therapy: stool softeners, suppositories, 115. The nurse is caring for a 12-month-old child
enemas who has a cleft palate. A cleft lip was repaired
2. Diet therapy: increased fluids and fiber when he was 2 months old. His mother asks the
C. Assessment findings nurse when he will be ready for a cleft palate
1. Less frequent stools, difficulty eliminating stool, repair. What response would best inform the
hard consistency compared to normal pattern parent when the cleft palate repair can be
(children do not have to stool every day)
performed?
2. Bleeding with stooling
3. Abdominal pain 1. Prior to development of speech.
D. Nursing interventions 2. When the child is toilet trained.
1. Assess for other pathologic causes of 3. When the child is completely weaned from
constipation. the bottle and pacifier.
2. Dietary modification, increase fiber and fluids. 4. When a large-holed nipple is ineffective for
3. Apply lubricant around anus.
his feedings.
4. Remove stool digitally if possible.
5. Provide prune juice (1 oz); add fruits to diet. 116. A 2-year-old has had a cleft palate repair. Which
6. Add small amount of Karo syrup to formula. priority teaching fact will be included when
7. Teach parents methods to prevent further educating the mother about the post-op period?
episodes.
1. Resume toilet training after he is up and
around.
2. Use a cup or wide bowl spoon for feeding.
Sample Questions 3. He will be more prone to respiratory
infections now that his airway is smaller.
4. No further treatment will be needed until his
112. A 9-year-old has celiac disease, which has been
adult teeth come in at age 6.
in good control since it was diagnosed 6 years
ago. She has now been admitted to the hospital 117. What is the appropriate feeding technique for the
for an emergency appendectomy. Which nurse to use with an infant who has a cleft palate?
preoperative procedure should the nurse
1. Suction client prior to feeding.
withhold?
2. Feed in sitting position.
1. A cleansing enema.
3. Have the nurse feed the client during
2. Starting an IV.
hospitalization.
3. Keeping her NPO.
4. Burp client after feeding to reduce risk

5
4. Obtaining a blood sample for a CBC. of aspiration.

PEDIATRIC NURSING 499


53155_05_Ch05_p421-532.qxd 2/27/09 1:32 PM Page 500

118. How would you evaluate that the new nurse is 3. Yes; you can use tap water after letting it run
using appropriate technique to feed a 3-day-old for one minute to clear any lead from the
with a cleft lip? pipes.
1. NG tube is patent. 4. No; tap water enemas are not allowed, but
2. Infant is seated in upright position. soap suds enemas are just as effective.
3. The nurse uses a Nuk nipple.
124. A 5-year-old boy has celiac disease. Which
4. The nurse adds rice to formula. statement by the child informs the nurse that he
is following his diet?
119. A baby girl is born prematurely to a mother with
polyhydramnios. The baby is diagnosed with 1. I had hot dogs and french fries for lunch.
esophageal atresia with tracheoesophageal 2. I ate chicken and vegetables for dinner.
fistula. What assessment finding would the 3. I had macaroni and cheese for lunch.
nurse be likely to note? 4. I ate soup and crackers for dinner.
1. Jaundice, high bilirubin.
2. Seedy yellow stools. 125. A 14-year-old is admitted to your unit following
an emergency appendectomy. What is the
3. Projectile emesis.
nurses goal for this client?
4. Frothy saliva, drooling.
1. Pain related to inflamed appendix.
120. A 5-month-old girl is admitted with 2. Patient will experience minimized risk of
gastroesophageal reflux. Her signs and symptoms spread of infection.
include emesis, poor weight gain, hemepositive 3. Maintain NG tube decompression until bowel
stools, irritability, and gagging with feeds. The motility returns.
nurse would include which intervention? 4. Child demonstrates resolution of peritonitis.
1. Urine dipstick each void.
2. Appropriate feeding positioning. 126. A 9-year-old girl comes into the clinic with a
diagnosis of pinworms. What is it essential for
3. Biweekly weights.
the nurse to teach?
4. Monitor WBC as indicator for infection.
1. Check for pinworms every morning for a
121. A 4-week-old is admitted for observation. Her week with a Scotch tape test.
assessment reveals projectile vomiting, visible 2. Save the girls next bowel movement to check
gastric peristalsis, and an olive-shaped mass in for pinworms.
the epigastrium. Which nursing diagnosis is of 3. Follow-up with local doctor in 6 months to
highest best priority? check for recurrence.
1. Altered nutrition. 4. Scrub hands and fingernails thoroughly before
2. Self-care deficit. each meal and after each use of the toilet.
3. Impaired gas exchange.
4. Fluid volume deficit.

122. The nurse would find which stool


characteristic consistent with a diagnosis of
Answers and Rationales
intussusception?
1. Yellow seedy stools. 112. 1. Enemas, cathartics, and heat to the abdomen
2. Currant jelly-like stools. should all be avoided in appendicitis because
3. Mucus-like stools. they may cause perforation of the appendix.
4. Hard black stools. 113. 2. The prescribed diet for children with celiac
disease is gluten free. Crackers contain gluten.
123. A 6-month-old boy is treated at home with saline
enemas due to his Hirschsprungs disease. His 114. 1. Administration of immune serum globulin
mother asks if she can use tap water to reduce will provide the child with passive immunity to
costs. Which is the best response by the nurse? prevent a full-blown case of measles or reduce
1. Yes, tap water is as effective as saline, just the severity of symptoms.
be sure to boil it first.
2. No, saline enemas must be used to maintain 115. 1. Cleft palate repair should be done before
his electrolyte balance. speech is well developed. This allows for the

5
formation of a more normal speech pattern.

500 NCLEX-RN Review


53155_05_Ch05_p421-532.qxd 2/27/09 1:32 PM Page 501

116. 2. Care must be taken not to put anything in the 122. 2. The obstruction causes bloody mucus known
mouth that could damage the suture line. as currant jelly stools.

117. 2. This position reduces the risk of aspiration. 123. 2. Repeated water enemas cause electrolyte
dilution.
118. 2. This position reduces the risk of aspiration.
124. 2. Chicken and vegetables do not contain gluten.
119. 4. Infants with esophageal atresia (EA) with Gluten is in barley, rye, oats, and wheat.
tracheoesophageal fistula (TEF) have difficulty
handling their secretions. 125. 2. This is an appropriate goal.

120. 2. It may be a challenge to find the optimum 126. 4. Handwashing prevents reinfection and/or new
position. Best positions include upright prone infections in other people.
and 30 head of bed elevation.

121. 4. Infants with pyloric stenosis are at high risk


for electrolyte imbalance and these need to be
corrected prior to a pyloromyotomy.

The Genitourinary System

VARIATIONS FROM THE ADULT Physical Examination


A. Nephrons continue to develop after birth. A. General appearance: note presence of edema.
B. Glomerular filtration rate is 30% below adult B. Abdomen and genitalia: note abdominal
levels at birth; reaches normal level by age distension, presence of undescended testicle,
2 years. tenderness to palpation, placement of urinary
C. Tubular functions immature at birth; tubular meatus, urinary stream during voiding
absorption and secretion reach adult levels by C. Vital signs: note presence of fever; increased blood
age 2 years. pressure (common in renal disease)
D. Urethra is shorter in children and more prone to
ascending infection (particularly true in girls); the
urethra is also closer to anus as source of ANALYSIS
contamination.
E. Many GU conditions in children become Nursing diagnoses for the child with a disorder of the
chronic. genitourinary tract may include:
A. Excess fluid volume
B. Impaired urinary elimination
ASSESSMENT C. Pain
D. Activity intolerance
E. Interrupted family process
History
A. Presenting problem: symptoms may include:
1. Change in appearance, color, or smell of PLANNING AND IMPLEMENTATION
urine
2. Change in amount, frequency, or pattern of Goals
urination
3. Abdominal or back pain A. Child will have normal urinary function.
4. Anorexia, nausea, vomiting, weight loss B. Childs fluid and electrolyte and acid-base
5. Headaches, seizures balances will be normal.
6. Fatigue, lethargy C. Child will be free from signs of infection.
7. Excessive thirst D. Childs blood pressure will be within normal
8. Drug use or accidental ingestions limits.

5
B. Family history: kidney disease, hypertension E. Parents will be able to care for child at home.

PEDIATRIC NURSING 501


53155_05_Ch05_p421-532.qxd 2/27/09 1:32 PM Page 502

Intervention c. Importance for girls to wipe perineum from


front to back
Pediatric Urine Collector (PUC) d. Increase in foods/fluids that acidify urine
A. Used when child is not toilet trained
B. Nursing care Vesicoureteral Reflux
1. Wash genitalia as for clean catch specimens.
A. General information
2. Apply bag directly to dry skin; do not use
1. Regurgitation of urine from the bladder via the
powder or creams.
ureters to the kidneys due to faulty valve
3. If child has not voided within 45 minutes,
mechanism at the vesicoureteral junction
remove bag and repeat process.
2. Predisposes child to:
a. UTIs from urine stasis
b. Pyelonephritis from chronic UTIs
EVALUATION c. Hydronephrosis from increased pressure
on renal pelvis
A. Child is adequately hydrated as evidenced by
B. Assessment findings: same as for urinary tract
normal serum electrolyte levels and normal urine
infections
output.
C. Nursing interventions for surgical reimplantation
B. Child is free from complications such as infection,
of ureters
skin breakdown, or hypertension.
1. Assist with preoperative studies as needed
C. Parents demonstrate ability to administer
(IVP, voiding cystourethrogram, cystoscopy).
appropriate medications and treatments.
2. Provide postoperative care.
a. Monitor drains; may have one from
DISORDERS OF THE bladder and one from each ureter (ureteral
stents).
GENITOURINARY SYSTEM b. Check output from all drains (expect
Urinary Tract Infection (UTI) bloody drainage initially) and record
carefully.
A. General information c. Observe drainage from abdominal dressing;
1. Bacterial invasion of the kidneys or bladder note color, amount, frequency.
2. More common in girls, preschool, and school- d. Administer medication for bladder spasms
age children as ordered.
3. Usually caused by E. coli; predisposing factors
include poor hygiene, irritation from bubble
baths, urinary reflux Exstrophy of the Bladder
4. The invading organism ascends the urinary A. General information
tract, irritating the mucosa and causing 1. Congenital malformation in which nonfusion
characteristic symptoms. of abdominal and anterior walls of the bladder
B. Assessment findings during embryologic development causes
1. Low-grade fever anterior surface of bladder to lie open on
2. Abdominal pain abdominal wall
3. Enuresis, pain/burning on urination, 2. Varying degrees of defect
frequency, hematuria B. Assessment findings
C. Nursing interventions 1. Associated structural changes
1. Administer antibiotics as ordered; prevention a. Prolapsed rectum
of kidney infection/glomerulonephritis b. Inguinal hernia
important. (Note: obtain cultures before c. Widely split symphysis
starting antibiotics.) d. Rotated hips
2. Provide warm baths and allow child to void in 2. Associated anomalies
water to alleviate painful voiding. a. Epispadias
3. Force fluids. b. Cleft scrotum or clitoris
4. Encourage measures to acidify urine c. Undescended testicles
(cranberry juice, acid-ash diet). d. Chordee (downward deflection of the penis)
5. Provide client teaching and discharge C. Medical management: two-stage reconstructive
planning concerning: surgery, possibly with urinary diversion; usually
a. Avoidance of tub baths (contamination delayed until age 36 months
from dirty water may allow D. Nursing interventions: preoperative
microorganisms to travel up urethra) 1. Provide bladder care; prevent infection.
b. Avoidance of bubble baths that might a. Keep area as clean as possible; urine on

5
irritate urethra skin will cause irritation and ulceration.

502 NCLEX-RN Review


53155_05_Ch05_p421-532.qxd 2/27/09 1:32 PM Page 503

b. Change diaper frequently; keep diaper 3. Surgery performed at age 39 months; 2 years
loose fitting. of age for complex repairs
c. Wash with mild soap and water. C. Assessment findings
d. Cover exposed bladder with Vaseline 1. Urinary meatus misplaced
gauze. 2. Inability to make straight stream of urine
E. Nursing interventions: postoperative D. Nursing interventions
1. Design play activities to foster toddlers need 1. Diaper normally.
for autonomy (e.g., Play-Doh, talking toys, 2. Provide support for parents.
books); child will be immobilized for extended 3. Provide support for child at time of surgery.
period of time. 4. Postoperatively check pressure dressing,
2. Prevent trauma; as child gets older and more monitor catheter drainage, assess pain.
mobile, trauma more likely; teach parents to
avoid areas such as sandboxes.
Enuresis
A. General information
Undescended Testicles (Cryptorchidism) 1. Involuntary passage of urine after the age of
A. General information control is expected (about 4 years)
1. Unilateral or bilateral absence of testes in 2. Types
scrotal sac a. Primary: in children who have never
2. Testes normally descend at 8 months of achieved control
gestation, will therefore be absent in b. Secondary: in children who have
premature infants developed complete control and lose it
3. Incidence increased in children having 3. May occur at any time of day but is most
genetically transmitted diseases frequent at night
4. Unilateral cryptorchidism most common 4. More common in boys
5. 75% will descend spontaneously by age 1 year 5. No organic cause can be identified; familial
B. Medical management tendency
1. Whether or not to treat is still controversial; if 6. Etiologic possibilities
testes remain in abdomen, damage to the testes a. Sleep disturbances
(sterility) is possible because of increased body b. Delayed neurologic development
temperature. c. Immature development of bladder leading
2. If not descended by age 8 or 9, chorionic to decreased capacity
gonadotropin can be given. d. Psychologic problems
3. Orchipexy: surgical procedure to retrieve and B. Medical management
secure testes placement; performed between 1. Bladder retention exercises
ages 13 years. 2. Behavior modification, e.g., bed alarm devices
C. Assessment findings: unable to palpate testes in 3. Drug therapy: results are temporary; side
scrotal sac (when palpating testes be careful not to effects may be unpleasant or even dangerous
elicit cremasteric reflex, which pulls testes higher a. Tricyclic antidepressants: imipramine HCI
in pelvic cavity) (Tofranil)
D. Nursing interventions b. Anticholinergics
1. Advise parents of absence of testes and c. DDAVP
provide information about treatment options. C. Assessment findings
2. Support parents if surgery is to be performed. 1. Physical exam normal
3. Post-op, avoid disturbing the tension 2. History of repeated involuntary urination
mechanism (will be in place for about 1 week). D. Nursing interventions
4. Avoid contamination of incision. 1. Provide information/counseling to family as
needed.
a. Confirm that this is not conscious behavior
Hypospadias and that child is not purposely
A. General information misbehaving.
1. Urethral opening located anywhere along the b. Assure parents that they are not
ventral surface of penis responsible and that this is a relatively
2. Chordee (ventral curvature of the penis) often common problem.
associated, causing constriction 2. Involve child in care; give praise and support
3. In extreme cases, childs sex may be uncertain with small accomplishments.
B. Medical management a. Age 56 years: can strip bed of wet sheets.
1. Minimal defects need no intervention b. Age 1012 years: can do laundry and
2. Neonatal circumcision delayed, tissue may be change bed.

5
needed for corrective repair 3. Avoid scolding and belittling child.

PEDIATRIC NURSING 503


53155_05_Ch05_p421-532.qxd 2/27/09 1:32 PM Page 504

Nephrosis (Nephrotic Syndrome) 6. Monitor I&O and vital signs and weigh daily.
7. Administer steroids to suppress autoimmune
A. General information response as ordered.
1. Autoimmune process leading to structural 8. Protect from known sources of infection.
alteration of glomerular membrane that results
in increased permeability to plasma proteins,
particularly albumin Acute Glomerulonephritis
2. Course of the disease consists of exacerbations A. General information
and remissions over a period of months to 1. Immune complex disease resulting from an
years antigen-antibody reaction
3. Commonly affects preschoolers, boys more 2. Secondary to a beta-hemolytic streptococcal
often than girls infection occurring elsewhere in the body
4. Pathophysiology 3. Occurs more frequently in boys, usually
a. Plasma proteins enter the renal tubule and between ages 67 years
are excreted in the urine, causing 4. Usually resolves in about 14 days, self-limiting
proteinuria. B. Medical management
b. Protein shift causes altered oncotic 1. Antibiotics for streptococcal infection
pressure and lowered plasma volume. 2. Antihypertensives if blood pressure severely
c. Hypovolemia triggers release of renin and elevated
angiotensin, which stimulates increased 3. Digitalis if circulatory overload
secretion of aldosterone; aldosterone 4. Fluid restriction if renal insufficiency
increases reabsorption of water and 5. Peritoneal dialysis if severe renal or
sodium in distal tubule. cardiopulmonary problems develop
d. Lowered blood pressure also stimulates C. Assessment findings
release of ADH, further increasing 1. History of a precipitating streptococcal
reabsorption of water; together with a infection, usually upper respiratory infection
general shift of plasma into interstitial or impetigo
spaces, results in edema. 2. Edema, anorexia, lethargy
5. Prognosis is good unless edema does not 3. Hematuria or dark-colored urine, fever
respond to steroids. 4. Hypertension
B. Medical management 5. Diagnostic tests
1. Drug therapy a. Urinalysis reveals RBCs, WBCs, protein,
a. Corticosteroids to resolve edema cellular casts
b. Antibiotics for bacterial infections b. Urine specific gravity increased
c. Thiazide diuretics in edematous stage c. BUN and serum creatinine increased
2. Bed rest d. ESR elevated
3. Diet modification: high protein, low sodium e. Hgb and HCT decreased
C. Assessment findings D. Nursing interventions
1. Proteinuria, hypoproteinemia, hyperlipidemia 1. Monitor I&O, blood pressure, urine; weigh
2. Dependent body edema daily.
a. Puffiness around eyes in morning 2. Provide diversional therapy.
b. Ascites 3. Provide client teaching and discharge
c. Scrotal edema planning concerning:
d. Ankle edema a. Medication administration
3. Anorexia, vomiting and diarrhea, malnutrition b. Prevention of infection
4. Pallor, lethargy c. Signs of renal complications
5. Hepatomegaly d. Importance of long-term follow-up
D. Nursing interventions
1. Provide bed rest.
a. Conserve energy. Hydronephrosis
b. Find activities for quiet play. A. General information
2. Provide high-protein, low-sodium diet during 1. Collection of urine in the renal pelvis due to
edema phase only. obstruction to outflow
3. Maintain skin integrity. 2. Obstruction most common at ureteral-pelvic
a. Do not use Band-Aids. junction (see Vesicoureteral Reflux) but may
b. Avoid IM injections (medication is not also be caused by adhesions, ureterocele,
absorbed into edematous tissue). calculi, or congenital malformation
c. Turn frequently. 3. Obstruction causes increased intrarenal
4. Obtain morning urine for protein studies. pressure, decreased circulation, and atrophy of

5
5. Provide scrotal support. the kidney, leading to renal insufficiency

504 NCLEX-RN Review


53155_05_Ch05_p421-532.qxd 2/27/09 1:32 PM Page 505

4. May be unilateral or bilateral; occurs more 1. The cosmetic appearance of the penis.
often in left kidney 2. Maintaining stable blood pressure in the child.
5. Prognosis good when treated early
3. Observing a straight stream when he voids.
B. Medical management: surgery to correct or remove
obstruction 4. His ability to void without discomfort.
C. Assessment findings
132. The nurse is teaching parents about post-op care
1. Repeated UTIs
2. Failure to thrive of their child who has had an orchiopexy. What
3. Abdominal pain, fever instructions will the nurse give the parents?
4. Fluctuating mass in region of kidney 1. You must tighten the rubber band around the
D. Nursing interventions: prepare child for multiple scrotum every 4 hours to maintain the testicle.
urologic studies (see also Vesicoureteral Reflux). 2. You must increase tension on the rubber
band every 4 hours.
3. You must check the rubber band every
4 hours to check for disconnection.
Sample Questions
4. Cut the rubber band after 24 hours.

127. A 4-year-old has just been diagnosed as having 133. A baby boy is born with a hypospadias. The
nephrotic syndrome. What is related to his parents decide to wait until the child is
potential for impairment of skin integrity? 6 months old for the repair. The father asks
the nurse why the doctor said not to have the
1. Joint inflammation.
baby circumcised. What is the nurses best
2. Drug therapy. response?
3. Edema. 1. It is best to wait until the baby is older and
4. Generalized body rash. understands the surgery.
2. Circumcision carries a high infection rate
128. A 20-month-old is admitted to the hospital with
and that may delay his hypospadias repair.
a diagnosis of cryptorchidism. What will
surgical correction help to prevent? 3. The foreskin may be used during the
hypospadias repair.
1. Difficulty in urinating.
4. He will need the foreskin to help anchor the
2. Sterility.
Foley catheter after the repair.
3. Herniation.
4. Peritonitis. 134. The nurse is planning care for a 2-year-old who
has nephrotic syndrome and is in remission.
129. A 3-day-old is diagnosed with hypospadias. His What type of diet would the nurse plan to feed
parents are very upset and have been willing this child?
listeners as the nurse has explained this problem 1. High protein, low calorie.
to them. In hypospadias, what primary problem
2. High calorie, low protein.
will the nurse discuss with the parents?
3. Low sodium, low fat.
1. Ambiguous genitalia.
4. Regular diet, no added salt.
2. Urinary incontinence.
3. Ventral curvature of the penis. 135. A 5-year-old girl recovered from a strep infection
4. Altered location of the urethral meatus. 2 weeks ago. She now presents with loss of
appetite, dark colored urine, and orbital edema.
130. The parents of a newborn who has hypospadias What is the nurses assessment?
ask about surgical repair. What age is the preferred 1. Nephrotic syndrome.
time to schedule surgical repair of hypospadias?
2. Glomerulonephritis.
1. 9 months old.
3. Renal tubular acidosis.
2. 5 years old.
4. Hemolytic uremic syndrome.
3. 12 years old.
4. 17 years old. 136. A 4-year-old boy is admitted with
glomerulonephritis. His mother asks why his
131. The parents of a baby boy who was born with eyes are so puffy. What is the nurses best
hypospadias want to know about the surgical response?
repair. The success of hypospadias surgery will 1. This is a common finding due to circulatory
be evaluated by what occurrence?

5
congestion in the kidneys.

PEDIATRIC NURSING 505


53155_05_Ch05_p421-532.qxd 2/27/09 1:32 PM Page 506

2. Children cry a lot with glomerulonephritis 131. 3. Observing the child void in a straight stream
and the puffiness should subside when he while standing is the expected successful
feels better. outcome of hypospadias repair.
3. Has he been rubbing his eyes excessively?
132. 3. The Torek procedure attaches a rubber band
4. Periorbital edema is associated with
from the testicle to the scrotal sac to the thigh to
hypertension.
maintain the testicle in the pouch. The family
must check the rubber band every 4 hours and
call the doctor if the rubber band breaks or
becomes disconnected.
Answers and Rationales
133. 3. The foreskin is frequently used as a flap
during the repair.
127. 3. A child with nephritic syndrome will have
massive edema. A child with edema is prone to 134. 4. The child who is in remission is allowed a
skin breakdown. regular diet; salt is restricted in the form of no
added salt at the table and excluding foods with
128. 2. If the testes remain in the abdomen beyond very high salt content.
the age of 5, damage resulting from exposure to
internal body temperature can cause sterility. 135. 2. Acute poststreptococcal glomerulonephritis is
the most common of the noninfectious renal
129. 4. In hypospadias, the urethral opening may be diseases in children.
anywhere along the underside of the penis.
136. 1. Periorbital edema is often associated with
130. 1. Most surgical repairs are scheduled for the circulatory congestion in the kidneys.
child between 6 and 18 months of age.

The Musculoskeletal System

VARIATIONS FROM THE ADULT C. Skeletal maturity is reached by age 17 in boys and
2 years after menarche in girls.
D. Certain characteristics of bone in children affect
Bones injury and healing, bones are more prone to injury,
A. Linear growth results from skeletal development and injury results from relatively minor accidents.
1. Centers of ossification 1. Metaphysis
a. Primary centers in diaphyses a. Absorbs shock, protects joints from
b. Secondary centers in epiphyses injury.
c. Used in assessment of bone age; number of b. Traumatic injury or infection to this
ossification centers in wrist equals age in growth plate can cause deformity.
years plus 1 c. If not injured, this growth plate
d. Centers appear earlier in girls than in participates in healing and straightening
boys of limbs by process of remodeling.
2. Metaphysis 2. Porous bone
a. Cartilaginous plate between diaphysis and a. Increases flexibility; absorbs force on impact.
epiphysis b. Allows bones to bend, buckle, and break in
b. The site of active growth in long bones greenstick or incomplete fracture.
c. Disappears over time with bony fusion of 3. Thicker periosteum
diaphysis and epiphysis a. More active osteogenic potential
d. Linear growth ends with epiphyseal fusion b. Healing more rapid
e. Assessment of bone age includes the 1) Neonatal period: may take 23 weeks
advancing bone edges 2) Early childhood: may take 4 weeks
B. Bone circumference growth occurs as new bone 3) Later childhood: may take 6 weeks
tissue is formed beneath the periosteum. 4) Adolescence: 810 weeks

5 506 NCLEX-RN Review


53155_05_Ch05_p421-532.qxd 2/27/09 1:32 PM Page 507

c. Stiffness after immobilization is rare unless PLANNING AND IMPLEMENTATION


joint has been injured.
E. Bone growth is affected by Wolffs law: bone will
grow in the direction in which stress is placed on it. Goals
A. Injury or deformity will be identified and treated
Muscles early.
B. Child will achieve maximum level of mobility.
A. Muscle growth is responsible for a large part of C. Pain will be relieved/controlled.
increase in body weight. D. Child will be free from injury.
B. The number of muscle fibers is constant E. Parents will be able to care for child at home.
throughout life; growth results from an increase in
the size of the muscle fibers and by an increased
number of nuclei per fiber. Interventions
C. Muscle growth most apparent in adolescence, Care of the Child with a Cast
influenced by growth hormone, adrenal androgens,
and, in boys, by testosterone. Also see Unit 4.
A. General information
1. Initial chemical hardening reaction may cause
ASSESSMENT a change in an infants body temperature;
monitor and intervene as needed.
2. Choose toys too big to fit down cast.
History 3. Do not use baby powder near cast because it
A. Presenting problem: symptoms may include: clumps and provides a medium for bacterial
1. Delayed motor development growth.
2. Injury 4. Prepare for anticipated casting by having child
3. Pain, loss of sensation, tingling help apply a cast to a doll the day before.
4. Muscle weakness, loss of function of an 5. Demonstrate the use of a cast cutter on a doll
extremity before using on child to show it does not cut skin.
5. Interference with normal activity or play B. Care of child in hip spica cast (cast encases child
6. Other parental concerns from nipples to knees; legs are abducted with a bar
B. Family history: genetic disorders, skeletal deformities between the thighs)
C. Inadequate nutrition (e.g., vitamin D deficiency 1. Use firm mattress to allow for increased
causes rickets) weight of plaster cast.
2. Do not lift cast by crossbar.
3. Protect cast from water and urine.
Physical Examination a. Put waterproof material over petaling
A. General appearance: note any asymmetry, visible (Chux, plastic diapers).
deformities, swelling (of joints or over bones), b. Elevate head of bed slightly to prevent
quality of movement (ROM, gait, guarding). urine and stool from seeping under cast;
B. Measure muscle strength. confirm that entire body is on a slant, not
C. Identify warmth or tenderness over bones and joints. just the head.
D. Assess pain: note type, location, onset, c. Use Bradford frame (canvas board with
relationship to activity. opening near genitalia) and place a bedpan
E. Perform examination in standing, lying, and sitting under opening.
positions. 4. Use pillows to support all parts of the cast.
5. Drape towel across top of chest part of cast
during feedings to prevent crumbs from
ANALYSIS entering cast.
6. Monitor for pain/pressure points due to
Nursing diagnoses for the child with a disorder of the growth if cast is on for a long time.
musculoskeletal system may include:
A. Risk for activity intolerance Care of the Child in Traction
B. Pain
C. Deficient diversional activity A. General information
D. Risk for injury 1. Infants and young toddlers do not have
E. Impaired physical mobility enough body weight to use traditional
F. Self-care deficit tractions effectively.
G. Disturbed body image 2. Children do not understand the necessity of
H. Risk for impaired skin integrity maintaining proper body alignment and will

5
I. Ineffective tissue perfusion need frequent repositioning.

PEDIATRIC NURSING 507


53155_05_Ch05_p421-532.qxd 2/27/09 1:32 PM Page 508

B. Nursing care: provide client teaching and


discharge planning concerning:
1. Importance of meticulous skin care
2. Need to wear protective clothing under brace
Bryant Traction 3. Potential problems of ill-fitting braces
a. Difficulty in balancing
b. Muscle stress and skin breakdown
4. Need for frequent checking and adjustment of
braces with growth

EVALUATION
Figure 5-10 Bryant traction A. Childs musculoskeletal development is normal as
evidenced by normal growth and activity.
B. Child experiences minimal discomfort.
C. Injuries are prevented.
D. Parents demonstrate ability to identify
B. Bryants traction: used primarily in children (See complications and administer treatments correctly.
Figure 5-10.)
1. Child is own counterweight.
2. Both legs are at 90 angle to bed. DISORDERS OF THE
3. Buttocks must be slightly off mattress in order
to ensure sufficient traction on legs.
MUSCULOSKELETAL SYSTEM
4. Used with children under age 2 years whose
weight is too low (under 30 lb [14 kg]) to Congenital Dislocation of the Hip
counterbalance without additional (Developmental Dysplasia of the Hip)
gravitational force.
5. Used for fractured femur and dislocated hip. A. General information
6. Monitor for vascular injury to feet with 1. Displacement of the head of the femur from
frequent neurovascular checks. the acetabulum; present at birth, although not
always diagnosed immediately
Care of the Child with a Brace 2. One of the most common congenital
malformations; incidence is 2 in 1000 live
A. General information births
1. Orthopedic device made of metal or leather 3. Familial disorder, more common in girls; may
applied to the body, particularly the trunk and be associated with spina bifida
lower extremities, to support the weight of the 4. Cause unknown; may be fetal position in utero
body, to correct or prevent deformities, and to (breech delivery), genetic predisposition, or
prevent involuntary movements in spastic laxity of ligaments
conditions 5. The acetabulum is shallow and the head of the
2. Types femur cartilaginous at birth, contributing to
a. Milwaukee brace the dislodgment.
1) Steel and leather brace fitted and B. Medical management
adapted to child individually 1. Goal is to enlarge and deepen socket by
2) Extends from chin cup and neck pad pressure.
to pelvis 2. The earlier treatment is initiated, the shorter
3) Used in scoliosis to correct curvature and less traumatic it will be.
4) Worn 23 hours/day, removed once 3. Early treatment consists of positioning the hip
daily for bathing in abduction with the head of the femur in the
5) Causes little interference with activity acetabulum and maintaining it in position for
b. Rotowalker several months.
1) Used to provide upright mobility in 4. If these measures are unsuccessful, traction
children with lower limb paralysis and casting (hip spica) or surgery may be
2) Child shifts weight to achieve mobility successful.
c. Leg brace C. Assessment findings
1) Designed to stabilize extremity and 1. May be unilateral or bilateral, partial or
offer support during ambulation complete
2) Special hinges permit hip, knee, and 2. Limitation of abduction (cannot spread legs to

5
ankle to flex during sitting change diaper)

508 NCLEX-RN Review


53155_05_Ch05_p421-532.qxd 2/27/09 1:32 PM Page 509

3. Ortolanis click (should only be performed by 3. Deformity almost always congenital; usually
an experienced practitioner) unilateral
a. With infant in supine position (on the 4. Occurs more frequently in boys than in girls;
back), bend knees and place thumbs on may be associated with other congenital
bent knees, fingers at hip joint. disorders but cause unknown
b. Bring femurs 90 to hip, then abduct. 5. General incidence: 1 in 7001000
c. With dislocation there is a palpable click B. Medical management
where the head of the femur snaps over 1. Exercises
edge of acetabulum. 2. Casting (cast is changed periodically to change
4. Barlows test angle of foot)
a. With infant on back, bend knees. 3. Denis Browne splint (bar shoe): metal bar with
b. Affected knee will be lower because the shoes attached to the bar at specific angle
head of the femur dislocates toward bed by 4. Surgery and casting for several months
gravity (referred to as telescoping of limb). C. Assessment findings: foot cannot be manipulated by
5. Additional skin folds with knees bent, from passive exercises into correct position (differentiate
telescoping from normal clubbing of newborns feet)
6. When lying on abdomen, buttocks of affected D. Nursing interventions
side will be flatter because head of femur falls 1. Perform exercises as ordered.
toward bed from gravity 2. Provide cast care or care for child in a brace.
7. Trendelenburg test (used if child is old enough 3. Child who is learning to walk must be
to walk) prevented from trying to stand; apply
a. Have child stand on affected leg only. restraints if necessary.
b. Pelvis will dip on normal side as child 4. Provide diversional activities.
attempts to stay erect. 5. Adapt care routines as needed for cast or brace.
D. Nursing interventions 6. Assess toes to be sure cast it not too tight.
1. Maintain proper positioning: keep legs 7. Provide skin care.
abducted. 8. Provide client teaching and discharge
a. Pavlik harness (place undershirt under planning concerning:
harness and socks on legs) a. Application/care of immobilization device
b. Frejka pillow splint (jumperlike suit to b. Preparation for surgery if indicated
keep legs abducted) c. Need to monitor special shoes for
c. Place infant on abdomen with legs in continued fit throughout treatment.
frog position
d. Other immobilization devices (splints,
casts, braces)
Tibial Torsion
2. Provide adequate nutrition; adapt feeding A. General information
position as needed for immobilization device. 1. Rotational deformity of tibia (greater than that
3. Provide sensory stimulation; adapt to normally found in newborn)
immobilization device and positioning. 2. Types
4. Provide client teaching and discharge a. Internal: knee forward and foot inward
planning concerning: b. External: knee forward and foot outward
a. Application and care of immobilization (rare, associated with muscle paralysis)
devices 3. Majority of cases resolve without treatment
b. Modification of child care using B. Medical management
immobilization devices 1. Splinting: use of Denis Browne splint at night
2. Surgical correction if still evident by age
3 years
Clubfoot (Talipes) C. Assessment findings: with child lying supine,
A. General information assess for straight line between tibial tuberosity
1. Abnormal rotation of foot at ankle and 2nd toe; in tibial torsion, the line intersects
a. Varus (inward rotation): would walk on the 4th or 5th toe.
ankles, bottoms of feet face each other D. Nursing interventions
b. Valgus (outward rotation): would walk on 1. If no treatment needed, encourage parents to
inner ankles be patient and emphasize that condition
c. Calcaneous (upward rotation): would walk usually resolves by itself
on heels 2. If stretching exercises are recommended, teach
d. Equinas (downward rotation): would walk parents normal ROM exercises and how to
on toes carry them out.
2. Most common deformity (95%) is talipes 3. Instruct parents on use of Denis Browne splint

5
equinovarus. if needed.

PEDIATRIC NURSING 509


53155_05_Ch05_p421-532.qxd 2/27/09 1:32 PM Page 510

Legg-Calv-Perthes Disease 2. Types


a. Osteogenesis imperfecta congenita:
A. General information autosomal recessive, prognosis poor
1. Aseptic necrosis of femoral head due to b. Osteogenesis imperfecta tarda: autosomal
disturbance of circulation to the area dominant, less severe form, involvement of
2. Primarily affects boys ages 410 years varying degrees
3. Stages: lasting from 18 months to a few years 3. Classic picture includes soft, fragile bones;
a. Initial stage: may not be distinguishable blue sclera; otosclerosis
from transient synovitis 4. Severity of symptoms decreases at puberty due
b. Avascular stage: often the first stage to hormone production and childs ability to
noticed prevent injury
c. Revascularization stage: regeneration of B. Medical management
vascular and connective tissue 1. Magnesium oxide supplements
d. Regeneration stage: formation of new bone 2. Reduction and immobilization of fractures
B. Medical management: goal is to minimize C. Assessment findings
deformity until healing process is completed 1. Osteogenesis imperfecta congenita
1. Initial bed rest with traction and then an a. Multiple fractures at birth
abduction brace b. Possible skeletal deformity due to
2. Possible surgery intrauterine fracture
C. Assessment findings c. Bones of skull are soft
1. Limp, limitation of movement d. Occasional intracranial hemorrhage
2. Pain in groin, hip, and referred to knee; often 2. Osteogenesis imperfecta tarda
difficult for child to localize pain a. Delayed walking, fractures, structural
3. Diagnostic test: X-ray reveals opaque scoliosis as child grows
ossification center of head of the femur b. Lower limbs more frequently affected
(softened in avascular stage) c. Hypermobility of joints
D. Nursing interventions d. Prone to dental caries
1. Provide care for a child with a cast or brace. D. Nursing interventions
2. Provide diversional activities. 1. Support limbs, do not stretch.
2. Position with care; use blankets to aid in
Slipped Femoral Capital mobility and provide support.
3. Instruct parents in bathing, dressing,
Epiphysis diapering.
4. Support parents; encourage expression of
A. General information feelings of anger or guilt (parents may have
1. Spontaneous displacement of proximal been unjustly suspected of child abuse).
femoral epiphysis in a posterior and inferior
direction
2. Onset insidious; usually occurs during fast
Scoliosis
growth period of adolescence (growth A. General information
hormones weaken epiphyseal plate) 1. Lateral curvature of the spine
3. Occurs most often in very tall and very obese 2. Most commonly occurs in adolescent girls
adolescents; boys affected more frequently 3. Disorder has a familial pattern; associated with
B. Medical management other neuromuscular disorders
1. Skeletal traction 4. Majority of the time (75% of cases) disorder is
2. Surgical stabilization with pinning idiopathic; others causes include congenital
C. Assessment findings abnormality of vertebrae, neuromuscular
1. Limp and referred pain to groin, hip, or knee disorders, and trauma
2. Limited internal rotation and abduction of hip 5. May be functional or structural
D. Nursing interventions a. Nonstructural/functional: C curve of spine
1. Suggest weight reduction program for obese 1) Due to posture, can be corrected
children to decrease stress on bones. voluntarily and disappears when child
2. Provide care for the child with a cast or lies down
traction. 2) Not progressive
3) Treated with posture exercises
Osteogenesis Imperfecta b. Structural/progressive: S curve of spine
1) Usually idiopathic
A. General information 2) Structural change in spine, does not
1. An inherited disorder affecting collagen disappear with position changes

5
formation and resulting in pathologic fractures 3) More aggressive intervention needed

510 NCLEX-RN Review


53155_05_Ch05_p421-532.qxd 2/27/09 1:32 PM Page 511

B. Medical management c. Instrumentation to stabilize position


1. Stretching exercises of the spine for 1) Harrington rod
nonstructural changes 2) Luque instrumentation: wires and
2. Bracing hooks
a. Milwaukee brace C. Assessment findings (structural scoliosis) (See
b. TSLO-custom molded plastic orthotic Figure 5-11.)
brace 1. Failure of curve to straighten when child
c. Braces worn 1623 hours/day; off only for bends forward with knees straight and arms
hygiene hanging down to feet (curve disappears with
3. Surgical correction functional scoliosis)
a. Spinal alignment 2. Uneven bra strap marks
b. Fusion with bone chips 3. Uneven hips
4. Uneven shoulders
5. Asymmetry of rib cage
6. Diagnostic test: X-ray reveals curvature
D. Nursing interventions
1. Teach/encourage exercises as ordered.
2. Provide care for child with Milwaukee brace
a. Child wears brace 23 hours/day; is
removed once a day for bathing.
b. Monitor pressure points, adjustments may
be needed to accommodate increase in
height or weight.
c. Promote positive body image with brace.
3. Provide cast/traction care.
4. Assist with modifying clothing for
immobilization devices.
5. Adjust diet for decreased activity.
6. Provide diversional activities.
7. Provide care for child with Harrington rod
insertion.
8. Provide client teaching and discharge
planning concerning:
a. Exercises
b. Brace/traction/cast care
A.
c. Correct body mechanics
d. Alternative education for long-term
hospitalization/home care
e. Availability of community agencies

Surgical Correction for Scoliosis


A. General information
1. Spinal fusion and installation of supports
along spine
2. Used for moderate to severe curvatures
3. Usually results in increase in height; positive
body image changes
B. Nursing interventions (see also Discectomy)
1. Provide general pre-op teaching and care.
2. In addition to routine post-op care.
a. Log roll.
b. Do not raise head of bed.
c. Usually out of bed to chair after 48 hours
with Luque procedure.
B. d. Discuss adapting home environment to
allow for privacy yet interaction with
Figure 5-11 Adolescent girl with scoliosis. family during recovery.
(A) Frontal view; (B) posterior view e. Discuss alternate methods of education

5
during recovery period.

PEDIATRIC NURSING 511


53155_05_Ch05_p421-532.qxd 2/27/09 1:32 PM Page 512

Muscular Dystrophy 5. Support child and parents and provide


information about availability of community
A. General information agencies and support groups.
1. A group of muscular diseases in children
characterized by progressive muscle weakness
and deformity Juvenile Rheumatoid Arthritis
2. Genetic in origin; biochemical defect is A. General information
suspected 1. Systemic, chronic disorder of connective
3. Types tissue, resulting from an autoimmune reaction
a. Pseudohypertrophic (Duchenne type): 2. Results in eventual joint destruction
most frequent type 3. Affected by stress, climate, and genetics
1) X-linked recessive 4. More common in girls; peak ages 25 and
2) Affects only boys 912 years
3) Usually manifests in first 4 years 5. Types
b. Facioscapulohumeral a. Mono/pauciarticular JRA
1) Autosomal dominant 1) Fewer than four joints involved
2) Mild form, with weakness of facial and (usually in legs)
shoulder girdle muscles 2) Asymmetric; rarely systemic
3) Onset usually in adolescence 3) Generally mild signs of arthritis
c. Limb girdle 4) Symptoms may decrease as child
1) Autosomal recessive enters adulthood
2) Affects boys and girls 5) Prognosis good
3) Onset usually in adolescence b. Polyarticular JRA
d. Congenital 1) Multiple joints affected
1) Autosomal recessive 2) Symmetrical symptoms of arthritis,
2) Onset in utero disability may be mild to severe
e. Myotonic 3) Involvement of temporomandibular
1) Autosomal dominant joint may cause earaches
2) More common in boys 4) Characterized by periods of remissions
3) Onset in infancy or childhood, or adult and exacerbations
onset 5) Prognosis poorer
4) Prognosis in childhood form is 6) Treatment symptomatic for arthritis:
guarded physical therapy, ROM exercises,
4. Disease causes progressive disability aspirin
throughout childhood; most children with c. Systemic disease with polyarthritis (Stills
Duchennes muscular dystrophy are confined disease)
to a wheelchair by age 810 years. 1) Explosive course with remissions and
5. Death occurs by age 20 in 75% of clients with exacerbations lasting for months
Duchennes muscular dystrophy. 2) Begins with fever, rash,
B. Assessment findings (Duchenne type) lymphadenopathy, anorexia, and
1. Pelvic girdle weakness is early sign (child weight loss
waddles and falls) B. Medical management, assessment findings, and
2. Gowers sign (child uses hands to push up nursing interventions: see Rheumatoid Arthritis
from the floor)
3. Scoliosis (from weakness of shoulder girdle)
4. Contractures and hypertrophy of muscles
5. Diagnostic tests Sample Questions
a. Muscle biopsy reveals histologic changes:
degeneration of muscle fibers and
replacement of fibers with fat. 137. An 18-month-old has a fractured femur and is in
b. EMG shows decrease in amplitude and Bryants traction. In evaluating the correct
duration of potentials. application of the traction, what will the nurse
6. Serum enzymes increased, especially CPK note?
C. Nursing interventions 1. The child is being continuously and
1. Prepare child for EMG and muscle biopsy.
gradually pulled toward bottom of bed.
2. Maintain function at optimal level; keep child
as active and independent as possible. 2. The childs buttocks are raised slightly.
3. Plan diet to prevent obesity. 3. The childs leg is at a 45 angle to the bed.
4. Continually evaluate capabilities. 4. The child can move the unaffected leg freely.

5 512 NCLEX-RN Review


53155_05_Ch05_p421-532.qxd 2/27/09 1:32 PM Page 513

138. A 14-year-old is in a hip spica cast. Which is the 1. Promoting adequate circulation is a top priority.
correct method to turn the adolescent? 2. Drying the cast is very important.
1. Use the cross bar. 3. Assessing the smell of a cast is a top priority.
2. Turn her upper body first, then turn the 4. Preserving skin integrity is of the utmost
lower body. importance.
3. Log-roll her.
4. Tell her to pull on the trapeze and sit up to 144. In examining a newborn, the nurse notes the
help in turning. following: asymmetric gluteal folds, shortened
right leg, and limited abduction of the right
139. A routine physical examination on a 2-day-old thigh. The nurse would correctly interpret these
uncovered evidence of congenital dislocation, or observations as which of the following?
dysplasia, of the right hip. When assessing the 1. Right congenital dislocated hip.
infant, what would be a sign of one-sided hip 2. Spastic cerebral palsy.
displacement? 3. Left hip dysplasia.
1. An unusually narrow perineum 4. Myelodysplasia.
2. Pain where her leg is abducted.
3. Symmetrical skin folds near her buttocks and 145. An infant with congenital hip dysplasia is
thigh. placed in a Pavlik harness. In the nurses
4. Asymmetrical skin folds over the buttocks teaching plan for the mother, which of the
and thigh. following would be important to include?
1. Adjustment of daily care routines as the
140. An infant is being treated for congenital hip harness is worn 24 hours a day.
dysplasia with a Pavlik harness. The babys 2. Clothing should not be worn under the harness.
mother asks if she can remove the harness if it 3. The harness should be removed for bathing
becomes soiled. What would be the nurses best and diapering only.
response?
4. The infant should be confined to the crib.
1. No, the harness may not be removed.
2. No, she will only be wearing it a few days. 146. In assessing a newborn for talipes equinovarus,
3. Yes, just long enough to clean the area. the nurse would note which of the following?
4. Yes, just overnight while she is sleeping. 1. The feet turn inward when the infant lies
still, but they are flexible.
141. A 10-year-old takes aspirin QID for Stills disease 2. The feet are rigid and cannot be manipulated
(juvenile rheumatoid arthritis). What symptoms to a neutral position.
would her mother observe that would be 3. Uneven knee length occurs when both knees
indicative of aspirin toxicity? are flexed.
1. Hypothermia. 4. Limited abduction is observed when
2. Hypoventilation. performing the Ortolani maneuver.
3. Decreased hearing acuity.
4. Increased urinary output. 147. The nurse would evaluate that the parents
correctly understand the care of their infant
142. Which of the following would the nurse include being treated for talipes equinovarus if the
in a plan of care for a toddler with a newly parents said which of the following?
applied hip spica cast? 1. We will unwrap the cast every night and
1. Petal the cast around the perineum area with massage his feet with lotion to prevent skin
waterproof tape. breakdown.
2. Teach the parents care of the child just before 2. Well petal the cast around the babys groin to
discharge. protect it from urine and bowel movements.
3. Give the child small blocks and beads to 3. Every day well check the babys toes for
promote eye-hand coordination. movement and color after we squeeze them.
4. Check neurovascular status every shift. 4. Were so glad that the casts will cure his
club feet.
143. The mother of a 6-year-old asks why she was
told not to use powder under her childs long leg 148. Which of the following comments by the school
cast. Which of the following is the most accurate nurse would be most appropriate in screening

5
basis for the nurses response? for scoliosis of a 13-year-old?

PEDIATRIC NURSING 513


53155_05_Ch05_p421-532.qxd 2/27/09 1:32 PM Page 514

1. You may leave your shirt on, but stand erect 138. 3. The client in a hip spica cast should be turned
and turn to the side. as a unit.
2. Do you have any back pain?
139. 4. Displacement of the hip on one side causes
3. Remove your clothes from the waist up and
asymmetry of skin folds.
bend over at your waist.
4. Have you noticed that your skirts dont hang 140. 1. The harness is not to be removed until the hip
evenly? is stable with 90 of flexion and X-ray
confirmation. This usually occurs after about
149. A child is admitted to the hospital for a spinal 3 weeks in a Pavlik harness.
fusion and Harrington rod insertion. What
would be a nursing priority in the first 8 hours 141. 3. Tinnitus or ringing in the ears is a side effect
postoperatively? of aspirin therapy. In salicylate poisoning the
1. Give fluids and fiber to promote bowel child will have hypothermia, hyperventilation to
elimination. compensate for metabolic acidosis, and may
2. Check neurovascular function in extremities. develop renal failure.
3. Log roll every 4 hours. 142. 1. It is important to protect the cast from urine
4. Monitor hourly urine output. and stool to prevent skin and cast breakdown.

150. The nurse would evaluate that a child 143. 4. Powder may irritate the skin, leading to skin
understood the effective use of her Milwaukee breakdown and infection.
brace for her scoliosis if she said which of the
following? 144. 1. These are all signs of right congenital
1. Im so glad that I dont have to sleep in this dislocated hip in a newborn.
brace.
145. 1. The harness is worn 24 hours a day so that
2. Ive toughened my skin so I can wear the parents must learn how to manage daily care
brace right next to my skin. (sponging and dressing the baby) with the
3. I cant believe that Im not allowed to chew harness on.
gum anymore.
4. Im going to look forward to my bath time 146. 2. Talipes equinovarus is a rigid deformity with
each day without this brace. forefoot adduction, inversion of the heel, and
plantar flexion of the feet.
151. A 4-year-old has recently been diagnosed with
Duchennes muscular dystrophy. His parents ask 147. 3. Parents should be taught to assess
if their 2-year-old daughter will get the disease. neurovascular status of the toes because babies
The nurses best response would be which of the grow quickly and may outgrow the casts.
following?
148. 3. This is part of the screening process for
1. Every child you have has a 25% chance of scoliosis. The nurse is checking for rib hump
developing the disease and a 50% chance of and flank asymmetry. Also included is visual
being a carrier. inspection of frontal and dorsal posture,
2. Sons are affected 50% of the time, whereas observation for uneven hip and shoulder levels
50% of the time daughters will become as well as for muscular disproportion.
carriers who have no symptoms.
3. Only your sons have a 25% chance of 149. 2. One of the greatest risks of spinal surgery is of
developing the disease. paralysis if the spinal cord is injured or
4. Every child has a 50% chance of developing compressed by swelling. Monitoring for
the disease. sensation and movement is the top priority.

150. 4. For best results in correction, the brace should


be worn for 2023 hours a day and only
removed for hygiene and skin care.
Answers and Rationales
151. 2. Duchennes muscular dystrophy is an
X-linked recessive disorder. The defective gene
137. 2. In Bryants traction both legs are in traction at is transmitted through carrier females to affected
a 90 angle and the buttocks are raised slightly sons 50% of the time depending on which X is

5
off the bed. transmitted. Daughters have a 50% chance of
becoming carriers.

514 NCLEX-RN Review


53155_05_Ch05_p421-532.qxd 2/27/09 1:32 PM Page 515

The Endocrine System

VARIATIONS FROM THE ADULT EVALUATION


A. Adenohypophysis (anterior lobe of pituitary gland) A. Child receives appropriate medication, and
1. Growth hormone nutritional requirements are met; symptoms of
a. Does not affect prenatal growth. endocrine disease are controlled.
b. Main effect on linear growth is through B. Child is free from complications of disease.
increase of cells in skeletal bones. C. Child is achieving growth and developmental tasks
c. Maintains rate of synthesis of body on as normal a timetable as possible.
protein. D. Child discusses feelings about body image and
2. Thyroid-stimulating hormone (TSH) uses coping mechanisms that promote a positive
a. Important for normal development of self-image.
bones, teeth, and brain.
b. Secretion decreases throughout childhood,
then increases at puberty. DISORDERS OF THE
3. Adrenocorticotropic hormone (ACTH)
a. Little is produced throughout childhood. ENDOCRINE SYSTEM
b. Becomes active in adolescence.
c. Stimulates adrenals to secrete sex Diabetes Mellitus
hormones.
d. Influences production of gonadotropic Also see Unit 4.
hormone by hypothalamus. A. General information
1) Gonadotropic hormones activate 1. Most common endocrine disease of children;
gonads. onset may be at any age
2) Gonads secrete estrogen or 2. Children typically develop Type 1: insulin-
testosterone, which stimulate dependent diabetes mellitus
development of secondary sex 3. Possible genetic predisposition to disease
characteristics. 4. Treatments vary based on rapid growth rate in
4. Estrogen has an inhibitory effect on children, increased incidence of infections,
epiphyseal growth. and dietary fads of peers; all include insulin
administration.
5. Risk of complications is high; most commonly
ANALYSIS retinopathy, neuropathy, nephropathy, skin
changes, predisposition to infection
Nursing diagnoses for the child with a disorder of the 6. Children sometimes have one honeymoon
endocrine system may include: period that occurs shortly after a child is
A. Ineffective health maintenance regulated on insulin for the first time
B. Impaired home maintenance a. Lasts from 1 month to 1 year.
C. Noncompliance b. Represents final effort of pancreas to
D. Disturbed body image provide insulin until beta cells are
E. Low self-esteem completely destroyed.
c. Parents may distrust the diagnosis of
diabetes and need to be reminded that
PLANNING AND IMPLEMENTATION symptoms will reappear and child will
need insulin for life.
B. Medical management
Goals 1. Insulin
A. Any endocrine imbalance in childhood will be 2. Diet therapy
identified and treated. 3. Exercise
B. Child will achieve a normal metabolic state. 4. Prevention of complications
C. Child will develop successful coping mechanisms C. Assessment findings
for manifestations of disease. 1. Rapid onset
D. Child will have no signs of complications of the 2. Polyuria, polydipsia, polyphagia, fatigue

5
disease. 3. Weight loss

PEDIATRIC NURSING 515


53155_05_Ch05_p421-532.qxd 2/27/09 1:32 PM Page 516

4. Ketoacidosis B. Medical management: administration of growth


5. Dry, flushed skin with hyperglycemia hormone (limited in supply since it is rendered
D. Nursing interventions from human cadavers)
1. Administer insulin (regular and NPH) as ordered. C. Assessment findings
2. Force fluids without sugar. 1. Newborn is of normal size, but child falls
3. Monitor blood glucose levels daily. below the third percentile by age 1.
4. Observe for hypoglycemia (insulin shock): 2. Child is well proportioned, but may be
behavior changes, sweating. overweight for height.
5. Provide client teaching and discharge 3. Underdeveloped jaw, abnormal position of
planning concerning: teeth, high voice, delayed puberty
a. Daily regimen for home care 4. Diagnostic tests
b. Urine and blood glucose monitoring a. X-rays reveal delayed closing of epiphyseal
c. Nutrition management plates of long bones
d. Effects of infection and exercise on b. Normal IQ
carbohydrate metabolism D. Nursing interventions
e. Prevention of acute and chronic 1. Interact with child according to chronologic
complications age/developmental level, and not according to
physical appearance.
Congenital Hypothyroidism (Cretinism) 2. Administer growth hormone as ordered
(because of delay in bone development, these
A. General information children can still grow even when their peers
1. Disorder related to absent or nonfunctioning have stopped).
thyroid 3. Monitor for signs and symptoms of additional
2. Newborns are supplied with maternal thyroid neurologic disorders.
hormones that last up to 3 months 4. Keep careful records of height and
B. Medical management weight.
1. Prevention: neonatal screening blood test 5. Encourage child/parents to express
(mandatory in many states) feelings.
2. Drug therapy: thyroid hormone replacement 6. Assist child in learning to interact normally
3. Without treatment mental retardation and with peers.
developmental delay will occur after age 3
months
C. Assessment findings Hyperpituitarism (Gigantism)
1. Altered body proportions; short stature with
legs shorter than they should be in proportion A. General information
to trunk 1. Hypersecretion of growth hormone (usually
2. Tongue is enlarged and protrudes from mouth; related to a tumor of the anterior pituitary)
may result in breathing and feeding difficulties resulting in enlargement of bones of head,
3. Hypothermia with cool extremities hands, and feet, and overgrowth of long
4. Short, thick neck; delayed dentition bones
5. Hypotonia 2. Especially noticeable at puberty
6. Low levels of T3 and T4 B. Medical management
D. Nursing interventions 1. Surgery to remove tumor
1. Administer oral thyroxine and vitamin D as 2. Radiation therapy if there is no tumor
ordered to prevent mental retardation. C. Assessment findings
2. Provide client teaching and discharge 1. Height beyond maximum upper
planning concerning: percentile
a. Medication administration and side effects 2. Proportional weight and muscle growth
b. Importance of continued therapy 3. Coarse facial features
4. Signs of increased ICP if caused by a
tumor
Hypopituitarism D. Nursing interventions
1. Record height and head circumference.
(Pituitary Dwarfism) 2. Provide nursing care for a client receiving
A. General information radiation therapy.
1. Hyposecretion of growth hormone by the 3. Provide care for the child with a brain
anterior lobe of the pituitary gland tumor.
2. Cause may be unknown or it may be due to 4. Assist child in interacting normally with
craniopharyngioma peers.

5 516 NCLEX-RN Review


53155_05_Ch05_p421-532.qxd 2/27/09 1:32 PM Page 517

nurse why she cannot take pills like her


Sample Questions grandmother who also has diabetes. Which
would be a correct response from the nurse?
1. How long has your grandmother been taking
152. An 8-year-old is newly diagnosed with diabetes oral medication?
mellitus. Which of the following symptoms is
2. Youll be able to stop taking insulin once you
different from what you would expect to find in
stop growing.
maturity-onset (Type 2) diabetes?
3. You have a different kind of diabetes and you
1. Increased appetite.
will need to take insulin throughout your life.
2. Increased thirst.
4. Youll be able to switch to pills when you
3. Increased urination. reach your grandmothers age.
4. Weight loss.

153. A 7-year-old is newly diagnosed with diabetes


mellitus. She had an injection of regular and
NPH insulin at 0730. At 1510 she complains that Answers and Rationales
she does not feel well. She is pale, perspiring,
and trembling. What instructions should the
nurse give the child? 152. 4. Weight loss is associated with juvenile
1. Tell her to lie down and wait for the dinner diabetes, whereas weight gain develops in
trays to arrive. maturity-onset diabetes.
2. Ask her to give a urine specimen and test it 153. 3. The symptoms suggest she is having a
for sugar and acetone. hypoglycemic reaction from the NPH insulin
3. Give her a carbohydrate snack. and needs an afternoon snack.
4. Administer the afternoon dose of regular insulin.
154. 3. Juvenile or Type 1 diabetics need lifetime
154. A 10-year-old with diabetes mellitus is learning insulin because they no longer produce their
how to administer her insulin. She asks the own.

The Integumentary System

VARIATIONS FROM THE ADULT B. History of present condition: onset, relationship to


eating or other activities, medication usage
A. Skin is only 1 mm thick at birth; approximately
twice as thick at maturity. Physical Examination
B. Evaporative water loss is greater in infants and
small children. A. Lesion type: note petechiae, erythema,
C. Skin is more susceptible to bacterial infection in ecchymosis; note secondary symptoms from
children. rubbing, scratching, or healing.
D. Children are more prone to toxic erythema as a B. Observe distribution pattern.
result of drug reactions and skin eruptions. C. Note presence of pain or altered sensation.
E. Childrens skin is more susceptible to sweat D. Check scalp for signs of lice or nits.
retention and maceration.

ANALYSIS
ASSESSMENT
Nursing diagnoses for the child with a disorder of the
integumentary system may include:
History A. Pain
A. Medical history: previous skin disease, allergic B. Disturbed body image
C. Disturbed sensory perception

5
conditions

PEDIATRIC NURSING 517


53155_05_Ch05_p421-532.qxd 2/27/09 1:32 PM Page 518

D. Risk for impaired skin integrity 4. Spreads peripherally


E. Low self-esteem 5. Most commonly found on face, axillae, and
extremities
6. Pruritus
PLANNING AND IMPLEMENTATION D. Nursing interventions
1. Implement skin isolation techniques.
2. Soften the skin and crusts with Burrows
Goals solution compresses.
A. Child will be free from discomfort. 3. Remove crusts gently.
B. Skin integrity will be restored. 4. Cover draining lesions to prevent spread of
C. Spread of infection and secondary infection will infection.
be prevented. 5. Administer antibiotics as ordered, both orally
and as bacteriocidal ointments.
6. Prevent secondary infection.
EVALUATION 7. Provide client teaching and discharge
planning concerning:
A. Child is free from discomfort. a. Medication administration
1. Minimal scratching or rubbing b. Proper hygiene techniques
2. Relaxed facial expression
3. Minimal restlessness
B. Childs skin is clean, dry, and free from redness or
Ringworm
signs of irritation. A. General information
C. Child is free from complications such as spread of 1. Dermatomycosis due to various species of fungus
infection. 2. Infected sites include:
D. Parents demonstrate satisfactory hygiene measures a. Scalp (tinea capitis)
when caring for child with disorder of skin or b. Body (tinea corporis)
scalp. c. Feet (tinea pedis or athletes foot)
3. May be transmitted from person to person or
acquired from animals or soil
DISORDERS OF THE B. Assessment findings
1. Scalp
INTEGUMENTARY SYSTEM a. Scaly circumscribed patches on the scalp
b. Base of hair shafts are invaded by spores of
Burns the fungus; causes hair to break off,
resulting in alopecia
Also see Unit 4. c. Spreads in a circular pattern
A. For children, the rule of nines is modified; the d. Detected by Woods lamp (fluoresces green
head of a small child is 1819%, the trunk 32%, at base of the affected hair shafts)
each leg 15%, each arm 912%. 2. Skin: red-ringed patches of vesicles; pain,
B. Burns in infants and toddlers are frequently due to scaling, itching
spills (pulling hot fluids on them or falling into C. Nursing interventions
hot baths); for older children, flame burns are more 1. Prevention: isolate from known infected
frequent. persons.
2. Apply antifungal ointment as ordered.
Impetigo 3. Administer oral griseofulvin as ordered.
A. General information
1. Superficial bacterial infection of the outer Pediculosis (Head Lice)
layers of skin (usually staphylococcus or A. General information
streptococcus) 1. Parasitic infestation
2. Common in toddlers and preschoolers 2. Adult lice are spread by close physical contact
3. Related to poor sanitation (sharing combs, hats, etc.)
4. Very contagious 3. Occurs in school-age children, particularly
B. Medical management: topical and systemic those with long hair
antibiotics B. Medical management: special shampoos followed
C. Assessment findings by use of fine-tooth comb to remove nits
1. Well-demarcated lesions C. Assessment findings
2. Macules, papules, vesicles that rupture, 1. White eggs (nits) firmly attached to base of
causing a superficial moist erosion hair shafts

5
3. Moist area dries, leaving a honey-colored crust 2. Pruritus of scalp

518 NCLEX-RN Review


53155_05_Ch05_p421-532.qxd 2/27/09 1:32 PM Page 519

D. Nursing interventions C. Nursing interventions


1. Institute skin isolation precautions (especially 1. Administer antihistamines and cortisone as
head coverings and gloves to prevent spread to ordered.
self, other staff, and clients) 2. Provide client teaching and discharge
2. Use special shampoo and comb the hair planning concerning:
3. Provide client teaching and discharge a. Plant identification
planning concerning: b. Need to wash with soap and water after
a. How to check self and other family contact with plant
members and how to treat them c. Importance of washing clothes to get the
b. Washing of clothes, bed linens, etc.; resin out
discouraging sharing of brushes, combs,
and hats Eczema
A. General information
Allergies 1. Atopic dermatitis, often the first sign of an
Diaper Rash allergic predisposition in a child; many later
develop respiratory allergies
A. General information 2. Usually manifests during infancy
1. Contact dermatitis B. Medical management
2. Plastic/rubber pants and linings of disposable 1. Drug therapy
diapers exacerbate the condition by prolonging a. Topical steroids
contact with moist, warm environment; skin is b. Antihistamines
further irritated by acidic urine c. Emollients
3. May also be caused by sensitivity to laundry d. Cautious administration of immunizations
soaps used e. Medicated or colloid baths
B. Medical management: exposure of skin to air/heat 2. Diet therapy: elimination diet to detect
lamp offending foods
C. Assessment findings C. Assessment findings
1. Erythema/excoriation in the perineal area 1. Erythema, weeping vesicles that rupture and
2. Irritability crust over
D. Nursing interventions 2. Usually evident on cheeks, scalp, behind ears,
1. Keep area clean and dry; clean with mild soap and on flexor surfaces of extremities (rarely on
and water after each stool and as soon as child diaper area)
urinates. 3. Severe pruritus; scratching causes thickening
2. Take off diaper and expose area to air during and darkening of skin
the day. 4. Dry skin, sometimes urticaria
3. Use heat lamp as ordered. D. Nursing interventions
4. Provide client teaching and discharge 1. Avoid heat and prevent sweating; keep skin
planning concerning: dry (moisture aggravates condition).
a. Proper hygiene/infant care 2. Monitor elimination diet to detect food cause.
b. Diaper laundering methods a. Remove all solid foods from diet (formula
c. Avoiding use of plastic pants or disposable only).
diapers with a plastic lining b. If symptoms disappear after 3 days, start
d. Avoiding use of cornstarch (a good one new food group every 3 days to see if
medium for bacteria once it becomes wet) symptoms reappear.
e. Need to avoid use of commercially c. The food that is suspected of causing the
prepared diaper wipes because they rash is withdrawn again to make sure
contain chemicals and alcohol, which may symptoms go away in 3 days and is then
be irritating introduced a second time (challenge test).
3. Check materials in contact with childs skin
Poison Ivy (sheets, lotions, soaps).
4. Tepid baths, mild soaps.
A. General information a. Provide lubricant immediately after bath.
1. Contact dermatitis; mediated by T-cell b. Pat dry gently with soft towel (do not rub)
response so rash is not seen for 2448 hours and pat in lubricant.
after contact. c. Avoid the use of harsh soaps (dry skin).
2. Poison ivy is not spread by the fluid in the 5. Administer topical steroids as ordered
vesicles; can be spread by clothes and animals (penetrate better if applied within 3 minutes
that retain the plant resin. after bath). Thin layer of topical steroid.
B. Assessment findings: very pruritic impetigo-like

5
6. Use cotton instead of wool clothing.
lesions

PEDIATRIC NURSING 519


53155_05_Ch05_p421-532.qxd 2/27/09 1:32 PM Page 520

7. Keep childs nails short to prevent scratching 156. A 7-year-old boy has a loss of scalp hair and is
and secondary infection; use gloves or elbow diagnosed with ringworm. What question will
restraints if needed. the nurse most likely ask?
8. Apply wet saline or Burrows solution 1. Whether the family owns any pets.
compresses.
2. From what economic background is the
9. Double-rinse laundry.
10. Assess skin for infection. family.
3. Whether other children in his classroom have
Acne ringworm.
4. Whether the child can read the medicine
A. General information directions.
1. Skin condition associated with increased
production of sebum from sebaceous glands at 157. Three school children have pediculosis capitus.
puberty. The school nurse has been instructing the
2. Lesions include pustules, papules, and parents of all three students on prevention.
comedones. Which statement made by one mother indicates
3. Majority of adolescents experience some an understanding of prevention?
degree of acne, mild to severe.
4. Lesions occur most frequently on face, neck, 1. I will put all of the stuffed animals in plastic
shoulders, and back. bags for 2 weeks.
5. Caused by a variety of interrelated factors 2. Since the sheets are now clean, the kids can
including increased activity of sebaceous share beds, too.
glands, emotional stress, certain medications, 3. Once I cut her hair, all the nits should be
menstrual cycle. gone.
6. Secondary infection can complicate healing of 4. I will now bathe my child every day to
lesions.
prevent reinfection.
7. There is no evidence to support the value of
eliminating any foods from the diet; if cause 158. Prior to discharge home with their new baby,
and effect can be established, however, a which of the following will demonstrate to the
particular food should be eliminated. nurse that the parents understand diaper rash
B. Assessment findings
prevention?
1. Appearance of lesions is variable and
fluctuating 1. They articulate that the baby should be
2. Systemic symptoms absent checked for wet diapers every half an
3. Psychologic problems such as social hour.
withdrawal, low self-esteem, feelings of being 2. They are observed wiping with soap and
ugly water at diaper changes.
C. Nursing interventions 3. The mother discusses needs to use tight
1. Discuss OTC products and their effects. rubber pants to keep diapers from
2. Instruct child in proper hygiene leaking.
(handwashing, care of face, not to pick or
squeeze any lesions). 4. The father wipes carefully and uses a mild
3. Demonstrate proper administration of topical ointment to protect the skin.
ointments and antibiotics if indicated.
159. A 3-year-old girl has had eczema since 4 months
of age. Which statement made by her father
indicates to the nurse that he understands the
Sample Questions management of eczema?
1. Benadryl should be given every night before
bedtime.
155. A 2-year-old was recently found to have 2. Its beneficial to keep her in the bubble bath
impetigo. What measures should be given the for as long as possible each day.
highest priority to prevent its spread while in
3. Typical eruption areas that need to be
the hospital?
treated include flexor surfaces of
1. Keeping it covered. joints.
2. Good handwashing. 4. Hot water is better in which to
3. Applying A&D ointment. bathe.
4. Placing the child in isolation.

5 520 NCLEX-RN Review


53155_05_Ch05_p421-532.qxd 2/27/09 1:32 PM Page 521

157. 1. Stuffed animals can harbor eggs and cause


Answers and Rationales reinfection.

158. 4. Careful cleansing on delicate skin and use of


155. 2. Good handwashing is of paramount ointments helps to preserve the skins integrity.
importance in preventing its spread.
159. 3. These are the joint areas typically affected in
156. 1. Pets are known to be carriers of ringworm. the childhood years.

Pediatric Oncology

OVERVIEW whether the disease is catching and


whether their friend will die
A. Cancer is the leading cause of death from disease D. Recurrence
in children from 114 years. 1. An event of enormous magnitude and a cause
B. Incidence of severe disappointment
1. 6000 children develop cancer per year. 2. May occur while still on treatment or after
2. 3000 children die from cancer annually. treatment has been completed
3. Boys are affected more frequently. E. Death
C. Leukemia is the most frequent type of childhood
cancer, followed by tumors of the CNS.
D. Etiologic factors include environmental agents, ASSESSMENT
viruses, familial/genetic factors, and host
factors. History
A. Family history: some cancers suggest patterns of
Major Stressful Events inheritance
Five events have been identified as major stressors: B. Prenatal exposure
A. Diagnosis: child is initially hospitalized to C. Children with chromosomal disorders have a
determine extent of disease, plan course of higher-than-average incidence of cancer
treatment, and to educate child and family. D. History may elicit symptoms that have been
B. Treatment: multimodal present for a period of time
1. May include surgery, radiation, chemotherapy E. Presenting problems: symptoms may include:
2. Side effects are serious and unpleasant; 1. Fever, pain, bleeding
child/family may complain that the treatment 2. Abdominal mass
is worse than the disease. 3. Night sweats, weight loss
C. Remission: child is without evidence of disease, 4. Hematuria, hypertension
treatment continues; goals for this period
include: Physical Examination
1. Maintenance of normal family patterns:
discipline and usual household chores A. General appearance
2. Maintenance of relationships among family 1. Skin: note color, bruises, or petechiae
and friends 2. Neurologic status: note fatigue, activity level,
a. Parents marriage may be strained behavior, headache, dizziness, gait
b. Siblings may feel neglected or jealous disturbances
3. Attendance at school 3. Pain: guarding of any body part, changes in
a. Child may fear rejection by peers due to range of motion
change in appearance or not being able to B. Measure vital signs including BP
keep up C. Plot height and weight on growth chart
b. Teacher may be unsure as to what to say or D. Inspect and palpate abdomen; note enlargement of
how to treat the child liver and spleen
c. Classmates need to be prepared for childs E. Palpate for enlarged lymph nodes
return; may have fears/concerns about F. Inspect eyes for nystagmus

PEDIATRIC NURSING

5 521
53155_05_Ch05_p421-532.qxd 2/27/09 1:32 PM Page 522

Laboratory/Diagnostic Test Chemotherapy


A. Blood studies, e.g., CBC Almost all pediatric cancer clients receive some form
B. X-rays, bone scans, CT scans, MRI, ultrasound of drug therapy.
C. Lumbar puncture A. Childhood cancers are more sensitive and
D. Bone marrow aspiration responsive to drugs than are adult cancers.
B. Childhood cancers tend to metastasize early and
systemic treatment is needed in addition to
ANALYSIS localized treatment.
Nursing diagnoses for the child with cancer may
include:
A. Risk for infection
Hematopoietic Stem Cell
B. Risk for injury Transplantation (HSCT)
C. Fear/anxiety [Previously known as Bone
D. Disturbed body image
E. Deficient knowledge Marrow Transplantation]
A. General information
PLANNING AND IMPLEMENTATION 1. Treatment alternative for a variety of
childhood diseases including:
Goals a. Definite: acute lymphoblastic leukemia,
A. Child will be free from infection acute nonlymphocytic leukemia, severe
B. Child will be free from pain aplastic anemia, immunodeficiencies, and
C. Optimum developmental level will be achieved malignant infantile osteopetrosis
D. Family will develop effective coping strategies b. Possible: chronic myelogenous leukemia,
solid tumors, some hematologic
disorders, and some inherited metabolic
disorders
Interventions 2. Types
a. Autologous: client transplanted with own
Surgery harvested marrow
A. May be performed for tumor removal, to obtain a b. Syngeneic: transplant between identical
biopsy, to determine extent of disease, or for twins
palliation c. Allogeneic: transplant from a genetically
B. Often used in conjunction with radiation or nonidentical donor
chemotherapy 1) Most common transplant type
2) Sibling most common donor
3. Procedure
Radiation Therapy a. Donor suitability determined through
A. Primarily used in children to improve prognosis tissue antigen typing; includes human
B. Goal is to achieve maximum effect on tumor while leukocyte antigen (HLA) and mixed
sparing normal tissue leukocyte culture (MLC) typing.
1. May be used for palliative relief from pain, b. Donor bone marrow is aspirated from
disfigurement multiple sites along the iliac crests under
2. May be curative, destroys cancer cells/reduces general anesthesia.
size of tumor c. Donor marrow is infused IV into the
3. Frequently used as an adjunct to recipient.
chemotherapy and surgery 4. Early evidence of engraftment seen during the
4. Must weigh gain versus risks of permanent second week posttransplant; hematologic
damage to normal tissue reconstitution takes 46 weeks; immunologic
5. Infants particularly susceptible to developing reconstitution takes months.
skeletal deformities in later years as a result of 5. Hospitalization of 2 or 3 months required.
radiation 6. Prognosis is highly variable depending on
6. Complications to growing child include indication for use.
scoliosis, arrested skeletal development, and B. Complications
pulmonary fibrosis (depends on site radiated) 1. Failure of engraftment
7. Dosage range varies; may be as low as 1000 rad 2. Infection: highest risk in first 34 weeks
to relieve bone pain in a specific area, to as high 3. Pneumonia: nonbacterial or interstitial
as 7000 rad to achieve a cure in Ewings sarcoma pneumonias are principal cause of death
8. Usually performed 5 days a week for 26 weeks during first 3 months posttransplant

5 522 NCLEX-RN Review


53155_05_Ch05_p421-532.qxd 2/27/09 1:32 PM Page 523

4. Graft vs host disease (GVHD): principal 2. Provide mouth care for stomatitis and
complication; caused by an immunologic mucositis (severe mucositis develops about
reaction of engrafted lymphoid cells against 5 days after irradiation).
the tissues of the recipient a. Note tissue sloughing, bleeding, changes in
a. Acute GVHD: develops within first color.
100 days posttransplant and affects skin, b. Provide mouth rinses, viscous lidocaine,
gut, liver, marrow, and lymphoid tissue and antibiotic rinses.
b. Chronic GVHD: develops 100400 days c. Do not use lemon and glycerin swabs.
posttransplant; manifested by multiorgan d. Administer parenteral narcotics as ordered
involvement if necessary to control pain.
5. Recurrent malignancy e. Provide care every 2 hours or as needed.
6. Late complications such as cataracts, 3. Provide skin care: skin breakdown may result
endocrine abnormalities from profuse diarrhea from the TBI.
C. Nursing care: pretransplant 4. Monitor carefully for bleeding.
1. Extensive time must be spent with child/ a. Check for occult blood in emesis and stools.
parents in preparing for this procedure. b. Observe for easy bruising, petechiae on
2. Recipient immunosuppression attained with skin, mucous membranes.
total body irradiation (TBI) and chemotherapy c. Monitor changes in vital signs.
to eradicate existing disease and create space d. Check platelet count daily.
in host marrow to allow transplanted cells to e. Replace blood products as ordered (all
grow. blood products should be irradiated).
3. Provide protected environment. 5. Maintain fluid and electrolyte balance and
a. Child should be in a laminar air flow room promote nutrition.
or on strict reverse isolation; surveillance a. Measure I&O carefully.
cultures done twice a week. b. Provide adequate fluid, protein, and
b. Encourage use of toys and familiar objects; caloric intake.
they must be sterilized before being c. Weigh daily.
brought into the room. d. Administer fluid replacement as ordered.
c. Encourage frequent contact with e. Monitor hydration status: check skin
schoolteacher/play therapist. turgor, moisture of mucous membranes,
d. Introduce new people where they can be urine output.
seen, but outside childs room so child can f. Check electrolytes daily.
see what they look like without isolation g. Check urine for glucose, ketones, protein.
garb. h. Administer antidiarrheal agents as needed.
4. Monitor central lines frequently; check 6. Provide client teaching and discharge
patency and observe for signs of infection planning concerning:
(fever, redness around site). a. Home environment (e.g., cleaning, pets,
5. Provide care for the child receiving visitors)
chemotherapy and radiation therapy to induce b. Diet modifications
immunosuppression. c. Medication regimen: schedule, dosages,
a. Administer chemotherapy as ordered, effects, and side effects
assist with radiation therapy if required. d. Communicable diseases and
b. Monitor side effects and keep child as immunizations
comfortable as possible. e. Daily hygiene and skin care
c. Monitor carefully for potential infection. f. Fever
d. Child will become very ill; prepare g. Activity
parents.
D. Nursing care: posttransplant
1. Prevent infection. STAGES OF CANCER TREATMENT
a. Maintain protective environment.
b. Administer antibiotics as ordered. A. Induction
c. Assess all mucous membranes, wounds, 1. Goal: to remove bulk of tumor
catheter sites for swelling, redness, 2. Methods: surgery, radiation/chemotherapy,
tenderness, pain. bone marrow transplant
d. Monitor vital signs frequently (every 3. Effects: often the most intensive phase; side
14 hours as needed). effects of treatment are potentially life
e. Collect specimens for cultures as needed threatening
and twice a week. B. Consolidation
f. Change IV set-ups every 24 hours. 1. Goal: to eliminate any remaining malignant

5
cells

PEDIATRIC NURSING 523


53155_05_Ch05_p421-532.qxd 2/27/09 1:32 PM Page 524

2. Methods: often chemotherapy/radiation 4. Provide contact with another parent or an


therapy organized support group such as Candlelighters.
3. Effects: side effects will still be evident 5. Try to keep daily life as normal as possible.
C. Maintenance C. Minimize side effects of treatment.
1. Goal: to keep child disease free 1. Skin breakdown
2. Method: chemotherapy (this phase may last for a. Keep clean and dry; wash with warm
several years) water, no soaps or creams.
D. Observation b. Do not wash off radiation markings.
1. Goal: to monitor the child at intervals for c. Avoid exposure to sunlight.
evidence of recurrent disease and d. Avoid all topical agents with alcohol
complications of treatment (perfumes and powders).
2. Method: treatment is complete; child may e. Do not use electric heating pads or hot
continue in this stage indefinitely water bottles.
E. Late effects of treatment 2. Bone marrow suppression
1. Impaired growth and development, usually a. Decreased RBCs
related to radiation of growth centers 1) Allow child to determine activities.
2. CNS damage resulting in intellectual, 2) Provide frequent rest periods.
psychologic, or neurologic sequelae b. Decreased WBCs
3. Impaired pubertal development including 1) Avoid crowds, isolate from children
hormonal or reproductive problems with known communicable disease.
4. Development of secondary malignancy 2) Evaluate any potential site of infection.
5. Psychologic problems (poor self-esteem, 3) Monitor temperature elevations.
depression, anxiety) related to living with a c. Decreased platelets
life-threatening disease and complex treatment 1) Make environment safe.
regimen 2) Select activities that are physically safe.
3) Avoid use of salicylates.
Side Effects d. Administer transfusions as ordered.
e. Interpret peripheral blood counts to guide
A. From combined effects of treatment: nausea, specific interventions and precautions.
vomiting, diarrhea, alopecia, anemia (low RBCs), 3. Nausea and vomiting
increased susceptibility to infection (low WBCs), a. Administer antiemetic at least half an hour
bleeding (low platelets), stomatitis, mucositis, before chemotherapy; repeat as necessary.
pain, learning problems b. Encourage relaxation techniques.
B. From radiation (findings differ according to site c. Eat light meal prior to administration of
radiated): sleepiness, reddened skin therapy.
C. From chemotherapy: drug toxicity specific to d. Ensure adequate oral intake or administer
agents used; liver and renal toxicity IV fluids as necessary.
D. Developmental: behavior problems, avoidance of 4. Alopecia
school and friends, low self-esteem or poor self- a. Reduce trauma of hair loss (especially in
image children over age 5 years).
b. Buy wig before hair falls out.
Nursing Interventions c. Discuss various head coverings with boys
and girls.
A. Help child cope with intrusive procedures. d. Avoid exposing head to sunlight.
1. Provide information geared to developmental e. Discuss feelings.
level and emotional readiness. 5. Stomatitis, mucositis (see Bone Marrow
2. Explain what is going to happen, why it is Transplant).
necessary, and how it will feel. 6. Nutrition deficits
3. Allow child to handle and manipulate a. Establish baseline prior to start of treatment.
equipment. b. Measure height and weight regularly.
4. Use needle play as indicated. c. Provide small, frequent meals.
5. Allow child some control in situations (e.g., d. Consult dietitian as needed.
positioning, selecting injection site). e. Provide high-calorie, high-protein
B. Support child and parents. supplements.
1. Maintain frequent clinical conferences to keep 7. Developmental delays
all informed. a. Discuss limit setting, discipline.
2. Always tell the truth. b. Some behavior problems might be side
3. Acknowledge feelings and encourage effects of drug therapy.
child/family to express them, assure them that c. Facilitate return to school as soon as able.

5
feelings are normal. d. Realize changing needs of child.

524 NCLEX-RN Review


53155_05_Ch05_p421-532.qxd 2/27/09 1:32 PM Page 525

CANCERS 5. Enlarged spleen and liver


6. Abdominal pain with weight loss and anorexia
7. Bone pain due to expansion of marrow
Leukemia D. Nursing interventions
A. General information 1. Monitor for signs of bleeding, anemia,
1. Most common form of childhood cancer thrombocytopenia, DIC.
2. Peak incidence is between 2 and 3 years of age 2. Provide care for the child receiving
3. Proliferation of abnormal white blood cells chemotherapy and radiation therapy.
that do not mature beyond the blast phase 3. Provide support for child/family; needs will
4. In the bone marrow, blast cells crowd out change as treatment progresses.
healthy white blood cells, red blood cells, and 4. Support child during painful procedures
platelets, leading to bone marrow depression (frequent bone marrow aspirations, lumbar
5. Blast cells also infiltrate other organs, most punctures, venipunctures needed).
commonly the liver, spleen, kidneys, and a. Use distraction, guided imagery.
lymph tissue b. Allow child to retain as much control as
6. Symptoms reflect bone marrow failure and possible.
associated involvement of other organs c. Administer sedation prior to procedure as
7. Types of leukemia, based on course of disease ordered.
and cell morphology
a. Acute lymphocytic leukemia (ALL) Brain Tumors
1) 75% of childhood leukemia
2) Malignant change in the lymphocyte A. General information
or its precursors 1. A space-occupying mass in the brain tissue;
3) Acute onset may be benign or malignant
4) 95% chance of obtaining remission 2. Males affected more often; peak age 37 years
with treatment 3. Second most prevalent type of cancer in
5) 75% chance of surviving 5 years or children
more 4. Cause unknown; genetic and environmental
6) Prognostic indicators include: initial factors may play a role; familial tendency for
white blood count (less than brain tumors, which are found with
10,000/mm3), childs age (29 years), preexisting neurocutaneous disorders.
histologic type, sex 5. Two thirds of all pediatric brain tumors are
b. Acute nonlymphocytic leukemia (ANLL) beneath the tentorium cerebelli (in the
1) Includes granulocytic and monocytic posterior fossa), often involving the
types cerebellum or brain stem.
2) 6080% will obtain remission with 6. Three fourths of brain tumors in children are
treatment gliomas (medulloblastoma and astrocytoma).
3) 3040% cure rate B. Types (See Figure 512.)
4) Prognostic indicators less clearly 1. Medulloblastoma: highly malignant tumor
defined usually found in cerebellum; runs a rapid
B. Medical management course
1. Diagnosis: blood studies, bone marrow biopsy a. Findings include increased ICP plus
2. Treatment stages unsteady walk, ataxia, anorexia, early
a. Induction: intense and potentially life morning vomiting
threatening b. Treated with radiation because complete
b. CNS prophylaxis: to prevent central removal is impossible
nervous system disease. Combination of 2. Astrocytoma: a benign, cystic, slow-growing
radiation and intrathecal chemotherapy. tumor usually found in cerebellum
c. Maintenance: chemotherapy for 2 to a. Onset of symptoms is insidious.
3 years. b. Findings include focal disturbances,
C. Assessment findings papilledema, optic nerve atrophy,
1. Anemia (due to decreased production of blindness.
RBCs), weakness, pallor, dyspnea 3. Ependymoma: a usually benign tumor that
2. Bleeding (due to decreased platelet arises in the ventricles of the brain, causing
production), petechiae, spontaneous bleeding, noncommunicating hydrocephalus and
ecchymoses damage (by pressure) to other vital tissues of
3. Infection (due to decreased WBC production), the brain
fever, malaise 4. Craniopharyngioma: tumor that arises from
4. Enlarged lymph nodes remnants of embryonic tissue near the

PEDIATRIC NURSING

5 525
53155_05_Ch05_p421-532.qxd 2/27/09 1:32 PM Page 526

Corpus Callosum
astrocytoma
oligodendroglioma Lateral Ventricle
lipoma ependymoma
choroid plexus papilloma

Third Ventricle and Area


ependymoma Cerebrum
choroid plexus papilloma astrocytoma
ependymoma
oligodendroglioma
(primitive neuroectodermal
tumors)

Optic Chiasm Pineal Area


astrocytoma germinoma
teratoma

Cerebellum
Pituitary Area medulloblastoma
craniopharyngioma astrocytoma
germ cell tumor dermoid cyst
pituitary adenoma
Brain Stem
astrocytoma
glioblastoma multiforme Fourth Ventricle
ependymoma
choroid plexus papilloma
dermoid cyst

Figure 5-12 Location of common childhood brain tumors. Source: From American Brain Tumor Association.
A primer of brain tumors: A patients reference manual. (2004). Des Plaines, IL: American Brain Tumor Association.
Used with permission.

pituitary gland in the sella turcica, causes 2. A definite diagnosis is difficult in children
pressure on the third ventricle because of the elasticity of childs skull and
a. Decreased secretion of ADH causes generally poor coordination of the young
diabetes insipidus (these children may child.
need Pitressin). 3. A decrease in school performance may be the
b. Additional symptoms include altered first sign.
growth pattern, visual difficulties, 4. Increased ICP
difficulty regulating body temperature. a. Morning headache
5. Brain stem glioma: slow-growing tumor, b. Morning vomiting without nausea;
indicated by cranial nerve palsies, ataxia vomiting without relation to feeding
C. Medical management schedule; projectile vomiting
1. Surgery: some tumors entirely or partially c. Personality changes
resected; others are not amenable to surgery d. Diplopia
because of proximity to vital brain parts 1) Difficult to assess in young children
2. Radiation therapy: often used to shrink tumors 2) Observe child for tilting of head,
3. Chemotherapy: vincristine, lomustine, closing or covering one eye, rubbing
procarbazine, intrathecal methotrexate; not as the eyes, or impaired eye-hand
effective with brain tumors as with other coordination
childhood cancers e. Papilledema: a late sign
D. Assessment findings f. Increased blood pressure with decreased
1. Symptoms dependent on location and type of

5
pulse: also a late sign
tumor.

526 NCLEX-RN Review


53155_05_Ch05_p421-532.qxd 2/27/09 1:32 PM Page 527

g. Cranial enlargement b. Provide stool softeners as ordered.


1) More readily noticeable prior to c. Avoid enemas, which increase ICP.
18 months when suture lines are 5. Provide care for the child undergoing brain
still open surgery.
2) Bulging, tense, pulsating fontanels 6. Provide care for the child undergoing radiation
3) Widened suture lines or chemotherapy.
4) 90% or more on head circumference 7. Provide client teaching and discharge
chart planning concerning:
5. Focal signs and symptoms a. Diagnostic tests (instruction needs to be
a. Ataxia appropriate to the childs developmental
1) In cerebellar tumors level)
2) May not be readily identified because 1) Machines will make clicking sounds
of uncoordinated movements of young 2) Wires attached to the head for an EEG
children will not electrocute child
b. Muscle strength 3) Head is immobilized for a CT scan
1) Weakness with cerebellar tumors 4) The use of contrast dye and expected
2) Weakness, spasticity, and paralysis of sensations if used
lower extremities with cerebral or 5) Need to lie still with the technician
brain stem tumors out of the room for most tests (younger
3) Change in handedness, posture, or children will be sedated for fuller
manual coordination: may be early cooperation)
signs b. Importance of family discussion of fears/
c. Head tilt anxiety about surgery and prognosis
1) In posterior fossa tumors c. Need to assist in implementing childs
2) Early sign of visual impairment interaction with peers
3) Associated with nuchal rigidity d. Available support groups and community
4) Due to traction on the dura agencies
d. Ocular signs
1) Nystagmus: corresponds to the same Brain Surgery
side as the infratentorial lesion
2) Diplopia/strabismus: from palsy of A. General information
cranial nerve VI with brain stem 1. Indications
glioma or increased ICP a. Removal of a tumor
3) Visual field deficit (child does not b. Evacuation of a hematoma
react to activity on periphery of c. Removal of a foreign body or skull
vision): with craniopharyngiomas fragments resulting from trauma
e. Seizures: with cerebral tumors d. Aspiration of an abscess
6. Diagnostic tests e. Insertion of a shunt
a. Skull X-ray reveals presence and location B. Nursing interventions: preoperative
of tumor 1. Assess the childs understanding of the
b. CT scan (with or without contrast dye) procedure; have the child draw a picture, tell a
reveals position, consistency, size of tumor, story; observe doll play.
and effect on surrounding tissue 2. Explain the procedure in terms according to
c. EEG may show seizure activity the childs developmental level.
E. Nursing interventions 3. Allow the child to visit the operating
1. Obtain baseline vital signs and perform room/intensive care unit, if permitted,
thorough neurologic assessment; monitor vital depending on the childs emotional and
signs and neurologic status frequently. developmental levels.
2. Prevent injury/complications. 4. Explain that pre-op symptoms such as
a. Institute seizure precautions. headache and ataxia may be temporarily
b. Monitor for fluid and electrolyte imbalance aggravated.
from vomiting. 5. Advise child/parents that blindness may
c. Observe for increased ICP. result, depending on the location of the
d. Provide safety measures (bed rails up). tumor.
3. Promote comfort/relief of headache. 6. Inform the child/parents that the head will be
a. Decrease environmental stimuli. shaved; long hair may be saved; hats or scarves
b. Administer analgesics as ordered. may be used to cover the head once the
4. Prevent constipation (straining increases ICP). dressings are removed.
a. Provide appropriate foods and fluids as 7. Support the child/family if a tumor cannot be

5
ordered. totally removed.

PEDIATRIC NURSING 527


53155_05_Ch05_p421-532.qxd 2/27/09 1:32 PM Page 528

8. Provide instruction about radiation and a. Lymphangiogram determines involvement


chemotherapy (may need to be delayed of all lymph nodes (reliable in 90% of
because detail may be overwhelming). clients); is helpful in determining radiation
9. Explain to the child/parents about the post-op fields
dressing, monitoring devices, and possibility b. Staging via laparotomy and biopsy
of facial edema. 1) Stage I: single lymph node involved;
C. Nursing interventions: postoperative usually in neck; 9098% survival
1. Prevent injury/complications. 2) Stage II: involvement of 2 or more
a. Monitor vital signs and neuro status lymph nodes on same side of
frequently until stable. diaphragm; 7080% survival
b. Apply hypothermia blanket as ordered. 3) Stage III: involvement of nodes on both
c. Assess respiratory status/signs of infection. sides of diaphragm; 50% survival
d. Observe the dressing for discharge/ 4) Stage IV: metastasis to other organs
hemorrhage. c. Laparotomy and splenectomy
e. Close or cover eyes, apply ice, instill saline d. Lymph node biopsy to identify presence of
drops or artificial tears. Reed-Sternberg cells and for histologic
f. Position as ordered according to the classification
location of the tumor and type of surgery. 2. Radiation: used alone for localized disease
g. Assess for increased ICP. 3. Chemotherapy: used in conjunction with
h. Institute seizure precautions. radiation therapy for advanced disease
2. Promote comfort. C. Assessment findings
a. Decrease environmental stimuli. 1. Major presenting symptom is enlarged nodes
b. Administer analgesics as ordered, first in lower cervical region; nodes are nontender,
assessing LOC. firm, and movable
3. Promote adequate nutrition. 2. Recurrent, intermittent fever
a. Administer fluids as ordered. 3. Night sweats
b. Monitor I&O. 4. Weight loss, malaise, lethargy
c. Provide diet as ordered. 5. Pruritus
4. Provide emotional support and encourage 6. Diagnostic test: presence of Reed-Sternberg
child/family to discuss prognosis. cells
5. Provide client teaching and discharge D. Nursing interventions
planning concerning: 1. Provide care for child receiving radiation
a. Wound care therapy.
b. Signs of increased ICP 2. Administer chemotherapy as ordered and
c. Activity level monitor/alleviate side effects.
d. Sensation and time period of hair growth 3. Protect client from infection, especially if
e. Peer acceptance splenectomy performed.
f. Radiation/chemotherapy, if indicated 4. Provide support for child/parents; specific
g. Availability of support groups/community needs of adolescent client must be considered.
agencies
Non-Hodgkins Lymphoma
Hodgkins Lymphoma
A. General information
A. General information 1. Tumor originating in lymphatic tissue
1. Malignant neoplasm of lymphoid tissue, 2. Significantly different from Hodgkins
usually originating in localized group of lymphoma
lymph nodes; a proliferation of lymphocytes a. Control of primary tumor is difficult
2. Metastasizes first to adjacent lymph nodes b. Disease is diffuse, cell type
3. Cause unknown undifferentiated
4. Most prevalent in adolescents; accounts for c. Tumor disseminates early
5% of all malignancies d. Includes wide range of disease entities:
5. Prognosis now greatly improved for these lymphosarcoma, reticulum cell sarcoma,
children; influenced by stage of disease and Burkitts lymphoma
histologic type 3. Primary sites include GI tract, ovaries, testes,
6. Long-term treatment effects include increased bone, CNS, liver, breast, subcutaneous
incidence of second malignancies, especially tissues
leukemia and infertility 4. Affects all age groups.
B. Medical management B. Medical management
1. Diagnosis: extensive testing to determine stage, 1. Chemotherapy: multiagent regimens including

5
which dictates treatment modality cyclophosphamide (Cytoxan), vincristine,

528 NCLEX-RN Review


53155_05_Ch05_p421-532.qxd 2/27/09 1:32 PM Page 529

prednisone, procarbazine, doxorubicin, Neuroblastoma


bleomycin
2. Radiation therapy: primary treatment in A. General information
localized disease 1. A highly malignant tumor that develops from
3. Surgery for diagnosis and clinical staging embryonic neural crest tissue; arises anywhere
C. Assessment findings along the craniospinal axis, usually from the
1. Depend on anatomic site and extent of adrenal gland
involvement 2. Incidence
2. Rapid onset and progression a. One in 10,000
3. Many have advanced disease at b. Males slightly more affected
diagnosis c. From infancy to age 4
D. Nursing interventions: provide care for child 3. Staging
receiving chemotherapy, radiation therapy, and a. Stage I: tumor confined to the organ of origin
surgery. b. Stage II: tumor extends beyond primary
site but not across midline
c. Stage III: tumor extends beyond midline
Wilms Tumor (Nephroblastoma) d. Stage IV: tumor metastasizes to skeleton
A. General information (bone marrow), soft tissue (liver), and
1. Large, encapsulated tumor that develops in lymph nodes
the renal parenchyma, more frequently in left B. Medical management: depends on the staging of
kidney (usually unilateral) tumor and age of child; includes surgery, radiation
2. Originates during fetal life from therapy, chemotherapy
undifferentiated embryonic tissues C. Assessment findings vary, depending on the tumor
3. Peak age of occurrence: 13 years site and stage
4. Prognosis good if there are no metastases. 1. If in the abdomen, may initially resemble
B. Medical management Wilms tumor
1. Nephrectomy, with total removal of tumor 2. Local signs and symptoms caused by pressure
2. Postsurgical radiation in treatment of stages II, of the tumor on surrounding tissue
III, and IV; stage I disease does not usually 3. Metastatic manifestations
require radiation, but it may be used if the a. Ocular: supraorbital ecchymosis,
tumor histology is unfavorable. periorbital edema, exophthalmos
3. Postsurgical chemotherapy: vincristine and b. Cervical or supraclavicular
daunorubicin, doxorubicin lymphadenopathy
C. Assessment findings c. Bone pain: may or may not occur with
1. Staging bone metastasis
a. Stage I: limited to kidney d. Nonspecific complaints; pallor, anorexia,
b. Stage II: tumor extends beyond kidney, but weight loss, irritability, weakness
is completely encapsulated 4. Diagnosis usually made after metastasis has
c. Stage III: tumor confined to abdomen occurred
d. Stage IV: tumor has metastasized to lung, 5. Diagnostic tests
liver, bone, or brain a. X-rays of the head, chest, or abdomen
e. Stage V: bilateral renal involvement at reveal presence of primary tumor or
diagnosis metastases
2. Usually mother notices mass while bathing or b. IVP: if tumor is adrenal, shows a
dressing child; nontender, usually midline downward displacement of the kidney on
near liver the affected side
3. Hypertension and possible hematuria, anemia, c. Bone marrow aspiration: to rule out
and signs of metastasis metastasis; neuroblasts have a clumping
4. Diagnostic test: IVP reveals mass pattern
D. Nursing interventions d. CBC: RBCs and platelets decreased
1. Do not palpate abdomen to avoid possible e. Coagulation studies: abnormal due to
dissemination of cancer cells. thrombocytopenia
2. Handle child carefully when bathing and f. Catecholamine excretion: VMA levels in
giving care. urine increased
3. Provide care for the client with a 1) Child must not ingest vanilla,
nephrectomy; usually performed within chocolate, bananas, or nuts for 3 days
2448 hours of diagnosis. prior to the test
4. Provide care for the child receiving 2) 24-hour urine specimen needed
chemotherapy and radiation therapy. D. Nursing interventions: same as for leukemia and

5
brain tumors.

PEDIATRIC NURSING 529


53155_05_Ch05_p421-532.qxd 2/27/09 1:33 PM Page 530

Bone Tumors D. Nursing interventions


1. Promote exercise of affected limb to maintain
Osteogenic Sarcoma function.
2. Avoid activities that may cause added stress to
A. General information
affected limb.
1. Primary bone tumor arising from the
mesenchymal cells and characterized by
formation of osteoid (immature bone)
2. Invades ends of long bones, most frequently Sample Questions
distal end of femur or proximal end of tibia
3. Occurs more often in boys, usually between
ages 10 and 20 years 160. A 10-year-old is being prepared for a bone
4. Lungs most frequent site of metastasis marrow transplant. Which statement by the boy
5. 5-year survival rate is 1020% will demonstrate to the nurse that he
B. Medical management understands this treatment?
1. Surgery: treatment of choice
a. Amputation: temporary prosthesis used 1. Ill be much better after this blood goes to
immediately after surgery; permanent one my bones.
usually fitted a few weeks later 2. I wont feel too good until my body makes
b. Limb salvage procedures healthy cells.
c. Lung surgery if there are metastases 3. This will help all of the medicine they give
2. Radiation: only in areas where tumor is not me to work better.
accessible to surgery 4. You wont have to wear a mask and gown
3. Chemotherapy: adjuvant therapy being
after my transplant.
studied
C. Assessment findings 161. A 4-year-old has leukemia. Her mother
1. Insidious pain, increasing with activity, understands the white count involvement in this
gradually becoming more severe disease but doesnt understand why her child
2. Tender mass, warm to touch; limitation of
has bruises and anemia. Which statement will be
movement
the best explanation for the bruises and anemia?
3. Pathologic fractures
D. Nursing interventions 1. All blood cells are made in the bone marrow
1. Prepare child for amputation: discuss fears, and therefore all types will be affected.
concerns, and facts of procedure; answer 2. The anemia is because her child hasnt been
questions regarding prosthetic devices, limited eating well; the bruises are from the multiple
activity needle sticks.
2. Assure child that phantom limb pain will 3. They are related to inactivity.
subside
4. This is indicative that the end is near.
Ewings Sarcoma 162. A 14-year-old has had an exacerbation of acute
A. General information lymphocytic leukemia. What is the primary
1. Primary tumor arising from cells in bone effect of leukemia on the bone marrow?
marrow 1. Crowding out of normal bone marrow
2. Invades bone longitudinally, destroying bone cells.
tissue; no new bone formation 2. Proliferation of cells producing blood
3. Femur most frequently affected site components.
4. More common in males, between ages 5 and
3. A selective reduction in the number of
15 years
neutrophils.
5. Lungs most frequent site of metastasis
B. Medical management 4. Leukopenia, thrombocytopenia, and anemia.
1. High-dose radiation is primary treatment
2. Chemotherapy 163. A 14-year-old girl has acute lymphocytic
3. Value of surgery presently being reassessed leukemia and is admitted. She is terminally ill.
C. Assessment findings What would be an appropriate nursing action?
1. Pain and swelling 1. Leave her alone as much as possible and
2. Palpable mass, may be tender, warm to whisper when in her room in order not to
touch disturb her.
3. 1030% of clients have metastatic disease at 2. Assist her in giving away her possessions to
time of diagnosis

5
friends and family.

530 NCLEX-RN Review


53155_05_Ch05_p421-532.qxd 2/27/09 1:33 PM Page 531

3. Encourage her parents to explain to her her care, which of the following would be
5-year-old sister that she will be asleep included in a nursing care plan?
for a long time. 1. Good handwashing by visitors and staff.
4. Reduce emotional stress by not having the 2. Daily CBCs drawn.
childs parents/family participate in her care. 3. Daily physical therapy.
164. A 10-year-old is receiving cranial irradiation for 4. Restriction of activity.
a brain tumor. He has developed alopecia.
169. A 5-year-old boy is newly diagnosed with an
Which of the following is an appropriate nursing
astrocytoma brain tumor. His symptoms include
intervention?
headache, nausea, and seizures. Based on this
1. Have the child identify famous movie stars information, which nursing diagnosis would be
and sports heroes who are bald. most appropriate for him?
2. Assure the child that his hair will grow in 1. High risk for infection.
before he leaves the hospital.
2. High risk for injury.
3. Wrap a bandage around his head.
3. Anticipated grief.
4. Help him select a variety of hats.
4. Impaired physical mobility.
165. A 6-year-old girl is newly diagnosed with acute
170. Which of the following statements made by
lymphoid leukemia (ALL). During your
parents of an 8-year-old boy who just had
assessment, which of the following signs and
surgery for a brain tumor reflect understanding
symptoms would you expect?
of safety needs?
1. Fever, pallor, bone and joint pain.
1. We will obtain a tutor to teach him at home.
2. Fever, ruddy complexion, petechiae.
2. We will not allow him to participate in
3. Abdominal pain, cystitis, swollen joints. sports anymore.
4. Enlarged lymph nodes, low grade fever, night 3. We will tell our other children to let him
sweats. have his way and not upset him.
166. A 12-year-old girl with ALL is receiving 4. He will wear a helmet for sports.
induction therapy with vincristine, prednisone,
171. A 16-year-old boy is admitted with Hodgkins
and L-asparaginase. She presents with
lymphoma. Which assessment finding would
paresthesia, alopecia, and moon face. Which of
you expect?
the following nursing diagnoses would be most
appropriate for this child? 1. Small, tender lymph nodes in the groin.
1. High risk for injury. 2. Enlarged, firm nontender nodes in the
supraclavicular area.
2. Impaired physical mobility.
3. Enlarged, tender nodes all over the body.
3. Body image disturbance.
4. Small, nontender, nonmoveable nodes in the
4. Altered nutrition: less than body
cervical area.
requirements.
172. A 3-year-old with a Wilms tumor is returning to
167. You are caring for a 10-year-old with ALL who
the unit after surgery to remove the tumor.
underwent a bone marrow transplant. To
Which of the following actions have the highest
provide a safe, effective care environment, what
priority in caring for this child?
would be included in a plan of care?
1. Maintaining NPO.
1. Rectal temperature every 4 hours to monitor
for infection. 2. Frequent blood pressure.
2. Encouraging the child to go to the playroom 3. Turning every 4 hours.
to limit isolation. 4. Administering pain medication every 4 hours.
3. Use of a pressure-reducing mattress.
173. A child is to receive radiation therapy following
4. Inserting a Foley catheter to monitor output. surgery for Wilms tumor. Which of the
following measures would be important to
168. A 15-year-old girl with ALL has been on
include in the care plan prior to radiation
maintenance therapy for 6 months. She is
therapy?
receiving chemotherapy of L-asparaginase,
methotrexate, and cytarabine. Her absolute 1. Give compazine every 6 hours for nausea.
neutrophil count is 500/mm3. In planning for 2. Place a sign over the bed that reads no

5
needle punctures.

PEDIATRIC NURSING 531


53155_05_Ch05_p421-532.qxd 2/27/09 1:33 PM Page 532

3. Practice lying in the required position. 168. 1. Because of the maintenance therapy and
4. Encourage play appropriate to age. neutrophil count, this client may have bone
marrow suppression, which increases her risk
174. A 6-year-old boy with Ewings sarcoma has just for infection. Good handwashing is essential to
finished his course of chemotherapy. Which of help prevent infection.
the following statements by his parents indicate
they understand the signs of complications from 169. 2. Seizure precautions should be instituted to
the chemotherapy? prevent an injury.
1. He will be playing football next week.
170. 4. To protect the skull while it is healing, a child
2. We will keep him on a liquid diet until he may need to wear a padded helmet for active
feels better. sports.
3. We understand he is more susceptible to
infections; we will keep him away from any 171. 2. The most common symptom of Hodgkins
sick family members. disease is enlarged, firm, nontender moveable
4. He will wear a baseball hat to bed. nodes in the supraclavicular area.

172. 2. Frequent blood pressure measurements are


needed to watch for signs of shock and as an
Answers and Rationales indication of the functioning of the remaining
kidney.

160. 2. The goal of a bone marrow transplant is to 173. 3. The child may stay in a fixed position during
have the donor cells produce functioning blood each therapy session, which may last 1020
cells for the client. minutes. Having the child practice the required
position prior to beginning radiation therapy can
161. 1. In leukemia, bone marrow is replaced by blast be helpful.
cells, resulting in decreased white cells, red
cells, and platelets. The bruises are due to the 174. 3. This client is likely to have bone marrow
childs decreased platelet count. suppression, which increases his risk for
infection and bleeding.
162. 1. Leukemia cells are capable of an increased
rate of production and a long cell life, causing
crowding out of the normal bone marrow cells. REFERENCES AND SUGGESTED READINGS
Cells producing normal blood components are
then unable to reproduce. American Academy of Pediatrics, Committee on Infectious
Diseases. (2003). The red book: Report of the committee on
163. 2. Adolescents who know they are dying infectious diseases. Elk Grove, IL: Author.
frequently want to give away their belongings. Axton, S., & Fugate, T. (2008). Pediatric nursing care plans for
the hospitalized child (3rd ed.). Upper Saddle River, NJ:
164. 4. Selecting hats to cover his head will help the Pearson Education.
Ball, J. W., & Bindler, R. C. (2007). Pediatric nursing: Caring for
child deal with the change in body image.
children (4th ed.). Upper Saddle River, NJ: Prentice-Hall.
Centers for Disease Control and Prevention. (2004). Most recent
165. 1. The signs and symptoms of leukemia are a
childhood immunization schedule can be found at: Retrieved
result of infiltration of the bone marrow. These on Sep. 7, 2008, from http://www.cdc.gov/nip/recs/
include fever, pallor, fatigue, anorexia, child-schedule.htm
petechiae, and bone and joint pain. Child passenger safety. Retrieved on Sep. 7, 2008, from
www.nhtsa.dot.gov
166. 3. This may be especially true for this child as Hockenberry, M. J., Wilson, D., Wilkenstein, M. L., & Kline,
she is entering adolescence. Her loss of hair and N. E. (2006). Wongs nursing care of infants and children
fat face will make her different from her (8th ed.). St. Louis, MO: Mosby.
Potts, N. & Mandleco, B. (2007). Pediatric nursing: Caring
friends. Adolescents need to belong and be
for children and their families (2nd ed.). Clifton Park, NY:
accepted by a group of peers. Delmar Learning.
Zitelli, B. J., & Davis, H. (2007). Atlas of pediatric physical
167. 3. Skin breakdown and impaired healing are diagnosis. (5th ed.). St. Louis, MO: Mosby.
common with bone marrow transplant. This is a
preventive measure for the integrity of the skin.

5 532 NCLEX-RN Review


53155_06_Ch06_p533-620.qxd 2/23/09 12:22 PM Page 533

U N I T 6

M AT E R N I T Y
AND FEMALE
REPRODUCTIVE
NURSING

This section covers the health care needs of females from adolescence UNIT OUTLINE
through late adulthood. Emphasis is placed on the childbearing cycle 534 Overview of Anatomy and
and the normal neonate, and frequently encountered health care Physiology of the Female
problems. Cultural differences are addressed. Reproductive System
538 The Childbearing Cycle
The unit begins with a review of the anatomy and physiology of
the female reproductive system as a basis for understanding the 543 The Antepartal Period
entire childbearing process. Nursing process is emphasized 560 Labor and Delivery
throughout, and nursing diagnoses are used to identify the 581 The Postpartum Period
clients health care needs and to select nursing interventions. 590 The Newborn
Nursing process always must be implemented with an 599 The High-Risk Infant
awareness of the interrelationship, during childbearing, of the 607 Conditions of the Female
maternal and fetal needs and their manifestations. The nurse Reproductive System
needs to keep in mind that interventions for the mother may
have an impact on the developing fetus, and vice versa.
Medications for maternal conditions may affect the fetus, and
fetal distress may require that the mother undergo surgery.

In this unit, a major assertion is that childbearing, for most


women, is a normal, healthy life event. Discomforts and
complications of childbearing also are covered, and these
conditions are presented after the content that reviews the
natural progression of events in pregnancy and childbearing. In
this manner, the factors that alter pregnancy from a normal life
event to a life crisis can be clearly identified.

In addition to childbearing, the unit includes a review of


frequently encountered health care conditions of women. These
conditions usually involve the reproductive system or its
accessory organs and include normally occurring states, such as
menopause, as well as pathologic conditions, such as
gynecologic cancer.
533
53155_06_Ch06_p533-620.qxd 2/23/09 12:23 PM Page 534

Overview of Anatomy and Physiology


of the Female Reproductive System

ANATOMY 1. Glands in the labia minora lubricate the


vulva.
2. The labia minora are very sensitive because of
External Structures their rich nerve supply.
See Figure 6-1. 3. The space between the labia minora is called
A. Mons veneris: rounded, soft, fatty, and loose the vestibule.
connective tissue over the symphysis pubis. Dark, D. Clitoris: small erectile organ located beneath the
curly pubic hair growth in typical triangular shape arch of the pubis, containing more nerve endings
begins here 1 to 2 years before the onset of than the glans penis; sensitive to temperature
menstruation. and touch; secretes a fatty substance called
B. Labia majora: lengthwise fatty folds of skin smegma.
extending from the mons to the perineum that E. Vestibule: area formed by the labia minora, clitoris,
protect the labia minora, the urinary meatus, and and fourchette, enclosing the openings to the
the vaginal introitus. urethra and vagina, Skenes and Bartholins glands;
C. Labia minora: thinner, lengthwise folds of easily irritated by chemicals, discharges, or
hairless skin, extending from the clitoris to the friction.
fourchette.

Figure 6-1 External structures of the female reproductive system

6 534 NCLEX-RN Review


53155_06_Ch06_p533-620.qxd 2/23/09 12:23 PM Page 535

1. Urethra: external opening to the urinary bladder 1. Underlying the perineum are the paired
2. Skenes glands (also called paraurethral muscle groups that form the supportive
glands): secrete a small amount of mucus; sling for the pelvic organs, capable of great
especially susceptible to infections distension during the birth process.
3. Bartholins glands: located on either side of the 2. An episiotomy can be made in the perineum
vaginal orifice; secrete clear mucus during if necessary during the birth process.
sexual arousal; susceptible to infection, as well
as cyst and abscess formation
4. Vaginal orifice and hymen: elastic, partial
Internal Structures
fold of tissue surrounding opening to the See Figure 6-2.
vagina A. Fallopian tubes: paired tubules extending from
F. Fourchette: thin fold of tissue formed by the the cornu of the uterus to the ovaries that serve
merging of the labia majora and labia minora, as the passageway for the ova. Mucosal lining
below the vaginal orifice. of tubes resembles that of vagina and uterus;
G. Perineum: muscular, skin-covered area between therefore, infection may extend from lower
vaginal opening and anus. organs.

Fallopian
tube

Ovary

Uterus

Bladder

Cervix

Vagina
Urethra

External
genitalia

Rectum

Figure 6-2 Internal structures of the female reproductive system

MATERNITY AND FEMALE REPRODUCTIVE NURSING

6 535
53155_06_Ch06_p533-620.qxd 2/23/09 12:23 PM Page 536

B. Uterus: hollow, pear-shaped muscular organ, freely The Breasts


movable in pelvic cavity, comprised of fundus,
corpus, isthmus, and cervix. Cervix has internal A. Paired mammary glands on the anterior chest wall,
and external os, separated by the cervical canal. between the second and sixth ribs, composed of
Wall of uterus has three layers. glandular tissue, fat, and connective tissue.
1. Endometrium: inner layer, highly vascular, shed B. Nipple and areola are darker in color than breasts.
during menstruation and following delivery C. Responsible for lactation after delivery.
2. Myometrium: middle layer, composed of smooth
muscle fibers running in three directions; expels
fetus during birth process, then contracts around PHYSIOLOGY
blood vessels to prevent hemorrhage
3. Parietal peritoneum: serous outer layer Menarche
C. Ovaries: oval, almond-sized organs on either side
of the uterus that produce ova and hormones. Onset of the menstrual cycle; puberty.
D. Vagina: muscular, distensible tube connecting
perineum and cervix; the birth canal. Menstrual Cycle
See Figure 6-3.
The Pelvis A. Complex pituitary/ovarian/uterine interaction.
Right and left innominate bones, sacrum, and coccyx B. Controls ripening and release of an ovum on a
form the bony passage through which the baby passes regular basis, and preparation for its fertilization
during birth. Relationship between pelvic size/shape and implantation in a thickened uterine lining
and baby may affect labor or make vaginal delivery (endometrium).
impossible. C. When fertilization/implantation do not occur, the
A. Pelvic measurements endometrium is shed (menstruation), and the cycle
1. True conjugate: from upper margin of begins again, due to follicle-stimulating hormone
symphysis pubis to sacral promontory, should (FSH) and luteinizing hormone (LH) from the
be at least 11 cm; may be obtained by X-ray or anterior pituitary.
ultrasound. D. Stages of cycle
2. Diagonal conjugate: from lower border of 1. Menstruation: first days of cycle when
symphysis pubis to sacral promontory; should endometrium is shed.
be 12.513 cm; may be obtained by vaginal 2. Proliferative phase: major hormone involved is
examination. estrogen, which influences build-up of
3. Obstetric conjugate: from inner surface of endometrium; also called follicular phase.
symphysis pubis, slightly below upper 3. Ovulation: release of ovum, usually 14 days
border, to sacral promontory, it is the most (plus or minus 2) before end of cycle.
important pelvic measurement; can be 4. Secretory phase: major hormone is
estimated by subtracting 1.52 cm from progesterone, which influences myometrium
diagonal conjugate. (decreased irritability); also called luteal phase.
4. Intertuberous diameter: measures the outlet
between the inner borders of the ischial Menopause (Climacteric)
tuberosities; should be at least 8 cm.
B. Pelvic shapes A. Decline in ovarian function and hormone
1. Android: narrow, heart shaped; male-type pelvis production.
2. Anthropoid: narrow, oval shaped; resembles B. Characterized by menstrual cycle irregularity and,
ape pelvis in some women, by vasomotor instability and loss
3. Gynecoid: classic female pelvis; wide and well of bone density.
rounded in all directions See also Menopause.
4. Platypelloid: wide but flat; may still allow
vaginal delivery Sexual Response in the Female:
C. Pelvic divisions
1. False pelvis: shallow upper basin of the pelvis; Four Phases
supports the enlarging uterus, but not A. Excitement phase: vaginal lubrication and
important obstetrically vasocongestion of external genitals.
2. Linea terminalis: plane dividing upper or false B. Plateau phase: formation of orgasmic platform in
pelvis from lower or true pelvis vagina.
3. True pelvis: consists of the pelvic inlet, pelvic C. Orgasmic phase: strong rhythmic contractions of
cavity, and pelvic outlet. Measurements of true vagina and uterus.
pelvis influence the conduct and progress of D. Resolution phase: cervix dips into seminal pool in

6
labor and delivery. vagina; all organs return to previous condition.

536 NCLEX-RN Review


53155_06_Ch06_p533-620.qxd 2/23/09 12:23 PM Page 537

anterior pituitary hormones


Blood levels of
FSH
LH

LTH

1 4 8 12 14 16 20 24 28

Estrogens
ovarian hormones
Blood levels of

Progesterone

Ovarian
cycle
Primary Growing Graafian Ovulation Corpus Degenerating
follicle follicle follicle luteum corpus luteum

Estrogens Progesterone and estrogens

Endometrial
changes during Menstrual flow
the menstrual
cycle
Functional
layer

Basal
layer

0 5 10 15 20 25 28
Days

Figure 6-3 Menstrual cycle illustrating the levels of pituitary and ovarian hormones, ovarian cycle,
and endometrial changes

MATERNITY AND FEMALE REPRODUCTIVE NURSING

6 537
53155_06_Ch06_p533-620.qxd 2/23/09 12:23 PM Page 538

The Childbearing Cycle

CONCEPTION NIDATION (IMPLANTATION)


See Figure 6-4. A. Burrowing of developing zygote into endometrial
A. The penetration of one ovum (female gamete) by lining of uterus.
one sperm (male gamete), resulting in a fertilized B. May take place 710 days after fertilization, while
ovum (zygote). Each gamete has haploid number zygote develops to trophoblastic stage.
of chromosomes (23). Zygote has diploid number C. Chorionic villi appear on surface of trophoblast
(46) with one of each pair from each parent. and secrete human chorionic gonadotropin (HCG),
B. Sex of child is determined at moment of which inhibits ovulation during pregnancy by
conception by male gamete. If X-bearing male stimulating continuous production of estrogen and
gamete unites with ovum, result is a female child progesterone. This secretion of HCG forms the
(X 1 X). If Y-bearing male gamete unites with basis of the various tests for pregnancy.
ovum, result is a male child (X 1 Y).
C. Usually occurs in the outer third of the fallopian
tube. DEVELOPMENTAL STAGES
D. Multiple pregnancies result from:
1. Two or more fertilized ova (fraternal or
dizygotic) Fertilized Ovum
2. Single fertilized ovum that dividesalways A. From conception through first 2 weeks of
same sex, only 1 chorion (identical or pregnancy.
monozygotic) B. Nidation complete by the end of this period.

Figure 6-4 Spermatogenesis and oogenesis. A primary spermatocyte produces four sperm,
but only one egg results from meiosis of a primary oocyte. The polar bodies are functionless.

6 538 NCLEX-RN Review


53155_06_Ch06_p533-620.qxd 2/23/09 12:23 PM Page 539

Embryo B. Average amount 1000 mL.


C. Protects fetus.
A. From end of second week through end of eighth 1. Allows free movement.
week (also called period of organogenesis). 2. Maintains temperature.
B. Critical time in development; embryo most 3. Provides oral fluid.
vulnerable to teratogens (harmful substances D. Can be aspirated and tested for various diseases
or conditions), which can result in congenital and abnormalities during pregnancy.
anomalies. E. Alkaline pH: can be tested when membranes
rupture to distinguish from urine.
Fetus
A. From end of eighth week to termination of Umbilical Cord
pregnancy. A. Connecting link between fetus and placenta.
B. Continued maturation of already-formed organ B. Contains two arteries and one vein supported by
systems. mucoid material (Whartons jelly) to prevent
kinking and knotting.
C. There are no pain receptors in the umbilical cord.
SPECIAL STRUCTURES
OF PREGNANCY Placenta
See Figure 6-5.
Fetal Membranes A. Transient organ allowing passage of nutrients and
waste materials between mother and fetus.
A. Arise from the zygote.
B. Also acts as an endocrine organ and as a sieve that
B. Inner (amnion) and outer (chorion).
allows smaller particles through and holds back
C. Hold the developing fetus as well as the amniotic
larger molecules. Passage of materials in either
fluid.
direction is effected by:
1. Diffusion: gases, water, electrolytes.
Amniotic Fluid 2. Facilitated transfer: glucose, amino acids,
minerals.
A. Clear, yellowish fluid surrounding the developing
3. Pinocytosis: movement of minute particles
fetus.
(e.g., fats).

Figure 6-5 Placental circulation. Through the placenta the fetus gets nourishment and excretes waste.

MATERNITY AND FEMALE REPRODUCTIVE NURSING

6 539
53155_06_Ch06_p533-620.qxd 2/23/09 12:23 PM Page 540

4. Leakage: caused by membrane defect; may C. Ductus venosus connects umbilical vein and
allow maternal and fetal blood mixing. inferior vena cava; bypassing portal circulation;
C. Mother also transmits immunoglobulin G (IgG) to closes after birth.
fetus through placenta, providing limited passive D. Foramen ovale allows blood to flow from right
immunity. atrium to left atrium, bypassing lungs. Closes
D. Hormones produced by the placenta include: functionally at birth because of increased pressure
1. HCG: early in pregnancy, responsible for in left atrium; anatomic closure may take several
continued action of corpus luteum, is basis weeks to several months.
of pregnancy tests. E. Ductus arteriosus allows blood flow from
2. Human chorionic somato-mammotropin/ pulmonary artery to aorta, bypassing fetal lungs;
human placental lactogen (HCS/HPL): similar closes after delivery.
to growth hormone; affects maternal insulin
production; prepares breasts for lactation.
3. Estrogen and progesterone: necessary for Fetal Growth and Development
continuation of pregnancy. A. Organ systems develop from three primary germ layers.
1. Ectoderm: outer layer, produces skin, nails,
Fetal Circulation nervous system, and tooth enamel.
2. Mesoderm: middle layer, produces connective
A. Arteries in cord and fetal body carry deoxygenated tissue, muscles, blood, and circulatory system.
blood. 3. Endoderm: inner layer, produces linings of
B. Vein in cord and those in fetal body carry gastrointestinal and respiratory tracts,
oxygenated blood. endocrine glands, and auditory canal.
B. Timetable (Figure 6-6 and Table 6-1).

Figure 6-6 Changes in the body size of the embryo and fetus during development in the uterus
(all figures are natural size)

6 540 NCLEX-RN Review


53155_06_Ch06_p533-620.qxd 2/23/09 12:23 PM Page 541

Table 6-1 Markers in Fetal Development

Date* Development
4 weeks All systems in rudimentary form; heart
chambers formed and heart is beating.
Embryo length about 0.4 cm; weight
about 0.4 (grams).
8 weeks Some distinct features in face; head large
in proportion to rest of body; some
movement. Figure 6-7 Ngeles rule
Length about 2.5 cm, weight 2 (grams).
12 weeks Sex distinguishable; ossification in most bones;
kidneys secrete urine; able to suck and
swallow.
Length 68 cm, weight 19 (grams). PHYSICAL AND PSYCHOLOGIC
16 weeks More human appearance; earliest movement
likely to be felt by mother; meconium in CHANGES OF PREGNANCY
bowel; scalp hair develops.
Length 11.513.5 cm, weight 100 (grams). Reproductive System
20 weeks Vernix caseosa and lanugo appear; movement
usually felt by mother; heart rate audible; A. External structures: enlarged due to increased
bones hardening. vascularity.
Length 1618.5 cm, weight 300 (grams). B. Ovaries
24 weeks Body well proportioned; skin red and wrinkled; 1. No ovulation during pregnancy
hearing established. 2. Corpus luteum persists in early pregnancy
Length 23 cm, weight 600 (grams). until development of placenta is
28 weeks Infant viable, but immature if born at this time. complete
Body less wrinkled; appearance of nails. C. Fallopian tubes: elongate as uterus rises in pelvic
Length 27 cm, weight 1100 (grams). and abdominal cavities.
32 weeks Subcutaneous fat beginning to deposit; D. Vagina
L/S ratio in lungs now 1.2:1; skin smooth 1. Increased vascularity (Chadwicks sign)
and pink. 2. Estrogen-induced leukorrhea
Length 31 cm, weight 18002100 (grams). 3. Change in pH (less acidic) may favor
36 weeks Lanugo disappearing; body usually plump; overgrowth of yeastlike organisms
L/S ratio usually 2:1; definite sleep/wake 4. Connective tissue loosens in preparation for
cycle. distention of labor and delivery
Length 35 cm, weight 22002900 (grams). E. Cervix
40 weeks Full-term pregnancy. Baby is active, with good 1. Softens and loosens in preparation for labor
muscle tone; strong suck reflex; if male, and delivery (Goodells sign).
testes in scrotum; little lanugo. 2. Mucous production increases, and plug
Length 40 cm, weight 3200 (grams) or (operculum) is formed as bacterial barricade.
more. F. Uterus
1. Hypertrophy and hyperplasia of muscle
*Dates are approximate, but developmental level should have cells
been reached by the end of the time period specified. 2. Development of fibroelastic tissue that
increases ability to contract
3. Shape changes from pearlike to ovoid
4. Rises out of pelvic cavity by 16th week of
C. Measurements of length of pregnancy pregnancy
1. Days: 267280 5. Increased vascularity and softening of isthmus
2. Weeks: 40, plus or minus 2 (Hegars sign)
3. Months (lunar): 10 6. Mild contractions (Braxton Hicks sign)
4. Months (calendar): 9 beginning in the fourth month through end
5. Trimesters: 3 of pregnancy
D. Estimated due date/estimated date of confinement G. Breasts
(Ngeles rule); see Figure 6-7. This calculation is 1. Increased vascularity, sensitivity, and
an estimation only. Most women deliver: due date fullness
1 or 2 2 weeks. Sonogram dating used to confirm 2. Nipples and areola darken

6
dates. 3. Nipples become more erectile

MATERNITY AND FEMALE REPRODUCTIVE NURSING 541


53155_06_Ch06_p533-620.qxd 2/23/09 12:23 PM Page 542

4. Proliferation of ducts and alveolar tissue C. Appearance of linea nigra, darkened line bisecting
evidenced by increased breast size abdomen from symphysis pubis to top of fundus.
5. Production of colostrum by the second trimester D. Striae (stretch marks): separation of underlying
connective tissue in breasts, abdomen, thighs, and
buttocks; fade after delivery.
Cardiovascular System E. Greater sweat and sebaceous gland activity.
A. Blood volume expands as much as 50% to meet
demands of new tissue and increased needs of all
systems.
Musculoskeletal System
B. Progesterone relaxes smooth muscle, resulting in A. Alterations in posture and walking gait caused by
vasodilation and accommodation of increased change in center of gravity as pregnancy progresses.
volume. B. Increased joint mobility as a result of action of
C. RBC volume increases as much as 30%; may be ovarian hormone (relaxin) on connective tissue.
slight decline in hematocrit as pregnancy C. Possible backache.
progresses because of this relative imbalance D. Occasional cramps in calf may occur with
(physiologic anemia). hypocalcemia.
D. Stroke volume and cardiac output increase.
E. WBCs increase.
F. Greater tendency to coagulation.
Neurologic System
G. Blood pressure may drop in early pregnancy; A. Few changes with a typical pregnancy.
should not rise during last half of pregnancy. B. Pressure on sciatic nerve may occur later in
H. Heart rate increases; palpitations possible. pregnancy due to fetal position.
I. Blood flow to uterus and placenta is maximized by
side-lying position.
J. Varicosities may occur in vulva and rectum as well
Gastrointestinal System
as lower extremities. A. Bleeding gums and hypersalivation may occur.
B. Tooth loss due to demineralization should not occur.
C. Nausea and vomiting in first trimester due to rising
Respiratory System levels of HCG.
A. Increased vascularity of mucous membranes of this D. Appetite usually improves.
system gives rise to symptoms of nasal and E. Cravings or desires for strange food combinations
pharyngeal congestion and fullness in the ears. may occur.
B. Shape of thorax shortens and widens to F. Progesterone-induced relaxation of muscle tone
accommodate the growing fetus. leads to slow movement of food through GI tract;
C. Slight increase in respiratory rate. may result in heartburn.
D. Dyspnea may occur at end of third trimester before G. Constipation may occur as water is reabsorbed in
engagement or lightening. large intestine.
E. Increased respiratory volume by 4050%. H. Emptying time for gallbladder may be prolonged;
F. Oxygen consumption increases by 15%. increased incidence of gallstones.

Renal System Endocrine System


A. Kidney filtration rate increases as much as 50%. A. Pituitary: FSH and LH greatly decreased; oxytocin
B. Glucose threshold drops; sodium threshold rises. secreted during labor and after delivery; prolactin
C. Water retention increases as pregnancy progresses. responsible for initiation and continuation of
D. Enlarging uterus causes pressure on bladder lactation.
resulting in frequency of urination, especially B. Progesterone secreted by corpus luteum until
during first trimester; later in pregnancy relaxed formation of placenta.
ureters are displaced laterally, increasing C. Principal source of estrogen is placenta,
possibility of stasis and infection. synthesized from fetal precursors.
E. Presence of protein (not an expected component of D. HCS/HPL produced by placenta; similar to growth
maternal urine) indicates possible renal disease or hormone, it prepares breasts for lactation; also
pregnancy-induced hypertension. affects insulin/glucose metabolism. May overstress
maternal pancreas.
E. Ovaries secrete relaxin during pregnancy.
Integumentary System F. Slight increase in thyroid activity and basal
A. Increased pigmentation of nipples and areolas. metabolic rate (BMR).
B. Possible appearance of chloasma (mask of G. Pancreas may be stressed due to complex
pregnancy): darkening of areas on forehead and interaction of glucose metabolism, HCS/HPL, and

6
cheekbones. cortisol, resulting in diminished effectiveness of
insulin, and demand for increased production.

542 NCLEX-RN Review


53155_06_Ch06_p533-620.qxd 2/23/09 12:23 PM Page 543

Psychosocial Changes in pregnant American Indians; increase in


parasitic infections and hepatitis in women from
A. First trimester Southeast Asia.
1. Mother needs accurate diagnosis of pregnancy. C. Behaviors during pregnancy (eating, sleeping,
2. Works through characteristic ambivalence of bathing, sexual activity, etc.) will differ from
early pregnancy. culture to culture.
3. Mother is self-centered, baby is part of her. D. Superstition, taboos, and old wives tales may
4. Grandparents are usually the first relatives to play an important role.
be told of the pregnancy. E. Most cultures consider childbirth to be the
B. Second trimester province of women; mens roles may be limited or
1. Mother demonstrates growing realization of excluded. This may include caregivers.
baby as separate and needing person. F. Perception of discomfort/pain will vary; may be
2. Fantasizes about unborn child. influenced by cultural expectations as well as
C. Third trimester mothers own experiences.
1. Nesting activity appears as due date G. Despite culture, many women prefer upright
approaches. position for labor and birth, rather than lying
2. Desire to be finished with pregnancy. down. Additionally, many women prefer to have
D. Anxiety over safe passage for self and baby the freedom to move around during labor,
through labor and delivery. whenever possible.
E. Reactions of father-to-be may parallel those of H. After delivery, cultural rituals may refer to rest,
mother (e.g., ambivalence, anxiety). Additionally, seclusion, and purification. The postpartum
as mothers pregnancy progresses he may women may be considered vulnerable.
experience similar physical changes. Applications of heat or cold, in air or water, need
F. Preparation of siblings varies according to their age to be specially assessed.
and experience. I. Other areas of culturally related practices may be:
nutrition, clothing, activity, resumption of contact
Transcultural Concerns in Pregnancy with the community, resumption of sexual
activities, and return to work.
A. Dominant philosophy concerning pregnancy and J. Infant care (feeding, cord care, circumcision,
birth may differ in non-U.S. cultures. May view clothing, sleeping arrangements) will vary among
this as healthy time, with little or no insight differing cultures.
concerning potential complications. K. Control of future fertility may vary from natural
B. Biological variations may occur, e.g., higher rate of methods such as breastfeeding to the use of any
complications in women with sickle cell trait or available means.
gene; diabetes mellitus seen with more frequency

The Antepartal Period

ASSESSMENT 2. Primipara: a woman who has given birth


to one baby more than 20 weeks gestation.
3. Multipara: Woman who has had two or more
Classification of Pregnancy births at more than 20 weeks gestation. . .
A. Gravida: number of times pregnant, regardless twins or triplets count as 1 para.
of duration, including the present pregnancy. 4. TPAL: Para subdivided to reflect births that
1. Primagravida: pregnant for the first time went to Term, Premature births, Abortions,
2. Multigravida: pregnant for second or and Living children.
subsequent time
B. Para: number of pregnancies that lasted more than Determination of Pregnancy
20 weeks, regardless of outcome.
1. Nullipara: a woman who has not given Diagnosis of pregnancy is based on pregnancy-related
birth to a baby beyond 20 weeks physical and hormonal changes and are classified as
gestation. presumptive, probable, or positive.

MATERNITY AND FEMALE REPRODUCTIVE NURSING

6 543
53155_06_Ch06_p533-620.qxd 2/23/09 12:23 PM Page 544

Presumptive Signs and 2. Dangerous behaviors during pregnancy (e.g.,


Symptoms (Subjective) smoking, using drugs [especially alcohol], use
of nonprescription medications)
These changes may be noticed by the mother/health B. In nutrition potential: individualized nutritional
care provider but are not conclusive for pregnancy. information will be needed
A. Amenorrhea (cessation of menstruation) C. Activity intolerance: need for additional rest and
B. Nausea and vomiting benefits of a moderate exercise program
C. Urinary frequency D. Anxiety
D. Fatigue E. Risk for constipation
E. Breast changes F. Disturbed body image
F. Weight change G. Ineffective coping
G. Skin changes H. Powerlessness
H. Vaginal changes including leukorrhea I. Noncompliance
I. Quickening J. Risk for deficient fluid volume
K. Health-seeking behaviors
Probable Signs and Symptoms (Objective)
These changes are usually noted by the health care PLANNING AND
provider but are still not conclusive for pregnancy.
A. Uterine enlargement IMPLEMENTATION
B. Changes in the uterus and cervix from increased
vascularity Goals
C. Ballottement: fetus rebounds against the
examiners hand when pushed gently A. Establish a diagnosis of pregnancy.
upwards. B. Gather initial data to form the basis for comparison
D. Braxton Hicks contractions: occur early in with data collected as pregnancy progresses.
pregnancy, although not usually sensed by the C. Identify high-risk factors.
mother until the third trimester. D. Propose realistic and necessary interventions.
E. Laboratory tests for pregnancy E. Promote optimal health for mother and baby,
1. Most tests rely on the presence of HCG in the providing any needed information.
blood or urine of the woman. F. Provide needed information for prepared childbirth.
2. Easy, inexpensive, but may give false readings
with any handling error, medications, or Interventions
detergent residue in laboratory equipment.
3. Exception is the radioimmunoassay (RIA), Prenatal Care
which tests for the beta subunit of HCG and
is considered to be so accurate as to be A. Time frame
diagnostic for pregnancy. 1. First visit: may be made as soon as woman
F. Changes in skin pigmentation. suspects she is pregnant; frequently after first
missed period.
2. Subsequent visits: Every month until the 7th
Positive Signs and Symptoms or 8th month, every 2 weeks during the 8th
These signs emanate from the fetus, are noted by the month, and weekly during the 9th month;
health care provider, and are conclusive for pregnancy. more frequent visits are scheduled if problems
A. Fetal heartbeat: detected as early as eighth week arise.
with an electronic device; after 16th week with B. Conduct of initial visit
a more conventional auscultory device. 1. Extensive collection of data about client in all
B. Palpation of fetal outline. pertinent areas in order to form basis for
C. Palpation of fetal movements. comparison with data collected on subsequent
D. Demonstration of fetal outline by either ultrasound visits and to screen for any high-risk factors
(after sixth week) or X-ray (after 12th week). a. Menstrual history: menarche, regularity,
frequency and duration of flow, last period
b. Obstetrical history: all pregnancies,
ANALYSIS complications, outcomes, contraceptive
use, sexual history
Nursing diagnoses for the antepartal period may c. Medical history: include past illnesses,
include: surgeries; current use of medications
A. Deficient knowledge: information on the following d. Family history/psychosocial data
topics needs to be given and reinforced e. Information about the father-to-be may also
1. Danger signals of pregnancy to be reported be significant

6
f. Current concerns

544 NCLEX-RN Review


53155_06_Ch06_p533-620.qxd 2/23/09 12:23 PM Page 545

2. Complete physical examination, including 5. Iron: needed by mother as well as fetus;


internal gynecologic exam and bimanual exam reserves usually sufficient for first trimester,
3. Laboratory work, including CBC, urinalysis, supplementation recommended after this time;
Pap test, blood type and Rh, rubella titer, iron preparations should be taken with source
testing for sexually transmitted diseases of vitamin C to promote absorption.
(STDs), other tests as indicated (e.g., TB test, 6. Calcium: 1200 mg per day needed; dairy
hepatitis viral studies, ECG, etc.) products most frequent source, with
C. Conduct of subsequent visits supplementation for those with lactose
1. Continue collection of data, especially weight, intolerance.
blood pressure, urine screening for glucose 7. Sodium: contained in most foods; needed in
and protein, evaluation of fetal development pregnancy; should not be restricted without
through auscultation of fetal heart rate (FHR) serious indication.
and palpation of fetal outline, measurement of 8. Vitamins: both fat- and water-soluble are
fundal height as correlation for appropriate needed in pregnancy; essential for tissue
progress of pregnancy. Fundus palpable above growth and development, as well as regulation
symphysis at 12 weeks, at the level of of metabolism. Generally not synthesized by
umbilicus at 20 weeks, then approximately body, nor stored in large amounts (folic acid
1 cm per week until 3638 weeks when head special concern as deficiency may cause fetal
often descends and fundal measurement may anomalies and bleeding complications).
drop somewhat. C. Dietary supplements: many health care providers
2. Additional tests supplement the pregnant womans diet with an iron-
a. Hemoglobin and hematocrit 2628 weeks fortified multivitamin to ensure essential levels.
b. Glucose screen 2428 weeks D. Special concerns
c. Antibody screen at 28 weeks 1. Religious, ethnic, and cultural practices that
d. Beginning of 9th month, test for STDs, influence selection and preparation of foods
strep, other infections 2. Pica (ingestion of nonedible or non-nutritive
3. Prepare for any necessary testing. substances)
a. Have client void (clean catch). 3. Vegan vegetarians: no meat products, may
b. Collect baseline data on vital signs. need B12 supplement
c. Collect specimen. 4. Adolescence
d. Monitor client and fetus after procedure. 5. Economic deprivation
e. Provide support to client. 6. Heavy smoking, alcohol consumption, drugs
f. Document as needed. 7. Previous reproductive problems

Nutrition during Pregnancy Education for Parenthood


A. Weight gain A. Provision of information about pregnancy, labor
1. Variable, but 25 lb usually appropriate for and delivery, the postpartum period, and lactation.
average woman with single pregnancy. B. Usually taught in small groups, may be
2. Woman should have consistent, predictable individualized.
pattern of weight gain, with only 23 lb in first C. Topics can be grouped into early and late
trimester, then average 12 oz gain every week pregnancy, labor and delivery, and
in second and third trimesters. postdelivery/newborn care.
3. Gains mostly reflect maternal tissue in first D. Emphasis placed on both physical and
half of pregnancy, and fetal tissue in second psychosocial changes seen in childbearing cycle.
half of pregnancy. E. Preparation for childbirth: intended to provide
B. Specific nutrient needs knowledge and alternative coping behaviors in
1. Calories: usual addition is 300 kcal/day, but order to diminish anxiety and discomfort, and
there will be specific guidelines for those promote cooperation with the birth process; see
beginning pregnancy either over- or Table 6-2 for specific methodologies.
underweight (never less than 1800 kcal/day).
2. Protein: additional 30 grams/day to ensure Determination of Fetal Status
intake of 7476 grams/day; very young and Risk Factors
pregnant adolescents and those with multiple
pregnancies will need more protein. A. Fetal diagnostic tests
3. Carbohydrates: intake must be sufficient for 1. Used to:
energy needs, using fresh fruits and vegetables a. Identify or confirm the existence of risk
as much as possible to derive additional fiber factor(s)
benefit; teach to avoid empty calories. b. Validate pregnancy

6
4. Fats: high-energy foods, which are needed to c. Observe progress of pregnancy
carry the fat-soluble vitamins.

MATERNITY AND FEMALE REPRODUCTIVE NURSING 545


53155_06_Ch06_p533-620.qxd 2/23/09 12:23 PM Page 546

15 and 18 weeks gestation. Elevated AFP


Table 6-2 Methods of Childbirth may be associated with neural tube defects,
renal anomalies. Low AFP seen with
Read method The so-called natural childbirth method.
chromosomal trisomies.
Underlying concept: knowledge
f. L/S ratio: uses amniotic fluid to ascertain
diminishes the fear that is key to pain.
fetal lung maturity through measurement
Classes include information as well as
of presence and amounts of the lung
practice in relaxation and abdominal
surfactants lecithin and sphingomyelin.
breathing techniques for labor.
At 3536 weeks, ratio is 2:1, indicative of
Lamaze method Psychoprophylactic method based on
mature levels; once ratio of 2:1 is achieved,
utilization of Pavlovian conditioned
newborn less likely to develop respiratory
response theory. Classes teach
distress syndrome. Phosphatidylglycerol
replacement of usual response to pain
(PG) is found in amniotic fluid after
with new, learned responses (breathing,
35 weeks. In conjunction with the L/S
effleurage, relaxation) in order to block
ratio, it contributes to increased reliability
recognition of pain and promote
of fetal lung maturity testing. May be done
positive sense of control in labor.
in laboratory or by shake test.
Bradley method Husband-coached childbirth.
g. Fetal movement count: teach mother to
A modification of the Read method
count 23 times daily, 3060 minutes each
emphasizing working in harmony
time, should feel 56 movements per
with the body.
counting time. Mother should notify care
Other methods Hypnosis, yoga.
giver immediately of abrupt change or no
movement.
h. PUBS (percutaneous umbilical blood
d. Identify optimum time for induction of sampling): uses ultrasound to locate
labor if indicated umbilical cord. Cord blood aspirated and
e. Identify genetic abnormalities tested. Used in second and third trimesters.
2. Types i. Biophysical profile: a collection of data on
a. Chorionic villi sampling (CVS): earliest test fetal breathing movements, body
possible on fetal cells (912 weeks); movements, muscle tone, reactive heart
sample obtained by slender catheter passed rate, and amniotic fluid volume. A score
through cervix to implantation site. of 0 to 2 is given in each category, and the
b. Ultrasound: use of sound and returning summative number interpreted by the
echo patterns to identify intrabody physician. Primary suggested use is to
structures. Useful early in pregnancy to identify fetuses at risk for asphyxia.
identify gestational sac(s) and to assist in B. Electronic monitoring
pregnancy dating. Later uses include 1. Nonstress test (NST) (see Table 6-3)
assessment of fetal viability, growth a. Accelerations in heart rate accompany
patterns, anomalies, fluid volume, uterine normal fetal movement
anomalies, and adnexal masses. Used as an b. In high-risk pregnancies, NST may be used
adjunct to amniocentesis; safe for fetus to assess FHR on a frequent basis in order
(no ionizing radiation). to ascertain fetal well-being.
c. Amniocentesis: location and aspiration of c. Noninvasive
amniotic fluid for examination; possible 2. Contraction stress test (CST) (see Table 6-4)
after the 14th week when sufficient a. Based on principle that healthy fetus can
amounts are present. Used to identify withstand decreased oxygen during
chromosomal aberrations, sex of fetus, contraction, but compromised fetus cannot.
levels of alpha-fetoprotein and other b. Types
chemicals indicative of neural tube defects 1) Nipple-stimulated CST: massage or
and inborn errors of metabolism, rolling of one or both nipples to stimulate
gestational age, Rh factor. uterine activity and check effect on FHR.
d. X-ray: can be used late in pregnancy 2) Oxytocin challenge test (OCT): infusion
(after ossification of fetal bones) to confirm of calibrated dose of IV oxytocin
position and presentation; not used in piggybacked to main IV line;
early pregnancy to avoid possibility of controlled by infusion pump; amount
causing damage to fetus and mother. infused increased every 1520 minutes
e. Alpha-fetoprotein screening: Maternal until three good uterine contractions
serum screens. Alpha-fetoprotein is are observed within 10-minute period.
glucoprotein produced by fetal yolk sac, 3) CST never done unless willing to

6
GI tract, and liver. Test done between deliver fetus.

546 NCLEX-RN Review


53155_06_Ch06_p533-620.qxd 2/23/09 12:23 PM Page 547

2. Maternal blood pressure, weight gain, and


Table 6-3 Nonstress Test (NST) other lab test findings are within normal range.
C. Pregnant woman/family have received adequate
Result Interpretation Significance
educational instruction.
Reactive 2 or more accelerations High-risk pregnancy 1. Pregnant woman/family express
of 15 beats/min allowed to understanding of childbirth experience and
lasting 15 sec or continue if twice begin transition to role of parenting.
more in 20-min weekly NSTs are 2. Any necessary testing procedures carried out
period (associated reactive. completely and correctly; client/fetus in stable
with each fetal condition.
movement)
Nonreactive No FHR acceleration, Need to attempt to
or accelerations less clarify FHR pattern;
COMPLICATIONS OF PREGNANCY
than 15 beats/min implement CST Pregnancy can be complicated by situations unique
or lasting less than and continue to childbearing (e.g., placental bleeding), or by long-
15 sec through fetal external standing conditions predating pregnancy and
movement monitoring. continuing into the childbearing process (e.g., age,
Unsatisfactory FHR pattern not able Repeat NST or socioeconomic status, cardiac problems); for common
to be interpreted do CST. discomforts of pregnancy, see Table 6-5.

General Nursing Responsibilities


A. Teach danger signals of pregnancy early in
Table 6-4 Contraction Stress Test (CST) prenatal period so that client is aware of what
needs to be reported to health care provider on an
Result Interpretation Significance immediate basis (see Table 6-6).
B. Be aware that early teaching allows the client to
Negative 3 contractions, Fetus should tolerate
participate in the identification and reporting of
4060 sec long, labor if it occurs
symptoms that can indicate a problem in her
within 10-min within 1 week.
pregnancy.
period, no late
C. Early recognition and reporting of danger signals
decelerations
usually results in diminishing the risk and controlling
Positive Persistent/consistent Fetus at increased the severity of maternal/fetal complications.
late decelerations risk. May need D. Interventions are specific for the individual risks.
with more additional testing, E. Evaluation centers around whether or not the risk
than 50% of may try induction was controlled or eliminated, and how the
contractions or cesarean birth. maternal/fetal reaction was controlled.
Suspicious Late decelerations Repeat CST in 24 hr,
in less than or other fetal First Trimester Bleeding Complications
50% of assessment tests.
contractions
Abortion
Unsatisfactory Inadequate pattern Same as for A. General information
or poor tracing suspicious. 1. Loss of pregnancy before viability of fetus;
may be spontaneous, therapeutic, or elective
(for additional information on therapeutic and
elective abortions see Control of Fertility).
(Clients may use term miscarriage for
EVALUATION spontaneous abortion.)
2. Types
A. Maternal/fetal assessment data remain within a. Threatened abortion
acceptable limits; fetus maintains growth and 1) Cervix closed
development pattern appropriate to gestational age 2) Some bleeding and contractions
(evidenced by maternal weight gain, normal 3) Fetus not expelled
increments in fundal height, fetal activity level, b. Inevitable
other antepartal tests). 1) Cervix open
B. No complications of pregnancy are evident. 2) Heavier bleeding and stronger
1. Pregnant woman receives prenatal care (initial contractions

6
and subsequent visits). 3) Loss of fetus usually not avoidable

MATERNITY AND FEMALE REPRODUCTIVE NURSING 547


53155_06_Ch06_p533-620.qxd 2/23/09 12:23 PM Page 548

Table 6-5 Common Discomforts during Pregnancy Table 6-6 Danger Signals of Pregnancy

Discomfort Trimester Intervention Any bleeding from vagina


Gush of fluid from vagina (clear, not urine)
Morning First Eat dry carbohydrate in Regular contractions occurring before due date
sickness AM; avoid fried, odorous,
Severe headaches or changes in vision
and greasy foods; small Epigastric pain
meals rather than large. Vomiting that persists and is severe
Fatigue First Rest frequently, as Change in fetal activity patterns
needed. Temperature elevation, chills, or sick feeling indicative of
infection
Urinary First, end Kegel exercises, perineal Swelling in upper body, especially face and fingers
frequency of third pad for leakage.
Heartburn Second, Small meals, bland foods,
third antacids if ordered.
c. Incomplete
Constipation Second, Sufficient fluids, foods high
1) Expulsion of fetus incomplete
third in roughage, regular
2) Membranes or placenta retained
bowel habits. No laxatives
d. Complete: all products of conception
unless ordered, including
expelled
mineral oil.
e. Missed: fetus dies in uterus but is not
Hemorrhoids Third Avoid constipation; expelled
promote regular f. Habitual
bowel habits. 1) Three pregnancies in a row
culminating in spontaneous abortion
Varicosities Third Avoid crossing legs and 2) May indicate need for investigation
long periods of sitting into underlying causes
or standing; rest with B. Assessment findings
feet and hips elevated; 1. Vaginal bleeding (observing carefully for
avoid elastic garters accurate determination of amount, saving all
and other constrictive perineal pads)
clothing. 2. Contractions, pelvic cramping, backache
Backache Third Use good posture and 3. Lowered hemoglobin if blood loss significant
body mechanics; 4. Passage of fetus/tissue
low-heeled shoes; C. Nursing interventions
exercises to strengthen 1. Save all tissue passed.
back muscles. 2. Keep client at rest and teach reason for bed
rest.
Insomnia Third Conscious relaxation; 3. Increase fluids PO or IV.
supportive pillows as 4. Prepare client for surgical intervention
needed; warm shower (D&C or suction evacuation) if needed
before retiring. (see also Termination of Pregnancy).
Leg cramps Third Flex toes toward knees 5. Provide discharge teaching about limited
for relief; ensure activities and coitus after bleeding ceases.
adequate calcium 6. Observe reaction of mother and others, provide
in diet. emotional support, and give opportunity to
express feelings of grief and loss.
Supine Third Left side-lying position. 7. Administer RhoGAM if mother Rh negative.
hypotensive
syndrome Incompetent Cervical Os
Vaginal discharge Second Correct personal hygiene, (Premature Dilation of Cervix)
refer to physician.
A. General information: painless condition in which
Do not douche.
the cervix dilates without uterine contractions and
Skin changes, All Interventions are allows passage of the fetus; usually the result of
dryness, symptomatic; cool prior cervical trauma.
itching baths, lotions, oils B. Medical management: may be treated surgically by
as indicated. cerclage (placement of fascia or artificial material

6
to constrict the cervix in a purse-string manner).

548 NCLEX-RN Review


53155_06_Ch06_p533-620.qxd 2/23/09 12:23 PM Page 549

When client goes into labor, choice of removal of 3. Dark red to brownish vaginal bleeding after
suture and vaginal delivery, or cesarean birth. 12th week
C. Assessment findings 4. Anemia often accompanies bleeding
1. History of repeated, relatively painless 5. Symptoms of preeclampsia before usual time
abortions of onset
2. Early and progressive effacement and dilation 6. No fetal heart sounds or palpation of fetal parts
of cervix, usually second trimester 7. Ultrasound shows no fetal skeleton
3. Bulging of membranes through cervical os C. Nursing interventions
D. Nursing interventions 1. Provide pre- and postoperative care for
1. Continue observation for contractions, rupture evacuation of uterus (usually suction curettage).
of membranes, and monitor fetal heart tones. 2. Teach contraceptive use so that pregnancy
2. Position client to minimize pressure on cervix. is delayed for at least one year.
3. Teach client need for follow-up lab work
Ectopic Pregnancy to detect rising HCG levels indicative of
choriocarcinoma.
A. General information 4. Provide emotional support for loss of
1. Any gestation outside the uterine cavity pregnancy.
2. Most frequent in the fallopian tubes, where the 5. Teach about risk for future pregnancies,
tissue is incapable of the growth needed to if indicated.
accommodate pregnancy, so rupture of the site
usually occurs before 12 weeks.
3. Any condition that diminishes the tubal Second Trimester Bleeding
lumen may predispose a woman to ectopic Complications
pregnancy
B. Assessment findings There are few unique causes of bleeding in the second
1. History of missed periods and symptoms of trimester. Bleeding may be a late manifestation of
early pregnancy condition usually seen in first trimester, such as
2. Abdominal pain, may be localized to one side spontaneous abortion or incompetent cervical os.
3. Rigid, tender abdomen; sometimes abnormal
pelvic mass Third Trimester Bleeding
4. Bleeding; if severe may lead to shock
5. Low hemoglobin and hematocrit, rising WBC
Complications
count Placental problems are the most frequent cause of
6. HCG titers usually lower than in intrauterine bleeding in the third trimester.
pregnancy
C. Nursing interventions Placenta Previa
1. Prepare client for surgery.
2. Institute measures to control/treat shock if A. General information
hemorrhage severe; continue to monitor 1. Low implantation of the placenta so that it
postoperatively. overlays some or all of the internal cervical os.
3. Allow client to express feelings about loss of Complete previa requires cesarean delivery.
pregnancy and concerns about future Partial may deliver vaginally if fetus in vertex
pregnancies. presentation.
2. Cause uncertain, but uterine factors (poor
Hydatidiform Mole (Gestational vascularity, fibroid tumors, multiple
pregnancies) may be involved.
Trophoblastic Disease) 3. Amount of cervical os involved classifies
A. General information placenta previa as marginal, partial, or
1. Proliferation of trophoblasts; embryo dies. complete.
Unusual chromosomal patterns seen (either no 4. Often diagnosed prior to 30 weeks by
genetic material in ovum, or 69 chromosomes). sonogram. Many resolve or migrate before
The chorionic villi change into a mass of clear, labor.
fluid-filled grapelike vessels. B. Assessment findings
2. More common in women over 40. 1. Bright red, painless vaginal bleeding after
3. Cause essentially unknown. seventh month of pregnancy is cardinal
B. Assessment findings indicator. Bleeding may be intermittent, in
1. Increased size of uterus disproportionate to gushes, or continuous.
length of pregnancy 2. Uterus remains soft.
2. High levels of HCG with excessive nausea and 3. FHR usually stable unless maternal shock

6
vomiting present.

MATERNITY AND FEMALE REPRODUCTIVE NURSING 549


53155_06_Ch06_p533-620.qxd 2/23/09 12:23 PM Page 550

4. No vaginal exam by nurse, may result in C. Nursing interventions


severe bleed, if done by physician, double 1. Begin NPO and IV fluid and electrolyte
set-up used. replacement. (Correction of F&E balance will
5. Diagnosis by sonography. decrease nausea, NPO will rest the stomach.)
C. Nursing interventions 2. Monitor I&O.
1. Ensure complete bed rest. 3. Gradually reintroduce PO intake, monitor
2. Maintain sterile conditions for any invasive amounts taken and retained.
procedures (including vaginal examination). 4. Monitor TPN and central line placement
3. Make provision for emergency cesarean birth if unable to eat.
(double set-up procedure). 5. Provide mouth care.
4. Continue to monitor maternal/fetal vital signs. 6. Offer emotional supportvery demoralizing
5. Measure blood loss carefully. and depressing to client.
6. Assess uterine tone regularly. 7. Refer to home health as appropriate for
continued IV or TPN therapy.
Abruptio Placentae
A. General information Pregnancy-Induced Hypertension
1. Separation of placenta from part or all of General Information
normal implantation site, usually
accompanied by pain A. Refers to condition unique to pregnancy where
2. Usually occurs after 20th week of pregnancy vasospastic hypertension is accompanied by
3. Increased risk of abruption with maternal proteinuria and edema; maternal or fetal condition
hypertension, previous abruption, cigarette may be compromised.
smoking, multiparity, history of abortions, 1. Probable cause: gradual loss of normal
cocaine use, abdominal trauma pregnancy-related response to angiotensin II
B. Assessment findings 2. May also be related to decreased production of
1. Painful vaginal bleeding some vasodilating prostaglandins
2. Tender, boardlike uterus (especially if concealed B. Onset after 20th week of pregnancy, may appear in
hemorrhage, then no vaginal bleeding) labor or up to 48 hours postpartum.
3. Fetal bradycardia and late decelerations, C. Characterized by widespread vasospasm.
absent FHT in complete abruption D. Cause essentially unknown, but incidence is high
4. Additional signs of shock in primigravidas, multiple pregnancies, maternal
C. Nursing interventions age under 17 or over 35, hydatidiform mole, poor
1. Ensure bed rest. nutrition, essential hypertension; familial
2. Check maternal/fetal vital signs frequently. tendency.
3. Prepare for IV infusions of fluids/blood as E. Occurs in 57% of all pregnant women.
indicated. F. Clinical classification of hypertensive disorders in
4. Monitor urinary output. pregnancy.
5. Anticipate coagulation problems (DIC). 1. Pregnancy-induced hypertension (PIH)
6. Provide support to parents as outlook for fetus a. Preeclampsiamild or severe
is poor. b. Eclampsia
7. Prepare for emergency surgery as indicated. 2. Chronic hypertension
3. Chronic hypertension with superimposed PIH
G. Classic triad of symptoms includes edema/weight
Hyperemesis Gravidarum gain, hypertension, and proteinuria. Eclampsia
A. General information includes convulsions and coma.
1. Excess nausea and vomiting of early H. Possible life-threatening complication: HELLP
pregnancy leads to dehydration and electrolyte syndrome (hemolysis, elevated liver enzymes,
disturbances, especially acidosis. lowered platelets).
2. Causes: possible severe reaction to HCG, not I. Only known cure is delivery. Delivery may
psychological, greater risk in conditions where be initiated early to reduce risks and further
HCG levels increased. HCG levels peak around complications.
6 weeks after conception, plateau, then begin
to decline after the 12th week. Symptoms Mild Preeclampsia
often improve later in pregnancy but may last
entire time. A. Assessment findings
B. Assessment findings 1. Appearance of symptoms between 20th and
1. Nausea and vomiting, progressing to retching 24th week of pregnancy
between meals 2. Blood pressure of 140/90 or 130 systolic/115
mm Hg diastolic on two consecutive occasions

6
2. Weight loss
at least 6 hours apart

550 NCLEX-RN Review


53155_06_Ch06_p533-620.qxd 2/23/09 12:23 PM Page 551

3. Sudden weight gain (13 lb/month in second 2. Coma may ensue.


trimester; 11 lb/week in third trimester; or 3. Labor may begin, putting fetus in great
14.5 lb/week at any time) jeopardy.
4. Slight generalized edema, especially of hands 4. Convulsion may recur.
and face C. Nursing interventions
5. Proteinuria of 300 mg/liter in a 24-hour 1. Minimize all stimuli.
specimen (11 in 1 specimen) a. Darken room.
B. Nursing interventions b. Limit visitors.
1. Promote bed rest as long as signs of edema or c. Use padded bedsides and bed rails.
proteinuria are minimal, preferably side-lying. 2. Check vital signs and lab values frequently.
2. Provide well-balanced diet with adequate 3. Seizure precautions: airway, oxygen, and
protein and roughage, no Na1 restriction. suction equipment should be available at
3. Explain need for close follow-up, weekly or bedside.
twice-weekly visits to physician. 4. Administer medications as ordered.
5. Prepare for cesarean delivery when seizures
Severe Preeclampsia stabilized.
6. Continue observations 2448 hours postpartum.
A. Assessment findings
1. Headaches, epigastric pain, nausea and
vomiting, visual disturbances, irritability PRE- AND COEXISTING DISEASES
2. Blood pressure of 150160/100110 mm Hg
3. Increased edema and weight gain OF PREGNANCY
4. Proteinuria (5 g/24 hours) (41) oliguria
5. Hyperreflexia of 41, possibly with clonus Cardiac Conditions
B. Medical management: magnesium sulfate
1. Magnesium sulfate acts upon the myoneural General Information
junction, diminishing neuromuscular
A. May be the result of congenital heart disease or the
transmission.
sequelae of rheumatic fever/heart disease.
2. It promotes maternal vasodilation, better tissue
B. May affect pregnancy, but are definitely affected by
perfusion, and has anticonvulsant effect.
pregnancy.
3. Nursing responsibilities
C. Classification
a. Monitor clients respirations, blood
1. Class 1: no limitation of activity
pressure, and reflexes, as well as urinary
2. Class 2: slight limitation of activity
output frequently.
3. Class 3: considerable limitation of activity
b. Administer medications either IV or IM.
4. Class 4: symptoms present even at rest
4. Antidote for excess levels of magnesium
sulfate is calcium gluconate or calcium
chloride. Prenatal Period
C. Nursing interventions A. Assessment findings
1. Promote complete bed rest, side-lying. 1. Evidence of cardiac decompensation especially
2. Carefully monitor maternal/fetal vital signs. when blood volume peaks (weeks 2832)
3. Monitor I&O, results of laboratory tests. 2. Cough and dyspnea
4. Take daily weights. 3. Edema
5. Do daily fundoscopic examination. 4. Heart murmurs
6. Institute seizure precautions. 5. Palpitations
a. Restrict visitors. 6. Rales
b. Minimize all stimuli. B. Nursing interventions
c. Monitor for hyperreflexia. 1. Promote frequent rest periods and adequate
d. Administer sedatives as ordered. sleep, decreased stress.
7. Instruct client about appropriate diet. 2. Teach client to recognize and report signs of
8. Continue to monitor 2448 hours postdelivery. infection, importance of prophylactic
9. Administer medications as ordered; vasodilator antibiotics.
of choice usually hydralazine (Apresoline). 3. Compare vital signs to baseline and normal
values expected during pregnancy.
Eclampsia 4. Instruct in diet to limit weight gain to 15 lb.,
low Na1.
A. Medical management (see Severe Preeclampsia).
5. Explain rationale for anticoagulant therapy
B. Assessment findings
(heparin used in pregnancy) if ordered.
1. Increased hypertension precedes convulsion
6. Teach danger signals for individual client.

6
followed by hypotension and collapse.

MATERNITY AND FEMALE REPRODUCTIVE NURSING 551


53155_06_Ch06_p533-620.qxd 2/23/09 12:23 PM Page 552

Intrapartal Period b. If the pancreas cannot respond by


producing additional insulin, excess
A. Labor increases risk of congestive heart failure: glucose moves across placenta to fetus,
milking effect of contractions and delivery where fetal insulin metabolizes it, and acts
increases blood volume to heart. as growth hormone, promoting
B. Nursing interventions macrosomia.
1. Monitor maternal ECG and FHT continuously. c. Maternal insulin levels need to be carefully
2. Explain to client that vaginal delivery is monitored during pregnancy to avoid
preferred over cesarean delivery. widely fluctuating levels of blood glucose.
3. Monitor clients response to stress of labor and d. Dose may drop during first trimester, then
watch for signs of decompensation. rise during second and third trimesters.
4. Administer oxygen and pain medication as e. Higher incidence of fetal anomalies and
ordered, epidural preferable. neonatal hypoglycemia (good control
5. Position client in side-lying/low semi-Fowlers minimizes).
position. B. Assessment findings: signs of hyperglycemia
6. Provide calm atmosphere. 1. Polyuria
7. Encourage open-glottal pushing during 2. Polydipsia
second stage of labor, forceps or vacuum 3. Weight loss
extractor used to minimize pushing. 4. Polyphagia
5. Elevated glucose levels in blood and urine.
Postpartal Period Urine tests for elevated blood glucose less
reliable in pregnancy. Blood tests (more
A. Nursing interventions accurate) used as follows:
1. Monitor vital signs, any bleeding, strict I&O, a. 1-hour glucose tolerance test: usually
lab test values, daily weight, rest and diet. done for screening on all pregnant women
2. Promote bed rest in appropriate position 2428 weeks pregnant.
(see Intrapartal Period). b. 3-hour glucose tolerance test: used when
3. Assist with activities of daily living (ADL) results from 1 hour GTT > 140 mg/dL.
as needed. c. HbA1c: reflects past 412-week blood levels
4. Prevent infection. of serum glucose.
5. Facilitate nonstressful mother/baby C. Nursing interventions
interactions. 1. Teach client the effects and interactions of
6. Help mother plan for rest and activity patterns diabetes and pregnancy and signs of hyper-
at home, as well as household help if and hypoglycemia.
indicated. 2. Teach client how to control diabetes in
pregnancy, advise of changes that need to be
Endocrine Conditions made in nutrition and activity patterns to
promote normal glucose levels and prevent
Diabetes Mellitus complications.
A. General information 3. Advise client of increased risk of infection and
1. Chronic disease caused by improper metabolic how to avoid it.
interaction of carbohydrates, fats, proteins, 4. Observe and report any signs of preeclampsia.
and insulin. 5. Monitor fetal status throughout pregnancy.
2. Interaction of pregnancy and diabetes may 6. Assess status of mother and baby frequently
cause serious complications of pregnancy. a. Monitor carefully fluids, calories, glucose,
3. Classifications of diabetes mellitus (see also and insulin during labor and delivery.
Unit 4) b. Continue careful observation in
a. Type 1, insulin-dependent diabetes postdelivery period.
mellitus, usually appears before age 30
b. Type 2, noninsulin-dependent, onset Renal Conditions
usually after age 30
c. Gestational diabetes, onset occurs during Urinary Tract Infections (UTI)
pregnancy A. General information
4. Significance of diabetes in pregnancy 1. Affect 10% of all pregnant women.
a. Interaction of estrogen, progesterone, 2. Dilated, flaccid, and displaced ureters are a
HCS/HPL, and cortisol raise maternal frequent site.
resistance to insulin (ability to use 3. E. coli is the usual cause.
glucose at the cellular level). 4. May cause premature labor if severe,

6
untreated, or pyelonephritis develops.

552 NCLEX-RN Review


53155_06_Ch06_p533-620.qxd 2/23/09 12:23 PM Page 553

B. Assessment findings such as Candida albicans, herpes, and


1. Frequency and urgency of urination toxoplasmosis.
2. Suprapubic pain 7. Treatment of the mother with AZT during
3. Flank pain (if kidney involved) pregnancy decreases the risk of transmission
4. Hematuria of the virus to the fetus.
5. Pyuria B. Nursing implications
6. Fever and chills 1. Thorough review of history and any physical
C. Nursing interventions symptoms.
1. Encourage high fluid intake. 2. Close attention to lab studies, especially CBC,
2. Provide warm baths to relieve discomfort and leukocyte count, T-cell count, and urinalysis
promote perineal hygiene. indicated.
3. Administer and monitor intake of prescribed 3. Strict attention to universal precautions as
medications (antibiotics, urinary analgesics). appropriate.
4. Stress good bladder-emptying schedule. 4. Protective coverings in delivery room.
5. Monitor for signs of premature labor from 5. Wear gloves to handle all infants until they are
severe or untreated infection. bathed.
6. Suction newborn with bulb or wall suction
devices only.
Other Infections 7. Special assessments: respiratory, neurologic,
A. General information psychosocial.
1. Pregnancy is not a prevention against pre- or
coexisting infections.
2. Toxoplasmosis, other infections, rubella,
Other Conditions of Risk in Pregnancy
cytomegalovirus, and herpes (TORCH Adolescence
infections) are especially devastating to the
fetus, causing abortions, malformations, and A. General information
even fetal death. 1. Pregnancy is a condition of both physical and
3. Rubella titer is assessed during early prenatal psychologic risk.
visit. If mother is deficient in rubella 2. Adolescent is frequently undernourished and
antibodies (titer less than 1.0), rubella virus not yet completely matured either physically
vaccine is recommended in immediate or psychosocially.
postpartum period. 3. Adolescent is uniquely unsuited for the
B. General nursing interventions stresses of pregnancy.
1. Instruct the pregnant woman in signs and 4. Frequency of serious complications increases
symptoms that indicate infection, especially in adolescent pregnancy, particularly toxemia
fever, chills, sore throat, localized pain, or rash. and low-birth-weight infants.
2. Caution pregnant women to avoid obviously B. Nursing interventions
infected persons and other sources of 1. Encourage adequate prenatal care.
infection, as danger exists for the fetus in all 2. Provide health teaching to prepare for
maternal infections. pregnancy, labor and delivery, and
3. May affect delivery options. motherhood.
3. Provide nutritional counseling.
4. Teach coping skills for labor and delivery.
AIDS and Pregnancy
5. Teach child care skills.
A. General information 6. Refer adolescent to Crisis Pregnancy Center.
1. Transmission of the human immunodeficiency
virus authenticated through blood, semen, Disseminated Intravascular Coagulation (DIC)
vaginal secretions, and breast milk.
2. Can be transmitted from mother to fetus A. General information
during pregnancy. 1. Also known as consumptive coagulopathy
3. Cesarean delivery will not avert mother-to-fetus 2. A diffuse, pathologic form of clotting,
transmission. secondary to underlying disease/pathology.
4. Breastfeeding not currently recommended for 3. Occurs in critical maternity problems such as
seropositive mothers. abruptio placenta, dead fetus syndrome,
5. Increase in prematurity, premature rupture of amniotic fluid embolism, preeclampsia/
membranes, low birth weight, and coexistent eclampsia, hydatidiform mole, and
STDs. hemorrhagic shock
6. Pregnancy-altered immune states may result in 4. Mechanism
the acceleration of opportunistic diseases, a. Precoagulant substances released in the
blood trigger microthrombosis in

MATERNITY AND FEMALE REPRODUCTIVE NURSING

6 553
53155_06_Ch06_p533-620.qxd 2/23/09 12:23 PM Page 554

peripheral vessels and paradoxical f. Fetus may display IUGR (intrauterine


consumption of circulating clotting factors. growth retardation), CNS dysfunction, and
b. Fibrin-split products accumulate, further craniofacial abnormalities (fetal alcohol
interfering with the clotting process. syndrome).
c. Platelet and fibrinogen levels drop. 2. Cocaine
B. Assessment findings a. Powerful stimulant; very addictive
1. Bleeding may range from massive, b. Causes vasoconstriction, elevated BP,
unanticipated blood loss to localized bleeding tachycardia
(purpura and petechiae) c. May precipitate seizures
2. Presence of special maternity problems d. Affects ability to transport O2 into the
3. Prolonged prothrombin and partial blood
thromboplastin times e. May cause spontaneous abortion, fetal
C. Nursing interventions malformation, placenta abruptio, neural
1. Assist with medical management of tube defects
underlying condition. f. Newborn may display irritability,
2. Administer blood component therapy (WBCs, hypertonicity, poor feeding patterns,
packed cells, fresh frozen plasma, increased risk of SIDS
cryoprecipitate) as ordered. 3. Opiates
3. Observe for signs of insidious bleeding (oozing a. Produce analgesia, euphoria, respiratory
IV site, petechiae, lowered hematocrit). depression
4. Institute nursing measures for severe b. If used IV, foreign substance contamination
bleeding/shock if needed. may cause pulmonary emboli or infections
5. Provide emotional support to client and family c. If used IV, places mother at greater risk of
as needed. contracting HIV, then passing it on to fetus
d. Newborns experience withdrawal within
Anemia 2472 hours after delivery
e. High-pitched cry, restlessness, poor
A. General information feeding seen in the newborn
1. Low RBC may be underlying condition 4. Other chemicals
2. May or may not be exacerbated by physiologic a. May include tranquilizers, prescription
hemodilution of pregnancy medications, paint thinners, other aerosols,
3. Most common medical disorder of pregnancy etc.
B. Assessment findings b. Major danger is overdose, with
1. Client is pale, tired, short of breath, dizzy. accompanying cardiac/respiratory arrest
2. Hgb is less than 11 gram/dL; hct less than 37%. C. Nursing interventions
C. Nursing interventions 1. Treatment during pregnancy may include
1. Encourage intake of foods with high iron in- or outpatient care. Alcoholics Anonymous-
content. based programs are widely utilized.
2. Monitor iron supplementation. 2. Treatment may include family therapy.
3. Teach sequelae of iron ingestion. 3. Efforts to treat the chemical abuse/dependency
4. Assess need for parenteral iron. should be maximized during pregnancy.
Withdrawal is best accomplished with
Prenatal Substance Abuse competent, professional help.
A. General information
1. Incidence: probably underestimated in our
society. Sample Questions
2. Morbidity/mortality: related to chemical used,
timing, and route of administration.
B. Assessment findings 1. A 22-year-old woman has missed two of her
1. Alcohol regular menstrual periods. Her doctor confirms
a. Elevates the mood, depresses the central an early, intrauterine pregnancy. This is her first
nervous system pregnancy. To determine her expected due date,
b. Affects every other system in the body of which of the following assessments is most
the mother important?
c. Displaces other nutritional food intake
d. Greatest risk from high blood alcohol 1. Date of first menstrual period.
levels 2. Date of last intercourse.
e. No safe level of maternal alcohol use in 3. Date of last normal menstrual period.

6
pregnancy has been established 4. Age at menarche.

554 NCLEX-RN Review


53155_06_Ch06_p533-620.qxd 2/23/09 12:23 PM Page 555

2. A 24-year-old woman is pregnant with her first 8. A woman is pregnant and diabetic. Why would a
baby. During her seventh month, she complains glycosylated hemoglobin level be ordered?
of backache. What teaching can the nurse 1. It is the most accurate method of determining
provide to help with comfort? present insulin levels.
1. Sleep on a soft mattress. 2. It will predict how well the pancreas can
2. Walk barefoot at least once/day. respond to the stress of pregnancy.
3. Perform Kegel exercises once/day. 3. It indicates mean glucose level over a 1- to
4. Wear low-heeled shoes. 3-month period.
4. It gives diagnostic information related to the
3. A woman is hospitalized for the treatment of peripheral effects of diabetes.
severe preeclampsia. Which of the following
represents an unusual finding for this condition? 9. A 25-year-old woman is 5 months pregnant and
1. Convulsions. has been suffering from morning sickness since
2. Blood pressure 160/100. early in her pregnancy. She is now admitted for
hyperemesis gravidarum and parenteral fluid
3. Proteinuria 41.
therapy is started. She has vomited twice within
4. Generalized edema. the last hour. What would be the priority
nursing intervention to perform?
4. A woman is admitted with severe preeclampsia.
What type of room should the nurse select for 1. Assist her with mouth care.
this woman? 2. Notify the physician.
1. A room next to the elevator. 3. Change the IV infusion to Ringers lactate.
2. The room farthest from the nursing station. 4. Warm her tray and serve it to her again.
3. The quietest room on the floor.
10. A woman in her seventh month of pregnancy
4. The labor suite. has a hemoglobin of 10.5 g. The nurse teaches
the woman about proper nutrition during
5. How does the action of hormones during
pregnancy. Which statement made by the client
pregnancy affect the body?
indicates to the nurse that teaching was
1. Raises resistance to insulin. effective?
2. Blocks the release of insulin from the pancreas. 1. I eat liver once a week.
3. Prevents the liver from metabolizing 2. I have an orange for breakfast.
glycogen.
3. I eat six small meals a day.
4. Enhances the conversion of food to glucose.
4. I have a green leafy vegetable occasionally.
6. A 28-year-old woman has had diabetes mellitus
11. A couple recently arrived in the United States
since she was an adolescent. She is 8 weeks
from East Asia. The man brings his wife to the
pregnant. Hyperglycemia during her first
hospital in late labor; his mother and the
trimester will have what effect on the fetus?
womans sister are also present. As the nurse
1. Hyperinsulinemia. directs the man to the dressing room to change
2. Excessive fetal size. into a scrub suit, his wife anxiously states, No,
3. Malformed organs. he cant come with me. Get my sister and
4. Abnormal positioning. mother-in-law! What would be the nurses best
response?
7. The nurse is caring for a young diabetic woman 1. Im sorry, but our hospital only allows the
who is in her first trimester of pregnancy. As the father into the delivery.
pregnancy continues the nurse should anticipate 2. Ill ask the doctor if thats OK.
which change in her medication needs?
3. When I talk to your husband, Im sure hell
1. A decrease in the need for short-acting want to be with you.
insulins.
4. Thats fine. Ill show your husband to the
2. A steady increase in insulin requirements. waiting area.
3. Oral hypoglycemic drugs will be given
several times daily. 12. During an initial prenatal visit, a woman states
4. The variable pattern of insulin absorption that her last menstrual period began on
throughout the pregnancy requires constant November 21; she also reports some vaginal

6
close adjustment.

MATERNITY AND FEMALE REPRODUCTIVE NURSING 555


53155_06_Ch06_p533-620.qxd 2/23/09 12:23 PM Page 556

bleeding about December 19. What would be the 17. A young woman had her pregnancy confirmed
calculated expected date of birth (EDB)? and has completed her first prenatal visit.
1. July 21. Considering that all data were found to be
2. August 28. within normal limits, how soon will the nurse
plan the next visit?
3. September 26.
1. One week.
4. October 1.
2. Two weeks.
13. A 24-year-old woman comes to the clinic 3. One month.
because she thinks she is pregnant. Which of the 4. Two months.
following is a probable sign of pregnancy that
the nurse would expect this client to have? 18. Which statement by a pregnant client would
1. Fetal heart tones. indicate to the nurse that diet teaching has been
2. Nausea and vomiting. effective?
3. Amenorrhea. 1. The most important time to take my iron
pills is during the early weeks when the baby
4. Chadwicks sign.
is forming.
14. A married 25-year-old housewife is 6 weeks 2. I dont like milk, but Ill increase my intake
gestation and is being seen for her first prenatal of cheese and yogurt.
visit. In relation to normal maternal acceptance 3. Ill be very careful about using salt while Im
of pregnancy, what would the nurse expect that pregnant.
the client feels? 4. Because Im overweight to begin with, I can
1. Some ambivalence now that the pregnancy is continue my weight loss diet.
confirmed.
2. Overwhelmed by the thought of future 19. A woman, age 40, gravida 3 para 2, is 8 weeks
changes. pregnant. She is a full-time office manager and
states she usually unwinds with a few glasses
3. Much happiness and enjoyment in the event.
of wine with dinner, smokes about five
4. Detached from the event until physical cigarettes a day, and was surprised by this
changes occur. pregnancy. After the assessment, which of the
following would the nurse select as the priority
15. A woman is entering the 20th week of
nursing diagnosis?
pregnancy. Which normal change would the
nurse expect to find on assessment? 1. Risk for an impaired bonding related to an
unplanned pregnancy.
1. Fundus just below diaphragm.
2. Risk for injury to the fetus related to
2. Pigment changes in skin.
advanced age.
3. Complaints of frequent urination.
3. Ineffective individual coping related to low
4. Blood pressure returning to prepregnancy self-esteem.
level.
4. Deficient knowledge related to effects of
16. A primigravida in the first trimester is blood substance abuse.
type A1, rubella negative, hemoglobin 12 g,
20. A young couple has just completed a
hematocrit 36%. During her second prenatal
preconception visit in the maternity clinic.
visit she complains of being very tired,
Before leaving, the woman asks the nurse why
experiencing frequent urination, and a white
she was instructed not to take any over-the-
vaginal discharge; she also states that her nausea
counter medications. What would be the nurses
and occasional vomiting persist. Based on these
correct reply?
findings, the nurse would select which of the
following nursing diagnoses? 1. Research has found that many of these drugs
have been linked to problems with getting
1. Activity intolerance related to nutritional
pregnant.
deprivation.
2. At conception, and in the first trimester,
2. Impaired urinary elimination related to a
these drugs can be as dangerous to the fetus
possible infection.
as prescription drugs.
3. Risk for injury related to hematologic
3. You should only take drugs that the
incompatibility.
physician has ordered during pregnancy.
4. Alteration in physiologic responses related to

6
4. Any drug is dangerous at this time; later on
pregnancy.
in pregnancy it wont matter.

556 NCLEX-RN Review


53155_06_Ch06_p533-620.qxd 2/23/09 12:23 PM Page 557

21. The pregnant couple asks the nurse what is the 26. A woman who is pregnant for the first time calls
purpose of prepared childbirth classes. What the clinic to say she is bleeding. To obtain
would be the nurses best response? important information, what question should be
1. The main goal of most types of childbirth asked by the nurse?
classes is to provide information that will 1. When did you last feel the baby move?
help eliminate fear and anxiety. 2. How long have you been pregnant?
2. The desired goal is childbirth without the 3. When was your pregnancy test done?
use of analgesics. 4. Are you having any uterine cramping?
3. These classes help to eliminate the pain of
childbirth by exercise and relaxation methods. 27. A woman is hospitalized with a possible ectopic
4. The primary aim is to keep you and your pregnancy. In addition to the classic symptoms
baby healthy during pregnancy and after! of abdominal pain, amenorrhea, and abnormal
vaginal bleeding, the nurse knows that which of
22. A woman in her 38th week of pregnancy is to the following factors in the womans history may
have an amniocentesis to evaluate fetal maturity. be associated with this condition?
The L/S (lecithin/sphingomyelin) ratio is 2:1. 1. Multiparity.
What is the indication of this finding? 2. Age under 20.
1. Fetal lung maturity. 3. Pelvic inflammatory disease (PID).
2. That labor can be induced. 4. Habitual spontaneous abortions.
3. The fetus is not viable.
4. A nonstress test is indicated. 28. A woman is being discharged after treatment for
a hydatidiform mole. The nurse should include
23. A woman is having a contraction stress test which of the following in the discharge teaching
(CST) in her last month of pregnancy. When plan?
assessing the fetal monitor strip, the nurse 1. Do not become pregnant for at least one year.
notices that with most of the contractions, the 2. Have blood pressure checked weekly for
fetal heart rate uniformly slows at mid- 6 months.
contraction and then returns to baseline about
3. RhoGAM must be received with next
20 seconds after the contraction is over. How
pregnancy and delivery.
would the nurse interpret this test result?
4. An amniocentesis can detect a recurrence of
1. Negative: normal.
this disorder in the future.
2. Reactive: negative.
3. Positive: abnormal. 29. A woman, 40 weeks gestation, is admitted to the
4. Unsatisfactory. labor and delivery unit with possible placenta
previa. On the admission assessment, what
24. A woman, 36 weeks gestation, is having a CST would the nurse expect to find?
with an oxytocin IV infusion pump. After two 1. Signs of a Couvelaire uterus.
contractions, the uterus stays contracted. What 2. Severe lower abdominal pain.
would be the best initial action of the nurse?
3. Painless vaginal bleeding.
1. Help the client turn on her left side.
4. A board-like abdomen.
2. Turn off the infusion pump.
3. Wait 3 minutes for the uterus to relax. 30. A woman, 30 weeks gestation, is being
4. Administer prn terbutaline sulfate (Brethine). discharged to home care with a diagnosis of
placenta previa. What statement by the client
25. A pregnant woman, in the first trimester, is to indicates she understands her care at home?
have a transabdominal ultrasound. The nurse 1. As I get closer to my due date I will have to
would include which of the following remain in bed.
instructions? 2. I can continue with my office job because
1. Nothing by mouth (NPO) from 6:00 A.M. the its mostly sitting.
morning of the test. 3. My husband wont be too happy with this
2. Drink one to two quarts of water and do not no sex order.
urinate before the test. 4. Im disappointed that I will need a cesarean
3. Come to the clinic first for injection of the section.
contrast dye.

6
4. No special instructions are needed for this test.

MATERNITY AND FEMALE REPRODUCTIVE NURSING 557


53155_06_Ch06_p533-620.qxd 2/23/09 12:23 PM Page 558

31. A teenage client, 38 weeks gestation, is


admitted with a diagnosis of pregnancy-induced Answers and Rationales
hypertension (PIH). Data include: blood pressure
160/100, generalized edema, weight gain of
10 pounds in last 2 weeks, and proteinuria of 1. 3. The dates of the last menstrual period,
13; the client is also complaining of a headache especially the first day of that period, will be
and nausea. In planning care for this client, used in applying Ngeles rule to determine the
which priority goal would the nurse establish? estimated date of delivery.
1. Demonstrate a decreased blood pressure
2. 4. A frequent cause of backache in the third
within 48 hours.
trimester of pregnancy is the combined effect of
2. Not experience a seizure prior to delivery. relaxation of the sacroiliac joints and the change
3. Maintain a strict diet prior to delivery. in the center of gravity of the pregnant woman
4. Comply with medical and nutritional regimen. due to the enlarging uterus. Wearing low-heeled
shoes, especially when on her feet for extended
32. A woman, 32 weeks gestation, has developed periods of time, will help to minimize this
mild PIH. What statement by the client would discomfort.
indicate understanding of her treatment regimen?
1. It is most important not to miss any of my 3. 1. Convulsions are associated with an eclamptic
blood pressure medication. condition when blood pressure is increased
2. I will watch my diet restrictions very above 160/110 mm Hg.
carefully. 4. 3. A quiet room in which stimuli are minimized
3. I will spend most of my time in bed, on my and controlled is essential to the nursing care of
left side. the severely preeclamptic client.
4. Im happy that this only happens during a
first pregnancy. 5. 1. Hormonal influences during pregnancy cause
a resistance to insulin utilization at the cellular
33. A pregnant client with class 3 cardiac disease is level. It allows sufficient glucose for placental
seen during an initial prenatal visit. The nurse transport to the fetus, and also prevents the
selects which of the following priority nursing blood sugar in the nondiabetic client from
diagnoses? falling to dangerous levels. In the diabetic client,
1. Knowledge deficit related to self-care during it requires increases in her insulin doses.
pregnancy.
6. 3. Major congenital malformations are noted in
2. Fear; client and family, related to pregnancy
the insulin-dependent diabetic mother with poor
outcome.
metabolic control.
3. Alteration in nutrition related to sodium-
restricted diet. 7. 2. During the first trimester of pregnancy, there
4. Activity intolerance related to compromised is little change in insulin requirements. In the
cardiac status. second trimester, gradually increasing amounts
of insulin are needed, with the insulin dose
34. The nurse includes the importance of self- doubling by the end of pregnancy.
monitoring of glucose in the care plan for a
diabetic client planning a pregnancy. What does 8. 3. Glycosylated hemoglobin measurements can
the goal of this monitoring prevent? be used to assess prior glycemic control, giving
1. Congenital malformations in the fetus. the average over the past 1 to 3 months.
2. Maternal vasculopathy. 9. 1. Frequent vomiting irritates the oral mucosa
3. Accelerated growth of the fetus. and leaves the mouth very dry and foul tasting.
4. Delayed maturation of fetal lungs. The first nursing action should be aimed at
relieving irritation and drying of the mouth by
35. What question will the nurse ask to assess a providing mouth care.
females highest risk for developing
toxoplasmosis during pregnancy? 10. 1. Liver contains more iron than any other food
1. Do you have any pets? source.
2. Do you consume any alcohol beverages?
11. 4. Within the traditional East Asian family, roles
3. Are you depressed? are clearly defined. One consideration is the East

6
4. Has your blood pressure been elevated?

558 NCLEX-RN Review


53155_06_Ch06_p533-620.qxd 2/23/09 12:23 PM Page 559

Asian husbands lack of involvement during 21. 1. All programs in prepared childbirth have
pregnancy and birth; this is a mutually agreeable some similarities; all have an educational
separation of mens and womens roles. component to help eliminate fear.

12. 2. If a woman has a menstrual period every 22. 1. Lecithin and sphingomyelin are
28 days and was not taking oral contraceptives, phospholipids produced by the type II alveolar
Ngeles rule may be a fairly accurate determiner cells. The L/S ratio increases with gestation and
of her predicted birth date. To use this method, a ratio of 2:1 indicates lung maturity.
begin with the first day of the last menstrual
period, subtract 3 months, and add 7 days. 23. 3. The CST subjects the fetus to uterine
contractions that compress the arteries supplying
13. 4. Probable signs of pregnancy are the result of the placenta, thus reducing placental blood flow
physiologic changes in the pelvic organs and and the flow of oxygen to the fetus; the fetus with
hormonal influences; for example, the mucous minimal metabolic reserve will have late
membranes of the vulva, vagina, and cervix decelerations where the fetal heart rate does not
become bluish (Chadwicks sign) as a result of return to the baseline until the contraction ends.
hyperemia and proliferation of cells. Fetal compromise is therefore suggested.

14. 1. During the first trimester of pregnancy, 24. 2. When IV oxytocin is being used to stimulate
women normally experience ambivalence about uterine contractions in a contraction stress test,
being pregnant. It is estimated that around 80% the oxytocin infusion is stopped if contractions
of women initially reject the idea of pregnancy; occur more often than every 2 minutes or last
even women who planned pregnancy may longer than 60 seconds, if uterine tetany
respond at first with surprise and shock. (remains contracted) takes place, or if continued
fetal heart rate decelerations are noted.
15. 2. From 2024 weeks gestation, pigment
changes in skin may occur from actions of 25. 2. To obtain clearer images during the first
hormones. These include the linea nigra, trimester, women are required to drink 1 to 2
melasma on the face, and striae gravidarum quarts of clear fluid to fill the urinary bladder and
(stretch marks). thereby push the uterus higher into the abdomen
where it can be more accurately scanned.
16. 4. All of the data stated are within the normal
expected range for a first trimester pregnancy. 26. 2. When a pregnant woman is bleeding
These factors are related to hormonal changes vaginally, the nurse should first ask her how
and the growing uterus. many weeks or months pregnant she is;
management of bleeding differs in an early
17. 3. In a low-risk pregnancy, the recommended pregnancy contrasted with bleeding in late
frequency of prenatal visits is: once very month pregnancy. Additional information would
until the 7th or 8th month, every 2 weeks during include if tissue amniotic fluid was discharged
the 8th month, then every week until birth. and what other symptoms, such as cramps or
pain, are present.
18. 2. To meet increased calcium needs, pregnant
women need to increase their intake of dairy 27. 3. The incidence of ectopic pregnancy in the
products or consider a calcium supplement that United States has increased by a factor of 4.9
provides 600 mg of calcium per day; it is not during recent years. This is attributed primarily
necessary to drink milk. to the growing number of women of childbearing
age who experience PID and endometriosis, who
19. 4. Evidence exists that smoking, consuming
use intrauterine devices, or who have had tubal
alcohol, or using social drugs during pregnancy
surgery.
may be harmful to the fetus.
28. 1. The follow-up protocol of critical importance
20. 2. It is best to avoid any medication when
after a molar pregnancy is the assessment of
planning a pregnancy and during the first
serum chorionic gonadotropin (HCG); HCG is
trimester; the greatest potential for gross
considered a highly specific tumor marker for
abnormalities in the fetus occurs during the first
gestational trophoblastic disease (GTD). The
trimester, when fetal organs are first developing.
HCG levels are assayed at intervals for 1 year; a
The greatest danger extends from day 31 after
rise or plateau necessitates further diagnostic
the last menstrual period to day 71.
assessment and usually treatment. Pregnancy

MATERNITY AND FEMALE REPRODUCTIVE NURSING

6 559
53155_06_Ch06_p533-620.qxd 2/23/09 12:23 PM Page 560

would obscure the evidence of choriocarcinoma and renal perfusion; urine output increases, and
by the normal secretion of HCG. blood pressure may stabilize or decrease.

29. 3. Placenta previa, when the placenta is 33. 4. Once pregnancy is established, the focus of
implanted in the lower uterine segment, often is management is on minimizing any extra cardiac
characterized by the sudden onset of bright red demands on the pregnant woman. In class 3
bleeding in the third trimester. Usually this cardiac disease, the client experiences fatigue,
bleeding is painless and may or may not be palpitation, dyspnea, or angina when she
accompanied by contractions. undertakes less than ordinary activity. Physical
activity is markedly restricted; this includes bed
30. 3. In placenta previa, any sexual arousal is rest throughout the pregnancy.
contraindicated because it can cause the release
of oxytocin, which can cause the cervix to pull 34. 1. There is increasing evidence that the degree
away from the low-lying placenta; this results in of control for an insulin-dependent diabetic
bleeding and potential jeopardy to the fetus. woman prior to conception greatly affects the
fetal outcome. Studies find that poor maternal
31. 2. Preeclampsia may progress to eclampsia, the glucose control underlies the incidence of
convulsive phase of PIH. Symptoms that herald congenital malformations in the infants of
the progression include headache, visual diabetic mothers.
disturbances, epigastric pain, nausea or
vomiting, hyperreflexia, and oliguria; classical 35. 1. Cats are intermediate hosts for toxoplasmosis.
signs of PIH also intensify. As transmission of the toxic parasite is via the
cats feces, have someone else change the litter
32. 3. Modified bed rest in the left lateral position box daily.
may be advised for the client with mild PIH. This
position improves venous return and placental

Labor and Delivery

OVERVIEW B. Fetal shoulders: may be manipulated during


delivery to allow passage of one shoulder at a time.
C. Presentation: that part of the fetus which enters the
Five Factors of Labor (Five Ps) pelvis in the birth process (Figure 6-9). Types of
Passenger presentation are:
1. Cephalic: head is presenting part; usually
The size, presentation, and position of the fetus. vertex (occiput), which is most favorable for
A. Fetal head (Figure 6-8) birth. Head is flexed with chin on chest.
1. Usually the largest part of the baby; it has 2. Breech: buttocks or lower extremities present
profound effect on birthing process. first. Types are:
2. Bones of skull are joined by membranous a. Frank: thighs flexed, legs extended on
sutures, which allow for overlapping or anterior body surface, buttocks presenting
molding of cranial bones during birth b. Full or complete: thighs and legs flexed,
process. buttocks and feet presenting (baby in
3. Anterior and posterior fontanels are the points squatting position)
of intersection for the sutures and are c. Footling: one or both feet are presenting
important landmarks. 3. Shoulder: presenting part is the scapula, and
a. Anterior fontanel is larger, diamond- baby is in horizontal or transverse position.
shaped, and closes about 18 months of age. Cesarean birth indicated.
b. Posterior fontanel is smaller, triangular, D. Position: relationship of reference point on
and usually closes about 3 months of age. fetal presenting part to maternal bony pelvis
4. Fontanels are used as landmarks for internal (Figure 6-10).
examinations during labor to determine 1. Maternal bony pelvis divided into four
position of fetus. quadrants (right and left anterior; right and left

6 560 NCLEX-RN Review


53155_06_Ch06_p533-620.qxd 2/23/09 12:23 PM Page 561

Bregna
Frontal
anterior
suture Vertex Sinciput
Sinciput fontanel
Anterior
fontanel
Frontal bone
s

Frontal
Parietal bone
Coronal Parietal Parietal bone
suture bone bone

x
Verte
Temporal
Sagittal
bone
suture Sphenoid
Me
fontanel
ntu
m

Occipital
Occipital bone
Lambdoid Occiput
bone
suture Posterior Occiput Mastoid
fontanel fontanel
(A) (B)

Figure 6-8 Fetal skullsutures and fontanels. (A) Superior view; (B) Lateral view

posterior). Relationship is expressed in a a. LOA (left occiput anterior): fetal occiput


three-letter abbreviation: first the maternal is on maternal left side and toward front,
side (R or L), next the fetal presentation, and face is down. This is a favorable delivery
last the maternal quadrant (A or P). Most position.
common positions are: b. ROA (right occiput anterior): fetal occiput
on maternal right side toward front, face is
down. This is a favorable delivery position.
c. LOP (left occiput posterior): fetal occiput is
on maternal left side and toward back, face
is up. Mother experiences much back
discomfort during labor; labor may be
slowed; rotation usually occurs before
labor to anterior position, or health care
provider may rotate at time of delivery.
d. ROP (right occiput posterior): fetal occiput is
on maternal right side and toward back, face
A. B.
is up. Presents problems similar to LOP.
2. Assessment of fetal position can be made by:
a. Leopolds maneuvers: external palpation
(4 steps) of maternal abdomen to determine
fetal contours or outlines. Maternal obesity,
excess amniotic fluid, or uterine tumors
may make palpation less accurate.
b. Vaginal examination: location of sutures
and fontanels and determination of
relationship to maternal bony pelvis.
C. D. c. Rectal examination: now virtually
completely replaced by vaginal examination.
d. Auscultation of fetal heart tones and
determination of quadrant of maternal
Figure 6-9 Breech presentations. (A) Complete;
abdomen where best heard. (Correlate with
(B) Frank; (C) Footling; (D) Shoulder

6
Leopold maneuvers.)

MATERNITY AND FEMALE REPRODUCTIVE NURSING 561


53155_06_Ch06_p533-620.qxd 2/23/09 12:23 PM Page 562

ROP LOP

Posterior

Right Left

ROT LOT
Anterior

ROA LOA

Figure 6-10 Positions of a vertex presentation

Passageway 2. If presenting part is above ischial spines,


station expressed as a negative number
Shape and measurement of maternal pelvis and (e.g., 1, 2).
distensibility of birth canal (see also Overview of 3. If presenting part is below ischial spines,
Anatomy and Physiology). station expressed as a positive number
A. Engagement: fetal presenting part enters true pelvis (e.g., 11, 12).
(inlet). May occur 2 weeks before labor in primipara; 4. High or floating terms used to denote
usually occurs at beginning of labor for multipara. unengaged presenting part.
B. Station: measurement of how far the presenting part C. Soft tissue (cervix, vagina): stretches and dilates
has descended into the pelvis. Referrant is ischial under the force of contractions to accommodate
spines, palpated through lateral vaginal walls. the passage of the fetus.
1. When presenting part is at ischial spines,
station is 0, engaged.

6 562 NCLEX-RN Review


53155_06_Ch06_p533-620.qxd 2/23/09 12:23 PM Page 563

B. Position will depend on whether client has had an


epidural; consider maternal needs as well as fetal
well-being.
Iliac crest
Psychologic Response
Factors that influence labor as a positive/negative
experience
Baby's head A. Culture: how society views childbirth
-5 B. Expectations/goals realistic or unattainable
-4
-3
-2
C. Feedback from others involved in labor and
-1
0
Ischial spine delivery process
+1
+2
+3
+4
+5

cm.
The Labor Process
Causes
Actual cause unknown. Factors involved include
Figure 6-11 Station, or relationship of the fetal A. Progressive uterine distension
presenting part to the ischial spines. The station B. Increasing intrauterine pressure
illustrated is 12. C. Aging of the placenta
D. Changes in levels of estrogen, progesterone, and
prostaglandins
E. Increasing myometrial irritability

Powers Mechanisms (Vertex Presentation)


Forces of labor, acting in concert, to expel fetus and A. Engagement
placenta. Major forces are: 1. The biparietal diameter of the head passes the
A. Uterine contractions (involuntary) pelvic inlet.
1. Frequency: timed from the beginning of one 2. The head is fixed in the pelvis.
contraction to the beginning of the next B. Descent: progress of the presenting part through
2. Regularity: discernible pattern; better the pelvis
established as labor progresses C. Flexion: chin flexed more firmly onto chest by
3. Intensity: strength of contraction; a relative pressure on fetal head from maternal soft tissues
assessment without the use of a monitor. May (cervix, vaginal walls, pelvic floor)
be determined by the depressability of the D. Internal rotation
uterus during a contraction. Described as mild, 1. Fetal skull rotates along axis from transverse to
moderate, or strong. anteroposterior at pelvic outlet.
4. Duration: length of contraction. Contractions 2. Head passes the midpelvis.
lasting more than 90 seconds without a E. Extension: head passes under the symphysis pubis
subsequent period of uterine relaxation may and is delivered, occiput first, followed by face
have severe implications for the fetus and and chin.
should be reported. F. External rotation: head rotates to full alignment
B. Voluntary bearing-down efforts with back and shoulders for shoulder delivery
1. After full dilation of the cervix, the mother can mechanisms.
use her abdominal muscles to help expel the G. When entire body of baby has emerged from
fetus. mothers body, birth is complete. This time is
2. These efforts are similar to those for recorded as the time of birth.
defecation, but the mother is pushing out the
fetus from the birth canal. Stages of Labor
3. Contraction of levator ani muscles.
A. Definitions
Position 1. Stage 1: from onset of labor until full dilation
of cervix
A maternal position during the labor process. a. Latent phase: from 04 cm
A. Several positions possible: b. Active phase: 48 cm
1. Not confined to lying supine in bed c. Transition phase: 810 cm
2. Lateral recumbent (usually most comfortable)
3. Sitting in rocking chair

6
4. Semi-reclined in bed

MATERNITY AND FEMALE REPRODUCTIVE NURSING 563


53155_06_Ch06_p533-620.qxd 2/23/09 12:23 PM Page 564

2. Stage 2: from full dilation of cervix to birth of Electronic Fetal Monitoring


baby
3. Stage 3: from birth of baby to expulsion of A. Placement of ultrasound transducer and
placenta tocotransducer to record fetal heartbeat and
4. Stage 4: time after birth (usually 12 hours) uterine contractions and display them on special
of immediate recovery graph paper for comparison and identification of
B. Cervical changes in first stage of labor normal and abnormal patterns.
1. Effacement B. Can be applied externally to mothers abdomen, or
a. Shortening and thinning of cervix. internally, within uterus.
b. In primipara, effacement is usually well 1. External application
advanced before dilation begins; in a a. Less precise information collected
multipara, effacement and dilation b. May be affected by maternal movements
progress together. c. Noninvasive: rupture of membranes not
2. Dilation required, can be widely used
a. Enlargement or widening of the cervical d. Little danger associated with use
os and canal. 2. Internal application
b. Full dilation is considered 10 cm. a. More precise information collected
b. Cervix must be dilated and membranes
Duration of Labor ruptured to be utilized
c. Physician applies scalp electrode and
A. Depends on: uterine catheter
1. Regular, progressive uterine contractions d. Sterile technique must be maintained
2. Progressive effacement and dilation of cervix during application to reduce risk of
3. Progressive descent of presenting part intrauterine infection
B. Average length e. Can yield specific short-term variability
1. Primipara C. Pattern recognition
a. Stage 1: 1213 hours 1. Nurse is responsible for assessing FHR
b. Stage 2: 1 hour patterns, implementing appropriate nursing
c. Stage 3: 34 minutes interventions, and reporting suspicious
d. Stage 4: 12 hours patterns to physician.
2. Multipara 2. Baseline FHR: 120160, when uterus is not
a. Stage 1: 8 hours contracting.
b. Stage 2: 20 minutes 3. Variability is normal, indicative of intact fetal
c. Stage 3: 45 minutes nervous system. Variability is result of
d. Stage 4: 12 hours interaction of sympathetic and
parasympathetic nervous systems. Two types
of variability are:
ASSESSMENT DURING LABOR a. Short-term (beat-to-beat): assessed as
present/absent.
b. Long-term (rhythmic fluctuations):
Fetal Assessment classified according to number of
Auscultation cycles/minute. Average is 6/minute.
4. Tachycardia
Auscultate FHR at least every 1530 minutes during a. FHR more than 160 beats/minute, lasting
first stage and every 515 minutes during second stage longer than 10 minutes.
(depends on risk status of client). b. May have multiple causes.
A. Normal range 120160 beats/minute c. Oxygen may be administered.
B. Best recorded during the 30 seconds immediately 5. Bradycardia
following a contraction a. FHR less than 120 beats/minute lasting
longer than 10 minutes.
Palpation b. May have multiple causes.
c. Oxygen may be administered.
Assess intensity of contraction by manual palpation of
6. Early deceleration
uterine fundus.
a. Deceleration of FHR begins early in
A. Mild: tense fundus, but can be indented with
contraction, stays within normal range,
fingertips.
returns to baseline by end of contraction.
B. Moderate: firm fundus, difficult to indent with
b. Believed to be the result of compression of
fingertips.
fetal head against cervix.
C. Strong: very firm fundus, cannot indent with
c. Not an ominous pattern, no nursing
fingertips.

6
interventions required.

564 NCLEX-RN Review


53155_06_Ch06_p533-620.qxd 2/23/09 12:23 PM Page 565

7. Late deceleration 3.Bloody show


a. Deceleration of FHR begins late in 4.Progressive fetal descent
contraction; depth varies with strength of 5.Walking intensifies contractions
contraction; does not return to baseline by 6.Discomfort begins in back then radiates to
end of contraction. abdomen
b. May be occasional or consistent. Gradual B. False labor
increase in number is always suspicious 1. Irregular, inefficient contractions not causing
and MUST be reported/charted. the progressive changes associated with true
c. Believed to be the result of uteroplacental labor
insufficiency. 2. No bloody show
d. An ominous pattern. 3. Discomfort primarily in abdomen, may be
e. Nurse should place client in side-lying relieved by walking
position, or change maternal position, 4. Need to assess client over period of time to
administer oxygen, discontinue any differentiate from true labor
oxytocin infusion, assess variability,
prepare for immediate delivery if pattern
remains uncorrected. FIRST STAGE OF LABOR
8. Variable deceleration
a. Onset of deceleration not related to uterine
contraction.
Latent Phase (04 cm)
b. Swings in FHR abrupt and dramatic, return Assessment
to baseline frequently rapid.
c. Believed to be the result of compression of A. Contractions: frequency, intensity, duration
the umbilical cord. B. Membranes: intact or ruptured, color of fluid
d. Although not an ominous pattern, C. Bloody show
continued nursing assessment required. D. Time of onset
e. Nurse should change maternal position to E. Cervical changes
relieve pressure on cord; if no F. Time of last ingestion of food
improvement seen, administer oxygen, G. FHR every 15 minutes; immediately after rupture
discontinue oxytocin if infusing, prepare of membranes
client for vaginal exam to assess for H. Maternal vital signs
prolapsed cord (see also Prolapsed 1. Temperature every 2 hours if membranes
Umbilical Cord). ruptured, every 4 hours if intact
f. If cord is prolapsed, relieve pressure on 2. Pulse and respirations every hour or prn as
cord; do not attempt to replace cord. indicated
g. Cesarean delivery will be needed. 3. Blood pressure every half hour or prn as
indicated
I. Progress of descent (station)
Maternal Assessment J. Clients knowledge of labor process
Premonitory Assessment K. Clients affect
L. Clients birth plan
Physiologic changes preceding labor
A. Lightening (engagement): occurs up to two weeks Analysis
before labor in primipara; at beginning of labor for
multipara Nursing diagnoses for the latent phase of first stage of
B. Braxton Hicks contractions: may become more labor may include:
noticeable; may play a part in ripening of cervix A. Anxiety
C. Easier respirations from decreased pressure on B. Ineffective breathing pattern
diaphragm C. Pain
D. Frequent urination, from increased pressure on D. Deficient knowledge
bladder
E. Restlessness/poor sleeping patterns, nesting Planning and Implementation
behaviors
A. Goals
1. Complete all admission procedures.
True vs False Labor 2. Labor will progress normally.
A. True labor 3. Mother/fetus will tolerate latent phase
1. Contractions increased in frequency, intensity, successfully.
and duration B. Interventions
2. Progressive cervical changes 1. Administer perineal prep/enema if

6
ordered/appropriate.

MATERNITY AND FEMALE REPRODUCTIVE NURSING 565


53155_06_Ch06_p533-620.qxd 2/23/09 12:23 PM Page 566

2. Assess vital signs, blood pressure, fetal heart, 8. Keep client/couple informed as labor
contractions, bloody show, cervical changes, progresses.
descent of fetus as scheduled. 9. With posterior position, apply sacral
3. Maintain bed rest if indicated or required. counterpressure, or have father do so.
4. Reinforce/teach breathing techniques as
needed. Evaluation
5. Support laboring woman/couple based on
their needs. A. Labor progressing through active phase, dilation
6. Have client attempt to void every 12 hours. progressing
7. Apply external fetal monitoring if indicated B. Mother/fetus tolerating labor appropriately
or ordered. C. No complications observed

Evaluation Transition Phase (810 cm)


A. Admission procedures complete Assessment
B. Progress through latent stage normal, cervix
dilated A. Progress of labor
C. Mother/fetus tolerate latent phase well, mother as B. Cervical changes
comfortable as possible, vital signs normal; FHR C. Maternal mood changes: if irritable or aggressive
maintained in response to contractions may be tiring or unable to cope
D. Signs of nausea, vomiting, trembling, crying,
irritability
Active Phase (48 cm) E. Maternal/fetal vital signs
Assessment F. Breathing patterns, may be hyperventilating
G. Urge to bear down with contractions
A. Cervical changes
B. Bloody show Analysis
C. Membranes
D. Progress of descent Nursing diagnoses for the transition phase of first stage
E. Maternal/fetal vital signs of labor may include:
F. Clients affect A. Ineffective breathing pattern
B. Powerlessness
Analysis C. Ineffective coping

Nursing diagnoses for the active phase of first stage Planning and Implementation
of labor may include:
A. Ineffective coping A. Goals
B. Impaired oral mucous membranes 1. Labor will continue to progress through
C. Deficient knowledge transition.
D. Pain 2. Mother/fetus will tolerate process well.
E. Ineffective tissue perfusion 3. Complications will be avoided.
F. Risk for injury B. Interventions
1. Continue observation of labor progress,
Planning and Implementation maternal/fetal vital signs.
2. Give mother positive support if tired
A. Goals or discouraged.
1. Progress will be normal through active phase. 3. Accept behavioral changes of mother.
2. Mother/fetus will successfully complete active 4. Promote appropriate breathing patterns
phase. to prevent hyperventilation.
B. Interventions 5. If hyperventilation present, have mother
1. Continue to observe labor progress. rebreathe expelled carbon dioxide to reverse
2. Reinforce/teach breathing techniques as respiratory alkalosis.
needed. 6. Discourage pushing efforts until cervix is
3. Position client for maximum comfort. completely dilated, then assist with pushing.
4. Support client/couple as mother becomes 7. Observe for signs of delivery.
more involved in labor.
5. Administer analgesia if ordered or indicated. Evaluation
6. Assist with anesthesia if given and monitor
maternal/fetal vital signs. A. Mother/fetus progressed through transition
7. Provide ice chips or clear fluids for mother to B. No complications observed

6
drink if allowed/desired. C. Mother/fetus ready for second stage of labor

566 NCLEX-RN Review


53155_06_Ch06_p533-620.qxd 2/23/09 12:23 PM Page 567

SECOND STAGE OF LABOR C. Maternal/fetal vital signs


D. Maternal pushing efforts
(See Figure 6-12) E. Vaginal distension
F. Bulging of perineum
Assessment G. Crowning
H. Birth of baby
A. Signs of imminent delivery
B. Progress of descent

Figure 6-12 Mechanisms of second stage labor

6
MATERNITY AND FEMALE REPRODUCTIVE NURSING 567
53155_06_Ch06_p533-620.qxd 2/23/09 12:23 PM Page 568

Analysis Table 6-7 Apgar Scoring


Nursing diagnoses for the second stage of labor may
include: Category 0 1 2 Score
A. Risk for injury
B. Noncompliance related to exhaustion Heart Rate Absent <100 >100 ______
C. Deficient knowledge Respiratory Absent Slow, Good cry
Effort Irregular
Muscle tone Absent/ Some Active ______
Planning and Implementation limp flexion motion
A. Goals Reflex No Grimace Cry ______
1. Safe delivery of living, uninjured fetus. irritability response
2. Mother will be comfortable after tolerating Color Blue, pale Body pink, All pink ______
delivery. extremities
B. Interventions blue
1. If necessary, transfer mother carefully to delivery Total Score ______
table or birthing chair; support legs equally to
prevent/minimize strain on ligaments.
2. Carefully position mother on delivery table, in
delivery chair, or birthing bed to prevent
popliteal vein pressure. A. Pain
3. Help mother use handles or legs to pull on as B. Risk for deficient fluid volume
she bears down with contractions.
4. Clean vulva and perineum to prepare for delivery.
5. Continue observation of maternal/fetal vital Planning and Implementation
signs. A. Goals
6. Encourage mother in sustained (57 seconds) 1. Placenta will be delivered without
pushes with each contraction. complications.
7. Support fathers participation if in delivery area. 2. Maternal blood loss will be minimized.
8. Catheterize mothers bladder if indicated. 3. Mother will tolerate procedures well.
9. Keep mother informed of delivery B. Interventions
progress. 1. Palpate fundus immediately after delivery of
10. Note time of delivery of baby. placenta; massage gently if not firm.
2. Palpate fundus at least every 15 minutes for
Evaluation first 12 hours.
3. Observe lochia for color and amount.
A. Delivery of healthy viable fetus 4. Inspect perineum.
B. Mother comfortable after procedure 5. Assist with maternal hygiene as needed.
C. No complications during procedure a. Clean gown.
b. Warm blanket.
THIRD STAGE OF LABOR c. Clean perineal pads.
6. Offer fluids as indicated.
7. Promote beginning relationship with baby and
Assessment parents through touch and privacy.
A. Signs of placental separation 8. Administer medications as ordered/needed
1. Gush of blood (pitocin added to IV if present).
2. Lengthening of cord
3. Change in shape of uterus (discoid to globular) Evaluation
B. Completeness of placenta
C. Status of mother/baby contact for first critical A. Placenta delivered without complications
12 hours B. Minimal maternal blood loss
1. Babys Apgar scores (see Table 6-7) C. Mother tolerated procedure well
2. Blood pressure, pulse, respirations, lochia,
fundal status of mother
FOURTH STAGE OF LABOR
Analysis
Assessment
Nursing diagnoses for the third stage of labor may
include: A. Fundal firmness, position

6
B. Lochia: color, amount

568 NCLEX-RN Review


53155_06_Ch06_p533-620.qxd 2/23/09 12:23 PM Page 569

C. Perineum 2. Cause is frequently unknown, but the


D. Vital signs following conditions are associated with
E. IV if running premature labor:
F. Infants heart rate, airway, color, muscle tone, a. Cervical incompetence
reflexes, warmth, activity state b. Preeclampsia/eclampsia
G. Bonding/family integration c. Maternal injury
d. Infection (urinary tract infection)
Analysis e. Multiple gestation, polyhydramnios
f. Placental disorders
Nursing diagnoses for the fourth stage of labor may 3. Preterm labor: prevention
include: a. Minimize or stop smoking: a major factor
A. Pain in preterm labor and birth.
B. Risk for deficient fluid volume b. Minimize or stop substance
C. Interrupted family processes abuse/chemical dependency.
c. Early and consistent prenatal care.
Planning and Implementation d. Appropriate diet/weight gain.
e. Minimize psychological stressors.
A. Goal: critical first hour(s) after delivery will pass f. Minimize/prevent exposure to infections.
without complications for mother/baby. g. Learn to recognize signs and symptoms of
B. Interventions preterm labor.
1. Palpate fundus every 15 minutes for first 4. Incidence of preterm labor is between 5% and
12 hours; massage gently if not firm. 10% in all pregnancies and is a major cause of
2. Check mothers blood pressure, pulse, perinatal mortality.
respirations every 15 minutes for first B. Medical management
12 hours or until stable. 1. Unless labor is irreversible, or a condition
3. Check lochia for color and amount every exists in which the mother or fetus would be
15 minutes for first 12 hours. jeopardized by the continuation of the
4. Inspect perineum every 15 minutes for first pregnancy, or the membranes have ruptured,
12 hours. the usual medical intervention is to attempt to
5. Apply ice to perineum if swollen or if arrest the premature labor (tocolysis).
episiotomy was performed (see Episiotomy). 2. Medications used in the treatment of
6. Encourage mother to void, particularly if premature labor
fundus not firm or displaced. a. Magnesium sulfate
a. Use nursing techniques to encourage 1) Stops uterine contractions with fewer
voiding. side effects than beta-adrenergic drugs.
b. If client unable to void, get order for 2) Interferes with muscle contractility.
catheterization. 3) Administered IV for 12 to 24 hours, PO
c. Measure first voiding. form of magnesium may be used for
7. Encourage early bonding; through maintenance.
breastfeeding if desired. a) Loading dose of 46 g IV over 20 to
30 minutes
Evaluation b) Maintenance dose IV 13 g/hr
(IV piggyback)
A. Mothers vital signs stable, fundus and lochia c) Must monitor client for magnesium
within normal limits toxicity
B. Evidence of bonding; parents cuddle, touch, talk to 4) Few serious side effects; initially client
baby feels hot, flushed, sweats, may complain
C. No complications observed for mother or baby of headache, nausea, diarrhea,
during crucial time dizziness, nystagmus, and lethargy.
5) Most serious side effect: respiratory
depression.
COMPLICATIONS OF LABOR 6) Most common fetal side effect is
hypotonia.
AND DELIVERY b. Beta-adrenergic drugsterbutaline and
ritodrine.
Premature/Preterm Labor 1) Decreases effect of calcium on muscle
activation to slow or stop uterine
A. General information
contractions.
1. Labor that occurs before the end of the 37th
2) Initially given IV, then PO brethine
week of pregnancy.

6
(terbutaline) for maintenance.

MATERNITY AND FEMALE REPRODUCTIVE NURSING 569


53155_06_Ch06_p533-620.qxd 2/23/09 12:23 PM Page 570

3) Terbutaline: 6) Support client through stressful period


a) 18 mg/min 3 812 hr of treatment and uncertainty.
b) 2.5 to 5 mg PO q 48 hr 7) Teach client necessity of continuing
4) Ritodrine oral medication at home if discharged.
a) 0.050.1 mg/min increased to b. Magnesium sulfate: carefully monitor
0.35 mg/min until contractions respirations, reflexes, and urinary output
stop (see also Severe Preeclampsia)
b) 1020 mg q 2 hr for 24 hours 5. Keep client informed of all progress/changes.
5) Side effects: increased heart rate, 6. Identify side effects/complications as early as
nervousness, tremors, nausea and possible.
vomiting, decrease in serum K1 level, 7. Carry out activities designed to keep client
cardiac arrhythmias, pulmonary edema. comfortable.
c. Nifedipine
1) Calcium channel blocker
2) 1030 mg loading dose, oral or
Postmature/Prolonged Pregnancy
sublingual; second dose may be given A. General information
in 30 min if contractions persist; 1. Defined as those pregnancies lasting beyond
1020 mg orally q 46 hr for the end of the 42nd week.
maintenance 2. Fetus at risk due to placental degeneration and
3) Side effects: facial flushing, mild loss of amniotic fluid (cord accidents).
hypotension, reflex tachycardia, 3. Decreased amounts of vernix also allow the
headache, nausea drying of the fetal skin, resulting in a dry,
d. Indomethacin parchmentlike skin condition.
1) Prostaglandin synthetase inhibitor B. Medical management
2) Loading dose: 50100 mg PO or 1. Directed toward ascertaining precise fetal
rectally: 25 mg q 46 hr for 2448 hr gestational age and condition, and determining
maintenance fetal ability to tolerate labor
3) Side effects: nausea, vomiting, 2. Induction of labor and possibly cesarean birth
dyspepsia C. Assessment findings
3. When premature labor cannot or should not be 1. Measurements of fetal gestational age for fetal
arrested and fetal lung maturity needs to be maturity
improved, the use of betamethasone 2. Biophysical profile
(Celestone) can improve the L/S ratio of lung D. Nursing interventions
surfactants. It is administered IM to the 1. Perform continual monitoring of maternal/fetal
mother, usually every 12 hr 3 2, then weekly vital signs.
until 34 weeks gestation. 2. Support mother through all testing and labor
C. Nursing interventions 3. Assist with amnio-infusion if ordered to
1. Keep client at rest, side-lying position. increase cushion for cord.
2. Hydrate the client and maintain with IV or
PO fluids.
3. Maintain continuous maternal/fetal
Prolapsed Umbilical Cord
monitoring. A. General information
a. Maternal/fetal vital signs every 10 minutes; 1. Displacement of cord in a downward
be alert for abrupt changes. direction, near or ahead of the presenting part,
b. Monitor maternal I&O. or into the vagina
c. Monitor urine for glucose and ketones. 2. May occur when membranes rupture, or with
d. Watch cardiac and respiratory status ensuing contractions
carefully. 3. Associated with breech presentations,
e. Evaluate lab test results carefully. unengaged presentations, and premature
4. Administer drugs as ordered/indicated. labors
a. Terbutaline 4. Obstetric emergency: if compression of the
1) Position client on side as much as cord occurs, fetal hypoxia may result in CNS
possible. damage or death.
2) Apply external fetal monitor. B. Assessment findings: vaginal exam identifies cord
3) Complete maternal/fetal assessment prolapse into vagina
before each increase in dosage rate. C. Nursing interventions
4) Special maternal assessment includes 1. Check fetal heart tones immediately when
respiratory status, blood pressure, membranes rupture, and again after next
pulse, I&O, lab values. contraction, or within 5 minutes; report

6
5) Notify physician of significant changes. decelerations.

570 NCLEX-RN Review


53155_06_Ch06_p533-620.qxd 2/23/09 12:23 PM Page 571

2. If fetal bradycardia, perform vaginal 2. Meconium-stained amniotic fluid with a


examination and check for prolapsed cord. vertex presentation
3. If cord prolapsed into vagina, exert upward 3. Fetal scalp sampling (may be needed for a
pressure against presenting part to lift part off definitive diagnosis)
cord, reducing pressure on cord. C. Nursing interventions
4. Get help to move mother into a position where 1. Check FHR on appropriate basis, institute fetal
gravity assists in getting presenting part off monitoring if not already in use.
cord (knee-chest position or severe 2. Conduct vaginal exam for presentation and
Trendelenburgs). position.
5. Administer oxygen, and prepare for immediate 3. Place mother on left side, administer oxygen,
cesarean birth. check for prolapsed cord, notify physician.
6. If cord protrudes outside vagina, cover with 4. Support mother and family.
sterile gauze moistened with sterile saline 5. Prepare for emergency birth if indicated.
while carrying out above tasks. Do not attempt
to replace cord.
7. Notify physician.
Dystocia
A. General information
1. Any labor/delivery that is prolonged or
Premature Rupture of Membranes difficult
A. General information 2. Usually results from a change in the
1. Loss of amniotic fluid, prior to term, interrelationships among the 5 Ps (factors in
unconnected with labor. labor/delivery): passenger, passage, powers,
2. Dangers associated with this event are position, and psyche of mother.
prolapsed cord, infection, and the potential 3. Frequently seen causes include
need for premature delivery. a. Disproportion between fetal presentation
B. Assessment findings (usually the head) and maternal pelvis
1. Report from mother/family of discharge of (cephalopelvic disproportion [CPD]).
fluid. 1) If disproportion is minimal, vaginal
2. pH of vaginal fluid will differentiate between birth may be attempted if fetal injuries
amniotic fluid (alkaline) and urine or purulent can be minimized or eliminated.
discharge (acidic). 2) Cesarean birth needed if disproportion
C. Nursing interventions is great.
1. Monitor maternal/fetal vital signs on b. Problems with presentation
continuous basis, especially maternal 1) Any presentation unfavorable for
temperature. delivery (e.g., breech, shoulder, face,
2. Calculate gestational age. transverse lie)
3. Observe for signs of infection and for signs of 2) Posterior presentation that does not
onset of labor. rotate, or cannot be rotated with ease.
a. If signs of infection present, administer 3) Cesarean birth is the usual
antibiotics as ordered and prepare for intervention.
immediate delivery. c. Problems with maternal soft tissue
b. If no maternal infection, induction of labor 1) A full bladder may impede the progress
may be delayed. of labor, as can myomata uteri, cervical
4. Observe and record color, odor, amount of edema, scar tissue, and congenital
amniotic fluid. anomalies.
5. Examine mother for signs of prolapsed cord. 2) Emptying the bladder may allow labor
6. Provide explanations of procedures and to continue; the other conditions may
findings, and support mother/family. necessitate cesarean birth.
7. Prepare mother/family for early birth if d. Dysfunctional uterine contractions
indicated. 1) Contractions may be too weak, too
short, too far apart, ineffectual.
2) Progress of labor is affected;
Fetal Distress progressive dilation, effacement, and
A. General information: common contributing factors descent do not occur in the expected
are: pattern.
1. Cord compression. 3) Classification
2. Placental abnormalities. a) Primary: inefficient pattern present
3. Preexisting maternal disease. from beginning of labor; usually a
B. Assessment findings prolonged latent phase.

6
1. Decelerations in FHR

MATERNITY AND FEMALE REPRODUCTIVE NURSING 571


53155_06_Ch06_p533-620.qxd 2/23/09 12:23 PM Page 572

b) Secondary: efficient pattern that


changes to inefficient or stops; may Table 6-8 Emergency Delivery of an Infant
occur in any stage.
If you have to deliver the baby yourself
B. Assessment findings
1. Progress of labor slower than expected rate of Assess the clients affect and ability to understand directions,
dilation, effacement, descent for specific client as well as other resources available (other physicians,
2. Length of labor prolonged nurses, auxiliary personnel).
3. Maternal exhaustion/distress Stay with client at all times; mother must not be left alone if
4. Fetal distress delivery is imminent.
C. Nursing interventions Do not prevent birth of baby.
1. Individualized as to cause. Maintain sterile environment if possible.
2. Provide comfort measures for client. Rupture membranes if necessary.
3. Provide clear, supportive descriptions of all Support babys head as it emerges, preventing too-rapid
actions taken. delivery with gentle pressure.
4. Administer analgesia if ordered/indicated. Check for nuchal cord, slip over head if possible.
5. Prepare oxytocin infusion for induction of Use gentle aspiration with bulb syringe to remove blood
labor if ordered. and mucus from nose and mouth.
6. Monitor mother/fetus continuously. Deliver shoulders after external rotation, asking mother to
7. Prepare for cesarean birth if needed. push gently if needed.
Provide support for babys body as it is delivered.
Precipitous Labor and Delivery Hold baby in head-down position to facilitate drainage of
secretions.
A. General information Promote cry by gently rubbing over back and soles of feet.
1. Labor of less than 3 hours Dry to prevent heat loss.
2. Emergency delivery without clients physician Place baby on mothers abdomen.
or midwife Check for signs of placental separation.
B. Assessment findings Check mother for excess bleeding; massage uterus prn.
1. As labor is progressing quickly, assessment Hold placenta as it is delivered.
may need to be done rapidly. Cut cord when pulsations cease, if cord clamp is available;
2. Client may have history of previous if no clamps, leave intact.
precipitous labor and delivery. Wrap baby in dry blanket, give to mother; put to breast
3. Desire to push. if possible.
4. Observe status of membranes, perineal area for Check mother for fundal firmness and excess bleeding.
bulging, and for signs of bleeding. Record all pertinent data.
C. Nursing interventions (see Table 6-8) Comfort mother and family as needed.

Amniotic Fluid Embolism


A. General information 3. Administer medications to control bleeding as
1. Escape of amniotic fluid into the maternal ordered.
circulation, usually in conjunction with a 4. Prepare for emergency birth of baby.
pattern of hypertonic, intense uterine 5. Keep client/family informed as possible.
contractions, either naturally or oxytocin
induced.
2. Obstetric emergency: may be fatal to the
Induction of Labor
mother and to the baby. A. General information: deliberate stimulation of
B. Assessment findings uterine contractions before the normal occurrence
1. Sudden onset of respiratory distress, of labor.
hypotension, chest pain, signs of shock B. Medical management: may be accomplished by
2. Bleeding (DIC) 1. Amniotomy (the deliberate rupture of
3. Cyanosis membranes)
4. Pulmonary edema 2. Oxytocins, usually Pitocin
C. Nursing interventions 3. Prostaglandin (PGE2) in gel/suppository form
1. Initiate emergency life support activities for to improve cervical readiness
mother. C. Assessment findings
a. Administer oxygen. 1. Indications for use
b. Utilize CPR in case of cardiac arrest. a. Postmature pregnancy
2. Establish IV line for blood transfusion and b. Preeclampsia/eclampsia
monitoring of CVP. c. Diabetes

6
d. Premature rupture of membranes

572 NCLEX-RN Review


53155_06_Ch06_p533-620.qxd 2/23/09 12:23 PM Page 573

2. Condition of fetus: mature, engaged vertex C. Examples of medications used systemically for
fetus in no distress labor analgesia
3. Condition of mother: cervix ripe for 1. Sedatives: help to relieve anxiety; may use
induction, no CPD secobarbital (Seconal), sodium pentobarbital
D. Nursing interventions (Nembutal), phenobarbital.
1. Explain all procedures to client. 2. Narcotic analgesics: help to relieve pain; may
2. Prepare appropriate equipment and use morphine, meperidine (Demerol),
medications. butorphanol (Stadol), fentanyl (Sublimaze),
a. Amniotomy: a small tear made in amniotic nalbuphine hydrochloride (Nubain).
membrane as part of sterile vaginal exam 3. Narcotic antagonists: given to reverse narcotic
1) Explain sensations to client. depression of mother or baby; may use
2) Check FHR immediately before and naloxone (Narcan), levallorphan (Lorfan).
after procedure; marked changes may 4. Analgesic potentiating drugs: given to raise
indicate prolapsed cord. desired effect of analgesic without raising dose
3) Additional care as for woman with of analgesic drug; may use promethazine
premature rupture of membranes. (Phenergan), promazine (Sparine),
b. Oxytocin (Pitocin): IV administration, propiomazine (Largon), hydroxyzine (Vistaril).
piggybacked to main IV D. Medication administration
1) Usual dilution 10 milliunit/1000 mL 1. IV: the preferred route; allows for smaller
fluid, delivered via infusion pump for doses, better control of administration, better
greatest accuracy in controlling dosage. prediction of action.
2) Usual administration rate is 0.51.0 2. IM: still widely used; needs larger dose,
milliunit/min, increased no more than absorption may be delayed or erratic.
12 milliunit/min at 4060-minute 3. SC: used occasionally for small doses of
intervals until regular pattern of nonirritating drugs.
appropriate contractions is established
(every 23 minutes, lasting less than Assessment
90 seconds, with 3045 second rest
period between contractions). A. Clients perception of pain/discomfort
3. Know that continuous monitoring and B. Baseline vital signs for later comparison
accurate assessments are essential. C. Known allergies
a. Apply external continuous fetal D. Current status of labor: medications best given in
monitoring equipment. active phase of first stage of labor
b. Monitor maternal condition on a E. Time of previous doses of medications
continuous basis: blood pressure, pulse,
progress of labor. Analysis
4. Discontinue oxytocin infusion when:
a. Fetal distress is noted. Nursing diagnoses for labor analgesia may
b. Hypertonic contractions occur. include:
c. Signs of other obstetric complications A. Pain
(hemorrhage/shock, abruptio placenta, B. Ineffective coping
amniotic fluid embolism) appear. C. Deficient knowledge
5. Notify physician of any untoward reactions.
Planning and Implementation
A. Goals
ANALGESIA AND ANESTHESIA 1. Medication will relieve maternal discomfort.
2. Maternal comfort will be achieved with least
Analgesia for Labor effect of medication on fetus.
B. Interventions
General Information 1. Administer medications on schedule to
A. Definition: the easing of pain or discomfort by the maximize maternal effect and minimize fetal
administration of medication that blocks pain effect.
recognition or the raising of the pain recognition 2. Continue to observe maternal/fetal vital signs
threshold. for side effects.
B. Sources of pain/discomfort 3. Explain to client that she must remain in bed
1. First stage of labor: stretching of cervix and with side rails up.
uterine contractions 4. Record accurately drug used, time, amount,
2. Second stage of labor: stretching of birth canal route, site, and client response.

6
and perineum

MATERNITY AND FEMALE REPRODUCTIVE NURSING 573


53155_06_Ch06_p533-620.qxd 2/23/09 12:23 PM Page 574

Evaluation keep flat for at least 68 hours


postspinal and encourage oral
A. Medication exerts intended effect. fluids postdelivery to facilitate
B. Mother reports positive response to medication. reversal of headache.
C. Fetus shows no ill effects from medication. 2) Peripheral nerve blocks
D. Labor is not affected by medication. a) Paracervical: instillation of
medication into cervix; rarely used
Anesthesia for Labor and Delivery during labor because of effect on
fetus; useful only in first stage of
General Information labor.
b) Pudendal: medication injected
A. Removal of pain perception by the administration
transvaginally to affect pudendal
of medication to interrupt the transmission of
nerve as it passes behind ischial
nerve impulses to the brain.
spines on either side of vagina;
B. May be administered by inhalation, IV, or regional
useful for delivery and episiotomy
routes.
if needed.
C. All methods of labor and delivery anesthesia have
c) Perineal (local): injected into the
their drawbacks; no one method is perfect.
perineum at the time of delivery in
D. Types of labor and delivery anesthesia
order to perform an episiotomy
1. Inhalation: mother inhales controlled
concentration of gaseous medication.
a. Administered by trained personnel only Assessment
b. Methoxyflurane (Penthrane) and nitrous A. Status of labor progress
oxide commonly used B. Maternal/fetal vital signs
c. Dangers include regurgitation and C. Allergies
aspiration, uterine relaxation, and D. Effects of medication on client (she may need help
hemorrhage postdelivery. pushing)
2. IV: rarely used in uncomplicated vaginal E. Level of anesthesia
deliveries; may be used for cesarean birth F. Return of sensation after anesthesia
(as induction anesthesia).
a. Administered by trained personnel only
Analysis
b. Sodium pentothal commonly used
3. Regional: medication introduced to specific Nursing diagnoses for anesthesia during labor and
areas to block pain impulses delivery may include:
a. Always administered by skilled personnel A. Impaired mobility
b. Medications used include tetracaine B. Ineffective tissue perfusion
(Pontocaine), lidocaine, bupivacaine C. Deficient knowledge
(Marcaine), mepivacaine (Carbocaine) D. Impaired urinary elimination
c. May block nerve at the root or in a
peripheral area Planning and Implementation
1) Nerve root blocks
a) Lumbar epidural: may be given A. Goals
continuously or intermittently 1. Pain relief will be obtained.
during labor, or at time of delivery; 2. Healthy maternal/fetal status will be
medication is injected over dura maintained.
through lumbar interspace; B. Interventions
absorption of drug is slower, with 1. Assist client to empty bladder.
less hypotension; client should 2. Assist client to assume appropriate position.
have no postspinal headache a. Inhalation anesthesia: supine with wedge
b) Caudal: may be given under right hip to displace gravid uterus
intermittently through labor, or at off inferior vena cava
time of delivery; medication is b. Pudendal and perineal: on back or left
injected through sacral hiatus into side
peridural space; client should have c. Other regional types: on left side or
no postspinal headache sitting up
c) Subarachnoid (low spinal, saddle 3. Check maternal blood pressure and fetal heart
block): given when delivery is rate every 35 minutes until stable, then every
imminent; medication is injected 15 minutes or prn.
into spinal fluid; mother may 4. If blood pressure drops, turn client on left
experience postdelivery headache; side, administer oxygen, and notify physician.

6 574 NCLEX-RN Review


53155_06_Ch06_p533-620.qxd 2/23/09 12:23 PM Page 575

Evaluation Reasons include maternal fatigue, medical


conditions, fetal distress, poor pushing, and
A. Client experiences expected pain relief. excessive infant size.
B. Fetus exhibits no untoward effects, FHR remains 2. Prerequisites include: head is engaged, no
relatively stable. CPD, membranes ruptured, cervix completely
C. Labor and delivery carried out as expected. dilated, empty bladder.
D. Clients vital signs remain within normal limits. B. Types of assisted deliveries
1. Low outlet forceps used when the head is
visible at the perineum.
OPERATIVE OBSTETRICAL 2. Mid and high forceps no longer used.
3. Vacuum extraction used when head is
PROCEDURES visiblesilastic suction cup applied to
presenting part and gentle traction exerted
Episiotomy while mother pushes.
C. Nursing interventions
A. General information
1. Anticipate request for forceps if possible.
1. Incision made in the perineum to enlarge the
2. Monitor FHTs continuously.
vaginal opening for delivery
3. Explain procedure to client if awake, advise
2. Client usually anesthetized in some manner
mother/family about possible presence of
3. Types
bruising that will go away but may contribute to
a. Midline or median: from posterior vaginal
jaundice, also risk of perineal or vaginal tearing.
opening through center of perineum
4. Newborn assessment should include careful
toward anal sphincter.
examination for bruising and facial nerve damage
1) Most frequently used
with forceps and cephalhematoma with vacuum.
2) Easily done, least discomfort for client
5. Ongoing newborn assessment includes careful
3) Danger of extension into anal sphincter
checking for jaundice.
b. Mediolateral: begins at posterior vaginal
opening but angles off to left or right at
45 angle (rarely in the United States). Cesarean Birth
1) Done when need for additional
enlargement of vaginal opening is a General Information
possibility A. Delivery of the baby through an incision into the
2) Mediolateral episiotomy usually more abdominal and uterine walls
uncomfortable than median B. Indications
4. Advantages of episiotomy are: 1. Fetal distress, disease, or anomaly
a. Enlarging of vaginal opening 2. Breech or other malpresentation,
b. Second stage of labor shortened cephalopelvic disproportion, macrosomia
c. Stretching of perineal muscles minimized 3. Placenta previa or abruptio
d. Tearing of perineum may be prevented 4. Prolapsed cord and other obstetric emergencies
5. Those opposed to episiotomies argue: 5. Failure to progress in labor
a. Kegel exercises can prepare the perineum 6. Multiple gestation
to stretch for delivery. 7. Maternal disease
b. Lacerations may occur anyway. 8. Previous uterine surgery
6. Side-lying delivery minimizes the strain on 9. Active herpes
the perineum and may, if used, reduce C. Types
incidence of episiotomies. 1. Classical: vertical incisions made into both
B. Nursing interventions abdomen and uterus
1. Apply ice packs to perineal area in first a. Used when rapid delivery is important, as
12 hours to help alleviate pain and swelling. in fetal distress, prolapsed cord, placenta
2. Help promote healing with warm sitz baths abruptio
after first 12 hours. b. Maternal bleeding greater with this
3. Observe episiotomy site for signs of infection method; client may have increased risk of
or hematoma. uterine rupture of scar tissue with future
4. Instruct client about perineal hygiene. pregnancies; not usually a candidate for
vaginal birth in future pregnancies.
Assisted Delivery: Vacuum or Forceps 2. Low cervical/low segment: transverse
incisions made in abdomen (above pubic
A. General information hairline) and in uterus
1. Used when there is a need to shorten the a. Most common method used
second stage or the second stage has stopped.

MATERNITY AND FEMALE REPRODUCTIVE NURSING

6 575
53155_06_Ch06_p533-620.qxd 2/23/09 12:23 PM Page 576

b. Procedure may take longer than classic a. Implement general postsurgical care and
because of need to deflect bladder, but blood general postpartum care.
loss is lessened and adhesions are fewer. b. Assist client with self-care as needed.
c. Vaginal birth after this type of cesarean c. Assist mother with baby care and handling
birth (VBAC) is a possibility. as needed.
d. Encourage client to verbalize reaction to all
Preoperative events.
e. Reinforce any special discharge
A. Assessment instructions from physician.
1. Maternal/fetal responses to labor D. Evaluation
2. Indications for cesarean birth 1. Mother and baby tolerated procedures well
3. Blood and urine test results 2. No postoperative complications or infection
B. Analysis: nursing diagnoses for the preoperative 3. Maternal/newborn bonding occurring
phase of cesarean birth may include:
1. Fear
2. Knowledge deficit
3. Powerlessness Sample Questions
4. Disturbance in self-concept
C. Planning and implementation
1. Goals 36. The nurse is caring for a woman who is admitted
a. Client prepared for surgery carefully and to the hospital in active labor. What information
competently. is most important for the nurse to assess to avoid
b. Client will have procedures explained to respiratory complications during labor and
her. delivery?
2. Interventions 1. Family history of lung disease.
a. Shave/prep abdomen and pubic area.
b. Insert retention catheter into bladder. 2. Food or drug allergies.
c. Administer preoperative medications as 3. Number of cigarettes smoked daily.
ordered. 4. When the client last ate.
d. Explain all procedures to client.
e. Provide emotional support to client/family 37. A woman who is gravida 1 is in the active phase
as needed. of stage 1 labor. The fetal position is LOA. What
f. Complete all preoperative charting should the nurse expect to see when the
responsibilities. membranes rupture?
D. Evaluation 1. A large amount of bloody fluid.
1. Client adequately prepared for surgery
2. A moderate amount of clear to straw-colored
2. Client understands all procedures
fluid.
Postoperative 3. A small amount of greenish fluid.
4. A small segment of the umbilical cord.
A. Assessment
1. Maternal vital signs 38. The nurse is caring for a woman in stage 1 labor.
2. Observation of incision for signs of infection The fetal position is LOA. When the membranes
3. I&O rupture, what should be the nurses first action?
4. Level of consciousness/return of sensation 1. Notify the physician.
5. Fundal firmness and location
6. Lochia: color, amount, clots, odor 2. Measure the amount of fluid.
B. Analysis: nursing diagnoses postoperatively for 3. Count the fetal heart rate.
cesarean birth may include: 4. Perform a vaginal examination.
1. Alteration in comfort: pain
2. High risk for fluid volume deficit 39. A woman has just delivered a 9 lb 10 oz baby.
3. High risk for alteration in parenting After the delivery, the nurse notices that the
4. Altered family processes mother is chilly and that her fundus has relaxed.
C. Planning and implementation The nurse administers the oxytocin that the
1. Goals physician orders. What occurrence will alert the
a. Healing will be promoted. nurse that the oxytocin has had the expected
b. Bonding between mother/couple and baby effect?
will be promoted. 1. The mother states that she feels warmer now.
c. No complications will ensue.
2. The mother falls asleep.

6
2. Interventions

576 NCLEX-RN Review


53155_06_Ch06_p533-620.qxd 2/23/09 12:23 PM Page 577

3. The baby cries. postpartum complication and are not normal?


4. The uterus becomes firm. Check all that apply.
_____ A foul lochial odor.
40. A woman had a midline episiotomy performed _____ Discomfort while sitting.
at delivery. What is the primary purpose of the
_____ Ecchymosis and edema of the perineum.
episiotomy?
_____ Separation of the episiotomy wound
1. Allow forceps to be applied.
edges.
2. Enlarge the vaginal opening.
3. Eliminate the possibility of lacerations. 46. The nurse is talking with a woman who is
4. Eliminate the need for cesarean birth. 36 weeks gestation during a prenatal visit.
Which statement indicates that the woman
41. A woman is admitted to the hospital in labor. understands the onset of labor?
Vaginal examination reveals that she is 8 cm 1. I need to go to the hospital as soon as the
dilated. At this point in her labor, which of the contractions become painful.
following statements would the nurse expect her 2. If I experience bright red vaginal bleeding
to make? I know that I am about to deliver.
1. I cant decide what to name my baby. 3. I need to go to the hospital when I am having
2. It feels good to push with each contraction. regular contractions and bloody show.
3. Take your hand off my stomach when I have 4. My labor will not start until after my
a contraction. membranes rupture and I gush fluid.
4. This isnt as bad as I expected.
47. Using Leopolds maneuvers to determine fetal
42. The nurse is caring for a woman who is in labor. position, the nurse finds that the fetus is in a
She is 8 cm dilated. How will the nurse best vertex position with the back on the left side.
support the woman during this phase of her Where is the best place for the nurse to listen for
labor? fetal heart tones?
1. Leave her alone most of the time. 1. In the right upper quadrant of the mothers
2. Offer her a back rub during contractions. abdomen.
3. Offer her sips of oral fluids. 2. In the left upper quadrant of the mothers
4. Provide her with warm blankets. abdomen.
3. In the right lower quadrant of the mothers
43. A woman in labor is placed on an external fetal abdomen.
monitor. The nurse notices that the fetal heart 4. In the left lower quadrant of the mothers
rate is erratic during contractions but returns to abdomen.
baseline at the end of each contraction. How will
this occurrence be recorded? 48. Which of the following is the best way for the
1. Early decelerations. nurse to assess contractions in a client
2. Variable decelerations. presenting to the labor and delivery area?
3. Late decelerations. 1. Place the client on the electronic fetal
monitor with the labor toco at the
4. Fetal distress.
fundus.
44. During labor, the nurse observes variable 2. Ask the client to describe the frequency,
decelerations on the external fetal monitor. What duration, and strength of her contractions.
is the best action for the nurse to take at this 3. Use Leopolds maneuvers to determine the
time? quality of the uterine contractions.
1. Apply an oxygen mask. 4. Place the fingertips of one hand on the
2. Change the womans position to left fundus to determine frequency, duration,
side-lying. and strength of contractions.
3. Get the woman out of bed and walk her
49. As the nurse assigned to a laboring woman, you
around.
are observing the fetal heart rate. Which of the
4. Move the woman to the delivery room. following findings would you consider abnormal
for a client in active labor?
45. During delivery, a mediolateral episiotomy is
performed and a 7 lb 8 oz baby delivered. Which 1. A rate of 160 with no significant changes

6
of the following nursing assessments indicate a through a contraction.

MATERNITY AND FEMALE REPRODUCTIVE NURSING 577


53155_06_Ch06_p533-620.qxd 2/23/09 12:23 PM Page 578

2. A rate of 130 with accelerations to 150 with 53. The nurse is providing care to a woman. During
fetal movement. the most recent vaginal examination the nurse
3. A rate that varies between 120 and 130. feels the cervix 6 cm dilated, 100% effaced, with
4. A rate of 170 with a drop to 140 during a the vertex at 1 station. Based on this information,
contraction. the nurse is aware the woman is in which phase
of labor?
50. A woman arrives at the birthing center in active 1. Active labor with the head as presenting part
labor. On examination, the cervix is 5 cm not yet engaged.
dilated, membranes intact and bulging, and the 2. Transition with the backside as presenting
presenting part at 21 station. The woman asks if part fully engaged.
she can go for a walk. What is the best response 3. Latent phase labor with the backside as
for the nurse to give? presenting part fully engaged.
1. I think it would be best for you to remain in 4. Active labor with the head as presenting part
bed at this time because of the risk of cord fully engaged.
prolapse.
2. Its fine for you to walk, but please stay 54. A woman is completely dilated and at 12
nearby. If you feel a gush of fluid, I will need station. Her contractions are strong and last
to check you and your baby. 5070 seconds. Based on this information, the
3. It will be fine for you to walk because that nurse should know that the client is in which
will assist the natural body forces to bring the stage of labor?
baby down the birth canal. 1. First stage.
4. I would be glad to get you a bean bag chair 2. Second stage.
or rocker instead. 3. Third stage.
4. Fourth stage.
51. A primigravida presents to the labor room
with rupture of membranes at 40 weeks 55. A 28-year-old primigravida is admitted to the
gestation. Her cervix is 2 cm dilated and 100% labor room. She is 2 cm dilated, 90% effaced,
effaced. Contractions are every 10 minutes. and the head is at 0 station. Contractions are
What should the nurse include in the plan every 10 minutes lasting 2030 seconds.
of care? Membranes are intact. Admitting vital signs are:
1. Allow her to ambulate as desired as long as blood pressure 110/70, pulse 78, respirations 16,
the presenting part is engaged. temperature 98.8 F, and fetal heart rate 144.
2. Assess fetal heart tones and maternal status What should the nurse monitor?
every five minutes. 1. Blood pressure and contractions hourly and
3. Place her on an electronic fetal monitor for fetal heart rate every 15 minutes.
continuous assessment of labor. 2. Temperature, blood pressure, and
4. Send her home with instructions to return contractions every 4 hours and fetal heart rate
when contractions are every 5 minutes. hourly.
3. Contractions, effacement, and dilation of
52. A woman who is in active labor at 4 cm cervix, and fetal heart rate every hour.
dilated, 100% effaced, and 0 station is
4. Contractions, blood pressure, and fetal heart
ambulating and experiences a gush of fluid.
rate every 15 minutes.
What is the most appropriate initial action for
the nurse to take? 56. A womans cervix is completely dilated with the
1. Send a specimen of the amniotic fluid to the head at 22 station. The head has not descended
laboratory for analysis. in the past hour. What is the most appropriate
2. Have the woman return to her room and initial assessment for the nurse to make?
place her in Trendelenburg position to 1. Assess to determine if the clients bladder is
prevent cord prolapse. distended.
3. Have the woman return to her room so that 2. Send the client for X-rays to determine fetal
you can assess fetal status, including size.
auscultation of fetal heart tones for one full 3. Notify the surgical team so that an operative
minute. delivery can be planned.
4. Call the womans physician because a 4. Assess fetal status, including fetal heart
cesarean delivery will be required.

6
tones, and scalp pH.

578 NCLEX-RN Review


53155_06_Ch06_p533-620.qxd 2/23/09 12:23 PM Page 579

57. A woman who has been in labor for 6 hours is cervix is 2 cm dilated and 70% effaced. What
now 9 cm dilated and has intense contractions should the nurse include in the plan of care for
every 1 to 2 minutes. She is anxious and feels this client?
the need to bear down with her contractions. 1. Discuss with the client the need to stop
What is the best action for the nurse to take? working after her discharge from the hospital.
1. Allow her to push so that delivery can be 2. Monitor the client and her fetus for response
expedited. to impending delivery.
2. Encourage panting breathing through 3. Assess the clients past pregnancy history to
contractions to prevent pushing. determine if she has experienced preterm
3. Reposition her in a squatting position to labor in the past.
make her more comfortable. 4. Start oral terbutaline to stop the contractions.
4. Provide back rubs during contractions to
distract her. 62. A woman was admitted in premature labor
contracting every 5 minutes. Her cervix is 3 cm
58. A newborn, at 1 minute after vaginal delivery, is dilated and 100% effaced, IV magnesium sulfate
pink with blue hands and feet, has a lusty cry, at 1 g per hour is infusing. How will the nurse
heart rate 140, prompt response to stimulation know the drug is having the desired effect?
with crying, and maintains minimal flexion, 1. The contractions will increase in frequency
with sluggish movement. What will the nurse to every 3 minutes, although there will be no
record as the Apgar score? further cervical changes.
1. 10. 2. The woman will be able to sleep through her
2. 9. contractions due to the sedative effect of the
3. 8. magnesium sulfate.
4. 7. 3. The contractions will diminish in frequency
and finally disappear.
59. A woman delivered a 7 lb boy by spontaneous 4. The woman will have diminished deep
vaginal delivery 30 minutes ago. Her fundus is tendon reflexes and her blood pressure will
firm at the umbilicus and she has moderate decrease.
lochia rubra. Which nursing diagnosis is highest
priority as the nurse plans care? 63. A woman has just received an epidural for
1. Risk for infection related to episiotomy. anesthesia during her labor. What should the
2. Constipation related to fear of pain. nurse include in the plan of care because of the
anesthesia?
3. Potential for impaired urinary elimination
related to perineal edema. 1. Assist the client in position changes and
observe for signs of labor progress.
4. Deficient knowledge related to lack of
knowledge regarding newborn care. 2. Administer 5001000 mL of a sugar-free
crystalloid solution.
60. A woman is in the fourth stage of labor. She and 3. Place a Foley catheter as soon as the
her new daughter are together in the room. What anesthesia has been administered.
assessments are essential for the nurse to make 4. Offer the client a back rub to reduce the
during this time? discomfort of her contractions.
1. Assess the pattern and frequency of
contractions and the infants vital signs. 64. A woman delivered her infant son 3 hours ago.
2. Assess the womans vital signs, fundus, She had an episiotomy to facilitate delivery. As
bladder, perineal condition, and lochia. the nurse assigned to care for her, which of the
Assess the infants vital signs. following would be the most appropriate action?
3. Assess the womans vital signs, fundus, 1. Place an ice pack on the perineum.
bladder, perineal condition, and lochia. 2. Apply a heat lamp to the perineum.
Return the infant to the nursery. 3. Take her for a sitz bath.
4. Assess the infant for obvious abnormalities. 4. Administer analgesic medication as ordered.
Assess the woman for blood loss and firm
uterine contraction. 65. A woman is scheduled for a cesarean section
delivery due to a transverse fetal lie. What is the
61. A woman, G3 P2, was admitted at 32 weeks best way for the nurse to evaluate that she
gestation contracting every 710 minutes. Her understands the procedure?

MATERNITY AND FEMALE REPRODUCTIVE NURSING

6 579
53155_06_Ch06_p533-620.qxd 2/23/09 12:23 PM Page 580

1. Ask her about the help she will have at home Ecchymosis and edema of the perineum should
after her delivery. be checked. This could be caused by a number of
2. Give her a diagram of the body and ask her to things but indicates recovery will be prolonged.
draw the procedure for you.
Separation of the episiotomy wound edges
3. Ask her to tell you what she knows about the
should be checked. This might be due to
scheduled surgery.
infection or trauma. In this case the episiotomy
4. Provide her with a booklet explaining wound would have to heal by second intention
cesarean deliveries when she arrives at the (wound left open to heal) or third intention
hospital. (wound resutured).

46. 3. Regular contractions coupled with bloody


show suggest that cervical changes are occurring
Answers and Rationales as a result of contractions.

47. 4. The left lower quadrant is the correct location


36. 4. Gastric motility is decreased during since the back is on the left and the vertex is in
pregnancy. Food eaten several hours prior to the the pelvis.
onset of labor may still be in the stomach
undigested. This will influence the type of 48. 4. The fingertips of one hand allow the nurse to
anesthesia the client may receive. feel when the contraction begins and ends and
to determine the strength of the construction by
37. 2. With the baby in a vertex, LOA presentation the firmness of the uterus.
and no other indicators of distress, amniotic
fluid should be clear to straw-colored. 49. 4. A rate of 170 is suggestive of fetal tachycardia.
A drop to 140 during a contraction represents
38. 3. Immediately after the rupture of membranes, some periodic change, which is not a normal
the fetal heart tones are checked, then checked finding.
again after the next contraction or after 510
minutes. 50. 2. Although there is always some risk of
complications when membranes rupture, it is
39. 4. Oxytoxic medications such as Pitocin, safe for this woman to ambulate as long as she is
Methergine, and Ergotrate are administered to rechecked if rupture of membranes occurs.
stimulate uterine contractility and reverse
fundal relaxation in the postdelivery client. 51. 1. Ambulation will help contractions more
effectively dilate the cervix. As long as the
40. 2. An episiotomy is an incision made in the presenting part is engaged, there is no increased
perineum to enlarge the vaginal opening, risk of cord prolapse.
allowing additional room for the birth of the
baby. 52. 3. The most important nursing action after
rupture of the membranes is careful fetal
41. 3. At 8 cm dilated the client is in the transition assessment, including fetal heart tones counted
stage of her labor. Many women experience for 1 minute.
hyperesthesia of the skin at this time and would
not want to be touched during a contraction. 53. 1. At 6 cm dilation and complete effacement,
active labor is occurring. A station of 1
42. 2. The counterpressure of a back rub during a indicates that the vertex is above the ischial
contraction may relieve discomfort. spines and not fully engaged.
43. 2. Variable decelerations are frequently caused 54. 2. The second stage of labor extends from complete
by transient fetal pressure on the cord and are cervical dilation to delivery of the infant.
not a sign of fetal distress. A change in the
mothers position will usually relieve the 55. 1. During early labor, blood pressure and
problem. contractions should be monitored hourly and
fetal heart rate every 15 minutes.
44. 2. Changing the position of the mother will
relieve transient pressure on the umbilical cord. 56. 1. A full bladder may prevent the head from
moving down into the pelvic inlet. Often clients
45. A foul lochial odor should be checked. This is a do not have the sensation of a full bladder late in

6
sign of infection. labor, despite significant distention.

580 NCLEX-RN Review


53155_06_Ch06_p533-620.qxd 2/23/09 12:23 PM Page 581

57. 2. Because the client is still in transition and not 62. 3. If the magnesium sulfate is effective, you
ready to deliver, encouraging her to pant will would expect the contractions to decrease and
diminish the urge to push. then disappear. You would not continue to
perform vaginal exams if the desired result is
58. 3. This infant has 2 points for heart rate, occurring.
respiratory effort, and reflex irritability. One
point is awarded for color and muscle tone for a 63. 1. Epidural anesthesia may diminish the clients
total of 8. sensation of painful stimuli and movement.
Assistance and frequent assessment are therefore
59. 3. Perineal edema may affect urinary essential.
elimination. If allowed to continue, it may also
lead to excessive postpartum bleeding because 64. 1. Ice during the first 12 hours after delivery
the uterus cannot stay firmly contracted when causes vasoconstriction and thereby prevents
the bladder is excessively full. edema. Ice also provides pain relief through
numbing of the area.
60. 2. Assessment of the mother during fourth stage
includes elements related to her recovery from 65. 3. Asking for clarification of what she knows is
childbirth. Infant assessment focuses on stability the best way to evaluate what she understands of
and transition to extrauterine life. the procedure. If the client has additional
questions, the nurse can then clarify or amplify
61. 3. As a G3 P2, the clients past pregnancy history the information.
may provide some important information that
may shape the care rendered at this time.

The Postpartum Period

OVERVIEW 3. The endometrial surface is sloughed off as


lochia, in three stages:
a. Lochia rubra: red color, days 13 after
Physical Changes of the delivery; consists of blood and cellular
Postpartum Period debris from decidua.
b. Lochia serosa: pinkish brown, days 410;
A. The postpartum period is defined as that period of mostly serum, some blood, tissue debris.
time, usually 6 weeks, in which the mothers body c. Lochia alba: yellowish white, days 1121
experiences anatomic and physiologic changes up to 6 weeks; mostly leukocytes, with
that reverse the bodys adaptation to pregnancy; decidua, epithelial cells, mucus.
may also be called involution. 4. Lochia has a particular, musty odor. Foul-
B. Begins with the delivery of the placenta and ends smelling lochia, however, may indicate
when all body systems are returned to, or nearly infection. Some small dark clots may be
to, their prepregnant state. normal immediately after delivery; large clots
C. May or may not include the return of the and bright red clots signify the need for close
ovulatory/menstrual cycle. investigation.
5. The placental site heals by means of
Specific Body System Changes exfoliative shedding, a process that allows the
upward growth of the new endometrium and
Reproductive System the prevention of scar tissue at the old
A. Uterus undergoes involutionreturn to placental site. This process may take 6 weeks.
prepregnant size and position in pelvis. B. Cervix
1. Fundus palpated at the umbilicus in midline 1. Flabby immediately after delivery; closes
at 1-hour postpartum. slowly.
2. Fundus may rise up to 1 cm above umbilicus 2. Admits one fingertip by the end of 1 week
within 12 hours then begin descent of 1 cm after delivery.
3. Shape of external os changed by delivery from

6
per day until no longer palpable by day 10.
round to slitlike opening.

MATERNITY AND FEMALE REPRODUCTIVE NURSING 581


53155_06_Ch06_p533-620.qxd 2/23/09 12:23 PM Page 582

C. Vagina Urinary System


1. Edematous after delivery
2. May have small lacerations A. May have difficulty voiding in immediate
3. Smooth-walled for 34 weeks, then rugae postpartum period as a result of urethral edema
reappear and lack of sensation.
4. Hypoestrogenic until ovulation and B. Marked diuresis begins within 12 hours of
menstruation resume delivery; increases volume of urinary output as
D. Ovulation/menstruation well as perspiration loss.
1. First cycle is usually anovulatory. C. Lactosuria may be seen in nursing mothers.
2. If not lactating, menses may resume in D. Many women will show slight proteinuria during
68 weeks. first 12 days of involution.
3. If lactating, menses less predictable; may
resume in 1224 weeks. Gastrointestinal System
E. Breasts
1. Nonlactating woman A. Mother usually hungry after delivery; good
a. Prolactin levels fall rapidly. appetite is expected.
b. May still secrete colostrum for 23 days. B. May still experience constipation from lack of
c. Engorgement of breast tissue resulting from muscle tone in abdomen and intestinal tract, and
temporary congestion of veins and perineal soreness.
lymphatic circulation occurs on third day, C. If hemorrhoids present, recommend sitz bath and
lasts 2436 hours, usually resolves foods to prevent constipation.
spontaneously.
d. Client should wear tight bra to compress Other
ducts and use cold applications to reduce
All other systems experience normal and rapid
swelling.
regression to prepregnancy status.
2. Lactating woman
a. High level of prolactin immediately after
delivery of placenta continued by frequent
contact with nursing baby.
POSTPARTAL PSYCHOSOCIAL
b. Initial secretion is colostrum, with CHANGES
increasing amounts of true breast milk
appearing between 4896 hours.
c. Milk let-down reflex caused by oxytocin
Adaptation to Parenthood
from posterior pituitary released by Motor Skills
sucking.
d. Successful lactation results from the New parents must learn new physical skills to care for
complex interaction of infant sucking infant (e.g., feeding, holding, burping, changing
reflexes and the maternal production and diapers, skin care).
let-down of milk.
Attachment Skills
Abdominal Wall/Skin A. Bonding: the development of a caring relationship
A. Muscles relaxed, separation of the rectus muscle with the baby. Behaviors include:
(diastasis recti) from 2 to 4 cm, usually resolves by 1. Claiming: identifying the ways in which the
6 weeks with exercise. baby looks or acts like members of the family.
B. Stretch marks gradually fade to silvery-white 2. Identification: establishing the babys unique
appearance. nature (assigning the baby his or her own
name).
Cardiovascular System 3. Attachment is facilitated by positive feedback
between baby and caregivers.
A. Normal blood loss in delivery of single infant is B. Sensual responses enhance adaptation to
less than 500 mL (up to 1000 mL normal blood parenthood.
loss for a cesarean delivery). 1. Touch: from fingertip, to open palm, to
B. Hematocrit usually returns to prepregnancy value enfolding; touch is an important
within 46 weeks. communication with the baby.
C. WBC count increases up to 20,000. 2. Eye-to-eye contact: a cultural activity that
D. Increased clotting factors remain for several weeks helps to form a trusting relationship.
leaving woman at risk for problems with thrombi. 3. Voice: parents await the babys first cry; babies
E. Varicosities regress. respond to the higher-pitched voice that

6
parents use in talking to the baby.

582 NCLEX-RN Review


53155_06_Ch06_p533-620.qxd 2/23/09 12:23 PM Page 583

4. Odor: babies quickly identify their own C. Lochia: color, amount, clots, odor
mothers breast milk by odor. D. Perineum
5. Entrainment: babies move in rhythm to 1. Healing of episiotomy if applicable
patterns of adult speech. 2. Hematoma formation
6. Biorhythm: babies respond to maternal 3. Development of hemorrhoids
heartbeats. E. Breasts: firmness, condition of nipples
F. Elimination patterns: voiding, flatus, bowels
G. Legs: pain, warmth, tenderness indicating
Maternal Adjustment thrombosis
Takes place in three phases. H. Perform foot dorsiflexion (Homans sign)

Dependent/Taking In Psychosocial Adjustment


A. 12 days after delivery. A. Overall emotional status of parents
B. Mothers needs predominate; mother passive and B. Parents knowledge of infant needs
dependent. C. Previous experience of parents
C. Mother needs to talk about labor and delivery D. Physical condition of infant
experiences to integrate them into the fabric of E. Ethnocultural background and financial status of
her life. parents
D. Mother may need help with everyday activities, F. Additional family support available to parents
as well as child care.
E. Food/sleep important.
ANALYSIS
Dependent/Independent/Taking Hold
Nursing diagnoses for the postpartum period may
A. By third day mother begins to reassert herself. include:
B. Identifies own needs, especially for teaching and A. Risk for constipation
help with her own and babys needs. B. Deficient knowledge
C. Some emotional lability, may cry for no reason. C. Self-care deficit
D. Mother requires reassurance that she can perform D. Impaired urinary elimination
tasks of motherhood. E. Interrupted family process
F. Risk for impaired parenting
Independent/Letting Go G. Ineffective role performance
H. Pain
A. Usually evident by fifth or sixth week.
B. Shows pattern of lifestyle that includes new baby
but still focuses on entire family as unit.
C. Reestablishment of father-mother bond seen in this PLANNING AND
period. IMPLEMENTATION
D. Mother may still feel tired and overwhelmed by
responsibility and conflicting demands on her
time and energies. Goals
A. Involution and return to prepregnancy state will be
accomplished without complication.
ASSESSMENT B. Parental role(s) will be successfully assumed.
C. New baby will be successfully integrated into
family structure.
Physical D. Successful infant feeding patterns (bottle- or
breastfeeding) will be established
A. Vital signs
1. Individual protocol until stable, then at least
once every 8 hours. Interventions
2. Temperature over 37.8C (100.4F) after first
24 hours, lasting more than 48 hours, Physical Care
indicative of infection.
B. Fundus A. Assess mother according to individual needs
1. Assessment done with empty bladder, one during first critical hours after delivery; implement
hand supporting base of uterus and one on nursing interventions as needed.
fundus B. Implement routine postpartum care after first
2. Assess for firmness and position hours.

MATERNITY AND FEMALE REPRODUCTIVE NURSING

6 583
53155_06_Ch06_p533-620.qxd 2/23/09 12:23 PM Page 584

1. Administer medications as ordered 2. Formula-feeding (bottle-feeding): utilizes


(e.g., oxytocins, analgesics). modified cows milk or soy formulas as basis
2. Teach perineal care. for provision of 20 kcal/oz.
3. Perform other care as needed (e.g., heat, cold a. Formulas are widely available in ready-to-
applications). feed, concentrated, and powdered forms.
4. Measure first voiding for sufficiency, observe b. They have supplemental vitamins; may
I&O for first 24 hours. also contain added iron.
5. Assist with breastfeeding as needed. c. Concentrated and powdered forms require
6. Instruct in breast care for bottle-feeding addition of prescribed amounts of water for
mothergood bra, limiting nipple appropriate reconstitution.
stimulation. d. Bottles and nipples should be carefully
C. Encourage measures to promote bowel function: cleaned daily by hand and rinsed with
roughage in diet, ambulation, sufficient fluids, boiling water or by electric dishwasher.
attention to urge to defecate. Reassure about e. Powdered or concentrated formulas
integrity of episiotomy. should be mixed with tap water, bottled
water or boiled water that has cooled.
Adjustment to Parenthood C. Nursing measures to promote successful infant
feeding
A. Provide time for parents to be alone with baby in 1. Assess previous experience and knowledge of
crucial early time after delivery. process of infant feeding.
B. Identify learning needs of parents. 2. Demonstrate how to hold baby for
C. Plan teaching to include both parents when breastfeeding and for feeding with formula.
possible. 3. Show how to burp baby.
D. Help parents realize that fatigue is normal at this 4. Allow time for practice with selected feeding
time. method.
E. Help parents identify and strengthen their own 5. Provide positive reinforcement for successful
coping mechanisms. actions.
F. Help parents identify resources available to 6. Give written instructions for at-home
them. reference.
G. Promote positive self-esteem on part of parents as 7. Help parents identify progress and pleasure in
they learn new role(s). feeding infant.
H. Provide anticipatory guidance for after discharge. 8. If bottle-feeding, demonstrate how to prepare
I. Provide information about contraception if formula using appropriate method.
requested. 9. If breastfeeding, assess breasts for tenderness
J. Prepare for discharge: reinforce physicians or discomfort, and examine nipples for cracks,
instructions about activities, rest, diet, drugs, bleeding, soreness, and erectility.
exercise, resumption of sexual intercourse, return 10. Assist mother with preparation: clean hands,
for postpartum examination. comfortable position, support as needed (extra
pillows). Demonstrate alternate infant
Infant Feeding positioning, e.g., football hold.
11. Bring infant to nurse as soon as possible after
A. General information delivery.
1. Infancy is a time of rapid growth and 12. Demonstrate positioning of baby at breast,
development; infant doubles birth weight in initiate rooting reflex, place entire nipple and
46 months, triples birth weight by 1 year. as much of areola as possible into babys
2. Newborns lose up to 8% of their birth weight, mouth, depress fleshy part of breast away from
then gain 47 ounces per week. babys nose if needed.
B. Choices in newborn nutrition 13. Allow baby to nurse in short frequent periods,
1. Breastfeeding: optimal infant nutrition, easily lengthening gradually in later days. Alternate
digested, contains antibodies to bolster the breast offered first.
immune system as well as all nutrients needed 14. Help mother release baby from nipple by
by the infant. breaking suction of baby on nipple. Check for
a. Helps mothers body return to prepregnant nipple trauma.
state faster. 15. Help mother move baby to alternate breast if
b. Provides some child-spacing but should needed.
not be relied on. 16. Remain with mother at each feeding until she
c. Prolactin, stimulated by the infants feels confident: see also Table 6-9 for
sucking, stimulates milk production. additional information on breastfeeding.
d. Oxytocin causes let-down or delivery of

6
milk to nursing baby.

584 NCLEX-RN Review


53155_06_Ch06_p533-620.qxd 2/23/09 12:23 PM Page 585

Table 6-9 Tips for successful breastfeeding Teaching: Postpartum/Discharge


A. Postpartum
Breast care Do not use soap on nipples or areola. 1. Normal events of postpartum period: physical,
Expose nipples to air to toughen them. psychosocial
Know how to pump breast milk if necessary 2. Information about feeding her infant
and how to store expressed breast milk. 3. Basic infant care, including cord care, bathing,
Nutrition Need for good maternal nutrition while circumcision care, dressing, handling, signs of
nursing. illness
Additional 300600 kcal/day 4. Safety needs of infant
23 liters fluid/day 5. Recommendations concerning activities
Know that certain foods may make the 6. Specific teaching about any medications
baby fussy and will need to be avoided B. Discharge
temporarily. 1. Reinforcement of all postpartum teaching,
Comfort Wear well-fitting bra; use absorbent pads allowing parent(s) time to ask questions.
without plastic coating if leaking occurs. 2. Referrals to professional assistance (MD, CNM,
Uterine cramping during nursing normal hospitals maternity unit, etc.).
at first. 3. Referrals to appropriate community assistance
Medications Avoid medications excreted in breast milk groups (Nursing Mothers, Mothers of Twins,
(mother should check with physician etc.) that meet individual needs.
before taking any drug while nursing). 4. Scheduled appointments for postpartal
Birth control pills should not be taken examination/newborns first well-baby
while nursing (decreases milk examination.
production). 5. Literature to reinforce all teaching. Excitement
Sources of help Inform mother of community support and anxiety of discharge may interfere with
groups available for nursing mothers. learning; literature will be available for quick
reference.

17. Assist bottle-feeding mother with suppression COMPLICATIONS OF THE


of lactation, accomplished primarily by POSTPARTUM PERIOD
mechanical inhibition.
a. Mechanical inhibition: usually takes
4872 hours Postpartum Hemorrhage
1) Snug breast binder for 23 days A. General information
postdelivery 1. Major cause of maternal death
2) Applications of cold (ice packs) and 2. Loss of more than 500-mL blood at the time of
analgesia to relieve discomfort delivery or immediately thereafter is
3) Avoidance of heat or other stimuli to considered postpartum hemorrhage.
breasts that increase milk production 3. Major causes include:
(including breast pumps) a. Uterine atony: loss of muscle tone in the
4) A well-fitting bra until lactation is uterus; may be the result of overdistension
suppressed (large baby, multiple pregnancy,
b. No approved medications to suppress polyhydramnios), overmassage, maternal
lactation. exhaustion, inhalation anesthesia
b. Laceration of the birth canal (cervix, vagina,
labia, perineum)
EVALUATION c. Retained placental fragments or
incomplete expulsion of placenta (usually
A. Involution successfully initiated and progressing the cause of late postpartum hemorrhage)
without complication d. Placenta accreta: penetration of the
B. Parents begin to assume new role behaviors and myometrium by the trophoblast, resulting
identities in abnormal adherence of the placenta to
C. Beginning integration of newborn into family the uterine wall. Rare; requires manual
structure; bonding established removal of the placenta
D. Infant feeding techniques mastered; infant growing B. Assessment findings
and developing appropriately 1. Boggy uterus, relaxed state indicating atony
E. Parents comfortable about infant care techniques 2. If uterus is firm with excess bleeding, may
and can demonstrate knowledge indicate lacerations.

MATERNITY AND FEMALE REPRODUCTIVE NURSING

6 585
53155_06_Ch06_p533-620.qxd 2/23/09 12:23 PM Page 586

3. Bright red blood, with clots 9. Observe client for signs of pulmonary
a. Large amounts with atony embolism.
b. Steady trickle with lacerations 10. Continue to bring baby to mother for feeding
4. Hemorrhage immediately after delivery with and interaction.
atony or lacerations
5. With retained placental fragments, delay of up
to 2 weeks
Subinvolution
6. With severe blood loss, signs and symptoms of A. General information
shock 1. Failure of the uterus to revert to prepregnant
7. Full bladder may displace uterus and prevent state through gradual reduction in size and
it from contracting firmly. placement
C. Nursing interventions 2. May be caused by infection, retained placental
1. Identify clients at risk for bleeding. fragments, or tumors in the uterus
2. Monitor fundus frequently if bleeding occurs; B. Assessment findings
when stable, every 15 minutes for 12 hours, 1. Uterus remains enlarged
then at appropriate intervals. 2. Fundus higher in the abdomen than
3. Monitor maternal vital signs for indications of anticipated
shock. 3. Lochia does not progress from rubra to serosa
4. Administer medications, IV fluids as ordered. to alba
5. Measure I&O. 4. If caused by infection, possible leukorrhea and
6. Remain with client for support and backache
explanation of procedures. C. Nursing interventions
7. Keep client warm. 1. Teach client to recognize unusual bleeding
8. Prepare for clients transfer to surgery if patterns.
needed for repair of laceration or removal of 2. Teach client usual pattern of uterine
placental fragment. involution.
9. Monitor for signs of DIC. 3. Instruct client to report abnormal bleeding to
physician.
4. Administer oxytoxic medications if ordered.
Thrombophlebitis
A. General information
1. Formation of a thrombus when a vein wall is
Postpartum Infection
inflamed A. General information
2. May be seen in the veins of the legs or pelvis 1. Any infection of the reproductive tract,
3. May result from injury, infection, or the associated with giving birth, usually occurring
normal increase in circulating clotting factors within 10 days of the birth
in the pregnant and newly delivered woman. 2. Predisposing factors include:
B. Assessment findings a. Prolonged rupture of membranes
1. Pain/discomfort in area of thrombus (legs, b. Cesarean birth
pelvis, abdomen) c. Trauma during birth process
2. If in the leg, pain, edema, redness over affected d. Maternal anemia
area e. Retained placental fragments
3. Elevated temperature and chills 3. Infection may be localized or systemic
4. Peripheral pulses may be decreased B. Assessment findings
5. Positive Homans sign 1. Temperature of 37.8C (100.4F) or more for 2
6. If in a deep vein, leg may be cool and pale consecutive days, excluding the first 24 hours
C. Nursing interventions 2. Abdominal, perineal, or pelvic pain
1. Maintain bed rest with leg elevated on pillow. 3. Foul-smelling vaginal discharge
Never raise knee gatch on bed. 4. Burning sensation with urination
2. Apply moist heat as ordered. 5. Chills, malaise
3. Administer analgesics as ordered. 6. Rapid pulse and respirations
4. Provide bed cradle to keep sheets off leg. 7. Elevated WBC count (may be normal for
5. Administer anticoagulant therapy as ordered postpartum initially), positive
(usually heparin), and observe client for signs culture/sensitivity report for causative
of bleeding. organism
6. Apply elastic support hose if ordered, with C. Nursing interventions
daily inspection of legs with hose removed. 1. Force fluids: client may need more than
7. Teach client not to massage legs. 3 liters/day.
8. Allow client to express fears and reactions to 2. Administer antibiotics and other medications

6
condition. as ordered.

586 NCLEX-RN Review


53155_06_Ch06_p533-620.qxd 2/23/09 12:23 PM Page 587

3. Treat symptoms as they arise (e.g., warm sitz


bath for infection in episiotomy). Sample Questions
4. Encourage high-calorie, high-protein diet to
promote tissue healing.
5. Position client in semi- to high-Fowlers to 66. On the second day postpartum, the nurse asks
promote drainage and prevent reflux higher the new mother to describe her vaginal bleeding.
into reproductive tract. What description does the nurse expect the
6. Support mother if isolated from baby. client to say?
1. Red and moderate.
Mastitis 2. Red with clots.
A. General information 3. Scant and brown.
1. Infection of the breast, usually unilateral 4. Thin and white.
2. Frequently caused by cracked nipples in the
nursing mother 67. A woman delivers a 6 lb 4 oz (2835 g) baby boy.
3. Causative organism usually hemolytic Which of the following statements would
S. aureus indicate to the nurse that the mother has begun
4. If untreated, may result in breast abscess. to integrate her new baby into the family
B. Assessment findings structure?
1. Redness, tenderness, or hardened area in the 1. All this baby does is cry. Hes not like my
breast other child.
2. Maternal chills, malaise
3. Elevated vital signs, especially temperature 2. I wish he had curly hair like my husband.
and pulse 3. My parents wanted a granddaughter.
C. Nursing interventions 4. When he yawns, he looks just like his
1. Teach/stress importance of hand washing to brother.
nursing mother, and wash own hands before
touching clients breast. 68. A diabetic woman plans to breastfeed her baby.
2. Administer antibiotics as ordered. Because the woman is hyperglycemic, what
3. Apply ice if ordered between feedings. explanation will the nurse give her?
4. Empty breast regularly: baby may continue to 1. The glucose content of her breast milk may
nurse or have mother use hospital-grade pump. be high.
2. The production of milk may be impaired.
Urinary Tract Infection 3. Her baby will receive insulin in the milk.
A. General information: may be caused postpartally 4. Her baby will not grow well.
by coliform bacteria, coupled with bladder trauma
during the delivery, or a break in technique during 69. A woman and her hospital roommate each
catheterization. delivered a child 2 days ago. One is
B. Assessment findings breastfeeding; one is using formula. Which of
1. Pain in the suprapubic area or at the the following instructions can the nurse give to
costovertebral angle both mothers?
2. Fever 1. Wear a good, well-supporting bra.
3. Burning, urgency, frequency on urination
2. Apply warm compresses to breast if too full.
4. Increased WBC count and hematuria
5. Urine culture positive for causative organism 3. Apply cold compresses to breast if too full.
C. Nursing interventions 4. Do not apply any soap to your nipples.
1. Check status of bladder frequently in
postpartum client. 70. What complaint would be a common occurrence
2. Use nursing measures to encourage client for a woman after delivery her third child?
to void. 1. Chest pain.
3. Force fluids: may need minimum of 3 liters/day. 2. Afterbirth cramps.
4. Catheterize client if ordered, using sterile
technique. 3. Burning on urination.
5. Administer medications as ordered. 4. Chills.
6. Monitor status of progress through continuing
lab tests. 71. A woman delivered a male infant yesterday.
7. Support mother with explanations of While caring for her on her first postpartum day,
interventions. which of the following behaviors would you

6
8. No need for baby to be separated from mother. expect?

MATERNITY AND FEMALE REPRODUCTIVE NURSING 587


53155_06_Ch06_p533-620.qxd 2/23/09 12:23 PM Page 588

1. Asking specific questions about home care of 77. A woman has just delivered her first baby who
the infant. will be breastfed. The nurse should include
2. Concern about when her bowels will move. which of the following instructions in the
3. Frequent crying spells for unexplained teaching plan?
reasons. 1. Try to schedule feedings at least every three
4. Acceptance of the nurses suggestions about to four hours.
personal care. 2. Wash nipples with soap and water before
each feeding.
72. A woman delivered this morning. Because this 3. Avoid nursing bras with plastic lining.
is her first child, which of the following goals is 4. Supplement with water between feedings
most appropriate? when necessary.
1. Early discharge for mother and baby.
2. Rapid adaptation to role of parent. 78. A womans prenatal antibody titer shows that
3. Effective education of both parents. she is not immune to rubella and will receive
the immunization after delivery. The nurse
4. Minimal need for expression of negative
would include which of the following
feelings.
instructions in the teaching plan?
73. A new mother is going to breastfeed her baby. 1. Pregnancy must be avoided for the next
What is the best indication that the let-down 3 months.
reflex has been achieved in a nursing mother? 2. Another immunization should be
1. Increased prolactin levels. administered in the next pregnancy.
2. Milk dripping from the opposite breast. 3. Breastfeeding should be postponed for 5 days
3. Progressive weight gain in the infant. after the injection.
4. Relief of breast engorgement. 4. An injection will be needed after each
succeeding pregnancy.
74. To prevent cracked nipples while she is
breastfeeding, what should the mother be 79. A woman had a normal vaginal delivery
taught? 12 hours ago and is to be discharged from the
birthing center. Which statement by the client
1. Apply a soothing cream prior to feeding.
demonstrates understanding about the teaching
2. Nurse at least 20 minutes on each breast each related to the episiotomy and perineal area?
feeding.
1. I know the stitches will be removed at my
3. Use plastic bra liners. postpartum clinic visit.
4. Wash the nipples with water only. 2. The ice pack should be removed for
10 minutes before replacing it.
75. What is the best indication that the breastfed
baby is digesting the breast milk properly? 3. The anesthetic spray, then the heat lamp,
will help a lot.
1. The baby does not experience colic.
4. The water for the Sitz bath should be warm,
2. The baby passes dark green, pasty stools.
about 102105 F.
3. The baby passes soft, golden-yellow stools.
4. The baby sleeps for several hours after each 80. A new mother is bottle-feeding her newborn.
feeding. Which statement by the client demonstrates
understanding how to safely manage formula?
76. Which of the following observations in the 1. Prepared formula should be used within
postpartum period would be of most concern to 48 hours.
the nurse?
2. I should mix the formula with water until it
1. After delivery, the mother touches the is a thin consistency.
newborn with her fingertips.
3. A dishwasher is not sufficient for proper
2. The new parents asked the nurse to cleaning.
recommend a good baby care book.
4. Prepared formula must be refrigerated until
3. A new father holds his son in the en face used.
position while visiting.
4. A new mother sits in bed while her newborn 81. A woman delivered her baby 12 hours ago.
lies awake in the crib. During the postpartum assessment, the uterus is

6 588 NCLEX-RN Review


53155_06_Ch06_p533-620.qxd 2/23/09 12:23 PM Page 589

found to be boggy with a heavy lochia flow. 1. Alteration in parenting related to material
What should be the initial action of the nurse? discomfort.
1. Notify the physician or nurse midwife. 2. High risk for injury related to spread of
2. Administer prn oxytocin. infection.
3. Encourage the woman to increase 3. Fluid volume excess related to urinary
ambulation. retention.
4. Massage the uterus until firm. 4. Knowledge deficit related to uterine
subinvolution.
82. A breastfeeding mother is visited by the home
health nurse 2 weeks after delivery. The woman
is febrile with flulike symptoms; on assessment
the nurse notes a warm, reddened, painful area Answers and Rationales
of the right breast. What is the best initial action
of the nurse?
66. 1. Lochia rubra is moderate red discharge and is
1. Contact the physician for an order for present for the first 23 days postpartum.
antibiotics.
2. Advise the mother to stop breastfeeding and 67. 4. Family identification of the newborn is an
pumping. important part of attachment. The first step in
3. Assess the mothers feeding technique and identification is defined in terms of likeness to
knowledge of breast care. family members.
4. Obtain a sample of breast milk for culture. 68. 1. Glucose can be transferred from the serum to
83. A woman had a vaginal delivery of her second the breast, and hyperglycemia may be reflected
child 2 days ago. She is breastfeeding the baby in the breast milk.
without difficulty. During a postpartum 69. 1. A well-fitting, supportive bra with wide straps
assessment, what normal finding would the can be recommended for both the nursing and
nurse expect? the non-nursing mother for the support of the
1. Complaints of afterpains. breasts and for comfort. The nursing mothers
2. Pinkish to brownish vaginal discharge. bra should have front flaps over each breast for
3. Voiding frequently, 5075 mL per void. easy access during nursing.
4. Fundus 1 cm above the umbilicus.
70. 2. Afterbirth cramps are most common in
84. A mother who had a vaginal delivery of her first nursing mothers and multiparas. This mother is
baby 6 weeks ago is seen for her postpartum both. The release of oxytocin from the posterior
visit. She is feeling well and is bottle-feeding her pituitary for the let-down reflex of lactation
infant successfully. During the physical causes the afterbirth cramping of the uterus.
assessment, what normal finding would the
71. 4. During the first few days after delivery, the
nurse expect?
mother is in a dependent phase, initiating little
1. Fundus palpated 6 cm below the umbilicus. activity by herself, and is usually content to be
2. Breasts tender, some milk expressed. directed in her activities by a health care
3. Striae pink but beginning to fade. provider.
4. Creamy, yellow vaginal discharge.
72. 3. Both parents will need education about the
85. A nurse collects the following data on a woman new babyhow to care for the baby, information
26 hours after a long labor and a vaginal about the baby, and how to be a parent.
delivery: temperature 38.3C (101F) blood
73. 2. The nursing infant will stimulate let-down,
pressure 110/70, pulse 90, some diaphoresis,
resulting in milk dripping from the other breast.
output 1000 mL per 8 hours, ankle edema,
lochia moderate rubra, fundus 1 cm above 74. 4. Nipples should be washed with water only
umbilicus and tender on palpation. The client (no soap) to prevent drying.
also asks that the infant be brought back to the
nursery. In the analysis of this data, the nurse 75. 3. Breastfed babies will pass 610 small, loose,
would select which of the following priority yellow stools per day.
nursing diagnoses?

MATERNITY AND FEMALE REPRODUCTIVE NURSING

6 589
53155_06_Ch06_p533-620.qxd 2/23/09 12:23 PM Page 590

76. 4. During the early postpartum period, evidence 81. 4. A soft, boggy uterus should be massaged until
of maladaptive mothering may include limited firm; clots may be expressed during massage and
handling or smiling at the infant; studies have this often tends to contract the uterus more
shown that a predictable group of reciprocal effectively.
interactions, between mother and baby, should
take place with each encounter to foster and 82. 1. These symptoms are signs of infectious
reinforce attachment. mastitis, usually caused by Staphylococcus
aureus; a 10-day course of antibiotics is
77. 3. Although plastic linings protect clothing from indicated.
leaking milk, the nipples may become sore and
prone to infection from trapped moisture; 83. 1. Afterpains occur more commonly in
disposable nursing pads can be used to protect multiparas than in primiparas and are caused by
clothing. intermittent uterine contractions. Because
oxytocin is released when the infant suckles,
78. 1. To prevent intrauterine infection, which can breastfeeding also increases the severity of the
result in miscarriage, stillbirth, and congenital afterpains.
rubella syndrome in the fetus, women who are
immunized should be advised not to become 84. 3. At 2 weeks postpartum, striae (stretch marks)
pregnant for 3 months. are pink and obvious; by 6 weeks they are
beginning to fade but may not achieve a silvery
79. 2. To attain the maximum effect of reducing appearance for several more weeks.
edema and providing numbness of the tissues,
the ice pack should remain in place 85. 2. The classic definition of puerperal morbidity
approximately 20 minutes and then be removed resulting from infection is a temperature of
for about 10 minutes before replacing it. 38.0C (100.4F) or higher on any of the first
10 days postpartum exclusive of the first
80. 4. Extra bottles of prepared formula are stored in 24 hours; additional signs are increased pulse
the refrigerator and should be warmed slightly rate, uterine tenderness, foul-smelling lochia,
before feeding. and subinvolution (uterus remains enlarged).

The Newborn

PHYSIOLOGIC STATUS OF THE G. Peripheral circulation established slowly; may


have mottled (blue/white) appearance for 24 hours
NEWBORN (acrocyanosis).
H. RBC count high immediately after birth, then falls
Circulatory after first week; possible physiologic anemia of
infancy.
A. Umbilical vein and ductus venosus constrict after I. Absence of normal flora in intestine of newborn
cord is clamped; these will become ligaments results in low levels of vitamin K; prophylactic
(23 months). dose of vitamin K given IM on first day of life.
B. Foramen ovale closes functionally as respirations
are established, but anatomic or permanent
closure may take several months. Respiratory
C. Ductus arteriosus constricts with establishment A. Thoracic squeeze in vaginal delivery helps drain
of respiratory function; later becomes ligament fluids from respiratory tract; remainder of fluid
(23 months). absorbed across alveolar membranes into
D. Heart rate ranges from 120160 beats/minute at capillaries.
birth, with changes noted during sleep and B. Adequate levels of surfactants (lecithin and
activity. sphingomyelin) ensure mature lung function;
E. Heart murmurs may be heard; usually have little prevent alveolar collapse and respiratory distress
clinical significance. syndrome.
F. Average blood pressure is 78/42 mm Hg.

6 590 NCLEX-RN Review


53155_06_Ch06_p533-620.qxd 2/23/09 12:23 PM Page 591

C. Normal respiratory rate is 3060 breaths/minute C. The liver of a mature infant can maintain the level
with short periods of apnea (<15 seconds); change of unconjugated bilirubin at less than 12 mg/dL.
noted during sleep or activity. Higher levels indicate a possible dysfunction and
D. Newborns are obligate nose breathers. the need for intervention.
E. Chest and abdomen rise simultaneously; no D. This physiologic jaundice is considered normal in
seesaw breathing. early newborns. It begins to appear after 24 hours,
usually between 4872 hours.
Renal
A. Urine present in bladder at birth, but newborn may
Temperature
not void for first 1224 hours; later pattern is 610 A. Heat production in newborn accomplished by:
voidings/day, indicative of sufficient fluid intake. 1. Metabolism of brown fat, a special structure
B. Urine is pale and straw colored; initial voidings in newborn that is source of heat.
may leave brick-red spots on diaper from passage 2. Increased metabolic rate and activity.
of uric acid crystals in urine. B. Newborn cannot shiver as an adult does to release
C. Infant unable to concentrate urine for first heat.
3 months of life. C. Newborns body temperature drops quickly after
birth; cold stress occurs easily.
D. Body stabilizes temperature in 810 hours if
Digestive unstressed.
A. Newborn has full cheeks due to well-developed E. Cold stress increases oxygen consumption; may
sucking pads. lead to metabolic acidosis and respiratory distress.
B. Little saliva is produced.
C. Hard palate should be intact; small raised white
areas on palate (Epsteins pearls) are normal.
Immunologic
D. Newborn cannot move food from lips to pharynx; A. Newborn has passive acquired immunity from
nipple needs to be inserted well into mouth. IgG from mother during pregnancy and passage
E. Circumoral pallor may appear while sucking. of additional antibodies in colostrum and breast
F. Newborn is capable of digesting simple milk.
carbohydrates and protein but has difficulty with B. Newborn develops own antibodies during first
fats in formulas. 3 months, but is at risk for infection during first
G. Immature cardiac (esophageal) sphincter may 6 weeks.
allow reflux of food when burped; may elevate crib C. Some immunizations are given before the infant is
after feeding. discharged. A complete list of immunizations is
H. Stomach capacity varies; approximately 1530 mL. listed in Table 5-1.
I. First stool is meconium (black, tarry residue from
lower intestine); usually passed within 1224 hours
after birth.
Neurologic/Sensory
J. Transitional stools are thin and brownish green in Six States of Consciousness
color; after 3 days, milk stools are usually passed
loose and golden yellow for the breastfed infant, A. Deep sleep
formed and pale yellow for the formula-fed infant. B. Light sleep: some body movements
Stools may vary in number from 1 every feeding to C. Drowsy: occasional startle; eyes glazed
12/day. D. Quiet alert: few movements, but eyes open and
K. Feeding patterns vary; newborn may nurse bright
vigorously immediately after birth, or may need as E. Active alert: active, occasionally fussy with much
long as several days to learn to suck effectively. facial movement
Provide support and encouragement to new F. Crying: much activity, eyes open or closed
mothers during this time as infant feeding is a very
emotional area for new mothers. Periods of Reactivity
A. First (birth through first 12 hours): newborn alert
Hepatic with good sucking reflex, irregular R/HR.
B. Second (48 hours after birth): may regurgitate
A. Liver responsible for changing hemoglobin (from
mucus, pass meconium, and suck well
breakdown of RBC) into unconjugated bilirubin,
C. Equilibrium usually achieved by 8 hours of age.
which is further changed into conjugated (water-
soluble) bilirubin that can be excreted.
B. Excess unconjugated bilirubin can permeate the Sleep Cycle
sclera and the skin, giving a jaundiced or yellow Newborn sleeps an average of 17 hours/day.

6
appearance to these tissues.

MATERNITY AND FEMALE REPRODUCTIVE NURSING 591


53155_06_Ch06_p533-620.qxd 2/23/09 12:23 PM Page 592

Hunger Cycle H. Ears


1. Should be even with canthi of eyes.
Varies, depending on mode of feeding. 2. Cartilage should be present and firm.
A. Breastfed infant may nurse every 23 hours. I. Eyes
B. Bottle-fed infant may be fed every 34 hours. 1. May be irritated by medication instillation,
some edema/discharge present.
Special Senses 2. Color is slate blue.
J. Nodule of tissue present in breasts.
A. Sight: eyes are sensitive to light; newborn will fix K. Female genitalia
and gaze at objects, especially those with black 1. Vernix seen between labia.
and white, regular patterns, but eye movements are 2. Blood-tinged mucoid vaginal discharge
uncoordinated. (pseudomenstruation) from high levels of
B. Hearing: can hear before birth (24 weeks); newborn circulating maternal hormones.
seems best attuned to human speech and its L. Male genitalia
cadences. 1. Testes descended or in inguinal canal
C. Taste: sense of taste established; prefers sweet- 2. Rugae cover scrotum
tasting fluids; derives satisfaction as well as 3. Meatus at tip of penis
nourishment from sucking. M. Legs
D. Smell: sense is developed at birth; newborn can 1. Bowed
identify own mothers breast milk by odor. 2. No click or displacement of head of femur
E. Touch: newborn is well prepared to receive tactile observed when hips flexed and abducted
messages; mother demonstrates touch progression N. Feet
in initial bonding activities. 1. Flat
2. Soles covered with creases in fully mature infant
O. Muscle tone
ASSESSMENT 1. Predominantly flexed
2. Occasional transient tremors of mouth and chin
Physical Examination 3. Newborn can turn head from side to side in
prone position
A. Weight 4. Needs head supported when held erect or lifted
1. Average between 2750 and 3629 g (68 lb) at P. Reflexes present at birth
term 1. Rooting, sucking, and swallowing.
2. Initial loss of 58% of body weight normal 2. Tonic neck, fencing attitude.
during first few days; should be regained in 3. Grasp: newborns fingers curl around anything
12 weeks placed in palm.
B. Length: average 45.755.9 cm (1822 in) 4. Moro reflex: symmetric and bilateral abduction
C. Head circumference: average 3335.5 cm (1314 in); and extension of arms and hands; thumb and
remeasure after several days if significant molding forefinger form a C; the embrace reflex.
or caput succedaneum present 5. Startle reflex: similar to Moro, but with hands
D. Chest circumference: average 1.9 cm (0.75 in) less clenched.
than head 6. Babinskis sign: flare of toes when foot stroked
E. Abdominal girth may be measured if indicated. from base of heel along lateral edge to great toe.
Consistent placement of tape is important for Q. Cry
comparison, identification of abnormalities. 1. Loud and vigorous.
Measurement is best done before feeding, as 2. Heard when infant is hungry, disturbed, or
abdomen relaxes after a feeding. uncomfortable.
F. Skin
1. Color in Caucasian infants usually pink; varies
with other ethnic backgrounds. Apgar Scoring
2. Pigmentation increases after birth. A. Used to evaluate the newborn in five specific
3. Skin may be dry. categories at 1 and 5 minutes after birth (see
4. Acrocyanosis of hands and feet normal for Table 6-7).
24 hours; may develop newborn rash B. The 1-minute score reflects transitional values.
(erythema toxicum neonatorum). C. The composite score at 5 minutes provides the best
5. Small amounts of lanugo and vernix caseosa direction for the planning of newborn care.
still seen. D. Composite score interpretations
G. Fontanels 1. 04: prognosis for newborn is grave.
1. Anterior: diamond shaped 2. 57: infant needs specialized, intensive care.
2. Posterior: triangular 3. 7 or above: infant should do well in normal

6
3. Should be flat and open newborn nursery.

592 NCLEX-RN Review


53155_06_Ch06_p533-620.qxd 2/23/09 12:23 PM Page 593

Gestational Age Assessment PLANNING AND


After birth, direct examination of the infant leads to an IMPLEMENTATION
accurate assessment of maturity. This is important, as
complications may vary with maturity level: pre- and
postmature infants, in general, have greater difficulty
Goals
adapting to extrauterine life. A. Newborn will adapt to extrauterine life.
A. Physical examination 1. Body temperature will be maintained.
1. Skin: thickens with gestational age; may be 2. Normal breathing and adequate oxygenation
dry/peeling if postmature. will be established.
2. Lanugo: disappears as pregnancy 3. Cardiovascular function will be stable.
progresses. 4. Nutrition and promotion of growth will be
3. Sole (plantar) creases: increase with established.
gestational age (both depth and number). B. Positive parent-infant relationship will be
4. Areola of breast: at term, 510 mm in established.
diameter. C. Potential dysfunctions will be identified early.
5. Ear: cartilage stiffens, recoil increases, and D. Needed interventions will be implemented early.
curvature of pinna increases with advancing
gestational age.
6. Genitalia: in the male, check for descended
Interventions
testicles and scrotal rugae; in the female, look Delivery Room
for the labia majora to cover the labia minora
and clitoris. A. Perform Apgar scoring at 1 and 5 minutes after
B. Neuromuscular assessment (best done after birth.
24 hours) B. Perform rapid, overall physical and neurologic
1. Resting posture: relaxed posture (extension) exam.
seen in the premature; flexion increases with 1. Identify obvious congenital anomalies.
maturity. 2. Count vessels in cord.
2. Square window angle: flex hand onto 3. Identify injuries from birth trauma.
underside of forearm, identify angle at which C. Prevent heat loss.
you feel resistance. Angle decreases with 1. Dry infant immediately after birth.
increasing gestational age. 2. Wrap newborn warmly, cover head, or place in
3. Arm recoil: flex infants arms, extend for specially warmed area.
5 seconds, then release. Note angle formed 3. Place newborn on warm surfaces (mothers
as arms recoil. Decreases with increasing body) or cover cool surfaces (e.g., scale).
gestational age. 4. Minimize placement of newborn near cooler
4. Popliteal angle: place infant on back, extend areas (windows, outside walls).
one leg, and measure angle at point of D. Maintain established respirations and heartbeat.
resistance. Angle becomes more acute as E. Identify mother and infant with matching bands.
gestation progresses. F. Perform cord clamping if physician has not
5. Scarf sign: draw one arm across chest until done so.
resistance is felt; note relation of elbow to G. Allow parents to hold infant, or place in warmed
midline of chest. Resistance increases with unit.
advancing gestational age. H. Suction gently prn with bulb syringe.
6. Heel to ear: attempt to raise foot to ear, I. Administer oxygen prn.
noting point at which foot slips from your J. Promote bonding through early nursing if mother
grasp. Resistance increases with gestational so desires, or by having parents hold newborn.
age.
In performing gestational age assessments, the use of a Nursery
specific form usually facilitates the ease and accuracy
of the process. A. Continue actions to prevent heat loss (temperature
done rectally on admission then axillary; tympanic
not accurate on infants).
B. When temperature stabilizes, perform complete
ANALYSIS physical and neurologic exam.
Nursing diagnoses for the normal newborn are related C. Administer medications as ordered.
to the potential for dysfunction in transition period 1. To prevent ophthalmia neonatorum,
and first few days of life. administration of 0.5% erythromycin or
1% tetracycline into conjunctival sac(s).

MATERNITY AND FEMALE REPRODUCTIVE NURSING

6 593
53155_06_Ch06_p533-620.qxd 2/23/09 12:23 PM Page 594

2. Vitamin K: prophylactic dose to prevent EVALUATION


hemorrhage.
3. Hepatitis B vaccine in first 12 hours. A. Newborn progress continually observed, normal
D. Measure and weigh newborn. vital signs for newborn maintained
E. After temperature has stabilized, bathe and B. No dysfunctional patterns discerned
dress newborn, place in open crib. C. No congenital anomalies identified
F. Institute daily care routine. D. Parents comfortable with infant, have initiated
1. Take weight. bonding
2. Monitor temperature, apical pulse, E. Parents comfortable with newborn care
respirations at least every shift. F. All necessary tests carried out at correct time
3. Suction prn. G. Evidence for continued growth and development
4. Bathe daily if ordered. at home is positive
5. Give diaper area care after each change.
6. Continue assessment for anomalies.
7. Allow umbilical cord to air dry by folding VARIATIONS FROM NORMAL
diaper below cord. Some institutions still use
alcohol wipes. NEWBORN ASSESSMENT
8. Institute feeding schedule as ordered.
9. Note voidings and stools on daily basis. FINDINGS
G. Assess for physiologic jaundice (see also
Some variations from the normal assessment findings
Hyperbilirubinemia).
in the newborn are not indicative of any disorders;
1. First manifests in the head area (test by
others, however, provide information about likely
depressing skin over bridge of nose) then
gestational age or the possibility of the existence of a
progresses to chest (depress skin over
more serious disorder.
sternum).
2. Early feedings promote excretion of bilirubin
in stool, diminishing incidence of jaundice. Weight
3. Prevention of cold stress in newborn
A. Under 2500 g (512 lb): small for gestational age
diminishes incidence of jaundice.
(SGA)
4. Loose, greenish stools and green-tinged
B. Over 4100 g (9 lb): large for gestational age (LGA)
urine are normal for these infants; advise
mother.
5. These infants need extra fluids to prevent Length
dehydration and replace fluids being
A. Under 45.7 cm (18 in): SGA
excreted.
B. Over 55.9 cm (22 in): LGA
6. Advise parents that breast-fed infants may
have increased jaundice.
a. May not feed frequently enough in first Head Circumference
2 days and become dehydrated
A. Under 31.7 cm (1212 in): microcephaly/SGA
b. Breastmilk jaundice not a confirmed
B. Over 36.8 cm (1412 in): hydrocephaly/LGA
problem, usually an underfed baby.
H. Provide phototherapy if ordered, not usually
required in physiologic jaundice unless levels rise Blood Pressure
rapidly or reach the high teens.
A. Variation with activity: normal
I. Monitor for pathologic jaundice.
B. Major difference between upper and lower
1. Appears at birth or in the first 24 hours.
extremities: possible aortic coarctation
2. Bilirubin levels over 12 mg/dL (see
Hyperbilirubinemia).
J. Male infants may need circumcision care. Pulse
1. Observe for bleeding.
A. Persistently under 120: possible heart block
2. Note first voiding after circumcision.
B. Persistently over 170: possible respiratory distress
3. Clean area appropriately.
syndrome
4. Vaseline to penis to prevent sticking to
diaper.
K. Perform screening tests before discharge (PKU, Temperature
hypothyroidism, galactasemia, etc.).
A. Elevated: possible dehydration or infection
L. Provide teaching and demonstrations as indicated
B. Temperature falls with low environmental
for parents (e.g., feeding, burping, holding,
temperature, late in cold stress, sepsis, cardiac
diapering, bathing, positioning, safety).
disease.

6 594 NCLEX-RN Review


53155_06_Ch06_p533-620.qxd 2/23/09 12:23 PM Page 595

Respirations Neck
A. Under 25/minute: possibly result of maternal Webbing; masses in muscle
analgesia
B. Over 60/minute: possible respiratory distress
Chest
Breast enlargement and milky secretion from breasts
Skin (witchs milk) is result of maternal hormones; self-
A. Milia (blocked sebaceous glands, usually on nose limiting.
and chin) are essentially normal.
B. Stork bites
1. Capillary hemangiomas above eyebrows and
Cord
at base of neck under hairline are essentially Fewer than three vessels may indicate congenital
normal. anomalies.
2. Raised capillary hemangiomas on areas other
than face or neck are not normal findings.
C. Newborn rash (erythema toxicum neonatorum) is
Female Genitalia
normal. Pseudomenstruation is normal.
D. Mongolian spots (darkened areas of pigmentation
over sacral area and buttocks) are normal and fade
in early childhood. (Seen in Asian and African-
Male Genitalia
American babies.) Misplaced urinary meatus
E. Fingernail scratches are normal. A. Epispadias: on upper surface of penis
F. Excess lanugo: possible prematurity. B. Hypospadias: on under surface of penis
G. Vernix
1. Decreases after 38 weeks, full-term usually has
only in creases
Upper Extremities
2. Excess: prematurity A. Extra fingers
B. Webbed fingers
C. Asymmetric movement: possible trauma or
Head fracture
A. Fontanels
1. Depressed: dehydration Lower Extremities
2. Bulging: increased intracranial pressure
B. Hair: coarse or brittle, possible endocrine disorder A. Extra toes
C. Scalp: edema present at birth (caput B. Webbed toes
succedaneum) from pressure of cervix against C. Congenital hip dysplasia
presenting part; crosses suture lines; disappears in D. Few creases on soles of feet: prematurity
34 days without intervention.
D. Skull: collection of blood between a skull bone Spine
and its periosteum (cephalhematoma) from
pressure during delivery; does not cross suture Tuft of hair: possible occult spina bifida; assess
line; appears 1224 hours after delivery; regresses pilonidal area for fistula.
in 36 weeks.
E. Eyes Anus
1. Edema from medications not uncommon
2. Strabismus (occasional crossing of eyes) is normal Lack of meconium after 24 hours may indicate
3. Wide space between eyes is seen in fetal obstruction, disease.
alcohol syndrome
F. Ears
1. Lack of cartilage: possible prematurity
2. Low placement: possible kidney disorder or Sample Questions
Downs syndrome
G. Nose: copious drainage associated with syphilis
H. Mouth 86. Which nursing action should be included in the
1. Thrush: appears as white patches in mouth; care of the infant with a caput succedaneum?
candida infection passed from mother during 1. Aspiration of the trapped blood under the
passage through birth canal. periosteum.
2. Tongue movement and excess salivation: 2. Explanation to the parents about the

6
possible esophageal atresia cause/prognosis.

MATERNITY AND FEMALE REPRODUCTIVE NURSING 595


53155_06_Ch06_p533-620.qxd 2/23/09 12:23 PM Page 596

3. Gentle rubbing in a circular motion to spots on the nose, and a red area at the nape of the
decrease size. neck. What would be the nurses next action?
4. Application of cold to reduce size. 1. Document findings as within normal range.
2. Isolate infant pending diagnosis.
87. A baby girl was born at 0915. At 0920 her heart
3. Request a dermatology consultation.
rate was 132 beats/minute, she was crying
vigorously, moving all extremities, and only her 4. Document as indicators of malnutrition.
hands and feet were still slightly blue. What will
93. While performing the discharge assessment on a
the nurse record for the Apgar score?
2-day-old newborn, the nurse finds that after
1. 7. blanching the skin on the forehead, the color
2. 8. turns yellow. What does this indicate?
3. 9. 1. A normal biologic response.
4. 10. 2. An infectious liver condition.
3. An Rh incompatibility problem.
88. Which of the following findings in a newborn
baby girl is normal? 4. Jaundice related to breastfeeding.
1. Passage of meconium within the first 24 hours. 94. A newborn is 2 days old and is being breastfed.
2. Respiratory rate of 70/minute at rest. The nurse finds that yesterday her stool was
3. Yellow skin tones at 12 hours of age. thick and tarry, today its thinner and greenish
4. Bleeding from umbilicus. brown; she voided twice since birth with some
pink stains noted on the diaper. What do these
89. The nursery nurse carries a newborn baby into findings indicate to the nurse?
his mothers room. The mother states, I think 1. Marked dehydration.
my babys afraid of me. Every time I make a loud 2. Inadequate initial nutrition.
noise, he jumps. What should be the nurses
3. Normal newborn elimination.
initial action?
4. A need for medical consultation.
1. Encourage her not to be so nervous with her
baby. 95. The nurse notes the following behaviors in a
2. Reassure her that this is a normal reflexive 6-hour-old, full-term newborn: occasional
reaction for her baby. tremors of extremities, straightens arms and
3. Take the baby back to the nursery for a hands outward and flexes knees when disturbed,
neurologic evaluation. toes fan out when heel is stroked, and tries to
4. Wrap the baby more tightly in warm blankets. walk when held upright. What do these findings
indicate to the nurse?
90. A new mother asks how much weight her 1. Signs of drug withdrawal.
newborn will lose. What will be the nurses 2. Abnormal uncoordinated movements.
reply?
3. Asymmetric muscle tone.
1. None.
4. Expected neurologic development.
2. 23%.
3. 58%. 96. While assessing a newborn, the nurse notes that
4. 1015%. the areola is flat with less than 0.5 cm of breast
tissue. What does this finding indicate?
91. Which of the following findings in a 3-hour-old 1. That infant is male.
full-term newborn would cause the nurse 2. Maternal hormonal depletion.
concern?
3. Intrauterine growth retardation.
1. Two soft spots between the cranial bones.
4. Preterm gestational age.
2. Asymmetry of the head with overriding
bones. 97. The nurses initial care plan for a full-term
3. Head circumference 32 cm, chest 34 cm. newborn includes the nursing diagnosis risk of
4. A sharply outlined, spongy area of edema. fluid volume depletion related to absence of
intestinal flora. What would be a related
92. The nurse collects the following data while nursing intervention?
assessing the skin of a 6-hour-old newborn: color 1. Administer glucose water or put to breast.
pink with bluish hands and feet, some pale yellow 2. Assess first void and passing of meconium.

6
papules with red base over trunk, small white

596 NCLEX-RN Review


53155_06_Ch06_p533-620.qxd 2/23/09 12:23 PM Page 597

3. Administer vitamin K injection. 3. Says she will apply Bacitracin ointment three
4. Send cord blood to lab for Coombs test. times per day.
4. Cleans the cord and surrounding skin with an
98. In the time immediately following birth, why alcohol pad.
might the nurse delay instillation of eye
medication to the newborn? 104. What statement by a new mother demonstrates
1. Check prenatal record to determine whether more instruction on care of the circumcised
prophylactic treatment is needed. infant is needed?
2. Ensure that initial eye saline irrigation is 1. I know to gently retract the foreskin after the
completed. area is healed.
3. Enable mother to breast feed the infant in the 2. At each diaper change I will squeeze water
first hour of life. over the penis and pat dry.
4. Facilitate eye contact and bonding between 3. I know not to disturb the yellow exudate
parents and newborn. that will form.
4. For the first day or so Ill apply a little A&D
99. The nurses should include which of the following ointment.
instructions in the care plan for a new mother
who is breastfeeding her full-term newborn? 105. Which statement by a new mother demonstrates
1. Put to breast when infant shows readiness to proper understanding of bottle feeding her
feed. infant?
2. Breastfeed infant every 3 to 4 hours until 1. I know not to prop the bottle until my baby
discharge. is older.
3. Offer water feedings between breastfeedings. 2. With these little bottles, he should be able to
4. Feed infant when he shows hunger by crying. finish them.
3. When I hold the bottle upside down, drops
100. In the delivery area, after ensuring that the of milk should fall.
newborn has established respirations, what is 4. I should burp the baby at the end of the
the next priority of the nurse? feeding.
1. Perform the Apgar score.
2. Place plastic clamp on cord.
3. Dry infant and provide warmth. Answers and Rationales
4. Ensure correct identification.

101. During the bath demonstration, a woman asks the 86. 2. Caput succedaneum (scalp edema) will regress
nurse if it is OK to use baby powder because warm in a few days without interventions and without
weather is coming. How should the nurse respond? residual damage.
1. Just dust in on the diaper area only.
2. Its best not to use powder on infants. 87. 3. Acrocyanosis, where hands and feet are still
slightly blue for the first 24 hours, is a normal
3. First use baby oil, then the powder.
variant in the newborn, but it rates a 1 on the Apgar
4. If the baby is just in a diaper hell be cool. scale. All the other descriptors are rated 2 on the
Apgar scale, giving this newborn a total of 9.
102. Which of the following muscles would the nurse
choose as the preferred site for a newborns 88. 1. Meconium is usually passed during the first
vitamin K injection? 24 hours of life.
1. Gluteus medius.
2. Mid-deltoid. 89. 2. The startle reflex, normally present in neonates,
is characterized by symmetric extension and
3. Vastus lateralis.
abduction of the arms with fingers extended. The
4. Rectus femoris. parent perceives this response as jumping.
103. What action by the mother of the newborn will 90. 3. Within 34 days of birth, a weight loss of
assure the nurse that she understands proper 58% is normal.
cord care for the newborn?
1. Views a videotape on newborn hygiene care. 91. 3. The circumference of the newborns head
2. Reads a booklet on care of the newborns cord should be approximately 2 cm greater than the

6
stump.

MATERNITY AND FEMALE REPRODUCTIVE NURSING 597


53155_06_Ch06_p533-620.qxd 2/23/09 12:23 PM Page 598

circumference of the chest at birth and will 98. 4. The initial parental-newborn attachment
remain in this proportion for the next few period can be enhanced if the care providers
months. Any differences in head size may keep routine investigations to a minimum, delay
indicate microcephaly (abnormal smallness of instillation of ophthalmic antibiotic for 1 hour,
head) or hydrocephalus (increased cerebrospinal keep the room dim, and provide privacy; eye
fluid within the ventricles of the brain). prophylaxis medication can cause chemical
conjunctivitis, which may interfere with the
92. 1. These findings of acrocyanosis (bluish babys ability to focus on the parents faces.
discoloration of the hands and feet), erythema
toxicum (newborn rash), milia, and a nevus 99. 1. It is important for the new mother to learn and
flammeus (port wine stain) are all within the respond to her infants early feeding cues. Early
normal range for a full-term newborn. cues that indicate a newborn is interested in
feeding include hand-to-hand or hand-passing-
93. 1. Physiologic jaundice occurs after the first mouth motion, whimpering, sucking, and
24 hours of life and is caused by accelerated rooting.
destruction of fetal RBCs, impaired conjugation
of bilirubin, and increased bilirubin 100. 3. After birth, the first priority is to maintain
reabsorption from the intestinal tract; there is respirations, the second priority is to provide
no pathologic basis. and maintain warmth; the newborns
temperature may fall 23C (3.65F) after birth
94. 3. Normal term newborns pass meconium within due mainly to evaporative losses; this triggers
824 hours of life; meconium is formed in utero cold-induced metabolic responses and heat
and is thick, tarry, black (or dark green) in production.
appearance. Transitional stool is a thinner brown
to green. Normal voiding is 2 to 6 times daily; 101. 2. Powders and oils are not recommended for
there may be innocuous pink stains (brick dust the neonates skin; oils may clog the pores, and
spots) on the diaper from urates. the small particles of powders may be inhaled
by the neonate.
95. 4. Tremors are common in the full-term
newborn; when a newborn is startled she will 102. 3. The middle third of the vastus lateralis
exhibit the Moro reflex, that is, she will muscle in the thigh is the preferred site for an
straighten arms and hands outward while the intramuscular injection in the newborn.
knees flex; in a newborn the Babinski reflex is
displayed by a fanning and extension of the toes 103. 4. Before discharge, parents should demonstrate
(in adults the toes flex); and when held upright proper cleaning of the cord stump by wiping it
with feet lightly touching a surface, the newborn with an alcohol pad; they should know to do
will put one foot in front of the other and this 2 to 3 times a day until the cord falls off in
walk. 714 days.

96. 4. At term gestation, the breast bud tissue will 104. 1. A circumcision is the surgical removal of the
measure between 0.5 and 1 cm (510 mm). prepuce or foreskin from the tip of the penis;
any foreskin that remains should not be
97. 3. The newborn is at a high risk for hemorrhage retracted.
due to an absence of intestinal flora (bacteria).
Vitamin K, needed for the formation of 105. 3. The nipple should have a hole big enough to
prothrombin and proconvertin for blood allow milk to flow in drops when the bottle is
coagulation, is usually synthesized by these inverted; too large an opening may cause
bacteria in the colon; however, they are absent in regurgitation, too small an opening can exhaust
the newborns sterile gut. This problem is and upset the infant.
prevented by the administration of vitamin K
following birth.

6 598 NCLEX-RN Review


53155_06_Ch06_p533-620.qxd 2/23/09 12:23 PM Page 599

The High-Risk Infant

OVERVIEW D. Needs of parents for closeness with infant will be


met; attachment/bonding will be promoted.
High-risk infants are those whose incidence of illness
or death is increased because of prematurity, Interventions
dysmaturity, postmaturity, physical problems, or birth
complications. They are frequently the result of a A. Constantly monitor infant for subtle changes in
high-risk pregnancy. condition and intervene promptly when necessary.
B. Conserve infants energy and decrease physiologic
stress.
ASSESSMENT C. Provide appropriate stimulation for infant growth
and development.
A. History of high-risk pregnancy or other factor D. Allow parents to express their reactions and
possibly affecting fetal development feelings and assist them in attachment behaviors.
B. Apgar scores in the delivery room E. Teach parents care of infant in preparation for
C. Head-to-toe assessment of the infant discharge.
(see Assessment of the Newborn)
D. Determination of gestational age
EVALUATION
ANALYSIS A. Infants physical condition stabilized and
improved on a steady basis
A. Alteration in respiratory function B. Infants growth and development steady and
B. Imbalanced nutrition: less than body requirements appropriate
C. Risk for impaired skin integrity C. Parents demonstrated acceptance of and comfort
D. Ineffective tissue perfusion with infants condition
E. Risk for injury D. Parents demonstrated comfort and confidence with
F. Impaired gas exchange infant care at discharge
G. Ineffective thermoregulation
H. For parents of high-risk infants
1. Ineffective coping HIGH-RISK DISORDERS
2. Deficient knowledge
3. Anticipatory grieving The Premature Infant
4. Powerlessness
5. Social isolation A. General information
1. Any infant born before the end of the 37th
week of pregnancy
PLANNING AND 2. Weight usually less than 2500 grams (512 lb)
3. Causes include:
IMPLEMENTATION a. Maternal factors: age, smoking, poor
nutrition, placental problems,
Goals preeclampsia/eclampsia
b. Fetal factors: multiple pregnancy,
A. Needs of infant for physical care will be met. infection, intrauterine growth retardation
1. Oxygen: respiratory functioning will be (IUGR)
maintained. c. Other: socioeconomic status,
2. Humidity and warmth: temperature will be environmental exposure to harmful
regulated and cold stress prevented. substance
3. Adequate nutrition will be provided. 4. Severity of problems related to level of
4. Tender handling: newborn will receive proper maturity: the earlier the infant is born, the
skin care and positioning. greater the chance of complications.
B. Infection or other complications will be prevented. 5. Major complicating conditions
C. Normal growth and development will be a. Respiratory distress syndrome

6
promoted. b. Thermoregulatory problems

MATERNITY AND FEMALE REPRODUCTIVE NURSING 599


53155_06_Ch06_p533-620.qxd 2/23/09 12:23 PM Page 600

c. Conservation of energy 16. Handle carefully; organize care to minimize


d. Infection disturbances.
e. Hemorrhage 17. Provide skin care with special attention to
B. Assessment findings cleanliness and careful positioning to prevent
1. Respiratory system breakdown.
a. Insufficient surfactant 18. Monitor heart rate and pattern at least every
b. Apneic episodes 12 hours; listen apically for 1 full minute.
c. Retractions, nasal flaring, grunting, seesaw 19. Monitor potential bleeding sites (umbilicus,
pattern of breathing, cyanosis injection sites, skin); these infants have
d. Increased respiratory rate lowered clotting factors.
2. Thermoregulation: body temperature 20. Monitor overall growth and development of
fluctuates easily (premature newborn has less infant; check weight, length, head
subcutaneous fat and muscle mass) circumference.
3. Nutritional status 21. Provide tactile stimulation when caring for or
a. Poor sucking and swallowing reflexes feeding infant.
b. Poor gag and cough reflexes 22. Provide complete explanations for parents.
4. Skin: lack of subcutaneous fat; reddened; 23. Encourage parental involvement in infants care.
translucent 24. Provide support for parents; refer to self-help
5. Drainage from umbilicus/eyes group or other parents if necessary.
6. Cardiovascular 25. Promote parental confidence with infant care
a. Petechiae caused by fragile capillaries and before discharge.
prolonged prothrombin time
b. Increased bleeding at injection sites
7. Neuromuscular
The Dysmature Infant (SGA)
a. Poor muscle tone A. General information
b. Weak reflexes 1. Birth weight in the lowest 10th percentile at
c. Weak, feeble cry term
C. Nursing interventions 2. Causes: discounting heredity, possibly
1. Maintain respirations at less than 60/minute, intrauterine growth retardation, infections,
check every 12 hours. malformations
2. Administer oxygen as ordered; check B. Assessment findings
concentration every 2 hours to avoid 1. Skin: loose and dry, little fat, little muscle
retrolental fibroplasia while providing mass
adequate oxygenation. 2. Small body makes skull look larger than
3. Auscultate breath sounds to assess lung normal
expansion. 3. Sunken abdomen
4. Encourage breathing with gentle rubbing of 4. Thin, dry umbilical cord
back and feet. 5. Little scalp hair
5. Suction as needed. 6. Wide skull sutures
6. Reposition every 12 hours for maximum lung 7. Respiratory distress; may have had hypoxic
expansion and prevention of exhaustion. episodes in utero
7. Monitor blood gases and electrolytes. 8. Hypoglycemia
8. Maintain thermoneutral body temperature; 9. Tremors
prevent cold stress. 10. Weak cry
9. Maintain appropriate humidity level. 11. Lethargic
10. Monitor for signs of infection; these infants 12. Cool to touch
have little antibody production and decreased C. Nursing interventions
resistance. 1. Care of SGA infant is similar in many
11. Feed according to abilities. instances to care of preterm infant.
12. Monitor sucking reflex; if poor, gavage feeding 2. Tailor high-level nursing care to meet specific
indicated. Most preterm infants require at least needs of infant with regard to functioning of
some gavage feeding as it diminishes the effort all body systems, psychologic growth and
required for sucking while improving the development, parental support and teaching,
caloric intake. and prevention of complications.
13. Use preemie nipple if bottle-feeding.
14. Monitor I&O, weight gain or loss; these infants
are easily dehydrated, with poor electrolyte
The Postmature Infant
balance. A. General information
15. Monitor for hypoglycemia and 1. Born after the completion of 42 weeks of

6
hyperbilirubinemia. pregnancy

600 NCLEX-RN Review


53155_06_Ch06_p533-620.qxd 2/23/09 12:23 PM Page 601

2. Problems caused by progressively less efficient C. Nursing interventions


actions of placenta 1. Identify conditions predisposing to
B. Assessment findings hyperbilirubinemia, especially positive
1. Skin coombs test (test on cord blood for presence of
a. Vernix and lanugo completely disappeared maternal antibodies).
b. Dry, cracked, parchmentlike appearance of 2. Prevent progression or complications of
skin jaundice.
c. Color: yellow to green from meconium 3. Assess jaundice levels (visually, lab tests) as
staining needed.
2. Depleted subcutaneous fat; old looking 4. Prevent conditions that contribute to
3. Hard nails extending beyond fingertips development of hyperbilirubinemia (i.e., cold
4. Signs of birth injury or poor tolerance of birth stress, hypoxia, acidosis, hypoglycemia,
process dehydration, infection).
C. Nursing interventions 5. Provide adequate hydration.
1. Nursing care of the postmature infant has 6. Implement phototherapy if ordered; use of
many characteristics in common with the care blue lights overhead, blanket-device wrapped
given to the premature infant. around infant (Wallaby), or bili-bed.
2. Design high-level nursing care to identify the a. Overhead unit
infants specific physical and psychologic 1) Unclothe infant for maximum skin
needs; monitor functioning of all body exposure; minimal diaper
systems, growth and development, parental 2) Cover eyes to prevent retinal damage
support and teaching, and prevention of 3) Carefully monitor temperature
complications. 4) Remove baby from warmer and
uncover eyes for feedings
5) Ensure feedings every 3 hours
SPECIAL CONDITIONS b. Wallaby blanket
1) Baby at bedsideexplain care of unit
IN THE NEONATE to mother
2) Keep unit on for feedings, eyes remain
Hyperbilirubinemia uncovered
3) Other care as above
A. General information 7. Explain all tests and procedures to parents.
1. Elevated serum level of bilirubin in the 8. Support parents with information on
newborn results in jaundice or yellow color of procedures.
body tissues.
2. In physiologic jaundice, average increase from
2 mg/dL in cord blood to 6 mg/dL by 72 hours; Hemolytic Disease of the Newborn
not exceeding 12 mg/dL. (Erythroblastosis Fetalis)
3. Level at which a newborn will sustain damage
to body cells (especially brain cells) from high A. General information
concentrations of bilirubin is termed pathologic. 1. Characterized by RBC destruction in the
4. May result from immaturity of liver, Rh or newborn, with resultant anemia and
ABO incompatibility, infection, birth trauma hyperbilirubinemia
with subsequent bleeding (cephalhematoma), 2. Possibly caused by Rh or ABO incompatibility
maternal diabetes, hypothermia, medications. between the mother and the fetus
5. Major complication is kernicterus (brain damage (antigen/antibody reaction)
caused by high levels of unconjugated bilirubin). 3. Mechanisms of Rh incompatibility
B. Assessment findings a. Sensitization of Rh-negative woman by
1. Pathologic jaundice usually appears early, up transfusion of Rh-positive blood
to 24 hours after birth; represents a process b. Sensitization of Rh-negative woman by
ongoing before birth presence of Rh-positive RBCs from her
2. Usual pattern of progression is from head to feet. fetus conceived with Rh-positive man
Blanch skin over bony area or look at conjunctiva c. Approximately 65% of infants conceived
and buccal membranes in dark-skinned infants by this combination of parents will be
3. Pallor Rh positive.
4. Dark, concentrated urine (often dehydrated) d. Mother is sensitized by passage of fetal
5. Behavior changes (irritability, lethargy) Rh-positive RBCs through placenta, either
6. Polycythemia during pregnancy (break/leak in
7. Increased serum bilirubin (direct, indirect, and membrane) or at the time of separation of
the placenta after delivery.

6
total)

MATERNITY AND FEMALE REPRODUCTIVE NURSING 601


53155_06_Ch06_p533-620.qxd 2/23/09 12:23 PM Page 602

e. This stimulates the mothers immune fewer anti-A, anti-B, or anti-AB antibodies are
response system to produce anti- produced.
Rh-positive antibodies that attack fetal 4. Clinical manifestations of ABO
RBCs and cause hemolysis. incompatibility are milder and of shorter
f. If this sensitization occurs during duration than those of Rh incompatibility.
pregnancy, the fetus is affected in utero; if 5. Care must be taken to observe for hemolysis
sensitization occurs at the time of delivery, and jaundice.
subsequent pregnancies may be affected. D. Assessment findings
4. ABO incompatibility 1. Jaundice and pallor within first 2436 hours
a. Same underlying mechanism 2. Anemia
b. Mother is blood type O; infant is A, B, 3. Erythropoiesis
or AB. 4. Enlarged placenta
c. Reaction in ABO incompatibility is less 5. Edema and ascites
severe. E. Nursing interventions
B. Rh incompatibility 1. Determine blood type and Rh early in
1. First pregnancy: mother may become pregnancy.
sensitized, baby rarely affected 2. Determine results of indirect Coombs test
2. Indirect Coombs test (tests for anti-Rh-positive early in pregnancy and again at 2832 weeks.
antibodies in mothers circulation) performed 3. Determine results of direct Coombs test on
during pregnancy at first visit and again about cord blood (type and Rh, hemoglobin and
28 weeks gestation. If indirect Coombs test is hematocrit).
negative at 28 weeks, a small dose (MicRho 4. Administer RhoGAM IM to mother as
GAM) is given prophylactically to prevent ordered.
sensitization in the third trimester. RhoGAM 5. Monitor carefully infants of Rh-negative and
may also be given after second trimester Type O mothers for jaundice.
amniocentesis. 6. Set up phototherapy as ordered by physician
3. If positive, levels are titrated to determine and monitor infant during therapy.
extent of maternal sensitization and potential 7. Instruct parents if home device will be used.
effect on fetus. 8. Support parents with explanations and
4. Direct Coombs test done on cord blood at information.
delivery to determine presence of anti-Rh-
positive antibodies on fetal RBCs.
5. If both indirect and direct Coombs tests are
Neonatal Sepsis
negative (no formation of anti-Rh-positive A. General information
antibodies) and infant is Rh positive, then 1. Associated with the presence of pathogenic
Rh-negative mother can be given RhoGAM microorganisms in the blood, especially gram-
(Rho[D] human immune globulin) to prevent negative organisms (E. coli, Aerobacter,
development of anti-Rh-positive antibodies as Proteus, and Klebsiella), and gram-positive
the result of sensitization from present (just- group B beta-hemolytic streptococci.
terminated) pregnancy. 2. Contributing factors
6. In each pregnancy, an Rh-negative mother who a. Prolonged rupture of membranes (more
carries an Rh-positive fetus can receive than 24 hours)
RhoGAM to protect future pregnancies if the b. Prolonged or difficult labor
mother has had negative indirect Coombs c. Maternal infection
tests and the infant has had a negative direct d. Infection in hospital personnel
Coombs test. e. Aspiration at birth or later
7. If mother has been sensitized (produced anti- f. Poor handwashing techniques among staff
Rh-positive antibodies), RhoGAM is not B. Assessment findings
indicated. 1. Behavioral changes: lethargy, irritability, poor
8. RhoGAM must be injected into unsensitized feeding
mothers system within first 24 hours if 2. Frequent periods of apnea
possible, by 72 hours at latest. 3. Jaundice
C. ABO incompatibility 4. Hypothermia or low-grade fever
1. Reaction less severe than with Rh 5. Vomiting, diarrhea
incompatibility C. Nursing interventions
2. Firstborn may be affected because type O 1. Perform cultures as indicated/ordered.
mother may have anti-A and anti-B antibodies 2. Administer antibiotics for 3 days until 72-hour
even before pregnancy. cultures backif negative, discontinue; if
3. Fetal RBCs with A, B, or AB antigens evoke positive, continue with full course of specific

6
less severe reaction on part of mother, thus antibiotics

602 NCLEX-RN Review


53155_06_Ch06_p533-620.qxd 2/23/09 12:23 PM Page 603

3. Prevent heat loss. Infant Born to Addicted Mother


4. Administer oxygen as indicated.
5. Maintain hydration. A. General information
6. Monitor vital signs (temperature, pulse, 1. Substance may be alcohol, heroin, morphine,
respirations) frequently. or any other addictive drug.
7. Weigh daily. 2. Mother usually seeks prenatal care only when
8. Stroke back and feet gently to stimulate labor begins and has frequently taken a dose of
breathing if infant is apneic. addictive substance before seeking help,
9. Promote parental attachment and involvement delaying withdrawal symptoms 1224 hours.
in newborn care. 3. Withdrawal symptoms in the neonate may be
noticed within 24 hours.
B. Assessment findings
Hypoglycemia 1. Infants born to alcohol-abusing mothers may
A. General information have facial anomalies, fine-motor dysfunction,
1. Less-than-normal amount of glucose in the genital abnormalities (especially females), and
blood of the neonate: cardiac defects; may be SGA (also called the
2. Common in infants of diabetic mothers (IDM), fetal alcohol syndrome)
especially type III: these infants usually LGA 2. Hyperirritability and hyperactivity
(macrosomic) due to high maternal glucose 3. High-pitched cry common
levels that have crossed placenta, stimulating 4. Respiratory distress, tachypnea, excessive
the fetal pancreas to secrete more insulin, secretions
which then acts as growth hormone. 5. Vomiting and diarrhea
3. At birth, with loss of supply of maternal 6. Elevated temperature
glucose, newborn may become hypoglycemic. 7. Other signs of withdrawal: sneezing; sweating;
4. Large size may have caused traumatic vaginal yawning; short, nonquiet sleep; frantic sucking
birth, or may have necessitated cesarean C. Nursing interventions
birth. 1. Reduce external stimuli.
5. Hypoglycemia can also occur in infants who 2. Handle minimally.
are full term, SGA, postterm, septic, or with 3. Swaddle infant and hold close when handling.
any condition that subjects the infant to 4. Monitor infants vital signs.
stress. 5. Suction/resuscitate as required.
B. Assessment findings 6. Feed frequently, with small amounts.
1. May be born prematurely due to complications 7. Measure I&O.
2. Although LGA, IDMs may be 8. Provide careful skin care.
immature/dysmature 9. Administer medications if ordered (may use
3. Higher incidence of congenital anomalies in phenobarbital or paragoric).
IDMs 10. Involve parents in care if possible.
4. General appearance 11. Inform parents of infants condition/progress.
a. Puffy body
b. Enlarged organs Respiratory Distress Syndrome (RDS)
5. Tremors, feeding difficulty, irregular
A. General information
respirations, lethargy, hypothermia
1. Symptoms found almost exclusively in the
6. Hypocalcemia and hyperbilirubinemia
preterm infant.
7. Respiratory distress
2. Deficiency of surfactant increases surface
8. Blood glucose levels below 40 mg/dL
tension, which causes alveolar collapse.
(20 mg/dL in premature infant) using
3. When women are likely to deliver
Dextristix
prematurely, betamethasone is given IM in two
C. Nursing interventions
doses, 12 hours apart.
1. Provide high-level nursing care similar to that
4. Additional factors: hypoxia, hypothermia,
for premature/dysmature infant.
acidosis
2. Assess blood glucose level at frequent
5. Sequelae of RDS may include
intervals, beginning 121 hour after birth in
a. Patent ductus arteriosus
at-risk infants.
b. Hyperbilirubinemia
3. Feed hypoglycemic infant according to
c. Retrolental fibroplasia: retinal changes,
nursery protocol (formula or breast preferred)
visual impairment and eventually
if suck/swallow reflex present and
blindness, resulting from too-high oxygen
coordinated.
levels during treatment
4. Poor suck-swallow or lack of serum response
d. Bronchopulmonary dysplasia (BPD):
to PO feeding, administer IV glucose.
damage to the alveolar epithelium of the

MATERNITY AND FEMALE REPRODUCTIVE NURSING

6 603
53155_06_Ch06_p533-620.qxd 2/23/09 12:23 PM Page 604

lungs related to high oxygen Apgar score, RDS, prematurity, neonatal


concentrations and positive pressure shock, and asphyxia).
ventilation. May be difficult to wean infant 4. Average age at onset is 4 days.
from ventilator, but most recover and have 5. Now that severely ill infants are surviving,
normal X-rays at 6 months to 2 years. NEC is encountered more frequently.
e. Necrotizing enterocolitis. 6. May ultimately cause bowel perforation and
B. Assessment findings death.
1. Respiratory rate of over 60/minute 7. Less common in breastfed premature infants.
2. Retractions, grunting, cyanosis, nasal flaring, B. Medical management
chin lag 1. Parenteral antibiotics
3. Increased apical pulse 2. Gastric decompression
4. Hypothermia 3. Correction of acidosis and fluid electrolyte
5. Decreased activity level imbalances
6. Elevated levels of carbon dioxide 4. Surgical removal of the diseased intestine
7. Metabolic acidosis C. Assessment findings
8. X-rays show atelectasis and density in alveoli. 1. History indicating high-risk group
C. Nursing interventions 2. Findings related to sepsis
1. Maintain infants body temperature at 36.4C a. Temperature instability
(97.6F). b. Apnea, labored respirations
2. Provide sufficient caloric intake for size, age, c. Cardiovascular collapse
and prevention of catabolism (usually IV d. Lethargy or irritability
glucose with gradual increase in feedings); 3. Gastrointestinal symptoms
nasogastric tube may be used. a. Abdominal distension and tenderness
3. Organize care for minimal handling of infant. b. Vomiting or increased gastric residual
4. Administer oxygen therapy as ordered. c. Poor feeding
a. Monitor oxygen concentration every d. Hematest positive stools
24 hours; maintain less than 40% e. X-rays showing air in the bowel wall,
concentration if possible. adynamic ileus, and bowel wall thickening
b. Oxygen may be administered by hood, D. Nursing interventions
nasal prongs, intubation, or mask. 1. Carefully assess infants at risk for early
c. Oxygen may be at atmospheric or recognition of symptoms.
increased pressure. 2. Discontinue oral feedings, insert nasogastric
d. Continuous positive air pressure (CPAP) or tube.
positive end-expiratory pressure (PEEP) 3. Prevent trauma to abdomen by avoiding
may be used. diapers and planning care for minimal
e. Oxygen should be warmed and handling.
humidified. 4. Maintain acid-base balance by administering
5. Monitor infants blood gases. fluids and electrolytes as ordered.
6. If intubated, suction (for less than 5 seconds) 5. Administer antibiotics as ordered.
prn using sterile catheter. 6. Stroke infants hands and head and talk to
7. Auscultate breath sounds. infant as much as possible.
8. Provide chest physiotherapy, postural 7. Provide visual and auditory stimulation.
drainage, and percussion if ordered. 8. Inform parents of progress and support them
9. Encourage parental involvement in care in expressing their fears and concerns.
(visiting, stroking infant, talking).
10. Administer surfactant via endotracheal tube
and other medications as ordered.
Phenylketonuria (PKU)
A. General information
1. Inability to metabolize phenylalanine to
Necrotizing Enterocolitis (NEC) tyrosine because of an autosomal recessive
A. General information inherited disorder causing an inborn error of
1. An ischemic attack to the intestine resulting in metabolism
thrombosis and infarction of affected bowel, 2. Phenylalanine is a composite of almost
mucosal ulcerations, pseudomembrane all proteins: the danger to the infant is
formation, and inflammation. immediate.
2. Bacterial action (E. coli, Klebsiella) 3. High levels of phenylketones affect brain cells,
complicates the process, producing sepsis. causing mental retardation.
3. May be precipitated by any event in which 4. Initial screening for diagnosis of PKU is made
blood is shunted away from the intestine to via the Guthrie test, done after the infant has

6
the heart and brain (e.g., fetal distress, low ingested protein for a minimum of 24 hours.

604 NCLEX-RN Review


53155_06_Ch06_p533-620.qxd 2/23/09 12:23 PM Page 605

5. Secondary screening 1 hour after birth, the baby exhibits tremors. The
a. Done when the infant is about 6 weeks old. nurse performs a heel stick and a Dextrostix test.
b. Test fresh urine with a Phenistix, which The result is 40 mg/dL. The nurse is aware these
changes color. symptoms are most likely caused by what
c. Parents send in a prepared sheet marking condition?
the color. 1. Hypoglycemia.
6. These tests, mandatory in many states, allow
the early diagnosis of the disorder, and dietary 2. Hypokalemia.
interventions to minimize or prevent 3. Hypothermia.
complications. 4. Hypercalcemia.
B. Assessment findings
1. Phenylalanine levels greater than 8 mg/dL are 110. A newborn weighs 1450 g, has weak muscle
diagnostic for PKU. tone, with extremities in an extended position
2. Newborn appears normal; may be fair with while at rest. The pinna is flat and does not
decreased pigmentation. readily recoil. Very little breast tissue is palpable.
3. Untreated PKU can result in failure to thrive, The soles have deep indentations over the upper
vomiting, and eczema; by about 6 months, one-third. Based on these data, what should the
signs of brain involvement appear. nurse know about the babys gestational age?
C. Nursing interventions
1. Full-term infant, 3842 weeks gestation.
1. Restrict protein intake.
2. Substitute a low-phenylalanine formula 2. Premature infant, less than 24 weeks gestation.
(Lofenalac) for either mothers milk or formula. 3. Premature infant, 2933 weeks gestation.
3. Provide special food lists for parents. 4. Postterm infant greater than 42 weeks
gestation.

111. A premature infant at 6 hours old, has respirations


Sample Questions of 64, mild nasal flaring, and expiratory grunting.
She is pink in room air, temperature is 36.5C.
The babys mother ruptured membranes 36 hours
106. In differentiating physiologic jaundice from
prior to delivery. Which measures should the
pathologic jaundice, which of the following facts
nurse include in the plan of care?
is most important?
1. Have respiratory therapy set up a respirator
1. Mother is 37 years of age.
because respiratory failure is imminent. Get
2. Infant is a term newborn. blood gases every hour.
3. Unconjugated bilirubin level is 6 mg/dL on 2. Encourage mother/infant interaction.
third day. Rooming in as soon as stable. Monitor vital
4. Appears at 12 hours after birth. signs every 8 hours.
3. Observe for signs of sepsis. Cultures if
107. A newborn is receiving phototherapy. What will
ordered. Monitor vital signs at least every 2
be included during the phototherapy to meet
hours for the first 24 hours. Encourage family
safety needs?
interaction with infant.
1. Limit fluid intake.
4. Radiant warmer for first 48 hours. Vital signs
2. Cover the infants eyes while he is under the every hour. Restrict visitation due to risk of
light. infection.
3. Keep him clothed to prevent skin burns.
4. Make sure the light is not closer than 24 inches. 112. During the assessment of a 2-day-old infant with
bruising and a cephalhematoma, the nurse notes
108. The morning temperature on a newborn is jaundice of the face and trunk. The baby is also
36.4C (97.6F). In order to prevent cold stress, being breastfed. Bilirubin level is 10 mg/dL. What
which action will be included in the plan of care? is the most likely interpretation of these findings?
1. Keep the babys head covered. 1. Hyperbilirubinemia due to the bruising and
2. Keep the baby unwrapped. cephalhematoma.
3. Turn up the thermostat in the nursery. 2. Pathologic jaundice requiring exchange
4. Use warm water for the bath. transfusion.
3. Breast milk jaundice.
109. A diabetic woman has a problem-free prenatal 4. Hyperbilirubinemia due to blood group
course and delivers a full-term 9 lb 2 oz girl. At

6
incompatibility.

MATERNITY AND FEMALE REPRODUCTIVE NURSING 605


53155_06_Ch06_p533-620.qxd 2/23/09 12:23 PM Page 606

113. A 6-hour-old newborn has been diagnosed with 117. You are caring for an infant. During your
erythroblastosis fetalis. What is the cause of this assessment you note a flattened philtrum, short
condition? palpebral fissures, and birth weight and head
1. ABO blood group incompatibility between circumference below the fifth percentile for
the father and infant. gestational age. The infant has a poor suck.
2. Rh incompatibility between the mother and Which of the following is the best interpretation
infant. of this data?
3. ABO blood group incompatibility between 1. Downs syndrome.
the mother and infant. 2. Fetal alcohol syndrome.
4. Rh incompatibility between father and infant. 3. Turners syndrome.
4. Congenital syphilis.
114. An Rh-negative mother has just given birth to an
Rh-positive infant. She had a negative indirect 118. A 2-week-old premature infant with abdominal
Coombs test at 38 weeks gestation and her distention, significant gastric aspirate prior to
infant had a negative direct Coombs test. What feeding, and bloody stools has also had episodes
should the nurse know about these tests? of apnea and bradycardia and temperature
1. Although her infant is Rh positive, she has no instability. What should the nurse include in the
antibodies to the Rh factor. RhoGAM should plan of care for this infant?
be given. 1. Increase feeding frequency to every 2 hours.
2. She has demonstrated antibodies to the Rh 2. Place the infant on seizure precautions.
factor. She should not have any more children.
3. Place the infant in strict isolation to prevent
3. She has formed antigens against the Rh infection of other infants.
factor. RhoGAM must be given to the infant.
4. Monitor infant carefully including blood
4. Because her infant is Rh positive, the pressure readings and measurements of
Coombs tests are meaningless. abdominal girth.
115. An infant was born at 38 weeks gestation to a
119. A mother is taking her newborn home from the
heroin-addicted mother. At birth, the baby had
hospital at 18 hours after birth. As the nurse is
Apgar scores of 5 at 1 minute and 6 at 5 minutes.
giving discharge instructions, which response
Birth weight was at the 10th percentile for
best validates her understanding of PKU testing?
gestational age. What should the nurse include
in the babys plan of care? 1. I know you stuck my babys heel today for
that PKU test and that my doctor will recheck
1. Administer methadone to diminish
the test when I bring her for her 1 month
symptoms of heroin withdrawal.
appointment.
2. Promote parent-infant attachment by
2. After I start my baby on cereal, I will return
encouraging rooming-in.
for a follow-up blood test.
3. Observe for signs of jaundice because this is a
3. I will have a visiting nurse come to the
common complication.
house each day for the first week to check the
4. Place in a quiet area of the nursery and PKU test.
swaddle close to promote more organized
4. I will bring my baby back to the hospital or
behavioral state.
doctors office to have a repeat PKU no later
116. A 36-week-gestation infant had tachypnea, nasal than 1 week from today.
flaring, and intercostal retractions that increased
over the first 6 hours of life. The baby was
treated with IV fluids and oxygen. Which of the
following assessments suggests to the nurse that Answers and Rationales
the baby is improving?
1. The baby has see-saw respirations with 106. 4. Time is one of the most important criteria in
coarse breath sounds. differentiating physiologic from pathologic
2. The babys respiratory rate is 50 and pulse is jaundice. Physiologic jaundice appears after 24
136, no nasal flaring is observed. hours. When jaundice appears earlier, it may be
3. The baby has a pH of 6.97 and pO2 of 61 on pathologic.
40% oxygen.
107. 2. The infant receiving phototherapy should
4. The baby has gained 150 grams in the
have a covering put over his eyes to protect them

6
12 hours since birth.
from light.

606 NCLEX-RN Review


53155_06_Ch06_p533-620.qxd 2/23/09 12:23 PM Page 607

108. 1. The babys head should be kept covered. The 113. 2. Erythroblastosis fetalis results when an Rh-
head is the greatest source of heat loss. negative woman makes antibodies against her
Rh-positive fetus. The antibodies attack fetal red
109. 1. Tremors are symptoms of the neonatal cells.
hypoglycemia. The baby of a diabetic mother is
at high risk for hypoglycemia because the 114. 1. Because the indirect and direct Coombs tests
infants insulin levels are high before birth and were negative, antibodies to Rh have not
continue to be high even though the infant has developed. She should have RhoGAM to prevent
suddenly lost the influx of glucose. Immediate antibody formation.
administration of IV glucose will be ordered for
the infant. 115. 4. Neonatal withdrawal is a common occurrence
in heroin addiction. Placing the baby in a quiet
110. 3. A birth weight of 1450 grams is the mean area and swaddling may promote state
weight for an infant at 30 weeks gestation, but organization and minimize some symptoms.
falls within the 1090th percentiles for infants Medication may be needed to control
between 29 and 33 weeks gestation. The hyperirritability.
diminished muscle tone and extension of
extremities at rest are also characteristic of this 116. 2. The babys respiratory rate and pulse are
gestational age. The sole creases described are within normal limits and the nasal flaring is no
actually most characteristic of an infant between longer present.
32 and 34 weeks gestation.
117. 2. Although a medical diagnosis cannot be made
111. 3. Prolonged rupture of membranes places this from the assessment data, all of the findings
premature infant at risk for sepsis. Frequent noted are commonly seen in infants with fetal
monitoring of vital signs, color, activity level, and alcohol syndrome.
overall behavior is particularly important because
118. 4. The infants prematurity is the major risk
changes may provide early cues to a developing
factor for necrotizing enterocolitis, which affects
infection. Family interaction with the infant
115% of all infants in NICU. Usual nonsurgical
should always be a part of the nursing plan.
treatment includes antibiotic therapy, making
112. 1. Although hyperbilirubinemia is common in the infant NPO, frequent monitoring, and
newborns, certain factors increase the likelihood respiratory and circulatory support as needed.
of early appearance of visible jaundice. Cold
119. 4. One additional PKU test within the first week
stress, bruising at delivery, cephalhematoma,
of life will validate whether PKU disease is
asphyxiation, prematurity, breastfeeding, and
present. The infant should have been on breast
poor feeding are all factors that may lead to
milk or formula for 48 hours prior to the test.
hyperbilirubinemia in otherwise normal infants.

Conditions of the Female Reproductive System

INFERTILITY AND FERTILITY D. May be primary (never been pregnant/never


impregnated) or secondary (pregnant once, then
unable to conceive or carry again)
Infertility E. Affects approximately 1015% of all couples
General Information F. Tests for infertility can include:
1. For the female
A. Inability to conceive after at least 1 year of a. Examination of basal body temperature
unprotected sexual relations and cervical mucus and identification of
B. Inability to deliver a live infant after three time of ovulation
consecutive pregnancies b. Plasma progesterone level: assesses corpus
C. For the male, inability to impregnate a female luteum
partner within the same conditions c. Hormone analysis: endocrine function

MATERNITY AND FEMALE REPRODUCTIVE NURSING

6 607
53155_06_Ch06_p533-620.qxd 2/23/09 12:23 PM Page 608

d. Endometrial biopsy: receptivity of warmth of body): changing these habits may


endometrium reverse low/absent fertility.
e. Postcoital test: sperm placement and C. Alternatives for infertile couples include
cervical mucus 1. Artificial insemination by husband or donor
f. Hysterosalpingography: tubal 2. In vitro fertilization
patency/uterine cavity 3. Adoption
g. Rubins test: tubal patency (uses carbon 4. Surrogate parenting
dioxide) 5. Embryo transfers
h. Pelvic ultrasound: visualization of pelvic D. Accepting childlessness as a lifestyle may also be
tissues necessary; support groups (e.g., Resolve) may be
i. Laparoscopy: visual assessment of helpful.
pelvic/abdominal organs; performance of
minor surgeries Nursing Interventions
2. For the male
a. Sperm analysis: assesses composition, A. Assist with assessment including a complete
volume, motility, agglutination. history, physical exam, lab work, and tests for both
b. There are fewer assessment tests as well as partners.
interventions and successes for male B. Monitor psychologic reaction to infertility.
infertility. C. Support couple through procedures and tests.
D. Identify any existing abnormalities and provide
Medical Management couple with information about their condition(s).
E. Help couple acknowledge and express their
A. Infertility of female partner, causes and therapy feelings both separately and together.
1. Congenital anomalies (absence of organs,
improperly formed or abnormal organs):
surgical treatment may help in some situations
Control of Fertility
but cannot replace absent structures. Voluntary prevention of conception through various
2. Irregular/absent ovulation (ovum released means, some of which employ devices or medications.
irregularly or not at all): endocrine therapy
with clomiphene citrate (Clomid)/menotropins Methods of Conception Control
(Pergonal) may induce ovulation; risk of
ovarian hyperstimulation and release of A. Natural methods
multiple ova. 1. Natural family planning
3. Tubal factors (fallopian tubes blocked or a. Periodic abstinence from intercourse when
scarred from infection, surgery, endometriosis, ovulating
neoplasms): treatment may include antibiotic b. Uses calculations intended to identify
therapy, surgery, hysterosalpingogram. those days of the menstrual cycle when
4. Uterine conditions (endometrium unreceptive, coitus is avoided.
infected): removal of an IUD, antibiotic 1) Basal body temperature: identification
therapy, or surgery may be helpful. of temperature drop before ovulation,
5. Vaginal/cervical factors (hostile mucus, sperm then rise past ovulation; identifies
allergies, altered pH due to infection): days on which coitus is avoided to
treatment with antibiotics, proper vaginal avoid conception.
hygiene, or artificial insemination may be 2) Cervical mucus method: identification
utilized. of changes in cervical mucus; when
B. Infertility of male partner, causes and therapy affected by estrogen and most
1. Impotence: may be helped by psychologic conducive to penetration by sperm,
counseling/penile implants, medication. cervical mucus is clear, stretchy, and
2. Low/abnormal sperm count (fewer than 20 slippery; when influenced by
million/mL semen, low motility, more than progesterone, cervical mucus is thick,
40% abnormal forms): there is no good cloudy, and sticky and does not allow
therapy, use of hormone replacement therapy sperm passage; coitus is avoided
has had little success. during days of estrogen-influenced
3. Varicocele (varicosity within spermatic cord): mucus.
ligation may be successful. 3) Sympto-thermal: combination of basal
4. Infection in any area of the male reproductive body temperature and cervical mucus
system (may affect ability to impregnate): method to increase effectiveness.
appropriate antibiotic therapy is advised. 2. Coitus interruptus
5. Social habits (use of nicotine, alcohol, other a. Withdrawal of the penis from the vagina

6
drugs; clothes that keep scrotal sac too close to before ejaculation.

608 NCLEX-RN Review


53155_06_Ch06_p533-620.qxd 2/23/09 12:23 PM Page 609

b. Not very safe; pre-ejaculatory fluids from D. Hormone therapy (oral contraceptives, birth
Cowpers glands may contain live, motile control pills)
sperm. 1. Ingestion of estrogen and progesterone on a
c. Demands precise male control. specific schedule to prevent the release of FSH
B. Chemical barriers and LH, thus preventing ovulation and
1. Use of foams, creams, jellies, and vaginal pregnancy.
suppositories designed to destroy the sperm or 2. Causes additional tubal, endometrial, and
limit their motility cervical mucus changes.
2. Available without a prescription, widely used, 3. Available in combined or sequential types.
especially in conjunction with the diaphragm 4. Usually taken beginning on day 5 of the
and the condom menstrual cycle through day 25, then
3. Need to be placed in the vagina immediately discontinued.
before each act of intercourse; messy 5. Withdrawal bleeding occurs within 23 days.
4. Some people may have allergic reaction to the 6. Contraindications
chemicals a. History of hypertension or vascular disorders
C. Mechanical barriers: diaphragm, condom, cervical b. Age over 35
cap, contraceptive sponge c. Cigarette smoking (heavy)
1. Diaphragm: shallow rubber dome fits over 7. Women using oral contraceptives need to be
cervix, blocking passage of sperm through sure to get sufficient amounts of vitamin B as
cervix metabolism of this vitamin is affected.
a. Efficiency increased by use of chemical 8. Minor side effects may include
barrier as lubricant a. Weight gain
b. Woman needs to be measured for b. Breast changes
diaphragm, and refitted after childbirth or c. Headaches
weight gain/loss of 10 lb d. Vaginal spotting
c. Device needs to be left in place 68 hours 9. Report vision changes/disorders immediately.
after intercourse. E. Intrauterine devices (IUD)
d. Woman needs to practice insertion and 1. Placement of plastic or nonreactive device into
removal, and to be taught how to check for uterine cavity
holes in diaphragm, store in cool place. 2. Mode of action thought to be the creation of a
2. Condom: thin stretchable rubber sheath worn sterile endometrial inflammation, discourages
over penis during intercourse implantation (nidation).
a. Widely available without prescription 3. Does not affect ovulation.
b. Applied with room at tip to accommodate 4. Device is inserted during or just after
ejaculate menstruation, while cervix is slightly open.
c. Applied to erect penis before vaginal 5. May cause cramping and heavy bleeding during
penetration menses for several months after insertion.
d. Man is instructed to hold on to rim of 6. Tail of IUD hangs into vagina through cervix;
condom as he withdraws from female to woman taught to feel for it before intercourse
prevent spilling semen. and after each menses.
3. Cervical cap: cup-shaped device that is placed 7. A distinct disadvantage is the increased risk of
over cervical os and held in place by suction. pelvic infection (PID) with use of the IUD.
a. Four sizes; client needs to be fitted F. Surgical sterilization
b. Women need to practice insertion and 1. Bilateral tubal ligation in the female to prevent
removal the passage of ova.
c. Spermicidals increase effectiveness 2. Bilateral vasectomy in the male to prevent the
d. May be left in place for up to 24 hours passage of sperm.
4. Contraceptive sponge: small, soft insert, with 3. Both of these operations should be considered
indentation on one side to fit over cervix; permanent.
contains spermicide 4. Female will still menstruate but will not
a. Moistened with water and inserted with conceive.
indentation snugly against cervix 5. Male will be incapable of fertilizing his
b. May be left in place up to 24 hours partner after all viable sperm ejaculated from
c. No professional fitting required vas deferens (6 weeks or 10 ejaculations).
d. May also protect against STDs 6. There should be no effect on male capacity for
e. Should not be used by women with history erection or penetration.
of toxic shock syndrome 7. Hysterectomy also causes permanent sterility
f. Problems include cost, difficulty in in the female.
removal, and irritation G. Steroid implants: approved in 1990 by

6
FDA; biodegradable rods containing

MATERNITY AND FEMALE REPRODUCTIVE NURSING 609


53155_06_Ch06_p533-620.qxd 2/23/09 12:23 PM Page 610

sustained-release, low-dose progesterone. Inhibits b. Prostaglandins


LH (luteinizing hormone) release necessary for 1) Injection of prostaglandin into uterus
ovulation. Effective over 5-year time frame. Need 2) Contractions initiated in under 1 hour
minor surgical procedure for insertion and removal. 3) Side effects may include nausea and
Removal causes total reversibility of effect. vomiting
H. Injectable progestin-same action as G; lasts 3 months c. Hysterotomy
1) Incision into uterus to remove fetus.
Nursing Responsibilities in Control of Fertility 2) May also be used for sterilization.
3) Client is hospitalized.
A. Assess previous experience of couple or individual. 4) Care is similar to that for cesarean birth.
B. Obtain health history and perform physical 3. Third trimester: same as second trimester, if
examination. permitted by state law.
C. Identify present needs for contraception.
D. Determine motivation regarding contraception. Assessment
E. Assist the client/couple in receiving information
desired; advise about the various methods available. A. Vaginal bleeding
F. Ensure that client/couple selects method best B. Vital signs
suited to their needs. C. Excessive cramping
1. Support choice of client/couple as right for
them. Analysis
2. Provide time for practice with method chosen,
if applicable. A. Risk for deficient fluid volume
3. Instruct in side effects/potential B. Risk for injury
complications. C. Deficient knowledge
G. Encourage expression of feelings about
contraception. Planning and Implementation
A. Goals
Termination of Pregnancy 1. Recovery from procedure will be free from
complications.
General Information
2. Client will be supported in her decision.
A. Deliberate interruption of a pregnancy in a B. Interventions
previable time. Legal in all states since Supreme 1. Explain procedure to client.
Court ruling of January 1973, as follows 2. Administer medications as ordered.
1. First trimester: determined by pregnant 3. Assist with procedure as needed.
woman and her physician. 4. Monitor client carefully during procedure.
2. Second trimester: determined by pregnant 5. Monitor client postprocedure.
woman and her physician; state can regulate 6. Administer postprocedure medications as
the circumstances to ensure safety. ordered (analgesics, antibiotics, oxytocins,
3. Third trimester: conditions determined by RhoGAM if mother Rh negative).
state law. 7. Provide contraceptive information as
B. Indications may be physical or psychologic, appropriate.
socioeconomic or genetic. 8. May need post-abortion counseling/therapy
C. Techniques vary according to trimester. (also available at Crisis Pregnancy Centers).
1. First trimester: vacuum extraction or dilation
and curettage (D&C) Evaluation
a. Cervix dilated
b. Products of conception either aspirated or A. Procedure tolerated without complications; vital
scraped out signs stable, no hemorrhage, products of
c. Procedure is short, usually well tolerated conception evacuated, no infection
by client, and has few complications. B. Client supported through procedure; emotionally
2. Second trimester stable
a. Saline abortion
1) Amniotic fluid aspirated from uterus,
replaced with same amount 20% MENSTRUAL DISORDERS
saline solution.
2) Contractions begin in 1224 hours; Menstruation is the periodic shedding of the
may be induced by oxytocin (Pitocin) endometrium when there has been no conception.
3) Client is hospitalized; infection or Onset is menarche (age 1114); cessation is
hypernatremia possible complications menopause (average age 50).

6 610 NCLEX-RN Review


53155_06_Ch06_p533-620.qxd 2/23/09 12:23 PM Page 611

Assessment Endometriosis
A. Menstrual cycle for symptoms and pattern See Figure 6-13.
B. Client discomfort with cycles A. Endometrial tissue is found outside the uterus,
C. Knowledge base about menses attached to the ovaries, colon, round ligaments,
etc.
Analysis B. This tissue reacts to the endocrine stimulation
cycle as does the intrauterine endometrium,
A. Deficient knowledge resulting in inflammation of the extrauterine sites,
B. Ineffective health maintenance with pain and fibrosis/scar tissue formation as the
eventual result.
Planning and Implementation C. Actual cause is unknown.
D. May cause dysmenorrhea, dyspareunia, and
A. Goals
infertility.
1. Client will receive necessary information.
E. Treatment may include the use of oral
2. Client will choose treatment/options best
contraceptives to minimize endometrial buildup or
suited to her needs.
medications to suppress menstruation (Danocrine,
B. Interventions
Synarel).
1. Explain menstrual physiology to client.
F. Pregnancy and lactation may also be
2. Explain options for treatment to client.
recommended as means to suppress menstruation.
3. Provide time for questions.
G. Surgical intervention (removal of endometrial
4. Reinforce good menstrual hygiene.
implants) may be helpful.
5. Administer medications if ordered.
H. Hysterectomy and salpingo-oophorectomy are
curative.
Evaluation
Client demonstrates knowledge of condition and
treatment options.

Specific Disorders
Dysmenorrhea
A. Pain associated with menstruation.
B. Usually associated with ovulatory cycles; absent
when ovulation suppressed.
C. Intensified by stress, cultural factors, and presence
of an IUD.
D. High levels of prostaglandins found in menstrual
flow of women with dysmenorrhea.
E. Treatment my include rest, application of heat,
distraction, exercise, analgesia (especially anti-
prostaglandins: NSAIDs).

Amenorrhea
A. Absence of menstruation.
B. Possibly caused by underlying abnormality of endo-
crine system, rapid weight loss, or strenuous exercise.
C. Treatment is individualized by cause.

Menorrhagia
A. Excessive menstrual flow
B. Possibly caused by endocrine imbalance, uterine
tumors, infection
C. Treatment individualized by cause

Metrorrhagia Figure 6-13 Endometriosis can be found outside


A. Intercyclic bleeding the uterus, attached to the ovaries, colon, and round
B. Frequently the result of a disease process ligaments

6
C. Treatment individualized by cause

MATERNITY AND FEMALE REPRODUCTIVE NURSING 611


53155_06_Ch06_p533-620.qxd 2/23/09 12:23 PM Page 612

INFECTIOUS DISORDERS Chlamydia


A. Currently most common STD
Sexually Transmitted Diseases (STD) B. Symptoms similar to gonorrhea (cervical/vaginal
Infections occurring predominantly in the genital area discharge) or may be asymptomatic
and spread by sexual relations. C. Can be transmitted to fetus at birth, causes
neonatal ophthalmia
D. Treated with erythromycin, prophylactic treatment
Assessment of neonates eyes
A. Sexual history/social practices E. If untreated, can lead to pelvic inflammatory
B. Physical examination for signs and symptoms of disease (PID)
specific disorder F. Receive regular PAP smears.

Analysis Gonorrhea
A. Deficient knowledge A. Caused by Neisseria gonorrhoeae.
B. Risk for injury B. Symptoms may include heavy, purulent vaginal
C. Ineffective health maintenance discharge, but often asymptomatic in female.
C. May be passed to fetus at time of birth, causing
Planning and Implementation ophthalmia neonatorum and sepsis.
D. Treatment is penicillin; allergic clients may be
A. Goals treated with erythromycin or (if not pregnant) the
1. Disease process will be identified and treated. cephalosporins.
2. Affected others will be identified and treated. E. All sexual contacts must be treated as well, to
3. Complications will be prevented. prevent ping-pong recurrence.
B. Interventions
1. Collect specimens for tests. Syphilis
2. Implement isolation technique if indicated.
3. Teach transmission/prevention techniques. A. Caused by Treponema pallidum (spirochete)
4. Assist in case finding. B. Crosses placenta after 16th week of pregnancy to
5. Administer medications as ordered. infect fetus.
6. Inform client of any necessary lifestyle changes. C. Initial symptoms are chancre and lymph-
adenopathy and may disappear without treatment
Evaluation in 46 weeks.
D. Secondary symptoms are rash, malaise, and
A. Client receiving treatment appropriate to specific alopecia; these too may disappear in several weeks
disorder, understands treatment regimen. without treatment.
B. Client demonstrates knowledge of disease process E. Tertiary syphilis may recur later in life and affect
and transmission. any organ system, especially cardiovascular and
C. Affected others have been identified and treated. neurologic systems.
F. Diagnosis is made by dark-field exam and
serologic tests (VDRL).
Specific Disorders G. Treatment is penicillin, or erythromycin if
penicillin allergy exists.
Herpes
A. Genital herpes is caused by herpes simplex virus Other Genital Infections
type 2 (HSV2). Cervical and vaginal infections may be caused by
B. Causes painful vesicles on genitalia, both external agents other than those associated with STDs. For
and internal. all female clients with a vaginal infection, nursing
C. There is no cure. actions should include teaching good perineal
D. Treatment is symptomatic. hygiene.
E. If active infection at the end of pregnancy,
cesarean birth may be indicated, because virus Trichomonas vaginalis
may be lethal to neonate who cannot localize
infection. A. Caused by a protozoan
F. Recurrences of the condition may be caused by B. Major symptom is profuse foamy white to greenish
infection, stress, menses. discharge that is irritating to genitalia.
G. Acyclovir (Zovirax) reduces severity and duration C. Treatment is metronidazole (Flagyl) for woman
of exacerbation. and all sexual partners.

6 612 NCLEX-RN Review


53155_06_Ch06_p533-620.qxd 2/23/09 12:23 PM Page 613

D. Treatment lasts 7 days, during which time a leaning forward. Assessment should include
condom should be used for intercourse. size, symmetry, shape, direction, color, skin
E. Alcohol ingestion with Flagyl causes severe texture and thickness, nipple size and shape,
gastrointestinal upset. rashes or discharges. Unusual findings should
be reported to health care provider.
Candida albicans 2. Palpation: to examine left breast, woman
should be lying down, with left hand behind
A. Caused by a yeast transmitted from GI tract to head and small folded towel or pillow under
vagina. left shoulder. Using flattened fingertips of right
B. Overgrowth may occur in pregnancy, with hand and a rotary motion, palpate along lines
diabetes, and with steroid or antibiotic therapy. of concentric circles from outer edges of breast
C. Vaginal examination reveals thick, white, cheesy to nipple area, or from outer edge to nipple
patches on vaginal walls. area following wedge or wheel-spoke lines.
D. Treatment is topical application of clotrimazole Also palpate in the left axillary area where
(Gyne-Lotrimin), nystatin (Mycostatin), or gentian multiple lymph nodes are present, as well as a
violet. tail of breast tissue. The nipple should be
E. Candida albicans causes thrush in the newborn by gently squeezed to assess for discharges.
direct contact in the birth canal. a. To examine right breast, positions are
reversed.
Bacterial Vaginitis b. Palpation activities are repeated for each
breast with the woman in the sitting
A. Caused by other bacteria invading the vagina position.
B. Foul or fishy-smelling discharge c. Unusual findings are reported to the health
C. Treatment is specific to causative agent, and care provider.
usually includes sexual partners for best results d. Breast self-examination (BSE) and
mammograms as indicated by age and risk
AIDS (see Unit 4) are primary screening tools.

Female Reproductive System Neoplasia Breast Cancer


The nursing diagnoses, general goals and
A. General information
interventions, and evaluation for the client with
1. Most common neoplasm in women
cancer of the reproductive system are similar to those
2. Leading cause of death in women age 4044
for any client with a diagnosis of cancer. Only nursing
B. Medical management
care specific to the disorder will be discussed here.
1. Usually surgical excision; options are simple
lumpectomy, simple mastectomy, modified
Fibrocystic Breast Disease radical mastectomy, and radical mastectomy.
2. Adjuvant treatment with chemotherapy,
A. Most common benign breast lesion.
radiation, and hormone therapy.
B. Cyst(s) may be palpated; surgical biopsy indicated
C. Assessment findings
for differential diagnosis.
1. Palpation of lump (upper outer quadrant most
C. Treatment includes surgical removal of cysts,
frequent site) usually first symptom
decreasing or removing caffeine from diet, and
2. Skin of breast dimpled
medication to suppress menses.
3. Nipple discharge
4. Asymmetry of breasts
Procedure for Breast 5. Surgical biopsy provides definitive diagnosis
Self-Examination (BSE) D. Nursing interventions
1. Assess breasts for early identification and
A. Age: routine BSE should begin as early in a treatment.
womans life as possible. Adolescence is not too 2. Support client through recommended/chosen
early. treatment.
B. Timing: regularly, on a monthly basis, 3 to 7 days 3. Prepare client for mastectomy if necessary.
after the end of the menses, when breasts are least
likely to be swollen or tender. After menopause,
BSE should be done on one particular day/date
Mastectomy
every month. A. General information
C. Procedure 1. Lumpectomy: removal of lump and surrounding
1. Inspection: stand before mirror and visually breast tissue; lymph nodes biopsied.
inspect with arms at sides; raised over head; 2. Simple mastectomy: removal of breast only,

6
hands on hips with muscles tightened; then lymph nodes biopsied.

MATERNITY AND FEMALE REPRODUCTIVE NURSING 613


53155_06_Ch06_p533-620.qxd 2/23/09 12:23 PM Page 614

3. Radical mastectomy: removal of breast, muscle Cancer of the Ovary


layer down to chest wall, and axillary lymph
nodes. A. Etiology unknown.
B. Nursing interventions B. Few early symptoms; palpation of ovarian mass is
1. Provide routine pre- and post-op care. usual first finding.
2. Elevate clients arm on operative side on C. Treatment of choice is surgical removal with total
pillows to minimize edema. hysterectomy and bilateral salpingo-oophorectomy.
3. Do not use arm on affected side for blood D. Chemotherapy may be used as adjuvant therapy.
pressure measurements, IVs, or injections.
4. Turn only to back and unaffected side. Cancer of the Vulva
5. Monitor client for bleeding, check under her.
6. Begin range-of-motion exercises immediately A. Begins as small, pruritic lesions
on unaffected side. B. Diagnosed by biopsy
7. Start with simple movements on affected side: C. Treatment is either local excision or radical
fingers and hands first, then wrist, elbow, and vulvectomy (removal of entire vulva plus
shoulder movements. superficial and femoral nodes).
8. Make abduction the last movement.
9. Coordinate physical therapy if ordered. Hysterectomy
10. Teach client about any necessary life-style
changes (special care of arm on affected side, A. General information
monthly breast self-examination on remaining 1. Total hysterectomy: removal of uterine body
breast, use of prosthesis). and cervix only
11. Encourage/arrange visit from support group 2. Subtotal hysterectomy: removal of uterine body
member. leaving cervix in place (seldom performed)
C. Medical therapy 3. Total abdominal hysterectomy with bilateral
1. Hormonal therapy: tamoxifen, anti-estrogen salpingo-oophorectomy (TAH-BSO): removal
effect. of uterine body, cervix, both ovaries, and both
2. Chemotherapy fallopian tubes
3. Radiation 4. Radical hysterectomy: removal of uterine
4. Chemotherapy and radiation used with body, cervix, connective tissue, part of vagina,
lumpectomy and pelvic lymph nodes
B. Nursing interventions
1. Institute routine pre- and post-op care.
Cancer of the Cervix 2. Assess for hemorrhage, infection, or other
A. Detected by Pap smear, followed by tissue biopsy. postsurgical complications (e.g., paralytic
1. Class Inormal pap smear ileus, thrombophlebitis, pneumonia).
2. Class IIatypical cells 3. Support woman and family through
3. Class IIImoderate dysplasia procedure, encourage expression of feelings
4. Class IVsevere dysplasia, cancer-in-situ and reactions to procedure.
5. Class VSquamous cell carcinoma, invasive 4. Explain implications of hysterectomy.
cancer a. No further menses.
B. Preinvasive conditions may be treated by b. If ovaries also removed, will have
colposcopy, cryosurgery, laser surgery, cervical menopause and may need estrogen
conization, or hysterectomy. replacement therapy.
C. Invasive conditions are treated by radium therapy c. Woman will be sterile after surgery; form
and radical hysterectomy. signed by client stating awareness of not
being able to become pregnant after
hysterectomy.
Cancer of the Uterus 5. Allow woman (and partner) to verbalize
A. May affect endometrium or fundus/corpus risk concerns about sexuality postsurgery.
increased by unopposed estrogen. 6. Provide discharge teaching.
B. Cardinal symptom: abnormal uterine bleeding,
either pre- or postmenopause Menopause
C. Diagnosis: by endometrial biopsy or fractional
curettage; cells washed from uterus under pressure The time in a womans life when menstruation ceases.
may also be used for diagnosis Fertility usually ceases, and symptoms associated
D. Usual intervention: total hysterectomy and with changing hormone levels may occur. Reactions
bilateral salpingo-oophorectomy to menopause may be influenced by culture, age at
E. Radium therapy and chemotherapy may also be used. menopause, reproductive and menstrual history, and

6
complications.

614 NCLEX-RN Review


53155_06_Ch06_p533-620.qxd 2/23/09 12:23 PM Page 615

Assessment Findings B. Other symptoms in the postmenopausal woman


include loss of height, back pain, and dowagers
A. Symptoms related to hormone changes hump.
1. Vasomotor instability (hot flashes and night C. Diagnosis by X-ray is not possible until more than
sweats) 50% of bone mass has already been lost.
2. Emotional disturbances (mood swings, D. Decreased bone porosity is inextricably linked
irritability, depression), fatigue, and headache with lowered levels of estrogen in the
B. Physical changes include postmenopausal woman. Estrogen plays a part in
1. Atrophy of genitalia the absorption of calcium and the stimulation of
2. Dyspareunia osteoclasts (new-bone-forming cells).
3. Urinary changes (frequency/stress E. Treatment includes
incontinence) 1. ERT unless contraindicated
4. Constipation 2. Supplemental calcium to slow the
5. Possibly uterine prolapse osteoporotic process (1 g taken daily at HS)
3. Increased fluid intake (23 liters/day will help
Interventions avoid formation of calculi)
4. High-calcium/high-phosphorus diet with
A. Estrogen replacement therapy (ERT); many avoidance of excess protein
controversial issues 5. Some exercise on a regular basis
1. Used to control symptoms, especially F. Prevention includes:
vasomotor instability and vaginal atrophy, and 1. Not smoking
to prevent osteoporosis 2. Regular weight-bearing exercise
2. Women with family histories of breast or 3. Good nutrition, including sources of calcium
uterine cancer, hypertension, and vitamin D
thrombophlebitis, or cardiac dysfunction are 4. Minimal use or exclusion of alcohol
not good candidates for ERT. 5. Regular physical examination
3. Women may need information about
contraception, as ovulation and ability to Cystocele/Rectocele
conceive may continue for up to 12 months
after menses cease. A. Herniations of the anterior and posterior
4. Sold under many pharmaceutical trade names; (respectively) walls of the vagina.
may be taken orally, or applied transdermally B. Usually the sequelae of childbirth injuries.
(patch). C. Herniation allows the bulging of the bladder and
5. Women who still have a uterus must take the rectum into the vagina.
progesterone to decrease risk of endometrial D. Treatment is surgical repair of these conditions:
cancer. New studies have shown long-term use anterior and posterior colporrhaphy.
carries serious risk.
B. Alternatives to ERT include: Prolapse of the Uterus
1. Vitamin E: from dietary sources and
supplements A. Usually the result of childbirth injuries or
2. Herbs: varied relief with combinations of roots relaxation of the cardinal ligaments.
and herbs, such as licorice and dandelion B. Allows the uterus to sag backward and downward
3. Other medications: Bellergal (phenobarbital, into the vagina, or outside the body completely.
ergotamine tartrate, and belladonna) C. Vaginal hysterectomy is the preferred surgical
4. Kegel exercises for genital atrophy: alternating intervention.
constriction and relaxation of pubococcygeal D. If condition does not warrant surgery, the insertion
muscles (muscles controlling the flow of of a pessary (supportive device) will help to
urine) done at least three times/day support and stabilize the uterus.
5. Vaginal lubricants for genital atrophy: water-
soluble lubricants can diminish dyspareunia
6. Maintenance of good hydration: at least 8
glasses of water/day Sample Questions
7. Good perineal hygiene
120. In collecting data for a health history of an
Complications of Menopause infertility client, which of the following findings
Osteoporosis is most important?
1. She is 5 ft 8 in tall and weighs 105 lb.
A. Increased porosity of the bone, with increased 2. She has never used any form of
incidence of spontaneous fractures.

6
contraception.

MATERNITY AND FEMALE REPRODUCTIVE NURSING 615


53155_06_Ch06_p533-620.qxd 2/23/09 12:23 PM Page 616

3. She has been married for 3 years. 126. A woman has advanced cancer of the breast. She
4. She has no brothers or sisters. is admitted to the medical unit for nutritional
evaluation. She weighs 101 lb and is 5 ft 8 in
121. What teaching should be included to a woman tall. She is started on leucovorin (Wellcovorin).
who has just been fitted with her first Which of the following would not be included
diaphragm? in the assessment of her nutritional health?
1. Specific amount of spermicide to be used 1. A diet history.
with diaphragm. 2. Anthropometric measurements.
2. Insertion at least 8 hours before intercourse. 3. Food preferences.
3. Specific cleaning techniques. 4. Serum protein studies.
4. Storage in the refrigerator.
127. What is the nurses primary role in relation to
122. A postmenopausal woman takes calcium sexually transmitted disease?
supplements on a daily basis. What instruction 1. Case reporting.
should be given the woman to reduce the danger 2. Sexual counseling.
of renal calculi?
3. Diagnosis and treatment.
1. Chewing her calcium tablets rather than
4. Recognizing symptoms and teaching clients.
swallowing them whole.
2. Swallowing her calcium tablets with 128. A female teen visits the local health clinic
cranberry juice. because her boyfriend was recently diagnosed as
3. Eliminating other sources of calcium from having gonorrhea. She asks the nurse about
her diet. possible consequences if she went without
4. Drinking 23 quarts of water daily. treatment. What would be an appropriate
answer?
123. A postmenopausal woman is having a routine 1. Disseminated systemic infections.
physical exam. Which of the following 2. Minor problems such as skin rashes.
assessments would yield critical information as
3. The need for delivery by cesarean section.
to her postmenopausal status?
4. Sterility, birth defects, and miscarriage.
1. Asking about weight loss of more than 5 lb in
the last year. 129. Several adolescent girls are discussing sexual
2. Asking about her nightly sleep patterns. activity with the nurse at the STD clinic. Which
3. Asking about her cultural background. comment indicates to the nurse that the client
4. Asking about her last pregnancy. has not understood the teaching regarding safe
sexual practices?
124. A woman is admitted to the hospital for a 1. We use KY jelly on condoms.
panhysterectomy. Which nursing strategy should 2. I douche after intercourse.
be included in the nursing care plan to meet her
3. I shower with my boyfriend.
body-image perception changes?
4. We use condoms and birth control pills.
1. Allowing her time to work out her feelings on
her own. 130. When discussing safe sex, which information
2. Discouraging fears about weight gain. about the use of condoms would be most
3. Helping her verbalize her concerns about her helpful?
femininity. 1. Lambskin condoms do not interfere with
4. Insisting that she look at the scar. sensation.
2. Latex condoms help prevent the transmission
125. Following a panhysterectomy, the woman is of germs.
placed on estrogen replacement therapy. What
3. Condoms are often inconvenient and
is the primary purpose of estrogen replacement
unnecessary.
therapy following surgical menopause?
4. Condoms prevent STDs but they are a poor
1. Arthritis.
choice for birth control.
2. Pregnancy.
3. Breast cancer. 131. A couple have come to your clinic because they
4. Vasomotor instability. have not been able to achieve a pregnancy after
trying for 2 years without using any form of birth

6
control. Which of the following tests could

616 NCLEX-RN Review


53155_06_Ch06_p533-620.qxd 2/23/09 12:23 PM Page 617

determine that the woman is ovulating 136. A woman comes to the office complaining of the
regularly? following symptoms: fatigue, weight gain, pelvic
1. Hysterosalpingogram. pain related to menstruation, heartburn, and
2. Serial basal body temperature graph. constipation. Which of the above symptoms
might indicate a diagnosis of endometriosis?
3. Postcoital test.
1. Weight gain and fatigue.
4. Semen analysis.
2. Heartburn.
132. A woman is preparing to take Clomid to induce 3. Constipation.
ovulation so she can have an in vitro 4. Pelvic pain related to menstruation.
fertilization. She asks if she should expect any
side effects from the drug. Your best answer 137. A woman has been diagnosed with Candida
should include which of the following? albicans. Which of the following types of vaginal
1. Weight gain with increased appetite and discharge would you expect to find?
constipation. 1. Thin, greenish yellow with a foul odor.
2. Tingling of the hands and feet. 2. Either a yellowish discharge or none at all.
3. Alopecia (hair loss). 3. Thick and white, like cottage cheese.
4. Stuffy nose and cold-like symptoms. 4. Thin, grayish white with a fishy odor.

133. A couple have been using a diaphragm for 138. A woman has just been diagnosed with genital
contraception. Which of the following herpes for the first time. You can expect which
statements indicates they are using it correctly? of the following treatments to be part of her plan
1. We use K-Y jelly around the rim to help of care?
with insertion. 1. Vaginal soaks with saline to keep the area
2. I wash the diaphragm each time and hold it moist.
up to the light to look for any holes. 2. Acyclovir 200 mg 5 times daily for 710
3. I take the diaphragm out about 1 hour after days.
intercourse because it feels funny. 3. Ceftriaxone 125 mg IM times 1 dose.
4. I douche right away after intercourse. 4. Topical application of podophyllin to the
lesions.
134. A 25-year-old wishes to take oral contraceptives.
When taking her history, which of the following 139. A woman is 10 weeks pregnant and tested
questions would determine whether she is an positive for syphilis but has no symptoms. She
appropriate candidate for this form of birth control? asks you why she needs to be treated since she
1. Do you currently smoke cigarettes and, if so, feels fine? Your best response to her would
how many? include which of the following?
2. Have you had any recent weight gain or loss? 1. Syphilis can be transmitted to the baby and
3. Do you douche regularly after intercourse? may cause it to die before birth if you are not
treated.
4. Is there any family history of kidney or
gallbladder disease? 2. If you do not receive treatment before the
baby is born, your baby could become blind.
135. A woman who is 18 weeks pregnant is 3. If syphilis is untreated, the baby may be
scheduled for a saline injection to terminate her mentally retarded at birth.
pregnancy. She asks the nurse what she should 4. Syphilis may cause your baby to have a
expect. What would be the nurses best answer? heart problem when it is born.
1. Contractions will begin immediately after
the instillation of saline and will be mild. 140. A woman has been diagnosed with fibrocystic
2. An amniocentesis will be performed with breast disease. Which of the following should be
amniotic fluid removal and saline included in the teaching plan for her?
replacement. 1. Limiting breast self-examinations to every
3. A tube will be inserted through the cervix 3 months because it may be painful.
and warm saline will be administered by 2. Wearing a bra as little as possible because
continuous drip. pressure on the breast may be painful.
4. The baby will be born alive but will die a 3. Limiting caffeine and salt intake.
short time later. 4. Using heat to the tender areas of the breast.

MATERNITY AND FEMALE REPRODUCTIVE NURSING

6 617
53155_06_Ch06_p533-620.qxd 2/23/09 12:23 PM Page 618

141. The local YWCA is having a series of seminars health issues. In discussing cancer of the
on health-related topics. You are invited to cervix, which of the following would be
discuss breast self-examination (BSE) with the accurate?
group. Which of the following would be 1. This cancer is very rapid growing, so early
appropriate to teach regarding when BSE should detection is difficult to achieve.
be performed by women of reproductive age? 2. A cervical biopsy is the screening test
1. At the end of each menstrual cycle. of choice for early detection of cervical
2. At the beginning of each menstrual cycle. cancer.
3. About 710 days after the beginning of each 3. All women have an equal chance to develop
menstrual cycle. cervical cancer because there are no high risk
4. About 710 days before the end of the factors.
menstrual cycle. 4. An annual Pap smear may detect cervical
dysplasia, a frequent precursor of cervical
142. You have been discussing breast self- cancer.
examination (BSE) with a woman. Which of the
following statements would best indicate she is 146. The nurse is talking to a woman who has been
doing BSE correctly? diagnosed with cancer of the ovary. She asks
1. I begin to examine my breasts by placing the you what she could have done so that the
palm of my right hand on the nipple of the cancer would have been found earlier. The best
left breast. response should include which of the
2. I dont like to press very hard because my following?
breasts are very tender. 1. She should have had more frequent, twice a
3. I use the tips of the middle three fingers of year, Pap smears.
each hand to feel each breast. 2. A yearly complete blood count (CBC) could
4. I feel for lumps in my breasts standing in have provided valuable clues to detect
front of a mirror. ovarian cancer.
3. Detection of ovarian cancer is easier if a
143. A woman had a simple mastectomy this yearly proctoscopy is done.
morning. Which of the following should be 4. There is little more she could have done for
included in your plan for her care? earlier detection.
1. Complete bed rest for the first 24 hours.
2. NPO with IV fluids for the first 48 hours. 147. The nurse is caring for a woman who has had a
vaginal hysterectomy and has an indwelling
3. Positioning on the operative side for the first
Foley catheter. After removal of the catheter, she
24 hours.
is unable to void and has little sensation of
4. Keep patient-controlled anesthesia (PCA) bladder fullness. She is also constipated and is
controller within easy reach for the first experiencing some perineal pain. Using a 2-part
48 hours. nursing diagnosis statement, which of the
following would be appropriately paired with a
144. The nurse is teaching a woman who had a
diagnosis of altered urinary elimination?
simple mastectomy. Which of the following
would be appropriate to tell her? 1. Infection as evidenced by inability to void
with frequency and urgency.
1. She should wait to be fitted for a permanent
prosthesis until the wound is completely 2. Retention as evidenced by inability to void
healed. and urinary distention.
2. Because she had a simple mastectomy, she 3. Gastrointestinal functioning as evidenced by
will probably not feel the need to attend inability to void and constipation.
Reach for Recovery meetings. 4. Dysuria as evidenced by inability to void and
3. She will have very little pain and the incision loss of bladder sensation.
will heal very quickly.
148. A 42-year-old had a simple vaginal
4. She should refrain from seeking male hysterectomy without oophorectomy due to
companionship because she will be seen as uterine fibroids. You have completed your
less than a woman. discharge teaching and she is preparing to go
home. Which of the following statements
145. A group of women have gathered at the local
indicates she understands the physical changes
library for a series of seminars about womens

6
she will experience?

618 NCLEX-RN Review


53155_06_Ch06_p533-620.qxd 2/23/09 12:23 PM Page 619

1. I hope my husband will still love me since engage in strenuous physical activity over
we cant have sexual intercourse anymore. prolonged periods of time may experience
2. I was hoping to stop having periods, but I changes in their menstrual cycle and their
guess that will need to wait a few more years. fertility.
3. It will be so nice to not need to use birth
121. 3. The client must be instructed to clean the
control anymore.
diaphragm with mild, plain soap, and warm
4. I just dont think I will ever feel feminine water; dust it lightly with cornstarch; and store
again since I can no longer experience orgasm. it in a cool, dry place. She should also be
instructed to check it regularly for perforations
149. The nurse has been discussing menopause with
or defects.
a 50-year-old woman who is experiencing some
bodily changes indicative of the perimenopausal 122. 4. The ingestion of sufficient amounts of water
period. Which of the following statements by a woman taking calcium supplements is
indicates the client understands what is important to prevent renal calculi.
happening to her body?
1. Even though I am only having periods every 123. 2. Postmenopausal women who are experiencing
few months, I should continue to use birth vasomotor instability may have night sweats and
control until at least 6 months after my interrupted sleep.
periods have stopped.
124. 3. Loss of the organs of reproduction are often
2. I am very upset to think that I will continue
equated with a loss of femininity. The client
to have these hot flashes for the rest of my
should be encouraged to explore her feelings
life.
and to adapt to body changes.
3. Now that I am an old woman, I guess Ill be
sick most of the time, so I should plan to 125. 4. Low-dose estrogen therapy is used to relieve
move to a retirement home. the vasomotor symptoms of menopausal women.
4. I may continue to bleed on and off
throughout the next 25 years. 126. 3. Food preferences are considered when
planning a program to meet the clients
150. A 55-year-old woman who has ceased having nutritional requirement after the nutritional
menses has a family history of osteoporosis and assessment has been completed.
increasing cholesterol levels over the past
several years. Hormone replacement therapy 127. 4. Early recognition of sexually transmitted
(HRT) has been prescribed with estrogen and diseases (STDs) reduces the risk of serious
progesterone. She asks you why she should take sequelae. The primary role of the nurse is to
the pills since she feels quite well. The nurses recognize symptoms of STDs in order to teach
answer would be: clients how to comply with treatment and how
to prevent reinfection.
1. HRT is thought to help protect women from
heart disease and osteoporosis. 128. 4. Lack of treatment or inadequate treatment of
2. HRT will help to reestablish the menstrual gonorrhea can result in serious sequelae such as
cycle, thus providing natural protection sterility, birth defects, and miscarriage. These
against heart disease and osteoporosis. are the most common complications and the
3. Even though she feels well now, she will ones most important to discuss.
soon begin having major health problems
and HRT will protect her against those 129. 2. Douching does not protect against infection
problems. and damages the natural protective barriers.
4. She will be protected from breast cancer by 130. 2. Condoms can prevent the transmission of
HRT. many STDs. This information is very important
to give.

131. 2. Serial basal body temperature graphs are a


Answers and Rationales baseline for determining when ovulation has
taken place during a menstrual cycle. If
120. 1. Because of the complex interaction between ovulation has occurred, the temperature will be
the hypothalamus, the ovary, and the amount of higher the second half of the cycle and lower the
body fat, women who are underweight or who first half.

MATERNITY AND FEMALE REPRODUCTIVE NURSING

6 619
53155_06_Ch06_p533-620.qxd 2/23/09 12:23 PM Page 620

132. 1. Weight gain associated with increased 145. 4. Cervical dysplasia is frequently a forerunner
appetite and constipation are fairly common of cervical cancer and is readily detected by Pap
side effects of Clomid. smear; thus follow-up Pap smears allow for early
detection and treatment of cervical cancer.
133. 2. The diaphragm should be washed and dried
and inspected for holes before being put away. 146. 4. Detection of ovarian cancer is very difficult
because it gives only vague, subtle symptoms
134. 1. Cigarette smoking significantly increases a and there are no diagnostic screening tests.
womans risk for circulatory complications and
may contraindicate oral contraceptive use. 147. 2. Retention of urine is common following
vaginal hysterectomy due to stretching of
135. 2. The procedure begins with an amniocentesis musculature and proximity of the surgery to the
where amniotic fluid is withdrawn and replaced bladder and its enervation.
with saline solution.
148. 3. After the loss of the uterus, pregnancy is
136. 4. Pelvic pain related to menstruation is the unachievable and birth control is not needed
most common symptom of endometriosis. The even if the ovaries remain.
pain usually ends following cessation of menses.
149. 1. Even though ovulation is erratic and many
137. 3. Thick, white cottage cheese-like discharge is periods are anovulatory, birth control should be
consistent with Candida albicans. continued for at least 6 months after the last
menses.
138. 2. This is the correct drug and dosage for an
initial infection of genital herpes. 150. 1. HRT appears to help protect many women
from heart disease and osteoporosis if used with
139. 1. Syphilis is associated with stillbirth,
exercise and calcium supplements.
premature birth, and neonatal death.

140. 3. Most women benefit from caffeine and salt


restriction because this reduces fluid retention REFERENCES AND SUGGESTED READINGS
and increases comfort.
Boyle, J. S., & Andrews, M. M. (2007). Transcultural concepts in
141. 3. The breasts are softer, less tender, and nursing care (5th ed.). Philadelphia: Lippincott-Raven.
Littleton, L., & Engebretson, J. (2005). Maternity nursing care (1st
swelling is reduced about a week after the
ed.). Clifton Park, NY: Delmar Cengage Learning.
beginning of the menstrual cycle. Lowdermilk, D., & Perry, S. (2007). Maternity and womens
health care (9th ed.). St. Louis, MO: Mosby.
142. 3. The ends of the three middle fingers are the Munoz, C., & Luckmann, J. (2004). Transcultural communication
most sensitive and should be used for BSE. in nursing (2nd ed.). Clifton Park, NY: Delmar Cengage
Learning.
143. 4. Adequate pain relief is important and the use Olds, S., London, M., Ladewig, P., & Davidson, M. (2004).
of PCA allows the client to control her own pain Maternal-newborn nursing and womens health care (7th ed.).
relief. Upper Saddle River, NJ: Prentice Hall.
Pillitteri, A. (2006). Maternal and child health nursing (5th ed.).
144. 1. The incisional site may change with time and Philadelphia: Lippincott.
healing, so a permanent prosthesis should be White, L. (2004). Foundations of maternal & pediatric nursing
(2nd ed.). Clifton Park, NY: Delmar Cengage Learning.
purchased only after complete healing has Wong, D., Perry, S., & Hockenberry, M. (2006). Maternal child
occurred. nursing care (3rd ed.). St. Louis, MO Mosby.

6 620 NCLEX-RN Review


53155_07_Ch07_p621-684.qxd 2/27/09 6:29 PM Page 621

U N I T 7

P S YC H I AT R I C -
M E N TA L H E A LT H
NURSING

An important aspect of professional nursing is the use of UNIT OUTLINE


therapeutic intervention for clients who are experiencing 623 Overview of Psychiatric-
emotional distress. A client does not have to have a psychiatric Mental Health Nursing
diagnosis to be in emotional distress, and often clients and their 638 Psychiatric Disorders
(DSM-IV-TR)
families may respond to illness or injury with anxiety and fear
that can be manifested in a variety of behaviors. The principles of 674 Psychologic Aspects of
Physical Illness
psychiatric-mental health nursing and therapeutic interventions
can be applied to any client, family, or group in need.

To plan appropriate interventions, nurses need to have an


understanding and knowledge of personality development and
other theories to analyze behaviors of the client or others.
Theories, principles, the nursing process, and treatment
modalities are the science of psychiatric-mental health nursing.

The manner the nurse selects to use the science of mental health
nursing is based in part on that nurses personal attributes.
Personal experiences, the ability to implement principles and
theories, and the willingness to use therapeutic communication
constitute the art of psychiatric-mental health nursing. This
creative aspect of each nurse is the therapeutic use of self
involved in planning and implementing effective nursing
interventions for dealing with clients who are experiencing
emotional distress.

621
53155_07_Ch07_p621-684.qxd 2/27/09 6:29 PM Page 622

Implementing the art of psychiatric-mental health nursing


is an important way to convey to clients the caring aspect
of nursing. Perceiving clients concerns and responding
therapeutically will encourage clients to share more
information with the nurse. Awareness of the clients
attitudes, values, and fears will enable the nurse to
individualize client care. Physical, psychologic, social, and
spiritual needs should be the concern of a nurse who wants
to provide holistic care. Nurses can determine specific client
needs by assessing verbal and nonverbal behaviors.
Application of the nursing process to meet client needs
will ensure comprehensive nursing care.

The following principles of psychiatric-mental health


nursing help form the basis of the therapeutic use of self:

Be aware of your own feelings and responses


Maintain objectivity while being aware of your own needs
Use empathy (recognizing/identifying somewhat with
clients emotions to understand behavior), not sympathy
(close identification/duplication of clients emotions)
Focus on the needs of the client, not on your own needs;
be consistent and trustworthy
Accept clients as they are; be nonjudgmental
Recognize that emotions influence behaviors
Observe a clients behaviors to analyze needs/problems
Accept clients needs to use defenses/behaviors to deal
with emotional distress
Accept clients negative emotions
Avoid verbal reprimands, physical force, giving advice, or
imposing your own values on clients
Avoid intimate relationships while maintaining a caring
attitude
Assess clients in the context of their social/cultural group
Recognize that client communication patterns (verbal and
nonverbal) vary with different cultural groups
Teach/explain on clients level of capability
Treat clients with respect, caring, and compassion

Asking yourself, What is this clients need at this time? can


assist you in determining the best response to questions.

7 622 NCLEX-RN Review


53155_07_Ch07_p621-684.qxd 2/27/09 6:29 PM Page 623

Overview of Psychiatric-Mental Health Nursing

THEORETICAL BASIS G. Freuds theory of personality


1. Id: present at birth; instinctual drive for
Medical-Biologic Model pleasure and immediate gratification,
unconscious. Libido is the sexual and/or
A. Emotional distress is viewed as illness. aggressive energy (drive). Operates on pleasure
B. Symptoms can be classified to determine a principle to reduce tension or discomfort
psychiatric diagnosis. (pain). Uses primary process thinking by
C. DSM-IV-TR* imagining objects to satisfy needs
1. Description of disorders (hallucinating).
2. Criteria (behaviors) that must be met for 2. Ego: develops as sense of self that is distinct
diagnosis to be made from world of reality; conscious, preconscious,
3. Axis: the dimensions and factors included and unconscious. Operates on reality principle
when assessing a client with a mental disorder which determines whether the perception has
a. I and II: clinical syndromes (e.g., bipolar, a basis in reality or is imagined. Uses
antisocial personality, mental retardation) secondary process thinking by judging reality
b. III: physical disorders and symptoms and solving problems.
(e.g., cystic fibrosis, hypertension) a. Functions of the ego
c. IV: psychosocial and environmental 1) Control and regulate instinctual drives
problems: acute and long-term severity of 2) Mediate between id drives and
stressors demands of reality; id drives versus
d. V: functioning of client, rating of superego restrictions
symptoms and their effect on activities of 3) Reality testing: evaluate and judge
daily living (ADLs) or violence to external world
self/others 4) Store up experiences in memory
D. Diagnosed psychiatric illnesses are within the 5) Direct motor activity and actions
realm of medical practice and have a particular 6) Solve problems
course, prognosis, and treatment regimen. 7) Use defense mechanisms to protect
E. Treatment can include psychotropic drugs, self
electroconvulsive therapy (ECT), hospitalization, b. Levels of awareness
and psychotherapy. 1) Preconscious: knowledge not readily
F. There is no proven cause, but theory is that available to conscious awareness but
biochemical/genetic factors play a part in the can be brought to awareness with
development of mental illness. Theories with effort (e.g., recalling name of a
schizophrenia and affective disorders include: character in a book)
1. Genetic: increased risk when close relative 2) Unconscious: knowledge that cannot
(e.g., parent, sibling) has disorder be brought into conscious awareness
2. Possible link to neurotransmitter activity without interventions such as
psychoanalysis, hypnotism, or drugs
Psychodynamic/Psychoanalytic 3) Conscious: aware of own thoughts and
perceptions of reality
Model (Freud) 3. Superego: develops as person unconsciously
A. Instincts (drives) produce energy. incorporates standards and restrictions from
B. There are genetically determined drives for sex parents to guide behaviors, thoughts, and
and aggression. feelings. Conscious awareness of
C. Human behavior is determined by past acceptable/unacceptable thoughts, feelings,
experiences and responses. and actions is conscience.
D. All behavior has meaning and can be understood. H. Freuds psychosexual developmental stages
E. Emotionally painful experiences/anxiety motivate 1. Oral
behavior. a. 018 months
F. Client can change behavior and responses when b. Pleasure and gratification through
made aware of the reasons for them. mouth
c. Behaviors: dependency, eating, crying,
*American Psychiatric Association. (2000). Diagnostic and Statistical

7
Manual of Mental Disorders (4th ed.), Text Revision. biting

PSYCHIATRIC-MENTAL HEALTH NURSING 623


53155_07_Ch07_p621-684.qxd 2/27/09 6:29 PM Page 624

d. Distinguishes between self and mother lack of self-confidence, overrestricting


e. Develops body image, aggressive drives actions
2. Anal 4. Industry versus inferiority
a. 18 months3 years a. 612 years
b. Pleasure through elimination or retention b. Creative; develop sense of competency
of feces versus sense of inadequacy
c. Behaviors: control of holding on or 5. Identity versus role diffusion
letting go a. 1220 years
d. Develops concept of power, punishment, b. Develop sense of self; preparation,
ambivalence, concern with cleanliness or planning for adult roles versus doubts
being dirty relating to sexual identity,
3. Phallic/Oedipal occupation/career
a. 36 years 6. Intimacy versus isolation
b. Pleasure through genitals a. 1825 years
c. Behaviors: touching of genitals, erotic b. Develop intimate relationship with
attachment to parent of opposite sex another; commitment to career versus
d. Develops fear of punishment by parent of avoidance of choices in relationships,
same sex, guilt, sexual identity work, or lifestyle
4. Latency 7. Generativity versus stagnation
a. 612 years a. 2145 years
b. Energy used to gain new skills in social b. Productive; use of energies to guide next
relationships and knowledge generation versus lack of interests, concern
c. Behaviors: sense of industry and mastery with own needs
d. Learns control over aggressive, destructive 8. Integrity versus despair
impulses a. 45 years to end of life
e. Acquires friends b. Relationships extended, belief that own
5. Genital life has been worthwhile versus lack of
a. 1220 years meaning of ones life, fear of death
b. Sexual pleasure through genitals
c. Behaviors: becomes independent of Interpersonal Model (Sullivan)
parents, responsible for self
d. Develops sexual identity, ability to love A. Behavior motivated by need to avoid anxiety and
and work satisfy needs
B. Sullivans developmental tasks
1. Infancy
Psychosocial Model (Erikson) a. 018 months
A. Emphasis on psychosocial rather than b. Others will satisfy needs
psychosexual development 2. Childhood
B. Developmental stages have goals (tasks) a. 18 months6 years
C. Challenge in each stage is to resolve conflict b. Learn to delay need gratification
(e.g., trust versus mistrust) 3. Juvenile
D. Resolution of conflict prepares individual for next a. 69 years
developmental stage b. Learn to relate to peers
E. Personality develops according to biologic, 4. Preadolescence
psychologic, and social influences a. 912 years
F. Eriksons psychosocial development tasks b. Learn to relate to friends of same sex
1. Trust versus mistrust 5. Early adolescence
a. 018 months a. 1214 years
b. Learn to trust others and self versus b. Learn independence and how to relate to
withdrawal, estrangement opposite sex
2. Autonomy versus shame and doubt 6. Late adolescence
a. 18 months3 years a. 1421 years
b. Learn self-control and the degree to which b. Develop intimate relationship with person
one has control over the environment of opposite sex
versus compulsive compliance or
defiance Therapeutic Nurse-Client
3. Initiative versus guilt
a. 35 years Relationship (Peplau)
b. Learn to influence environment, evaluate A. Based on Sullivans interpersonal model

7
own behavior versus fear of doing wrong, B. Therapeutic relationship is between nurse (helper)
and client (recipient of care). The goal is to work

624 NCLEX-RN Review


53155_07_Ch07_p621-684.qxd 2/27/09 6:29 PM Page 625

together to assist client to grow and to resolve relationship. Countertransference is a normal


problems. occurrence, but must be recognized so that
C. Differs from social relationship in which both supervision or consultation can keep it from
parties form alliance for mutual benefit. undermining the nurse-client relationship. For
D. Therapeutic use of self example: Nurse is sarcastic and judgmental to
1. Focus is on client needs but nurse is also client who has history of drug abuse. Client
aware of own needs. represents (unconsciously to nurse) the nurses
2. Self-awareness enables nurse to avoid having brother who has abused drugs.
own needs influence perception of client. 3. Interventions
3. Determine what client/family needs are at the a. Reflect on reasons for behaviors of client or
time. nurse.
E. Three phases of nurse-client relationship b. Establish therapeutic goals for this
1. Orientation relationship.
a. Nurse explains relationship to client, c. If unable to control these occurrences,
defines both nurses and clients roles. transfer client to another nurse.
b. Nurse determines what client expects from
the relationship and what can be done for
the client.
Human Motivation/Need Model
c. Nurse contracts with client about when (Maslow)
and where future meetings will take
A. Hierarchy of needs in order of importance
place.
1. Physiologic: oxygen, food, water, sleep, sex
d. Nurse assesses client and develops a plan
2. Safety: security, protection, freedom from
of care based on appropriate nursing
anxiety
diagnoses.
3. Love and belonging: freedom from
e. Limits/termination of relationship are
loneliness/alienation
introduced (e.g., We will be meeting for
4. Esteem and recognition: freedom from sense of
30 minutes every morning while you are in
worthlessness, inferiority, and helplessness
the hospital.).
5. Self-actualization: aesthetic needs, self-
2. Working phase
fulfillment, creativity, spirituality
a. Clients problems and needs are identified
B. Primary needs (oxygen, fluids) need to be met
and explored as nurse and client develop
prior to dealing with higher-level needs (esteem,
mutual acceptance.
recognition).
b. Clients dysfunctional symptoms, feelings,
C. Focus on provision of positive aspects such as
or interpersonal relationships are identified.
feeling safe, having someone care, affiliation
c. Therapeutic techniques are employed to
reduce anxiety and to promote positive
change and independence. Behavioral Model (Pavlov, Skinner)
d. Goals are evaluated as therapeutic work
A. Behavior is learned and retained by positive
proceeds, and changed as determined by
reinforcement (e.g., more studying produces higher
clients progress.
grades).
3. Termination
B. Motivation for behavior is not considered.
a. Relationship and growth in nurse and
C. Behaviors that are not adequate can be replaced by
client are summarized.
more adaptive behaviors.
b. Client may become anxious and react with
increased dependence, hostility, or
withdrawal. Community Mental Health Model
c. These reactions are discussed with client.
A. Emotional distress stems from personal and social
d. Feelings of nurse and client concerning
factors
termination should be discussed in context
1. Family problems (e.g., divorce, single
of finiteness of relationship.
parenthood)
F. Transference and countertransference
2. Social factors (e.g., unemployment, lack of
1. Transference: occurs when client transfers
support groups, changing mores)
conflicts/feelings from past to the nurse. For
B. Health care a right
example: Client becomes overly dependent,
C. Decreased need for hospitalization, increased
clinging to nurse who represents
community care
(unconsciously to client) the nurturing desires
D. Collaboration of social and health care services
from own mother.
E. Comprehensive services
2. Countertransference: occurs when nurse
1. Emergency care
responds to client emotionally, as if in a
2. Inpatient/outpatient services

7
personal, not professional/therapeutic,

PSYCHIATRIC-MENTAL HEALTH NURSING 625


53155_07_Ch07_p621-684.qxd 2/27/09 6:29 PM Page 626

3. Substance-abuse treatment joyful mood may be expressed by smiling or happy


4. Transitional living arrangements (temporary affect.
residence instead of inpatient care) A. Appropriateness
5. Consultation and education to increase B. Description: flat, sad, smiling, serious
knowledge of mental health C. Stability
F. Prevention D. Specific feelings and moods
1. Primary prevention
a. Minimize development of serious
emotional distress: promote mental health,
Cognitive Assessment
identify persons at risk. Evaluation of thought, sensorium, intelligence
b. Anticipate problems such as A. Intellectual performance
developmental crises (e.g., birth of first 1. Orientation to person, place, and time
child, midlife crisis, death of spouse). 2. Attention and concentration
2. Secondary prevention: early case finding and 3. Knowledge/educational level
treatment (drug therapy, outpatient, short-term 4. Memory: short and long term
hospitalization). 5. Judgment
3. Tertiary prevention: restore client to optimal 6. Insight into illness
functioning; facilitate return of client to home 7. Ability to use abstraction
and community by use of social agencies. B. Speech
1. Amount, volume, clarity
2. Characteristics: pressured, slow or fast, dull or
NURSING PROCESS lively
3. Specific aberrations, i.e., echolalia (imitating
A. Applies to all clients, not only to those with and repeating anothers words or phrases) or
psychiatric diagnosis; incorporates holism. neologisms (making up of own words that
B. Utilized in a unique manner for psychosocial have special meaning to client).
assessment. C. Thoughts
C. Sets goals (with client, whenever possible) that can 1. Content and clarity
be measured in behavioral terms (e.g., client will 2. Characteristics: spontaneity, speed, loose
dress self and eat breakfast before 9 A.M.). associations, blocked, flight of ideas,
D. Uses principles of therapeutic communication for repetitions
interventions.
E. Evaluates whether, how well goals were met.
Social/Cultural Considerations
A. Age: assess for developmental tasks and
Physical Assessment developmental crises, age-related problems.
A. Subjective reporting of health history 1. 018 months: development of trust and sense
B. Objective data (general status and appearance) of self, dependency
1. Age: clients appearance in relation to 2. 18 months3 years: development of autonomy
chronologic age and beginning self-reliance, toilet training
2. Attire: appropriateness of clothing to 3. 36 years: development of sexual identity,
age/situation relationships with peers, adjustment to school
3. Hygiene: cleanliness and grooming, or lack 4. 612 years: mastery of skills, beginning self-
thereof esteem, identification with others outside
4. Physical health: weight, physical distress family, social relationships
5. Psychomotor: posture, movement, activity 5. 1218 years: sense of self solidifies, separation
level and individuation often follow some
6. Sleep and rest disorganization and rebellion, substance abuse
C. Neurologic assessment/level of consciousness 6. 1825 years: identification with peer group,
setting of personal and career goals to master
future
Mental Status Assessment 7. 2538 years: take place in adult world,
Emotional Status Assessment commitments made relating to career,
marriage, parenthood
Observation of mood (prolonged emotion) and affect 8. 3865 years: review of past accomplishments;
(physical manifestations of mood). That is, sad mood may set new and reasonable goals; midlife
may be evidenced by crying or downcast appearance; crises when present achievements have
not met goals set in earlier stages of
development

7 626 NCLEX-RN Review


53155_07_Ch07_p621-684.qxd 2/27/09 6:29 PM Page 627

9. 65/70 to death: loss of friends/spouse, PLANNING AND


retirement, loss of some social/physical
functions IMPLEMENTATION
B. Family/community relationships
1. Role of client in family Goals
2. Family harmony, family support for or
dependency on client A. Client will:
3. Clients perception of family 1. Participate in treatment program.
4. Availability of community support groups to 2. Be oriented to time, place, and person and
client (include government social agencies; exhibit reality-based behavior.
religious, ethnic, and volunteer agencies) 3. Recognize reasons for behavior and develop
C. Socioeconomic group/education alternative coping mechanisms.
1. Factors that relate to how client is approached 4. Maintain or improve self-care activities.
and how client perceives own present state 5. Be protected from harmful behaviors.
2. Determination of level of teaching and need B. There will be mutual agreement of nurse and
for social services client whenever possible.
D. Cultural/spiritual background C. Short-term goals are set for immediate problems;
1. Assess behaviors in context of clients culture. they should be feasible and within clients
2. Avoid stereotyping persons as having capabilities.
attributes of their culture/subculture. D. Long-term goals are related to discharge planning
3. Note clients religious/philosophic beliefs. and prevention of recurrence or exacerbation of
symptoms.

ANALYSIS Interventions
Select nursing diagnoses based on collected data. The nurse will use therapeutic intervention and the
Decide which is most important. Specific nursing nurse-client relationship to help the client achieve the
diagnoses will be given when discussing particular goals of therapy. Interventions must be geared to the
disorders, but those nursing diagnoses generally level of the clients capability and must relate to the
appropriate to the client with psychiatric-mental specific problems identified for the individual client,
health disorders include: family, or group.
A. Anxiety
B. Chronic sorrow Therapeutic Communication
C. Decisional conflict
D. Defensive coping A. Facilitative: use the following approaches to
E. Deficient knowledge intervene therapeutically
F. Disturbed body image 1. Silence: client able to think about
G. Disturbed sleep pattern self/problems; does not feel pressure or
H. Disturbed through processes obligation to speak.
I. Dysfunctional family processes 2. Offering self: offer to provide comfort to client
J. Fatigue by presence (Nurse: Ill sit with you. Ill
K. Fear walk with you.).
L. Hopelessness 3. Accepting: indicate nonjudgmental acceptance
M. Impaired adjustment of client and his perceptions by nodding and
N. Impaired social interaction following what client says.
O. Impaired verbal communication 4. Giving recognition: indicate to client your
P. Ineffective coping awareness of him and his behaviors (Nurse:
Q. Ineffective denial Good morning, John. You have combed your
R. Ineffective role performance hair this morning.).
S. Ineffective therapeutic regimen management 5. Making observations: verbalize what you
T. Low self-esteem perceive (Nurse: I notice that you cant seem
U. Noncompliance to sit still.).
V. Powerlessness 6. Encouraging description: ask client to
W. Rape-trauma syndrome verbalize his perception (Nurse: Tell me
X. Risk for injury when you need to get up and walk around.
Y. Risk for violence What is happening to you now?).
Z. Risk-prone health behavior 7. Using broad openings: encourage client to
AA. Self-mutilation introduce topic of conversation (Nurse:
BB. Social isolation Where shall we begin today? What are you

7
CC. Spiritual distress thinking about?).
DD. Stress overload

PSYCHIATRIC-MENTAL HEALTH NURSING 627


53155_07_Ch07_p621-684.qxd 2/27/09 6:29 PM Page 628

8. Offering general leads: encourage client to 6. Defending: protecting person or institutions


continue discussing topic (Nurse: And then? (Client: Ms. Jones is a rotten nurse. Nurse:
Tell me more about that.). Ms. Jones is one of our best nurses.).
9. Reflecting: direct clients questions/statements 7. Approving: giving approval to clients
back to encourage expression of ideas and behavior or opinion (Client: Im going to
feelings (Client: Do you think I should call my change my attitude. Nurse: Thats good.).
father? Nurse: What do you want to do?). 8. Disapproving: telling client certain behavior or
10. Restating: repeat what client has said (Client: opinions do not meet your approval (Client:
I dont want to take the medicine. Nurse: I am going to sign myself out of here. Nurse:
You dont want to take this medication?). Id rather you wouldnt do that.).
11. Focusing: encourage client to stay on topic/point 9. Agreeing: letting client know that you think
(Nurse: You were talking about . . .). and feel alike; nurse verbalizes agreement.
12. Exploring: encourage client to express feelings 10. Disagreeing: letting client know that you do
or ideas in more depth (Nurse: Tell me more not agree; telling client that you do not believe
about . . . How did you respond to . . . ?). he is right.
13. Clarification: encourage client to make idea or 11. Probing: questioning client about a topic he
feeling more explicit, understandable (Nurse: has indicated he does not want to discuss.
I dont understand what you mean. Could 12. Denial: refusing to recognize clients perception
you explain it to me?). (Client: I am a hopeless case. Nurse: You are
14. Presenting reality: report events/situations as not hopeless. There is always hope.).
they really are (Client: I dont get to talk to my 13. Changing topic: letting client know you do not
doctor. Nurse: I saw your doctor talking to want to discuss a problem by introducing a
you this morning.). new topic (Client: I am a hopeless case.
15. Translating into feelings: encourage client to Nurse: Its time to fill out your menu.).
verbalize feelings expressed in another way
(Client: I will never get better. Nurse: You Therapeutic Groups
sound rather hopeless and helpless.).
Table 7-1 lists types of therapeutic groups.
16. Suggesting collaboration: offer to work with
A. Groups of clients meet with one or more therapists.
client toward goal (Client: I fail at everything
They work together to alleviate client problems in:
I try. Nurse: Maybe we can figure out
1. Interpersonal relations/communication
something together so that you can accomplish
2. Coping with particular stressors (e.g., ostomy
something you want to do.).
groups)
B. Ineffective communication styles: the following
3. Self-understanding
nontherapeutic approaches tend to block
B. Purposes
therapeutic communication and are sometimes
1. Increase self-awareness
used by nurses to avoid becoming involved with
2. Improve interpersonal relationships
clients emotional distress; often a protective
3. Make changes in behavior
action on part of nurse.
4. Deal with particular stressors
1. Reassuring: telling client there is no need to
5. Enhance teaching/learning
worry or be anxious (Client: Im nervous
about this test. Nurse: Everything will be all
right.).
2. Advising: telling client what you believe
should be done (Client: I am going to . . . Table 7-1 Types of Therapeutic Groups
Nurse: Why dont you do . . . instead? or
I think you should do . . .). Type Goal(s) Example
3. Requesting explanation: asking client to provide Task Accomplish Select field trip
reasons for his feelings/behavior. The use of outcome
why questions should be avoided (Nurse:
Why do you feel, think, or act this way?). Teaching/ Gain knowledge/ Identify side effects
4. Stereotypical response: replying to client with learning skills of medications
meaningless clichs (Client: I hate being in Social/support Give and receive Postmastectomy
the hospital. Nurse: Theres good and bad support clients
about everything.).
5. Belittling feelings: minimizing or making light Psychotherapy Insight/behavioral Resolve loss
of clients distress or discomfort (Client: Im change
so depressed about . . . Nurse: Everyone feels Activity Increase social Grooming,
sad at times.). interaction/ manicures
self-esteem

7 628 NCLEX-RN Review


53155_07_Ch07_p621-684.qxd 2/27/09 6:29 PM Page 629

C. Structure of groups Milieu Therapy


1. Leader(s) chosen
2. Selection of members A. Total environment (milieu) has an effect on
3. Size: 5 to 10 members individuals behavior, including:
4. Physical arrangements 1. Physical environment (i.e., cleanliness, noise,
5. Time/place of meetings colors, fresh air, light)
6. Open: accept members anytime 2. Relationships of staff to staff, staff to clients,
7. Closed: do not add new members and client to clients
D. Group dynamics 3. Atmosphere of safety, caring, mutual respect
1. System of interactions (e.g., client-run community meeting,
2. Collective activity community-set standards for behaviors)
3. Process: all activities/interactions B. Purposes
4. Content: topics discussed 1. Improve clients behavior
E. Stages of group development 2. Involve client in decision making of unit
1. Beginning stage 3. Increase clients sense of autonomy
a. Anxiety in new situation 4. Increase communication among clients and
b. Information given between clients and staff
c. Group norms established 5. Set structure of unit and behavioral limits
2. Middle stage 6. Form a sense of community
a. Group cohesiveness C. Role of the nurse
b. Members confronting each other 1. Involve clients in decision making
c. Reliance on group member leading to 2. Promote involvement of all staff
self-reliance 3. Promote development of social skills of
d. Sense of trust established individual clients (e.g., nurse serves as role
3. Termination stage model)
a. Individual member may leave abruptly 4. Encourage sense of community in staff and
b. Group decides work is done clients
c. Ambivalence felt about termination
d. Ideally, group members have met goals Crisis Intervention
F. Role of the nurse
1. Explain purpose and rules of group A. Client cannot resolve problem with usual problem-
2. Introduce group members solving skills. Problem is so serious that
3. Promote group cohesiveness functioning (homeostasis) is threatened. Crisis can
4. Focus on problems of group and group process be developmental (e.g., birth of first child), a
5. Encourage participation sudden death (e.g., car accident), a result of
6. Role model interpersonal violence (e.g., arson), terrorist attacks
7. Facilitate communication and war (e.g., September 11, 2001), or situational
8. Set limits (e.g., home destroyed by fire). Adapting to and
coping with the crisis can be considered within
Family Therapy the normal range up to 1 year.
B. Purposes
A. Client is whole family, although a family member 1. Support client during time of crisis
may be identified client. 2. Resolve crisis
B. Purposes 3. Restore client at least to precrisis level of
1. Improve relationships among family members functioning or assist client to integrate the
2. Promote family function crisis and reinvest in life
3. Resolve family problem(s) 4. Allow client to attain higher level of
C. Process functioning through acquiring greater skill in
1. Problem(s) are identified by each family member. problem solving
2. Members discuss their involvement in C. Process
problem(s). 1. Crisis event occurs: client unable to solve
3. Members discuss how problem(s) affect them. problem.
4. Members explore ways each of them can help 2. Increase in level of clients anxiety.
resolve problem(s). 3. Client may use trial and error approach.
D. Role of the nurse 4. If problem unresolved, anxiety escalates and
1. Assess interactions among family members client seeks help.
2. Make observations to family members D. Role of the nurse
3. Encourage expression of feelings by family 1. Assess clients perception of problem:
members to one another realistic/distorted
4. Assist family in resolving problems

PSYCHIATRIC-MENTAL HEALTH NURSING

7 629
53155_07_Ch07_p621-684.qxd 2/27/09 6:29 PM Page 630

2. Determine situational supports (e.g., family, EVALUATION


neighbors, agencies)
3. Explore previous coping behaviors of client A. How well have goals been met? If not met,
4. Offer support and education in resolving crisis why not?
5. Enlist help of situational supports 1. Review prior steps of nursing process.
6. Help client develop new, more effective a. Do you need more assessment data?
coping behaviors b. Were nursing diagnoses prioritized?
7. Convey hope to client that crisis can be c. Were goals feasible and measurable?
resolved d. Were interventions appropriate?
8. Work with client as he resolves crisis 2. Revise goals as necessary.
9. Encourage the client to attend a debriefing B. Client
session (if one is available and appropriate for 1. Enrolled/participates in appropriate treatment
the crisis) program.
2. Expresses concerns/needs and develops a
Behavior Modification therapeutic relationship with nurse.
3. Identifies causes for behavior; learns and uses
A. Based on theory that all behavior is learned as a
alternative coping mechanisms.
result of positive reinforcement. Behaviors can be
4. Demonstrates ability to care for self at
changed by substituting new behaviors.
optimum level and to identify areas where
B. Purpose: change unacceptable or maladaptive
assistance is needed.
behaviors
5. Does not engage in harmful behaviors; shows
C. Process
increased ability to control destructive
1. Determine the unacceptable behavior.
impulses.
2. Identify more adaptive behavior to replace the
C. Clients behavior demonstrates optimal orientation
unacceptable behavior.
to reality (e.g., can state name, place); interacts
3. Apply learning principles.
appropriately with others.
a. Respond to unacceptable behavior by
negative reinforcement (punishment) or by
withholding positive reinforcement (ignore
behavior).
BEHAVIORS RELATED TO
b. Determine what client views as reward. EMOTIONAL DISTRESS
4. When desired behavior occurs, present
positive reinforcement (reward).
5. Consistently reward desired behavior.
Anxiety
6. Consistently respond to unacceptable behavior A. General information
with negative reinforcement/ignoring 1. One of the most important concepts in
behavior. psychiatric-mental health nursing.
D. Types 2. Anxiety is present in almost every instance
1. Counterconditioning: specific stimulus evokes where clients are experiencing emotional
a maladaptive response that is replaced with a distress/have a diagnosed psychiatric illness.
more adaptive response. 3. Experienced as a sense of emotional or
2. Systematic desensitization physical distress as the individual responds to
a. Expose to small amount of stimulus while an unknown threat or thwarting of unmet
ensuring relaxation (client cannot be needs.
anxious and relaxed at same time). 4. The ego protects itself from the effects of
b. Continue relaxing client while increasing anxiety by the use of defense mechanisms
amount of stimulus. (see Table 7-2).
c. Fear response to stimulus is eventually 5. Physiologic responses are related to autonomic
extinguished. nervous system response and to level of
3. Token economy: Tokens (rewards such as anxiety.
candy) are used to reinforce desired behaviors. a. Subjective: client experiences feelings of
tension, need to act, uneasiness, distress,
Psychotropic Medications and apprehension or fear.
b. Objective: client exhibits restlessness,
A variety of agents is used to control disordered inability to concentrate, tension, dilated
thinking, anxiety, and mood disorders. Effects, side pupils, changes in vital signs (usually
effects, and nursing implications are summarized with increased by sympathetic nervous system
each disorder. response, may be decreased by
parasympathetic reactions).

7 630 NCLEX-RN Review


53155_07_Ch07_p621-684.qxd 2/27/09 6:29 PM Page 631

Table 7-2 Defense Mechanisms

Type Characteristics Examples


Denial Refusal to acknowledge a part of reality A client on strict bed rest is walking down the hall;
shows refusal to acknowledge need to stay in bed
because of illness.
A client states admission to the mental hospital is
for reasons other than mental illness.
Repression Threatening thoughts are pushed into the I dont know why I have to wash my hands all the
unconscious, anxiety and other symptoms are time, I just have to.
observed; client unable to have conscious
awareness of conflicts or events that are source
of anxiety
Suppression Consciously putting a threatening/distressing A nurse must study for the NCLEX, but she has had
thought out of ones awareness. a heated argument with her boyfriend. She decides
not to think about the problem until she finishes
studying, then she will attempt to resolve it.
Rationalization Developing an acceptable, justifiable (to self) A friend tells you that he has been in an automobile
reason for behavior accident because the car skidded on wet leaves in
the road; you go to the scene of the accident, but
there are no leaves; friend admits to you and to
self that he was probably driving too fast.
Reaction-formation Engaging in behavior that is opposite of true A man has an unconscious desire to view
desires pornographic films; he circulates a petition to
close the theater where such films are shown.
Sublimation Anxiety channeled into socially acceptable A student is upset because she received a failing
behavior grade on a test; she knows that she will feel better
if she goes jogging and runs a few miles.
Compensation Making up for a deficit by success in another area A young man who cannot make any varsity teams
becomes the chess champion in his school.
Projection Placing own undesirable trait onto another; A student who would like to cheat on an exam
blaming others for own difficulty states that other students are trying to cheat; a
paranoid client claims that the FBI had him
committed to the mental hospital.
Displacement Directing feelings about one object/person The head nurse reprimands you; you do not argue
toward a less-threatening object/person even though you do not agree with her
reprimand; when you return home that evening
you are hostile toward your roommate.
Identification Taking onto oneself the traits of others that one You greatly admire the clinical specialist in your
admires hospital; unconsciously you begin to use the
approaches she uses with clients.
Introjection Symbolic incorporation of another into ones own John becomes depressed when his father dies;
personality Johns feelings are directed to the mental image
he has of his father.
Conversion Anxiety converted into a physical symptom that is A young woman unconsciously desires to strike her
motor or sensory in nature mother; she develops sudden paralysis of her
arms.
Symbolization Representing an idea or object by a substitute A man who was spurned by a librarian develops a
object or sign dislike of books and reading.
Dissociation Separation or splitting off of one aspect of mental A student who prides herself on being prompt does
process from conscious awareness not recall the times that she arrived late for class.
(continues)

PSYCHIATRIC-MENTAL HEALTH NURSING

7 631
53155_07_Ch07_p621-684.qxd 2/27/09 6:29 PM Page 632

Table 7-2 Defense Mechanisms (continued)

Type Characteristics Examples


Undoing Behavior that is opposite of earlier unacceptable Joan tells an ethnic joke to a coworker, Sally; Sally, a
behavior or thought member of that ethnic group, is offended; the
following week Joan offers to work the weekend
for Sally.
Regression Behavior that reflects an earlier level of When a new baby is brought home, 5-year-old Billy
development; adults hospitalized with serious begins to wet his pants although he had not done
illnesses sometimes will engage in regressive this for the past 212 years.
behaviors
Isolation Separating emotional aspects of content from A client discusses his terminal diagnosis in clinical
cognitive aspects of thought terms. He does not express any emotion.
Splitting Viewing self, others, or situations as all good or A client tells you that you are the best nurse. Later
all bad tells you that you are incompetent and she will
report you.

6. Anxiety can be viewed positively (motivates 3. Encourage client to move from affective
us to change and grow) or negatively (feeling) mode to cognitive (thinking) behavior
(interferes with problem-solving ability and (e.g., ask client, What are you thinking?).
affects functioning). Stay with client. Reduce anxiety by remaining
a. Trait anxiety: individuals normal level of calm yourself; use silence, or speak slowly and
anxiety. Some people are usually rather softly.
intense while others are more relaxed; may 4. Help client recognize own anxious behavior.
be related to genetic predisposition/early 5. Provide outlets (e.g., talking, psychomotor
experiences (repressed conflicts). activity, crying, tasks).
b. State anxiety: change in persons anxiety 6. Provide support and encourage client to find
level in response to stressors (environmental ways to cope with anxiety.
or any internal threat to the ego). 7. In panic state nurse must make decisions.
7. Levels of anxiety a. Do not leave client alone.
a. Mild: increased awareness; ability to solve b. Encourage ventilation of thoughts and
problems, learn; increase in perceptual feelings.
field; minimal muscle tension c. Use firm voice and give short, explicit
b. Moderate: optimal level for learning, directions (e.g., Sit in this chair. I will sit
perceptual field narrows to pay attention to here next to you.).
particular details, increased tension to d. Engage client in motor activity to reduce
solve problems or meet challenges tension (e.g., We can take a brisk walk
c. Severe: sympathetic nervous system around the day room. Lets go.).
(flight/fight response); increase in blood
pressure, pulse, and respirations; narrowed Defense Mechanisms
perceptual field, fixed vision, dilated
pupils, can perceive scattered details or Usually unconscious processes used by ego to defend
only one detail; difficulty in problem itself from anxiety and threats (see Table 7-2).
solving
d. Panic: decrease in vital signs (release of Disorders of Perception
sympathetic response), distorted
perceptual field, inability to solve Occur with increased anxiety, disordered
problems, disorganized behavior, feelings thinking/impaired reality testing
of helplessness/terror A. Illusions
B. Nursing interventions 1. General information: stimulus in the
1. Determine the level of clients anxiety by environment is misperceived (e.g., car
assessing verbal and nonverbal behaviors and backfiring is perceived as a gunshot; a
physiologic symptoms. bathrobe in an open closet is perceived as a
2. Determine cause(s) of anxiety with client, person in the closet); may be visual, auditory,
if possible. tactile, gustatory, olfactory.

7 632 NCLEX-RN Review


53155_07_Ch07_p621-684.qxd 2/27/09 6:29 PM Page 633

2. Nursing intervention: show/explain stimulus Withdrawal


to client to promote reality testing.
B. Delusions A. General information: withdrawal from social
1. General information: fixed, false set of beliefs interaction by not talking, walking away, turning
that are real to client. away, sleeping, or feigning sleep
a. Grandiose: false belief that client has power, B. Nursing interventions
wealth, or status or is famous person 1. Use silence.
b. Persecutory: false belief that client is the 2. Offer self.
object of anothers harassment or harmful 3. Discuss nonthreatening topics that will not
intent provoke increased anxiety.
c. Somatic: false belief that client has some 4. Be consistent; keep promises, promote trust.
physical/physiologic defect
2. Nursing interventions Hostility and Aggression
a. Avoid arguing: client cannot be convinced,
even with evidence, that the belief is A. General information
false. 1. Hostile behavior: responding to nurse with
b. Determine clients need (grandiose anger, insults, threats.
delusion may indicate low self-esteem; 2. Assaultive behavior: attempting to physically
provide opportunities to succeed at task harm others.
that will enhance self-concept). 3. Usually nurse is not real object of clients
c. Reduce anxiety to encourage decreased anger, but is convenient target for angry
need to use delusions. feelings/verbalizations.
d. Accept clients need for delusion, present B. Nursing interventions
(but do not insist that client accept) reality. 1. Hostility
e. After therapeutic relationship has been a. Recognize own response of anger or
established, you can express doubt about defensiveness.
delusions to client. b. Determine source of clients anger (e.g.,
f. Direct clients attention to nondelusional, intoxicated, psychotic, recent argument
nonthreatening topics (e.g., current events, with parent).
clients hobbies or interests). c. Accept angry feelings.
C. Ideas of reference d. Attempt to have client verbalize feelings
1. General information: belief that events or and channel into acceptable behaviors.
behaviors of others relate to self (e.g., e. Assess the need for prn medications based
telephone rings in nurses station, client on the possible source of the hostility.
believes they are calling for him; two nurses 2. Physical aggression/assaultive behaviors (client
are talking and laughing, client believes nurses may act on increased anxiety by throwing
are talking/laughing at him). objects or attempt to physically harm others)
2. Nursing interventions are the same as for a. Assess for increased anxiety.
Delusions. b. Maintain distance, at least arms length.
D. Hallucinations c. Attempt to have client verbalize feelings.
1. General information: sensory perceptions that d. Talk client down.
have no stimulus in environment; most e. Obtain help if client becomes assaultive.
common hallucinations are auditory and
visual (e.g., hearing voices; seeing persons, Self-Mutilation
animals, objects).
2. Nursing interventions A. General information: behaviors cause physical
a. Encourage client to describe hallucination. injury but are not motivated by the desire to die.
b. Accept that this is a real experience for B. Nursing interventions
client. 1. Assess for suicide risk.
c. Present reality. 2. Offer support.
d. Example: nurse sees client in listening 3. Protect client from carrying out self-mutilation
attitude or responding to auditory actions.
hallucinations. Nurse: You seem to be 4. Remove objects that can be used for
listening/talking. Client: The voices are self-harm.
telling me to hurt myself. Nurse: I dont 5. Observe for changes in behaviors and
hear the voices. Tell me what the voices attitudes.
are saying to you.

PSYCHIATRIC-MENTAL HEALTH NURSING

7 633
53155_07_Ch07_p621-684.qxd 2/27/09 6:29 PM Page 634

Suicide Table 7-4 Lethality Assessment


A. General information
1. Ideation: verbalization of wish to die (overt or Plans for suicide: when? where? how?
disguised) Means available: what will be used? Is it available to client?
2. Gestures: engaging in nonlethal behaviors (e.g., Lethality of means (e.g., tranquilizers are less lethal when
superficial scratches, ingestion of medication used alone than when combined with alcohol; guns are
in amounts that are not likely to cause serious more lethal than plan to cut wrists)
injury/death) - Most lethal: gunshot, hanging, jumping from high places,
3. Actions: engaging in behaviors or planning to carbon monoxide, potent poisons (e.g., cyanide)
engage in behaviors that have potential to - Less lethal: nonprescription drugs, wrist cutting,
cause death tranquilizers without CNS depressants
4. May or may not be associated with a - Males tend to use more lethal means
psychiatric disorder Possibility of rescue
5. Groups at risk (see Table 7-3) Support systems available or sense of isolation
B. Assessment findings
Availability of alcohol or drugs
1. Verbal cues
a. Overt: Im going to kill myself. Severe/panic level of anxiety
b. Disguised: I have the answer to my Hostility
problems. Disorganized thinking
2. Behavioral cues Preoccupation with thought of suicide plan
a. Giving away prized possessions
Prior suicide attempts
b. Getting financial affairs in order, making
a will
c. Suicidal ideation/gestures
d. Indications of hopelessness, depression
e. Behavioral and attitudinal changes (e.g.,
neat person becomes sloppy, depressed b. Ask client if he has formulation of plan;
person suddenly becomes alert/positive, if details are worked out, when? where?
increased use of drugs and/or alcohol, how?
alcohol withdrawal). c. Check availability of method (e.g., gun,
3. For lethality assessment, see Table 7-4. pills).
C. Nursing interventions 3. Keep client under constant observation.
1. Contract with client to report suicide ideation 4. Remove any objects that can be used in suicide
with intent and/or suicide attempt. attempt (e.g., shoe laces, sharp objects).
2. Assess suicide risk. 5. Therapeutic intervention
a. Ask client if he thinks about, intends to a. Support aspects of wish to live; clients
harm himself. often ambivalent: wish to live and wish
to die.
b. Use one-to-one nurse/client relationship
(let client know you care for him).
c. Allow client to express feelings of
Table 7-3 Groups at Increased Risk for Suicide hopelessness, helplessness, worthlessness.
d. Provide hope.
Adolescents/young adults (ages 1524) e. Provide diversionary activities.
Elderly f. Utilize support groups (e.g., family, clergy).
g. Notify psychiatrist or responsible
Terminally ill
practitioner for the medical management
Persons who have experienced loss/stress of the client to evaluate medications and
Survivors of persons who have committed suicide precaution level.
Individuals with bipolar disorders 6. Following a suicide
Depressed persons (when depression begins to lift) a. Encourage survivors to discuss clients
Substance abusers death, their feelings and fears.
b. Provide anticipatory guidance to family
Persons who have attempted suicide previously
who may experience problems at holidays,
Schizophrenics anniversaries.
More women attempt suicide; more men complete suicide c. Hold staff meetings to ventilate feelings
and provide a debriefing to process the
event.

7 634 NCLEX-RN Review


53155_07_Ch07_p621-684.qxd 2/27/09 6:29 PM Page 635

2. Ask him to sit down and orient him to the


Sample Questions nurses name and the need for information.
3. Check his vital signs, ask him about allergies,
and call the physician for sedation.
1. The nurse is talking with a mother to assess her
4. Explain the importance of accurate
child. A positive response to which question
assessment data to him.
would indicate the child is in the anal stage of
psychosexual development as described by 6. A woman has been referred for help in managing
Freud? her children. The woman arrives late for
1. Does he put everything in his mouth? appointments and focuses on her busy schedule,
2. Does he say No! to everything you say? the difficulty in parking, and other reasons for
3. Does he like to dress up and pretend to be being late. How would the nurse best interpret
his father? this behavior?
4. Does he seem jealous when you show 1. Transference.
affection to his father? 2. Counter-transference.
3. Identification.
2. The nurse is assessing a 70-year-old woman.
4. Rationalization.
Which statement by the client indicates that she
has achieved integrity according to Ericksons 7. A woman has remained at the side of the nurse
stages of personality development? all day. When the nurse talked with other clients
1. My life has been wasted. during dinner, the client tried to regain the
2. My children no longer visit me. I am just nurses attention and then began to shout,
waiting to die. Youre just like my mother! You pay attention
3. I was a good nurse when I was younger, but to everyone but me! What is the best
now I am nothing. interpretation of this behavior?
4. I have a good life and I still enjoy it, but I 1. She is exhibiting sublimation.
feel ready to go when it is time. 2. She has been spoiled by her family.
3. The nurse has failed to meet her needs.
3. Which cognitive skill would the nurse expect a
4. She is demonstrating transference.
6-year-old child to be in the process of
developing? 8. A nurse is part of a community task force on
1. Understanding of basic rules. teenage suicide. The task force is considering all
2. Ability to understand abstract concepts. of the following steps in an effort to reduce teen
3. Recognition of object permanence. suicide. Which action represents primary
prevention?
4. Imitation of others actions.
1. Encourage emergency room staff to request
4. The nurse is meeting a new client on the unit. psychiatric consultation for adolescents who
Which action, by the nurse, is most effective in overdose.
initiating the nurse-client relationship? 2. Educate teachers, counselors, and school
1. Introduce self and explain the purpose and nurses in recognition and early intervention
the plan for the relationship. with suicidal teens.
2. Describe the nurses family and ask the client 3. Provide community programs, such as
to describe his family. Scouts, which increase self-esteem for
3. Wait until the client indicates a readiness to children and adolescents.
establish a relationship. 4. Increase the number of inpatient adolescent
4. Ask the client why he was brought to the psychiatric beds available in the community.
hospital.
9. Two nurses are discussing plans for their client
5. An adult has just been brought to the psychiatric group. What should be in the plan to promote
unit and is pacing up and down the hall. The group cohesiveness?
nurse is to admit him to the hospital. To establish 1. Let the group know which clients are
a nurse-client relationship, which approach behaving in ways approved by the nurses.
should the nurse try first? 2. Help the group identify group goals that are
1. Assign someone to watch him until he is consistent with the individual members
calmer. goals.

PSYCHIATRIC-MENTAL HEALTH NURSING

7 635
53155_07_Ch07_p621-684.qxd 2/27/09 6:29 PM Page 636

3. Make most decisions about the group in 14. An adult is pacing about the unit and wringing
advance and make each group member aware his hands. He is breathing rapidly and complains
of the nurses decisions. of palpitations and nausea and he has difficulty
4. Seat the most talkative members nearest the focusing on what the nurse is saying. He says he
nurses where they can be more clearly heard is having a heart attack but refuses to rest. How
by the group. would the nurse interpret his level of anxiety?
1. Mild.
10. The nurse is the leader of a client group. The
2. Moderate.
members of the group test each other and the
groups rules, as well as compete for the nurses 3. Severe.
attention. This behavior is typical of which 4. Panic.
phase of the nurse-client relationship? 15. Each time a client is scheduled for a therapy
1. Orientation. session she develops a headache and nausea.
2. Working. How would the nurse interpret this behavior?
3. Feedback. 1. Conversion.
4. Termination. 2. Reaction formation.
3. Projection.
11. A family was referred to family therapy after
their teenage son experienced behavioral 4. Suppression.
problems in school. Which statement by the 16. A man is admitted to the intensive care unit
father indicates that he understands the purpose with chest pain, an abnormal ECG, and elevated
of family therapy? enzymes. When the significance of this is
1. Our son will realize the consequences of his explained to him, he says, I cant be having a
actions and try harder to behave. heart attack. No way. You must be mistaken.
2. It will help us learn to communicate and The nurse suspects the client is using which
problem solve better as a group. defense mechanism?
3. I expect the therapist to tell my wife how to 1. Sublimation.
discipline our son. 2. Regression.
4. The therapist will tell us how to make our 3. Dissociation.
son behave better in school. 4. Denial.
12. A client walks in to the mental health outpatient 17. An adult is admitted for panic attacks. He
center and states, Ive had it. I cant go on any frequently experiences shortness of breath,
longer. Youve got to help me. The nurse asks palpitations, nausea, diaphoresis, and terror.
the client to be seated in a private interview What should the nurse include in the care plan
room. Which action should the nurse take next? when he is having a panic attack?
1. Reassure the client that someone will help 1. Calm reassurance, deep breathing, and
him soon. medication as ordered.
2. Assess the clients insurance coverage. 2. Teach him problem solving in relation to his
3. Find out more about what is happening to the anxiety.
client. 3. Explain the physiologic responses of anxiety.
4. Call the clients family to come and provide 4. Explore alternate methods for dealing with
support. the cause of his anxiety.

13. The nurse is caring for a client with anorexia 18. A client on an inpatient psychiatric unit refuses
nervosa who is to be placed on behavior to eat and states that the staff is poisoning her
modification. Which is appropriate to include in food. Which action should the nurse include in
the nursing care plan? the clients care plan?
1. Remind the client frequently to eat all the 1. Explain to the client that the staff can be
food served on the tray. trusted.
2. Increase phone calls allowed the client by 2. Show the client that others eat the food
one per day for each pound gained. without harm.
3. Include the family with the client in therapy 3. Offer the client factory-sealed foods and
sessions 2 times per week. beverages.
4. Reduce the clients TV time for any weight loss. 4. Institute behavior modification with

7
privileges dependent on intake.

636 NCLEX-RN Review


53155_07_Ch07_p621-684.qxd 2/27/09 6:29 PM Page 637

19. A woman is being treated on the inpatient unit 7. 4. Transference is the unconscious transfer of
for depression. She tells the nurse, I dont see qualities originally associated with another
how I can go on. Ive been thinking of ways to relationship to the nurse. These are often
kill myself. I can see several ways to do it. What qualities associated with a parent or sibling and
is the best initial action for the nurse to perform? may provoke responses from the client that are
1. Notify her family about her statements. not appropriate to the situation.
2. Explain to the client the consequences of 8. 3. Primary prevention involves making changes
suicide on her family. in the community that promote health and
3. See that someone is with the client at all prevent disease.
times.
4. Help the client identify alternate means of 9. 2. Goals that are best met by a group and that are
coping. consistent with the goals of the individual
members foster cohesive groups.
20. An adult has been admitted to the inpatient unit
with a diagnosis of depression. He states that he 10. 1. During the orientation phase, group members
continues to think of suicide. Which is most demonstrate these behaviors as they try to
essential for the nurse to include in his nursing identify and develop trust with the group.
care plan? 11. 2. Family therapy is aimed at improving
1. Encourage the client to participate in all unit communication and problem solving within the
activities. family group. The focus is on the family as a
2. Ask the client if he has a knife. group, not on correcting the behavior of any one.
3. Allow the client time alone to relax and think.
12. 3. The nurse must assess the client and his
4. Have someone stay with the client 24 hours a situation before the appropriate action can be
day. determined.

13. 2. In behavior modification, rewards are tied to


specific goals.
Answers and Rationales
14. 4. Terror, physiologic changes, and inability to
focus on the real world are characteristic of the
1. 2. Negativism is common to the toddler in the most extreme form of anxiety, panic.
anal stage of development (age 1 to 3) who is
learning to assert his independence and mastery. 15. 1. Conversion changes anxious feelings into
somatic symptoms.
2. 4. Integrity includes acceptance of changes; a
sense of continuity of past, present, and future; 16. 4. Denial helps the person escape unpleasant or
and acceptance of death. intolerable reality by refusing to perceive the
facts. It can serve as a normal protection in the
3. 1. Preoperational-preconceptual children (5 to early stages of crisis, but if the denial persists it
7 years old) are learning to integrate concepts will prevent the client from coping.
based on relationships and can comprehend the
basic rules. 17. 1. Before any other interventions can be used,
the client in panic must reduce his anxiety to a
4. 1. The client needs orientation to the nurse and manageable level. The other interventions might
the situation. An open, honest approach in be used when the client is less anxious.
sharing these initial data will set the tone for the
relationship. 18. 3. The client may be able to eat food if she
knows the staff has not handled it.
5. 2. Many clients are anxious at the time of
19. 3. Maintaining client safety is the first priority.
admission and are often reassured by a calm,
When a client is actively suicidal, one to one
competent professional approach, which should
observation is necessary.
always be tried first. If the client is unable to
respond to this, then other measures, such as 20. 4. The client who is actively suicidal needs
medication, may be necessary. constant observation to prevent him from
carrying out his plan. Any objects that could
6. 4. Rationalization is characterized by providing
be used in a suicide attempt would be
the client acceptable reasons (to her) why she is
automatically removed at admission.

7
having some difficulty.

PSYCHIATRIC-MENTAL HEALTH NURSING 637


53155_07_Ch07_p621-684.qxd 2/27/09 6:29 PM Page 638

Psychiatric Disorders (DSM-IV-TR)

4. Disturbed sensory-perceptual
DISORDERS OF INFANCY, 5. Fear
CHILDHOOD, AND ADOLESCENCE 6. Ineffective coping
7. Low self-esteem
8. Risk for injury
Overview 9. Risk for violence
A. A specific group of disorders beginning in infancy, 10. Sexual dysfunction
childhood, or adolescence. 11. Total incontinence
B. Clients in these age groups may also evidence B. Parents/family
other disorders such as depression or 1. Anticipatory grieving
schizophrenia. 2. Anxiety
C. Intellectual, behavioral, and/or emotional 3. Deficient knowledge
dysfunction of the young client also has an effect 4. Disabled family coping
on the family, which may require nursing 5. Dysfunctional family process
intervention. 6. Impaired parenting
7. Interrupted family processes
8. Parental role conflict
Assessment 9. Risk for care-giver role strain
Newborn/Infants
Planning and Implementation
A. Maturation
B. Developmental level Goals
C. Sensorimotor capabilities
D. Bonding A. Client will:
E. Response to cuddling 1. Communicate thoughts and feelings about
self-concept.
2. Perform tasks at optimal level of capability.
Children/Adolescents 3. Develop trusting relationship with caregivers.
A. Motor skills B. Parents/family will:
B. Communication abilities 1. Communicate feelings and responses to child
C. Vocational/academic skills and to disorder.
D. Social and behavioral problems 2. Demonstrate knowledge of disorder.
E. Behavioral changes 3. Formulate plans for childs care.
F. Growth and development: physical/emotional
G. Self-concept Interventions
H. Knowledge of disorder A. Client
1. Establish a therapeutic relationship by
Parent/Family accepting client and clients limitations.
2. Promote communication by use of therapeutic
A. Response to infant/child/adolescent with disorder
techniques, play therapy.
B. Guilt, sense of loss
3. Encourage independence in task performance
C. Sibling jealousy/resentment
with guidance and support.
D. Knowledge of disorder
B. Parents/family
E. Expectations
1. Promote communication by accepting family
F. Plans for future (home care/institutionalization)
responses.
2. Provide information about disorder.
Analysis 3. Contact appropriate person/agency for
consultation with family about care and
Nursing diagnoses for a child/family with a
assistance with the child.
psychiatric-mental health disorder may include:
A. Client
1. Anxiety
Evaluation
2. Deficient knowledge A. Client

7
3. Deficient self-care 1. Demonstrates trust in caregivers.

638 NCLEX-RN Review


53155_07_Ch07_p621-684.qxd 2/27/09 6:29 PM Page 639

2. Relates feelings about self verbally or 2. Provide opportunities for client/family to


symbolically. communicate thoughts, feelings.
3. Performs activities of daily living (ADLs) and 3. Provide positive reinforcement for every
tasks at optimal level. success.
B. Parents/Family 4. Accept clients limitations and set goals
1. Relate positive/negative responses to child. accordingly.
2. Demonstrate understanding of disorder and 5. Provide support and information about
childs potential. disorder to family.
3. With consultant, formulate a plan for childs 6. Accept familys response to client.
care.
Other Disorders of Childhood/Adolescence
Specific Disorders A. General information
Mental Retardation 1. Separation anxiety: excessive anxiety and
worry about being separated from
Note: This is coded on Axis II. person(s)/places to which child has become
A. General information attached (e.g., refusal to leave mother/home to
1. Significant subaverage intelligence (IQ of 70 or attend school)
below) resulting in maladaptive behaviors 2. Reactive/attachment disorder: reluctance to
with onset before age of 18 years enter social relationships with others, creating
2. Etiology an interference with social growth
a. Heredity 5% 3. Overanxious disorders: pervasive, unrealistic
b. Early alterations in embryonic worry or concern about competency; somatic
development 30% complaints without physical basis
c. Perinatal problems 10% B. Assessment findings: excessive anxiety related to
d. Acquired in infancy/early childhood 5% separation, social interaction, and achievements
e. Environmental/other mental disorders C. Nursing intervention: provide information
1520% regarding available mental health services for child
f. Unknown etiology 3040% and family.
3. Degrees of retardation
a. Mild mental retardation (IQ 5070) Disorders with Physical Manifestations
1) 85% of cases
2) Educable to 6th grade level A. General information
3) Able to become self-supporting 1. Important to rule out any physiologic cause
b. Moderate mental retardation (IQ 3549) 2. Often related to stress or conflict in the
1) 10% of cases family
2) Educable to 2nd grade level 3. May affect childs family/social interactions
3) Able to perform skills but will need and development
supervision at work B. Assessment findings
c. Severe mental retardation (IQ 2034) 1. Enuresis: urinary incontinence (bedwetting)
1) 34% of cases after age 5 not caused by physical disorder
2) May learn to talk/communicate 2. Encopresis: fecal incontinence after age 4 not
3) Able to perform simple tasks and caused by physical disorder
elementary hygiene 3. Tics: involuntary, repetitive movements
d. Profound mental retardation (IQ below 20) 4. Stuttering: repetition of sounds, words or
1) 12% of cases frequent hesitations in speaking
2) Some speech/communication possible C. Nursing interventions
B. Assessment findings 1. Provide information about the disorders and
1. Intellectual impairment (determine degree) emphasize that they are treatable.
2. Sensorimotor impairment 2. Determine whether family therapy may be
3. Communication, social, behavioral indicated, as well as individual therapy for
impairment child.
4. Lack of self-esteem and poor self-image 3. Offer support and help child/family overcome
5. Sense of loss, guilt, nonacceptance or feelings of shame or guilt.
unrealistic expectations on part of 4. For enuresis and encopresis, utilize toilet
parents/family training techniques.
C. Nursing interventions 5. Encourage discussion of client/family
1. Promote optimal functioning in ADL and response to symptoms.
feelings of accomplishment, self-worth.

PSYCHIATRIC-MENTAL HEALTH NURSING

7 639
53155_07_Ch07_p621-684.qxd 2/27/09 6:29 PM Page 640

PERVASIVE DEVELOPMENTAL Eating Disorders


DISORDERS A. General information
1. Gross disturbances in eating behaviors
2. Pica: persistent eating of nonfood substances
Autistic Disorder such as, paint, sand, ice, paper
A. General information 3. Bulimia nervosa: binge eating; the ingestion of
1. Usually develops prior to 3 years of age large amounts of food in short time, often
2. Categorized with a group of disorders known followed by self-induced vomiting. May be
as autism spectrum disorders (ASD) accompanied by affective disorders and fear of
3. 1 in 150 individuals is diagnosed with being unable to stop this behavior. Manifested
disorder; more common than pediatric cancer, by fluctuations in weight caused by binges of
diabetes, and AIDS combined eating and fasting. Antidepressant medications
4. Occurs in all racial, ethnic, and social groups; can be used in the treatment of bulimia.
affects males to females 4:1 4. Anorexia nervosa: refusal to eat or aberration
5. Symptoms range from very mild to quite severe in eating patterns, resulting in severe
6. Fastest-growing serious developmental emaciation that can be life threatening.
disability in the United States Characterized by a fear of becoming fat, and a
7. No known medical detection or cure body-image disturbance in which clients claim
8. Special education is necessary. to feel fat even when extremely thin. This
B. Assessment findings disorder is most common (95%) in adolescent
1. Infant not responsive to cuddling; may even and young adult females. There is a mortality
show an aversion to being touched rate of 718%. Antidepressant medications
2. No eye contact or facial responsiveness can be used in the treatment of anorexia.
3. Impaired or no verbal communication B. Assessment findings (anorexia nervosa)
4. Echolalia (repetition of words/phrases spoken 1. Weight loss of 15% or more of normal body
by others) weight for age and height
5. Inability to tolerate change 2. Electrolyte imbalance
6. Ritualistic or repetive behavior 3. Depression
7. Fascination with movement, spinning objects 4. Preoccupation with being thin; inability to
8. Labile moods recognize degree of own emaciation (distorted
9. Unresponsive or overresponsive to stimuli body image)
10. Symptoms may appear from 6 months to 5. Social withdrawal and poor family and
2 years of age individual coping skills
11. High risk for developing seizure disorders 6. History of high activity and achievement in
12. Medications that have been used to assist in academics, athletics
treatment of behavior are haloperidol, 7. Amenorrhea
clomipramine, and SSRIs. C. Nursing interventions
C. Nursing interventions 1. Monitor vital signs.
1. Provide parents/family with support and 2. Measure I&O.
information about the disorder, opportunities 3. Weigh client 3 times per week at the same time
for therapy and education for the child. (check to be sure client has not hidden heavy
2. Assist child with ADLs. objects or water loaded before being weighed,
3. Promote reality testing. weigh in hospital gown).
4. Encourage child to develop a relationship with 4. Do not comment on weight loss or gain.
another person. 5. Set limits on time allotted for eating.
5. Maintain regular schedule for activities. 6. Record amount eaten.
6. Provide constant routine for child (place for 7. Stay with client during meals, focusing on
eating, sitting, sleeping). client, not on food.
7. Controversial issues: Maintain a gluten-free 8. Accompany client to bathroom for at least
or casein-free diet; multiple vaccines given at 1
2 hour after eating to prevent self-induced
18 months. vomiting.
8. Protect child from self-injury. 9. Individual/family therapy may be necessary.
9. Provide safe environment. 10. Encourage client to express feelings.
10. Institute seizure precautions if necessary. 11. Help client to set realistic goal for self and to
11. Refer parents to support groups and reduce need for being perfect.
websites. 12. Encourage client to discuss own body image;
present reality; do not argue with client.
13. Teach relaxation techniques.

7
14. Help client identify interests and positive
aspects of self.

640 NCLEX-RN Review


53155_07_Ch07_p621-684.qxd 2/27/09 6:29 PM Page 641

DELIRIUM, DEMENTIA, AND B. Nutritional status


C. Ability to perform ADLs, self-care
OTHER COGNITIVE DISORDERS D. Presence of confabulation (making up information
to fill in memory gaps)
Overview E. Behavioral/social changes
F. Disorders of perception
A. A group of disorders with a known or presumed G. Impaired motor skills, coordination
etiology. H. Change in sleep patterns
B. Frequently manifest as dementia or delirium. I. Elimination: constipation/incontinence
C. May be substance induced (drugs or alcohol) or J. Family response to clients condition
caused by a disease process; etiology may be
unknown.
D. It is important for the nurse to assess behaviors
Analysis
rather than focus on medical diagnoses. Nursing diagnoses for clients with these disorders may
E. Behaviors related to impaired brain functioning include:
may be temporary or permanent, with increasing A. Anxiety
degeneration and eventual loss of brain function. B. Impaired verbal communication
F. Not exclusive to old age, may complicate illnesses C. Ineffective individual/family coping
in any age group. D. Altered family processes
E. Risk for fluid volume deficit
Types F. Risk for injury
G. Imbalanced nutrition: less than body requirements
A. Delirium/rapid development H. Self-care deficits
1. Manifested by reduced awareness of I. Low self-esteem
environment, disorders of perception, thought, J. Disturbed sleep pattern
speech, and attention deficits. K. Disturbed thought processes
2. Usually of brief duration. L. Risk for violence
3. May occur postoperatively or following head
injury, intoxication from drugs/alcohol, acute
disease, or injury. Planning/Implementation
B. Dementia/gradual development Goals
1. Loss of intellectual abilities resulting in
impaired social and occupational functioning. A. Client will:
2. May be temporary, or progressive loss may 1. Be protected from injury.
occur. 2. Retain optimal cognitive function and self-care
3. Found predominantly in elderly. abilities.
4. Personality changes are usually an 3. Have fear/anxiety minimized.
exaggeration of former character traits 4. Maintain adequate nutrition/hydration.
(e.g., suspicious, nontrusting person becomes B. Family will communicate feelings about client.
paranoid); but alteration can also occur
(e.g., formerly neat and orderly person pays Interventions
no attention to hygiene, becomes sloppy and
dirty). A. Institute safety measures: side rails, appropriate
5. Memory impairment; short-term memory loss lighting in room, bed should be at lowest setting,
may be most obvious. frequent checks. Restraints should only be used as
6. Organic etiology may be known; conditions a last resort and for protection of client as ordered
include intoxication, infections, tumors, by physician and based on state and federal
circulatory disorders (cerebral atherosclerosis), regulations.
trauma, Huntingtons chorea, Korsakoffs B. Maintain reality orientation
syndrome, Creutzfeld-Jakob disease, 1. Client may not be capable of reality testing.
neurosyphilis. 2. Continue to address client by name.
7. Specific etiology may not be known (e.g., 3. Maintain awareness of clients limitations in
Alzheimers disease, Picks disease). this area.
8. Frequently these clients cannot perform basic 4. Do not tell client to remember; severe
ADLs. memory loss may make client incapable of
memory.
C. Assist/support with self-care needs; arrange for
Assessment necessary assistive devices, help with feeding;
A. Mental status assessment, especially orientation to encourage fluids.

7
time and place, memory, and judgment

PSYCHIATRIC-MENTAL HEALTH NURSING 641


53155_07_Ch07_p621-684.qxd 2/27/09 6:29 PM Page 642

D. Avoid insight therapy and discussion of PSYCHOACTIVE SUBSTANCE-


impaired mental functioning as this may increase
anxiety. INDUCED ORGANIC MENTAL
E. Provide spouse/family with information about
clients capabilities. DISORDERS
F. Provide support for spouse/family; encourage
The use of substances that result in intoxication or
continued interaction with client.
withdrawal syndromes, delirium, hallucinations,
G. Administer ordered medications (based on
delusions, mood disorders.
etiology), assess response, and provide education
to the client and family. Medications might
include short-acting benzodiazepines, Assessment
antidepressants, cholinesterase inhibitors, or
A. Determine substances used, amount and last time
low-dose antipsychotics.
taken, and if combined with other drugs
H. Provide information on support organizations or
B. Pupillary changes, changes in vital signs or level of
groups.
consciousness
C. Presence of dehydration
Evaluation D. Presence of nutritional and vitamin deficiencies
E. Suicide potential: ideation, gestures
A. Client
F. Level of anxiety
1. Remains free from injuries.
G. Use of denial/projection
2. Retains cognitive functions and self-care
H. Symptoms of overdose (will be drug-specific;
ability as far as possible; interacts with others
see Table 7-5)
appropriately.
I. Drug-use patterns: what, when, why substances are
3. Maintains appropriate weight.
used
B. Family
1. Expresses sense of loss or frustration related to
clients condition. Analysis
2. Continues contact with client.
Nursing diagnoses for clients with a psychoactive
3. Participates in support or group organizations.
substance abuse disorder may include:
A. Anxiety
B. Ineffective coping
SUBSTANCE USE DISORDERS C. Fear
D. Risk for fluid volume deficit
Overview E. Risk for injury
F. Imbalanced nutrition: less than body requirements
A. The use of chemical agents (alcohol and drugs) to
G. Self-care deficit
change behavior and mood
H. Low self-esteem
B. Abuse: continued use despite problems (social,
I. Disturbed sensory-perceptual
occupational, psychologic) that are caused by
J. Sleep deprivation
substance or continued use in hazardous situations
K. Disturbed thought processes
(e.g., operating machinery, driving)
L. Risk for violence
C. Dependence
M. Ineffective denial
1. Need for larger amounts (tolerance)
2. Unsuccessful attempts to decrease/
discontinue use Planning and Intervention
3. Inability to function as usual in work, social
activities Goals
4. Withdrawal symptoms (psychologic/physical Client will:
distress when substance is reduced/ A. Be protected from injury.
discontinued) B. Receive adequate hydration and nutrition.
D. Addiction: compulsive use of a substance; C. Terminate use of substance being abused without
physiologic and psychologic dependence withdrawal symptoms; emergency care will be
provided if symptoms cannot be avoided.

7 642 NCLEX-RN Review


53155_07_Ch07_p621-684.qxd 2/27/09 6:29 PM Page 643

Table 7-5 Commonly Abused Drugs

Assessment Nursing Interventions


Drug Effect Dependence Findings Overdose for Overdose

Barbiturates
Antianxiety drugs, Reduction in Psychologic at first, Irritability, weight Slurred speech, Keep person awake
hypnotics anxiety, escape then physiologic; loss, changes in lethargy, and moving to
from stress withdrawal mood or motor respiratory prevent coma;
similar to alcohol coordination depression, maintain airway.
withdrawal, to coma; use
point of delirium; combined with
cross-tolerance to alcohol can be
other depressants lethal
Opioids/Narcotics
Heroin, Euphoria, Psychologic Pinpoint pupils, Depressed Provide emergency
morphine, dysphoria, dependence mental clouding, consciousness support of vital
meperidine, and/or apathy rapidly leading to lethargy, impaired and respirations, functions.
methadone physical; signs of memory and dilated pupils In withdrawal,
withdrawal: judgment, with anoxia or administer
cramps, nausea, evidence of polydrug use methadone or
vomiting, needle tracks, Narcan as ordered.
diarrhea; sleep inflamed nasal
disturbance, mucosa if drug is
chills and snorted
shaking
Stimulant
Cocaine/crack Increased self- Dopamine Increased vital Delirium, tremors, Emergency support of
esteem, deficiency results signs, headache, high fever (106+) vital functions,
energy, sexual in psychologic chest pain, convulsions, reduce CNS
desire, dependency to depression cardiac/ stimulation.
euphoria; produce feelings and/or paranoia, respiratory arrest
decreased of well-being inflamed nasal
anxiety passages if
snorted
Amphetamines
Amphetamine, Depressed Long-term use or Same as cocaine, Same as cocaine Same as cocaine, plus
dextroamphetamine, appetite; high doses may plus suicidal suicide precautions.
methamphetamine increased produce delirium, ideation Observe for
activity, paranoid-like increased anxiety to
awareness, delusions, panic, which may
sense of well- withdrawal, potentiate assaultive
being depression, behavior.
fatigue, sleep
disturbances Monitor vital signs.
Violent behavior,
Phencyclidine (PCP) Euphoria, Not reported Vomiting, suicide, respiratory Observe for suicidal or
psychomotor arrest, delirium, assaultive behavior.
hallucinations,
agitation, coma, increased Provide
paranoid
emotional blood pressure nonthreatening
ideation,
lability and pulse environment, reality
agitation
orientation, support.

Hallucinogens
LSD, mescaline Disordered Not reported Bad trip, high Reduced LOC Same as PCP, plus talk
perceptions, anxiety to panic; client down.
depersonal- hallucinations
ization may occur long
after drug has
been
metabolized;
flashbacks may
produce long-
lasting psychotic
diorders

(continues)

PSYCHIATRIC-MENTAL HEALTH NURSING

7 643
53155_07_Ch07_p621-684.qxd 2/27/09 6:29 PM Page 644

Table 7-5 Commonly Abused Drugs (continued)

Assessment Nursing Interventions


Drug Effect Dependence Findings Overdose for Overdose

Cannabis
Marijuana, hashish, Euphoria, Not reported Increased pulse Panic reaction, In panic, talk down.
THC intense rate and nausea, vomiting,
perceptions, appetite; depression and In severe depression,
relaxation, impaired disorders of institute suicide
lethargy judgment perception precautions.
and coordination
Benzodiazepines
Anti-anxiety drugs, Reduction in Physical: Calm effect unless Mild sedation to Support vital body
muscle relaxants: anxiety; dependence is drug withdrawn stupor functions.
clonazepam, anticonvulsant, low with oral abruptly dependent on Provide
diazepam, and reduces dosing Mild withdrawal dose nonthreatening
others muscle Psychologic: including CNS depression, environment.
spasms, withdrawal confusion, sedation to Administer Narcan as
reduces syndrome may anterograde stupor, dose ordered. Must be
insomnia resemble an amnesia dependent closely monitored.
anxiety disorder (impaired Oral unlikely to
Must differentiate recall of events cause significant
withdrawal after dosing), respiratory
syndrome from anxiety, depression
anxiety disorders diaphoresis, without
tremors concomitant
Effect may agents such as
resemble alcohol
alcohol Intravenous may
intoxication cause severe
respiratory
depression and
death

D. Have decreased feelings of anxiety. 11. Involve staff in negotiating care plan revisions.
E. Receive information and consider help for C. Rehabilitation/longer-term care
substance-abuse disorder (e.g., AA or NA). 1. Provide nonthreatening environment.
2. Set limits on unacceptable behavior.
Interventions 3. Provide adequate diet and fluids.
4. Provide information relating to substance
A. Assess drug use pattern: identity, recent use, and abuse and rehabilitation programs.
frequency of use of prescription and nonprescription
drugs, other substances (e.g., alcohol, nicotine).
B. Support client during acute phase of detoxification Evaluation
or withdrawal. A. Client experiences no injury.
1. Stay with client; reassure that current B. Vital signs are stable.
manifestations are temporary. C. Withdrawal proceeded without symptoms; client
2. Monitor vital signs, level of consciousness. remains drug/alcohol free.
3. Institute suicide precautions (if appropriate). D. Client can discuss substance-abuse problem and
4. Administer medications (to prevent requests or agrees to consider rehabilitation/
withdrawal) as ordered. therapy for problem.
5. If client is experiencing panic, talk down,
possibly with assistance of family/friends.
6. If client is hallucinating, reinforce reality, Specific Disorders
speak in a calm voice. Alcohol Abuse/Dependence
7. Confront clients use of denial.
8. Monitor your own responses of sympathy/anger. A. General information
9. Be aware of transference/countertransference. 1. Alcohol is a legal substance and there are
10. Maintain course of action in plan of care; millions of social drinkers.

7
client must follow plan.

644 NCLEX-RN Review


53155_07_Ch07_p621-684.qxd 2/27/09 6:29 PM Page 645

2. Alcohol is classified as a central nervous b. Delirium tremens (DTs)


system depressant. 1) History of alcohol abuse usually for
3. Alcohol abuse/dependence is a major problem more than 5 years.
in this country with over 18 million adults 2) May be preceded by seizures.
identified as alcohol abusers (see Table 7-6) 3) Symptoms occur 23 days after
4. Only approximately 5% of alcohol abusers are alcohol reduced/discontinued.
the skid row type. 4) Signs include tachycardia, increased
5. Incidence is increasing in women and blood pressure, agitation, delusions,
adolescents. hallucinations.
6. Considered a disease that can be arrested but c. Alcohol hallucinosis: hallucinations only
not cured. d. Alcohol-related dementia: caused by poor
7. Important to assess history of alcohol nutrition
consumption for clients admitted to hospital 1) Korsakoffs psychosis is sometimes
for nonalcohol-related disorders, because preceded by Wernickes
they may go into withdrawal. encephalopathy. Confusion and ataxia
8. Socioeconomic as well as a physiologic are predominant symptoms.
problem, resulting in increased health care 2) Thiamine deficiency results in
costs and loss of productivity if ability to Korsakoffs dementia/psychosis;
maintain a job is impaired. symptoms include chronic
9. Alcohol used with other substances disorientation, confabulation. It is
(barbiturates, antianxiety drugs) may have irreversible.
lethal consequences. 3) Large doses of thiamine may prevent the
10. Long-term use may result in loss of health development of Korsakoffs psychosis.
(gastritis, pancreatitis, cirrhosis, hepatitis, B. Medical management
malnutrition, cardiac and neural disorders) 1. Vitamin and nutrition therapy
and life (suicide, automobile accidents). 2. Antianxiety drugs (Librium or Ativan)
11. Directly related problems include withdrawal, 3. Disulfiram (Antabuse)
delirium tremens, and alcohol-related a. Produces unpleasant reaction (thirst,
dementia sweating, palpitations, vomiting, dyspnea,
a. Withdrawal respiratory and cardiac failure) when taken
1) Alcohol consumption with alcohol.
reduced/discontinued following b. 500 mg/day for 12 weeks; usual
continuous consumption for many maintenance dose is 250 mg/day.
days or longer c. Duration of action is 12 to several hours; no
2) Withdrawal is progressive and has four alcohol should be taken at least 12 hours
stages: before taking drug.
I: At least 8 hours after last drink; d. Increases effects of antianxiety drugs and
symptoms include mild tremors, oral anticoagulants.
tachycardia, increased blood
pressure, diaphoresis, nervousness
II: Gross tremors, hyperactivity, Table 7-6 Phases of Alcohol Addiction
profound confusion, loss of
appetite, insomnia, weakness, Phase Features
disorientation, illusions, auditory
and visual hallucinations Prealcoholic Drink almost every day to reduce tension
III: 1248 hours after last drink: Increase in amount of alcohol ingested
symptoms include (in addition to Addiction Blackouts
those found in I and II) severe Secret drinking
hallucinations, grand mal seizures Large amounts ingested
IV: 35 days after last drink (2472 Dependence Physical craving for alcohol
hours if untreated): delirium Makes up reasons for drinking
tremens, confusion, agitation, Reduced nutrition
severe psychomotor activity, Aggressive behavior
hallucinations, insomnia, Pressure from family and/or employer
tachycardia to reduce/stop drinking
3) Withdrawal may last less than a week
or may evolve into alcohol withdrawal Chronic Long periods of intoxication
delirium (delirium tremens). Impaired thinking
4) 1015% mortality rate from Less alcohol produces sedation tremors

7
hypoglycemia/electrolyte imbalances.

PSYCHIATRIC-MENTAL HEALTH NURSING 645


53155_07_Ch07_p621-684.qxd 2/27/09 6:29 PM Page 646

e. Side effects include headache, dry mouth, d. Cocaine, hydrochloride cocaine (crack)
somnolence, flushing. e. Phencyclidine (PCP)
f. Nursing responsibilities f. Hallucinogens: LSD, mescaline, DMT
1) Teach client the nature of severe g. Cannabis: marijuana, hashish, THC
reaction and importance of avoiding B. Assessment findings and nursing interventions for
all alcohol (including cough medicine, overdoses vary with particular drug; see Table 7-5
foods prepared with alcohol, etc.). C. Polydrug abusers
2) Teach client to carry an identification 1. Common pattern of drug use.
card in case of accidental alcohol 2. Synergistic effect: drugs interact so that effect
ingestion. is greater than if each drug is taken separately.
3) Monitor effects of antianxiety drugs if 3. Additive effect: two or more drugs with same
being taken at the same time. action are taken together (e.g., barbiturates
4) Monitor for bleeding if taking oral with alcohol will result in heavy sedation).
anticoagulants.
4. High doses of chlordiazepoxide (Librium) to Impaired Nurses
control withdrawal in acute detoxification. A. General information
C. Assessment findings 1. Most nursing licenses are suspended or
1. Dependent personality; often using denial as a revoked for substance abuse while on duty.
defense mechanism 2. Substances include alcohol and/or prescription
2. Tendency to minimize and underreport drugs stolen from unit drug stocks.
amount of alcohol consumed 3. Stealing drugs may result in criminal
3. Intoxication: blood alcohol level 0.15 (150 mg prosecution.
alcohol/100 mL blood). Legal level 0.080.10. 4. Work-related stress and easy access to drugs
4. Signs of impaired judgment, motor skills, and are factors relating to nurses substance abuse.
slurred speech 5. Substance use results in impaired judgment
5. Behavior may be boisterous, euphoric, and psychomotor abilities, resulting in unsafe
aggressive, or may be depressed, withdrawn nursing practice.
6. Signs of withdrawal, DTs, or alcohol-related B. Assessment of impairment
dementias 1. Alcohol odor on breath
D. Nursing interventions 2. Frequent lateness/absences
1. Stay with client. 3. Shortages in narcotics
2. Monitor vital signs and blood sugar levels. 4. Clients do not experience pain relief after
3. Observe for tremors, seizures, increased receiving pain reduction medication from
agitation, anxiety, disorders of perception. nurse
4. Administer medications as ordered; observe 5. Nurse makes frequent trips to bathroom/locker
effects/side effects of tranquilizers carefully. room
5. If disorders of perception occur, explain that 6. Changes in locomotion, psychomotor skills,
these are part of the withdrawal process. pupil size, and mood/affect
6. Provide fluids, adequate nutrition, and quiet C. Nurses responsibilities related to impaired nurse
environment. colleague
7. When client is stable, provide information 1. Client safety is first priority.
about rehabilitation programs (Alcoholics 2. ANA code of ethics (and most state laws)
Anonymous); at this stage client may be require nurse to safeguard clients.
willing to consider a program to stop drinking. 3. Interventions for suspected substance abuse by
8. Provide information about Alanon (for spouse coworker
and adult family members), Alateen (for a. Obtain information about legal issues,
children), and ACOA (for adult children of treatment options, and institutional policies.
alcoholics). b. Document observations related to
Psychoactive Drug Use behaviors and narcotic charting.
c. If possible, have other coworkers verify
A. General information your information.
1. Drugs abused may be prescription or street d. Arrange meeting with peer(s), nurse,
drugs supervisor, nurse advocate (where possible)
2. Types of drugs frequently abused and confront nurse with documentation.
a. Barbiturates, antianxiety drugs, hypnotics e. Let nurse know you care and will help.
b. Opioids (narcotics): heroin, morphine, f. Help nurse work through denial.
meperidine, methadone, hydromorphone g. Provide plan to offer recovery program
c. Amphetamines (speed): amphetamine, (e.g., include recovering nurse buddy).
dextroamphetamine, methamphetamine, h. Offer hope, support (moral and financial)

7
some appetite suppressants to aid nurse in treatment.

646 NCLEX-RN Review


53155_07_Ch07_p621-684.qxd 2/27/09 6:29 PM Page 647

i. Explain institutional policies regarding B. Any changes in thoughts, speech, affect


future employment. C. Ability to perform self-care activities, nutritional
j. If nurse continues to deny substance abuse, deficits
consider following steps: D. Suicide potential
1) Advocate should protect nurses rights. E. Aggression
2) Suspension/dismissal from job. F. Regression
3) Report to licensing board. G. Impaired communication
4) If theft of drug from unit has occurred,
report to law enforcement agency. Analysis
Nursing diagnoses for clients with schizophrenic
SCHIZOPHRENIA AND OTHER disorders may include:
A. Anxiety
PSYCHOTIC DISORDERS B. Disturbed sensory-perceptual
C. Disturbed sleep pattern
Overview D. Disturbed thought process
E. Imbalanced nutrition; less than body requirements
A. Characterized by disordered thinking, delusions, F. Impaired verbal communication
hallucinations, depersonalization (feeling of being G. Ineffective coping
strange, not oneself), impaired reality testing H. Low self-esteem
(psychosis), and impaired interpersonal relationships. I. Powerlessness
B. Regression to the earliest stages of development is J. Risk for injury
often noted (e.g., incontinence, mutism). K. Risk for violence
C. Onset is usually in adolescence/early adulthood L. Self-care deficit
(15 to 35 years of age). M. Social isolation
D. Client may be seriously impaired and unable to
perform ADLs.
E. Etiology is not known; theories include:
Planning and Implementation
1. Genetic: 1% of population. Goals
2. Biochemical: neurotransmitter dysfunction
i.e., dopamine, serotonin. Client will:
3. Interaction of predisposing risk and A. Develop a trusting/therapeutic relationship with
environmental stress. nurse.
F. Prior to onset (premorbid) client may have been B. Be oriented, able to test reality.
suspicious, eccentric, or withdrawn. C. Be protected from injury.
D. Be able to recognize impending loss of control.
E. Adhere to medication regimen.
Classifications
F. Participate in activities.
A. Disorganized: incoherent; delusions are not G. Increase ability to care for self.
organized; social withdrawal; affect blunted, silly,
or inappropriate Interventions
B. Catatonic: psychomotor disturbances
1. Stupor: mute, little reaction or movement A. Offer self in development of therapeutic
2. Excitement: purposeless, excited motor activity relationship.
3. Posturing: voluntary, inappropriate, bizarre B. Use silence.
postures C. Set time for interaction with client.
C. Paranoid: delusions and hallucinations of D. Encourage reality orientation but understand that
persecution/grandeur delusions/hallucinations are real to client.
D. Undifferentiated: disorganized behaviors, E. Assist with feeding/dressing as necessary.
delusions, and hallucinations F. Check on client frequently, remove potentially
harmful objects.
G. Contract with client to tell you when anxiety is
Assessment becoming so high that loss of control is possible.
A. AssessFour As H. Administer antipsychotic medications as ordered
1. Affect: flat, blunted (see Table 7-7 for side effects and dosages); observe
2. Associative looseness: verbalizations are for effects.
disorganized 1. Reduction of hallucinations, delusions,
3. Ambivalence: cannot choose between agitation
conflicting emotions 2. Postural hypotension
4. Autistic thinking: thoughts on self, extreme a. Obtain baseline blood pressure and

7
withdrawal, unable to relate to outside world monitor sitting/standing.

PSYCHIATRIC-MENTAL HEALTH NURSING 647


53155_07_Ch07_p621-684.qxd 2/27/09 6:29 PM Page 648

Table 7-7 Antipsychotic Medications

Dosages
Drug Classification Acute Symptoms Maintenance/Day Range/Day Significant Side Effects
Chlorpromazine 25100 mg IM 200600 mg PO 252000 mg PO Sedation
(Thorazine) q14h prn Anticholinergic effects: dry mouth,
blurred vision, constipation,
urinary retention, postural
hypotension
Fluphenazine HCl 1.25 mg IM, max 15 mg PO 130 mg PO Extrapyramidal effects
(Prolixin) 10 mg IM,
divided doses
Fluphenazine 25 mg IM q2wk 25100 mg IM Extrapyramidal
decanoate/enanthate
(Prolixin)
Trifluoperazine 12 mg IM q4h; 24 mg PO 280 mg PO Extrapyramidal
(Stelazine) 24 mg PO, max
10 mg qd
Haloperidol (Haldol) 210 mg IM in 28 mg PO 1100 mg PO Extrapyramidal
divided doses
Thiothixene (Navane) 816 mg IM in 610 mg PO 660 mg PO Extrapyramidal
divided doses
Loxapine (Loxitane) 60100 mg PO 30250 mg PO Extrapyramidal
Olanzapine (Zyprexa) 1020 mg PO 520 mg PO 2.520 mg PO Sedation, weight gain, increased
glucose and lipid levels
Quetepine (Seroquel) 400800 mg PO 200600 mg PO 25800 mg PO Sedation, may accelerate cataract
formation
Ziprasidone (Geodon) 4080 mg PO BID 2080 mg PO BID 2080 mg PO Nausea, anxiety, insomnia (transient);
with food or BID 1040 mg QTC prolongation
10-20 mg IM BID IM
Aripiprazole (Abilify) 1030 mg PO 1030 mg PO 1030 mg PO Nausea, insomnia
Clozapine (Clozaril) 300450 mg PO 75700 mg PO Agranulocytosis; sedation

Risperidone (Risperdal) 26 mg PO 26 mg PO 0.258 mg Increased prolactin levels, EPS at


PO higher doses, sedation

b. Client must lie prone for 1 hour following 6. Sedation


injection. a. Avoid use of heavy machinery.
c. Teach client to sit up or stand up slowly. b. Do not drive.
d. Elevate clients legs while seated. 7. Extrapyramidal symptoms (EPS)
e. Withhold drug if systolic pressure drops a. Dystonic reactions
more than 2030 mm Hg from previous 1) Sudden contractions of face, tongue,
reading. throat, extraocular muscles
3. Photosensitivity 2) Administer antiparkinson agents prn
a. Advise use of sun screen. (e.g., benztropine [Cogentin] 18 mg PO,
b. Avoid exposure to sunlight. IM, IV; diphenhydramine [Bendadryl]
4. Agranulocytosis 1050 mg PO, IM, IV; trihexyphenidyl
a. Instruct client to report sore throat or fever. [Artane] 315 mg PO only).
b. Institute reverse isolation if necessary. 3) Remain with client; this is a
5. Elimination frightening experience and usually
a. Measure I&O. occurs when medication is started.
b. Check bladder distension.

7
c. Keep bowel record.

648 NCLEX-RN Review


53155_07_Ch07_p621-684.qxd 2/27/09 6:29 PM Page 649

b. Parkinson syndrome B. Bipolar disorder: components of both depression


1) Occurs within 13 weeks and elation (formerly called manic-depression)
2) Tremors, rigid posture, masklike facial C. Cyclothymic disorder: milder symptoms of both
appearance mania and depression, often separated by long
3) Administer antiparkinson agents prn. periods of normal mood
c. Akathisia D. Dysthymic disorder: long-standing symptoms of
1) Motor restlessness depression alternating with short periods of
2) Need to keep moving normal mood; client usually able to maintain roles
3) Administer antiparkinson agents. in job, school, etc.
4) Do not mistake this for agitation; do E. Etiology is unknown; theories include:
not increase antipsychotic medication. 1. Genetic: approximately 7% of general
5) Reduce medications to see whether population; risk is 20% if a close relative has
symptoms decrease. depression
6) Determine if movement is under 2. Biochemical: dysregulation in norepinephrine
voluntary control. and serotonin
d. Tardive dyskinesia 3. Psychoanalytic: anger turned inward (i.e.,
1) Involuntary movements of tongue, anger toward significant other is turned into
face, extremities anger toward self)
2) May occur after prolonged use of
antipsychotics
e. Neuroleptic malignant syndrome
Assessment
1) Occurs days/weeks after initiation of A. Mood: dysphoric; blue/sad or elated/aggressive
treatment in 1% of clients B. Presence of psychomotor agitation, retardation, or
2) Elevated vital signs, rigidity, and hyperactivity
confusion followed by incontinence, C. Disorders of cognition: narrowed perception and
mutism, opisthotonos, retrocollis, interests, impaired concentration, grandiose
renal failure, coma, and death delusions, flight of ideas in elation stage
3) Discontinue medication, notify D. Sexual functioning changes
physician, monitor vital signs, E. Appropriateness of appearance/dress
electrolyte balance, I&O F. Appetite
f. Elderly clients should receive doses reduced G. Potential for suicide
by one-half to one-third of recommended level
I. Encourage participation in milieu, group, art, and
occupational therapies when client able to tolerate
Analysis
them. Nursing diagnoses for clients with affective disorders
may include:
A. Constipation
Evaluation B. Impaired verbal communication
Client C. Ineffective coping
A. Stays with nurse prescribed period of time. D. Risk for injury
B. Is oriented to reality, can state name, place, and E. Imbalanced nutrition: less than body requirements
date. F. Self-care deficit
C. Can feed/dress self with specified amount of G. Disturbed self-esteem
assistance. H. Disturbed sleep pattern
D. Has not attempted/will not attempt to injure self or I. Disturbed thought processes
others.
E. Adheres to medication regimen with minimal side
effects.
Planning and Implementation
F. Participates in activities. Goals
Client will:
MOOD DISORDERS A. Be protected from injury.
B. Receive adequate rest and sleep.
C. Maintain adequate intake of fluids and nutrients,
Overview regular elimination.
A. Characterized by disturbance in mood (affect) that D. Develop trusting/therapeutic relationship with
is either depression or elation (mania); occur in nurse.
a variety of patterns, alone or together (see E. Be oriented to reality.
Figure 7-1). Disturbance is beyond normal range F. Participate in planned activities.

7
of mood experienced by most people.

PSYCHIATRIC-MENTAL HEALTH NURSING 649


53155_07_Ch07_p621-684.qxd 2/27/09 6:29 PM Page 650

Figure 7-1 Patterns of mood disturbances in affective disorders

7 650 NCLEX-RN Review


53155_07_Ch07_p621-684.qxd 2/27/09 6:29 PM Page 651

Interventions b. Maintenance dosage levels: 300 mg


TID/QID, to maintain a blood serum level
A. Assess for suicide potential. of 0.41.0 mEq/liter; checked monthly.
B. Encourage verbalization of feelings of hopelessness c. Toxicity when blood levels higher than
and helplessness. 2.0 mEq/liter: tremors, nausea and
C. Provide quiet environment for rest and sleep. vomiting, thirst, polyuria, coma, seizures,
D. Provide small, attractive meals; encourage intake cardiac arrest
of fluids. 2. Antipsychotics may also be given for
E. Maintain bowel record. hyperactivity, agitation, psychotic behavior.
F. Use silence and broad openings, focus on clients Chlorpromazine (Thorazine) and haloperidol
verbal/nonverbal behaviors. (Haldol) are most commonly used (see
G. Present reality but accept clients need for Table 7-7).
delusions. D. Nursing interventions
H. Accept clients negative responses, hostility. 1. Determine what client is attempting to tell
I. Provide activities and tasks to raise clients you; use active listening.
self-esteem. 2. Assist client in focusing on a topic.
J. Assist with self-care as needed. 3. Offer finger foods, high-nutrition foods, and
K. If client is agitated fluids.
1. Work with client on a one-to-one basis. 4. Provide quiet environment, decrease stimuli.
2. Walk with client; provide some diversional 5. Stay with client, use silence.
activity. 6. Remove harmful objects.
3. Reduce environmental stimuli (e.g., quiet 7. Be accepting of hostile statements.
room, dim lights). 8. Do not argue with client.
9. Use distraction to divert client from behaviors
Evaluation that are harmful to self or others.
10. Administer medications as ordered and
Client observe for effects/side effects.
A. Has gained or maintained weight. a. Teach clients early signs of toxicity.
B. Reports any suicidal ideation. b. Maintain fluid and salt intake.
C. Sleeps a specified number of hours. c. Avoid diuretics.
D. Can meet own needs for ADLs. d. Monitor lithium blood levels.
E. Has realistic appraisal of self. 11. Assist in dressing, bathing.
12. Set limits on disruptive behaviors.
Specific Disorders
Major Depression
Bipolar Disorder (Manic Episode)
A. General information
A. General information 1. Characterized by loss of ambition, lack of
1. Onset usually before age 30 interest in activities and sex, low self-esteem,
2. Characterized by hyperactivity and euphoria and feelings of boredom and sadness.
that may become sarcasm or hostility 2. Etiology may be physiologic or response to an
B. Assessment findings actual or perceived loss.
1. Hyperactivity to the point of physical 3. These clients are at high risk for suicide,
exhaustion especially when depressed mood begins to lift
2. Flamboyant dress/makeup and/or energy level increases.
3. Sexual acting out B. Medical management (see Table 7-8)
4. Impulsive behaviors 1. Tricyclic antidepressants: amitriptyline HCl, etc.
5. Flight of ideas: inability to finish one thought 2. Monoamine oxidase inhibitors (MAOIs):
before jumping to another isocarboxazid (Marplan), etc.
6. Loud, domineering, manipulative behavior 3. Atypical antidepressants: fluoxetin (Prozac),
7. Distractibility sertraline (Zoloft), etc.
8. Dehydration, nutritional deficits 4. Electroconvulsive therapy (ECT)
9. Delusions of grandeur C. Assessment findings
10. Possible short-term depression (risk of suicide) 1. Feelings of helplessness, hopelessness,
11. Hostility, aggression worthlessness
C. Medical management 2. Reduction in normal activities or agitation
1. Lithium carbonate (Eskalith, Lithobid, Lithotabs) 3. Slowing of body functions/elimination
a. Initial dosage levels: 600 mg TID, to maintain 4. Loss of appetite
a blood serum level of 1.01.5 mEq/liter; 5. Inappropriate guilt

7
blood serum levels should be checked 12 6. Self-deprecation, low self-esteem
hours after last dose, twice a week.

PSYCHIATRIC-MENTAL HEALTH NURSING 651


53155_07_Ch07_p621-684.qxd 2/27/09 6:29 PM Page 652

Table 7-8 Antidepressant Medications

Drug Initiating Dosage Maintenance Side Effects


SSRIs
Fluoxetine (Prozac, 20 mg PO 2040 mg/day or Sexual dysfunction; nausea, diarrhea,
Prozac Weekly) 90 mg once per headache, anxiety (transient)
week (once stable)
Sertraline (Zoloft) 50 mg PO 50200 mg/day As above
Paroxetine (Paxil, 20 mg PO 2040 mg/day As above
Paxil CR) 25 mg PO 2550 mg/day
Citalopram (Celexa) 20 mg PO 2080 mg/day Sexual dysfunction; nausea,
headache, nervousness
(transient)
Escitalopram (Lexapro) 10 mg PO 1020 mg/day As above
Fluvoxamine (Luvox) 50 mg PO HS 50300 mg HS Tiredness, sexual dysfunction; headache,
nausea, nervousness (transient) (used in
treatment of OCD)
Atypical Antidepressants
Bupropion (Wellbutrin SR) 100 mg PO 150 mg BID Anxiety, insomnia; can lower seizure
threshold in overdose
Mirtazapine (Remeron) 15 mg PO HS 3045 mg HS Sedation at 15 mg (less at higher doses);
increased appetite
Nefazodone (Serzone) 50 mg PO HS 50600 mg HS Sedation, dry mouth, postural hypotension;
liver toxicity
Venlafaxine (Effexor XR) 37.5 mg PO 75375 mg Sexual dysfunction; headache, nausea,
nervousness; can increase BP
at doses > 300 mg/day
Trazodone (Desyrel) 150 mg PO in 150400 mg PO in Sedation, anxiety, hypotension, priapism
divided doses divided doses (commonly used as sleep aid)
Tricyclics
Amitriptyline (Elavil, 75100 mg PO 50150 mg PO at Constipation, blurred vision, drowsiness,
Endep) bedtime; 80100 mg orthostatic hypotension, urinary retention,
IM in divided doses dry mouth, increased appetite, sexual
dysfunction
Doxepine (Apo-Doxepin, 25 mg PO TID 75150 mg/day Sedation, confusion, constipation,
Novo-Doxepin) (or up to 150 mg decreased libido
can be given at
bedtime)
Clomipramine (Anafranil) 75250 mg PO in 50150 mg PO at HS As above
divided doses
at HS
Desipramine (Norpramin) 50 mg HS 100150 mg HS As above
Nortriptyline 50 mg HS 50125 mg HS As above
Monoamine Oxidase Inhibitors (MAOIs)
Isocarboxazid (Marplan) 30 mg PO in 1030 mg PO As for tricyclics, plus angina, hypoglycemia,
divided doses hypertensive crisis precipitated by ingestion
of foods with tyramine or concurrent use of
tricyclics
Phenalizine (Nardil) 60 mg PO in 3060 mg PO As above
divided doses

7 652 NCLEX-RN Review


53155_07_Ch07_p621-684.qxd 2/27/09 6:29 PM Page 653

7. Inability to concentrate, disordered thinking d. Check vital signs after the procedure.
8. Poor hygiene e. Reorient and assure that any memory loss
9. Slumped posture is temporary.
10. Crying, ruminating (relates same incident over f. Assist to room or to care of responsible
and over) party if outpatient.
11. Dependency
12. Depressed children: possible separation Dysthymic Disorder
anxiety
13. Elderly clients: possible symptoms of A. General information: chronic mood disturbance of at
dementia least 2 years duration for adults, 1 year for children
14. Somatic and persecutory delusions and B. Assessment findings
hallucinations 1. Normal moods for a period of weeks, followed
D. Nursing interventions by depression
1. Monitor I&O. 2. Insomnia/hypersomnia
2. Weigh client regularly. 3. Social withdrawal
3. Maintain a schedule of regular appointments. 4. Loss of interest in activities
4. Remove potentially harmful articles. 5. Recurrent thoughts of suicide and death
5. Contract with client to report suicidal ideation, C. Nursing interventions: same as for major
impulses, plans; check on client frequently. depression.
6. Assist with dressing, hygiene, and feeding.
7. Encourage discussion of negative/positive
aspects of self. NEUROTIC DISORDERS
8. Encourage change to more positive topics if
self-deprecating thoughts persist. In DSM-IV-TR, the disorders formerly categorized as
9. Administer antidepressant medications (see neurotic disorders are included in Anxiety,
Table 7-8) as ordered. Somatoform, and Dissociative Disorders. Reality
a. Tricyclic antidepressants (TCAs) testing is intact.
1) Effectiveness increased by
antihistamines, alcohol,
benzodiazepines ANXIETY DISORDERS
2) Effectiveness decreased by
barbiturates, nicotine, vitamin C Overview
b. Monoamine oxidase inhibitors (MAOIs)
1) Effectiveness increased with A. Common element is anxiety, manifested in a
antipsychotic drugs, alcohol, variety of behaviors (see also Behaviors Related to
meperidine Emotional Distress).
2) Avoid foods containing tyramine (e.g., B. Therapy relates to reduction of anxiety; when
beer, red wine, aged cheese, avocados, anxiety is reduced, the symptoms will be
caffeine, chocolate, sour cream, alleviated.
yogurt); these foods or MAOIs taken C. Types include generalized anxiety disorder, panic
with TCAs may result in hypertensive disorder, phobic disorders, and obsessive-
crisis. compulsive disorders.
c. Be sure client swallows medication. If
side effects disappear suddenly, cheeking/ Assessment
hoarding may have occurred. These
medications can be used to attempt suicide. A. Level of anxiety: may be to point of panic
d. Antidepressant medications do not take B. Vital signs: may be elevated
effect for 23 weeks. Encourage client to C. Reality testing: should be intact; can recognize that
continue medication even if not feeling thoughts are irrational but cannot control them
better. Be aware of suicide potential during D. Physical symptoms: no organic basis
this time. E. Memory: possible memory loss or loss of identity
e. Warn client not to take any drugs without F. Pattern of symptoms: chronic with a pattern of
consulting physician. waxing and waning or sudden onset
10. Assist with electroconvulsive therapy as
ordered. Analysis
a. Give normal pre-op preparation, including
informed consent (see Perioperative Nursing diagnoses for the client with an anxiety
Nursing). disorder may include:
b. Remove all hairpins, dentures. A. Anxiety

7
c. Ensure client is wearing loose clothing. B. Deprivation of sleep

PSYCHIATRIC-MENTAL HEALTH NURSING 653


53155_07_Ch07_p621-684.qxd 2/27/09 6:29 PM Page 654

C. Disturbed thought processes 3. Client can recognize that fear of these


D. Fear objects/situations is irrational, but cannot
E. Ineffective coping control emotional response when confronting
F. Ineffective tissue perfusion or thinking about confronting the particular
G. Powerlessness object/situation.
B. Assessment findings
1. Agoraphobia: most serious phobia; fear of
Planning and Implementation being alone or in public places; may reach
Goals point where client panics at thought of being
in public places and cannot leave home.
Client will: 2. Social phobias: fear of being in situations
A. Develop a trusting/therapeutic relationship with where one may be scrutinized and
nurse. embarrassed by others.
B. Recognize causes of anxiety and develop 3. Specific phobias: irrational fear of specific
alternative coping mechanisms. objects/situations (e.g., snakes, insects,
C. Reduce/alleviate symptoms of anxiety. heights, closed places).
C. Nursing interventions
Interventions 1. Know that behavior modification and systematic
desensitization most commonly used; client
A. Encourage discussion of anxiety and relationship
cannot be reasoned out of behavior.
to symptoms.
2. Do not force contact with feared
B. Provide calm, accepting atmosphere.
object/situation; may result in panic.
C. Administer antianxiety medications (for short-term
3. Administer benzodiazepines (alprazolam or
use only) as ordered and monitor effects/side
clonazepam), SSRIs, venlafaxine, or buspirone
effects.
as ordered.
1. Diazepam (Valium): 520 mg PO daily;
4. Instruct in and encourage use of relaxation
210 mg IM or IV daily
techniques.
2. Chlordiazepoxide (Librium): 20100 mg PO
daily; 50100 mg IM or IV daily
3. Alprazolam (Xanax) 0.754 mg PO daily Generalized Anxiety Disorder
4. Oxazepam (Serax) 30120 mg PO daily A. General information
5. Triazolam (Halcion) 0.250.5 mg, PO HS 1. Persistent anxiety for at least 1 month
6. Side effects 2. Cannot be controlled by client or displaced,
a. Client may become addicted. remains free-floating and diffuse
b. Additive effect with alcohol. B. Assessment findings
c. Dizziness may occur when treatment 1. Motor tensions: trembling, muscle aches,
initiated. jumpiness
d. Lower doses for elderly client. 2. Autonomic hyperactivity: sweating,
e. Do not stop abruptly; taper doses. palpitations, dizziness, upset stomach,
D. Teach client about self-medication regimen and increased pulse and respirations
side effects. 3. Affect: worried and fearful of what might
happen
Evaluation 4. Hyperalert: insomnia, irritability
C. Nursing interventions
Client 1. Stay with client.
A. Can discuss causes of anxiety with nurse. 2. Encourage discussion of anxiety and its source.
B. Demonstrates constructive coping mechanisms 3. Provide calm, relaxing atmosphere.
and ability to reduce anxiety. 4. Administer antianxiety drugs, as ordered.
C. Demonstrates knowledge of effects and hazards of 5. Observe for effects and side effects.
antianxiety medications. 6. Monitor vital signs.
7. Assess for level of anxiety.
Specific Disorders
Panic Disorder (with/without Agoraphobia)
Phobic Disorders
A. General information: acute, panic-like attack
A. General information lasting from a few minutes to an hour.
1. Irrational fears resulting in avoidance of B. Assessment findings
objects or situations. 1. Sudden onset of intense fear/terror
2. Repressed conflicts are projected to outside 2. Symptoms: include dyspnea, palpitations,
world and eventually are displaced onto an

7
chest pain, sensation of smothering or
object or situation. choking, faintness, fear of dying, dizziness

654 NCLEX-RN Review


53155_07_Ch07_p621-684.qxd 2/27/09 6:29 PM Page 655

3. When severe, symptoms mimic acute cardiac 4. Symptoms of depression


disease that must be ruled out. 5. Possible violent outbursts
4. Client may be seen in ER. 6. Memory impairment
C. Nursing interventions: same as for generalized 7. Panic attacks
anxiety disorder. 8. Substance abuse
C. Nursing interventions
Obsessive-Compulsive Disorder (OCD) 1. Arrange for individual or group psychotherapy
with others who experienced same trauma
A. General information (e.g., Iraq or Vietnam war veterans).
1. Obsession 2. Provide crisis counseling, family therapy as
a. Recurrent thoughts that client cannot needed.
control; often violent, fearful, or doubting 3. Provide referrals.
in nature (e.g., fear of contamination).
b. Client cannot keep thoughts from intruding
into consciousness; eventually resort to SOMATOFORM DISORDERS
defense of undoing (performing ritual
behavior).
2. Compulsion Overview
a. Action (ritual behavior) that serves to A. Anxiety is manifested in somatic (physical)
reduce tension from obsessive thought. symptoms.
b. Client may not desire to perform behavior B. There is organic pathology but no organic etiology.
but is unable to stop, as this is the only C. Symptoms are real and not under voluntary
relief from distress. control of the client.
c. May interfere with social/occupational D. Defense used is somatization or conversion:
functioning. anxiety is transformed to a physical symptom.
B. Nursing interventions
1. Allow compulsive behavior, but set reasonable
limits. Specific Disorders
2. Permit client to complete behavior once Somatization Disorder
started; aggression may result if behavior is not
allowed or completed. A. General information
3. Engage client in alternative behaviors (client 1. Multiple, recurrent somatic complaints
will not be able to do this alone). (fatigue, backache, nausea, menstrual cramps)
4. Provide opportunities to perform tasks that over many years
meet need for perfectionism (e.g., stacking and 2. No organic etiology for these complaints
folding linens). B. Assessment findings
5. As compulsive behavior decreases, help client 1. Complaints chronic but fluctuating
to verbalize feelings, concerns. 2. History of seeking medical attention for many
6. Help client to make choices, participate in years
decisions regarding own schedule. 3. Symptoms of anxiety and depression
7. Administer clomipramine (Anafranil) as 4. Somatic complaints may involve any organ
ordered. Gradual decrease in symptoms may system
take 23 months. Often used with behavior C. Nursing interventions
modification therapy (see Table 7-8). 1. Be aware of own response (irritation/
impatience) to client.
Post-Traumatic Stress Disorder (PTSD) 2. Rule out organic basis for current complaints.
3. Focus on anxiety reduction, not physical
A. General information symptoms.
1. Disturbed/disintegrated response to significant 4. Minimize secondary gain.
trauma
2. Symptoms can occur following crisis event Conversion Disorder
such as war, earthquake, flood, airplane crash,
rape, or assault A. General information
3. Reexperiencing of traumatic event in 1. Sudden onset of impairment or loss of motor
recollections, nightmares or sensory function.
B. Assessment findings 2. No physiologic cause.
1. Psychic numbing: not as responsive to persons 3. Defenses used are repression and conversion;
and events as to the traumatic experience anxiety is converted to a physical symptom.
2. Sleep disturbances (e.g., nightmares) 4. Temporal relationship between distressing
event and development of symptom

7
3. Avoidance of environment/activities likely to
arouse recall of trauma

PSYCHIATRIC-MENTAL HEALTH NURSING 655


53155_07_Ch07_p621-684.qxd 2/27/09 6:29 PM Page 656

(e.g., unconscious desire to hit another may 5. Avoid negative response to clients demands
produce paralysis of arm). by discussing in staff conferences.
5. Primary gain: client is not conscious of 6. Provide client with correct information.
conflict. Anxiety is converted to a symptom
that removes client from anxiety-producing
situation. DISSOCIATIVE DISORDERS
6. Secondary gain: gain support and attention
that was not previously provided. Tends to
encourage client to maintain symptoms.
Overview
B. Assessment findings A. Sudden change in clients consciousness, identity,
1. Sudden paralysis, blindness, deafness, etc. or memory.
2. La belle indiffrence: inappropriately calm B. Loss of memory, knowledge of identity, or how
when describing symptoms individual came to be in a particular place.
3. Symptoms not under voluntary control C. Defenses are repression and dissociation.
4. Usually short term; symptoms will abate as
anxiety diminishes
C. Nursing interventions
Specific Disorders
1. Focus on anxiety reduction, not physical Dissociative Amnesia
symptom.
2. Use matter-of-fact acceptance of symptom. A. General information: inability to recall information
3. Encourage client to discuss conflict. about self with no organic reason
4. Do not provide secondary gain by being too B. Assessment findings
attentive. 1. No history of head injury
5. Provide diversionary activities. 2. Retrograde amnesia, may extend far into
6. Encourage expression of feelings. past
C. Nursing interventions
1. Rule out organic causes.
Pain Disorder 2. Reassure client that personal identity will be
made known to client.
A. General information: complaint of severe and 3. Provide safe environment.
prolonged pain 4. Establish nurse-client relationship to reduce
B. Assessment findings anxiety.
1. Pain impairs social/occupational function
2. Pain often severe
3. Sleep may be interrupted by experience of
Dissociative Fugue
pain A. General information
C. Nursing interventions 1. Client travels to strange, often distant place;
1. Pain management unaware of how he traveled there, and unable
2. Encourage participation in activities. to recall past.
2. May follow severe psychologic stress.
B. Assessment findings
Hypochondriasis 1. Memory loss
A. General information 2. May have assumed new identity
1. Unrealistic belief of having serious 3. No recall of fugue state when normal functions
illnesses. return
2. Belief persists despite medical reassurance. C. Nursing interventions: same as for psychogenic
3. Defenses used are regression and somatization. amnesia.
B. Assessment findings
1. Preoccupation with bodily functions, which
are misinterpreted. PERSONALITY DISORDERS
2. History of seeing many doctors, many
diagnostic tests. Note: This is coded on Axis II.
3. Dependent behavior: desires/demands great
deal of attention. Overview
C. Nursing interventions
1. Rule out presence of actual disease. A. Patterns of thinking about self and environment
2. Focus on anxiety, not physical symptom. become maladaptive and cause impairment in
3. Set limits on amount of time spent with client. social or occupational functioning or subjective
4. Reduce anxiety by providing diversionary distress.
B. Usually develop by adolescence.

7
activities.
C. Most common is borderline personality disorder.

656 NCLEX-RN Review


53155_07_Ch07_p621-684.qxd 2/27/09 6:29 PM Page 657

Specific Disorders
Sample Questions
Borderline Personality Disorder
A. General information: clients are impulsive and
21. A 6-year-old has been diagnosed with enuresis
unpredictable, have difficulty interacting;
after tests revealed no organic cause of bed
characterized by behavior problems
B. Assessment findings wetting. The childs mother is upset and blames
1. Unstable, intense interpersonal relationships the problem on his father. Its all his fathers
2. Impulsive, unpredictable, manipulative fault! What is your initial response?
behavior; prone to self-harm 1. Why do you say that?
3. Marked mood shifts from anger to dysphoric 2. Its usually nobodys fault.
4. Uncertainty about self-image, gender identity, 3. You seem really upset by this.
values
5. Chronic intolerance of being alone, feelings of 4. Why are you blaming his father?
boredom
22. An adolescent is admitted with anorexia
6. Splitting: distinct separation of love and hate;
nervosa. You have been assigned to sit with her
views others as all good or all bad.
while she eats her dinner. The client says to you,
7. Use of projection and regression
C. Nursing interventions My primary nurse trusts me. I dont see why
1. Protect from self-mutilation, suicidal you dont. What is your best response?
gestures. 1. I do trust you, but I was assigned to be with
2. Establish therapeutic relationship, be aware of you.
own responses to manipulative behaviors. 2. Id like to share this time with you.
3. Maintain objectivity. 3. OK. When I return, Ill check to see how
4. Use a calm approach. much you have eaten.
5. Set limits.
6. Apply plan of care consistently. 4. Who is your primary nurse?
7. Interact with clients when they demonstrate
23. A teenager is hospitalized for the treatment of
appropriate behavior.
anorexia nervosa. She is 64 inches tall and
8. Teach relaxation techniques.
weighs 100 pounds. What is the primary
objective in the treatment of the hospitalized
anorexic client?
Antisocial Personality Disorder
1. Decrease the clients anxiety.
A. General information 2. Increase insight into the disorder.
1. Chronic history of antisocial behaviors (e.g.,
3. Help the mother to relinquish control.
fighting, stealing, aggressive behaviors,
substance abuse, criminal behaviors). 4. Get the client to eat and gain weight.
2. These behaviors usually begin before the age
24. A female adolescent is hospitalized for treatment
of 15 and continue into adult life.
3. May be hospitalized for injuries. of anorexia nervosa. While admitting the client,
B. Assessment findings the nurse discovers a bottle of pills. She states
1. Manipulative behavior, may try to obtain they are antacids and she takes them because her
special privileges, play one staff member stomach hurts. What would be the nurses best
against another initial response?
2. Lack of shame or guilt for behaviors 1. Tell me more about your stomach pain.
3. Insincerity and lying 2. These do not look like antacids. I need to get
4. Impulsive behavior and poor judgment an order for you to have them.
C. Nursing interventions
3. Tell me more about your drug use.
1. Provide model for mature, appropriate
behavior. 4. Some girls take pills to help them lose
2. Observe strict limit-setting by all staff. weight.
3. Monitor own responses to client.
4. Demonstrate concern, interest in client. 25. The nurse assesses an adolescent who has
5. Reinforce positive behaviors (socialization, dropping grades, low motivation, somatic
conforming to limits). complaints, and dental caries. What disorder
6. Avoid power struggles. would the nurse suspect?
1. Anxiety.

7
2. Depression.

PSYCHIATRIC-MENTAL HEALTH NURSING 657


53155_07_Ch07_p621-684.qxd 2/27/09 6:29 PM Page 658

3. Acute mania. 3. Frequency of substances taken over the past 2


4. Dissociative fugue. weeks.
4. Frequency of substances taken over the past
26. An elderly client was recently admitted to a month.
nursing home because of confusion,
disorientation, and negativistic behavior. Her 31. What is a characteristic common to most
family states that she is in good health. The substance abusers that is difficult for them to
woman asks you, Where am I? What would be achieve?
the best response from the nurse? 1. Coping with stress and anxiety.
1. Dont worry. Youre safe here. 2. Interacting socially.
2. Where do you think you are? 3. Performing in work-related settings.
3. What did your family tell you? 4. Setting goals.
4. Youre at the community nursing home.
32. A client is developing impending alcohol
27. Which of the following would be an appropriate withdrawal delirium. Besides tremors, what
strategy in reorienting a confused client to where other signs and symptoms would be present?
her room is? 1. Bradycardia and hypertension.
1. Place pictures of her family on the bedside 2. Bradycardia and hypotension.
stand. 3. Tachycardia and hypertension.
2. Put her name in large letters on her 4. Tachycardia and hypotension.
wristband.
3. Remind the client where her room is. 33. What is the most widely accepted treatment
4. Let the other residents know where the modality for substance abuse?
clients room is. 1. Individual therapy with a psychodynamically
oriented therapist.
28. An elderly client was recently admitted to a 2. Individual therapy with a systems-oriented
nursing home because of confusion, therapist.
disorientation, and negativistic behavior. Which
3. Group therapy with others with personality
activity would you engage the client in at the
disorders.
nursing home?
4. Group therapy with other substance abusers.
1. Reminiscence groups.
2. Sing-alongs. 34. A client was voluntarily admitted to the inpatient
3. Discussion groups. unit with a diagnosis of paranoid schizophrenia.
4. Exercise class. As the nurse approaches the client, he says, If
you come any closer, Ill die. Which disorder of
29. A 78-year-old was recently admitted to a nursing perception does this client exhibit?
home because of confusion, disorientation, and 1. Hallucination.
negativistic behavior. She has had difficulty 2. Delusion.
sleeping since admission. Which of the
3. Illusion.
following would be the best intervention?
4. Idea of reference.
1. Provide her with a glass of warm milk.
2. Ask the physician for a mild sedative. 35. The nurse is approaching an adult client who is
3. Do not allow her to take naps during the day. admitted with a diagnosis of paranoid
4. Ask her family what they prefer. schizophrenia. As the nurse approaches the
client, he says, If you come any closer, Ill die.
30. A middle aged client is on the verge of losing his What is the best response for the nurse to make
job because of a drinking problem. He to this behavior?
voluntarily enters an alcohol detoxification 1. How can I hurt you?
program. Along with the amount and type, what 2. I am your nurse today.
information is most important that he needs to
3. Tell me more about this.
inform the staff?
4. Youre not going to die.
1. Time substances were taken over the past 24
hours. 36. A young man admitted with a diagnosis of
2. Frequency of substances taken over the past paranoid schizophrenia is pacing the halls and

7
week. is agitated. The nurse hears him saying, I have

658 NCLEX-RN Review


53155_07_Ch07_p621-684.qxd 2/27/09 6:29 PM Page 659

to get away from those doctors! They are trying 42. A 38-year-old was admitted to the psychiatric
to commit me to the state hospital! The nurses service after a failed suicide attempt by drug
continued assessment should include: overdose. The client sought help when her
1. Clarifying information with the doctor. husband informed her of his decision to leave
2. Observing the client for rising anxiety. her and the children after 19 years of marriage.
Her suicide attempt was made after she and her
3. Reviewing history of involuntary
husband had had a fierce argument about
commitment.
property settlement. Upon initial contact with
4. Checking dosage of prescribed medication. the nurse, the client looked exhausted, affect
was sad, movements and responses were
37. After 2 days in the hospital, the nurse assesses a
slowed, and self-care impairments were evident.
client diagnosed with schizophrenia as
She is convinced that a blemish on her face is a
exhibiting flat affect with little interest in other
melanoma that is invading her brain and eating
clients. What describes this characteristic of the
away at the tissue. What type of disorder is
schizophrenic process?
being shown?
1. Paranoia.
1. Bipolar disorder.
2. Ambivalence.
2. Depression with melancholia.
3. Cyclothymic.
3. Dysthymic disorder.
4. Undifferentiated.
4. Major depression.
38. What would be an appropriate activity for the
43. An adult is admitted to the psychiatric service
nurse to recommend for a client who is
after a failed suicide attempt by drug overdose.
extremely agitated?
She presents with a sad affect and moves and
1. Competitive sports. responds slowly. Which nursing diagnosis is of
2. Bingo. greatest priority at the time of her admission?
3. Trivial Pursuit. 1. Imbalanced in nutrition: less than body
4. Daily walks. requirements.
2. Ineffective coping.
39. A client who is diagnosed with a bipolar
3. Risk for violence: self-directed.
disorder is admitted to the hospital in the manic
phase. What is the initial plan of care? 4. Bathing/hygiene self-care deficit.
a. Put the client in seclusion. 44. An adult is admitted following a suicide
b. Put the client on one to one for safety. attempt. She took sleeping pills. She has been
c. Provide a quiet environment for the client. receiving therapy for depression since her
d. Stabilize the client on medication. husband left her after 23 years of marriage. Upon
admission she looks very tired, has a sad affect,
40. A 34-year-old is hospitalized with bipolar and moves slowly. What intervention would be a
disorder. At 2 A.M. the nurse finds him phoning priority in helping to stabilize the client?
friends all across the country to discuss his new 1. Allow her to catch up on lost sleep for the
plan for eradicating world hunger. His excited first 3 days of her hospitalization.
explanations are keeping the entire unit awake, 2. Have her fully involved in all therapeutic
but he wont quiet down. Which drug is most activities.
likely to be prescribed for this client?
3. Encourage her husband to visit for brief
1. A tricyclic antidepressant. periods of time.
2. An MAO-inhibitor antidepressant. 4. Schedule balanced periods of rest and
3. Lithium carbonate (Eskalith). therapeutic activity.
4. An antianxiety drug.
45. When a client is experiencing severe anxiety,
41. Which supportive therapy for a client who is what would be the priority nursing intervention?
exhibiting manic behavior would be 1. Give the client medication immediately.
inappropriate to use as treatment? 2. Offer the client psychotherapy to calm her
1. Psychoanalysis. down.
2. Cognitive therapy. 3. Isolate the client in a quiet environment.
3. Interpersonal therapy. 4. Put the client in seclusion temporarily.

7
4. Problem-solving therapy.

PSYCHIATRIC-MENTAL HEALTH NURSING 659


53155_07_Ch07_p621-684.qxd 2/27/09 6:29 PM Page 660

46. A client is admitted to the hospital because her the nurse that the hand and face washing was
family is unable to manage her constant quite repetitive and ritualistic. However, she
handwashing rituals. Her family reports she refused to bathe or wash her clothing. Which
washes her hands at least 30 times each day. The nursing diagnosis describes the most prominent
nurse noticed the clients hands are reddened, difficulty that the client is experiencing?
scaly, and cracked. What is the main nursing goal? 1. Impaired skin integrity.
1. Decrease the number of hand washings a day. 2. Disturbed thought processes.
2. Provide a milder soap. 3. Ineffective coping.
3. Provide good skin care. 4. Social isolation.
4. Eliminate the handwashing rituals.
51. An adult is admitted because of ritualistic
47. An adult is admitted to the psychiatric hospital behavior. She is also constipated and
for handwashing rituals. The day after dehydrated. Which nursing intervention would
admission she is scheduled for lab tests. How the client be most likely to comply with?
will the nurse ensure that the client is there on 1. Drinking Ensure between meals.
time? 2. Drinking extra fluids with meals.
1. Remind the client several times of her 3. Drinking 8 oz water every hour between
appointment. meals.
2. Limit the number of hand washings. 4. Drinking adequate amounts of fluid during
3. Tell her it is her responsibility to be there on the day.
time.
4. Provide ample time for her to complete her 52. An adult is admitted because of excessive hand
rituals. and face washing rituals. What would be the
most effective way for the nurse to intervene
48. An adult who is hospitalized with an obsessive- with her hand and face washing?
compulsive disorder washes her hands many 1. Allow her a certain amount of time each shift
times a day. Which of the following is an to engage in this behavior.
appropriate treatment for this client? 2. Interrupt the activity briefly and frequently.
1. An unstructured schedule of activities. 3. Lock the door to her room and restrict access
2. A structured schedule of activities. to the bathroom.
3. Intense counseling. 4. Tell her to stop each time she is observed
4. Negative reinforcement every time she doing it.
performs the ritual.
53. A client was admitted for ritualistic behavior
49. A woman is admitted to the psychiatric hospital. involving frequent hand and face washing. Upon
She was found walking on a highway. She is admission, the client was also dehydrated and
unkempt and appears thin and dirty. What is the underweight. When will the nurse know to
most thorough way to conduct a nursing initiate discharge planning for this client?
assessment of her nutritional status? 1. The clients normal body weight is regained.
1. Observe her at mealtime. 2. The client will express a desire to leave the
2. Request a medical consult. hospital.
3. Explore her recent dietary intake. 3. The client is able to start talking about her
4. Compare current weight with her usual guilt and anxiety.
weight. 4. The client limits her hand and face washing
to a few times a day.
50. A client is admitted to the psychiatric unit. She
was found wandering on a major four-lane 54. A young adult was admitted on a voluntary basis
highway and cannot recall her activities from to psychiatric services. During the last 3 years,
the past 3 days. During the assessment, the nurse he has been under psychiatric care and has a
observes that her face and hands are very red long history of petty crimes. Once on the unit,
and excoriated, her hair is matted and dirty, her the client is difficult to manage because he is
clothing is dirty, and she is quite thin. When the arrogant and manipulative. When a scheduled
client asked to be excused, she went directly to group therapy session is announced, he refuses
her room, and washed her hands and face. to go. He uses other clients to his own ends and
Within a very short while, it became apparent to often pioneers causes that are disruptive to the

7 660 NCLEX-RN Review


53155_07_Ch07_p621-684.qxd 2/27/09 6:29 PM Page 661

milieu. What diagnostic title best describes his fitting which of the following diagnostic
behavior? categories?
1. Antisocial personality disorder. 1. Antisocial personality disorder.
2. Borderline personality disorder. 2. Borderline personality disorder.
3. Somatization disorder. 3. Generalized anxiety disorder.
4. Bipolar disorder. 4. Post-traumatic stress disorder.

55. An adult is admitted to a psychiatric unit with a 59. A client is admitted to the psychiatric unit with
diagnosis of antisocial personality disorder. In a diagnosis of borderline personality disorder.
planning care for the client, which of the Which of the following components would be
following would be least likely to occur? needless to obtain for the history/data base?
1. Staff and client agree when setting treatment 1. Ego-strength assessment.
goals. 2. Social history.
2. Staff and client are in a constant struggle for 3. Cognitive aspect of mental status exam.
control of the milieu. 4. Past psychiatric treatment history.
3. Allow client to set limits.
4. Staff and client use the same defense 60. An adult was admitted to the psychiatric unit
mechanisms when interacting. after cutting herself on the forearm. She has
numerous scars which are from prior self-
56. A client is admitted with an antisocial mutilation. Should the client attempt self-
personality disorder. Which key intervention mutilation while in the hospital, which
would be contraindicated with this client? implementation should the nurse execute?
1. Assisting him to identify and clarify his 1. Focus on the how, when, and where of the
feelings. injury.
2. Changing staff assigned to a client at his 2. Care for the injury and explore the clients
request. activities and feelings immediately before the
3. Making expectations about his behavior clear episode.
as well as consequences for same. 3. Care for the injury and leave the client alone
4. Setting firm limits with clear consequences. for awhile to let her settle down.
4. Care for the injury and seclude, and possibly
57. A client has been hospitalized with an antisocial restrain, the client to prevent further injury.
personality disorder on a voluntary basis as an
alternative to serving a jail sentence. Following 61. A female client was admitted with a borderline
discharge, what will be the most likely result of personality disorder following an episode of
the client? self-mutilation. Her husband recently left her
1. Be committed to another facility for a longer and she reports that she has injured herself in
length of stay. the past so she could feel something. Which of
2. Be committed to a virtuous and socially the following would be excluded during the
acceptable lifestyle. discharge planning?
3. Continue to use sublimation. 1. Cognition.
4. Revert to pre-hospitalization behaviors. 2. Identity.
3. Dealing with anger.
58. A 28-year-old is admitted to the psychiatric 4. Separation/individuation.
unit under an involuntary petition after a
perceived suicide attempt. Initially, she 62. While collecting data about a 7-year-old boy, the
presented as very tearful and highly anxious. school nurse learned that he has minimal verbal
As the staff became more familiar with her, it skills and expresses his needs by acting out
became apparent that she had had many behaviors. The communication capabilities of
episodes of self-mutilation and would do so so this boy indicate which of the following levels of
I can feel something. While she could appear mental retardation?
quite intact most of the time, when stressed she 1. Mild
would respond very impulsively, express anger, 2. Moderate.
report hearing voices of a depreciative nature,
3. Severe.
and require a high level of observation. This
clients symptoms can best be described as 4. Profound.

PSYCHIATRIC-MENTAL HEALTH NURSING

7 661
53155_07_Ch07_p621-684.qxd 2/27/09 6:29 PM Page 662

63. What nursing care would be included for a 3. Give the client step-by-step instructions for
4-year-old boy with severe autistic disorder? dressing himself.
1. Psychotropic medications. 4. Allow enough time for the client to dress
2. Social skills training. himself.
3. Play therapy.
69. Which question made by the family of a client
4. Group therapy. with Alzheimers disease indicates to the nurse
64. The nurse makes the following assessment of a an understanding of the prognosis?
14-year-old gymnast: underweight, hair loss, 1. Does another hospital have a better
yellowish skin, facial lanugo, and peripheral treatment?
edema. These findings are suggestive of which 2. Will a change in diet help his memory?
of the following disorders? 3. Wont his new medicine cure him?
1. Anorexia nervosa. 4. What supports are available for the future?
2. Bulimia nervosa.
3. Acquired immunodeficiency. 70. A 75-year-old man was brought to the emergency
room confused, incoherent, and agitated after
4. Ulcerative colitis.
painting his lawn furniture earlier in the day. He
65. An adolescent gymnast presents in the eating has no current history of illness. Which one of
disorders clinic severely emaciated, with sallow the following interpretations would be
skin color, 20% body weight loss, amenorrhea appropriate for the nurse to make about his
for the past 12 months, and facial lanugo. Based condition?
on these findings, which one of the following 1. Depression related to aging.
nursing diagnoses would be most appropriate for 2. Dementia related to organic illness.
the nurse to make?
3. Delirium related to toxin exposure.
1. Impaired nutrition: less than body
4. Distress related to unaccomplished tasks.
requirements.
2. Impaired tissue integrity. 71. A student with a history of barbiturate addiction
3. Ineffective individual coping. is brought to the infirmary with suspected
4. Deficient knowledge, nutritional. overdose. Which of the following assessments is
the nurse likely to make?
66. Which observation of the client with anorexia
1. Watery eyes, slow and shallow breathing,
indicates the client is improving?
clammy skin.
1. The client eats meals in the dining room.
2. Dilated pupils, shallow respirations, weak
2. The client gains 1 pound per week. and rapid pulse.
3. The client attends group therapy sessions. 3. Constricted pupils, respirations depressed,
4. The client has a more realistic self-concept. nausea.
67. A client with severe Alzheimers disease has 4. Responsive pupils, increased respirations,
violent outbursts, wanders, and is incontinent. increased pulse and blood pressure.
He can no longer identify familiar people or
72. A teenage girl is admitted to a detoxification
objects. In developing the nursing care plan,
unit with a history of cocaine abuse. Her pupils
the nurse would give highest priority to which
are dilated and she complains of nausea and
nursing diagnosis?
feeling cold. She states that she is not addicted,
1. High risk for injury. but uses cocaine occasionally with friends.
2. Impaired verbal communication. Which one of the following nursing diagnoses is
3. Self-care deficits. appropriate for the nurse to make?
4. Altered pattern of urinary elimination: 1. Impaired verbal communication related to
incontinence. substance use as evidenced by giving untrue
information.
68. A client with Alzheimers disease has a self-care
deficit related to his cognitive impairment. Because 2. Altered growth and development related to
the client has difficulty dressing himself, what substance use as evidenced by age of client.
would be the best action for the nurse to take? 3. Perceptual alteration related to substance use
1. Have the client wear hospital gowns. as evidenced by distortion of reality.
2. Explain to the client why he should dress 4. Ineffective denial related to substance use as

7
himself. evidenced in refusal to admit problem.

662 NCLEX-RN Review


53155_07_Ch07_p621-684.qxd 2/27/09 6:29 PM Page 663

73. The nurse is caring for a client in early alcohol late for work and often calls in sick. When she is
withdrawal. What would most likely be at work, she complains about everything. Which
included in the nursing care plan? of the following is the most likely cause of these
1. Using physical restraints. problems?
2. Providing environmental stimulation. 1. The nurse is dissatisfied with her job.
3. Taking pulse and blood pressure. 2. The nurse is having problems at home.
4. Administering antipsychotic medications. 3. The nurse may be abusing drugs or
alcohol.
74. A client in a detox program is being 4. The nurse realizes she is in the wrong
manipulative by trying to split staff. The client profession.
tells the nurse that he is the best staff member
on the unit. What would be the best response 79. A nurse is evaluating an adult client from the
from the nurse? substance abuse unit. Which statement by the
1. Thank the client for the compliment. client reveals that the client may be ready for
2. Identify the clients manipulative discharge?
behavior. 1. Ill take my Antabuse when I need it.
3. Ignore the clients comment. 2. I cant wait to hang out with my old buddies.
4. Ask the client why he feels that way. 3. Ill drink in moderation and only on the
weekend.
75. In developing a teaching plan for adolescents on 4. Attending daily AA meetings will help me
the topic of cocaine abuse, the nurse would not drink again.
highlight which of the following?
1. Cocaine is a naturally occurring depressant. 80. Which of the following assessment findings
2. Cocaines physical effects differ according to would the nurse observe in a client with
the method of ingestion. schizophrenia?
3. The bodys peak reaction occurs 30 minutes 1. Associative looseness, affect disturbance,
after it is taken. ambivalence, autistic thinking.
4. Smoking cocaine is particularly dangerous to 2. Euphoria, distractibility, dramatic
the cardiovascular system. mannerisms, energetic.
3. Argumentative, anhedonia, poor judgment,
76. A 14-year-old male client is admitted to the manipulative.
emergency room after ingesting a high dose of 4. Psychomotor retardation, intense sadness,
PCP and subsequently injuring himself in a fall. loss of energy, suicidal.
What would be an effective action for the nurse
to take? 81. A client with a diagnosis of paranoid
1. Attempt to talk the client down. schizophrenia reports to the nurse that he hears a
2. Withhold fluids. voice that says, Dont take those poisoned pills
from that nurse! Which one of the following
3. Place the client in a quiet, dimly lit room.
nursing diagnoses would it be appropriate for the
4. Administer a prn phenothiazine. nurse to make regarding this statement?
77. The nurse on a medical unit smells alcohol and 1. Disturbed sensory perceptual: auditory,
notices that the relief nurses words are slurred related to anxiety as evidenced by auditory
and she is giggling inappropriately. What is the hallucination.
best initial action for the nurse to take? 2. Disturbed thought processes related to anxiety
1. Double assign the nurses clients. as evidenced by delusions of persecution.
2. Ask the relief nurse if she has been 3. Defensive coping related to impaired reality
drinking. testing as evidenced by paranoid ideation.
3. Report the nurse to the licensing board. 4. Impaired verbal communication related to
disturbances in form of thinking as evidenced
4. Refer the nurse to an employee assistance
by use of symbolic references.
program.
82. An adult is admitted with a diagnosis of
78. A nurses coworker is argumentative and
catatonic schizophrenia, excited phase. She
resistant to change. Her appearance has become
shouts and paces continuously and seems to be
sloppy over the last 6 months; she is frequently

PSYCHIATRIC-MENTAL HEALTH NURSING

7 663
53155_07_Ch07_p621-684.qxd 2/27/09 6:29 PM Page 664

responding to internal stimuli. What would be a 1. Administer an antiparkinsonism medication.


short-term goal for the nurse to formulate? 2. Stop the neuroleptic medication.
1. The client will groom self daily. 3. Withhold fluids.
2. The client will maintain adequate nutrition. 4. Administer an antianxiety medication.
3. The client will sleep 8 hours per night.
4. The client will attend unit social activities. 88. A client with paranoid schizophrenia has a
delusion of persecution. He tells the nurse,
83. A client with schizophrenia stops talking mid The CIA is out to get me. Theyre spying on
sentence and tilts her head to one side. The me. What is the nurses best initial response?
nurse suspects that the client is experiencing 1. I dont want to hurt you.
auditory hallucinations. What is an appropriate 2. How would they spy on you here?
response from the nurse? 3. Tell me how theyre trying to get you.
1. Ask the client what she is experiencing. 4. I know the CIA wouldnt want to hurt you.
2. Change the topic of conversation.
3. Explain that hallucinations are not real. 89. Which of the following statements indicates to
4. Deny that she hears anything. the nurse that a client with obsessive-
compulsive disorder has developed insight into
84. In teaching a client for whom clozapine her problem?
(Clozaril) has been prescribed, the nurse would 1. I realize that the dangers are more in my
include which of the following? mind.
1. The drug will be given every 4 weeks by 2. I dont hear the voices anymore.
intramuscular injection. 3. I check on my family 12 times every day.
2. The drug will probably cause weight 4. I slept 8 hours last night.
reduction.
3. There is a high incidence of extrapyramidal 90. An adult is brought to the emergency room after
side effects. he attempted to walk across the roof of a
4. Blood work may be required weekly. building in an attempt to fly like a jet plane. In
addition to impulsiveness, which of the
85. An adult is to go on a 3-day pass and has his following behaviors would the nurse assess in a
maintenance supply of chlorpromazine client diagnosed as bipolar, manic type?
(Thorazine). Which statement indicates to the 1. Hallucinations and delusions.
nurse that he understands instructions regarding 2. Euphoria and increased motor activity.
his medication? 3. Paranoia and ideas of reference.
1. Ill take my pills when I hear those voices. 4. Splitting and manipulation.
2. Ill drink beer but no wine while Im away.
3. Ill cover up when I go to the beach. 91. During the focused assessment of a client with
4. Ill stop taking it if my mouth stays dry. major depression, the nurse may ask which of
the following questions?
86. Which of the following behaviors indicates to 1. You seem to have a lot of energy; when did
the nurse that the clients antipsychotic you last have 6 or more hours of sleep?
medication is having a desired effect? 2. You seem to be angry with your family now;
1. The client states that her voices are not as when was it that you last got along?
threatening. 3. Have you had any thoughts of harming
2. The client reports having inner feelings of yourself?
restlessness. 4. You seem to be listening to something.
3. The client sleeps all day. Could you tell me about it?
4. The client reports muscular stiffening in her
face and arms. 92. Which of the following nursing diagnoses would
be most appropriate for a client who is
87. A client taking trifluoperazine (Stelazine) diagnosed as bipolar I disorder, single manic
exhibits severe extrapyramidal symptoms, a episode and is intrusive, argumentative, and
temperature of 40.5C (105F), and diaphoresis. severely critical of peers?
The nurse suspects neuroleptic malignant 1. Impaired social interaction related to
syndrome. What is the nurses best action? narcissistic behavior as evidenced by

7
inability to sustain relationships.

664 NCLEX-RN Review


53155_07_Ch07_p621-684.qxd 2/27/09 6:29 PM Page 665

2. Risk for injury related to extreme should the nurse ask during the prescreen
hyperactivity as evidenced by increased assessment?
agitation and lack of control over behavior. 1. How long have you thought about harming
3. Social isolation related to feelings of yourself?
inadequacy in social interaction as evidence 2. What is it that makes you think about
by problematic interaction with others. harming yourself?
4. Defensive coping related to social learning 3. How has your concentration been?
patterns as evidenced by difficulty 4. What specifically have you thought about
interacting with others. doing to harm yourself?
93. An adult is in an acute manic phase of bipolar 98. A 19-year-old recently broke off her 1-year
disorder. He talks and paces incessantly, engagement. Her mother states, She does
frequently shouting and threatening other nothing but cry and sit and stare into space. I
clients. The nurse expects the clients care plan cant get her to eat or anything! She feels she
to include which of the following? cant go on without her boyfriend. The nurse
1. Monitor blood lithium levels. should make which priority nursing diagnosis?
2. Monitor client during phototherapy. 1. Impaired nutrition: less than body
3. Monitor client after electroconvulsive requirements.
therapy. 2. Dysfunctional grieving.
4. Teach client to avoid foods with tyramine. 3. Risk for self-directed violence.
4. Social isolation.
94. The nurse is preparing to administer lithium
(Eskalith) to a client with bipolar disorder. The 99. A client is admitted for treatment of a major
client complains of nausea and muscle depression. She is withdrawn, appears
weakness, and his speech is slurred. His lithium disheveled, and states, No one could ever love
level is 1.6 mEq/liter. What would be the nurses me. What would the nurse expect to be ordered
best action? for this client?
1. Chart the clients symptoms after giving the 1. Antiparkinsonism medication.
lithium.
2. Suicide precautions.
2. Explain that these are common side effects.
3. A low-salt diet.
3. Withhold the clients lithium.
4. Phototherapy.
4. Administer a prn antiparkinsonism drug.
100. A mans wife complains that her husbands
95. Which of the following behaviors indicates to depression isnt any better after 1 week on
the nurse that the client understands teaching amitriptyline (Elavil). What is the nurses best
related to lithium treatment? response?
1. Taking lithium 1 hour after meals. 1. Tell her she will contact the physician.
2. Stopping taking her lithium when her mania 2. Question the wife about what response she
subsides. expects.
3. Going on a low-salt diet to counter weight 3. Explain that it may take 1 to 3 weeks to see
gain. any improvement.
4. Withholding her lithium if episodes of 4. Suggest that the client change
diarrhea, vomiting, and diaphoresis occur. antidepressants.
96. An adult is recovering from a severe depression. 101. Which of the following behaviors indicates to
Which of the following behaviors alerts the the nurse that a clients major depression is
nurse to a risk for suicide? improving?
1. The client sleeps most of the day. 1. Displaying a blunted affect
2. The client has a plan to kill herself. 2. Losing an additional 2 pounds
3. The client loses 5 pounds. 3. Stating one good thing about himself
4. The client does not attend unit activities. 4. Sleeping about 16 hours per day
97. A man has been severely depressed for 2 weeks. 102. An adult is hospitalized for treatment of
He had mentioned ending it all prior to obsessive-compulsive disorder (OCD). The nurse

7
admission. Which of the following questions recognizes which of the following as an

PSYCHIATRIC-MENTAL HEALTH NURSING 665


53155_07_Ch07_p621-684.qxd 2/27/09 6:29 PM Page 666

indication that the clients sertraline (Zoloft) is 107. A client is prescribed buspirone hydrochloride
having the desired effect? (BuSpar). Which statement alerts the nurse that
1. The client experiences nervousness and additional medication teaching is required?
drowsiness. 1. Ill take my drugs as soon as I feel anxious.
2. The clients delusions are less entrenched. 2. I wont drink any alcohol.
3. The client engages in fewer rituals. 3. Ill report any troubles with my heart or
4. The client sleeps 4 hours per night. seeing.
4. Ill have my blood checked every month.
103. A client with major depression is scheduled for
electroconvulsive therapy (ECT) tomorrow. The 108. In teaching a client about her new antianxiety
nurse would plan for which of the following medication, alprazolam (Xanax), the nurse
activities? should include which of the following?
1. Force fluids 6 to 8 hours before treatment. 1. Caution the client to avoid foods with
2. Administer succinylcholine (Inestine, tyramine.
Anectine) during pretreatment care. 2. Caution the client not to drink alcoholic
3. Encourage the clients spouse to accompany beverages.
him. 3. Instruct the client to take the Xanax 1 hour
4. Reorient the client frequently during after meals.
posttreatment care. 4. Instruct the client to double up a dose if she
forgets to take her medication.
104. A severely depressed client received ECT this
morning. Which of the findings listed below 109. A client experiencing thanataphobia is afraid to
would the nurse least expect to assess leave her aging, ailing husband alone for any
posttreatment? reason. She has not left her husband alone since
1. Headache. her mother and sister died 4 years ago. Which of
2. Memory loss. the following statements would be appropriate
for the nurse to make during the initial
3. Paralytic ileus.
assessment of this client?
4. Disorientation.
1. Are you afraid that your husband might die
105. A client for whom Nardil was prescribed for while you are away from him?
depression is brought to the ER with severe 2. There must be someone you are able to trust
occipital headaches after eating pepperoni pizza to stay with your husband.
for lunch. Which of the following interpretations 3. Dont you have children who are willing to
is it important for the nurse to make regarding stay with your husband when you need to be
these findings? away?
1. Allergic reaction related to ingestion of 4. It must be very confining to have constantly
processed food. attended to your husband for so long.
2. Hypertensive crisis related to drug and food
reaction. 110. A newly admitted client is fearful of elevators.
She needs to take one in 10 minutes to attend
3. Panic anxiety related to unresolved issues.
therapy on the 10th floor. Which of the following
4. Conversion disorder related to uncontrolled actions would be best for the nurse to take?
anxiety.
1. Explain to her that she needs to attend
106. The nurse explains the major difference between therapy.
neurotic and psychotic disorders. What is a 2. Have another client go with her.
major difference in clients with psychotic 3. Accompany her to the 10th floor.
disorders? 4. Explore with her why she is afraid of
1. The clients are aware that their behaviors are elevators.
maladaptive.
2. The clients are aware they are experiencing 111. A man, with a family of five, was recently laid
distress. off and now has financial concerns. He is
experiencing muscle tension, breathlessness,
3. The clients experience no loss of contact with
and sleep disturbances. Which one of the
reality.
following nursing diagnoses would be
4. The clients exhibit a flight from reality.

7 666 NCLEX-RN Review


53155_07_Ch07_p621-684.qxd 2/27/09 6:29 PM Page 667

appropriate for the nurse to make regarding his 4. I had another horrible nightmare last night
condition? and went through the same trauma and
1. Post-trauma response related to loss of anxiety all over again.
economic support as evidenced by job loss.
116. A client with OCD has an elaborate handwashing
2. Parental role conflict related to perceived
and touching ritual that interferes with her
inability to meet his familys economic and
activities of daily living. She misses meals and
physical needs as evidenced by job loss.
therapy sessions. What effective strategy could
3. Ineffective individual coping related to recent the nurse initiate to limit her ritual?
unemployment as evidenced by physical
1. Teach thought stopping techniques.
manifestations.
2. Prevent the ritualistic behavior.
4. Powerlessness related to inability to deal
with anxiety as evidenced by physical 3. Use adjunctive therapies for distraction.
manifestations. 4. Facilitate insight regarding the need for the
ritual.
112. A woman appears to be having a panic attack
during group therapy. She is agitated, pacing 117. A client with an OCD has checking rituals and
rapidly, and not responding to verbal stimuli. thoughts that her family will be harmed. Which
What would be the nurses initial intervention? of the following indicates to the nurse that the
1. Remove her from the group. client is improving?
2. Encourage her to express her feelings. 1. Obsessing about her familys health.
3. Facilitate her recognizing her anxiety. 2. Adhering to the unit schedule.
4. Ignore her. 3. Losing 2 pounds in 1 week.
4. Awakening 8 times during the night.
113. The nurse is assessing a client who presents
with OCD. In addition to gathering information 118. A 4-year-old girl, who is a victim of a bomb blast
about the clients anxiety and rituals, the nurse that demolished the building which housed
should assess for which of the following? her daycare, constantly builds block houses
1. Handwringing and foot-tapping behaviors. and blows them up. She also has nightmares
frequently. Which one of the following
2. Use of abusive substances and gambling.
diagnoses is appropriate for the nurse to make
3. Tics, stuttering, or other unusual speech regarding this child?
patterns.
1. Post-trauma response related to terrorist
4. Diaphoresis and rapid breathing. attack as evidenced by destructive behaviors
and sleep disturbances.
114. Which of the following statements by a client
with delusions indicates to the nurse that the 2. Explosive disorder related to dysfunctional
client is improving? personality as evidenced by destructive
behaviors.
1. I dont feel those crawling bugs anymore.
3. Sleep disturbance related to emotional
2. I wont talk about my crazy thoughts at work.
trauma as evidenced by nightmares.
3. I feel less jumpy inside.
4. Ineffective individual coping related to
4. I must check my room for bugs. internal stressors as evidenced by destructive
behaviors and nightmares.
115. During the assessment phase of the nurse-client
interaction, which of the following statements 119. The nurse recognizes that the client with post-
made by the client is suggestive of post- traumatic stress disorder (PTSD) is improving
traumatic stress disorder? when which of the following occurs?
1. My dad had trouble swallowing before he 1. States he feels numb most of the time.
died and I always feel as if I have a lump in
2. Drinks alcohol to cope with his feelings.
my throat.
3. Talks about a benefit of the traumatic
2. After I contracted meningitis on vacation
experience.
last summer, I cant control this horrible
thought that all people who work in park 4. Attends weekly group therapy.
restaurants are dirty.
120. A young woman is found wandering on campus
3. I continue to have the same dream over and after a fraternity party. She is disheveled and does
over again. not know who she is. She has no recollection of

PSYCHIATRIC-MENTAL HEALTH NURSING

7 667
53155_07_Ch07_p621-684.qxd 2/27/09 6:29 PM Page 668

the evening. At the student health service she is 4. Receive secondary gain from his physical
diagnosed with dissociative amnesia subsequent symptoms
to a rape. What is the most appropriate nursing
diagnosis for the nurse to formulate? 125. A man is brought into the police station after he
1. Ineffective individual coping. ran toward a boy who resembled his son. At the
police station he was unable to recall any
2. Personal identity disturbance.
personal information. The prescreening nurse
3. Anxiety related to alteration in memory. inferred that the man has which one of the
4. Risk for violence, self-directed. following dissociative disorders?
1. Amnesia
121. The nurse finds, during the initial assessment of
the star player on the basketball team, that he is 2. Fugue
not concerned about the sudden paralysis of his 3. Personality disorder
shooting arm. What is this behavior known as? 4. Stress disorder
1. Secondary gain
126. Which of the behaviors listed below would
2. La belle indiffrence
assist the nurse in establishing the diagnosis of
3. Malingering borderline personality disorder?
4. Hypochondriasis 1. Impulsivity
122. A mans family brought him into the hospital 2. Hallucinations
because of his many somatic complaints. He has 3. Self-mutilation
been seen by many medical specialists in the 4. Narcissism
past without discovery of organic pathology. The
nurse assesses that the client is probably 127. A woman is admitted to the unit with a
experiencing which of the following problems? diagnosis of borderline personality disorder. She
1. Conversion disorder has angry outbursts and is impulsive and
manipulative. She has lacerations on her arm
2. Body dysmorphic disorder
from self-mutilation. Which of the following
3. Malingering would be a priority nursing diagnosis?
4. Hypochondriasis 1. Ineffective individual coping.
123. An adult is hospitalized for treatment of a 2. Disturbed body image.
conversion disorder. She complained of 3. Disturbed personal identity.
paralysis of her right side after her husband 4. Risk for violence to self.
threatened to leave her and their children. She
seems unconcerned about her paralysis. What 128. A client with borderline personality disorder
would be an appropriate long-term goal for the tells the nurse she hates her doctor because he
nurse to formulate for the client? denied her a pass because she returned high
1. Cope effectively with stress without using from her last pass. What would be the nurses
conversion best action?
2. Identify stressors 1. Ask the client why she is feeling so angry.
3. Express feelings about the conflict 2. Suggest that the client bring it up in
community meeting.
4. Develop an increased sense of relatedness to
others 3. Offer to contact the doctor and discuss the
situation.
124. An adult has hypochondriasisbelieving he is 4. Set limits and point out that the denial is a
dying of stomach cancer despite repeated and consequence of her inappropriate behavior.
extensive diagnostic testing that has all been
negative. He has become reclusive and is 129. The nurse would formulate which of the
preoccupied with his physical complaints. The following outcome criteria for a client with
nurse would include which of the following in borderline personality disorder?
the nursing care plan as a client outcome? 1. Displays anger frequently.
1. Focus on the signs and symptoms of stomach 2. Acts out neediness.
cancer 3. Experiences troubling thoughts without self-
2. Attend a support group for persons with cancer mutilation.
3. Complete a contract to attend social and 4. Idolizes assigned nurse.

7
diversional activities daily

668 NCLEX-RN Review


53155_07_Ch07_p621-684.qxd 2/27/09 6:29 PM Page 669

130. A client with antisocial personality disorder is 26. 4. Responding factually helps to orient the
charming, seductive, and highly manipulative. client.
He has a history of multiple jobs and marriages,
which have all failed, and problems with the 27. 3. The nurse should be someone the client can
law. Which of the following is an appropriate turn to for guidance.
short-term goal for the nurse to formulate in
28. 4. Providing the client with structured activities
relation to a nursing diagnosis of ineffective
will allow her to release tension. Exercises also
individual coping?
help older people with balance and mobility and
1. The client will avoid situations that provoke reduce falls.
aggressive acts.
2. The client will adhere to unit rules. 29. 4. Including the family in the plan of care
3. The client will assume a leadership role in ensures a more effective plan.
unit governance.
30. 1. Although a complete substance abuse history
4. The client will acknowledge manipulative
is necessary eventually, on admission the most
behaviors pointed out by staff.
important information is the type and amount of
131. Which of the following indicates to the nurse substances taken by the client in the past 24
that a client with antisocial personality disorder hours.
is improving?
31. 1. While a substance abuser has difficulty in all
1. Complimenting the nurse for on outstanding areas listed, problems handling stress and
job on the unit. anxiety underlie all the others.
2. Testing the limits on personal behavior.
3. Acknowledging some manipulative behavior. 32. 3. Delirium tremens is characterized by
increased blood pressure, pulse, and
4. Sleeping 8 hours per night.
respirations, and an increase in psychomotor
activity.

Answers and Rationales 33. 4. Group therapy with other substance abusers is
the most highly prescribed therapy. It is the
model for Alcoholics Anonymous, the most
21. 3. Upon hearing her sons diagnosis, the mother effective treatment group.
is experiencing emotional turmoil and projecting
blame. Acknowledging her feelings would build 34. 2. A delusion is a fixed false belief.
further trust and encourage her to discuss her 35. 2. The nurse needs to present reality to the client
thoughts and feelings. and not encourage the delusion.
22. 2. The nurse can offer himself to the client to 36. 2. Assessing increasing signs of anxiety and
establish trust. The nurse will stay with the agitation gives clues to the clients ability to
client while eating. maintain control and suggests further
23. 4. Because the anorexic client is experiencing nursing interventions to protect the client
starvation, her well-being is dependent on and others.
establishing an adequate nutritional state. Eating 37. 2. There are four characteristics of schizophrenia
and gaining weight are the primary goals of that help in an assessment. One of the key
hospitalization. indicators is the overwhelming attitude of
24. 1. While there might be some concern that the ambivalence toward the environment and any
client is abusing drugs and possibly using them to emotional involvement with others. The other
induce further weight loss, the primary concern is three indicators are affect, associative looseness,
that the client is experiencing abdominal pain. and autistic thinking.
This may be a clue to an impending medical 38. 4. Daily walks provide time for the nurse to
crisis needing further assessment. develop trust. Walking allows expenditure of
25. 2. Dropping grades, low motivation, somatic energy without increasing paranoia.
complaints, and poor mouth hygiene are signs 39. 3. This client does not need additional stimuli
and symptoms of depression. from the environment.

PSYCHIATRIC-MENTAL HEALTH NURSING

7 669
53155_07_Ch07_p621-684.qxd 2/27/09 6:29 PM Page 670

40. 3. A drug frequently used to treat manic clients 53. 4. The major issue is control of behavior and
is lithium carbonate (Eskalith). thoughts. When the client is able to control her
compulsive behavior, i.e., limit her hand and
41. 1. Psychoanalysis is an in-depth, insight- face washing to a few times a day, she will then
oriented psychotherapy, not appropriate in be able to resume normal activities of daily
treatment of bipolar disorders. living.
42. 4. The client shows many signs of classic 54. 1. A long history of petty crimes, a high level of
depression as evidenced by psychomotor manipulative behavior, use of other clients to his
retardation, impairment of self-care, inability to own end, and fostering behavior that is
sleep, a suicide attempt, and somatic delusion. disruptive to the milieu are all signs of the
diagnosis of antisocial personality disorder.
43. 3. The priority at this time is maintenance of
client safety. This client is at particular risk for 55. 1. The staff and client will most likely disagree
self-directed violence because of her recent when setting treatment goals.
failed suicide attempt and her obsession with
what she perceives to be her impending death. 56. 2. The client will compare and attempt to split
staff, so it is very important to keep staff
44. 4. Even though the client is probably exhausted, assignments as consistent as possible.
the most therapeutic plan would allow for both
rest and activity. 57. 4. People who have this type of personality
disorder typically seek psychiatric care as a
45. 3. The client who is experiencing severe panic lesser of two evils. In this case in-hospital care
needs a quiet environment with supportive care is preferable to jail. The chances of this client
to decrease anxiety enough to cope. making any great change in his lifestyle as a
result of short-term hospitalization are slim.
46. 1. Obsessive-compulsive behavior represents
The client will likely be committed to another
displacement of anxiety. A concrete measurable
facility when he is again arrested for deviant
goal is to decrease the number of handwashings.
behavior.
47. 4. Providing ample time for the client to
58. 2. The clustering of self-mutilation, impulsivity,
complete her handwashing rituals will lessen
transient psychosis, intense anger, and feeling
her anxiety.
empty is most typically found in borderline
48. 2. Planning a structured schedule of activities personality disorder.
provides the client with ways other than
59. 3. The mental status exam is conducted when
handwashing to reduce anxiety.
the nurse suspects a client is disoriented. The
49. 4. Current weight as it relates to usual weight is client with a borderline personality disorder has,
the best determinant of nutritional status and for the most part, intact reality testing.
weight change when the client is unable to be
60. 2. A matter-of-fact approach to the injury with
specific about recent activities and eating habits.
emphasis on the events leading to the episode of
50. 3. Ineffective individual coping encompasses all mutilation is the most therapeutic approach.
of the other nursing diagnoses. This area will be
61. 1. Impairments involving cognition are most
the primary focus of nursing interventions, and
commonly found in psychoses.
positive changes in the clients ability to cope
will be the criteria for discharge readiness. 62. 3. Individuals with severe mental retardation
possess minimal verbal skills. They often
51. 3. Building the intake of a specified amount of
communicate wants and needs by acting out
liquid into a daily schedule of activities is very
behaviors.
consistent with the obsessive-compulsive
clients need to control as many aspects of her 63. 3. Play therapy would be most effective given
life as possible. his developmental level and autism. In autistic
disorder, communication with others is severely
52. 1. Allowing the client a certain amount of time
impaired. Through one-to-one play therapy, the
to engage in the activity alleviates some of the
therapist may establish rapport through
clients anxiety.
nonverbal play.

7 670 NCLEX-RN Review


53155_07_Ch07_p621-684.qxd 2/27/09 6:29 PM Page 671

64. 1. Anorexia nervosa, usually occurring in 76. 3. Environmental stimuli need to be reduced for
individuals ages 1322 years, is an eating the client in PCP intoxication to reduce danger
disorder characterized by self-starvation, weight to self, paranoia, delusions, and hallucinations.
loss (25% below normal weight), disturbance in These clients are sensitive to stimuli and quickly
body image, and physiologic and metabolic become combative and assaultive.
changes.
77. 2. There is usually a chain of command policy
65. 1. The assessment data and history of the client that begins with a direct discussion of the
support the diagnosis of altered nutrition related involved parties. If the relief nurse denies
to anorexia. drinking, the nurse has a duty to intervene.

66. 2. Weight gain is the best indication that the 78. 3. Signs of possible substance abuse are social
clients anorexia is improving. A realistic isolation; requesting to work nights; changes in
expectation is for the client to gain 1 pound per appearance and mood; excessive tardiness,
week. accidents, and absences; excuses for being
unavailable when on duty; resistance to change;
67. 1. Safety is of highest concern for this client. His defensive when questioned about client
wandering and memory loss pose hazards for complaints or drug discrepancies; failure to meet
accidents, falls, and injuries. schedules and deadlines; and inaccurate and
sloppy documentation. The situation requires
68. 3. The client may need step-by-step instructions
further professional assessment. The nurse
so he can focus on small amounts of
should follow agency policies and board of
information. This allows him to perform at his
nursing guidelines to report his suspicions.
optimal level. Clients with dementia may not
remember how to dress themselves. 79. 4. Daily attendance at AA meetings is necessary
for most discharged clients to remain sober and
69. 4. This response indicates that the family is
continue their rehabilitation.
expecting to need support during the process of
the clients increasing cognitive impairment. 80. 1. Eugen Bleulers 4 As of schizophrenia are
loosening of associations (L.O.A.), which are
70. 3. Paint is a toxin that could cause delirium.
representative of thought disorders, disturbance
Delirium is a state of mental confusion and
in affect, ambivalence, and autistic thinking.
excitement. The mind wanders, speech is
incoherent, and the client is often in a state of 81. 1. Hallucinations are sensory experiences of
continual, aimless physical activity. The onset is perception without corresponding stimuli in the
rapid (hours to days). environment.
71. 2. The effects of overdose of barbiturates are 82. 2. It is important for the nurse to monitor dietary
shallow respirations, cold and clammy skin, intake and weight so the person does not lose
dilated pupils, weak and rapid pulse, coma, and calories and fluids due to hyperactivity. Finger
possible death. foods may need to be provided, e.g.,
sandwiches and fruit.
72. 4. Denial is the minimizing or disavowing of
symptoms or a situation to the detriment of 83. 1. The best initial action is to focus on the cues
health. and elicit the clients description of her
experience. It is important for the nurse to
73. 3. Pulse and blood pressure should be checked
determine that she is hallucinating and the
hourly for the first 812 hours after admission.
content. This is vital in relation to safety issues
They are usually elevated during withdrawal
and command hallucinations.
and the pulse is a good indication of progress
through withdrawal. Elevation may indicate 84. 4. Weekly white blood cell counts may be
impending alcohol withdrawal delirium. required due to the side effects of possible life-
threatening agranulocytosis.
74. 2. A priority in intervening in manipulative
behavior is to identify it and then set limits by 85. 3. The client should avoid the sun or cover up
stating expected behaviors. and use sunscreen to protect himself from severe
photosensitivity.
75. 4. A total cardiac collapse may occur. Smoking
crack cocaine is the method that most often

7
leads to myocardial infarction.

PSYCHIATRIC-MENTAL HEALTH NURSING 671


53155_07_Ch07_p621-684.qxd 2/27/09 6:29 PM Page 672

86. 1. A desired effect of the antipsychotics is to 99. 2. Maintaining safety for the client is a priority
reduce the disturbing quality of hallucinations because she may have suicidal ideation and/or a
and delusions. plan.

87. 2. The neuroleptic should be immediately 100. 3. The client may need to take Elavil 1 to 3
discontinued. Medical treatment should be weeks before any improvement or a therapeutic
instituted because this is a potentially fatal effect is noticed.
syndrome.
101. 3. This behavior may indicate an increase in
88. 1. The nurse should first clarify her intent and self-esteem that accompanies an improvement in
then empathize with the underlying feeling. depression. A depressed person often cannot
problem solve or acknowledge any positive
89. 1. This statement indicates that the client has aspects of their lives.
some insight into the underlying reason for her
rituals. 102. 3. Zoloft is a selective serotonin reuptake
inhibitor (SSRI) that is effective in treating
90. 2. The client diagnosed as bipolar, manic clients with obsessive-compulsive disorder.
exhibits behaviors of elation, euphoria, and is Using fewer rituals would indicate an
full of energy, which may lead to exhaustion. improvement.
91. 3. Clients with major depression are often 103. 4. Common side effects of bilateral treatment
suicidal. The first concern of assessment is the include confusion, disorientation, and short-
risk of suicide potential in the immediate future. term memory loss. The nurse should provide
frequent orientation statements that are brief,
92. 2. The client who invades the space of others,
distinct, and simple.
creates arguments, and attacks others is at risk
for injury by those in the environment. 104. 3. ECT is treated as an operative procedure;
however, paralytic ileus (intestinal obstruction,
93. 1. Lithium is the drug of choice for manic clients
especially failure of peristalsis) frequently
with an antimanic effectiveness of 78%. It
accompany peritonitis and usually result from
reduces the intensity, duration, and frequency of
disturbances in the bowel.
manic and depressive episodes. Blood levels are
monitored for therapeutic levels in the acute 105. 2. Severe occipital and/or temporal pounding
phase (1.01.5 mEq/liter) and during headaches, manifestations of hypertensive crisis,
maintenance. occur when processed meats are eaten by
individuals currently taking Nardil (MAOI).
94. 3. The client is exhibiting symptoms and signs
of lithium toxicity. Another blood level should 106. 4. In psychotic responses to anxiety, clients
be drawn and the dose evaluated. escape from reality into hallucination and/or
delusional behavior.
95. 4. These are early signs of lithium toxicity. The
drug should be withheld and a lithium blood 107. 1. BuSpar must be taken as a maintenance drug,
level drawn and evaluated to determine an not as a prn response to symptoms. Improvement
appropriate dosage. may be noted in 710 days, but it may take 3 to
4 weeks to note therapeutic effects.
96. 2. Having a suicide plan is a risk factor. The
lethality needs to be assessed. When a 108. 2. The depressant effects of alcohol and
depression is lifting, the client may have the alprazolan will be potentiated and may cause
energy and resources to carry out a plan. harmful sedation.
Behavioral, somatic, and emotional cues may be
overt or covert. 109. 1. Confronting fear diminishes the phobic
response and the anticipatory anxiety that
97. 4. This question assists in determining suicidal precedes it.
intent and lethality.
110. 3. This is the best action because the nurse is
98. 3. The depressed client often feels hopeless and conveying her support. Later, she would need to
helpless with self-directed anger. Suicidal further assess the clients fear of elevators and
ideations are often expressed and warrant respond accordingly.
immediate intervention.

7 672 NCLEX-RN Review


53155_07_Ch07_p621-684.qxd 2/27/09 6:29 PM Page 673

111. 2. Parental role conflict is the state in which a 121. 2. This lack of concern is identified as la belle
parent experiences role confusion and conflict indifference and is often a clue that the problem
in response to crisis. Loss of economic base may be psychological rather than physical.
constitutes a crisis state.
122. 4. Hypochondriasis is excessive preoccupation
112. 1. The nurse should remove the client from the with ones physical health, without organic
group to provide a safe environment for her and pathology.
others. The nurse should stay with the client
and provide comfort and reality orientation. 123. 1. This is an appropriate long-term goal related
to the clients ineffective coping (use of
113. 3. There is comorbidity between Tourettes conversion symptom, paralysis) related to
syndrome and obsessive-compulsive disorder. unresolved conflicts and anxiety.

114. 2. Improvement in relation to delusional content 124. 3. This goal is related to the clients impaired
includes a reduction in the disturbing quality of social interaction in response to his
the delusions and the clients ability to control preoccupation with illness.
and/or not respond to them.
125. 1. In dissociative amnesia, an individual is
115. 4. Symptoms of post-traumatic stress disorder unable to recall important personal information
range from emotional numbness to vivid such as name, occupation, and relatives.
nightmares in which the traumatic event is
recalled. 126. 3. Self-mutilation is characteristic of borderline
personality disorder.
116. 1. Thought stopping techniques, flooding, and
response prevention have proven effective in 127. 4. A safe environment for the client is a priority.
treating clients with OCD. Clients may shout or Her self-mutilation, poor impulse control, and
think stop or snap a rubber band on their wrist temper are characteristic of persons with
to dismiss the obsessive thought. borderline personality disorder who have self-
directed violence.
117. 2. If the client adheres to the unit schedule, it is
likely that her obsessions and compulsive rituals 128. 4. The clients acting out and demanding
have lessened. They no longer preoccupy her to behavior indicates her need for ego boundaries
the point of interfering with activities. and control, which the nurse provides.

118. 1. Post-trauma response is the state of an 129. 3. Clients with borderline personality disorder
individual experiencing a sustained painful frequently engage in impulsive suicidal or self-
response to an overwhelming traumatic event. mutilating behaviors. The behavior described in
choice 3 indicates less acting-out of feelings
119. 3. Cognitive treatment for PTSD includes and less impulsiveness in response to more
redefining the event by considering benefits of effective coping.
the experience and finding meaning in the
experience. 130. 4. This is an appropriate short-term goal in
relation to his use of manipulative behavior to
120. 2. The clients behavior is indicative of personal meet his needs.
identity disturbance related to a traumatic event,
the rape. The client is unable to recall her identity, 131. 3. This would indicate that the client may be
which is a factor in dissociative disorders. The improving related to recognizing his
person loses the ability to integrate consciousness, manipulative behavior. This is a first step in
memory, identity, or motor behavior. reducing the need for manipulation and
attaining more effective coping strategies.

PSYCHIATRIC-MENTAL HEALTH NURSING

7 673
53155_07_Ch07_p621-684.qxd 2/27/09 6:29 PM Page 674

Psychologic Aspects of Physical Illness

STRESS-RELATED DISORDERS B. Recognize relationship of stress to physical


symptom(s).
C. Acknowledge coping patterns that may affect
Overview recurrence of physical symptoms.
A. Actual physiologic change in structure/function of D. Recognize relationship of self-concept, self-esteem,
organ or system role performance to disorder.
B. May be referred to as psychosomatic or E. Develop alternative coping behaviors.
psychophysiologic disorders
C. Theorized that clients response to stress is a factor Interventions
in etiology of disease
A. Provide nursing care specific to physical
D. Stress/anxiety not the sole cause but may be a
symptoms.
causative factor in the development/exacerbation
B. Establish nurse-client relationship.
of physical symptoms
C. Encourage discussion of psychosocial problems.
E. See Table 7-9 for types of disorders with a stress
D. Explain relationship of stress to physiologic
component.
symptoms.
E. Encourage client to devise alternative coping
Assessment behaviors, changes in environment, attitude.
A. Health history, family history F. Role play new behaviors with client.
B. Physical symptoms
C. Social/cultural considerations Evaluation
D. Coping behaviors
A. Goals specific to clients physical symptoms have
Analysis been met.
B. Client
Nursing diagnoses for stress-related disorders 1. Is able to relate stress to physical symptoms.
may include any nursing diagnosis specific to 2. Develops alternative coping behaviors.
the physiologic problem as well as: 3. Engages in role playing of new behaviors.
A. Ineffective coping
B. Deficient knowledge
C. Health-seeking behaviors
VICTIMS OF ABUSE
Planning and Implementation
Overview
Goals
A. Abuse is physical or sexual assault, emotional
Client will: abuse, or neglect.
A. Receive appropriate treatment for any physical B. Victims are helpless or powerless to prevent the
symptoms (e.g., maintenance of blood pressure assault on their bodies or personalities.
within normal range). C. Sometimes victims blame themselves for the assault.
D. The abusers often blame the victims, have poor
Table 7-9 Types of Stress-Related Disorders impulse control, and use their power (physical
strength or weapon) to subject victims to their
Systems Examples assaults.
E. Victims include children, spouse, elderly, or rape
Respiratory Asthma, common cold victims; each will be described separately.
Circulatory Hypertension, migraine headaches
Digestive Peptic ulcers, colitis Child Abuse
Skin Hives, dermatitis
Overview
Musculoskeletal Rheumatoid arthritis, chronic backache
Nervous Fatigue A. Over one million cases reported each year
B. Suspected child abuse must be reported
Endocrine Dysmenorrhea, diabetes mellitus

7 674 NCLEX-RN Review


53155_07_Ch07_p621-684.qxd 2/27/09 6:29 PM Page 675

C. Abusing adults (parents) often have been victims 2. Conduct interview in private with child and
of abuse, substance abusers, have poor impulse parent(s) separated.
control 3. Inform parent(s) of requirement to report
D. Battered-child syndrome: multiple traumas suspected abuse.
inflicted by adult 4. Do not probe for information or try to prove
E. Sexual abuse/incest: common types of child abuse abuse.
F. Health care workers often experience negative 5. Be supportive and nonjudgmental.
feelings toward abuser 6. Provide referrals for assistance and
G. See Child Abuse, Unit 5 therapy.
C. Evaluation
Assessment 1. Physical symptoms have been treated.
2. Child safety has been ensured.
A. Physical signs/behaviors of physical/sexual abuse 3. Parent(s) have agreed to seek help.
(see Table 7-10)
B. Signs of neglect: hunger, poor hygiene/nutrition,
fatigue Spouse Abuse
C. Signs of emotional abuse: habitual behaviors Overview
(thumb sucking, rocking, head banging),
conduct/learning disorders A. Estimates of five million women assaulted by mate
each year
Analysis B. Stages
1. Tension builds: verbal abuse, minor physical
A. Situational low self-esteem assaults
B. Fear a. Abuser: often reduces tension with
C. Pain alcohol/drugs
D. Altered parenting b. Abused: blames self
E. Post-trauma response 2. Acute battering: brutal beating
F. Powerlessness a. Abuser: does not recall incident
G. High risk for injury b. Abused: depersonalizes, may seek
separation/divorce
Planning and Implementation c. Both parties in shock
3. Honeymoon: make-up stage
A. Goals a. Abuser: apologizes and promises to control
1. Client (child) will be safe until home self
assessment made by child welfare agency. b. Abused: feels loved/needed;
2. Child will participate with nurse (therapist) forgives/believes abuser
for emotional support. 4. Cycle repeats with subsequent battering,
3. Client (parent[s]) will be able to contact usually more severe
agencies to deal with own rage/helplessness.
4. Parent(s) will participate in therapy (group or Assessment
other required).
B. Interventions A. Headache
1. Provide nursing care specific to B. Injury to face, head, body, genitals
physical/emotional symptoms. C. Reports accidents
D. Symptoms of severe anxiety
E. Depression
F. Insomnia
Table 7-10 Symptoms of Child Abuse G. X-rays reveal previously healed fractures/broken
bones
Physical Abuse Sexual Abuse
Analysis
Pattern of bruises/welts Pain/itching of genitals
Burns (cigarette, scalds, rope) Bruised/bleeding genitals A. Risk for injury
B. Anxiety
Unexplained fractures/ Stains/blood on underwear
C. Pain
dislocations
D. Disabled family coping
Withdrawn or aggressive Withdrawn or aggressive E. Ineffective coping
behavior behavior F. Spiritual distress
Unusual fear of parent/desire Unusual sexual behaviors G. Fear
to please parent

PSYCHIATRIC-MENTAL HEALTH NURSING

7 675
53155_07_Ch07_p621-684.qxd 2/27/09 6:29 PM Page 676

Planning and Implementation 3. Client will notify nurse if further abuse takes
place.
A. Goals 4. Caregiver will verbalize plans to meet own
1. Client will admit self and/or children are needs.
victims of abuse 5. Caregiver will seek assistance to meet clients
2. Client will describe plan(s) for own/childrens needs when necessary.
safety B. Interventions
3. Client will name agencies that will assist in 1. Refer to state laws for reporting elder abuse
maintaining a safe environment and nurses liability.
B. Interventions 2. Obtain clients consent for treatment and/or
1. Crisis stage transfer.
a. Provide safe environment 3. Document physical/emotional condition of
b. Treatment of physical injuries; document client.
c. Encourage verbalization of actual home 4. Refer client/caregiver to agencies for
environment assistance.
d. Provide referral to shelters 5. Encourage client and caregiver to discuss
e. Encourage decision making problems.
2. Rebuilding stage: therapy (individual, family 6. Encourage communication between client and
and/or group) caregiver.

Evaluation Evaluation
Client will be protected from further injury. A. Client will remain free of injury, effects of neglect.
B. Caregiver will utilize support systems for self.
Elder Abuse
Overview Rape
A. Estimates one-half million to over one million
Overview
cases per year. A. Estimates of occurrence vary; only 10% reported
B. Women, over age 70, with some B. Most victims are female between ages of 15 and
physical/psychological disability are most frequent 24 years
victims. C. Response to rape
C. Neglect is most common, followed by physical 1. Shock: panic to overly controlled
abuse, financial exploitation, and sexual 2. Outward adjustment: manages life but may
abuse/abandonment. make drastic changes (e.g., moves, leaves
D. Victims do not always report abuse because of fear school/job)
of more abuse/abandonment by caretaker(s). 3. Integration: acknowledges response (e.g.,
depression, fear, rage)
Assessment
A. Malnutrition
Assessment
B. Poor hygiene, decubiti A. Physical injury
C. Omission of medication/overmedication B. Emotional response: controlled/hysterical
D. Welts, bruises, fractures
Analysis
Analysis
A. Rape trauma syndrome
A. Risk for injury 1. Compound reaction: immediate to 2 weeks
B. Fear (anger, fear, self-blame)
C. Anxiety 2. Long-term: nightmares, phobias, seeks support
D. Imbalanced nutrition: less than body requirements B. Silent reaction: anxiety, changes in relationships
E. Powerlessness with men, physical distress, phobias
F. Situational low self-esteem C. Post-trauma response

Planning and Implementation Planning and Implementation


A. Goals A. Goals
1. Client will be free from injury. 1. Client will express response to assault
2. Client will receive adequate nutrition, 2. Client will verbalize plan to handle immediate

7
hydration, prescribed medication. needs

676 NCLEX-RN Review


53155_07_Ch07_p621-684.qxd 2/27/09 6:29 PM Page 677

3. Client will seek assistance from rape counselor Planning and Implementation
4. Client will discuss need for follow-up
counseling Goals
5. Client will report (long-term) reduction of
A. Client will:
physical and emotional symptoms.
1. Receive treatment for physiologic problems.
B. Interventions
2. Experience decrease in level of anxiety/fear.
1. Give emotional support in nonjudgmental
3. Discuss anxiety/fears with nurse.
manner.
B. Family will:
2. Maintain confidentiality: client must give
1. Be informed of clients condition on regular
consent for reporting rape and for medical
basis.
examination.
2. Discuss anxiety/fears with nurse.
3. Listen to client, encourage expression of
3. Provide appropriate support to client.
feelings.
4. Document physical findings. Put evidentiary
garments in paper bag. Interventions
5. Provide referral to rape counselor and follow- A. Provide nursing care specific to physiologic
up care. problems.
B. Stay with client.
Evaluation C. Explain all procedures slowly, clearly, concisely.
D. Provide opportunities for client to discuss fears.
A. Client seeks support from family/agencies.
E. Provide opportunities for client to make decisions,
B. Client verbalizes emotional response to rape.
have as much control as possible.
C. Long-term: client reports return to prerape
F. Encourage family to ask questions.
lifestyle.
G. Recognize negative family responses as coping
behaviors.
CRITICAL ILLNESS H. Encourage family members to support each other
and client.
Overview
A. Individuals in critical life-threatening situations Evaluation
have realistic fears of death or of permanent loss of
A. Goals specific to clients physiologic status have
function.
been met.
B. Clients and their families may respond to these
B. Client
crises with denial, anger, hostility, withdrawal,
1. Demonstrates a decrease in anxious
guilt, and/or panic.
behaviors.
C. Loss of control and a sense of powerlessness can
2. Is able to express fears verbally.
be overwhelming and detrimental to chance of
3. Has participated in decisions whenever
recovery.
possible.
C. Family members
Assessment 1. Have discussed fears.
2. Demonstrate support for each other and for
A. Physiologic needs (first priority)
client.
B. Anxiety level of client/family
C. Client/family fears
D. Coping behaviors of client/family CHRONIC ILLNESS
E. Social and cultural considerations
Overview
Analysis A. Chronic illnesses, such as diabetes mellitus,
multiple sclerosis, or illnesses/injuries resulting in
Nursing diagnoses for the psychologic component of
loss of function or loss of a body part necessitate
critical illness may include:
adaptation to the inherent changes imposed.
A. Anxiety
B. Clients/families may respond to the losses associated
B. Hopelessness
with chronic illness with a variety of behaviors and
C. Ineffective coping
defenses, including recurrent depression, anger and
D. Deficient knowledge
hostility, denial, or acceptance.
E. Fear
F. Powerlessness

PSYCHIATRIC-MENTAL HEALTH NURSING

7 677
53155_07_Ch07_p621-684.qxd 2/27/09 6:29 PM Page 678

Assessment and Analysis B. Family


1. Discusses positive and negative responses to
Same as stress-related disorders as well as: clients illness.
A. Ineffective coping 2. Plans/engages in appropriate activities with
B. Risk for violence, self-directed client.
C. Spiritual distress

Planning and Intervention AIDS


Goals
Overview
A. Client will:
A. In the United States, many thousands of reported
1. Receive appropriate treatment for any
cases and deaths, estimates between 1 and 2
physiologic symptoms.
million infected.
2. Be able/willing to discuss responses to illness.
B. Highest risk populations: homosexual/bisexual
3. Recognize effect of illness on aspects of self-
men, IV drug users and their sexual partners,
concept.
hemophiliacs, newborns from infected mothers,
4. Develop realistic plans for activities and role
and black females between the ages of 15 and
functions.
44 years.
5. Contract with nurse to report
C. Approximately 60% of persons with AIDS develop
depression/suicidal ideation.
neurological symptoms.
B. Family will:
D. Health care workers may have difficulty caring for
1. Be able to discuss responses to client illness.
these clients because of fear of contagion,
2. Develop plans to deal with alterations in
knowledge deficit, bias against lifestyle, or
clients behaviors and functions.
burnout.
E. Families/partners will require support, education,
Interventions and counseling.
A. Provide nursing care specific to physiologic
problems. Assessment
B. Develop nurse/client relationship through active
listening, acceptance of positive and negative A. Physical symptoms
client responses. 1. Fever
C. Encourage client to plan activities within present 2. Fatigue
capabilities. 3. Weight loss
D. Provide information about illness, suggestions for 4. Diarrhea
activities. 5. Opportunistic infections
E. Contract with client to request support in times of B. Neurological and emotional responses
depression and to report suicidal ideation. 1. Depression
F. Encourage family members to discuss their 2. Panic disorders
response to clients illness. 3. Paranoid reaction
G. Be accepting and nonjudgmental of negative 4. HIV dementia complex
responses (e.g., anger, hopelessness). C. See AIDS (Unit 4) for other physical assessment
H. Support family efforts to develop plans for their findings.
participation in clients care.
Analysis
Evaluation A. Anxiety
A. Client B. Fear
1. Receives appropriate treatment for any C. Ineffective denial
physiologic problems. D. Anticipatory grieving
2. Recognizes/discusses positive and negative E. Ineffective coping
responses to illness. F. Powerlessness
3. Understands effects of feelings about body G. Risk for violence, self-directed
image, self-esteem, role function. H. Social isolation
4. Agrees to report depression or suicidal
thoughts.

7 678 NCLEX-RN Review


53155_07_Ch07_p621-684.qxd 2/27/09 6:29 PM Page 679

Planning and Implementation D. Desire to discuss impending death, value of own life
E. Level of consciousness
Goals F. Family needs
A. Client will:
1. Communicate responses (physical and Analysis
psychologic) to disease process
Nursing diagnoses for the dying client may include:
2. Maintain ADLs as long as possible
A. Anxiety
3. Report suicidal ideation/impulses
B. Pain
B. Family/partners will:
C. Ineffective coping
1. Seek support and education relating to care of
D. Fear
HIV-positive client
E. Anticipatory grieving
2. Communicate responses to clients illness to
F. Hopelessness
nurse/support group
G. Impaired mobility
C. Health care workers will:
H. Powerlessness
1. Discuss feelings of homophobia,
I. Self-care deficit
addictophobia, and fear of infection
J. Social isolation
2. Attend groups for education and support

Interventions Planning and Implementation


A. Monitor cognitive and affective domain. Goals
B. Encourage communication of fears and concerns. A. Client will:
C. Maintain nonjudgmental attitude. 1. Be maintained in optimum comfort.
D. Assist client/family through grieving process. 2. Not be alone.
E. Provide opportunities for decision making to client 3. Have opportunity to discuss what death means
and/or caregivers. and to progress through stages of dying.
B. Family will have opportunity to be with client as
Evaluation much as they desire.
A. Client participates in care decisions.
B. Client and caregivers discuss responses to illness.
Interventions
C. Client expresses anger but does not harm self. A. Recognize clients/families have own way of
dealing with death and dying.
DEATH AND DYING B. Support clients/families as they work through
dying process.
Overview C. Accept negative responses from clients/families.
D. Encourage clients/families to discuss feelings
A. One of the most difficult issues in nursing practice related to death and dying.
B. Often difficult for nurses to maintain objectivity E. Support staff and seek support for self when
because of identification and response to death dealing with dying client and grieving family.
based on own value system and personal
experiences
Evaluation
Assessment A. Client
1. Takes opportunity to discuss feelings about
A. Stage of dying (Kubler-Ross); see Table 7-11 impending death and eventually
B. Physical discomfort acknowledges inevitable outcome.
C. Emotional reaction (withdrawal, anger, 2. Is comfortable and participates in self-care for
acceptance) and stage of dying as long as possible.
B. Family discusses feelings about loss of loved one.

Table 7-11 Stages of Dying


GRIEF AND MOURNING
1. Denial and isolation
2. Anger Overview
3. Bargaining
4. Depression A. Response to loss (person, body part, role)
5. Acceptance B. Biologic, psychologic, social implications
C. Family system effects

PSYCHIATRIC-MENTAL HEALTH NURSING

7 679
53155_07_Ch07_p621-684.qxd 2/27/09 6:29 PM Page 680

D. Mourning is process to resolve grief mother told the nurse that the child grabbed for
1. Shock, disbelief are short term the hot coffee cup and spilled it on herself.
2. Resentment, anger Legally, what is the nurse required to do?
3. Concentration on loss 1. Testify in court on the injuries.
a. Possible auditory, visual hallucinations
2. Report suspected child abuse.
b. Possible guilt
c. Possible fear of becoming mentally ill 3. Have the mother arrested.
4. Despair, depression 4. Refer the mother to counseling.
5. Detachment from loss
6. Renewed interest, investment in 133. A toddler was admitted for second-degree burns
others/interests surrounding the genital area. Her mother told
the nurse that the child grabbed the hot coffee
cup and spilled it on herself. The toddlers
Assessment mother is 17 years old. In which of the areas
A. Weight loss would the nurse provide health teaching?
B. Sleep disturbance 1. Normal growth and development.
C. Thoughts centered on loss 2. Bonding techniques.
D. Dependency, withdrawal, anger, guilt
E. Suicide potential 3. How to childproof the apartment.
4. Parenting skills.
Analysis 134. A young woman was returning home from work
late and was sexually assaulted. She was
A. Ineffective coping
B. Hopelessness brought to the emergency room upset and
C. Sleep pattern disturbance crying. What is the nurses main goal?
D. Disturbed thought processes 1. Assist her in crisis.
E. Risk for violence, self-directed 2. Notify the police of the alleged assault.
3. Understand she will have a long recovery
Planning and Implementation period.
4. Provide support and comfort.
Goals
135. The nurse is caring for a young woman who was
Client/family will:
A. Discuss responses to loss. sexually assaulted. Which of the following is
B. Resume normal sleeping/eating patterns. indicative of successful adjustment to the trauma?
C. Resume ADLs as they accept loss. 1. She moves to another city.
2. She resumes her work and activities.
Interventions 3. She takes classes in the martial arts.
A. Encourage client/family to express feelings. 4. She remains silent about the assault.
B. Accept negative feelings/defenses. 136. A young man has recently begun experiencing
C. Employ empathic listening.
forgetfulness, disorientation, and occasional
D. Explain mourning process and relate to
lapses in memory. The client was diagnosed
client/family responses.
with AIDS dementia. His family began sobbing
E. Refer client/family to support groups.
on hearing the diagnosis. What would be an
appropriate response from the nurse?
Evaluation 1. You must never give up hope.
Client/family 2. He was in a high-risk group for AIDS.
A. Express feelings. 3. I can understand your grief.
B. Progress through mourning process.
4. This must be very difficult for you.
C. Seek necessary support groups.
137. The nurse is planning care for a young man who
has AIDS dementia. What is the primary goal in
his care?
Sample Questions
1. Enhance the quality of life.
2. Teach him about AIDS.
132. An 18-month-old has been admitted for second- 3. Discuss his future goals.
degree burns surrounding the genital area. Her

7
4. Provide him with comfort and support.

680 NCLEX-RN Review


53155_07_Ch07_p621-684.qxd 2/27/09 6:29 PM Page 681

138. What is one of the major fears experienced by 142. Which of the following statements made by a
people with AIDS? victim of spouse abuse would indicate to the
1. Dying. nurse that the woman was admitting that she
2. Debilitation. was a victim of spousal abuse?
3. Stigma. 1. It would be nice to be out of the situation,
but I cannot afford to leave. I have no skills.
4. Poverty.
2. My husband has never visited me when Ive
139. A school nurse is assessing a second-grade child been in the hospital. He even said he will
for symptoms of sexual abuse. Which of the take me out more often.
following behavioral symptoms would support 3. Last time it happened I tried to talk to his
the possibility of sexual abuse? mother. She said he was never like this
1. Enuresis, impulsivity, decline in school growing up.
performance. 4. I have the shelter number and Ive decided
2. Thumb sucking, isolating self from peers on to work on my high school diploma while the
playground, excessive fearfulness. kids are in school each day.
3. Hyperactivity, rocking, isolating self from
143. A 78-year-old male with a history of cancer of
peers on playground.
the prostate is admitted to the medical unit for
4. Stuttering, rocking, impulsivity. the fourth time in 6 weeks. On admission, the
client is confused and has a decubitis ulcer the
140. A 21-year-old college student is seen in the ER
size of a fifty cent piece on the sacral area. The
following an incident of date rape. During the
client did not have this breakdown on discharge
nursing assessment, the client describes the
10 days ago. The nurse also notes what appear
entire chain of events with a blank facial
to be friction burns on both wrists. Which of the
expression. She ends her comments by saying,
following nursing diagnosis statements takes
Its like it didnt happen to me at all. Which of
priority in the care of this patient?
the following statements most accurately
explains that patients reaction? 1. Impaired skin integrity.
1. This client is using dissociation/isolation as a 2. Disturbed thought processes.
defense mechanism to cope with the attack. 3. Ineffective health maintenance.
2. This client is using denial as a defense 4. Risk for injury.
mechanism to cope with the attack.
144. A 27-year-old is admitted to the medical unit
3. This client is in the shock phase of a crisis
with severe abdominal pain, dehydration, and
and is repressing feelings associated with the
renal insufficiency associated with substance
traumatic event.
abuse. The patients admitting chest X-ray shows
4. This client is using reaction formation to diffuse interstitial infiltrates and the physician
manage the hostility she feels toward the asks that the client give consent for HIV testing.
attacker. The client consents and the test returns positive.
After learning of the positive results, the client
141. A 38-year-old mother of three children is seen in
says to the nurse, I never thought this would
the medical clinic with complaints of chronic
happen to me. I dont know if I can go through
fatigue. The woman looks sad, makes only brief
this. Which of the following nursing diagnosis
eye contact, and startles easily. The nurse
statements is of highest priority for this patient?
acknowledges these observations and the
woman says, My husband has started to hold a 1. Anticipatory grieving.
gun to my head when I dont do exactly what he 2. Risk for infection.
wants. Which of the following is the most 3. Risk for self-directed violence.
appropriate response by the nurse? 4. Thought process, altered.
1. What is it you wont do that makes him do
this? 145. The nurse is changing the dressing on a client
2. Tell me what has influenced your decision who has had a modified radical mastectomy
to stay with your husband? 2 days ago. The client refuses to look in the
direction of the nurse or the operative site. The
3. That is abusive behavior; there are resources
nurse notices a tear running down the clients
which can help you.
cheek. Which of the following responses would
4. How often does this happen? most appropriately facilitate the clients grief

7
resolution?

PSYCHIATRIC-MENTAL HEALTH NURSING 681


53155_07_Ch07_p621-684.qxd 2/27/09 6:29 PM Page 682

1. You look very sad, it might help you feel


better if you let yourself cry. Answers and Rationales
2. Tell me whats the worst part about losing
your breast.
132. 2. Legal statutes require health professionals to
3. Everything is going to be all right; you can be
report suspected cases of child abuse. The burn
fitted for a new bra and no one will notice.
pattern described is consistent with being placed
4. Are you crying because you are concerned in a tub of very hot water.
about how your partner will respond?
133. 4. Because the toddlers mother is only 17 years
146. A 42-year-old male is admitted to the medical old, she needs information and role modeling on
unit for insertion of an access site for how to provide an emotionally and physically
hemodialysis. The client relates that his safe environment for her child. This response is
transplant graft failed, he has lost his job due to more inclusive and includes the other responses.
corporate downsizing, and his wife left him
recently. He has now moved back into his 134. 1. A sexual assault is a crisis situation that
parents home. Which of the following nursing requires crisis intervention.
diagnosis statements takes priority in planning
nursing care for this client? 135. 2. The goal of adjustment is to have the woman
1. Fluid volume deficit. return to her precrisis level of functioning.
2. Ineffective denial. 136. 4. AIDS is an illness that generates intense
3. Ineffective tissue perfusion; renal. emotional reactions and fears. Acknowledging
4. Powerlessness. these feelings allows the family to discuss them
with the nurse in a nonthreatening environment.
147. The condition of a client diagnosed with chronic
obstructive pulmonary disease (COPD) and cor 137. 1. Because the clients illness has no cure and
pulmonale is deteriorating. The client is very the progression is dependent on the body system
hypoxemic, obtunded, and easily fatigued by affected, the primary goal is to ensure his
any activity. The nurse who has been working personal dignity and make plans to fulfill
with this client throughout this hospitalization personal goals.
is repositioning the client. Which of the
following remarks made by the client indicates 138. 2. Research has found that many clients with
that the client has come to terms with death? AIDS are most fearful of the debilitating effects
1. It is finally spring and that is my favorite of the disease.
time of year. 139. 2. Behavioral symptoms of children who are
2. Am I going to die? victims of sexual abuse include: regression
3. Im very tired, but content and ready to go. (thumb sucking would be regressive behavior in
4. Im feeling stronger by the moment today. a second-grade child who is probably around
7 years old), disturbed sleep patterns, clinging
148. A family member whose mother is terminally ill behaviors, lack of peer friendships, sexual acting
asks to speak to the nurse. Which of the out, running away or threats to do so, and
following statements made by this family suicide attempts.
member should indicate to the nurse that this
family member understands the emotional 140. 1. One of the defense mechanisms that a person
response to death and dying? can use to manage the anxiety associated with
1. Mother seems very comfortable; so were an attack/rape is dissociation/isolation in which
able to recall some of our good times spent the client strips an event of its emotional
together. significance and affective content.
2. My mother is irate because she says you all 141. 3. This response identifies the husbands
told her she had to have an advanced behavior as abusive and offers help for the wife
directive. if she is ready to consider other options. It does
3. My mother is talking about redoing her not cast judgment on her or question why she
bedroom when shes discharged. Doesnt she stays.
know shes dying?
4. My mother is crying so much these days. 142. 4. This statement acknowledges that the victim
Wheres all this sadness coming from? has admitted the need for protection in case of

7
emergency and is making plans to work on

682 NCLEX-RN Review


53155_07_Ch07_p621-684.qxd 2/27/09 6:29 PM Page 683

establishing some degree of autonomy, which is there may be few remaining times to share these
a factor that keeps many women in abusive memories. This sharing indicates both have
relationships. accepted death of mother and its finality.

143. 4. The highest priority for this client based on


the available data is the increased risk for injury
because of confusion. The nurses immediate REFERENCES AND SUGGESTED READINGS
concern must be the clients safety in the present
American Psychiatric Association. (2000). Diagnostic and
environment. statistical manual of mental disorders: Text revision (4th ed.).
Washington, DC: Author.
144. 3. Based on the patients comment, the highest Antai-Otong, D. (2008). Psychiatric nursing, biological and
priority of care for this client immediately is the behavioral concepts (2nd ed.). Clifton Park, NY: Delmar
risk of suicide. He states he doesnt know if he Cengage Learning.
can go through this. Suicide is a common reaction Austism Speaks, Inc. (2008). Retrieved on Sept 7, 2008, from
of persons who learn they are HIV-positive, http://www.autismspeaks.org
Boyd, M. A. (2008). Psychiatric nursing: Contemporary practice
which is associated with stigma and many losses.
(4th ed.). Philadelphia: Lippincott.
The client does not have a history of positive Centers for Disease Control and Prevention (2008). Autism
coping, which increases the risk of suicide. Information Center. Retrieved on Sept. 7, 2008, from
http://www.cdc.gov/ncbddd/autism.
145. 1. This acknowledges the clients mood and Fortinash, K. M., & Holodey Worret, P. A. (2007). Psychiatric
gives her permission to cry. Crying puts the mental health nursing (4th ed.). St. Louis. MO: Elsevier Health
client in touch with the sadness/pain over the Sciences.
loss. Offering permission to cry facilitates Frisch, N. C., & Frisch, L. E. (2002). Psychiatric mental health
expression of feelings related to the loss. nursing (3rd ed.). Clifton Park, NY: Delmar Cengage
Learning.
Gorman, L., Sulton, D., & Raines, M. (2008). Daviss manual of
146. 4. Powerlessness, or feelings of uncertainty
psychosocial nursing for general client care (3rd ed.).
about the future, may be present in this client Philadelphia: F. A. Davis.
due to the uncertainty about his future in several Mullins, D. (2006). 501 human diseases. Clifton Park, NY:
areas: job, another transplant, long-term Delmar Cengage Learning.
hemodialysis, reconciliation in his marriage, Sadock, B., & Sadock, V. (2007). Kaplan and Sadocks synopsis of
whether he will have to be dependent on parents psychiatry: Behavioral sciences/clinical psychiatry (10th ed.).
long term, and concern about their own possible Philadelphia: Lippincott, Williams & Wilkins.
Spratto, G. R., & Woods, A. L. (2009). Delmar Nurses Drug
declining health.
Handbook 2009. Clifton Park, NY: Delmar Cengage
Learning.
147. 3. This response indicates that the client is Townsend, M. C. (2008) Nursing diagnosis in psychiatric
exhausted and ready to let the natural processes nursing: care plans and psychotropic medications (7th ed.).
take their course. The client is at peace. Philadelphia: F. A. Davis.
Varcarolis, E. M., Carson, V. B., & Shoemaker, N. C. (2006).
148. 1. This statement indicates that the family Foundations of psychiatric mental health nursing: A clinical
member and mother have been able to reminisce approach (5th ed.). Philadelphia: W. B. Saunders.
about good times together, acknowledging that

PSYCHIATRIC-MENTAL HEALTH NURSING

7 683
53155_07_Ch07_p621-684.qxd 2/27/09 6:29 PM Page 684
53155_08_APP_p685-698.qxd 3/3/09 2:00 PM Page 685

A P P E N D I X

SPECIAL DIETS

APPENDIX OUTLINE
686 Diabetic Diet and Exchange
Lists
690 Renal Diet
691 Bariatric Diet
692 High-Fiber Diet
692 1500-Kilocalorie Diet
694 1000-MilIigram Sodium-
Restricted Diet
695 Bland Diet
696 Low-Residue Diet
696 20-Gram Fat-Restricted Diet
697 Fat-Controlled Diet

685
53155_08_APP_p685-698.qxd 3/3/09 2:00 PM Page 686

Diabetic Diet and Exchange Lists

DIABETIC DIET
Description
A diabetic diet is prescribed for clients with diabetes mellitus. Purposes include: attain or maintain ideal body weight,
ensure normal growth, and maintain plasma glucose levels as close to normal as possible. Food preparation includes:
Distribution of kcal: protein 1220%; carbohydrates 5560%; fats (unsaturated) 2025%.
Daily distribution of kcal: equally divided among breakfast, lunch, supper, snacks.
Foods high in fiber and complex carbohydrates.
No simple sugars, jams, honey, syrup, frosting.

EXCHANGE LISTS FOR MEAL PLANNING

Food Exchange Lists*

List Measure Carbohydrates (g) Protein (g) Fat (g) Energy (kcal)
Milk, nonfat (see List 1) 1 cup 12 8 trace 80
Milk, whole (see List 1) 1 cup 12 8 9 160
Vegetables (see List 2) 1
2 cup 5 2 25
Fruits (see List 3) Varies 10 40
Breads, cereals, and starchy Varies 15 2 70
vegetables (see List 4)
Meat, low fat (see List 5) 1 oz 7 2.5 50
Meat, medium fat (see List 5) 1 oz 7.5 5 75
Meat, high fat (see List 5) 1 oz 7 7.5 95
Fat (see List 6) Varies 5 45

*Resource: Committees of the American Diabetes Association, Inc., and the American Diatetic Association: Exchange Lists for Meal Planning. Chicago: The
American Dietetic Association and the American Diabetes Association, in cooperation with the National Institute of Arthritis, Metabolism and Digestive
Diseases and the National Heart, Blood and Lung Institute, Public Health Service, U.S. Department of Health, Education and Welfare, 1976.

List 1: Milk Exchanges

Amount to Use
Nonfat, fortified
Use only this list for diets restricted in saturated fat.
Skim or nonfat milk 1 cup
Powdered (nonfat dry) 1/3 cup
Canned, evaporated, skim 1
2 cup
Buttermilk made from skim milk 1 cup
Yogurt, made from skim milk, plain, unflavored 1 cup

Low fat, fortified


1% fat, fortified (omit 12 fat exchange) 1 cup
2% fat, fortified (omit 1 fat exchange) 1 cup
Yogurt made from 2% fortified, plain, unflavored (omit 1 fat exchange) 1 cup

Whole
Whole milk 1 cup
Canned evaporated 2 cup
1

Buttermilk made from whole milk 1 cup


Yogurt made from whole milk, plain, unflavored 1 cup

686 NCLEX-RN Review


53155_08_APP_p685-698.qxd 3/3/09 2:00 PM Page 687

List 2: Vegetable Exchanges

One-half cup equals one exchange.


Asparagus* Greens* Onions
Bean sprouts Beet greens Rhubarb
Beets Chard Rutabaga
Broccoli* Collards Sauerkraut
Brussels sprouts Dandelion greens String beans, green or yellow
Cabbage* Kale Summer squash
Carrots Mustard greens Tomatoes*
Celery Spinach Tomato juice
Cauliflower* Turnip greens Turnips
Cucumbers Mushrooms Vegetable juice cocktail
Eggplant Okra Zucchini
*Good sources of ascorbic acid.
Good sources of vitamin A.

These vegetables can be used as desired: chicory, Chinese cabbage, endive, escarole, lettuce, parsley, radishes, and watercress.
See List 4, Bread Exchanges, for starchy vegetables.

List 3: Fruit Exchanges

Amount to Use Amount to Use


Apple 1 small Mango* 1
2 small
Apple juice 1/3 cup Cantaloupe* 1
4 small
Applesauce (unsweetened) 1
2 cup Honeydew* 1/8 medium
Apricots, fresh 2 medium Watermelon 1 cup
Apricots, dried 4 halves Nectarine 1 medium
Bananas 1
2 small Orange* 1 small
Blackberries 1
2 cup Orange juice* 1
2 cup
Blueberries 1
2 cup Papaya* 3
4 cup
Raspberries 1
2 cup Peach 1 medium
Strawberries 1
2 cup Pear 1 small
Cherries 10 large Persimmon 1 medium
Cider 1/3 cup Pineapple 1
2 cup
Dates 2 Pineapple juice 1/3 cup
Figs, fresh 1 Plums 2 medium
Figs, dried 1 Prunes 2 medium
Grapefruit* 1
2 Prune juice 1
4 cup
Grapefruit juice 1
2 cup Raisins 2 tbsp
Grapes 12 Tangerine* 1 large
Grape juice 1
4 cup
*Good sources of ascorbic acid.
Good sources of vitamin A.

Tanberries may be used as desired if no sugar is added.

List 4: Bread, Cereal, and Starchy Vegetable Exchanges

Amount to Use Amount to Use


Bread English muffin, small 1
2
White (including French and Italian) 1 slice Frankfurter roll 1
2
Whole wheat 1 slice Hamburger bun 1
2
Rye or pumpernickel 1 slice Plain roll (bread) 1
Raisin 1 slice Dry bread crumbs 3 tbsp
Bagel, small 1
2 Tortillas, 6 inch 1
(Continues)

Appendix: SPECIAL DIETS 687


53155_08_APP_p685-698.qxd 3/3/09 2:00 PM Page 688

List 4: Bread, Cereal, and Starchy Vegetable Exchanges (Continued)

Amount to Use Amount to Use

Cereal Starchy Vegetables


Bran flakes 1
2 cup Corn 1
3 cup
Other ready-to-eat unsweetened cereal 3
4 cup Corn on cob 1 small
Puffed cereal, unfrosted 1 cup Lima beans 1
2 cup
Cereal, cooked 1
2 cup Parsnips 2
3 cup
Grits, cooked 1
2 cup Peas (green, fresh, canned, or frozen) 1
2 cup
Rice or barley, cooked 1
2 cup Potato, white 1 small
Pastas, cooked 1
2 cup Potato, mashed 1
2 cup
Popcorn, popped 3 cups Pumpkin 3
4 cup
Cornmeal, dry 2 tbsp Winter squash, acorn or butternut 1
2 cup
Flour 212 tbsp Yam or sweet potato 1
4 cup
Wheat germ 1
2 cup
Prepared Foods
Crackers Biscuit, 2 inch diam. (omit 1 fat
Arrowroot 3 exchange) 1
Graham, 212 inch 2 Corn bread, 2 3 2 3 1 inch (omit 1 fat
Matzoh, 4 3 6 inch 1
2 exchange) 1
Oyster 20 Corn muffin, 2 inch diam. (omit 1 fat
Pretzels, 318 inch 3 18 inch 15 exchange) 1
Rye wafers, 2 3 312 inch 3 Crackers, round, butter type (omit 1 fat
Saltines 6 exchange) 5
Soda, 218 inch square 4 Muffin, plain, small (omit 1 fat exchange) 1
Pancake, 5 3 12 inch (omit 1 fat exchange) 1
Dried Beans, Peas, and Lentils
Potatoes, french fried, 2 inch to 312 inch
Dried beans, peas, and lentils cooked 2 cup
1
(omit 1 fat exchange) 15
Baked beans, no pork 4 cup
1
Waffle, 5 3 12 inch (omit 1 fat exchange) 1

List 5: Meat and Protein-Rich Exchanges

Amount to Use
Lean Meat, Protein-Rich Exchanges
Use only this list for diets low in saturated fat and cholesterol.
Beef
Baby beef (very lean), chipped beef, chuck, flank steak, tenderloin, plate
ribs, plate skirt steak, round (bottom, top), all cuts rump, spare ribs, tripe 1 oz
Lamb
Leg, rib, sirloin, loin (roast and chops), shank, shoulder 1 oz
Pork
Leg (whole rump, center shank), smoked ham (center slices) 1oz
Veal
Leg, loin, rib, shank, shoulder, cutlets 1 oz
Poultry without skin
Chicken, turkey, Cornish hen, guinea hen, pheasant 1 oz
Fish, any fresh or frozen
Canned crab, lobster, mackerel, salmon, tuna 1
4 cup
Clams, oysters, scallops, shrimp 5 or 1 oz
Sardines, drained 3
Cheeses containing less than 5% butterfat 1 oz
Cottage cheese: dry or 2% butterfat 1
4 cup
Dried peas and beans (omit 1 bread exchange) 1
2 cup

688 NCLEX-RN Review


53155_08_APP_p685-698.qxd 3/3/09 2:00 PM Page 689

List 5: Meat and Protein-Rich Exchanges (Continued)

Amount to Use

Medium-Fat Meal, Protein-Rich Exchanges


Beef
Ground, 15% fat; comed beef, canned; rib eye, round, ground (commercial) 1 oz
Pork
Loin, all cuts tenderloin, shoulder arm (picnic); shoulder blade, Boston
butt, Canadian bacon; boiled ham 1 oz
Liver, heart, kidney, and sweetbreads (high in cholesterol) 1 oz
Cottage cheese, creamed 1
4 cup
Cheese
Mozzarella, ricotta, farmers, Neufchtel 1 oz
Parmesan 3 tbsp
Eggs (high in cholesterol) 1
Peanut butter (omit 2 fat exchanges) 2 tbsp

High-Fat Meal, Protein-Rich Exchanges


Beef brisket; corned beef brisket; ground beef (over 20% fat); hamburger
(commercial); chuck, ground (commercial); rib roast, club and rib steak 1 oz
Lamb, breast 1 oz
Pork, spare ribs; loin (back ribs); pork, ground; country style ham, deviled ham 1 oz
Veal, breast 1 oz
Poultry: capon, duck (domestic), goose 1 oz
Cheese: cheddar type 1 oz
Cold cuts, 412 3 1/8 inch 1 slice
Frankfurter small

List 6: Fat Exchanges

Amount to Use

For a diet low in saturated fat and higher in polyunsaturated fat


select only from this list.
Margarine: soft, tub, or stick (made with corn, cottonseed,
safflower, soy, or sunflower oil) 1 tbsp
Avocado, 4 inch diam. 1/8
Nuts
Almonds* 10 whole
Peanuts*
Spanish 20 whole
Virginia 10 whole
Pecans* 2 large, whole
Walnuts 6 small
Other nuts* 6 small
Oil, corn, cottonseed, safflower, soy, sunflower 1 tsp
Oil, olive or peanut* 1 tsp
Olives* 5 small
Salad dressings, if made with corn, cottonseed,
safflower or soy oil
French dressing 1 tbsp
Italian dressing 1 tbsp
Mayonnaise 1 tsp
Salad dressing, mayonnaise type 2 tsp
(Continues)

Appendix: SPECIAL DIETS 689


53155_08_APP_p685-698.qxd 3/3/09 2:00 PM Page 690

List 6: Fat Exchanges (Continued)

Amount to Use

The following fats should not be used on a diet low in saturated fat.
Margarine, regular stick 1 tsp
Butter 1 tsp
Bacon fat 1 tsp
Bacon crisp 1 strip
Cream, light 2 tbsp
Cream, sour 2 tbsp
Cream, heavy 1 tbsp
Cream cheese 1 tbsp
Lard 1 tsp
Salad dressings (permitted on restricted diets
if made with allowed oils)
French dressing 1 tbsp
Italian dressing 1 tbsp
Mayonnaise 1 tsp
Salad dressing, mayonnaise type 2 tsp
Salt pork 3
4 inch cube
*Fat content is primarily monounsaturated.

Renal Diet

Description
A renal diet consists of controlling the intake of fluids, potassium, phosphorus, and sodium (salts). A typical renal diet
could be written as 80-3-3, which means 80 grams of protein, 3 grams of sodium, and 3 grams of potassium a day.

Food Preparation
Foods should be selected that are restrictive in sodium and potassium levels. Fluid intake will also need to be limited.

Diet Guidelines

Type of food Allow Avoid


Milk Cool Whip, mocha mix, rice milk Eggnog
Eggs Egg substitute
Cheese All More than one 4-oz serving
Meat, fish, poultry Beef, chicken, fish, fresh pork, lamb, tuna, turkey Bacon, bologna, canned meat,
ham, sausage, hot dogs, lunchmeat
Vegetables, potatoes 1
2 cup baked, boiled, fried
Other vegetables Beans, cabbage, cauliflower, celery, eggplant, leek, Artichoke, beet, dried beans, pumpkin, spinach,
lettuce, mushrooms, onion, peppers, radishes, turnips sweet potato, tomato, winter squash
Cereals Dry, low salt; Cream of Rice, hominy grits, Malt-o-Meal,
puffed rice, puffed wheat, wheat farina
Bread Bagels, hamburger, hot dog buns; hard or dinner rolls Dark, whole, unrefined grains; croutons
Crackers Unsalted Salted crackers
Other cereal Couscous, macaroni, noodles, rice, spaghetti Chow mein noodles
Fats Butter, canola oil, cream cheese, margarine,
mayonnaise, Miracle Whip, nondairy creamers,
olive oil, salad dressings, sour cream
(Continues)

690 NCLEX-RN Review


53155_08_APP_p685-698.qxd 3/3/09 2:00 PM Page 691

Diet Guidelines (Continued)

Type of food Allow Avoid


Fruits Berries, cranberries, figs, fruit cocktail, grapes, lemons, Avocado, bananas, dried fruits,
limes, mandarin oranges, peaches, pears, plums, kiwis, mangoes, melons, nectarines,
rhubarb oranges, tangeloes, papayas, raisins
Desserts Animal crackers, cookies, hard candy, Fig Newtons Banana pudding
Beverages Coffee, fruit punch, grape juice, lemon/lime Orange juice, prune juice, unsalted tomato
soda, tea juice, dark-colored sodas
Sauces Bottled sauces
Soup Canned soups
Condiments
Sweets Hard candy Chocolate

Bariatric Diet

The bariatric diet is used for clients who have had bariatric surgery for obesity. Depending on the type of bariatric
surgical procedure, some foods are restricted while others are limited. Weight loss is considerably faster with gastric
banding and gastroplasty. To avoid nutritional problems, additional vitamin and mineral supplements may be needed.

Food Preparation
Food intake is started in liquid form and progresses to a regular diet but with limited proportions.
Postoperative 12 daysIce chips, water, sugar-free noncarbonated beverage (up to 4 oz per hour)
Postoperative 512 daysAdditions to above; sugar-free popsicles, diet Jell-O, decaffeinated coffee/tea,
chicken/beef broth. Client can avoid clumping syndrome by restricting high carbohydrate foods and drinking
liquids between meals instead of with meals.

Diet Guidelines

Type of food Allow Avoid


Milk Nonfat, 1% milk, plain soy milk High fat milk, yogurt
Eggs Egg Beaters Those prepared with fats other than
vegetable oils
Cheese Low-fat cheese Whole milk cheese
Meat, fish, poultry Turkey, chicken, fresh fish, low fat deli meat, water-packed Fatty meat or fish
tuna; reduced fat ham
Vegetables Fresh vegetables; all (baked, boiled) Fried, French fries
Potatoes
Other vegetables Vegetable broth Fried vegetables
Cereals Cream of Rice, Cream of Wheat, whole grain cereal, regular Sugary cereals
or unflavored oatmeal
Breads Whole wheat or oat bran White
Other cereal products Pasta noodles, rice
Fats Spray olive oil, nonfat mayonnaise, light margarine, Palm oil, full-fat condiments
reduced-calorie salad dressing
Fruits All fresh fruits Canned in syrup fruit
Dessert Sugar-free pudding, custard Highly concentrated sweets (cakes,
cookies, doughnuts)
Beverages Unsweetened, pulp-free juice, coffee (no sugar), unsweetened Sweetened juice, tea
tea, Crystal Light, bottled water, herbal tea

Appendix: SPECIAL DIETS 691


53155_08_APP_p685-698.qxd 3/3/09 2:00 PM Page 692

High-Fiber Diet

Description
The diet is essentially a normal diet with increased amounts of cellulose, hemicellulose, lignin, and pectin. It
increases the volume and weight of the stool; increases gastrointestinal motility; and decreases intraluminal
colonic pressure in clients suffering from increased pressure, including clients with constipation, hemorrhoids,
and long-term management of diverticulosis.

General Characteristics
Consume a regular diet with increased fiber content.
1. Include raw fruits and vegetables instead of canned or cooked ones.
2. Substitute whole-grain bread and cereals for refined grains.
3. Include dried fruits and nuts in meals. (Nuts may be eliminated in clients with diverticulosis.)
4. Prepare soups with high-fiber vegetables.
5. Eat a fresh vegetable or fruit salad daily.
6. Add 12 tablespoons of bran to other foods daily.
7. Initiate the high-fiber diet gradually to prevent gas and loose stools.

Possible Complications
l. Osmotic diarrhea.
2. Decreased serum levels of minerals, such as iron, calcium, magnesium, etc.

1500-Kilocalorie Diet

Description
1500-kcal diet will permit a steady weight loss of body fat without loss of body tissue and other essential body
components. The diet meets the Recommended Dietary Allowances for the adult for protein, minerals, and vitamins.

Food Preparation
Foods should be prepared without added sugar and flour, using only that amount of fat allowed in the diet. Meats
may be broiled, braised, stewed, or roasted. All visible fat should be trimmed. The measure or weight of food refers
to the food in its cooked form.

Diet Guidelines
Food Exchange Lists

Daily Allowance
List 1: Milk: 1 serving = 8 ounces 2 cups
Skim milk (1% fat)
Buttermilk (1% fat)
Yogurt made from
skim milk

692 NCLEX-RN Review


53155_08_APP_p685-698.qxd 3/3/09 2:00 PM Page 693

Food Exchange Lists (Continued)

Daily Allowance
List 2: Vegetables: 1 serving = 2 cup
1
3 servings
Asparagus Cauliflower Onions
Been sprouts Celery Sauerkraut
Beets Eggplant String beans
Broccoli Greens Summer squash
Brussels sprouts Green peppers Tomatoes
Cabbage Mushrooms Tomato/vegetable juice
Carrots Okra Turnips
Zucchini
List 3: Fruits (fresh or unsweetened) 5 servings
Small apple 2 dates Medium peach
1/3 cup apple juice 1
2 grapefruit 1
2 cup pineapple
2 apricots 1
2 cup grapefruit juice 1/3 cup pineapple juice
1
2 small banana 2 tbsp raisins 2 plums
1
2 cup berries 12 grapes 2 prunes
3
4 cup strawberries 1/8 honeydew melon 1
4 cup prune juice
1
4 cantaloupe Small orange 1 cup watermelon
10 cherries 1
2 cup orange juice Medium tangerine
List 4: Bread, Cereal, Starchy Vegetables 6 servings
1 slice bread 1
2 cup rice, grits 1
2 matzoh
1
2 bagel 1
2 cup peas, beans 25 small pretzels (3 1/8 long, 1/8 inch diam)
1
2 English muffin 1
4 cup baked beans 3 cups popcorn
1
2 hamburger roll 1
4 cup sweet potato 1
2 cup pastas
3
4 cup dry cereal 2 graham crackers 1 small potato
1
2 cup cooked cereal 6 saltine crackers 1/3 cup corn
List 5: Meat and Poultry Foods Lean 3 oz total or 34 cup
1 oz beef: leg, round, chipped, rump, loin
1 oz lamb; leg, rib, loin, sirloin
1 oz veal: leg, loin, rib, cutlets
1 oz pork: leg, rump, center slice
1 oz poultry: without skin (no duck or goose)
1
4 cup tuna, salmon, crab, shrimp, lobster
1
4 cup dry cottage cheese
Medium Fat 3 oz total or 34 cup
1 oz ground beef (15% fat)
1 oz pork shoulder, boiled ham, Canadian bacon
1 oz liver
1 egg
1 oz mozzarella, ricotta, farmers cheese
1
4 cup cottage cheese
List 6: Fats 5 servings
1 tsp butter 10 peanuts 1 tsp bacon fat
1 tsp margarine 3
4 inch cube salt pork 5 small olives
1 tsp oil 1 tbsp cream cheese 1 tsp mayonnaise
1 tbsp French dressing 2 tbsp cream 1 tsp lard
1 tbsp Italian dressing 1 slice crisp bacon

Foods Allowed
Cucumbers Dill pickles Chinese cabbage
Endive Escarole Lettuce
Parsley Radishes Bouillon
Unflavored gelatin Coffee Tea
Spice

Appendix: SPECIAL DIETS 693


53155_08_APP_p685-698.qxd 3/3/09 2:00 PM Page 694

1000-Milligram Sodium-Restricted Diet

Description
The aim of this diet is to promote the loss of excess sodium and water from the extracellular fluid compartments of
the body. It is used primarily for clients with ascites/edema associated with advanced liver or renal disease, clients in
congestive heart failure, as a treatment for essential hypertension, and with clients receiving adrenocorticosteroids.

Food Preparation
All food should be prepared without the addition of salt, regular baking powder, and baking soda. No salt should
be used at the table.

General Principles
Select foods that have not been processed or preserved with large amounts of salt. Include all fruits and fruit juices,
fresh, canned, frozen, dried. Use only unsalted snack foods.

Diet Guidelines

Type of food Allow Avoid


Milk Whole or skim milk Limit: 9 cups/day Milk shakes, malted milk,
commercial chocolate, or
buttermilk
Eggs Any form prepared without the addition of
salt. Limit: 4/week
Cheese Dry cottage cheese and low-sodium American cheese All types except those allowed
Meat, fish, poultry Meat or poultry without added salt or sodium Lunchmeat, sausage, hot dogs,
products. Fresh or canned fish without added shellfish, organ meats, bacon,
salt. Unsalted peanut butter. ham, corned beef, dried beef,
canned meat, anchovies, salted
canned fish, dried cod
Soups and sauces All made with allowed milk and vegetables Broth, bouillon, gravy, canned
soup, consomme, and cream
sauce
Vegetables, potatoes Fresh, frozen or canned white and sweet potatoes Potato chips
without added salt
Other vegetables Unsalted asparagus, green beans, wax beans, corn, Beet greens, kale, frozen lima
fresh or canned lima beans, eggplant, endive, escarole, beans, olives, frozen peas,
lettuce, mushrooms, fresh or canned peas, squash, pickles, sauerkraut, Swiss chard,
tomatoes. Also, if tolerated: broccoli, Brussels, sprouts, mustard greens, dandelion
cabbage, cauliflower, turnips, onions, parsnips, greens
radishes, rutabagas, and cucumbers
Following vegetables should be limited to one serving
per day: beets, carrots, celery, spinach, or turnips
Cereals Puffed rice, puffed wheat, shredded wheat, and All other ready-to-eat cereals.
regular cooked whole-grain or enriched cereals Quick Cream of Wheat, quick-
cooling farina, hominy
Bread Regular white, whole wheat, or rye. Limit: All others
3 slices/day. If additional bread is desired, use
low-sodium bread.
Crackers Unsalted crackers All others
Other cereal Macaroni, spaghetti products, noodles, rice
Fats Unsalted butter, margarine, oil Salted butter or margarine, salad
dressings, salt pork, bacon fat

694 NCLEX-RN Review


53155_08_APP_p685-698.qxd 3/3/09 2:00 PM Page 695

Bland Diet

Description
This diet excludes foods that may be chemically or mechanically stimulating or irritating to the gastrointestinal
tract. Small, frequent meals may be indicated. Prescribed for clients with ulcers and postoperatively after some
types of surgery.

Food Preparation
Meats may be baked, broiled, stewed, or roasted, but not fried. Fruits and vegetables should be cooked or canned.
Avoid meat extracts, pepper, and chili powder.

Diet Guidelines

Type of food Allow Avoid


Milk Milk, buttermilk, cocoa, milk beverages None
Eggs Soft cooked, hard cooked, poached, steam scrambled Fried, deviled
Cheese Cottage cheese, cream, mild cheddar in sauces or Strongly flavored cheese
in combination dishes
Meat, fish, poultry Tender beef, lamb, pork, liver, poultry, veal, fish, Smoked, salted, or fatty meat
crisp bacon or fish
Vegetables, potatoes Baked (without skin), boiled, creamed, escalloped, Sweet potatoes, fried potatoes
or mashed white potatoes
Other Vegetables Cooked asparagus tips, beets, carrots, peas, chopped All others
spinach, winter squash, green beans, mushrooms,
waxed beans, strained corn pudding
Cereals Cream of Rice, Cream of Wheat, farina, cornmeal, Bran, whole-grain cereals
hominy grits, strained oatmeal, corn flakes, puffed rice,
Rice Krispies, Special K
Breads White, Italian, French, rye bread without seeds, melba toast Whole-grain breads
Crackers Soda crackers, saltines Graham crackers, crackers
with seeds
Other cereal products Macaroni, noodles, rice, spaghetti
Fats Butter, margarine, cream, salad oil All others
Fruits Avocado, ripe bananas, canned peaches, pears, Raw fruits except avocado and
Royal Anne cherries, peeled apricots, applesauce, bananas; berries, figs, pineapple
baked apple (no skin), all strained fruit juices
Desserts Plain sugar cookies, vanilla wafers, lady fingers, Any containing fruits, nuts, or
angel food cake, sponge cake, plain sherbet, spices; pastries, pies, doughnuts
plain ice cream, custard, Jell-O, Junket, Bavarian
cream, simple puddings, fruit whips
Beverages Decaffeinated beverages Caffeine-containing soft drinks,
coffee, tea; alcohol

Appendix: SPECIAL DIETS 695


53155_08_APP_p685-698.qxd 3/3/09 2:00 PM Page 696

Low-Residue Diet

Description
The low-residue diet is low in fiber, soft in texture, and easily digested. It decreases the weight and look of the stool.

Food Preparation
Fruits and vegetables should be well cooked and pureed. Meats may be baked, broiled, stewed, or roasted. The
meats that must be ground may be made from cooked meats that have had the gristle and excess fat removed, or
ground meats may be purchased and made into patties or meatloaf. The food may be mildly seasoned.

Diet Guidelines

Type of food Allow Avoid


Milk 1 pint Any additional
Eggs Hard cooked, soft cooked, steam scrambled, poached All others
Cheese Cottage, cream, mild American All others
Meat, fish, poultry Ground lean beef, lamb, veal, liver, sliced white Luncheon meats, sausages;
meat of chicken or turkey, fish, crisp bacon, smooth smoked, highly seasoned or
peanut butter highly salted meats and fish
Vegetables, potatoes White potato, strained sweet potato Potato chips or fried potatoes
Other vegetables Cooked and pureed: beets, peas, lima beans, squash, All others
string beans, spinach, asparagus, pumpkin, whole
asparagus tips and carrots cooked tender, mushrooms
tomato juice
Cereals Cream of Rice, Cream of Wheat, farina, commeal, Whole-grain cereals
strained oatmeal, corn flakes, puffed rice, Rice
Krispies, Special K
Breads White bread, melba toast, zwieback All others
Crackers Soda crackers, saltines All others
Other cereal products Macaroni, noodles, refined rice, spaghetti Whole-grain rice
Fats Butter, margarine, cream, mayonnaise, salad oil, shortening All others
Fruits All canned or strained fruit juices, cooked or canned All others
applesauce, Royal Anne cherries, peeled apricots,
peaches, pears, bananas, peeled baked apple
Desserts Simple puddings, ice cream, sherbet, plain cakes All others. Allowed desserts
and cookies, flavored gelatin, custards with nuts, seeds, or coconut
Beverages Coffee, tea, Postum, Sanka, carbonated beverages, All others
fruit juices, milk in allowed amount

20-Gram Fat-Restricted Diet

Description
The 20-gram fat-restricted diet is designed for clients with an acute intolerance for fat and for clients with high
serum cholesterol levels. Lean meat is the only source of fat. For a 40-gram fat-restricted diet, add any combination
of 4 teaspoons of the following: butter, margarine, shortening oil, mayonnaise.

696 NCLEX-RN Review


53155_08_APP_p685-698.qxd 3/3/09 2:00 PM Page 697

Diet Guidelines

Type of food Allow Avoid


Yogurt, milk Skim milk, skim milk yogurt, skim milk buttermilk Whole milk, buttermilk, yogurt
Eggs Egg whites only Whole eggs and egg yolks
Cheese Dry cottage cheese, skim milk cheese Whole milk cheese
Meat, fish, poultry Two 2-oz servings of lean beef, veal, pork, lamb, Corned beef, sausages, goose,
poultry, liver, or fish duck, fish canned in oil such as
sardines, tuna fish, and salmon;
fried meats, fish, and poultry
Vegetables, Potatoes Sweet or white Those prepared with fat
Other vegetables All prepared without fat, oil, or cream Frozen vegetables in butter or
cream sauces, au gratin;
potato chips; casseroles
Cereals Cooked and ready-to-eat cereals None
Bread White, whole wheat, and rye bread; hard, All others
water rolls, simple yeast buns
Crackers Saltine and graham All others
Other cereal products Macaroni, noodles, rice, and spaghetti
Fats None All
Fruits Any fruit or juice Olives, avocados
Desserts Fruit ices and sherbets, gelatin desserts, puddings Pies, cakes, pastries, chocolate,
prepared with skim milk, fruit whips made with egg rich desserts, ice cream, any
whites, angel food cake containing nuts, whole milk,
cream, or butter
Beverages Coffee, tea, carbonated beverages All beverages made with
whole milk
Sauces Tomato sauce and white sauce made with skim Gravies with fat, cheese sauces,
milk, gravy made with fat-free broth or drippings meat sauces, rich dessert sauces
Soups Fat-free broth soups and creamed soups made Soups made with whole milk,
with skim milk cream, or additional fat
Condiments and In moderation: salt, pepper, spices, herbs, Chocolate, nuts
miscellaneous flavoring extracts
Sweets Sugar, jelly, honey, jams, syrups, molasses, Candy with chocolate and nuts
plain hard candy

Fat-Controlled Diet

Description
The fat-controlled diet limits foods containing cholesterol and saturated fatty acids and increases foods high in
polyunsaturated fatty acids. (Cholesterol intake should be limited to 300 mg daily if diet is followed.)

Food Preparation
Only lean meats, fish, and poultry are used. The allowed vegetable oils may be used in preparing meats, fish, or
poultry; used in salad dressings; or in baked products. A portion of the total fat is allowed in the form of margarine
each day. Margarine labels should be read carefully to ensure that the one selected contains liquid polyunsaturated
oils, preferably corn, soy, or safflower.

Appendix: SPECIAL DIETS 697


53155_08_APP_p685-698.qxd 3/3/09 2:00 PM Page 698

Diet Guidelines

Type of food Allow Avoid


Milk Yogurt made from skim milk, skim milk, skim Whole milk and cream; creamed
buttermilk buttermilk
Eggs Three whole eggs per week; egg whites as desired Those prepared with fats other
than vegetable oils
Cheese Dry cottage cheese, skim milk cheese All others, whole milk cheese
Meat, fish, poultry Lean beef, veal, lamb, and pork limited to 9 oz/week; Fat meats such as bacon, duck,
fish, poultry without skin goose, sausages, luncheon
meats, frankfurters, and
spareribs; glandular and organ
meats, caviar, and shrimp
Vegetables, Potatoes Sweet or white Those prepared with fats other
than a special margarine or
vegetable oil
Other vegetables All None
Cereals All None
Bread White or whole grain, hot breads made with Breads made with other
vegetable oil shortenings and egg yolk
Crackers Soda and graham All others
Other cereal Macaroni, rice, and spaghetti Noodles made with egg
Fats Margarine (polyunsaturated), corn oil, safflower oil, Butter, mayonnaise, cream, lard,
soybean oil; salad dressings made with these oils hydrogenated vegetable
shortening, olive oil, coconut oil
Fruits All None
Desserts Fruit ices, gelatin, fruits, angel food cake, puddings Ice cream; sherbet; baked
prepared with skim milk, fruit whips made with egg products made with egg yolk,
whites, and baked products made with allowed oils shortening (other than
vegetable oil) or cream; other
desserts prepared with same
Beverages Coffee, tea made with skim milk, carbonated beverages Beverages made with whole
milk or cream
Sauces Tomato sauce and sauces made with skim milk Gravies, meat sauces, and rich
dessert sauces
Soups Homemade fat-free soups made with allowed ingredients Soups made with cream and
whole milk
Condiments All desired No restrictions
Sweets All except chocolate Chocolate

698 NCLEX-RN Review


53155_09_PT 01_p699-722.qxd 2/27/09 6:30 PM Page 699

C O M P R E H E N S I V E

PRACTICE TESTS

This section contains eleven 100-question tests similar in structure and content to those you will find on the
NCLEX-RN examination.
At the end of each test are the correct answers and a comprehensive rationale for the correct answers. Also
included are identifiers for the phases of the nursing process, the categories of client needs, the cognitive level,
and the subject area for each question.
Following the directions for test taking described in Unit 1, allow 100 minutes for each practice test. The
following codes are used in the answers and rationales to categorize the test items.
NP 5 PHASES OF THE He/3 5 Health Promotion and CL 5 COGNITIVE LEVEL
NURSING PROCESS Maintenance K 5 Knowledge
As 5 Assessment 5 Growth and Development Co 5 Comprehension
An 5 Analysis Through the Life Span Ap 5 Application
Pl 5 Planning 5 Prevention and Early An 5 Analysis
Im 5 Implementation Detection of Disease
SA 5 SUBJECT AREAS
Ev 5 Evaluation Ps/4 5 Psychosocial Integrity
1 5 Medical-Surgical
5 Coping and Adaptation
CN 5 CLIENT NEED 2 5 Psychiatric and Mental
5 Psychosocial Adaptation
Sa 5 Safe Effective Care Health
Ph 5 Physiological Integrity
Environment 3 5 Maternity and Womens
Ph/5 5 Basic Care and Comfort
Sa/1 5 Management of Health
Ph/6 5 Pharmacological and
Care 4 5 Pediatric
Parenteral Therapies
Sa/2 5 Safety and Infection 5 5 Pharmacologic
Ph/7 5 Reduction of Risk Potential
Control Ph/8 5 Physiological Adaptation

The above categories are discussed in more detail in Unit 1. The following sample answer should help you
understand how to interpret these codes. The correct answer is given, followed by the comprehensive rationale.
The codes are listed beside each question.
ANSWER RATIONALE NP CN CL SA
#1. 4. Hemorrhagic reactions are a result of banked blood that is low in An Ph/6 Co 1
platelets and coagulation factors. The other choices describe allergic
and hemolytic reactions, plus circulatory overload.
The elements are as follows:
#1 is the question or item number in the test; 4 is the correct answer.
A comprehensive rationale explains the correct answer, and may include information on the incorrect answers.
The phase of the nursing process is analysis.
The category of client need is physiologic integrity; pharmacological and parenteral therapies.
The cognitive level is comprehensive.
The subject area is medical-surgical.

COMPREHENSIVE PRACTICE TESTS 699


53155_09_PT 01_p699-722.qxd 2/27/09 6:30 PM Page 700

Practice Test 1

1. An adult who has a fractured right hip with 5 lb 1. I need to sleep with this metal eye shield at
of Bucks traction needs to be transferred to night, but I can wear my glasses during the
another bed. What instructions should the nurse day.
tell the team? 2. I should avoid coughing, sneezing, and
1. Slowly lift the traction to release the weight, vomiting.
support the right leg, and lift the client to the 3. Its okay to bend over to pick something up
new bed. from the floor as long as I put the eye shield
2. Slowly lift the 5 lb weight from the traction on.
set up, and apply 10 lb of manual traction 4. I should call the doctor for any bad pain in
during the move. my eyes that the pain medicine doesnt help,
3. It is not safe to move the client with Bucks or if I start seeing double or light flashes.
traction. Support her position changes with
pillows until traction is no longer needed. 5. A client is diagnosed with hypertension and
4. Decrease the weight of traction over a 2-hour prescribed hydrochlorothiazide (HCTZ). What
period; then discontinue the traction and teaching instruction by the nurse should be
move the client into the new bed. included?
1. Take this medication in the evening to
2. When assigning the proper precautions for a prevent falls due to hypotension.
client with HIV, which of the following 2. Make sure to eat a banana or salad
transmission-based precautions would be the everyday.
most appropriate?
3. Notify your health care provider if your
1. Contact urine output increases.
2. Airborne 4. Be aware that your heart rate may be
3. Universal slower.
4. Reverse
6. A woman who has cystitis is receiving Pyridium
3. A two-year-old begins to scream, kick, and wave 200 mg PO TID. Which assessment best
his arms angrily when the nurse lowers his side indicates to the nurse that the medication is
rails to take his temperature and other vital effective?
signs. The child and nurse are alone in the 1. The clients urine is reddish-orange in color.
room. What is the best action for the nurse to 2. There is a decrease in pain and burning on
take? urination.
1. Leave the child alone until his mother comes 3. There is a decrease in the clients
to visit and can be there to help hold him on temperature.
her lap for the procedures.
4. The clients white blood cell count has
2. Immediately call another nurse to come and returned to normal.
help hold the child still for the procedures.
3. Hold the child and talk calmly while 7. An adult client is now ready for discharge
showing him something of interest and following a bilateral adrenalectomy for treatment
explain what is going to be done. of Cushings syndrome. Which statement the
4. Tell the child he will be left alone for client makes indicates to the nurse that further
2 minutes without his toys and he must discharge teaching is needed?
quiet down during that time. 1. I will begin to look more normal soon.
2. I should not lift heavy objects for 6 weeks.
4. The nurse is providing discharge instructions to
3. I will gradually discontinue the hormone
an adult client who has had a cataract extraction
pills in a few months when I feel better.
with a lens implant performed on an outpatient
basis. Which statement by the client indicates a 4. I will not go grocery shopping or run the
need for further instruction? vacuum cleaner until the doctor says I can.

700 NCLEX-RN Review


53155_09_PT 01_p699-722.qxd 2/27/09 6:30 PM Page 701

8. An adult woman is recovering from a mastectomy 12. The nurse is assessing a woman admitted for a
for breast cancer and is frequently tearful when possible ectopic pregnancy. The nurse should
left alone. The nurses approach should be based ask the client about the presence of which of the
on which of these understandings? following?
1. Clients need a supportive person to help 1. Profuse, bright-red vaginal bleeding.
them grieve for the loss of a body part. 2. Right or left colicky abdominal pain.
2. The clients family should take the leadership 3. Nausea and vomiting.
in providing the support she needs. 4. Dyspareunia.
3. The nurse should explain to the client that
breast tissue is not needed by the body. 13. A 19-year-old woman is admitted with a
4. The client should focus on the cure of her diagnosis of anorexia nervosa. Which of the
cancer rather than the loss of the breast. following should the nurse include in the care
plan?
9. An adult has been hospitalized for 1 week for 1. Allow her as much time as she needs for each
severe depression and suicidal thinking. Last meal.
night, he was tearful with his wife present, but this 2. Explain the importance of an adequate
morning he is relaxed and says, Now I have it all diet.
figured out. I know exactly what Im going to do.
3. Observe her during and one hour after each
What does the nurse deduct from this statement?
meal.
1. A sudden lifting of depression may indicate
4. Use a random pattern for surprise weights.
that the client has formed a suicide plan.
2. Support from his wife may have convinced 14. A 28-year-old client with schizophrenia is sitting
the man that life is worth living. alone in his room. He alternates quiet, listening
3. Antidepressant drugs may require several behaviors with agitated talking. The nurse enters
weeks before an effect is felt. his room and observes this behavior. What
4. An absence of sadness and the ability to plan should the nurse say first?
may indicate improvement in depression. 1. You need to come out to the day area with
the group now.
10. An adult client has visible jaundice and tests 2. Why are you hearing voices again?
positive for asterixis. Palpation reveals
3. You appear to be listening to something.
hepatomegaly. The clients labs show an increase
in AST, ALT, and LDH. Based on these findings, 4. I know you hear something but there is no
which nursing diagnosis should the nurse plan one here.
to address first?
15. A client has just returned to the surgical unit
1. Activity intolerance related to weakness following a femoral arteriogram. Which
secondary to liver failure. assessment data would require immediate
2. Risk for injury related to reduced intervention by the nurse?
prothrombin synthesis and reduced vitamin 1. The client is keeping the affected extremity
K absorption. straight.
3. Ineffective health maintenance related to 2. The clients right pedal pulse is 31.
insufficient knowledge of etiology of
3. The client is complaining of numbness in the
condition and treatment.
right foot.
4. Fluid volume excess related to retention.
4. The pressure dressing to the right femoral
11. The nurse is evaluating a new mother feeding area is intact.
her newborn. Which observation indicates the
16. A 28-year-old client with schizophrenia has
mother understands proper feeding methods for
been taking a phenothiazine drug,
her newborn?
chlorpromazine (Thorazine) 50 mg PO QID for
1. Holding the bottle so the nipple is always 4 days. Which observation by the nurse
filled with formula. indicates a desired effect of the drug?
2. Allowing her 7-pound baby to sleep after 1. The client reports fewer episodes of
taking 112 ounces from the bottle. hallucinations.
3. Burping the baby every 10 minutes during 2. Sleeping 10 hours at night plus a 2-hour
the feeding. afternoon nap.
4. Warming the formula bottle in the microwave
for 15 seconds and giving it directly to the baby.

COMPREHENSIVE PRACTICE TESTS 701


53155_09_PT 01_p699-722.qxd 2/27/09 6:30 PM Page 702

3. The client reports feelings of stiffness in his 21. The nurse is caring for a client who has been
neck and face. placed in cloth wrist restraints. What should the
4. The client is increasingly responsive to his nurse do to ensure the clients safety?
delusional system. 1. Remove the restraints every 2 hours and
inspect the wrists.
17. The nurse is to give medication to an infant. 2. Wrap each wrist with gauze dressing beneath
What is the best way to assess the identity of the the restraints.
infant?
3. Keep the head of the bed flat at all times.
1. Ask the mother what the childs name is.
4. Tie the restraints using a square knot.
2. Look at the sign above the bed that states the
clients name. 22. An adult client is scheduled for gallbladder
3. Compare the bed number with the bed X-rays in the morning for suspected
number of the MAR. cholelithiasis. What question will be important
4. Compare the ankle band with the name on for the nurse to ask the client in preparation for
the MAR. the X-ray?
1. Have you ever had trouble with uncontrolled
18. An adult client sustained a fractured tibia bleeding?
3 hours ago and had a long cast applied. The 2. Do you have any known allergies?
client is now complaining of increasing pain
3. Have you received teaching on the low-fat
and the nurse suspects compartment syndrome.
diet?
What initial action will the nurse take?
4. Do you understand the procedure for local
1. Prepare for emergency fasciotomy.
anesthesia?
2. Raise the casted leg to the level of the heart
and notify the physician. 23. A client is scheduled for a glycosylated
3. Administer the ordered pain medication. hemoglobin assay (Hgb A1c). What explanation
4. Instruct client to wiggle his foot and toes will the nurse provide to the client regarding the
more frequently. purpose for this test?
1. It is used to diagnose thyroid levels.
19. The nurse is caring for a client who is scheduled 2. It reveals heart inflammation.
for an magnetic resonance imaging (MRI) study.
3. It measures liver enzymes.
Which statement made by the client warrants
further assessment by the nurse? 4. It reflects blood glucose level over a
23 month period.
1. I am allergic to iodine and seafood.
2. I had a total hip replacement 5 years 24. An adult clients telemetry monitor has been
ago. showing normal sinus rhythm with occasional
3. Ive been taking a blood thinner and PVCs. When there is a sudden change on the
bleed easily. monitor screen to a ventricular fibrillation
4. My doctor told me never to take pattern, what should be the most appropriate
laxatives. action by the nurse?
1. Administer a precordial thump.
20. An adult is admitted to the psychiatric unit 2. Obtain the defibrillator.
with a diagnosis of obsessive-compulsive
3. Begin cardiopulmonary resuscitation.
disorder. His hands are red and rough and he
tells the nurse that he washes them many times 4. Check the clients ECG electrodes.
a day. What would be an appropriate short-term
25. An adult client presents with the sudden onset
goal for him?
of the appearance of floating black spots in her
1. He would explain why his hand washing is right eye. The client sees a black shadow in her
inappropriate. peripheral vision. There is no pain but the client
2. He is prevented from accessing the sink in is very frightened. What should the nurse expect
his room. to do in the care of this client?
3. He records the number of times he washes 1. Place patches on both eyes and plan for strict
his hands each day. bed rest.
4. He verbalizes the anxiety underlying each 2. Patch the right eye and let the client resume
episode of handwashing. activity after 24 hours.

702 NCLEX-RN Review


53155_09_PT 01_p699-722.qxd 2/27/09 6:30 PM Page 703

3. Plan for emergency surgery as the client is in 3. If I cant get to Granddaddys house until
danger of losing her eyesight. lunch time, I can give him a little more
4. Administer a cholinergic eye drop insulin in case his sugar went up in the
(Pilocarpine) to decrease intraocular morning.
pressure. 4. Its very important to keep insulin shots on
schedule and for him to eat at regular times.
26. The nurse is caring for a woman in labor. When
she is 8 cm dilated she tells her support person 30. An elderly woman received digoxin 0.25 mg for
she wants to go home for a few hours of sleep. treatment of her congestive heart failure. Which
The womans statement reveals the womans of the following physiological responses
desire for what action? indicates that the digoxin is having the desired
1. Have others tell her what she needs. effect?
2. Have a soothing back rub. 1. Increased heart rate.
3. Be rid of this difficult situation. 2. Decreased cardiac output.
4. Be left alone. 3. Increased urine output.
4. Decreased myocardial contraction force.
27. A 22-year-old woman comes into the obstetrics
clinic requesting oral contraceptives. Which 31. An adult is admitted to the hospital with
item in the nursing history would indicate that anorexia, weight loss, and ascites. Serum SGOT
she is not a good candidate for this method of (AST), SGPT (ALT), LDH, and total bilirubin are
contraception? significantly elevated. Based on the lab results,
1. She has a history of heavy menstrual what would the nurse expect to find while
periods. performing an admission assessment?
2. She has diabetes mellitus. 1. Pallor.
3. The client reports a broken leg when she was 2. Dry mucous membranes.
10 years old. 3. Jaundice.
4. The client had a baby 6 months ago. 4. Peripheral edema.

28. The nurse is caring for a client who has just had 32. The nurse is preparing a client for an IVP
a craniotomy. The client has an intracranial tomorrow. The client tells the nurse that she gets
pressure monitor in place and is becoming more a rash and becomes short of breath after eating
lethargic. The intracranial pressure is high. How lobster. Given this information, what should the
should the nurse position the client? nurse plan for the client?
1. Elevate the head of the bed 90. Position the 1. A dietitian should visit the client while in
client upright with pillow support under the the hospital.
head. 2. The client is not a candidate for IVP.
2. Place the client flat in bed with the legs 3. The client is at risk for an allergic reaction.
elevated 15 on pillows. 4. An antihistamine will be required before the
3. Position the client on the left side with IVP.
pillow support to the back.
4. Elevate the head of the bed 30. 33. An elderly client requiring abdominal wound
packing TID complains about his wound care to
29. An adult will be administering daily insulin to the nurse making morning rounds. He states that
her 84-year-old blind grandfather. The insulin everyone does it differently and at any time
dose is 15 units NPH, 5 units regular every they feel like it. He is angry at being awakened
morning at 0745. Which statement best indicates at night for this procedure. What is the nurses
that the granddaughter needs further instruction best response?
in insulin administration prior to her 1. The wound care is being done as ordered by
grandfathers discharge from the hospital? your doctor.
1. The regular insulin acts quickly. NPH 2. I understand youre upset at losing sleep.
insulin is milky colored and lasts longer, You can have medication to help you get
usually the whole day. back to sleep.
2. I need to keep track of where I give his 3. Tell me whats really bothering you.
insulin so that I dont use the same site over 4. After rounds Ill be back and we can plan
and over. your wound care.

COMPREHENSIVE PRACTICE TESTS 703


53155_09_PT 01_p699-722.qxd 2/27/09 6:30 PM Page 704

34. The nurse is planning care for a client with 3. A double room with another toddler who also
cervical radiation implants. Which nursing has vomiting and diarrhea.
intervention will be included in the plan of care? 4. A bed in the pediatric intensive care unit, in
1. Implement strict isolation protocol. case dehydration develops.
2. Provide a lead apron for the client.
39. The nurse is caring for a client who is to have a
3. Use only disposable supplies and equipment
lumbar puncture (LP). How should the client be
in the clients room.
positioned during the procedure?
4. Limit visitors to 30 minutes per day.
1. Prone with head turned to the left.
35. The nurse reviews a clients laboratory data and 2. Side-lying in a fetal position.
notes the following hematology values: 3. Sitting at the edge of the bed.
hematocrit (hct) 43%; hemoglobin (Hgb) 4. Trendelenburg position.
15 g/dL; RBCs 5 million; WBCs 7500; platelet
count 30,000. What nursing care is indicated in 40. The physician has ordered a Schilling test for a
relation to these lab values? client with possible pernicious anemia. What
1. Plan a diet high in iron. implementation will be required by the nurse?
2. Plan for frequent rest periods throughout the 1. Administer a mild laxative.
day. 2. Initiate a 24-hour urine collection.
3. Avoid invasive procedures and injections. 3. Administer an intramuscular dose of iron.
4. Implement protective isolation precautions. 4. Insert an intravenous catheter.

36. The nurse is planning care for a client who is 41. The nurse has given discharge instructions on
having a gastroscopy performed. What will be how to care for a newly applied cast to an adult
included in the plan of care for the immediate client. Which statement indicates the client
postgastroscopy period? understands the instructions?
1. Maintain nasogastric tube to intermittent 1. I should pack the casted leg in ice for
suction. 24 hours to help it dry.
2. Assess gag reflex prior to administration of 2. I can use my hair dryer to help the cast dry
fluids. faster.
3. Assess frequently for pain and medicate 3. A good way to relieve the itching under the
according to orders. cast is to gently scratch under the cast with a
4. Measure abdominal girth every 4 hours. soft knitting needle.
4. Putting the casted leg up on fabric-covered
37. An elderly client has suffered a cerebrovascular pillows is the best way to dry the cast.
accident (CVA) and as a result has left
homonymous hemianopia. Based on this fact, 42. The nurse is caring for a client who has just had
what measure will the nurse include in this a bone marrow biopsy. What is essential for the
clients plan of care? nurse to do at this time?
1. Supporting the clients left arm and hand 1. Apply firm pressure over the puncture site.
with pillows. 2. Maintain the client on bed rest for 24 hours.
2. Applying a patch to the clients left eye. 3. Apply an occlusive dressing to the puncture
3. Encouraging the client to use his right hand site.
for activities of daily living. 4. Refrigerate the biopsy specimen.
4. Placing the clients meal on the right side of
the overbed table. 43. An adult client is one day post subtotal
thyroidectomy. What intervention is most
38. A toddler is admitted with a history of vomiting important for the nurse to include in the care plan?
and diarrhea for 2 days, accompanied by 1. Carry out range-of-motion exercises to the
abdominal pain. The admitting diagnosis is neck and shoulders every shift.
gastroenteritis. What type of room assignment 2. Maintain bed rest with client in supine
should the nurse make? position at all times.
1. A room near the nurses station so that he can 3. Ask client questions every hour or two to
be checked frequently and heard if he vomits. assess for hoarseness.
2. A single room with a sink near the doorway 4. Provide tracheostomy care every shift and
for isolation use. suction prn to maintain a patent airway.

704 NCLEX-RN Review


53155_09_PT 01_p699-722.qxd 2/27/09 6:30 PM Page 705

44. An adult client is 4 hours post-op abdominal 1. The amount of alcohol and other drugs
hysterectomy. She has an IV at 125 mL per hour, usually taken and the type and amount taken
an indwelling catheter that has drained 100 mL in the last few days.
since surgery, and her pain is 3 out of 10. 2. The events prompting the client to seek
Which would be the priority nursing diagnosis? treatment.
1. Alteration in comfort, pain. 3. The factors that trigger the clients drinking
2. Alterations in patterns of elimination. episodes.
3. Disturbance in self-concept, body image. 4. Any work, legal, or family problems that
4. Fluid volume deficit, actual or risk for. relate to his use of alcohol.

45. An adult client has meperidine HCl (Demerol) 49. A woman who is 9 months pregnant is attending
50 mg100 mg IM every 34 hours ordered. He a luncheon and fashion show. Suddenly, her
received Demerol 50 mg IM 3 hours ago but hes membranes rupture and contractions come so
still complaining of pain at 8 out of 10. The rapidly that she yells, The baby is coming.
client is asking for pain medication even before What is the most appropriate action for the
it is due and refuses to get out of bed because of nurse to take?
the pain. He was heard telling jokes to the 1. Ask for boiled water, towels, string, and
cleaning personnel. What is the best action for scissors.
the nurse to take? 2. Ask someone to call her doctor.
1. Give the client 50 mg of Demerol IM now. 3. Take her via cab to the nearest hospital.
2. Wait 1 hour and give the client 75 mg of 4. Have her lie on her left side in a less-crowded
Demerol IM. area and be prepared to help with the
3. Give the client 100 mg of Demerol IM now delivery.
and repeat 100 mg Demerol IM in 3 hours if
the pain is still greater than 5 out of 10. 50. While attending a basketball game, a woman
4. Do not medicate the client now. Laughing who is 9 months pregnant suddenly goes into
and joking behavior indicate the pain is not labor and delivers her baby within 5 minutes.
as severe as the client claims. What is the most appropriate course of action for
the nurse to take?
46. An elderly male with undiagnosed respiratory 1. Tie the cord with a shoelace and cut the cord
symptoms is to receive a diagnostic test for with a penknife.
histoplasmosis. How will the nurse administer 2. Have the mothers friend hold the baby until
the histoplasmin skin test? an ambulance arrives.
1. Apply a patch to the skin on the forearm. 3. Place the naked baby on the mothers bare
2. Make a shallow scratch on the skin surface. chest, cover both, and encourage
3. Use a 25-gauge needle placed parallel to the breastfeeding.
skin. 4. Ask people to clear the area so more air
4. Use a 19-gauge needle and Z track injection. can circulate around the mother and
baby.
47. A 35-year-old woman is admitted for treatment
of depression. Which of these symptoms would 51. A young man with newly diagnosed acquired
the nurse be least likely to find in the initial immune deficiency syndrome (AIDS) is being
assessment? discharged from the hospital. The nurse knows
1. Inability to make decisions. that teaching regarding prevention of AIDS
transmission has been effective when the client
2. Feelings of hopelessness.
expresses what thought?
3. Family history of depression.
1. He verbalizes the role of sexual activity in
4. Increased interest in sex. spread of the disorder.
48. An adult male, who appears about 40 years old, 2. He states he will make arrangements to drop
is admitted to the psychiatric unit for alcohol his college classes.
detoxification. He is tremulous and irritable, and 3. He acknowledges the need to avoid all
complains of nervousness and nausea. Which contact sports.
information is most important for the admitting 4. He says he will avoid close contact with his
nurse to obtain? 3-year-old niece.

COMPREHENSIVE PRACTICE TESTS 705


53155_09_PT 01_p699-722.qxd 2/27/09 6:30 PM Page 706

52. A client in the intensive care unit is on a 1. Keep the salicylate medication at the bedside
volume-cycled mechanical ventilator. The high- and take before getting out of bed.
pressure alarm (PAP) begins to sound repeatedly. 2. Take a hot tub bath or shower upon rising.
The client is sleeping quietly. What is the most 3. Ask the physician to order splints to be worn
appropriate initial response by the nurse? at night to maintain anatomical position.
1. Call the respiratory therapist to check the 4. Increase activity to work out the stiffness.
ventilator.
2. Turn the client to stimulate coughing. 58. An adult client has a central line placed for IV
3. Obtain arterial blood for blood gas analysis. fluids. When the nurse enters the room the IV
4. Check the ventilator tubing. bottle is empty, the IV line is full of air, and the
client is dyspneic. What is the best initial
53. A woman is 4 cm dilated and wants to walk nursing action?
about the labor and delivery nursing unit. Which 1. Notify the physician and administer oxygen
of the following criteria will help the nurse via nasal cannula immediately.
determine whether she should walk? 2. Hang another IV bag as soon as possible, then
1. Whether her membranes are intact. remove the air from the IV line.
2. Her contraction frequency. 3. Clamp the tubing and place the client on the
3. The fetal position. left side with head down.
4. The fetal station. 4. Begin CPR and call the code team.

54. Which statement by the client to her partner 59. The nurse is caring for a client who has just
demonstrates understanding of the diaphragm as returned to the nursing unit following a left
a contraceptive device? above-the-knee amputation. How should the
1. It is good for 5 years. client be positioned?
2. It has to be used with a condom. 1. Place the stump on a pillow to decrease edema.
3. It must be left in place for at least 6 hours 2. Place the stump flat on the bed to prevent
after intercourse. contractures.
4. It has to be removed between each sexual 3. Place the client in a prone position to prevent
intercourse encounter. contractures.
4. Place the client in reverse Trendelenburg
55. The nurse is caring for a woman four hours position to promote arterial flow.
following a cesarean birth. Because there are
surgical effects that hinder the womans 60. The nurse is planning care for a child with
resumption of eating, the nurse should include diabetes. Which concept is essential to include
which of the following in the plan of care? when developing the care plan?
1. Ambulation at this time. 1. Most of the family and child education about
2. Applying an abdominal binder. diabetes and its management takes place in
the first 3 or 4 days after the initial diagnosis
3. Administering a Dulcolax suppository.
is made.
4. Listening for bowel sounds.
2. The morning short-acting insulin dosage is
56. An adult client is admitted to the nursing care usually determined by the previous days late
unit with intestinal obstruction and has a Miller- morning and noon blood glucose levels.
Abbott tube in place. How should the nurse 3. The majority of the total daily dose of insulin
assess for proper placement and function of the is given in the evenings to cover the days
tube? intake of food.
1. Inject air and auscultate over the stomach. 4. Snacks for children with diabetes should be
2. Aspirate the tube for stomach contents. given during an exercise episode, rather than
before it.
3. Check the distance markings on the tube.
4. Assess for signs of respiratory compromise. 61. An adult has had diabetes mellitus for many
years. When the nurse enters the room to
57. An adult client who has rheumatoid arthritis administer the morning dose of regular and NPH
reports that the pain and stiffness are greatest insulin, the client complains of dizziness,
upon arising early in the morning. What advice diaphoresis, and nausea. The nurse does a blood
should the nurse give to help the client decrease glucose, which is 30. What is the next nursing
the pain? action?

706 NCLEX-RN Review


53155_09_PT 01_p699-722.qxd 2/27/09 6:30 PM Page 707

1. Give the usual dose of regular insulin and get 3. A 35-year-old with an ulnar fracture.
the clients breakfast tray. 4. A 75-year-old with rib fractures.
2. Hold the NPH insulin but give the regular
insulin. 66. A man with a 10-year history of asthma presents
3. Hold the regular and NPH insulin and call with respiratory distress with labored breathing,
the physician. use of accessory muscles, and audible inspiratory
and expiratory wheezes. Which of the following
4. Give the client a glass of orange juice, hold
would indicate his condition is worsening?
all insulin, and call the physician.
1. Audible expiratory wheezes with lessening
62. An adult had a thyroidectomy this morning. The inspiratory wheezes.
nurse assesses a positive Chovsteks sign and a 2. Increasing expectoration of thick, tenacious
positive Trousseaus sign. The nurse understands sputum with decreasing wheezing lung
that the most common cause of these symptoms sounds.
is which of the following? 3. Absence of audible inspiratory and
1. Inadvertent removal of the parathyroid expiratory wheezes with increasing
glands during the thyroidectomy surgery. somnolence.
2. Overuse of radioactive iodine given 4. Decreasing respiratory rate with decreased
preoperatively to clients undergoing use of accessory muscles.
thyroidectomy.
3. A history of insufficient intake of iodine. 67. The nurse has instructed an adult in crutch-
walking technique. Which statement best
4. Overstimulation of parathormone during the
indicates that the client understands the proper
thyroid surgery.
way to bear weight on crutches while ambulating?
63. The nurse is to begin bladder training with a 1. I should bear my weight on my hands while
young woman who has a T-2 spinal cord injury. walking.
What should the nurse plan to do? 2. Its OK to lean on my crutches, bearing the
1. Teach her to change the indwelling catheter weight under my arms, as long as I dont
drainage bag to a leg bag at night. walk like that.
2. Plan a consistent intermittent catheterization 3. I should bear weight on my underarms
schedule with her and teach her self- while I walk.
catheterization technique as she is able. 4. I should avoid bearing weight on the crutch
3. Plan to place her on the bedside commode to that is on my injured side as much as possible.
void every 2 hours until consistent urination
is achieved. 68. An adult has undergone a total hip replacement
and is now ready for discharge. Which of his
4. Clamp the indwelling catheter for longer
statements indicates good understanding of what
periods of time each day until a bladder
activities are allowed?
capacity of 1500 mL is achieved.
1. I cant wait to see my daughter. She lives 8
64. An adult client has a comminuted fracture of the hours away and until now my hip hurt too
ulnar bone. He asks the nurse what type of much to travel such a long distance.
fracture this is. The nurses response is based on 2. I will really have to be careful not to cross my
which of these understandings? legs. Thats the way I used to sit all the time.
1. The ulnar bone has been crushed and broken 3. It will be great to be able to put on my socks
in several places. and shoes by myself.
2. The two ends of the fractured ulnar bone are 4. As soon as I get home, I wont have to use
pulled apart and separated from each other. this walker.
3. The ulnar bone has been broken in two and
one end of the bone broke through the skin. 69. The nurse is giving discharge instructions to an
adult client who is to be discharged taking
4. Only one side of the ulnar bone is broken.
hydantoin (Dilantin). Which of the following is
65. The nurse is caring for several clients with correct and must be included in the discharge
fractures. Which client is most at risk for fat teaching?
embolus? 1. If there are problems with taking Dilantin
1. A 4-year-old with a wrist fracture. orally, the drug is easily given
intramuscularly.
2. A 20-year-old with a femur fracture.

COMPREHENSIVE PRACTICE TESTS 707


53155_09_PT 01_p699-722.qxd 2/27/09 6:30 PM Page 708

2. Alcohol interferes with the absorption of 3. Exercise.


Dilantin. Do not drink alcohol while taking 4. Metabolism.
Dilantin.
3. Dilantin builds up in the body and achieves 74. Where would the nurse place the stethoscope to
blood levels that prevent seizures. Skipping a assess the clients apical pulse before
day or two will not affect the Dilantin blood administering a dose of digitalis?
levels. 1. Left fifth intercostal space, midclavicular
4. Slurred speech and confusion are side effects of line.
Dilantin and are normal while taking Dilantin. 2. Right second intercostal space, midclavicular
line.
70. An adult client underwent an exploratory 3. Site of carotid pulsation.
laparotomy 24 hours ago. The nurse assesses her
4. Left third intercostal space, sternal border.
abdominal incision and observes a bulging area
at the lower part of the midline abdominal 75. The nurse is assessing a person with long-
incision. This finding most likely indicates standing chronic obstructive pulmonary
which of the following? disease (COPD). Which findings would the
1. Normal postoperative incisional edema. nurse expect to find?
2. A hematoma beneath the skin. 1. Low-grade fever.
3. Abdominal distention. 2. Weak, thready pulse.
4. Impending wound infection with purulent 3. Increased chest diameter.
drainage. 4. Crepitus.
71. An adult client is to receive a unit of whole 76. The nurse is assessing a female client who says
blood. The clients vital signs before starting the she has gained a lot of weight recently, feels cold
transfusion are BP 120/70, P 80, and T 98.4F. all the time and is always tired. In addition she
Five minutes after the transfusion was started reports her hair is falling out. Given the signs
the vital signs are: BP 100/70, P 100, and and symptoms, which of the following disorders
T 99.4F. What should the nurse do initially? is the woman most likely to have?
1. Slow down the rate of the transfusion, 1. Addisons disease.
reassess the client in 15 minutes.
2. Cushings syndrome.
2. Stop the transfusion, keep vein open with
3. Myxedema.
normal saline.
4. Graves disease.
3. Slow down the infusion, notify the physician
immediately. 77. The nurse is caring for a client who underwent a
4. Administer acetaminophen (Tylenol), total hip replacement yesterday. Which of the
continue to monitor closely throughout the following assessments is inappropriate at this
transfusion. time?
1. Presence of Homans sign.
72. A woman is in the outpatient clinic to have a
pelvic sonogram for a suspected ovarian mass. 2. Ability to flex and adduct affected hip.
What will the nurse ask the client to do in 3. Temperature.
preparation for the sonogram? 4. Complaints of pain.
1. Sign a permit.
78. A client is admitted with the diagnosis of gout.
2. Drink several glasses of water.
What is the client most likely to relate when the
3. Take a mild sedative. history is taken?
4. Completely empty her bladder. 1. A slow gradual onset of pain, redness,
swelling, and warmth of the affected joint.
73. The community health nurse is making an initial
home visit for an elderly client to assess the 2. Waking up at night with severe pain in the
clients ability to provide self-care in the home. foot.
Which one of the following areas of concern 3. Pain and muscle spasms that occurred in the
should be assessed for determining the clients morning hours.
ability to remain at home? 4. A sudden onset of discomfort, redness, and
1. Elimination. swelling of the affected joint, occurring
2. Cognitive abilities. mostly during waking hours.

708 NCLEX-RN Review


53155_09_PT 01_p699-722.qxd 2/27/09 6:30 PM Page 709

79. The nurse is caring for a person during a seizure. 3. That picture on the wall looks like my
What is the priority assessment at this time? mother.
1. Presence of an aura. 4. I think my mother plans to get rid of me.
2. Length of the seizure.
84. A woman who is 32 years old and 35 weeks
3. What precipitated the seizure.
pregnant has had rupture of membranes for
4. Type and progression of seizure activity. 8 hours and is 4 cm dilated. Because she is a
candidate for infection, the nurse should include
80. The school nurse initiates a screening program
which of the following in the care plan?
for pediculosis capitis. What else might the
nurse also find when searching for nits clinging 1. Universal precautions.
to the hair shafts? 2. Oxytocin administration.
1. Bites, pustules, and excoriated areas on the 3. Frequent temperature monitoring.
scalp from scratching. 4. More frequent vaginal examinations.
2. Pruritic, scaling, erythematous papules,
plaques, and patches with well-defined 85. A man is hospitalized with probable bacterial
borders. pneumonia. The physician has ordered a
sputum specimen for culture and sensitivity.
3. Beefy-red erythematous areas with a few
What should the nurse do to obtain a good
surrounding papules and pustules.
specimen?
4. An inflammation of the hair follicles with
1. Teach the client deep breathing and coughing
pus-filled nodules.
techniques.
81. An 8-year-old girl suffered a partial thickness 2. Use nasotracheal suction.
scald burn over most of her anterior thigh and 3. Obtain the specimen after starting antibiotics.
lower leg. What admission assessment would 4. Keep client NPO until sputum specimen
give the nurse the most data about the obtained.
probability of shock occurring?
1. Edema, weeping blisters, high serum 86. The nurse is caring for a mother and her
potassium, low serum sodium. newborn son. Which statement the mother
2. Tachycardia, hyperventilation, and a pale makes indicates understanding of newborn care?
appearance. 1. The face and neck are washed first, then the
3. Variations in hyperthermia and hypothermia, eyes, going from the outer corners inward.
and decreased gastric motility. 2. As soon as the cord looks dried, my baby
4. Anemia from red blood cell loss through can sit in a tub bath instead of being
damaged capillaries. sponged.
3. After applying alcohol to the cord once a
82. The nurse is caring for a client on a hypothermia day with the bath, the diaper is applied over
blanket. The nurse turns the client every 2 hours the umbilicus to keep it dry.
for which of the following reasons? 4. The yellow-white covering over the end of
1. The client will accept the treatment more the penis is part of the healing process and
readily if allowed to change positions. should not be removed, but washed gently
2. Turning frequently helps to prevent with water.
shivering.
87. The pregnant client with diabetes on insulin
3. Frequent turning helps the clients
needs to be evaluated for correct medication
autoregulatory mechanism to reestablish
dosage. What is the most effective method to
itself.
assist the nurse in determining the clients need
4. Hypothermia causes vasoconstriction, which for insulin management?
may result in skin damage.
1. Home serum glucose testing.
83. The nurse is assessing a 26-year-old man whose 2. Weight gain.
diagnosis is schizophrenia. Which statement the 3. Daily dietary diary.
client makes indicates he is experiencing 4. Home urine glucose monitoring.
hallucinations?
1. I dont get along very well with my mother. 88. An adult is admitted for further evaluation of a
2. I hear my mother talking to me when Im very high white blood cell count, which may
alone. indicate leukemia. A bone marrow aspiration

COMPREHENSIVE PRACTICE TESTS 709


53155_09_PT 01_p699-722.qxd 2/27/09 6:30 PM Page 710

and biopsy are scheduled. What is the purpose socialization best by implementing which of the
of this test? following interventions?
1. Determine whether Reed-Sternberg cells are 1. Grouping clients together by age and gender
present in the marrow. to encourage the development of friendships
2. Identify the number and type of white blood based on a common characteristic.
cells in all stages of development. 2. Assign a different nurse to group activities
3. Determine whether Epstein-Barr virus is each day to familiarize client with staff.
present in the marrow. 3. Avoid discussion of clients life outside the
4. Identify metastatic changes in the bone day care setting to encourage participation in
structure that are characteristic of leukemia. current activities.
4. Get to know the clients and accompany them
89. A 4-year-old boy with acute epiglottitis is to group events such as singing, crafts,
admitted to the emergency room. He has a fever communal meals, etc.
of 102F, is agitated, drools, and insists upon
sitting up and leaning forward with the chin 93. An elderly client who has diabetes mellitus and
thrusting outward. The nurse expects which of severe cataracts has been given instructions for
the following? administering insulin. Which of the following
1. Intravenous fluids and an antibiotic will be client behaviors signals to the nurse that he has
started before anything else is done. a need for assistance with administration of his
2. The child will cry and resist lying supine insulin?
when he needs to be examined and X-rayed. 1. He uses a magnifier to read the insulin
3. The child will be intubated in the emergency syringe.
room or operating room and then transferred 2. He states he will pinch the skin at the site
to the pediatric intensive care unit. and inject the insulin at a 90 angle.
4. A croup tent with an oxygen source available 3. He mixes NPH and regular insulins by
will be ordered on the regular pediatric unit. drawing up the NPH first.
4. He rotates sites only after using all available
90. A few days ago a child had red, swollen, itchy, areas within each site.
poison ivy lesions that are now becoming fluid-
filled vesicles. Which statement from the child 94. The nurse is teaching a client with an L-3 spinal
demonstrates that she understands how to keep cord injury regarding a bladder training regimen.
from getting poison ivy again? Which of the following instructions should be
1. If Im careful not to touch the leaves of the included in the bladder training process?
plant, I can play with the berries and pretend 1. Drink 12001500 mL of liquid a day.
Im baking. 2. Drink adequate fluids until 10:00 P.M. at
2. My shoes, clothes, and dolls all have to be night.
washed to get the poison ivy off them so I 3. Tighten the abdominal muscles to void.
wont get sores from touching them. 4. Pour cool water on the perineum.
3. Our dog doesnt get poison ivy when she
lies in the plants, so I can hug her all I want. 95. A female was diagnosed with breast cancer
4. When I come inside from playing house, I 12 weeks ago. She was admitted to the hospital
have to scrub myself with soap and hot 4 days ago and is undergoing chemotherapeutic
water. treatment of her cancer. Since her admission,
she has communicated very little with the staff,
91. An elderly client is in for her annual health stays in her room, eats almost none of the food
checkup. Which of the following findings during provided, and is occasionally seen punching her
the physical assessment is of greatest concern to pillow. While caring for her today, the nurse also
the nurse? finds out that she has not been sleeping and
1. Altered pupillary constriction and dilation. feels as though she is somehow being punished
2. Sluggish bowel sounds. for not doing regular breast-self exams. Based
on the noted observations, which of the
3. Kyphosis.
following nursing diagnoses should the nurse
4. Hyperactive deep-tendon reflexes. select as most appropriate for this client?
92. When working with groups of older clients in a 1. Anticipatory grieving.
day care setting, the nurse can promote 2. Fear.

710 NCLEX-RN Review


53155_09_PT 01_p699-722.qxd 2/27/09 6:30 PM Page 711

3. Ineffective individual coping. 98. A 17-year-old female has been admitted with a
4. Anxiety. diagnosis of anorexia nervosa. What is the most
appropriate short-term nursing goal?
96. The nurse is reviewing the breast self-exam with a 1. Client will admit that she has a fear of weight
client who is being discharged after a spontaneous gain.
vaginal delivery. She is breastfeeding. The nurse 2. Client will adhere to a nutritionally balanced
should determine that the client understands diet appropriate for her age.
which of the following?
3. Client will identify her problems and develop
1. Breast self-exam should not be done during new coping methods to deal with them.
lactation.
4. Client will accept herself as having self-worth.
2. Breast should be examined between the 4th
and 7th day after menstrual bleeding begins 99. The client has right hemiplegia as a result of a
or at least once a month. cerebrovascular accident. What finding indicates
3. Breast self-exam should be done with the that the caregivers understand the importance of
woman lying flat on her back. positioning a client with hemiplegia?
4. The breasts must be checked in a circular 1. The right shoulder is adducted and internally
method and assessed for any lumps or rotated.
bumps. 2. The right hip is externally rotated with knee
flexion.
97. An adult has been placed on coumadin therapy
3. The right foot shows plantar flexion.
after prosthetic valve replacement. Which
statement by the client demonstrates correct 4. The right fingers are extended with the
understanding of the teaching about coumadin thumb abducted.
therapy?
100. An adult is to have a pulse oximeter applied to
1. If I miss a dose, I will double the next dose. assess arterial oxygen saturation level. What
2. I should eat plenty of green and leafy action will be included in the correct
vegetables. application?
3. If my arthritis flares up again, Ill take only 1. Placement over the apical area of the chest.
two aspirins every 6 hours. 2. Covering the probe with an opaque material.
4. I will use a soft toothbrush and stop flossing 3. Insertion of an arterial catheter.
my teeth.
4. Insertion of a venous catheter.

Answers and Rationales for Practice Test 1

ANSWER RATIONALE NP CN CL SA
#1. 1. Five to eight lb of traction is applied temporarily to provide Im Ph/7 Ap 1
immobilization prior to surgery. No additional treatment is
required, such as manual traction or pillows. Once the transfer
is complete, the weight should be maintained until no longer
needed.
#2. 3. Universal precautions are utilized as the HIV virus is transmitted Pl Sa/2 Ap 1
by body fluids, not by contact or airborne means. Reverse isolation
would be used if a client was immunocompromised, in which case
the client had AIDS and was more susceptible to infection.
#3. 3. A 2-year-old may respond to distraction to regain some sense of Im He/3 Ap 2
control so he can listen to the explanation of what the nurse wants
him to do. A comforting voice may help calm the child even if he
cannot listen while screaming. Vital signs are essential to evaluate

COMPREHENSIVE PRACTICE TESTS 711


53155_09_PT 01_p699-722.qxd 2/27/09 6:30 PM Page 712

ANSWER RATIONALE NP CN CL SA
ongoing treatment and must be performed whether or not the
parent is present; another nurse in the room may foster more anger
in the child; a time-out should only be used if discipline is required.
#4. 3. Bending over should be avoided as it increases intraocular pressure. Ev Ph/7 An 1
The client should wear the eye shield at night to protect the eye
from accidental injury during sleep; coughing, sneezing would
increase intraocular pressure; and pain, double vision, or light
flashes may indicate glaucoma or retinal detachment, in which
case medical attention should be sought.
#5. 2. HCTZ is a thiazide diuretic which acts on the distal tubules to Im Ph/6 Ap 5
block Na1 reabsorption and increase potassium and water
excretion; therefore, potassium-rich foods should be encouraged.
The client should be aware that the medication should be taken
in the morning to prevent interrupted sleep due to increased
urine excretion. Tachycardia is a possible side effect.
#6. 2. Pyridium acts locally on the urinary tract mucosa to produce an Ev Ph/6 An 5
analgesic effect. A side effect is the urine will turn reddish-orange,
yet does not indicate a therapeutic effect, such as a decrease in
pain. Pyridium does not have microbial properties, so the
temperature or WBC will not be affected by it.
#7. 3. Clients undergoing a bilateral adrenalectomy require lifelong Ev Ph/7 An 1
glucocorticoid and mineralocorticoid replacement, not for just a
few months. The client will gradually lose the Cushings syndrome
features as the hormones are adjusted. After abdominal surgery
clients should abstain from lifting heavy objects or strenuous
activity until given approval by the physician.
#8. 1. The nurse must support the client through the steps and An Ps/4 Co 3
importance of grief by encouraging discussion of the loss, its
meaning to the client, the reactions of others, and the ways of
compensating. Families also may need support first before
providing support to the client; and the loss of breast tissue may
represent a loss of femininity and self-esteem.
#9. 1. Reassessment for suicide risk is essential when depression An Ps/4 An 2
suddenly improves, as the client may appear to feel better
once the decision to commit suicide has been made. Even if
the wifes visit, medications, or ability to plan may have
decreased the depression, it is still vital to reassess.
#10. 4. The clients assessment findings point to cirrhosis of the liver. Pl Sa/1 An 1
Fluid retention is the most immediate concern due to fluid/
electrolyte fluctuations and overload. It would be expected
that the client have fatigue and possible bleeding, so in following
with Maslows Hierarchy of Needs, the physiological problem
would be addressed first.
#11. 1. Holding the bottle so the nipple is always filled with formula Ev He/3 An 3
prevents the baby from sucking air, which can cause gastric
distention and intestinal gas pains. Based on the infants
weight, it should be 50 calories per pound, which would
calculate to 23 oz per feeding. Burping could be performed
halfway through the feeding and at the end; the temperature
should be checked first before feeding.

712 NCLEX-RN Review


53155_09_PT 01_p699-722.qxd 2/27/09 6:30 PM Page 713

ANSWER RATIONALE NP CN CL SA
#12. 2. In ectopic pregnancy, the abdominal pain is usually on one side, As He/3 An 3
is vague, cramping, or colicky from tubal distention, and lasts from
one day to a week or longer. Other reports may be dark red blood as
the uterine deciduas is sloughed off, nausea and vomiting after a
rupture. Dyspareunia is not a complaint.
#13. 3. Left alone at mealtime, clients with anorexia nervousa may hide or Pl Ps/4 Ap 2
discard food, or induce vomiting after a meal. Maladaptive behaviors
may be reinforced if meal times do not have a time limit, or include
discussing food. Weights of clients should always be done at the
same time, wearing a hospital gown and voiding prior to weighing.
#14. 3. This response shares the nurses observation and allows for Im Ps/4 Ap 2
validation by the client. Group participation may not be
appropriate at this time; avoid using why questions which
imply blame; validate whether the client is hallucinating and
its content before confrontation with reality.
#15. 3. Any neurovascular assessment data that are abnormal require As Ph/7 Ap 1
intervention by the nurse; numbness may indicate decreased
blood supply to the right foot. The affected leg should be kept
straight for at least 68 hours (may vary) to prevent any arterial
bleeding from the insertion site at the right femoral artery; +3 is
a normal finding; and a normal finding would be an intact
pressure dressing at the site.
#16. 1. Phenothiazine drugs, like chlorpromazine, are antipsychotic Ev Ph/6 Ap 2
drugs and the desired action is to reduce the symptoms of
psychosis, such as hallucinations. Drowsiness is a common
side effect in early treatment and should diminish over time;
a dystonic reaction with stiffness in the neck and face is also
a side effect and can be treated with an antiparkinson drug;
increased delusions indicate the psychosis may be worsening.
#17. 4. Two parameters are required to assure right client and medication. As Sa/2 Ap 3
If a name band is missing, a new one should be put on as soon as
possible. Mistakes may have occurred if the nurse only identifies
the bed number or room. Always administer the 5 rights of
medication for each client.
#18. 2. To decrease the pressure within the compartment, the affected Pl Ph/8 An 1
extremity is raised to the level of the heart, and if this does not
relieve pressure, a fasciotomy may be necessary. An accurate
assessment should be performed before pain medication is given;
foot exercises will not relieve the pressure from compartment
syndrome.
#19. 2. Implanted medical devices (pacemaker, screws, pins, etc.) may As Ph/7 An 1
render the client unsuitable for the MRI procedures. No contract
media is utilized so allergies are not a concern; the MRI is a
non-invasive procedure so bleeding is not a risk; and a bowel
prep is not required.
#20. 3. The client participation in obtaining baseline data is the first step Pl Ps/4 Ap 2
to decreasing that behavior. Clients with compulsive behavior
cannot stop without increasing anxiety; physical prevention of
the behavior may initiate a panic attack or other extreme behavior;
verbalization is a long-term goal.

ANSWERS AND RATIONALES FOR PRACTICE TEST 1 713


53155_09_PT 01_p699-722.qxd 2/27/09 6:30 PM Page 714

ANSWER RATIONALE NP CN CL SA
#21. 1. Wrists must be inspected for breakdown/trauma. Wrist restraints As Sa/2 Ap 1
are sufficiently padded; position of the head of the bed has no
relation to use of restraints; even though a square knot is used,
it does not ensure client safety.
#22. 2. Iodine contract medium is used for gallbladder X-rays. The client As Ph/7 Ap 1
must be assessed for a history of iodine allergy. This procedure is
non-invasive; diet is recommended but not related to X-ray
preparation; local anesthesia is not used.
#23. 4. The Hgb A1c assay provides information about long-term control An Ph/7 An 1
of DM. The assay reflects glucose level within erythrocytes,
providing an average level over 23 months preceding the test.
It is not related to thyroid, heart or liver findings.
#24. 4. Sudden changes in ECG patterns may be a result of loose An Ph/8 An 1
electrodes (artifact) rather than a lethal dysrhythmia. The
client should be assessed upon any abnormal monitor activity.
A precordial thump, defibrillation and cardiopulmonary
resuscitation all could cause injury and would not be utilized
unless the client has a true lethal dysrhythmia.
#25. 1. The client is displaying signs of a detached retina, which Pl Ph/8 An 1
requires patching of both eyes to minimize eye movement
and bed rest with a flat or slightly raised head of bed to
prevent separation of retina and choroid layers. Emergency
surgery is not the initial plan, but scleral buckling
or laser reattachment are treatment options; Pilocarpine
is used for glaucoma.
#26. 3. The pain may be unbearable and she wishes to get away An He/3 An 3
from it. Her desires include to be in control, yet turn
inward and shut out external stimuli; a back rub would
be more appropriate in early labor.
#27. 2. Diabetes is a contraindication for taking oral contraceptive, as DM As Ph/6 An 3
is linked with cardiovascular disease. The contraceptives would
decrease menstrual flow; the broken leg is too far in the past and
the baby is old enough that contraceptives wont be a concern.
#28. 4. Elevation to 30 to promote optimum venous outflow causing Im Ph/8 Ap 1
reduction in intracranial pressure and prevention of aspiration.
Pillows should be avoided because they may cause head flexing,
which decreases venous outflow. Avoid elevation above 30 or
leg elevation which will increase blood to the brain. Side-lying
does not affect intracranial pressure, but the choice does not
specify elevation of head, as supine would be avoided.
#29. 3. It is important not to change insulin dosage without consulting Ev Ph/6 An 1
the physician. This statement indicates that the client needs
further instruction before her grandfather leaves the hospital.
All other statements are correct statements.
#30. 3. Urine output increases due to the increased cardiac output and Ev Ph/6 An 1
myocardial contraction force, increasing perfusion of the kidney.
The other choices are opposite from the true action of digoxin.

714 NCLEX-RN Review


53155_09_PT 01_p699-722.qxd 2/27/09 6:30 PM Page 715

ANSWER RATIONALE NP CN CL SA
#31. 3. Elevated liver enzymes and total bilirubin, along with the An Ph/7 An 1
symptoms, anorexia, weight loss, and ascites, all suggest
liver disease. Jaundice occurs with liver disease because
of the inability of diseased liver cells to clear bilirubin from
the blood. Bile is deposited in the skin and sclera, producing
the yellow discoloration. Pallor, dry mucous membranes, and
peripheral edema are associated with anemia, dehydration,
and congestive heart failure, respectively.
#32. 3. People who are allergic to shellfish (iodine) are at risk for allergic An Ph/7 An 1
reactions to the contract material (iodine) used for an IVP. A
dietitian is not needed, and the test can be performed using a
less allergic contract material. The physician will be the one to
order an antihistamine, which may be combined with steroids.
#33. 4. The nurse arranges to plan wound care with the client, thereby An Sa/1 Ap 1
allowing him to participate in his own care and addressing the
source of his anger. The other responses discount the clients
feelings, only address part of the problem, and show a
misunderstanding of the clients complaints.
#34. 4. Limited time in the clients room reduces exposure to radiation Pl Sa/2 Ap 1
for nursing staff and visitors. Strict isolation is not needed; time
and distance limits are needed. The lead apron could be worn for
staff or visitors and disposable supplies and equipment are not
necessary. Bed linens are handled according to radiation protocol.
#35. 3. The platelet count is low. Normal platelet count is 150,000450,000. Pl Ph/7 Ap 1
A low platelet count places the client at risk for bleeding. Trauma,
injections, and invasive procedures should be avoided. All other
values are within normal limits.
#36. 2. Because a local anesthetic is used to numb the pharyngeal area Pl Ph/7 Ap 1
for gastroscopy, the nurse must be certain the client is able to
swallow before giving food or fluids. It may take 24 hours for
the gag reflex and swallowing ability to return. An NG tube or
pain will not be present after the procedure, and measuring
abdominal girth is not indicated following procedure.
#37. 4. This disorder involves blindness on the left half of the visual field Pl Ph/7 Ap 1
of both eyes. Therefore, the client can only see objects placed
within the right visual field. The other choices are related to
hemiplegia or double vision.
#38. 2. The child should be placed on enteric isolation until the lab Pl Sa/2 Ap 4
reports no contagious organisms in the stool. If the stool is
infected, isolation is continued after the antibiotics are completed
until 3 consecutive daily stool specimens are negative. Priority
placement of room is dependent on prevention of communicability,
not exposing other children, and dehydration can be managed on
a regular floor.
#39. 2. The fetal position increases space between lumbar vertebrae, Im Ph/7 Co 1
facilitating easier entry of the needle into the subarachnoid space.
Sitting is a possible position but not for obtaining cerebrospinal
fluid and may cause a headache. The prone and Trendelenburg
are acceptable positions for this procedure.

ANSWERS AND RATIONALES FOR PRACTICE TEST 1 715


53155_09_PT 01_p699-722.qxd 2/27/09 6:30 PM Page 716

ANSWER RATIONALE NP CN CL SA
#40. 2. A Shilling test measures the percent of vitamin B12 excreted Im Ph/7 Co 1
in a 24-hour urine sample following an intramuscular loading of
vitamin B12 and a radioactive oral dose of vitamin B12. Laxatives
could interfere with the B12 absorption, iron treats iron-deficiency
anemia, and an intravenous catheter is not required.
#41. 4. Cloth-covered pillows or blankets are breathable materials that Pl Ph/7 Ap 1
allow the cast to air dry. No plastic should be used. Ice should
only be used in 20-minute intervals; the listed heat sources will
cause uneven drying of the cast, and no objects should be inserted
into a cast.
#42. 1. Bleeding may occur from the puncture site. Firm pressure is Im Ph/7 Ap 1
required for several minutes to prevent this. The client can
resume normal activity after the sedation has worn off, no
occlusive dressing is needed, and the specimen is sent
immediately to the lab.
#43. 3. Damage to the recurrent laryngeal nerve is a major complication Pl Ph/7 Ap 1
of thyroid surgery. Hoarseness immediately following surgery is
often related to intubation during surgery. However, report
persistent or worsening hoarseness immediately to the physician
because it may be the first sign of nerve injury. Semi-fowlers
position is preferred with pillow support. A tracheostomy
set should be available, but only used in emergency situations.
#44. 4. All abdominal surgery clients have a potential for third-spacing Pl Ph/8 Ap 3
of fluids, causing a fluid volume deficiency. Post-op urine output
should be maintained to at least 30 mL/hr. 100 mL indicates a
beginning deficit. Remembering Maslows Hierarchy of Needs,
physiological needs are addressed first.
#45. 3. Pain is what the person says it is and occurs when the person Pl Ph/6 Ap 1
says it does. The clients report of 8 out of 10 validates the
nurse administering 100 mg of Demerol. The other responses
do not meet the clients need for pain relief and clients use
various mechanisms to deal with their pain, which may be
laughter, exercise, or being quiet.
#46. 3. Using a 25-gauge needle inserted between the skin layers Im Ph/7 Co 1
angling the needle parallel to the skin of the forearm. A patch,
the scratch method, or the Z-track IM injection are not suitable
methods for this test.
#47. 4. Interest in sex is markedly decreased in depression, not As Ps/4 Co 2
increased. The other symptoms are commonly found in
clients with depression.
#48. 1. Knowledge of the types and amounts of alcohol and other As Ps/4 An 2
drugs consumed are necessary to plan the program of
detoxification and anticipate physical complications. The
other choices may be helpful to determine the reason for the
episodes; however the staff will need to be on the alert for
delirium tremors. As treatment progress, the additional
information will be important in the later stages of the
clients treatment.

716 NCLEX-RN Review


53155_09_PT 01_p699-722.qxd 2/27/09 6:30 PM Page 717

ANSWER RATIONALE NP CN CL SA
#49. 4. Lying on the left side provides the best perfusion to the uterus Im He/3 Ap 3
and the infant while waiting for delivery. The nurse should have
the mother in as clean and uncrowded a place as possible. It would
be advisable to call her doctor after placing her in the left-side
position, to keep her where she is rather than risk having the baby
in transit to the hospital, and the umbilical cord would not be cut if
hospital care is likely within 1 hour.
#50. 3. Skin-to-skin contact is recommended so that the mothers warm Im He/3 Ap 3
body will warm the infant. Covering both will help keep them warm.
Breastfeeding will help contract the mothers uterus and reduce
bleeding. The umbilical cord would not be cut if hospital care is
likely within 1 hour.
#51. 1. The AIDS virus is spread through direct contact with body fluids Ev Sa/2 An 1
such as blood, and through sexual intercourse. All the other activities
are casual contact, which do not spread AIDS. Contact sports may
post a risk if there is potential for direct contact with blood.
#52. 4. Unless the client is coughing, has decrease airway compliance, Ev Ph/7 An 1
or has an airway obstruction, a high pressure alarm usually indicates
water collection in or kinking of ventilator tubing. The RN should
check the tubing first.
#53. 4. If the fetal station is engaged, that is, at 0 station or +1 or more, As He/3 Co 3
cord prolapse will be prevented whether her membranes are ruptured
or not. The most important criteria is the fetal station. Presentation
refers to the anatomical part of the fetus closest to the birth canal.
#54. 3. For effective action, the diaphragm must be left in place for Ev He/3 An 3
68 hours after intercourse. The diaphragm should have regular
inspections, is more effective when used with other contraceptives,
and should be removed at least once in a 24-hour period.
#55. 4. Bowel sounds indicate a beginning return of peristalsis. Ice chips Pl Ph/7 Ap 3
could be offered until either gas or bowel sounds are present.
#56. 3. The Miller-Abbott intestinal tube is weighted with mercury Ev Ph/7 Co 1
to decompress the small intestine. As the tube moves through the
intestine, progress can be assessed by comparing distance marking
on the tube. The Salem-sump or feeding tube placement is assessed
by either injecting air and auscultating or aspiration for stomach
contents.
#57. 2. A hot tub bath/shower helps to shorten the period of stiffness. Im Ph/5 Ap 1
Salicylate should be avoided on an empty stomach, splints would
decrease joint mobility, and activity may need to be decreased
if pain is present.
#58. 3. Air embolism occurs frequently with central lines with sudden Im Ph/8 Ap 1
onset of dyspnea, hypotension, chest pain, and cyanosis. The best
initial nursing action is to clamp the IV line, and turn the client on
to the left side to trap the air on the right side of the heart, so it
does not enter the pulmonary artery. Then call the physician and
administer oxygen. The client does not warrant CPR at this time,
another IV bag of fluids would be hung, but only after the tubing
is unhooked from the client and re-primed.

ANSWERS AND RATIONALES FOR PRACTICE TEST 1 717


53155_09_PT 01_p699-722.qxd 2/27/09 6:30 PM Page 718

ANSWER RATIONALE NP CN CL SA
#59. 1. Elevating the stump will decrease edema. However, Im Ph/7 Co 1
elevation on a pillow is indicated only for the first day,
because prolonged hip flexion can lead to contracture.
Following that, the foot of the bed should be elevated on
shock blocks. Reverse Trendelenburg is contraindicated as
it promotes venous congestion.
#60. 2. The morning short action insulin dosage is usually Pl Ph/6 Ap 4
determined by the previous days late morning and noon
blood glucose levels. To depend on todays BG levels to
determine dose of regular insulin, would put the child into
a situation of constant overtreatment or undertreatment.
Education is an ongoing activity, insulin is given before
intake of food, and snacks would be given before exercise.
#61. 4. The symptoms indicate hypoglycemia. Ten grams of Pl Ph/6 Ap 1
rapidly absorbing carbohydrate is the treatment for
hypoglycemia. This should be repeated in 5 minutes
if the client does not feel better. The physician should
be notified for new orders if glucose and insulin parameters
have not already been determined. Regular insulin should
be held, as it will lower the blood glucose.
#62. 1. The symptoms suggest hypocalcemia. The four pea-sized An Ph/8 Ap 1
parathyroid glands, which regulate calcium and phosphorus
balance, are embedded in the thyroid. Inadvertent removal
during a thyroidectomy is a common cause of post-operative
hypocalcemia. Radioactive iodine is used to shrink the thyroid
and causes hypothyroidism, insufficient intake of iodine may
cause a goiter, and over stimulation of parathormone causes
hyperthyroidism.
#63. 2. Early intermittent catherization is essential in bladder Pl Ph/7 Ap 1
training. A high thoracic spinal cord injury may have
some arm, shoulder, and hand movement that would
enable client to learn self-catherization techniques. An indwelling
catheter should be removed as soon as possible; the client
will be able to void on her own; a clamp would be
contradicted as an areflexic neurogenic bladder would
accommodate the urine.
#64. 1. Comminuted fracture usually results from a crush injury An Ph/8 Co 1
and results in fractured and crushed bones. Bones pulled
apart are displaced, compound fracture is through the skin,
and greenstick involves only one side of the bone.
#65. 2. Fat embolism occurs most often in the client with long An Ph/7 Co 1
bone lower extremity fractures or multiple fractures,
regardless of age.
#66. 3. Absence of audible wheezes can be a sign of improvement. An Ph/8 An 1
However, when coupled with somnolence, a sign of
hypercapnia, absence of wheezing is a sign of worsening
bronchospasm. This is a respiratory emergency period. All
of the other choices demonstrate improvements in respiratory
function.

718 NCLEX-RN Review


53155_09_PT 01_p699-722.qxd 2/27/09 6:30 PM Page 719

ANSWER RATIONALE NP CN CL SA
#67. 1. Client should be taught to support his weight on the crutch Ev Ph/5 Ap 1
handpieces. Weight should be avoided on the axilla and
should be placed on crutch side.
#68. 2. The client should not cross his legs or abduct or assume any Ev Ph/7 An 1
position that requires acute flexion of more than 90 degrees.
Traveling long distances should be avoided as the person
remains in hip flexion for an extended time; and a walker
will be used until sufficient muscle tone has developed.
#69. 2. Alcohol interferes with Dilantin and causes it to remain at Im Ph/6 Co 1
subtherapeutic level causing the client to be prone to seizure
activity. Dilantin is not given IM, it needs to be given on a
consistent basis, and side effects include ataxia, nystagmus,
hypotension, rash, gingival hyperplasia, and ventricular
fibrillation.
#70. 2. A hematoma beneath the skin may cause the skin to bulge, but Ev Ph/7 An 1
this does not mean the client has abdominal distention. This may
require evacuation of the hematoma so that healing may take
place. No evidence is given that an infection is present.
#71. 2. The symptoms suggest acute hemolytic transfusion reaction. The Im Ph/6 An 1
priority nursing action is to stop the infusion immediately, remove
the existing tubing and blood, flush the IV site, and administer
normal saline. The physician should be notified and blood tubing
and remaining blood should be sent back to the lab.
#72. 2. The bladder should be full for a pelvic sonogram to serve as a Ev Ph/7 Ap 3
reference point and sonic window to the pelvic organs. The
procedure is noninvasive (no permit needed) and a sedative is
not required.
#73. 2. Alzheimers disease is the most common cognitive impairment As He/3 Ap 1
affecting older adults. As the disease progresses, it requires ongoing
assessment to determine the clients ability to maintain himself in
the home environment. All other needs will depend on his cognitive
abilities.
#74. 1. This is the appropriate place for the stethoscope, as the others are As He/3 Co 1
used for aortic heart sounds and Erbs point (murmurs).
#75. 3. The anterior-posterior diameter increases over time as As Ph/7 An 1
compensation for chronic hypoxemia, and is known as a barrel
chest. Fever is associated with pneumonia; shock is associated
with a weak pulse; and crepitus is associated with a pneumothorax.
#76. 3. Myxedema is a condition of the thyroid gland in which there is As Ph/7 Co 1
thyroid hypofunction. Addisons is hypofunction of the adrenal
cortex (low blood sugar, hypotension, bronze-colored skin);
Cushings syndrome is the hyperfunction of the adrenal cortex
(high blood sugar, hypertension, buffalo hump, moon face,
hirsutism); Graves disease is hyperfunction of the thyroid
gland (weight loss, tremor, tachycardia, tachypnea, heat
intolerance).

ANSWERS AND RATIONALES FOR PRACTICE TEST 1 719


53155_09_PT 01_p699-722.qxd 2/27/09 6:30 PM Page 720

ANSWER RATIONALE NP CN CL SA
#77. 2. The client should not flex and adduct affected joint for 24 hours As Ph/7 Co 1
after surgery. Usually an abductor pillow maintains the hip in
slight abduction to prevent dislocation. Nursing actions would
include assessing for a deep vein thrombosis by checking for a
positive Homans sign, taking vital signs that could signal
an infection, and administering pain medication as needed.
(Note: Although a positive Homans sign is still used in
clinical practice, it is not used as the single diagnostic
indicator for a deep vein thrombosis. Further tests include
venography and ultrasonography.)
#78. 2. An acute attack of gout may be caused by trauma, alcohol As Ph/7 An 1
ingestion, dieting, surgical stress or medications. It often
occurs at night, which awakens the patient due to pain in
the affected area. Osteoarthritis is associated with pain &
muscles spasms upon rising in the morning hours.
#79. 4. The priority nursing action should be to observe the type As Ph/8 An 1
and progression of the seizure activity. It is also important
to ensure the client is safe and does not injure himself
during the seizure. The client can be asked about the
presence of an aura when he is conscious.
#80. 1. The scalp itches from the crawling and saliva of the adult As Ph/7 Co 4
louse. The childs fingernails scratch the skin, leaving red
marks. The others describe ringworm, candidiasis, and
folliculitis, respectively.
#81. 1. Shifts in fluid and electrolytes are caused by the loss of fluid As Ph/8 An 4
and proteins through damaged tissues and blood vessels. Loss
of protein causes the interstitial area to fill with fluid, decreasing
the intravascular volume. Broken cells release potassium into the
circulation. Sodium is retained when aldosterone is released into
the bloodstream in response to stress. From the bloodstream, it
goes into the interstitial spaces through broken capillaries, thus
lowering serum levels and drawing more fluid from the circulating
volume. Shock is a major complication during the first 12 days.
#2 is related to pain; #3 is incorrect as tissue cannot maintain
temperature balance; and #4 occurs 47 days after injury.
#82. 4. Turning frequently reduces skin damage, as it does not An Ph/7 An 1
prevent shivering or help autoregulatory systems.
#83. 2. A hallucination is a sensory experience with no stimuli, As Ps/4 An 2
as in hearing voices when alone.
#84. 3. Temperature elevation will indicate beginning infection. Pl He/3 Ap 3
This is the most important measure to help assess the client
for infections, because the lost mucous plug and the ruptured
membranes increase the potential for ascending bacteria
from the reproductive tract. This will infect the fetus,
membranes, and uterine cavity.
#85. 1. Deep breathing and coughing are essential for obtaining Im Ph/7 Ap 1
mucus from the bronchi. The optimal collection time is
early in the morning, after the client has brushed his teeth.
The specimen should be obtained before antibiotics are
given. NPO status is not necessary.

720 NCLEX-RN Review


53155_09_PT 01_p699-722.qxd 2/27/09 6:30 PM Page 721

ANSWER RATIONALE NP CN CL SA
#86. 4. This statement is correct, as soap may cause irritation if used. Ev He/3 An 3
Eyes are washed first; the cord must have fallen off and the area
healed completely before immersing in water; avoid placing
diaper in contact with an unhealed umbilicus to prevent
contamination by fecal organisms.
#87. 1. This is the best method as it is the only one that indicates Pl Ph/7 An 3
metabolism of food and the bodys response to intake.
#88. 2. Leukemia is diagnosed by identifying abnormal white An Ph/7 Co 1
blood cells and their precursors in the bone marrow and
noting how many of these cells are present. Reed-Sternberg
cells are found in Hodgkins lymphoma and Epstein-Barr
virus has not been shown to have a direct relation with
leukemia. Metastasis is not found by a bone marrow test.
#89. 3. Epiglottitis is always a medical emergency. Intubation is An Ph/8 An 4
best facilitated in the operating room where all equipment
is readily available, and where the epiglottis can be visualized
with a laryngoscope. Laryngospasm is prevented by starting
an IV after the intubation, keeping the child on the mothers
lap, and antibiotics with the procedure.
#90. 2. Shoes, toys, and clothes can all transfer the oil (urushiol) to An He/3 Ap 4
the skin, where it sets up an immediate reaction. Items that
have touched any part of the plant should be washed in hot
water and detergent. All parts of the plant are poisonous,
including dried or burning leaves. The fur on animals can
contain the plant oils. The first contact (within 15 minutes)
should be with cool water, as soap will wash off the natural
protective oils in skin.
#91. 4. Clonus is a phenomenon where reflexes are very hyperactive An He/3 An 1
and suggests the presence of central nervous system disease.
The client required further evaluation. The other findings
are a normal process of aging.
#92. 4. Socialization is fostered when nurses and other caregivers Pl He/3 Ap 1
take time to talk with the clients and show a genuine concern
for their well-being, for their present and past life stories.
There is no need to separate by age or gender and continuity
of care has proven to enhance client outcomes.
#93. 3. The correct preparation of insulin should be the regular (clear) An Ph/6 An 1
insulin is drawn up first, then the NPH (cloudy). This action is
performed to keep the short-acting regular insulin free from
potential contamination by the intermediate-acting NPH.
Remember clear to cloudy. The client would be correct in
all the other actions.
#94. 3. Clients with injuries to the lumbosacral area usually have a Im Ph/7 Ap 1
lower motor neuron (flaccid) bladder. The emptying of the
bladder may be achieved by performing a Valsalva maneuver
or tightening the abdominal muscles. Fluid intake should be
20002500 mL during the day and decrease fluids after 6 P.M.
Pouring water on the perineum may help clients with upper
motor neuron injuries.

ANSWERS AND RATIONALES FOR PRACTICE TEST 1 721


53155_09_PT 01_p699-722.qxd 2/27/09 6:30 PM Page 722

ANSWER RATIONALE NP CN CL SA
#95. 1. Anticipatory grieving is the state in which an individual An Ps/4 An 3
experiences multiple feelings in response to an expected
significant loss. A clients behaviors may include emotional,
physical, spiritual, social, and intellectual responses, in which
the nurse recognizes the client is experiencing real grief
related to the perception of potential loss.
#96. 2. This is the appropriate time when hormones have the least Ev He/3 Ap 3
effect on the breasts, making them easier to examine. When
not menstruating, breasts should be examined once a month,
even when lactating. Proper positioning during a self-breast
exam should be both lying and standing, using one of the
3 methods: circular, vertical strip, or wedge.
#97. 4. Clients should be cautious about any injuries while on Ev Ph/6 Ap 1
anticoagulants, which may precipitate bleeding. Anticoagulants
should not be increased without physician and/or lab indication.
The foods mentioned contain vitamin K, which is the antidote for
coumadin and should not be eaten in large amounts. Salicylate
drugs, such as aspirin, produce inhibition of platelet aggregation
and could cause further bleeding.
#98. 1. To aid in promoting positive expectations regarding body image, Pl Ps/4 Ap 2
fears must be discussed openly before management can be
approached. The other choices involve long-term goals in all
or part of the selection.
#99. 4. Spasms in the flexors and adducts in a hemiplegic can result Ev Ph/7 Ap 1
in flexion and adduction contractures. Proper positioning of
joints in an extended, abducted position prevents contractures.
The hand tends to form a fist around the thumb unless properly
positioned in a semi-extended position. A pillow under the axilla,
a trochanter roll by the hip, and a posterior split are additional
positional supports.
#100. 2. The pulse oximeter is a painless, noninvasive procedure, Im Ph/7 Ap 1
in which the probe is attached over a pulsating vascular bed,
such as the finger or earlobe. Covering the probe with an
opaque material helps to prevent inaccurate readings from
bright external lighting.

722 NCLEX-RN Review


53155_09_PT 02_p723-743.qxd 2/26/09 8:09 AM Page 723

Practice Test 2

1. The nurse is administering medication in an 3. I cant concentrate at work but I do fine at


extended care facility. The client answers to home.
Mr. Smith and Mr. Brown. What is the best way 4. My wife worries that I am drinking more
for the nurse to correctly identify the client now.
before administering the medications?
1. Check with the picture identification on file. 6. An adult client comes to the nurses station
2. Check the arm band. complaining of shortness of breath, choking,
dizziness, and nausea. He says, I think Im
3. Check the name on the bed.
going crazy or dying or something. I dont know
4. Check the name on the room door. what happened. Help me, help me. When the
nurse tries to ask about what happened, the
2. An adult is scheduled to undergo an exploratory
client can only say, Help me, help me. What
laparotomy in 1 hour. The nurse has just
level of anxiety is the client displaying?
received the order to administer his preoperative
medication. What assessment is essential for the 1. Mild.
nurse before administering the medication? 2. Moderate.
1. The clients ability to cough and deep 3. Severe.
breathe. 4. Panic.
2. Any drug hypersensitivity or allergy.
7. A woman completes a session with her
3. The clients understanding of the surgical
psychotherapist and becomes increasingly
procedure.
anxious. The nurse talks with her and the
4. Whether the clients family is present and physician orders Xanax (alprazolam) 0.25 mg
supportive. PO. Which action by the nurse is appropriate for
evaluating the effectiveness of this drug?
3. The nurse is assessing a male client who has
been admitted for treatment of alcoholism. 1. Ask her whether she remembers not to drive
Which question by the nurse is least or use alcohol.
appropriate? 2. Assess her anxiety level 1 hour after giving
1. How much do you drink? the drug.
2. What other drugs do you use? 3. Encourage her to use a relaxation tape along
with the medication.
3. How is your general health?
4. Ask her if the medication caused her to be
4. Why do you drink so much?
nauseated.
4. The nurse is planning care for a client who has a
8. The nurse is assessing a client who was raped a
phobic disorder manifested by a fear of
month ago. The client cannot describe any feelings
elevators. Which goal would need to be
related to her rape. She focuses on the need for
accomplished first?
better law enforcement and rape prevention
1. Demonstrates the relaxation response when programs. The client is using which defense?
asked.
1. Denial.
2. Verbalizes the underlying cause of the
2. Conversion.
disorder.
3. Introjection.
3. Rides the elevator in the company of the nurse.
4. Isolation.
4. Role plays the use of an elevator.
9. A client confides to the nurse that he is being
5. Which statement would be least expected when
pursued by the Federal Bureau of Investigation
the nurse assesses a client with post-traumatic
because he has access to information that will
stress disorder (PTSD)?
prevent future wars. This behavior most likely
1. Sometimes my heart pounds and I cant get represents which of the following?
my breath.
1. Ideas of reference.
2. I have nightmares about my time in the war.
2. Delusions.

COMPREHENSIVE PRACTICE TESTS 723


53155_09_PT 02_p723-743.qxd 2/26/09 8:09 AM Page 724

3. Hallucinations. 3. Ask the client what precipitated his


4. Dissociation. overdose.
4. Find out how many pills were in the bottle.
10. A middle-age woman is admitted in the manic
phase of bipolar disorder. She has eaten very 14. During her first trimester, a woman experiences
little food from her meal trays and has lost many physiological changes that lead her to
2 pounds in the 3 days since admission. Which think she is pregnant. Which of the following
approach by the nurse is most likely to meet her changes will the nurse most likely tell her are
nutritional needs? normal for an 8-week pregnancy?
1. Provide frequent snack foods that are high in 1. Dysuria.
nutrition. 2. Colostrum secretion.
2. Limit her intake of between-meal 3. Nosebleeds.
snacks.
4. Dependent edema.
3. Allow her to eat in the cafeteria and choose
her own foods. 15. The nurse is assessing a healthy neonate upon
4. Set up a system in which greater intake is admission to the newborn nursery. Which
rewarded by privileges. characteristic would the admitting nurse record
as normal?
11. An adult male was admitted to the psychiatric 1. Hypertonia.
unit 3 days ago with a diagnosis of
2. Irregular respiratory rate of 50 breaths/minute.
schizophrenia. Today the nurse finds him facing
the window, and he seems to be talking and 3. Head circumference measuring 31cm.
listening. After validating that the client is 4. High-pitched or shrill cry.
hallucinating, which is the best action by the
nurse? 16. Following her babys birth, the womans uterine
fundus is soft, midline, 2 cm above the
1. Allow him to continue his conversation
umbilicus, and she has saturated two pads
without interruption.
within 30 minutes. Which immediate need by
2. Offer him a choice of IM or PO for his PRN the client should be addressed?
medication.
1. Be cleaned and have another pad change.
3. Tell him that his behavior is inappropriate
2. Empty her bladder.
and should stop.
3. Have an increase in her intravenous fluids of
4. Involve him in some concrete game or other
Ringers lactate.
activity.
4. Have her fundus massaged.
12. The nurse is evaluating a woman who has been
treated for major depression for 3 weeks. Which 17. The nurse is caring for a child with hemophilia
is the best indication that her depression is who is actively bleeding. Which nursing action
improving? is most important in the prevention of the
crippling effects of bleeding?
1. She takes her medication as prescribed.
1. Active range of motion.
2. She says, I think I can set more realistic
goals now. 2. Avoidance of all dental care.
3. Her husband demonstrates more 3. Encourage genetic counseling.
understanding of his wifes condition. 4. Elevate and immobilize the affected
4. She ventilates about how sad and hopeless extremity.
she feels. 18. The nurse is planning care for an infant with
13. The nurse enters the room of a depressed client Hirschsprungs disease who is admitted to the
to find the client poorly responsive and an hospital prior to surgery. Which would not be
empty pill bottle and water pitcher on the night included in the nursing care plan?
stand. What is the best initial action by the 1. Measure abdomen every 4 hours.
nurse? 2. Administer tap water enemas until clear
1. Determine if the client is on suicide results.
precautions. 3. Begin colostomy care teaching with the
2. Check the clients level of consciousness and parents.
vital signs. 4. Complete daily intake and output.

724 NCLEX-RN Review


53155_09_PT 02_p723-743.qxd 2/26/09 8:09 AM Page 725

19. The nurse is developing a care plan for a woman 23. A client with a head injury is moaning and
with cystitis. Which is most appropriate to complaining of head pain. The family requests
include in the care plan? that some pain medication be given. The nurse
1. Testing urine for protein with a dipstick. explains that most pain medications are usually
2. Promoting elimination of excess fluid by not given to clients with head injuries. The
maintaining NPO. family says, Aspirin wont hurt him. What is
the best rationale for withholding the aspirin
3. Encouraging voiding every 2 to 3 hours.
from a client with a head injury?
4. Telling her to take a tub bath to sooth the
1. Gastrointestinal distress.
urethra.
2. Tinnitus, making neuro assessments more
20. A man underwent an exploratory laparotomy difficult.
yesterday. He is on strict intake and output. 3. Increased likelihood of intracranial bleeding.
Calculate his intake and output for an 8-hour 4. Constriction of the pupils, making pupil
period. assessment more difficult.
Intake Output
24. A client has a closed head injury. Vital signs are
IVD5LR at 125 mL/hr Foley urine output
T 103F rectally; pulse 100; respirations 24;
850 mL
BP 110/84. Hourly urine output is 200 mL/hr.
PO1 ounce ice chips NG tube200 mL What is the best understanding of the cause of
NG irrigantNS these findings?
15 mL q 2 hr 1. Damage to the hypothalamus resulting in
Intake _________ Output _________ decreased hormone production.
2. Movement of fluid from the tissue into the
21. An adult is receiving Coumadin (warfarin) 5 mg
intravascular space, resulting from sepsis.
PO QD for treatment of a resolving deep vein
thrombosis. When asked, the client states his 3. An increase in antidiuretic hormone (ADH)
gums have been bleeding when he brushes his as a result of injury to the hypothalamus.
teeth. Which nursing action is most appropriate? 4. Fluid shifts from the tissue into the
1. Administer the daily dose of Coumadin, then intravascular space due to administration of
notify the physician so tomorrows dose can normal saline used during fluid resuscitation.
be adjusted.
25. An adult client has been on bed rest for several
2. Administer the daily dose of Coumadin. These months. Which statement best describes the
are the expected side effects of Coumadin. relationship between complications of
3. Hold the Coumadin and notify the physician prolonged bed rest and nursing interventions to
of the assessment findings. prevent these complications?
4. Hold the Coumadin until the next daily dose 1. Turning and positioning will help decrease
is due. the potential for calcium loss from bones.
2. Adequate fluid intake is vital to decrease the
22. A client has had Parkinsons disease for several
risk of brittle bones.
years. He exhibits all of the typical signs and
symptoms of advanced Parkinsons. He is being 3. Leg exercises are important to decrease the
discharged to his home with his 70-year-old wife loss of calcium from the bones and the risk of
as his primary caregiver. Which statement best pathological fractures.
indicates that his wife understands his 4. Encouraging milk intake will help decrease
symptoms and needs? the loss of calcium from the bones.
1. Since my husband is on Levodopa, I need to
26. An adult client had an exploratory laparotomy
watch him closely for things like facial
3 days ago. The nurse assesses the clients
grimacing and involuntary movements of his
incision and observes the following: edges of
trunk, legs, and arms.
incision well approximated; a small amount of
2. It is very important for my husband to rest edema noted the entire length of incision;
in bed or his chair most of the day. moderate amount of serosanguinous drainage.
3. My husband may have bad diarrhea due to What do these findings indicate?
his Parkinsons. 1. That the wound is likely to develop an
4. My husband may be embarrassed by his infection.
difficulties with eating so we should not go 2. An abnormal amount of wound drainage.
out like we used to do.

COMPREHENSIVE PRACTICE TESTS 725


53155_09_PT 02_p723-743.qxd 2/26/09 8:09 AM Page 726

3. A healthy postoperative wound. 3. Tamoxifen is used to treat your breast


4. That the wound is infected. cancer. It will help stop the tumor from
growing. You may have some nausea,
27. An order is written to start an IV on a 75-year- vomiting, and hot flashes from this drug.
old client who is getting ready to go to the 4. Tamoxifen is an antiulcer medication.
operating room for a total hip replacement. What Constipation is the major side effect.
gauge of catheter would best meet the needs of
this client? 31. An adult has terminal cancer. She is receiving
1. 18 morphine sulfate by PCA. She is grimacing and
2. 20 moaning occasionally. She sleeps for short
intervals. Her respiratory rate is 20, heart rate
3. 21 butterfly
100, and BP 140/90. Which is the most accurate
4. 25 assessment of this clients pain?
28. An adult client is taking prednisone. Which 1. As long as the client is sleeping for short
statement by the client best indicates periods, the pain is manageable.
understanding of the possible side effects of 2. The client may be hurting some, but her
prednisone therapy? respirations should not be depressed any
1. I really need to limit how much potassium I further.
take in since hyperkalemia is a side effect of 3. The client may need additional pain
this medicine. medication or an increase in dosage.
2. I must take this medicine exactly as 4. As long as the client does not voice
my doctor ordered it. I shouldnt skip complaints of pain, the nurse can assume she
doses or double up on them if I should is comfortable.
forget.
32. A child fell off her bicycle and was seen in the
3. This medicine will protect me from getting
emergency room for a mild concussion. The
any colds or infections.
X-rays and physical exam were normal and the
4. Since Im taking prednisone, my incision child was discharged. Which statement indicates
will heal much faster. the parents have a need for further instruction?
29. The nurse is inserting an indwelling urinary 1. I should let her sleep as much as possible
catheter. Which action is essential to decrease since she needs the rest.
the risk of complications associated with 2. I should report any vomiting episodes.
catheter insertion? 3. She should be seen in 1 or 2 days for a
1. Cleanse the female client using betadine- follow-up exam.
soaked 4 3 4s, cleaning from the rectal area 4. I can give her Tylenol (acetaminophen) as
to the clitoris. needed for pain.
2. Utilize a catheter that is slightly larger than
the external urinary meatus. 33. An adult is seen in the outpatient medical clinic
for an upper respiratory infection. The physician
3. Utilize clean technique.
prescribes erythromycin 500 mg PO q 8 h.
4. Test the retention balloon prior to Which statement indicates the client
insertion. understands the possible side effects of
erythromycin?
30. A client who is receiving tamoxifen (Tamofen)
20 mg PO BID asks the nurse the reason for this 1. I may have to increase my theophylline dose
medication and what side effects are possible. while Im taking this medication.
What is the nurses best response? 2. Erythromycin is supposed to cause
1. Tamoxifen is a vasodilator-antihypertensive. constipation, so I should take it with a glass
It will lower your blood pressure. The main of fruit juice.
side effects include dizziness, headache, 3. Taking antibiotics may cause me to get a
nasal congestion, and nausea. yeast infection.
2. Tamoxifen will help you manage your 4. This drug may cause me to feel light-headed
nausea and vomiting associated with when I stand up.
chemotherapy. It may cause you to be a little
sleepy and constipated, and to have dry
eyes.

726 NCLEX-RN Review


53155_09_PT 02_p723-743.qxd 2/26/09 8:09 AM Page 727

34. An infant is being treated for talipes equinovarus. 3. Your child can return 6 days after the first
Which statement by the childs mother indicates lesions appear, because the crusts will be
the best understanding of the casting process? formed.
1. My child will have successive casts until the 4. Your child must first learn to cough with her
desired results are achieved. mouth covered, put tissues in the trash, and
2. Wearing a cast is very painful, so Ill need to wash her hands after touching her nose and
medicate her every 4 hours. mouth.
3. Once the cast is on, it will remain on until
39. A 1-year-old child has a staph skin infection.
the deformity is corrected.
Her brother has also developed the same
4. My child will be immobilized and confined infection. Which behavior by the children is
to an infant seat. most likely to have caused the transmission of
the organism?
35. The nurse is caring for an adult client with
cirrhosis. What is the best explanation for the 1. Bathing together.
development of edema? 2. Coughing on each other.
1. Shunting of the blood from the portal vessels 3. Sharing pacifiers.
into vessels with lower pressure. 4. Eating off the same plate.
2. Inadequate formation, use, and storage of
vitamins A, C, and K. 40. The nurse is caring for a client who has a
nasogastric tube attached to low wall suction.
3. Decreased concentration of plasma
albumin. Which of the following is the nurse likely to
note when assessing the client?
4. Decreased production of aldosterone, causing
sodium and water retention. 1. Client vomits.
2. Client has a distended abdomen.
36. A man is admitted to the nursing care unit with a 3. There is no nasogastric output in the last
diagnosis of cirrhosis. He has a long history of 2 hours.
alcohol dependence. During the late evening 4. Large amounts of nasogastric output.
following his admission, he becomes increasingly
disoriented and agitated. Which of the following 41. An adult was placed in four-point restraints
would the client be least likely to experience? 3 hours ago after he attempted to hit a nurse.
1. Diaphoresis and tremors. Which observation by the nurse is the best
2. Increased blood pressure and heart rate. indication that the clients restraints could be
3. Illusions. discontinued?
4. Delusions of grandeur. 1. The client has had one hand and one leg free
for the past hour and has made no aggressive
37. The nurse is caring for an adult client who is moves.
scheduled for an intravenous pyelogram (IVP). 2. The client apologizes to the nurse and
Which nursing intervention is most essential? explains that he doesnt want to hurt anyone.
1. Encourage large amounts of fluids prior to the 3. The nurse has explained the importance of
test. not striking out in anger and the client
2. Assess for any indications of allergies. verbalized understanding.
3. Administer a laxative. 4. The medication administered to the client
4. Restrict fluids only in clients with marginal has been effective and he is now sleeping.
renal reserve or uncontrolled diabetes.
42. The nurse is planning care for a client who has
38. A child has chickenpox. Her mother calls a just had a renal biopsy. Which would the nurse
nurse friend to find out when the child can expect?
return to school. What is the best response for 1. The clients urine will be red.
the nurse to make? 2. The client will experience severe
1. All the lesions must be completely gone excruciating pain in the flank that radiates to
before contact with others is resumed. the groin.
2. Within 2 to 3 weeks, the itching should be 3. The client will be encouraged to drink at
under control and good hand washing least 3000 mL of fluid per day.
established so that contact with others can be 4. The client will ambulate 4 hours after the
started. procedure.

COMPREHENSIVE PRACTICE TESTS 727


53155_09_PT 02_p723-743.qxd 2/26/09 8:09 AM Page 728

43. The nurse is planning care for an adult client 3. Order clear liquids for the client.
who has just undergone a liver biopsy. Which 4. Withhold food from the client at this time.
nursing action is of highest priority? The physician may be notified of the absence
1. Making sure the client can void right away. of bowel sounds.
2. Measuring and recording the clients blood
pressure, pulse, and respiratory rate every 48. An adult is receiving O2 at 3 liters per nasal
10 to 20 minutes. cannula. His roommate lights a cigarette and
tosses the match, catching the curtain on fire.
3. Positioning the client on his left side with a
What is the priority action for the nurse?
pillow placed under his costal margin.
1. Turn off the oxygen.
4. Ambulating the client to the chair, placing a
pillow against his abdomen. 2. Sound the fire alarm.
3. Try to extinguish the flames.
44. An elderly client was admitted with a diagnosis 4. Remove the clients from the room.
of left-sided heart failure. Furosemide (Lasix)
80 mg IVP was given. Which indication shows 49. An 84-year-old male client has been bedridden
that the medication is not having the desired for 2 weeks. Which of the following complaints
effect? by the client indicates to the nurse that he is
1. Oliguria. developing a complication of immobility?
2. Hypotension. 1. Stiffness of the right ankle joint.
3. Absence of rales. 2. Soreness of the gums.
4. Polydipsia. 3. Short-term memory loss.
4. Decreased appetite.
45. Zantac is ordered for an adult client. The nurse
mistakenly administered Xanax. What is the 50. A woman has been diagnosed with cervical
most appropriate action for the nurse to take? cancer and will be undergoing internal radiation
1. Notify the physician, document in nurses in addition to surgery. The nurse is planning her
notes of error occurrence. nursing care. Check all that are appropriate in
2. Notify the supervisor, complete medication maintaining a safe environment.
error report, and document in nurses notes of Minimizing staff contact with the client.
error occurrence. Utilizing required shielding.
3. Notify the charge nurse, assess client hourly, Encouraging staff to stay at the foot of the
and document if adverse effects occur. bed or at the entrance to the room.
4. Notify the physician, complete incident Wearing isolation gowns when entering the
report, document notification of physician, room.
and any assessments made.
51. The lab results of a 68-year-old male reveal
46. A client who has ascites is admitted to the hospital an elevated titer of Helicobacter pylori.
and will be undergoing a paracentesis. What Which of the following statements, if made
should be included in the nursing care plan? by the nurse, indicates an understanding of
1. Monitor client closely for evidence of this data?
vascular collapse. 1. Treatment will include Pepto-Bismol and
2. Place client in Trendelenburg position for the antibiotics.
procedure. 2. No treatment is necessary at this time.
3. Encourage client to drink plenty of fluids to 3. This result indicates a gastric cancer caused
distend the bladder prior to the procedure. by the organism.
4. Have client remain on bed rest for 24 hours 4. Surgical treatment is indicated.
following the procedure.
52. Which of the following nursing interventions
47. A client had an exploratory laparotomy 2 days ago indicate an understanding on the part of the
and now has a new order for a soft diet. The nurses nurse concerning proper care of pressure ulcers?
assessment includes absence of bowel sounds in 1. Rub reddened skin to increase
any quadrant. What is the best nursing action? circulation.
1. Follow the physicians order and feed the client. 2. Use a heat lamp 4 times a day to dry the
2. Cancel the physicians order and make the wound surface.
client NPO.

728 NCLEX-RN Review


53155_09_PT 02_p723-743.qxd 2/26/09 8:09 AM Page 729

3. Cleanse a noninfected pressure ulcer with 57. A woman underwent a D&C under general
isotonic saline. anesthesia and was placed in lithotomy
4. Cleanse a noninfected pressure ulcer with position during surgery. Because she was
povodone-iodine. placed in lithotomy position, what assessment
is essential in the immediate postoperative
53. A female client, scheduled for a mammogram, is period?
called the day before regarding pre-procedure 1. Check anxiety level.
instructions. Which statement by the client best 2. Check for foot drop.
indicates adequate understanding of the
3. Check for sensation in lower extremities.
preparation?
4. Check for equal, bilateral radial pulses.
1. I know that I cant use deodorant, so I will
use powder instead that morning. 58. An adult will have to change the dressing on
2. I should eat a low-fat diet today and drink her injured right leg twice a day. The dressing
extra water. will be a sterile dressing, using 4 3 4s, normal
3. I should not use deodorant, powders, or saline irrigant, and abdominal pads. Which
creams under my arms. statement best indicates that the client
4. The technician will be able to tell me understands the importance of maintaining
immediately if my mammogram is okay. asepsis?
1. If I drop the 4 3 4s on the floor, I can use
54. A client has been placed in blood and body fluid them as long as they are not soiled.
isolation. Which statement by the nursing assistant 2. If I drop the 4 3 4s on the floor, I can use
indicates the best understanding of the correct them if I rinse them with sterile normal
protocol for blood and body fluid isolation? saline.
1. Masks should be worn with all client contact. 3. If I question the sterility of any dressing
2. Gloves should be worn for contact with material, I should not use it.
nonintact skin, mucous membranes, or soiled 4. I should put on my sterile gloves, then open
items. the bottle of saline to soak the 4 3 4s.
3. Isolation gowns are not needed.
4. A private room is always indicated. 59. A teen has his arm in suspension traction.
Which nursing assessment is highest
55. Autonomic dysreflexia may be manifested by priority?
flushed skin above the level of the lesion, pallor 1. Skin integrity.
below; severe hypertension; tachycardia; and 2. Neurovascular status of the affected
piloerection. How would these symptoms best extremity.
be explained?
3. Level of discomfort.
1. Parasympathetic nervous system stimulation
4. Knowledge about his injury.
with release of epinephrine.
2. Sympathetic nervous system hyperactivity 60. A 74-year-old client has been admitted with a
with release of norepinephrine. 3-day history of severe diarrhea. The nurse is
3. Disruption in the communication between assessing for fluid volume deficit. Which
upper motor neurons and lower motor neurons. findings are seen in the client with a fluid
4. Muscles served by a lower motor neuron no volume deficit?
longer receive stimuli and are unable to 1. Pedal edema.
contract. 2. Orthostatic hypotension and tachycardia.
3. Increased urine output.
56. An adult client is to participate in a double-
blind research study of a new medication. 4. Elastic skin turgor.
Which statement by the client indicates that the
61. The nurse is assessing a client who is
client does not understand the study risks?
developing slow progressive hydrocephalus.
1. I can drop out of the study at any time. Which is the nurse least likely to find in the
2. I must sign an informed consent form to be assessment?
in the study. 1. Client reports a headache.
3. They will tell me exactly what medication I 2. Client reports blurred vision.
am getting.
3. Rapid thready pulse.
4. My confidentiality will be protected in the
4. Decreased level of consciousness.
study.

COMPREHENSIVE PRACTICE TESTS 729


53155_09_PT 02_p723-743.qxd 2/26/09 8:09 AM Page 730

62. While assessing the client with a history of 67. Which finding would alert the nurse to potential
allergic asthma, the nurse questions the client problems in a newly delivered term infant of a
about what precipitates an attack. Which is the mother whose blood type is O negative?
clients response least likely to include? 1. Pallor.
1. Climate changes. 2. Negative direct Coombs.
2. Exposure to animal dander. 3. Infants blood type is O negative.
3. Exposure to high pollen and mold counts. 4. Resting heart rate of 155.
4. Seasonal changes.
68. A 10-year-old is admitted to the hospital with
63. A young child is admitted with acute epiglottitis. sickle cell crisis. Which client goal is most
Which is of highest priority as the nurse plans care? appropriate for this child?
1. Assessing the airway frequently. 1. The client will participate in daily aerobic
2. Turning, coughing, and deep breathing. exercises.
3. Administering cough medicine as ordered. 2. The client will take an antibiotic until the
4. Encouraging the child to eat. temperature is WNL.
3. The client will increase fluid intake.
64. A young child with bronchial asthma is 4. The client will utilize cold compresses to
admitted for the second time in 1 month. Cystic control pain.
fibrosis is suspected. Which physiological
assessment is most likely to be seen in the child 69. The nurse is caring for a client who has had a
with cystic fibrosis? total gastrectomy. The client complains of
1. Expectoration of large amounts of thin, frothy weakness, palpitations, cramping pains, and
mucus with coughing, and bubbling rhonchi diarrhea. He is also experiencing reactive
for lung sounds. hypoglycemia. What is the best explanation for
2. High serum sodium chloride levels and low these signs and symptoms?
sodium chloride levels in the sweat. 1. Rapid distention of the jejunal loop
3. Large, loose, foul-smelling stools with normal anastomosed to the stomach.
frequency or a chronic diarrhea of unformed 2. Lack of fluid intake at mealtime.
stools. 3. There is only a small opening from the gastric
4. Obesity from malabsorption of fats and remnant to the jejunum.
polycythemia from poor oxygenation of tissues. 4. The hypotonic intestinal contents draw
extracellular fluid from the circulating blood
65. A physician has prescribed tetracycline 500 mg volume into the jejunum to dilute the high
PO q 6 h. While completing the nursing history concentration of electrolytes and sugars.
for allergies, the nurse notes that the client is
also taking oral contraceptives. What is the most 70. An adult presents with severe rectal bleeding,
appropriate initial nursing intervention? 16 diarrheal stools a day, severe abdominal pain,
1. Administer the dose of tetracycline. tenesmus, and dehydration. Because of these
2. Notify the physician that the client is taking symptoms the nurse should be alert for
oral contraceptives. complications associated with which of these
diseases?
3. Tell the client she should stop taking oral
contraceptives because they are inactivated 1. Crohns disease.
by tetracycline. 2. Ulcerative colitis.
4. Tell the client to use another form of birth 3. Diverticulitis.
control for at least 2 months. 4. Peritonitis.

66. A client with an acute exacerbation of 71. An adult underwent a below-the-knee


rheumatoid arthritis is admitted to the hospital amputation 4 days ago. He is complaining of
for treatment. Which drug, used to treat clients burning, crushing pain in his amputated foot.
with rheumatoid arthritis, has both an anti- What is the best action for the nurse to take in
inflammatory and immunosuppressive effect? relation to administering his ordered pain
1. Gold sodium thiomalate (Myochrysine) medication?
2. Azathioprine (Imuran) 1. Explain to the client that he really is not
3. Prednisone (Deltasone) having pain and encourage him to wait until
later for his pain medication.
4. Naproxen (Naprosyn)

730 NCLEX-RN Review


53155_09_PT 02_p723-743.qxd 2/26/09 8:09 AM Page 731

2. Acknowledge the clients pain and 76. Which of the following statements, if made by a
administer the pain medication. 43-year-old female, would indicate to the nurse
3. Acknowledge the clients pain but tell him he in a cancer health screening clinic that further
really shouldnt use pain medication for his follow-up is needed?
pain. 1. My diet is low in fat and high in
4. Explain to the client that he really is not residue.
having pain and administer his pain 2. My mother and uncle died of colon
medication. cancer.
3. I have a yearly rectal exam.
72. A client with antisocial personality uses
4. I was born and raised in a rural
manipulation to gain access to a vending room
area.
close to the hospital entrance, where he attempts
to leave the hospital grounds. Which is the 77. A 212-year-old child is hospitalized for
best nursing intervention for manipulative severe otitis media. He was toilet trained prior
behavior? to being hospitalized but is having accidents
1. Place client in restraints for attempting to now that he is in the hospital. What is the
escape. best explanation for this change in
2. Help client identify patterns of manipulative behavior?
behaviors and the consequences as 1. It is unrealistic for a child at age 212 to be
determined by the team plan. toilet trained.
3. Deal with each incident of client 2. The nurse did not show the child where the
manipulation on individualized basis, bathroom is located.
dependent on the situation and nurse 3. A child of this age needs a parent available to
involved. assist with toileting.
4. Restrict the client from all activities to reflect 4. It is normal for a child to experience
on social behavior. regressive behavior due to the stress of
hospitalization.
73. An adult has undergone surgery for a detached
retina. What is essential to include in the 78. The nurse has been instructing the parents of
postoperative care plan? a toddler about nutrition. Which of the
1. Up ad lib with assistance. following statements best indicates the parents
2. Notify physician of severe pain or understanding of an appropriate diet for a
nausea/vomiting. toddler?
3. Bed rest in supine position. 1. Its unusual for a toddler to be a picky
4. Maintain a low carbohydrate diet. eater.
2. A multivitamin each day will meet my
74. A child is admitted to the rehabilitation center childs nutritional needs.
for gait training and use of adaptive devices. He 3. A toddler needs servings from each food
has cerebral palsy and a history of falls related to group daily.
spasticity. Which of the
4. Toddlers should still be eating prepared
following nursing goals has highest
junior foods.
priority?
1. Prevent deformity. 79. The nurse is caring for a woman who is having
2. Prevent physical injury. labor induced with an oxytocin (Pitocin) drip.
3. Establish locomotion. Which assessment of the client indicates there is
4. Ensure a balanced diet. a problem?
1. The fetal heart rate is 160 beats per
75. A 2-year-old is admitted to the hospital with minute.
meningitis. What is the highest priority? 2. The woman has three contractions in
1. Inform the parents of the childs 5 minutes.
condition. 3. Contraction duration is 60 seconds.
2. Maintain a quiet environment. 4. Early fetal heart rate decelerations are
3. Monitor for changes in intracranial occurring.
pressure.
4. Maintain bed rest.

COMPREHENSIVE PRACTICE TESTS 731


53155_09_PT 02_p723-743.qxd 2/26/09 8:09 AM Page 732

80. A client is in labor and taking three cleansing dilation. The nurse notes that the fetal heart rate
breaths followed by four slow, deep breaths with has dropped to 80 and suspects a prolapsed
each contraction. She is experiencing much cord. What is the most appropriate immediate
discomfort with her contractions. What action is nursing action?
most appropriate for the nurse to take? 1. Call for an emergency cesarean section.
1. Demonstrate to the woman a different 2. Place the woman in knee-chest position.
breathing pattern during contractions. 3. Place the expelled cord back into the vagina.
2. Ask the physician for an order for pain 4. Open up the main intravenous line.
medication.
3. Have the man take a break and instruct the 86. A woman is 25% over her ideal weight of 140
woman in another breathing pattern. pounds. She would like to lose weight before
4. Leave the couple alone as they have their becoming pregnant. The woman is 2 months into
routine established. her weight loss program. Which indicates she is
following proper weight management
81. The nurse is teaching childbirth education principles?
classes. What topic should be included during 1. Carefully selects only carbohydrate and fat
the second trimester? choices for meals.
1. Overview of conception. 2. Has lost a total of 4 pounds.
2. Medication usage and breastfeeding. 3. Is now 5% over her ideal weight.
3. Infant care. 4. Goes to beginning aerobics 3 times a week.
4. Strategies to relieve the discomforts of
pregnancy. 87. The nurse is caring for a client admitted with
herpes zoster, or shingles. What should the nurse
82. A woman 30 weeks pregnant is admitted to the expect to find during the initial assessment?
hospital with a diagnosis of placenta previa. She 1. Rhinorrhea, small red lesions, including
and the fetus are stable. To help achieve the goal some with vesicles that are widespread over
of avoiding premature delivery, what the face and body.
intervention will be initiated? 2. A painful vesicular eruption following a
1. Receive a blood transfusion. nerve pathway.
2. Be placed on bed rest. 3. Blisters on the lips and in the corners of the
3. Receive betamethasone. mouth.
4. Avoid sexual intercourse upon discharge. 4. Painful fluid-filled vesicles in the genital
area.
83. A 37-week-gestation neonate has just been born
to a woman with insulin-dependent diabetes 88. The mother of a 4-month-old who received his
mellitus and is admitted to the term nursery. second DTP immunization yesterday calls the
Which of the following is most essential when office nurse to report he has a temperature of
planning immediate care for the infant? 104F and a hard red area as big as a quarter on
1. Glucose monitoring. his thigh. What is the best interpretation of these
2. Daily weights. data about the child?
3. Supplemental formula feedings. 1. Reacting normally to the immunization.
4. An apnea monitor. 2. May be allergic to the vaccine.
3. Is developing symptoms of the disease.
84. The nurse is caring for a woman in labor who 4. Has developed a secondary infection.
suddenly complains of dizziness, becomes pale,
and has a 30-point drop in her blood pressure 89. The nurse is administering insulin to an adult
with an increase in pulse rate. What is the most client. Which is the correct method of
appropriate initial nursing action? administering insulin?
1. Turn her to her left side. 1. Administer via intramuscular injection
2. Have her breathe into a paper bag. holding the needle at a 90 angle.
3. Notify her physician. 2. Inject via Z track intramuscular injection.
4. Increase her intravenous fluids. 3. Use a 12 inch 27-gauge needle at a 90 angle
into subcutaneous tissue.
85. A client on the labor and delivery unit has 4. Inject at a 10 angle into the intradermal area.
spontaneous rupture of membranes at 2 cm

732 NCLEX-RN Review


53155_09_PT 02_p723-743.qxd 2/26/09 8:09 AM Page 733

90. The nurse is caring for a client from an Asian 3. The specific gravity of his urine is 1.001.
culture. The client refuses to look at the nurse 4. His apical pulse is 120 and his blood
and does not maintain eye contact. What is the pressure is 70/40.
best interpretation of this behavior?
1. The client is angry at the nurse. 95. Which statement by a postoperative client
2. The nurse is not effective in communicating suggests that he is ready to learn how to care for
with the client. his new ileostomy?
3. The client does not understand English. 1. I think Ill be able to wear one of those
pouches my doctor told me about without
4. The client is treating the nurse with
any problems. What do you think, Nurse?
respect.
2. I want to know all about my ileostomy, right
91. A male client tells the office nurse that his wife after you give me my pain medication.
does not let him change his colostomy bag 3. I suppose I have to learn how to take care of
himself. Which response by the nurse indicates this thing eventually, but it sure is
an understanding of the situation? disgusting.
1. Your wifes need to help you is a reality you 4. My wifes always been the one to do the
should accept. nursing in our house. Shell come in and
2. Do you think your wife might benefit from learn how to take care of it and Ill watch.
counseling?
96. An adult who is 1-day post abdominal
3. You feel you need privacy when changing
cholecystectomy complains to the nurse that she
your colostomy?
is having abdominal pain. The nurse goes in to
4. Have you discussed the situation with your assess her pain utilizing a visual analog scale.
doctor? Which action by the nurse would be initially
most important?
92. Which of the following nursing interventions
would the nurse perform prior to administering 1. Have the client describe the type and location
a tube feeding? of pain.
1. Check for placement by aspirating for gastric 2. Check the chart for the pain medication
contents with a syringe and test pH with order.
Testape. 3. Assess the clients abdomen.
2. Advance the tube 35 inches prior to the 4. Ask the client to locate the pain on a linear
feeding. scale.
3. Instruct the client to swallow.
97. An elderly widow is being discharged after
4. Instill 30 mL of sterile water into the treatment for chronic renal disease. Dietary
tube. teaching is being done to her daughter, with
whom she lives. The physician has ordered a
93. The nurse is caring for an adult who is 1-day
high-carbohydrate, low-protein, low-sodium
post-op following an abdominal
diet. Which of the following choices best reflects
cholecystectomy. Which finding by the nurse
the information the nurse should provide for the
indicates the patient-controlled analgesia (PCA)
clients family?
is controlling her pain?
1. Avoid canned and processed foods, do not
1. The clients vital signs have returned to
use salt replacements, substitute herbs and
preoperative status.
spices for salt in cooking, and when
2. The client is observed laughing and talking seasoning foods, call a dietitian for help.
with her family.
2. Use potassium salts in place of table salt
3. The client rates her pain as 8 on a 010 pain when cooking and seasoning foods, read the
scale. labels on packaged foods to determine
4. The client states that she is comfortable. sodium content, and avoid snack foods.
3. Limit milk and dairy products, cook separate
94. An adult male is admitted to a medical floor
meals that are low in sodium, and encourage
with a diagnosis of sepsis, dehydration. Which
increased fluid intake.
finding by the nurse indicates that he is now
rehydrated within normal limits? 4. Avoid eating in a restaurant, soak vegetables
well before cooking to remove sodium, omit
1. His urine output is 40 mL per hour.
all canned foods, and remove salt shakers
2. His skin tents when it is pinched. from table.

COMPREHENSIVE PRACTICE TESTS 733


53155_09_PT 02_p723-743.qxd 2/26/09 8:09 AM Page 734

98. An adult client is being treated in the burn unit 3. Blood pressure 80/50.
for partial- and full-thickness burns of the left 4. Complaints of constipation.
foot, ankle, and leg. Skin autografts are taken
from the right thigh and a skin graft is performed. 100. A client who weighs 380 pounds has had a
The nurse planning care for the client on return Roux-en-Y gastric bypass surgery. When the
from the operating room includes which of the clients sister asks questions about the procedure
following nursing interventions? and the postoperative care, which will be a
1. Change dressing on graft sites every shift. correct statement?
2. Cover donor site with fine mesh gauze and 1. The stomach was removed.
expose to air. 2. Several pounds of extra skin were removed.
3. Lubricate donor site with skin cream every 3. The client will be allowed 60 cc of fluid
shift. 6 times a day.
4. Hydrotherapy to graft sites daily. 4. It will help reduce the clients ability to
absorb nutrients and calories.
99. The client is being admitted with acute adrenal
insufficiency (Addisonian crisis). Which
assessment finding would be consistent with
this diagnosis?
1. Pulse 70.
2. Respirations 12.

Answers and Rationales for Practice Test 2

ANSWER RATIONALE NP CN CL SA
#1. 1. Having a current picture ID for each resident allows the nurse As Sa/1 Ap 1
to positively identify the client. This helps to decrease errors in
a population that may not always be able to respond appropriately.
The client may have taken off or put on another clients armband;
the client may be on another clients bed or in anothers room.
The visual picture will be the most accurate verification.
#2. 2. Drug hypersensitivity and allergic reactions should be documented As Ph/7 Co 1
on every perioperative client before administration. Deep breathing
exercises should have been taught at an earlier period as well as the
surgeons discussion of the procedure with the client. By this
time, the family would have already seen the client and be
in the waiting area. It is not the priority at this time if the
family is supportive.
#3. 4. Questions that begin with Why should be avoided, as it appears As Ps/4 An 1
blaming.
#4. 1. The ability to use relaxation is basic to the treatment of phobia. Pl Ps/4 Ap 2
Choices 3 and 4 are long-term goals. The client may not
know the cause of the phobia.
#5. 3. Work and family life are usually both affected by PTSD. The other As Ps/4 An 2
choices are symptoms of PTSD.
#6. 4. The client has typical symptoms of a panic attack, in which An Ps/4 An 2
he cannot focus on the nurses questions or current events.

734 NCLEX-RN Review


53155_09_PT 02_p723-743.qxd 2/26/09 8:09 AM Page 735

ANSWER RATIONALE NP CN CL SA
Anxiety is categorized into four levels of mild, moderate,
severe, and panic: mildlearning can occur; moderate
focus only immediate concerns; severeperceptual field
reduced; panicunable to follow directions or communicate.
#7. 2. Because Xanax is an anti-anxiety medication, assessment is Ev Ps/6 Ap 2
an appropriate action, and the only choice that evaluates the
effectiveness.
#8. 4. Isolation is separating unacceptable feelings from ones thoughts. An Ps/4 An 2
Denial is refusing to believe a stressful event has occurred;
conversion is changing unacceptable feelings into physical
symptoms; and introjection is taking in feelings/attitudes
of another.
#9. 2. Delusions are fixed false beliefs. They occur when the clients An Ps/4 An 2
unacceptable feelings are projected and rationalized. Ideas of
reference occur when events are directly related to him;
hallucinations are sensory perceptions without stimuli; and
dissociation involves a change in consciousness, such
as amnesia.
#10. 1. The nutritional problem in mania is the clients decreased Pl Ps/4 Ap 2
attention span and difficulty sitting still long enough to eat.
Snack foods that are easy to eat and have good nutritional
value may prevent malnutrition until the client is in better
control. The cafeteria may impose too much stimulation
and her attention span does not allow goal-oriented rewards.
#11. 4. Involvement in reality may decrease the clients preoccupation Im Ps/4 Ap 2
with his hallucinations. Medication is offered only if a threat
of danger to himself or others is apparent; a judgmental
approach may increase the clients hallucinations and agitation.
#12. 2. Clients with depression often have unrealistic expectations Ev Ps/4 An 2
for themselves and cannot set goals that are possible to reach.
Severely depressed clients may be unable to visualize any
future and thus are unable to set any goals. The other choices
do not show a true improvement in the condition.
#13. 2. Assessing the clients LOC will help determine the next actions. Im Ph/8 Ap 2
Whether the client is on precautions is irrelevant at this time;
questions to the client will be asked later when a stable condition
is obtained; determining the amount of medication is helpful,
however assessment and treatment is performed first.
#14. 3. Epistaxis (nosebleed) occurs in the first trimester. It is related As He/3 An 3
to capillary dilation. Dysuria is an abnormal condition with
urinary track infection; colostrum occurs at 16 weeks gestation;
and dependent edema may occur in the third trimester.
#15. 2. The normal respiratory rate is between 30 and 60, characterized As He/3 An 3
by shallow, irregular breaths, often interrupted by short periods
of apnea lasting 5 to 15 seconds. Hypertonia or a high-pitched/
shrill cry may indicate neurologic impairment or drug
withdrawal and normal head circumference is 3335 cm.

ANSWERS AND RATIONALES FOR PRACTICE TEST 2 735


53155_09_PT 02_p723-743.qxd 2/26/09 8:09 AM Page 736

ANSWER RATIONALE NP CN CL SA
#16. 4. Massaging the fundus is most important because her uterus is An He/3 An 3
soft and higher than normal. Fundal massage causes uterine
contraction leading to vasoconstriction, which will lead to
decreased bleeding. Cleaning and pad change along with
replacing IVF are important, but not before an action to
decrease bleeding. Information given does not indicate
the bladder is full.
#17. 4. Repeated hemarthrosis may result in flexion contractures and Pl Ph/7 Ap 4
joint fixations. During bleeding episodes, the affected joint must
be elevated and immobilized to prevent the crippling effects of
bleeding. Active range of motion is contraindicated during a
bleeding episode. Dental care and genetic counseling are
both appropriate, but neither is a priority action during a
bleeding episode.
#18. 2. Tap water enemas are contraindicated in children, as the Pl Ph/5 Ap 4
hypotonic solution can cause rapid fluid shift and fluid
overload. The other choices are appropriate interventions.
#19. 3. This prevents overdistention of the bladder and a Pl Ph/7 Ap 3
compromised blood supply to the bladder wall. Protein
is not in the urine in a client with cystitis; fluids
should be increased to promote flushing out the bacteria;
tub baths should be avoided due to the chance of bacteria
in the water entering the urethra.
#20. Intake 5 1090 mL; Output 5 1050 mL. Im Ph/6 Ap 1
125 mL/hr (125 3 8 hr) is 1000 mL. One ounce of ice chips
is 30 mL; NG irrigant 15 mL q 2 (15 mL 3 4) is 60 mL for
a total of 1090 mL. Output is 850 mL urine and 200 mL of
nasogastric drainage for a total of 1050 mL.
#21. 3. The physician should be notified prior to administration Im Ph/6 Ap 1
because bleeding gums are an adverse side effect of
Coumadin and may indicate overdose. The dose should
be held until blood tests are performed.
#22. 1. Dyskinesias (abnormal involuntary movements) are fairly Ev Ph/6 Ap 1
common side effects of Levodopa. This may be due to
the effect of the bodys disappearance of dopamine. The
dose of Levodopa may need adjusting. An exercise
program is important; constipation is common due
to inactivity or inadequate fluid intake; and drooling
is common with Parkinsons.
#23. 3. Aspirin (ASA) is an anticoagulant and increased the An Ph/6 An 1
clients potential for further bleeding. Large doses may
cause GI bleeding or tinnitus. ASA does not cause
constriction of the pupils.
#24. 1. Injury to the hypothalamus usually leads to decreased secretion An Ph/8 An 1
of antidiuretic hormone (ADH), which is manifested by large
amounts of very dilute urine output. The hypothalamus also
controls temperature. Injury causes a very high temperature.
#25. 3. The ideal exercises will have some resistance of weight bearing An Ph/7 An 1
as tolerated. Turning and positioning is essential for skin

736 NCLEX-RN Review


53155_09_PT 02_p723-743.qxd 2/26/09 8:09 AM Page 737

ANSWER RATIONALE NP CN CL SA
protection: fluids help mobilize pulmonary secretions; milk
intake will not decrease loss of calcium.
#26. 3. The findings are typical of healing, common health incision. Ev Ph/7 An 1
No evidence is present to indicate an infected wound.
#27. 1. Clients going to the operating room ideally should have an Im Ph/6 Ap 1
18 gauge catheter. This is large enough to handle blood products
safely and to allow rapid administration of large amounts of fluid
if indicated during the perioperative period. A 20 gauge would be
a second choice, and the others are too small.
#28. 2. Prednisone should be taken exactly as ordered. It is very important Ev Ph/6 An 5
not to skip doses. Stopping the medication suddenly may result
in adrenal insufficiency manifested by anorexia, nausea, fatigue,
weakness, hypotension, dyspnea, and hypoglycemia. If these
appear, the physician should be notified immediately as this can
be life threatening. Prednisone can cause hypokalemia,
immunosuppression, and slow wound healing.
#29. 4. The balloon should be checked for inflation and leaks prior to Im Ph/7 Ap 1
insertion, preventing repeated catherizations if the balloon fails.
Sterile technique is used after cleansing from front to back,
using a catheter slightly smaller than the meatus.
#30. 3. It is given following a mastectomy to prevent recurrence. Side Ev Ph/6 Ap 5
effects include nausea, vomiting, hypercalcemia, and hot flashes.
The medication suppresses tumor growth, and is not for use in
hypertension, nausea, or ulcers.
#31. 3. The clients actions of grimacing and moaning, along with the Ev Ph/6 An 1
elevated vital signs may indicate that she has not had adequate
pain relief. It would be advisable to assess whether she is using
the PCA machine correctly, or if cultural issues are prohibiting the
client from voicing complaints. Depending on the findings,
the physician may need to be notified for a change in dosage.
#32. 1. A child with a concussion should be aroused every 2 hours and Ev Ph/7 Ap 4
evaluated for responsiveness. All the other actions are appropriate.
#33. 3. Erythromycin may lead to superimposed infection including Ev Ph/6 Ap 5
yeast infection. Other side effects include increased theophylline
blood levels so the dose may need to be decreased; as diarrhea may
also occur, extra water instead of juices should be encouraged to
prevent a yeast infection; light-headedness is associated with
antihypertensives.
#34. 1. Cast changes will be repeated throughout the course of treatment, Ev Ph/7 An 4
usually every 12 week period. Although casts may feel heavy,
continuous pain would need to be reported to the physician.
Age appropriate activity should be encouraged.
#35. 3. The late symptoms of cirrhosis can be attributed to chronic failure An Ph/8 An 1
of liver function. The concentration of plasma albumin is reduced,
leading to the formation of edema. Fibrotic changes in the liver will
cause the development of collateral vessels, which can form varices/
hemorroids; the vitamins noted are involved in the clotting factor;
and overproduction of aldosterone causes sodium and water
retention.

ANSWERS AND RATIONALES FOR PRACTICE TEST 2 737


53155_09_PT 02_p723-743.qxd 2/26/09 8:09 AM Page 738

ANSWER RATIONALE NP CN CL SA
#36. 4. Delusions of grandeur are symptomatic of manic clients, not As Ps/4 Co 1
clients withdrawing from alcohol. The symptoms and history
of alcohol abuse suggest this client is in alcohol withdrawal.
#37. 2. The client should be assessed for allergic reactions to iodine, As Ph/7 Ap 1
i.e., shellfish allergy or previous allergic reaction to contrast
material. Liquids may be restricted up to 10 hours prior to the test
to concentrate the urine; laxatives may have been administered the
night before, but is not essential; fluids are generally not restricted
in clients with renal or diabetic conditons to prevent dehydration.
#38. 3. Varicella zoster is found in the respiratory secretions of infected Im He/3 Co 4
person and also in the skin lesions that are not scabbed over. Scabs
are not infectious, which are usually crusted over by 6 days.
#39. 1. Direct contact is the mode of transmission for staphylococcus. An Sa/2 An 4
#40. 4. The purpose of the NG tube is to remove stomach contents, As Ph/7 Ap 1
therefore the first three choices would be assessed as reasons
that output is not occurring. If large amounts are present,
then the tube is performing adequately.
#41. 1. The controlled behavior demonstrates an ability to remain in An Sa/2 Ap 1
control. Apologizing, verbalizing, and sleeping do not
demonstrate the capability to maintain behavior control.
#42. 3. Increasing fluid intake will decrease hematuria, unless the Pl Ph/7 Ap 1
client has renal insufficiency. Gross hematuria should not be
expected. Pain may be from a clot in the ureter; bed rest is
recommended for 24 hours following the procedure.
#43. 2. Vital signs are the priority action, due to the possibility of hepatic Pl Ph/7 Ap 1
bleeding. The client will be placed on his right side with a pillow
under the costal margin and kept on bed rest for several hours to
decrease the risk of bleeding.
#44. 1. Lasix is a loop diuretic that should increase urinary output. Ev Ph/6 Ap 1
Oliguria is decreased urinary output. All the other symptoms
would be expected during the diuresis.
#45. 4. This would be the proper protocol. The incident report will not Im Sa/1 Ap 5
be included in the clients medical record. It would be advisable to
also inform the client of the error and to perform appropriate
monitoring, i.e., vital signs, labs.
#46. 1. Removing large amounts of fluid may cause vascular collapse, Pl Ph/7 Ap 1
therefore vitals sign are essential. The position for the procedure
is usually an upright position: the client voids before the
procedure to minimize puncturing the bladder; bed rest
is not necessary.
#47. 4. It would be advisable to withhold food as peristalsis is not Im Ph/5 Ap 1
present; food would set the patient up for nausea and a
probable nasogastric tube if administered before the return of
bowel motility.
#48. 1. If the client is not in immediate danger, turn off the oxygen, then Im Sa/2 Ap 1
follow the RACE protocol: Rescue, Alarm, Contain, Extinguish.

738 NCLEX-RN Review


53155_09_PT 02_p723-743.qxd 2/26/09 8:09 AM Page 739

ANSWER RATIONALE NP CN CL SA
#49. 1. Stiffness of a joint is the only choice that may indicate the As Ph/5 Co 1
beginning of a contracture and/or early muscle atrophy.
#50. All are appropriate except for wearing an isolation gown, Pl Ph/7 Ap 3
which does not offer protection from radiation.
#51. 1. H. pylori is the bacteria believed to cause most chronic gastritis An Ph/7 Co 1
or peptic ulcers. The use of these medications suppresses and
eradicates the bacteria, which can be predisposing to cancer.
#52. 3. Cleansing should include an isotonic solution to prevent Im Ph/5 Ap 1
disruption of healing. Evidence-based practice has shown not
to rub reddened areas; heat lamps are no longer used.
#53. 3. Aluminum chlorhydrate (found in many deodorants, powders, Ev Ph/7 Ap 3
creams) may mimic calcium clusters, so the client is instructed
not to wear these. Diet is not affected; results are received from
the physician.
#54. 2. Gloves would be protection from blood and body fluids, as the Ev Sa/2 An 1
other choices would not be the correct protocol.
#55. 2. This condition is usually seen above the 6th thoracic vertebra in An Ph/7 An 1
spinal cord injuries. This results from uninhibited sympathetic
discharge with release norepinephrine. Choice 3 is related to a
client experiencing shock.
#56. 3. A double-blind research study of a new medication generally will Ev Sa/1 An 1
have a placebo and test drug. The client will not know which drug
is being administered.
#57. 4. Positioning changes can cause hypotension, so evaluation of As Ph/7 An 3
cardiovascular stability is performed first, followed by the pulse
circulation checks. Sensation would be assessed later when the
client is more alert; footdrop occurs when pressure is placed on
the peroneal nerve, which is unlikely in the lithotomy position.
#58. 3. If there is any doubt about the sterility of an instrument or Ev Sa/2 An 1
dressing, it should not be used. The sterile field will be within
visual and physical proximity of the person performing the action.
#59. 2. It is essential to first assess to the neurovascular status (color, As Ph/7 Ap 4
temperature, capillary refill, edema, pulses, sensations,
movement) frequently and to compare to unaffected extremity.
#60. 2. These are seen in a volume depleted person. The other choices are As Ph/8 An 1
apparent in a client with either proper hydration or fluid overload.
#61. 3. A rapid thready pulse is a sign of shock, not hydrocephalus, As Ph/8 An 1
which causes increased intracranial pressure.
#62. 1. Exacerbation triggered by climate changes (cold air, air pollution) As Ph/7 An 1
is most often associated with non-allergic allergy. The other
choices can trigger an attack.
#63. 1. Airway occlusion frequently occurs with epiglottis. No liquid
medications or food should be administered at this time,
because of the swelling of the infected tissue in the throat which Pl Ph/7 Ap 4
may block the airway and cut off breathing. Turning, coughing and
deep breathing are not a priority at this time as for a client with a
lung condition, such as pneumonia.

ANSWERS AND RATIONALES FOR PRACTICE TEST 2 739


53155_09_PT 02_p723-743.qxd 2/26/09 8:09 AM Page 740

ANSWER RATIONALE NP CN CL SA
#64. 3. The obstruction of the pancreatic duct with thick mucus prevents As Ph/7 An 4
digestive enzymes from entering the duodenum, thus preventing
digestion of food. Undigested food (mainly fats and proteins) are
excreted in the stool, increasing the bulk to twice the normal
amount. Expectoration is very difficult because the excess mucus
produced is tenacious and viscous. Elevated sweat chloride above
60 mmol/L is consistent with the diagnosis of cystic fibrosis.
#65. 2. Tetracycline decreases the effectiveness of oral contraceptives. Im Ph/6 An 5
Document on nurses notes or in chart that physician was notified.
The client would not be instructed by the nurse to stop
contraceptive use or use another method unless by physician order.
#66. 3. Prednisone is the only drug presented that has both properties. An Ph/6 K 5
Gold and Imuran have only an immunosuppressive effect;
naproxen has only an anti-inflammatory effect.
#67. 1. When maternal sensitization occurs, maternal antibodies destroy An Ph/7 An 3
the fetuss red blood cells, leading to anemia and pallor. Negative
direct Coombs indicates no development of maternal antibodies;
O negative would not present an incompatibility; HR of 155 is a
normal finding.
#68. 3. Adequate hydration prevents sickling and delays the stasis Pl Ph/7 Ap 4
thrombosis-ischemic cycle. Exercise should be avoided because
it causes cellular metabolism; antibiotics are given only for
710 days; and cold enhances vasoconstriction.
#69. 1. The sign and symptoms suggest dumping syndrome. The An Ph/7 An 1
exact cause is unknown, but rapid emptying in the small
intestine is associated with the symptoms. It is prevented
by taking in small meals at frequent intervals. Ingestion of
fluids usually increases signs and symptoms; there is a
large opening from the gastric remnant to the jejunum;
intestinal contents are hypertonic.
#70. 2. The signs and symptoms described are associated with ulcerative An Ph/8 An 1
colitis. Less frequent diarrhea, crampy pain, and abdominal pains
are associated with the other diseases, respectively.
#71. 2. Phantom-limb pain is a commonly occurring complication of Im Ph/6 Ap 1
amputation. The nurse should recognize that the pain is real to
the client and administer pain medication as ordered. The other
choices are not therapeutic to the client.
#72. 2. Help the client to develop a relationship between his behavior Im Ps/4 Ap 1
and the consequences. Do address the client with a consistent
team plan to minimize manipulation and to help the client
understand and live within set limits. The client needs
continued opportunities to interact with staff and peers.
#73. 2. The physician should be notified of any severe pain that does not Pl Ph/7 Ap 1
respond to therapy, severe nausea, vomiting, swelling, cloudy
vision, a halo around lights, or purulent or excessive mucoid
drainage. Activity may be restricted to bed rest with bathroom
privileges and may include face down or on the clients side if a
gas tamponade is used (so that gas bubble will float into the best
position). Diet is not a factor in the postcare instructions.

740 NCLEX-RN Review


53155_09_PT 02_p723-743.qxd 2/26/09 8:09 AM Page 741

ANSWER RATIONALE NP CN CL SA
#74. 2. All are important, but the history of falls and spasticity would Pl Ph/7 Ap 1
initiate this as a priority nursing goal.
#75. 3. All are important: however, changes in intracranial pressure Pl Ph/7 An 4
can be life-threatening.
#76. 2. A family history of colon cancer is a known risk factor. As He/3 An 3
Susceptibility to some forms of colon cancer is inherited.
The other choices are not risk factors.
#77. 4. Regressive behavior is frequently seen in children who are under An He/3 Ap 4
stress. This age is appropriate for toilet-training and the child
could be assisted by anyone.
#78. 3. Toddlers present a challenge to parents because they are picky Ev He/3 An 4
eaters, so food choices would include a variety of food servings
from all food groups.
#79. 2. If the woman has more than three contractions in 5 minutes, the Ev Ph/6 Ap 3
oxytocin should be discontinued. Normal fetal heart rate is
120160 beats/min; normal contraction is 4090 seconds; early
decelerations indicate fetal head compression but not distress.
#80. 1. Appropriate demonstration does not belittle the man or diminish Im He/3 Ap 3
his wifes confidence in him. This allows the man to maintain
continued control in the situation.
#81. 4. Many discomforts arise during the second trimester and Pl He/3 Ap 3
information regarding relief will make pregnancy much
more comfortable. The other topics would be discussed at
other periods of pregnancy.
#82. 2. Restricted activity is the most important measure to minimize Pl He/3 Ap 3
stress on the cervix and reduce chances of premature labor. The
other choices do not prevent premature delivery.
#83. 1. Because the infant is no longer exposed to the mothers high Pl He/3 Ap 3
circulating glucose levels and its own pancreas is still
secreting insulin in response to the glucose, the infant is subject
to hypoglycemia.
#84. 1. The signs and symptoms described are those of vena caval Im He/3 Ap 3
syndrome. It is most important to remove the gravid uterus from
the interior vena cava and the aorta. Turning the woman to the
left side will accomplish this.
#85. 2. Knee-chest position removes pressure from the cord, which is Im Ph/8 Ap 3
caught between the presenting part and womans pelvis.
A cesarean will most likely be performed, but the first action
will be to remove the pressure.
#86. 4. Traditional weight loss programs combine dieting, exercise, Ev He/3 An 3
psychosocial support, and behavior modification. Protein
should be included in the diet; a 4 lb weight loss is inadequate
for 2 months; or has occurred too quickly, respectively.
#87. 2. Herpes zoster (shingles) is an acute infectious disease caused by As Ph/8 An 1
the Varicella zoster virus, accompanied by painful vesicular
eruptions. The others choices are associated with true chicken pox,
Herpes simplex I, and Herpes simplex II.

ANSWERS AND RATIONALES FOR PRACTICE TEST 2 741


53155_09_PT 02_p723-743.qxd 2/26/09 8:09 AM Page 742

ANSWER RATIONALE NP CN CL SA
#88. 2. The description is of an adverse reaction to the immunization, An He/3 An 4
showing an allergic response.
#89. 3. Insulin is administered via a short needle at a 90 angle into Im Ph/6 Ap 1
the SQ tissue. In a person with adipose tissue, it can be
administered at a 60 angle, but always in SQ tissue. Z-track
is used only in IM injections.
#90. 4. In many Asian cultures, a person does not look directly at a person An Ps/4 An 1
who is in a position of authority or who is greatly respected.
No evidence of anger or non-English speaking actions are indicated.
#91. 3. This type of communication technique, making an observation, An Ps/4 Ap 1
enables the nurse to acknowledge that something exists or has
changed in some way. This acknowledgement made by the nurse
should open communication with the client. The nurse should
avoid making assumptions and jumping to conclusion without
involving the client.
#92. 1. Placement is always checked first before any administration of As Ph/7 Ap 1
liquids, by either aspiration of gastric contents or auscultating
over stomach area with injected air. Because the tube was
initially placed in the stomach, no advancement is required
and swallowing was performed during insertion of tube
and does not assess placement.
#93. 4. The only true evidence that a clients pain is controlled is Ev Ph/7 An 1
that the client says it is.
#94. 1. A normal urinary output is indicative of adequate hydration Ev Ph/8 An 1
and renal perfusion. Urine output for adults should approximate
0.5 mL/kg/hr or 30 mL/hr. The other symptoms are associated
with either overhydration or hypovolemic shock.
#95. 1. By soliciting the nurses opinion, the client is indicating he is As Ps/4 An 1
ready to learn. Pain inhibits the learning process; the Disturbed
Body Image statement also inhibits learning; and lastly, the
client is pushing care to someone else.
#96. 4. A visual analog pain scale is a line indicating the intensity of pain As Ph/7 Ap 1
with visual anchors at either end, one end indicating the worse
possible pain and the other end indicating no pain. The other
actions will be performed also, but the questions pertains to using
a visual analog scale.
#97. 1. Salt is used as a preservative in most canned and processed foods. Pl Ph/5 Ap 1
Salt-substitutes may contain high potassium, which would be
contraindicated. Limit milk and dairy products as they are high
in protein; eating out in restaurants is possible as long as care
in selections is utilized.
#98. 2. The donor site may be treated in a variety of ways, but the most Pl Ph/7 Ap 1
common method is to cover the wound with a fine mesh gauze
or impregnated gauze that is open to the air or exposed to a heat
lamp to allow the wound to dry. Dressings are usually changed
every 4872 hours; the healed wound (not donor site) is lubricated
to prevent drying and itching. Hydrotherapy is used in cleaning
wounds, removing eschar and necrotic tissue, but autografts would
be dislodged by the currents.

742 NCLEX-RN Review


53155_09_PT 02_p723-743.qxd 2/26/09 8:09 AM Page 743

ANSWER RATIONALE NP CN CL SA
#99. 3. As Addisonian crisis develops, this condition is characterized by As Ph/8 Co 1
signs of cyanosis, shock, apprehension, rapid and weak pulse, rapid
respiration, and low blood pressure with additional complaints
of nausea and diarrhea.
#100. 4. This procedure is recommended for long-term weight loss and is a As Ph/8 Ap 1
combined and mal absorptive procedure. A small pouch will only
allow approximately 30 mL of fluid to be ingested at a time. The
jejunum is divided and anastomosed to the new pouch. After the
client has lost 1001 pounds, the client may elect to have a
panniculectomy, which will remove excess fat and skin.

ANSWERS AND RATIONALES FOR PRACTICE TEST 2 743


53155_09_PT 03_p744-764.qxd 2/26/09 8:13 AM Page 744

Practice Test 3

1. A woman is admitted for a suspected duodenal 1. Accommodating his frequent need for the
ulcer. The nurse is interviewing her for an bedpan.
admission history. Which description of her 2. Maintaining the gastric pH.
pain would be most characteristic of a duodenal 3. Monitoring vital signs on an hourly
ulcer? basis.
1. Aching in the epigastric area that wakens her 4. Rapid blood and fluid administration.
from sleep.
2. Right upper quadrant pain that increases after 5. An adult has just had a broken left ankle casted
meals. in the emergency department. He will be going
3. Sharp pain in the epigastric area that radiates home to a second floor apartment. What
to the right shoulder. teaching instructions will be given on how to
4. A sensation of painful pressure in the walk upstairs with crutches?
midsternal area. 1. Resting his weight on his right foot while he
lifts the crutches and his left foot to the next
2. The nurse is playing with a 2-year-old child with step, then resting his weight on the crutches
tetralogy of Fallot, who suddenly squats on the while he brings his right foot up onto the
floor. What is the best initial nursing action? same step.
1. Return the child to bed immediately. 2. Resting his weight on the crutches while he
2. Allow the child to remain in that position. lifts his right foot to the next step, then
3. Place the child in a chair. moving the crutches and his left foot to the
same step.
4. Call the physician immediately.
3. Holding both crutches with his left arm and
3. The nurse is caring for a client with cirrhosis of using the crutches to bear part of his weight
the liver who has developed esophageal varices. while he lifts his right foot to the next step,
The nurse understands that the best explanation then moving his crutches and his left foot to
for development of esophageal varices is which the same step.
of the following? 4. Sitting on the steps and using his right leg
1. Chronic low serum protein levels result in and left arm to lift his weight and place his
inadequate tissue repair, allowing the buttocks on the next step, while pulling the
esophageal wall to weaken. crutches with his right hand.
2. The enlarged liver presses on the diaphragm,
6. A child has cerebral palsy and is hospitalized for
which in turn presses on the esophageal wall,
corrective surgery for muscle contractures. What
causing collapse of blood vessels into the
is the most important immediate postoperative
esophageal lumen.
goal?
3. Increased portal pressure causes some of the
1. Ambulate using adaptive devices.
blood that normally circulates through the
liver to be shunted to the esophageal vessels, 2. Demonstrate optimal oxygenation.
increasing their pressure and causing 3. Verbalize pain control.
varicosities. 4. Complete daily self-care needs.
4. The enlarged liver displaces the esophagus
toward the left, tearing the muscle layer of the 7. A 5-year-old child with a terminal illness is
esophageal blood vessels, which allows small talking to the nurse. Which of the following
aneurysms to form along the lower esophageal best reflects a 5-year-olds understanding of
vessels. death?
1. Ill see Grandma in heaven.
4. An adult with esophageal varices begins to 2. Will it hurt when I die?
experience severe gastrointestinal bleeding. To 3. Can Mommy go with me?
meet the clients fluid needs, what priority
4. It isnt fair. Why me? Im too young
should be included in the plan of care?
to die.

744 NCLEX-RN Review


53155_09_PT 03_p744-764.qxd 2/26/09 8:13 AM Page 745

8. The nurse is teaching the parents of a child who 3. Pull objects rather than push them.
is being treated in clinic for otitis media. Which 4. Wear supportive stockings when working.
of the following statements is essential to
include in the teaching? 14. The nurse is caring for a woman in labor. The
1. Do not take acetaminophen as this is woman is irritable, complains of nausea and
contraindicated. vomits, has heavier show, and the membranes
2. Take the medication until the pain and fever have ruptured. What does this indicate?
are gone. 1. The woman is in transition stage of labor.
3. Do not apply heat to the ear. 2. The woman is having a complication and the
4. Take all of the medication as ordered. doctor should be notified.
3. Labor is slowing down and the woman may
9. The nurse is planning care for a child who must need oxytocin.
remain in a croup tent continuously. Which goal 4. The woman is emotionally distraught and
is of highest priority? needs assistance in dealing with labor.
1. The tent will remain closed, except for
feedings and hygiene. 15. The nurse is caring for a woman who had a
2. The child will maintain normal body vaginal delivery an hour ago without
temperature and have dry linens. complications. She has a boggy fundus after
voiding 500 mL. What would be the highest
3. The tent will deliver mist and cooled air
priority for the nurse to address?
simultaneously while the child is inside.
1. Massaging the fundus until it is firm.
4. The child will find entertainment within the
tent to encourage compliance. 2. Assessing the lochia.
3. Adding Pitocin to the intravenous solution
10. A woman comes to the prenatal clinic because being administered.
she thinks she might be pregnant. She tells the 4. Calling the health care provider.
nurse that her menstrual periods are irregular but,
since her last menses 7 weeks ago, shes noticed 16. A client who is having a saline abortion is being
some physiologic changes in her body. Which cared for on the labor floor. The clients vital
finding should the nurse expect when assessing signs are temperature 101F, pulse 100,
the woman for a probable sign of pregnancy? respirations 24, blood pressure 120/90. How
1. Morning sickness. does the nurse interpret this data?
2. Urinary frequency. 1. The blood pressure is elevated from receiving
3. A positive pregnancy test. the saline injection.
4. Auscultation of fetal heart sounds. 2. The vital signs are within normal limits for a
client undergoing saline abortion.
11. A woman in the prenatal clinic tells the nurse that 3. The client is at extreme risk for shock.
the first day of her last normal menstrual period 4. The client may be developing an infection,
was June 15th. The nurse uses Ngeles rule to such as chorioamnionitis.
calculate the due date as being about _______.
17. A woman who is taking oral contraceptives tells
12. A woman who is 6 months pregnant is seen in her nurse neighbor that she is experiencing a
antepartal clinic. She states she is having trouble vaginal discharge. What would be the most
with constipation. To minimize this condition, appropriate advice the nurse should give the
what instruction would the nurse provide? woman?
1. Increase her fluid intake to 3 liters/day. 1. Purchase an over-the-counter remedy.
2. Request a prescription for a laxative from her 2. Change undergarments.
physician. 3. See the physician as soon as possible.
3. Stop taking iron supplements. 4. Stop taking the oral contraceptives.
4. Take 2 tablespoons of mineral oil daily.
18. The nurse is assessing a woman for sexually
13. A nurse works 12 hour shifts in a hospital transmitted diseases. Which symptom would be
setting. What intervention would be beneficial most apt to be present in a woman with a
for the nurses health? Trichomonas vaginalis vaginal infection?
1. Drink at least 1 liter of fluid per day. 1. A profuse, white, bubbly discharge.
2. Elevate legs at least twice during the shift. 2. White cheese-like patches in the vagina.

COMPREHENSIVE PRACTICE TESTS 745


53155_09_PT 03_p744-764.qxd 2/26/09 8:13 AM Page 746

3. A perineal deformity. 3. Tremors, muscular weakness, and mask-like


4. An ulcer or lesion of the vulva or vagina. face.
4. Vertical and horizontal nystagmus.
19. The nurse is assessing a newborn 5 minutes
after birth. He has full flexion of the extremities, 24. A group of eight psychiatric clients have been
is acrocyanotic, has a heart rate of 124, a together in group therapy for 12 sessions. There
full, lusty cry, and resists the suction has been an expression of warm feelings, self-
catheter. The nurse should record the disclosure, and an awareness of events in the
Apgar score as ________. here and now. The group finds it difficult to deal
with two members who must now join the group.
20. A client in the active phase of labor has just The nurse recognizes that group members are
been given continuous epidural anesthesia. experiencing which phase of the group process?
Which assessment finding indicates to the nurse 1. Beginning phase.
that the client is experiencing a common side
2. Transition phase.
effect of this type of anesthesia?
3. Middle phase.
1. Blood pressure of 50/30.
4. Termination phase.
2. Uterine pain.
3. Fetal heart rate of 140. 25. An adult has been informed that he needs
4. Euphoria. surgery for rectal cancer. His response is,
Theres nothing wrong with me. I just have
21. An adult has had a cerebrovascular accident hemorrhoids. The nurse knows this response to
(CVA) and has severe right-sided weakness. She be which defense mechanism?
has been taught to walk with a cane. The nurse 1. Projection.
is evaluating her use of the cane prior to
2. Repression.
discharge. Which of the following reflects
correct use of the cane? 3. Denial.
1. Holding the cane in her left hand, the client 4. Displacement.
moves the cane forward first, then her right
26. A female client is hospitalized for depression.
leg, and finally her left leg.
One evening after an argument with her
2. Holding the cane in her right hand, the client husband, she discusses with the evening nurse
moves the cane forward first, then her left her intent to cut her wrists. Her husband has
leg, and finally her right leg. threatened to divorce her and retain custody of
3. Holding the cane in her right hand, the client the children. What is the most appropriate
moves the cane and her right leg forward, initial action for the nurse to take?
then moves her left leg forward. 1. Attempt to convince the client of the need to
4. Holding the cane in her left hand, the client address her husbands threats instead of
moves the cane and her left leg forward, then using self-destructive behavior.
moves her right leg forward. 2. Place the client on suicide precautions,
which restrict her leaving the nursing unit.
22. The nurse is caring for a client who has just
had a splenectomy. When planning care in the 3. Place the client on suicide precautions,
immediate postoperative period the nurse requiring close observation and one-to-one
should avoid using which position? monitoring by nursing staff.
1. Left side-lying. 4. Recognize the suicidal remarks as less serious
because the client is in a safe environment.
2. Right side-lying.
3. Semi-Fowlers. 27. A teen is brought to the emergency room in a
4. Supine. semiconscious state. The friend who
accompanies him reports that he fell while
23. A 23-year-old is admitted to the hospital after playing ball. The nurse assesses the client and
having taken LSD. What should the nurse assess notes Kussmaul respirations, hypotension,
for? tachycardia, decreased reflexes, and an acetone
1. Dilated pupils, flushing, and tremors. odor to his breath. Which question does the
2. Pupillary constriction, constipation, and nurse appropriately ask the friend at this time?
sleepiness. 1. Does he have diabetes?
2. Does he have epilepsy?

746 NCLEX-RN Review


53155_09_PT 03_p744-764.qxd 2/26/09 8:13 AM Page 747

3. Did he hit his head? nursing strategies should be included in the plan
4. Did he use drugs? of care to reduce the clients edema?
1. Establishing limits on activity.
28. An adult client has been passing blood in his 2. Fostering a relaxed environment.
feces. What would be the best way for the nurse
3. Identifying goals for self-care.
to assess whether the hematochezia is a
symptom of gastric bleeding? 4. Restricting IV fluids.
1. Ask him how long he has been bleeding. 33. An elderly woman admitted with congestive
2. Check his vital signs. heart failure and +34 peripheral edema
3. Monitor his laboratory results. complains that she is always tired. Which of the
4. Obtain his complete past medical history. following would be the most appropriate
suggestion by the nurse while the client is still
29. A construction worker complains of low back on bed rest?
pain that increases when he bends over, coughs, 1. Continue to exercise your legs.
or lifts objects. A diagnosis of herniated disc is 2. Try not to think about the fatigue.
made. When asked about the cause of his pain,
3. Eat larger meals.
the nurses response is based on the knowledge
that pain associated with a herniated disc results 4. Sleep as much as possible.
from what?
34. An adult is admitted with early left-sided
1. Compression of the spinal nerve root. congestive failure. Which symptom should the
2. Spasms of the paraspinal muscles. nurse expect to find?
3. A friction rub created by degeneration of 1. Bradycardia.
vertebrae. 2. Rales.
4. Edema and swelling of nerve endings. 3. Liver engorgement.
30. The nurse is caring for a client who is being 4. Jugular vein distention.
transfused for severe gastrointestinal bleeding.
35. An adult is given digoxin (Lanoxin) 0.25 mg
How can the nurse decrease the danger of
daily. What signs of digitalis toxicity would the
hypothermia?
nurse provide to the client?
1. Administering blood with normal saline.
1. Auditory hallucinations and
2. Administering blood products through a bradycardia.
central line.
2. Dry mucous membranes and diarrhea.
3. Giving only packed cells.
3. Heart block and brittle hair and nails.
4. Warming blood to body temperature before
4. Visual disturbances and premature
administering.
heartbeats.
31. A client complains of a sudden onset of pain in
36. Which serum potassium level reported for an
the ankle, which is swollen, red, and extremely
adult requires no immediate nursing
sensitive to pressure. The client asks the nurse
intervention?
about gout. What explanation will the nurse
provide about gout? 1. 3.2 mEq/liter.
1. A metabolic disorder that results in elevated 2. 4.0 mEq/liter.
serum uric acid levels. 3. 5.7 mEq/liter.
2. An infection of the synovial membrane by 4. 6.0 mEq/liter.
microorganisms, resulting in inflammation.
37. The mother of a newborn learns that her infant
3. A disease of cartilage resulting in destruction
son has lost 8 ounces since his birth 2 days ago.
of the cartilage and the underlying bone,
The nurse explains that this weight loss is
causing severe pain.
normal. What explanation will the nurse provide
4. Inflammation of the bursal sac accompanied for the weight loss result?
by formation of large calcium deposits, which
1. Feeding infants every 4 hours instead of
cause swelling and joint pain.
every 3 hours.
32. A client is admitted to the hospital with 2. Loss of fluid from the cord stump.
congestive heart failure. She has shortness of 3. Limited food intake since birth.
breath and a +34 peripheral edema. What 4. Regurgitation of feedings.

COMPREHENSIVE PRACTICE TESTS 747


53155_09_PT 03_p744-764.qxd 2/26/09 8:13 AM Page 748

38. A 16-year-old Type 1 diabetic takes his morning 3. Birth of the baby and delivery of the placenta.
dose of insulin and leaves for school. At 10 A.M. 4. Readjustment to the nonpregnant state.
he feels faint and is brought to the nurses office.
He has tachycardia and diaphoresis and is 44. A female woman is diagnosed with somatization
unresponsive. What would be the appropriate disorder. She experiences palpitations, nausea,
intervention by the nurse at this time? abdominal pain, and headaches. Physical exam
1. 5 units regular insulin SC. and diagnostic tests do not reveal pathology. The
2. 8 ounces of orange juice. client comes to the nurse stating she has
palpitations. Which plan of care by the nurse
3. Glucagon SC.
best reduces secondary gain?
4. Glucose 50% IV push.
1. After investigation of her palpitations, do not
39. A client is given discharge instructions following continue to take her vital signs with each
a thoracotomy. Which of the following would be complaint of the problem.
included in the teaching? 2. Inform her that the palpitations are not real
1. Cough when necessary. and she must learn to relax.
2. Keep arm of affected side in sling. 3. Convey an intense interest in her palpitations
by encouraging her to talk about her
3. Remove dressing from groin area after 24 hours.
symptoms.
4. Explain the use of a Passy-muir valve.
4. Reassure her that she will be assisted in
40. The nurse in a well-baby clinic is assessing a meeting her dependency needs.
12-month-old child. He is 30 inches tall and weighs
45. The nurse is assessing a client with borderline
30 lb. How does the nurse interpret this data?
personality disorder. What behavior will the
1. Normal height, increased weight. nurse assess for?
2. Normal height, decreased weight. 1. Aggression.
3. Small for age, normal weight. 2. Depression.
4. Tall for age, but weight appropriate for height. 3. Sleep disturbances.
41. The nurse has been discussing promotion of 4. Splitting.
growth and development with a family whose
46. An elderly woman is addicted to pain killers
15-month-old son has a cyanotic heart defect.
prescribed for her arthritis. She denies it is a
Which statement by the father indicates a need
problem because it was prescribed by her
for further teaching?
physician. What is the best interpretation of the
1. I need to feed him slowly and allow clients view of her addiction?
frequent rest periods.
1. The client is using rationalization to support
2. I need to play quiet games and activities her denial.
with my son.
2. The client really does not have a problem; it
3. I need to provide highly nutritious foods. is the physician who does.
4. I need to limit my sons interactions with 3. The client is transferring blame by denying
other children. she has a problem.
42. A child with hemophilia cut his hand while 4. The client is an uneducated woman so she
working on a craft project in the hospital play couldnt understand her problem.
room. What will be the nurses initial action?
47. An adult is prepared for discharge following a
1. Apply pressure to the bleeding area for at bilateral adrenalectomy. Which statement by the
least 10 to 15 minutes. client demonstrates understanding of the
2. Apply an ice pack. discharge teaching?
3. Cover the wound with a sterile dressing. 1. The surgery cured my disease, now I wont
4. Notify the physician immediately. have to take any medications.
2. I should wear a Medic Alert bracelet or
43. The nurse is caring for a woman in the fourth necklace at all times.
stage of labor. What should the woman be
3. I will need to take replacement doses of
monitored for?
steroids daily for 1 to 2 months.
1. Uterine contractions.
4. I will probably develop a round face and gain
2. Cervical dilation. weight now that I will take cortisol daily.

748 NCLEX-RN Review


53155_09_PT 03_p744-764.qxd 2/26/09 8:13 AM Page 749

48. Sandra, an RN, reports to work looking 53. The nurse is caring for a client with severe back
unkempt. Nancy, another RN, approaches when strain. The nurse administers diazepam (Valium)
she notices her using uncoordinated movements. 10 mg QID. Which observation is most
Sandras breath reeks of peppermints and Nancy indicative of the need to reassess this order?
suspects Sandra may be intoxicated. What is the 1. Drowsiness.
best initial nursing action for Nancy to take? 2. Hyperesthesia of the arms.
1. Call the supervisor and report Sandra. 3. Loss of appetite.
2. Confront Sandra with the concerns and 4. Severe muscle spasms.
relieve her of her nursing duties immediately.
3. Ignore the situation. 54. An adult has been receiving physical therapy
4. Give Sandra a lecture about substance abuse following a cerebrovascular accident. His left leg
and do nothing else. is weak and he is instructed in the use of a cane.
What documentation by the nurse shows the
49. An adult is experiencing a panic attack. The clients ability to use the cane correctly?
nurse intervenes by escorting him to his room, 1. Holds the cane in his left hand.
using short sentences, and conveying a calm 2. Leans his body toward the cane when
demeanor. Which action by the client indicates walking.
the nursing interventions are effective?
3. Advances the left leg and cane
1. Releases his anxiety by punching his fist on a simultaneously.
bedside table.
4. Advances the right leg and cane
2. States he wants to be alone to deal with his simultaneously.
feelings.
3. Expresses verbally his demands to the nurse. 55. The nurse is caring for a client who is
4. Makes connections between events and his hypertensive. To facilitate the clients ability to
anxious response. lower his blood pressure to a normal range, the
nurse should teach him to avoid which of the
50. A young adult is readmitted to the rehabilitation following foods?
unit after a T4 spinal cord injury that occurred 1. Cooked cereal.
6 months ago. He is about to begin an intensive 2. Broccoli.
rehabilitation program. Which of the following
3. Catsup.
statements made by this client best indicates
that he understands the extent of his injury? 4. Sugar.
1. I want to use an electric wheelchair. 56. An adult client has hypertension. The nurse
2. My goal is to be independent in transfers. takes his blood pressure in lying and standing
3. Soon Ill be walking. positions. What condition is this test used for?
4. There is little I can do, but I will try. 1. Central nervous system depression.
2. Malignant hypertension.
51. The nurse is caring for an adult with a T4 spinal
3. Orthostatic hypotension.
cord transection. Which activity by the client
indicates adequate learning regarding urinary 4. Vascular insufficiency.
tract care?
57. The nurse is formulating a teaching plan for an
1. Avoiding the Valsalva maneuver when the adult client with severe emphysema. What are
bladder is full. the recommended activities the nurse should
2. Cleaning the urinary meatus every 2 hours. instruct the client to select?
3. Checking the bladder distention frequently. 1. Avoid movement.
4. Limiting fluids to 100 mL per 24 hours. 2. Build strength.
3. Conserve energy.
52. To evaluate the effectiveness of bowel training
for an adult with a T6 spinal cord injury, what 4. Test his limits of tolerance.
should the nurse expect him to be able to do?
58. What is an appropriate expected outcome for a
1. Avoid laxatives and stool softeners. client with chronic obstructive lung disease
2. Experience no incontinence. (COPD)?
3. Move his bowels daily. 1. Deemphasizes expirations.
4. Resume previous bowel habits. 2. Increases his respiratory rate.

COMPREHENSIVE PRACTICE TESTS 749


53155_09_PT 03_p744-764.qxd 2/26/09 8:13 AM Page 750

3. Reduces the use of his diaphragm. 63. A high school student with a history of sexual
4. Utilizes abdominal breathing. abuse was admitted to the psychiatric unit
experiencing depersonalization. What is an
59. An adult is hospitalized for treatment of deep appropriate short-term goal for the nurse?
electrical burns. Burn wound sepsis develops 1. Help the client develop coping skills.
and mafenide acetate 10% (Sulfamylon) is 2. Orient the client to the staff and unit.
ordered BID. What physiological response will
3. Place the client on q 15 minute checks.
the nurse inform the client to expect from the
topical application? 4. Teach the client about her medications.
1. Severe burning pain for a few minutes 64. A delusional client is admitted to the hospital.
following application. What is the most appropriate action for the
2. Possible severe metabolic alkalosis with nurse to take?
continued use. 1. Attempt to disprove the clients delusion.
3. Black discoloration of everything that comes 2. Focus on the reality aspects of the clients
in contact with this drug. communication.
4. Chilling due to evaporation of solution from 3. Place the client on room restriction to
the moistened dressings. decrease stimuli.
60. A client is admitted to the burn unit with partial 4. Agree with the delusion until psychotropic
and full thickness burns of both legs, which medications take effect, then focus on reality.
occurred when a charcoal grill tipped over on 65. The nurse evaluates a delusional client for
her. Blister formation and a large amount of fluid improvement. Which of the following statements
exudate is noted. Urine output is 30 mL/hr, BP indicates a positive outcome for a delusional
90/60, and pulse 110. What is the primary client?
nursing diagnosis during the initial 4872 hours
1. Client states he hears voices, but only when
following the burn?
alone.
1. Body image disturbance related to disfiguring
2. Client states people are observing him but are
burns of both legs.
not talking about him.
2. High risk for infection related to skin
3. Client expresses less fear in using the public
breakdown.
phone on the hospital unit.
3. Potential for ineffective airway clearance
4. Client states he can now use the unit shower
related to smoke inhalation.
room because he realizes the shoe left by
4. Fluid volume deficit related to increased another client is not a rat.
capillary permeability.
66. A child who is 2 years and 6 months old has had
61. While assessing the client with burns on the one bout of nephrosis (nephrotic syndrome). His
back and trunk, the nurse notes areas that are mother suspected a recurrence when she
not painful, grayish-white in color, and leathery observed swelling around his eyes. The nurse
in appearance. What type of burns will the nurse helps to confirm this condition by recognizing
document? what additional symptom?
1. Superficial burns. 1. Blood pressure of 140/90.
2. Superficial partial thickness burns. 2. Marked proteinuria.
3. Deep partial thickness burns. 3. Cola-colored urine.
4. Full thickness burns. 4. A history of positive streptococcal
infection.
62. A middle-aged woman has no memory of her past.
She assumed the name Blanche as she created a 67. The nurse is evaluating a child who is being
new identity during her hospital stay. Why does treated for nephrosis. Which observations
the nurse feel it is important for her to do this? indicate successful treatment of nephrosis?
1. It decreased the clients anxiety level. 1. Diuresis and weight loss.
2. All people need a name and a history. 2. Improved appetite and weight gain.
3. The hospital needs to have a name for its 3. Increase in the sedimentation rate and urine
records for payment purposes. specific gravity.
4. It increased the clients self-esteem by 4. Return of temperature to normal and
developing a possible self. indications that the child is more comfortable.

750 NCLEX-RN Review


53155_09_PT 03_p744-764.qxd 2/26/09 8:13 AM Page 751

68. The mother of a 2-year-old tells the nurse that color pink with tremors and irritability. What is
her son has temper tantrums, demanding the best nursing intervention at this time?
cookies in the supermarket, and asks how she 1. Feed the infant 5% dextrose water via gavage.
can best handle these temper tantrums. What 2. Take the infant to the mother to feed
suggestion should the nurse give to the immediately.
mother?
3. Check the infants blood sugar.
1. Buy one box of cookies for each shopping
4. Ask a coworker to call the pediatrician
trip.
immediately.
2. Leave him home while she goes shopping.
3. Remain calm and ignore his behavior. 73. When caring for a client with a casted extremity,
4. Discipline the child immediately when he frequent assessments of neurologic and
demands cookies. circulatory status of the affected extremity are
required. Which of the following assessment
69. A young woman is in her fifth month of findings should be recognized by the nurse as
pregnancy. She has been taking 20 units of NPH abnormal?
insulin for diabetes mellitus daily for 6 years. 1. Client reports the extremity feels like its
Which of the following statements indicates that asleep.
the woman understands the teaching regarding 2. Capillary refill time is less than 5 seconds.
her insulin needs during her pregnancy?
3. The area distal to the cast is warm to
1. Are you sure all this insulin wont hurt my touch.
baby?
4. Client reports dull aching in the casted
2. Ill probably need my daily insulin dose extremity.
raised.
3. I will continue to take my regular dose of 74. An adult is receiving peritoneal dialysis. His
insulin. acid-base balance and electrolytes are now
4. These finger sticks make my hand sore. within normal limits. Which of the following
Can I do them less frequently? best explains the mechanism of action for
peritoneal dialysis?
70. A woman who delivered a healthy baby 18 hours 1. Hypotonic fluid is instilled into the
ago has just been given Rho (D) immune globulin. peritoneal cavity and waste products
Which finding indicates the need for passively diffuse into it.
administration of this medication? 2. Sodium and bicarbonate from the dialysate
1. The mother is Rho (D) negative with Rho (D) in the peritoneal cavity are exchanged for
antibodies. excess potassium and hydrogen ions from
2. The infant is Rho (D) positive. the blood.
3. There is a positive indirect Coombs test of 3. Increased intra-abdominal pressure caused by
cord blood. the dialysate solution in the abdomen creates
4. The mother is Rho (D) positive. a filtration pressure similar to that in the
kidney, causing wastes and electrolytes to
71. What would the nurse expect when evaluating move out of the blood.
the effectiveness of IV Pitocin for a client with 4. Glucose added to the dialysate solution
secondary dystocia (uterine inertia)? increases the osmotic pressure of the
1. A precipitate delivery. dialysate, causing fluid to move from the
2. Cervical effacement without delivery. blood into the dialysate along with wastes
3. Infrequent contractions lasting longer than and electrolytes.
90 seconds.
75. An adult with chronic renal failure is receiving
4. Progressive cervical dilation with peritoneal dialysis. His acid-base balance and
contractions lasting less than 90 seconds. electrolyte levels are now within normal limits.
His hemoglobin is 9.2 and his hematocrit is
72. A client with type 2 diabetes delivered a 3700-
30. What is the most likely cause for his
gram live girl via cesarean delivery for a breech
anemia?
position 15 minutes ago. She wishes to
breastfeed as soon as possible. The nurse caring 1. Hemodilution secondary to fluid retention.
for the infant makes the following assessments 2. Eating insufficient protein due to taste
of vital signs: T 98.8F, P 148, respirations 22, changes that occur with dialysis.

COMPREHENSIVE PRACTICE TESTS 751


53155_09_PT 03_p744-764.qxd 2/26/09 8:13 AM Page 752

3. Failure of his kidneys to produce the Inspect hair and skin several times a day
hormone necessary to stimulate bone marrow for ticks.
to produce red blood cells. Look for bites that have a fine petechial rash.
4. Hemolysis of red blood cells as they move
past the membrane containing the dialysis 81. An adult is scheduled for a liver biopsy. What
solution. should the nurse include when planning the
postprocedure care?
76. An adult is scheduled for an intravenous 1. Administering narcotic analgesics every 3 to
pyelogram (IVP). What action should the nurse 4 hours for the first 24 hours.
do before sending her to the test? 2. Positioning her on her right side for at least
1. Ask if she is allergic to barium. the first 2 hours.
2. Ask if she is allergic to shellfish. 3. Monitoring for pain referred to the left arm.
3. Give her a full glass of water. 4. Changing the dressing over the puncture site
4. Instruct her not to urinate until after the test. frequently, until bile leakage has stopped.

77. A child is recovering from chickenpox. At what 82. The nurse is evaluating whether nonprofessional
point will he be allowed to return to school? staff understand how to prevent transmission of
1. After he has been on antibiotics for 48 hours. HIV. Which of the following behaviors indicates
correct application of universal precautions?
2. After his temperature is normal and the
itching has subsided. 1. A lab technician rests his hand on the desk to
steady it while recapping the needle after
3. When all lesions are crusted and scabbed.
drawing blood.
4. When his skin is clear of all lesions.
2. An aide wears gloves to feed a helpless client.
78. A woman calls her neighbor, who is a nurse, to 3. An assistant puts on a mask and protective
say that her 5-year-old has had a stomach virus eye wear before assisting the nurse to suction
with vomiting for 24 hours. The doctor a tracheostomy.
recommended the child eat nothing for 4 to 6 4. A pregnant worker refuses to care for a client
hours. There has been no vomiting during that known to have AIDS.
time and now she wants to know what is best to
give him. What does the nurse recommend? 83. An adult is ready for discharge following
1. Broth and water. creation of a sigmoid colostomy. Which of the
following statements by the client should be
2. Flat ginger ale and tea.
evaluated by the nurse as an indication that
3. Jell-O and a soft boiled egg. he has understood his discharge instructions
4. Skim milk and dry toast. correctly?
1. I will irrigate the colostomy with tap water
79. The nurse is assessing a child who is admitted
every day.
with pyloric stenosis. Which of the following
findings is most likely to be reported/observed? 2. I can eat anything as long as I chew
thoroughly.
1. The child has greenish-yellow mucus-like
emesis that has a strong odor. 3. I will change the pouch every day.
2. The vomiting began gradually and then 4. I should not drink more than six glasses of
increased until there is no retention of fluid a day.
feedings.
84. The nurse is caring for a premature 33-week
3. The infant is content between feedings and baby girl who is 5 days old and weighs 2000
shows hesitancy to feed. grams. Which should be included in the nursing
4. There is a palpable lump in the epigastrum care plan?
directly under the xyphoid process. 1. Teach her parents how to do gavage feedings,
because the baby has no sucking reflex.
80. A family is planning a camping trip. The mother
calls the clinic to obtain information regarding 2. Allow the mother to breastfeed when she visits.
Lyme disease. Which information is appropriate 3. Inform the parents that the infant will need to
for the nurse to give? Select all that apply. stay in an isolette until she is discharged.
Wear long pants and long-sleeved shirts 4. Instruct the parents on how to take rectal
when hiking. temperatures.
Use a tick repellent.

752 NCLEX-RN Review


53155_09_PT 03_p744-764.qxd 2/26/09 8:13 AM Page 753

85. The mother of an infant who has had a cleft lip 90. An adult has received one unit of packed red
repair has been taught the postoperative care blood cells after sustaining severe trauma to his
needed. What does the nurse hope to see when legs with profuse bleeding. What action will the
evaluating this mothers understanding of this nurse perform to evaluate the transfusions
care? effectiveness?
1. Positioning the child on his abdomen to 1. Take his blood pressure.
facilitate drainage of oral secretions. 2. Auscultate lung sounds.
2. Comforting the child as soon as he starts to 3. Check hemoglobin and hematocrit results.
fuss, to prevent his crying. 4. Take his temperature.
3. Using a regular bottle nipple to feed the
infant in a semi-reclining position. 91. A client who underwent a right total hip
4. Cleaning the suture line with warm water replacement arrived on the nursing unit from the
and a washcloth once a day. post anesthesia care unit at 2 P.M. What should
be the initial assessment by the 3 to 11 nurse
86. The school nurse is assessing a child who has during initial rounds at change of shift?
fallen in the gymnasium. The child exhibits all 1. The dressing.
of the following. Which finding indicates to the 2. Urine output.
nurse that the child most likely has a fracture in
3. Circulation to the leg.
addition to soft tissue injury?
4. Breath sounds.
1. Localized swelling.
2. Abnormal motion. 92. An adult who had gastric surgery is passing
3. Ecchymosis. large amounts of blood via nasogastric suction.
4. Pain. Which finding, if assessed by the nurse, is an
early sign of shock?
87. The nurse is planning care for a 3-year-old child 1. Distended neck veins.
who has just returned to the unit following a 2. Rapid shallow breathing.
cardiac catheterization. The nurse should
3. Bradycardia.
include which of the following on the care plan?
4. Constricted pupils.
1. Monitor for response to general anesthetic.
2. Bed rest for 24 hours. 93. The nurse is administering tracheostomy care to
3. NPO for 12 hours. an adult. Which of the following should be
4. Observe for severe pain and medicate as included in the procedure?
needed. 1. Soaking the outer cannula with saline solution.
2. Performing the procedure utilizing medical
88. The school nurse is assisting a school teacher to asepsis.
understand the classroom capabilities of a child
3. Soaking the inner cannula in half-strength
with athetoid cerebral palsy. What action will
hydrogen peroxide solution.
the child most likely demonstrate?
4. Cutting a sterile gauze pad to place between
1. Exaggerated hyperactive reflexes.
the neck and the tracheostomy tube.
2. Normal intelligence levels.
3. Slow, worm-like, writhing movements. 94. Which of the following teachings should the
4. Unsteady gait and clumsy, uncoordinated nurse include when establishing a bowel training
upper extremity function. regimen for a client with chronic constipation?
1. Avoid laxatives.
89. A woman with severe pregnancy-induced 2. Decrease exercise.
hypertension was delivered 2 hours ago. Which
3. Increase the fiber content of your diet.
nursing action should be included in the plan of
care for her postpartum hospital stay? 4. Increase fluid intake 4500 to 5000 mL.
1. Continuing to monitor blood pressure, 95. Medicare will pay for a limited number of home
respirations, and reflexes. care visits for clients with hypertension. What
2. Encouraging frequent family visitors. must the nurse assess on an elderly hypertensive
3. Keeping her NPO. client on a regular basis?
4. Maintaining an IV access to the circulatory 1. Ability to ambulate.
system. 2. Dehydration.

COMPREHENSIVE PRACTICE TESTS 753


53155_09_PT 03_p744-764.qxd 2/26/09 8:13 AM Page 754

3. Effectiveness of medication. 1. Arrange for a volunteer to stay with the client


4. Awareness of advanced directives. during the day to provide for socialization
needs and to facilitate communication with
96. The clinic nurse is teaching the client about staff.
breast cancer. Which one of the following risk 2. Explain to the family that consistency in
factors for breast cancer should be assessed enforcing rules is important to prevent
regularly by the nurse? complaints from the families of other clients.
1. Dietary habits. 3. Suggest that the family visit in shifts during
2. Socioeconomic status. the normal visiting hours, because the client
3. Early menarche. needs to sleep at night.
4. Being over 30 years of age. 4. Make an exception to visiting regulations
because of the long-term nature of the clients
97. The home health nurse is visiting a 90-year-old recovery and the need for family support.
man who lives with his 88-year-old wife. He is
legally blind and suffered a broken hip 5 years 99. An adult client has congestive heart failure and
ago. He ambulates with difficulty with the aid of is receiving spironolactone (Aldactone). Which
a walker. What is the highest priority nursing of the following is least appropriate for the nurse
diagnosis? to teach the client about his medication?
1. Self-care deficit, toileting. 1. Use calcium-based salt substitutes.
2. Knowledge deficit regarding blindness. 2. Swelling and tenderness of the breasts may
3. High risk for injury. occur with long-term therapy.
4. Impaired adjustment. 3. Take the medicine in the morning if possible.
4. Include high-potassium foods in the diet to
98. A client with Guillain-Barr syndrome has been decrease the possibility of hypokalemia.
on a ventilator for 3 weeks and can communicate
only with eye blinks because of quadriplegia. 100. A young child is admitted to the hospital with
The intensive care nursing staff sometimes have Tay-Sachs disease. What order can the nurse
no time for this tedious communication process. anticipate that this child will be placed on?
The clients family comes infrequently because 1. Seizure precautions.
they run a family-owned restaurant that does not 2. A cooling blanket.
close until visiting hours are over. How should 3. Strict intake and output.
the nurse respond to the familys request for
4. Protective isolation.
exemption from visiting hours?

Answers and Rationales for Practice Test 3

ANSWER RATIONALE NP CN CL SA
#1. 1. Pain from a duodenal ulcer is often aching or burning in As Ph/7 An 1
character, and occurs when the stomach is empty. It is likely
to occur in the mid-upper abdomen, whereas pain in the
epigastric/shoulder area is characteristic of gallbladder disease
and pain in the midsternal area with cardiac problems.
#2. 2. The squatting position serves to decrease venous return by Im Ph/7 Ap 4
occluding the femoral vein through hip flexion, to lessen the
workload on the right side of the heart, and to increase arterial
oxygen saturation. Returning the child to bed or placing in a
chair would not lessen the hearts workload as the squatting
does. The physician would already be aware of this condition.

754 NCLEX-RN Review


53155_09_PT 03_p744-764.qxd 2/26/09 8:13 AM Page 755

ANSWER RATIONALE NP CN CL SA
#3. 3. The fibrosed liver obstructs flow through portal vessels, An Ph/8 An 1
which normally receive all blood circulating from the
gastrointestinal tract. The increased pressure in portal
vessels shunts some of the blood into the lower pressure
veins around the lower esophagus. Because these veins are
not designed to handle the high-pressure portal blood flow,
they develop varicosities, which often rupture and bleed.
#4. 4. Administration of blood and fluid is vital in maintaining Pl Ph/8 An 1
blood volume in a client with severe gastrointestinal
bleeding. Vital signs would need to be monitored more
frequently than hourly. A client with severe bleeding
would have decreased urine output, so the bedpan
would not be used frequently as with diuretics and the
gastric pH is not related to the clients fluid needs.
#5. 2. When going up steps the client must always provide for Im Ph/5 Ap 1
either the crutches or the unaffected leg to be bearing his
weight at all times. Moving the unaffected leg up first
allows the strong leg muscle of the unaffected leg to do
the lifting to raise him to the next step. The other choices
involve positions that would place the arms trying to lift the
body, cause the client to lean and be off balance or sitting
down which would be difficult to raise back up; all placing
the client at risk for falling.
#6. 2. Oxygenation is the most important immediate goal. Pl Ph/7 Ap 1
Remember the ABCs of client care. The other choices
are appropriate goals, but not as important as oxygenation.
#7. 3. Children ages 3 to 5 often think of death as sleep or a An He/3 An 4
departure. To them, death is reversible. Children ages
5 to 9 view death as irreversible and permanent; children
ages 7 to 10 view death as inevitable and final. Teenagers
view life in the present and can become angry at the
injustice of death.
#8. 4. To prevent reinfection, the entire prescribed antibiotic Im Ph/6 Ap 4
needs to be taken, with a course of treatment lasting
7 to 10 days. Acetaminophen is the drug of choice instead
of aspirin; heat helps to decrease pain.
#9. 3. The delivery of room air and oxygen will keep carbon dioxide Pl Ph/7 Ap 4
levels decreasing, preventing hypoxia. The humidity will
prevent the drying of the mucous membranes and subsequent
edema. The tent can be opened or even remain open with the
understanding that the treatment will not be as effective.
#10. 3. The nurse would be looking for objective signs which would As He/3 An 3
be a positive urine test. Morning sickness would be a subjective
sign; urinary frequency could also be a sign of a urinary tract
infection; and fetal heart tones would not be auscultated this
early.

ANSWERS AND RATIONALES FOR PRACTICE TEST 3 755


53155_09_PT 03_p744-764.qxd 2/26/09 8:13 AM Page 756

ANSWER RATIONALE NP CN CL SA
#11. March 22. Ngeles rule for calculating the estimated date As He/3 Ap 3
of confinement (EDC) or birth is to add 7 days to the first
day of the last menstrual period, subtract 3 months, and
add 1 year.
#12. 1. In pregnancy, constipation results from decreased gastric Pl He/3 Ap 3
motility and increased water reabsorption in the colon
caused by increased levels of progesterone. Prenatal
vitamins have iron in them, which also contributes to
constipation. The best instruction is to increase fluid
intake and to avoid laxatives as they may cause cramping.
#13. 4. Supportive stockings will help circulate blood flow back to Pl Ph/7 An 1
the heart because the nurse will be standing for most of the
shift. They will also help prevent varicose veins. It would
be beneficial to drink more than 1 liter for the entire shift;
elevation could be done at home, but not usually done on
a working shift; proper body mechanics would entail
pushing objects, not pulling them.
#14. 1. These signs and symptoms describe the transition phase An He/3 Co 3
of labor.
#15. 1. Gentle massage of the uterine fundus is indicated. The bladder, Im He/3 Ap 3
lochia, fundal firmness and placement should be assessed.
If uterine atony is not resolved, the physician or midwife
should be called for further evaluation.
#16. 4. There is risk of infection from saline abortion because saline An Ph/7 An 3
is injected into the amniotic sac. The woman also may labor
at length with ruptured membranes. The elevated temperature
and blood pressure are associated with infection.
#17. 3. Vaginal discharge can be from Chlamydia infection and can Im Ph/6 Ap 3
be seen in conjunction with other infections such as gonorrhea
or trichomonas. Cultures and blood studies are needed and
contacting a physician is necessary.
#18. 1. The discharge would also be foul-smelling, Choice 2 is As Ph/7 Co 3
associated with monilia infection; warts or ulcers are
usually seen in herpes simplex virus types I and II and
in syphilis.
#19. Nine (9). The baby gets 2 points for full flexion of the As He/3 Ap 3
extremities, 1 point for being acrocyanotic, 2 points for
heart rate, 2 points for respirations (full, lusty cry), and
2 points for resisting the suction catheter.
#20. 1. The most common side effect of epidural anesthesia is a Ev Ph/6 An 3
sudden drop in material blood pressure, which can
compromise fetal blood flow.
#21. 1. When a person with weakness on one side uses a cane, Ev Ph/5 An 1
there should always be two points of contact with the
floor. When the client moves the cane forward, she has
both feet on the floor. As she moves the weak leg, the
cane and the strong leg provide support. Finally, the
cane, which is even with the weak leg, provides stability
while she moves the strong leg.

756 NCLEX-RN Review


53155_09_PT 03_p744-764.qxd 2/26/09 8:13 AM Page 757

ANSWER RATIONALE NP CN CL SA
#22. 4. Supine position would provide less room for lung expansion Pl Ph/7 Co 1
and cause increased pressure of abdominal organs. The
most beneficial position would be semi-Fowlers to allow
for lung expansion.
#23. 1. These are characteristics seen with someone taking As Ps/4 An 2
hallucinogens. The other choices are characteristic of
opioid use, Parkinsons, and PCP, respectively.
#24. 3. The cohesiveness is apparent in the group. Joining or An Ps/4 An 2
leaving the group results in strong emotions due to
disruption of the sharing group. In the beginning phase,
members are getting to know each other, transition is not
a phase in group process; termination phase may bring
various emotions, but not what is described.
#25. 3. Denial is the blocking out of thoughts or feelings perceived As Ps/4 Co 1
as painful. Projection is blaming others; repression is a
forgotten memory; and displacement is the expression of
emotion on to another person or object.
#26. 3. Suicide attempts are more common on evenings, night shift Im Ps/4 Ap 2
or weekends when the unit structure is lessened. The client
feels threatened from her husbands actions and is
expressing tunnel vision in regards to her situation.
The safety of the client is the first concern and all suicidal
remarks and gestures must be taken seriously. Therapeutic
sessions will be held at a later time.
#27. 1. All the findings suggest diabetic ketoacidosis. However, An Ph/8 An 4
a negative answer does not rule out diabetes. The other
choices would include a seizure noted; or manifestations
of increased intracranial pressure (slow, labored respirations,
bradycardia, pupillary dysfunction, changes in motor
function); a drug overdose would present with shallow
respiration, constricted pupils, and circulatory collapse.
#28. 4. Hematochezia (blood in the stool) may come from a source As He/3 An 1
in either the upper or lower GI tract. The clients past
medial history will aid in determining the location of
the bleeding, as the others will not suggest a location.
#29. 1. In herniation of the disc, the nucleus of the disc protrudes An Ph/8 Co 1
into the annulus, which causes pain in the nerve distribution.
Paraspinal muscles are next to the spine and spasms will
cause the muscles to tighten up, causing a painful, burning
sensation; the third choice is related to arthritis of the spine
in which pain comes and goes; edema and swelling occur
after a spinal cord injury.
#30. 4. Hypothermia with cardiac arrhythmias may occur when Im Ph/6 Ap 1
infusing the large quantities needed in GI bleeding. Blood
warming equipment should be used to prevent this problem.
#31. 1. Gout, or gouty arthritis, is a systemic disease in which urate An Ph/7 Co 1
crystals are deposited in joints and other body tissues.

ANSWERS AND RATIONALES FOR PRACTICE TEST 3 757


53155_09_PT 03_p744-764.qxd 2/26/09 8:13 AM Page 758

ANSWER RATIONALE NP CN CL SA
Elevated uric acid levels occur as a result of improper
metabolism of purines, resulting in excessive production of
uric acid, which the kidneys are unable to adequately eliminate.
It is not an infection; osteoarthritis is a destruction of bone
and cartilage; bursitis is an inflammation of the bursa and
is usually the result of trauma or strain to the joint.
#32. 4. Fluid restrictions should be implemented to reduce excess Pl Ph/7 Ap 1
vascular volume. The other activities do not affect the
edematous state.
#33. 1. Active and passive leg exercises are important with clients As Ph/7 An 1
on bed rest as a method of preventing thrombophlebitis. It will
be therapeutic to allow the client to verbalize her feelings
about fatigue or other problems associated with the disease
process.
#34. 2. Left-sided failure caused by ventricular dysfunction, results As Ph/8 An 1
in increased pressure in the pulmonary veins, which leads to
the development of rales. Choices 3 and 4 are associated with
right-sided heart failure, and tachycardia would be present
in both.
#35. 4. Digitalis toxicity would include nausea/vomiting, irregular Ev Ph/6 Ap 5
pulse, diarrhea, and yellow vision.
#36. 2. Normal serum potassium levels are 3.55.3 mEq/L. Immediate An Ph/7 An 1
interventions are required for all the other levels and the
physician should be notified. The client also should be placed
on cardiac monitoring.
#37. 3. Weight loss occurs through excessive extracellular fluid loss, An He/3 An 3
meconium loss, and limited food intake. Infants take in small
amounts of feedings and energy expenditure exceeds intake.
#38. 3. Glucagon is the drug of choice in the treatment of hypoglycemia Im Ph/6 Ap 4
due to excess insulin when the client cannot safely take
glucose by mouth. Glucagon begins to raise the blood sugar
within 5 minutes, whereby raising the level of consciousness
to allow the client to eat carbohydrates.
#39. 1. Coughing is necessary to move retained secretions. A sling is Im Ph/8 Ap 4
not necessary, whereas arm and shoulder exercises are to be
performed to regain previous range of motion; the dressing
will be on the thoracic area and not the groin (as in a cardiac
catherization); a Passy-muir is an assistive device used after a
tracheostomy for speaking purposes.
#40. 1. Normal height is 2932 inches; normal weight is between An He/3 An 4
19 and 27 pounds.
#41. 4. The parents should be encouraged to foster normal socialization Ev He/3 An 4
of their child. Parents may need additional information regarding
fears they may have. Resting periods while feeding and engaging
in quiet activities will reduce energy expenditure; a good diet
will foster growth and development.

758 NCLEX-RN Review


53155_09_PT 03_p744-764.qxd 2/26/09 8:13 AM Page 759

ANSWER RATIONALE NP CN CL SA
#42. 1. Applying pressure to allow for clot formation is the initial Im Ph/8 Ap 4
action.
#43. 4. The fourth stage of labor is the first hour or two after delivery Pl He/3 An 3
and is a critical period for maternal systems to stabilize after
giving birth. The other choices are in various other stages of labor.
#44. 1. The plan is to reduce secondary gain, which is the avoidance Pl Ps/4 Ap 2
of an unpleasant activity. Focusing on symptoms only promotes
secondary gain. Somatic symptoms are perceived as real to the
client and not under voluntary control. Goals would be to show
an interest in the client rather than symptoms and to foster
independence.
#45. 4. Splitting is the primitive defense mechanism, seen in clients As Ps/4 An 2
with borderline personality disorder, that presents as an
inability to integrate both good and bad aspects of self and
others into an integrated whole. This results in both an
idealization and a devaluation of others and self.
#46. 1. Denial is often used for maintaining self-esteem when control An Ps/4 An 2
is lost. The physician should have been monitoring the
situation closer to prevent its occurrence.
#47. 2. The Medic Alert bracelet is essential to warn health care Ev Ph/7 An 1
providers that his adrenals have been removed and that
glucocorticoid and mineralocorticoid replacement is
essential for life. Failure to supply replacement doses will
precipitate severe hypotension, shock, coma, and vasomotor
collapse. Careful adjustment of the replacement hormones
can prevent the moon face, weight gain, and edema that
is associated with steroid use.
#48. 2. Sandra needs to be relieved from her duties, as client safety Im Sa/1 Ap 1
is the primary concern. Notifying the supervisor will be
the secondary measure, as the supervisor may not be
available right away. Ignoring the situation is against the
professional code of conduct for nurses and Sandra
would not benefit from a lecture in her condition.
#49. 4. With reduced levels of anxiety, the clients perceptual Ev Ps/4 Ap 2
field broadens, allowing the client to focus on the cause
of anxiety and to connect the cause with his anxious
response. He is able to learn from the experience. High
levels of anxiety, as expressed in the other choices,
prevent this from occurring.
#50. 2. Clients with a T4 injury will have sufficient upper As Ph/7 An 1
extremity strength to master the technique of
independent transfer and not need an electric wheelchair.
Given the level of the spinal injury, he will not be
walking, but will still be able to do a great deal.
#51. 3. Bladder distention may cause urinary tract infections, Im Ph/7 Ap 1
distention of the ureters and renal pelvis, and autonomic
dysreflexia. Checking for bladder distention should be

ANSWERS AND RATIONALES FOR PRACTICE TEST 3 759


53155_09_PT 03_p744-764.qxd 2/26/09 8:13 AM Page 760

ANSWER RATIONALE NP CN CL SA
done every 34 hours and assessing whether intermittent
self-catherization is required, especially if this is on a
scheduled basis. The Valsalva maneuver can help to expel
urine, but is dangerous for clients with cardiovascular disease.
Cleaning is only necessary 23 times a day and fluid intake
of 2500 mL/day is encouraged.
#52. 2. The goal with this client is to prevent incontinence by having Ev Ph/7 An 1
the client control defecation. The client should not need
laxatives, nor expect a bowel movement every day.
#53. 4. The muscle spasm indicates that the Valium is not effective Ev Ph/6 An 1
as a muscle relaxant. Choices 1 and 3 are possible side effects
of Valium. Hyperesthesia (sensitivity to touch or painful
stimuli) is not seen with Valium administration.
#54. 3. The cane should be held in the hand opposite the affected leg Ev Ph/5 An 1
and should be advanced at the same time as the weak leg is
advanced to maximize support.
#55. 3. Catsup, like all canned tomato products, is high in sodium Pl Ph/5 Ap 1
and should be avoided. The cooked cereals are low in
sodium; broccoli is high in vitamins A, K, calcium,
and fiber and should be eaten regularly; sugar may be
restricted if weight reduction is desired or diabetes is
present.
#56. 3. A decrease in systolic blood pressure when the client moves Im He/3 Co 1
from a lying to a standing position is evaluated and will
result in the client reporting dizziness (also known as
postural hypotension). A neurological exam would be
performed for CNS depression; malignant HTN is
characterized by elevated BP in both standing and lying
positions; vascular insufficiency involves occlusion of
vessels with atherosclerotic plaques.
#57. 3. The client must work hard to breathe, so the plan of care Pl Ph/7 Ap 1
should structure a balance between rest and activity.
#58. 4. Abdominal breathing elevates the diaphragm, thereby Ev Ph/7 An 1
improving breathing effectiveness in clients with COPD.
#59. 1. Because of the burning pain upon application, an analgesic Im Ph/6 Co 1
may be required before ointment application. Sulfamylon
is a strong carbonic anhydrase inhibitor that affects the renal
tubular buffering system, resulting in metabolic acidosis.
#60. 4. Fluids are a primary intervention to replace lost fluids and An Ph/8 An 1
prevent irreversible shock. The critical hours after the burn
are characterized by a rapid shift of fluid from the vascular
compartment into interstitial spaces. As the burns are located
on the lower extremities, the client is not at risk for
pulmonary inhalation problems. After the primary goals are
met, then care of the wounds and infection begins, usually
4872 hours once the client has stabilized. Dealing with
the psychological aspects of disfiguring of a burn is a
long-term goal.

760 NCLEX-RN Review


53155_09_PT 03_p744-764.qxd 2/26/09 8:13 AM Page 761

ANSWER RATIONALE NP CN CL SA
#61. 4. The epidermis and dermis are destroyed in full thickness An Ph/8 An 1
burns. Because the nerve endings are destroyed, there is
an absence of pain.
#62. 4. By developing an identity the client is able to reckon An Ps/4 An 2
with negative and positive feelings to establish self-esteem.
This should provide motivation to cope.
#63. 2. The client experiencing depersonalization sees herself as Pl Ps/4 Ap 2
changed or the situation as unreal. It is essential to orient
the client to the unit to create a sense of reality and
security in her environment.
#64. 2. Delusions are fixed false beliefs. The nurse focuses on Im Ps/4 Ap 2
reality aspects of communication in an effort to promote
health rather than focus on delusions, which could become
further entrenched. The nurse should not disprove or agree
with the delusion.
#65. 2. The intensity of the ideas of reference has diminished, Ev Ps/4 An 2
showing improvement in the clients delusional thinking.
The other choices reflect auditory hallucination, phobia,
and illusions.
#66. 2. In nephritic syndrome, plasma proteins are excreted in the As Ph/8 An 4
urine due to an abnormal permeability of the glomerular
basement membrane of the kidney to protein molecules,
particularly albumin. The cause of nephrosis is unknown,
with the average age of onset at 212 years, more commonly
in boys than girls. Blood pressure and dark urine is not
associated in nephritic syndrome. A history of strep
infection is associated with glomerulonephritis.
#67. 1. The primary goal in the treatment of nephritic syndrome is Ev Ph/8 An 4
to control edema. Diuretics are used to promote diuresis and
subsequent weight loss. Corticosteroids are also given.
A decrease will be seen in the sedimentation rate and urine
specific gravity; temperature elevations are not common.
#68. 3. The best technique for handling temper tantrums includes Im He/3 Ap 4
being consistent, remaining calm, and ignoring the behavior.
It is advisable to explain to the child how to act in the store
before entering and to have one or more trial runs in
educating the child how to behave in a public place.
#69. 2. As a result of placenta maturation and placental production Ev Ph/6 An 3
of lactogen, insulin requirements begin increasing in the 2nd
trimester and may double or even quadruple by the end of
pregnancy. Newer glucometers allow blood glucose
to be taken from other areas beside the fingers only.
#70. 2. Rho (D) immune globulin is given to prevent maternal An Ph/6 An 3
sensitization by promoting destruction of Rh positive red
blood cells circulating in the mothers bloodstream. Two
of the criteria for administration of the Rho (D) immune
globulin are: Rho (D) negative mother without Rh antibodies
(nonsensitized) and an Rho (D) positive infant.

ANSWERS AND RATIONALES FOR PRACTICE TEST 3 761


53155_09_PT 03_p744-764.qxd 2/26/09 8:13 AM Page 762

ANSWER RATIONALE NP CN CL SA
#71. 4. Intravenous Pitocin should produce progressive cervical Ev Ph/6 An 5
dilation with contractions lasting no longer than 90 seconds.
Longer contractions may be dangerous to the unborn baby.
#72. 3. The infant has signs of hypoglycemia: tremors, irritability, Im Ph/7 Ap 3
and a decreased respiratory rate. Checking the blood sugar
is necessary to determine whether hypoglycemia is the
problem before any other interventions are done.
#73. 1. Paresthesias, such as numbness or tingling occur when Ev Ph/7 An 1
compression of the tissues deprives the nerves of part of
the circulation or when something presses directly on the
nerve. The capillary refill is normally 5 seconds or less;
dull aching would be expected.
#74. 4. This is the correct explanation of peritoneal dialysis (PD), An Ph/8 An 1
in which it removes toxic substances from the body. PD can
be dangerous and may cause death if not done with adequate
supervision of body fluid and electrolyte balance.
#75. 3. In chronic renal failure the hormone, erthythropoietin, is not An Ph/8 An 1
produced, which stimulates red blood cell production.
The trade name is Epogen. Hemodilution can produce a
drop in hematocrit, however sodium would also be lower,
which it is not in this case. Renal clients need to monitor
their protein intake. Hemolysis does not occur as RBCs
do not move outside the clients own blood vessels.
#76. 2. Dye is injected intravenously, and it contains iodine. Allergy As Ph/7 Ap 1
to shellfish often reflects iodine allergy and would place the
client at high risk. If this is the case, the physician would
need to be notified for further orders.
#77. 3. Once chickenpox lesions are crusted over, they are no longer Ev Sa/2 Ap 4
infectious. Antibiotics are not ordered.
#78. 2. These are well tolerated and not irritating to the gastrointestinal Pl Ph/5 Ap 4
tract after a stomach virus. Broth could be irritating and the
other choices are harder to digest.
#79. 2. Although there is variability in the pattern and type of vomiting, As Ph/8 An 4
it usually starts gradually, rather than suddenly and becomes
more projectile (1 to 4 feet away) and more frequent with the
tightening and further obstruction of the pyloric channel. It takes
about 4 to 6 weeks for complete obstruction to occur. Greenish-
color emesis is indicative of an obstruction below the stomach
level; an infant will want to feed after a vomitus episode.
#80. 1,2,4 All are correct except a Lyme disease rash is characterized Pl He/3 Ap 4
by a papule with a circular border, known as the bulls
eye rash.
#81. 2. The client will need to be positioned on her right side for the Pl Ph/7 Ap 1
first 2 hours or longer, to put pressure on the liver. Pressure
will decrease the bleeding from the very vascular liver and
also reduce leakage of bile into the peritoneal cavity (which
would not be present on the outside of the body). Pain, which

762 NCLEX-RN Review


53155_09_PT 03_p744-764.qxd 2/26/09 8:13 AM Page 763

ANSWER RATIONALE NP CN CL SA
may be referred to the right shoulder, should not persist
up to 24 hours, nor be severe enough to require narcotics.
The pressure dressing on the site should be not disturbed,
and bile leakage would be an abnormal finding.
#82. 3. Universal precautions will be utilized with all clients, and Ev Sa/2 Ap 1
always if there is a risk the transmission of blood and body
fluids. The other choices are inappropriate for the situation
given, unless blood/body fluids are going to be a transmission
risk.
#83. 2. There are no dietary restrictions with a colostomy, but high Ev Ph/7 Ap 1
flatulence foods may be limited due to the odor produced.
Irrigation should only be used as an enema would be used;
the pouch should last 3 to 5 days to prevent skin excoriation;
and fluid intake should be encouraged to prevent
constipation.
#84. 2. At 33 weeks, the infants sucking reflex is developed and Pl He/3 Ap 3
breastfeeding should be encouraged if this is the mothers
wish. The move to a crib is appropriate, rectal temps are
avoided due to the risk of perforation.
#85. 2. Crying pulls the edges of the suture line and may widen Ev Ph/7 Ap 4
the scar line. The baby should be prevented from crying as
much as possible by keeping the infants needs met and
providing postoperative analgesia. Prone position is avoided
as the infant can move the face back and forth on the bed,
putting tension on the sutures and Logan bar. Drainage
secretions are suctioned by a bulb syringe or placing the infant
on his side. Special nipples are available to allow closure of the
jaw without damaging the lip repair. Cleaning is performed
as a sterile procedure with the use of cotton applications
dipped in saline (as ordered).
#86. 2. Following a fracture or break in a bone, the extremity cannot As Ph/8 An 4
be used and tends to move unnaturally instead of remaining
rigid as it normally would. Swelling, ecchymosis, and pain
does not differentiate between a fracture or a soft tissue
injury, as they are present in all.
#87. 2. The prolonged bed rest is to prevent bleeding at catherization Pl Ph/7 Ap 4
insertion site. Mild sedation, not general anesthesia, is used;
fluids are encouraged to flush out the injected dye; pain
would not be an expected syndrome.
#88. 3. Athetoid cerebral palsy (CP) is characterized by involuntary, As Ph/7 Co 4
purposeless movements. Normal intelligence is common in
this type of disorder; hyperactive reflexes and unsteady gait
are seen with spastic CP.
#89. 1. Post delivery management of the mother includes close Pl He/3 Co 3
observation for BP elevation, CNS irritability (visitors are
limited), and respiratory function. The client is at risk for
seizure for 24 hours after delivery.

ANSWERS AND RATIONALES FOR PRACTICE TEST 3 763


53155_09_PT 03_p744-764.qxd 2/26/09 8:13 AM Page 764

ANSWER RATIONALE NP CN CL SA
#90. 3. Hemoglobin and hematocrit are expected to rise. All the Ev Ph/6 An 1
other choices are interventions that are performed
during the transfusion.
#91. 4. Respiratory function is always of prime importance in As Ph/7 Ap 1
assessing a postoperative client. The other choices are
essential also, after the respiratory assessment.
Post-operative clients are at risk for pneumonia and
pulmonary embolism.
#92. 2. Shock is a syndrome in which the peripheral blood As Ph/8 Ap 1
flow is inadequate to return sufficient blood to the heart
for normal function. The most outstanding symptoms
are skin paleness, cyanosis, staring of the eyes, pulse
weak and rapid, and rapid breathing rate is increased
and shallow.
#93. 3. Using a sterile technique, the inner cannula is removed Im Ph/7 Ap 1
utilizing sterile gauze and is soaked in the hydrogen
peroxide solution, cleaned with a small brush/pipe cleaner,
then rinsed with normal saline and dried. Outer cannulas
are not removed; gauze is not cut due to risk of filaments
working their way into the stoma.
#94. 3. Bowel training is to manipulate factors within the clients Im Ph/5 Ap 1
control (food, fluid, exercise, time for defecation) to
produce the elimination of a soft formed stool at regular
intervals. Chronic laxative use will create a dependency
on them.
#95. 3. In clients with cardiovascular disease, the effectiveness As He/3 Ap 1
of medications, as well as side effects, should be monitored.
This is the highest priority. The others are related to
management of hypertension.
#96. 1. Dietary habits is the only risk factor that can be assessed. As He/3 Ap 1
A high-fat diet put the woman at risk for breast cancer as
well as other cancer, so a high-fiber, low-fat diet is
recommended.
#97. 3. The fact that the client is legally blind and has difficulty An Sa/2 Ap 1
ambulating place him at extreme risk for injury.
#98. 4. Guillain-Barr syndrome is characterized by the onset of Im Sa/1 Ap 1
ascending paralysis, which may include respiratory muscles.
The client may be ventilator-dependent for weeks but may
have full consciousness. The prognosis is good but dependent
upon the level of supportive care during the acute stage.
#99. 4. Aldactone is a potassium sparing diuretic so dietary Pl Ph/6 An 5
potassium should be limited, not increased. Substitutes used
should contain calcium versus potassium; breast swelling
may occur with long-term therapy; morning administration
prevents sleep deprivation due to voiding.
#100. 1. Tay-Sachs disease is a degenerative neurologic disorder, which Pl Ph/7 An 4
is often characterized by seizures. The other choices are not
indicated for this disease.

764 NCLEX-RN Review


53155_09_PT 04_p765-785.qxd 2/27/09 6:30 PM Page 765

Practice Test 4

1. Which statement by the client would suggest the 6. A 4-year-old has been blind since birth. She has
client has hyperthyroidism? been attending a nursery program for the visually
1. I feel more nervous than usual. impaired. When her lunch tray arrives, what
2. I have had to wear a sweater all the action by the nurse will continue independence
time. in her ADLs?
3. My appetite has really been decreased 1. Offer to feed her.
lately. 2. Explain that foods on her tray are set up like
4. Should I be taking a laxative to prevent this a clock.
constipation? 3. Put food on her fork and hand her the fork.
4. Tell her that two foods are in front of her,
2. An 8-year-old is admitted with rheumatic fever. one at the top of the tray and one at the
Which clinical finding indicates to the nurse bottom.
that the client needs to continue taking the
salicylates he had received at home? 7. A nurse identifies that an infant displays
1. Chorea. the abduction, extension, and adduction of
2. Polyarthritis. arms to an embracing position when startled.
How would this finding be explained to the
3. Subcutaneous nodules.
parent?
4. Erythema marginatum.
1. This is a normal occurrence of the Babinski
3. The nurse is caring for a client with advanced reflex.
cancer of the breast. She complains of 2. Your child needs to see a neurologist.
hypoguesia. What recommendation should the 3. This is called the Moro reflex and
nurse give? disappears around 34 months.
1. Eating dry crackers. 4. Placing your child on his abdomen will help
2. Monitoring intake and output. reduce these twitches.
3. Using spices to enhance food flavors.
8. Discharge instructions are given to a woman
4. Weighing her before and after meals. who had been admitted with placenta previa.
Which statement by the client to her husband
4. An adult is admitted to the hospital to undergo a
best demonstrates she understands the
stapedectomy for the treatment of otosclerosis.
teaching?
Which findings elicited during physical
assessment are most indicative of 1. We cant have sex.
otosclerosis? 2. I have to return in a few days for a vaginal
1. Bone conduction is greater than air exam.
conduction. 3. I will have to have a cesarean for this and
2. Bone conduction is equal to air other pregnancies.
conduction. 4. I can go back to part-time work beginning
3. Air conduction is greater than bone tomorrow.
conduction.
9. Which of these statements would be appropriate
4. Sound lateralizes to the unaffected ear. for a nurse to give to a client who is scheduled
to have surgery in 15 minutes?
5. The nurse is caring for a client who has had a
stapedectomy. What will be appropriate 1. You need to remove your underwear
postoperative communication by the nursing now.
staff? 2. You may have sips of water for that dry
1. Overarticulate. mouth.
2. Shout in the affected ear. 3. Let me show you how to use an incentive
spirometer (IS).
3. Speak at a moderate rate.
4. How long have you smoked?
4. Use long, easily understood phrases.

COMPREHENSIVE PRACTICE TESTS 765


53155_09_PT 04_p765-785.qxd 2/27/09 6:30 PM Page 766

10. A young adult is involuntarily admitted to the 1. Roast chicken sandwich and ice cream cone.
psychiatric unit in a manic state. Upon arrival on 2. Roast beef sandwich and vanilla pudding.
the unit he is unable to sit, he is very difficult to 3. Fruit salad with cottage cheese and frozen
understand because of his rapid rate of speech, yogurt.
and he refuses to eat or drink. What area of
4. Bacon, lettuce, and tomato sandwich and an
disturbance poses the greatest physical danger to
apple.
this client?
1. Activity. 15. The nurse in the delivery room is caring for the
2. Perceptual. newborn. Which action is the most important
3. Sensory. and most immediate action for the nurse to take?
4. Social. 1. Do the Apgar score.
2. Dry the baby completely.
11. A young man was arrested by the police for 3. Place identification bracelets on the infant
indecent exposure, loitering, and disturbing the and the mother.
peace. He was also reported stripping off his
4. Prevent infection by doing eye care.
clothes at his mothers grave (who has been
deceased for 12 years). Upon admittance to the 16. The nurse caring for a mother who is
psychiatric unit, he was speaking rapidly, breastfeeding her full-term 2-day-old baby boy
refusing food or drink, and refusing to sit. Which instructed her on proper breast care this
nursing diagnosis would describe the behavior morning and wishes to evaluate her learning.
that is of greatest concern? Which of the following would demonstrate that
1. Anxiety. the mother has an adequate knowledge base?
2. Potential for violence. 1. She states she should not be concerned if
3. Spiritual distress. hard lumps develop in her breasts at home
4. Alteration in nutrition: less than body because engorgement may cause lumps.
requirements. 2. She states she will continue to feed the infant
as she has been, even if mild skin breakdown
12. A young woman with a history of bipolar disorder occurs on the nipple.
is admitted to the psychiatric unit. She is talking 3. She assesses her nipples carefully before and
excitedly and walking rapidly around the unit. after each feeding.
What intervention would most likely be initiated
4. She states she does not have to worry about
during the initial period of hospitalization?
good hand washing because her baby is not
1. Encourage the client to participate in group premature.
and therapeutic activities.
2. Observe the client closely until she calms 17. The nurse is caring for a 30-weeks gestation baby
down. girl who is currently receiving 15 mL of breast
3. Place the client in four-point restraints for milk via oral gastric tube every 3 hours. As part
protection of self and others. of the routine assessment the nurse should
4. Place the client in seclusion but maintain assess which of the following?
frequent one-to-one contact with her. 1. Assess for heme in the stool at each bowel
movement.
13. Which of the following is least likely to 2. Assess abdominal girth once every 3 days.
influence the potential for a client to comply 3. Assess for residual once per shift.
with lithium therapy after discharge?
4. Assess for tube placement once every
1. The impact of lithium on the clients energy 24 hours.
level and lifestyle.
2. The need for consistent blood level monitoring. 18. The nurse is caring for a 2-week-old baby who is
3. The potential side effects of lithium. showing clinical manifestations of heart
4. What the clients friends think of his need to murmur, widened pulse pressure, cardiomegaly,
take medication. bounding pulses, and tachycardia. The
assessment findings indicate that which of the
14. A teen who is 20 weeks pregnant has attended a following shunt systems from fetal circulation
prenatal nutrition course at her high school. has failed to close?
Which meal chosen by the female would warrant 1. Ductus venosus.
further instruction on proper protein intake? 2. Ductus arteriosus.

766 NCLEX-RN Review


53155_09_PT 04_p765-785.qxd 2/27/09 6:30 PM Page 767

3. Ligamentum arteriosum. the client to which of the following common


4. Foramen ovale. side effects?
1. Vertigo.
19. The nurse is caring for a 48-hour full-term infant 2. Skin rash.
whose mother abused cocaine and heroin
3. Tingling in the feet.
throughout pregnancy. The mother does not
wish to go into rehabilitation at this time. The 4. Orange-tinged body fluids.
nursing care plan should include which of the
24. The nurse is caring for an elderly woman who has
following?
been admitted to the hospital. The woman is
1. Feeding the infant whenever the infant cries upset and confused and repeatedly tells the nurse
or acts hungry. she is worried about being constipated while in
2. Allowing extra time to assist the woman with the hospital. Which question by the nurse would
breastfeeding and promote attachment. elicit information about the clients bowel status?
3. Organizing all necessary care around feeding 1. Do you realize you are confused?
times. 2. What laxative do you take at home?
4. Covering the baby loosely with a blanket to 3. When was your last bowel movement?
allow the body to cool down from the fever.
4. Why are you so worried about your bowels?
20. A mother brings her baby in for his 1-month
25. An elderly client has had Bucks traction applied
checkup. The mother reports that she has
while awaiting surgery for repair of a fractured
difficulty spreading the babys right leg when
left hip. Which intervention must be included in
she is diapering him. The nurse suspects a
the plan of care?
dislocated hip. Further assessment for the
possibility of a dislocated hip on the right side 1. Turn from side to side every 2 hours.
would include what observation? 2. Maintain high-Fowlers position.
1. Absence of Ortolanis sign. 3. Remove boot every shift.
2. Presence of Trendelenburgs sign. 4. Use footboard to position left foot.
3. An increase in skin folds on the unaffected
26. The nurse is caring for a client who has just
side.
returned to the nursing unit from the recovery
4. Shortening of the affected femur when supine room (postanesthesia care unit) after surgery.
with knees bent. What is the first action the nurse should perform?
21. A client is receiving intravenous (IV) therapy for 1. Take vital signs.
correction of a fluid volume deficit. Which 2. Administer pain medication.
observation by the nurse indicates infiltration of 3. Connect drainage systems.
the IV has occurred? 4. Check dressings.
1. Pallor at the infusion site.
2. Increased temperature at the infusion site. 27. An adult client received a kidney transplant 8
days ago. During this first period when an acute
3. Erythema around the infusion insertion
rejection of the kidney could occur, what is an
device.
essential assessment?
4. Seepage of blood around the infusion
1. Increased output of very dilute urine.
insertion device.
2. Hypotension.
22. An adult woman is admitted to an isolation unit 3. Anemia.
in the hospital after tuberculosis was detected 4. Fever.
during a pre-employment physical. Which
method would be responsible for the 28. What advice by the school nurse would ensure a
contamination? safe health care environment for the child with
1. Hands. impetigo?
2. Droplet nuclei. 1. You need to see your doctor this week to get
3. Milk products. started on some antibiotics so the sores on
your face will clear up.
4. Eating utensils.
2. Its O.K. to share personal items, such as
23. An adult is receiving rifampin (Rifadin). towels, with the rest of the family, because
Teaching by the nurse should include alerting good handwashing will prevent the spread of
impetigo.

COMPREHENSIVE PRACTICE TESTS 767


53155_09_PT 04_p765-785.qxd 2/27/09 6:30 PM Page 768

3. Be sure to drink 6 to 8 glasses of fluid each time (APTT), and decreased platelet count and
day and let me know right away if your urine fibrinogen level. What would be the explanation
turns dark like the color of cola. for the occurrence of these changes?
4. If the itching bothers you, put some rubbing 1. Formation of clots in small blood vessels
alcohol on your face several times a day. throughout the body has used up her clotting
factors.
29. A male client is admitted to the emergency 2. Damage to her liver during childbirth has
department with a medical diagnosis of closed- resulted in impaired production of clotting
angle glaucoma. He is placed on miotic therapy factors.
and receives 75% glycerin (Glycol). In planning
3. Exposure to fetal blood of a type different
care for the client, which of the following should
from hers has caused her to form antibodies,
be a teaching priority during the acute phase of
which are attacking her bone marrow.
his illness?
4. Internal bleeding has resulted in loss of the
1. Eyedrop administration.
clotting factors from the intravascular space
2. Eye patch changes every hour. into the interstitial spaces.
3. Measuring intake and output.
4. Keeping bright lights on in the room. 34. A 4-year-old girl was in a car accident with her
family. Upon arrival in the emergency room, she
30. Which nursing action represents unsafe nursing is noted to have lacerations on her head and
care for a client with closed-angle glaucoma? arms, a temperature of 39C, BP 158/102, pulse
1. Administering morphine sulfate 8 mg IM prn 60, and sluggish pupil reactions. She is crying
for pain. and does not recognize her family. With what
2. Allowing him to ambulate to the bathroom condition would the nurse suspect these
with assistance. manifestations to be associated?
3. Occluding the puncta during the 1. Elevated intracranial pressure.
administration of eye drops. 2. Reyes syndrome.
4. Wearing unsterile gloves when examining the 3. Guillain-Barr syndrome.
eye. 4. Anxiety attack related to being in a strange
environment.
31. A client who has glaucoma is receiving
pilocarpine (Pilocar). Which of these statements 35. An adult male is admitted in alcohol
would assure the nurse that the client withdrawal. The nurse plans his care to include
understands the reason for treatment? all of the following. What is the most important
1. It will reduce my intraocular pressure. goal to approach first?
2. It will improve my vision. 1. Client will be able to identify reality.
3. It will relieve the pain. 2. Client will remain free from injury.
4. It will restore my peripheral vision. 3. Client will remain free of alcohol use.
4. Client will maintain optimal nutrition intake.
32. An elderly man has closed-angle glaucoma. He
tells the nurse that he has heard that glaucoma 36. A man is admitted with a diagnosis of antisocial
may be hereditary. When he expresses concern personality. He has a long history of fights,
about his children, which is the most incarcerations for stealing and forgery, lying,
appropriate response for the nurse to make? lack of remorse for his actions, inconsistent
1. Are your children complaining of eye employment, and impersonal relationships with
problems? others. As a child, he was often truant, in trouble
2. There is no need for concern because with school officials, and cruel to his family dog.
glaucoma is not a hereditary disorder. Based on his background, which explanation is
associated with antisocial personality?
3. There may be a genetic factor with glaucoma
and your children should be screened. 1. A low I.Q.
4. Your son should be evaluated because he is 2. Failure to develop a stabilized and socialized
over 40. ego and superego during early childhood.
3. Rebelliousness despite parental discipline
33. A woman develops disseminated intravascular and moral values in the home.
coagulation (DIC) following childbirth. Lab 4. Poverty and resulting need to meet basic
studies show she has elevated prothrombin time needs independent of family.
(PT), elevated activated partial thromboplastin

768 NCLEX-RN Review


53155_09_PT 04_p765-785.qxd 2/27/09 6:30 PM Page 769

37. A young adult suffered from depression and was proportions. What would be the most adaptive
withdrawn when admitted to the unit. She has way she might try to deal with this situation?
responded well to treatment and, though still 1. Attempt thought-control methods to decrease
depressed, now attends unit group meetings. pervasiveness of thoughts.
How can the nurse best determine whether the 2. Request prn medication whenever such
clients condition has improved? thoughts intrude.
1. The client has been compliant with her 3. Share her concerns with another client
medications. whenever they arise.
2. Ask another client if she has improved in her 4. Withdraw to her room whenever such
participating in the milieu. thoughts arise.
3. Observe whether she socializes appropriately
with other clients outside of unit group 42. Several clients are participating in group
meetings. therapy. Which is least likely to be a benefit of
4. Observe that the client attends unit group group therapy for clients?
meetings. 1. Focusing strictly on personal situations.
2. An increase in the sense of belonging and
38. A child is admitted with idiopathic thrombocy- worthiness.
topenic purpura with a platelet count of
3. A decrease in isolation and an increase in
18,000/mm3. What will the nurse expect to be
reality testing.
ordered for this child?
4. The opportunity to ventilate and problem
1. Aspirin every 4 hours.
solve.
2. Seizure precautions.
3. Restricted activity level. 43. The nurse is caring for a woman who is 35 weeks
4. Tracheostomy set at bedside. pregnant. She comes to the emergency department
with painless vaginal bleeding. This is her third
39. A older client with arthritis is experiencing pregnancy and she states that this has never
increased alterations in mobility. In planning her happened to her before. What would be avoided
care, which of the following measures would be in caring for this client?
the best approach for the nurse to safeguard the 1. Allowing her husband to stay with her.
client? 2. Keeping her at rest.
1. Using a vest restraint at all times. 3. Shaving the perineum.
2. Teaching crutch walking. 4. Performing a vaginal examination.
3. Removing excess room furniture and
clutter. 44. The nurse is caring for a woman with a placenta
4. Placing the bedside table away from the previa who has been hospitalized for several
client. weeks. She is now at 38 weeks gestation and her
membranes have ruptured. The amniotic fluid
40. An adult client states that it hurts too much to has a greenish color and the woman has started
cough and deep breathe following abdominal to bleed again. What would be the nurses first
surgery. Which of the following approaches action?
would the nurse take first? 1. Administer oxygen.
1. Inform the client that coughing is not a 2. Place her in Trendelenburgs position.
matter of choice and must be done. 3. Call the physician and prepare for a cesarean
2. Call the respiratory therapist in to talk with birth.
the client. 4. Move her to the delivery room
3. Notify the surgeon that the client refuses to immediately.
cough.
4. Coordinate a pain medication and respiratory 45. A child has been brought to the emergency room
exercise schedule. with an asthma attack. What signs and
symptoms would the nurse expect to see?
41. An adult client has been admitted to the 1. A prolonged inspiratory time and a short
psychiatric unit. She is convinced that a blemish expiratory time.
on her face is a malignant melanoma. By the end 2. Frequent productive coughing of clear, frothy,
of the third day of hospitalization, her fear of thin mucus progressing to thick, tenacious
dying from the melanoma has reached psychotic mucus heard only on auscultation.

COMPREHENSIVE PRACTICE TESTS 769


53155_09_PT 04_p765-785.qxd 2/27/09 6:30 PM Page 770

3. Hypoinflation of the alveoli with resulting 3. Asking the client if she feels dizzy.
poor gas exchange from increasingly shallow 4. Outlining drainage on the dressing and
inspirations. noting the time.
4. Swelling of the bronchial mucosa, with
wheezes starting on expiration and spreading 50. A woman who has had a lumbar laminectomy
to continuous. and a spinal fusion is getting out of bed for the
first time. What action by the client will indicate
46. A 2-month-old baby who has a rash on his that the teaching plan is considered effective?
cheeks, trunk, and extremities that wont heal is 1. Bends only from the waist.
brought in for a well checkup. Infantile eczema 2. Moves rapidly.
is diagnosed and the nurse provides educational
3. Thinks through every movement.
teaching for this problem. Upon returning for the
3-month checkup, what reported activity 4. Refuses to use a walker.
indicates the mother has been properly caring
51. A 16-year-old client has acute infectious
for the babys skin?
mononucleosis. Which statement by the client
1. She bathes him twice a day to remove crusts. indicates to the nurse that he understands the
2. She leaves his skin exposed to air whenever necessary home care?
possible. 1. Im excited about going to the football game
3. She gently pats lubricant into the skin. tonight.
4. She uses only natural fibers against his skin. 2. My friends are coming over here to help me
with my school work.
47. The parents of a 2-month-old infant who has an
3. I plan to work out with the swim team
apnea monitor are visiting the pediatrician for a
tomorrow.
checkup. When asked how the monitoring is
going at home, the parents indicate 4. I have to stay in bed all the time.
dissatisfaction with the process, saying it keeps
52. A 70-year-old woman with severe macular
everyone awake and on edge while the baby is
degeneration is admitted to the hospital the day
okay. What can the nurse do to promote a safe,
before scheduled surgery. What would the
effective health care environment for this infant?
nurses preoperative goals include for her?
1. Order another monitor for them, because
1. Independently ambulating around the unit.
there are several brands to choose from.
2. Reading the routine preoperative education
2. Ask the parents to apply the monitor and
materials.
turn it on, so the nurse can see what happens.
3. Maneuvering safely after orientation to the
3. Stress the importance of continuation of
room.
monitoring during the first year for all high-
risk infants. 4. Using a bedpan for elimination needs.
4. Recommend that the infants crib be placed 53. The nurse is caring for a client with a newly
beside the parents bed so the baby can be implanted pacemaker. When monitoring
heard if any distress occurs. pacemaker functioning, which of the following
should the nurse initially assess?
48. The nurse is caring for a woman who has had a
lumbar laminectomy with a spinal fusion. 1. Electrocardiogram.
Immediately after surgery, which of the following 2. Pulse.
should the nurse expect the client to manifest? 3. Blood pressure.
1. Absence of lower extremity movement. 4. Incision site.
2. Response to pinprick sensation.
54. The nurse is assessing a child with
3. Severe muscle spasms.
conjunctivitis (pink eye). Which of the following
4. Weak pedal pulses. findings would the nurse most likely observe?
49. The nurse is caring for a client who has had a 1. Serous drainage from the eyes.
spinal fusion. The donor site for the graft begins to 2. Crusting of the eyelids.
hemorrhage and then ooze blood. What is the most 3. Severe eye pain.
appropriate way to determine whether nursing 4. Only one eye is affected.
interventions to stop bleeding have been effective?
1. Monitoring output. 55. An adult client is receiving cancer
2. Taking hourly vital signs. chemotherapy. Which action the client makes

770 NCLEX-RN Review


53155_09_PT 04_p765-785.qxd 2/27/09 6:30 PM Page 771

indicates a need for further instruction to 3. Inform client that most reactions usually do
prevent stomatitis? not occur until the end of the administration.
1. Brushing teeth with a soft bristle brush. 4. Gather blood filter tubing and IV catheters
2. Lubricating lips with petroleum jelly. with a 18 to 20 gauge needle.
3. Avoiding hard or spicy foods.
60. An adult is admitted for bipolar illness, manic
4. Rinsing with an alcohol-based mouthwash. phase, after assaulting his landlord in an
argument over the client staying up all night
56. The nurse is conducting a mental status
playing loud music. The client is hyperactive,
examination. What is used in the component of
intrusive, and has rapid, pressured speech. He
the examination that tests for the clients ability
has not slept in 3 days and appears thin and
to think abstractly as well as reason?
disheveled. Which of the following is the most
1. Proverbs. essential nursing action at this time?
2. Item identification. 1. Providing a meal and beverage for him to eat
3. Presidents names. in the dining room.
4. Serial sevens. 2. Providing linens and toiletries for the client
to attend to his hygiene.
57. A young adult was seen by the psychiatric nurse
3. Consulting with the psychiatrist to order a
and the client states he hears the voice of his
hypnotic to promote sleep.
former girlfriend calling to him to help her. In an
attempt to find her, he breaks into various 4. Providing for client safety by limiting his
buildings and enters others homes uninvited. privileges.
He rarely sleeps and has lost a job; afraid he will
61. The nurse is assessing a 2-year-old child with
miss a visit or call from her. He now lives with
tetralogy of Fallot. Which of the following is most
his parents who have threatened to evict him if
characteristic of a child with this condition?
he does not get help. Which of the following
nursing diagnoses is least appropriate? 1. Normal growth and development.
1. Altered thought processes. 2. Hypotonia of upper extremities.
2. Bathing hygiene self-care deficit. 3. Epistaxis.
3. Sensory/perceptual alterations. 4. Assuming a squatting position.
4. Sleep pattern disturbance. 62. A client has had a radical neck dissection. He is
having difficulty breathing and secretions are
58. A young adult was seen in outpatient clinic. He
visible in the laryngectomy tube. What should
states he hears the voice of a former girlfriend
be the initial nursing intervention?
calling to him to help her. He does not sleep and
has lost his job because he is afraid he will miss 1. Obtain the vital signs.
a visit or a phone call. What would the nurse 2. Notify the physician.
plan to do using the community mental health 3. Remove the secretions.
model? 4. Start oxygen via a tracheostomy collar.
1. Encourage the client to admit himself to a
community hospital psychiatric unit. 63. The nurse is caring for a client who has had a
2. File a petition for involuntary total laryngectomy. What is nursing management
commitment. in the early postoperative period directed
toward?
3. Maintain the client in treatment in a
community-based setting. 1. Alleviation of pain.
4. Refer the client to a psychiatrist for 2. Decreasing the clients concern about
medication as sole treatment. appearance.
3. Improving the nutritional status of the client.
59. The nurse is preparing to administer 2 units of 4. Observing the client for hemorrhage.
packed red blood cells. Which action should be
included at this time? 64. The nurse is caring for a toddler who has
1. Prime the blood administration tubing with infantile eczema. What will be included in the
3% saline solution. nursing care plan?
2. Add prescribed antibiotics when blood is 1. Applying the emollient preparation to the skin
infusing to ensure proper distribution before allowing the child to sit in the bathtub
throughout the body. to protect the skin from water damage.

COMPREHENSIVE PRACTICE TESTS 771


53155_09_PT 04_p765-785.qxd 2/27/09 6:30 PM Page 772

2. Removing the gloves, cotton stockings, or 3. Do not allow the client to perform any self-
elbow protector devices when the child is care activities for 48 hours.
sleeping. 4. Maintain the client NPO for 24 hours.
3. Measures to protect the family from the
childs lesions. 70. A young woman comes to the gynecology clinic
4. Teaching the parents that permanent remission to be fitted for a diaphragm. Which nursing
will usually take place around age 2 or 3. action would best prevent incorrect placement
of a diaphragm when the client is inserting it for
65. The nurse is assessing a newborn baby girl and the first time?
finds the following: in a supine position with 1. Allowing her supervised practice time.
hips and knees flexed, the right knee is higher 2. Providing a brochure.
than the left; there are more gluteal and thigh 3. Teaching her to lie on her back.
folds on the left than the right. What is the best
4. Teaching her sex partner to insert it.
interpretation for this data?
1. The right hip is dislocated. 71. The nurse in the gynecology clinic is assessing a
2. The left hip is dislocated. young woman. The client states that she gets her
3. Both hips are dislocated. menstrual period every 18 days. She states that
4. The baby has normal newborn joint laxity. her flow is very heavy and lasts 6 days. How
does the nurse identify this pattern?
66. A client with schizophrenia is admitted to the 1. Dysmenorrhea.
hospital experiencing auditory hallucinations 2. Dyspareunia.
that others are after him and intend to harm him. 3. Menorrhagia.
What should the nursing plan of care include?
4. Metrorrhagia.
1. Seclusion until hallucinations lessen.
2. Placement in a reality-oriented therapy group. 72. A young woman is seen in the womans clinic.
3. Advising the client that antipsychotic drugs She states that she has many little blisters on
will cure him. my privates. After examining her labia and
4. Presenting reality by stating that the nurse perineum, the nurse finds multiple vesicles,
does not hear the voices. some ruptured and crusted over. There is no
unusual vaginal discharge. What would the
67. An adult client underwent a cardiac nurse suspect?
catherization in which atherosclerotic plaque 1. Chlamydia.
formations were seen on his coronary arteries. 2. Gonorrhea.
Blood work revealed cholesterol of 260 and HDL 3. Herpes.
of 30. After dietary teaching, the client states I
4. Syphilis.
can eat red meat as long as I dont see any fat on
it. Which nursing diagnosis is most appropriate 73. A young child is admitted to the hospital with a
related to the clients statement? diagnosis of Reyes syndrome. Which of the
1. Altered nutrition: risk for more than body following would the nurse expect to see in the
requirements. childs history?
2. Altered nutrition: dysfunctional eating 1. Temperature elevations of 103F or higher in
behaviors. the past 8 hours.
3. Knowledge deficit: lack of information. 2. Enlarged spleen.
4. Knowledge deficit: information 3. Influenza 1 week ago.
misinterpretation. 4. Family history of Reyes syndrome.
68. One liter of fluid every 6 hours is ordered for 74. An older man with a 10-year history of
an adult client. If the adminstration set delivers Parkinsons disease is admitted to the hospital
10 gtts/mL, then the drip rate is ________. because his condition is deteriorating. What is
an obvious symptom of Parkinsons disease that
69. The nurse is caring for a woman who is 1 day
could be present on admission?
post radical mastectomy. What must be included
in the care plan? 1. Confusion.
1. Elevate the arm on the operative side for 2. Intention tremor.
24 to 48 hours. 3. Pallor.
2. Maintain complete bed rest for 24 to 48 hours. 4. Pill rolling.

772 NCLEX-RN Review


53155_09_PT 04_p765-785.qxd 2/27/09 6:30 PM Page 773

75. Amantadine hydrochloride (Symmetrel) is 3. Provides ROM exercises with the child
prescribed for a client with Parkinsons disease. admitted with rheumatic fever.
The client asks how the drug works. What 4. Positions the head of bed slightly elevated
response by the nurse indicates the correct for a child diagnosed with bacterial
action of the drug? meningitis.
1. The drug allows accumulation of dopamine.
2. The drug corrects mineral deficiencies. 81. An adult is admitted to the hospital with a
femoral neck fracture of the left leg. A total hip
3. The drug elevates the clients mood.
replacement is performed. While planning care
4. The drug replaces enzymes. for 2 days following the surgery, the nurse
includes which of the following nursing
76. The nurse is planning care for an elderly client
interventions?
who has severe Parkinsons disease. Which of
the following is of highest priority? 1. Ambulate in room with weight bearing on
both legs for 5 minutes.
1. Positioning.
2. Out of bed in the chair for 1 hour, elevating
2. Encouraging independence.
both legs on another chair.
3. Increasing activity.
3. Turn from side to side q 2 h, support upper
4. Preventing aspiration. leg with pillows from thigh to heel.
77. In planning care for a client with advanced 4. Turn from supine to right side q 2 h while
Parkinsons disease, which activity is most likely maintaining the left leg in abduction.
to be effective in alleviating fatigue?
82. An elderly client is admitted with the following
1. Getting him to bed on time. problems: oliguria, extreme fatigue, dyspnea.
2. Avoiding high-carbohydrate foods. Vital signs are as follows: T 100.2F (oral), HR
3. Collaborating with him when scheduling 62, R 28, BP 152/94. Assessment reveals 3+
activities. bilateral pedal edema, crackles in bilateral lungs
4. Providing for morning and afternoon naps fields, blood glucose of 150, hypoactive bowel
while he is in the hospital. sounds, and a pressure ulcer on the left lateral
ankle. Which nursing diagnosis would be
78. What intervention should the nurse include assigned the highest priority?
when planning care for the client with multiple 1. Activity intolerance.
myeloma? 2. Ineffective breathing patterns.
1. Fluid restriction. 3. Constipation.
2. Administration of potassium supplements. 4. Skin integrity impairment.
3. Assisting with mobility.
4. Administration of aspirin to control bone pain. 83. An adult client is hospitalized for treatment of
diabetes insipidus. The nurse is performing the
79. The nurse is evaluating a client who has been in initial assessment. Which finding should the
a long leg cast for 3 weeks. Which finding nurse expect?
indicates the client is free of neurological or 1. Daily urine output of 10 liters.
circulatory complications? 2. Urine specific gravity of 1.050.
1. The toes on the casted foot are cool to the 3. Serum sodium levels of 120 mEq/liter.
touch.
4. Daily fluid intake of 12 liters.
2. The nail beds have a blue tinge when pressed
lightly. 84. A 1-day-old infant is admitted to the
3. The client reports pain under the cast near intensive care nursery. She is suspected
the fracture site. of having esophageal atresia. What
4. The dorsalis pedis pulse is +3. assessment findings should the nurse expect
to find?
80. An unlicensed assistive personnel (UAP) is 1. Bile-stained vomitus and a weak cry.
assisting children in play. Which action by the 2. Diarrhea and colicky abdominal pain.
UAP would need further instruction by the RN?
3. Excessive drooling and immediate
1. Works a puzzle with a child recovering with regurgitation of feedings.
Reyes syndrome.
4. Visible peristaltic waves and projectile
2. Places an infant in side-lying position due to vomiting.
diaper rash.

COMPREHENSIVE PRACTICE TESTS 773


53155_09_PT 04_p765-785.qxd 2/27/09 6:30 PM Page 774

85. The nurse is assessing a 6-month-old child. 90. An adult has continued slow bleeding from the
Which developmental skills are normal and graft after repair of an abdominal aortic
should be expected? aneurysm and is in the intensive care unit. The
1. Speaks in short sentences. client insists on having a visit from a medicine
2. Sits alone. man whom the family visits regularly. How
should the nurse interpret this request?
3. Can feed self with a spoon.
1. The principle of justice prohibits giving one
4. Pulling up to a standing position.
client a privilege that other clients are not
86. The parents of a 1-year-old are discussing the permitted.
safety needs of their daughter with the nurse. 2. Faith healers do not meet the standards for
Which statement indicates a need for further clergy exemption from visitation rules.
education on safety practices? 3. Medicine men are not approved by the
1. We should fence in our yard soon. hospital as legitimate health care providers.
2. One of us will always be with her while she 4. Provision of holistic care requires that the
is in the bathtub. clients belief system is honored.
3. We dont need the stair gate anymore; shes
91. The nurse is caring for a client who had a left knee
so good at walking.
replacement. The nurse is adjusting the passive
4. The safest position for her car seat is in the motion device on his third post-op day. Which of
middle of the back seat. the following indicates correct technique?
87. The nurse is caring for a client with Raynauds 1. Allow passive motion prn as desired.
phenomenon. The nurse should instruct the 2. Monitor alignment at hinged joint of machine.
client to avoid which of the following 3. Monitor pressure areas on shins.
situations? 4. Allow client to choose speed and degree of
1. Living in a warm climate. extension.
2. Active exercising.
92. An adult is admitted to the surgical floor with a
3. Exposure to cold temperatures.
diagnosis of a tumor, right lung. Upon return to
4. Alcohol consumption. the surgical unit following a right
pneumonectomy, the nurse should place the
88. The nurse is caring for an elderly client who has
client in which position?
been diagnosed as having sundowners
syndrome. The nurse asks the client and his 1. Left lateral decubitus.
family to list all of the medications, prescription 2. Right lateral decubitus.
and nonprescription, he is currently taking. 3. Semi-Fowlers.
What is the primary reason for this action? 4. High-Fowlers.
1. Multiple medications can lead to
dementia. 93. An adult comes to the clinic because she has a
2. The medications can provide clues regarding productive cough. She smokes two packs of
his medical background. cigarettes a day and has a family history of lung
cancer and emphysema. Using the principles of
3. Ability to recall medications is a good
health promotion, the nurse would make what
assessment of the clients level of
interpretation of the clients behavior?
orientation.
1. Using denial to deal with being at high risk
4. Medications taken by a client are part of
for lung cancer.
every nursing assessment.
2. Not assuming self-responsibility for her health.
89. The nurse must report to another nurse about a 3. Exhibiting a laissez-faire attitude toward
clients problem and is using the SBAR smoking and her risk of cancer.
technique for communication. Which of the 4. Demonstrating passive suicidal tendencies.
following would be included in this particular
type of communication? 94. The nurse is teaching an adult who has
1. S sensory capabilities ulcerative colitis. In developing the teaching
2. B background information plan which of the following foods should the
nurse plan to instruct the client to avoid?
3. A ADLs
1. Roast chicken and cooked spinach.
4. R respiratory status
2. Broiled liver and white rice.

774 NCLEX-RN Review


53155_09_PT 04_p765-785.qxd 2/27/09 6:30 PM Page 775

3. Cottage cheese and canned apricots. 3. I believe he is having pain and I want to
4. Pork chop and brown rice. help him deal with it.
4. If he continues taking pain medication, he
95. A 46-year-old female with chronic constipation will become a drug addict even though he
is assessed by the nurse for a bowel training really has pain.
regimen. Which factor indicates further
information is needed by the nurse? 98. The nurse is assessing a client following
1. The clients dietary habits include foods high hemodialysis. Which of the following findings
in bulk. indicates the treatment was effective?
2. The clients fluid intake is between 1. Hypertension.
25003000 mL per day. 2. Hyperkalemia.
3. The client engages in moderate exercise each 3. Fluid volume decrease.
day. 4. Cardiac dysrhythmias.
4. The client has bowel sounds in all four
quadrants. 99. The nurse is discontinuing an intravenous
catheter. Which action should be included at
96. A 26-year-old obese female is assessed for a this time?
weight reduction diet by a clinic nurse. Which 1. Apply a tourniquet proximal to the catheter
of the following statements by the client insertion site.
presents most concern to the nurse? 2. Flush the catheter with a heparin solution to
1. I understand the food pyramid. ensure patency of the catheter before
2. My family is in support of my weight removal.
reduction. 3. Assess the insertion site for signs of
3. I have gained and lost weight over the last infiltration or inflammation.
5 years. 4. Wear only sterile gloves to perform the
4. I do not have diabetes. procedure.

97. A man suffered a traumatic amputation of his left 100. The client diagnosed with asthma has visible
arm in a factory accident about 7 months ago and oral candidiasis. What question should the nurse
has had severe chronic phantom pain for the last ask this client?
6 months. Which statement, if made by his wife, 1. Will you show me how you use your
who assists her husband with his daily care, inhaler?
indicates an understanding of this clients pain? 2. When was your last dentist visit?
1. Phantom pain is not real pain; his body is 3. Have you had respiratory infection lately?
just tricked into thinking he has pain.
4. Do you floss your teeth daily?
2. Because he lost his arm so long ago, his pain
must be caused by something besides his
injury.

Answers and Rationales for Practice Test 4

ANSWER RATIONALE NP CN CL SA
#1. 1. Excess output of the thyroid hormones increase the metabolic An Ph/8 An 1
rate causing an increased demand for food. Other symptoms
are the presence of a goiter, fine tremor of the fingers, increased
nervousness, weight loss, altered bowel activity, heat intolerance,
excessive sweating and increased heart rate.

COMPREHENSIVE PRACTICE TESTS 775


53155_09_PT 04_p765-785.qxd 2/27/09 6:30 PM Page 776

ANSWER RATIONALE NP CN CL SA
#2. 2. Polyarthritis is characterized by swollen painful, hot joints An Ph/6 An 4
that respond to salicylate. Chorea is irregular movement;
SC nodules and erythema marginatum (nonpruritic rash)
are typical with rheumatic fever.
#3. 3. It is thought that hypoguesia (altered taste sensation) occurs As Ph/5 Ap 3
when cancer cells release substances that resemble amino
acids and stimulate the bitter taste buds. Food-enhancing
seasoning can mask the taste alternations. This phenomenon
is also reported in the aging population.
#4. 1. Otosclerosis is the formation of spongy bone in the capsule of As Ph/8 An 1
the ear labyrinth. As it advances, it causes progressive fixation
of the footplate of the stapes. With oval window obstruction
by otosclerosis, hearing by air conduction is reduced.
#5. 3. Speaking at a moderate rate allows the client to observe the lips Ev Ph/7 Ap 1
of the speaker and to hear normal voice tones, while using short
phrases and speaking slowly.
#6. 4. Placing the food in a recognizable location fosters autonomy as Im Ph/5 Ap 4
well as independence. The 4-year-old is too young to understand
the clock method.
#7. 3. The nurse recognizes a normal occurrence in a 13 month old An He/3 An 4
infant, known as the Moro reflex. No abnormality is occurring
that warrants seeing a neurologist; a Babinski reflex involves the
feet; and it is recommended that infants are not placed prone
(unless otherwise indicated).
#8. 1. Sexual intercourse is avoided as it causes uterine contractions, Ev He/3 An 3
contributing to further placental separation or dislodge the
placenta. The client will not have vaginal examinations (as it
can cause further separation of the placenta); cesarean will be
evaluated at a later time; bed rest is recommended.
#9. 1. The usual requirement for dress to the operating room is a Im Ph/7 An 1
hospital gown, with all jewelry, dentures, and contact lenses
removed. Liquids are contraindicated to prevent aspiration
(unless ordered specifically by the physician or anesthesiologist);
IS instructions should have given at a previous time as the client
may be experiencing some anxiety at this time; although it is
important to know the clients smoking habits, which will
influence postoperative healing, this information would already
be known when obtaining a client history.
#10. 1. The clients high activity level poses the most danger because An Ps/4 An 2
it can lead to absence of food, fluid, and rest with resultant
dehydration, electrolyte imbalance, and physical collapse.
#11. 2. Many characteristics of a client who is manic (i.e., irritability, An Ps/4 An 2
excitement, agitation, provocative behavior) contribute to the
potential for violence. Maintaining the safety of the client and
those around him is the greatest priority.
#12. 4. Manic clients cannot calm down without assistance. Decreasing Im Ps/4 Ap 2
the level of sensory stimulation is of paramount importance and
provides the greatest therapeutic effect until proper medication

776 NCLEX-RN Review


53155_09_PT 04_p765-785.qxd 2/27/09 6:30 PM Page 777

ANSWER RATIONALE NP CN CL SA
levels (often lithium) are established. Restraints would further
agitate the client.
#13. 4. While the clients social network can influence the client in terms Ev Ph/6 An 5
of compliance, the influence is typically secondary to that of the
other factors listed. Side effects of lithium include fine tremor,
drowsiness, diarrhea, polyuria, thirst, weight gain, and fatigue,
which can be disturbing to the client.
#14. 4. This is the only choice that only has one source of protein, where- Ev Ph/5 An 4
as the others have two sources.
#15. 2. Drying prevents heat loss and reducing of body temperature, the Im He/3 Ap 3
most important part of newborn care. The other interventions
will be done following the drying or within 1 hour after birth.
#16. 3. Careful assessment of the breasts before and after each feeding is Ev He/3 An 3
extremely important for noting any early skin breakdown, which
can get infected. A lump can signify a clogged duct; position
changes may be needed if skin breakdown is occurring;
handwashing is appropriate for all infants.
#17. 1. Heme in the stool can be an early warning sign of necrotizing As Ph/7 An 4
enterocolitis. All the other activities should be assessed prior
to each feeding.
#18. 2. The baby shows clinical manifestations of patent ductus An Ph/7 An 3
arteriosus (PDA): failure of the fetal ductus arteriosus to
completely close after birth. The ductus venous is a major
blood channel that develops through the embryonic liver
from the left umbilical vein to the inferior vena cava; after
the ductus arteriosus closes, the remains are called the
ligamentum arteriosus; the foramen ovale closes at birth,
failure of closure is manifested by dyspnea.
#19. 3. Because infants born to addicted mothers are highly Pl He/3 Ap 3
irritable, it is best to organize all care around the feedings
and then try to disturb them as little as possible. The infants
also have a strong sucking reflex, may have frequent vomiting,
and are prone to temperature instability, so it is advisable to
keep the infant wrapped snugly to maintain temperature.
Breastfeeding would not be recommended as drugs will cross
the breast milk.
#20. 4. Gravity will cause the head of the femur to drop toward the As Ph/7 An 4
bed, causing the affected thigh to appear shorter. Ortolanis
sign (a popping sensation when hip joint is internally and
externally rotated) is normal; Trendelenburgs sign is seen with
an abnormality of the pelvis associated with congenital hip
dislocation; skinfolds would be increased in the affected thigh.
#21. 1. Infiltration is the infusion of fluid into tissue. The accumulation Ev Ph/6 An 1
of fluid causes pressure, which reduces circulation to the area,
resulting in pallor. Increased warmth or redness at the site would
suggest phlebitis or infection. Blood seeping around the needle
may be from anticoagulant overdose or the insertion site has
been stretched and needs restarting at a new site.

ANSWERS AND RATIONALES FOR PRACTICE TEST 4 777


53155_09_PT 04_p765-785.qxd 2/27/09 6:30 PM Page 778

ANSWER RATIONALE NP CN CL SA
#22. 2. The most frequent means of transmission of the tubercle bacillus Pl Sa/2 Co 1
is droplet nuclei. The bacillus is present in the air as a result of
coughing, sneezing, laughing, singing, and expectorating of
sputum by an infected person.
#23. 4. A side effect of rifampin is orange-tinged tears, sweat, urine, and Im Ph/6 Co 5
it may stain soft contact lenses. Tingling in the feet is a side
effect of isoniazid (INH), which is sometimes taken in conjunction
with rifampin.
#24. 3. Determining when the client had her last bowel movement As Ph/5 Ap 1
provides baseline data as a first part of bowel history. Avoid
questions that begin with why as it may appear threatening
to the client.
#25. 3. The boot should be removed at least once a shift for skin care Pl Ph/5 Ap 1
and to assess for skin breakdown and nerve damage. The left leg
must be immobilized by one person and traction applied while
the second person removes the boot, provides the skin care, and
performs the assessment. Turning may cause bone fragments to
move against each other resulting in damage to blood vessels
and nerves. A trapeze would be placed for the client to lift herself,
while also being encouraged to cough and deep breathe.
#26. 1. Monitoring vital signs is the most important aspect of assessing Im Ph/7 Ap 1
respiratory and cardiovascular status. All the other actions
would follow assessment.
#27. 4. Sign and symptoms of acute rejection include temperature of As Ph/7 Co 1
100F or greater, enlarged tender kidney, fluid retention,
increased blood pressure, fatigue and lethargy. Anuria or
oliguria, not polyuria, occurs with acute rejection. An increase
in blood pressure would increase, not decrease, due to the
fluid overload. Anemia is a symptom of chronic renal
failure, not acute rejection.
#28. 3. Several weeks after the lesions have healed, the child who had Im Ph/7 An 4
beta-hemolytic streptococci infection is at risk for acute
glomerulonephritis. Puffiness around the eyes would also be
seen. Medical referral is needed promptly. An antibiotic would
be started after the infection is diagnosed; personal items do not
need to be separate, as secretions harbor the organism; Neosporin
could be a topical antibiotic applied several times a day.
#29. 3. Because glycerin, a rapid-acting osmotic diuretic, is being used, the Pl Ph/6 Ap 1
clients intake and output would be monitored. During the acute
phase, eye patches may not be present and bright lights would be
irritating. Eyedrop administration would be done at a later time.
#30. 1. Morphine is contraindicated for a client with glaucoma because An Ph/6 Ap 1
it is constipating. Straining at stool raises intraocular pressure.
Occluding the puncta prevents the eye medication from
entering the systemic circulation. Unsterile gloves can be worn
to prevent exposure of viruses to the examiner.
#31. 1. Pilocar is a cholinergic agent that reduces intraocular pressure by An Ph/6 Co 5
producing miosis (constriction of the pupil), thus increasing
outflow of aqueous humor. It will not perform any of the other
choices.

778 NCLEX-RN Review


53155_09_PT 04_p765-785.qxd 2/27/09 6:30 PM Page 779

ANSWER RATIONALE NP CN CL SA
#32. 3. There is a strong hereditary factor in glaucoma. Therefore, family Im He/3 Co 1
members of all ages should have intraocular pressures measured
yearly.
#33. 1. The pathophysiology of DIC includes formation of multiple An Ph/7 An 3
microscopic clots in very small vessels, which uses up the
clotting factors, leaving the client vulnerable to bleeding at
other sites. The abnormal lab values are the indication of the
cause of the problem.
#34. 1. Head injury is one situation that may cause elevated intracranial An Ph/8 An 4
pressure, as evidence by the manifestations listed. Reyes
syndrome affects the central nervous system; Guillain-Barr
presents with ascending paralysis; and the childs crying would
be expected, but the presenting assessment would point to a
different problem.
#35. 2. Safety is the highest priority. A client in withdrawal suffers from Pl Ps/4 Ap 2
altered cognition and sensory disturbances, as well as tremors,
which increase the potential for injury.
#36. 2. This is the psychodynamic view and usually begins prior to age 15. An Ps/4 An 2
His antisocial personality stems from a failure to develop a
stabilized and socialized ego and superego during early childhood.
A low I.Q. is associated with mental retardation, however many
antisocial persons have an above average intelligence. A child
with the stated behaviors are the result of a lack of consistent or
effective behavior, which also does not provide an atmosphere to
instill society morals and values. Poverty is not a direct link to
antisocial personality.
#37. 4. The clients attendance displays that she is a participant in her Ev Ps/4 An 2
treatment and has made a step forward from her withdrawn
behavior.
#38. 3. Prevention of injury, bruising, and bleeding is high priority when Pl Ph/7 Ap 1
the platelet count is low. As normal is 150,000500,000/mm3,
this childs count is extremely low. Aspirin is contraindicated
in bleeding disorders; seizure precautions are associated
with a central nervous system disorder, not thrombocytopenic;
there is not a risk for airway obstruction.
#39. 3. Removing excess items from the room is the best way to safeguard Pl Ph/5 Ap 1
the client. Crutch walking would be difficult due to the arthritis
and the bedside table should be close to avoid overreaching
and the chance of injury or falling.
#40. 4. Pain medication should be given when available, even on a Pl Ph/7 Ap 1
PRN basis. If the pain is lessened the client will be more
cooperative and achieve the goal of coughing and deep breathing.
#41. 1. Thought-control methods are applicable to this situation and are Im Ps/4 An 2
the least restrictive method of achieving symptom control which is
designed to subvert an individuals control of her own
thinking, behavior, emotions, or decisions.
#42. 1. This action is handled in individual therapy, not group therapy. An Ps/4 An 2
All the other choices are a beneficial goal of group therapy.

ANSWERS AND RATIONALES FOR PRACTICE TEST 4 779


53155_09_PT 04_p765-785.qxd 2/27/09 6:30 PM Page 780

ANSWER RATIONALE NP CN CL SA
#43. 4. Painless vaginal bleeding is symptomatic of placenta previa. Im He/3 Ap 3
Vaginal exams are contraindicated before 36 weeks unless done
in the delivery room set up for emergency cesarean section if
needed. Bed rest is essential and shaving is not necessary.
#44. 3. Green amniotic fluid is indicative of fetal distress. This combined Pl He/3 Ap 3
with bleeding from the placenta previa may require a cesarean
section. Oxygen and movement to the delivery room may be
performed, but notifying the physician would be a definite
plan.
#45. 4. Asthma causes spasm of the smooth muscles in the bronchi and As Ph/8 Co 4
bronchioles, resulting in prolonged exhalation. Inspirations
increase in rate in an effort to relieve hypoxia. The cough would
begin as nonproductive, then progress to a profuse mucous; gas
trapping is caused by allowing more air to enter the alveoli than
can escape, which causes increased depth and rate of respirations.
#46. 3. Lubricants applied to the skin after bathing seal in moisture and Ev Ph/7 Ap 4
rehydrate, lubricate, and moisturize the skin. Wool is an example
of a natural fiber, which would not be used.
#47. 2. The nurse should observe how the family applies and positions Ev Ph/7 Ap 4
the monitors leads. This will provide the nurse more data to rule
out faulty technique. When monitors frequently alarm for breathing
infants, it is usually due to loose leads or low batteries.
#48. 2. Sensation and movement should be present, along with normal As Ph/7 An 1
pedal pulses. Spasms are indicative of nerve damage during
surgery.
#49. 4. It will be important to outline the drainage so that a quantitative Ev Ph/7 An 1
measure can be obtained.
#50. 3. Good body alignment and avoiding sudden movements will be to Ev Ph/5 Ap 1
her advantage as she thinks through her moves. A walker may not
be indicated, but does not indicate effective teaching.
#51. 2. Rest is a primary treatment; however adolescents have a great Ev Ph/7 Ap 4
need for socialization with peers. Activities during the
acute phase should be restricted.
#52. 3. This is a realistic goal to be oriented to the room and bathroom, Pl Sa/2 Ap 1
as the others could either cause injury or not be able to be seen
well enough to read.
#53. 1. The ECG reflects the heart rate, pacer spikes, and dysrhythmias. As Ph/7 Ap 1
#54. 2. This is associated with purulent drainage and is usually bilateral. As Ph/7 Ap 4
Serous drainage is characteristic of a viral infection and pain is
characteristic of a foreign body in the eye.
#55. 4. An alcohol-based mouthwash will break down the tissues. All the Ev Ph/7 Ap 1
other choices are appropriate.
#56. 1. The use of proverbs test for the clients ability to abstract meaning As Ps/4 Co 2
as well as reason. Item identification tests knowledge of object
in the environment; presidents names test long-term memory;
serial sevens tests calculation ability.

780 NCLEX-RN Review


53155_09_PT 04_p765-785.qxd 2/27/09 6:30 PM Page 781

ANSWER RATIONALE NP CN CL SA
#57. 2. There is nothing in the case information to support the An Ps/4 An 2
diagnosis of a self-care deficit. The client does exhibit all
the other diagnoses.
#58. 3. As the client is unlikely to admit himself to a psychiatric unit, Pl Ps/4 Ap 2
he would best be managed in a community setting with partial
hospitalization and outpatient care, as he will need
psychotherapy as well as medication.
#59. 4. The large bore needle prevents lysis of the red blood cells, and Pl Ph/6 Ap 1
the special blood filter prevents emboli or contamination matter
from flowing into the bloodstream. Blood tubing is always primed
with normal saline (0.9%), and no medications are infused with
the blood. Blood reactions mostly occur during the first 15 to
60 minutes of an infusion.
#60. 4. It is reasonable to expect that client may be assaultive with peers Im Ps/4 Ap 2
and staff. His mental illness produces a hyperactive state and poor
judgment and impulse control. External controls such as limiting
of unit privileges will assist in feelings of security and safety. His
hyperactivity interferes with food and hygiene needs; he will
receive an evaluation before medication is ordered.
#61. 4. This position allows for relief of dyspnea, which improves As Ph/7 Ap 4
the hemodynamics. Children with tetralogy have poor growth;
hypotonia and epistaxis may be present, but these are not the
most characteristic manifestations.
#62. 3. Secretions that are visible in the tube may be partially occluding Im Ph/7 Ap 1
the airway and should be removed by suctioning. After removing
secretions, oxygen may be required, but if no relief is obtained,
the physician would be called to report the condition and
provide vital signs.
#63. 4. Life-threatening disorders such as hemorrhage and breathing Pl Ph/7 Ap 1
difficulties are a priority in the immediate period. The others
are important, but remember the ABCs of nursing care.
#64. 4. Until the time of spontaneous remission, there will be many Pl Ph/7 Ap 4
exacerbations and remissions. Sometimes these children develop
asthma-type respiratory reactions or other allergies. Lotion is
applied after the bath; protector devices are left on to prevent
scratching; and eczema runs in families.
#65. 2. The shorter leg is on the affected side, because the femur head An Ph/7 An 3
slips further upward into the acetabulum. This causes the extra
skin folds in the thigh on the affected leg.
#66. 4. Reality is presented to decrease fear in the client which results Pl Ps/4 An 2
from internal stimuli. Seclusion and group therapy will not
be beneficial at this time; antipsychotic drugs do cure the
problem.
#67. 4. The client is displaying denial or has misinterpreted the An Ph/5 An 1
information given to him by the dietitian. He should abstain
from red meat as it contains hidden fat that is not visible.

ANSWERS AND RATIONALES FOR PRACTICE TEST 4 781


53155_09_PT 04_p765-785.qxd 2/27/09 6:30 PM Page 782

ANSWER RATIONALE NP CN CL SA
#68. 28 gtts/min. Using the formula: amount of solution in mL 4 time Im Ph/6 Ap 5
in minutes x the drop factor, the correct rate of low is 28 gtts/min.
6 hrs 3 60 min. 5 360 min.
1000 mL
5 2.8 3 10 5 28
360
#69. 1. It is elevated and placed at a right angle to the chest. Early Pl Ph/7 Ap 3
ambulation and post operation exercises are encouraged to
promote functioning of all body systems and to prevent
postoperative complications. Food and fluid are also
encouraged.
#70. 1. Correct placement is accomplished best if the client is allowed Im He/3 Ap 3
time to practice insertion of the device under professional
supervision. Lying on the back is not necessary and the client
needs to take responsibility for proper insertion.
#71. 3. This is abnormal menstrual flow. Dysmenorrhea is painful As Ph/7 K 3
menstruation; dyspareunia is painful intercourse; and
metrorrhagia is uterine bleeding other than that caused by
menstruation.
#72. 3. This condition is descriptive of genital herpes, which is highly As Ph/7 Co 3
contagious and causes severe morbidity and recurrences. There is
no cure, but acyclovir helps to reduce the number of occurrences.
Chlamydia has a non-odorous white discharge; gonorrhea has
yellowish-green discharge; and syphilis has a cancre present.
#73. 3. Reyes syndrome follows a common viral illness such as influenza As Ph/7 Co 4
or varicella and an enlarged liver may accompany the condition.
There is no genetic predisposition for Reyes syndrome.
#74. 4. Rhythmic flexion and contraction of the muscles cause a As Ph/7 Ap 1
characteristic tremor called a pill rolling tremor. A staring
mask-like facial expression may be apparent due to muscle
tension. Confusion and pallor are not characteristics of
Parkinsons disease; tremors are unintentional at rest and
tend to disappear with motion.
#75. 1. Because Parkinsons disease is characterized by a dopamine An Ph/6 Ap 5
deficiency, the medication allows dopamine to accumulate
in extracellular or synaptic sites. The disease is not related to
mineral or enzyme deficiencies, or mood disorders.
#76. 4. Clients with advanced Parkinsons disease usually have difficulty Pl Ph/7 Ap 1
swallowing and are in danger of choking. Families need much
instruction on this slowly progressing disorder, as the swallowing
disorder may require hospitalization. Preventing aspiration
pneumonia must be a high priority.
#77. 3. Scheduling activities in collaboration with the client will allow Pl Ph/7 Ap 1
him to proceed at his own pace and maximize his strength. All
activities, including naps, should be planned with the client, as
well as providing a high-carb diet to provide energy.
#78. 3. Mobility is important for the client with multiple myeloma. Pl Ph/7 Ap 1
Weight bearing promotes movement of calcium back into

782 NCLEX-RN Review


53155_09_PT 04_p765-785.qxd 2/27/09 6:30 PM Page 783

ANSWER RATIONALE NP CN CL SA
weakened bones, helping to maintain their strength. This will
also reduce the risk of hypercalcemia, which is a common
complication with multiple myeloma. Fluids are encouraged to
prevent renal failure due to the high levels of uric acid released
as plasma cells are destroyed; aspirin is avoided due to decreased
platelet count from chemotherapy, which would predispose
to bleeding.
#79. 4. The casted extremity should have palpable pulses of +2 to +3, Ev Ph/7 Ap 1
feel warm to touch, the toes should be able to move freely, have
pink nail beds, and a report of decreasing pain at fracture.
Increasing pain would indicate a warning of complications,
such as compartment syndrome.
#80. 3. Rheumatic fever is an inflammatory disease that may develop Pl Ph/5 An 4
after an infection with streptococcus bacteria and develops as a
type of arthritis. To relieve discomfort due to the arthritic pain,
the child should not perform ROM exercises, should not be
massaged or have splints applied, as these treatments will cause
additional pain. The other actions are appropriate for the
specified diagnosis.
#81. 4. The leg must be maintained in abduction and the hip is not to Pl Ph/7 Ap 1
be flexed more than 4560. Recommended activities would
include standing on nonoperative leg and weight bearing when
permitted and turning to the right side. Activities to avoid would
be sitting in a chair that requires more than a 60 bend at the hip
or elevating the legs on a chair.
#82. 2. All nursing diagnoses are appropriate for the client; however An Ph/7 An 1
the need for air is a priority, cited on Maslows Hierarchy of
Needs, as a physiological need. The need for air is a higher
priority than activity, bowel movements or skin impairment.
#83. 1. Clinical manifestations in diabetes insipidus include marked As Ph/7 Co 1
polyuria, extreme dilution of the urine resulting in a specific
gravity of 1.0001.005, polydipsia, high serum osmolarity,
and hypernatremia. This disorder is caused by a deficiency
of vasopressin.
#84. 3. The esophagus is closed at some point and there is a fistula to As Ph/7 Co 3
the trachea. Because of the blockage, excessive mucus builds up
in the nasopharynx and the child has difficulty breathing and
becomes cyanotic. After being suctioned, the baby becomes
pink again. As food is not getting to the intestine, there would
be no evidence of diarrhea or colicky abdominal pain.
#85. 2. A 6-month-old is learning to sit alone. Language skills begin As He/3 K 4
between ages 1 and 3; spoon use begins at 1215 months; and
pulls himself to a standing position at 812 months.
#86. 3. Most children at 12 months are not proficient walkers. They Ev Sa/2 Ap 4
may know how to climb upstairs but not how to come down.
Their sense of balance is not stable and they lack judgment.
#87. 3. Raynauds phenomenon (excessive and prolonged painful Im Ph/7 Ap 1
vasoconstriction of the extremities, especially the hands) is
precipitated by exposure to cold and aggravated by smoking.
The other activities do not cause an attack.

ANSWERS AND RATIONALES FOR PRACTICE TEST 4 783


53155_09_PT 04_p765-785.qxd 2/27/09 6:30 PM Page 784

ANSWER RATIONALE NP CN CL SA
#88. 1. Polypharmacy (concurrent use of several drugs) increases the An Ph/6 An 1
potential for adverse side effects, one being dementia.
Sundowners syndrome involves behaviors that are seen in the
late afternoon or early evening when the sun sets, which include
disorientation, emotional upset, or confusion.
#89. 2. The SBAR technique for communication is a method that helps Im Sa/1 Ap 1
guide required information to be passed on to another. The
acronym is as follows:
S Situation identify the clients name and problem
B Background state pertinent background information
A Assessment state concern
R Recommendation state what you want
#90. 4. The clients spiritual needs must be met within the framework An Sa/1 An 1
of his personal belief systems, even if those beliefs differ from
those of the nursing staff.
#91. 2. It is imperative that correct alignment at the knee joint is regarded Im Ph/7 Ap 1
while in the continuous passive motion (CPM) machine. The client
will complain of increased pain if alignment is not maintained, so
frequent checking is necessary. The CPM time and degrees are
ordered by the physician. No pressure is exerted on the shins.
#92. 3. Semi-Fowlers position would promote respiratory function. Im Ph/7 Ap 1
High-Fowlers would probably cause too much fatigue for the
client; decubitus is a lying down position.
#93. 2. There are four principles of health promotion: self-responsibility, An He/3 An 1
nutrition, stress management, and exercise. Self-responsibility
includes avoiding high-risk behaviors, such as smoking, abusing
alcohol, overeating or driving while intoxicated.
#94. 4. A low-fiber diet is recommended, which will be limited in high Pl Ph/5 Co 1
roughage content (which stimulates peristalsis and makes
symptoms of ulcerative colitis worse). Foods to avoid would
include whole grains, nuts, raw fruits and vegetables, caffeine,
alcohol, tough meats, pork, and highly spiced meats.
#95. 4. Even though the client has positive bowel sounds in all four An Ph/5 Co 1
quadrants, this does not provide information about bowel habits
of the client. This information would help the nurse determine
the normal patterns of the client at present.
#96. 3. To design an effective care plan for an obese client, the nurse An Ph/5 Ap 1
should be aware of past weight problems, along with eating
behaviors, exercise habits, medical diagnoses, family support,
and reasons for desiring weight loss.
#97. 3. Nurses and caregivers must realize that pain is whatever the Ev Ph/7 Ap 1
client says it is, when it is. Phantom pain can last for many years.
Addiction is the seeking of drugs for a psychic, not physical,
action.
#98. 3. One of the cardiovascular manifestations of chronic renal Ev Ph/7 An 1
failure and uremia is fluid volume excess. Blood pressure

784 NCLEX-RN Review


53155_09_PT 04_p765-785.qxd 2/27/09 6:30 PM Page 785

ANSWER RATIONALE NP CN CL SA
and potassium would decrease following hemodialysis, due
to the fluid reduction.
#99. 3. Extravasation of intravenous fluid may cause tissue ischemia Im Ph/6 Ap 1
or necrosis. A catheter is inserted using a tourniquet, heparin
is used when flushing a PICC or central line, and this is not
a sterile procedure.
#100. 1. This condition appears as a cheesy white plaque on the tongue As Ph/6 An 1
and is also called thrush. The client may not be using the
metered dose inhaler (MDI) without a spacer. After using the
spacer with the medication, the mouth should be rinsed to further
prevent thrush. The other choices would not be the cause for
thrush.

ANSWERS AND RATIONALES FOR PRACTICE TEST 4 785


53155_09_PT 05_p786-805.qxd 2/27/09 6:31 PM Page 786

Practice Test 5

1. An adult client is admitted to the hospital with a 2. Keep flat on back with minimal movement to
diagnosis of tuberculosis. Which room reduce risk of hemorrhage following surgery.
assignment is most appropriate for a client with 3. Administer hydrocortisone until vital signs
active tuberculosis? stabilize, then discontinue the IV.
1. A semiprivate room. 4. Teach him how to care for his wound because
2. A room with laminar flow. he is at high risk for developing postoperative
3. A reverse isolation unit. infection.
4. A negative pressure room.
6. When a 10-year-old went to see the school nurse
2. A sputum specimen is ordered for a client. What about a circle of ringworm on his scalp, he was
is an important factor to consider when asked some questions for a data base. Which of
obtaining a sputum specimen for culture? his environmental factors most likely
contributed to the childs acquisition of
1. A copious amount must be collected.
ringworm?
2. Sputum collected must not be diluted.
1. He rides a public bus to and from school each
3. It should be coughed up from deep in the day and likes to sit behind the driver so they
lungs. can talk about baseball.
4. The specimen must be refrigerated 2. He has a pet kitten that stays outside during
immediately. the day but comes inside to sleep with him at
night.
3. An adult client with tuberculosis asks the nurse
if she needs to follow any special diet. Which 3. He loaned his baseball hat to a friend last
suggestion would be most appropriate for the week but hasnt gotten it back yet.
nurse to give? 4. He forgets to wash his hair sometimes, and
1. Eat a high-carbohydrate diet. his mother has to remind him.
2. Eat a low-calorie, low-protein diet. 7. When planning immediate postoperative care for
3. Eat frequent small, high-calorie meals. the adolescent with a Harrington rod insertion,
4. Consume only high-carbohydrate liquids. what would be the priority nursing focus?
1. Assessment of paralytic ileus.
4. An adult has been in the burn unit 3 days
2. Cast care and repair of rough edges.
following second- and third-degree burns of both
legs. The nurse plans to assess the client for 3. Neurological assessments.
indications of the complication that is the major 4. Vital signs and urinary output.
cause of death in this period and includes which
of the following? 8. When a child vomits a bright-red liquid several
hours after a tonsillectomy, the nurse needs to
1. Monitor arterial blood gases (ABGs) daily.
determine whether the child is bleeding from
2. Monitor intake and output every shift. the operative site. Which nursing action would
3. Monitor results of wound cultures daily. be most informative?
4. Monitor daily caloric intake. 1. Visualizing the posterior throat with use of a
tongue depressor and flashlight.
5. An adult is admitted to the surgical ICU
2. Asking if the child had received red Koolaid
following a left adrenalectomy. An IV containing
for oral intake during the last hour.
hydrocortisone is running. The nurse planning
care for him knows it is essential to include 3. Taking vital signs, including the blood
which of the following nursing interventions at pressure, and checking oral mucous
this time? membranes for color changes.
1. Monitor blood glucose levels every shift to 4. Examine the blood to see if the membrane
detect development of hypo- or from the operative site is present.
hyperglycemia.

786 NCLEX-RN Review


53155_09_PT 05_p786-805.qxd 2/27/09 6:31 PM Page 787

9. The nurse has been teaching the family of a 3. First-degree heart block.
child with croup about emergency care. 4. Frequent bursts of tachycardia.
Which statement made by the parent indicates
that teaching was effective? 14. When caring for an elderly client it is important
1. If he wakes up coughing a barky cough, Ill to keep in mind the changes in color vision that
try sitting in a steamy bathroom with him. If may occur. What colors are apt to be most
he isnt better in an hour, Ill bring him to the difficult for the elderly to distinguish?
hospital for an aerosol of epinephrine. 1. Red and blue.
2. If the X-ray shows no swelling of the 2. Blue and gold.
epiglottis, we can probably go back home and 3. Red and green.
use the humidifier there.
4. Blue and green.
3. Symptoms of breathing hard, inward
movement of the ribs and neck with 15. Which statement by the client who has Type 2
breathing, and a continuous loud breathing diabetes mellitus requires additional teaching on
noise, are usual signs of spasmodic croup foot care?
and can be treated at home. 1. I should use a mirror to examine all surfaces
4. If he has an episode of loud, labored of my feet for cuts, cracks, or redness.
breathing and retractions, becomes 2. I should wear shoes that fit well and allow
frightened, sweaty, and thrashes around, then room for my toes to wiggle.
falls asleep, the croup attack is over. 3. I should use a bath thermometer to ensure
10. The nurse is assisting a child with congestive that my bath water is between 85 and 90
heart failure (CHF). Which of the following before I step into the tub.
would the child be least likely to manifest? 4. I should remove corns and calluses by using
1. Weakness and fatigue. the special medicated pads available at the
drugstore.
2. Dyspnea.
3. Tachycardia. 16. An elderly client is diagnosed as having sick
4. Oliguria. sinus syndrome and is being prepared for the
insertion of a demand pacemaker. What
11. A woman is scheduled for radiation therapy explanation will be provided to the client on
following a mastectomy. What symptoms can how the pacemaker works?
the nurse expect the client to report? 1. It senses changes in blood pressure.
1. Increased energy after treatment. 2. It stimulates the SA node at 60 beats per
2. Increased appetite after treatment. minute.
3. Skins changes at radiation site. 3. It beats when there is decreased coronary
4. Diarrhea. blood flow.
4. It senses the heart rate and starts a beat as
12. The nurse is assessing an elderly client who needed.
wears glasses and a hearing aid and is in
generally good health. What is a common theme 17. Which information would be most accurate
in the physical assessment and evaluation of the when describing pacemakers to a client who is
elderly client? to receive a pacemaker?
1. The elderly are living a shorter period of 1. Batteries are no longer necessary.
time. 2. Todays pacemakers are smaller than earlier
2. Reserve capacity is diminished. models.
3. Changes are usually related to disease. 3. The generator will be implanted in the upper
4. Clients with good health habits experience arm.
few age-related changes. 4. Modern pacemakers can be inserted in the
clients room.
13. The nurse is performing a cardiovascular
assessment on an elderly client. What findings 18. Which information should the nurse include in
would be expected? the discharge teaching plan of a client who had a
1. A bounding radial pulse. pacemaker implanted?
2. An early systolic murmur. 1. Remember to take all medications as
directed.

COMPREHENSIVE PRACTICE TESTS 787


53155_09_PT 05_p786-805.qxd 2/27/09 6:31 PM Page 788

2. Avoid sudden changes in temperature. diseases would the nurse explain that routine
3. Keep the pacemaker insertion site covered. childhood immunizations protect against?
4. Follow a low-cholesterol diet carefully. 1. Poliomyelitis, Haemophilus influenzae type
B, and mononucleosis.
19. The nurse is teaching a client about symptoms of 2. Measles, mumps, rubella, and herpes
pacemaker failure. Which symptoms would be simplex.
excluded in the teaching? 3. Diphtheria, tetanus, and Calmette-Gurin
1. Nausea. bacillus.
2. Syncope. 4. Poliomyelitis, Haemophilus influenzae type
3. Dizziness. B, and pertussis.
4. Palpitations.
25. The mother of a child in well-baby clinic asks
20. The client has a 2-year history of back pain and the nurse which immunizations contain live
sciatica. Which of the following is he least likely virus. What is the nurses best response?
to report having been included during 1. MMR and varicella.
conservative therapy for his back problem? 2. Hib and PPV.
1. Analgesics. 3. DTaP and IPV.
2. Enzymes. 4. DTaP and Hib.
3. Muscle relaxants.
4. Anti-inflammatory agents. 26. A 6-month-old child is seen in well-baby clinic.
The child has had the routine immunizations up
21. What is a correct statement about an objective to this point. At this visit, which immunizations
approach for eliciting the severity of pain from a should the nurse expect to administer?
client? 1. IPV.
1. Asking the client to describe the pain on a 2. MMR.
010 scale, record the information, and base 3. DTaP.
future assessments on it. 4. Smallpox.
2. Asking the client to compare the current pain
experience to that of previous experiences. 27. What nursing intervention should be included
3. The pain experience is a subjective one and prior to electroconvulsive therapy (ECT)?
not amenable to a standardized assessment 1. Providing an opportunity for the client to ask
approach. questions and express concerns about ECT.
4. The best approach is to medicate the client 2. Telling the client that it is not helpful to
sufficiently to control the pain. concentrate on the therapy.
3. Reassuring the client that ECT is no worse
22. What assessment finding would be expected in a than having a venipuncture.
client with polycythemia?
4. Telling the client she will recover completely
1. Pallor. as a result of ECT.
2. Tachycardia.
3. Leg pain with exercise. 28. The nurse is discussing electroconvulsive
4. Shortness of breath. therapy (ECT) with a client who asks how long it
will be before she feels better. How soon will the
23. Which of the following actions should the nurse nurse state the beneficial effects of ECT occur?
take in assisting an above-the-knee amputee who 1. 1 week.
is 2 weeks post-op and experiencing phantom 2. 3 weeks.
pain? 3. 4 weeks.
1. Provide and encourage client activities. 4. 6 weeks.
2. Keep the client on bed rest.
3. Tell the client the pain will disappear in one 29. Nursing assessment before electroconvulsive
week. therapy (ECT) is aimed at establishing
4. Instruct the client to ignore the pain. parameters that reflect the clients mental and
physical status. Which assessment is excluded
24. A mother brings her 6-month-old daughter to the in the assessment before ECT therapy?
well-baby clinic for her regular checkup. Which 1. Activity level.
2. Bowel habits.

788 NCLEX-RN Review


53155_09_PT 05_p786-805.qxd 2/27/09 6:31 PM Page 789

3. Pain tolerance. 35. An adult who has had an abdominal perineal


4. Sleep habit. resection asks the nurse when he can expect his
bowel function to return. Which answer by the
30. What side effects would the nurse expect to be nurse would be most appropriate?
present following electroconvulsive therapy 1. Upon returning to the floor from the post
(ECT)? recovery unit.
1. Cardiac arrhythmias, elevated SED rate, and 2. After voiding once.
fractures of the spine. 3. In approximately 6 hours.
2. An increase in norepinephrine at synapses of 4. In 2 to 3 days.
the brain, an increase in MAO platelet levels,
and euphoria. 36. An elderly female client is dying from heart
3. Slowing of electrical impulses in the brain, failure. The clients daughter only cries when
temporary amnesia, and confusion. alone with the nurse, not in front of her mother.
4. Mild organic changes in the neurons of the Which statement made by the daughter indicates
brain, dementia, and mild aphasia. to the nurse that the daughter is adjusting to the
dying process her mother is experiencing?
31. The nurse is planning care for a client who has 1. Im so depressed. I try to do my best but I
had a bowel resection. It is important for the find myself crying a lot of the time.
nurse to assess the client for which complication 2. My mother isnt dying. She has so many
after this type of surgery? relatives to live for.
1. Atelectasis. 3. I really dont know what I would do if
2. Parotitis. mother died.
3. Temporary ileus. 4. We dont talk about death. Its a morbid topic.
4. Transient ischemia.
37. The nurse is caring for an adult client who had
32. The nurse is to irrigate a nasogastric tube every an abdominal resection. On the fourth
2 hours. Which solution should the nurse select postoperative day, the clients wound dehisces.
to irrigate the tube? What is the appropriate nursing intervention at
1. Normal saline. this occurrence?
2. Tap water. 1. Cover the wound with sterile, moist saline
3. Ringers lactate. dressings.
4. Half-strength peroxide. 2. Approximate the wound edges with tape.
3. Irrigate the wound with sterile saline.
33. To prevent postoperative pulmonary 4. Hold the abdominal contents in place with a
complications, which of the following will be sterile, gloved hand.
most effective in helping a client who has had
abdominal surgery to breathe deeply? 38. An adult client has been experiencing double
1. Oxygen via nasal prongs. vision and frequent headaches. Which
2. Incentive spirometry. diagnostic procedure most likely would be used
to confirm the presence of a brain tumor?
3. Periodic ambulation.
1. A CT scan.
4. Verbal encouragement.
2. A myelogram.
34. An adult has received albumin to treat ascites 3. Skull X-rays.
secondary to cirrhosis. What action will the 4. A lumbar puncture.
nurse perform to evaluate if the albumin has
been effective? 39. The nurse is caring for an adult admitted with a
1. Assess whether pedal edema has diagnosis of a brain tumor. Shortly after her
decreased. admission, the client suffers a seizure. What will
2. Observe whether the client is less short of the nurses initial intervention be directed
breath. toward?
3. Measure the clients abdominal girth. 1. Controlling the seizure.
4. Check lab study results for an increase in 2. Protecting the client.
serum albumin. 3. Restraining the client.
4. Reducing circulation to the brain.

COMPREHENSIVE PRACTICE TESTS 789


53155_09_PT 05_p786-805.qxd 2/27/09 6:31 PM Page 790

40. Following a craniotomy, the client asks the 45. A newly admitted client with a conversion
nurse why a bone flap is necessary. What disorder says he cannot move his legs. What is
explanation does the nurse give for the purpose the best nursing response?
for removing the bone flap? 1. The physical tests and examinations state no
1. Allow for the insertion of an ICP bolt. physiological reason for your paralysis.
2. Accommodate postoperative brain 2. Let me help you out of bed to the
swelling. wheelchair. I will show you where the dining
3. Allow free flow of fluid into the room is. Dinner is served at 5:30 P.M. Ill be
Jackson-Pratt (JP) drain. telling you more about the typical routine
4. Permit reoperation if necessary as access later.
will be easier. 3. Ill plan to have your meals served to you in
bed. Because of your physical problem you
41. The nurse is caring for a client who had a will receive special privileges.
craniotomy performed this morning. What is the 4. You are here to get an understanding of how
importance of positioning the clients head? your physical symptoms related to the
1. Maintain a patent airway. conflicts in your personal life. Maybe
2. Facilitate venous drainage. you should reflect on this awhile and
3. Provide for client comfort. Ill be back in one hour to discuss it
with you.
4. Prevent hemorrhage from the suture line.
46. Which of the following assessments made by the
42. Following craniotomy, which of the following
nurse would be essential in understanding
measures is contraindicated for postoperative
behavior of a client with a conversion
pulmonary toilet?
disorder?
1. Coughing.
1. Physical symptoms are not under voluntary
2. Deep breathing. control.
3. Turning. 2. Physical symptoms are under voluntary
4. Suctioning. control but without intent to reduce
secondary gain.
43. Following a craniotomy, what is the rationale for
3. Physical symptoms are experienced as a
giving glucocorticoid dexamethasone
means to manipulate others to meet
(Decadron)?
narcissistic needs.
1. It creates a feeling of euphoria, which is
4. Physical symptoms are produced through
beneficial in the early postoperative
purposeful means to reduce anxiety and
period.
maintain dependency.
2. It promotes excretion of water, which aids in
reducing ICP. 47. After a young woman witnesses a traumatic
3. It enhances venous return and thus reduces vehicle accident, she suddenly reports changes
ICP. in her vision and claims to be developing
4. It reduces cerebral edema, thus reducing blindness. A conversion disorder is diagnosed,
ICP. when no physical problems are present. Which
of the following responses by family members
44. An adult client is diagnosed as having indicate to the nurse that they understand their
psychogenic amnesia. The nurse would find daughters symptoms?
which of the following symptoms during the 1. Shes afraid to get involved as a witness of
assessment? the event, so she claims to be blind.
1. Client states he feels detached from his 2. Shes trying to avoid her civic
body. responsibilities, so shes manipulating the
2. Client states he can recall some things but situation and being childish.
not everything. 3. Seeing the accident was very traumatic for
3. Client states he cant move his arm since he her.
saw a man killed. 4. Perhaps the physical examinations arent
4. Client states hes told he does things that he true. Maybe glass splinters are in her eyes
cant remember. and are too small to be seen.

790 NCLEX-RN Review


53155_09_PT 05_p786-805.qxd 2/27/09 6:31 PM Page 791

48. Which of the following would best indicate to 3. Validate eradication of the infection.
the nurse that a depressed client is improving? 4. Provide an opportunity for sexual counseling.
1. Reduced levels of anxiety.
2. Changes in vegetative signs. 54. A client has newly diagnosed Type 1 diabetes
and asks when she will have to test her urine for
3. Compliance with medications.
ketones. For what condition will the nurse state
4. Requests to talk to the nurse. this action needs to be done?
49. In assessing a client for posttraumatic stress 1. She is overhydrated.
disorder (PTSD), which symptoms would the 2. She begins to gain weight.
nurse perceive as key in the clients response to 3. The glucometer reading is abnormal.
trauma? 4. Her blood glucose level is more than
1. Emotional numbing and detachment 240 mg/dL for 6 hours.
followed by irritability, anxiety,
aggressiveness, and hyperalertness. 55. The nurse is discussing ketones with a newly
2. Depression and social withdrawal. diagnosed client with Type 1 diabetes. In
answering the clients question about how
3. Intrusive, hyperactive behavior and use of
ketones will affect her, the nurse should base the
alcohol to soothe symptoms.
answer on which concept?
4. Drug-seeking behavior and sexual
1. The client with diabetes is no different from
promiscuity as a means to cope.
others in the capacity to handle ketones.
50. The nurse is talking with a young female client 2. Ketones overpower the clients adaptive
in the health clinic who is concerned she may mechanisms.
have a sexually transmitted disease. What reason 3. Most clients with diabetes are allergic to the
does the nurse provide for the delayed treatment by-products of ketone metabolism.
of the majority of STDs? 4. It is impossible to predict the reaction to
1. The client is embarrassed. ketones.
2. Symptoms are thought to be caused by
something else. 56. Which special precaution must the nurse take
when assisting a client with self-monitoring of
3. Symptoms are ignored.
blood glucose?
4. The client never has symptoms.
1. Give the client a machine for his use only.
51. A female client tells the nurse that her boyfriend 2. Wear gloves when performing the test.
has told her he has gonorrhea and they had their 3. Rinse the lancet between uses.
last sexual experience three days ago. How long 4. Recalibrate the glucometer before
should the nurse tell the client to expect each use.
symptoms from the initial infection?
1. 2 to 5 days. 57. An adult clients insulin dosage is 10 units of
2. 5 to 7 days. regular insulin and 15 units of NPH insulin in
the morning. What will the nurse state as the
3. 1 to 2 weeks.
first insulin peak?
4. 2 to 3 weeks.
1. As soon as food is ingested.
52. What organism is linked with up to 90% of 2. In 2 to 4 hours.
cervical malignancies and may be linked to 3. In 6 hours.
other genital cancers? 4. In 10 to 12 hours.
1. Neisseria gonorrhoeae.
2. Chlamydia trachomatis. 58. Dietary teaching for a client with Type 1 diabetes
includes information on the glycemic impact
3. Human papilloma virus.
of a meal. Which statement by the client
4. Herpes simplex virus. indicates she has a good understanding of the
teaching?
53. The nurse is teaching a client about the
treatment for gonorrhea. What explanation does 1. Foods high in protein raise blood sugar
the nurse provide on why follow-up cultures are rapidly.
taken after treatment? 2. Simple sugars or carbohydrates cause a
1. Evaluate for complications. predictable rise in blood sugar.
2. Check the labs work.

COMPREHENSIVE PRACTICE TESTS 791


53155_09_PT 05_p786-805.qxd 2/27/09 6:31 PM Page 792

3. The protein, fat, and carbohydrate 64. The nurse is assessing an adult client admitted
composition of a meal affect the blood in ketoacidosis. What would be the expected
glucose level. condition of the clients skin?
4. Dairy beverages contain lactose, which 1. Clammy.
dramatically increases the need for insulin. 2. Flushed.
3. Diaphoretic.
59. Teaching a client who is insulin-dependent
will include guidelines for managing sick days. 4. Silky.
What is the recommended treatment for an
65. The nurse is caring for an adult client who is
insulin dependent diabetic if nausea is
admitted in diabetic ketoacidosis. The client was
present?
diagnosed 10 months ago. This is the first
1. Take the prescribed insulin. episode of ketoacidosis since the client
2. Go to the emergency department. was diagnosed. What focus should be
3. Administer regular insulin only. discussed?
4. Take nothing by mouth if vomiting. 1. An extremely poor prognosis.
2. The clients noncompliance.
60. An adult with Type 1 diabetes tells the clinic
3. Reinforcement of client teaching.
nurse that she plans to accompany her husband
on a business trip. When traveling, the client can 4. The potential for a long, painful, chronic
use which food or beverage as a substitute for a disorder.
delayed meal?
66. The nurse is doing discharge teaching with an
1. Diet cola. adult client who had diabetic ketoacidosis. What
2. Raisins. reminder should the client be given?
3. A candy bar. 1. The symptoms of ketoacidosis can vary;
4. A glass of wine. therefore, all changes in status should be
monitored.
61. An adult with Type 1 diabetes tells the nurse 2. Weight loss and fatigue are early symptoms of
that she would like to lose 15 pounds. What ketoacidosis.
would be the best way for the client to lose
3. Headache is a serious diagnostic sign of
weight?
ketoacidosis.
1. Increase her insulin dosage.
4. In ketoacidosis mucous membranes will be
2. Reduce calories and walk daily. pale.
3. Do an aerobic exercise program daily.
4. Restrict all carbohydrates from diet. 67. In assessing adult clients for early signs of
cancer, which of the following findings reported
62. The nurse is caring for a adult client who has to the nurse would indicate a priority for
been taking insulin for 8 months. Which follow-up?
diagnostic study is the most valuable in 1. Bowel movements twice a day for the past
evaluating long-term management of a diabetic 5 years.
client? 2. Monthly breast self-exam.
1. A 2-hour postprandial test. 3. Lingering cough 1 week after a cold.
2. A 6-hour glucose tolerance test. 4. Mole that has become larger in the past
3. A glycosylated hemoglobin test. 4 weeks.
4. The diary of glucometer test results.
68. The nurse is caring for the mother of a newborn.
63. An adult client known to have diabetes is What action by the mother indicates to the nurse
brought to the emergency department with that more teaching is needed?
complaints of fever, vague abdominal pain, 1. Keeps the cord exposed to the air.
nausea, and vomiting for the past several days. 2. Washes her hands before sponge bathing her
For what sign of ketoacidosis would the nurse be baby.
particularly observant?
3. Washes the cord with water at each diaper
1. Polyuria. change.
2. Abnormal reflexes. 4. Checks the cord daily for bleeding and
3. Increased thirst. drainage.
4. Mental deterioration.

792 NCLEX-RN Review


53155_09_PT 05_p786-805.qxd 2/27/09 6:31 PM Page 793

69. What is recommended to minimize discomfort 3. Use of accessory muscles during inspiration.
and embarrassment when the nurse is assessing 4. Presence of barrel chest and dyspnea.
an adolescent girl?
1. Make sure a parent is present. 75. Which statement by the client indicates the
2. Provide a gown and private area. discharge teaching for the client diagnosed with
pulmonary embolus is effective?
3. Examine two adolescents at the same time.
1. I am going to use a regular-bristle
4. Postpone the exam until the adolescent is
toothbrush.
older.
2. I will avoid being around large crowds.
70. A 10-month-old child is brought to the clinic for 3. I will take enteric-coated aspirin if I have a
the first time. During the assessment interview, headache.
the mother states that her baby is allergic to eggs. 4. I will drink extra fluids while on long trips.
The child will need testing before receiving
which immunization? 76. What should the nurse include in the care plan
1. DTaP. for a client with sundowner syndrome regarding
2. Smallpox vaccine. his room environment during his sleeping hours?
3. OPV. 1. Keep the room brightly lit.
4. MMR. 2. Use subdued lighting.
3. Keep the room dark with a night light.
71. Which of the following interventions would be 4. Ask the client how he is most comfortable.
appropriate to add to the plan of care for a client
diagnosed with cancer, who is receiving 77. The nurse is caring for a client who is having a
radiation and chemotherapy? panic attack. Which symptom will the client be
1. Provide air sprays to mask odors. least likely to exhibit?
2. Administer Demerol IM for complaints of 1. Bradycardia.
pain. 2. Sweating.
3. Encourage rinsing sore mouth with 3. Chest pain.
commercial mouthwashes. 4. Fear of going crazy.
4. Avoid fresh fruit and vegetables.
78. The nurse is caring for a man who has angina.
72. What should the nurse do when an elderly client He complains of chest pain. For what reason is
with sundowner syndrome becomes mildly nitroglycerin given?
disoriented? 1. Slows and strengthens the heart rate.
1. Ignore the disorientation. 2. Assists smooth muscles to contract.
2. Prepare a normal saline IV. 3. Increases venous return to the heart.
3. Turn off the lights in the room. 4. Reduces both preload and afterload.
4. Remind him where he is and why he
is there. 79. To combat the most common adverse effects of
chemotherapy, what medication would the
73. A young male is admitted to the emergency nurse administer?
department suffering from a gunshot wound. 1. Antibiotic.
What assessment finding would be of most 2. Antiemetic.
concern to the nurse?
3. Anticoagulant.
1. Nausea.
4. Anti-inflammatory.
2. Headache.
3. BP 104/54. 80. Discharge teaching for an adult client with
4. Tracheal deviation. angina includes a complete review of
nitroglycerin usage. After opening, what time
74. When assessing the client with the diagnosis of frame does the nurse tell the client that the
chronic obstructive pulmonary disease (COPD), nitroglycerin may be used in?
which data would require the nurse to take 1. 1 week.
immediate action? 2. 1 month.
1. Large amounts of thick white sputum. 3. 4 months.
2. Oxygen via nasal cannula set on 8 liters. 4. 6 months.

COMPREHENSIVE PRACTICE TESTS 793


53155_09_PT 05_p786-805.qxd 2/27/09 6:31 PM Page 794

81. What advice should the nurse give an adult 86. To prevent complications for a client who has
client who takes a sublingual nitroglycerin tablet developed thrombocytopenia secondary to
without relief of pain? radiation therapy, what instruction will the
1. Go to the emergency department. nurse provide to the client?
2. Take another tablet sublingually. 1. Brush the teeth with a hard bristle
3. Take two more tablets orally. brush.
4. Double the strength of the next dose. 2. Shave with an electric razor.
3. Continue with sports activities.
82. Which statement indicates the need for further 4. Continue with intramuscular pain
teaching for the client diagnosed with sleep medications.
apnea?
1. Im trying to lose weight and stop smoking. 87. A laboring client has just been told that she will
2. The continuous airway pressure prevents the be delivering her baby by cesarean birth because
collapse of my airway. of a contracted pelvis. To ensure a positive
outcome for the parents, what will be given the
3. Using the CPAP at night will help me stay
highest priority by the nurse?
awake during the day.
1. Keeping the woman clean and dry.
4. Im glad they found out I have sleep apnea
from all the X-rays they took of my mouth. 2. Keeping the woman informed.
3. Escorting the husband to the waiting room.
83. What would be an expected assessment finding 4. Maintaining comfort.
in a client with iron-deficiency anemia?
1. Bradycardia. 88. Following circumcision of a 1-day-old infant,
2. Jaundice. what is the most effective strategy for ensuring
urinary elimination?
3. Hunger.
1. Feeding the infant.
4. Fatigue.
2. Having nonconstrictive gauze over the
84. The nurse has been teaching self-care to an adult penis.
client who is receiving external radiation 3. Keeping the infant on his side.
therapy to the facial area. Which of the following 4. Checking for first void postcircumcision.
client actions indicates a need for further
teaching? 89. An adult client is scheduled for gallbladder
1. Sitting next to his wife and holding hands. surgery in 4 weeks. During his preadmission
2. Applying lotion and powder to the radiated office visit he states that he smokes two packs of
site. cigarettes a day. What instructions will the nurse
give to the client about this activity?
3. Gently cleaning mouth and teeth with a
sponge. 1. Demonstrate how to use an incentive
spirometer.
4. Resting between activities.
2. Try to decrease smoking.
85. The nurse is caring for a 28-week-premature 3. Stop smoking now.
infant on a ventilator. Which action is essential 4. Join a nonsmokers group and reschedule
for the nurse to take? surgery.
1. Assess the oxygen saturation of the infant
once per 8-hour shift using a pulse oximeter. 90. An adult had abdominal surgery 2 days ago.
2. Notify the physician if the oxygen saturation Which of the following statements would
falls below 95% on the pulse oximeter, and indicate to the nurse that normal bowel
plan to increase the oxygen settings. peristalsis is returning?
3. Notify the physician if the oxygen saturation 1. My belly seems bigger this afternoon.
is continually above 95%. 2. I keep burping.
4. Suction the infant every 2 hours around the 3. I passed some rectal gas today.
clock. 4. I feel like vomiting.

794 NCLEX-RN Review


53155_09_PT 05_p786-805.qxd 2/27/09 6:31 PM Page 795

91. A client was admitted with a diagnosis of left- 3. Feeding self-care deficit related to poor food
sided cerebrovascular accident (CVA) and choices.
placed on cardiac monitoring. There were 4. Altered health maintenance related to
indiscernible P waves and the QRS complex was inadequate health teaching.
normal but the ventricular rhythm was irregular
with a rate of between 100 and 180 beats per 95. An adult client with hemiplegia and right
minute. What is the supraventricular arrhythmia hemianopia expresses concern about how to
associated with CVA called? operate the vacuum cleaner and washing
1. Wide complex junctional tachycardia. machine at home. Which of the following
2. Sinus tachycardia. nursing diagnoses is appropriate for this
client?
3. Second-degree AV block referred to as
Wenckebach. 1. High risk for injury related to right-sided
weakness.
4. Atrial fibrillation initiated by an ectopic
focus outside the SA node. 2. Impaired home maintenance management
related to paralysis and visual impairment.
92. An adult is admitted with a diagnosis of 3. Altered health management related to altered
probable Graves disease with thyrotoxic crisis. mobility and sensory perception.
Which of the following assessments will provide 4. Hygiene and self-care deficit related to
the nurse with the best measure of the severity inability to operate appliances.
of the clients disease?
1. Blood glucose. 96. Which assessment is the most important to
2. Heart rate. include in an older client with a family history
of diabetes mellitus?
3. Urine output.
1. Palpation of pedal pulses and auscultation
4. Blood pressure.
for carotid bruit.
93. An adult with diabetes mellitus has a 2. Palpation of liver and observation of sclera.
glycosolated hemoglobin (hemoglobin A1c) 3. Palpation of spleen and pulse oximetry.
reading of 10% and the blood glucose reading is 4. Palpation of abdomen and auscultation of
100 mg/dL. How should the nurse proceed? breath sounds.
1. Ask the client for daily blood glucose
monitoring records, and ask the client to 97. The nurse is teaching a male client to examine
describe self-care practices. his testicular area for abnormal masses. Which
2. Congratulate the client on the excellent level of the following would be included in the
of diabetic control achieved and suggest that appropriate assessment of the scrotum?
the client continue the present regimen. 1. Observing that the right side is lower than the
3. Observe the client for the signs and left.
symptoms of diabetic ketoacidosis 2. Including the inguinal and femoral areas for
and refer the client to the physician bulges.
immediately. 3. Requesting he inhale during the exam.
4. Observe the client for signs and symptoms of 4. Holding the penis down during the exam.
hypoglycemia and provide orange juice
immediately. 98. The nurse is discussing risk factors for
osteoporosis with a middle-aged client. Which
94. An elderly client with Type 2 diabetes mellitus assessment finding indicates a risk factor for
lives alone. Admission assessment includes the osteoporosis?
following information: eats one meal a day 1. The client has lactose intolerance and does
(mostly carbohydrate foods); wears poor not drink milk, but eats cheese and dark
fitting shoes and often goes barefoot. Which green vegetables.
nursing diagnosis would be appropriate if the 2. The client is 5 feet 2 inches tall and weighs
client is receptive to a change in these 90 pounds.
behaviors?
3. The client participates in aerobics classes
1. Impaired home maintenance management twice weekly.
related to declining health.
4. The client has been taking estrogen since her
2. Activity intolerance related to pedal pain. ovaries were removed 2 years ago.

COMPREHENSIVE PRACTICE TESTS 795


53155_09_PT 05_p786-805.qxd 2/27/09 6:31 PM Page 796

99. The nurse is performing a breast examination for 100. The nurse is assessing an adult client who has
a nonpregnant woman during her annual had a kidney transplant. Which of the following
gynecological visit. The nurse will be concerned assessment findings would indicate to the nurse
if which one of the following findings is that the client might be developing acute
present? rejection of the kidney?
1. Nipple discharge. 1. Oliguria.
2. Tail of Spence. 2. Temperature range of 37.2C (98.6F)
3. Soft axilla. 37.7C (99.8F).
4. Consistent patterns of veins. 3. Blood pressure of 110/76.
4. Serum Creatine of 0.8 and BUN at 12.

Answers and Rationales for Practice Test 5

ANSWER RATIONALE NP CN CL SA
#1. 4. A negative pressure room is always a private room, in which the Pl Sa/2 Ap 1
air is vented to the outside, ensuring that contaminated air cannot
escape from the room into other parts of the facility. This type of
room usually also has a separate side room in which to enter
that has a sink and personal protective equipment (PPE) is housed.
#2. 3. The client should be instructed the day before how to cough and Im Ph/7 Ap 1
obtain the specimen, as the best time secretions are easily obtainable
is early in the morning. The client may brush the teeth first, but
not swallow. The specimen will be taken to the lab where it will
be placed on an appropriate culture medium and incubated for
at least 24 hours.
#3. 3. The goal will be to maintain normal weight or allow for weight gain. Pl Ph/5 Ap 1
Weight lost may have occurred during the disease process, so
small, frequent meals would be tolerated best.
#4. 3. Infection, which begins in the burn wound and travels to the Pl Ph/7 Ap 1
bloodstream, is the primary cause of death in persons who survive
the first few days following extensive burns. Cultures are taken to
monitor the colonization of the wound by organisms, alerting staff
to early detection of infection and treatment with antibiotics. ABGs
would indicate a need for intubation; fluid overload is monitored
to prevent congestive heart failure; and a high-calorie, high-protein
diet is ordered.
#5. 1. Hydrocortisone promotes gluconeogenesis and elevates blood Ev Ph/6 Ap 1
glucose levels. Following adrenalectomy the normal supply of
hydrocortisone is interrupted and must be replaced to maintain the
blood glucose at normal levels. The medication will be changed
when the client is able to take it by mouth and will be necessary
for 6 months to 2 years until his remaining gland recovers. Wound
care will be done at a more appropriate time.
#6. 2. It is common for children to acquire ringworm from their pets, An He/3 Ap 4
especially if the pets are outside during the day and only spend
the night in the house. The pet will also need to be assessed for
ringworm lesions. The child is most likely leaning forward while
talking with the bus driver (not placing head on headrest); the

796 NCLEX-RN Review


53155_09_PT 05_p786-805.qxd 2/27/09 6:31 PM Page 797

ANSWER RATIONALE NP CN CL SA
child who has his hat should be checked for ringworm. Not
washing the hair is not the cause for the current infection.
#7. 3. Any sign of paresthesia or paralysis needs to be reported promptly. Pl Ph/7 Ap 4
Spinal nerve damage is a risk and may require emergency removal
of the instrumentation. It is also imperative to monitor kidney
perfusion which could result in acute renal failure, and would be
a good second choice. A paralytic ileus is not an immediate
complication, but if it occurs, treatment would be nasogastric
intubation. Cast care is addressed 8 to 12 days postoperatively,
when the child is removed from the immobilization of the
Stryker frame bed.
#8. 1. Because the operative site can be visualized, the nurse should Im Ph/7 Ap 4
look for oozing of blood using good lighting and a tongue
depressor. The surgical membrane does not pull apart until
410 days after surgery.
#9. 1. Laryngotracheobronchitis (LTB) will fatigue the child unless the Ev Ph/7 Ap 4
airway is opened more. In worsening signs of respiratory distress,
epinephrine is given to cause vasoconstriction and a reduction
of airway swelling. The child will require monitoring in a hospital
setting for side effects and any rebound signs and symptoms. No
swelling of the epiglottis is present with LTB; breathing hard
describes stage II of the progression of symptoms of LTB, which
necessitates being observed in a croup tent along with an
oximeter to indicate oxygenation status.
#10. 4. Due to the administration of diuretics in CHF, oliguria is usually As Ph/8 Ap 4
not seen. All the others are expected symptoms seen in CHF.
#11. 3. Skin changes would include thinning, altered pigmentation, Pl Ph/8 Ap 3
ulceration, or necrosis. The client should be taught only to use
creams/lotions approved by the radiation oncologist. Fatigue and
anorexia are expected after treatment, and diarrhea would be
expected if the intestinal area were being radiated.
#12. 2. Body organs experience a decrease in functional capacity as the As He/3 Ap 1
individual ages; the cardiac and respiratory system are
particularly vulnerable to decline. Changes are always related to
disease and good health habits may help prevent disease.
#13. 2. Cardiac valves thicken and stiffen with age which can be a cause As He/3 An 1
of the common systolic murmur heard in the elderly. The pulse
may become weaker with age and tachycardia would not be
constant, but have frequent bursts.
#14. 4. The elderly have poor blue-green discrimination because of the An He/3 K 1
difference in wavelengths. This is due to the yellowing of
the lens with age.
#15. 4. The client did not hear the importance of seeing a podiatrist Ev Ph/7 Ap 1
for the treatment of problems of the foot. Special shoes may be
necessary to prevent recurrence of these problems.
#16. 4. A pacemaker is an electrical device that provides repetitive Im Ph/7 Ap 1
electrical stimuli to the heart muscle for the control of heart rate.
It is set at a rate determined individually for each client and is

ANSWERS AND RATIONALES FOR PRACTICE TEST 5 797


53155_09_PT 05_p786-805.qxd 2/27/09 6:31 PM Page 798

ANSWER RATIONALE NP CN CL SA
either inhibited by ventricular response or initiated by the atria.
It is not able to sense coronary blood flow.
#17. 2. The pulse generators are smaller in size, but still require a battery, Im Ph/7 Ap 1
usually a lithium cell that lasts 812 years. The generator is a
smooth, lightweight case containing a tiny computer and a
battery which is implanted under the collarbone in a pocket
underneath the skin. The insertion is done by
fluoroscopic control in a cardiovascular laboratory
or operating room.
#18. 1. Many clients will require cardiac medication following the Im Ph/7 Ap 1
insertion of a pacemaker. Avoidance of sudden changes in
temperature and covering the site is not necessary. The diet will
depend on the clients serum cholesterol level.
#19. 1. Nausea is the only symptom listed that is usually not Ev Ph/7 An 1
associated with pacemaker failure.
#20. 2. Conservative treatment would not include enzyme therapy. As Ph/7 An 1
Chemonucleolysis is a surgical procedure that injects an
enzyme in the nucleus pulposus of the intravertebral disc.
#21. 1. A numerical scale can be used to compare it with future Im Ph/7 Ap 5
assessments. As the goal of pain management is to use as
little medication as possible to control pain, alternative methods
should be used in conjunction with drug-induced analgesia.
#22. 4. Polycythemia is an excess of red blood cells, in which blood As Ph/7 Ap 1
viscosity is increased, making circulation through capillary
beds sluggish. The decreased oxygenation of the tissues leads
to shortness of breath, headache, flushing of the face, and
paresthesias. Leg pain with exercise is associated with
intermittent claudication.
#23. 1. Phantom pain may occur several months after amputation and Im Ph/7 Ap 1
the caregiver should acknowledge the pain as real, providing
medication and client activities.
#24. 4. Routine childhood immunizations are given to prevent Pl He/3 Ap 4
poliomyelitis, diphtheria, pertussis, tetanus, Haemophilus
influenzae type B, measles, mumps, and rubella. There are no
vaccines for mono or herpes simplex; the Calmette-Gurin
bacillus (BCG) is given in some countries to protect against
tuberculosis.
#25. 1. The measles, mumps, and rubella vaccine (MMR) and varicella An He/3 Ap 4
(chickenpox) contain live virus.
#26. 3. DTaP is given at 2, 4, 6, 18 months and between 4 and 6 years Im He/3 Ap 4
of age. IVP is given at 2, 4, and 18 months and between
4 and 6 years of age. MMR is given at 15 months and between
11 and 12 years of age. The smallpox vaccine is no longer given
since the diseases eradication in the United States in 1972.
#27. 1. The opportunity to ask questions helps to reduce anxiety and Im Ps/4 Ap 2
misinformation while enlisting the client and familys support
and cooperation in the treatment. The treatment often results
in significant reduction in depression but the results cannot
be guaranteed.

798 NCLEX-RN Review


53155_09_PT 05_p786-805.qxd 2/27/09 6:31 PM Page 799

ANSWER RATIONALE NP CN CL SA
#28. 1. Treatments are administered at intervals of 48 hours, with Ev Ps/4 Co 1
beneficial effects usually evident after the first several
treatments, which is within 1 week.
#29. 3. Pain is not associated with ECT, but activity level, As Ps/4 An 2
bowel habits, and sleep habits and/or depression provide
insight into the clients physical and mental status.
#30. 3. Common side effects of ECT include slowing of As Ph/7 Co 1
electrical impulses in the brain and temporary
confusion and amnesia.
#31. 3. An ileus is a result of bowel manipulation during surgery. As Ph/7 An 1
Bowel sounds need to be assessed as measures will need to be
taken if they are not present within the first few days following
surgery. Atelectasis is possible after any surgery, however an ileus
is a complication that is associated with abdominal surgery.
Parotitis is an inflammation of the parotid gland and a TIA is a
episode of cerebrovascular insufficiency.
#32. 1. Normal saline will not cause a loss of sodium when it is removed Pl Ph/7 Ap 1
by suction. Tap water would cause the cells to swell, and sodium
would be lost when fluid is suctioned. Ringers lactate (or
Lactated Ringers) is used intravenously to replace electrolytes;
hydrogen peroxide is not indicated for internal use.
#33. 2. Incentive spirometry (IS) will help the client to breathe deeply by An Ph/7 Ap 1
providing visual reinforcement to the breathing effort. Ambulation
will be encouraged, but the IS can be performed on an hourly basis.
#34. 3. Because ascites is the accumulation of fluid in the abdominal Ev Ph/7 An 1
cavity, measuring the girth before and after treatment will be the
most effective way to determine success of treatment.
#35. 3. Peristalsis (bowel motility) usually occurs within 6 hours after Ev Ph/7 An 1
surgery and food absorption is tolerated. All segments of the
bowel may take up to 3 to 4 days to achieve full motility. The
nurse should assess that bowel sounds and flatulence is present
before liquids or food is given.
#36. 1. Depression is a response to impending loss, chronic illness, Ev Ps/4 Ap 2
or death. The daughter recognizes this as a healthy response in
adjusting to this life event. The other statements show the
daughter is not adjusting.
#37. 1. This action prevents infection and drying of the wound until the Im Ph/8 Ap 1
physician arrives and decides on a plan of action.
#38. 1. Computerized tomography (CT scan) is a highly accurate An Ph/7 An 1
neurological diagnostic test that provides definitive information
on presence, size, and location of brain tumors. A myelogram is
associated with the spinal cord, skull x-rays would only view the
bones, and a lumbar puncture would examine the cerebrospinal fluid.
#39. 2. Safety is the first priority of care during a seizure. Protect Im Ph/7 Ap 1
head/body from hitting other objects and turn client to the
side if vomiting occurs.

ANSWERS AND RATIONALES FOR PRACTICE TEST 5 799


53155_09_PT 05_p786-805.qxd 2/27/09 6:31 PM Page 800

ANSWER RATIONALE NP CN CL SA
#40. 2. A bone flap allows for accommodation of postoperative brain An Ph/8 An 1
tissue swelling. It can be removed, preserved in a freezer, and
re-implanted several months later. The ICP bolt is inserted through
the skull without removing a bone flap; a JP drain is inserted
during surgery; removal only occurs should postoperative
brain swelling is present.
#41. 2. Elevating the head of the bed 3045 promotes venous return and Im Ph/7 Ap 1
improves cerebrospinal circulation, thus minimizing the most
serious potential problem of increased intracranial pressure. This
is the one case in which maintenance of a patent airway is
secondary to facilitating venous drainage.
#42. 1. Coughing or sneezing will elevate intra-abdominal or intrathoracic Pl Ph/7 Co 1
pressure, preventing venous return from the cranial vault, resulting
in increased intracranial pressure. Suctioning would be performed
through the nose for no longer than 1015 second intervals.
#43. 4. Decadron has an anti-inflammatory action that is effective in An Ph/6 An 5
reducing cerebral edema, which reduces ICP.
#44. 2. Selective amnesia is recalling some things but not everything. As Ps/4 An 2
Depersonalization is feeling detached; conversion reaction is
associated with no organic reason for the inability to move the arm;
multiple personality disorder may be the reason for not
remembering whats been said.
#45. 2. Explanation of normal routine reduces anxiety and decreases Im Ps/4 Ap 2
secondary gain. It is too early in the relationship to uncover the
conflict underlying the conversion.
#46. 1. The disorder is a loss or alteration in physical functioning due As Ps/4 An 2
to psychological causes, but the symptoms are not produced on a
conscious level. Symptoms are involuntary.
#47. 3. This response shows an understanding of the traumatic event Ev Ps/4 Ap 2
and the conflict she feels in terms of the consequences of
being a witness to it.
#48. 2. Vegetative signs such as insomnia, anorexia, psychomotor Ev Ps/4 Co 2
retardation, constipation, diminished libido, and poor
concentration are biological responses to depression.
Improvement in these signs indicates a lifting of the depression.
#49. 1. Psychic numbing and detachment are followed by somatic and An Ps/4 Ap 2
cognitive symptoms. Depression and social withdrawal can
occur in other disorders; alcohol is common in response to PTSD,
but the described behavior is typical of manic depressive; and
sexual activity is not a symptom of PTSD.
#50. 4. Chlamydia is the #1 STD and clients are asymptomatic. Many Ev He/3 An 3
females with Neisseria gonorrhea and syphilis are
asymptomatic also.
#51. 1. The usual incubation period between infection and onset An Ph/7 K 3
of symptoms is 25 days.
#52. 3. Human papilloma virus (genital warts) has been strongly linked An He/3 An 3
to cervical malignancies, as a shift from the Pap test to the HPV

800 NCLEX-RN Review


53155_09_PT 05_p786-805.qxd 2/27/09 6:31 PM Page 801

ANSWER RATIONALE NP CN CL SA
test is being recommended to be the primary detection method
for cervical cancer.
#53. 3. A repeat culture is important to validate eradiation of Ev He/3 An 3
disease, thus preventing spread of infection.
#54. 2. The urine should be tested for ketone bodies when there is a Ev Ph/7 An 1
persistent increase in the blood sugar. Ketones in the urine signal
that the body is breaking down fat stores for energy.
#55. 2. A lack of insulin stimulates ketoacidosis. The hyperglycemia of An Ph/7 An 1
ketoacidosis produces large fluid losses (polyuria) and the
client becomes thirsty (polydipsia). The body is unable to
compensate for the renal losses and dehydration results.
#56. 2. Gloves, and any additional PPE, should be worn any time Pl Sa/2 Ap 1
contact with blood or body fluids is anticipated.
#57. 2. Regular insulin is classified as rapid acting and will peak Ev Ph/6 An 1
2 to 4 hours after administration. The second peak will be
6 to 12 hours after the administration of NPH insulin. This is
why a snack should be eaten mid-morning and also 34 hours
after the evening meal.
#58. 3. The glycemic index is the result of research that revealed Ev Ph/5 Ap 1
that factors other than chemical composition impact the blood
glucose level. The protein and fat composition of a meal appears
to delay gastric emptying, resulting in a slower rise in blood sugar.
Protein foods raise blood sugar slowly, simple sugars cause a rapid
rise, and lactose does not increase the need for insulin.
#59. 1. Sick-day procedure includes taking all prescribed insulin as usual Pl Ph/6 Ap 1
to prevent diabetic ketoacidosis (DKA). The physician will need
to be notified if vomiting is consistent and fluids cannot be
tolerated. Sick-day rules consist of liquids or soft foods.
#60. 2. Nonperishable foods such as raisins are the most appropriate Pl Ph/5 Ap 1
food to serve as a substitute for a delayed meal. One-quarter cup
provides one fruit exchange, containing 10 grams of carbohydrate
and 40 calories. The other choices would either elevate the blood
sugar too quickly followed by a rebound fall or does not provide
any sustained carbohydrates.
#61. 2. Clients who need to lose weight should have a reduced-calorie Pl Ph/5 Ap 1
diet plan. Exercise must be carefully selected and used in
combination with diet control. Increased insulin might cause
hypoglycemia, and restricting carbs would not balance the diet
plan. Fifty to sixty percent of calories must be derived from
carbs and help maintain blood glucose levels.
#62. 3. This test is also referred to as the A1c, which shows a pattern of Ev Ph/7 An 1
blood glucose levels over a 3-month period. The other methods are
used for a more current glucose reading.
#63. 4. The buildup of ketone bodies causes a decline in tissue perfusion, As Ph/8 An 1
resulting in hypoxia. Early detection of cerebral hypoxia is achieved
by assessing orientation of person, place and time, along with simple
questions and commands. Identifying the subtle changes in these

ANSWERS AND RATIONALES FOR PRACTICE TEST 5 801


53155_09_PT 05_p786-805.qxd 2/27/09 6:31 PM Page 802

ANSWER RATIONALE NP CN CL SA
areas can facilitate the early diagnosis and initiate treatment of
ketoacidosis and prevent progression to coma. Slow reflexes are a
late sign of ketoacidosis. Polyuria and polydipsia would be
expected to be present.
#64. 2. Ketoacidosis causes dehydration resulting in flushed, dry skin. As Ph/8 Co 1
Clammy and diaphoresis is seen in hypoglycemia.
#65. 3. Because this is the first time in 10 months that the client Pl Ph/7 Ap 1
has had any problems, reinforcement of client teaching is
the priority to avoid future episodes.
#66. 2. Symptoms include fatigue, weight loss, polydipsia, Im Ph/7 Ap 1
polyuria, nausea, vomiting, flushed membranes, and
change in the level of consciousness (LOC).
Physical examination reveals dehydration and
fruit odor of the breath, with a blood sugar greater
than 250 mg/dL.
#67. 4. The seven warning signals of cancer are: As He/3 Ap 1
C: Change in bladder/bowel habits
A: A sore that does not heal
U: Unusual bleeding or discharge
T: Thickening or presence of lump
I: Indigestion/difficulty swallowing
O: Obvious changes to moles or warts
N: Nagging cough or hoarseness that lingers
#68. 3. Wetting the cord keeps it moist and predisposes it to Ev He/3 Ap 3
infection. Air exposure helps to dry the cord.
#69. 2. Meeting with the adolescent alone, and providing gown and Im He/3 Ap 4
private area would be recommended to ensure privacy
needs are met.
#70. 4. The measles, mumps and rubella (MMR) vaccine is the only An He/3 Ap 4
choice that contains eggs. Any child who is allergic to eggs
should receive a skin test before receiving the vaccine. If the
child tests positive, the vaccine would be given in very small
doses at 20-minute intervals with adrenaline available
should anaphylaxis occur.
#71. 4. If the clients WBC is , 1000/mm3, fresh fruits and vegetables Pl Ph/7 An 1
may harbor bacteria and increase the risk of infection.
Refrain from sprays which may stimulate nausea/vomiting;
avoid IM infections to prevent intramuscular bleeding; alcohol
content in mouthwashes will potentiate breakdown.
#72. 4. When the clients cognitive level declines, the nurse should Im Ps/4 Ap 1
provide clear and simple explanations and cues to minimize
confusion and disorientation. Turning the lights off may produce
more confusion during this time of day when the occurrence is
most frequent (late afternoon or early evening).

802 NCLEX-RN Review


53155_09_PT 05_p786-805.qxd 2/27/09 6:31 PM Page 803

ANSWER RATIONALE NP CN CL SA
#73. 4. The wound in the chest wall may cause air to be trapped and As Ph/7 An 1
not be expelled during expiration, thereby causing the lung to
collapse and the heart, vessels, and trachea to shift toward the
unaffected side of the chest. This would compromise respirations
and circulatory function.
#74. 2. A client with COPD is usually treated with low-flow oxygen As Ph/6 An 1
delivery of 2 L/min to avoid depressing the respiratory drive in
some clients. Nasal cannula is used for oxygen flow up to 6 liters,
and then a different type mask would be used. The other choices
are expected to be present in a client with COPD.
#75. 4. Extra fluids will help avoid hemoconcentration if there is a fluid Ev Ph/7 An 1
deficit while traveling or in warm weather. The client may be
taking warfarin (Coumadin) for several weeks following the
incident and should not take aspirin, or any NSAIDs, while on
the ordered anticoagulant. A soft-bristle toothbrush will prevent
gum injury/bleeding.
#76. 3. The lighting in the room should be adjusted for normal Pl Sa/2 Ap 1
circadian rhythm. Darkening the room will signify bedtime
and a night light is a safety measure to prevent falls.
#77. 1. A panic attack stimulates the sympathetic nervous system, An Ps/4 An 2
resulting in increased heart rate, chest pain, anxiety, and choking.
#78. 4. Nitrates can cause venous pooling, resulting in reduced blood An Ph/6 Ap 1
return to the heart. This reduces preload. The systemic arterial
bed is also relaxed, causing a fall in blood pressure. The result is
decreased afterload. Digitalis therapy slows and strengthens the
heart.
#79. 2. Antiemetics are used for nausea and vomiting, which are common Pl Ph/6 Ap 1
side effects of chemotherapy, and may persist from 24 to 48 hours.
Antibiotics are used for infection, anticoagulants are used in blood
coagulation, and anti-inflammatory medications are used for
inflammatory problems.
#80. 4. Nitroglycerin may be used up to 6 months after the vial is opened, Im Ph/6 Ap 1
but should be kept in a cool, dark place.
#81. 2. Up to three sublingual nitroglycerin tablets should be taken at Ev Ph/6 Ap 5
5-minute intervals before the client seeks further medical
intervention for the relief of pain.
#82. 4. Diagnosis of sleep apnea is based on clinical features plus Ev He/3 An 1
polysomnographic findings from a sleep study. Using the CPAP
at night while sleeping will prevent the nocturnal hypoxemia;
this may result in cardiac problems.
#83. 4. Fatigue is often the only symptom of the condition in its early As Ph/7 Ap 1
stages. Inadequate iron stores result in inadequate production of
red blood cells. This decreases the amount of oxygen carried to all
tissues. When anemia is severe enough to affect heart rate, the
change will be seen in tachycardia, as the heart is trying to
compensate to increase the amount of oxygen to reach tissues.
Jaundice may be seen in sickle cell anemia, hemolytic
anemia or hepatic diseases.

ANSWERS AND RATIONALES FOR PRACTICE TEST 5 803


53155_09_PT 05_p786-805.qxd 2/27/09 6:31 PM Page 804

ANSWER RATIONALE NP CN CL SA
#84. 2. Lotions and powders are not applied as they may cause skin Ev Ph/7 Ap 1
irritation. Clients receiving external radiation are not radioactive;
oral hygiene is done to prevent irritation; fatigue is a common
side effect.
#85. 3. If the infant continually has high pulse oxygenation readings Im Ph/7 Ap 4
and other vital signs are stable, the infant may be ready to be
weaned to lower oxygen settings. Hyperoxemia increases the
potential for retrolental fibroplasia. The oxygen saturation is
monitored on an hourly basis; check connections first if
pulse ox drops; suctioning would be on a prn basis.
#86. 2. All precautions to prevent bleeding must be taken: using electric Im Ph/7 Ap 1
razors, soft bristle toothbrush, stopping sports activities, and
discontinuing IM injections (due to decreased platelets
from radiation).
#87. 2. A couple may be prepared for cesarean delivery at the last minute. Im He/3 Ap 3
The nurse should make the birth experience a positive one by
making collaboration between client and staff a priority.
Maintaining comfort is important, but given the new planned
action, anxiety will be reduced with providing information.
#88. 1. The infant has had feeding restrictions prior to the circumcision Pl Ph/7 Ap 3
so feeding him afterwards will satisfy his nutritional needs and
provide him with fluid to help him void. The gauze is to prevent
irritation/friction from covers on the penis; side-lying position
has no influence on the urethra to expel urine; the first void is
an evaluate measure and does not lead to voiding as feeding would.
#89. 3. All clients should be encouraged to stop smoking 46 weeks Pl Ph/7 Ap 1
before surgery, with explanation to the client that the cessation
will decrease the chances of respiratory complications
during and after surgery.
#90. 3. Passage of flatus is usually a sign of positive peristaltic activity. Ev Ph/7 Ap 1
It is after this finding that the client may start eating.
#91. 4. Atrial fibrillation is the most rapid of atrial dysrhythmias. An Ph/8 An 1
The atria beat chaotically at rates of 350 to 600 beats per minute.
Cardiac output is reduced due to loss of atrial kick. Mural
thrombi tend to form, resulting in pulmonary or cerebral
thrombosis. Atrial fibrillation is characterized by no definite
P wave and an irregular ventricular rhythm.
#92. 2. The metabolic rate and body temperature are elevated in As Ph/7 Co 1
Graves disease (hyperthyroidism). The clients heart rate
increases to provide additional oxygen required to meet
metabolic demands. Thyrotoxic crisis (thyroid storm) is a
state of extreme hyperthyroidism characterized by a heart rate
over 130 beats per minute. Because it can be fatal if untreated,
it must be recognized immediately.
#93. 1. Glycosylated hemoglobin is formed each time blood glucose An Ph/7 Ap 1
levels are elevated and remains attached to the red blood cell for up
to 120 days. Although the reading is normal this day, the Hgb A1c
reflects the average levels of diabetic control over the past

804 NCLEX-RN Review


53155_09_PT 05_p786-805.qxd 2/27/09 6:31 PM Page 805

ANSWER RATIONALE NP CN CL SA
three months. Asking the client for more information about daily
self-care will reveal the cause of the poor control. The Hgb A1c
values should range between 4 and 6%. The normal reading at
the present rules out ketoacidosis or hypoglycemia.
#94. 4. This would be the appropriate diagnosis as the client expresses An Ph/7 Ap 1
a desire to learn better health management techniques. Poor
nutrition and unsafe foot care habits place a client with diabetes
at risk for complication of diabetes, such as infection, vascular
disease, hyperglycemic hyperosmolar nonketotic coma,
nephropathy, neuropathy, and retinopathy.
#95. 2. Hemiplegia is one-sided paralysis. Hemianopia (or hemianopsia) An Ph/7 Ap 1
is the loss of one-half of the visual field in each eye. The diagnosis
would be appropriate when a client expresses concern about the
ability to maintain the home properly.
#96. 1. The vascular complications of diabetes may be so mild that As He/3 Ap 1
detection of the disease is delayed. Pedal pulses may be absent
in the presence of peripheral vascular disease of the lower limbs.
The presence of a carotid bruit may indicate partial occlusion of the
carotid artery related to arteriosclerosis. Other organs typically
affected in diabetes include the eye, kidney, and nerves.
#97. 2. In teaching the client to examine the scrotal area, it is important to Im He/3 Ap 1
also inspect the inguinal and femoral areas for bulges that indicate
herniation. The left testicle is usually lower than the right; inhaling
would make palpation more difficult; the penis is held up
during the exam.
#98. 2. Women who are short and slender are at increased risk and An He/3 Ap 1
should be encouraged to increase their source of calcium, perform
weight-bearing exercises, and take estrogen if prescribed.
#99. 1. A woman who is not pregnant or lactating who has nipple As He/3 Ap 3
discharge should be further evaluated. The Tail of Spence is the
tissue of the mammary gland that extends into the axillary region
and becomes enlarged premenstrually and during lactation. The
other choices are normal findings.
#100. 1. Acute rejection of a kidney transplant can be differentiated from As Ph/7 An 1
chronic rejection. Oliguria or anuria are signs of acute rejection.
The other choices are values within normal limits.

ANSWERS AND RATIONALES FOR PRACTICE TEST 5 805


53155_09_PT 06_p806-823.qxd 2/26/09 8:16 AM Page 806

Practice Test 6

1. The nurse is caring for a middle-age client whose to clamp off the suction so the patient can leave
blood pressure over the past 2 months has ranged the room, what would be the nurses response?
between 140/88 and 148/94. When explaining 1. Ok, but have him back as soon as possible to
the condition to the client, which limit would be be put back on suctioning.
viewed as normal systolic pressure? 2. We will have to call the doctor and get an
1. 110 mm Hg. order before we can do that.
2. 120 mm Hg. 3. It should be clamped already to maintain
3. 135 mm Hg. lung pressure.
4. 145 mm Hg. 4. No, I cannot clamp the suction tube; let me get
a portable suction that you can take with you.
2. An adult client is diagnosed with mildly
elevated blood pressure. According to the Joint 7. A adult who has mild hypertension asks if blood
Committee which of the following studies would pressure medicine is needed. When is
be excluded in evaluating this client? pharmacologic therapy usually added to the
1. An ECG. therapeutic regime?
2. A urinalysis. 1. When symptoms appear.
3. Serum calcium levels. 2. Any time a client is noncompliant with diet
4. White blood cell count. therapy.
3. When the history indicates the client is at
3. An adult clients blood pressure (BP) has risk for cardiovascular disease.
ranged between 142/90 to 148/96. She is 55 lb 4. When the difference between systolic and
overweight for her height and age. What lifestyle diastolic blood pressure is greater than
change should the nurse suggest for this client? 60 mm Hg.
1. Lose weight as rapidly as possible.
2. Plan a gradual exercise program. 8. The nurse is teaching a client who has mild
hypertension. What is the major goal of treatment
3. Begin vigorous exercise immediately.
in a teaching plan for an adult with hypertension?
4. Avoid exercise until your blood pressure is
1. Control of the disease.
within the normal range.
2. A healthier lifestyle.
4. A client diagnosed with primary hypertension 3. Avoidance of renal complications.
asks the nurse what is the cause. What is the 4. Restoration of her prior state of health.
nurses best response?
1. Atherosclerosis. 9. The gynecologist has referred a young couple to
2. Renal disease. the infertility clinic. They have been married
for 5 years, are in their late 20s, and have been
3. Diabetic vessel changes.
trying to conceive a child for over a year. What
4. The cause is unknown. is a primary tool in the initial assessment of
this infertile couple?
5. The nurse is discussing dietary teaching with an
overweight woman who has mild hypertension. 1. A complete history and physical.
What should the nurse advise the woman to do? 2. A psychological examination.
1. Eat fewer vegetables. 3. An explanation of all surgical options.
2. Reduce calcium intake. 4. Hormonal assessment of ovulatory function.
3. Read contents labels on processed foods.
10. What is the purpose of a semen analysis for a
4. Add a water softening system for drinking couple in an infertility clinic?
water.
1. Chromosomal disorders.
6. A patient with a chest tube needs to leave the 2. Hormone levels.
floor for another procedure. When the person 3. Sperm motility.
comes to transport the patient he asks the nurse 4. Temperature.

806 NCLEX-RN Review


53155_09_PT 06_p806-823.qxd 2/26/09 8:16 AM Page 807

11. The nurse in an infertility clinic is discussing out of traction for feeding. What is the nurses
tests that will be done to evaluate an infertile best response?
couple. What does the nurse provide in the 1. Yes, but only for feedings.
teaching plan that is a simple method of 2. Yes, this is intermittent traction.
determining ovulatory function?
3. No, parenteral feedings will be given.
1. Laparoscopy.
4. No, continuous traction is necessary to bring
2. Cervical mucous testing. the femoral head fully into position.
3. Postcoital sampling.
4. Testing urine LH levels. 17. The nurse is caring for an infant who has had a
hip spica cast applied. What should the nurse do
12. When the cuff pressure from an endotracheal to keep the cast free of urine and stool?
tube is too high, above 20 mm Hg, which of the 1. Use a Bradford frame.
following would be excluded in possible 2. Use a Denis Browne splint.
complications?
3. Catheterize the baby prn.
1. Risk of aspiration.
4. Insert an indwelling catheter.
2. Tracheal bleeding.
3. Pressure necrosis on the trachea. 18. The nurse is caring for an infant who is in a hip
4. Ischemia of the tracheal lining. spica cast. Which action is least appropriate for
the nurse to take to prevent skin irritation at the
13. A woman who is being evaluated and treated for edges of the babys cast?
infertility is instructed to graph her ovulatory 1. Give meticulous skin care.
function by taking her basal body temperature 2. Petal the edges with moleskin.
upon awakening each morning. What other
3. Use baby powder around the edges.
information should be documented at this
time? 4. Tuck plastic wrap under the edges.
1. All food consumed. 19. Adult clients with acute pancreatitis often have
2. Her daily weight. H2 blockers or antacids ordered. What is the
3. The temperature of the room. primary purpose of giving these drugs to a client
4. The presence of a sore throat. with pancreatitis?
1. Coat the stomach to protect it from the effects
14. An infant who is fitted with a Pavlik harness is of bile reflux.
given home care instructions. Which of the 2. Reduce gastric pH to inactivate digestive
following would be excluded from the enzymes.
instructions?
3. Counteract excessive gastric acid secretion
1. Turn her every 3 to 4 hours. stimulated by release of gastrin from the
2. Keep her off the affected side. damaged pancreas.
3. Watch for signs of skin breakdown. 4. Raise gastric pH to decrease stimulation of
4. Give her sponge baths, not tub baths. excessive release of pancreatic enzymes.

15. After a newborn has spent 6 weeks in the Pavlik 20. In caring for an adult client with varicose veins,
harness, an open reduction is planned. Awaiting which of the following measures is most
surgery, the newborn is in Bryants traction to essential for the nurse to include in the plan
bring the femoral head fully into position. What of care?
position of traction will demonstrate that the 1. Discussing cosmetic techniques to improve
mother understands the placement? appearance.
1. The hips will be flexed at a 45 angle. 2. Discouraging stair climbing or walking.
2. The buttocks will be flat. 3. Teaching activities to promote circulation.
3. The buttocks will be slightly elevated off the 4. Encouraging activities that cause venous
bed. stasis.
4. The knees will be bent.
21. After swallowing a dime, a 22-month-old toddler
16. An infant who has congenital hip dysplasia not is brought to the emergency room by her
responding to a Pavlik harness is placed in frightened mother. What assessment would alert
Bryants traction prior to surgery. The babys the nurse to the possibility of esophageal
mother asks the nurse if she can take the baby blockage?

COMPREHENSIVE PRACTICE TESTS 807


53155_09_PT 06_p806-823.qxd 2/26/09 8:16 AM Page 808

1. Dim breath sounds in upper right lobe. that would have indicated her mother had
2. Choking, gagging, wheezing, and osteoporosis. What is the nurses best
coughing. response?
3. Increased salivation, painful swallowing. 1. Did she experience cramps after
4. Inability to speak, cyanosis, and collapse. exercising?
2. Were her nails and hair brittle?
22. Which of the following manifestations indicates 3. Is she shorter now than she was in the
to the nurse that an infant needs further fluid past?
therapy for dehydration? 4. Had she been eating less and still gaining
1. Fontanel level with skull and sutures. weight?
2. Liquid, loose stools.
3. Specific gravity of 1.010. 28. When teaching about osteoporosis, the nurse
stresses the importance of prevention. What
4. Urinary output 12 mL/kg/hour.
vitamin would the nurse emphasize that the
23. The nurse is caring for an adult client with clients diet should include?
chronic venous insufficiency. He now has deep, 1. Vitamin A.
draining, foul-smelling ulcers on his legs. The 2. Vitamin D.
nurse can anticipate that the client is likely to be 3. Vitamin E.
given which of the following vitamins? 4. Vitamin K.
1. Vitamin A.
2. B complex vitamins. 29. The nurse is teaching a client about
3. Vitamin C. osteoporosis. The client reports all of the
following. Which should the nurse recommend
4. Vitamin D.
the client stop doing to help reduce the risk of
24. The treatment of an adult client with chronic osteoporosis?
venous insufficiency and severe leg ulcers 1. Smoking.
includes the application of a gelatin bandage 2. Overeating.
around the stasis ulcers. What is the correct term 3. Biting her nails.
for this bandage? 4. Skipping breakfast.
1. A Jobst stocking.
2. An Unnas paste boot. 30. The nurse has been teaching a client about
3. A specialized Ace bandage. factors to reduce the risk of development of
osteoporosis. Adding which food to her diet
4. A plaster of Paris bandage.
indicates that the client understands the role of
25. A client with chronic venous insufficiency is nutrition in preventing osteoporosis?
given discharge instructions. What activity will 1. Oatmeal.
he be told to avoid? 2. Peaches.
1. Walking. 3. Canned salmon.
2. Sexual intercourse. 4. Poached flounder.
3. Elevating the legs.
31. An elderly man was admitted for surgery for
4. Standing for long periods.
benign prostatic hypertrophy. Preoperatively
26. The nurse is discussing the prevention of he was alert, oriented, cooperative, and
osteoporosis with a group of adult clients. The knowledgeable about his surgery. Several hours
nurse explains factors required to keep bones after surgery, the evening nurse found him
strong. What information is excluded from the acutely confused, agitated, and trying to climb
nurses discussion? over the protective side rails on his bed.
What will be the most appropriate nursing
1. An adequate calcium intake.
intervention that will calm an agitated
2. Maintenance of a low weight. client?
3. Sufficient estrogen levels. 1. Limit visits by staff.
4. Weight-bearing exercise. 2. Encourage family phone calls.
27. A client whose mother has developed 3. Position in a bright, busy area.
osteoporosis asks the nurse if there was anything 4. Speak soothingly and provide quiet music.

808 NCLEX-RN Review


53155_09_PT 06_p806-823.qxd 2/26/09 8:16 AM Page 809

32. A young woman is admitted to the eating 38. The nurse is examining the feet of an older man
disorders clinic for treatment of bulimia. What is suspected of having peripheral arterial disease
the primary issue for the bulimia client? (PAD). Which finding indicates an inadequate
1. Delusions. nutritional supply to the feet?
2. Depersonalization. 1. Coarse body hair.
3. Fear and suspicion of others. 2. Muscle hypertrophy.
4. Poor impulse control. 3. Thick, ridged nails.
4. Rough, reddened skin.
33. The nurse is assessing a client with bulimia.
Which characteristic is least likely to be evident 39. When you read a clients chart and see that a
in the history? syngeneic bone marrow transplant is scheduled
1. Repeated crash dieting. in the morning, who would you automatically
2. Repeated weight fluctuations. know the donor is?
3. Rigorous exercise regimens. 1. The client.
4. Self-induced vomiting. 2. The clients identical twin.
3. A family member.
34. In planning care for a client with bulimia, the 4. A friend of the client.
nurse expects that the client may be given which
pharmacologic agent? 40. The nurse understands that a client with a long
1. An anticonvulsant. history of diabetes is at increased risk for
2. An antidepressant. developing peripheral arterial disease due to
what occurrence?
3. A major tranquilizer.
1. Hypoglycemic episodes.
4. A minor tranquilizer.
2. Capillary rupture.
35. The nurse is caring for a client admitted with 3. Early atherosclerotic changes.
severe diarrhea. Why should the nurse observe 4. Fluctuating levels of insulin.
this client for hyponatremia?
1. Sodium is concentrated in gastrointestinal 41. A client has peripheral arterial disease and long-
fluid. term diabetes mellitus. If the client develops
2. Water lost in diarrhea causes sodium to diabetic neuropathy, why is the risk of
follow it. complications of peripheral arterial diseases
increased because of diabetic neuropathy?
3. Diarrhea triggers renal mechanisms to waste
sodium. 1. It dilates peripheral vessels.
4. Hyponatremia occurs as a result of treatment 2. It decreases sensation.
for diarrhea. 3. It increases cardiac output.
4. It alters renal function.
36. A client with Type 1 diabetes mellitus is diagnosed
with peripheral arterial disease. How would the 42. The nurse is caring for a client with long-
nurse expect the client to describe his discomfort? standing diabetes. Which finding is most
1. Incapacitating. consistent with damage to the autonomic
2. Throbbing. nervous system?
3. Mild. 1. Flushed, warm extremities.
4. Aching weakness in the lower extremities. 2. Dry, cracked skin.
3. Absent pulses.
37. An older adult with a long history of Type 1 4. Burning sensation on the soles of the feet.
diabetes mellitus is being evaluated for
peripheral arterial disease. The nurse assesses 43. The nurse is teaching a client with diabetes
his feet after elevating them for 30 seconds and mellitus and peripheral arterial disease how to
notes the color is pale. How would the nurse care for his feet. Which instruction should the
correctly interpret the color? nurse include?
1. Normal. 1. Avoid deodorant soaps.
2. Dependent rubor. 2. Examine your feet weekly.
3. Pregangrenous. 3. Use only hot water.
4. Cadaveric pallor. 4. Soak your feet daily.

COMPREHENSIVE PRACTICE TESTS 809


53155_09_PT 06_p806-823.qxd 2/26/09 8:16 AM Page 810

44. The nurse is caring for a client who is scheduled 1. Having vitamin K available if bleeding
for surgery tomorrow. It is expected that he will occurs.
have patient-controlled analgesia (PCA) 2. Observing for hematomas at IV puncture
following surgery. Which statement by the sites.
nurse provides the primary reason for using the 3. Suggesting that the client use a soft bristled
PCA? toothbrush.
1. It is very cost effective. 4. Using an IV control device for drug
2. It requires less pain medication. administration.
3. It allows for families to assist in pain
management. 50. Two days after admission with deep venous
4. It allows clients to control their own pain. thrombosis (DVT), an adult client develops a
cough with slight hemoptysis and complains of
45. The nurse knows that which criteria is most shortness of breath and sharp pain under the
important in determining whether a client is a right shoulder blade. What will the
good candidate for PCA? ventilation/perfusion scan show if the client has
1. He is alert. a pulmonary embolism (PE) but no other
pulmonary disease?
2. He is not overweight.
1. Decreased ventilation; decreased perfusion.
3. His pain will be constant.
2. Decreased ventilation; normal perfusion.
4. His surgical procedure will be relatively
short. 3. Normal ventilation; decreased perfusion.
4. Normal ventilation; normal perfusion.
46. What information will the nurse provide that
explains why the possible side effect of 51. The nurse is caring for a small child. Child
respiratory depression is reduced using a PCA? abuse is suspected. Who does the nurse know
1. It eliminates peaks in serum drug levels. is most frequently the abuser of small
children?
2. It uses drugs without respiratory side effects.
1. Babysitter.
3. It requires very little medication for pain
relief. 2. Relative.
4. There are intervals when the client receives 3. Teacher.
no medication. 4. Casual acquaintance.

47. The nurse is caring for a client who is at risk for 52. In evaluating risk factors for child abuse, which
developing deep venous thrombosis. Which family would be at least risk for abusing?
nursing care measure is not appropriate? 1. Moves frequently.
1. Careful leg massages. 2. Owns their own home.
2. Elastic stockings. 3. Has experienced divorce.
3. Elevating the legs. 4. Has problems with chronic illnesses.
4. Leg exercises.
53. The nurse is assessing a 4-year-old girl who has
48. The nurse is caring for a client who has a deep been brought to the emergency room with a high
venous thrombosis. Which nursing care measure fever. Child abuse is suspected. Which test is
would be excluded in the care? least likely to indicate that the child has been
1. Nursing measures to help the client avoid sexually abused?
straining at stool. 1. A Pap smear.
2. Telling the client to avoid sudden 2. Urine culture.
movements. 3. Throat culture.
3. Assisting the client to dangle on the side of 4. Vaginal culture.
the bed 3 times a day.
4. Teaching the client to avoid bumping the legs 54. What should the nurse do when interviewing a
against other objects. child suspected of being sexually abused?
1. Ask leading questions.
49. A client with deep vein thrombosis is started on 2. Have the parents present.
heparin therapy. Which of the following actions 3. Have a security guard present.
would be inappropriate during heparin
4. Use the childs words to describe body parts.
administration?

810 NCLEX-RN Review


53155_09_PT 06_p806-823.qxd 2/26/09 8:16 AM Page 811

55. An adult client has a diagnosis of severe 60. What is the drug of choice for treatment of
hypovolemia. The medication administration Pneumocystis carinii pneumonia?
record (MAR) states that the client should 1. Amphotericin B (Fungazone).
receive a hypertonic solution. How should the 2. Ethambutol (Myambutol).
nurse respond to the order?
3. Pentamidine (Pentam).
1. Hold the solution until further clarification
4. Zidovudine (Retrovir).
from the physician.
2. Administer the solution as ordered. 61. The nurse is planning care for an adult client
3. It doesnt matter if the solution is hypertonic who has Pneumocystis carinii pneumonia and
or hypotonic, as long as the patient is getting AIDS. If this client were to participate in IV drug
the necessary fluids. use, why would he be at risk for infective
4. The nurse should question the order because endocarditis?
the client has too much fluid and should not 1. HIV-associated arrhythmias.
be getting any more. 2. Increased workload on the heart.
3. Resistance of bacteria to antibiotics.
56. A 4-year-old girl is brought to the emergency
room with a high fever. She is clinging to two 4. Introduction of bacteria into the bloodstream.
dolls and has them engaging in explicit sexual
62. The nurse is planning care for an HIV-infected
behaviors. What will be the nurses
drug abuser. Which goal is unrealistic?
interpretation of this activity?
1. Quitting the drug addiction.
1. The child is mimicking behavior seen
on TV. 2. Cooperating with unit goals.
2. She is acting out a personal experience. 3. Learning to clean drug equipment.
3. Such play is a healthy expression of sexual 4. Remaining for the full treatment course.
development.
63. The nurse is assessing a client who has infective
4. The child needs to be directed to more endocarditis secondary to AIDS. Which
appropriate play. symptoms indicate that the client is
experiencing endocarditis?
57. When child abuse is suspected, what is the least
appropriate nursing action to initiate? 1. A pronounced S1 and S2.
1. Take a wait-and-see position. 2. Chronic low-grade fever.
2. Call a local social service agency for help. 3. Tachycardia and hypertension.
3. Prevent the childs return to a dangerous 4. Shortness of breath and chest pain.
environment.
64. A 6-year-old is brought to the emergency
4. Confront the parent with security department unconscious after having been hit by
present. a car. Which of the following would be absent
from a baseline neurologic exam?
58. Which question is least useful in the assessment
of a client with AIDS? 1. Motor function.
1. Are you a drug user? 2. Visual acuity.
2. Do you have many sex partners? 3. Vital signs.
3. What is your method of birth control? 4. Level of consciousness.
4. How old were you when you became 65. A young adult has Type 1 diabetes mellitus. If
sexually active? she should plan to have a child, what will be of
primary importance for her to consider?
59. When planning care for the client with
Pneumocystis carinii pneumonia (PCP), what is 1. Perform a review of the dietary modifications
the nurse aware of? that will be necessary.
1. It is usually fatal. 2. Seek early prenatal medical care.
2. The client has few symptoms. 3. Look into adoption instead of conception.
3. It is highly contagious. 4. Knowing that pregnancy is a major health
risk to the mother.
4. Treatment is more successful now than in the
past. 66. A young woman who has Type 1 diabetes
mellitus is pregnant. She asks the nurse how

COMPREHENSIVE PRACTICE TESTS 811


53155_09_PT 06_p806-823.qxd 2/26/09 8:16 AM Page 812

much weight she can gain during her pregnancy. 3. Taping the catheter to the top of the
What is the nurses best response? clients leg.
1. 10 to 15 pounds. 4. Filling the balloon up with normal saline.
2. 25 to 30 pounds.
72. What would be a short-term goal (to be met in
3. Less than 40 pounds.
1 week following admission) planned by a nurse
4. The weight of the baby plus 3 pounds. for a delusional client?
67. The nurse is teaching a pregnant woman who is 1. Reduce the frequency and intensity of the
also diabetic about her diet during pregnancy. delusional thinking.
What statement conveys to the nurse that the 2. Verbalize why he uses delusions to deal with
client understands her dietary needs? life.
1. I will eat a low-protein, low-salt diet. 3. Communicate in only reality-oriented terms.
2. I will continue my normal intake of simple 4. Recognize his delusions as nonreality-based
carbohydrates. statements.
3. I will increase my water consumption.
73. The nurse and a severely depressed client
4. I will eat lots of fruits, vegetables, and grains. mutually plan a short-term goal regarding self-
esteem needs. Which of the following would be
68. The nurse is caring for a pregnant woman who is
appropriate to meet in 1 week?
also diabetic. What statement conveys to the
nurse that the client understands her blood 1. The client will be able to describe one
sugar levels during pregnancy? positive attribute about himself to the nurse.
1. I will keep my blood sugar between 80 to 2. The client will be able to attend and fully
110 mg/dL. participate in all groups and therapeutic
activities.
2. My blood sugar should stay within 150 to
200 mg/dL. 3. The client verbalizes to the nurse that he is
now able to solve his problems.
3. My goal should be between 200 and
250 mg/dL. 4. The client verbalizes to the nurse that he
feels good enough to run the next community
4. I will not allow my blood sugar to go over
meeting.
300 mg/dL.
74. An adult client states, That TV newsman is
69. The nurse is assessing a client with diabetes for
talking about me. The nurse recognizes the
signs and symptoms of hyperglycemia. Which
statement as what type of thought process?
symptom is least likely to be stated by a client
with hyperglycemia? 1. Thought broadcasting.
1. I am very tired. 2. Delusion of reference.
2. I am voiding more than normal. 3. Thought insertion.
3. I am very thirsty. 4. Delusion of persecution.
4. My bed needs to be remade, as I am 75. Which statement would indicate further
sweating so much. discharge teaching is required for a client who is
prescribed warfarin (Coumadin)?
70. A pregnant woman asks the nurse if there are
any special problems that she might encounter 1. I am going to have to keep coming back to
during labor and delivery because she has get my blood tested.
diabetes. Which condition will the nurse state 2. An electric razor would be better to shave
that may develop during the birth process? with.
1. Hypoglycemia. 3. I will use less salt when cooking my food.
2. Hyperglycemia. 4. I will contact my doctor if I see blood in my
3. Metabolic alkalosis. bowel movements.
4. Hyperosmolar nonketotic coma. 76. Which of the following behaviors indicates to
the nurse that the client with agoraphobia is
71. Which would be of the most importance when
improving?
inserting an indwelling urinary catheter?
1. Client is able to offer complaints to the boss
1. Putting the catheter bag on the lowest part of
regarding the workload.
the bed.
2. Client is able to travel five flights on an elevator.
2. Maintaining aseptic technique.

812 NCLEX-RN Review


53155_09_PT 06_p806-823.qxd 2/26/09 8:16 AM Page 813

3. Client is able to shop alone at a local mall 81. An 11-year-old girl is ready for hospital discharge
without intense anxiety. after being newly diagnosed with Type 1 diabetes
4. Client is able to resist washing hands after mellitus. Which statement by the child alerts the
touching a dirty object. nurse to do further teaching before the child
leaves?
77. A client has just completed the detoxification 1. I never know when my sugar is low, because
process and is due for discharge. His nurse must I dont feel any different, so I guess Ill really
evaluate his comprehension of the long-term have to use my blood glucose monitor on
nature of his addiction. Which statement schedule.
indicates the best understanding? 2. On my constant carb diet I can have a
1. I can have a social drink for special holidays dessert every day, as long as I eat it at the
without a problem. same time each day.
2. I must attend 90 AA meetings in 90 days. 3. I need to pinch up my skin for the injections
3. I know I must never drink alcohol again. because if the insulin gets into my muscle
4. Once I finish the 12 stages, Im cured. instead of fat it wont work right.
4. I think Ill have Mom get me some of those
78. An adult has been taking tricyclic medication for glucose tablets to carry with me, because I
a week with no improvement in mood. What is wont be tempted to eat them for snacks, like
the nurses best explanation for this situation? I would with Life Savers.
1. The client has been ordered an
antidepressant that is ineffective due to client 82. An adult has acute cholecystitis secondary to
insensitivity. cholelithiasis. Which factor in the history is
2. The client should consider electroconvulsive most often associated with cholelithiasis?
therapy for a more rapid change in 1. Low-fat diet for many years.
mood. 2. A period of unusually strenuous exercise.
3. The client needs to wait longer because drug 3. Use of oral hypoglycemic drugs.
onset takes 7 to 10 days. 4. Being a descendant of the Pima Indians.
4. The client requires a second opinion with
regards to her medical diagnosis and 83. An adult client was discharged following
treatment. treatment for partial and full thickness burns of
the upper body, and an elastic pressure garment
79. A client with a spinal cord injury develops the was prescribed. When examining the wounds a
signs and symptoms of autonomic dysreflexia. year later, how will the nurse document that the
What would be the nurses initial action? wounds are healing without complications?
1. Administer an analgesic to relieve the 1. Red, raised, and hard.
headache. 2. Pink, flat, and soft.
2. Instruct the client on preventive measures. 3. Hard, raised, and shiny.
3. Examine the rectum for a fecal mass. 4. Open, pink, and draining.
4. Sit the client up to lower the blood
pressure. 84. A child with head lice is seen in the clinic and
receives home care instructions. Which
80. A client who is scheduled for a bowel resection statement reflects understanding of the
tomorrow has just completed preoperative teaching?
teaching by the nurse. Which of the following 1. Im really embarrassed about this situation,
statements to the nurse indicates the client because our home isnt a dirty place.
needs further instruction on postoperative 2. I guess well have to wash all the towels and
care? sheets in the house, along with all your
1. I know Ill have pain after surgery, but I can clothes!
call the nurses for medicine. 3. Ill have to get seven bottles of this special
2. They will be taking my pulse and blood shampoo and use one every day for a week
pressure many times after the operation. on my childs hair. Then she can return to
3. The intravenous needle will come out in the school.
recovery room. 4. Im supposed to leave this special shampoo
4. Ill show you how I can deep breathe and on her for 10 minutes, rinse, and then comb
cough. out the nits with a fine-toothed comb.

COMPREHENSIVE PRACTICE TESTS 813


53155_09_PT 06_p806-823.qxd 2/26/09 8:16 AM Page 814

85. Knowing that a test for phenylketonuria (PKU) is 90. The nurse is administering CPR. Which is most
conducted on all babies in the United States, important for the nurse to evaluate to determine
what instructions does the nurse provide to the whether the procedure is being done effectively?
mother? 1. Feeling the carotid pulse during compressions.
1. Keep the infant NPO before the test. 2. Observing the chest rise and fall during
2. Maintain the infants feeding schedule. rescue breathing.
3. Request that it be done within 8 hours of 3. Monitoring arterial blood gases.
delivery. 4. Monitoring the electrocardiogram rhythm.
4. Give the infant water before the test.
91. The nurse is assessing an elderly client in a long-
86. The nurse has just completed education with term care facility. Which is a normal finding?
parents of a newborn recently diagnosed with 1. Deposits of melanin.
phenylketonuria. Which statement the parents 2. Thickening of the epidermis.
make indicates the best understanding of
3. Increase in hair follicles.
phenylketonuria?
4. Increase in subcutaneous tissue.
1. The child only needs to be on a special diet
until the age of 1 year. 92. An adult has had a left hip replacement. He is
2. The child must eat a diet low in now 3 days post-op. Which parameter should
phenylalanine. the nurse monitor to determine if the client is
3. There is medicine that the child can take to meeting goals related to the nursing diagnosis of
avoid being on any special diet. high risk for infection?
4. The child must avoid all foods that contain 1. Nutrition status.
phenylalanine. 2. Hemoglobin and hematocrit.
3. Vital signs every 4 hours.
87. The nurse has just completed teaching a woman
regarding the use of the diaphragm and is 4. Amount and character of drainage from
assessing her understanding. Which statement incision.
demonstrates correct learning by the client?
93. The nurse suspects a complication in a client
1. I will use Vaseline to lubricate the rim of the who is receiving peritoneal dialysis. Which of
diaphragm prior to insertion. the following observations would support this
2. I should get refitted for the diaphragm if I evaluation?
gain or lose more than 5 pounds. 1. Pain during the inflow of dialysate.
3. I should put spermicide only on the rim of 2. Occasional diarrhea.
the diaphragm before insertion.
3. Cloudy or opaque effluent.
4. I will leave the diaphragm in place at least
4. Clear or light yellow effluent.
6 hours after intercourse.
94. The clinic nurse is administering tuberculin skin
88. An elderly client has reported to a local health
tests. When administering the purified protein
department to receive a flu shot. The nurse
derivative (PPD), which angle and location will
should make which one of the following
the nurse select?
assessments prior to administering the flu
vaccine? 1. 90 angle into the deltoid.
1. Mental status. 2. 45 angle into the subcutaneous tissue of the
arm.
2. Gastrointestinal system.
3. 60 angle into the hypodermal space of the
3. Integumentary system.
gluteal.
4. Egg allergies.
4. 10 angle into the forearm.
89. While teaching self-testicular examination,
95. A client received a PPD 72 hours ago.
when does the nurse instruct as the best time to
Assessment finds an erythematosus circle of
perform the exam?
10 mm in diameter and an induration of 1.5 mm.
1. After a warm bath or shower. How will the nurse interpret these findings?
2. In the morning before getting out of bed. 1. Meaningless, as the test must be read at 24 and
3. At bedtime. 48 hours. The test will need to be repeated.
4. After exercise. 2. Possible exposure to tuberculosis because of
the size of the erythema.

814 NCLEX-RN Review


53155_09_PT 06_p806-823.qxd 2/26/09 8:16 AM Page 815

3. Positive for active tuberculosis because there Hardwire cardiac monitoring was instituted
is an induration and erythema. using lead sites for MCL1. What anatomical
4. There is no evidence of tuberculosis because point will the nurse identify to properly place
the induration is small. the electrodes?
1. McBurneys point.
96. A tracheostomy was performed and mechanical 2. Angle of Louis.
ventilation instituted on an adult. What will the
3. Suprasternal notch.
nurse include when performing tracheal
suctioning? 4. Costovertebral angle.
1. Wearing clean gloves, goggles, and a mask. 99. The nurse provides instructions on a low-fat,
2. Applying constant suction while inserting high-fiber diet. Which of the following food
the catheter. choices, if selected by the client, indicate an
3. Hyperoxygenating the client with 100% understanding of a low-fat, high-fiber diet?
oxygen only after the procedure is completed. 1. Tuna salad sandwich on whole wheat bread.
4. Applying intermittent suction and rotating 2. Vegetable soup made with vegetable stock,
the catheter as the suction catheter is drawn carrots, celery, and legumes served with
from the tracheostomy tube. toasted oat bread.
3. Chefs salad with hard-boiled eggs and fat-
97. The physician has ordered total parenteral
free dressing.
nutrition to be delivered through the central
venous line. When changing the tubing to 4. Broiled chicken stuffed with chopped apples
institute the TPN, the nurse should perform and walnuts.
which of the following activities to prevent the
100. An adult female asks the nurse why she should
occurrence of an air embolism?
have a mammogram. What is the nurses best
1. Cleanse the central line insertion site with response?
povidone-iodine ointment.
1. Mammograms can diagnose breast cancer
2. Wrap sterile Vaseline gauze around the hub with nearly 100% accuracy.
of the open central venous line while priming
2. Every sexually active woman needs to have
the TPN line.
a mammogram, since there is a correlation
3. Clamp the central venous line while between sexual intercourse and breast
connecting the primed TPN administration set. cancer.
4. Place an alcohol wipe over the open end of 3. You are 38 years old. This is the appropriate
the central venous catheter while preparing time to have a baseline mammogram done.
to insert the primed TPN tubing.
4. The dye, or contrast medium, used when
98. An adult was admitted to the coronary care unit you have a mammogram helps the radiologist
(CCU) for complaints of substernal chest pain of see the difference between a tumor and
one-hour duration unrelieved by nitrogylcerin. a cyst.

Answers and Rationales for Practice Test 6

ANSWER RATIONALE NP CN CL SA
#1. 2. Current upper limit for normal systolic blood pressure is Im He/3 Co 1
120 mm Hg.
#2. 4. A white blood cell count is not necessary, as it is associated with An He/3 An 1
an infection. The Joint National Committee recommends: an
electrocardiogram; urinalysis; blood glucose; hematocrit;
serum potassium, creatinine, calcium, lipid profile.

COMPREHENSIVE PRACTICE TESTS 815


53155_09_PT 06_p806-823.qxd 2/26/09 8:16 AM Page 816

ANSWER RATIONALE NP CN CL SA
#3. 2. Exercise would be the best choice, but one that will not be Pl He/3 Ap 1
too vigorous for the cardiovascular system. Rapid weight
loss would not be healthy.
#4. 4. Primary hypertension is a disorder in which the cause cannot An He/3 Co 1
be identified. The other conditions cause hypertension.
#5. 3. Salt and fat in the diet should be limited. Processed foods Im He/3 Ap 1
usually have a high sodium and fat content. Vegetables and
calcium are encouraged, as they are low in sodium and fat
and help to reduce blood pressure. If a water softening
system has added sodium, it should be avoided.
#6. 4. Clamping can result in a tension pneumothorax. The chest Im Ph/7 Ap 1
tube should not be clamped unless ordered by the
physician.
#7. 3. First-line drugs used in the management of hypertension Ev Ph/6 An 1
are added when the client is considered high risk for
cardiovascular disease.
#8. 1. The goal is to control the disease with diet, exercise and Pl He/3 Ap 1
possibly medication, and to avoid complications involving
other organs.
#9. 1. After a thorough history is taken, and both partners have As He/3 Ap 3
expressed concerns, possible surgical intervention will be
discussed at a later time.
#10. 3. Motile sperm (20120 million per mL) should compose at Ev He/3 Ap 3
least 50% of the specimen sent for analysis.
#11. 4. Ovulation occurs 1630 hours after the luteinizing hormone Pl He/3 Ap 3
(LH) surge. Testing the urine on a daily basis throughout the
cycle helps to pinpoint the time of ovulation. Cervical
mucous testing gives an indication of when ovulation occurs
but is not as accurate as testing urine for LH levels.
#12. 1. Risk for aspiration is a problem that occurs when the cuff is An Ph/7 An 1
inflated below what is recommended.
#13. 4. A sore throat may be indicative of a cold or other infection, Ev He/3 An 3
which will distort the interpretation of the graph. The other
items do not affect the basal body temperature.
#14. 2. The infant in a Pavlik harness can be turned from back to Pl Ph/7 Ap 4
abdomen but should not be positioned on either side.
#15. 3. The buttocks need to be raised slightly off the bed and the Im Ph/7 Ap 4
hips are flexed at a 90 angle.
#16. 4. Continuous traction is needed to bring the femoral head Im Ph/5 Ap 4
into position. The child easily learns to play with toys tied
to the crib and to eat in this position.
#17. 1. The Bradford frame facilitates collection of urine and stool Im Ph/7 Ap 4
for an infant or child in a spica cast. The Denis Browne
splint is a splint used for the correction of club foot.

816 NCLEX-RN Review


53155_09_PT 06_p806-823.qxd 2/26/09 8:16 AM Page 817

ANSWER RATIONALE NP CN CL SA
#18. 3. Baby powder coats the skin and causes skin irritation. It is Im Ph/7 Ap 4
also not advised in infants as it can cause respiratory irritation.
#19. 4. In acute pancreatitis, the pancreas itself is exposed to the An Ph/6 An 5
digestive action of pancreatic enzymes. During the acute
phase it is desirable to remove the stimulation to release
these enzymes and thus reduce autodigestion. NPO and
H2 blockers are also commonly ordered for this reason.
#20. 3. The client should be taught activities that include walking Pl Ph/7 Ap 1
and climbing stairs, promote circulation and avoid activities
that decrease circulation (or venous stasis).
#21. 3. A dime-size object usually can pass through the gastrointestinal As Ph/8 Ap 4
tract and be eliminated in the stool within a week. However,
if it occludes the esophagus, the child will not be able to
swallow saliva effectively and will begin drooling. Swallowing
causes pain from the tightening motion of esophageal tissues
around the coin. A foreign object in the air passageway could
be evident by choking, gagging, or the inability to speak.
#22. 2. Liquid, loose stools contribute to dehydration in infants and Ev Ph/8 Ap 4
would warrant further liquid therapy.
#23. 3. Vitamin C, the healing vitamin, will aid wound healing through Pl Ph/7 An 1
mechanisms that maintain capillary integrity. Vitamin A is for
skeletal growth; the B complex vitamins aid in normal metabolism;
and Vitamin D aids in absorption of calcium and phosphorus.
#24. 2. An Unnas paste boot is a gelatin-based bandage that is An Ph/7 K 1
frequently used to treat stasis ulcers occurring in a client with
venous insufficiency. A Jobst stocking is a custom-made
support hose; an Ace bandage provides support to the extremity;
and a plaster of Paris bandage dried into a cast is used to
support a fractured bone.
#25. 4. Pain and venous congestion can be the result when standing Im Ph/7 Ap 1
for a long period of time which aggravates venous insufficiency.
#26. 2. Low weight is not a requirement for bone strength. It is Pl He/3 An 1
indicated for persons with osteoarthritis. Calcium intake
should be started in childhood and continued through all ages.
Weight-bearing exercise is vital to maintaining bone strength.
#27. 3. There is a loss of height in clients with osteoporosis. Leg cramps As He/3 An 1
relate to arterial insufficiency; brittle hair and nails may
indicate iron deficiency anemia; and weight gain may
indicate hypothyroidism.
#28. 2. Vitamin D is a fat-soluble vitamin essential for the normal Pl He/3 An 1
formation of bone and teeth and for the absorption of calcium
and phosphorus from the gastrointestinal tract. It is present
in saltwater fish, sardines, organ meats, fish liver oils, and
egg yolk. However, requirements are usually met from
vitamin D-fortified breads, milk, and dairy products and by
exposure to sunlight. Vitamin A is for skeletal growth;
Vitamin E is for reproduction and muscle development;
and Vitamin K is involved in the clotting of blood.

ANSWERS AND RATIONALES FOR PRACTICE TEST 6 817


53155_09_PT 06_p806-823.qxd 2/26/09 8:16 AM Page 818

ANSWER RATIONALE NP CN CL SA
#29. 1. Smoking causes a decrease in bone density. As He/3 Ap 1
#30. 3. Canned salmon is a good source of calcium if the bones are Ev He/3 An 1
not removed. The other food choices are not high in calcium,
but contain grains (oatmeal), vitamin A and vitamin C (peaches),
and protein (flounder).
#31. 4. The environment is an important factor in the prevention Pl Sa/2 An 1
of injuries. Talking softly and providing quiet music have a
calming effect on the agitated client.
#32. 4. The bulimic clients awareness of the inappropriateness of An Ps/4 An 2
the eating pattern coupled with the clients inability to control
eating activity indicates lack of impulse control. The other
choices describe paranoia or schizophrenia.
#33. 3. This activity is seen in anorexia nervosa. The others are As Ps/4 An 2
commonly associated with bulimia.
#34. 2. Antidepressants have been found to be the most promising Pl Ph/6 Ap 2
pharmacologic treatment of bulimia. The others listed are for
seizure, psychotic, and anxiety disorders, respectively.
#35. 1. Gastrointestinal fluid has a high concentration of sodium, as As Ph/8 Ap 1
water follows sodium. Treatment for diarrhea helps restore
electrolyte balance.
#36. 4. Intermittent claudication is pain occurring when walking that As Ph/7 An 1
subsides with rest. Resulting from inadequate blood supply,
it may be due to arterial spasm, atherosclerosis, arteriosclerosis
obliterans, or an occlusion of an artery to the extremity.
Aching weakness is a common description.
#37. 4. When the extremities of a client with peripheral arterial An Ph/7 An 1
disease are elevated for 30 seconds, a cadaverous pallor
(skin color is pale gray) often results. Dependent rubor is
when the leg is held in a dependent position and becomes
reddened in color. Pregangrenous would present as cyanotic,
cold skin, and absent pulses.
#38. 3. Due to a lack of nutrients to the area, the nails are often thick An Ph/7 An 1
and hardened. A client with peripheral arterial disease (PAD)
would reveal thin, shiny, hairless, and muscular atrophy in
extremities.
#39. 2. Syngeneic means that the donor is the clients identical twin. As Sa/1 An 1
Autologous means that the donor would be the client and
allogeneic means the donor is either a family member or a
matched donor.
#40. 3. The client with diabetes often experiences early atherosclerotic An Ph/7 An 1
changes due to alterations in fat and carbohydrate
metabolism.
#41. 2. Decreased sensation is the only choice that can lead to injury An Ph/7 An 1
or the client may be unaware of injury. Healing is slow in
persons with peripheral arterial disease.

818 NCLEX-RN Review


53155_09_PT 06_p806-823.qxd 2/26/09 8:16 AM Page 819

ANSWER RATIONALE NP CN CL SA
#42. 2. The blood vessels of the skin constrict in response to impulses As Ph/7 An 1
from the autonomic nervous system. The result can be dry,
cracked skin when vascular constriction is of a long-standing
nature. The damage to the feet would be indicative of diabetic
neuropathy related to the central nervous system.
#43. 1. To prevent drying and cracking, only mild soaps should be Pl Ph/7 Ap 1
used on the feet. Feet should be examined every day, using a
mirror if needed; avoid hot water, especially if decreased
sensation is present; soaking too frequently may cause the skin
to become too soft, and meticulous care to dry the feet, especially
between the toes.
#44. 4. Current research shows that pain medication is more effective Pl Ph/6 Ap 5
when it is given before the clients pain gets too intense.
The client can stay on top of pressing the button to receive
some medication before the pain escalates. Instructions are
given that only the client is the one to push the button for
medication.
#45. 1. The use of a PCA is best in a client who is alert enough to Pl Ph/6 An 1
activate the system. Short periods of sleep may occur, but
when the client awakes, he should be reminded to perform
deep breathing exercises.
#46. 1. A PCA is set up to only give small doses, thus eliminating peaks An Ph/6 An 5
in the serum drug levels. The client still needs monitoring
regarding respiratory depression, as the dose may be too high
or family members may be pushing the PCA button when the
client is asleep. Deep breathing between pushing the button
should be encouraged.
#47. 1. Leg massages are contraindicated for deep venous thrombosis or Pl He/3 Ap 1
those at risk because of the danger of dislodging part of the clot
and causing it to become an embolus. The other choices
would be ordered for this client.
#48. 3. A client with a DVT should be on bed rest. Dangling would Im Ph/7 Ap 1
promote movement of the clot and formation of additional
clots by putting pressure on the leg veins. The client would
be instructed to avoid straining (which causes the Valsalva
maneuver and could dislodge the clot), or sudden
movements.
#49. 1. The antidote for heparin is protamine sulfate. Vitamin K is Im Ph/6 Ap 5
the antidote for Coumadin. Bleeding tendencies would be
monitored and heparin should always be administered via
an IV pump.
#50. 3. Pulmonary embolism causes a decrease in perfusion due to An Ph/7 An 1
obstruction of the vascular system from the clot. Ventilation
will be normal.
#51. 2. In 90% of child physical abuse cases, the abuser is a relative An Ps/4 An 4
whom the child trusts.

ANSWERS AND RATIONALES FOR PRACTICE TEST 6 819


53155_09_PT 06_p806-823.qxd 2/26/09 8:16 AM Page 820

ANSWER RATIONALE NP CN CL SA
#52. 2. The family who owns their own home provides some stability As Ps/4 An 4
and is less likely to be at risk for abusing their children. Risk
factors in child abuse include isolation, unemployment, poverty,
marital problems, or chronic illness.
#53. 1. A Pap smear does not detect sexual abuse, but detects changes As Ps/4 An 4
in cells that may indicate cancer and precancer changes.
#54. 4. Using words the child uses to describe body parts ensure that Im Ps/4 Ap 4
the child understands what is being said. The child should be
asked to describe things in her own words, and in a private
interview so the child will feel free to express her feelings.
#55. 1. Hypovolemia means that the client is lacking water or is Pl Ph/6 An 1
dehydrated so an isotonic solution should be administered to
expand the extracellular fluid. The clients lab values should
be given to the physician so a correct solution could be ordered.
#56. 2. Demonstrating explicit sexual activity is not within the An Ps/4 An 4
normal 4-year-olds realm of understanding. These are acts that
could only have been known through actual experience, not
seen on TV. Observation of this action will help the nurse to
be able to explore feelings with the child.
#57. 1. The primary goal is to prevent further abuse and to ensure the Im Ps/4 An 4
childs safety. Taking a wait-and-see stance can prove deleterious
to the child. If the situation indicates, the nurse may confront
the parents in a nonjudgmental manner with a security guard
present.
#58. 4. The age at which sexual activity began is not relevant as an As Sa/2 Ap 1
identifier for a risk factor for AIDS. Drug use and multiple sex
partners are risk factors.
#59. 4. Great progress has been made in the treatment of PCP, and Pl Ph/7 An 1
research is continuing. It is not highly contagious in a person
with an intact immune system.
#60. 3. Pentam is used in the treatment of PCP. The other choices are Pl Ph/6 An 5
used as an antifungal, antitubercular, and antiviral, respectively.
#61. 4. The direct introduction of bacteria into the bloodstream increases An Ph/7 An 1
the risk of the IV drug user developing infective endocarditis.
#62. 1. Counseling may be insufficient to obtain desired behaviors when Pl Ps/4 An 1
the negative consequences seem distinct. Objectives must take
into consideration the lifestyle of the individual and where
changes can be made with the clients cooperation. Therefore,
quitting the drug addiction can be unrealistic or inappropriate
for clients seeking only care for their medical problems.
#63. 4. Signs of endocarditis include shortness of breath, chest pain, As Ph/7 An 1
murmurs, high fever, and tachycardia (but not hypertension).
#64. 2. A neurological exam would include level of consciousness (LOC), As Ph/7 An 4
motor function and vitals signs, but it is impossible to assess
visual acuity in an unconscious client.

820 NCLEX-RN Review


53155_09_PT 06_p806-823.qxd 2/26/09 8:16 AM Page 821

ANSWER RATIONALE NP CN CL SA
#65. 2. Pregnancy makes metabolic control of diabetes more difficult. Pl He/3 An 1
It is essential that prenatal care starts early so that potential
complications can be controlled or minimized. In a pregnant
woman with diabetes, the greater risk is to the fetus.
#66. 2. The woman of average size should gain between 25 and Pl He/3 Ap 3
30 pounds during pregnancy.
#67. 4. The recommended diet for diabetes is high fiber, low fat. Ev Ph/5 Ap 3
The pregnant client with diabetes should follow a high-fiber,
moderate-fat diet with adequate amount of protein.
#68. 1. The ideal goal is to maintain blood sugar as near to a normal Ev Ph/7 Ap 1
level as possible, 80 to 110 mg/dL.
#69. 4. Sweating is a symptom of hypoglycemia. An Ph/7 An 1
#70. 1. The metabolic demands on the mother during labor and delivery An Ph/7 An 3
are great and glucose may be insufficient to meet these demands.
Blood sugars will be monitored every hour to detect hypoglycemia.
#71. 2. While all the components of inserting a catheter are important, Pl Sa/2 Ap 1
maintaining aseptic technique is most important because it
limits the chances for infection and extended hospital stays.
#72. 1. Within one week, there may be minimal to moderate changes Pl Ps/4 Ap 2
in thought process, depending on the clients diagnosed mental
illness. An appropriate goal is for the client to feel less threatened
and less anxious, lessening the requirement for delusional thought.
If the client is compliant with psychotropic medications, the
client may respond positively by decreased frequency and
intensity of delusions after 1 week of medications. The other
choices are long-term goals.
#73. 1. Stating a positive attribute about himself is the only choice that Pl Ps/4 Ap 2
would be possible to meet within 1 week. Additional symptoms
of depression, pessimisms, or thoughts of failure will prohibit the
client from accomplishing the other tasks listed. As self-esteem
improves, the clients activity involvement should advance.
#74. 2. A delusion of reference is a fixed false belief that events or people As Ps/4 An 2
are directly related to the individual person. The other choices
are a disturbance in thought pattern or a belief that others are
attempting to harm a person.
#75. 3. Limiting sodium is not necessary for a patient on Coumadin. Im Ps/4 Ap 5
Dietary recommendations for foods high in vitamin K would
be given.
#76. 3. Agoraphobia is the fear of open or public places. The other choices Ev Ps/4 Ap 2
are social, simple, and compulsive phobia, respectively.
#77. 3. Stating and abstaining from alcohol for life will be essential to his Ev Ps/4 Ap 2
long-term addiction and recovery.
#78. 3. Drug onset with tricyclic antidepressants begins between 7 and An Ph/6 Ap 5
10 days after initial treatment with full effects taking up to 1 month.

ANSWERS AND RATIONALES FOR PRACTICE TEST 6 821


53155_09_PT 06_p806-823.qxd 2/26/09 8:16 AM Page 822

ANSWER RATIONALE NP CN CL SA
#79. 4. Autonomic dysreflexia is an emergency situation which may be Im Ph/8 Ap 1
triggered by distension of the bladder or colon. The priority action
is to lower the blood pressure by placing the client in a sitting
position and monitoring blood pressure and other vital signs until
the episode is resolved. Then check bladder and/or rectum for
distension as the possible triggering response.
#80. 3. Intravenous fluids are necessary post-op to maintain fluid and Ev Ph/7 Ap 1
electrolyte balance and as a route for medications. The intravenous
infusion will be kept in place until fluids can be taken by mouth.
#81. 2. The principle of the constant carbohydrate diet is to keep the CHO Ev Ph/7 Ap 4
intake consistent by time of day, each day. But the dessert may
differ considerably in content from day to day.
#82. 4. Seventy-five percent of elderly Pima Indians have evidence of As Ph/7 Ap 1
gallstones. Cholelithiasis is also more common in people of
northern European descent. A high-fat or high-cholesterol diet,
sedentary lifestyle are associated with gallbladder disease.
#83. 2. The continuous use of the elastic pressure garment is designed Ev Ph/7 Ap 1
to reduce vascularity and cellularity of the scar tissue and
promote the growth of soft, pale scar tissue that is free of collagen
nodules. A red wound indicates the development of hypertrophic
scars, a hard area would be characteristic of keloid formation,
and an open area indicates failure to heal.
#84. 4. The shampoo loosens the nits and kills the adult lice. The Ev He/3 An 4
fine-tooth comb pulls the nits from the hair shaft. People from all
socioeconomic groups get head lice; only wash items that have
had direct contact and place into the dryer on a hot cycle; one
initial shampooing is sufficient with a possible additional one
710 days later.
#85. 2. The infants feeding schedule should be maintained because Im He/3 An 4
phenylalanine is an essential amino acid that is converted into
tyrosine by the enzyme phenylalanine hydroxylase. Therefore,
protein in the diet is necessary to see if phenylalanine is
converted.
#86. 2. A diet low in phenylalanine is necessary for an indefinite period Ev He/3 Ap 3
of time and possibly throughout life. If initiated within the first
days of life, a low-phenylalanine diet ensures a normal
development and life span.
#87. 4. It should be left in for at least 6 hours after the last intercourse to Ev He/3 Ap 3
allow the spermicidal cream or jelly to work. Vaseline is avoided
as it can cause breakdown of the latex; a weight loss/gain of
25 lb would necessitate a refitting; spermicide is applied on
the rim and inside the dome.
#88. 4. Because of the albumin in the influenza vaccine, it should be As He/3 Co 1
not given to individuals who are allergic to eggs or egg
products. The other choices are not essential to flu shot
administration.
#89. 1. The most appropriate time for examination is when the scrotum Im He/3 Ap 1
is relaxed, as after a warm bath/shower.

822 NCLEX-RN Review


53155_09_PT 06_p806-823.qxd 2/26/09 8:16 AM Page 823

ANSWER RATIONALE NP CN CL SA
#90. 2. If the airway is open and breaths are being delivered correctly, Ev Ph/8 Ap 1
then the chest is rising and falling. Oxygen being delivered to
the lungs is the most important factor during CPR.
#91. 1. As the skin ages, there is an increase in melanin. These are As He/3 Co 1
commonly called age spots. The other choices are opposite
occurrences of what happens in aging.
#92. 3. Monitoring vital signs would provide the first means of Ev Ph/7 Ap 1
detecting an infection that is not otherwise visible or obvious.
Hgb and Hct provide information on loss of blood/bleeding,
not infection.
#93. 3. The major complication for peritoneal dialysis is peritonitis. Ev Ph/7 Ap 1
Cloudy or opaque effluent (the flowing outward liquid) is an
early sign, along with fever, rebound abdominal tenderness,
malaise, nausea, and vomiting.
#94. 4. The PPD skin test should always be given intradermally. When Im Ph/7 Co 5
injected properly, the PPD will form a wheal just beneath the
skin surface.
#95. 4. Indurations less than 5 mm are not significant and erythema is Im Ph/7 An 1
always insignificant. Further assessment is required of indurations
of 10 mm or more, which is highly suggestive of tuberculosis;
indurations of 59 mm are inconclusive and should be repeated
in another site. If the client is at high risk for tuberculosis, further
testing should be initiated.
#96. 4. The method described is the proper way for tracheal suctioning. Im Ph/7 Ap 1
The procedure is sterile; suction should not be applied while
inserting the catheter; hyperoxygenation should be performed
before and after suctioning.
#97. 3. Clamping the central venous line will prevent air embolism Im Ph/7 Ap 1
and blood backup. The tubing is always primed before attachment
to the existing line. Cleaning helps to prevent infection.
#98. 2. The sternal notch or Angle of Louis identified the second rib and Im Ph/7 Ap 1
thereby assists in locating the fourth intercostal space. McBurneys
point is associated with appendix location; suprasternal notch is
located above the sternum; costovertebral refers to the joining of a
rib and vertebral and assessing kidney pain.
#99. 2. The choice of a low-fat soup (which would have been higher in fat Ev Ph/5 An 1
if made with chicken or beef stock) and high-fiber bread are correct
choices. Mayonnaise in tuna salad is high in fat; hard-boiled eggs
are high in fat; walnuts are high in fat.
#100. 3. The schedule for mammogram testing recommend by the American Im He/3 Co 3
Cancer Society is a baseline between the ages of 35 and 40; once
every 12 years between 40 and 50; yearly after age 50. The test
can detect tumors and lesions that are still too small to be palpated
but should not be promised as 100% accurate; a strong family
history of cancer would initiate a mammogram performed at a
younger age, not sexual activity; no contrast media is used.

ANSWERS AND RATIONALES FOR PRACTICE TEST 6 823


53155_09_PT 07_p824-844.qxd 2/27/09 6:31 PM Page 824

Practice Test 7

1. An adult client has been taking aluminum 1. She does biweekly grocery shopping.
hydroxide (Amphojel) for hyperphosphatemia. 2. She has stopped attending a widow support
What will the client need to be taught about this group.
medication? 3. She babysits her two grandchildren whenever
1. To inform the physician if he has constipation. asked.
2. The tablets tend to be more effective than the 4. She participates in a local senior citizen
liquid. group.
3. To take large amounts of water to ensure
passage of the medication to the stomach. 6. An adult resident is in a long-term care facility
4. To report signs of muscle weakness, anorexia, with a medical diagnosis of organic brain
and malaise. syndrome. Her mental status assessment
documents an untidy, suspicious, easily agitated
2. An adult client has chronic idiopathic woman who speaks in nonsense syllables. In
hypoparathyroidism. Which is not appropriate caring for her, the nurse should anticipate which
to include in the nursing care plan? nursing actions to promote socialization?
1. Low-calcium, high-phosphorus diet. 1. Limiting visitation by family and friends.
2. Oral calcium (Os-Cal) for chronic 2. Utilizing the pet-animal companion program.
hypocalcemia. 3. Discussing the need for a speech therapist.
3. Seizure precautions. 4. Touching the client only when necessary.
4. Private room to reduce environmental stimuli.
7. In completing an assessment of an elderly client
3. An adult has hyperthyroidism and is scheduled who has been a victim of abuse, who does the
for a thyroidectomy. The physician has ordered nurse know is at the highest risk?
Lugols solution for the client. What is the 1. A Caucasian female who is physically or
primary reason for giving Lugols solution cognitively impaired.
preoperatively? 2. A Caucasian male who has a physical
1. Decrease the risk of agranulocytosis disability.
postoperatively. 3. An African-American female whose physical
2. Prevent tetany while the client is under or mental conditions cause dependency on
general anesthesia. family members.
3. Reduce the size and vascularity of the thyroid 4. An African-American male whose cognitive
and prevent hemorrhage. impairment causes behavioral problems.
4. Potentiate the effect of the other preoperative
medication so less medicine can be given 8. Which would best indicate to the nurse that a
while the client is under anesthesia. client is depressed?
1. Feelings of worthlessness.
4. The nurse is caring for a client who had a 2. Poor hygiene and grooming.
thyroidectomy this morning. The nurse must 3. Intense anxiety.
monitor for possible adverse effects. Which is
4. Thought insertion.
least likely to occur in this client?
1. Chvosteks sign. 9. The nurse in an outpatient mental health clinic
2. Laryngeal damage. has identified marital discord as a significant
3. Brudzinskis sign. problem for one of the clients. A client with this
4. Trousseaus sign. type of problem would be most likely to be dealing
with issues in which developmental phase?
5. The nurse is caring for an older adult widow 1. Trust vs. mistrust.
whose husband died 6 months ago. Which 2. Identity vs. role confusion.
action best indicates to the nurse that the client 3. Intimacy vs. isolation.
is making progress in resocialization?
4. Generativity vs. stagnation.

824 NCLEX-RN Review


53155_09_PT 07_p824-844.qxd 2/27/09 6:31 PM Page 825

10. Which of the following statements best indicates 3. The mother verbalizes a need for a shot after
that the client understands the nurses teaching giving birth to an Rh-positive baby.
about effective coping mechanisms? 4. The mother verbalizes a need for a shot after
1. Talking to you really helped me put things breastfeeding.
into perspective.
2. Talking to you really helped me solve my 14. A client who is 25 weeks pregnant with no
problems. previous medical or obstetrical problems is
admitted to the hospital in premature labor. The
3. I dont have time at home to do the
nurse can expect that orders for this client, in
relaxation techniques.
addition to bed rest, will include which of the
4. The relaxation techniques helped me to go following?
right to sleep.
1. A fetal monitor and a tocolytic.
11. An adult was admitted to the chemical 2. A fetal monitor and a tranquilizer.
dependency unit with a history of daily alcohol 3. A maternal cardiac monitor and fluid therapy.
use for the past 15 years. Which of the following 4. A fetal monitor and a maternal cardiac monitor.
nursing diagnoses should the admitting nurse
select to be the primary focus during the initial 15. Upon admission, a client tells the nurse that she
phase of his treatment? has weak blood but doesnt know the name of
1. Sensory/perceptual alteration related to her disease. She has been taking vitamin B12
withdrawal seizures secondary to alcohol injections for 5 years. The nurse explains that
cessation. vitamin B12 is the drug management for which
2. High risk for injury related to suicidal type of anemia?
thoughts secondary to alcohol cessation. 1. Iron deficiency anemia.
3. Ineffective denial related to inability to 2. Aplastic anemia.
identify effect of alcohol on life secondary to 3. Pernicious anemia.
alcohol use. 4. Hemolytic anemia.
4. High risk for injury related to withdrawal
seizures secondary to alcohol cessation. 16. A client has been admitted with possible
pernicious anemia. Various diagnostic tests are
12. The labor and delivery unit called to give report ordered. What test will the nurse expect to give
on a woman who delivered a full-term live baby a definitive test?
girl 2 hours ago via spontaneous vaginal 1. A positive Schilling test.
delivery. She had a first-degree laceration, which 2. A gastric analysis with decreased free HCl acid.
was repaired. She plans to breastfeed. Both
3. A bone marrow biopsy showing abnormal
mother and infant will be coming to the mother
erythrocyte and defective leukocyte
baby unit. What should the nurse expect will be
maturation.
included in the care plan?
4. An elevated LDH.
1. Keep the client NPO for 24 hours.
2. An order for ice packs to the breasts. 17. An adult male is hospitalized for urolithiasis.
3. An order for ice packs to the perineum prn A stone he passed in his urine was sent to the
for 4 hours. laboratory this morning. The lab identifies the
4. An indwelling catheter that will remain in stone as an oxalate stone. Which modifications
place for 12 hours. should the nurse teach him to make in his diet?
1. Limit milk and dairy products.
13. The nurse has been giving a mother who is at 2. Limit intake of tea, chocolate, and spinach.
risk for having a baby with Rh incompatibility
3. Eat an acid ash diet to keep his urine acidic.
instruction about preventing isoimmune
hemolytic disease in future neonates. Which 4. Limit food high in purine.
statement indicates that she understands the
18. The nurse is teaching an older teen how to
need for Rho(D) immune globulin in the
perform a testicular self-exam. Which is an
future?
abnormal finding that indicates he should see
1. The mother asks when her baby will get the his physician?
shot.
1. His left testis hangs lower than his right testis.
2. The mother verbalizes a need for a shot after
2. His testes feel smooth, rubbery, and oval
donating blood.
shaped.

COMPREHENSIVE PRACTICE TESTS 825


53155_09_PT 07_p824-844.qxd 2/27/09 6:31 PM Page 826

3. His left testis is larger than his right testis. 24. An adult client is scheduled for a variety of tests
4. His testes are slightly tender when he for diarrhea and other gastrointestinal
examines them. complaints. The doctor has ordered an antacid
prn for upset stomach. Which antacid is least
19. An adult is in acute renal failure and must likely to be ordered for this client because it may
undergo hemodialysis. Which medication must have a laxative effect?
the nurse withhold prior to dialysis? 1. Aluminum hydroxide (Amphogel).
1. NPH insulin. 2. Kaopectate.
2. Pilocarpine. 3. Magnesium hydroxide (MOM).
3. Dipyridamole (Persantine). 4. Dihydroxy-aluminum sodium carbonate
4. Cholestyramine (Questran). (Rolaids).

20. The nurse is monitoring an adult who is 25. An adult client is admitted for bowel surgery.
undergoing hemodialysis. The client suddenly The nurse teaches the client what to expect in
becomes cyanotic and complains of dyspnea and preparation for surgery. Which is least likely to
chest pain. His blood pressure is 70/40 and his be included in the nurses explanation?
pulse is weak and rapid. The nurse calls the 1. Cleansing enemas will be given the night
physician immediately because the signs and before surgery.
symptoms suggest which complication of dialysis? 2. Antibiotics are given 3 to 5 days preoperatively
1. Disequilibrium syndrome. to decrease bacteria in the intestine.
2. Air embolism. 3. A nasogastric tube will be inserted on the
3. Internal bleeding. morning of surgery.
4. Hemorrhage at the shunt. 4. Laxatives will be given the morning of
surgery to relax the bowel.
21. An adult who has urolithiasis is being treated
conservatively in hopes that surgery will not be 26. A client recovering from rectal surgery is
necessary. Which of these nursing measures ordered Colace. What action does Colace have
should the nurse plan to do? on the bowels?
1. Provide fluid intake of 3000 mL or more. 1. Attracts and hold large amounts of fluids,
thereby increasing the bulk of stools.
2. Restrict citrus juices and milk products.
2. Coats the feces with an oily film and prevent
3. Insert an indwelling catheter as ordered.
the colon from reabsorbing water from the
4. Administer ordered narcotic analgesics feces.
whenever the client requests them.
3. Softens stool to prevent straining during
22. Prior to discharge, the client with COPD will defecation.
need to be taught self-care. The nurse should plan 4. Stimulates peristalsis.
to include which instruction to the client?
27. An adult client is diagnosed with a hiatal hernia.
1. Increase the oxygen flow rate to 4 liter/min. He has listed all of the following on his
when you plan to exercise. admission form. Which activity is most likely to
2. Stay indoors if possible when the weather is be aggravating his condition?
very cold. 1. Experiencing added stress because he gave
3. Limit fluid intake to 200 mL or less. up smoking recently.
4. When short of breath, sit in a recliner 2. Lying down and falling asleep on the couch
chair with the backrest at a semi-Fowlers after a big dinner each evening.
position. 3. Taking an antacid before and after each meal.
23. An adult has developed dumping syndrome 4. Eating six small meals a day.
following a subtotal gastrectomy. Which should 28. When planning care for the client during the
the nurse include in the plan of care? immediate postoperative period after a total
1. Sit upright for at least 30 minutes after meals. laryngectomy, which of these measures would
2. Take sips of fluid between bites of solid food. be included in the plan of care?
3. Eat something every 2 to 3 hours. 1. Provision of a nonverbal means of
4. Reduce the amount of simple carbohydrate in communication.
the diet. 2. Positioning the client on the side with the
head of the bed flat.

826 NCLEX-RN Review


53155_09_PT 07_p824-844.qxd 2/27/09 6:31 PM Page 827

3. Administering cough suppressants. 2. Pass a catheter gently into the esophagus to


4. Suctioning the tracheostomy every 4 hours. detect resistance.
3. Give 10 mL of glucose water to check for
29. Which common side effect should the nurse swallowing and color changes with
anticipate for a client receiving chemotherapy? feeding.
1. Nausea and vomiting. 4. Place the infant with the head in a downward
2. Cardiac arrhythmias. position to drain mucus.
3. Paralytic ileus.
35. The nurse performs a cardiovascular assessment
4. Diuresis.
on an elderly client which reveals a blood
30. An adult suffered a detached retina of the right pressure of 162/86. The finding is a likely result
eye while playing racquetball. He presents in the of what problem?
emergency department with which symptom? 1. Less muscle mass.
1. Sharp pain OD. 2. Calcification of arteries.
2. Redness OU. 3. Dehydration.
3. Increase in intraocular pressure OD. 4. Impaired lung capacity.
4. Blank areas in the field of vision.
36. A 2-year-old child with congestive heart failure
31. An adult has undergone surgery to correct a has been receiving digoxin for one week. What
detached retina. What does the postoperative occurrence will alert the nurse to an early sign of
care plan include? digitalis toxicity?
1. Turn, cough, and deep breathe every 2 hours. 1. Bradypnea.
2. Position on the operative side to keep the 2. Failure to thrive.
retina next to the choroid. 3. Tachycardia.
3. A patch over the operative eye to prevent 4. Vomiting.
further detachment.
37. A 5-year-old child is being discharged to home
4. Administer pilocarpine eye drops for pupil
after a bone marrow transplant. Which of the
constriction.
following statements to the nurse by the childs
32. The nurse is admitting a client with probable mother indicates understanding of home care for
Mnires disease. Which symptom is the client this child?
most likely to exhibit? 1. My child needs to wear a mask when
1. Vomiting. contact with others is necessary.
2. Tinnitis. 2. My child can return to school tomorrow.
3. Nystagmus. 3. My child should stay in bed.
4. Blurred vision. 4. My child will need a nurse 24 hours
a day.
33. An adult has Mnires disease. Which
statement indicates that more teaching about the 38. The nurse working in the newborn nursery
management of her disease is needed? notices that a newborn has one foot that curves
inward. What information does the nurse know
1. I was trying to stop smoking anyway, so Im
about congenital clubfoot?
glad to have to now.
1. It rarely occurs in boys and is usually a
2. I have to remember to move slower even
bilateral defect.
when Im in a hurry on my job.
2. It can be manipulated to various range-of-
3. Im going to miss using a lot of salt on my
motion positions if muscle stretching
food.
exercises are performed first.
4. Ill have to fit in the extra fluid I need to
3. It will not need to be actually diagnosed until
drink throughout the day so I dont feel
the infant is ready to be dismissed from the
bloated all at once.
hospital.
34. A term newborn is admitted to the nursery with 4. It is most commonly of the equinovarus type
excessive drooling, coughing, and sneezing. with plantar flexion and inversion of the
How will the nurse assess for esophageal atresia? ankle and foot.
1. Check the mothers prenatal record for a
history of oligohydramnios.

COMPREHENSIVE PRACTICE TESTS 827


53155_09_PT 07_p824-844.qxd 2/27/09 6:31 PM Page 828

39. A nurse on the orthopedic ward takes report on Mommy! I am going to marry Mommy! His
four clients in traction. Of the four, which mother is embarrassed saying, I dont know
clients traction is intermittent and can be where he gets such ideas. What is the nurses
released? most appropriate response?
1. A 112-year-old in Bryants traction for hip 1. Its pretty normal behavior for a 4-year-old.
dislocation. 2. Are you and your husband having
2. A 24-year-old in Russell traction for a difficulties in your marriage?
fractured femur. 3. If you discipline him when he says that, he
3. A 40-year-old in Bucks extension traction for will stop and eventually forget about it.
a fractured hip. 4. Have you considered getting counseling for
4. A 32-year-old in cervical traction for cervical Sam? It is not normal to want to marry your
disc disease. mother.

40. The nurse is caring for an obese male client who 44. A 2-month-old infant is admitted to the pediatric
has had a herniorrhaphy for a strangulated unit in congestive heart failure. He has a history
hernia. Which is important postoperative care of ventricular septal defect diagnosed at birth.
and teaching? He is placed on digoxin by mouth in liquid
1. Turn, cough, and deep breathe every 2 hours, form. Before administering this medication,
making sure to splint the incision. what is the nurses most appropriate action?
2. Assess for a distended bladder. 1. Placing the medication in a small amount of
3. Place a heating pad on the scrotal area to formula and having the infant suck.
reduce the swelling. 2. Taking the apical heart rate and withholding
4. Restrict physical activities for 2 weeks. if the rate is below 70.
3. Drawing the medication up in a syringe and
41. A mother has brought her daughter to the verifying the correctness with a second nurse.
pediatricians office for her 9-month checkup. The 4. Giving the medication when the mother is
nurse assesses the baby and finds all of the available to hold the infant and preventing
following data. Which finding would cause the him from spitting it out.
nurse to be concerned about developmental delay?
1. The child plays with a toy for only a few 45. A 2-month-old infant is admitted with a history
minutes, then moves on to something else. of projectile vomiting for the last 2 weeks. The
2. The child cannot pull herself up to a standing infant has gone from a birth weight of 9 lb to a
position. current weight of 8 lb. He looks emaciated, acts
hungry, and is crying. Given this data, which
3. She does not sit up without assistance.
nursing diagnosis is appropriate for this
4. When something is taken away from her, she client?
cries and protests.
1. Alteration in growth and development
42. A 6-year-old is in the hospital for surgery. He has related to poor food intake.
preoperative medications ordered by injection. 2. Alteration in fluid and electrolyte balance
When the nurse brings it to him, he cries, No, I due to vomiting and poor intake.
wont be bad! Dont give me a shot! What is the 3. Potential for altered family process related to
nurses best response? situational crisis.
1. You have to have this shot, but if you are 4. Nutritional deficit.
good, there wont be any more.
2. What have you done that makes you say you 46. For a school nurse in a junior high school, it is
are bad? important to check young teenage girls for
scoliosis. What is the best way for the nurse to
3. You need this shot to get ready for your
assess for this problem?
operation. It has nothing to do with being
good or bad. 1. Have each girl walk in a straight line.
4. I know you try to be good. Ill call the doctor 2. Have each girl bend over and measure
and ask if you really have to have this shot. shoulder height.
3. Run fingers down the spine to feel for
43. A young child is admitted with rheumatic fever. abnormalities.
His mother must go home and child does not 4. Watch as each girl does physical education
want his father staying with him, stating I want activities to see if any abnormality is evident.

828 NCLEX-RN Review


53155_09_PT 07_p824-844.qxd 2/27/09 6:31 PM Page 829

47. A 5-year-old boy is admitted in acute respiratory 3. It is a good indication hell need to be
distress. He is sitting upright, drooling, unable to circumcised, but there is no hurry.
swallow, with a look of panic on his face. The 4. Discuss this with your pediatrician.
nurse plans to place which essential equipment Circumcision is controversial.
at the bedside?
1. Croup tent. 52. A client who is 24 hours post cesarean delivery
2. Padded bedsides for seizure precautions. has orders to advance diet as tolerated. She has
been on full liquids and asks if she can have real
3. Tracheotomy set.
food. Which question is most appropriate for the
4. Suction. nurse to ask before changing her to a regular diet?
48. A patient is admitted with an ulcer due to 1. Have you had a bowel movement yet?
venous insufficiency. What characteristics 2. Are you passing gas?
would the nurse expect to find on assessment? 3. Do you notice rumblings in your belly?
1. Location on the toes or heels. 4. Are you still hungry after eating your liquid
2. Circular in shape. tray?
3. Black in color, dry, and gangrenous.
53. A client is admitted to labor and delivery for an
4. Moderate to severe edema. induction of labor. She is receiving Pitocin and
has progressed to 5 cm dilation. Her contractions
49. An 18-year-old has been sexually active for
have steadily become stronger and longer until
2 years and has come to the clinic for birth
the nurse notices a contraction lasting 2 minutes.
control pills. Her history reveals she is 15 pounds
What is the nurses best initial action?
underweight, a nonsmoker, exercises 34 times
per week, and has numerous sexual partners. 1. Assess the fetal heart rate and observe a little
Which of the following would be the least longer.
appropriate birth control device? 2. Turn the client on her left side and encourage
1. Oral contraceptives. transition breathing.
2. Condoms and foam. 3. Give the client oxygen through a nasal
cannula and decrease the rate of the
3. Intrauterine device.
infusion.
4. Diaphragm.
4. Stop the pitocin infusion.
50. A woman has just been admitted to the postpartum
54. A woman who is 30 weeks pregnant has been
after delivery of a baby girl. When the nurse brings
diagnosed with gestational diabetes. Her
in the baby to assist her in breastfeeding, the
physician has ordered a 2000-calorie ADA diet,
mother states she does not want to try yet and
moderate exercise, and weekly appointments for
begins talking about how difficult her labor and
prenatal care. She is very upset and wants to
delivery was. The nurse recognizes this is
know everything about her condition and how she
indicative of what type of behavior?
can have a healthy baby. Which of the following is
1. Risk for alteration in parenting related to an appropriate goal for the client at this time?
the mothers lack of interest in her babys
1. Discuss how pregnancy causes diabetes.
needs.
2. Demonstrate insulin injections.
2. Fatigue from labor and delivery.
3. Keep a food diary for 48 hours.
3. Inability to accept the reality of
parenthood. 4. Identify risks to her fetus if she doesnt
follow her diet rigidly.
4. Normal developmental phase of taking-in
during the early puerperium. 55. A woman is admitted to the antepartal unit with
pregnancy-induced hypertension. Assessment
51. While caring for a newborn baby boy, the nurse
findings include: 34 weeks gestation, BP
notices the foreskin on the penis cannot be
160/100, +3 protein in urine, generalized edema,
retracted. The babys mother asks if this means
headache, seeing spots before her eyes. She
her baby must be circumcised immediately.
states concern about her two preschool children
What is the nurses best response?
who are with a neighbor. What would be the
1. It is normal for a newborn. You cannot priority nursing diagnosis?
retract the foreskin until he is older.
1. Alteration in fluid volume.
2. Yes, the foreskin should retract or bacteria
2. Powerlessness.
can grow and cause infection.

COMPREHENSIVE PRACTICE TESTS 829


53155_09_PT 07_p824-844.qxd 2/27/09 6:31 PM Page 830

3. Ineffective family coping. schizophrenia. If the medication is effective,


4. Alteration in tissue perfusion. what would the nurse expect to see?
1. Decreased hallucinations and decreased
56. A woman has been in labor for 8 hours, pushing sleep.
for 112 hours without progress and is extremely 2. Increased sleep and improved personal
tired and discouraged. The fetus remains at hygiene.
minus one station. The nurse suggests changing
3. Decreased suspiciousness and increased
to a nontraditional position. Which assessment
appetite.
is most important to make before proceeding?
4. Decreased hallucinations and decreased
1. The condition of the fetal membranes.
suspiciousness.
2. The level of maternal fatigue.
3. Effacement and dilation. 61. A physically abused wife says to her nurse, Id
4. Status of maternal bladder. be nothing without my husband. To increase
the womans self-esteem, which statement by the
57. A newborn is admitted to the nursery 20 minutes nurse would be most appropriate?
after birth. His vital signs are stable. He is crying 1. I see you as a survivor in a difficult
vigorously. The nurse begins the admission situation.
procedure. Which of the following is most 2. It sounds like you dont think you are
appropriate? important.
1. Wear latex gloves during the admission. 3. Lets not talk about that right now.
2. Do the admission in an open crib with good 4. I can see that youre upset.
light.
3. Wear mask and gown when approaching the 62. The nurse is planning nursing interventions for
infant. parents who abuse their children. What will be
4. Perform the admission procedures while the important for the nurse to recall about abusive
infant is asleep. parents?
1. They plan ahead as to when and how to
58. An infant has just been delivered with a abuse their children.
myelomeningocele. The infant is immediately 2. They ask for help generally only after feeling
transferred to the nursery. The nurse should overwhelmed with the problem.
place the infant in what position?
3. They usually feel no guilt concerning the
1. Semi-sitting with support of an infant seat. abuse.
2. Side-lying with his head lower than the rest 4. They are always a product of abuse themselves.
of his body to promote drainage.
3. Supine to place counterpressure on the 63. The nurse is caring for a client with antisocial
defect. personality. What behavior exhibited by the
4. Prone to reduce the risk of rupture and client would indicate an improvement?
infection. 1. Attending therapeutic groups daily.
2. Compliance with antipsychotic medications.
59. Shortly after administering Prolixin
3. Abiding by hospital rules and regulations.
(fluphenazine) 10 mg PO to a client who has
schizophrenia, the nurse notices that he is 4. Interacting with peers in the day room.
pacing, appears restless, is drooling, and
64. The nurse is caring for an adult client who is to
complains that his tongue feels thick. Which of
undergo a cystoscopy in the morning. What
the following prn orders would the nurse
should be included in the plan of care?
administer to alleviate these symptoms?
1. Explaining that this is a painless test.
1. Ativan (lorazepam) 2 mg IM.
2. Instructing the client to drink several glasses
2. Atropine 4 mg IM.
of water prior to the test.
3. Cogentin (benztropine mesylate) 2 mg IM.
3. Giving instructions to breathe rapidly and
4. Thorazine (chlorpromazine hydrochloride) deeply throughout the procedure.
50 mg IM.
4. Telling the client to administer a small
60. The nurse is evaluating the effectiveness of package enema in the evening before the test.
antipsychotic medication 2 weeks after starting
65. A young woman is to undergo a Tensilon test.
treatment in a client who has paranoid
The nurse is explaining the test to the client.

830 NCLEX-RN Review


53155_09_PT 07_p824-844.qxd 2/27/09 6:31 PM Page 831

Which statement the client makes indicates the 70. The nurse is caring for a client who is scheduled
best understanding of the test? for a ureterosigmoidostomy. Which information
1. A positive test will be evident within is inappropriate and will not be a part of the
1 minute of the Tensilon injection. preoperative teaching plan?
2. The test is of diagnostic value in only about 1. Liquid diet for 24 hours prior to the surgery.
20% of persons with myasthenia gravis. 2. Assessment of the adequacy of the rectal
3. If the test is positive I will feel an immediate sphincter.
decrease in muscle strength. 3. Administration of neomycin sulfate for
4. My blood sugar will decrease to a normal 3 days prior to surgery.
level after the test. 4. Application of full-length elastic
stockings.
66. The client asks why it is necessary to have a
serum creatinine and BUN drawn before the CT 71. An adult client who has chronic obstructive
scan. What is the nurses best response? lung disease needs frequent monitoring of
1. These tests will determine if you are allergic arterial blood gases. What is an essential action
to iodine contrast media. that should be performed after the drawing of
2. The tests determine if the kidneys are arterial blood gases?
functioning and can eliminate contrast 1. Encourage the client to cough and deep
media. breathe.
3. The tests serve as baseline information to 2. Apply pressure to the puncture site for
determine if the scan has caused damage. 5 minutes.
4. The blood tests give additional information 3. Shake the vial of blood before transporting it
about the presence of possible tumors. to the lab.
4. Keep the client on bed rest for 2 hours.
67. The nurse is performing an ophthalmologic
examination on an elderly client. The client 72. A client with chronic obstruction pulmonary
states, my peripheral vision is decreased. disease (COPD) is on oxygen by nasal cannula at
What is the nurses best response during the 2 liters per minute. Which is most useful in
exam? assessing the success of the oxygen therapy?
1. You should be grateful you are not blind. 1. Respiratory rate.
2. As one ages, peripheral vision decreases. 2. Color of mucous membranes.
This is normal. 3. Pulmonary function tests.
3. You should rest your eyes frequently. 4. Arterial blood gases.
4. You may be able to improve your vision if
you move slowly. 73. The nurse is interpreting the results of a blood
gas analysis performed on an adult client. The
68. An adult is admitted with post cerebral vascular values include pH of 7.35, pCO2 of 60, HCO3 of
accident (CVA) with right-sided paralysis. What 35, and O2 of 60. Which interpretation is most
documentation will be correct if the client is accurate?
having difficulty speaking due to the 1. The client is in metabolic acidosis.
impairment of the facial muscles? 2. The client is in compensated metabolic
1. Semantic aphasia. alkalosis.
2. Receptive aphasia. 3. The client is in respiratory alkalosis.
3. Dysarthria. 4. The client is in compensated respiratory
4. Dysphagia. acidosis.

69. The nurse is assessing a client for local 74. An elderly woman is admitted with a fractured
inflammation following an injury. What is left hip after a fall in her home. During the
one of the cardinal signs the nurse should nursing assessment, the nurse would expect to
observe? see which of the following?
1. Fever. 1. The client cannot move her left leg but can
2. Confusion. wiggle her toes.
3. Impaired function. 2. The left leg will have internal rotation and
4. Malaise. appear longer.

COMPREHENSIVE PRACTICE TESTS 831


53155_09_PT 07_p824-844.qxd 2/27/09 6:31 PM Page 832

3. The client can voluntarily move the left leg tubercle bacillus without development of
without pain. tuberculosis infection.
4. The left leg will have involuntary tremors. 2. The skin test is only a screening test.
3. BCG vaccine is not effective against
75. The nurse is evaluating the care given to a client tuberculosis.
who has had a total hip replacement. Which
4. BCG vaccine stimulates formation of
position indicates the client has been positioned
antibodies against tuberculosis.
appropriately?
1. The affected leg is abducted and externally 80. An adult has had a low-sodium, low-fat diet
rotated. prescribed for heart disease. The nurse is
2. The affected leg is adducted and externally evaluating her understanding of the diet. Which
rotated. statement she makes indicates a need for further
3. The affected leg is abducted and internally instruction?
rotated. 1. Whenever I go out to eat at a restaurant,
4. The affected leg is adducted and internally I get the salad bar and use vinegar for a
rotated. dressing.
2. When I eat chicken, I take the skin off and
76. A male client is admitted with Guillain-Barr broil the chicken in the oven.
syndrome and complains of severe weakness, 3. I like to put catsup on my noodles.
numbness in both hands, and is extremely
4. We use skim milk for drinking and
anxious. What would be the priority nursing
cooking.
action?
1. Raise the head of the bed to high-Fowlers to 81. The nurse is administering eyedrops to an
prevent increased intracranial pressure. elderly client. Which action is least appropriate
2. Place respiratory support equipment at the for the nurse to take?
bedside. 1. Inform the client that the drops may cause
3. Reassure the client that in time the strength blurred vision and difficulty focusing for a
will return to his legs. period of time.
4. Place the client in reverse isolation to prevent 2. Apply gentle pressure to the nasolacrimal
spreading the virus. canal for 1 to 2 minutes after instillation to
prevent systemic absorption.
77. The nurse is assessing an adult who has a 3. Encourage the client to lie down with eyes
cataract in his right eye. What symptom is the closed after instillation to prevent systemic
client likely to exhibit? absorption.
1. Acute eye pain. 4. Gently pull the lower lid down and place the
2. Redness and itching in the right eye. medicine in the center of the lid.
3. Gradual blurring of vision.
82. A woman is admitted for internal radiation for
4. Severe headaches and dizziness. cancer of the cervix. The nurse knows the client
understands the procedure when she makes
78. An adult is scheduled for a magnetic resonance
which of the following remarks the night before
imaging (MRI) test because of a back injury.
the procedure?
Which question is it essential for the nurse to
ask the client before the procedure? 1. She says to her husband, Please bring me a
hamburger and french fries tomorrow when
1. Are you allergic to iodine or shellfish?
you come. I hate hospital food.
2. Are you afraid of heights?
2. I told my daughter who is pregnant to either
3. Do you get dizzy easily? come to see me tonight or wait until I go
4. Do you have any metal in your body? home from the hospital.
3. I understand it will be several weeks
79. An adult was born and raised in another country
before all the radiation leaves my
and received the BCG vaccine as a child. Upon
body.
taking a tuberculin skin test, a positive result is
seen. What information will the nurse base a 4. I brought several craft projects to do while
response on? the radium is inserted.
1. The only cause for a positive skin test and 83. The nurse is teaching auxiliary staff on the unit
negative chest X-ray is exposure to the about standard precautions. Which statement

832 NCLEX-RN Review


53155_09_PT 07_p824-844.qxd 2/27/09 6:31 PM Page 833

made by one of the aides indicates the best 3. A 1-year-old child who keeps pulling up in
understanding of the procedure? the crib and tries to climb out.
1. I should wear gloves when I give an enema 4. An adult who is supposed to be on bed rest
to a client who cant hold it. following surgery who tried to get up during
2. If I see a blood spill I will get iodine the night.
immediately and wipe up the spill.
88. The nurse is to administer an intramuscular
3. When I see used needles in the treatment
injection to a 1-year-old child. Which site is
room or the clients room, I will recap them
most appropriate for the nurse to select?
and put them in the Sharps box.
1. Dorsal gluteal.
4. I will wear a gown and mask whenever I go
into the room of a person with AIDS. 2. Ventral gluteal.
3. Ventral forearm.
84. The nurse is caring for a client who has patches on 4. Vastus lateralis.
both eyes following eye surgery. When entering the
room the nurse should do which of the following? 89. An adult client is being treated for
1. Announce presence and name clearly before hypertension. A low-sodium, low-fat diet is
entering the room. prescribed. The nurse knows the client
2. Speak in a louder tone than usual. understands his diet when he selects which
of the following menus?
3. Enter the room quietly and touch the client
before speaking. 1. Fried chicken, mashed potatoes, green beans,
and milk.
4. Refrain from saying things like I see to the
client. 2. Macaroni and cheese casserole, tossed salad
with dressing, and hot chocolate.
85. A client is admitted after being stabbed by a 3. Baked chicken, steamed broccoli and
knife and a chest tube is inserted. Immediately cauliflower, steamed rice, and hot tea.
following insertion of the chest tube, which 4. Steak, baked potato, peas, and hot
observation by the nurse best indicates the coffee.
drainage system is functioning adequately?
1. There is no bubbling in the water seal 90. An adult client is to have a pelvic sonogram
chamber. today to diagnose possible ovarian cysts.
2. The fluid level in the suction control When she arrives in the clinic area, which
chamber fluctuates with each respiration. question is the most important for the nurse
to ask?
3. The collection chambers are filling with
sanguinous drainage. 1. When was your last menstrual period?
4. The client reports pain relief. 2. What have you had to drink this
morning?
86. An adult is admitted to the medical unit with 3. When was your last bowel movement?
symptoms of angina. Nitroglycerin is 4. Did you use powder or deodorant
administered. Which assessment indicates the today?
client is responding positively to the
administration of nitroglycerin? 91. The nurse is working with a nursing assistant
1. The clients blood pressure drops. (NA). Which action should be delegated to the
2. The client reports he has developed a NA?
headache. 1. Ask the NA to apply the ice bag while the
3. The client asks to be discharged because his nurse performs decubitus ulcer care.
pain is relieved. 2. Ask the NA to assess the bowel sounds before
4. The client reports he has developed feeding the client.
nausea. 3. Ask the NA to perform decubitus care while
the nurse applies an ice bag.
87. The nurse is caring for several clients. Which 4. Ask the NA to feed the first meal to a client
client may need an order for wrist restraints? after surgery.
1. An elderly woman who is confused and pulls
out her intravenous line. 92. A man on the phone states he is a government
2. An adult man who refuses to have an official who needs access to a clients medical
intravenous line started. file. He states his name and position in the

COMPREHENSIVE PRACTICE TESTS 833


53155_09_PT 07_p824-844.qxd 2/27/09 6:31 PM Page 834

government. According to HIPPA, what should 1. The client is showing normal signs of pain
the nurse do next? and anxiety after surgery.
1. Give him the information, as all government 2. The client may be in the early stages of
officials are allowed access. congestive heart failure.
2. Ask for a number to reach the official and 3. The client will need to perform active range-
contact the charge nurse with the information. of-motion exercises for his legs.
3. Inform the official that only the medical staff 4. The client may be experiencing a pulmonary
are allowed access to the file. embolus.
4. Ask the client why the government official is 97. An unconscious client is receiving a transfusion
needing information about their file. of whole blood. Upon assessment, the nurse
finds a weak pulse, fever, and hypotension.
93. A middle-aged client was admitted for treatment
What would be the priority nursing action?
of secondary hypertension that has not
responded to lifestyle modifications over the last 1. Notify the physician.
5 weeks. Using the stepped-care approach, the 2. Stop the blood transfusion.
medications included in his treatment were 3. Recheck the vital signs.
furosemide (Lasix) and quinipril hydrochloride 4. Check the amount of urine output.
(Accupril). What type of information should the
nurse include in the discharge teaching? 98. The nurse is caring for an adult who has had an
1. Cholesterol restriction, weight reduction. acute myocardial infarction. The nurse finds
him very restless with a heart rate of 110,
2. Potassium restriction, limited activity.
respiratory rate of 28, and blood pressure 80/50.
3. Sodium restriction, increased activity. What is the most appropriate nursing action?
4. Magnesium restriction, limited alcohol intake. 1. Prepare for administration of vasoconstrictive
drugs.
94. A clients renal disease has progressed and has
decided with his physician to use continuous 2. Limit IV intake to 100 mL the first 2 hours.
ambulatory peritoneal dialysis (CAPD) as his 3. Prepare for insertion of CVP or pulmonary
treatment option. Which of the following choices artery catheter.
best reflects the information that the nurse 4. Prepare to apply MAST trousers/suit.
should include in the CAPD teaching plan?
99. The nurse is visiting a client at home and is
1. Low-sodium diet and Foley catheter care. assessing him for risk of a fall. What is the most
2. Low-protein/high-carbohydrate diet and care important factor to consider in this assessment?
of the AV shunt site. 1. Illumination of the environment.
3. Complications and aseptic technique for the 2. Amount of regular exercise.
Tenckhoff catheter.
3. The resting pulse rate.
4. Assessing for the bruit and thrill daily.
4. Status of salt intake.
95. A client has been admitted to the coronary care 100. An elderly client who lives at home is alert and
unit with the diagnosis of an anterior wall oriented and being treated for polyarthritis,
myocardial infarction. While on telemetry, his primary biliary cirrhosis, mild hypertension,
rhythm strip demonstrates frequent premature and glaucoma. He is prescribed four different
ventricular beats as well as occasional episodes medications, some of which are prescribed once
of second-degree heart block episodes. Which a day, some twice a day and one at different
alterative drug to suppress the ventricular times of the day. He states, Im not used to
ectopic beats is most likely to be taking all these medicines and sometimes I miss
prescribed? them all. Which of the following suggestions
1. Atenolol (Tenormin). would be most beneficial to the client to
2. Calcium gluconate. promote his medication regimen?
3. Verapamil (Calan). 1. Purchase a pill sorting box to arrange dosages.
4. Diltiazem (Cardizem). 2. Make arrangements for the public health
nurse to visit daily.
96. A client is 2 days post-op surgery. He is now 3. Require family members to administer
complaining of shortness of breath and had a medications.
positive Homans sign yesterday. How will the
4. Explain the importance of these medicines
nurse interpret the findings?
and tell the client he needs to find a better
way to remember.

834 NCLEX-RN Review


53155_09_PT 07_p824-844.qxd 2/27/09 6:31 PM Page 835

Answers and Rationales for Practice Test 7

ANSWER RATIONALE NP CN CL SA
#1. 4. These are symptoms of hypophosphatemia that may occur with Pl Ph/6 Ap 5
prolonged use of Amphojel. Constipation is an expected side
effect; the liquid tends to be more effective; large amounts of
liquid would dilute the medicine.
#2. 1. Hypoparathyroidism results in decreased calcium and increased Pl Ph/7 Ap 1
phosphorus levels. A high-calcium, low-phosphorus diet will be
prescribed, as well as the other interventions listed.
#3. 3. Lugols solution (iodine solution) may be given 10 to 14 days An Ph/6 Ap 1
before surgery to decrease vascularity of the thyroid and thus
prevent excess bleeding.
#4. 3. Brudzinskis sign is flexion at the hip and knee in response to As Ph/7 Co 1
forward flexion of the neck and may be present in a client with
meningitis. Chvosteks sign and Trousseaus sign may occur due
to the parathyroid glands being inadvertently removed and in
response to hypocalcemia; the laryngeal nerve may be damaged
during the surgery, in which sudden hoarseness develops.
#5. 4. The clients social network is expanded by attending a local Ev Ps/4 Ap 2
senior citizen group, and socializing with others outside of
the family.
#6. 2. Pets can provide an opportunity for touching and can promote Pl Ps/4 Ap 2
socialization and speech when it otherwise would not be
performed by the client. Family and friends are encouraged to
visit; speech therapy cannot correct the progression of organic
brain syndrome.
#7. 1. According to the National Elder Abuse Incident Study of 1998, As Ps/4 Co 2
the elderly Caucasian female who has physical and/or cognitive
impairment is at greatest risk for elder abuse by a family member.
#8. 1. Depressive symptoms include exaggerated feelings of sadness, As Ps/4 Co 2
dejection, worthlessness, hopelessness, and emptiness. Poor
grooming and hygiene are signs of mental decompensation
in mental illness such as in dementia and schizophrenia.
Intense anxiety alone is not a symptom and thought insertion
is a symptom of schizophrenia.
#9. 3. This stage of Ericksons developmental stages targets intimate An Ps/4 Ap 2
relationships. Trust vs. mistrust is in the infant stage; identity vs.
role confusion is in the adolescent stage; generativity vs. stagnation
is in the middle adult stage with concerns related to productivity
and contributing to society.
#10. 1. The client learned a method to use in dealing with problems/ Ev Ps/4 Ap 2
stressors such as verbalization rather than looking to quick and
sometimes superficial answers to problems.

ANSWERS AND RATIONALES FOR PRACTICE TEST 7 835


53155_09_PT 07_p824-844.qxd 2/27/09 6:31 PM Page 836

ANSWER RATIONALE NP CN CL SA
#11. 4. Using Maslows hierarchy of needs as well as basic concepts of An Ps/4 Ap 2
alcohol detoxification, the nurse needs to initially assess and
attend to the potential for physical problems associated with
withdrawal. There is no data to support suicidal thoughts;
ineffective denial would be a focus later in treatment after safe
detoxification has been achieved.
#12. 3. An ice pack helps to soothe the area by constricting vessels and Pl He/3 Ap 3
reducing inflammation. NPO status would be appropriate
following a cesarean section until bowel sounds are audible;
ice packs to the breasts would suppress lactation and are not
appropriate for a woman who plans to breastfeed; a catheter is
usually in place for a woman undergoing a cesarean section until
the first postpartum day so bed rest can be maintained.
#13. 3. Rho(D) should be given within 72 hours of delivery to the mother Ev Ph/6 Ap 3
when the mother is Rho(D) negative and the infant is Rho(D)
positive. Rho(D) is indicated following the termination of a
pregnancy, after amniocentesis, after abdominal trauma during
pregnancy, and after receiving a transfusion of Rho(D) positive
blood.
#14. 1. Monitoring fetal heart rate assesses fetal well-being and fetal Pl He/3 Ap 3
distress. A tocolytic is administered to relax the smooth muscles
of the uterus and inhibit uterine contractility. A tranquilizer may
depress the neonates respiratory center.
#15. 3. Persons with pernicious anemia are unable to absorb vitamin B12 An Ph/6 Ap 1
from the gastrointestinal tract, in which injections must be taken
for life. Iron is taken for iron deficiency anemia; blood transfusion
and corticosteroids help manage aplastic and hemolytic anemias.
#16. 1. A Schilling test utilizes radioactive vitamin B12 for gastrointestinal An Ph/7 Co 1
absorption of vitamin B12. The other choices do not provide a
definitive diagnosis.
#17. 2. With an oxalate stone, the client should limit excess intake of Pl Ph/7 Ap 1
food high in oxalate and maintain an alkaline ash diet for alkaline
urine. Foods high in purine are limited for persons with uric acid
stones or gout.
#18. 3. A warning sign of testicular cancer is a slight enlargement or Ev He/3 An 4
change in the consistency of the testes. The other findings are
considered normal.
#19. 3. Persantine is a peripheral vasodilator and should be withheld, Im Ph/6 Ap 1
along with antihypertensives and sedatives, to prevent a
hypotensive episode. The other medications listed are not
contraindicated before dialysis.
#20. 2. Air embolism is a potentially fatal complication characterized by An Ph/8 Ap 1
sudden hypotension, dyspnea, chest pain, cyanosis and weak,
rapid pulse. Complications of disequilibrium include: headache,
muscle twitching, backache, nausea, vomiting, seizures; internal
bleeding and hemorrhage present as restlessness; pale, cold
clammy skin; rapid, weak, thready pulse; increased respiration.

836 NCLEX-RN Review


53155_09_PT 07_p824-844.qxd 2/27/09 6:31 PM Page 837

ANSWER RATIONALE NP CN CL SA
#21. 1. The goal of conservative treatment is to pass the stone without Pl Ph/7 Ap 1
need for invasive procedures. Fluids will be forced to help flush
the stone through the urinary tract quickly and dilute the urine to
reduce the risk of forming additional stones. If obstruction is
present, conservative treatment would not be selected. Narcotic
analgesics would be required for severe pain on an ordered
schedule.
#22. 2. Very cold air, especially if it is dry, is likely to cause Pl Ph/7 Ap 1
brochospasms, which make breathing even more difficult.
Oxygen flow rate is typically kept at 23 liter/min because high
oxygen levels can reduce the hypoxic drive to breathe. Fluid will
help to liquefy the secretions and make them easier to clear from
the airways. The position of choice is a forward-leaning position
or high-Fowlers.
#23. 4. Large amounts of simple carbohydrates in the diet produce a Pl Ph/7 Ap 1
high osmotic pressure within the intestine, which draws fluid
into the intestine from surrounding cells, causing the early
dumping syndrome. The hypoglycemic effect noted in late
dumping syndrome develops from production of large amounts
of insulin when the intestinal contents are high in simple carbs.
Reducing dietary carbs, using complex carbs, increasing fat and
protein delays gastric emptying time. The client should be
encouraged to lie on the left side, withhold fluid during the
meal and eat six small meals rather than three large ones to
slow gastric emptying.
#24. 3. Antacids containing magnesium have diarrhea as a side effect. An Ph/6 Co 5
Choices 1 and 4 cause constipation; 2 is an antidiarrheal.
#25. 4. Laxatives are given, if needed, the night before surgery to Im Ph/7 Ap 1
help cleanse the bowel.
#26. 3. Colace is a stool softener and the laxative of choice for clients An Ph/6 Ap 1
who should not strain during defecation. It causes water and
fats to penetrate the stool, making it easier to move the feces
along. A bulk-forming laxative would hold a large amount of
water, such as Metamucil; mineral oil is a laxative that retards
colonic absorption of water; Lactulose is a laxative that retards
colonic absorption of water and increases colonic peristalsis.
#27. 2. A hiatal hernia is a structural defect of a weakened diaphragm An Ph/7 Ap 1
and is aggravated by reclining and activities that cause an increase
in intra-abdominal pressure. The client should be instructed to
wait at least 2 hours after eating before lying down.
#28. 1. A total laryngectomy leaves the client unable to speak, so it will Pl Ph/7 Ap 1
be important to plan ahead for an alternate method of
communication.
#29. 1. Nausea and vomiting are the most common side effects of cancer Pl Ph/6 Ap 1
chemotherapy, the other choices are not.
#30. 4. There is no pain, bilateral redness, or increase in intraocular As Ph/8 An 1
pressure with a detached retina. Symptoms include gaps in
vision, flashes of light floating particles before the eyes, a curtain
over the field of vision, and blindness, if not treated.

ANSWERS AND RATIONALES FOR PRACTICE TEST 7 837


53155_09_PT 07_p824-844.qxd 2/27/09 6:31 PM Page 838

ANSWER RATIONALE NP CN CL SA
#31. 2. Positioning to keep the retina next to the choroid and the area Pl Ph/7 Ap 1
of detachment dependent is important. This may be on the
operative or the nonoperative side depending on the position of
the detachment. The client is to avoid coughing, wear patches on
both eyes, and eyedrops are given for dilation and to paralyze
the eye muscles postoperatively.
#32. 2. Mnires disease is a chronic disease of the inner ear As Ph/7 An 1
characterized by recurrent episodes of vertigo, tinnitus, and
progressive unilateral nerve deafness. Nausea may result
from the vertigo the client experiences.
#33. 4. Clients with Mnires disease are on a restricted fluid intake Ev Ph/7 Ap 1
and may also be on diuretics. Other management includes
smoking cessation, moving slowly, and a low-sodium diet.
#34. 2. The catheter should be sufficiently stiff so as not to coil in the As Ph/7 Ap 3
esophageal pouch and should never be forced when resistance
is felt. Air can be instilled into the stomach or gastric contents
aspirated to confirm a patent esophagus. Placing the infant with
head downward is an intervention rather than an assessment.
If the defect is the most common type (upper atresia with lower
fistula into the trachea), air will collect in the stomach with
crying, causing upward pressure on the diaphragm. Because
air rises, the infants head should be elevated to reduce gastric
distension from air. Continuous suction of the pouch with a
catheter in place keeps the mucus away from the upper trachea.
#35. 2. Increased systolic pressure is a result of fibrosis of blood As He/3 An 1
vessels and calcification and elongation of arteries, which
frequently occur in aging. Dehydration is likely to cause
hypotension. Less muscle mass or impaired lung capacity
are both normal occurrences in aging but do not cause increased
systolic pressure.
#36. 4. The earliest sign of digitalis toxicity is vomiting, although one Ev Ph/6 Ap 4
episode does not warrant discontinuing medication. Bradycardia
is also associated with digitalis toxicity.
#37. 1. It is of the utmost importance to protect the child from infections, Ev Ph/7 Ap 4
by wearing a mask and avoiding crowded areas. The childs
energy level can dictate his activity; and family can learn the
specifics of care required.
#38. 4. Talipes equines refers to plantar flexion, which lowers the toes As Ph/7 An 3
below the level of the heel. Talipes varus refers to the inversion
of the whole foot. Boys have a 2:1 higher incidence of clubfoot
than girls. Manipulation and casting are usually begun immediately
upon discovery of the defect, as the infants bones are most
flexible during the newborn period.
#39. 4. Cervical traction for cervical disc disease is the only one listed Im Ph/7 An 1
that can be intermittent.
#40. 2. The two major complications of a herniorrhaphy are a distended Im Ph/7 Ap 1
bladder and scrotal swelling. It is important to assess for
difficulty in urinating postoperatively.

838 NCLEX-RN Review


53155_09_PT 07_p824-844.qxd 2/27/09 6:31 PM Page 839

ANSWER RATIONALE NP CN CL SA
#41. 3. All of the choices are normal for a 9-month-old except sitting up An He/3 Ap 4
without support and would need further investigation.
#42. 3. The nurse is firm in carrying out the order and separating it Im He/3 Ap 4
from behavior.
#43. 1. This is known as the oedipal stage, in which it is normal for the Im He/3 Ap 4
child to want to marry the parent of the opposite sex. The child
will outgrow this and only needs to be reminded that he cant
marry his parent.
#44. 3. A syringe is used for accuracy. A second nurse should always Im Ph/6 Ap 4
verify potent drugs that require measurement. Placing medication
in formula will alter the taste causing the infant to refuse it and it
may be unknown whether all the medication is ingested. The apical
pulse should not be below 100 to 110 for a young infant. The nurse
is responsible for giving the medication and will demonstrate to the
mother as part of discharge teaching.
#45. 2. All the diagnoses are relevant at some point in the care of this An Ph/7 Ap 4
child, but the fluid and electrolyte balance is the most immediate
concern.
#46. 2. A quick assessment is to look for uneven shoulders. Ask the girl As He/3 An 4
to bend over and look at bra strap marks to see if one side is deeper.
Walking a straight line, palpating the spine, or watching
physical activities are all unreliable assessments.
#47. 3. The biggest risk to this child is that his airway will close off. If this Pl Ph/7 Ap 4
occurs, a tracheotomy will be necessary to save his life. Suctioning
could be dangerous as it may irritate, increase swelling, and cause
complete blockage of the airway.
#48. 4. This is the only correct answer that would be found related to As Ps/4 An 1
venous ulcers. The other choices are descriptors of arterial
insufficiency.
#49. 3. An intrauterine device is least appropriate. With a history of An He/3 Ap 3
numerous partners, she is at increased risk of infection, a common
problem with IUDs. There are no risk factors presented that
would contraindicate the use of oral contraceptives; condoms,
foams or the diaphragm are acceptable methods but do require
motivation to use.
#50. 4. Reva Rubin identified the phases of adjustment following An He/3 An 3
delivery as taking-in, taking-hold, and letting-go. Lack of interest
in infant care and the need to talk about herself are perfectly
normal in this initial phase of taking-in.
#51. 1. It is rare to be able to retract the foreskin of a newborn. This does Im He/3 An 3
not indicate a need for circumcision.
#52. 2. Passing flatus would indicate normal peristaltic activity. It would As Ph/5 Ap 3
be unlikely to have a bowel movement so soon; the nurse would
assess the bowel sounds with a stethoscope, (not by asking the
client to describe them); hunger alone is not a criterion for the
beginning of eating solid foods.

ANSWERS AND RATIONALES FOR PRACTICE TEST 7 839


53155_09_PT 07_p824-844.qxd 2/27/09 6:31 PM Page 840

ANSWER RATIONALE NP CN CL SA
#53. 4. A contraction lasting longer than 90 seconds increases the risk Im Ph/6 Ap 3
of fetal distress and uterine rupture. Safe practice requires the
nurse to immediately discontinue the Pitocin.
#54. 3. Assessment is done first by seeing what her normal eating Pl Ph/7 An 3
habits are. The diet can then be individualized to her needs
and compliance will be better. Insulin is not needed at this time.
#55. 4. All choices are appropriate but the most important is the risk of An Ph/8 An 3
seizures and the probability of already suffering intrauterine
growth retardation. Her physical well-being and that of the fetus
are in jeopardy.
#56. 4. Given the mother is pushing, the cervix is already completely As He/3 An 3
effaced and dilated. A full bladder may hold the fetus back.
Most likely the fetal membranes are ruptured.
#57. 1. Wearing gloves is part of standard precautions, especially since Im Sa/2 Ap 3
the newborn is coated with amniotic fluid. The admission is
usually done under a warmer and is it not practical to wait until
the newborn is sleeping, although it is better to obtain heart
rate and respiratory rate while the infant is quiet.
#58. 4. Prone is the best position for minimal pressure on the defect. Im Ph/7 An 3
Rupture presents a surgical emergency and all efforts are taken
to avoid it.
#59. 3. The symptoms described are side effects of antipsychotic Im Ph/6 An 5
medication and are not the symptoms of agitation often seen
in a schizophrenic client. Cogentin is the only medication listed
that will decrease the side effects of the antipsychotic medication.
#60. 4. Because the DM-IV-TR criteria for paranoid schizophrenia focus Ev Ph/6 An 2
on delusions and suspiciousness, a decrease in these symptoms
would be the expected outcome from antipsychotic medication.
#61. 1. This response points out the clients strength in a realistic manner Im Ps/4 Ap 2
and is an attempt to improve self-concept. The other choices are
examples of judging, rejecting, or not promoting self-esteem.
#62. 2. Abusive behavior often occurs when parents lose control or Pl Ps/4 An 2
feel overwhelmed.
#63. 3. The most important improvement would be the clients ability Ev Ps/4 An 2
to live within the guidelines, rules, and regulations, as well as
a decrease in testing limits.
#64. 2. A cystoscopy is the examination of the bladder with the lighted Pl Ph/7 Ap 1
cystoscope. Water may be drunk the night before, as the client
is likely to be NPO after midnight. Enemas are given if the
bowels are examined.
#65. 1. A Tensilon test yields immediate result in evaluating Ev Ph/7 An 1
myasthenia gravis. If positive, the client almost immediately
has an increase in muscle strength by increasing the amount
of acetylcholine available.
#66. 2. Serum creatinine and blood urea nitrogen (BUN) are tests of An Ph/7 An 1
kidney function. Before giving contrast media, it is essential
to be sure the kidneys can excrete the dyes.

840 NCLEX-RN Review


53155_09_PT 07_p824-844.qxd 2/27/09 6:31 PM Page 841

ANSWER RATIONALE NP CN CL SA
#67. 2. As one ages, the eyes undergo changes including a decreased As He/3 Ap 1
ability to focus on near objects, increased difficulty with color
discrimination, and a lessened field of peripheral vision.
#68. 3. Dysarthria is the term used to describe difficulty speaking when As Ph/7 Ap 1
muscle impairment is present. Semantic is the inability to
understand the meaning of words; receptive aphasia is the
inability to understand spoken or written words; and dysphagia is
difficulty swallowing.
#69. 3. Swelling, heat redness, impaired function, and pain are cardinal As Ph/7 An 1
signs of local inflammation. Fever and malaise are signs of
systemic inflammation. Confusion could be a sign of a systemic
infection, impaired oxygenation or brain dysfunction, not a local
inflammation.
#70. 1. A liquid diet should be given for 5 days prior to surgery to ensure Pl Ph/7 Ap 1
adequate cleansing of the bowel before surgery.
#71. 2. Arterial blood gases are usually done by the respiratory therapist, Im Ph/7 Ap 1
however, it is essential that pressure is applied to the puncture site
for 5 minutes to ensure the client does not bleed from the arterial
puncture.
#72. 4. The ABGs provide the most specific information about the Ev Ph/7 An 1
adequacy of the oxygen therapy. Other factors may influence the
other choices.
#73. 4. A pH of 7.35 is on the acid side of normal. All the other values An Ph/7 An 1
are abnormal so the client has compensated. The CO2 is sharply
elevated and will lower the pH. The HCO3 is also elevated and
is responsible for bringing the pH up to the normal range. An
abnormal O2 suggests that the problem is a respiratory one.
#74. 1. The client should be able to move the toes, but be unable to As Ph/8 Ap 1
move and will complain of pain if the leg is moved. The injured
extremity will appear shorter and be in external rotation.
#75. 1. To keep the hip in the socket, the affected leg should be kept Ev Ph/7 An 1
abducted and externally rotated. An abduction pillow can be
used to achieve the position.
#76. 2. Guillain-Barr is characterized by an ascending paralysis that Im Ph/7 Ap 1
usually paralyzes the respiratory muscle before descending. It is
thought to be an autoimmune response following a viral infection.
If the client receives good respiratory support, the paralysis will
descend after a few days.
#77. 3. Cloudy vision and gradual blurring are symptomatic of As Ph/8 An 1
cataracts. The other choices are from a foreign body,
acute glaucoma, infection, allergy, or migraine headaches.
#78. 4. Metal in the body, such as pacemakers, aneurysm clips, and As Ph/7 Ap 1
hip prostheses can cause serious injury and/or cause artifacts
in the images. Dye is not used, but the client will be placed in
a cylindrical scanner so explain that some suffer a feeling of
claustrophobia, and they will have to remain totally still.

ANSWERS AND RATIONALES FOR PRACTICE TEST 7 841


53155_09_PT 07_p824-844.qxd 2/27/09 6:31 PM Page 842

ANSWER RATIONALE NP CN CL SA
#79. 4. BCG vaccine is given in many parts of the world to immunize An He/3 An 1
against tuberculosis. It causes formation of antibodies and
consequently a positive reaction to a tuberculin skin test.
A positive skin test indicates the vaccine is working and
producing antibodies.
#80. 3. Catsup is high in sodium so other choices should be suggested. Ev Ph/5 Ap 1
#81. 3. Lying down is not required after eyedrops. The other choices Im Ph/6 Ap 5
are correct information.
#82. 2. Pregnant visitors and children under 16 are not allowed in the Ev Sa/2 An 1
room if internal radiation therapy is used. The client will be on
a clear liquid or low residue diet; the client will no longer be
contaminated with radioactivity once the source is removed
(probably 36 to 72 hours after insertion); crafts may require
sitting, but the client will need to lie flat with very little head
elevation.
#83. 1. Fecal material is a body fluid and could transmit AIDS or hepatitis. Ev Sa/2 Ap 1
Blood spills are cleaned with chlorine bleach; needles are never
recapped; a gown and mask are unnecessary to wear in an AIDS
clients room unless contact is to be made with body fluids.
#84. 1. Announcing ones name will prevent the client from being startled, Im Sa/2 Ap 1
especially before touching the client. A normal tone of voice and
language is the most appropriate method.
#85. 3. Filling of the collection chambers with sanguinous (bloody) Ev Ph/7 Ap 1
drainage indicates the drainage system is functioning. There
should be intermittent bubbling in the water seal chamber.
Absence of bubbling shortly after the chest tube insertion indicates
an obstruction in the tubing. Fluid level fluctuation with
respiration may occur in the fluid in the water seal chamber.
#86. 3. The purpose of administering nitroglycerin is to improve blood Ev Ph/6 Ap 1
flow to the myocardium and relieve chest pain. Side effects of
nitroglycerin may cause a blood pressure drop and a headache.
Nausea is an unwarranted symptom and may indicate the
condition is getting worse.
#87. 1. Wrist restraints may be indicated for a confused client who is As Sa/2 Ap 1
pulling out essential lines; however the least restraint possible
should be used for the shortest period of time.
#88. 4. The vastus lateralis is the most appropriate site as the other Im Ph/6 Ap 5
sites are not used until the child has been walking and develops
some muscle, and there is danger of hitting vessels and nerves.
The forearm is for skin tests.
#89. 3. All the foods are low in sodium and low in fat. Fried chicken Ev Ph/5 Ap 1
and milk are high in fat, as is the cheese, salad dressing, hot
chocolate, and steak. A baked potato is acceptable without
butter, cheese, or sour cream.
#90. 2. Persons having a pelvic sonogram should drink several As Ph/7 Ap 1
glasses of water before the procedure so the bladder is full.
No bowel prep is required and the procedure is safe even
if pregnant.

842 NCLEX-RN Review


53155_09_PT 07_p824-844.qxd 2/27/09 6:31 PM Page 843

ANSWER RATIONALE NP CN CL SA
#91. 1. Delegation of routine care is given to nonprofessional staff. An Sa/1 Ap 1
All activities, except applying ice, require assessment, skills,
or safety precautions that the registered nurse should
perform.
#92. 2. The medical staff has access to the clients file only if they As Sa/1 Ap 1
are assigned to that particular client. This does not mean
that government officials are not allowed access. Sometimes
police/courts need access for legal reasons. The nurse should
first alert the charge nurse, who will handle the situation or
contact her superior for further decision making.
#93. 3. Discharge information should include reducing sodium in Pl Ph/7 Ap 1
the diet, losing weight, increasing exercise, avoiding tobacco
use, and reducing stress, along with the actions and side
effects of drug therapy. There is no need to limit activity,
restrict potassium or magnesium.
#94. 3. In addition to review of the disease process, a brief review of Pl Ph/7 Ap 1
anatomy and physiology is needed and the importance of good
handwashing with aseptic technique during the exchange
procedure. Possible complications and appropriate responses
are also essential parts of the teaching plan. The client will not
have an indwelling Foley catheter, nor an AV shunt or fistula.
#95. 1. A cardioselective beta blocker is less likely to cause or worsen Ev Ph/6 Co 5
the heart block while still effectively suppressing the
ventricular ectopic beats. Calcium gluconate is contraindicated
in ventricular fibrillation; verapamil is contraindicated in AV
block; diltiazem is contraindicated in second-degree heart
block.
#96. 4. There is a risk of developing a pulmonary embolism as a result As Ph/7 An 1
of venous thrombosis in the lower extremity following surgery.
The Homans sign is pain in the calf when the foot is passively
dorsiflexed, which may indicate a deep vein thrombosis. More
diagnostic testing should follow. Shortness of breath may
indicate the thrombosis has traveled and may be in the lungs.
Emergency intervention should be initiated.
#97. 2. The symptoms are indicative of a transfusion reaction. When Im Ph/6 An 1
a reaction is suspected, the transfusion should be stopped
immediately and the IV line kept open with normal saline. The
physician should be notified and the blood and tubing will be
sent back to the lab. Vital signs will continue to be recorded.
#98. 3. A CVP or pulmonary artery catheter monitor fluid levels in the Im Ph/8 An 1
blood, which will provide early detection of high or low levels
and promote management of complication. Fluids may or may
not be restricted and MAST (Medical Anti-Shock Trousers)
trousers are indicated if severe abdominal blood is occurring
to slow the progress of shock.
#99. 1. Nightlights would help the client see to prevent falls. Other As Sa/2 An 1
factors to assess include removing loose scatter rugs, cleaning
up spills, and installing handrails/grab bars as appropriate. The
other choices are not related to preventing falls.

ANSWERS AND RATIONALES FOR PRACTICE TEST 7 843


53155_09_PT 07_p824-844.qxd 2/27/09 6:31 PM Page 844

ANSWER RATIONALE NP CN CL SA
#100. 1. Older adults make medication administration errors for many Pl Ph/7 Ap 1
reasons, the most common of which is forgetfulness. Pill boxes
come in an assortment of styles and are very useful in organizing
dosages, and are used for any client who is taking several
medications either daily or more than once each day. The client
should be able to be responsible for his own medication because
he is alert and oriented. Encouragement, teaching, and follow-up
are important to promote adherence to medication regimens.

844 NCLEX-RN Review


53155_09_PT 08_p845-864.qxd 2/27/09 6:32 PM Page 845

Practice Test 8

1. The nurse is assessing an elderly client. Which 5. A young child is brought to the emergency room
finding is most apt to be seen in the elderly by her parents with a fractured arm, which they
client with dementia? say she sustained when she fell down the stairs.
1. Good hygiene and grooming. Which of the following would the nurse expect
2. Rapid mood swings. to find in the assessment of the child if she has
been abused?
3. Agnosia.
1. A child who is very trusting of her nurse
4. Phobias and unwanted thoughts and
since she has not been able to trust her
behaviors.
parents.
2. An adult is being treated for second- and third- 2. The child will be constantly asking for her
degree burns over 25% of his body and is now parents to comfort her even if they abused her.
ready for discharge. The nurse evaluates his 3. A child who doesnt cry much and who
understanding of discharge instructions relating responds very little to her environment.
to wound care and is satisfied that he is prepared 4. The child will be bouncing around, feeling
for home care when he makes which statement? safe and happy to be away from an abusive
1. I will need to take sponge baths at home to home.
avoid exposing the wounds to unsterile bath
water. 6. A pregnant woman is diagnosed as being
2. If any healed areas break open I should first anemic. Her physician has told her to eat an
cover them with a sterile dressing and then iron-rich diet and to take iron supplements BID.
report it. What instruction should the nurse plan to give
about taking iron?
3. I must wear my Jobst elastic garment all day
and can only remove it when Im going to 1. Iron should be taken only on an empty
bed. stomach at least 1 hour before meals.
4. I can expect occasional periods of low-grade 2. It is good to increase consumption of dairy
fever and can take Tylenol every 4 hours. products while taking iron.
3. Citrus juice taken with iron will increase
3. The nurse is developing a care plan for a 2-year- absorption.
old girl with Hirschsprungs disease. Which 4. Iron supplements often cause diarrhea and
would be contraindicated for the care plan should be discontinued if diarrhea develops.
interventions?
1. Administer stool softeners. 7. During parenting classes, the nurse teaches
2. Give isotonic enemas. parents the importance of immunizations and
the schedule that will be implemented. Which of
3. Have client follow a low-fiber diet.
the following findings indicate the nurses
4. Place on fluid restriction. teaching has been effective?
4. A 2-year-old is admitted with flu and dehydration. 1. The parents are able to list three reasons to
The history assessment reveals high fevers, little immunize and when to begin immunization.
food or fluid intake for several days, has slept 2. Taking the infant for his first immunization at
almost constantly, and weight has dropped from 2 weeks of age.
30 lb to 21 lb. The mother has given him baby 3. The parents state their intent to follow a
aspirin, decongestants, and leftover amoxicillin printed immunization schedule.
from a past ear infection. What fact increases his 4. By 6 months of age, the infant has received
risk for Reyes syndrome? the recommended immunizations.
1. The use of aspirin.
2. His high fevers. 8. A woman delivered her first baby 12 hours ago.
She calls the nurse in tears stating that she has
3. The use of antibiotics previously prescribed.
been unable to get the baby to nurse. All she
4. Severe dehydration as evidence by weight does is cry when I try to get her to nurse. The
loss.

COMPREHENSIVE PRACTICE TESTS 845


53155_09_PT 08_p845-864.qxd 2/27/09 6:32 PM Page 846

nurse comes to assist her. What is the nurses 3. The warm sitz baths relieve the discomfort
best initial action? I feel from the incision on my bottom.
1. Explain the basics of breastfeeding to the 4. This T-binder helps support my abdominal
client. incision.
2. Assess her nipples and the measures she has
tried. 13. An adult had a tuberculin skin test, which the
nurse reads as positive. Which of the following
3. Demonstrate proper positioning of mother
is true about the tuberculin skin test?
and baby.
1. The intradermal test does not differentiate
4. Find out if she really wants to breastfeed or
active tuberculosis from dormant infections.
would rather bottle-feed.
2. The induration is measured in cm.
9. A postpartum client complains of sore nipples, a 3. A positive test has a diameter of 5 mm.
sore bottom, cramping, fatigue, and lack of 4. Results of a tuberculin skin test must be read
ability to satisfy her newborn, who is crying. within 24 hours.
Based on these data, which of the following
would be an appropriate nursing diagnosis? 14. A man fractured his femur yesterday. In writing
1. Ineffective parenting. his care plan, the nurse notes to observe for a fat
2. Alteration in comfort. embolism from the long-bone fracture. Which of
the following is likely to be seen with a fat
3. Anxiety related to new role of parenting.
embolism?
4. Knowledge deficit.
1. Bradycardia.
10. A client in labor has been taught to use 2. Dyspnea.
breathing to help her cope with the discomfort 3. Edema in lower extremities.
of contractions. How would the nurse best 4. Altered level of consciousness.
evaluate the effectiveness of teaching?
1. Ask the client to demonstrate each of the 15. An adult has chronic renal failure. She begins
techniques and state the appropriate time in complaining to the nurse of increasing
labor to use it. numbness of her right hand and leg cramps. Of
2. Observe the clients use of breathing with the medications ordered, which is the
contractions. appropriate medication for the nurse to
administer to help alleviate these symptoms?
3. Have the client list the two main respiratory
techniques and the variations. 1. Lasix.
4. Identify the clients request for pain 2. Amphojel.
medications or refusal as evidence of good 3. Dilantin.
breathing. 4. Magnesium sulfate.

11. The nurse is to give Coumadin (warfarin) 10 mg 16. An adult had a barium enema for complaints of
PO. The nurse is comparing the anticoagulants chronic diarrhea, right lower quadrant pain,
Coumadin and heparin. Which statement is weight loss, and weakness. The enema revealed
correct regarding the therapies? the characteristic string sign. Which
1. Heparin is measured in mg; Coumadin inflammatory disorder would the nurse suspect?
dosage is measured in units. 1. Ulcerative colitis.
2. Both have few drug interactions. 2. Crohns disease.
3. Heparin therapy is monitored by APTT; 3. Diverticulosis.
Coumadin therapy is monitored by PT. 4. Gastritis.
4. Heparin dissolves existing clots; Coumadin
does not. 17. A baby boy who is 8 hours old is in his mothers
room. Which finding by the nurse indicates the
12. An adult is post-op abdominoperineal resection newborn needs his environment altered to
(AP resection) for colon and rectal cancer. Which promote adjustment to extrauterine life?
statement indicates to the nurse that he requires 1. The baby just regurgitated his formula.
further teaching concerning his recovery? 2. Petechiae are present on his head.
1. Im glad this colostomy is only temporary. 3. His hands and feet have a blue-tinged color.
2. Ill have to cut back on eating coleslaw, my 4. His axillary temperature is 36C (96.8F).
favorite type of salad.

846 NCLEX-RN Review


53155_09_PT 08_p845-864.qxd 2/27/09 6:32 PM Page 847

18. A 3-year-old child has been diagnosed with 3. He will not rock back and forth as he tries to
Wilms tumor and is scheduled for surgery. The crawl.
nurse performing the preoperative assessment 4. Though he wont cuddle, his body is relaxed
must modify the usual procedure. Which when picked up.
procedure would be contraindicated for this
child? 23. What would be an appropriate intervention for a
1. Auscultation of the lungs. young child who has atopic dermatitis?
2. Assessment of parents understanding of the 1. Avoid bathing until condition has
childs condition. subsided.
3. Measurement of vital signs. 2. Avoid all eggs and milk products.
4. Palpation of abdomen. 3. Keep socks on hands.
4. Use lotions with a higher water content.
19. An elderly client is being treated for chronic
open-angle glaucoma. Which medication is 24. When a client who is experiencing a conversion
contraindicated for her? disorder exhibits paralysis, the nurse should
1. Pilocarpine eyedrops. provide which therapeutic approach?
2. Diamox. 1. Develop trust through a therapeutic one-to-
3. Mannitol. one relationship in which acceptance of the
disorder is conveyed.
4. Neo-Synephrine eyedrops.
2. Confront the client gently about the fact
20. The nurse is caring for an adult who has just that no physical basis exists for the
returned to the nursing care unit following a conversion.
radical neck dissection for squamous cell 3. Probe into the nature of the recent conflict-
carcinoma of the mouth. Which nursing action producing event and the resultant paralysis.
would be inappropriate during the early 4. Provide a treatment approach in which the
postoperative period? paralysis receives negative reinforcement so
1. Provide mouthwash and lemon and glycerin the client will eventually give up the
swabs at the bedside to maintain the clients symptom.
comfort and oral hygiene.
2. Place the client in a side-lying position 25. What would be important for the nurse to
initially, then in Fowlers position. provide for a newly admitted client with
schizophrenia?
3. Place oral fluids in the back of the throat with
an asepto syringe. 1. An environment that makes minimal
demands on the client.
4. Monitor for facial drooping and circumoral
numbness or tingling. 2. An environment that provides maximal
stimulation for the client.
21. A client is suffering from rejection of a kidney 3. A climate in which the client can reflect on
transplant and is told that kidney dialysis is the her problems.
next treatment of choice. The client states, Im 4. A climate of social relatedness for the
not going on that machine. My kidneys will hold client.
out until I find another kidney. The nurse
recognizes what defense mechanism is being 26. A middle-aged client is admitted following an
used? overdose of prescribed antidepressant
1. Rationalization. medication. He tells the nurse he may be losing
2. Intellectualization. his job and he would rather die than be faced
with unemployment. What is the most
3. Denial.
appropriate short-term goal for him?
4. Suppression.
1. To look at the help-wanted ads in the local
22. A 10-month-old is totally unresponsive to his newspaper every day.
parents talking to him or being cuddled by them. 2. To identify one new adaptive coping
What will the nurse most likely find upon mechanism by the end of the week.
assessment if the childs behavior is a product of 3. To contract for safety while on the unit.
infant autism? 4. To contact his supervisor at work to discuss
1. The child is responsive to other adults. job possibilities within the company by the
2. Babble is less than usual. time of his discharge.

COMPREHENSIVE PRACTICE TESTS 847


53155_09_PT 08_p845-864.qxd 2/27/09 6:32 PM Page 848

27. What would be an appropriate short-term goal 1. The presence of edema.


for a client with borderline personality disorder? 2. Complaints of pain in great toe.
1. Discuss feelings of self-destruction with the 3. Urine output increased.
nurse rather than to act out impulses. 4. Decreased blood pressure.
2. State ability to use problem solving as a
means to deal with life problems. 32. The nurse is interviewing the mother of a child
3. Control impulses through use of prn who is scheduled for a tonsillectomy. Which
medications. question is most essential for the nurse to ask
4. Identify the process of splitting before acting the mother?
out the behaviors. 1. Have you had your tonsils out?
2. Does your child get colds easily?
28. A client with a history of alcoholism is admitted 3. Does your child have any bleeding
for pneumonia. What will be the nurses reason tendencies?
for investigating the amount of alcohol the client
4. Has your child ever had an operation?
has consumed during the 24 to 48 hours before
admission? 33. A client is showing sinus tachycardia on the
1. To determine how far the disease has telemetry monitor. Which medication would the
progressed. nurse expect to give?
2. To determine the severity of withdrawal. 1. Atropine.
3. To determine whether the client will 2. Lidocaine.
experience delirium tremens. 3. Diltiazem.
4. To recommend Alcoholics Anonymous. 4. Epinephrine.
29. The nurse is assessing a 17-year-old female who 34. A 2-year-old is to be admitted to the pediatric
is admitted to the eating disorders unit with a unit. His diagnosis is febrile seizures. In
history of weight fluctuation, abdominal pain, preparing for his admission, which of the
teeth erosion, receding gums, and bad breath. following is the most important nursing
Which of the following assessments will be the action?
least useful as the nurse develops the care
1. Order a stat admission CBC.
plan?
2. Place a urine collection bag and specimen
1. Information regarding recent mood
cup at the bedside.
changes.
3. Place a cooling mattress on his bed.
2. Family functioning using a genogram.
4. Pad the side rails of his bed.
3. Ability to socialize with peers.
4. Whether she has a sexual relationship with a 35. The nurse is caring for a 2-year-old who is
boyfriend. admitted for febrile seizures. Which of the
following is the primary goal of nursing care
30. The mother of a 6-day-old breastfed infant calls following admission?
the nurse and asks how she can be sure her baby
1. Recognition of abnormal signs or behavior
is getting enough milk since she cannot see how
indicative of a neurological deficit.
much milk there is. He is breastfeeding
approximately every 3 hours. The nurse would 2. Early detection and recognition of elevated
tell the mother to evaluate the effectiveness of temperatures.
breastfeeding by what action? 3. Identification of the etiology of the childs
1. Weigh the baby before and after feedings. febrile condition.
2. Offer the baby water after a feeding to 4. Promotion of rest and comfort in an
determine if he is still hungry. atmosphere of limited stimulation.
3. Compare the amount of time the baby cries 36. A 2-year-old child is admitted with febrile
with his contented time. seizures. A Tylenol (acetaminophen) suppository
4. See if the baby has at least 6 wet diapers in was given as ordered. Which intervention
24 hours. should be included in the care plan?
1. Remove extra blankets to decrease the
31. A client is administered furosemide (Lasix)
metabolic rate.
100 mg IV. What finding would signal a possible
complication is occurring? 2. Ask the mom to report any seizure activity.

848 NCLEX-RN Review


53155_09_PT 08_p845-864.qxd 2/27/09 6:32 PM Page 849

3. Retake the temperature in 30 minutes. 1. Do you have syrup of ipecac in your


4. Prepare an ice collar for the child. house?
2. How much does your child weigh?
37. A new graduate is to administer 2 units of PRBC 3. When was the childs last bowel movement?
(packed red blood cells) for a client whose
4. When did your child last eat?
hemoglobin is 7.2. What action will warrant
intervention by the observing nurse? 42. A child has taken approximately 10 childrens
1. The blood tubing has been primed with aspirin. The physician places a nasogastric tube
normal saline. in the child and administers activated charcoal.
2. The blood is piggybacked into the tubing with The mother asks the nurse what the charcoal is
the clients maintenance fluids of D5 12 NS. used for. What is the nurses best response?
3. The client has signed a consent for the 1. It induces vomiting.
administration of blood products. 2. It will correct electrolyte imbalances.
4. The graduate only obtains 1 unit of blood 3. It will absorb the aspirin in the stomach.
from the blood bank. 4. It will prevent bleeding tendencies.
38. When implementing any method to reduce a 43. A child who has swallowed several aspirin is
childs fever, chilling should be avoided. Which admitted to the emergency room.
of the following best explains the reason for this Aquamephyton is ordered. For what reason does
principle? the nurse tell the mother that it is given?
1. Chilling makes the child irritable and 1. To promote metabolism of aspirin.
uncooperative.
2. To prevent bleeding.
2. Chilling causes peripheral vasodilation.
3. To decrease absorption of aspirin.
3. Chilling can lead to shivering.
4. To prevent hepatotoxicity.
4. Chilling can lessen the childs ability to fight
infection. 44. The nurse is giving instructions to a client who
has a prescription for penicillin V potassium
39. The nurse is caring for a client admitted with (Pen Vee K) 500 mg tablets to be given 4 times a
pneumococcal pneumonia in the left lower lobe. day. Which statement by the client indicates the
Percussion of the clients left lower lobe would best understanding of the instructions regarding
most likely produce which of the following taking this medication?
findings?
1. I will take the pill with a full glass of orange
1. Rales. juice to make it work better.
2. Rhonchi. 2. I will call my doctor when I feel better so
3. Hyperresonance. I can stop taking the medicine.
4. Dullness. 3. I should take the pill 1 or 2 hours before
meals or 2 to 3 hours after meals.
40. Percussion and vibration are ordered as part of
4. I know I could get a rash, so I wont worry if
chest physical therapy for a client who has
I start to itch.
pneumonia. How often does the nurse understand
that percussion should be performed? 45. The nurse is caring for a client who had major
1. Intermittently during postural drainage. abdominal surgery. At 3:00 P.M. the day
2. At each bronchopulmonary segment following surgery, the clients temperature is
immediately prior to postural drainage. 102.6F. What should the nurse do first to
3. At each bronchopulmonary segment determine the most likely cause of the
immediately following postural drainage. temperature elevation?
4. Only during the expiratory phase of 1. Auscultate the lungs.
respirations. 2. Obtain a urine sample for culture.
3. Culture the wound.
41. A mother calls the emergency department, stating
4. Ask the client about exposure to
her daughter has just taken 8 to 10 childrens
communicable diseases.
aspirin. After verifying the child is awake,
what question would the nurse ask before telling 46. The charge nurse observes a new staff nurse who
her the child needs to be brought to the is changing a dressing on a surgical wound.
emergency department? After carefully washing her hands the nurse

COMPREHENSIVE PRACTICE TESTS 849


53155_09_PT 08_p845-864.qxd 2/27/09 6:32 PM Page 850

dons sterile gloves to remove the old dressing. 3. A nurse with open, weeping lesions of the
After removing the dirty dressing, the nurse hands puts on gloves before giving direct
removes the gloves and dons a new pair of client care.
sterile gloves in preparation for cleaning and 4. The nurse puts on a mask, a gown, and gloves
redressing the wound. What would be the most before entering the room of a client on strict
appropriate action of the charge nurse? isolation.
1. Interrupt the procedure to inform the staff
nurse that sterile gloves are not needed to 50. A young adult is admitted to the emergency
remove the old dressing. room. He is comatose. Initial assessment shows a
2. Congratulate the nurse on the use of good pulse of 90 and respirations of 32 and deep. His
technique. face is flushed and his skin is dry. He is wearing
a medical alert bracelet identifying him as
3. Discuss dressing change technique with the
diabetic. What initial order for this client would
nurse at a later date.
the nurse expect?
4. Interrupt the procedure to inform the nurse of
1. Administration of glucagon.
the need to wash her hands after removal of
the dirty dressing and gloves. 2. Oxygen at 6 liters/minute.
3. Administration of sweetened orange juice.
47. An adult male is scheduled for exploratory 4. Starting an IV of normal saline.
surgery this morning. After he is premedicated
for surgery the nurse reviews his chart and 51. A young adult is admitted to the emergency room
discovers that he has not signed a consent form. with a rapid pulse, rapid, deep respirations,
The nurses action is based on which of the flushed face, and dry skin. He is known to be
following understandings? diabetic. He responds to treatment and regains
1. Because the client came to the hospital, consciousness. Which statement he makes is most
consent is implied even if the consent for the likely related to the onset of his current problem?
surgery has not been signed. 1. Ive been eating at a lot of restaurants since
2. All invasive procedures require a consent form. I moved into my own apartment.
3. The nurse should have him sign a consent 2. I like my new job at the manufacturing
form immediately. plant.
4. The nurse should have the next of kin sign 3. I recently joined the health club and I work
the necessary consent form. out 3 times a week.
4. I have a new kitten that I like a lot.
48. The nurse has administered an intramuscular
injection. Following the procedure which is the 52. A client with diabetes who was admitted in
best technique to use for disposal of the needle ketoacidosis receives 30 units of regular insulin
and syringe? at 0730. When is he most likely to experience a
1. Recap the needle and discard in the waste hypoglycemic reaction?
container in the clients room. 1. Midmorning.
2. Recap the needle and dispose of the entire 2. At the midday meal.
unit in a special container in the utility room. 3. Midafternoon.
3. Carefully break the needle before placing the 4. At the evening meal.
needle in a needle box and the syringe in a
plastic-lined container. 53. The nurse is caring for a client with diabetes.
4. Do not recap the needle, and place syringe One morning at 1000 he becomes very irritable
with needle attached in a puncture-resistant to the nurse. What is the nurses first priority to
container. determine?
1. What is actually upsetting the client.
49. The nurse is evaluating the infection control 2. When he took his insulin and if he ate his
procedures on the unit. Which finding indicates breakfast.
a break in technique and the need for education
3. How well he slept the previous night.
of staff?
4. Which of the nurses behaviors is upsetting
1. The nurse aide is not wearing gloves when
the client.
feeding an elderly client.
2. A client with active tuberculosis is asked to 54. The nurse is teaching a young adult who is
wear a mask when he leaves his room to go to diabetic about management of his disease.
another department for testing.

850 NCLEX-RN Review


53155_09_PT 08_p845-864.qxd 2/27/09 6:32 PM Page 851

Which statement indicates the greatest need for 59. An adult client is scheduled for a magnetic
further instruction? resonance imaging test. Before scheduling the
1. Im glad Ill be able to eat out sometimes. test it is most essential for the nurse to ask the
2. I will take a snack when I go to the health client which question?
club to exercise. 1. Are you afraid of heights?
3. Ill be glad when I get off the shots and start 2. Do you have any metal in your body?
the pills. 3. Are you allergic to shellfish?
4. Its hard for me to remember to read labels 4. Are you pregnant?
on cans and boxes.
60. A 24-hour urine specimen is ordered for an
55. A client had a cystectomy with ileal conduit for adult client. The nurse goes to the client at
a diagnosis of bladder cancer. During the first 8:00 A.M. to start the specimen collection.
48 hours post-op which symptoms should be What does the nurse instruct the client to do?
reported to the physician? 1. Empty her bladder and save the
1. Absence of urinary output over a period of specimen. Collect all urine until 8:00 A.M.
1 to 2 hours. tomorrow.
2. Swelling of the abdominal stoma. 2. Empty her bladder and discard the specimen.
3. Pain along the incision site. Collect all urine for 24 hours including that
4. Absent bowel sounds. voided at 8:00 A.M. tomorrow.
3. Drink large amounts of fluid during the test.
56. The nurse is teaching an adult who had a Collect all urine for the next 24 hours.
cystectomy and ileal conduit. Which statement 4. Note the time when she next voids and
made by the client indicates a need for further collect urine for 24 hours from that time.
instruction? Notify the nurse when the collection is
1. Now that Ive had the surgery, Ill have to be completed.
careful that I dont get frequent urinary tract
infections. 61. An adult client had a stapedectomy and has just
2. My stoma is 112 inches in size now, but I returned to the nursing care unit following an
understand it will get smaller. Therefore, I uneventful stay in the postanesthesia care unit.
will need to measure it again in several What is essential for the nurse to include in the
weeks. care plan?
3. Im glad that once I get home and am better 1. Instruct the client to ask for help when
regulated, I will only have to wear an wanting to get out of bed.
appliance at night. 2. Encourage the client to drink plenty of fluids
4. I certainly dont want the stoma to close up during the day.
so I will gently dilate it with my finger once a 3. Remind the client to remain in bed for
week. 24 hours.
4. Tell the client to speak only when it is
57. An adult client has an IV infusing. The current essential for the next 24 hours.
fluid order is for Ringers lactate 1000 mL to run
in over an 8-hour period. The drop factor is 12 62. The nurse is performing an admission
gtt/mL. What is the drip rate? gtt/min. assessment on a client admitted for outpatient
surgery today. In addition to obtaining vital
58. An adult client is admitted with a diagnosis of signs, what information is most essential for the
urinary tract calculi. The physicians orders nurse to obtain?
read: vital signs every shift, morphine 10 mg for 1. Time and amount the client last
pain, Probanthine (propantheline bromide) 15 voided.
mg PO with meals, OOB as tolerated, limit fluid
2. Characteristics of clients stools.
intake to 1000 mL/24 hours, strain all urine.
Which medical order should the nurse 3. When the client last had anything to eat or
question? drink.
1. Morphine 10 mg. 4. The clients understanding of the surgical
procedure to be performed.
2. Limit fluid intake.
3. OOB as tolerated. 63. The nurse is performing an admission
4. Strain all urine. assessment on a client admitted with a diagnosis

COMPREHENSIVE PRACTICE TESTS 851


53155_09_PT 08_p845-864.qxd 2/27/09 6:32 PM Page 852

of pernicious anemia. Which assessment is the 69. A female teenager is admitted in sickle cell
nurse most likely to find? crisis. She is anemic, has painful joints,
1. Pallor, gingivitis, and fever. abdominal pain, a leg ulcer, and hematuria.
2. Jaundice, hepatomegaly, and fatigue. What should the nurse expect to include in the
nursing care plan during the acute stage?
3. Ruddy complexion, ecchymotic areas, and
distended veins. 1. Promotion of hydration.
4. Glossy red tongue, paresthesias, and fatigue. 2. Application of cold to swollen and painful
joints.
64. An infant was born 3 months ago to a mother who 3. Administration of aspirin for pain.
was diagnosed with syphilis late in her pregnancy. 4. Active exercises to involved joints.
Which information would be most useful in
determining if the baby has congenital syphilis? 70. An adult client is scheduled for a colonoscopy.
1. Irritability. Which statement by the client indicates he
2. Red rash around anus. understands the prescribed preparation
regimen?
3. Rhinitis.
1. All I need to do is give myself a packaged
4. Positive serology.
enema the morning of the procedure.
65. The nurse is caring for an infant who has 2. I will eat only jello and drink clear liquids
congenital syphilis. The baby is started on for 2 days before the test.
penicillin. Which statement is true about the 3. I will take the dye tablets with water the
babys ability to transmit the disease now that night before the test.
treatment is started? 4. All I have to do is not eat anything after
1. She will not be contagious after 48 hours of midnight the night before the test.
penicillin therapy.
2. After 10 days of antibiotic therapy she will 71. The nurse is caring for a woman who had a
not be contagious. vaginal hysterectomy 2 days ago. The indwelling
catheter has been removed. The nurse has
3. She will always be infected and be contagious.
performed a catheterization for residual urine.
4. Congenital syphilis is not contagious. Which urine amount indicates the client is
without complications?
66. The nurse is caring for an infant who is being
treated for congenital syphilis. The baby 1. 30 mL.
develops vesicular lesions on the soles of her 2. 150 mL.
feet and has a rash on her face. What is the most 3. 300 mL.
appropriate initial intervention for the nurse? 4. 500 mL.
1. Call the physician immediately.
2. Apply Neosporin ointment to the rash. 72. An 11-month-old infant is brought to the
pediatric clinic. The nurse suspects that the
3. Cover the infants hands with mittens.
child has iron-deficiency anemia. Because iron-
4. Give Benadryl (diphenhydramine) by mouth. deficiency anemia is suspected, which of the
following is the most important information to
67. The nurse is caring for a child with eczema. To
obtain from the infants parents?
prevent infection, what will be important use
with the bath? 1. Normal dietary intake.
1. Baby oil. 2. Relevant sociocultural, economic, and
educational background of the family.
2. Tepid water.
3. Any evidence of blood in the stools.
3. Bubble bath.
4. A history of maternal anemia during pregnancy.
4. Perfumed soap.
73. The nurse is assessing a 6-month-old infant. He
68. The nurse is caring for a 4-year-old child who
has acquired all the expected developmental
has eczema. Which toy is most appropriate to
milestones. Which will he have acquired most
give to this child while she is in the hospital?
recently?
1. Fuzzy teddy bear.
1. Imitates sounds.
2. Tabletop toy piano.
2. Balances head well in a sitting position.
3. Stuffed doll.
3. Smiles at mirror image.
4. 1,000-piece jigsaw puzzle.
4. Is able to grasp objects voluntarily.

852 NCLEX-RN Review


53155_09_PT 08_p845-864.qxd 2/27/09 6:32 PM Page 853

74. The nurse is assessing a 2-month-old infant. The 79. The client has had a central venous access
mother says the baby has colic. Which of the device (Hickman catheter) inserted. Correct
following is the most appropriate initial step in placement has been confirmed by X-ray. The
managing colic? nurse is to set up parenteral nutrition to be
1. Obtain a detailed history of normal daily connected to the Hickman catheter. The nurse
events surrounding the infant. should place the client in which position for the
2. Eliminate cows milk from the infants diet or procedure?
from the diet of the lactating mother. 1. Trendelenburg.
3. Request that the physician prescribe 2. Semi-Fowlers.
antispasmodic and antiflatulent medication 3. Side lying.
and instruct the mother on its use. 4. Supine.
4. Identify the mothers feelings regarding
mothering and the infant. 80. An adult client has an order for a nasogastric
tube. Before inserting the tube the nurse
75. The nurse is talking with the parents of a normal measures the amount of tube needed. How
2-month-old. Which of the following should be would the nurse determine the amount of tube
included in anticipatory guidance for the next needed?
month of life? 1. Measure from the forehead to the ear and
1. Stranger anxiety will begin. from the ear to the umbilicus.
2. The posterior fontanel will close. 2. Measure from the chin to the back of the
3. The first tooth will erupt. throat and from the back of the throat to the
4. The child will begin to show awareness of umbilicus.
strange situations. 3. Measure from the mouth to the xiphoid
process and add 2 inches.
76. The clinic nurse is performing anticipatory
4. Measure from the tip of the clients nose to
guidance with parents of a 2-month-old infant.
his earlobe and from the earlobe to the
Instruction aimed at the prevention of accidents
xiphoid process.
would best be planned with which as a reference?
1. Mouthing of objects is very prominent. 81. An adult client is admitted with an asthma
2. Grasps and manipulates objects well. attack. Aminophylline IV is prescribed. How
3. Dislikes being restrained. will the nurse know if the desired results are
4. Crawling and Moro reflexes are present. achieved in the client?
1. Pulse rate increases.
77. A young woman had surgery today. Her father, a 2. Breathing effort is less.
physician but not her surgeon, enters the
3. Pain is relieved.
nursing station and asks for her chart. What is
the best action for the nurse to take? 4. Respiratory rate increases.
1. Allow him to read the chart as requested. 82. An adult client has just returned to the nursing
2. Do not allow him to read the chart. care unit following a gastroscopy. Which
3. Ask the attending surgeon for permission for notation is essential for the nurse to include on
him to read the chart. the nursing care plan?
4. Ask the client if she wants him to read the 1. Throat lozenges PRN for sore throat.
chart. 2. Supine position for 6 hours.
78. An adult client is being prepared for abdominal 3. NPO for 4 hours.
surgery. She refuses to remove her plain gold 4. Clear liquid diet for 24 hours.
wedding band before going to surgery. What is
the best action for the nurse to take? 83. An adult has myasthenia gravis and is admitted
in myasthenic crisis. The nurse should include
1. Firmly insist that it must be removed or
which of the following on the nursing care plan
surgery cannot be performed.
immediately after admission?
2. Ask her husband to assist you in discussing
1. Suction equipment at bedside.
this with his wife.
2. Active exercises QID.
3. Cover the wedding band with adhesive tape
and tape it to her finger. 3. Give medicines following meals.
4. Premedicate her and remove the wedding 4. Prepare client for the Tensilon test.
band after she falls asleep.

COMPREHENSIVE PRACTICE TESTS 853


53155_09_PT 08_p845-864.qxd 2/27/09 6:32 PM Page 854

84. The nurse is evaluating a client who is receiving 3. In semi-Fowlers


total parenteral nutrition (TPN) therapy. Which 4. Prone
observation best indicates the client is having
the desired therapeutic effects from TPN? 89. An adult is to have a pelvic sonogram tomorrow.
1. The client has regular bowel movements. The nurse knows the client understands
2. The client maintains blood sugars in the preprocedure instruction when which statement
normal range. is made?
3. The client is gaining weight. 1. I wont eat for 12 hours before the
procedure.
4. The client has normal urine output.
2. I will take a laxative the night before the
85. A man fractured his right ankle, which was procedure.
plaster casted in the emergency room. Before 3. I will empty my bladder immediately before
discharge from the emergency room the nurse the test.
gives him instructions for his care. Which 4. I will drink several glasses of water just
statement by the man indicates a need for before I come.
further instruction?
1. I will keep my foot elevated when I go 90. An adult has had a total hip replacement and is
home. to be discharged home. The client can ambulate
2. My wifes hair dryer should work well to with a walker. The nurse is evaluating the home
dry the cast. care environment. Which finding alerts the
nurse to a potential safety problem?
3. I wont let anyone sign my cast for a couple
of days. 1. The bathroom has a shower but not a tub.
4. It is okay to wiggle my toes on my right 2. The bathroom door is narrow.
foot. 3. The kitchen is equipped with a microwave.
4. There are scatter rugs on the floor in the
86. A woman who is 40 weeks pregnant, is admitted entrance hall and the living room.
to the labor suite. While the nurse is assessing
her, her membranes rupture. The nurse 91. A young adult is admitted for a broken arm,
immediately performs a vaginal exam. During ruptured spleen, increased intracranial pressure,
the exam, the nurse feels a loop of cord and damage to the spinal cord at the level of C-5
protruding into the vagina. What is the best and C-6 from a motor vehicle accident. His
immediate action for the nurse to take? Glasgow Coma scale score is 10 and he is
1. Attempt to lift the fetal head off the cord. aphasic at present. Which nursing diagnosis
2. Prepare for immediate vaginal delivery. would be the highest priority?
3. Call the physician. 1. Impaired verbal communication.
4. Place the tocometer on the mothers abdomen. 2. Self-care deficit.
3. High risk for infection.
87. An adult is in acute renal failure secondary to 4. Decreased intracranial adaptive capacity.
hemorrhagic shock. Which statement by his wife
best indicates to the nurse that she understands 92. An adult client was experiencing vertigo and
her husbands condition? severe headaches and a blood pressure of
1. I understand my husband will need dialysis 180/104. The medication orders include
for the rest of his life. furosemide (Lasix) and quinipril hydrochloride
2. I need to watch our son for evidence of renal (Accupril). What specific drug information
failure as he grows up. should the nurse include in the discharge plan
of this client?
3. My husband will be on a special diet for a
long time. 1. Take the Lasix in the morning and the
Accupril at bedtime.
4. My husband has a good chance to recover
normal kidney function. 2. Change positions slowly and increase dietary
potassium.
88. The nurse is planning care for an adult who has 3. Discontinue the Lasix once blood pressure is
just had a liver biopsy. What would be the within normal range and increase fluid
proper positioning for the client? intake.
1. On the left side 4. Increase sodium intake and monitor blood
2. On the right side pressure monthly.

854 NCLEX-RN Review


53155_09_PT 08_p845-864.qxd 2/27/09 6:32 PM Page 855

93. An adult was admitted to the respiratory floor 97. An elderly client lives alone at home and has a
with COPD. The nurse finds him extremely history of hypertension and constipation, but
restless, incoherent, and showing signs of acute takes no medications. Which of the following
respiratory distress. He is using accessory assessment statements by the nurse will help to
muscles for breathing and is diaphoretic and devise a strategy to help the client?
cyanotic. What is the nurses best initial action? 1. When did you have your last bowel
1. Administer oxygen as ordered. movement?
2. Assess vital signs and neural vital signs. 2. Tell me what you had to eat yesterday.
3. Administer medication which has been 3. Why do you not take any
ordered for pain. antihypertensives?
4. Call respiratory therapy for a prescribed 4. Tell me how much you walk during the
arterial blood gas (ABG) analysis. day.

94. The nurse is evaluating an adult with respiratory 98. The nurse is assessing an elderly woman. Which
disease. Which finding, if observed, indicates statement by the client indicates an abnormal
compliance with breathing exercises? finding and one that needs to be further
1. Decreased use of pursed lip breathing. evaluated?
2. Decreased coughing after exhalation when 1. I move a little slower these days.
using resisted breathing exercises. 2. I cant seem to remember whats going on
3. Increased coughing after exhalation when now.
using resisted breathing exercises. 3. I enjoy reading, but have to use a magnifying
4. Inhaling through the mouth and exhaling glass.
through the nose. 4. My skin is thin and dry and there are little
brown spots on my hands.
95. The nurse is caring for a client who has an order
for a cooling blanket. What does the nurse need 99. The nurse is instructing a client how to perform
to assess before starting the procedure? Kegels exercises. Which is most appropriate for
1. If the client is improperly exposed. the nurse to include in the instructions?
2. The skin for areas of breakdown. 1. Squeeze your buttocks.
3. The amount of shivering the client exhibits. 2. Hold your urine as long as possible before
4. The skin color for cyanosis. voiding.
3. Push down on your lower abdomen while
96. The nurse is instructing a homebound client holding your breath.
how to apply a warm compress to her wrist. 4. Practice stopping your urine in midstream.
Which comment by the client indicates to the
nurse that she understands the safety aspects of 100. The school nurse is called to the playground
the procedure? where an 8-year-old child is lying on the ground
1. I will take my temperature before and after with bright red blood spurting from a wound in
apply

You might also like